{"text1":"异涡虫(学名:\"Xenoturbella\") 是一类生活在海底的两侧对称动物,现在仅知1属6种,分布于波罗的海和太平洋东部。这种生物早在1949年就被发现了,不过,那时它被认为是一种原始的涡虫。此后也有研究认为这是一种软体动物(Noren & Jondelius,1997),不过它的身体构造却与软体动物有极大的差异。直到2003年根据DNA研究的结果,异涡虫被确认为一类独立的后口动物(Bourlat et al., 2003),并确认其主要以软体动物的卵为食品,正因为如此,导致大量软体动物的DNA混入其标本中,从而导致此前被误认为是软体动物。经过更加详细的研究后,建立了仅有异涡虫的异涡动物门(Bourlat et al., 2006)。异涡虫根据18S rRNA被分在后口动物基部。然而根据形态学上的特征,比如上皮细胞的纤毛结构,异涡虫与无腔动物应当具有紧密的关联。近期的分子生物学研究显示原本的异涡虫可以与无腔动物组成一个单系群异无腔动物。2016年发表的对 \"X. profunda\" 等的研究结论支持异无腔动物是肾管动物的旁系群这一分类。异涡虫身体呈两侧对称。身体结构简单,其没有大脑、消化道、排泄系统和性腺(但在囊中有配子、卵子和晶胚产生[Israelsson and Budd]),具有扩散神经系统和纤毛。体型方面,较大的种 \"X. monstrosa\" 可长达,而较小的种 \"X. hollandorum\" 则仅有。异涡动物门下仅有1属6种,其中 \"X. bocki\" 和 \"X. westbladi\" 较早发现于瑞典、苏格兰、冰岛和挪威附近海域,而其余4种均发现于东太平洋。其中体型较小的两个种 \"X. bocki\" 与 \"X. hollandorum\" 生存于以上水域中,而体型较大的三个种 \"X. churro\"、\"X. monstrosa\" 与 \"X. profunda\" 生存于以下深水中。","text2":"最早发现异涡虫是在什么时候?","label":1} {"text1":"褐拟鳞鲀(学名:),又称绿拟鳞鲀、胡麻皮剥鲀、黄褐砲弹、剥皮鱼、褐拟板机鲀,为鳞鲀科拟鳞鲀属的鱼类。分布于印度洋及太平洋的热带海内、在印度洋南达莫三鼻给的德拉果阿、北达红海及印度的安达曼群岛、东经马来半岛、印度尼西亚、菲律宾、往南到太平以及西沙群岛和海南岛等海域等,属于暖水性鱼类。该物种的模式产地在毛里求斯。本鱼分布于红海及印度太平洋区,包括东非、红海、模里西斯、塞席尔群岛、马尔地夫、印度、斯里兰卡、缅甸、中国、台湾、日本、越南、泰国、圣诞岛、印尼、新几内亚、菲律宾、吉里巴斯、密克罗尼西亚、东加、帛琉、萨摩亚群岛、澳洲、土木土群岛及新喀里多尼亚等海域。水深0至40公尺。本鱼体呈椭圆形、吻小,体色淡黄褐色,齿白色,具缺刻,各鳞片中央大多有灰黑色小点。自眼到胸鳍基部附近有一黑褐色横斜纹形宽带,两眼间距及其附近灰褐色。上唇后方的光皮处呈红黑色,口角后方纵光皮的背缘红黑色。第一背鳍淡红色,散布红黑色,第一棘粗大,奇鳍上有蓝绿外缘,尾柄上有数列平行的突起。幼鱼体色较黄,奇鳍上无蓝绿色外缘。背鳍硬棘3枚;背鳍软条24至26枚;臀鳍软条22至24枚,体长可达65公分。本鱼栖息于岩礁,成鱼常单独或成对地在礁区外围出现,幼鱼则大多在有隐蔽物附近出现如珊瑚礁,杂食性,以海胆、甲壳类、水螅等为食。可食用,一般多作观赏鱼,为蛮受欢迎的鱼种之一。因食物链的关系,体内可能含有鱼毒。","text2":"褐拟鳞鲀有多少枚背鳍软条?","label":1} {"text1":"霍弋(,卒于271年或以前),字绍先。荆州南郡枝江人,三国时期蜀汉及西晋将领。父亲为霍峻,同样是蜀汉将领。蜀汉昭烈帝末年即223年,为太子舍人。刘禅登位后,除谒者。任在公元226年,随诸葛亮北伐,诸葛亮任他为记室,让他和儿子(实为侄子)诸葛乔相处。诸葛亮死后,任黄门侍郎。刘禅立刘璿为太子,以霍弋为太子中庶子。刘璿好骑射,出入无度,霍弋援引古书进行规劝,表现很得体。后来参军,任庲降都督「副贰都督」,转任「护军」,统事如前。霍弋曾领永昌太守讨伐贼军,后迁监军翊军将军,兼任建宁太守,统治南中诸郡。263年,升为安南将军。同年,霍弋闻魏兵入蜀(魏灭蜀之战),欲赴成都,刘禅以备敌既定,不听霍弋言。霍弋闻成都失守、蜀汉亡国后,素服望西大哭三日。后降于魏,司马昭拜为南中都督,与巴东太守罗宪守备蜀地。霍弋后来遣兵救援吕兴,吕兴被其手下功曹杀害,霍弋表奏建宁爨谷为交趾太守,又率牙门将董元、毛炅、孟干、孟通、爨能、李松、王素等人出征,平定交趾、日南、九真三郡,封为列侯,进号崇赏焉。《资治通鉴》上说:霍弋自宁州(今云南)遗杨稷等人经略交、广二州,采取水、陆二路并进。交州平定后,霍弋令杨稷、毛炅等戌守交趾,并与他们起誓说:「如果贼兵围城不满百日而投降,你们的家属就要受诛;但如果超过百日而救兵不至,以至于城池陷落,就由我霍弋担起责任。」271年,交州又被吴军陶璜夺回,杨稷、毛炅粮尽援绝,守城百日后投降而就死。在三国演义称霍戈,内容与正史无大差别,主要描述尽在三国演义第119回。","text2":"霍弋是哪个时期的人?","label":1} {"text1":"克里雍大饭店(Hôtel de Crillon)是巴黎的一座古老的五星级豪华酒店,位于协和广场10号(北侧),香榭丽舍大街东端。克里雍大饭店开始于蜂蜜色的大理石大堂,近旁是充满水晶吊灯和装饰有7种不同大理石的Les Ambassadeurs 餐厅。酒店的大厅充满了17和18世纪挂毯,吊灯,镀金和织锦家具,精致的雕塑,和路易十六的柜橱和椅子。其顶层的伦纳德·伯恩斯坦套房的阳台可以看到巴黎壮观的景色,也有一架Maestro的钢琴。克里雍大饭店有103间客房和44间套房,占据着被皇家路分开的两座相同的石头建筑中的一座,由路易十五委任建筑师路易斯·弗朗索瓦Trouard 兴建于1758年。起初两座建筑均为作为政府机构,东侧的建筑今天仍是法国海军部,然而,西侧的建筑建成了豪华酒店,很快,王后玛丽·安托瓦内特和她的精英朋友就经常光顾此处。她来到这家酒店学习钢琴课。","text2":"Les Ambassadeurs 餐厅装饰有多少种不同的大理石?","label":1} {"text1":"怀邵衡铁路是中国一条建设中连接湖南省怀化市与衡阳市的高速铁路。怀邵衡铁路西起怀化西站,经安江、江口、洞口、石下江(预留)、隆回、岩口铺;与洛湛铁路邵永段三线并行至邵阳站,再与新娄邵段经邵阳东站四线并行至邵东新站,然后自邵东站货运场东侧分道扬镳;次第过金兰、西渡、松木园、麻元,最后接入颜家垄,并在此对接衡茶吉铁路,同时将兴建联络线连入衡阳站。怀邵衡铁路全长大约318km。该线路于2008年七月初进入《国家中长期铁路网规划调整方案》,原拟于2010年内动工。2011铁道部高层人事变动及其后的温州7·23动车追尾事故,使得国内所有铁路新工程上马延迟。2012年4月14日铁道部同意在得到湖南省部分资金垫付的前提下,怀邵衡铁路与娄邵铁路四线并行的32km路段可以率先开工。怀邵衡铁路有限责任公司于2012年11月2日成立。2013年12月24日可行性研究报告获得国家发改委批复。2014年1月6日全线开工。怀邵衡铁路设计技术标准:","text2":"怀邵衡铁路连接了哪两个城市?","label":1} {"text1":"1 + 1 + 1 + 1 + …,亦写作 formula_1, formula_2或formula_3,是一个发散级数,表示其部份和形成的数列不会收敛。数列1可以视为公比为1的等比级数。不同于其他公比为有理数的等比级数,此级数不但在实数下不收敛,在某些特定数字p的p进数下也不收敛。若在扩展的实数轴中,因为部份和形成的数列单调递增且没有上界,因此级数的值如下此发散级数无法用切萨罗求和及阿贝尔和的求和法求和。当出现于物理运用时,它也解释为,它是黎曼ζ函数在零点的取值。上述二个公式在formula_6时不成立,必需利用解析连续定义。用上式求得(假设formula_8)以下在时的级数展开:也是这种意义下此级数的和:也可用其他的s值来为其他的级数求和,例如ζ(-1)=1 + 2 + 3 + 4 + ⋯=–1\/12,ζ(-2)=1 + 4 + 9 + ... = 0,其通式为其中\"B\"为伯努利数。在同一年内,有两位杰出的物理学家斯拉夫诺夫(A. Slavnov)和F. Yndurain 分别在巴塞罗那作了学术演讲。两场学术演讲的主题不同,但是在这两个人的介绍当中,都说到了一句令观众非常难忘的话:“各位都知道,1 + 1 + 1 + 1 + … = −1⁄2”,某程度意味著「如果观众不知道这个,那么继续听下去是没有意义的。」","text2":"物理学家斯拉夫诺夫(A. Slavnov)和F. Yndurain 分别在哪里作了学术演讲?","label":1} {"text1":"野山羊(\"Capra aegagrus\")是一种野生的山羊,分布在欧洲及小亚细亚至中亚及中东。群族中野山羊的数量可以多达500头。较老的公羊会在发情期躯使年幼的公山羊进到母群中交配。平均妊娠期为70日。母山羊一般会产下一只幼山羊。幼山羊出生后会立即跟随母山羊,在6个月后断奶。母山羊在岁半至两岁半时就达至性成熟,公山羊则要到三岁半至四岁。牠们的寿命介乎12至22岁之间。虽然野山羊在一些地区是原生种,但做为入侵物种则会是非常具破坏性的。在加拉帕戈斯群岛,外来的野山羊大量啃食当地的植物,破坏了生态的平衡,后来人们只能将牠们猎杀至岛内灭绝来解决问题。野山羊分布在阿富汗、亚美尼亚、阿塞拜疆、孟加拉国、塞浦路斯、格鲁吉亚、希腊、印度、以色列、伊朗、伊拉克、义大利、黎巴嫩、阿曼、巴基斯坦、克什米尔、俄罗斯、斯洛伐克、叙利亚、土耳其及土库曼斯坦。","text2":"野山羊分布在哪些地方?","label":1} {"text1":"彰化基督教医疗财团法人,系由财团法人台湾基督长老教会于西元1995年10月23日捐助设立的公益法人,创立时名为财团法人彰化基督教医院,西元2013年4月3日更名为彰化基督教医疗财团法人。院史可溯源自西元1896年,英格兰长老教会海外宣道会(Foreign Missions Committee of the Presbyterian Church of England)所派遣的宣教师,梅鉴雾牧师及兰大卫医师在彰化市开设的诊所,同时进行医疗传道。其所属彰化基督教医院,为彰化地区唯一的医学中心。前身为彰化市的一间诊所,于西元1896年由梅鉴雾( Rev. Campbell Moody)牧师及兰大卫( David Landsborough III)医师建立,西元1907年10月19日正式命名为「英立彰化基督教医院」。彰化基督教医疗财团法人为深入就近服务社区,近几年已发展为体系运作方式,除了自身筹办之彰化基督教医院、二林基督教医院、鹿港基督教医院、鹿东基督教医院,接手承办的云林基督教医院外,为提供民众可近性医疗服务,也进一步与彰化县员林市的员生医院、南投县南投市的南基医院、南投县草屯镇的佑民医院进行医疗合作,密集提供医疗服务,形成紧密之彰基医疗服务体系。而员生医院已于2016年8月31日正式结束合作关系。云林基督教医院原名为慈爱综合医院,创立于1896年8月8日,座落在云林县西螺镇,在2007年通过新制医院及教学医院评鉴,成为区域教学医院,2009年3月1日,改制为彰化基督教云林分院,又随彰化基督教医院改制,更名为云林基督教医院。资讯部、医工部、工务部、新建工程室、体系人力资源处、国际医疗事业部、医疗勤务部、秘书室、体系财务处、体系稽核辅导处、体系资材运筹管理处、院史文物馆、公共关系部、体系营运中心、医疗事务部、体系营养暨膳食部。内科部、外科部、妇产部、儿科部、耳鼻喉暨头颈部、牙科部、中医部、急诊医学部、其他科别、医疗中心、社区医学部教学部、研究部、医疗品质部、医学图书部、教研创新行政中心、实证医学中心、实验动物中心、医学伦理中心、人体试验委员会1896年11月29日,来自英国的传教士兰大卫医师和梅监雾牧师在台湾中部进行医疗传道工作以来,就开始聘任牧师的院牧工作。目前主要发展的工作:","text2":"何时开始开始聘任牧师的院牧工作?","label":1} {"text1":"Grace是「GRaphing, Advanced Computation and Exploration of data」的缩写。它是在X Window系统和Motif下的所见即所得(所见及所得)的二维绘图软件。Grace可以运行在任何类Unix系统操作系统下。它也成功的运行在OpenVMS、OS\/2和Microsoft Windows 9*\/NT\/2000\/XP(Cygwin)。Grace从Paul Turner的ACE\/gr演化而来(Grace也被称为ACE\/gr的翻版或者改进版)有时也被称为Xmgr. Grace小组在Evgeny Stambulchik的领导下从Xmgr分化发展了grace。他们遵循GNU通用公共许可证原则发行了grace。Paul Terner依旧在维护原来Xmgr的一个封闭版本。Grace能创造印刷质量的图形输出。它支持鼠标操作和脚本代码(该程序语言或者通过其它计算机程序语言)。它能对用户定义的任何复杂方程进行线性和非线性最小二乘法拟合--不论是否有限制。其他的分析工具包括快速傅立叶变换,积分,求导,样条函数,插值和图形平滑处理.","text2":"Grace又被称为什么?","label":1} {"text1":"美国在台协会商务组()是美国商务服务局在台湾推广贸易的单位,隶属美国商业部(U.S. Department of Commerce)。美国商务服务局的宗旨是推广美国产品与服务的出口,尤其是促进中小企业的外销,维护美商在海外市场的商业利益,以及协助美商寻找理想的国际贸易伙伴等。美国在台协会商务组的服务内容包括帮助美国出口商安排与当地商业伙伴与主管机关的会议,提供台湾市场的产业调查报告,并建立线上美国供应厂商名单供台湾进口商洽询。美国在台协会商务组于台北市、高雄市均有办公室。目前台北办事处有四位商务官与十七位职员,高雄则有一位商务官与一位产业专员,为当地进口商服务。美国商务服务局的核心优势为拥有庞大的国际贸易人才网络,包括全美的美国出口协助中心(U.S. Export Assistance Center),以及遍及海外八十多个国家、超过一百五十个城市的美国大使馆与领事馆。","text2":"美国在台协会商务组是什么?","label":1} {"text1":"胎生贝湖鱼,是辐鳍鱼纲鲉形目杜父鱼亚目胎生贝湖鱼科的其中一种。本鱼分布于俄罗斯贝加尔湖。本鱼体延长,头中大,吻尖长,眼中大,上侧位。口大,前位。眶下骨,前鳃盖骨和下颔多孔。背鳍两个,第一背鳍短小;第二背鳍长,后端几达尾鳍,但不与尾鳍相连,臀鳍与第二背鳍相对且同形,后端也几达尾鳍;胸鳍长,后端超过臀鳍起点;无腹鳍;尾鳍截形。背鳍硬棘7至9枚、背鳍软条31至34枚、臀鳍软条32至36枚。体长可达16公分。本鱼习性类似大洋性鱼类,巡游于湖泊,深度达100至250公尺间。繁殖季节在7至9月;春天来临时,随著波浪被打至岸边。生活时,身体透明且富含脂肪,几占身体25%,所以死后鱼体不沉,反而浮至水面而结冻。卵胎生鱼类。鱼体小型,较不具经济价值。","text2":"胎生贝湖鱼栖息在多深的湖面下?","label":1} {"text1":"本列表显示于无线电视J2播放的动画。本类别包括第一次于无线电视免费频道播放的节目,只列出首播时段及时间。年份以首播日计算。J2自5月28日进行节目表调动,取消星期一至四21:30-22:00、星期五21:30 - 22:30; 24:00-24:30 和星期日23:30-24:00动画时段。J2在9月16日起将动画时段由星期一至四22:00-22:30及星期六23:30-24:30移动至星期一至四24:00-25:00及星期六24:00-24:30;至11月9日起星期六动画时段移动至23:00-24:00。J2在10月15日起为配合黄金时段的改革,将维持数年的平日24:05-24:35的动画改由逢周末,于星期六23:45-24:15及星期日23:35-24:35播出,并取消周末的深夜重播时段。于11月19日增设中午动画档期,逢非赛马日的12:30-13:30播出。本年起再度恢复深夜动画之重播时段,时间为逢星期六29:30-30:00及逢星期日28:00-29:00播出以下未有确切首播日期之预定节目,一般为无线节目巡礼、J2动漫热及一些大型活动上所公布,亦包含TVB周刊及官方时间表所记载的资料。本类别包括曾于无线电视其他免费频道播放过的动画节目,大部份改为双语广播。有部份原使用粤语片头曲的动画,亦于另一声道提供日语版,甚至改回使用日语片头曲。此外,16:9制作的动画改以全萤幕播放。","text2":"J2自5月28日进行节目表调动。取消了哪些动画时段?","label":1} {"text1":"《无双大蛇Z》()是光荣旗下ω-force开发的动作游戏,于2009年3月12日登陆索尼PlayStation 3,并于2009年11月27日推出Windows版。《无双大蛇Z》是真·三国无双系列和战国无双系列的融合作品——无双大蛇系列的第三作,将《无双大蛇》和《无双大蛇:魔王再临》的所有内容合并移植到最新的游戏及平台上,并添加了新的角色和关卡。得益于次世代游戏机的强大机能,该游戏在当时为无双系列中同屏幕人数最多的一款。该作的PlayStation 3日文版于2009年3月12日在日本和台湾地区发售,而PlayStation 3中文版则于2009年7月30日由台湾光荣推出,Windows中文版则于2009年12月24日开始发售。由于《无双大蛇Z》是先前两部作品的结合之作,因此游戏操作方式也一沿其旧:玩家选择三名角色组成小队,并在某一关卡中使用。不过,游戏不少某些细节得以加强。但是删减敌兵模组的状况依旧。PC版游戏画面较前一代有较大改观,且支持宽屏幕模式画面,同屏人数也明显增加。本作中,第二玩家可以选择有别于第一玩家的角色队伍,而不像前两作一样,两名玩家必须使用相同的角色。剧情模式也有部分改动,《无双大蛇Z》新增了12个关卡,使得剧情模式关卡的总数达到40个。而且剧情模式的关卡可以使用其他武将。游戏新增了以下的设置:在无双大蛇系列前两作92名角色的基础上,《无双大蛇Z》再度追加了两名新角色。此外,百百目鬼和牛鬼原本在《魔王再临》中只能在对战模式和生存模式中使用,而在《无双大蛇Z》中则可使用于故事模式和剧情模式,并且改变了他们的动作模组 。以下新增的角色:可在故事模式和剧情模式中使用的以下角色:加上这四位新增角色,在《无双大蛇Z》中共有96名角色登场,成为当时登场人数最多的无双系列游戏。尽管《无双大蛇Z》沿袭了光荣一贯的移植作风,整体并无太多新意,但在日本销量依然看好,上市首日便获得了售出7.9万套的好成绩。","text2":"《无双大蛇Z》新增了多少个关卡?","label":1} {"text1":"迅销公司(Fast Retailing Co., Ltd.,ファーストリテイリング,、)是日本的零售控股公司。持有的品牌包括知名的UNIQLO(优衣库),以及ASPESI、Comptoir des Cotonniers、Foot Park、National Standard等。是世界第四大休闲服公司,同时是亚洲最大的服装公司。2007年迅销公司在纽约巴尼的竞标案中以900万美元输给了美国琼斯服饰集团(Jones Apparel Group)。迅销公司的历史最早是由柳井正的父亲柳井等,在1963年5月1日个人创业的「小郡商事」,以经营男性服饰店为主要业务。之后在1963年5月成立了「小郡商事株式会社」。1984年6月开设了「Unique Clothing Warehouse」,为UNIQLO的前身。1991年公司正式改名为「Fast Retailing」。迅销将于2014年3月5日以香港预托证券(HDR)形式在联交所主板第二上市及买卖,迅销在香港最多有5亿份HDR作供申请,每手300份HDR。迅销上市后,会以港元报价及买卖。","text2":"迅销公司早期从事什么业务?","label":1} {"text1":"《粉红色的火烈鸟》(Pink Flamingos)是一部由约翰·沃特斯执导的美国荒诞幽默电影。当1972年上映时,这部片子因其变态的内容引发了大规模争论,并使它成为了最臭名昭著的非主流电影。影片的主角是一个肥胖的女人,自认为是“世界上最污秽”的人。另一个主角是一对同样变态的夫妇,这对夫妇专门强迫被绑架的妇女受孕,然后将出生的孩子卖给别人。影片就在双方不可避免的冲突中展开,最后,胖女人赢得了胜利,解放了那些被关押的妇女,又枪决了这对夫妇。此片最具争议之处是因为展现了人工授精、口交、吃屎、兽奸等镜头,大大超出了普通观众的心理承受能力。甚至该片在影院放映时会为观众准备呕吐袋。","text2":"《粉红色的火烈鸟》是一部什么样的电影?","label":1} {"text1":"明清音系指行用于明朝和清朝的汉语官话音系,是现代普通话的前身。和之前历朝一样,明朝和清朝都编有官方的韵书,如《洪武正韵》、《佩文诗韵》等。但这些韵书都承袭平水韵的音系,不能作为分析当时口语的依据。所幸的是,有一些私修的描述声韵的册子,不同程度地反映了当时的实际语音,如明兰茂作《韵略易通》,清无名氏作《圆音正考》。从明季开始,陆续有西方人到中国,他们用拉丁字母标注汉字的读音,为语音的实际音值留下了宝贵的材料。如明朝有利玛窦作《西字奇迹》,金尼阁作《西儒耳目资》。除此之外,还有一些对音材料作为补充。《韵略易通》的创作约当十五世纪,上有一首早梅诗,归纳了当时的声母系统,一共四句二十字,每个字代表一个声母。原诗为:东风破早梅,向暖一枝开。
冰雪无人见,春从天上来。由此得到明初声母如下表:一百多年后,意大利传教士罗明坚和利玛窦留下的手稿和书籍用拉丁字母记录了当时的官话语音,他们记录的声母系统比早梅诗多了和,但这套语音系统对中古汉语的疑母的划分十分混乱,部分疑母字的确有声母脱落的现象,但同时有大量影母的字被派分了疑母的声母。《西字奇迹》是用拉丁字母为汉字注音写的几篇文章,《西儒耳目资》是用拉丁字母注汉字音的汇集,两者之间有传承关系,使用了统一体系的拉丁拼音方案。从这套拉丁拼音系统可以归纳出明朝的韵母系统。继塞音韵尾消失之后,鼻音韵尾-m至此也消失了,因此只剩下元音韵尾-i、-u和鼻音韵尾-n、-ŋ。同时发展出卷舌元音。《西字奇迹》等书用元音字母上的标号表示声调,一共有5种标号,分表代表阴平、阳平、上、去、入5调。早梅诗的音系中古汉语见、溪、群、晓、匣等母的字仍然保持舌根音、、,精、清、从、心、邪等母的字也仍然是平舌音、、。但到了清朝,这种局面改变了。清朝后期的《圆音正考》针对当时尖团混淆的现象,要求严格区别尖团音。其序言说:“试取三十六母字审之,隶见溪郡晓匣五母者属团,隶精清从心邪五母者属尖,判若泾渭。”可见原作、、的细音字,在这之前已经变为舌面音(团音)、、了。而且该书写作的当时,、、的细音也有混入舌面音的迹象。至此,现代普通话的声母格局就完全形成了。","text2":"《西字奇迹》等书用元音字母上的标号表示声调,一共有5种标号,分表代表什么调?","label":1} {"text1":"卡利亚·基拔(,)生于英国汉默史密斯,是一名英格兰籍职业足球员,于2010年夏季约满离开母会阿仙奴。直到2005\/06年,基拔通常在阿仙奴的青年后备队效力。他在首次在2005年11月29日的联赛杯赛事上场,并于12月7日,在一个欧洲联赛冠军杯比赛对阿积士,作为替代左后卫,入替受伤的劳伦。2006年7月21日阿仙奴宣布,将基拔出借卡迪夫城整个2006-07赛季,其后转借给修安联。2008年1月3日返回阿仙奴授予46号码。2008年2月11日,阿仙奴的英超联赛比赛中对布莱克本作为后备球员。但2008年7月10日,基拔被出借莱斯特城的一个赛季之久。2009年3月3日主场对-{zh-hans:斯托克港;zh-hk:史托港}-,开赛后仅两分钟,基拔的传中球「挞Q」却直入网角,是他个人首个入球。基拔在外借期间成为常规正选,整季上阵达39场及射入1球,协助莱斯特城赢取英甲联赛冠军及重返英冠。2009\/10年上半季仅于两场英格兰联赛杯及一场无关痛痒的欧联分组赛上阵,将于季后约满的基拔获外借到英冠榜末球会彼德堡直到球季结束,期间上阵10场。2010年夏季基拔约满阿仙奴成为自由球员,仅为母会合共上阵10场,英超「升班马」黑池有意罗致,其后前往-{zh-hans:谢菲尔德联; zh-hk:锡菲联;}-参加试训,惟未有获得录用。","text2":"卡利亚·基拔的职业是什么?","label":1} {"text1":"梳邦()是马来西亚雪兰莪州八打灵县的一个市郊,行政上隶属于莎亚南市政厅(MBAS)。其范围包括梳邦新村和。苏丹阿都阿兹沙机场--吉隆坡旧有的国际机场枢纽就坐落于此,主要提供东南亚区域的航班。梳邦区内有许多高尔夫球场如格林玛丽高尔夫球场、和 主要住宅区种类有高级公寓、半独立式、洋房等。梳邦也是马来西亚的一个国会下议院远区,选民背景多元。目前该区议员为自2008大选至今,来自希望联盟人民公正党的西华拉沙。截止2004年的选区划分,梳邦下有武吉兰樟、 哥打白沙罗和巴也加拉斯三个州议会选区。马航和飞萤航空皆在 梳邦苏丹阿都阿兹沙机场设立总部。而 和拉亚航空也在这设有总部。梳邦柏兰岭商业中心是梳邦天空花园航站楼最近的商业枢纽。吉隆坡日本学校位于梳邦的,是马来西亚除槟城以外唯一一间日本学校。梳邦是马来西亚最多雷暴的地区,其中:梳邦交通完善,可通往临近城镇如吉隆坡、梳邦再也、莎亚南和灵市市郊如双威镇、、哥打白沙罗、万达镇、和珍珠白沙罗。主要道路有 联邦大道、 (KESAS)、新巴生河流域大道 (NKVE),、 (NPE)、,、和白沙罗-蒲种大道。","text2":"梳邦区内有几个州议会选取?","label":1} {"text1":"陈锋(Fung Permadi,),为出生于印尼中爪哇省普禾格多的已退休男子羽毛球运动员。他专攻单打,原本参加印尼国家队,后来入籍中华民国而改代表中华台北参赛。虽然陈锋早在1990年即已展现令人印象深刻的技术能力,但由于当时印尼国家队好手如云,男子单打世界排名更高的还有阿迪、魏仁芳、佐戈·苏普里昂托、哈里扬托·阿比与蔡祥林等人,因而经常无法代表国家出赛。1990年代中期,陈锋移居台湾并入籍后,开始他的羽毛球第二春。到了1990年代末期,已经超过30岁的他才逐渐打出好成绩。他除了赢得多项国际赛冠军外,还以31岁的「高龄」获得1999年世界羽毛球锦标赛亚军,仅败给中国队的孙俊。只列出曾进入准决赛的国际赛事成绩:","text2":"陈锋是谁?","label":1} {"text1":"面波,又称表面波(),是沿不同介质(常常是两密度不同的流体)界面传播的波。折射率梯度波导下的电磁波也可以成为表面波。地波(沿地面传播的无线电波)也是一种表面波。在地震中,面波也是地震波之一,在地球表面扩散,因此测量面波震级也是除测量矩震级外的另一种估计地震规模的方法。对于浅源地震,中国习惯使用面波震级,美国则习惯使用体波震级或矩震级,这三个震级无法进行对比,但可以通过经验公式换算。在无线电学中,有表面波特征的电磁波也被称为地波,长波即是其中之一,它可以沿着地球表面传播。由于地球表面和大气的折射率不同以及大气中的折射率渐变,低频的电磁波(如长波)会发生衍射,造成了“电磁波没有沿直线传播”的假象。另外,电离层也能反射长波。","text2":"估计地震规模的方法有哪些?","label":1} {"text1":"太的黄鲫(学名:\"\"),为辐鳍鱼纲鲱形目鲱亚目鳀科的其中一种。本鱼广泛分布印度西太平洋区,包括印度、孟加拉湾、斯里兰卡、缅甸、泰国、马来西亚、柬埔寨、越南、中国南海、台湾、印尼苏门答腊、婆罗洲、爪哇岛附近等海域。水深5至50公尺。本鱼体甚扁,背缘窄,腹缘有强锐的棱鳞。口大倾斜,口裂窄长。上颌稍长于下颌。上颌骨细长,不达鳃盖开口。胸鳍第一鳍条延长为丝状,向后达臀鳍起点。体背部青灰色,体侧和腹部银白带有黄色。吻部和头侧中部淡金黄色,鳃盖内面橘黄色。背鳍前方至头顶黑褐色。背鳍和尾鳍金黄色,背鳍末端、尾鳍上缘和后缘灰黑色。腹鳍白色,尖端黄色。体长可达20公分。本鱼近海洄游性鱼类,偶会进入岸边、港湾觅食。食用鱼,但体薄肉少,常晒成鱼干或是当下杂鱼处理。","text2":"太的黄鲫的生活习性是什么?","label":1} {"text1":"牛眼马钱(学名:\"\"),别称狭花马钱、马钱子、牛眼珠、牛眼球、牛眼睛、牛目周及车前树等,为马钱科马钱属植物,分布于中国广西、广东及云南等华南地区,常见于低海拔灌木丛中。在香港,被称之为「香港四大毒草」之一,也是香港原生物种。牛眼马钱是一种木质藤本灌木植物。枝干无毛,小枝上具有与叶对生的螺旋状卷曲钩子,钩子长约2.5-5厘米,老枝有时变为硬刺。叶椭圆形或卵形,顶端钝或短渐尖或急尖,基部阔急尖至圆形或微心形,全缘,对生,长约4-8厘米,宽约2.5-4厘米;叶柄长约4-6毫米;老叶草质;叶基脉3-5条,紧靠边缘的2条叶脉极纤细。花为聚伞花序长于小枝之顶端,花基数为5,长约8-11毫米;花苞片极小,直径约3-4厘米,花萼裂片呈卵状三角形,急尖,披极短的柔毛,长约1毫米,花冠白色或淡黄色,冠管长约4-5毫米,裂片长圆形,外弯,约与冠管等长,冠管喉部及基部披长柔毛,子房无毛,花柱长约8毫米。花期4-6月。果为球型浆果,果皮光滑,表皮呈红色或橙红色,直径约3.5-4厘米,远看形状貌似小桔梗。种子扁圆形,中间凹陷,直径约1-2厘米,1-2颗,果期6-12月、。性味苦、性寒及大毒。全株具毒性,当中果实、种子及木质部毒性最强,树皮及幼叶则次之。毒性成份为番木鳖碱及马钱子碱等生物碱。误服可引致面部肌肉僵硬、全身肌肉痉挛、呼吸困难、散瞳及脊髓瘫痪等症状,严重甚至可引致死亡。茎及种子可入药,外用可通经活络、消肿止痛,主治风湿痺痛、手足麻木及跌打损伤等。","text2":"牛眼马钱生长在什么环境中?","label":1} {"text1":"吴彦晟,是一名中国足球守门员,现效力于中国足球超级联赛球队上海申鑫。2001年,上海中远收购了吴彦晟所在的上海万申青年队。2004年,吴彦晟进入上海国际一线队名单。2005年9月7日,他在2005年中国足协杯四分之一决赛第二回合客场挑战山东鲁能泰山由于主力门将张晨突然生病而得到出场机会,上演职业生涯首秀。11月5日,他在联赛最后一轮上海国际1比3不敌重庆力帆的比赛首发出场,第一次亮相中超联赛。2006年,吴彦晟随球队迁到陕西。2007年,他在主力门将张晨转会上海申花后,逐渐坐稳了主力位置。2008年一度保持着当时联赛最少丢球的纪录,并且用自己稳健的发挥使陕西浐灞成为当赛季的半程冠军。不过在2010年,朱广沪与科萨先后担任球队的主教练后,吴彦晟淡出了球队的主力阵容。2011年,吴彦晟在整个赛季都无球可踢赋闲在家。2012年,吴彦晟加盟上海申鑫。","text2":"吴彦晟加盟上海申鑫是在哪一年?","label":1} {"text1":"查理斯·哈契特(,),生于英国伦敦,化学家,其最重要的成就是发现了铌元素。1801年,在伦敦大英博物馆工作的查理斯·哈契特对博物馆收藏的一份钶铁矿(Columbite)进行了分析。钶铁矿是一种成分非常复杂的矿物,查尔斯发现其中存在一种新的元素。哈契特将这种新元素称作“钶”(,)。11月26日,他在英国皇家学会宣读了《北美一矿物分析含有的一种至今未知的金属》的论文,正式公布他的发现。实际上哈契特发现的是钶钽的混合物。后来该元素被重新发现,并命名为“铌”。为了纪念这位伟大的发现者,巴西矿冶公司-CBMM与其他组织一道设立了查理斯·哈契特(Charles Hatchett)优秀论文奖。","text2":"11月26日,查理斯·哈契特在英国皇家学会宣读了什么论文?","label":1} {"text1":"西环大楼(),曾位于石塘咀皇后大道西450至456G号,于1961年建成,是香港岛西环的一组私人屋苑,共有8幢,自设小社区及商店,石塘咀市政大厦之东边及毗邻港铁香港大学站B1出口。2015年10月,西环大楼已全面清拆,现址将会兴建一幢单幢式37层高,共645伙单位的翰林峰。地段现址分成A座及B座,楼高11层,每幢的1层至10层为住宅,每幢各160个单位,地下除商舖外,亦有14个单位作住宅用途,整个地盘共涉及364份业权。西环大楼前身为西环中华煤气公司4个煤气鼓所在地点,供应全港煤气,1934年5月14日早上其中一个煤气鼓泄漏,起火爆炸,烧至山道加伦台,酿成42死46伤。当时一位巴基斯坦籍的看更没有逃生,勇敢冲入火场,启动紧急装置,把鼓内余下的煤气,通过安全管道排出大海,避免其它煤气鼓波及,化解了更大的灾难。可惜这位巴国职员已经葬身火海。为记念英勇行为,拆卸前的西环大楼第8座地下设有一个小型灵位。恒地经过长达5年的时间收购,终于在2014年11月18日通过强制拍卖统一业权,顺利以底价29.57亿元成交,创下历来最大宗银码强拍个案。恒地执行董事黄浩明表示,日后拟建1幢38层高商住物业,提供600个小型单位,连同建筑费约十多亿元,预算总投资额将达45亿元。2017年4月20日恒基兆业地产营业(一)部总经理林达民表示,西营盘皇后大道西460号(前西环大楼)项目命名为翰林峰(NOVUM WEST),项目分别提供约650伙,间隔由开放式、1房及2房,预计落成日期为2019年9月16日。","text2":"西环大楼现在被哪个集团收购?","label":1} {"text1":"电磁波引发透明(,EIT),一般是用两束光同时照射到原子介质(如大量原子组成的气体),使得其中一束光能够在与原子跃迁共振时通过原子介质而不产生吸收和反射的现象。观测 EIT 需要两种相干光源(例如 激光)和介质(一般为原子气体,如Rb85,Rb87)的三种 量子态. 两种相干光源分别称为\"探测光\"和\"耦合光\",通常耦合光数十倍强于探测光。置\"耦合光\"频率恰等于|2>和|3>能级差,调节\"探测光\"的频率于|1>和|2>能级差自然衰减的数倍左右.EIT 是基于跃迁光在原子能级之间破坏性的相干,和相干原子数量陷阱 (CPT) 现象紧密相连.第一种理论: 将 密度矩阵图像处理二能级系统Rabi 振荡 扩展到三能级. 在此图像下, 概率幅 在不同路径(态与态之间的跃迁)之间的 干涉 而变小, 阻止探测光被吸收. 这里, \"干涉\" 指的是 \"量子事件\" (跃迁) 之间的干涉,绝对不是光学干涉. 举例, 考虑 lambda 能级的情形. 探测光的吸收定义为 formula_1到 formula_2的跃迁. 光场可以驱动原子直接从 formula_1-formula_2 或者从formula_1-formula_2-formula_7-formula_2. 不同路径的概率波幅互损. 如果 formula_7 有较长的寿命(衰变常数),则透明窗口位于 formula_1-formula_2 吸收线内部.第二种理论:\"穿戴态\"图像, 原子能级系统 + 耦合光场 Hamiltonian 对角化且各个态的概率计算建立在新的态下. 在此图像下, EIT 如同Autler-Townes 分裂和穿戴态之间的Fano 干涉的结合. 双峰之间,透明窗户的中心,利用探测光跃迁的概率波幅相互抵消.第三种理论: 极化声子 图像 在慢化光的方面具有重要作用. 这里, 探测光的光子 同步地传输到\"黑暗态声子\",同时也是原子的激发态. 这些\"激子\"存在的时间长度只跟态的衰变常数有光.","text2":"电磁波引发透明(,EIT)是一种什么现象?","label":1} {"text1":"林翠(英文名:Jeanette Lin Chui,),生于上海,籍贯广东中山,本名曾懿贞(Jeanette Tsang Yi Cheng),香港电影红星,著名粤语演员曾江之胞妹,歌手王馨平的母亲。她被香港影坛誉为「学生情人」、「NO小姐」,主演和参演的影片无数,亦正亦邪皆胜任。1949年随家人迁居香港,就读圣士提反女子中学。1953年考入由导演黄卓汉领导的自由影业,林黛正因《翠翠》(1953)名声鹊起。1954年,凭处女作《女儿心》大红,成为台柱明星。与自由影业三年片约期间,陆续主演《终身大事》(1955)、《马车伕之恋》(1956)、《山地姑娘》(1956)、《馥兰姐姐》(1956)、《蔷薇处处开》(1956)。同时外借给其他公司,如艺华影业的《化身姑娘》(1956) ,新天影业的《马路小天使》(1957)、〈流浪儿〉(1958)等。也曾为邵氏拍摄《夜来香》(1957)、《移花接木》(1957)、《千金小姐》(1959)等。1957年11月加盟国际电影懋业(电懋),开启其电影事业的黄金时期,主要作品包括《四千金》、《兰闺风云》(1959)、《豆腐西施》(1959)、《啼笑姻缘》(1964)、《空谷兰》(1966)、《苏小妹》(1967),期间也为邵氏拍摄《金菩萨》(1966)等。国泰电影出品的《游龙戏凤》(1968),是息影前的最后一部作品。她曾经历两次婚姻,第一任丈夫为名导演秦剑,两人于1959年结婚,婚后育有一子陈山河。1967年林翠与秦剑分居,当时香港规定要分居3年才可离婚。1968年8月,林翠为王羽生下王馨平。1969年6月秦剑自杀,同年年底林翠与王羽结婚。林翠与王羽育有王馨平、王加露、王美怡三女,可惜仍于1975年离婚。恢复单身后,创办真纳影业公司,出品唐书璇导演的《十三不搭》(1975) 及自导的《香港式离婚》(1976)。1977年移居美国旧金山,转而从事餐饮及租赁业。1980年末复出影坛,参与电影《海峡两岸》(1988)、《胭脂》(1991)及台湾电视剧《不了情》、《婆媳过招七十回》、《初恋三十年》演出,并有意转往幕后发展。1995年2月22日在台湾台北市突发气喘病殁于家中,享年59岁。","text2":"林翠1949年随家人迁居香港后在哪里就读?","label":1} {"text1":"威廉·杰克逊·胡克(,)是英国植物学家。胡克出生于诺里奇。父亲是一位神学家,精通德国文学,尤其喜好种植稀奇的植物。他在诺里奇中学毕业后,学习了鸟类学和昆虫学,后来开始对植物感兴趣。1809年夏季,他到冰岛考察,搜集了许多标本,但回程时发生了火灾,差点丧命,标本几乎都被毁,但他凭记忆写出了《冰岛记游》,记录了冰岛的植物和居民生活。1814年,他用了9个月时间到法国、瑞士和意大利考察植物,1816年,出版了第一部学术著作《英国叶苔属》,1820年,他受聘担任格拉斯哥大学的植物学教授,第二年,出版了《苏格兰植物》,发起成立了格拉斯哥皇家植物学会和格拉斯哥植物园。1841年,他被任命为皇家植物园首任园长,在他的主持下,皇家植物园从4公顷扩大到30公顷,种植面积达1.1平方千米,建立了许多新温室和一个经济植物博物馆。他的园长职务后来由他的儿子,同样著名的植物学家约瑟夫·道尔顿·胡克接替了。","text2":"什么记录了冰岛的植物和居民生活?","label":1} {"text1":"海勒卡拉国家公园()是美国位于夏威夷州茂宜岛的一个国家公园。公园面积33,265英亩(134.62平方公里)。海勒卡拉(Haleakalā)在夏威夷语是\"太阳之家(house of the sun)\"之意。海勒卡拉国家公园分为两个截然不同的部分:峰顶区和海滨区(Kipahulu)。园区每年平均参观人数为1,450,000人次。()设立于1916年,海勒卡拉国家公园与夏威夷岛的夏威夷火山国家公园(Hawaiʻi Volcanoes National Park)最初是夏威夷国家公园的一部分(夏威夷岛上的冒纳罗亚火山和基拉韦厄火山亦是夏威夷国家公园的一部分)。1961年,夏威夷岛的夏威夷火山国家公园被改设成一个独立的国家公园,1980年,夏威夷火山国家公园并被指定为生物圈保护区。","text2":"这个公园分为两部分,分别是哪两部分?","label":1} {"text1":"陶鹰鼎是仰韶文化陶器,出土于陕西省华县(现渭南市华州区)太平庄。2002年列为中华人民共和国禁止出国(境)展览文物之一,现藏中国国家博物馆。1993年,中国第一次申办奥运,当时在瑞士的洛桑市的奥林匹克博物馆正举行申奥国家文化展,当时国际奥委会主席萨马兰奇亲自于中国北京及西安等地挑选参展文物,最终陶鹰尊与青铜大面具、错金银龙凤青铜方案、金缕玉衣、兵马俑、鎏金银盘、凤冠和皇帝龙袍共七件文物成为参展品。陶鹰鼎通高36厘米,整体结构质朴,周身光洁,未加纹饰。鼎上雕有一只两眼圆睁的猫头鹰,以鹰的前胸为鼎腹,双翅后收构成鼎之中后部,鹰背开有鼎口,鹰足和尾部为鼎足。鹰的双目圆睁,喙部呈勾状,整体造型颇具气势,十分生动。","text2":"陶鹰鼎现在在什么地方?","label":1} {"text1":"尚元(;)是琉球国第二尚氏王朝第五代国王。他是第四代国王尚清王的第二王子。1556年至1572年在位。神号日始按司添()。童名金千代(),一作鹤千代()。据《中山世谱》记载,尚元原是尚清王生前指定的继承人。但1555年尚清王死后,法司葛可昌(城间亲方秀信)、和为美(国头亲方景明)却突然变心,欲拥立尚清王第四子尚鉴心为君。法司毛龙吟(新城亲方安基)提薙刀叱责二人。群臣多有支持毛龙吟者,因此和、葛二人畏惧而不敢复言。于是尚元在继承权争夺之中胜利,并于次年即位。1559年,将和为美流放久米岛,葛可昌流放伊比屋岛。1562年(嘉靖四十一年),明世宗派遣刑科给事中郭汝霖、行人李际春为册封正副册封使封尚元为王。尚元王体弱多病,常常无法正常管理国事。为了保证朝政的正常运行,琉球王府推举了三名法司为其代理。从此琉球王府中法司的人数定例为三名,这三名法司被统称为「三司官」。朝政基本上掌握在毛龙吟等三司官手里,因此尚元王被称人为「哑巴国王」()。 尚元王死后,三司官的制度被保留了下来。尚元王在位期间,琉球与明朝保持良好的外交关系,多次送还漂流至琉球境内的中国船民。1565年(嘉靖44年),倭寇过境琉球,在北山地区为琉将郑都所败。尚元王遣使将被倭寇虏获的中国人口送还明朝。尚元王也曾一度与日本萨摩藩保持良好的关系,然而在1570年至1572年,双方关系出现两次危机。1570年,萨摩岛津贵久的使者雪岑和尚至琉球,要求尚元王向萨摩朝贡,遭琉方拒绝。岛津大怒,煽动奄美大岛酋长发动叛乱,并向奄美派遣军队,给奄美的酋长提供军事上的支持。1571年,尚元率船五十余只御驾亲征,在奄美大岛两次与萨摩军交战,败萨军,俘杀叛变的酋长。归军途中,尚元王病危,三司官马顺德(国头亲方正格)对天祈祷愿代王死,不久尚元王竟果然病愈。尚元王大为感动,封马顺德之子为按司,称国头御殿,世代袭爵。1572年4月1日,45岁的尚元王病逝,次子尚永继位。","text2":"尚元王是哪个国家的国王?","label":1} {"text1":"独龙语,是中国云南省独龙族和怒族独龙支系采用的语言。属汉藏语系藏缅语族侬语支。有人认为接近同语族的景颇语。贡山独龙族怒族自治县独龙江乡的独龙族居民使用独龙江方言,约有4000人;贡山丙中洛乡和西藏自治区察隅县瓦龙一带自称阿侬的怒族居民使用怒江方言,约有6000人。两种方言差异不大,可相互交流,两种方言主要差异是借词来源不同而使语音有所不同。独龙江方言有舌面塞音、擦音和鼻音;怒江方言分清塞音、送气和不送气塞擦音,l 声母读成 n。独龙语有声调,但需要靠声调来分辨词义的词约只有 3%。独龙语原有一种一种拉丁字母形式的拼音文字(简称日汪文),是外国传教士为缅甸境内独龙族支系的日汪人设计。在1983年12月,云南省民语委通过一种与日汪文拼法接近的《独龙语拼音方案》,以独龙江方言为标准,有声母40个、韵母76个、声调3个(但不表示)。声母:韵母","text2":"独龙语,是哪个族采用的语言?","label":1} {"text1":"兔子四部曲,是美国作家约翰·厄普代克的作品。1960年厄普代克出版《兔子,快跑》(Rabbit, Run),书以「兔子」哈利·安斯特朗(Harry 'Rabbit' Angstrom)为主角,是一个二十六岁的年轻人,长得又高又帅,这位兔子有些自恋又脆弱,高中时曾是学校中的篮球明星,曾破纪录投篮得分62分,他的妻子Janice大他两岁,酗酒、不理家事,育有一子Nelson,一女。他结交了篮球教练介绍的妓女Ruth,不久Ruth怀孕,妻子淹死女婴,兔子不知所措,于是匆忙逃走了。1971年厄普代克继续创作有《兔子归来》(Rabbit Redux), 描写1969年7月16日兔子回家后,成为了一个产业工人,妻子Janice跟情人同居,他接连遇上种族对峙、性革命、反文化运动等,兔子还收留了黑人逃犯斯基特。1981年又发表《兔子富了》(Rabbit Is Rich),故事是兔子继承了岳父的日本汽车事业,经济开始富裕,兔子加入乡村俱乐部、打高尔夫球、到加勒比海度假。1990年发表的《兔子安息》(Rabbit at Rest)写兔子年老病患,岛田先生(Mr. Shimada)非常不满意兔子的作为,兔子用心倾听日本老板的严厉批评,儿媳妇Pru因为长期闺房失和,晚上主动要求和他上床,Pru从睡袍口袋里掏出保险套,两度达到性高潮,兔子在事发之后离家出走,前往佛罗里达州,在篮球场试图重温当年风采时,结果因心脏病发作去世。兔子四部曲记录了美国自二战后40年来的社会历史的全貌,描绘出了一幅中产阶级的历史画卷,这些白领阶级有点铁石心肠,共塑造了约一百五十个大小不等的鲜活人物,内容涉及越南战争、登陆月球、女权运动、能源危机,有人称之为「美国断代史」。2000年,厄普代克又写了一部182页的中篇小说《兔子回忆》(“Rabbit Remembered”),交代兔子去世的后续情节,Pru跟Nelson离了婚,带着孩子回Ohio,兔子Ruth所生的女儿Annabelle以护士的身份出现,Nelson看著她想到自己的妹妹,大家共进感恩节晚餐,却因政治话题吵架收场。“兔子”的骨灰罐遗留在一家汽车旅馆的橱架里。本篇故事收录短篇小说集《Licks of Love》,有人说这是第五部曲,亦有人批评是狗尾续貂。","text2":"1960年厄普代克出版什么?","label":1} {"text1":"花旗参蜜,一种以为花旗参及蜂蜜调合而成的饮品,可热饮或冷饮。花旗参:具有二十多种「人参皂」的主要活性成份,对中枢神经系统有镇静和新陈代谢的作用,防止抑压性溃疡、增强胃肠蠕动、抗疲劳、影响内分泌系、及加快糖类、脂肪和蛋白的新陈代谢及合成。并能够滋补中气、增强活力、补肺消热、宁神安睡、开胃健脾、 生津、止渴。蜜糖:一种高浓度的糖液,能产生很高热量,成分包括是单糖、果糖和葡萄糖、有机酸、人体必需微量元素及氨基酸等,并富含多种维他命,不但有助身体排毒,更能补虚润肺燥、滋润喉咙。李时珍在《本草纲目》,指蜜糖能清热、补中、解毒、润燥及止痛五大功效。参蜜早已在香港的餐饮业占了一定的地位。在一般的茶餐厅加少许钱已能尝到;在快餐店、酒楼等亦不难找到参蜜的踪影;甚至在部份cafe及自助餐中也有参蜜提供;而因为参蜜有润喉的功能,在近年出现的烧烤场及卡拉OK场所中参蜜是必然之选来的。一般餐厅都会用蜜糖开水再加几片花旗参冲制参蜜,因此,餐厅中提供的大多是热参蜜。部份餐厅会用平价、较少的花旗参,令冲制出来的参蜜出现没有参味、过甜甚至淡而无味等问题。随著时代发展,近年开始出现参蜜与其他饮品的配搭如柠檬参蜜。","text2":"一般餐厅是如何冲制参蜜的?","label":1} {"text1":"多线天竺鲷(学名:),又称黄体天竺鲷,俗名大目侧仔,为辐鳍鱼纲鲈形目鲈亚目天竺鲷科的其中一种。本鱼分布于印度西太平洋区,包括菲律宾、澳洲西北部、印尼、越南、台湾及新几内亚等海域。该物种的模式产地在雅加达、爪哇。水深5至40公尺。本鱼体延长而侧扁,眼大,口大略下位。鱼体呈淡黄色,稚鱼期体侧具有排列紧密的深色纵纹,成鱼期则渐渐变模糊,尾鳍凹形,各鳍也呈淡黄色,体长可达10公分。本鱼栖息于水质清澈的珊瑚礁、岩礁,白天躲藏于岩架下或岩洞中,夜间出来觅食,属肉食性,以多毛类或其它底栖甲壳类为食。繁殖期时,雄鱼具有口孵习性,卵约7日化成仔鱼,由雄鱼吐出,具短暂的仔鱼飘浮期。可食用,但多做为下杂鱼处理。","text2":"它们的模式产地是哪里?","label":1} {"text1":"哥-{zh-hant:德;zh-hans:特}-体或哥-{zh-hans:德;zh-hant:特}-体是约1150年起至17世纪在西欧广泛使用的字体,特点是非常夸张华丽。在德国,哥特体沿用至二战结束。Fraktur 为哥特体的典型字体,在西方有时指代全部哥特体;有时也称为「老英文字体(Old English)」。数学用的哥特体(Fraktur)字母在 Unicode 的数学字母数字符号区段,分为 U+1D504-1D537 和 U+1D56C-1D59F (粗体)两个区间。字母长s 则收录于字母式符号区段(在 U+017F)。这些字母通常仅用于数学,用来与其他字体的字母形成比对。一般用途的文字选择哥特类字体即可。","text2":"哥特体是什么时期广泛使用的字体?","label":1} {"text1":"冲绳若梅鲷,又称梭地拟乌尾鮗,俗名鸡仔鱼,为辐鳍鱼纲鲈形目鲈亚目笛鲷科的一个种。本鱼分布于印度西太平洋区,包括东非、马达加斯加、模里西斯、葛摩、马尔地夫、塞席尔群岛、红海、斯里兰卡、印度、缅甸、泰国、马来西亚、圣诞岛、琉球群岛、台湾、新加坡、印尼、新几内亚、澳洲、新喀里多尼亚、吉里巴斯、密克罗尼西亚、马绍尔群岛、马里亚纳群岛、东加、吐瓦鲁、帛琉、所罗门群岛、万纳杜、斐济群岛、萨摩亚群岛等海域。水深5至200公尺。本鱼眼大,吻短,大约等于或短于眼径。上,下颌骨等长,胸鳍长,延伸到肛门。体色为淡紫蓝色,尾鳍深分叉,且特别延长,几乎呈丝状。背和尾部鳍褐色略黄,其他鳍白色至半透明。背鳍硬棘10枚,软条9至10枚;臀鳍硬棘3枚,软条8枚。体长可达48公分。栖息在较深的珊瑚礁外缘,常成群活动,以浮游生物为食。食用鱼,味美,清蒸、红烧或做成鱼松均适合。","text2":"此鱼的体长约是多少公分?","label":1} {"text1":"楠型护卫舰(,JMSDF PF Kusu-class)是日本海上自卫队初创时曾使用过的一个护卫舰舰级,刚开始时原归类为「警备舰」。此级船舰的前身是美国海军的(Tacoma-class)巡防舰。1953年时美国出借18艘予日本运用,直到1972年时全级退役。塔科马级巡防舰原是二次大战时期美国海军建造、使用的警戒巡防舰(Patrol frigate)。美方共建造了96艘,战后还有75艘于美军服役,1945年时曾出借28艘给苏联作为对日作战用。在意外沉没1艘后,剩下27艘停留于横须贺由美军控制,直到1950年韩战爆发时又重新翻修后投入。昭和27年(1952年)海上警备队开始筹备成立,美国方面出借了50艘LSSL登陆艇和这18艘警备舰给日本,然而当时针对PF编号是「艇」而不算「舰」还引起过一些争论。本级护卫舰是海上自卫队自战后直到日本国产护卫舰服役前所长期使用的主要战力。昭和40年(1965年)以后,由于过于老旧而逐步除役淘汰,同级舰中最后一艘退役的是昭和47年(1972年)时变更用途成为纪念船的(,JMSDF Kaya PF-288)。在自卫队的舰号编列中,本级舰的编号是延续使用DE级(日本方面称为「乙型护卫舰」,等同于美国海军的等级)的流水编号,但舰类编号并没有改成护卫舰的DD或乙型护卫舰的DE,而是沿用塔科马级在美国海军服役时所使用的PF编号。","text2":"1965年后,楠型护卫舰为何被淘汰?","label":1} {"text1":"丁谓(),字谓之,后更字公言,北宋时期苏州长州(今江苏苏州)人。善言谈,喜欢作诗,于图书、博奕、音律无一不精。出自寇准门下,太宗淳化三年(992年)进士,授大理寺评事、通判饶州事。真宗咸平初除三司户部判官,大中祥符初,权三司使。咸平五年(1012年)任户部侍郎。官至参政知事。当政后极力排斥寇准,干兴元年(1022年)二月,再贬寇准为雷州司户参军。丁谓同党雷允恭因先帝陵寝工程事故,坐“擅移皇堂”罪,丁谓受牵连,贬为太子太保。后以“丁谓前后欺罔”罪,被贬崖州(今海南省琼山县)司户参军。以秘书省监致仕归里。景祐四年(1037年)病卒。著有《丁谓集》8卷、《虎丘集》50卷、《刀笔集》2卷、《青衿集》3卷、《知命集》1卷,皆佚。大中祥符年间,皇宫内失火,丁谓奉命修缮被烧毁的宫殿,但是苦于取土的地方很远。丁谓竟然命令工人挖大街上的土,没几天大街就成了深深的水沟,丁谓又命令工人把汴水引入沟中,再用很多竹筏、木筏与船运送建材,顺著水沟,运到宫中。修完宫殿后,把废弃的瓦砾泥土填到沟里,又填成街道。一举三得,省下了亿万文钱。丁谓机敏有智谋,然有才无德,寇准喝汤时弄污胡须,丁谓立刻帮寇擦胡子,寇准调侃他说:“参政是国家的大臣,竟然会帮长官擦胡子啊!”丁谓怀恨,拜相后排挤寇准,“只要跟寇准要好的,一律贬斥。”丁谓被贬崖州,路过雷州时,寇准派人送一蒸羊在路迎他。寇准还怕自己的仆人找丁寻仇。。有僧妙海者,昔出入丁谓门下,丁谓移住光州时,妙海往见之,丁谓与之相别曰「吾不死,五年当复旧位。」后五年,元昊反,边事起,朝廷更用大臣,丁谓沐浴衣冠,卧佛堂中而薨。尚书省左仆射、资政殿大学士,判郓州事王曾闻丁谓卒,语人曰「斯人智数不可测,在海外犹用诈得还。若不死,数年未必不复用。斯人复用,则天下之不幸,可胜道哉!吾非幸其死也。」","text2":"何时丁谓担任了户部侍郎这个官职?","label":1} {"text1":"中国政府为了奖励在科技进步活动中作出突出贡献的公民,推动中国科技事业的发展,国务院设立五个国家科学技术奖。包括国家最高科学技术奖、国家自然科学奖、国家技术发明奖、国家科学技术进步奖和中华人民共和国国际科学技术合作奖。奖项中规格最高的国家最高科学技术奖每年评审一次,每次选出不超过两名科技成就卓著、社会贡献巨大的公民,由国家主席亲自签署并颁发荣誉证书和高额奖金。每位获奖者的奖金总额均为人民币500万元,其中50万元直接授予个人,另外450万元作为科学研究经费由获奖人全权管理具体用途。国家自然科学奖是授予在数学、物理学、化学、天文学、地球科学、生命科学等基础研究和信息、材料、工程技术等领域的应用基础研究中,阐明自然现象、特征和规律、做出重大科学发现的中国公民的奖项。国家自然科学奖不授予组织。国家自然科学奖设一、二等两个奖励等级。国家自然科学一等奖是自然科学领域的国家最高奖。国家技术发明奖授予在产品、工艺、材料及其系统等有重大技术发明的中国公民。国家技术发明奖不授予组织。国家技术发明奖设一、二等两个奖励等级。国家技术发明奖表彰的是当年中国国民经济中新的、先进的、效益好的新技术。国家科学技术进步奖授予在技术研究、技术开发、技术创新、推广应用先进科学技术成果、促进高新技术产业化,以及完成重大科学技术工程、计划等过程中做出创造性贡献的中国公民和组织。每年国家科学技术进步奖总数不超过400项,分为特等奖,一等奖, 二等奖3个等级。中华人民共和国国际科学技术合作奖是由中国国务院设立的国家科学技术奖励,以奖励对中国科技事业作出贡献的外国人或组织。其前身为国家科学技术委员会设立的中国国际科技合作奖,根据1993年7月经全国人大常委会八届二次会议通过的《中华人民共和国科学技术进步法》的规定,国务院设立“中华人民共和国国际科学技术合作奖”(简称国际科技合作奖)。国家科学技术奖励委员会是对奖励进行评选的机构,设立主任委员,副主任委员,秘书长,委员等职位,一般主任委员,秘书长各一名,副主任委员两名,另外有委员若干名。委员会成员主要是中国科技部,中国科学院,中国工程院及各知名大学的专家学者。","text2":"获奖者奖金及其用途是什么?","label":1} {"text1":"散手,又称散打,散打是两人按照一定的规则,并运用武术中的踢、打、摔等攻防技法制服对方的、徒手对抗的武术项目,它是中国武术的重要组成部分。分为古传散手(杀伤性强)、现代散打(限制较多)。 古传散手作为散打的最早发展要能对抗单人和兵器或多人的格斗,用头、指、掌、拳、肘、肩、膝、腿、胯、臂等部位攻击,主要的技法为打、踢、拿、跌、摔等其中还有肘膝等技法,在格斗中讲究出其不意,不讲究花法只讲究打赢实用。 现代散打就是常见的以直拳、摆拳、抄拳、鞭拳、鞭腿、蹬腿、踹腿、摔法等技法组成的以踢、打、摔结合的攻防技术。 散打没有套路,只有单招和组合,见招拆招。散手比赛一般采用淘汰制,但也可使用循环制进行比赛。比赛场地为高8公寸,长宽各80公寸的木台,台中央有直径12公寸的国际武术联合会会徽。台上铺巧拼垫,台下四周铺有高3公寸,宽20公寸的保护巧拼垫。比赛一般进行3局,采取3局2胜,每局净比赛时间为2分钟,局间休息1分钟,与其他格斗类比赛(柔道、跆拳道、空手道)等类似,在介绍运动员、每局比赛开始、宣布结果和边裁换人时,相应人员都要行礼(抱拳礼),为了保护运动员,运动员必须穿戴包括拳套、护头、护胸、护齿、护裆等在内的完整的护具进行比赛。比赛中禁止用危险动作攻击对方,具体可参见参考资料中规则。符合规则的击中对方的头部、躯干、大腿等有效部位根据难度分别可以得到1分或2分,而若违反规则则根据严重程度判对方得1分或2分,具体得分规则可以参考参考资料中规则。(一)优势胜利(二) 得3分(三)得2分(四)得1分(五)不得分","text2":"散手是什么样的武术项目?","label":1} {"text1":"广州白云足球队是中国广州市昔日的一支足球队,现时广州恒大足球俱乐部前身。其前身是广州市足球一队,于1961年4月组建,1967年停办。1977年10月26日广州市人民政府批准恢复广州市足球运动队,称广州市青年足球队,1978年8月复办。1979年6月国务院发文要求发展足球重点地区,广州列首批重点地区的首位。1980年广州青年队改名为广州市足球队,并参加丙级联赛,获分区赛第2名,晋升乙级,次年再获乙级联赛冠军,进入顶级联赛。1984年10月1日由白云山制药总厂出资赞助每年20万元,与广州市体委合办,改名为广州白云山制药厂体协足球队,成为中国大陆首支由体育部门与企业合办的运动队。1989年广州白云队扩大为足球俱乐部。1993年1月8日广州太阳神集团和广州市体委合作成立中国首家股份制职业足球俱乐部,改名为广州太阳神足球俱乐部。","text2":"中国首家股份制职业足球俱乐部名称是什么?","label":1} {"text1":"东林寺是位于中国江西省九江市庐山西北麓的一座著名佛寺,始建于384年(东晋太元九年),为东林慧远所创。慧远在此创立净土宗。当时名仕谢灵运,钦敬慧远,替他在东林寺中开东西两池,遍种白莲。绍兴六年岳飞曾在东林寺中为母守孝。东林寺正对香炉峰,四周群山环抱,门前为虎溪桥。东林寺在历史上曾经数次被毁。如1645年毁于左良玉军队,1853年再毁于太平军。文化大革命期间又遭到破坏,挪作他用。直到1978年,方被列为江西省文物保护单位,并得到重修。由东林寺投资兴建、高48米的东林寺阿弥陀佛大铜像——东林大佛坐落于庐山脚下,于2000年经国家宗教局批准建造,2012年底完工,是目前最高的阿弥陀佛铜像。","text2":"谢灵运为什么要替慧远在东林寺中开东西两池,遍种白莲?","label":1} {"text1":"国际聚变材料放射测试设施(International Fusion Material Irradiation Facility,缩写:IFMIF),是一项国际科学研究项目,目的是测试核聚变反应堆所用材料的可用性。IFMIF将使用基于粒子加速器的中子源在适当的时间周期内产生大的但合适的中子流,以测试在极端情况下材料的长期行为,这些极端情况类似于反应堆内壁处的情况。IFMIF将由两个平行的加速器构成,每个长约50米,用来产生氘核粒子束。用这些粒子束撞击锂元素组成的标靶后,可得到高能中子,进而照射材料样本和被测试成分。IFMIF的最初策划者是日本、欧盟、美国和俄罗斯,管理者是国际能源委员会(International Energy Agency)。IFMIF的建设准备工作按预期已经在2006年开始,尽管发挥其实际的测试功能至少被排在2017年之后。","text2":"IFMIF的管理者是谁?","label":1} {"text1":"两女一杯(2 Girls 1 Cup)是巴西色情片《饥饿婊子》(\"Hungry Bitches\")的预告片,由MFX Media发行。预告片中包含两嗜粪癖女子相互爱抚,吞吃对方的排遗物,然后呕吐到对方的嘴里的情节。该片在网路通过病毒传播,引起巨大反响,成为网路迷因,在各大网志和留言板上传播。2007年10月中旬左右,YouTube上的反应和声讨达到顶峰。尽管2008年后方才逐渐被禁,该影片仍有传播的余地,甚至部分人特地为此开设站点。2013年1月16日,美国法院以涉嫌拍摄吃屎、人兽交和恋物癖等淫秽影片的罪名,将导演萨克斯判刑4年,全案仍可上诉,其辩护律师戴蒙(Roger Diamond)表示:「他的客户作品明显有别于主流A片,应受美国宪法第一修正案保护。」","text2":"两女一杯是巴西哪部色情片的预告片?","label":1} {"text1":"冲绳若梅鲷,又称梭地拟乌尾鮗,俗名鸡仔鱼,为辐鳍鱼纲鲈形目鲈亚目笛鲷科的一个种。本鱼分布于印度西太平洋区,包括东非、马达加斯加、模里西斯、葛摩、马尔地夫、塞席尔群岛、红海、斯里兰卡、印度、缅甸、泰国、马来西亚、圣诞岛、琉球群岛、台湾、新加坡、印尼、新几内亚、澳洲、新喀里多尼亚、吉里巴斯、密克罗尼西亚、马绍尔群岛、马里亚纳群岛、东加、吐瓦鲁、帛琉、所罗门群岛、万纳杜、斐济群岛、萨摩亚群岛等海域。水深5至200公尺。本鱼眼大,吻短,大约等于或短于眼径。上,下颌骨等长,胸鳍长,延伸到肛门。体色为淡紫蓝色,尾鳍深分叉,且特别延长,几乎呈丝状。背和尾部鳍褐色略黄,其他鳍白色至半透明。背鳍硬棘10枚,软条9至10枚;臀鳍硬棘3枚,软条8枚。体长可达48公分。栖息在较深的珊瑚礁外缘,常成群活动,以浮游生物为食。食用鱼,味美,清蒸、红烧或做成鱼松均适合。","text2":"冲绳若梅鲷身体的颜色是什么?","label":1} {"text1":"晋侯稣编钟是西周的青铜乐器,铸于前9世纪中叶,具体年份有争议,有厉王三十三年、宣王三十三年、宣王十三年等诸说。现多倾向为厉王三十三年。属于晋献侯,出土于山西省曲沃县晋侯墓葬。全套共16件,其中14件为盗墓者盗掘,后由上海博物馆从香港的古玩市场购得,现藏上海博物馆;2件为考古发掘出土,现藏山西博物院。山西省侯马市曲沃县曲村的附近,曾是晋国最早的都城所在地,有九代晋侯及夫人的墓葬。1992年12月,上海博物馆馆长马承源在香港古玩市场中发现此套编钟14件,并根据铭文命名为“晋侯稣钟”。1993年初,北京大学考古学系和山西省考古研究所对古墓进行抢救性发掘,得10数件青铜器,其中2件编钟的铭文亦为凿刻,形制与14件晋侯稣钟相同,铭文也可以连缀,证实这16件编钟原出同墓。编钟大小不一,最大者高52厘米,最小者高22厘米,皆为甬钟。16件编钟上均有铭文,为利器刻凿,刀痕明显,共355字。铭文可以连缀,完整记载晋侯稣受命讨伐夙夷的过程。铭文叙述周王三十三年,周王亲征东国、南国。晋侯稣(左鱼右木)奉命征伐夙夷。晋侯稣率领晋军奋勇作战。周王召见晋侯稣,亲赐马匹、美酒和弓箭。晋侯稣制作这套编钟以弘扬天子美德。晋侯稣钟铭文所记载的战争,史籍中无从查考,对研究西周历史和晋国历史极为重要。此外铭文中的记时历日对西周的断代研究也有重要价值。","text2":"晋侯稣编钟铸于什么时候?","label":1} {"text1":"让-弗朗索瓦·科佩(,),法国政治家,现任人民运动联盟主席。1964年5月5日出生于巴黎西南郊区布洛捏—比昂固赫(Boulogne-Billancourt)市的一个来自罗马尼亚的犹太家庭,毕业于法国国立行政学院(ENA)自由、平等、博爱届(1987-1989),自1995年起任塞纳-马恩省地区首府牟市(Meaux)市长,1997-2002年在巴黎第八大学教经济和金融。2002-2007年出任国务秘书、部长级代表及让-皮埃尔·拉法兰和多米尼克·德维尔潘两届政府的发言人。2007年,尼古拉·萨科齐担任总统后,未进入政府,以其国民议会议员身份竞选为国民议会人民运动联盟(UMP)党派主席。2010年11月17日出任人民运动联盟(UMP)书记。在尼古拉·萨科齐连任总统失败后,他宣布辞去人民运动联盟党主席一职。2012年11月18日的党主席选举中,以微弱优势战胜竞选对手前总理弗朗索瓦·菲永,但后者也宣称以微弱优势战胜对手。次日党内选举活动组织与检查委员会(COCOE)宣布科佩以50.03%的优势获胜,但菲永阵营仍然不承认这一结果。在经过一周的激烈论战与互相攻击后,其党内特况处理国家委员会(CONARE)宣布科佩以 50,28 %的选票胜选。但菲永阵营始终拒绝承认对方获胜的选举结果。12月末他们两个最终达成协议,科佩接受在2013年进行新一轮选举并修改选举章程,菲永则解散他在国民议会成立的党内独立小组,人民运动联盟党政治局也重申了这一决定。","text2":"科佩和菲永最终达成了什么协议?","label":1} {"text1":"乔尔·格里菲斯(Joel Griffiths,)是一名澳大利亚足球运动员,现在效力于澳超球队纽卡素喷射机。1997年,乔尔·格里菲斯在悉尼联开始职业生涯,在2001年加盟纽卡斯尔联喷气机。2003年乔尔首次到国外踢球効力瑞士球队纳沙泰尔,其后加盟利兹联转战英格兰联赛,但只是出场2次。2006年,乔尔回归纽卡斯尔联喷气机効力。2008年被短期租借日本球队福冈黄蜂効力9场进3球。2009年,乔尔被纽卡斯尔联喷射机租借至中国足球超级联赛球队北京国安一年。乔尔在中超联赛第一轮就打入加盟北京国安后的首个入球。2009年4月及10月,乔尔因肘击对方球员、向客队球迷做出了不雅的手势,两次遭到中国足协的处罚,合共停赛12场,罚款8万元。2009年乔尔最终随北京国安队获得中超冠军,并以8个进球和弟弟瑞恩·格里菲斯并列一起成为球队的最佳射手。在租借期满后,北京国安打算让乔尔永久转会到球队,纽卡斯尔联喷气机拒绝放人。乔尔选择了罢赛、罢训方式要求纽卡斯尔联喷气机同意其转会。2010年1月,纽卡斯尔喷射队最终同意了乔尔以45万澳元的身价正式转会北京国安。2011赛季结束后,乔尔拒绝和北京国安续约。2012年1月19日,上海申花官方宣布乔尔·格里菲斯加盟。3月10日,乔尔在2012赛季中超第一轮的比赛中打进一球,这使得他在中超联赛连续四个赛季第一轮都可以取得进球,这个进球也帮助上海申花主场1比1战平江苏舜天。2012年12月13日下午,澳大利亚媒体证实,在中超效力效力了4个赛季的乔尔·格里菲斯已签约澳超悉尼FC。他从德罗巴的搭档,变为皮耶罗的搭档。2013年7月16日,悉尼FC证实已经和乔尔·格里菲斯解约。一周后,他重返中国,加盟另一支中超球队青岛中能。但青岛中能在赛季末降级,未获续约。2014年1月11日,纽卡斯尔喷气机俱乐部正式宣布签下乔尔•格里菲斯,合同期至本赛季结束。这是乔尔在纽卡斯尔喷气机的“三进宫”。格里菲斯的其中一个比他小两岁不过生日同为8月21日的弟弟瑞恩·格里菲斯曾与他一起在北京国安效力两年,此外他还有一个双胞胎兄弟亚当·格里菲斯也是职业球员。2010年6月,亚当·格里菲斯与杭州绿城签约一年,这也使得格里菲斯三兄弟在2010赛季都在中超赛场聚齐。","text2":"乔尔·格里菲斯是哪个国家的足球队员?","label":1} {"text1":"烧录卡是一种用于任天堂DS或Game Boy Advance上的一种存储介质,通过它,可以使NDS或GBA机器对存储芯片内的各种文件进行读取。目前种类上分两种不同规格的烧录卡:一种是插SLOT-2接口(GBA卡槽)的烧录卡,也是早期比较老的烧录卡;另一种是插SLOT-1接口(NDS卡槽)的烧录卡,这类卡推出时间较前者晚,但是由于其可使用存储容量相对于任天堂游戏卡要大的多且能够对数据擦写的TF卡或mini-sd卡,并可在上述存储卡片上直接运行NDS的游戏,因此这类卡在NDS玩家中深受欢迎。目前这类卡包括R4、DSTT、M3DSR、AceKard等。其中中国使用烧录卡的人数要远远多于日本美国等国家","text2":"烧录卡是什么?","label":1} {"text1":"紫城镇是中国广东省河源市紫金县下辖的一个镇,位于紫金县中部,2009年7月经省政府批准由原紫城、乌石、附城三镇合并而成。是县政府驻地,全县政治、经济、文化、交通的中心。辖区总面积384.8平方公里,人口18万多,是省定中心镇并纳入为河源市副中心城区。紫城镇共辖12个居委会:九田、城东、永安、安民、西城、新安、城南、上坝、安良、通惠、附城、乌石,30个行政村:白溪村、城西村、林田村、蓝坑村、中洞村、中埔村、衙前村、水澄村、黄花村、横径村、新庄村、上庄村、鹧鸪村、璜光村、璜坑村、乌石村、石陂村、龙鸣村、荷岗村、南光村、龙湖村、龙潭村、上书村、下书村、澄且村、升车村、升平村、仕贵村、石坑村、榕林村、仙湖村。","text2":"紫城镇是由哪三个镇子组合而成的?","label":1} {"text1":"安德海(),一说名安得海,直隶南皮县(今河北省南皮县)人。宦官,清朝咸丰皇帝、慈禧太后的宠臣。安德海出生年份有多种说法:1837年,1842年,1844年。自幼自阉入宫,成为咸丰帝身边御前太监,人称「小安子」,颇知书,可以读《论语》、《孟子》。咸丰十一年(1861年)发生的辛酉政变中,安德海把遗诏密报慈禧太后那拉氏,深得慈禧太后的欢心,升为总管太监。同治八年(1869年)七月,安德海出京办货,乘上两艘太平船从京杭大运河一路南下,“招摇煽惑,声势赫然”,七月二十日,驶入山东境内,抵鲁北古城德州。山东巡抚丁宝桢以顺治帝祖训「宦竖非经差遣,不许擅出皇城」表示:“宦竖私出,非制。且大臣未闻有命,必诈无疑”为由,命泰安县知县何毓福将安德海与其随从逮捕,派人秘密向慈安太后请示,慈安命军机处回文,内称:“该太监擅离远出,并有种种不法情事,若不从严惩办,何以肃宫禁而儆效尤?著丁宝桢迅速派委干员于所属地方将六品蓝翎安姓太监严密查拿,令随从人等指证确实,毋庸审讯即行就地正法,不准任其狡饰。如该太监闻风折回直境,即著曾国藩饬属一体严拿正法。倘有疏纵,惟该督抚是问。其随从人等,有迹近匪类者,并著严拿,分别惩办,毋庸再行请旨。”八月七日,丁宝桢于济南西门外丁字街(今饮虎池街北段)斩首安德海,暴尸三日,随行二十余人,一律处死。安德海死后,李连英取代其地位。","text2":"安德海做了什么讨得了慈禧太后欢心?","label":1} {"text1":"南雅镇是中国福建省南平市建瓯市南部的一个镇,位于建溪中游,与延平区及顺昌县相邻。《建瓯县志》记载“东汉献帝建安初年,即隶属建安县,因系建瓯南面门户,初名‘南瓦口’。后人因这里的山水相依,四季山青水秀,环境幽雅,因而更其名为南雅。”。南雅镇属中低山地,境内高山有云龙岩、龙岗岩、打四门山、莲花山、老保岗、通天阳、九洞岩、高世林岗等。溪河属建溪流域,主要有高阳溪、集北溪、白沙溪、中湾溪等8条。南雅镇距离建瓯市区23公里,距离南平市区45公里。205国道、横南铁路从南雅镇经过,境内设有火车货车站和浦南高速公路互通口。南雅镇共辖1个居委会,21个行政村,2个镇办场,117个自然村,分别是:四龙居委会、南雅村、新建村、伊村村、新村村、小雅村,十字街村、集瑞村、杉溪村、小康村、叶康村、皇康村、大康村、梅村村、鲁口村、白水源村、房村村、仁墩村、黄园村、山安村、白沙村、太平村。南雅镇的主要产业有林业、茶叶、柑桔、葡萄及锥栗等,素有“柑桔之乡”、“南国葡萄之乡”、“闽北竹乡”之称。竹制品加工、茶叶加工、食用菌加工、绿色食品加工、奶牛饲养为南雅镇的五大龙头产业。","text2":"南雅镇的主要产业有哪些?","label":1} {"text1":"老巴拉克·侯赛因·欧巴马(Barack Hussein Obama, Sr.,),肯尼亚经济学家。卢欧族人。奥巴马的父亲奥尼安戈先生年轻时曾广泛游历,加入英国殖民地军队,旅游欧洲、印度和桑给巴尔。在那里,奥尼安戈先生从罗马天主教改信伊斯兰教,并改了名字侯赛因。在内罗毕他成为一名传教士的厨师和当地的草药师。奥巴马在1959年获得奖学金,到美国的夏威夷大学留学。1961年与美国女子斯坦利·安·邓纳姆在夏威夷结婚,同年儿子巴拉克·奥巴马出生。1962年,奥巴马获得哈佛大学的奖学金,进入该校修读研究生课程。1964年奥巴马与安·邓纳姆离婚,女方取得巴拉克·奥巴马的抚养权。奥巴马未能在哈佛完成博士课程,只取得经济学硕士学位。奥巴马回国后,成为运输部的经济顾问、财政部的高级经济顾问。期间奥巴马发表文章批评政府的计划经济政策。在与乔莫·肯雅塔总统的斗争中失败退出政坛。1982年,他在内罗毕一场车祸中丧生。他的儿子巴拉克·奥巴马是美国第44任总统,也是美国第一位拥有非裔血统(50%)的总统。","text2":"奥巴马发表文章批评政府的什么政策?","label":1} {"text1":"石中玉的父母石清和闵柔,年轻时曾和梅花拳掌门人梅芳姑有一段三角恋情,梅芳姑由爱生恨,在石中玉满周岁不久跟石中坚还没满月时,趁石清外出,到玄素庄攻击闵柔母子,闵柔防备不力痛失石中坚,因此开始溺爱石中玉,导致石中玉逐渐养成顽劣狡猾的性格。后来石清夫妇将他送上雪山派习武,拜风火神龙之称的封万里为师,封万里管教不善,让石中玉企图强奸掌门人白自在的孙女白阿绣。石中玉为躲避门规重罚,当日便逃下山远离雪山派追杀。石中玉偶遇长乐帮,因长乐帮上下无人出任帮主,接受赏善罚恶铜牌令赴侠客岛送死,于是拱他出任长乐帮帮主。石中玉不久也明白长乐帮的用意,于是不告而别(据长乐帮众所言,前帮主不愿独自赴会打算隐居,而石破天将其打倒夺得帮主,作威作福),躲在市井妓院间醉生梦死,逍遥快活。军师贝海石无奈,只得找了忠厚老实但相貌酷似石中玉的石破天来顶替他做长乐帮帮主。赏善罚恶使者查明长乐帮现任帮主石破天非石中玉,于是将石中玉找了出来,石中玉不愿被白万剑等人捉回雪山派,和情人丁珰密谋,骗取石破天的善良要他和自己对调。石中玉重获自由后,回长乐帮时被上门寻找石破天的谢烟客误以为是石破天,对他提出关于玄铁令的要求,石中玉命他歼灭雪山派。但是谢烟客在雪山派见到真正的石破天,得知自己被石中玉欺骗,原本打算将石中玉击毙。石破天不忍闵柔痛失爱子,决定要求谢烟客管教石中玉,同时告诉石中玉得好生伺候谢烟客的饮食起居,从此石中玉就在摩天崖和谢烟客一起生活。","text2":"石中玉明白长乐帮的真实用意后,怎么做的?","label":1} {"text1":"在结构力学上的自由度(Degrees of freedom),或称动不定度(Degrees of kinematic indeterminacy),意指分析结构系统时,有效的结构节点上的未知节点变位数。其中称之为「有效」是因为结构构件上的任一点,都应有机会具有自由度,我们只选择其中对分析整体结构有用的节点变位来讨论,而称为「未知」则因为为求解容易,我们通常尽可能减少自由度的数量,因此扣除已知的变位。自由度大致有两种型式:在平面中,只有三个自由度,一者为面旋转,二者为前后及左右两个移动。在立体中,有六个自由度,三个为前后、上下及左右三个移动和前后、上下及左右三面旋转。简单来说就是沿三个坐标轴的移动和绕三个坐标轴的转动把构建相对于参考系具有独立运动参数的数目称为构件的自由度自由度作为结构力学中的重要概念,是描述一个结构基本情况的基本参数。在结构分析中,将自由度作为主要未知数,基本求解方法有两种:利用变形谐合条件求解的方法,称为力法,此法的应用范围是未知的自由度较少的情况。利用力平衡条件求解的方法,称为位移法,此法应用较为广泛,尤其在求解高阶超静定结构的情况下较力法容易,适合利用线性代数(矩阵)的方式配合程式撰写来求得欲知的自由度。","text2":"结构力学上的自由度是指什么?","label":1} {"text1":"法医昆虫学(),为应用昆虫及其他自然科学的理论与技术,研究并解决司法实践中有关昆虫问题的一门科学。在自然界中,昆虫不仅取食动物的尸体,还在其尸体中活动,促使大量微生物进入尸体,使之加速崩溃。为法医昆虫学一切工作的基础。由宋朝法医学家宋慈在淳祐七年(1247年)成书的法医学案例著作《洗冤集录》含括了已知最古老的法医昆虫学案件。该谋杀案于端平二年(1235年)发生,一名村民被人用刀刺死。仵作们认为死者的创伤是由镰刀造成的,而镰刀是种在收获季节用于割稻的工具,这一事实使他们怀疑同伴农工涉有重嫌。地方县令将村民都聚集在衙门广场,以便暂时收缴镰刀。几分钟内,一大群苍蝇循著肉眼无法识别的血迹气味而来,围著一把镰刀,对其他的不屑一顾。很显然地,该镰刀的所有者便是罪魁祸首,后来该农工被捕快拿下后请求宽恕。","text2":"法医学案例著作《洗冤集录》成书于哪个朝代?","label":1} {"text1":"褐牙鲆(学名:)为牙鲆科牙鲆属的鱼类,俗名比目鱼、偏口、牙片、油牙鲆、沙地、左口。分布于国外达朝鲜、日本及库页岛等海区以及中国自珠江口到鸭绿江口外附近海域等,属于暖温性底层海鱼。该物种的模式产地在日本长崎。形态特征体侧扁,呈长卵圆形。一般体长25~50厘米。体重1500-3000克,大者可达5000克。口大、斜裂,两颌等长,上下颌各具一行尖锐牙齿、尾柄短而高、2只眼睛均在头的左侧,眼球隆起。鳞小,有眼一侧被栉鳞,体呈深褐色并具暗色斑点;无眼一侧被圆鳞,体呈白色。胸鳍稍小;腹鳍基部短、左右对称;尾鳍后缘双截形、侧线明显,在胸鳍上方有一弓状弯曲部。背鳍、臀鳍和尾鳍均有暗色斑纹,胸鳍有暗色点列成横条纹。","text2":"褐牙鲆一般体长为多少厘米?","label":1} {"text1":"《妖怪医生》()是在《周刊少年Magazine》连载的佐藤友生的漫画作品。2006年以『』刊载短篇作品,从2007年开始连载。于《周刊少年Magazine》2008年35号结束连载,续篇于《Magazine Special》连载。外表朴素的高中生护国寺黑郎,在学校被认为是个怪人,而实际上他就是治疗妖怪的妖怪医生。意外知道他秘密的春日琴子不但成为他的朋友,还自愿当他的助手,一起帮助妖怪。在帮助医治妖怪的过程中也增加了许多朋友跟伙伴,黑郎与伙伴们也不断打破妖怪与人类之间的障碍,但同时也有不认同黑郎作法的妖怪或人类,想除掉黑郎等人。这是关于妖怪与人类之间的羁绊的冒险故事。也称为四长老。由一群上级且高年龄的妖怪组成,依四神的称号有学有样,自命为妖界的统治者。很排斥人类,相当傲慢,只对权力有兴趣。","text2":"春日琴子在知道了护国寺黑郎的秘密后,是如何做的?","label":1} {"text1":"手机女孩(又名iPhone女孩,)是富士康在中国深圳工厂生产苹果公司iPhone 3G手机的员工。这个女孩出名是因为MacRumors论坛一个用户名为markm49uk的会员发布有关这个女孩的照片。markm49uk来自英国赫尔河畔金斯顿,他发现他从手机仓库(Carphone Warehouse)买的由富士康深圳工厂生产的一部iphone手机中有这个女孩的很多照片。2008年8月20日,他把这些照片发布到论坛上,数小时内这些图片在全球传开了。在线社区对这个女孩的关注急速上升,国内外都已经有专门为她建立的网站。富士康的一名负责人否认女孩被开除的消息。一位富士康发言人称这个有趣的事件为“一个美丽的过失”。越来越多的人也关注这个女孩的年龄问题。2008年8月25日下午,有记者前往富士康位于深圳观澜园的厂区寻找这个女孩。在质检部门记者大约询问了60多名工人,但没有一个人表示认识或者见过这名女孩。网络推手立二认为:这是一次完美的策划推广。9月5日,一个叫Ebuzzing的网络口碑工作室在天涯社区爆料,该事件是他们为富士康操作,并公布了部分与富士康工作人员的QQ聊天记录。8月20日之后,这个事件在全球主流媒体中快速传播。南华早报报道中称:","text2":"用户名为markm49uk的会员来自哪里?","label":1} {"text1":"BA-6()是苏联在1930年代早期研制的装甲车。BA-6使用的45毫米炮(BT炮塔)和T-26坦克一样,另装备1挺7.62毫米DT机枪于车身中部,一挺DT同轴机枪于炮塔。BA-6重5.1吨,车长4.65米,宽2.1米,高2.2米,乘员4名。BA-6使用美国制GAZ-AAA底盘,这种底盘只适合在公路等良好路况下行驶。这情况后来在增加一对适合路轮时得到一点改善。BA-6外形接近BA-3,但BA-6取消了右后方一扇车门。BA-3装甲过于笨重,而BA-6装甲较薄(10毫米)且性能更好。BA-6最后被更佳的BA-10取代。在30年代早期,BA-6等装甲车可轻易击毁大部份同期装甲车,但其薄薄的装甲还是容易被小口径炮击破。BA-6参加了早期东线的战斗。由于其装甲不足以应付德军火力,所以其侦察角色被T-60坦克及T-70坦克等取代。","text2":"BA-6相比较于BA-3有什么优势?","label":1} {"text1":"《诸病源候论》,又名《巢氏病源》、《巢氏诸病源候论》,共五十-{卷}-,古代中国医学典籍。隋代太医博士巢元方等人于大业六年(公元610年)奉敕所编著,是现存中国第一本病因、病理与证候学专论。全书共分67门,载列证候1739条,包括内、外、妇产、小儿、五官诸证。详细记载它的病因、病理、及证状,又附论导引养生之法,“荟萃精说,沉研精理,形脉证治,罔不该集”,对于中国医学的发展,有很重大的影响。《诸病源候论·卷三十八·无子候》指出夫妇双方有病可导致无子,而不是女方单方面的原因。《诸病源候论》记载的赤虫即姜片虫。它其中又多引用《伤寒论》的条文,为后世提供了另一个了解《伤寒论》原文的途径。但《诸病源候论》也有不少错误,例如在〈九虫候〉中称:“蛲虫在人肠内,变化多端,发动亦能为癣,而癣内实有虫也。”事实上蛲虫跟癣症是两回事。〈九虫候〉又称“肺虫,状如蚕。”又“肺虫令人咳嗽。”“蛔虫,贯心则杀人。”,实属不确。《诸病源候论》有时也涉及迷信,以为女子不孕是风水影响,“妇人无子者,其事有三也。一者坟墓不祀,二者夫妇年命相克,三者夫病妇疹,皆使无子。”。《诸病源候论》多缺佚,王焘所撰《外台秘要》40卷中共引用《诸病源候论》370候,对《诸病源候论》条文的辑佚有很大帮助。后世学者认为《诸病源候论》最大的缺陷就是缺少了治疗的药方。","text2":"《诸病源候论》是哪国的医学典籍?","label":1} {"text1":"巴林国际机场(,),是巴林王国全国唯一的民用机场,位于该国首都麦纳麦东北方向约7公里处的穆哈拉格岛上。它是海湾航空主要的枢纽机场,也是海湾地区第一座国际机场。巴林国际机场与阿布扎比国际机场、素万那普国际机场、多哈国际机场、迪拜国际机场、奥利弗·坦博国际机场、科威特国际机场、伦敦希思罗机场和马德里-巴拉哈斯机场一起被公司评为三星级机场。第一架预定抵达巴林的商务班机是1932年从伦敦飞往德里的名为“汉尼拔”的汉德利·佩季·H·P·42型飞机。这架飞机只搭载了24名乘客,从伦敦出发后用了好几天才抵达目的地,时速100英里。这条航线固定后,巴林建起了海湾地区第一座国际机场。","text2":"巴林国际机场是哪个地方的唯一的民用机场?","label":1} {"text1":"海榄雌科只有一属—海榄雌属,广泛分布于全世界热带和亚热带海岸潮汐地带,是构成红树林的主要树种之一,南半球种类最多。中国有一种,多个变种,分布在南方沿海污泥滩涂带。海榄雌科植物多为灌木,也有的品种为乔木,有气根;叶对生,全缘,常呈灰白色;花小,花萼5裂;花冠4裂;果实为肉质蒴果。具有保持土壤,防止海岸侵蚀的作用,红树林是滩涂动物的重要栖息地。1981年的克朗奎斯特分类法将海榄雌属列入马鞭草科内,属于唇形目,1998年根据基因亲缘关系分类的APG 分类法单独分出一个科,仍然放在唇型目中,2003年经过修订的APG II 分类法不承认有这一个科,将海榄雌属列入爵床科中。","text2":"海榄雌科植物在外观上有什么特点?","label":1} {"text1":"富纳角箱鲀(学名:)为箱鲀科角箱鲀属的鱼类,俗名福氏角箱鲀。分布于印度洋-太平洋海域的热带地区,从东非和南非至台湾岛、日本南部、夏威夷、豪勋爵岛等。该物种的模式产地在莫桑比克。它喜欢珊瑚礁和珊瑚礁外区的沙地或者石地。最深的生活区为130米的深处。它生活在离海底很近的水域。富纳角箱鲀的一个特征是在它背部正中有一根大刺。它的颜色是朱红至淡棕色,占据地盘的雄性是黄色的,带有蓝色的线条。组成装甲似的骨头往往明显显示出是多边形的。体长在23至25厘米间。雄性富纳角箱鲀的地区性非常强烈,它们占据约500平方米的地域。在这个地域上他们可以与至四条雌性富纳角箱鲀一起生活。有时地域上也会有一些未成年的雄性鱼。对富纳角箱鲀的繁殖的研究很彻底。交配在日落后不久的黄昏中。雌鱼和雄鱼并行向上游。雄鱼发出嗡嗡声。在它们游到最上方的时候排卵和精。一些雄鱼也会变色成雌鱼的颜色,游到其它地域里,假如它在那里遇到要排卵的雌鱼,它会变色回到雄鱼的颜色,然后和雌鱼交配。","text2":"雄性富纳角箱鲀的地区性非常强烈,它们占据约多少平方米的地域?","label":1} {"text1":"三棱龙属(学名:\"Trilophosaurus\")是种已灭绝双孔亚纲爬行动物,生存于三叠纪晚期,外表类似蜥蜴,跟主龙类有接近亲缘关系。三棱龙是种草食性动物,身长可达2.5公尺。头骨短而厚重,有宽广平坦的颊齿,表面锐利,可切断坚硬的植物。前上颌骨与下颌前部缺乏牙齿,牠们生前可能有角质覆盖的喙状嘴。三棱龙的头骨也很独特,因为牠们的下颞孔消失了,使牠们看起来像是阔孔类爬行动物的头骨,所以三棱龙类最初跟楯齿龙目、鳍龙超目一起被归类于阔孔亚纲。在1988年,Robert L. Carroll提出下颞孔消失是为了使头骨强化。到目前为止,三棱龙的化石发现于北美与欧洲的晚三叠纪地层。","text2":"三棱龙的头骨独特支出体现在哪里?","label":1} {"text1":"《爱的根源》是香港歌手谭咏麟发行第八张粤语专辑,与《雾之恋》相比,《爱的根源》在风格上来了一个大的变化,全碟共有7首歌为改编歌,改编歌曲亦继续以日文歌为主。专辑中多首歌曲均为流行极品之作,《爱的根源》、《夏日寒风》、《爱在深秋》、《谁可改变》、《捕风的汉子》和《酒红色的心》等首首皆为首本名曲,而《酒红色的心》更同蔡枫华主唱的Kitty Records作品《月蚀》改编自安全地带同一日文作品。专辑封面位于香港九龙的飞鹅山拍摄。《爱的根源》是谭咏麟中前期的巅峰之作,而香港新秀帮李克勤更采用其作品参赛,歌曲流行程度达到全张专辑。在这情况下,他在该年度的颁奖礼上以雷霆万钧之势横扫各大奖项。此专辑销量超过七白金,这销量在当时可说是几乎绝无仅有。这张专辑在歌曲质素和流行元素取得比较好的平衡,成为很多谭咏麟歌迷的至爱,可见经过这专辑后他已成功奠定其乐坛天王霸者地位。而专辑大部分歌曲到现时仍然流行,更有歌手会在公开场合或翻唱专辑中翻唱。","text2":"《爱的根源》是香港歌手谭咏麟发行第几张粤语专辑?","label":1} {"text1":"江西理工大学是一所全日制普通高等院校,位于江西省南昌市和赣州市。创办于1958年,并开始大学本科教育,原名江西冶金学院,1988年改为南方冶金学院,2004年经教育部教学评估,学校更名为现在所使用的“江西理工大学”。2013年6月,江西理工大学成为江西省人民政府、工业和信息化部、教育部共建大学。目前江西理工大学有红旗大道校本部、西校区、应用科学学院、南昌校区4个校区,校园总面积近2500亩,建筑总面积70多万平方米。其中校本部坐落于江西省赣州市红旗大道,位于赣州市中心地带,始建于1958年,后经多次新建,特别是近几年大力发展。校园内绿化覆盖率达60%,绿树成荫、环境优美、景色怡人。学校周围有赣南师范学院科技学院和赣南医学院等一些院校。校本部地址:江西省赣州市红旗大道86号 邮编:341000西校区地址:江西省赣州市章贡区宋城路83号 邮编:341000应用科学学院(独立学院)地址:江西省赣州市客家大道156号 邮编:341000南昌校区地址:江西省南昌市昌北开发区双港东大街1180号 邮编:330013学校建校于1958年,建校之初名为江西冶金学院,隶属于江西省领导。1962年,由江西省改属国家冶金工业部领导。1983年,改为中国有色金属工业总公司直属领导。1986年,经国务院学位委员会会议通过,取得硕士学位授予权。1988年,学校改名为南方冶金学院。1998年9月,实施中央与地方共建,主要为江西省管理。同年,原南昌有色金属工业学校并入,在其址建设为现江西理工大学南昌校区。2000年1月,原江西省商业技工学校并入,在其址新建江西理工大学西校区。2004年5月,通过国家教育部本科教育水平评估,经教育部批准改名为江西理工大学。2013年6月,经国家教育部批准,由江西省人民政府、工业和信息化部、教育部三方共建江西理工大学。世界大学列表 | 中国大学列表","text2":"学校周围有哪些高校?","label":1} {"text1":"于斯纳尔斯贝里市(瑞典文:)是瑞典中部厄勒布鲁省的一个自治市。其治所位于。于斯纳尔斯贝里市境内主要是含有矿脉的火山岩,在东部也有花岗岩。在这些往往布满裂缝的岩层上面则覆盖着一层冰碛。在冰碛上生长有针叶林。在谷地里往往有沼泽。在许多地方采矿把过去的森林砍掉了,在市区的东部还有大片的森林和许多湖泊。考古已经发现一些石器时代的居民点。今天的市区是17世纪里由于矿业的发展形成的,在18世纪里达到顶峰,当时市内有许多矿井和冶炼厂。1635年当地造了一座教堂,教堂内的牧师说芬兰语。一直到20世纪内矿山依然是当地的主要经济部门,最后一座大矿井是1987年关闭的。从1960年开始市区内的矿井逐渐被关闭。到1989年为止还有一座开采白钨矿的矿井,但是在当年也被关闭了。所有矿井关闭后市内最大的企业是一座生产纤维产品的工厂,此外市内经济由一座啤酒厂、小企业和公共设施组成。由于矿山及其关闭市内的病例和失业率均很高。","text2":"一直到20世纪内什么依然是当地的主要经济部门?","label":1} {"text1":"桩(Stub \/ Method Stub)是指用来替换一部分功能的程序段。桩程序可以用来模拟已有程序的行为(比如一个远端机器的过程)或是对将要开发的代码的一种临时替代。因此,打桩技术在程序移植、分布式计算、通用软件开发和测试中用处很大。以下是桩程序的一个例子(伪码):上例中的伪码调用了 ThermometerRead函数,其返回一个温度。由于ThermometerRead需要去读取硬件设备,而这个函数现在还没能开发完成,不能正常工作。ThermometerRead只是简单的返回了一个合理的值,这样主程序就能正常调用这个函数,并继续接下来的开发了。可以注意到,虽然它接受了一个Source类型的参数,表明需要返回的温度是内部还是外部的,实际上并没有对这个参数进行任何使用。桩程序是一段并不执行任何实际功能的程序,只对接受的参数进行声明并返回一个合法值。这个返回值通常只是一个对于调用者来讲可接受的值即可。桩通常用在对一个已有接口的临时替换上,实际的接口程序在未来再对桩程序进行替换。在远程方法调用(RMI)中将客户辅助对象称之为Stub(桩);将服务辅助对象称之为skeleton(骨架)。RMI的过程是:客户对象一旦被调用,客户对象调用stub,stub调用网络远端的skeleton,而skeleton最终调用真正的服务对象。由此,在调用客户对象的时候,感觉上就是直接调用了真正的服务对象。","text2":"RMI的过程是什么?","label":1} {"text1":"天主教康布雷总教区(拉丁文:Archdiocesis Cameracensis)是罗马天主教在法国北部设立的一个总教区,范围相当于北部省的Avesnes-sur-Helpe、康布雷、杜埃、瓦伦谢讷专区。康布雷教区成立于第6世纪,管辖范围包括今比利时。1559年西班牙国王分出梅赫伦总教区和另外11个教区以抵抗宗教改革,康布雷教省的范围大为缩减,附属教区还有Saint Omer教区、图尔奈教区和那慕尔教区。1802年,康布雷失去所有附属教区,自身也附属于巴黎总教区。1817年,教宗和法国国王希望成立里尔教区,遭到康布雷主教Louis de Belmas的激烈反对。1841年,康布雷恢复总教区,附属教区为阿拉斯教区。现任总主教为2000年就任的 François Charles Garnier。2002年,康布雷失去教省地位,转而附属于天主教里尔总教区。","text2":"天主教康布雷总教区范围相当于什么?","label":1} {"text1":"参薯(学名:)为薯蓣科薯蓣属的植物,有一种独特的香草味,肉心呈紫色。与甘薯(\"D. esculenta\")、淮山(\"D. opposita\")、日本山芋(\"D. japonica\")等是近亲。分布在美洲、台湾、太平洋热带岛屿、非洲、东南亚、孟加拉、马来西亚以及中国大陆的福建、云南、贵州、广东、江西、浙江、湖南、四川、广西、湖北等地,生长于海拔200米至1,800米的地区,目前已由人工引种栽培。玻里尼西亚人将原产于东南亚的参薯带至夏威夷,并逐渐在19世纪成为夏威夷的主要农产品之一。大薯、田薯、云饼山药(云南腾冲)、脚板薯(通称)、香芋等。用作食材时,常被称为香芋,例如制作香芋冰淇淋,从西方传至华人地区时被译作「香芋雪糕」。人工培育的品种众多,例如台湾的山药台农1号、山药台农2号、大汕二品系、大汕三品系、中国长品系、二刺品系、紫玉血薯品系(\"D. a.\" var. \"purpurea\")、自然品系、大陆淮山品系等等。本种亦是菲律宾保和省的代表植物,当地每年1月都会举“参薯节”,用以提高参薯的品质,交流新裁种技术,推广新参薯产品等等。当地特产的参薯品种被称为“ubi kinampay”,芳香非常,比一般香芋的味道更浓郁,呈深紫色,用kinampay制成的深紫色菲律宾香芋雪糕,味道非常香浓。在中文中使用“香芋”一名时,本种经常会与芋头混淆,日语中则容易把本种与紫番薯混乱。以下是三者的基本分别:由于香芋的名称和颜色吸引,部份华人在制作芋头批、芋头酥等芋头类食品时,会加入紫薯与芋头混合作为馅料,来模仿参薯的颜色和味道,虽然没有参薯的香味,但却会称作「香芋批」、「香芋酥」。现时部份华人食肆甚至会把芋头直称为「香芋」,例如把芋头西米露称为「香芋西米露」。但这些混用,却使「香芋」一词产生歧义,并产生一些实际问题。","text2":"参薯当做食材的时候,被称之为什么?","label":1} {"text1":"伙伴本作火伴,约定俗成为-{伙}-伴或-{伙}-伴,指一同生活或者共事者,源起于军人同火之意思。此词亦引申解作工作搭档或商业之间的合作关系。此称呼仍然流行于香港纪律部队,俗称「火记」。中国古代军队每十名士兵共用一火煮食,同火者互称火伴;因偏旁类化作-{伙}-伴,沿用至今。而中国大陆将-{伙}-简化归并为-{伙}-,使很多人以为-{伙}-伴才是正写。相关词汇还有「火拼」,原解作内哄,即一「火」的人互斗。火记乃差人(警察)用以互相称呼,是火伴的意思,亦即一同出生入死的火伴。-{伙}-和-{伙}-字均为误写,正写实为火字。另外,广东人同时习惯以记字代替商号或名字全称。例如已故香港歌手罗文,人称罗记;麦当劳餐厅,人称麦记,就是例子了。伙伴或弟兄(,略为「\"battle\"」) 在美军口语是指在中分配的二人组密友,使他们能够互相监测和帮助,避免军人产生失落、自杀或其他的负面情绪,是军队编制中最小的单位, 在加拿大、以色列和新加坡也有类似的用语。","text2":"广东人一般用什么代替商号或名字全称?","label":1} {"text1":"南美貘(学名:'),又名巴西貘或低地貘,是一种貘,在南美洲是最大的野生陆地动物。南美貘呈深褐色,由头顶至颈背有一道短而直立的鬃毛。牠们体长达1.8-2.5米,尾巴长5-10厘米,重230公斤。牠们肩高达77-108厘米。南美貘栖息在南美洲亚马逊雨林及亚玛逊盆地近水的地方。牠们的分布地北临委内瑞拉、哥伦比亚及圭亚那,南至巴西、阿根廷及巴拉圭,西至玻利维亚、秘鲁及厄瓜多尔。南美貘能游善跑,就算在崎岖的山地也能奔走自如。牠们的寿命约25-30岁。在野外,牠们的天敌主要是鳄鱼及大型的猫科,如美洲豹及美洲狮。南美貘当受惊时会走入水中。牠们每个月也会交配。南美貘是草食性的。牠们利用灵活的吻,来吃叶子、芽、嫩枝及细小的树枝。南美貘的数量减少是因被烹吃及失去栖息地所致。牠们一般被认为是濒危物种,但灭绝的可能性则较其他的貘为低。","text2":"南美貘分布在哪儿?","label":1} {"text1":"迪西岛,旧译“度栖岛”、“度西岛”、是一个人迹罕至的珊瑚岛,从1902年开始成为皮特凯恩群岛的一部分,目前无永久居民。迪西岛位于皮特凯恩岛以东540公里处,坐标为,包括潟湖在内一共有3.9平方公里的面积。从东北方向到西南方向大概有2.4公里长,有1.6公里宽。一共有4个小岛围成一个珊瑚岛环:迪西岛有0.7平方公里的陆地面积,70%的面积被银毛树覆盖(这是一种常见于太平洋岛屿的树),这种树能长到6米高。另外1971年有报告说还有另外两种树种,但在1987年没有被找到。这个珊瑚岛环里面水很深而且需要注意有毒的鱼和危险的鲨鱼。葡萄牙水手\"Pedro Fernandes de Queirós\"在1606年1月26日发现了迪西岛并且命名为\"La Encarnación\"。迪西岛在1791年被英国的船长\"Edward Edwards\"重新发现了,他当时正指挥着\"HMS Pandora(1779)号\"奉命搜寻邦蒂号上的叛变船员。迪西岛以皇家海军船长\"Baron Francis Ducie\"的名字命名。美国在1867年根据宣称拥有迪西岛的主权,英国在1902年吞并了迪西岛。经过无线电业余爱好者的努力,迪西岛在2001年11月16日成为了DXCC的一员,并且在Kan Mizoguchi、JA1BK的指挥下在2002年3月使用VP6DI信号进行了第一次探险。在2003年,加拿大无线电业余爱好者在迪西岛建立起了一个使用VP6DI信号的。1821年12月17日的伦敦泰晤士报","text2":"葡萄牙水手发现迪西岛后将它命名为什么?","label":1} {"text1":"赵汇川(),安徽省宿县人,中国人民解放军海军将领。早年曾组织地方武装抗日并负伤。1939年在张爱萍领导下率部开辟皖东北抗日根据地,并由张爱萍改编为新四军,任宿县抗日游击队副司令、新四军6支队11团团长,淮北军区三分区司令等。第二次国共内战中历任华中军区7分区副司令、华东野战军9纵参谋长、淮北挺进支队副司令员,江淮军区参谋长、三野九兵团司令部参谋处长,华东军区海军司令部作战处处长。中华人民共和国成立后,参加创建中国人民解放军海军,任中国人民解放军海军航空学校校长,后任海军一航校校长。1952年赴京筹建海军航空部,是中国人民解放军海军航空兵的创建人之一,任海军航空部参谋长,随后任海军航空部副司令员。次年入伏罗希洛夫海军学院,1957年毕业回国,任海军航空兵部副司令员兼参谋长、北海舰队副司令员。1961年晋升中国人民解放军少将军衔并获二级独立自由勋章、一级解放勋章、二级红旗勋章。在1964年6月11日海军航4师独立5大队使用“照明攻击法”击落台湾P-2V侦察机的战斗中负主要指挥责任。1996年5月27日逝世,享年84岁。","text2":"哪一年赵汇川病逝?","label":1} {"text1":"冲绳海槽(,),狭义说法是位于日本九州琉球群岛西方和台湾本岛东北方之间;广义说法是位于中华民国宜兰县和日本岛根县之间。中华人民共和国以此作为与日本的海上边界,宣称其为中国大陆与日本的海上分界线,但日本从未承认。冲绳海槽是由欧亚板块的两个小板块(华南板块、冲绳板块)的裂张运动形成冲绳海槽西侧及以西由于水深较浅关系,海面呈青绿色。经过东海大陆架的边缘进入东侧琉球界后,由于琉球海沟的岩石圈扩展而形成的弧后盆地,大部分深度逾1,000米,最大深度2,716米,因此海面看来呈黑色,因此明清两代又称黑水沟、黑水洋、分水洋。清代自汪楫始开创了“过沟祭海”制度。历届册封使对冲绳海槽多有记载和题咏:关于郭汝霖「渐有清水,中国山将可望」,日本学者认为此处清水在钓鱼屿西方大陆沿岸关于夏子阳「中国界」,日本学者认为此处中国界在钓鱼屿西方大陆沿岸。关于张学礼「分水洋」,日本学者认为此处分水洋在钓鱼屿西方台湾海峡北部。关于汪楫「中外之界」,日本学者认为汪楫船抵台湾海峡时,船中福建船员和琉球船员发生争执,汪楫一开始采用福建人的偏南航线,第二天最终改用琉球人的偏北航线,其后驶近琉球时,船员告诉汪楫有「中外之界」,当然是琉球船员所告诉的,因此汪楫船中的过郊祭(亦作过沟祭)可推测是琉球人的祭祀活动,并且中外即内外,是琉球国的内外,非中琉的国界。关于徐葆光「海水见绿色,夜过沟祭海神」,日本学者认为此沟在大陆棚浅绿色水中,等待夜间才举行祭祀,不是冲绳海槽。关于周煌「黑水沟」,日本学者认为此处黒水沟无法确定在东在西,且夏子阳早已记载沧水黑水之界离大陆不远,周煌据之,当指钓鱼屿西方的黑潮;周煌自身诗中也说「岂知中外原无界」「相传中外分界处」,意谓中外界只是汪楫所记载的「相传」之谈,原来无法寻觅。关于王文治「黑水之沟」,日本学者认为王文治与册使全魁同航,全魁在钓鱼屿西方记载沟界,王文治所见当与此相同。","text2":"狭义说法冲绳海槽位于哪儿?","label":1} {"text1":"怀仁堂,是中国北京市中南海内主要建筑之一,位于中海西门内,丰泽园东北。清朝光绪11年(1885年),慈禧命醇亲王奕𫍽在此处开始营造仪銮殿,费用主要由各地海关关税中提取,光绪14年(1888年)落成。此后,仪銮殿成为慈禧日常起居之所,由于她才是清朝实际统治者,故仪銮殿也因而取代养心殿,成为清朝实际权力中心。1900年,八国联军攻占燕京,仪銮殿被联军统帅,德国的瓦德西元帅占用,成为联军指挥部,但在1901年4月18日,该殿因为瓦德西所雇中国厨师用火不当,起火焚毁,导致一名德国人丧生。但也有指责此火为德国人所为者。1902年慈禧回都后,耗资500余万两白银对该殿加以重建,改名佛照楼。1908年,慈禧病逝于此。1911年,中华民国成立后,大总统袁世凯将此楼改名为怀仁堂,并在此接见外宾、接受元旦朝贺。袁世凯死后,灵堂设于此处。其后黎元洪、徐世昌也沿用怀仁堂。曹锟就任总统后,将怀仁堂改为眷属居住场所。北洋政府结束后,怀仁堂长期闲置,成为当时北平市政府举办集体婚礼的场所。1949年,中国人民政治协商会议第一届全体会议在怀仁堂召开。中华人民共和国成立后,怀仁堂成为中央政府的礼堂,经常举行各种政治会议和文艺晚会。1953年,为筹备亚太和平会议,周恩来亲自制定了翻修方案,将怀仁堂改建为中式二层楼阁样式礼堂。其后又多次进行翻修。怀仁堂正门位于南侧,进门后为前厅,北接礼堂,可供上千人开会。礼堂东西两侧各有一休息室。礼堂北有一门,进门为怀仁堂正厅,正厅北门通往后花园。","text2":"仪銮殿为何被焚毁?","label":1} {"text1":"冯素弗 (),北燕太祖之长弟。侍中、车骑大将军、录尚书事、辽西公。死于太平七年(415年)。慷慨有大志,姿貌魁伟,雄杰不群,任侠放荡,不修小节。北燕事业,素弗居功厥伟。初为京尹,及镇营丘,百姓歌颂。及为宰辅,谦虚恭慎,非礼不动。虽厮养之贱,皆与之抗礼。车服屋宇,务于俭约,修己率下,百僚惮之。论者归其有宰衡之度。冯素弗在后燕慕容熙属下担任侍御郎,冯跋兄弟因事获罪于后燕天王慕容熙,因此慕容熙有杀冯跋兄弟之意,冯跋兄弟遂逃匿深山。冯跋兄弟商量说:「熙今昏虐,兼忌吾兄弟,既还首无路,不可坐受诛灭。当及时而起,立公侯之业。事若不成,死其晚乎!」于是与从兄冯万泥等二十二人合谋。后燕建始元年(407年)冯跋兄弟乘车,由妇人御,潜入都城龙城(今辽宁朝阳),匿于北部司马孙护家。趁慕容熙送葬苻后之际起事,推高云(慕容云)为燕王,改元正始,不久擒杀慕容熙,冯素弗担任昌黎尹。408年,为司隶校尉。正始三年(409年),高云为宠臣离班、桃仁所杀,乱事平定后,众人推冯跋为主,冯跋遂即位,改元太平。冯素弗被任命为侍中、车骑大将军、录尚书事。1965年考古的发掘了位于辽宁北票西官营子村的冯素弗墓,出土了世界至今最早的马镫实物,以及珍贵艺术品鸭形玻璃注等。","text2":"冯素弗是何时去世的?","label":1} {"text1":"湖南路街道是中国上海市徐汇区下辖的一个街道办事处。,位于徐汇区东北部,东以陕西南路与黄浦区瑞金二路街道相邻,西以兴国路毗邻长宁区,北以长乐路、华山路毗邻静安区,南临复兴中路、淮海中路,面积1.73平方公里,有居委会16个,户籍人口5.34万人(2008年)。办事处设在复兴西路62号。湖南路街道辖区原为上海法租界西部的高级住宅区,至今仍保留700多幢花园住宅,居住此区域的居民中,知名人士、离休干部多。湖南路街道下辖淮中居委会、安福居委会、兴武居委会、金波居委会、武康居委会、春华居委会、华康居委会、复永居委会、新乐居委会、张家弄居委会、复襄居委会、复中居委会、东湖居委会、陕新居委会、淮海居委会、延庆居委会。","text2":"街道办事处设在什么地方?","label":1} {"text1":"《粉红色的火烈鸟》(Pink Flamingos)是一部由约翰·沃特斯执导的美国荒诞幽默电影。当1972年上映时,这部片子因其变态的内容引发了大规模争论,并使它成为了最臭名昭著的非主流电影。影片的主角是一个肥胖的女人,自认为是“世界上最污秽”的人。另一个主角是一对同样变态的夫妇,这对夫妇专门强迫被绑架的妇女受孕,然后将出生的孩子卖给别人。影片就在双方不可避免的冲突中展开,最后,胖女人赢得了胜利,解放了那些被关押的妇女,又枪决了这对夫妇。此片最具争议之处是因为展现了人工授精、口交、吃屎、兽奸等镜头,大大超出了普通观众的心理承受能力。甚至该片在影院放映时会为观众准备呕吐袋。","text2":"电影的结局如何?","label":1} {"text1":"武汉长江隧道位于湖北省武汉市长江一桥和长江二桥之间,设计为左右两条单洞式隧道,隧道为单向两车道。2008年12月28日通车,为中国首条穿越长江的隧道,也是武汉首条连接武昌和汉口的河底隧道。工程包括长江两岸明挖、暗挖隧道、江中盾构隧道、匝道、引道以及设备采购和安装等项目,涉及深坑、大断面泥水平衡盾构掘进(盾构直径为11.38m)浅埋大跨软弱围岩暗挖施工等工程技术。隧道总长5049.2单线米,其中左线长2550m,右线长2499.2m。包括过长江段2610m(左右线均为1305m)。盾构隧道工程左线2550m,右线2499m,在二条盾构隧道之间设置了5条联络通道,左右线隧道各在最低点设置一座江中泵房,盾构隧道穿越富含水的全新统新近沉积粉细砂、全新统沉积粉细砂、中粗砂、砂卵石层以及志留系砂岩泥岩不同的地层。该隧道由铁道第四勘察设计院设计,中铁隧道集团有限公司联合体施工。2004年11月28日,隧道正式动工建造。隧道于2008年4月19日双线贯通,并进入路面铺设阶段,2008年12月28日通车,该隧道是长江上第一条过江隧道。","text2":"武汉长江隧道位于哪两座大桥之间?","label":1} {"text1":"青年公园,位于沈阳市沈河区青年大街中段东侧,为沈阳市南运河带状公园之一,与彩电塔比邻。青年公园始建于1952年,于1958年建成。占地面积29万平方米,其中水面面积14万平方米。公园设有三个出入口,主出入口在青年大街。园内建有多个具有特色的游园小景点。公园正门内为大型水幕式喷泉,在花鱼廊南侧设有假山,假山上安装有电控井抽式自来水管道,开启后可形成宽2.5米,落差6.65米人工瀑布。1984年在东南部的湖心岛上增建了“振兴园”,园内参照中国工农红军长征途径的雪山、草地和铁索桥等地理环境设置了部分景观,作为青少年教育活动的基地,是沈阳市内中小学组织春游的几个主要场所之一。青年公园曾是沈阳市80至90年代元宵冰灯会的主要承办地点之一。市内可乘坐214、238、244路公交到青年公园站点下车。","text2":"青年公园占地面积多少平方米?","label":1} {"text1":"2006年泛美棒球锦标赛(Pan-American Baseball Championship 2006)是指2006年8月25日至9月7日在古巴哈瓦那所举行的泛美棒球锦标赛(Pan-American Baseball Championship),也是2008年夏季奥林匹克运动会棒球比赛的美洲区资格赛(Americas Olympic Qualifier)。本届赛事前两名可获得参加2008年夏季奥林匹克运动会棒球比赛的资格,第三名与第四名则可以获得参加2008年夏季奥运棒球最终资格排名赛的资格。本届赛事前六名可获得参加2007年世界杯棒球赛的资格,本届赛事前七名可以获得2007年泛美运动会棒球比赛的参赛资格。本届赛事共有12个国家参赛,除了主办国古巴之外,其余的11个国家皆由分区进行的资格赛中选出,分别为美国、加拿大、尼加拉瓜、巴拿马、墨西哥、多明尼加、波多黎各、委内瑞拉、巴西、厄瓜多与哥伦比亚。12个参赛国家在预赛先分成两组进行单循环赛,各组前四名可以晋级复赛,复赛亦是单循环赛,以战绩排名。美国获得此届赛事的冠军,与亚军古巴皆获得参加2008年夏季奥林匹克运动会棒球比赛的资格。季军墨西哥与殿军加拿大则取得参加2008年夏季奥运棒球最终资格排名赛的资格。以上四队与委内瑞拉和巴拿马皆获得参加2007年世界杯棒球赛的资格。另外加上尼加拉瓜一起共同获得2007年泛美运动会棒球比赛 的参赛资格。2006年泛美棒球锦标赛资格赛于2005年分成三个地区比赛,分别是北美中美洲、加勒比海地区与南美洲。从参赛的18个国家中选出11个国家参加2006年泛美棒球锦标赛。加勒比海地区的资格赛中,牙买加、美属维京群岛与库拉索弃权。","text2":"2006年泛美棒球锦标赛指的是什么?","label":1} {"text1":"杨林(),是中国足球运动员,身材高大,主要出任高中锋,也可出任中后卫,曾经入选过中国国家队。2002年,杨林加盟甲B球队大连赛德隆。2003年,大连赛德隆被转卖到珠海,更名珠海中邦,杨林随队。2004年,球队又搬到上海,更名为上海中邦,杨林继续作为球队主力。2007年,上海申花与上海联城合并后,杨林转会到了河南建业,表现依然稳定。2009年,杨林从河南建业租借至家乡的大连实德效力,不过被视为中后卫的人选。2010年,杨林加盟新组建的乙级联赛球队大连阿尔滨,前半赛季和联城时代的老队友常琳搭档球队的中后卫,表现稳定,后半赛季顶替离队的徐一文出任球队的高中锋,并攻入3粒进球。2011年,在大连阿尔滨大力引援的情况下,杨林失去了主力位置,不过做一名老将,杨林有效的丰富了球队的板凳深度,继续为阿尔滨队发挥着余热。最后更新:2011年10月30日","text2":"2010年杨林加盟了哪里?","label":1} {"text1":"Astro AOD是马来西亚Astro拥有的最新TVB剧集频道,该频道是由香港电视广播(国际)有限公司(TVBI)与马来西亚Astro合作开办,该频道全天候24小时播出。Astro On Demand剧集首映于2007年7月16日正式启播,该频道的标语是「港剧潮流,顶级享受」,该频道与香港同步播出最新、最热播的香港TVB剧集。此外Astro On Demand HD于2013年6月3日正式启播,该频道备有粤语及华语双声道服务,同时也备有中文及马来语字幕服务。Astro On Demand剧集首映于2017年5月,正式改名为「Astro AOD」。Astro AOD HD - 频道350 (HD)Astro AOD - 频道351 - 353,频道361 - 363 (SD)从2010年起,Astro AOD和MY FM(主办单位)开始与TVB协办“MY AOD我的最爱颁奖典礼”以取代现已停办的Astro华丽台电视剧大奖。此颁奖礼依据前一年11月至当年10月间在Astro AOD播出的剧集,让剧迷们上网投选心目中最爱的艺人和电视剧集,并送出丰富奖品,尤其回馈玉龙配套用户。此颁奖在2013年起改由TVB娱乐新闻台主办、Astro协办,并更名为“TVB 马来西亚星光荟萃颁奖典礼”。下表列出未曾在Astro AOD播出的TVB剧集。","text2":"“MY AOD我的最爱颁奖典礼”取代了什么典礼?","label":1} {"text1":"《王菀之Ivana首张国语创作专辑》为王菀之在2007年推出的首张全国语创作专辑,台湾版发行日期为2007年12月6日,香港版发行日期为2007年12月18日,第二版发行于2008年2月15日。第一版包括一只CD,共收录了十一首新歌及两首Bonus Track。除了四首歌曲为全新创作外,其余歌曲均从王菀之旧作改编。香港版特别收录全新广东歌《奇异恩典》;第二版包括一只CD及一只DVD,DVD中收录了三首新歌的MV。全碟歌曲均由王菀之作曲(《奇异恩典》除外)。全碟由王菀之作曲,除了《奇异恩典》由William Walker作曲及王菀之附加旋律。备注:","text2":"第二版专辑有什么不同?","label":1} {"text1":"《莎华丽雅》(;)该部影由环球影业2007年9月出品,严格来说是第一部产于宝莱坞的美国影片。山杰·李拉·班莎里导演的意图是向大众推介两名新人兰比尔·卡浦尔和松楠·卡浦尔。他们分别是印度著名导演拉吉·卡浦尔的孙子和孙女。电影推出半年后,兰比尔获得印度电影观众奖——最佳新人奖。影片音乐优美,主题曲得到传媒的广泛好评,明星拉妮·穆科吉在影片中演出一支舞曲。沙莱曼·罕也出演一个仅有十句对白的配角。一个年轻人兰比尔,某晚来到某城市红灯区的RK酒吧卖唱。他先遇到一个好心的妓女,后来同一个晚上在小桥上遇到一个美丽而伤心的姑娘。他想和姑娘搭讪,姑娘却跑得远远的。兰比尔根据妓女的提示去寻住处。他找到一所房子里的老太太,好不容易说服老太太让他住下来。当他听说老太太的儿子三十七年前当兵阵亡,他给予老人一点安慰。第二天,他又在同一条小桥上遇到同一个姑娘。渐渐地他们相互信任,姑娘告诉他自己叫莎金娜,是织地毯的。有一天,兰比尔邀请莎金娜到酒吧向她表白对她的爱意,莎金娜却同时向她道出来自己伤心的原因是她去年曾经爱上茵曼,他们相约一年以后在小桥上会面,茵曼却没有出现。她要求兰比尔为她写一封信并交给茵曼。兰比尔不情愿地写下信,随即仍进了火炉,他骗姑娘说寄出了信,也去很多地方但是没有找到茵曼,姑娘渐渐也死心了,和兰比尔发展友谊。就在几天后,茵曼突然出现在小桥上……","text2":"沙莱曼·罕也出演什么角色?","label":1} {"text1":"科克大学(直译为科克大学学院,,)是爱尔兰国立大学的一部分。大学建于1845年,原名“皇后学院”(Queen's College, Cork);1908年爱尔兰大学法案出台后,学校更名为“科克大学学院”(University College, Cork)。1997年大学法案将学校更名为'“爱尔兰国立大学科克”(National University of Ireland, Cork);1998年教育部将大学更名为“科克大学”(University College Cork - National University of Ireland, Cork)。科克大学被星期日泰晤士报选为2003-2004年度爱尔兰大学,2005-2006年和2011-2012年再度当选年度爱尔兰大学。2007年,科克大学在泰晤士高等教育世界大学排名286位,2008年226位,2009年207位。科克女王学院(Queen's College, Cork)乃根据维多利亚女王发布的\"为发展爱尔兰的高等教育\"法案建立。根据该法案,三所学院于1845年10月30日分别在贝尔法斯特(贝尔法斯特女王大学)科克和戈尔韦(爱尔兰国立大学戈尔韦)建立。科克学院1849年正式办学,当时有23位教授及181位学生,一年后便加入爱尔兰女王大学。医学楼建于1860年至1880年,医学院也迅速因其毕业生而建立了名声。爱尔兰最早的两名女医学学生于1898年医学院毕业(20年后牛津大学才开始招收女医学学生)。在随后的一个世纪中,爱尔兰大学法案(1908)成立了爱尔兰国立大学,包括都柏林、科克和戈尔韦的三所成员学院。三所学校也有大学学院的地位,例如“大学学院,科克”。1997年大学法案使大学学院成为爱尔兰国立大学的成员大学,也使成员大学成了全方位的独立大学,但是学位和文凭仍由爱尔兰国立大学颁发。科克大学是爱尔兰的研究型大学,其政府科研经费全国最高。现有艺术、凯尔特研究与社会科学学院、商学与法学院、医学与健康学院、科学、工程和食品科学学院。科克大学还是爱尔兰中国研究院的所在地。学生在中国研究院能够通过艺术和商业学习中国语言和文化。中国研究院最近荣获2008年欧洲语言奖。","text2":"1849年科克大学正式办学时有多少人?","label":1} {"text1":"马蹄大黄(学名:')是一种源于亚洲的蓼科植物。根据《本草》记载,马蹄大黄源产于中国,别名黄良、将军。在中国地区的文献里,「大黄」指的往往是马蹄大黄。为分辨两者,有时会把「马蹄大黄」称为「药用大黄」。马蹄大黄是一种四五尺高的多年生草本蓼科植物,有粗壮的根状茎,茎直立单一,上部有分枝;基出叶很大,丛出,掌状浅裂,茎生叶较小,互生,圆锥花序,分枝展开,夏季开淡绿色或黄白色花,三角形瘦果,有翅。多生于排水良好的山地,分布于中国的湖北、四川、陕西、云南等省,当中以四川出产的最好,陕西次之。马蹄大黄的根和茎是中医常用作通便泻火的药物,「药性峻利」,「性大苦大寒」;此外,马蹄大黄亦有活血的作用。这是由于马蹄大黄含有蒽甙衍生物,其中以番泻甙的泻下作用最强,另还含有大黄鞣质及相关物质,如没食子酸、儿茶精和大黄四聚素等。在东南亚,有研究中医药的学者研究把药用大黄用于治疗乙型肝炎。","text2":"马蹄大黄在夏季时花的颜色是什么?","label":1} {"text1":"黄兆伦 () 别名幽,生于香港,Cosplay爱好者,是一名女性Cosplayer。在香港的Cosplay界十分出名,热爱玩Cosplay已有十多年,2005年创办Cosplay队伍BaLance。有参加香港动漫电玩节(前名:漫画节\/动漫节)的动漫Cosplay大赛(前名:Cosplay扮野大赛),至在2004年开始直到2006年的动漫Cosplay大赛,连续3届赢得Cosplay比赛冠军。并在香港动漫电玩节2008的动漫Cosplay大赛2008,扮演Monster Hunter的银火龙猎人,夺得冠军。她表示,参赛者愈来愈专业,她今次刻意往内地学习舞台技巧,为角色设计特别的舞台效果,结果自04至06年间得到冠军后,在08年的动漫Cosplay大赛再次夺得冠军。在香港动漫电玩节2009的动漫Cosplay大赛2009以扮演Final Fantasy VIIDirge of Cerberus的Genesis再度参战,但最后没有得奖!由她的队友区可儿夺得两项奖项,分别是分组冠军和全场总冠军。2011年至2013年香港动漫电玩节的动漫Cosplay大赛中均获得全场总冠军。","text2":"黄兆伦参加动漫Cosplay大赛获得几届冠军?","label":1} {"text1":"芝加哥哥伦布纪念博览会(World's Columbian Exposition),亦称芝加哥世界博览会,简称芝加哥博览会,是于1893年5月1日至10月3日在美国芝加哥举办的世界博览会,以纪念哥伦布发现新大陆400周年。共有19国参加,2750万人参观。因1871年的芝加哥大火而复兴重建的芝加哥市,于1890年由美国政府指定为博覧会的主办都市。在密西根湖畔,花了近3年兴建67万坪以上的公园,在展示的Court of Honor地区以及娱乐的Midway地区等广大会场建造了约200个建筑物。主会场Court of Honor地区,有美术馆、联邦政府馆、园艺馆、工艺馆、农业馆、机械馆、管理栋等等,为展示美国的繁荣建造了豪华的新古典主义建筑。以美国为中心,展示了各国来的工艺、美术、机械等等主题。除了以红砖造涂漆的美术馆外,其余建筑统一为白色,因夸示白人文明,而被称为「White City」。会场有由小乔治·华盛顿·盖尔·费里斯设计的机械式大摩天轮,直径75.5m,可乘坐2160人。大井浩二,《ホワイト・シティの幻影―シカゴ万国博覧会とアメリカ的想像力》,研究社出版,1993\/12。","text2":"芝加哥哥伦布纪念博览会的举办时间是什么时候?","label":1} {"text1":"溪头龙蜥(学名:),又名牧氏攀蜥,俗称竹虎,旧称牧茂氏攀蜥,为飞蜥科龙蜥属下的物种。仅分布于台湾,正模标本采集自南投县的针叶林中,由日本动物学家牧茂市郎采集。其生存的海拔范围为1500至1500米。体长 10~25 公分,最大可达 27 公分。头部有一条粗黑的过眼线,嘴部外缘与腹部体色较浅。雌雄体色差异甚大,体色会随环境而小幅度改变;雄性最显著特征在于身上有许多粗而黑的斑块(斑块颜色有个体差异),鬣鳞明显;雌性体色会因个体差异而有所不同,虽然大多以绿色为主(有些个体也有与雄性相似的粗斑块,但颜色较接近绿色)但有些个体除了斑块外,自头部经背脊中央至尾部均为暗褐色,形成一条宽纵带,称之为「棕背型」。为树栖性生物,日行性,白天常在森林边缘空旷处活动,且常在树丛面向光的叶面。领域性强,雄蜥遇惊扰时喉部会扩张,并做出类似伏地挺身的姿势,多以小型无脊椎动物为食","text2":"溪头龙蜥多以什么为食?","label":1} {"text1":"马克·雅各布斯(,)生于纽约,是位美国时尚设计师。马克是同名品牌以及副牌的领衔设计师。2013年10月2日LVMH集团主席Bernard Arnault证实与LV合作长达十六年的Marc Jacobs10月约满后将离任,专注发展其个人品牌。雅各布斯的祖母在他幼时教导他如何缝制衣物,并成了影响雅各斯一生最大的人。15岁时,雅各布斯到路易威登在纽约的前卫精品店「胡闹」担任采购员,并认识了佩瑞·艾里斯,雅各布斯曾表示艾里斯「让我觉得很酷、给了我很多希望。」1981年,雅各布斯于纽约艺术与设计高级中学毕业、前往帕森设计学院就读,并于1984年获得佩瑞·艾里斯金环奖、切斯特温伯格金环奖、帕森设计学院的年度设计学生等最高荣誉。2007年,雅各布斯被OUT杂志评价为「美国五十大同性恋男女」。同年3月12日,雅各布斯因毒品及酗酒而接受治疗。雅各布斯是公开的同性恋者,现与巴西广告经理罗伦佐·马尔同(Lorenzo Martone)交往。2009年3月,女性时装日报报导两人将在交往一年后订婚。MARC BY MARC JACOBS主要走青春时尚路线,深受时下青少年欢迎。另外,MARC BY MARC JACOBS在香港亦设有特卖场。","text2":"马克·雅各布斯是公开的同性恋者,他现在的交往对象是谁?","label":1} {"text1":"在代数几何中,一个代数群(或群簇)是一个群是一个代数簇,其簇之乘与逆由正则函数提供。以范畴论描述,一个代数群是一个于代数簇范畴 (数学)中的群对象。在数学中,域formula_1上的代数群有几种等价的描述:可以将代数群设想为李群的代数几何版本,代数群一样有切空间及李代数,却没有指数映射(某些幂零群除外);李群可以表成formula_9-代数群的覆叠空间。代数群的典型例子包括formula_10及椭圆曲线。仿射代数群必可表为formula_10的子群,因此又称线性群。当formula_1是完美域时,Chevalley定理断言:设formula_5为formula_1-代数群,则存在短正合序列在此formula_16是线性群、formula_17是阿贝尔簇。准此,线性群与阿贝尔簇是代数群的基本构件。既非线性亦非阿贝尔簇的典型例子是带奇点的代数曲线之广义雅可比簇formula_18。","text2":"代数群的典型例子有哪些?","label":1} {"text1":"氯化铬(化学式:CrCl)也称三氯化铬、氯化铬(III),是常见的铬(III)化合物之一。有无水物和六水合物两种。无水物为强烈发光的紫色结晶,几乎不溶于水。六水物 formula_1 是一个配合物,有三种水合异构体:formula_2formula_3,分别为紫色、浅绿色和暗绿色的固体。一般买到的都是暗绿色的异构体。三氯化铬晶体中含有连接成层的 CrCl 八面体单元,结构中存在螺旋状位错,不含有金属-金属键。它与三碘化铬同构。氯化铬是较硬的路易斯酸。其中含 d 构型的三价铬,对于配体置换反应来说是惰性的。为了提高其活性,可以加入少量的还原剂(如锌 \/ 盐酸),将其还原为氯化亚铬,很快发生配体交换反应,然后使其与 CrCl 通过氯桥发生电子转移,获得三价铬的配合物,并再生成少量活性的Cr(II),直至所有Cr(III)都发生了取代反应。无水三氯化铬几乎不溶于水,但在还原剂(如锌)存在时会缓慢溶解,原因可能是生成了电荷转移的桥连配合物 [Cr-X-Cr…X]。溶解的产物是紫色的 [Cr(HO)] 离子。如果配体是吡啶,则产物是 [CrCl(CHN)]。Cr(III)的配位数为6的配合物大多都是八面体型的。与碱金属氯化物(如氯化钾)熔融时,三氯化铬生成含八面体型 [CrCl] 离子,以及多聚生成的 CrCl 等离子的盐类。无水三氯化铬可通过单质高温化合制备。或者800°C时,用三氧化二铬与氯气在碳存在下反应也可以得到无水三氯化铬。六水合三氯化铬在650°C时与四氯化碳蒸汽反应,可以得到不含 CrOCl 的无水三氯化铬。也可以用亚硫酰氯脱水。水合三氯化铬可由金属铬与盐酸反应得到。无水三氯化铬是有机金属化学中的重要原料。以它为原料可以制得很多有机铬化合物,比如结构上与二茂铁类似的二苯铬(下图)。三氯化铬也是很多铬(III)配合物的起始原料。有机合成中,CrCl 被原位还原生成的 CrCl 是常用的有机还原试剂之一。它可以(A)将 C-Cl 键还原为 C-H 键,也可以(B)与醛作用,将其还原为烯基卤化物。第二个还原反应中通常用2:1摩尔比的三氯化铬和氢化铝锂。三氯化铬的路易斯酸性可以用于催化某些反应,例如用亚硝基化合物作亲双烯体的狄尔斯-阿尔德反应。","text2":"三氯化铬的路易斯酸性可以用于什么反应?","label":1} {"text1":"库克山(Mount Cook)是纽西兰南岛骨干南阿尔卑斯山脉的最高峰,高3,724公尺,位于南岛中西部坎特伯雷地区。库克山顶部有大量积雪(雪线仅1070米),以巨大冰川著名,最大的塔斯曼冰川(Tasman Glacier)长约30公里,平均宽2公里,景色壮观,现被纽西兰政府将此地域划为库克山国家公园。库克山的正式全名为Aoraki\/Mount Cook,Aoraki是当地原住民的称呼,意为「云之巅」;或译插云峰,库克山之名则是为纪念库克船长,他是最早发现南半球的纽西兰、澳大利亚东海岸的英国人。此外,今日纽西兰北岛和南岛间的海峡,就依他的名字命名为库克海峡。南太平洋中也有一个群岛以他命名为库克群岛。","text2":"库克山最早是被哪国人发现的?","label":1} {"text1":"姜英勋(,),韩国陆军中将、驻外大使、韩国总理,在任日期为1988年12月5日至1990年12月27日,并于1988年12月16日前出任韩国的临时总理。日治朝鲜时期平安北道昌城郡出身。本贯晋州(),号青农()。1936年宁边农业高中毕业,1940年广岛高等师范大学毕业。后留学满州国,考入建国大学经济系。1946年5月1日成为军事英语学校第1期生,并曾参加韩战。1951年起担任韩国国防部经理局长、管理局长,1952年担任韩国驻美国大使馆武官,1954年以少将阶级担任联合参谋本部本部长,1955年任国防部动员助理次官,1956年任陆军本部管理副参谋长,1958年至进修,1959年任。1960起担任陆军官校校长,但因其反对官校生参与支持五一六政变的示威活动,而作为「头号反革命将领」()被关入西大门监狱服刑。1961年获释出狱后以陆军中将阶级退役。1970年前往美国南加州大学留学,获得政治学博士学位。1978年年初进入外务部任外交安保研究员院长,1981年1月13日任兼爱尔兰大使(至1984年)和驻梵蒂冈大使(1985年至1987年)1988年4月26日第13届总选挙中当选国会议员(1988年5月至12月),同年12月16日任第21代国务总理,1990年去职。2016年5月10日逝世。","text2":"姜英勋在1960年起担任了什么职务?","label":1} {"text1":"帕拉克西龙(学名:\"Paluxysaurus\")是种原始巨龙形类恐龙,化石发现于美国德州胡德郡的双子山组地层,年代为下白垩纪的晚阿普第阶到早阿尔比阶,约1亿1200万年前。目前已在当地的尸骨层中,发现至少四个个体的化石。如同其他蜥脚类恐龙,帕拉克西龙是种大型、四足、草食性恐龙,具有小型头部与长颈部。在2012年的一个重新研究,显示帕拉克西龙、波塞东龙是相同动物,较晚被命名的帕拉克西龙成为波塞东龙的一个次异名。帕拉克西龙是在2007年由Peter J. Rose所命名。其学名意为「帕拉克西蜥蜴」,是以德州胡德郡的帕拉克西镇与邻近的帕拉克西河为名。Peter J. Rose并提出一个亲缘分支分类法研究,他认为帕拉克西龙与腕龙共同属于腕龙科演化支。但是腕龙科的分类仍有争论,目前只有一个共有衍征:股骨中段的横向相当宽。无论帕拉克西龙是否属于腕龙科,牠们应该是种原始巨龙形类恐龙。在帕拉克西河流域,经常发现蜥脚类的骨头与足迹化石,它们过去时常被归类于侧空龙,包含一个部份骨骸。这些化石被发现于玫瑰谷组,位于双子山组地层上方。在1980年代中期,德克萨斯大学奥斯汀分校的学生在德州胡德郡的一个牧场,发现一个尸骨层,并挖掘到1987年。在1993年,由南卫理公会大学、福斯沃斯科学和历史博物馆、塔尔顿州立大学组成的团队再度展开挖掘工作。这些新发现的蜥脚类化石似乎是同一种恐龙。当地也发现了矽化木。根据砂岩沉积层研判,当地在下白垩纪曾有河流经过,化石即是从砂岩中挖掘出的。帕拉克西龙的正模标本(编号FWMSH 93B-10-18)是个相连的左上颌骨与鼻骨,以及牙齿。其他的标本还有:七节颈椎、十三节背椎、三十节尾椎,以及除了手部与脚掌以外的一些四肢与肩带骨头。帕拉克西龙与其他蜥脚类恐龙的差异在于脊椎特征;而与下白垩纪北美洲的其他蜥脚类恐龙相比,其他部位的骨头也有形态上的不同处。帕拉克西龙的化石目前仅发现于当地的尸骨层。近年在附近的德州怀斯郡新发现的化石(编号SMU 61732),被归类于侧空龙的未定种(\"Pleurocoelus\" sp.),而非帕拉克西龙。帕拉克西龙与这个侧空龙的新种有不同处,但仍有争论。在2012年的一个重新研究,显示帕拉克西龙、波塞东龙是相同动物,较晚被命名的帕拉克西龙成为波塞东龙的一个次异名。","text2":"帕拉克西龙是怎么被命名的?","label":1} {"text1":"苏保罗(Paul O'Sullivan、),人称「阿苏」、「Paul苏」,是纽西兰出生的练马师,现时于香港从练。苏保罗1976年开始练马事业,其父是纽西兰传奇练马师苏礼云,胞弟苏利云为骑师,他与父亲共同名义训练马匹,其胜出的主要大赛有:1989年凭好利时赢得日本杯、1986年「Our Waverley Star」赢日本富士锦标。此外亦凭滑浪天堂赢得1991年觉士盾。曾胜出澳洲多项一级赛,而在纽西兰,差不多所有大赛都赢过,曾经训练出三匹纽西兰全年度马王以及十一匹不同组别的冠军赛驹。于纽西兰期间,总共有11年成为了练马师冠军。1998年,其父苏礼云光荣退休,苏保罗独立展开其练马事业,夺得2002-2003年度纽西兰冠军练马师。2004-2005年马季开始,苏保罗获得香港赛马会发给练马师牌照,展开于香港的练马事业,家族在纽西兰的事业,便交由胞弟苏利云接管,并于2004年10月30日,凭同欢畅团体名下的「领奖」取得在港从练首场头马。2006-2007年马季成为亚军练马师,2007年凭活力金刚赢得香港打吡大赛,赢得自来港开仓以来第一个分级赛冠军。2010-2011及2011-2012年度,马房成绩陷入低潮,之后成绩渐见起色。2013-2014年度马季,苏保罗训练的「友莹格」取得一季五连捷佳绩。2015年3月29日,苏保罗凭「友莹格」为香港夺得首次高松宫纪念赛冠军,扬威日本。5月17日,苏保罗凭「友莹格」为香港赛驹连续三届胜出KrisFlyer国际短途锦标冠军,扬威新加坡。苏保罗在2014-2015年马季于港夺得39场头马,并位列练马师榜的前六名。2015年11月14日,苏保罗凭「尚华盛甲」取得在港从练的第三百场头马。2015-2016马季成绩突出,取得50场头马位列练马师榜的第五名。","text2":"他主要胜出的大赛有哪些?","label":1} {"text1":"埃及圣鹮(学名:),又名埃及圣朱鹭、埃及圣鹭、圣鹮,是撒哈拉以南非洲、伊拉克东南部及以往埃及的一种朱鹭。牠们在埃及备受尊敬,经常被制成木乃伊当做托特的象征。牠们也被引入到法国、义大利、西班牙及美国。现在,在台湾西部滨海地区也可看到牠们。保守估计全台湾约有1200至1500只的族群量。埃及圣鹮成鸟长68厘米,身体全白,后羽黑色。头部及颈部都是秃及黑色的,喙厚而弯,脚也是黑色的。飞行时双翼张开后边有一道黑色羽毛。雄鸟及雌鸟相似,雏鸟呈灰白色,喙较少,颈上有一些羽毛。埃及圣鹮在树上筑巢,会与其他水鸟(如鹭科)一同生活。牠们很多时会在猴面包树上以树枝筑巢,每次产2-3颗蛋。埃及圣鹮在沼泽湿地及泥泞出没,有时会到访农地及垃圾站。牠们主要吃鱼类、青蛙及其他水中生物,包括昆虫与螺类。埃及圣鹮很静,但有时也会发出一些哇声。引入到南欧的埃及圣鹮急速破坏了燕鸥等动物的栖息地,且与牛背鹭及白鹭竞争筑巢的地方。牠们很能适应环境,在冬天的时候会吃垃圾来补充牠们的食物。埃及圣鹮是《非洲-欧亚大陆迁徙水鸟保护协定》中所保护的物种之一。在古埃及,埃及圣鹮倍受尊崇,且会被制成木乃伊,成为托特的象征。希罗多德及老普林尼都指埃及圣鹮可以对抗蛇。另亦有指只要将埃及圣鹮作为祭牲,就可以杀死带来瘟疫的苍蝇。","text2":"保守估计全台湾约有多少只的族群量?","label":1} {"text1":"副剑齿虎(学名:Paramachairodus)是一种已灭绝的剑齿虎亚科剑齿虎。分布于1500万至900万年前中新世末期的欧洲和亚洲。副剑齿虎是已知最古老的剑齿虎,大量副剑齿虎化石发现于西班牙的马德里附近的一个中新世晚期化石点Cerro Batallones。有两具豹形化石比较出名,一具是大约处在瓦西里期到Turolian期的“Paramachairodus ogygia”,另一具则是完全处在Turolian期“Paramachairodus orientalis”。大多数学者认为发现的第三个物种“Paramachairodus maximiliani”可能和第一具化石是同种的。副剑齿虎肩高约58厘米,与美洲豹相似,但体型更加轻盈。四肢形状显示牠们是灵活的攀爬者,且可以猎杀相对大型的猎物。","text2":"副剑齿虎有什么外貌特征?","label":1} {"text1":"托尼·埃斯唐盖(Tony Estanguet,),法国皮划艇激流回旋男子运动员,曾在2000年夏季奥林匹克运动会、2004年夏季奥林匹克运动会和2012年夏季奥林匹克运动会上夺得单人项目金牌。托尼是皮划艇运动员亨利·埃斯唐盖的儿子,亨利在20世纪70年代赢得世界锦标赛奖牌。他的哥哥,帕特里斯·埃斯唐盖[1996年夏季奥林匹克运动会|1996年亚特兰大奥运会]]赢得了一枚铜牌。托尼·埃斯唐盖共在奥运会上获得3枚C-1项目金牌,分别为2000年、2004年和2012年。2004年的金牌很有争议,裁判判罚米哈尔·马尔季坎2秒的惩罚,使其屈居亚军,其最后成绩仅比落后埃斯唐盖0.12秒。埃斯唐盖是法国在2008年北京夏季奥运会开幕式的旗手。他在C-1半决赛获得第9位,因此被最终淘汰。2012年伦敦奥运会,他成为法国第一位在同一项目获得3枚金牌的运动员。他在皮划艇世界锦标赛一共获得12枚奖牌,其中5枚金牌(C-1:2006年,2009年,2010年;C-1团体:2005年,2007年),6枚银牌(C-1:2003年,2005年,2007年;C-1团体:1997年,2003年,2009年),一枚铜牌(C-1团体:1999年)。2012年,他入选为国际奥委会运动员委员会,他将担任国际奥委会委员,任期为8年。2012年10月30日,他宣布退役。","text2":"托尼·埃斯唐盖在皮划艇世界锦标赛一共获得多少枚奖牌?","label":1} {"text1":"大同钟楼位于山西省大同市旧城清远街中段,建于明代。1951年被拆除。钟楼基座呈正方形。楼高两层,平面三间,檐三重,上层绕以腰檐。顶层为歇山顶,上檐单杪单昂重拱,明间补间铺作一朵,稍间无;腰檐斗栱甚小,为单杪重栱,各间补间铺作两朵;下层为单杪重栱,各间补间铺作一朵。志称钟楼建于明代,根据梁思成考证,建筑手法确为明代风格。钟楼的铁钟号称9999斤(合4999.5公斤),实重3400公斤,铸于明景泰年间。 钟楼拆除后,铁钟保存在华严寺。1966年4月23日,铁钟列入第一批大同市文物保护单位。事实上铁钟属于可移动文物,列入文物保护单位并不合适。2005年国家文物局出版的资料中仍将铁钟记载为市级文物保护单位,但2011年大同市文物局发布的市级文物保护单位列表中去掉了铁钟。2013年元旦,华严寺举办了第一次敲钟迎新活动。据统计,有三千余名市民在2012年12月31日晚至2013年1月1日凌晨参与了敲钟活动。1951年大同市将钟楼拆除后不久,时任中央人民政府副主席宋庆龄来大同考察,得知大同市拆除了钟楼和华严寺的海会殿,批评了大同市政府。尽管如此,大同市之后为了给大同市一医院建设病房,又拆除了明代建筑县文庙大成殿。此事导致大同市再次遭到批评,市政府被迫向中央提交了书面的检讨。1974年,大同市政府又打算拆掉鼓楼,王民选上书市政府,提及拆除钟楼和海会殿被中央批评一事,市政府才作罢。1979年,市政府以影响交通为由再次提出拆除鼓楼,赖时任文化局副局长王民选死保,鼓楼才躲过一劫,得以保存至今。 大同市计划于近年重建钟楼。","text2":"请简要说明钟楼的外形?","label":1} {"text1":"教育路(Kau Yuk Road)是香港新界西元朗市中心的一条主要街道,呈东西走向,全长约700米,整条街道横贯元朗市心脏地带。教育路被山贝河(区内人多俗称为大坑渠)分成东西两部份,西端与元朗体育路相接,东端与大棠路交会,道路中段与多条道路相接,分别为安康路、丰年路、马田路、元朗康乐路和西裕街等。元朗市中心另外两条主要街道分别为青山公路元朗段(惯称元朗大马路)及元朗安宁路,两条均与教育路平行。千色广场坐落教育路东端多年,现已成为区内地标之一。东部街内拥有多所著名食肆(如B仔凉粉)、名商店及娱乐场所。紧接教育路东端的牡丹街、元朗新街等更是区内繁华市集。教育路上部分商店都是24小时营业,故教育路被区内人称为「不夜街」和新界西铜锣湾,也成为年青人的消费集中地,每日可谓车水马龙,与此同时也令街内治安变得较为复杂。相对于东面部分,教育路西面部分人流较稀少,原因是西面较少零售店舗而多装修材料店等与工业相关的店舗。每年的天后诞(农历三月廿三)元朗会举行天后宝诞会景巡游,有舞龙舞狮,教育路是重要路线之一。","text2":"教育路中段与哪些道路相接?","label":1} {"text1":"褐带少女鱼,又称黑尾蝴蝶鱼,俗名大斑马,为辐鳍鱼纲鲈形目蝴蝶鱼科的其中一种。本鱼分布于印度西太平洋区,包括安达曼海、马来西亚、泰国、菲律宾、印尼、中国南海、东海、日本、台湾、越南、新几内亚、所罗门群岛、澳洲、马里亚纳群岛、马绍尔群岛、密克罗尼西亚、帛琉、诺鲁、斐济群岛等海域。水深3至30公尺。本鱼吻尖,但不延长管状。体白色,具黑眼带,体前端有2黑横带,它们在腹鳍上方合而为一,腹鳍黑色。另有一条颜色较淡且较宽的横带起自背鳍鳍条前半部延伸至臀鳍中央,尾柄末端亦有一细黑带。背鳍、臀鳍黄且具白边。幼鱼在背鳍鳍条部中央有一眼状斑,但随成长而消失。鳍硬棘13枚、软条31至33枚;臀鳍硬棘3枚、软条18至22枚。体长可达15公分。本鱼幼鱼常港内或内湾区出现,成鱼则出现在较深的岩礁区。属杂食性,以藻类和底栖小生物为食。为高价值观赏性鱼类,不供食用。","text2":"幼鱼在背鳍鳍条部中央有什么?","label":1} {"text1":"朱慈炫(又作朱慈煊,),明昭宗朱由榔第三子、母孝刚王皇后。永历二年(1648年)出生,出生时受洗成为天主教基督徒,并取教名「当定」(Constantine,今译“君士坦丁”)。这个教名与罗马帝国的君士坦丁大帝相同,永历帝希望借由该皇子的受洗与即位,使大明能如同罗马帝国般,开始转天主教为国教。永历五年(1651年)封为皇太子,成为中国史上第一个改信基督信仰之太子。也是南明最后一位太子。永历十五年(1661年),吴三桂擒朱慈煊,他大骂吴三桂:「逆贼!大明负了你甚么?我父子俩负了你甚么?使我有此下场?」遂于永历十六年(1662年)与其父一同被杀于滇京五华山逼死坡,年十四岁,郑经追谥为哀愍太子。","text2":"朱慈炫是谁?","label":1} {"text1":"桑比亚半岛(俄语:)是俄罗斯加里宁格勒州的一座半岛,位于波罗的海东南沿岸。桑比亚半岛得名于已经消亡的古普鲁士人的分支——桑比安人。桑比亚半岛在德语及日耳曼语族中称为桑兰(Samland),在波兰语和拉丁语中称为桑比亚(Sambia),在立陶宛语中称为森巴(Semba)。桑比亚半岛最初由桑比安人居住,13世纪时该地区被条顿骑士团占领。1243年,桑兰教区划定,成为普鲁士四个教区之一。来自神圣罗马帝国的移民逐渐定居桑比亚,而普鲁士桑比安人则慢慢被同化。古普鲁士语在18世纪初消失,在此之前,桑比亚半岛是使用该种语言的最后一个地区。1525年,条顿骑士团国宣布世俗化,桑比亚半岛成为普鲁士公国的一部分。1701年,霍亨索伦家族建立普鲁士王国。1773年,桑比亚成为东普鲁士省的一部分。1871年,随着德意志统一,桑比亚半岛加入德意志帝国。第一次世界大战后,桑比亚半岛成为魏玛共和国的飞地。1945年第二次世界大战后,桑比亚半岛成为苏联加里宁格勒州的一部分,其名源于附近的城市加里宁格勒,而该地区的德国人也逐渐被驱逐,最终被俄罗斯人和白俄罗斯人取代。卡尔·贝德克尔对桑比亚半岛如此描述:“一个肥沃和森林茂密的地区,有多个湖泊,位于柯尼斯堡以北(a fertile and partly-wooded district, with several lakes, lying to the north of Königsberg)”。半岛最高点位于一处滑雪胜地附近,海拔360英尺。桑比亚半岛拥有两处滨海旅游胜地:泽列诺格拉茨克和斯韦特洛戈尔斯克。桑比亚半岛发现琥珀的历史已经超过一百年,特别是在加里宁格勒附近的沿海地区。1900年,琥珀主要被出口至东方,用来制作烟嘴和工艺品。1918年以前,开采琥珀的权利被霍亨索伦王朝所垄断,去海边的游客被禁止拾取任何物品。据说,有一条古老的贸易路线被称作琥珀之路,从考普和埃尔布隆格附近的特鲁索出发,将琥珀贩卖到黑海及更远的东方。","text2":"尔·贝德克尔对桑比亚半岛如何描述的?","label":1} {"text1":"柯青华,尔雅出版社创办人、发行人,亦为台湾作家,笔名隐地。隐地的代表作《心的挣扎》、《人啊人》、《众生》被称为「人性三书」系列,曾被翻译为韩文出版。籍贯浙江永嘉,1937年生于上海,1947年来台;政工干校毕业后,先后担任过《青溪杂志》、《新文艺月刊》、《书评书目》等杂志的主编,同时从事写作。1975年创办尔雅出版社,任发行人。曾先后发起编辑、出版「年度小说选」、「年度诗选」、「年度文学批评选」等丛书。作品以散文为主,1980年代开始写小说,近年亦有散文、自传、日记等作品问世。隐地也是早期自由句的重要作者,1981年夏天开始创作一句话的作品,集结收录于1984年出版的《心的挣扎》、1987年出版的《人啊人》、1989年出版的《众生》,合此三册称为「人性三书」系列。此外在隐地的诗集《一天里的戏码》,同样散见不少的一句话作品。隐地不仅自己创作一句话,更致力推广一句话,1987年尔雅出版了《十句话》第一集,在七年内陆续出版六集,总计有一百多位作者共集合了1380句话,是白话诗史上第一次「句」对「诗」的集体挑战。1997年荣获中华文艺协会颁发的中华文艺奖章文艺出版奖。2000年获得「年度诗奖」。2003年曾任北一女驻校作家。","text2":"隐地什么时候创办了尔雅出版社?","label":1} {"text1":"大鳞鮻(学名:),也称大鳞鲻、大鳞鲻鱼(台湾学术界使用),俗名塭豆、豆仔鱼、小乌,为辐鳍鱼纲鲻形目鲻科鮻属的其中一种。本鱼分布于印度洋-太平洋区,包括东非、南非、马达加斯加、塞席尔群岛、阿曼、斯里兰卡、巴基斯坦、孟加拉、印度、泰国、日本、中国、越南、缅甸、马来西亚、台湾、新加坡、菲律宾、印尼、澳洲、密克罗尼西亚、帛琉、新喀里多尼亚、萨摩亚群岛、万那杜、法属玻里尼西亚等海域。该物种的模式产地在南非。水深0至20公尺。本鱼体延长,前部圆筒型,后部侧扁。脂性眼睑不甚发达。主上颔骨后端向下弯曲至口角。第一背鳍之起点至吻端之距离,大于由彼至尾基之距离。背面呈青灰色,腹面银白色、胸鳍基部金黄色。背鳍硬棘4至5枚、背鳍软条8至9枚;臀鳍硬棘3枚、臀鳍鳍条8至10枚。一纵列鳞片30至33枚。体长可长60公分。本鱼主要栖息在沙泥底沿岸及河口区,甚至河川下游。亦是吞食底层淤泥,以摄食矽藻及有机碎屑。食用鱼,但没有乌鱼那么饱满的卵,体型也较小,为养殖鱼类。红烧、煮汤均宜,肉较腥。","text2":"大鳞鮻作为食用鱼有什么特点?","label":1} {"text1":"慕容延钊(),字化龙。太原人。出身将门,其父慕容章官至襄州马步军都校。延钊少年以勇敢干练著称,后汉时为枢密使郭威的部下,广顺元年(951年),补西头供奉官,历官尚食副使、铁骑都虞侯。柴荣即位后,为殿前散指挥使都校,显德五年(958年),以功迁殿前副都指挥使、为当淮南节度使。恭帝即位,改镇宁军节度。宋太祖即位,加殿前都点检。建隆二年(961年),改为山南东道节度、西南面兵马都部署。干德元年(963年)一月,宋太祖决定向荆湘地区发动进攻,以延钊为湖南道行营前军都部署、枢密副使李处耘为都监,以讨张文表,出兵湖南,顺便向高继冲借道荆南。不久宋军灭荆南,渡江南下,向潭州(今湖南长沙)进发。这时张文表已经被湖南周保权的属下杨师璠所杀,周保权拒绝宋军入境湖南。慕容延钊分兵两路,水陆并进,陆路出澧州(今湖南澧县),水师东趋岳州(今湖南岳阳)。一月底,水师占领江陵城,二月底,大破武平军于三江口(今岳阳北),斩首四千余级;李处耘由陆路至澧州,楚军望风而遁。三月,占领岳州(今湖南岳阳),三月,攻克朗州(今湖南常德),斩杀张崇富,将其首级悬挂于闹市高杆之上。当时慕容延钊抱病在身,太祖命他“肩舆即戎事。”李处耘与慕容延钊素不和,延钊的士兵一旦有错,李处耘直接杀伐决断,造成延钊一病不起。北宋干德元年(964年)卒。","text2":"为什么太祖命慕容延钊“肩舆即戎事”?","label":1} {"text1":"三色龙胆(学名:)为龙胆科龙胆属的植物,是中国的特有植物。分布于中国大陆的青海、甘肃等地,生长于海拔2,200米至3,200米的地区,多生在湖边漫滩草地、河滩草地、沼泽化草甸、林下以及路边,目前尚未由人工引种栽培特征一年生草本,高3-5厘米,茎紫红色,下部光滑,上部具细乳突,在基部多分枝,枝铺散,斜上升。基生叶大,在花期枯萎,宿存,倒卵圆形,长7-11毫米,宽5-6.5毫米,先端钝圆,具短小尖头,边缘光滑,软骨质,两面光滑,中脉软骨质,在下面突起,叶柄膜质,光滑,长2-3毫米,连合成长0.5毫米的筒;茎生叶对折,疏离,长于或等于节间,矩圆披针形至线状披针形,长4-7毫米,宽1.5-2.5毫米,愈向茎上部叶愈长,先端钝至渐尖,边缘膜质,狭窄,平滑,两面平滑,中脉在下面呈脊状突起,叶柄背面有细乳突或光滑,连合成长2-2.5毫米的筒。花数朵,单生于小枝顶端;花梗紫红色,具细乳突,长2-5.5毫米,藏于上部叶中;花萼倒锥形,长7-8毫米,光滑,裂片披针形或三角形,长2.5-3毫米,先端急尖,边缘膜质,狭窄,平滑,两面光滑,中脉绿色或白色,在背面呈脊状突起,并向萼筒下延,弯缺窄,截形;花冠常闭合,上部淡蓝色或蓝色,下部黄绿色,外面具绿色宽条纹,筒形,长11-13毫米,裂片卵形或卵状椭圆形,长1.2-2毫米,先端钝,具短小尖头,褶卵形,长1.5-2毫米,先端钝,具极短小尖头,全缘或有不整齐细齿;雄蕊着生于冠筒中部,整齐,花丝丝状钻形,长4-4.5毫米,花药直立,稀微弯曲,矩圆形,长约1毫米;子房狭椭圆形,长2.5-3.5毫米,两端渐尖,柄粗壮,长2-2.5毫米,花柱线形,连柱头长约2毫米,柱头2裂,裂片矩圆形。蒴果外露,倒卵状矩圆形,长5.5-6.5毫米,先端钝圆,具宽翅,两侧边缘具狭翅,基部渐狭成柄,柄粗壮,长至15毫米;种子黑褐色,矩圆形,长1.1-1.4毫米,表面具细网纹。花果期6-8月。","text2":"三色龙胆的雄蕊有什么特点?","label":1} {"text1":"民主共和党()是美国建国早期的一个政党。由美国开国元勋汤玛斯·杰佛逊和詹姆斯·麦迪逊在西元1789年创建。民主共和党的对立政党是联邦党(又叫做联邦同盟党)。与联邦党相比,民主共和党更强调各州的权力(States' rights),并且重视自耕农的权益,反对过份的联邦主义和君主主义。主要支持地区是中西部和南部。支持者大部份都是乡村地区的农民还有城市地区的工人,民主共和党可被视为今日美国两大党派中民主党的前身。1789年,美国开国元勋汤玛斯·杰佛逊和詹姆斯·麦迪逊创建美国民主共和党。他们因为反对联邦党的领袖、时任美国财政部长的亚历山大·汉密尔顿的经济政策,退出联邦政府,成为反对党。1796年的美国总统选举,民主共和党领袖汤玛斯·杰佛逊输给了联邦党候选人、时任美国副总统的约翰·亚当斯。但由于汤玛斯·杰佛逊是该次选举得票数第二多的候选人,因而成为副总统。但在随后宪法修正使总统和副总统分开改选,使总统的对手不会成为副总统。1800年的美国总统选举,汤玛斯·杰佛逊击败寻求连任的约翰·亚当斯,终于当选总统,并于西元1804年,获得连任。之后,民主共和党人詹姆斯·麦迪逊和詹姆斯·门罗赢得之后的四次美国总统选举,使得民主共和党连续执政长达24年,也使得主要竞争对手联邦党瓦解。1824年的美国总统选举,由于民主共和党未能推派单一候选人,四名民主共和党人同时参选,最终普选票和选举人票为四人中最多的安德鲁·杰克逊在众议院选举中未能当选,而是由亨利·克莱支持的约翰·昆西·亚当斯当选总统,而亨利·克莱则获约翰·昆西·亚当斯任命为国务卿,此一选举结果导致民主共和党瓦解。1828年和1832年的美国总统选举,分别由约翰·昆西·亚当斯、亨利·克莱与安德鲁·杰克逊对垒,约翰·昆西·亚当斯和亨利·克莱的支持阵营,被称为国民共和党,而安德鲁·杰克逊的支持阵营,以老共和党人(Old Republicans)为主,被称为民主党,今日的民主党以民主共和党的成立作为创党年份。","text2":"汤玛斯·杰佛逊是何时当选的总统?","label":1} {"text1":"恒安国际有限公司(),简称恒安国际、恒安集团,是中国最大的妇女卫生巾和婴儿纸尿裤生产商,从事生产及销售「安尔乐」、「心相印」、「安乐」、「安儿乐」、「安而康」等品牌的妇幼卫生用品和生活用纸。公司在1985年由主席施文博先生及副主席兼行政总裁许连捷先生共同创办,1998年在香港交易所上市。总部设在福建晋江。2011年6月7日,恒安国际与友邦保险获纳入恒生指数成份股。恒安国际发行54.34亿元零息可换股债券,于2018年到期,换股价每股120.0825元,较2013年5月21日收市价88.75元溢价达35.3%,倘悉数换股共逾4,525万股,占扩大后股本约3.5%。集资用作还债及资本开支等。","text2":"恒安国际是中国最大的什么生产商?","label":1} {"text1":"Syfy(原名Sci-Fi 频道)是美国国家广播公司(NBC)的一个有线电视频道,于1992年9月24日开播。Syfy专门播放科幻、奇幻、惊悚、超自然等电视影集,制作播放的著名影集包括《异形庇护所》(\"Sanctuary\")、《第十三号仓库》(\"Warehouse 13\")、2004年版本的《太空堡垒卡拉狄加》(\"Battlestar Galactica\")、《星际之门》(\"Stargate\")系列影集等等,亦有播出世界摔角娱乐所属节目SmackDown。Syfy一名于2009年7月7日开始采用。1989年,佛罗里达州博卡拉顿,沟通律师Mitchell Rubenstein和他的妻子Laurie Silvers想出了科幻频道的概念,并计划在1990年12月开始广播,但缺乏资源来实施。1992年3月,这个概念被收购,当时是派拉蒙电影与环球影业之间的合资公司。该频道被视为与经典电影和电视剧天生配合,这此作品在其两个电影制作公司的保险库中,包括环球影业的《'》、《科学怪人》和的电视剧《'》,以及派拉蒙电影的《星际争霸战》。《星际争霸战》的创作者金·罗丹贝利和作家以撒·艾西莫夫是最初的咨询委员会成员之一,但是在1992年9月24日终于启播的时候,他们都已经去世了。Rubenstein回忆说:「在放映上做的第一件事是『献给纪念以撒·艾西莫夫和金·罗丹贝利』。」伦纳德·尼莫伊是在曼哈顿的海登天文馆(Hayden Planetarium)举行该频道启播派对上的仪式主持。艾西莫夫的遗孀和罗丹贝利的遗孀都双双出席。电视网上广播的第一个节目是电影《星球大战》(1977年)。1994年,派拉蒙电影被出售给,然后在下年公司收购了松下电器产业株式会社所控制的股份公司(其中环球影业是一家子公司)。1997年,Viacom将其在USA电视台的股份出售给环球影业,环球影业在下一年便将所有的电视资产剥离至。三年后,迪勒将这些资产卖回环球影业, 当时是威望迪公司的子公司(当时被称为威望迪环球)。威望迪公司的电影和电视制作和有线电视资产随后在2004年与通用电气的全国广播公司(NBC)合并成NBC环球。频道的高清版本于2007年10月3日在DirecTV上推出。在2013年,Syfy被授予的。","text2":"《星际争霸战》的创作者是谁?","label":1} {"text1":"金花蛇属(学名:\"Chrysopelea\")是蛇亚目游蛇科下的一个蛇属,又称为飞蛇,此蛇分泌毒性较轻的毒素,对人类甚少会造成重大威胁,但始终仍是毒蛇成员之一。金花蛇主要集中分布于东南亚、美拉尼西亚群岛及印度。目前共有5个物种已被确认。金花蛇是树栖性蛇类,擅长攀爬树木;平日多于日间活动,主要捕食鼠类、鸟类及蜥蜴等。属卵生蛇类。金花蛇俗称「飞蛇」,主要是因为牠们会于高处地区弹跳穿梭,并在半空中作出类似飞翔的动作,因而得名。事实上此蛇当然不懂得飞行,牠们只是利用身体肌肉的摆动,在空中作出短距离降落式的滑翔而已。当牠们要进行「飞翔」时,牠们会先爬行到高处,压缩肌肉将身体压得扁平(其身体宽度甚至可达身体水平高度的两倍),借此加强其降落时的空气阻力,再将身体弹出,并滑翔至其目的地处。即使没有翅膀之类的滑翔辅助器官或肢体,金花蛇的滑翔技巧仍可媲美鼯鼠或其他擅长滑翔的动物,甚至有更佳的表现。金花蛇能以弹道学的原理,准确到达其滑翔的目的地;牠们亦能在身处空中时,以摆动身体的方式,稍控制其飞行方向。根据芝加哥大学最近的一项科学研究认为,体型较短小的金花蛇,可以作出更远的飞翔距离,故此得出总结:金花蛇的体型,与其飞行距离,是反比例关系。金花蛇属目前共有5个物种已被确认,分别是:在上述5个物种中,首三种金花蛇较常被视为金花蛇属的成员。而摩鹿加金花蛇及印度金花蛇则由于物种本身缺乏清晰资讯,因此其从属问题尚有争议的空间。","text2":"金花蛇的滑翔技巧利用了什么原理?","label":1} {"text1":"凯特·维多莉亚·“凯蒂”·彤丝朵(\"Kate Victoria \"KT\" Tunstall\",),是一位苏格兰的创作歌手。她以在英国BBC电视台的音乐节目「裘斯·荷兰的晚间秀」(\"Later ... with Jools Holland\")上一首现场独唱的自创曲黑马与樱桃树(\"Black Horse and the Cherry Tree\")首度在观众面前亮相。凯蒂·彤丝朵至今在评论界享有不错的评价,并曾获全英音乐奖及葛莱美奖提名。她的外祖母是华人。凯蒂选择以同音的「KT」代替本名「Katie」作为她的名字。她说,「『Kate』总让我联想到一个娇媚的小姑娘,在农田里为她的男人烤著面包。我对那没有任何偏见,我只是觉得,那不是我所想像作为一位摇滚明星的样子。」同时,也与歌手凯蒂·玛露(\"Katie Melua\")做个区隔。2005年,彤丝朵在一些粉丝把她与创作歌手蒂朵(\"Dido\")的音乐做比较后,因公然抨击蒂朵「她X的根本不会唱歌」而惹了一些争议。凯蒂后来道了歉,并表示她不想因此卷入公开的争执。彤丝朵并否认关于她是同性恋的传言,同时澄清她首张专辑封面上的彩虹吊袜带并不是代表同志的身分,她当时对此象征并不知情。2007年4月,彤丝朵动了外科手术,来矫正她因童年感染,而尺寸较一般人小的肾脏的位置。2003年,彤丝朵开始与她乐团里的鼓手路克·布伦交往。2007年的圣诞节,布伦在彤丝朵位于苏格兰圣安德鲁斯的老家向她求婚,两人并于2008年9月6日于苏格兰斯凯岛的弗洛地格瑞乡村别墅旅馆结婚。","text2":"凯特·维多莉亚·“凯蒂”·彤丝朵首度在观众面前亮相是在何时?","label":1} {"text1":"《第一季单车二势力》为2008年台湾年代MUCH台所制播的户外活动性节目,主持人为黑涩会美眉瑶瑶以及演艺圈新进MIA两位台湾艺人共同主持。该节目以主持人骑乘单车造访台湾各处知名景点,透过活泼风趣介绍当地有趣的人事物。《第二季单车二势力》全台湾走透透、吃透透、玩透透,此季单车二势力以校园周边美食为主轴,每集由主持人MIA及PARTY率领两所大专院校学生推荐的周边美食来PK,在介绍最好吃美食的同时,在节目中还加入了两校学生推荐PK的元素,让节目不只是提供吃喝玩乐讯息,还有竞赛性的趣味,增加内容张力。同时,用学生推荐美食也可以增加节目的互动性,拉近与一般观众的距离,节目也不将再是纯粹的单向介绍美食给观众。","text2":"《第一季单车二势力》的主持人是谁?","label":1} {"text1":"兰大卫(,),暱称老兰医师,是一位出身苏格兰的医师及长老教会传教士,亦是彰化基督教医院创建人之一。长年在彰化地区行医,使当地产生一句俗谚:「南门妈祖宫,西门兰医生」。其子乃兰大弼医师。1870年8月2日,兰大卫出生于联合王国苏格兰。1895年,他毕业于爱丁堡大学医学院,之后在台展开长达40年的医疗传道工作,他经常和梅监雾牧师一起工作。 此外,兰大卫亦致力于培养台湾本土西医。1928年,一名13岁少年周金耀腿部溃疡,无法长皮,兰大卫将妻子连玛玉(Miss Marjorie Learner)四块腿皮移植予周金耀,后虽因排斥而手术失败,在施行自体植皮手术及悉心照顾下仍逐渐痊愈。1936年,兰大卫退休返回英国。1957年10月,兰氏因车祸去世。","text2":"兰大卫常年在哪里行医?","label":1} {"text1":"广州府,明清时广东省的府。清代隶广肇罗道。元朝时为广州路,明朝洪武十一年(1378年),改广州府。下领一州:连州(领阳山县、连山县),十五县:南海县、番禺县、顺德县、东莞县、新安县、三水县、增城县、龙门县、香山县、新会县、新宁县、从化县、清远县、阳山县、连山县。治所在南海县、番禺县。清朝时,两广总督旧驻肇庆,乾隆十一年徙。光绪二十四年裁巡抚,寻复。三十一年,仍与粤海关监督、粮道同裁。布政、提学、提法、盐运四司,巡警、劝业二道,广州将军,满洲、汉军副都统,广东水师提督驻。明领县十三。康熙中增置花县。领县十四:南海县、番禺县、顺德县、东莞县、从化县、龙门县、新宁县、增城县、香山县、新会县、三水县、清远县、新安县、花县。1912年中华民国成立后废州府,成立广东军政府。","text2":"广州路在什么时间改为广州府?","label":1} {"text1":"在广义相对论中,裸奇异点()是一种理论推测出的重力奇异点,其外没有事件视界包围住。一个黑洞是由重力奇异点与包围住它的事件视界所构成,速度最快的光也无法逃脱到事件视界之外,因此理论上外界观察者无法直接观测到黑洞内部的现象。裸奇异点则与之相反,光与其他粒子有机会逃离奇异点至远方,而事件视界因此不存在;外界观察者有机会观察到发生在奇异点附近剧烈扭曲时空的现象。裸奇异点的存在对于天文物理等领域来说有其重要性,其中之一是可能得以观察到星体塌缩成无限大密度的点的一些过程。另一方面,其存在与特性乃是对量子重力理论进行检验的良好机会。数学物理学家彭罗斯(Roger Penrose)于1969年提出一项自然法则的猜想,被称之为宇宙审查假说。此猜想陈述星体塌缩成奇异点的过程,必须伴随事件视界的产生;由于事件视界的存在,处于其内的奇异点无法被直接窥视。宇宙审查假说并未经过直接验证,而有些物理学家也质疑宇宙审查假说的真确性。其透过理论预测,指出裸奇异点的存在是有可能的。一些不同结构、形状的星体在塌缩过程中,并不如过去所分析的必然伴随事件视界的产生。其中的例子如德国物理学家塞费德(Hans Jürgen Seifert)与其同僚在1973年所提出的密度不均匀星体的例子。","text2":"为什么说理论上外界观察者无法直接观测到黑洞内部的现象?","label":1} {"text1":"谢永常(),字平山,明末福建省海户人,明郑将领,后来与荷兰东印度公司在台南卫(今台南市安平区)一役中,与一千多名士兵战亡。经过数百年,当地的台南市政府土木课在中山国中的操场的地底里,挖掘到谢永常、其他士兵与战马的灵骨,不久在台南市东区裕农路的灵骨塔供奉,被民众尊称为「谢府元帅」。李登财和刘还月将得到的神明生平及传统祭典知识,合著四本《神佛正传与祭拜须知》,其中谢永常在《夏之卷》有详细的介绍。明思宗崇祯三年农历十一月初五(公元1630年12月8日),诞生于福建省海户。少年时,臂力超乎常人,十分威武。而且经常打抱不平,扶助善良驱逐邪恶。在海户附近一带的人,对于谢永常的英勇事迹有所赞扬,让他的名声传播千里。后来满族清兵入关,顺治正式入主中原,建立清朝。谢永常悲痛万分,就拜别自己的父母,追随郑成功。在永历七年(1653年),正式成为明朝将领郑成功的部将后来因参与数十役大小战争,而晋升为马部先锋官,为宜毅前镇陈泽的前锋。永历十五年(1661年)春夏两季之间,郑成功率领自己的军队,往东边航行,这时谢永常也跟著郑成功的行列。终于到了农历四月初一(4月29日),谢永常他们正式登陆台南鹿耳门。不久,他们与由荷兰台湾长官揆一军打仗,苦斗数月。农历八月廿四(9月17日)未时,谢永常在台南安平一役,跟荷军战斗,不幸右臂中枪而亡,得年三十二。另外追随他的部将赵胜,也在同役死亡。民国五十三年(1964年)台南市立初级女子中学(今台南市立中山国民中学)在进行操场扩建工程时,挖出一千零六十七具灵骨。而后因为有谢永常显灵托梦的事件,让许多人捐献土地和金钱,准备建立寺庙供奉这些士兵,最后决定在台南市东区后甲公墓一带立庙,同时取庙名为庆隆庙。该庙除了供奉谢永常外,另供奉赵胜与吉原小造。吉原小造为日本东京人,是日本军军官,阶级为大佐,以北白川宫能久亲王部属的身分来台,参与乙未攻台之役,在安平遭原住民攻击而阵亡,骨骸与谢将军同殁一处,故合奉之。","text2":"谢永常的灵骨是在哪里被挖掘到的?","label":1} {"text1":"欧阳玄(),字元功,号圭斋,因避清讳,其名又作欧阳元,其先家庐陵,为欧阳修之后,后迁居浏阳,故为浏阳人。生于是元世祖至元二十年(1283年),祖籍江西,后迁居湖南浏阳。其父欧阳龙生,入元后仕至道州路教授。欧阳玄自幼聪明,八岁能成诵,年十四下笔辄成章。元仁宗延祐二年(1315年),中式第三名进士。曾任翰林待制,兼国史院编修官,元统六年(1333年),任翰林院直学士,奉诏编修《泰定帝实录》、《明宗实录》、《文宗实录》和《宁宗实录》。官至翰林学士承旨。编有《辽史》、《金史》、《宋史》三史。又编有《太平经国》、《至正条格》、《经世大典》。至正十七年(1357年)病逝大都。赠大司徒、柱国,封楚国公,谥曰“文”。有《圭斋文集》15卷、《睽东记》传世。","text2":"在科举考试中,欧阳玄取得了什么成绩?","label":1} {"text1":"高鳍刺尾鱼(学名:),又称高鳍刺尾鲷,俗名粗皮鱼、大帆倒吊、太平洋帆吊,是辐鳍鱼纲鲈形目刺尾鱼亚目刺尾鱼科的一种。本鱼分布于印度太平洋区,包括东非、红海、马达加斯加、模里西斯、留尼旺、葛摩、塞席尔、马尔地夫、斯里兰卡、安达曼群岛、印度、日本、中国沿海、台湾、菲律宾、马来西亚、印尼、新几内亚、泰国、澳洲、罗德豪岛、圣诞岛、新喀里多尼亚、马里亚纳群岛、马绍尔群岛、帛琉、密克罗尼西亚、索罗门群岛、斐济群岛、万那杜、诺鲁、夏威夷群岛法属玻里尼西亚、吉里巴斯、吐瓦鲁、复活节岛、东加、美属萨摩亚等海域。水深0至30公尺。本鱼体呈卵圆形而侧扁。口小,端位,上下颌齿较大,齿固定不可动,扁平,边缘具缺刻。体色具变异,由深橄榄棕色到几乎全黑,杂有暗黄色之垂直窄斑纹及较宽的白色斑纹。头部色白,布暗黄色斑点,一条深棕色垂直斑纹横过眼睛。臀鳍及背鳍与体同色,具有卷曲环状的白色窄斑纹。尾鳍棕色布满暗黄色小斑点。背鳍硬棘4至5枚、背鳍软条29至33枚、臀鳍硬棘3枚、臀鳍软条23至26枚。体长可达40公分。本鱼栖息于礁湖或向海礁坡上。幼鱼通常单独在水较混浊的浅水礁石区或石缝等有隐藏处活动。以底栖动物及藻类为食。可食用,具观赏价值,食用时,可先以姜丝煎半熟,在加些葱及味增,即成美味之味增汤。","text2":"高鳍刺尾鱼是属于什么科目的物种?","label":1} {"text1":"《伊斯坦布尔:一座城市的记忆》(土耳其文:İstanbul: Hatıralar ve Şehir)大体上是奥尔罕·帕慕克一部忧伤的自传回忆录,于2005年被译成英文,同年获得德国书业和平奖。该书是一段撼动了整个土耳其的巨大文化变迁的记录——现代化与不断退却的传统之间的斗争,它也是一逝去家庭传统的挽歌,更多的,它还是一本博斯普鲁斯,以及伊斯坦布尔和这海峡之间历史的书。动手写此书时,帕慕克处于抑郁症爆发的边缘。在一次访谈中,他说:“当时我的生活,因为很多事情,处在一场危机之中;细节就不多说了:离婚,父亲去世,职业上的问题,这个问题,那个问题,所有的事都很糟糕。如果我软弱的话,一定会得抑郁症的,但每天我起来洗个冷水澡,坐下来,回忆然后写作,永远记得要把这本书写得美。”他的家庭对这本书很是不满,特别是他的哥哥。帕慕克说因为这本书他失去了他的哥哥,他还承认这本书也伤害了他母亲的感情。书中作者个人的回忆和与伊斯坦布尔紧密相关的作家、艺术家交织在一起。书中有一整章献给了十九世纪制作过君士坦丁堡版画的西方艺术家梅林。帕慕克本人最喜欢的伊斯坦布尔作家,他灵感的来源,雅哈雅、科丘、希萨尔和坦皮纳都被他写入了书中。他最钟爱的西方行旅作家对他而言,就像是他个人的奈瓦尔、歌德和福楼拜。本书所采用的插图大部分是由古勒拍摄的,由帕慕克亲自挑选的。帕慕克认为古勒的照片有一种忧伤的韵味。此外,帕慕克还加入了一些家庭照片。","text2":"《伊斯坦布尔:一座城市的记忆》中有哪些伊斯坦布尔作家?","label":1} {"text1":"詹姆斯·切斯特(,),是英格兰出生的威尔斯职业足球运动员,司职中后卫,出身曼联青训系统,现时效力英冠球会阿士东维拉。切斯特8岁时就被曼联邀请到青训营接受训练,2005年7月正式签约。2007\/08年赛季,他是曼联预备队出场最多的球员和后防绝对主力,因此进入一线队。2009年1月20日,他在联赛杯半决赛对阵德比郡的比赛中替补出场,第一次代表曼联参加正式比赛。2月2日被外借到英甲球队彼德堡一个月,期间上阵5场。2009年9月15日切斯特再获外借到英冠球会普利茅夫3个月,但仅作赛3场便因伤提前被送返曼联。2010年8月3日切斯特获外借到英甲球会卡素尔直到年底才完结。期间表现出色,在各项赛事上阵23场及射入3球,于借用期满前表示希望继续留队,于2011年元旦主场对哈德斯菲尔德近门顶入为球队扳平2-2完场。赛后曼联召回切斯特,转会窗重开后动作多多的英冠球会侯城提出约30万英镑罗致旗下。","text2":"詹姆斯·切斯特现在效力哪个球队?","label":1} {"text1":"埔里华吸鳅(学名:),俗名石贴仔、畚箕鱼、棕蓑贴、簸箕鱼、木箕贴仔,为辐鳍鱼纲鲤形目爬鳅科的其中一种,是台湾的特有物种。台湾农委会2009年4月1日公告为应予保育之野生动物。本鱼仅分布于台湾大甲溪至高屏溪等河川中、上游。该物种的模式产地在埔里大肚溪。水深0至10公尺。本鱼体纵扁延长呈圆筒状,腹部扁平。眼小,口下位,具4对须。鱼体为浅黄或黑绿色,鳞片细小。胸鳍极宽大平展,腹鳍后缘愈合呈吸盘。尾柄部具4至6个鞍状斑,尾鳍凹形,体长可达9公分。本鱼为初级淡水鱼,栖息于溪流中、上游水流湍急之高溶氧区域。为底栖性鱼类,平时平贴于石头上,属杂食性,以岩石附生藻类和水生昆虫为食。非食用鱼,但因花纹及生态特殊,可做为观赏鱼。","text2":"埔里华吸鳅有着怎样的生活习性?","label":1} {"text1":"长腰鲔(\"Thunnus tonggol\"),又名小黄鳍鲔、黑鳍串、串仔、长实、长翼,为为鲭科金枪鱼属的其中一种。可做为食用鱼、游钓鱼。本鱼身体呈纺锤形,体长可达0.7米。第二个背鳍比第一背鳍更高; 胸鳍是短的到中等长度; 泳鳔不存在或不发达。下侧与腹面银白色有无色的被排列成水平导向的列的细长的椭圆形的斑点,背鳍、胸鳍与腹鳍是黑色;第二个背鳍与臀鳍的顶端褪色成黄色;臀鳍是银色;背部与肛门的离鳍是黄色的有浅灰色的边缘;尾鳍是黑色。栖息深度50-2473公尺,栖息在沿海水质混浊的海域,成群活动,属肉食性,以鱼类、头足类、甲壳类为食。分布于印度西太平洋区,从红海至新几内亚,北从日本至澳洲海域。","text2":"长腰鲔哪个背鳍更高?","label":1} {"text1":"曾博君(),绰号伊藤(起这个绰号的主要原因是因为他个人十分喜欢日本的文化以及事物。),是第二届模范棒棒堂的重心人物之一。曾博君于2008年中旬参加在台北举行的《模范棒棒堂》征选会,以自弹自唱入选,于2008年8月正式加入《模范棒棒堂》参加录影,成为第二届第一代成员。他是本届节目主打的音乐路线的代表人物之一,也是节目内少数与小杰相同的以唱作型歌手为卖点的节目成员,本身也懂得演奏钢琴及结他,他亦擅长即兴作词;他也曾被《模范棒棒堂》堂主兼资深歌手范玮琪指具有极佳音乐天分(他甚至在节目中表演过将结他背在肩上弹奏),很看好他。另外,在第二届《模范棒棒堂》中期开始,他开始凭努力建立起个人的独特风格,开始担起支撑节目的重任,成绩获一众底迪及堂主所认同。值得一提的是,他也曾经参加过第三届《超级星光大道》,可惜在比赛初期已经被淘汰。由于他在三军选拔中因技艺稍嫌不足而未能选上成为模范三军,于第二届模范棒棒堂结束后退出节目。在加入《模范棒棒堂》后在发型改造的时候,被发型师指有秃头危机,而他个人也十分关心这个贴心问题,在节目上被突击查包子(突击搜袋)的时候被发现带著有关秃头的单张以及问卷。而他这个问题也常在节目上被揶揄。","text2":"曾博君参加《模范棒棒堂》凭借什么入选?","label":1} {"text1":"裂带天竺鲷(学名:),为辐鳍鱼纲鲈形目鲈亚目天竺鲷科的一种,俗名粗体天竺鲷。本鱼分布于印度西太平洋区,包括马来西亚、印尼、越南、琉球群岛、菲律宾、所罗门群岛、台湾、澳洲、密克罗尼西亚、万那杜、帛琉等海域。该物种的模式产地在菲律宾。水深2至20公尺。本鱼体延长而侧扁,眼大,口大略下位。体色呈白色或略带桃色,在体侧边上有大约6条红褐色的斑纹;在尾鳍基底有3至4个深色斑点。眼具蓝色圆圈包围。稚鱼有黄色的尾梗且在中心有一黑色斑点,背鳍硬棘8枚;背鳍软条9枚;臀鳍硬棘2枚;臀鳍软条9枚,体长可达12公分。本鱼栖息于珊瑚礁区,白天躲藏于岩洞中,夜间出来觅食,属肉食性,以软体动物或其它底栖甲壳类为食。繁殖期时,雄鱼具有口孵习性,卵约7日化成仔鱼,由雄鱼吐出,具短暂的仔鱼飘浮期。可做为观赏鱼。","text2":"裂带天竺鲷一般栖息在什么地方?","label":1} {"text1":"猫叫综合症(,也称猫哭症、猫啼症)——又称5号染色体短臂缺失症候群()——是一种由于第五号染色体短臂缺损而引起的罕见基因异常病症。患病婴儿的哭声特别,似猫啼,因而得名;其英文名称系借自法语,意即「猫哭」或「猫叫」。1963年Jérôme Lejeune首次在文献中报告该病,该疾病因而又称Lejeune症候群。每年出生的活产婴儿中,约每两万至五万名新生儿之中有一人患有此症。此症可发生于任何族裔,男女比例约为3:4。因为婴儿的喉头和神经系统异常,所以患有此症的婴儿会发生猫啼般的哭声,大约三分之一的患儿在两岁之后就不会再发出这种哭声,除此之外,猫哭症还有以下症状﹕其他常见症状包括低肌肉张力、小脑症、发育不良、圆脸、胀面颊、小下巴、两眼过份分离、内眦赘皮、上眼线下钭、斜视、扁鼻梁、嘴角下垂、低耳、短手指、断掌、先天性心脏病(例如﹕心室间隔缺损、心房间隔缺损、动脉导管未闭、法乐氏四联症),但患有猫哭症的人,其生育能力则不受影响。较少见的症状包括兔唇、颚裂、耳边有瘘管、胸腺发育不良、肠道转位不全、巨结肠、腹股沟疝、髋关节脱臼、隐睾症、尿道下裂、罕见肾脏畸形(例如﹕马蹄形肾脏、肾脏异位、发育不全、肾积水)、尾指内弯、马蹄形内翻足、扁平足、第二及第三只手指和脚趾连趾、可过度伸展的关节等在较年长的小朋友和青少年的身上,患者会表现出明显的智力不全、小脑症、脸部特征变得粗鲁、突出的眼眉骨、深陷的眼睛、扁鼻梁、严重的咬合不正、脊椎侧弯。受影响的女性患者会踏入青春期,第二性征、月经会如常出现,生殖器官正常,但曾发现心型的子宫。受影响男性的睾丸通常会比较细小,但可正常制造精子。猫哭症是因为第五号染色体部份缺损,亦叫做5p单倍体,大约九成的问题基因都是由突变形成的,其余的问题基因是由于父或母一方的两条第五色体连在一起,令受影响的婴儿成为5p染色体三倍体(trisomy),这些婴儿通常会有较严重的病症和病状。诊断主要根据受影响婴儿特有的哭声和其他症状,受影响的家庭可以接受基因咨询和基因测试。患有猫哭症的儿童可以接受言语治疗、声音治疗和职业治疗,如有先天性心脏缺陷,则通常需要接受手术。","text2":"受影响的女性患者有什么影响?","label":1} {"text1":"马永贞()回族,山东省临清直隶州邱县陈村(今河北省邱县陈村回族乡陈村)人。清朝末年拳师。马永贞自幼习武,擅长查拳、弹腿功,还善于驯马、骑马。离开家乡后,靠武艺生活,曾在河南周口、江苏扬州多次和人比武。后到上海卖艺收徒,是最早在上海传授查拳的武师。他在上海还以贩马维生,曾多次赴蒙古、热河、察哈尔贩马。咸丰十一年(1861年)秋季,在上海跑马厅(1851年建,今人民广场和人民公园)举办的赛马会上,马永贞战胜了前两届赛马冠军洋人史蒂夫。马永贞曾任清朝松江府正营武备教官。光绪五年三月二十二日(1879年4月13日),马永贞应邀到上海“一洞天”茶楼(原址位于今南京东路金华路)吃茶,遭马贩顾忠溪纠集打手暗算,经朋友送到上海体仁医院抢救无效,当夜死亡。1927年电影《山东马永贞》中,还虚构了马永贞的胞妹马素贞得知后,只身到上海手刃杀兄仇人而回的情节。","text2":"马永贞自幼习武,擅长什么?","label":1} {"text1":"《柳林风声》(The Wind in the Willows)或译「柳林中的风声」,是英国小说家肯尼思·格拉姆的代表作,也是经典的儿童文学作品,出版于1908年。肯尼思·格雷厄姆是英国作家,出生于爱丁堡一个传统的苏格兰家庭,父亲是律师,却有严重的酗酒恶习。肯尼思·格雷厄姆的母亲因猩红热病逝后,外公外婆把他带到乡间抚养长大。他小时候流连的田野风光,后来成为《柳林风声》中鼹鼠、河鼠、獾与癞蛤蟆先生结伴畅游的世界。虽然肯尼思·格雷厄姆的童年充满烦恼,但他创作出的《柳林风声》的基调却是甜美的。这本书曾经引起当时美国总统罗斯福的注意,他曾写信告诉作者,他把《柳林风声》一口气读了3遍。《柳林风声》也是《哈利波特》的作者J·K·罗琳最喜欢的文学作品,在哈利波特当中赫夫帕夫的象征獾也是以书里憨厚的獾先生为原型的。《柳林风声》曾经多次被改编成舞台剧、电影与电视剧。其无人类存在、动物著衣拟人化亦影响迪士尼2016年3D动画电影《-{zh-cn:疯狂动物城;zh-tw:动物方城市;zh-hk:优兽大都会;}-》。","text2":"在哈利波特当中赫夫帕夫的象征獾是以什么为原型的?","label":1} {"text1":"短头真银汉鱼为辐鳍鱼纲银汉鱼目银汉鱼科的其中一种。本鱼分布于东南大西洋,局限在纳米比亚的卢德里兹(Lüderitz)至南非的纳塔尔。本鱼体延长而细,略高大而侧扁;头小,外侧无齿状小棘;两腭齿甚小,绒毛状;口盖骨及锄骨有齿,中翼状骨(mesopterygoids)齿有强脊;舌上具乳状突起;前上颚骨末端略达瞳孔前缘之下方,鳍上突起较细长,突起长为宽之2倍;侧突起小而平坦。齿骨向后上方斜走,其后半部高耸。主鳃盖骨后缘无明显缺刻。背鳍两枚,第一背鳍位于体中央上方之体背,具5至8棘,其起点在腹鳍末端之上方;第二背鳍在第一背鳍后方,具1棘及15至18枚软条;臀鳍与第二背鳍对在而略前,具1棘极15至18枚软条;脊椎骨数约46枚。肛门开口于腹鳍末端或略前之腹缘。体具圆鳞,7至11纵列,依纵列约44至50枚鳞片。体呈银白色,略透明,头背部暗色,体侧具一明亮之纵走带;吻端灰色;体背各鳞具细小色素胞黑点、其上有时具有条纹;体侧下方灰白色,无色素胞存在;各鳍多少均透明,惟尾鳍暗色。体长可达11公分。为海水鱼,于春季时成群出现在南非东岸及河口,以浮游生物为食。食用鱼,也为海鸟及其他鱼类的饵料鱼。","text2":"此鱼的第二背鳍的具体位置在哪里?","label":1} {"text1":"阿扎迪自由纪念塔(),位于伊朗首都德黑兰梅赫拉巴德国际机场附近,是德黑兰的地标,也是伊朗的象征。阿扎迪自由纪念塔原称作沙希亚德塔或国王纪念塔(),建成于1971年,为纪念波斯帝国建国2500周年献礼。1979年伊朗发生伊斯兰革命,巴列维王朝被推翻,伊朗伊斯兰共和国建立,国王纪念塔也改名为自由塔,象征伊朗自由神权时代的来临。塔高45米,塔基长63米,宽42米,呈灰白色,采用钢筋水泥和2500块产自伊斯法罕的大理石建成,象征波斯帝国建国的2500个岁月。阿扎迪塔附近设有一系列配套设施,包括博物馆和影像剧院。以阿扎迪塔为中心的阿扎迪广场有五万平方米大,广场有大型喷泉和绿地,每年伊朗国庆日,此处会举行盛大的阅兵仪式。","text2":"伊朗伊斯兰共和国建立后,该塔改名意味着什么?","label":1} {"text1":"天文航空(Astro Air International Inc.)是一家菲律宾廉价航空公司,基地设于邦板牙省克拉克自由港区的克拉克国际机场(Clark International Airport,又称「马嘉柏臬国际机场」,Diosdado Macapagal International Airport,DMIA)。天文航空的前身为马尼拉精神航空(Spirit of Manila Airlines),主要航线皆集中于东南亚地区。2011年冬季,该公司由于内部进行重组和融资,因此航班处于停飞状态。该公司已于2013年5月1日恢复台北(台湾桃园国际机场)到长滩岛(卡利博国际机场)的航线 。","text2":"天文航空的主要航线是哪里?","label":1} {"text1":"BC信用卡杯世界围棋公开赛 ( BC card Cup World Baduk Championship) , 简称BC卡杯,是围棋主要国际赛事之一。前身为韩国国内赛事BC信用卡杯新人王战,韩国棋院主办,围棋TV,NHN协办 , BC信用卡公司赞助。2008年韩国棋院通过决议,2009年起赛事采奖金制,公开赛制,以创新现有围棋体制,创建新的世界围棋大赛。2012年BC信用卡公司被韩国KT公司(电信会社)收购,此前两公司均致力于赞助围棋比赛。BC信用卡杯世界围棋公开赛历经四届赛事之后将停办,KT杯(韩国电信会社杯)有望接替填补空白,如今两公司合并后,两大赛事可能合二为一。比赛每方2小时,60秒读秒3次。2011年1月24-26日,首尔:综合预选2012年2月27-29日,首尔:综合预选","text2":"BC信用卡杯世界围棋公开赛的赛时是多久?","label":1} {"text1":"蛋白质四级结构()是生物化学中用于描述多亚基蛋白质复合物中各个折叠蛋白质亚基的排列组合。许多蛋白质实际上是多个多肽链的组装。 四级结构是指蛋白质亚基相对于彼此的数目和排列。具有四级结构的蛋白质的实例包括血红蛋白,DNA聚合酶和离子通道。由具有不同功能的亚基组成的酶有时被称为全酶,其中一些部分可以称为调节亚基,功能核被称为催化亚基。作为多蛋白质复合体的其它组件也具有四级结构。实例包括核小体和微管。四级结构的变化可以通过个体亚基内的构象变化或通过亚基相对于彼此的重新取向而发生。 正是通过这种变化,其在“多聚”酶中协同性和别构调节的基础,许多蛋白质经历调节并且执行它们的生理功能。上述定义遵循生物化学的经典方法,其在蛋白质和功能性蛋白质单元之间的区别难以阐明的时候建立。最近,人们在讨论蛋白质的四级结构时考虑蛋白质 - 蛋白质相互作用,并考虑蛋白质的所有组装作为蛋白质复合体(protein complexes.)。具有四级结构的蛋白质复合物通常可以根据其亚基数目来描述,如含两个亚基称为二聚体,三个则为三聚体,以此类推(只有一个蛋白质多肽链的称为单体)。通常情况下,细胞中的蛋白质复合物很少超过八聚体。但也有一些例外,如蛋白酶体(28聚体)、外切体、剪接体等,这些具有多个亚基的蛋白质在细胞中往往可以形成分子机器以完成某一特定的复杂生物学过程;病毒衣壳甚至可以为60聚体。","text2":"四级结构的变化是如何发生的?","label":1} {"text1":"张永德(),字抱一,并州阳曲(今山西省太原市阳曲县)人,五代时后周禁军统帅之一,太祖郭威第四女寿安公主之婿。后周广顺元年(951年),太祖郭威即位,以外甥李重进为内殿直都知,女婿张永德为内殿直小底四班都知;又升李重进为小底都指挥使,而以张永德接任内殿直都知。翌年(952年),郭威以李重进为大内都点检兼马步都军头,张永德为小底第一军都指挥使;后又以李重进为殿前都指挥使,张永德为殿前都虞候,掌管殿前亲军。两年后,郭威病危,传位养子柴荣,临终前特命李重进向柴荣行君臣之礼,以免其觊觎皇位。显德元年(954年),世宗柴荣即位,以姑表兄李重进为侍卫亲军马步军都虞侯,妹夫张永德接任殿前都指挥使,分掌侍卫亲军和殿前亲军,张永德时年二十四岁。李重进、张永德本以姻亲之故,在数年间不次擢升,但后来都在战争中展现出过人的军事才能。在决定后周生死存亡的高平之战后,李重进以战功加使相衔,升侍卫亲军都指挥使,母福庆长公主追封燕国大长公主;而张永德以战功加检校太傅,授义成军节度使,妻寿安公主进封晋国长公主。显德三年冬,擢为殿前都点检。四年,从克寿州还,制授检校太尉、领镇宁军节度使。五年夏,契丹扰边,命永德率步骑兵二万拒之。回军途中,周世宗发现一个韦编袋子,里面有一块三尺长的木板,上有“点检做天子”五字,下诏免去张的殿前都点检一职,解兵柄,加检校太尉、同中书门下平章事。恭帝嗣位,移忠武军节度使,赵匡胤代其职。宋初,甚受宋太祖礼遇,加侍中,入朝授武胜军节度使,太祖每呼驸马而不呼其名。太平兴国二年入朝,拜左卫上将军。五年,坐市秦、陇竹木所过矫制免关市算,降为左卫大将军。数月,复旧秩。六年,进封邓国公。雍熙中,连知沧、雄、定三州。端拱元年,拜安化军节度使。召还,为河北两路排阵使,屯定州。尝与契丹战,斩获甚众。二年,丁内艰,起复。淳化初,又代田重进知镇州。二年,改泰宁军节度使兼侍中,出判并州兼并代都部署。真宗即位,进封卫国公。未几,判左金吾街仗事。咸平初,屡表请老,授太子太师,分司西京,仍以其孙大理寺丞文蔚厘务洛下,以便就养。二年冬,契丹入边,帝将北巡,以永德宿将,召入对便殿,赐坐,访以边要。以老不可从行,留为东京内外都巡检使。三年,制授检校太师、彰德军节度使、知天雄军。俄以衰耄,命还本镇。是秋卒,年七十三。遣内园使冯守规护柩还京师、赠中书令。诸孙迁秩者五人。","text2":"在决定后周生死存亡的高平之战后张永德接任什么职位?","label":1} {"text1":"崔真实(,),已故韩国女演员。20岁时以拍摄MBC剧集《朝鲜王朝500年-恨中录》出道,1995年凭借电影《杀妻秘笈》获得第33届韩国大钟奖最佳女主角奖及第31届韩国百想艺术大奖人气奖。2008年10月2日,因深受被指迫害安在焕烧炭自杀谣言困扰,在住所浴室利用绷带上吊自杀,享年岁。事后韩国政府拟于11月立法信息通信网法施行令修正案,防止匿名网民发放流言。曾经与日本职棒选手赵成珉于2000年结婚,因此息影,育有2名子女。之后在2002年10月因为家庭暴力,崔真实怀孕被推落楼梯,而在2004年9月离婚。2008年5月崔真实的两名子女依照当地的户籍法改随母姓。生前崔真实最后的日子与母亲同住。其弟崔真永亦为演员,2010年3月29日自杀身亡,得年39岁。2013年1月6日,其前夫赵成珉被发现在家自杀身亡。2013年11月27日,前经理人朴尚浩自杀。","text2":"崔真实有几个孩子?","label":1} {"text1":"刘胜(),汉景帝第九子。生母贾夫人,同母兄赵敬肃王刘彭祖,汉武帝的异母兄,封中山王,谥号靖,即中山靖王。前155年,汉景帝封其同母兄刘彭祖为广川王。前154年,赵王刘遂谋反后,改封刘彭祖为赵王,另封刘胜为中山王。刘胜好酒色。赵王刘彭祖批评亲弟弟,作为藩王,不佐天子抚慰百姓,只知道奢淫。刘胜反驳,说哥哥作为王爵,却干一些官吏干的事,国王就该听音乐,御声色。前113年,刘胜病卒,嫡子刘昌继位。刘胜的儿子和孙子总数一百二十多人1968年,在河北省保定专区满城县,发掘了刘胜和王妃窦氏的坟墓。两人遗体分别包裹着金缕玉衣、银缕玉衣。发掘作业中又发现长信宫灯、漏壶。","text2":"刘胜一母同胞的兄弟叫什么名字?","label":1} {"text1":"八斗子渔港()位于台湾基隆市中正区的八斗子,是一座多功能的渔港,也是台湾北部最大的渔港。依《渔港法》分类,其渔港等级为第一类渔港,主管机关为农委会渔业署。八斗子渔港位处八斗子半岛西南侧与台湾本岛间,基隆港东南岸2.5浬处的岩岸岬角,行政区分上隶属基隆市中正区。原为天然海湾,三面环山,西北通海,水域深广,潮差温和,又少漂沙,港口宽度60公尺,航道水深6至18公尺, 条件优良,而附近海域渔获丰富,早期就已经是当地人生活中不可缺乏的食物来源,更是成为渔港的重要条件,为一岩岸岬角之天然良港。1974年当时的行政院长蒋经国任内巡视基隆市时,希望配合基隆地区渔业的快速发展,为解决当时基隆市唯一的大型渔港正滨渔港(位于基隆港内)渔船使用地不足的拥挤问题,于是决定在八斗子兴建大型渔港,并计划以此新渔港取代正滨渔港的功能,以扩大基隆港的商港用地(但日后并未执行)。1975年正式动工兴建,兴建时八斗子煤矿全面停采,到1979年兴建完成。八斗子渔港历经多年的整修后,现今各项公共设施齐全,已成为台湾北部地区最大的渔港。八斗子渔港的兴建不仅提供基隆地区渔业生存空间,也促进了相关工商业的引进及蓬勃发展,更成为基隆市重要的旅游景点,每逢假日吸引大量外地游客前往。为了因应民众假日休闲游憩的需要,基隆市政府所成立的管理所利用渔港的第二泊区陆路地范围,设置休闲绿带成为滨海公园。八斗子渔港面对太平洋,受东北季风直接吹袭,其强度对渔港作业有极大影响,每年9月至翌年5月,多为北北东风,6月至8月多为西南风。八斗子地区的渔业以沿、近海为主,主要渔获物有以下几种:八斗子地区常用的渔法","text2":"港口的宽度是多少公尺?","label":1} {"text1":"新一代设计展(Young Designers' Exhibition,简称YODEX)是一项每年在举办的大型创意设计展览,参展单位以大专院校为主轴,在台湾也是一项大型的毕业成果联展。2007获国际工业设计社团协会选为ICSID50周年系列活动之一。新一代设计展在1982年,由中华民国对外贸易发展协会设计推广中心开办当时,被称之「大专院校产品与包装设计展」。其后,于1987年,隶属于「台湾产品设计展」的其中一项范畴,而当时的台湾产品设计展,则依照内外销性质的不同,依序有专为产业界设计的「国际优良设计观摩展」及「手工业产品评选展」,以及针对一般消费者(民众、学生)设计的「国际优良设计选拔展」及「大专院校设计展」。这项展览是由中华民国对外贸易发展协会与台湾创意设计中心联合举办,借此以发掘创意设计人才与精品,并将设计产业,与国际进行接轨。适逢台湾创意设计中心拿下2011年世界设计大会的关系,进而使得这项展览更受到国内外设计产业的高度关注。新一代设计展是一个跨国性,以台湾为主的设计院校毕业作品联展,规模在国际设计学界甚少见,提供设计新秀展现设计创意的平台,此项展览是台湾凝聚设计能量的年度盛事,每年有超过50所国内外设计学院参予展出毕业作品,领域横跨产品设计、平面设计、视觉传达设计、空间及景观设计、流行时尚设计及服装设计等领域。2011年第30届新一代设计展中,承揽主视觉设计的陈永基设计有限公司抄袭Lomography Spinner 360°相机广告,消息曝光后,执行单位台湾创意设计中心立刻撤除了所有文宣及广告刊物,另赶工重制海报、路灯旗、网站广告、导览手册、展场设计等文宣制作物。负责人陈永基总监向Lomography台湾区总经理Jeansman Lee当面致歉,并在联合报、自由时报、中国时报、工商时报及经济日报等各大报刊登道歉启事;Lomography李总经理口头接受道歉,李总经理亦立即向该公司总部报告此事,该公司最后决定与陈永基有限公司签订和解书,并不再追究后续责任。。2014年5月,由于新一代设计展参展费用过高,台湾科技大学、成功大学、实践大学、辅仁大学等设计科系的学生在脸书上发起「反对新一代」活动。经济部12日表示,行政院已核定自2014年起场租减半,以5折价收场地租金,16日起将受理申请退费。","text2":"经济部为什么表示行政院核定自2014年起场租减半?","label":1} {"text1":"广州白云足球队是中国广州市昔日的一支足球队,现时广州恒大足球俱乐部前身。其前身是广州市足球一队,于1961年4月组建,1967年停办。1977年10月26日广州市人民政府批准恢复广州市足球运动队,称广州市青年足球队,1978年8月复办。1979年6月国务院发文要求发展足球重点地区,广州列首批重点地区的首位。1980年广州青年队改名为广州市足球队,并参加丙级联赛,获分区赛第2名,晋升乙级,次年再获乙级联赛冠军,进入顶级联赛。1984年10月1日由白云山制药总厂出资赞助每年20万元,与广州市体委合办,改名为广州白云山制药厂体协足球队,成为中国大陆首支由体育部门与企业合办的运动队。1989年广州白云队扩大为足球俱乐部。1993年1月8日广州太阳神集团和广州市体委合作成立中国首家股份制职业足球俱乐部,改名为广州太阳神足球俱乐部。","text2":"1980年广州青年队改名之后称为什么?","label":1} {"text1":"阎相闯(),中国足球运动员,司职前卫。1993年,阎相闯进入秦皇岛足校开始足球训练。2000年,阎相闯进入八一青年队。2003年,阎相闯进入八一队的一线队球员名单,八一队在当年甲A联赛降级后宣布解散。2004年,阎相闯加盟北京国安。2009年以主力身份随北京国安获得当年中超冠军。2010年5月,阎相闯首次入选中国国家足球队并于同年6月中国4-0胜塔吉克斯坦的比赛中,打进其在国家队的首个进球。2011年7月,球员二次转会期间闫相闯被北京国安租借到大连实德效力。2012年,阎相闯完成正式转会与大连实德签约三年。2013年,大连实德解散后闫相闯转会贵州人和。2014年7月二次转会,之前错过注册的阎相闯正式加盟哈尔滨毅腾。2015年1月,加盟中甲北京北控队。","text2":"阎相闯什么时候开始的足球训练?","label":1} {"text1":"《至Net奇兵:追求无限》(\",)是至Net奇兵的第二款游戏,由The Game Factory制作,在2007年11月16日于美国推出在Wii平台的动作游戏,之后亦于澳洲及欧洲推出。此游戏的故事情节概括《至Net奇兵》动画版的第四辑。而这款游戏在2008年7月21日推出在Playstation Portable及Playstation 2这两个游戏平台上。威廉因为被山拿控制而成为山拿的手下,为了令威廉回复清醒,所以Lyoko小组的五名Lyoko战士——亚烈达、杰理明、治狼、尤美及阿奇便继续和邪恶的山拿战斗,希望能够令威廉摆脱山拿控制回复清醒以及消灭山拿。本游戏和《至Net奇兵》的动画版一样,一共分为现实世界和虚拟世界两部份,在现实世界的部份是以第一人称的角度游戏,玩家可选择玩不玩现实世界的部份;而在虚拟世界的部份则是以第三人称的角度游戏。","text2":"《至Net奇兵:追求无限》游戏主要情节是什么?","label":1} {"text1":"安东尼·刘易斯·约翰·卢彻涅特(Antoine Louis John Ruchonnet,)是一位瑞士政治家,瑞士联邦委员会委员(1881年-1893年)。他于1875年12月10日被首次选为联邦委员会委员,但是他回拒了选举结果,没有履职。1881年3月3日他被再次选为委员,并接受这一任命,任职至1893年9月14日在任内去世。刘易斯·卢彻涅特是瑞士自由民主党成员。在任期内,他主要主持领导了以下部门的工作:刘易斯·卢彻涅特于1883年和1890年两度出任瑞士联邦总统。在洛桑,一条大道被以他的名字命名为刘易斯·卢彻涅特大道(Avenue Louis-Ruchonnet)。刘易斯·卢彻涅特的肖像画由瑞士出生的美国艺术家阿道夫·穆勒-乌里(Adolfo Müller-Ury,1862年-1947年)作于1888年,现收藏于洛桑美术学院博物馆。这幅画像是1894年赠送给他的儿子欧内斯特·卢彻涅特(Ernest Ruchonnet)的礼物。","text2":"肖像画现在收藏在哪里?","label":1} {"text1":"运动系统是动物体用来进行诸如移动,抓取,进食,眼动以及言语等骨骼肌运动的器官组成的功能整体。运动系统的主要部分包括位于外周的骨骼肌和神经以及位于中枢的脑和脊髓。脑和脊髓支配所有骨骼肌的运动。从脊髓的前角投射到肌肉的运动神经元与骨骼肌形成神经肌肉接头,通过动作电位和乙酰胆碱的突触来使肌肉发生收缩。肌肉一般通过肌腱与骨骼相连。肌肉的收缩导致骨骼之间角度的变化,从而使得身体的结构发生形状变化。骨骼肌内含有两种感受器,称为高尔基腱器官和肌梭。两种器官通过感觉纤维(Ia, Ib和II型纤维)与脊髓相连,提供关于肌肉的张力,长度和收缩速度的信息。这些感觉纤维在脊髓内形成神经元回路,来支配肌肉的反射,例如伸张反射。此类低级反射的主要作用是保证肢体姿势的稳定性。脑是较脊髓更高一级的运动中枢。支配脊髓运动神经元的输入来做多个脑内的结构。包括脑干内的下行网状结构,小脑深部核团,中脑的红核,以及大脑皮质的运动区。除了这些结构,基底核也与运动功能有密切的关系。从大脑皮质的运动去直接投射到脊髓运动神经元的通路称为皮层脊髓束。这种非常简单直接的通路与灵长类对手指的精确控制有关。除了支配肢体运动,眼球的运动也是运动系统的重要一部分,与动物的视觉感知,注意力和抓取等功能密切相关。中脑的上丘内含有眼动神经元,直接支配控制眼球运动的肌肉。常见的运动系统疾病包括瘫痪,运动失调症,帕金森氏病,亨廷顿氏病等","text2":"低级反射的主要作用是什么?","label":1} {"text1":"欧尼特(Nathaniel Fitzwilliam Archibald)(剧中生日:1991年5月7日),美国热门电视剧《花边教主》中的男主角。欧尼特(Nate)昰Howard和Anne Archibald的独生子。他是上东城最英俊的男生,被叫作“Golden Boy”。欧尼特,韦倩莲(Serena),胡贝雅(Blair)从小孩子就是非常要好的朋友。Nate和Blair从幼儿园就约会,但Nate的心中最爱的还是Serena。在Sheppard家的婚礼上,Nate和Serena在酒吧台上发生性行为,Nate失了他的“第一次”。Nate和Serena之间的感情很复杂。","text2":"Nate和Blair最早从什么时候开始约会?","label":1} {"text1":"硫氰酸盐是硫氰酸根离子SCN所成的盐,常见的包括无色的硫氰酸钾、硫氰酸钠、硫氰酸铵和硫氰酸汞。硫氰酸酯指含有SCN官能团的有机化合物。硫氰酸根离子与氰酸根离子[OCN]同类,只是氧原子被硫原子替代。[SCN]与卤离子相似之处很多,是拟卤离子之一。硫氰酸盐可由硫或硫代硫酸盐与氰化物反应制备:<\/chem>+S8->8SCN-<\/chem><\/chem>+S2O3^2-<\/chem>->SCN-<\/chem>+SO3^2-<\/chem>磺基转移酶硫氰酸酶可催化第二个反应,可能是体内氰化物解毒的机理。Riemschneider硫代氨基甲酸酯合成涉及有机硫氰酸酯水解为硫代氨基甲酸酯的反应。硫氰酸根存在键合异构体。溶液中,Fe会与[SCN]反应生成血红色的[Fe(SCN)(HO)]<\/nowiki>,可用于检验三价铁离子,有些电影中也用该法制作假血。","text2":"硫氰酸盐可以怎样制备?","label":1} {"text1":"休·马斯顿·海夫纳(,),美国企业家。《花花公子》杂志的创刊人及主编,以及的创意总监(Chief Creative Officer)。出生在芝加哥,是一对教师夫妻的第一个孩子,高中毕业后曾一度参军。他于1953年9月创办了《花花公子》杂志。偏向自由派,曾经多次捐款给民主党。海夫纳曾于1959年及1986经历两度婚姻,都离异收场。2010年,他与年轻六十岁的克莉丝朵·哈里斯(1986年4月29日出生)宣布订婚,本来准备在6月18日结婚,但哈里斯临时取消婚礼,理由是“不想和其他女人公用一个男人”。后在2012年12月24日结婚,31日婚宴。此外,他为了维持性欲大量服用威而钢导致右耳失聪。据英国《镜报》报道,根据婚前协议,赫夫纳31岁的妻子克莉丝朵·海夫纳从他的遗产中将“得不到一分钱”,尽管她将“得到照顾”。2017年9月27日,休·赫夫纳逝世,享耆寿91岁。","text2":"休·海夫纳一个有几段婚姻?","label":1} {"text1":"阿部氏是日本的一个氏族。三河松平氏(德川氏)的谱代家臣。孝元天皇的第一皇子阿部大彦命被认为是他们的始祖。战国时期中,阿部氏是在柳营秘鉴中记载的“安祥谱代七家”之一的古老氏族。有如下记载:。当时,在名人物德川家康的祖父·松平清康的时代,阿部定吉侍奉着松平氏。“守山崩”事件,危及到了主上家的存亡。远征中的主君·清康,被定吉之子·弥七郎正丰所暗杀。阿部正丰在远征途中立即被处刑了,但是阿部家却平安无事。此后阿部定吉继续侍奉松平氏,最后阿部氏在此断绝了(井上氏被认为是定吉留下的后代)。除了侍奉着松平氏的阿部氏,旁支阿部正胜的系谱也十分清楚,但是,和定吉的关系不甚明了。在阿部正胜的家系中,正胜之子·正次趁在江户幕府做大坂城代的时候,使分家都扩大为大名。正次的后辈中,担任重任者人才辈出。尤其是德川家光时代担任老中的阿部忠秋,和幕府末期的老中笔头、备后福山藩藩主阿部正弘为世人皆知。图注:实线为嗣子,虚线为养子,粗体字为历任老中。1 = 历代宗家当主、① = 历代分家(丰后守家)当主、Ⅰ = 历代分家(因幡守家)当主。","text2":"谁是三河松平氏(德川氏)的谱代家臣?","label":1} {"text1":"《苏姗娜之歌》()是美国小说家史蒂芬金所写的长篇奇幻小说黑塔系列中的第六集。台湾由皇冠出版社出版,内地简体版则由人民文学出版社于2008年3月出版,英文原版早于2004年6月出版,当中相距近四年。曾于2005年提名轨迹奖「最佳奇幻小说」。枪侠们打败了卡拉的狼群后,苏姗娜的另一个意志米亚抢夺了苏姗娜的身体逃到了1999年的纽约,由于逃走时同时带走了黑十三,使得门洞失去了魔力,由此枪侠只好求助古老的民族曼尼族,透过-{zh-hans:曼尼族; zh-hant:曼宁}-的磁石,时空之门重新恢复了魔力,但枪侠并不能随心所欲选择目的地,结果杰克、卡拉汉神父及仔仔去到了苏姗娜的时空,罗兰及埃蒂则去到了二十三岁史蒂芬金的年代,1977年。米阿到了纽约后,由于人生路不熟,不知道如何应付,苏姗娜便以协助米阿延下婴孩为交换条件,换取婴孩的秘密,结果发现婴孩原来是罗兰的,米阿原来是吸取杰克精力的神喻,又是强暴苏姗娜的-{zh-hans:恶灵; zh-hant:通灵魔}-。虽然苏姗娜一直拖延时间,并且不断透过精神向埃蒂发出讯号,但还是等不到枪侠的救援,被送到吸血鬼及低等人的地方。由于米阿的告密,黑帮老大巴拉扎预早埋伏,枪侠一离开时空之门便遇到偷袭,幸好得到管理员卡伦的帮助,枪侠才可以逃出生天,后来他们找到了-{zh-hans:塔尔; zh-hant:卡文·塔}-及-{zh-hans:深纽; zh-hant:亚伦·狄普诺}-两人,原来深纽一直都不希望卖掉玫瑰下的土地,而且一直高调行事,惹得枪侠很不满,最终在枪侠的威逼利诱及朋友的劝告下,-{zh-hans:塔尔; zh-hant:卡文}-心有不甘地将给枪侠的泰特集团。两人后来又去了找史蒂芬金,发觉自己的确是史蒂芬金的笔下人物,但在血王的扰乱下,史蒂芬金只写了第一部的手稿就放弃了,罗兰于是利用催眠促使他继续写下去。","text2":"《苏姗娜之歌》的作者是谁?","label":1} {"text1":"北海是一列日本国有铁道(国铁)于1967年3月1日至1986年11月1日之间,行驶于函馆~旭川间的函馆本线特急列车。日本国有铁道为了使青函连络船的旅客能经由更多途径连络道内,在函馆本线上设立了该列车,“北海”也是目前为止唯一经由「山线」(函馆本线长万部~小樽间路段)的特急列车。最初投入运行时使用日本国铁Kiha80系柴联车行驶,运行初期为7节车厢编成、一天仅一往返。1973年10月2日,因运行成果佳,故将编成调为十节一组,也有经由钏网本线延驶至网走站的班次,此时的特急北海达到了全盛期。1981年10月1日,班次调为一天二往返,因Kiha80系逐渐老化而将北海3、4号改用为新制的Kiha183系车辆。以Kiha80系行驶的班次调为九节车厢一组,Kiha183系行驶的班次则是十辆编组。1984年2月1日,编组再度缩为六节,偶尔才会增结为七节,而且只使用Kiha80系行驶,直到翌年才恢复Kiha183系行驶,编组也都固定为六节。1986年11月1日,特急「北海」废止,特急列车从此自山线上消失。因平坦的室兰本线(海线)逐渐被重用,「山线」已有没落的趋势。虽然经由室兰本线来往函馆与札幌的里程数较山线长,但因为弯道比函馆山线少的多,所费时间也较短。「山线」一路上都是急转弯和陡坡,马力低的车辆要通过是十分困难的。因此后来由道南前往道央的优等列车全部改走室兰本线。「北斗」即是「北海」的海线版本。「北海」停驶后,其做为道南连络道央用列车的重要性,改由「北斗」取代。在「北海」行驶的同时,山线上也有急行列车「二世谷」()行驶,但在北海号废除的同时,二世谷号调整为仅有夏季行驶的临时列车。现在「山线」上仅有定期行驶的快速列车二世谷快车()、普通车和临时特急列车二世谷滑雪特快()、SL二世谷号行驶。","text2":"北海号列车是目前为止唯一经由什么线路(函馆本线长万部~小樽间路段)的特急列车?","label":1} {"text1":"太阳向点是太阳在星际空间中运动时按本地静止标准(LSR)所对著的方向。这不应与太阳通过黄道带星座的视运动混淆,因为太阳通过黄道带星座的视运动是假想的运动,并不实际发生,而是由地球绕太阳公转产生的。太阳向点的大致方向是武仙座附近的织女一西南向。太阳向点可以选取多个坐标,视觉坐标(由视运动的目视观测获得)为赤经(RA),赤纬(dec)+30°(银道坐标:银经56.24°,银纬22.54°)。太阳向太阳向点运动的速度是19.5千米\/秒。这不能与太阳围绕的银心的轨道速度混淆。太阳围绕的银心的轨道速度约为220千米\/秒,包含在本地静止标准的运动中。太阳在银河系中的运动比直接的想像要复杂。太阳运动的本性和范围最初于1783年由威廉·赫歇尔阐述。太阳背点是太阳向点的相反方向,位于小犬座ζ星附近。","text2":"为什么不应与太阳通过黄道带星座的视运动混淆?","label":1} {"text1":"白蛋白(,又称“清蛋白”)是属于球状蛋白的一种蛋白质,但并不是球蛋白。在人体内它最重要的作用是维持胶体渗透压。在奶和蛋里也有白蛋白。人体内白蛋白的正常值为:新生儿28~44g\/L,14岁后38~54g\/L,成人35~50g\/L,60岁后为34~48g\/L。身体缺少白蛋白会导致浮肿。肝硬化病人的血浆白蛋白含量比正常人低。尿中白蛋白的含量的变化能反映肾脏的某些病变。白蛋白的分子量为68,500道尔顿,由584至590个氨基酸组成。由于它的半胱氨酸含量比较高因此相对而言它含有比较多的硫。白蛋白可溶于水中,每克水约可溶18毫升白蛋白。它的等电点为4.6。在热(凝固温度60—70℃)或酶等条件下可凝固和变性。白蛋白存在于动物、植物、细菌中。动物性白蛋白在(NH)SO的饱和度为0.64—0.90时开始盐析,而植物性白蛋白在(NH)SO的饱和度为0.5时则有相当一部分被沉淀。","text2":"人体内白蛋白的正常值是多少?","label":1} {"text1":"尚元(;)是琉球国第二尚氏王朝第五代国王。他是第四代国王尚清王的第二王子。1556年至1572年在位。神号日始按司添()。童名金千代(),一作鹤千代()。据《中山世谱》记载,尚元原是尚清王生前指定的继承人。但1555年尚清王死后,法司葛可昌(城间亲方秀信)、和为美(国头亲方景明)却突然变心,欲拥立尚清王第四子尚鉴心为君。法司毛龙吟(新城亲方安基)提薙刀叱责二人。群臣多有支持毛龙吟者,因此和、葛二人畏惧而不敢复言。于是尚元在继承权争夺之中胜利,并于次年即位。1559年,将和为美流放久米岛,葛可昌流放伊比屋岛。1562年(嘉靖四十一年),明世宗派遣刑科给事中郭汝霖、行人李际春为册封正副册封使封尚元为王。尚元王体弱多病,常常无法正常管理国事。为了保证朝政的正常运行,琉球王府推举了三名法司为其代理。从此琉球王府中法司的人数定例为三名,这三名法司被统称为「三司官」。朝政基本上掌握在毛龙吟等三司官手里,因此尚元王被称人为「哑巴国王」()。 尚元王死后,三司官的制度被保留了下来。尚元王在位期间,琉球与明朝保持良好的外交关系,多次送还漂流至琉球境内的中国船民。1565年(嘉靖44年),倭寇过境琉球,在北山地区为琉将郑都所败。尚元王遣使将被倭寇虏获的中国人口送还明朝。尚元王也曾一度与日本萨摩藩保持良好的关系,然而在1570年至1572年,双方关系出现两次危机。1570年,萨摩岛津贵久的使者雪岑和尚至琉球,要求尚元王向萨摩朝贡,遭琉方拒绝。岛津大怒,煽动奄美大岛酋长发动叛乱,并向奄美派遣军队,给奄美的酋长提供军事上的支持。1571年,尚元率船五十余只御驾亲征,在奄美大岛两次与萨摩军交战,败萨军,俘杀叛变的酋长。归军途中,尚元王病危,三司官马顺德(国头亲方正格)对天祈祷愿代王死,不久尚元王竟果然病愈。尚元王大为感动,封马顺德之子为按司,称国头御殿,世代袭爵。1572年4月1日,45岁的尚元王病逝,次子尚永继位。","text2":"被推荐给尚元王代理国事的三名法司统称为什么?","label":1} {"text1":"刺鲳(学名:)为辐鳍鱼纲鲈形目鲳亚目长鲳科的一种,俗名䖳鲳、肉鱼、瓜核、肉鲫、南鲳、玉昌、海仓。本鱼分布于西太平洋区,日本、中国东海、黄海、南海、朝鲜半岛、台湾、香港、越南及泰国等海域。该物种的模式产地在日本。水深30-60公尺。本鱼体短而高侧扁,呈椭圆形,头稍呈圆形。被圆鳞,极易脱落,体表面分泌有粘液。鳃盖上方有一模糊黑斑,体浅灰蓝色,外罩以银白色光泽。背鳍硬棘6-7枚;背鳍软条27-33枚;臀鳍硬棘3枚;臀鳍软条24-28枚,成鱼体长可达30公分。本鱼栖息在沙质底区的底层到表层的整个海域,以浮游性动物、小鱼及甲壳类为食。幼鱼生活于表层,成鱼生活于底层。刺鲳是一种重要食用鱼,野生数量稳定,适合煎或清蒸食用。","text2":"刺鲳适合什么烹饪方式来食用?","label":1} {"text1":"红尾蝴蝶鱼,又称黄蝴蝶鱼,俗名黄网蝶,为辐鳍鱼纲鲈形目蝴蝶鱼科的其中一种。本鱼分布于西太平洋区,包括菲律宾、印尼、中国南海、东海、日本、台湾、越南、马来西亚、澳洲北部、新几内亚、所罗门群岛、诺鲁、斐济群岛、马里亚纳群岛、马绍尔群岛、密克罗尼西亚、帛琉等海域。该物种的模式产地在安汶岛。水深4至20公尺。本鱼鱼体背面为淡褐色,腹面为青灰色,上有许多平行的棕色横线,及斜向臀鳍方向的斜线互相交错,造成许多近于菱形的格子,其越往身体后方越小。而约在背鳍鳍条后半部处有块橘色区域自背鳍延伸至臀鳍处,其前缘略呈弧形。尾鳍上亦有块弧形橘色带。具黑色眼带,在额部亦有一黑斑。背鳍硬棘13枚、软条22枚;臀鳍硬棘3枚、软条16至17枚。体长可达14公分。本鱼栖息在珊瑚礁区或港湾的外缘珊瑚礁较茂密处,属肉食性,以底栖小动物为食。色彩鲜艳的观赏性鱼类,不供食用。","text2":"红尾蝴蝶鱼分布于哪里?","label":1} {"text1":"独立学校(\"independent school\")是指不依赖国家或地方政府拨款,而依赖学费收入或捐赠而运作的学校。在美国和加拿大,独立学校是私立学校的同义词。独立学校可能从属于宗教机构,但是该术语更精确的用法则排除了教会学校或其他经济上依靠外部机构的学校。在英格兰、 威尔士和北爱尔兰,最有声望的独立学校称为公学,有时归类为大小公学。校长大会(\"Headmasters' and Headmistresses' Conference\")的成员通常被列为公学,尽管其中包括许多独立的文法学校。当他们成立时,这些学校确实是“公众的”,面向那些付不起学费者;批评公学者认为他们早已失去了存在的理由,通常更喜欢使用“私立学校”一词来称呼。在苏格兰,不依赖政府资助的学校称为独立学校。在澳大利亚,独立学校这个术语也可与私立学校交替使用,独立学校是增长最快的教育部门,其中超过85%都从属于宗教机构。2006年,在澳大利亚有1,078所独立学校,491,000 名学生。一些独立学校享有盛誉,入学竞争激烈,学费昂贵,不过自1980年代起,迎合普通澳大利亚人的低学费,而且不从属于教会的学校的数目显著增加。澳大利亚的独立学校占总入学人数的15%,而通常收费较低的天主教学校也占有客观的份额(18%),通常也包括在广义的独立学校之列。1970年以来,非公立学校的入学人数稳定增长,而公立学校的入学人数已经从78%下降到67%。澳大利亚的独立学校与美国略有不同,因为澳大利亚政府根据社会经济地位得分,向包括独立学校的所有学校都提供经费。学校的社会经济地位得分得自于从澳大利亚统计署人口普查中一个人口普查区中选择的样本,家庭收入和教育数据用于为每所学校评出社会经济地位得分,然后据此核算资助额度。平均而言,对独立学校的资助为开办公立学校所需的47%,剩余部分的费用由父母支付。在美国,就读于独立学校的学龄人口只占很小的比例(略超过1%)。独立学校与其他私立学校之间的本质区别是自主管理和自主筹措经费。与此相对照,公立学校由政府提供资金和管理,而大部分教会学校由教会组织拥有、管理和资助。全国独立学校协会(NAIS)是一个美国大学预科独立学校的成员组织。独立高等教育机构的成员组织是全国独立学院与大学协会。","text2":"为什么澳大利亚的独立学校与美国略有不同?","label":1} {"text1":"Miro(原名为Democracy Player及DTV)是由开发的集媒体播放器、网络电视于一身的开源软件,自身可播放多种文件格式,也可通过下载功能下载互联网上的影片并且加以管理,支持Microsoft Windows、Mac OS X及Linux作业系统。Miro是遵循GNU通用公共许可证发布的开源软件。Miro可播放如Quicktime、WMV、MPEG、AVI、XVID等在内的多种格式。Miro可从YouTube、Google影片、Blip以及其他网站下载影片,也可以使用搜索功能在互联网上寻找自己需要的影片。Miro能自动从使用RSS的「频道」下载影片,并进行管理和播放。通过YouTube的高清影片功能,可以获得高质量的影片。","text2":"Miro可从哪些网站下载影片?","label":1} {"text1":"地址,是一串的字符,内含国家、省份、城市或乡村、街道、门牌号码、屋邨、大厦等建筑物名称,或者再加楼层数目、房间编号等。一个有效的地址应该是独一无二,有助邮差等物流从业员派送邮件,或者上门收件。除了物流业,其它例如朋友聚会、登门拜访、求职招聘、公司注册登记等,也必须有一个地址。由于文化差异,世界各地的地址字符顺序有所不同。例如中式地址顺序如上,而英式的顺序刚好是倒转,即是先写房号、门牌号数,最后是写出国家名称。中华民国门牌是由户政机关管理,而邮件主要由中华邮政寄送,因此在不同的情况必需依不同单位的要求填写地址。在实际操作中,从国外寄香港的国际邮件,只需要在信封上的详细地址最后一行写上英文「HONG KONG」即可,无需写上邮政编码。为了便于在香港内部的递送,需要用香港当地使用的语言来写清详细地址;包括地区、街道(街号如有)、大厦、楼层、门号,以便让香港的邮局分发、邮递员递送。邮编格式:目前香港政府决定香港内部继续不用邮政编码系统。地址写法英文示范:投递到家庭住宅:投递到邮政信箱:","text2":"在实际操作中,从国外寄香港的国际邮件应怎么填写地址?","label":1} {"text1":"寒鸦(学名:)为鸦科寒鸦属的鸟类,旧时归类于鸦属寒鸦亚属,别名慈鸟、小山老鸹。全球活动范围约为15,600,000平方千米。该物种的保护状况被评为无危。寒鸦的平均体重约为233.5克,体长可达35厘米;上体除颈后羽毛呈灰白色外,其余为黑色;胸和腹部为灰白色。分布于欧洲,西亚和北非各地。栖息地包括温带草原、牧草地、城市、亚热带或热带的旱林、耕地、亚热带或热带的(低地)干草原、海崖和多岩石的离岸岛屿和温带森林。一般栖息于戈壁荒漠有绿洲的树林中、亦见于河流两岸的峻峭岸壁上以及巢主要在石质或土质的洞穴或缝隙中;冬季常和秃鼻乌鸦混合成群。该物种的模式产地在瑞典。寒鸦包括四个认定的亚种 分布于欧洲的亚种混合交配:","text2":"寒鸦别名叫做什么?","label":1} {"text1":"石谷站()是一个城市铁路东郊及伊拉瓦拉线的车站,位于悉尼南部的石谷,这里附近亦是石谷地方政府区域的总部所在地。车站跟随著路线在1884年10月开幕。每小时会有4班列车,繁忙时间会加设班次。由于往邦迪联运的列车会由2号或4号月台开出,因此乘客需在大堂的电子显示器上留意下一班列车会在那一个月台上开出;同一样地,往瀑布\/克罗努拉的列车会由3号或5号月台开出,亦需要在电子显示器上留意下一班列车会在那一个月台上开出。在2006年的时间表,南海岸线列车会在繁忙时间进入此站。1号月台\"(不常用)\"2号月台3号月台4号月台
5号月台
Stand A:Stand B:Stand C:Stand D:Stand E:Stand F:Stand G:Stand H:车站设有升降机并有职员在车站开放时间当值。","text2":"该车站的路线是什么时候开幕的?","label":1} {"text1":"克里斯蒂安·尼斯林·巴纳德(Christiaan Neethling Barnard,),南非外科医生,世界上首例人类心脏移植手术的实施者。巴纳德的童年在南非开普省西博褔特度过,生活贫困,1940年,巴纳德被Beaufort West High School录取。1945年进入开普敦大学医学院,随后在那里获得学士学位。医学院毕业后,他作为一个家庭医生的助手,却不能融洽相处,最终被赶出来。他又去了开普敦大学学普通外科学。巴纳德1946年自开普敦大学毕业,1953年获得硕士学位,1956年赴美国明尼苏达大学访问学习,并开始人体器官移植方面的研究。1958年他回到南非,1967年成功实施了南非第一例肾脏移植手术。他先后尝试给几十条狗实施了心脏移植手术。1967年他终于成功地实施了世界上第一例人体心脏移植手术,病人接受移植后存活了18天。其后他又实施了不少次心脏移植手术,其中最成功的一位病人存活了超过23年。巴纳德1983年退休,2001年在塞浦路斯度假时猝死于当地。","text2":"心脏移植手术最成功的病人活了多久?","label":1} {"text1":"吉冈秀隆(),日本男演员、创作歌手。出生于埼玉县蕨市。血型为A型。亚细亚大学经济学部二年级肄业。身高172公分,体重55公斤。5岁进入若草剧团,75年参与东京电视台时代剧《大江戸捜查网》的演出。1977年出演野村芳太郎监督的『八つ墓村,饰主人公・寺田辰弥的幼年时代。之后在1980年公映的山田洋次监督的『远山的呼唤』中演出女主角的儿子“武志”一角,继而成为山田洋次的长年御用演员之一。1981年开始出演山田洋次监督的长篇系列电影『男人真命苦』),从第27部开始一直到最终章出演主人公寅次郎的外甥“诹访満男”一角,并从第42部开始,因为主演渥美清的身体欠佳,情节作以调整,寅次郎的外甥满男和满男的女朋友阿泉也成为故事的主角,和寅次郎展开了双线的情节。直至1996年渥美清病逝,『男人真命苦』也于第48部完结。在电视剧方面,从1981年开始,在富士电视台的长寿经典名作『来自北国』(日文名“北の国から”,仓本聪脚本)中饰主角之一的黑板纯,之后20余年中,一直持续拍摄特别篇,直至2002年『遗言篇』终于完结。遂后也与『遗言篇』中饰演黑板纯女友的女优内田有纪共结连理,并在『来自北国』所在的外景地北海道富良野市举行婚礼,遂成一段佳话。但由于婚后两人聚少离多,终究于2005年12月21日协议离婚。2003年首次主演民放台趋势剧,富士电视台的小孤岛大医生,得到了极高的视聴率与口碑,并在接下来相继推出了2004的特别篇与2006年的续篇,也都获得了极高的视聴率。而在2005年主演的电影「永远的三丁目的夕阳」中,因良好口碑与票房也于2007年续拍,而他也凭精湛的演技创下以同影片同角色两度拿下日本电影金像奖最优秀主演男优赏的记录。从童星时代出道至今,除了演员发展,音乐才华亦受瞩目,94年配合同名电影推出第一张自己做词作曲的单曲『ラストソング』(Last Song),于当年畅销47万张,其后他所推出的几张专辑几乎都一人包办了词曲的创作。","text2":"吉冈秀隆的身份是什么?","label":1} {"text1":"延斯·克里斯蒂安·斯科(Jens Christian Skou,),丹麦化学家以及诺贝尔奖得主。斯科生于丹麦莱姆维的一个富有家庭。他父亲马格努斯·马丁努斯·斯科是一名木材和煤炭商,母亲安妮·玛格丽特·斯科在他父亲去世后接管了公司。15岁时斯科进入西兰岛海斯莱乌的一所寄宿学校,1944年从哥本哈根大学获得医学学位,随后1954年获得博士学位。1947年他开始在奥胡斯大学工作,并于1977年被聘为该校的生物物理学教授。斯科于1988年从奥胡斯大学退休,但仍保留了在学院的办公室。1997年,他同保罗·博耶、约翰·沃克一起发现了钠钾-ATPase(钠钾帮浦),获得诺贝尔化学奖。","text2":"斯科出生在什么样的家庭?","label":1} {"text1":"黄额闭壳龟(学名:)又称梅花箱龟、梅花盒龟,为龟科闭壳龟属的爬行动物,又称梅花盒龟、花背盒龟、海南闭壳龟等。分布于北部湾,尤其是越南北部以及中国大陆的海南,亦见于广西及寮国等地,但尚未确定是否存在于柬埔寨东北部。一般生活于山区的溪流中,但海南岛的黄额闭壳龟是生活于低地的。因陆栖性强,有部分学者认为应将黄额闭壳龟和黄缘盒龟列入Cistoclemmys属。黄额闭壳龟是杂食性的,圈养个体每次产一至三枚卵。黄额闭壳龟虽被列为极危物种,并是华盛顿公约附录2、越南红皮书物种,但仍面对野生个体被捕捉食用的威胁。中国是黄额闭壳龟主要的消费国,多用作食物和医药。除此之外,分布局限、生境的破坏、不易繁殖和捕捉作宠物等问题也令黄额闭壳龟数量锐减。黄额闭壳龟的人工饲养十分困难,因此龟种适应环境能力欠佳,且十分神经质难以开食,提高了饲养的难度。日本第一个成功驯养和繁殖该物种的动物园是札幌市丸山动物园。而香港动植物公园亦曾饲养黄额闭壳龟。","text2":"黄额闭壳龟属于什么类的动物?","label":1} {"text1":"陶申·特罗曼(Dawson Trotman,),又译“道森·卓门”是福音传道士,导航会(The Navigators)的创始人。特罗曼在1933年创建导航会,导航会渐渐发展成世界性的基督徒组织。其宗旨为以基督为中心过圣灵充满的生活、活在神的话语中、强调个人跟进的重要性、一对一门徒训练还有门徒、同工和传道人的倍增。1956年6月18日,在纽约休伦湖为了拯救落水的人,特罗曼献出了自己的生命。布道家葛培礼博士说:“我知道陶申·特罗曼感动了很多人,比我认识的任何其他人都要多。”特罗曼的工作和著作在创建校园传道会(the Campus Outreach ministry)里起了重要作用。校园传道会强调门徒训练是建造大学校园基督徒团体的方式。陶申·特罗曼和他那个时代的很多布道家一起同工,包括 Henrietta Mears, Jim Rayburn, Charles E. Fuller和Dick Hillis. Lorne Sanny接替他担任导航会的领袖。1932年7月3日特罗曼和Lila Mae Clayton结婚。 其妻Lila死于2005年10月27日享年90岁,夫妻育有4名儿女。","text2":"因为什么事情,特罗曼献出了自己的生命?","label":1} {"text1":"捷克爱乐乐团是一支捷克交响乐团。2006年一次由法国杂志《Le Monde de la Musique》的调查中,它被评为欧洲十大乐团第九位。乐团成立于1896年。它的原始成员来自布拉格国家歌剧院。1905年乐团独立,1945年被国有化。1946年,乐团出席第一届布拉格之春国际音乐节以庆祝其成立五十周年纪念。乐团的第一场音乐会由捷克音乐家德沃夏克指挥。而它与其他捷克音乐家的关系也非常密切,如斯美塔纳,扬那切克和马替奴。两战期间,乐团名声鹊起,当时领导乐团的是指挥家瓦茨拉夫·塔利赫()。随后是有名的捷克指挥家拉法埃尔·库贝里克(任期为1942-1948),卡罗尔·安切尔(Karel Ančerl,任期为1950-1968)和瓦茨拉夫·纽曼(任期为1968-1989)。吉里·贝洛拉维克曾两次出任首席指挥。乐团曾和很多有名的指挥家合作,如古斯塔夫·马勒,理查德·施特劳斯,保罗·欣德米特,赫伯特·冯·卡拉扬,卡尔·伯姆和祖宾·梅塔。","text2":"捷克爱乐乐团成立于哪一年?","label":1} {"text1":"丁不四是金庸小说《侠客行》里头的人物。丁不四和两位哥哥丁不二、丁不三以六合丁氏名闻江湖,个人品性不端,原先和梅文馨是一对情侣,并生下梅芳姑,可是后来丁不四移情迷恋史小翠,途中将梅氏母女抛弃,娶不成史小翠后便居住在碧螺山。许多年后,丁不四在武昌府再度遇上已是祖母的史小翠,丁不四发挥死缠烂打邀她前往碧螺山聚会,不料被石破天解围,于是丁不四和哥哥丁不三上凌霄城打算抢劫,无论如何要史小翠沾到碧螺山的的泥土才甘心。但是史小翠因和丈夫尚未和好不在凌霄城,丁不四就捏造谎言欺骗白自在,宣称史小翠已到碧螺山和他幽会过,导致白自在醋意大发,加上石中玉和白阿绣事件处理未果,神智开始不清在凌霄城内滥杀无辜。丁不四也受到侠客岛邀请前往赴会,在那里遇到昔日老相好梅文馨,后来大家平安回归中土,梅文馨便立刻揪他前往熊耳山寻找女儿梅芳姑。","text2":"丁不四的女儿是谁?","label":1} {"text1":"《苏姗娜之歌》()是美国小说家史蒂芬金所写的长篇奇幻小说黑塔系列中的第六集。台湾由皇冠出版社出版,内地简体版则由人民文学出版社于2008年3月出版,英文原版早于2004年6月出版,当中相距近四年。曾于2005年提名轨迹奖「最佳奇幻小说」。枪侠们打败了卡拉的狼群后,苏姗娜的另一个意志米亚抢夺了苏姗娜的身体逃到了1999年的纽约,由于逃走时同时带走了黑十三,使得门洞失去了魔力,由此枪侠只好求助古老的民族曼尼族,透过-{zh-hans:曼尼族; zh-hant:曼宁}-的磁石,时空之门重新恢复了魔力,但枪侠并不能随心所欲选择目的地,结果杰克、卡拉汉神父及仔仔去到了苏姗娜的时空,罗兰及埃蒂则去到了二十三岁史蒂芬金的年代,1977年。米阿到了纽约后,由于人生路不熟,不知道如何应付,苏姗娜便以协助米阿延下婴孩为交换条件,换取婴孩的秘密,结果发现婴孩原来是罗兰的,米阿原来是吸取杰克精力的神喻,又是强暴苏姗娜的-{zh-hans:恶灵; zh-hant:通灵魔}-。虽然苏姗娜一直拖延时间,并且不断透过精神向埃蒂发出讯号,但还是等不到枪侠的救援,被送到吸血鬼及低等人的地方。由于米阿的告密,黑帮老大巴拉扎预早埋伏,枪侠一离开时空之门便遇到偷袭,幸好得到管理员卡伦的帮助,枪侠才可以逃出生天,后来他们找到了-{zh-hans:塔尔; zh-hant:卡文·塔}-及-{zh-hans:深纽; zh-hant:亚伦·狄普诺}-两人,原来深纽一直都不希望卖掉玫瑰下的土地,而且一直高调行事,惹得枪侠很不满,最终在枪侠的威逼利诱及朋友的劝告下,-{zh-hans:塔尔; zh-hant:卡文}-心有不甘地将给枪侠的泰特集团。两人后来又去了找史蒂芬金,发觉自己的确是史蒂芬金的笔下人物,但在血王的扰乱下,史蒂芬金只写了第一部的手稿就放弃了,罗兰于是利用催眠促使他继续写下去。","text2":"《苏姗娜之歌》在台湾由哪个出版社出版?","label":1} {"text1":"B2,香港二人女子演唱团体,成员分别是谭嘉荃(Amy)及郭可颖(Sandy,郭可盈堂妹)。B2参加新城电台举办的「一级班少女歌唱大募集」分别获得亚军及季军,获得BMA唱片公司合约。2000年正式出道,她们的曲风以快歌为主。于2000年10月推出首张EP《B2 Attack》。她们于2000年「新城劲爆颁奖礼」获得最受欢迎组合的第三名。及后于2001年推出了《B2同名专辑》。她们的主打歌《型男索女》的歌词被认为粗俗,而被各大电台及电视台禁播。2002年1月,她们推出了首张大碟《娥世代》。及后她们于同年的6月在台湾推出了《娥世代》的台湾版。2003年,宣布退出乐坛。Amy于无线电视担任兼职,及为节目《娱乐大搜查》担任记者及写稿工作,Sandy则到外国进修。2006年,Amy离开了无线电视,转投「香港宽频电视」任主持。直到2012年,Amy加盟汉洋主理的自由行娱乐,正式重返乐坛。","text2":"宣布推出乐坛之后,Amy做什么工作?","label":1} {"text1":"凯旋门大赛(Prix de l'Arc de Triomphe)是每年10月第一个星期日在法国巴黎隆尚马场举行的比赛。距离是草地2400米,国际一级赛,总奖金达400万欧元。3岁或以上雄马及雌马可以出战此赛事(阉马不得参与)。凯旋门大赛是目前为止最高奖金的欧洲草地赛事。第一次世界大战完结后,为了复兴赛马比赛以及寻找与巴黎大赛匹敌的赛事。因而以凯旋门命名为凯旋门大赛,第一届总奖金为15万法郎。此赛事曾经在1939-1940年因第二次世界大战影响停办。1943年在Tremblay马场举行。2008年获得卡塔尔赛马会赞助,总奖金由200万欧元增至400万欧元,成为欧洲草地赛事中最高奖金的比赛。Cadum在1925以第一名完成比赛,不过被竞赛小组研究后,降为亚军。","text2":"为什么被命名为凯旋门大赛?","label":1} {"text1":"瑞士经济是世界最为稳定的经济体之一。 其政策的长期性、安全的金融体系和银行的保密体制使瑞士成为避税投资者的安全避风港。瑞士是世界上最为富裕的国家之一,人均收入处在世界最高行列,同时有著很低的失业率和财政赤字。由于拥有发达的金融产业,服务业在瑞士经济中也占有日益重要的地位。瑞士是一些重要国际政经组织,如联合国、WTO、IMF、世界银行、OECD的会员,WEF「全球竞争力报告」,瑞士长期高居总排行世界第一。当附近欧洲国家失业率飙到20%以上,连德国失业率也达8%到10%间,瑞士失业率约3%,堪称欧洲最低。主要建立于精致工业品出口和观光业之上,但令人争议的秘密银行制度也让许多富人定期前往瑞士办理财产事务顺便进行高档旅游,人才政策采取全面开放,境内有多达124.9万名外籍工作人口,占整体就业人口超过1∕4,多数是高技术人士,并重视师徒制的技职教育体系,全国已发展出职训课程(VET)及产学专班(PET),通常是每周在校二天、工厂三天的师徒制训练。","text2":"瑞士为什么境内有大量外籍人才?","label":1} {"text1":"白阿绣,是金庸小说《侠客行》里头的人物,小说中只称呼为「阿绣」,「白阿绣」不一定为其本名。白阿绣是雪山派掌门白自在的孙女,美丽秀气,凌霄城上下将她像小公主一样疼爱。但是在她十三岁时,被十五岁的师兄石中玉绑住手脚脱光衣服企图强奸,幸亏仆人发现即时挽回名节,她仍然不堪耻辱,哭了三天后跳下山崖自尽。从雪山派出走的奶奶史小翠在山下救起白阿绣后,祖孙二人到中原散心,在武昌府偶遇对史小翠痴恋的丁不四。她和奶奶不愿接受丁不四的要胁上碧螺山,于是加紧修练武功但导致身子动弹不得,后来石破天的误打误撞,将祖孙二人救到紫烟岛。众人见到石破天均以为他是石中玉,但是白阿绣凭借直觉认定他跟石中玉不一样,让石破天犹如含冤得到昭雪,又在紫烟岛的相处,两人进而相惜相恋,日后也得到雪山派的成全。","text2":"在雪山派企图侮辱白阿绣是谁?","label":1} {"text1":"裴蓓(),中国籍模特,世界著名模特儿经纪公司IMG Models签约模特。裴蓓窜起主要凭著清纯的脸庞和俏皮的个性,不只Dior设计总监约翰·加利亚诺把她从一堆高知名度的名模中拉拔出来,裴蓓自己也透露之前参加08年秋冬Ralph Lauren秀,成为这个品牌多年来第一个华人面孔。「那个年纪很大的老头(拉尔夫·洛朗本人)看了我的照片,直接叫助理扫瞄我所有照片,然后我就受邀了。」裴蓓说。2002年参加新丝路模特大赛,获得安徽赛区的季军和最上镜奖两个奖项,开始进入模特界。在校学习设计系期间,多次参加GUCCI、Max&Co、MaxMara等名牌展示会,开始立足国际模特界。2007年签约国际最大的模特儿经纪公司IMG MODELS。曾经获得「中国模特之星」亚军与最上镜奖、中国风尚大典风尚女模提名。今年1月,裴蓓正式进军巴黎,短短两个月的时间,她自然的表现和举手投足间流露著的迷人气质,让整个法国时尚界为之惊艳。Dior创意总监约翰·加利亚诺更曾说裴蓓为他最喜爱的模特。","text2":"裴蓓是哪家模特公司的签约模特?","label":1} {"text1":"品质国际控股有限公司 ,简称品质国际控股或品质国际 ( ,),是一家在香港交易所上市的工业公司,成立于1982年。公司主要业务为制造及销售集成电路引线框、散热器与加强杆等半导体元件。公司总办事处设于荃湾合福工业大厦。2016年1月宣布每4股合并为1股,合并股份后以每股0.32港元价格配售最多8.75亿股予独立第三方,另外又以「1供5」比例进行供股,供股价每股(合并后)0.32港元。2016年10月7日,该公司向乐亚国际提出收购要约,以1股品质国际股份换取25股乐亚国际股份,对于存在的2亿份尚未行使的乐亚购股权(行使价为0.0256港元),收购方提出以3股品质国际股份换取注销500份乐亚购股权。2017年3月接纳要约期限届满后,只有约18%的乐亚股份接纳收购要约,要约因此失效。2017年7月14日举行股东特别大会,以大比数否决削减股份溢价账的提案,导致原先宣布的每股分派0.133港元的提案未能实行。","text2":"2017年股东大会做出了什么决定?","label":1} {"text1":"红翅旋壁雀(学名:),俗名爬树鸟、石花儿、爬岩树,是鸣禽亚目的一种小型鸟类。旧属䴓科,今独立出来成为旋壁雀科,之下只有旋壁雀属一个属。分布于欧洲、西南亚地区以及中国大陆的新疆、西藏、青海、甘肃、宁夏、内蒙古、四川、中国东北、河北、北京、河南、陕西、湖北、江西、安徽、江苏、云南、福建?、广东等地,多生活于非树栖高山型、栖息在悬崖和陡坡壁上以及或栖于亚热带常绿阔叶林和针阔混交林带中的山坡壁上。该物种的模式产地在欧洲南部。有关本物种的分类,鸟类学家有些少争议:在鸟类传统分类系统中,林奈最初把本物种归入旋木雀科(Certhiidae)。现时基本上确定本属物种是一个单系群,不过并所有人认同Clements (2007)把本属独立成科的主张。Dickinson (2003)认为本属充其量只能成为䴓科之下的一个亚科(Tichodromadinae)。本物种有两个亚种,基本上有地域分别:","text2":"红翅旋壁雀属于哪一科?","label":1} {"text1":"重怪龙属(学名:\"Peloroplites\")是种结节龙科恐龙,化石发现于美国犹他州艾麦里县,年代属于白垩纪早期。重怪龙的身长约5到5.5公尺,与同时代的蜥结龙相当,是已知最大型的结节龙科恐龙之一。重怪龙的正模标本(编号CEUM 26331)是一个部分颅骨,发现于犹他州的雪松山组的Mussentuchit段,年代属于阿普第阶与阿尔比阶,约1亿1600万到1亿900万年前。相同地点还发现许多骨骼化石,被标记为副模标本。模式种是雪松山重怪龙(\"P. cedrimontanus\"),是由肯尼思·卡彭特(Kenneth Carpenter)等人在2008年命名。属名意为「异常重的」;种名则是以雪松山脉为名。重怪龙的颅骨长度为56公分,宽度为35.5公分。头顶略呈圆顶状。重怪龙的前上颌骨没有牙齿;目前只有发现一颗上颌骨牙齿,形状类似孔牙龙(另一种相同时代的结节龙科)的牙齿。目前只有发现下颌的后段。除了后肢以外,已发现身体的各部分化石。重怪龙具有6节愈合的荐椎,与林木龙相同。肠骨的形状异于其他结结龙科,但这可能是化石化过程遭到外力变形的结果。重怪龙的尺骨长而笔直,与大部分结节龙科不同。距骨与胫骨之间没有愈合。重怪龙的化石发现于泥岩层,该地点还出土乌龟、翼龙类、腕龙科、甲龙科的雪松甲龙、以及禽龙类…等化石。重怪龙、雪松甲龙、蜥结龙都生存于白垩纪早期的北美洲,皆是已知最大型的结节龙科恐龙。蜥结龙的化石主要发现于怀俄明州与蒙大拿州的Cloverly组;某些发现于雪松山组的蜥结龙,可能属于重怪龙。甲龙类恐龙是群四足草食性恐龙,以低矮的植被为食。科学家们根据重怪龙的大型牙齿、结实颌部,推测牠们以较坚硬的植物为食。","text2":"什么是已知最大型的结节龙科恐龙之一?","label":1} {"text1":"《28日后》(),是一部于2002年上映的英国灾难片,为丹尼·鲍伊执导,监制,亚力克斯·嘉兰编剧。主演包括基利安·墨菲、娜奥米·哈里斯、及布兰顿·葛利森。作曲。电影于2002年11月1日在英国上映。英国某深夜,一群维护动物权益人士闯入剑桥灵长类动物研究所,欲释放在里面用作医学研究的黑猩猩。维权人士无视研究员作出「黑猩猩已感染凶恶病毒」的警告,释放了一只黑猩猩。该只黑猩猩随即袭击并感染了在场的维权人士及研究员……28天后,快递吉姆(基利安·墨菲 饰演)在伦敦一座废弃的医护中心从昏迷中苏醒。困惑的他,不停的在病房和走廊徘徊,想看看其他人都在哪里。上街察看后,却发现整个伦敦市已空空如也,仿佛已遭遇到一场神秘大屠杀。进入教堂后,却发现成堆的尸体躺在地板上,一阵突如其来的嘈杂声,牧师突然出现并攻击吉姆,吉姆立刻吓得夺门而出,惊慌又不知道到底发生了什么事的吉姆,被一群「感染者」追杀。这时,两名生还者的出现拯救了吉姆,莎伦娜(娜奥米·哈里斯 饰演)和马克( 饰演)他们利用汽油弹消灭了「感染者」,并把吉姆带到他们在地下铁的基地。并解释感染的整个状况,说这种病毒经由血液传染,全英国的人几乎都染上了这种病毒,也说病毒可能已经扩散至全世界。隔天,莎伦娜和马克帮忙吉姆回家去找父母,却已发现他们已服毒自杀。这时,天色已晚他们决定在这休息,但没多久「感染者」破窗攻击他们,「感染者」被消灭后,发现马克已遭到咬伤,莎伦娜只好把他活活打死。没多久,他们发现一栋大楼上有灯光,他们前往寻找是否有生还者。并找到了一对父女,法兰克(布兰顿·葛利森 饰演)和汉娜( 饰演),他们因为水快喝完了,急著在找下一个藏身的地方。法兰克拿出录音机并播放,听到曼彻斯特为据地的士兵在亨利·威斯特(克里斯多弗·伊莱斯顿 饰演)领军下,宣称有办法解决病毒感染的问题,并邀请生还者们加入他们的行列。在无计可施的情况下,他们动身往北方曼彻斯特所在的方向去。烂番茄新鲜度87%,基于219条评论,平均分为7.4\/10,而在Metacritic上得到73分,收获普遍好评。2006年9月1日,片商正式宣布於伦敦开拍续集,且导演一职改由担任,原导演丹尼·鲍伊则改任监制,电影于2007年5月11日上映。","text2":"《28日后》的主演有哪些?","label":1} {"text1":"威廉·杰克逊·胡克(,)是英国植物学家。胡克出生于诺里奇。父亲是一位神学家,精通德国文学,尤其喜好种植稀奇的植物。他在诺里奇中学毕业后,学习了鸟类学和昆虫学,后来开始对植物感兴趣。1809年夏季,他到冰岛考察,搜集了许多标本,但回程时发生了火灾,差点丧命,标本几乎都被毁,但他凭记忆写出了《冰岛记游》,记录了冰岛的植物和居民生活。1814年,他用了9个月时间到法国、瑞士和意大利考察植物,1816年,出版了第一部学术著作《英国叶苔属》,1820年,他受聘担任格拉斯哥大学的植物学教授,第二年,出版了《苏格兰植物》,发起成立了格拉斯哥皇家植物学会和格拉斯哥植物园。1841年,他被任命为皇家植物园首任园长,在他的主持下,皇家植物园从4公顷扩大到30公顷,种植面积达1.1平方千米,建立了许多新温室和一个经济植物博物馆。他的园长职务后来由他的儿子,同样著名的植物学家约瑟夫·道尔顿·胡克接替了。","text2":"胡克在诺里奇中学毕业后,学习了什么学科?","label":1} {"text1":"巴塞罗那-埃尔普拉特机场(,;),多简称为埃尔普拉特机场(),是西班牙第二大城市巴塞罗那的联外国际机场,乃西班牙仅次于马德里-巴拉哈斯机场的第二大机场,也是加泰隆尼亚最大的机场,由()经营,以提供西班牙国内,以及欧洲、北非的国际航线为主。埃尔普拉特机场位于巴塞罗那市区西南方,开航于1918年,是巴塞隆纳第二座启用的机场。巴塞隆纳至马德里间的穿梭航班(Air shuttle)服务,被称为「天桥」(、),在马德里-巴塞隆纳高速铁路于2008年开通前,是欧洲最繁忙的航线。在2010年,超过292万人次使用巴塞隆纳机场,较2009年成长6.5%,使其跻身欧洲第十大繁忙机场。","text2":"埃尔普拉特机场开航于哪一年?","label":1} {"text1":"中国劳动关系学院是中华全国总工会所属普通高等院校,前身是中国工运学院,2003年5月升格为普通本科院校,遂更名。本学校的前身最早可以追溯到建于1946年的中共晋察冀边区行政干部学校。学科建设以社会科学、管理科学为主,学院具有五十多年的发展历史,专门培养从事工会工作、劳动法律、劳动关系、经济管理、公共管理方面的人才。学校拥有2个校区,校本部在北京市海淀区花园村,河北省保定市涿州市东城坊镇宁村亦有分校区。这一时期,中国的改革开放事业迅速推进,社会主义市场经济体制逐步建立和发展。学院贴近工会工作实际、贴近劳动关系的变化、贴近市场展开科研和教学,在工会学、劳动法学、劳动关系、经济管理等一些学科领域积累了厚重的基础,一些领域居于国内领先水平。一个新的发展契机逐渐呈现。在学院“三位一体”的办学格局中,本科教育处于基础的地位。目前,中国劳动关系学院开设有社会工作、行政管理、劳动关系、人力资源、法学、财务管理、工商管理、劳动与社会保障、行政管理、安全工程、汉语言文学、新闻学、公共事业管理等十三个普通高等本科专业,涵盖了法学、经济学、文学、管理学、工学等5大学科门类。目前,中国劳动关系学院已初步建立起以劳动关系为主线的特色学科专业群,它由劳动关系、劳动法学、社会工作、劳动与社会保障等专业方向构成。其中:工会学专业方向形成于1985年,为国内首创;劳动关系专业为教育部批准的全国首家劳动关系本科专业;行政管理专业开设了中国首个“企事业行政文化建设”方向。","text2":"该学校处于基础地位的是哪方面的教育?","label":1} {"text1":"弗来明·戴夫斯(,Flemming Delfs),已退休丹麦男子羽毛球运动员。弗来明·戴夫斯的黄金时代为1970年代初期至1980年代初期,曾赢得丹麦全国赛及国际赛冠军。戴夫斯最特别值得注意的成就,是1977年在瑞典马尔默赢得首届世界锦标赛男子单打冠军。在那个球季(1976\/1977年)里,他几乎赢得所有其他重要比赛,包括最有名望的全英公开赛。他曾连续3届赢得欧洲锦标赛男子单打冠军(1976、1978及1980)。在球员生涯当中,他代表丹麦队参加1972年至1982年共4届的汤姆斯杯男子国际团体赛,其中两度(1973、1979)晋级决赛,但皆输给印尼而屈居亚军。戴夫斯身材高大、姿势优雅,而且反手拍力量强。在他的全盛时期,是个令人印象深刻的球员,但在湿热的远东地区比赛时往往无法称心如意。由于在1977年世界锦标赛获得第一个「真正的」世界冠军,使戴夫斯得以和当时的世界一流高手,如史温·普利、钱谷钦治、林水镜平起平坐。而其号称当世最佳的反手拍能力,居功甚伟。","text2":"弗来明·戴夫斯的黄金时代是什么时候?","label":1} {"text1":"汇豪山()位于香港九龙慈云山,是一52层高的单幢式住宅,于2008年5月落成,发展商为爪哇控股。汇豪山前身为香岛中学慈云山分校,亦是慈云山十年来的新屋苑。由于建在山丘上,一些单位可以侧面观赏到港岛东之景色。汇豪山的名字源自美国纽约著名的传统豪宅区「」,爪哇集团将这元素注入汇豪山,希望以此制造出高级住宅的形象。汇豪山地址为九龙慈云山蒲岗村道99号,同时邻近慈云山中心等建筑。汇豪山亦是位于45区小学校网中。汇豪山为一52层建筑(不设4、13、14、24、34、44字楼),合共304个单位(一梯八伙),每伙面积少于1,000平方呎 ,单位间隔分别为2房、3房(3房分2款,其中1款为套房)。而位于51层的单位则超过1,000平方呎,发展商称为「特色单位」,另52顶层为3层复式单位连天台。汇豪山设有25,000平方呎名为「Forest Place」「汇豪坊」的商场,商场共有两层,佳宝食品市场曾租用地下舖位,而上层则由食肆租用,但商场各商户已于2014年12月迁出。现今佳宝食品超级市场再次迁入,还打算扩充至第二层。","text2":"汇豪山在什么地方?","label":1} {"text1":"杨廷艺(,,部份文献写成「杨延艺」()),越南史上亦称为「杨正公」,交趾爱州人,原为静海节度使曲氏将领,当其主曲承美被中国南汉灭亡后,杨廷艺奋起抵抗,夺取了交趾的统治,因而被视为是越南自立建国的重要人物之一。杨廷艺是中国五代十国时期爱州人士,曾任交趾静海节度使曲颢的将领。当时,位于中国岭南的南汉与曲氏势成水火,曲颢之子曲承美继位后,于大有三年,930年(《大越史记全书·外纪全书·南北分争纪》作923年),南汉君主刘䶮出兵,擒获曲承美,占领交趾地区,并派骁将梁克正、交州刺史李进驻守,而杨廷艺则获南汉授以爵命,名义上成为了南汉的臣僚。据《安南志略》,刘䶮虽已得胜,但洞悉到「交趾民好乱,但可覊縻而已」,认为当地民众不会服从其统治。而杨廷艺亦为驱逐南汉而作好准备,《大越史记全书》提到,到了大有四年(931年)农历十二月,南汉官员李进向刘䶮告发「杨廷艺养假子三千人,图恢复」,廷艺随即先发制人,攻破李进所在的大罗城,李进逃归。适值南汉派遣程宝(《新五代史·南汉世家·刘隐》作「程宝」,《大越史记全书·外纪全书·南北分争纪》作「陈宝」)率兵赶来,包围杨廷艺,杨廷艺出战,击杀程宝。杨廷艺取得军事胜利后,便自称节度使,领州事。关于杨廷艺得胜后与南汉的关系,中国学者郭振铎、张笑梅作以下分析:「杨廷艺虽战败南汉军,领有交州。但他深恐南汉再次派大军来攻,为了本身的政权不受威胁,只好臣服南汉,服从南汉的诏谕,接受为交州节度使,刘䶮和杨廷艺之战宣告结束。」而在交趾方面,杨廷艺与本地的土豪结好,例如把女儿嫁给「世为贵族」的牙将吴权。南汉大有十年(937年)农历三月,杨廷艺被牙将矫公羡所杀。矫公羡行凶的原因,则是「恶廷艺所为,起所部攻杀廷艺,而代为节度。」矫公羡遂向南汉臣服,但其后亦被杨廷艺另一员牙将吴权所杀,并继续争取交趾政权的独立。","text2":"杨廷艺为什么被视为是越南自立建国的重要人物之一?","label":1} {"text1":"珍娜·荷兹(,)是一位美国电视色情演员,于2001年进入色情电影界。在2002年至2005年,她是吉儿·凯莉制作公司旗下女星。这段期间,荷兹大多只与女性表演,以表示对当时的摄影师男朋友的忠诚。她在2006年获得多项大奖的作品《Jenna Haze Darkside》中重新与男性演出。荷兹赢过多项成人产业大奖,包括2003年成人影带新闻奖的最佳新人和2009年的成人影带新闻奖的年度最佳女演出者。荷兹出生于加利福尼亚州富勒顿,成长于,也曾住在兰开斯特和明尼苏达州 。荷兹有两位姐姐,一位哥哥。 她有西班牙、德国和爱尔兰血统。她小时在学校的成绩出色,但是在初中时「发现了男孩和性」。于15岁时决定退学,开始在家自学,也找到了第一份工作。她做过一系列的低薪工作,像是速食经理及换油女郎等等。18岁时,荷兹尝试脱衣舞表现。 于19岁时,·荷兹在安纳罕一间她最爱的夜总会中,男朋友将他介绍给好友──知名色情男星彼得·诺斯和演员兼导演克雷文·穆尔黑德(Craven Moorehead),荷兹欣然接受穆尔黑德所提出的拍片邀请。两日后,她拍摄了第一部性爱场面。 在进入成人业之前,荷兹取好了艺名;珍娜此名是她在派饼餐厅工作时,为了与名为珍妮佛(Jennifer)的同事区分而所使用;荷兹是为了纪念当时的未婚夫和她最爱的歌曲:吉米·罕醉克斯的《Purple Haze》。","text2":"在2002年至2005年,珍娜·荷兹就业于哪家公司旗下?","label":1} {"text1":"丹后探索者号(,)是一列由北近畿丹后铁道所拥有、与西日本旅客铁道(JR西日本)联合营运的特别急行列车,也是该列车所使用的车型——北近畿丹后铁道KTR001形柴油车的暱称。列车名称中的「丹后」是指列车所服务的京都府北部地区在令制国时代的旧名,丹后国。丹后探索者号是北近畿丹后铁道与西日本旅客铁道联合经营、服务范围涵盖京都、大阪与丹后等地区的特急列车路网——北近畿大X网路()的成员列车之一。该列车行驶于新大阪至宫津之间,沿途经过JR福知山线、北近畿丹后铁道宫福线与宫津线等几条路线。新大阪 - 大阪 - 尼崎 - 宝冢 - 三田 -(新三田)-(相野)- 筿山口 -(谷川)- 柏原 -(黑井)- 福知山 - 大江 - 宫津 - 天桥立 - 野田川 - 丹后大宫 - 峰山 - 网野 - 木津温泉 - 久美滨 - 丰冈","text2":"丹后探索者号途径哪些站?","label":1} {"text1":"西贡户外康乐中心(英文:Sai Kung Outdoor Recreation Centre)是一个位于香港西贡的度假营,由康乐及文化事务署管理。西贡户外康乐中心位于西贡对面海区康健路21号,占地6.45公顷。西贡户外康乐中心前身原本为西贡军营,于1976年关闭,并于1985年移交予区域市政总署(康乐及文化事务署前身)改建为营地。在2003年沙士影响香港期间,中心曾用作隔离有可能染病的家庭,后来在同年5月16日重新开放供度假人士使用。西贡户外康乐中心楼高三层,有合共31个单位,每个单位有8张睡床,可供最多248人同时居住。另外亦提供日营和黄昏营设施。2006年,康乐及文化事务署管理的四个度假村,包括麦理浩夫人度假村、西贡户外康乐中心、曹公潭户外康乐中心和鲤鱼门公园及度假村,共有496000人次使用过其宿营及日营服务,而黄昏营则共有超过5万人次使用。","text2":"西贡户外康乐中心楼的结构是啥样的?","label":1} {"text1":"《小咩的管家》(),是日本漫画家宫城理子的一部爱情漫画,2006年于《玛格丽特》上连载,台湾中文版由东立出版社代理,单行本20卷已发售。从小是乌冬面店老板女儿的东云芽衣(荣仓奈奈饰演),某天父母因事故身亡只留下她一个人,却突然出现一名男子,自称是她的管家柴田理人(水嶋斐吕饰演),并告知芽衣是日本第一财团本乡家的继承人。为了把芽衣培养成真正的名门淑女,她被送进了著名的圣露丝亚女子学院。这里所有人也和芽衣一样,身边都跟著一个英俊的男管家。不同的是,大家都是真正的淑女名媛,无法适应上流社会生活的芽衣和所有人都格格不入。青梅竹马的玩伴柴田剑人(佐藤健饰演)为保护芽衣成为见习管家进入学院,再加上喜欢芽衣管家的人是学校里的老大——露丝亚,本乡诗织(山田优饰演),芽衣本来平凡简单的生活真实发生了翻天覆地的变化……2011年日本宝冢歌剧团改编成舞台剧,1~2月由宝冢歌剧团星组在宝冢Bow Hall和日本青年馆初演。","text2":"《小咩的管家》单行本有多少卷?","label":1} {"text1":"澳门瑞吉金沙城中心酒店(St. Regis Macao, Cotai Central),位于路氹城塡海区金光大道地段,是路氹金光大道中一个五星级酒店发展项目.毗连澳门喜来登酒店、澳门金沙城中心假日酒店以及澳门金沙城中心康莱德酒店。酒店于2012年第四季开始动工,提供约400间客房及套房,附设有Iridium Spa水疗中心、The St.Regis Athletic Club健身中心、Outdoor Swimming Pool 室外游泳池及两间餐厅,未来将会增设酒店式公寓服务。酒店在2015年12月17日开幕。邻近金沙城中心的Conrad,Holiday Inn和Shereton酒店.对面就是Four Seasen和威尼斯人.澳门瑞吉酒店官方网站<\/div>","text2":"酒店目前有哪些配套设施?","label":1} {"text1":"红点镜(red dot sight),部份型号又名反射式瞄准镜(reflex sight),亦作内红点,是一种无放大率的光学瞄具,有光亮的瞄准点(通常为一红点)。一些望远镜及内置取景器的「傻瓜相机」亦会载有红点镜。由于红点镜中射入眼睛的光点(光射)永远与红点镜平衡,即使眼睛不在红点镜的中轴上,也能透过红点准确瞄准,可提升高速移动或身体摇晃时的射击精确度,因此被各国大量使用于军事枪械甚至战机的抬头显示器上。另外,特殊光线除了可由发光二极管生成外,亦可由自然光或极低放射性的放射物(半永久有效)提供。红点镜与激光瞄准器不同:激光指示器是一种能将一激光红点投射于目标上的器材,而红点镜则把特殊光线由镀膜镜片(析光镜)点状反射进射手眼睛;所以目标无法发现自己身上有光点。","text2":"有哪些物品会载有红点镜?","label":1} {"text1":"三联科技(股票代号:5493)(三联科技股份有限公司,SAN LIEN TECHNOLOGY CORP),简称三联。三联科技成立于1967年(民国56年),总部设立于台湾台北,早期从事配电盘的生产、单品仪器的买卖;后以感测监控技术为核心,转型为提供各领域所需之仪控系统、生产系统,提供各种专业设备服务。其领域包含半导体、土木营建、工业感测与电脑周边等。三联科技是以自动化感测监控技术整合为产业核心,陆续引进多项先进产品与技术,延伸应用在不同领域。拥有工业机械设备事业处、系统整合事业处、微电子制程设备事业处、量测事业处,提供客户从可行性研究、采购咨询、装机、调整、试车、检校、教育训练等全方位服务。1. 自动控制系统2. GraphtecCAD\/CAM3.其他产品1. 工程安全监控2. 工业机电监控三联(Sanlien)Logo各字母所代表之意义:","text2":"三联科技成立于什么时候?","label":1} {"text1":"日本乌鲂(学名:)为辐鳍鱼纲鲈形目乌鲂科的其中一种。本鱼分布于太平洋区,包括日本太平洋沿岸、朝鲜半岛南部沿沿岸、中国、台湾、美国、加拿大、中美洲、秘鲁、夏威夷、菲律宾、越南、俄罗斯远东地区等海域。该物种的模式产地在日本。水深0至620公尺。本鱼体高呈卵圆形,吻钝,背鳍起点在鳃盖之后,鳍上覆有小鳞,体侧纵列鳞数65至75枚,尾呈叉形尾。体色为银白色,死亡后会变成黑褐色。背鳍软条33至36枚;臀鳍软条27至30枚,体长可达41公分。本鱼为深海底栖性鱼类,白天在150至300公尺深的底层栖息,夜晚则到水表层活动觅食,属肉食性,以其他鱼类、甲壳类、头足类等为食。食用鱼,全年皆可捕获,但市场上不多见。","text2":"日本乌鲂主要分布在什么地方?","label":1} {"text1":"《决战玄武门》()是香港电视广播有限公司拍摄制作的古装武打剧集,全剧共20集,监制王天林。此剧于1985年10月下午,1988年4月及1990年10月中午在翡翠台重播。以下所有影碟发行均由电视广播(国际)有限公司授权:香港发行代理商现代音像(国际)有限公司于2001年推出发行了《决战玄武门》VCD影碟零售版本,此影碟将已播放的集数并制作成12只碟作为片段完整及无删剪版本,每只碟跟电视剧的每集片长约60分钟一样,设有粤语及国语发音版本并配上繁体中文字幕。台湾发行代理商弘音多媒体科技股份有限公司于2007年推出发行了《决战玄武门》DVD影碟零售版本,此影碟烧录VCD香港版的12只碟作为集数,设有粤语及国语发音版本并配上非隐藏繁体中文字幕。","text2":"《决战玄武门》的所有影碟由哪个公司授权?","label":1} {"text1":"洛伦佐·德西尔维斯特里(Lorenzo De Silvestri,),是一名意大利足球运动员,司职右后卫,而且喜欢频繁助攻,攻防能力都相当突出。德西尔维斯特里是拉齐奥从罗马本地选拔培养的球员中最出名的一个,年仅18岁就进入一线队参加了意甲。2007年4月22日,他首次代表拉齐奥参加意甲赛事。在2006-07年球季,他上阵了两场意甲联赛。至2007-08年球季他终于崭露头角。该季他上阵了24场联赛,以出色的助攻站稳了正选位置。2007年12月19日意大利杯,他还取得了一个入球。他获得了拉素球迷的钟爱,还被拿来跟前拉素球员,现效力AC米兰的尼斯达比较。2007年11月19日,英国足球杂志《世界足球》把德西尔维斯特里选为五十位最出色的新秀之一。2009年8月,他自拉齐奥转会至佛罗伦萨。他随意大利廿一岁以下国家足球队出战2008年北京奥运,但没有取得奖牌。","text2":"德西尔维斯特几岁时参加了意甲?","label":1} {"text1":"田恒,名恒,原为陈恒,古音田、陈音近,也称为田恒。东周春秋战国时期齐国的政治人物。是齐国田氏家族的第八任首领。当时卿大夫多敬称为子,陈恒卒后谥成,故称陈成子或田成子。汉朝时,为汉文帝刘恒避讳,称为田常。宋朝时,为宋真宗赵恒避讳,«资治通鉴»亦称之为“田常”。前485年,田恒承袭父亲田乞之位,而后唆使齐国大夫鲍息弑杀齐悼公,立齐简公。田恒和阚止(又名监止,字子我)任齐国的左右相。前481年,田恒发动政变,杀死了阚止和齐简公,拥立齐简公的弟弟为国君,就是齐平公。之后,田恒独揽齐国大权,尽诛鲍、晏诸族。田成子的封邑,大于齐平公直辖的地区。史称田恒挑选齐国女子身高七尺以上为姬妾,后宫以百数,而没有禁止宾客舍人出入后宫。在田恒死的时候,有七十个儿子在旁。田恒死后,由儿子田盘继位。当陈恒(田恒)杀害齐简公的消息传到鲁国,孔子斋戒三日,请求鲁哀公讨伐齐国,鲁哀公表示鲁国势力弱小,让孔子去问季孙肥,结果遭到拒绝。后世因他的作为,指他为逆臣,导致君权更加集中、专制。庄子《庄子.胠箧》记载田成子盗齐国之事,指他为诸侯大盗。韩非称之为「田成子取齐」。也是后世常引用之成语「窃钩者诛,窃国者侯」的由来。","text2":"后世哪一常引用之成语是由田成子而来?","label":1} {"text1":"泥鳅(学名:、pond loach、oriental weather loach、weather loach)为辐鳍鱼纲鲤形目鳅科的一种,俗名鳅鱼、鱼鳅、土鳅、旋鳅、胡鰡、雨鰡。本鱼分布于缅甸与东北亚向南至中国中部。在中国分布于云南六大水系及其附属水体、四川、湖北、湖南、重庆各河流干支流、湖泊、水库、沟渠、池塘和稻田等水体,主要栖息于江河湖泊。该物种的模式产地在浙江舟山。本鱼体极修长,体侧扁或略带筒状沿长,体色为斑驳的金褐色,带有黑色斑点,背鳍1枚,约与背部同等大小;臀鳍短;位于背鳍之后,尾鳍后缘圆形,尾柄末端上方有一斑点,背鳍软条9枚,臀鳍有软条7枚。具有10个触须,体长可达28公分。本鱼栖息在河、湖泊、池塘和水田中,性情温和,属杂食性,以蠕虫、小型甲壳动物、昆虫等为食,容忍温度在 2 与 30℃之间, 而且在水中氧气不足时,能直接呼吸水面上的空气。在气压低时,牠会变得十分活跃。可食用鱼、饵钓鱼及观赏鱼,本种亦被引进至不少地方(如夏威夷),并有报告指引入后造成不利的生态冲击。","text2":"泥鳅喜欢以什么为食?","label":1} {"text1":"《皇黎一统志》(越:),又名《安南一统志》,是一部成书于约18世纪末19世纪初的越南汉文历史小说。作者是吴氏三兄弟的吴时俧、吴时悠、吴时任。小说描述了越南后黎朝的灭亡和西山朝的崛起这段时期的历史故事。《皇黎一统志》的作者及编者为吴氏三兄弟的吴时俧、吴时悠、吴时任,越南方面有时将其合称为“吴家文派”()。小说的前七回由吴时俧所撰写,吴时俧又名吴俧,字学逊,号渊密,越南山南青威人(今属河内市),为吴时任之弟,领乡荐亚元,官历佥书平章事。第八回至十四回,为其从弟吴时悠所续。吴时悠又名吴悠,字征甫,号文博,有诗文集传世。这部小说后经吴时任编辑。吴时任又名吴任,字希尹,号达轩。30岁中进士,授户科都给事中,33岁兼太原督同行参政,后为郑王世子棕日讲,世子废,擢任为工部右侍郎,曾撰写或参与编写过《十七史撮要》、《四书说谱》、《海洋志略》等书。《皇黎一统志》的最后三回未注作者,是否为吴时任所补不得而知。《皇黎一统志》共十七回,约十二万字。小说反映了公元十八世纪末期黎郑王朝内部的斗争史。小说始于郑森宠邓妃,废嫡立少,致骄兵为变,卒招西山之兵,灭郑扶黎。因朝廷空虚郑氏诸王再次向皇帝要权,黎氏少主(即昭统帝黎维祁)重用权臣阮整,与南方军发生冲突,阮整被杀。昭统帝逃离京城,沿途招募义勇与西山党领袖阮光平对抗,屡为南方军所败,最后不得不向清政府求救,乾隆皇帝命两广总督孙士毅率清军南下以扶黎氏,孙士毅骄傲轻敌,为北平王阮光平所败,只得撤离升龙城。昭统帝亦逃到南宁、桂林,后与皇太后及皇妃被清政府召到燕京(今北京),由于清政府承认阮氏政权,昭统帝被软禁,于乾隆五十八年冬十月(1794年)郁郁而死,时年二十八岁。不久,越南最后一个王朝阮朝的帝王阮福映乘西山朝内部分裂之机回国,夺取嘉定(柴棍的另一名称),打败了阮光平之继任者阮光缵,建立阮朝,越南自此南北统一。小说最后以昭统帝归葬越南作为全书的结尾。","text2":"吴时任曾经参与编写过什么书目?","label":1} {"text1":"美国教育部(United States Department of Education,缩写:ED)为美国内阁层级的联邦政府部门,由《教育部门组织条例》(Department of Education Organization Act, Public Law 96-88)成立,由吉米·卡特总统于1979年10月17日签署生效,并于次年5月4日开始运行。《教育部门组织条例》将原先的“美国卫生、教育及福利部”重组为“美国教育部”以及“美国卫生及公共服务部”,教育部由美国教育部长管理。教育部是目前最小的内阁层级联邦部门,只有约5000名雇员。教育部正式的英文缩写是ED,而不是DOE(美国能源部英文缩写)。不同于其他国家的教育部,美国教育部不干涉课程标准的设置(现今的「有教无类法案」,或称「不让任何孩子落后法案」,是个例外),地方及州政府掌管教育权并决定大部分课程。教育部主要的职权在于编列联邦补助方案,以及执行联邦关于民权及隐私的教育法案。","text2":"美国教育部与其他国家相比有什么不同之处?","label":1} {"text1":"福州义序机场现是一座位于福建省福州仓山区南台岛高盖山南侧的军用机场。在1997年长乐国际机场启用之前曾是福州唯一的机场。位于高盖山南麓,净空环境不佳,曾经有东航班机在此失事。1944年福州被日军第二次占领后,为了与盟军争夺制空权,日军在占领区加快修建机场,福州机场即是其中之一。当时日军强迫机场周围的福州仓山地区村民修建机场,包括十几岁的童工。抗日战争结束后转为民用,1949年后转为解放军军用机场,至1974年开始成为军民合用机场,曾开辟多条国内外航线,一度成为国内赢利良好的机场,于1997年福州长乐国际机场启用后转为纯军用机场。2005年后,两岸的战争可能性大大减少,曾为抗台第一线的义序机场现为南京军区驻福州空军基地的机场。2009年,中央政府大力支持海西发展,该机场已限制福州城区南扩,比如,对建筑物限高等,解放军正计划将其搬迁,但是暂无搬迁时间表。","text2":"为什么日军第二次占领福州后,加快机场修建工作?","label":1} {"text1":"海南鳅鮀(学名:),又称中间鳅鮀,为辐鳍鱼纲鲤形目鲤科鳅鮀属的鱼类。是中国的特有物种。本鱼分布于中国珠江支流山溪之中、台湾、福建、广东、海南岛和广西沿海各水系等,一般栖息于河流的底层。该物种的模式产地在海南岛。本鱼体修长,前部略圆,头胸部腹面平坦,尾柄侧扁。头平扁,吻稍尖而平扁,被有微细皮质颗粒,吻长略大于眼后头长。鼻孔后方稍隆起,眼大,口下位,上唇边缘具皱折,下唇光滑。具有4对短须,颐须3对,其基部之间有许多小乳突。体被圆鳞,腹鳍基部区前的胸腹部裸露无鳞,体背黄褐色,腹面灰白色,体侧中央有一条7至9个黑色斑块所组成的浅灰色纵纹,眼下缘至口角处有灰黑色线纹。背鳍及尾鳍呈灰黑色,其余各鳍透明,体长可达10.3公分。本鱼为初级淡水鱼,喜栖息于溪水湍急且溶氧量高的溪流底层,对环境的忍受力较低,摄食时,将吻部钻入沙中,以滤食其中的有机碎屑及其他小生物。遇敌时,会钻入沙中躲藏。可作为食用鱼。","text2":"海南鳅鮀有怎样的生活习性?","label":1} {"text1":"本尼迪克特·理查德·奥格曼·安德森(,),生于中国昆明,是美国著名的学者,专门研究民族主义和国际关系。为康乃尔大学荣休教授。著有《想像的共同体》(\"Imagined Communities\"),提出民族是想像的共同体。安德森的父亲名叫詹姆斯·卡鲁·奥格曼·安德森(James Carew O'Gorman Anderson)是盎格鲁爱尔兰人,其母为英格兰人。其弟为历史学家佩里·安德森。其祖母法兰西斯·安德森(Frances Anderson),来自一个长期支持爱尔兰独立运动的家庭,外曾祖父为梅杰·普赛尔·奥格曼(Major Purcell O'Gorman)。其祖父来自于爱尔兰沃特福德郡,其家族为苏格兰安德森家族的后裔,在1700年代移居到爱尔兰。其祖父是大英帝国武官,奉派至英国海峡殖民地槟榔屿。其父在槟榔屿出生,是中国海关总税务司中的官员。安德森在中国昆明出生,在5岁时,其父亲带全家移民美国加州,安德森在此长大。9岁时,其父去世,安德森全家移居爱尔兰。安德森中学进入英国伊顿公学,大学就读剑桥大学,在此取得学士。研究所就读于美国康乃尔大学,在此取得博士学位。安德森于2015年12月13日在印尼巴图过世于睡眠中。","text2":"安德森的著作是什么?","label":1} {"text1":"布龙齐诺 Bronzino()全名阿纽洛·迪·科西莫\"Agnolo di Cosimo\",也称尼奥洛·布龙齐诺。是意大利佛罗伦萨的风格主义画家。布龙齐诺这个昵称的来历已不可考,但是应该和他黝黑的肤色或者他常用的肖像画主题有关。现代医学有研究证明他的肤色可能是来自爱迪生氏病,因为肾上腺功能不足导致皮肤上的色素沉积。1503生于佛罗伦萨。根据他同时代的乔尔乔·瓦萨里,布龙齐诺是蓬托莫的学徒,并且他的画风受了他老师的很多影响。布龙齐诺也是一位出色的肖像画家,在大部分职业生涯里,充任美第奇家族的宫廷画家。1563年参与创建绘画学院。布龙齐诺出生于佛罗伦萨的一个屠户家。按照他同时代的乔尔乔·瓦萨里的说法,布龙齐诺最先从师于Raffaellino del Garbo, 14岁以后跟着蓬托莫学习绘画。蓬托莫曾把年幼的布龙齐诺画入他的一系列\"旧约圣经中的约瑟\"(Joseph (Hebrew Bible)|Joseph in Egypt) 肖像画作品中,这些作品现今珍藏在伦敦国家美术馆 。他的画风受了他老师的很多影响,对他老师的一生而言,两人一直相互合作。在佛罗伦萨阿诺河老桥边的Santa Felicita教堂的小礼堂里,我们时常可以看到布龙齐诺的早期手迹。礼堂内室、祭坛大作 Deposition from the Cross 和侧墙湿壁画\"Annunciation\"是由蓬托莫完成的。很显然,布龙齐诺被安排了创作穹顶湿壁画的任务,令人惋惜的是,这些作品并没有幸存下来。据乔尔乔·瓦萨里,其中四幅圆形福音绘画是由布龙齐诺完成的。然而由于他的技巧和他导师的是如此相似,至今学者们仍在为此争辩不止。在他生命的最后阶段,布龙齐诺投身于佛罗伦萨Accademia delle Arti del Disegno的诸多活动中,他是这家学院的创办人之一。画家Alessandro Allori 是他最喜爱的徒弟,公元1572年布龙齐诺过世年间,他是和Allori一家住在一起的(Alessandro是Cristofano Allori的父亲) 。布龙齐诺一生大部分时光是在佛罗伦萨度过的。","text2":"布龙齐诺是谁的学徒?","label":1} {"text1":"韩国围棋联赛地区性围棋对抗赛改名韩国梦幻联赛,只有6支队伍,总规模为3.6亿韩元改名韩国围棋联赛,发展到了有8家企业俱乐部参加,总预算达到10亿韩元。每队4人共32人中,2003年度奖金排名前16名的棋手自动获得参赛权,其他16个名额由其余的180名棋手竞争获得。此外还设立了1000万韩元的最佳棋手奖和300万韩元的多胜奖。分为季前赛、常规赛和季后赛三个阶段。奖金总规模15亿韩元,从4月28日起开始正式比赛。冠名“KB国民银行”,常规赛棋队排名将依次按照积分--胜局数--两队间胜负--两队间比赛净胜局--主将至第5台各台次胜局数的累计情况来确定。  1、BGF retail队 主教练:白大铉九段  正规:姜东润九段、李志贤五段、李元荣五段、柳珉滢四段、崔精五段  次级:李昌锡二段、洪茂镇初段  2、tbroad队:主教练:李相勋九段(大)  正规:朴廷桓九段、李东勋五段、金升宰六段、姜儒泽七段、朴珉奎四段  次级:金东昊五段、李鱼德东初段  3、韩国物价信息队 主教练:韩钟振九段  正规:元晟溱九段、白洪淅九段、安国铉五段、朴承华七段、韩态熙五段  次级:洪旼杓九段、宋泰坤九段  4、SK能源队: 主教练:崔圭丙九段  正规:朴永训九段、安成浚六段、闵详然四段、李泰贤六段、姜升旼四段  次级:黄宰渊三段、崔显宰二段  5、新安天一盐队主教练:李相勋九段(小)  正规:李世石九段、赵汉乘九段、睦镇硕九段、申旻埈三段、李浩范五段  次级:安正己初段、朴炫洙初段  6、Kixx队:主教练:金荣桓九段  正规:金志锡九段、尹畯相九段、许映皓九段、金起用七段、崔宰荣初段  次级:宋知勋初段、吴长煜二段  7、浦项队: 主教练:金成龙九段  正规:崔哲瀚九段、罗玄六段、卞相壹四段、尹灿熙五段、柳秀沆四段  次级:金民浩二段、朴材根初段  8、正官庄皇真丹队:主教练:金荣三九段  正规:申真谞五段、李昌镐九段、金明训三段、韩升周二段、朴进率七段  次级:洪基杓七段、李炯珍三段  华城koriyo队:主教练:李廷宇九段  正规:李映九九段、洪性志九段、金庭贤五段、朴正祥九段、安祚荣九段  次级:高根台九段、朴河旼初段","text2":"韩国围棋联赛的冠名商是谁?","label":1} {"text1":"布袋线,为位于台湾台南市新营区与嘉义县布袋镇间,由台湾糖业股份有限公司新营总厂经营之轻便铁路,今既废止。此线为糖业铁路客运化之始,开办于1909年。纵贯线铁路在选线时,在嘉义曾文溪间即有经过盐水港(今盐水)或新营庄(今新营)之议。详见纵贯线 (南段)。当时虽曾研议另建新营经由盐水至北门屿(今北门区)之官线铁道支线弥补不足,唯此案未成真。新营=盐水=布袋间铁道运输稍后由制糖会社完成,属于盐水港制糖兼办之客运业务。据铁道部之资料,新营庄=盐水港间五哩三分于1909年(明治42年)5月20日开始营业,亦为台湾首条糖业铁路定期营业线。然而,根据同年3月4日《台湾日日新报》资料,在官方核准开办营业线之前,当时新营-{庄}-=岸内-{庄}-间即对外办理客运(一日4往复),车资内地人(即日本人)15钱、本岛人(台湾人)10钱;1913年(大正2年)3月8日,营业区间延至布袋嘴(今布袋)。战后布袋线曾进行数次改动。包括糖铁新营车站(原址位于台铁车站旁,今已成停车场)因破烂不堪,1950年迁移百余米至今址。另外,布袋车站因位于市区之外,曾应民众要求,利用盐业铁路(台盐公司所有)延伸营业间区750m至贴近市区的半路店、但仅维持数年。沿线各车站亦有重修,大部份皆非日治时期原貌。布袋线为762mm狭轨铁路,但在新营=岸内间,另有一条并行之新岸线,为762mm及1067mm轨距之三线轨道。后者可允许台铁货车驶至岸内。新营 - 厂前 - (南信号所) - 工作站前 - 东太子宫 - 太子宫 - 南门 - 盐水 - 岸内 - 义竹 - 埤子头 - 安溪寮 - 前东港 - 振寮 - 布袋 (- 半路店)支线:东子宫 - 纸浆厂","text2":"台湾首条糖业铁路是何时开始营业的?","label":1} {"text1":"施云奴(,),全名施维奥·文迪斯·甘波斯·祖利亚(Sylvio Mendes Campos Junior),出生于圣保罗,是一名巴西足球运动员。他是一名左路球员,常担任后卫或翼卫。他是一个进攻型的后卫,经常助攻予队友入球。他在哥连泰斯展开职业生涯,从1994年开始效力,直至1999年,他被英超豪门阿仙奴看中,成为首位巴西籍的阿仙奴球员。他担任偶像温特本的左后卫位置。但他只在阿仙奴逗了两年。在这短时间,他认识了很多朋友和追求者,亦曾在对锡周三和车路士的赛事射入「世界波」。离队后,他被艾殊利高尔代替。2001年,施云奴加盟切尔达,效力了3年。2004年,他以200万英镑转会费加盟巴塞隆拿,他为球会赢得2005和06年西甲联赛冠军和2006年欧联冠军。他和云邦贺斯、奥华马斯、法比加斯、美列达、希比、亨利一样,都是曾效力巴塞和阿仙奴的。2008年他在球会有好表现,使球会和他续约至2009年。施云奴于2009年与巴塞完成合约后以自由身加盟英超球会曼城,签约一年,仅为球队上阵10场后于季后被放弃。施云奴虽曾为巴西国家队效力6场,但他却有西班牙护照,居住西班牙3年后,使他成为「欧盟球员」,不受西甲「非欧盟球员政策」限制。","text2":"巴塞隆拿球为什么会和施云奴续约至2009年?","label":1} {"text1":"《绝对冲激 ~柏拉图之心~》()是由日本多家企业结盟参与的跨媒体制作作品。各家公司取用故事设定和八宝备仁设计的角色,分别展开活动。目前已有漫画、OVA等作品。原文表记为「」,「」是误植。有这么一个都市传说:「如果能够收集到宝石『柏拉图之心』,不论什么愿望都能实现。」据说只有被选上的人可以参加宝石的争夺战,然而一切细节仍然是个谜。「」的参加企业与参与领域如下。计划参与企业共有10家,但有1家公司未公开。预定由松竹发售全5卷OVA。其中第1卷(BATTLE1)已于10月29日发售。动画音乐、主题曲由Lantis制作。OVA『』音乐集,LHCA-5093 (2008年10月22日)双叶社《Comic High!》2008年9月号至2010年1月号连载,由仓上淳士作画,单行本全3集。2008年7月7日起在手机用官方网站开始提供。各个角色的小说游戏。设定上柏拉图之心并非宝石,而是解释为特异功能,传奇色彩浓厚。","text2":"日本多家企业取用故事设定和角色做了什么?","label":1} {"text1":"中国-苏丹关系是中国和苏丹之间的外交关系。苏丹于1956年独立。中华人民共和国与苏丹共和国于1959年2月4日建立外交关系。早在1964年,中国总理周恩来、副总理陈毅与苏丹总统易卜拉欣·阿布德()进行互访,最近的一次最高级别的互访为2006年奥马尔·巴希尔总统访问中国和2007年国家主席胡锦涛访问苏丹。中国自1970年开始向苏提供一定数量的经济援助,1971年起向苏丹开始提供医疗援助,1995年中苏二国开始合作开采石油项目,1997年两国外交部建立定期政治磋商机制。2007年中苏双边贸易额56.6972亿美元,苏丹仅次于安哥拉和南非,为中国非洲第三大贸易伙伴。","text2":"中国和苏丹之间的外交关系叫做什么?","label":1} {"text1":"笔迹分析 (Graphoanalysis) 指透过书写人的笔迹,分析其行为及性格,或者核对笔迹之真伪。笔迹分析工作一般由笔迹专家进行。笔迹可以显示出性格、智力水平、思维及逻辑的具体反映。人书写的内容,能够反映其显意识;人书写的方法,能够反映其潜意识。笔迹分析可以从写者的书写风格,透析其的内心世界、个性特质、心理状态及情绪等。笔迹分析包括著重字体被书写的力度、斜度、字体结构、字与字之间的结构、格式、空格与空白位置及签名风格等等。现时于美国,有300多家公司聘请了笔迹分析师担任顾问。公司高层认为,通过笔迹分析,可以进一步地了解应征者的心理状态,把笔迹分析技术应用于公司的招聘和管理中,将雇员分配到适合自己性格的岗位,以达至人尽其才。笔迹学
","text2":"美国公司为什么注重笔迹分析?","label":1} {"text1":"南峇山()位于马来西亚柔佛州居銮,座落在居銮市东侧约5公里处,主峰高度海拔510米。那里的森林保留地是当地的旅游景点。南峇山的山路径清晰,适合初次攀爬热带山林的旅客。山可分为南山和北山,各有一条山道,登山者可选择其中一条路径,也可以同时攀北、南两峰和两个小峰,全程需时2到4个小时。北山后山有一棵圆周十多人的黄叶子巨树,是南峇山的亮点。前往后山的山路上,可见山藤和双层飞瀑。其中北山山高海拔505米,可由度假村水池旁的洋灰道,经情人桥和石山水道到半山,或由度假村后山路直上半山。半山宽阔的平台矗立着一座“欢迎登上南峇山”的牌楼,还有一座“茶亭”,石凳石桌,山水水喉,两个水池。北峰顶约500平方呎的光秃秃土地,向东可俯瞰丰盛港路工业区和山林,西边因被大树遮挡,无法看见居銮市容。南山和北山同样高度,较隐蔽。从度假村水池旁右拐,即是登南山的山径,一路上有许多树木,山顶大树林立。南峇山位于令金森林保护区内,山区里热带木种繁多,可在当地找到东革阿里,一种罕见可用药的树木。山脚斜坡也设有休闲中心。休闲中心由10余座独立小楼和一个泳池组成。","text2":"南峇山海拔多少?","label":1} {"text1":"海军陆战队两栖侦搜大队,又称中华民国蛙人部队或中华民国陆战蛙人,为中华民国海军陆战队指挥部直属大队之一。在成为正式侦搜队员之前,每位学员都必须在集训队中先受8到12周不等的「两栖专长训」,训练课程大致分为跑步、游泳、操舟等三大部分。在这8到12周的集训生活中,是每位侦搜队员最痛苦却也是最美好的回忆。在集训阶段完成前,每一期的集训队都必须通过「综合考验周」(又俗称为「克难周」)的洗礼。综合考验周共分为22堂课,这22堂课也就是先前在集训队受训期间所学到的所有课程。每位学员必须在这短短的六天五夜中,将受训的成果展现出来。综合考验周的最后一关,俗称为「天堂路」,因为-{只}-要通过该项考验就能结束犹如地狱般痛苦的训练,宛若飞上天堂一样,而得名。天堂路的考验在军教片及新闻媒体的宣传下,成为台湾民众对海军陆战队训练的一项基本印象。此部队与陆军海龙蛙兵训练内容大至相同。若要比较差异,陆战队两栖较注重毅力以及忍耐度的训练,例如爬天堂路、或是跳粪坑用粪便刷牙;陆军海龙蛙兵较注重近身搏斗及作战技能部份,例如自由搏击、格斗刀术等。天堂路是一条以棱角尖锐的咾咕石铺成、长约50公尺的路面,全身仅著游泳短裤的海军陆战队员们必须在这样的路况上以匍匐前进、翻滚等方式通过,还要做出各种指定的战技动作,而且-{只}-要教官认定动作不合格便必须重来,沿途还会被泼洒盐水。所以凡通过天堂路的队员没有不遍体鳞伤的,但许多合格的队员们都将这些伤疤视为军人荣誉的象征与珍贵的人生经历。侦搜大队目前编制共有七个中队,分别如下(括弧内是精实案前分别所属国军特种单位,现已不存)两栖侦搜中队,拥有陆海及空中侦查渗透能力,蛙人的座右铭是「高山向我低头,海水为我让路」一旦战争爆发,蛙人是最早投射到敌人阵营的部队。特勤中队,俗称黑衣部队,格言「忠孝 勇敢 敏捷 机智」,行动隐密,快速打击敌方,负责台湾南部反劫持,反恐及反劫船任务,一旦接到上级任务,特勤队能够在15分钟内完成战备整备,每位成员必须能够独立作战,三分之二队员具有七年以上的资历。爆破中队,早年接受美国海军指导,负责爆破清除水中障碍,渗透特战任务。","text2":"侦搜大队目前编制共有几个中队?","label":1} {"text1":"胡马雍陵建于1570年,位于印度首都新德里的东南郊亚穆纳河畔。此陵是莫卧儿帝国创始人巴卑尔之子,帝国第二代君主胡马雍及其皇妃的陵墓。胡马雍陵是伊斯兰教与印度教建筑风格的典型结合,并为印度第一座花园陵寝,著名的泰姬陵也是以此为范本所建。1993年被被联合国教科文组织第十七次会议列入世界文化遗产。胡马雍陵坐北朝南,墓中有一个内巴蒙式的方形水塘,陵墓内种有棕榈、丝柏等植物,主体主要采用带有浓郁印度风格的黑白色大理石及红砂岩为建筑材料。胡马雍陵的主体建筑矗立于一开有壁龛基座上,高24米的正方形陵墓,陵体的四周是四扇线条柔美的圆形孤门,陵墓顶部是一个以白色大理石雕成的半球形体,圆顶上竖著一支金属小尖塔,这是典型的伊斯兰教建筑特色。胡马雍陵另一特色是其拱门及窗户上皆雕有极为细密的格纹和几何图形。","text2":"胡马雍陵是哪两种风格的典型?","label":1} {"text1":"任天堂DSi()是任天堂所开发及贩售的携带型游戏机,它是任天堂DS及任天堂DS Lite的上级机种,也是任天堂DS系列的第三代机种。本机名称任天堂DSi中所使用的小写英文字母i是英语用作第一人称的「」,即「我」的意思,与另一任天堂家用游戏机Wii所代表的「We」,即「我们」的意思相对,而DSi就是My DS,即「我的DS」,一人一机的意思。本游戏机于2008年11月1日于日本发售,2009年4月5日和2009年4月3日分别于美国及欧洲地区发售,神游科技在2009年12月17日于中国大陆发售“iQue DSi”。2010年4月15日于韩国发售。任天堂于2008年10月2日在日本召开「秋季发表会」上公布任天堂DS第三版主机。NDSi内建两个30万像素摄影镜头,可作为拍摄用途。继续使用由Opera提供的为DSi优化的浏览器(神游iQue DSi和韩国版本不支持此功能)。可以使用主机内置的DSi商店购买DSi专用游戏,购买时需要使用任天堂点数(中国大陆命名为iQue点数)。下表比较三代任天堂DS的规格。任天堂DSi LL\/XL(英文:Nintendo DSi LL\/XL),是任天堂继任天堂DSi的衍生机型。最大特色为双萤幕尺寸加大。解析度和功能上则无任何差异。任天堂DSi LL\/XL将会与DSi并行发售,用户可以自行决定购买何种版本。DSi可以透过主机设定内的系统更新,来新增更多功能、服务或修正错误。日版DSi出厂时的版本编号是“Ver 1.0 J型”。更新时请勿关闭主机电源或切断无线网络,另外更新后也将无法复原。系统更新历史详见任天堂DSi系统软件。","text2":"任天堂DSi LL\/XL又是什么?","label":1} {"text1":"鳞首方头鲳(学名:),又称鳞首鲳,为辐鳍鱼纲鲈形目鲳亚目圆鲳科的其中一种。本鱼分布于印度西太平洋区,包括南非、莫三比克、肯亚、索马利亚、留尼旺、模里西斯、印度、日本、韩国、台湾、香港、印尼、菲律宾、澳洲等海域。该物种的模式产地在阿拉伯海。表层至水深80公尺。本鱼中等细长且侧扁,背鳍2枚,硬棘部发育完善,腹鳍比较短,起点在胸鳍基底略后,鳞较大,吻部上方亦有鳞,背鳍、臀鳍鳍条均在20枚左右,体淡色到深褐色, 鱼鳍深色;眼蓝色;幼鱼在尾部基底附近有深色条纹。吻钝,前额些微弓形。上颌无法伸出, 眼大, 直径几乎等于吻的长度, 颌齿小,成单一列,体长可达21公分。本鱼属外洋性鱼类,稚鱼时期喜群聚于漂流物之下,或随潮境漂流或栖息于水母触须下,与水母共生。食用鱼,肉嫩,适合煎食或红烧。","text2":"鳞首方头鲳幼鱼时在哪里有深色条纹?","label":1} {"text1":"利府车站()是一由东日本旅客铁道(JR东日本)所经营的铁路车站,位于日本宫城县宫城郡利府町森乡字柱田。利府是JR东日本东北本线利府支线的终点站,管辖上位于JR东日本仙台支社的范围之内,由岩切车站管理,设有绿窗口()。利府车站在初设时,原本只是东北本线沿线的一个中途站而已,但1944年起被称为「海线」、坡度较缓的新线通车之后,原本经由利府的「山线」就成了使用率越来越低的旧线。终于在1962年时,在利府~(旧)松岛之间的山线路段废线之后(至于松岛~品井沼之间的路段则在此三个月前就已废线),利府成了剩余路段的终点站。配合这改变,原本是东北本线主线的山线剩余路段,也正式改为利府支线继续营运。港湾式月台2面2线的地面车站。","text2":"利府车站是哪一年建立的?","label":1} {"text1":"组合博弈论引入了一类数学对象,称为尼姆数,它们被定义为尼姆堆的值。但是由于斯普莱格–格隆第定理,它们可以用于一大类游戏的研究。事实上,尼姆数是在序数的真类上赋予尼姆加法和尼姆乘法的运算之后形成的概念。这些运算和通常施行于序数类上的加法和乘法并不相同。斯普莱格–格隆第定理指出:每个无偏博弈等价于一个特定大小的尼姆堆。尼姆数的加法运算(叫做尼姆加法)可以用于计算等价于多个堆的单一尼姆堆大小。这被定义为对于序数的集合\"S\",mex(\"S\")定义为“局外最小序数”,也就是说序数中不是\"S\"的元素的最小一个。对于有限序数,尼姆和即是两个数进行异或运算的结果,这个结果也可以简单地通过将相加的各个数字的二进制表示逐位进行不进位的加法而得到(例如,100010+110010=10000)。尼姆数的乘法运算(尼姆乘法)可以递归地定义如下:全体尼姆数不能组成普通集合而只是真类。要是把它当作普通集合,或者考虑其任意的一个对尼姆加法和乘法封闭的子集,那么尼姆数的类可以构成一个特征为2的代数封闭域。尼姆加法的单位元是序数0,而尼姆乘法的单位元则是序数1。由于特征为2,α的尼姆加法逆元是α自身。非零序数α的尼姆乘法逆元是mex(\"S\"),这里\"S\"是满足以下条件的序数集合:若\"n\"是自然数,小于formula_2的尼姆数组成一个formula_2阶的有限域formula_4。正如尼姆加法,有限序数的尼姆积也有一些有意思的结果:尼姆数组成的最小代数封闭域是由小于formula_6的序数构成的,这里ω是最小的无限序数。因此,作为尼姆数的formula_6是尼姆数“域”上最小的超越数。以下表格列出了最小16个尼姆数的加法和乘法表。因为16是一个费马幂(形如formula_2),因此这个子集是封闭的。","text2":"斯普莱格–格隆第定理指出了无偏博弈和尼姆数的什么关系?","label":1} {"text1":"结构性失业(Structural unemployment)是指市场竞争的结果或者是生产技术改变而造成的失业,通常由于就业市场并不平衡,某些行业正扩张,另一些则衰退,造成部分工人失业。结构性失业通常较摩擦性失业持久,因为失业人员需要再训练或是迁移才能找到工作。结构性失业的出现是因为经济结构、体制、增长方式等的变动,改变了工作技能的要求,导致失业的发生。由于失业工人并不具有合适的技能,因此,若失业工人并没有接受再培训或进一步的教育,他们便不能再获聘任,结构性失业问题因而会持续,影响长远的经济发展。摩擦性失业和结构性失业被归类为自然失业,经济学家视这两种失业为正常现象。劳动人口中,属于该两种失业的劳动人口数目占劳动人口总数的百分比等于自然失业率。自然失业率:失业率(%)=(摩擦性失业 + 结构性失业)÷劳动人口×100%","text2":"为什么会产生结构性失业的现象?","label":1} {"text1":"李良年(),又名法远、兆潢,字武曾,号秋锦,晚号芋田叟。浙江秀水人。李绳远之弟。先世原籍江阴,后徒居嘉兴王店镇。生于明思宗崇祯八年(1635年),少时曾与邻女浣衣相爱,“后经年不复相见,张夜梦之”,家贫,以诸生游食四方,曹申吉延入贵州巡抚幕府,后因获悉朝廷要撤三藩,恐祸乱将作,遂离开贵州。康熙十八年举博学鸿词,冒名虞兆潢,且未及第。以后不再复出,谙建筑,筑秋锦山房于长水上梅会里之漾葭湾。康熙二十九年(1690年)应邀赴徐干学洞庭山书局,助修《大清一统志》。与沈进等友好,又与朱彝尊、李符、沈皞日、沈岸登、龚翔麟号为浙西六家。康熙三十三年(1694年)卒,朱彝尊为之作墓志。有《秋锦山房集》。良年初娶钱氏,兵科给事中桐乡钱允鲸之孙,继娶陆氏。子李潮偕,为国子监生。","text2":"浙西六家中除了李良年外,还有哪些人?","label":1} {"text1":"埔里华吸鳅(学名:),俗名石贴仔、畚箕鱼、棕蓑贴、簸箕鱼、木箕贴仔,为辐鳍鱼纲鲤形目爬鳅科的其中一种,是台湾的特有物种。台湾农委会2009年4月1日公告为应予保育之野生动物。本鱼仅分布于台湾大甲溪至高屏溪等河川中、上游。该物种的模式产地在埔里大肚溪。水深0至10公尺。本鱼体纵扁延长呈圆筒状,腹部扁平。眼小,口下位,具4对须。鱼体为浅黄或黑绿色,鳞片细小。胸鳍极宽大平展,腹鳍后缘愈合呈吸盘。尾柄部具4至6个鞍状斑,尾鳍凹形,体长可达9公分。本鱼为初级淡水鱼,栖息于溪流中、上游水流湍急之高溶氧区域。为底栖性鱼类,平时平贴于石头上,属杂食性,以岩石附生藻类和水生昆虫为食。非食用鱼,但因花纹及生态特殊,可做为观赏鱼。","text2":"埔里华吸鳅是哪里特有的物种?","label":1} {"text1":"《密谍》(原题:The Prisoner)为1967年10月1日~1968年2月4日,于英国ITV电视台播出的电视影集,由英国演员派屈克·麦古恩(Patrick McGoohan,1928~2009)身兼主演・企画・导演(部份集数),全17集。台湾则曾由台视在1968年9月24日至11月26日期间,于每周二20:30~21:30时段播出,并曾于1975年重播,但两次播出时均未播毕即告腰斩。 身为英国谍报员的主角(片中从未透露主角的本名),某天向上级递出辞呈决定辞职。可是当他回到家里收拾行李时,却被人用催眠瓦斯给迷昏。醒来之后,他才发现自己身在一处名为「村子」的国籍不明之地。在「村子」里,也有许多跟他一样的「囚犯」(the prisoner)被绑来,而每个人的真实身份都遭到隐匿,并以编号来称呼。 被赋予「6号」(No.6)这个编号的主角,每集都会遇到村中的新领导者「2号」(No.2,且担任人物会屡次更替)前来质问他「辞职的理由」与「所知的情报」,但主角却坚拒回答。「6号」屡次尝试逃出「村子」,却总是遭到「2号」阻拦,并以各种手段隐藏幕后的秘密。剧情主轴虽著重于两者之间的斗智,但其间也有许多故事开始发展。 本片虽然归类于科幻谍报类型,但却随处可见崭新前卫的表现手法与哲学风的概念,至今仍颇受推崇。括号内为英国首播日期。","text2":"醒来之后,主角位于什么地方?","label":1} {"text1":"圣若望广场()是佛罗伦萨的一个广场,得名于圣若望洗礼堂,实际上是主教座堂广场的向西延伸。目前的广场建于19世纪时,当时拆除了一部分总主教府,使洗礼堂显露出来,并使via de' Martelli与罗马路形成一条直线。圣若望洗礼堂是一座基督教建筑,可能建立在5世纪建筑的基础上,外部和内部装饰已经改变很多。绿色和白色大理石的外墙,可追溯到11世纪的罗曼式风格。在内部,是佛罗伦萨马赛克和壁画的杰作:有契马布埃和库伯·迪·马克瓦多,和威尼斯马赛克大师的作品。最古老的是安德里亚·皮萨诺的作品,作于1330年到1336年。原来放在东侧,主教座堂前面的重要位置,后来被转移到南侧。第二件在北侧,是洛伦佐·吉贝尔蒂在比赛中击败伯鲁乃列斯基的结果。第三件是洛伦佐·吉贝尔蒂的杰作,著名的《天堂之门》这组极其丰富,表现力和深度透视感的浅浮雕,面对着主教座堂的正立面。总主教府位于洗礼堂前面。原来的总主教府1533年毁于大火,1573年到1584年重建。","text2":"圣若望广场位于哪里?","label":1} {"text1":"变色树蜥(学名:)为鬣蜥科树蜥属的爬行动物,俗名马鬃蛇、鸡冠蛇。变色树蜥体长10~12厘米,尾巴长约为头体的3倍。鳞片十分粗糙;背部有一例像鸡冠的脊突,所以又叫鸡冠蛇。头较大,吻端钝圆,吻棱明显。眼硷发达。鼓膜裸露,无肩褶。体背鳞片具棱呈复瓦状排列;背中线上,由颈至尾基部有一列侧扁而直立的鬣鳞,颈部的较长,形如马鬃。四肢发达,前后肢有五指、趾,均具爪。背面浅棕色,杂有深棕斑块;眼四周有辐射状黑纹。生殖季节雄性头部为红色。体色可随环境干湿、光线强弱而变。分布于印度、安达曼群岛、中南半岛、阿富汗、斯里兰卡以及中国大陆的云南、广东、海南、广西等地,多栖息于热带和亚热带地区以及常见于林下、山坡草丛、坟地、河边、路旁、住宅附近的草丛或树干上。其生存的海拔范围为80至2000米。","text2":"变色树蜥为何又被称为鸡冠蛇?","label":1} {"text1":"拉脱维亚大学(,缩写LU),坐落在拉脱维亚首都里加市区,成立于1919年,是该国历史最悠久、规模最大的大学。2007-08学年注册学生数达23801,其中博士生900人。现任校长为Mārcis Auziņš教授。拉脱维亚大学的历史可以追溯到1862年成立的里加理工学院(Rīgas Politehniskā augstskola)。1919年9月28日,在里加理工学院的基础上成立了拉脱维亚学院(Latvijas Augstskola),1923年改称“拉脱维亚大学”。在苏联时期,拉脱维亚大学的一些学科独立出来,成立了拉脱维亚农业大学、拉脱维亚药学院、里加工业大学等高等院校。1991年,拉脱维亚从苏联独立后不久,拉最高委员会在9月18日通过了拉脱维亚大学章程,规定拉脱维亚大学是“一所为拉脱维亚国家和人民的需求服务的国立教育、科学、文化机构”。拉脱维亚大学设有13个学院,分别是:生物学院、化学学院、经管学院、教育与心理学院、地理与地球科学学院、历史与哲学学院、法学院、医学院、现代语言学院、哲学与艺术学院、数理学院、社会科学学院、以及神学院。拉脱维亚大学还设有10多个研究所,涉及到众多不同的领域和学科。另外,由拉脱维亚、瑞典两国政府以及Soros基金会共同设立的里加法学研究生院也是拉脱维亚大学的一个独立、自治单元。拉脱维亚大学是IAU、EUA、乌得勒支网络、欧洲首都大学联盟(UNICA)和波罗的海区域大学网络(BSRUN)等大学联盟的成员。除了通过欧盟的伊拉斯谟(Erasmus)等项目与欧洲范围内的大学进行交流,拉脱维亚大学还与世界上29个国家的76所大学签署了双边合作协议。","text2":"拉脱维亚大学成立于哪一年?","label":1} {"text1":"台北国际牛肉面节,为台湾一项以牛肉面为主轴的新兴节庆活动,主办单位为台北市政府产业发展局台北市商业处。该活动创立于2005年,当时名为「台北牛肉面节」。由于反应热烈,自第2年即扩大邀请国际美食家及牛肉面业者参与,并更名为「台北国际牛肉面节」。2011年台北国际牛肉面节已将台湾牛肉面的英译定为「New Row Mian」(仿日本寿司、韩国泡菜之英译,以当地语言直译,突显专有特色)2011年台北国际牛肉面节的主题是「牛香一世情」,谐音「留香一世情」,寓含尝过的人难以忘记它的香味;此外,另一层意涵则指「记忆里的留香」,主要活动项目包括:等。「2008年台北国际牛肉面节」主要活动项目如下:","text2":"2011年台北国际牛肉面节的主题是什么?","label":1} {"text1":"荷兰公共安全与司法部(Ministerie van Veiligheid en Justitie)是主掌荷兰司法的政府部门,由司法大臣担任部门主官,设有一位国务秘书(Staatssecretaris)协助大臣。公共安全与司法部原名为「司法部」(Ministerie van Justitie),吕特内阁上任后,由于原内政部掌管的部分公共安全职权转移至司法部,10月14日正式改名。司法部的法定责任有:另外,也负责反恐政策的协调。由于司法部与内政及王国关系部共同分担许多职责,也位在同一栋大楼,因此有双子部会之称。司法部首长为司法大臣与一位国务秘书,30,000名公职人员分布在海牙(本部所在地)及荷兰各地。司法部与内政及王国事务部位于同一栋大楼。正副秘书长率领整个公职部门,下辖三个总司:检察长会议(Raad van Procureurs-Generaal)率领整个检察部门(Openbaar Ministerie),是一个相对独立的组织,为司法系统的一部分。荷兰法医研究所(Nederlands Forensisch Instituut)是司法行政及执法总署底下的自治部门。","text2":"司法部的本部在哪里?","label":1} {"text1":"五牛图为唐代画家韩滉的作品,是现时存世作品中最早以牛作为题材的纸绢画。五牛图先后被宋高宗赵构、元人赵伯昂、赵孟𫖯及乾隆收藏,乾隆一直将五牛图收于紫禁城内,直至1900年,八国联军侵华,五牛图自此下落不明。到了二十世纪五十年代,五牛图才出现在香港的一个拍卖会上,最终由中方以6万港元购回,为中国十大传世名画之一,被誉为「镇国之宝」,现存于北京故宫博物院 。《五牛图》为唐德宗时期的宰相韩滉所画,图为设色纸本,纸质为麻纸,长20.8公分,总宽139.8公分,是目前中国最早的纸质动物画作,已有1300多年的历史。此图卷从右至左,一字排开,画了五头姿态各异的牛,图画中除了五头牛外,只有一小丛荆棘,没有其他任何衬景,以这种完全突显主题的方式来构图,极其大胆,充份显示韩滉高超的写实描绘能力。韩滉笔下的五牛造型非常准确,笔法简洁老练,线条劲健有力,将五头牛不同的体势姿态,筋肉骨髂,肌理皱折,甚至口鼻细毛都作了精细的刻画,生动入微。尤其对五头牛不同情绪神态的刻画格外传神,如画中最右的一头老黄牛正伸长脖子倚著荆棘蹭痒,露出舒适惬意的眼神,加之牛角呈弯月状,将老黄牛酥痒的表情刻画入骨。又如图面最左边那头伫立不动的牛, 可以看到牠的表情郁闷,炯炯牛眼似乎正瞪著看画的人;牛角朝前,有股随时要斗争撒牛气的倔态。仔细一看,这头牛的头上正套著缰绳,是图中唯一被拴束的牛,其郁闷心情不难理解。《五牛图》达到了神形兼备的高超艺术境界,元代的大书画家赵孟𫖯盛赞:「五牛图,神气磊落,稀世名笔也。」。","text2":"这幅画有什么传世价值?","label":1} {"text1":"方顺吉(),台湾高雄市凤山区人,台湾男歌手、男演员、主持人。方顺吉是小童星起家,中华艺术学校毕业。在校期间留级,霸凌同学。1992年参加台视五灯奖(与当时的冠军打成平手)。1993年参加三立《二十一世纪新人歌唱排行榜》儿童组歌唱比赛,闯关成功。1994年10月举辨「方强强滚反毒演唱会」,创国内年纪最小开三万人演唱会之歌手。1994年6月15日出版第一张个人专辑《翘脚髯嘴须》(与方婉真(方宥心)、萧玉芬两人),创国内年纪最小专辑破一百万张之歌手。1994年9月24日出版第二张个专辑《强强滚》,专辑销售破50万张。1994年11月14日与同比赛出身的童星歌手钟旎菱、邬兆邦、萧玉芬、张俊豪、慈袖静等人出合辑《美梦成真》,销售破25万张。其后又在方婉真(方宥心)《彩虹》专辑中与其合唱《彩虹》一曲。1995年1月26日出版第三张个人比赛歌曲专辑,销售破30万张。1995年6月29日出版第四张人专辑《彩色精灵》专辑,销售破30万张。1996年1月31日参与《让爱无所不在》合辑,其中收录强强滚,翘脚髯嘴须。2000年大旗唱片出版《大旗2000超级音乐快递》合辑,收录了《让爱无所不在》与大人变声版的《鼓励》两首歌曲。2002年12月参与中视闽南语连续剧《漂浪之女》记者招待会,于剧中饰演男主角「蔡顺吉」。2003年2月22日服食摇头丸被查缉,勒戒24天。2003年5月20日在高雄县燕巢乡看守所进行勒戒,6月12日完成勒戒。2003年8月19日贴钱发专辑《鼓励》,举行发片记者会,并发誓不再碰毒。2003年9月6日接拍大爱戏剧《后山姐妹》。2004年2月出版第6张个人专辑《异乡情梦》。2004年7月演出民视《蓝色水玲珑》借种大丈夫。2004年8月演出民视《亲戚不计较》。2004年8月演出台视《兄弟姊妹》。2006年卷入华视《快乐星期天》评审包小柏被殴事件,带嫌犯进摄影棚,被疑教唆打人,同年携大麻被捕,因未吸食以5万元交保。2012年1月加入天帝海国际股份有限公司顺瑛堂生技电视主持人。","text2":"方顺吉在哪一年被查出服食摇头丸?","label":1} {"text1":"造纸胡蜂(学名:\"\")是欧洲一种很普遍及著名的胡蜂。牠们在加拿大及美国是入侵物种。过冬后的造纸胡蜂蜂后会在春天的一个时间先行筑巢,并产子。最先出生的雌蜂会成为工蜂继续筑巢。雄蜂会在后来出生,一些雌蜂此时会与雄蜂交配,并离开蜂巢,在下一季成为新的蜂后。蜂群于夏天末就会消失,只余下雄蜂及未来的蜂后聚集在一起过冬。牠们很少会在原有的蜂巢过冬。在形态上,造纸胡蜂蜂后与其他负责繁殖的雌蜂没有分别。不过,由于在蜂群相互作用下不同的角色,牠们在行为上却有著分别。一般而言,蜂后是蜂群之首,牠会产大部份的卵,并且吃下参差的卵。蜂后会花大部份时间在蜂群的相互作用,而其他雌蜂则更多在觅食及照顾雏蜂。这种行为上的差别并非永恒:若蜂后离开了蜂巢,另一只雌蜂(第二大的)就会取代牠的位置。事实上,每只造纸胡蜂都会按牠们的个别的级别位置而变更其行为模式。一些研究显示蜂后会透过行为来压抑其他雌蜂卵巢的发育。蜂后相信是透过摆动腹部来发出支配的讯号,但有研究发现被去卵巢的蜂后失去了限制其他雌蜂繁殖的能力,但却仍维持其蜂后的地位。另外,雌蜂们的身份亦可以由其他因素所影响。若将可能成为蜂后的雌蜂放在蜂巢较早筑成的部份,这只雌蜂就会转而成为工蜂,相反亦然。这显示蜂群在阶级地位上有明显的灵活性。造纸胡蜂蜂后在外表上与工蜂有所分别,蜂后会以腹部打击蜂巢的表面,且会较其他下属多留在蜂巢中,故蜂巢的气味大部份是来自蜂后的。造纸胡蜂的分布地覆盖大部份的南欧及北非、亚洲的温带至中国。牠们亦被引入澳洲、北美洲及南美洲。造纸胡蜂的学名很多时会被误写为\"Polistes gallicus\",但这其实是柞蚕马蜂的学名。造纸胡蜂最初于1791年被命名为\"Vespa dominula\"。种小名的\"dominula\"意为「小妇人」,但一直以来很多学者都将之误并为\"dominulus\"。","text2":"如果蜂后离开了蜂巢会发生什么?","label":1} {"text1":"世界羽毛球联合会(),原羽毛球组织,于1978年2月成立,1981年5月26日与国际羽毛球联合会(现称羽毛球世界联合会,BWF)合并后停止运作。成立于20世纪30年代的国际羽毛球联合会(国际羽联,IBF)一直是管理国际羽毛球的机构。几个国家对国际羽联的数项政策不满:国际羽联接受中华民国为国家组织;拒绝因南非政府的种族隔离政策而中断南非的会员资格及拒绝修改国际羽联章程中的不合理条款。这几个国家在非IBF成员中华人民共和国的带领下,成立世界羽毛球联合会。于1977年9月召开由亚洲羽球联盟由发起的会议,提出了成立世界羽毛球联合会。同年11月,在世界羽联筹委会第二次会议上,非洲羽毛球联合会宣布退出国际羽联。1978年2月在香港召开的第3届会议上正式宣布成立世界羽毛球联合会。几乎全部来自亚洲和非洲的二十二个国家和地区加入世界羽联:孟加拉国、文莱、中国、加纳、圭亚那、香港、印度、伊朗、肯尼亚、马来西亚、毛里裘斯、尼泊尔、尼日利亚、朝鲜、巴基斯坦、菲律宾、新加坡、韩国、斯里兰卡、坦桑尼亚、泰国和赞比亚。 奥地利、法国、墨西哥(代表泛美羽毛球联合会)、瑞典、联邦德国和南斯拉夫派出没有投票权的观察员列席会议。之后世界羽毛球联合会组织了两届自己的世界锦标赛,一届于1978年在曼谷举行,另一届于1979年在杭州举行。在中华民国主管组织更改名称为「Chinese Taipei Association(-{中华台北羽球协会}-)」及国际羽协禁止南非羽毛球协会参加汤姆斯杯、尤伯杯和世界羽毛球锦标赛后,两个羽毛球联合会决定合并。1981年3月举行了特别会议,投票结果57对4支持合并。1981年5月26日,世界羽毛球联合会正式与国际羽毛球联合会正式合并。","text2":"非洲羽毛球联合会什么时候宣布退出国际羽联?","label":1} {"text1":"所罗门行动()是指1991年以色列政府把埃塞俄比亚犹太人空运回以色列的行动。早在海尔·马里亚姆·门格斯图成为总统之前,1984至1985年的摩西行动和约书亚行动中已有约8000名埃塞俄比亚犹太人送回以色列。在1990年,以色列政府和军方已注意到门格斯图政权不稳,1991年5月,衣索比亚的门格斯图政权倒台,厄利垂亚和提格雷族的反政府武装部队取得政权,以色列政府和犹太组织决定把在衣索比亚的犹太人空运回以色列,以助他们躲避内乱及重返以色列。5月24日行动正式开始,该行动在36小时内,利用以色列空军的C-130和以色列航空的波音747货机,把14324名犹太籍衣索比亚人运回以色列。5月24日,一架以色列航空的波音747货机则一次运载1122人(已登录的只有1087人,不少未登录的都是在母亲怀内的小童),其中有5人是在该航班上出生的。然而该行动并没有把所有犹太籍衣索比亚空运回以色列,。","text2":"以色列政府选择用什么工具运送犹太人?","label":1} {"text1":"丁丁在西藏(法语: Tintin au Tibet ;英语: Tintin in Tibet )是丁丁历险记的第20部作品。作者是比利时漫画家埃尔热。从1958年9月到1959年11月每周连载于丁丁杂志,并在1960年出版书籍。据埃尔热表示,这是他最喜欢的一部作品。《丁丁在西藏》已被翻译成32种语言,一方面被高度批评,另一方面被赞扬,被达赖喇嘛授予真理之光奖。丁丁最重要、也最珍视的中国朋友——张充仁,在乘坐飞机时失事,丁丁梦到张在雪地中满身是血向他求救,出发前往失事地点西藏,决定在冰天雪地中找到他深信还活着的朋友。尽管所有的人都认为这是不可能的事,也无法改变丁丁的决心。1957年10月,埃尔热将绘制好的《红海鲨鱼》的封面寄给了卡斯特曼出版社,并在之后的几天时间构思他的下一部作品。2006年6月1日,在布鲁塞尔,国际声援西藏运动组织达赖授予埃尔热的遗孀范妮·罗德威尔此书真理之光奖。《丁丁在西藏》被改编成电视 、广播 、纪录片、戏剧 和游戏。《丁丁历险记:独角兽号的秘密》(英语:The Adventures of Tintin)是一部2011年上映的美国动画影片,改编自著名比利时漫画家埃尔热的同名漫画作品。本片由史蒂芬·斯皮尔伯格执导,彼得·杰克逊担任制片人。本片获得重大的成功,上画后不久便在夺下法国全年总票房的第四名宝座,更得一致的好评。过了不久更得到第39届安妮奖等一共43项提名,其中包括“最佳动画”。其后更在第69届金球奖获得“最佳动画”名誉。本片被外界传媒认定为“本世纪最成功的真人动画”。本片是史蒂芬·斯皮尔伯格的第一套动画。埃尔热. 中国少年儿童出版社. 译者: 张敏. 2002-4-1. ISBN 9787500760825.","text2":"这部作品在何时何地被连载的?","label":1} {"text1":"扁咽齿鱼(学名:)为辐鳍鱼纲鲤形目鲤科扁咽齿鱼属的鱼类,俗名鳇鱼、小嘴鱼、小嘴巴鱼、草(地)鱼,是黄河的主要经济鱼类之一,分布于黄河上游及湖泊等。 本鱼体色土黄色背面与上侧面,腹鳍与臀鳍淡黄色、背鳍与尾鳍鳍灰蓝色。吻钝圆,身体裸露,侧线鳞片阴暗,尾柄短,体长可达60公分,可做为食用鱼。极边扁咽齿鱼曾是黄河上游的优势种群,生活在海拔3000米以上黄河上游宽谷河段。这种鱼能生长至10公斤以上,由于过度捕捞和生态环境恶化,逐渐稀少被列入《中国濒危动物红皮书:鱼类》,一度甚至被认为已灭绝。高原裂腹鱼类专家曹文宣院士表示:『以极边扁咽齿鱼为代表的黄河上游特有鱼类的不饱和脂肪酸含量高,有较好的食用价值和药用价值。...不仅要保护和恢复黄河上游鱼类资源和水域生态,还要进一步探索规模化养殖技术,发展成为甘肃的特色产业。』","text2":"扁咽齿为什么有较好的食用价值和药用价值?","label":1} {"text1":"李良年(),又名法远、兆潢,字武曾,号秋锦,晚号芋田叟。浙江秀水人。李绳远之弟。先世原籍江阴,后徒居嘉兴王店镇。生于明思宗崇祯八年(1635年),少时曾与邻女浣衣相爱,“后经年不复相见,张夜梦之”,家贫,以诸生游食四方,曹申吉延入贵州巡抚幕府,后因获悉朝廷要撤三藩,恐祸乱将作,遂离开贵州。康熙十八年举博学鸿词,冒名虞兆潢,且未及第。以后不再复出,谙建筑,筑秋锦山房于长水上梅会里之漾葭湾。康熙二十九年(1690年)应邀赴徐干学洞庭山书局,助修《大清一统志》。与沈进等友好,又与朱彝尊、李符、沈皞日、沈岸登、龚翔麟号为浙西六家。康熙三十三年(1694年)卒,朱彝尊为之作墓志。有《秋锦山房集》。良年初娶钱氏,兵科给事中桐乡钱允鲸之孙,继娶陆氏。子李潮偕,为国子监生。","text2":"李良年又名什么?","label":1} {"text1":"法尔奈克一世(希腊语:), 本都王国第五任国王,是米特里达梯三世的儿子。统治期间不确定,可能约在前183年-约前155年。关于法尔奈克一世即位时间不是很确定,但至少在前183年当上本都国王,并拿下之前本都国王意图染指的锡诺普。之后,罗德岛向罗马抱怨法尔奈克一世的侵略企图,但没有后文。同一时间,法尔奈克与邻近的帕加马国王欧迈尼斯二世发生纠纷,两国都派出大使到罗马,互相指控对方的恶行。前181年春天,不等出使罗马的大使归国,法尔奈克率军入侵帕加马,并与卡帕多细亚国王阿里阿拉特四世和比提尼亚国王普鲁西阿斯二世交战,战争很快得被罗马元老院所派的代理人中止,并厘清这次冲突因果。但法尔奈克一世以罗马的要求极不合理而拒绝,使协商破局,战争再度开启。期间经过不同缘由中断几次,最终于前179年夏天结束,因法尔奈克觉得在也无力与帕加马、卡帕多细亚和比提尼亚等势力对抗,被迫失去所有获得的加拉太和帕夫拉戈尼亚领土来换取和平,但本都依旧保有锡诺普。法尔奈克的统治期不是很确定,在一些资料透露他在前170年仍在位,并确定于前155年逝世,因其兄弟米特里达梯四世此时已当上国王。古希腊历史学家波利比乌斯描述法尔奈克是个自大且充满暴力的君主。","text2":"为什么一些资料透露法尔奈克在前170年仍在位,并确定于前155年逝世?","label":1} {"text1":"新白河车站()是一位于日本福岛县西白河郡西乡村字道南东,由东日本旅客铁道(JR东日本)与日本货物铁道(JR货物)所共用的铁路车站。新白河车站是JR东日本所属的东北新干线与在来线干线东北本线的交会车站,其实际位置位于西乡村与隔邻的白河市之交界附近(部分新干线月台则已经伸入白河市的范围内),是全日本唯一一个设址于村级地方自治体的新干线车站,但实际上却是白河市的门户站。新白河车站当初是日本国有铁道在1944年时设的一个号志站(),原名磐城西乡,1959年时升格为磐城西乡车站。但一直到1982年东北新干线通车后,才配合日本国铁的惯例,将车站以主要服务城市命名,而改为新白河。除了客运服务外,目前JR货物在新白河仅设有公路货运()服务的代收站,除此之外并无定期的货物列车在此站停靠,也没有相关的装卸设备。在1994年之前,新白河原本设有通往三菱制纸白河工厂的专用支线,主要是用在燃料用石油、化学药品与纸制品的输送,但1994年时工厂减产之后专用线也随之停用,目前纸制品主要是靠货柜或篷车(有盖货车)来运输。新干线为对向式月台2面2线的高架车站,并设有通过线2条。在来线为侧式月台1面1线与岛式月台1面2线,合计2面3线的地面车站。","text2":"新白河车站的原名是什么?","label":1} {"text1":"叶齿鳄(学名:\"Phyllodontosuchus\")是种原始鳄形类动物,属于喙头鳄亚目。目前已发现一个颅骨、下颌化石,发现于中国云南省,年代为侏罗纪晚期。叶齿鳄的牙齿形状为叶状,类似草食性恐龙的牙齿,显示牠们是草食性动物,与其他鳄形类动物不同。叶齿鳄的正模标本(编号BVP568-L12)是个被压碎的颅骨与下颌,发现于中国云南省的下禄丰组,年代属于侏罗纪晚期的锡内穆阶。同一地点还发现一个摩尔根兽的颅骨。叶齿鳄的颅骨长7.14公分,保存状态不佳。这个标本最出被认为是个早期鸟臀目恐龙的化石。颅骨的骨头间没有缝合处,显示这个小型动物已经是个成年的个体。叶齿鳄是在2000年由Jerald Harris等人所叙述、命名,模式种是禄丰叶齿鳄(\"P. lufengensis\")。属名意为「叶状牙齿的鳄鱼」;种名则是以发现化石的禄丰组为名。叶齿鳄的上颌两侧各有17或18颗牙齿。最前5、6颗牙齿呈圆锥状,微弯,而后方12颗牙齿呈叶状、铲状。牙齿的形状并不一致,显示牠们是异齿型动物。叶齿鳄的牙齿型态类似某些原蜥脚类、或早期鸟臀目恐龙。但是,叶齿鳄的牙齿缺乏棱脊,与早期鸟臀目恐龙(例如赖索托龙)不同;叶齿鳄的牙齿也缺乏锯齿状边缘,与上述草食性恐龙不同。叶齿鳄也缺乏鸟臀目恐龙的前齿骨。科学家根据上述特征,认为叶齿鳄最类似喙头鳄类。目前已发现数种异齿型的喙头鳄类,例如:裂头鳄、黄昏鳄、\"Pedeticosaurus\"、喙头鳄。这些喙头鳄类的颌部牙齿有类似的型态,前方皆为尖状、微弯的牙齿,中间的牙齿略尖,而后段是接近叶状的牙齿。另外,有数种后期的小型鳄形类也具有类似异齿型牙齿,例如:贫齿鳄、喀迈拉鳄、马拉威鳄。科学家认为上述的鳄形类动物可能是草食性动物,而非一般鳄形类的肉食性、食鱼性食性。如果叶齿鳄是草食性动物,牠们可能咬碎植物直接吞下,而不经由研磨的阶段。","text2":"叶齿鳄的颅骨有多长?","label":1} {"text1":"姆拉登·克尔斯塔伊奇(塞尔维亚语:Младен Крстајић,)出生于前南斯拉夫的泽尼察,是一名已退役的塞尔维亚足球运动员,司职后卫。卡斯达积出生于波斯尼亚和黑塞哥维那境内的工业城市泽尼察,父母均为塞尔维亚人,其父亲来自扎布利亚克(Žabljak),而母亲则来自比耶利纳(Bijeljina)。这名身高达到1.91米的后卫自小在波斯尼亚成长,出身于家乡球队NK些里克(NK Čelik Zenica)的青训系统。由于波斯尼亚在1992年发生波士尼亚战争,年仅18岁塞尔维亚裔的卡斯达积举家迁移到塞尔维亚的基金达(Kikinda),加盟当地球队在塞尔维亚甲级联赛角逐的基坚达(FK Kikinda)。卡斯达积表现出色,于1995年获得国内两支劲旅柏迪逊及贝尔格莱德红星的青睐,由于卡斯达积全家均为柏迪逊的支持者,故此选择加盟柏迪逊。效力柏迪逊四年期间,他共赢得3次塞尔维亚和黑山联赛冠军(1996、1997及1999年)及1次南斯拉夫杯(Yugoslav Cup,1998年)。卡斯达积于2000年转战德甲,以250万马克身价加盟云达不莱梅,签约三年,在2004年协助球队赢得德甲及德国足协杯双料冠军。同年7月17日卡斯达积转投史浩克零四。2007年12月卡斯达积与队友拉杰迪(Ivan Rakitić)在赛后夜游而被球队在欧联对洛辛堡禁赛。于2009年3月17日接替博登担任球队队长。2009年夏季,在德甲效力9年后,卡斯达积鸟倦知还,自由转会加盟母会柏迪逊,签约两年。卡斯达积于1999年9月首次披甲代表塞黑国家足球队出战马其顿,他是塞尔维亚被称为「四大铁卫」之一员,其余三位为柏迪逊的加维兰锡(Goran Gavrančić)、曼联的维迪(Nemanja Vidić)及西维尔的德古天奴域(Ivica Dragutinović),四人合作在2006年世界杯外围赛10战只失1球。在2006年世界杯分组赛以0-6惨败给阿根廷后,卡斯达积一度想在世界杯完结后退出国家队。","text2":"姆拉登·克尔斯塔伊奇的父亲来自哪儿?","label":1} {"text1":"人民路是纵贯中国江苏省苏州市古城区的一条交通主干道,南北走向,位于苏州古城中心略偏西处,北到平门桥,南到人民桥,全长4681米,宽40米,双向4车道。属于姑苏区管辖。途径苏州的观前和南门两大商业区。苏州人民路在唐宋时期原名大街,后因其南起文庙书院巷,北到北寺塔,形如卧龙,名为卧龙街。清代乾隆南巡,苏州百官在此护驾,于是改称护龙街。为3米宽的石板街。书院巷以南至沧浪亭,即文庙东侧,原名三元坊,此处原有1781年为连中三元的钱棨所立的牌坊。1928年,察院场(观前街口)以北段拓宽为10米左右的碎石马路,并向北延伸至平门桥,火车站,形成直接沟通观前街与火车站的干道。以后南段也逐渐拓宽。1980年代以后,在拓宽到目前的宽度。","text2":"人民路经过苏州的哪两个商业区?","label":1} {"text1":"白兰氏(英语:Brand's),是一家食品公司的品牌,著名于鸡精饮料,产品已经有170年历史以上。白兰氏品牌由食益补太平洋有限公司(英文名:Cerebos Pacific Limited)所有,而食益补是新加坡交易所主版上市公司,在1990年已经由日本三得利公司收购其股权。在1820年,白兰氏鸡精由英国白金汉宫御厨韩温白兰(H.W.Brand)发明,之后白兰先生于1835年创立白兰氏公司,在英国设厂生产白兰氏鸡精。2003年,白兰氏健康博物馆于台湾彰化县鹿港镇彰滨工业区开馆,除了企业相关主题外,其鸡精制造工厂以「空中走廊」的方式,开放一般民众参观。鸡精是白兰氏的主要产品,除传统鸡精外,亦开发添加冬虫夏草、花旗参等传统保健药材的鸡精。白兰氏另开发多种保健食品,饮品类包括红胶原青春饮、燕窝、蚬精及养参饮等。锭类包括维他命、深海鱼油、五味子芝麻锭。","text2":"白兰氏另开发的多种保健食品中,饮品类包括哪些?","label":1} {"text1":"陈衡哲(),中国女作家,原名陈𪆒,字乙睇,笔名莎菲。她祖籍湖南衡山,出生于江苏省武进县。1914年,她考取清华大学在上海留美奖学金,卜美国瓦萨学院(Vassar College)获文学学士后入芝加哥大学(University of Chicago)攻读西洋史获硕士. 1920年9月27日,与任鸿隽结婚。在1917年1月胡适的《文学改良刍议》出现之后,陈衡哲的白话小说《一日》于1917年5月发表在《留美学生季报》上。因此引起了中国文学史上第一篇白话小说是否是《狂人日记》的争议。。1920年,获硕士学位的陈衡哲,受蔡元培之邀回到北京大学任教,开讲西洋史和英文课,成为中国第一位女教授。一年之后,她怀孕生女,辞职养育孩子的同时,写作了“女性视角”的《西洋史》。她认为,女性应该尽力发展双重角色:母职和独立人格。陈衡哲是当期之才子也。病逝于上海。她与丈夫任鸿隽都是胡适多年的好友,1920年时胡适还曾在《新青年》上作诗《我们三个朋友》赠予他们夫妇二人。胡适的女儿素斐的名字则来源于陈衡哲的英文名Sophia。","text2":"陈衡哲的白话小说《一日》发表后引起了什么争议?","label":1} {"text1":"天正10年(1582年),准备进攻阿波国的长宗我部元亲与十河存保在中富川交战,最后长宗我部元亲胜利,而十河存保则退却到胜瑞城。在充当三好氏靠山的织田信长死于本能寺之变后,亟欲攻打阿波国的长宗我部元亲在8月上旬领两万三千大军攻打十河存保,十河存保果断地放弃一宫、夷山两城,把兵力收拢在胜瑞城抗战,长宗我部军在8月27日集结在中富川南岸的中岛,十河存保则将先锋两千人安置在中富川河畔的川砂,并将本阵设置在矢上城。翌28日,长宗我部军分两路渡过中富川夹击十河军,在三好方的矢上城主矢野虎村被香宗我部亲泰讨取,且七条兼仲、寒川元隣、奈良元政等将领相继战死后,三好方越趋不利,十河存保接受东村备后守却劝谏撤回胜瑞城防备,但旋即被长宗我部元亲的两万大军重重包围,十河存保苦苦支撑到9月21日后终究开城逃跑,撤往赞岐国的虎丸城继续对抗长宗我部元亲。","text2":"中富川之战发生在什么时候?","label":1} {"text1":"菜豆(学名:)通称包括云藊豆、白肾豆、架豆、芸豆、刀豆、玉豆、去豆、四季豆等,一年生豆科植物,是餐桌上的常见蔬菜之一。油豆角(\"Phaseolus vulgaris\" var. \"chinensis\" Hort.)是蔓生菜豆在中国东北的优质软荚变种。以荚内油分多得名,有500年的栽培历史。 油豆角主要是以食荚为主,烹煮后豆荚软面,纤维少,豆香味浓,蛋白质含量较高。矮菜豆,或者矮生菜豆(\"Phaseolus vulgaris\" L. var. \"humilis\" Alef.)是菜豆种的一个变种,就是广州常见的龙牙豆。中南美原产。一般认为16世纪末经由欧洲传到中国,17世纪传到了日本。四季豆的多个品种均含植物血凝素(phytohaemagglutinin,PHA),当中以红芸豆的含量最高,白芸豆的植物血凝素只有红芸豆的三分之一,而扁豆(\"Dolichos lablab\")的植物血凝素含量则只有红芸豆的5%到10%。所以在煮食四季豆时,要在100度的沸水中煮至少10分钟,以去除豆内的植物血凝素,然后才可继续烹煮。至于干豆,根据美国食物及药品管理局(FDA)的建议,在食用前应把干豆浸在水里至少五小时,然后把浸过豆的水倒走。假若四季豆没有在沸水里煮过,植物血凝素的毒性不单不能消除,还可能会增加。FDA指四季豆在摄氏80度的水里煮,植物血凝素会增加至未煮前的五倍。斑豆()是一种有斑点的菜豆,是等北美洲南部最常见的豆类,在美国及墨西哥西北部均有生长。斑豆的烹调方法通常都是用来煮肉汤或制成斑豆泥;而煮好的斑豆或斑豆泥,均是卷饼的饀料。斑豆的幼嫩豆荚亦可吃,有时会被当作青豆或四季豆的一种。","text2":"矮菜豆是什么类型的豆类?","label":1} {"text1":"铁电性()是某些材料存在自发的电极化,并在外加电场的作用下可以被反转的特性。该术语被用于类比铁磁性,其中,材料表现出永久磁矩。当铁电性于1920年被Valasek在酒石酸钾钠中发现时,铁磁性就已经被知道。其英文术语的前缀\"ferro\",意思是铁,被用来描述属性尽管大多数铁电材料不含有铁。多数材料的极化是与外加电场成线性正比的,非线性效应是不显著的。这种极化叫做介电极化。有些称作顺电体的材料,其线性之极化效应更加显著。于是与极化曲线斜率相对应的介电常数为一个外加电场之函数。除了非线性效应外,铁电材料中还存在自发极化。铁电材料的不同之处在于它的自发极化可以在外加电场作用下被反转,产生一个电滞曲线。一般来说,材料的铁电性只存在于某一相应温度以下,称为居里温度。在这个温度以上,材料变为顺电体。铁电材料的非线性性质可以用来制造电容可调的电容器。一个铁电电容器的典型结构是两个电极夹一层铁电材料。铁电材料的介电常数不仅可以调节,而且在相变温度附近值非常大。这使得铁电电容器与其他电容器相比体积非常小。带有滞归特性的自发极化的铁电材料可以用来制造存储器。在实际应用中,铁电材料可以用来制造电脑和RFID卡。这些应用通常是基于铁电薄膜,这样用一个不太大的电压就可以产生一个强大的矫顽场。铁电材料内部的电偶极子与材料的晶格密切相关,于是材料晶格的变化将导致材料自发极化的变化。自发极化的变化将产生一个表面电荷。由此,在铁电电容器当中,即使没有外加电压,电流也会产生。改变晶格的两个因素是力和温度。外加的机械应力可以产生表面电荷的性质称作压电性,温度的变化导致自发极化的变化的性质称作焦电性。","text2":"一个铁电电容器的典型结构是什么?","label":1} {"text1":"《银河骑士》(The Adventures of the Galaxy Rangers)是美国与日本东京ムービー于1986年所合作,自1986年9月14日~12月11日之间,共播映65集(每周一~周五带状播出)的动画作品。台湾于1987年至1988年间于每周五下午5:30~6:00时段由台视引进播出。公元2086年两位外星人和平使节来到地球寻求人类的合作与协助,也为了报答人类的对于合作的好意,他们慷慨捐出一项非常珍贵的礼物——『超驱动能(hyperdrive)』,使人类首次克服时空之限制,能够更轻易跨入大宇宙中的银河,展开了外太空探索的崭新一页。在地球本身方面,为了地球自身的生存安全与大宇宙的命运前途,立即组成了『宇宙防卫总部』,担负起打击恶魔的神圣使命,以便共同对付一帮存心在大宇宙中破坏和平安宁的恶徒。每位队员都有配戴藏有能源传继系统的『第五系列太空警徽(Series Five Implant)』,只要当负担打击来敌时按动所佩带的警徽,立刻就可以使自己的「能」力因扩增而达于极点,拥有风驰雷掣难有匹敌者之力。","text2":"《银河骑士》是什么?","label":1} {"text1":"XEvil 为横向卷轴动作游戏,由Steve Hardt在麻省理工学院开发,属于自由软体。游戏模式以free-for-all对战为主,玩家互相对抗,或是对抗电脑。本游戏被评为\"有史以来最生动暴力的电脑游戏之一\",其暴力成份引发了不少争议。本游戏首建于1994年,现已存在多种不同平台的版本可供下载。The game is available for anyone, and leaves a large pool of characters to pick from. 如同多数的线上游戏,其故事线并不完整,而就其属的类别来说,其实也非必要。The game has been reviewed and被认为是暴力且具成瘾性,本质上却极为简单。\"XEvil\" 为第三人称二维横向卷轴游戏。随机生成关卡,玩家也用随机角色攻击敌人但是也可以通过梯子以及电梯过关。作为自由软件,能在世界各地各种不同的作业系统上进行游戏。游戏中还包含了\"Kill-Kill-Kill\"-模式,这个选项会让游戏变得更加困难。游戏的唯一目标是生存,可以单人或多人模式进行。\"XEvil\"背后的故事是,你是死后的灵魂,正被送往地狱。。。为了决定你在地狱中的位阶,你必须在死亡对战中竞争,来决定你的地位。玩家的角色为随机决定,各有各的优势跟劣势。角色有忍者,步行机器人,英雄,异型,螺旋桨人,杀手(由螺旋桨人制造),火恶魔,狗,雪怪,龙等等。\"XEvil\" been called by the online community \"fast action\"and \"is a must-have for anyone interested in straightforward mayhem.\"Finally the game \"features an incredibly addictive gameplay.\"GamezWorld, who gave the game a 7 out of 10, commented \"If you like little shooting orgys, you may like XEvil.\"重要开源游戏列表","text2":"这个游戏最受争议的部分是什么?","label":1} {"text1":"突尼斯航空(),是突尼斯的国家航空公司,成立于1948年,总部设在突尼斯首都突尼斯市。该公司以突尼斯—迦太基国际机场为枢纽,经营国内航线以及前往非洲、亚洲及欧洲一些城市的国际航线。突尼斯航空是阿拉伯航空运输组织的成员。1948年,突尼西亚政府与法国航空达成同意,建立了突尼斯航空。于1949年开始服务。在1957年,法国航空把部分股份数目由降低,然后突尼斯政府就成为最大股东。1990年,突尼斯航空买了2架空中巴士A320,并保持以前的涂装。1993年-1998年期间,突尼斯航空开始扩展业务至欧洲,增加了往布拉格、布达佩斯、华沙、布拉迪斯拉发、里斯本、林兹、萨尔茨堡、格拉茨、莫斯科、贝鲁特和斯德哥尔摩。但只有往贝鲁特和斯德哥尔摩是正常的定期班次,往里斯本是季节性班次,其他欧洲目的地都是包机班次。之后往莫斯科的航班还中止了。1998年10月21日,突尼斯航空庆祝50周年成立,同时也开始了使用波音727型飞机。1999年,突尼斯航空和法国航空签署了一项协议,内容是组织联盟。空中巴士A330进入了突尼斯航空机队后,突尼斯航空有可能会开设航班往加拿大蒙特利尔。在2008年9月,突尼斯航空共有53个目的地飞越非洲、亚洲和欧洲。截至2017年3月,突尼斯航空机队平均机龄15.1年,拥有以下飞机:","text2":"突尼斯航空共有多少个目的地飞越非洲、亚洲和欧洲?","label":1} {"text1":"刺海马(学名:)为辐鳍鱼纲棘背鱼目海龙科海马属的鱼类,俗名长棘海马。分布于印度太平洋区,包括日本、新加坡、红海、台湾岛以及东海、南海、夏威夷群岛、法属波里尼西亚等海域,栖息深度可达82公尺。该物种的模式产地在日本。 本鱼背鳍软条18枚;臀鳍软条4枚;胸鳍18枚。体环11+35-36。头冠不高,尖端具4-5细而尖锐的小棘。吻细长,呈管状;吻长大于或等于眶后之头长。体上各骨环接结处及头部的小棘特别发达,仅后部尾环的小棘不甚明显。这是刺海马有别于其他种类的特征。体为淡黄褐色,背鳍近尖端具一纵列斑点,臀、胸鳍淡色,体上小棘尖端呈黑色,体长可达17公分。栖息在有遮蔽的珊瑚礁区,属肉食性,以小型无脊椎动物为食,卵胎生,可作为观赏鱼。","text2":"刺海马的模式产地是哪里?","label":1} {"text1":"裸盖鱼(学名:')又名银鳕鱼、裸头鱼,是辐鳍鱼纲鲉形目黑鲉亚目裸盖鱼亚目黑鲉科的其中一种,经常被与真正的鳕鱼混为一谈。后在南极海域发现南极犬齿细鳞鱼的鱼类,外观虽与裸盖鱼完全不同,但也被称为银鳕鱼,且市价更高。本鱼分布于北太平洋,包括日本北部、勘察加半岛的白令海海岸、阿拉斯加、加利福尼亚州等海域。一般栖于近底层(水深0-2740米),属冷水性底栖鱼类。水深0至2740公尺。本鱼体相当延长而稍侧扁,呈圆筒形。头尖形;口中大,上颌略长于下颌。齿细小,呈齿带,上下颌、锄骨、腭骨均具齿。头部及身体均被弱小圆鳞。背鳍分离,具有硬棘19至27枚、软条15至19枚;臀鳍硬棘3枚、软条15至19枚;尾鳍深叉型。体背深蓝灰色或绿灰色;体腹灰色或淡色;除第一背鳍后,各鳍均具有黑色外缘。体长可达120公分。冷水域之深海鱼类,本鱼卵为大洋性漂浮卵,随著成长逐渐由水表面而不断迁移至较深水域。杂食性,大都以虾、蠕虫或小鱼为食。食用鱼,一般皆以烟薰食之。其肝具有丰富的维他命A及D。在中国大陆市场上被以“鳕鱼”的名称出售。然而,另一种较常见的名称“油鱼” ,即“异鳞蛇鲭”,常被拿来冒充裸盖鱼,但肌肉及内脏富含蜡质油脂,多吃会导致腹泻。","text2":"裸盖鱼还有什么名字?","label":1} {"text1":"丁丁在美洲(法语:Tintin en Amérique;英语:Tintin in America)是《丁丁历险记》的第三部作品。作者是比利时漫画家埃尔热。丁丁在美洲于1931年9月3日开始在比利时报刊二十世纪小伙伴上连载。1932年集结成册出版。1945年,进行彩色化,并精简重编为62页的标准格式。美国版于1973年出版。故事的舞台是在1930年代,黑帮与盗匪横行的美国城市芝加哥。从踏足芝加哥的第一刻开始,匪帮已经牢牢盯著丁丁,一心要除之而后快!一场接一场的正邪搏斗旋即展开,丁丁不断在鬼门关外徘徊:被绑架掳劫、被囚禁、被毒气弄昏、被投进水去淹死、被嫁祸吊死、甚至被缚在铁路轨上给火车碾过!见义勇为的丁丁与好拍挡米路如何逃过一次次的劫数?如何把匪帮头子一个个地绳之于法?","text2":"此作品在何时何地刊载的?","label":1} {"text1":"科提斯·史东(,) 出生于澳大利亚 ,是一位厨师、烹饪书籍作者、与电视节目主持人。科提斯在澳洲的Penleigh and Essendon Grammar学校念书。他的父亲William是位会计师。他在五岁时就跟随奶奶下厨了。科提斯在对食物产生兴趣前就读商业学士学位,18岁时他开始从事厨艺事业,他到欧洲体验生活,并且学习义大利,法国,西班牙的美食。2008年,史东主持《帅哥厨师来我家》,并且著作了系列的食谱书,著作的\"Recipes to Put You in My Favorite Mood\"于2009年4月上市。2013年,主持《决战全球餐厅》。他在电视节目Martha中,宣称自己是单身。史东被People杂志列为最性感男性,几近追上乔治·克隆尼和Brad Pitt。他于2009年开始与女星 Lindsay Price 交往,2011年11月6号,他们的儿子Hudson出世,他们也于2012年11月宣布订婚。于2013年6月8号结婚。地点The Hilton's Sa Torre Mallorca Resort. 2014 年九月他们第二个儿子Emerson于加州出世","text2":"科提斯出过一本关于美食的书叫什么名字?","label":1} {"text1":"阿德姆·利亚伊奇(,),是一名塞尔维亚职业足球运动员,司职进攻中场,也能担任翼锋。目前效力意甲球队-{zh-hans:都灵;zh-hk:拖连奴;zh-tw:杜里诺;}-。拿积于2005年,他14岁的时候加入柏迪逊。他首次在一队上阵是在2008年7月29日,2008\/09赛季欧洲联赛冠军杯第二轮资格赛第一回合,他以后备身份上阵。他取得的第一个入球是在2008年11月23日联赛对阵OFK贝尔格莱德的赛事。2009年1月,英超球队曼联宣布利亚伊奇与队友苏兰·托锡一起转会曼联,苏兰·托锡将直接进入曼联阵容,而拿积则将以租借形式留在游击队效力。2009年12月3日曼联在网站宣布球队虽然有收购权,但由于中场人材过剩及财政问题,决定取消交易。2010年1月13日,意甲球队费伦天拿宣布签下拿积。通过体检后,拿积签订了为期5年的合约,转会费为750万欧元。拿积首次为费伦天拿上阵是在2010年1月31日联赛作客2-2战平卡利亚里的赛事,他在82分钟后备上阵。2012年5月2日,在一场意甲联赛中,费伦天拿时任主教练德里奥·罗西在把拿积换下后,拿积在鼓掌后被德里奥·罗西认为是在讽刺他,德里奥·罗西立即痛殴在后备席的拿积。比赛结束后,德里奥·罗西马上被俱乐部解雇。事后,拿积的队友认为拿积并没有冒犯德里奥·罗西的意思。2012-13赛季,拿积在主场对阵国际米兰时与队友祖维迪各自梅开二度,以4-1击败对手。拿积首次为塞尔维亚U-21上阵是在2008年9月7日,对匈牙利U-21的赛事。","text2":"2012年意甲联赛后,德里奥·罗西为什么被俱乐部解雇了?","label":1} {"text1":"米点箱鲀(学名:),又称白点箱鲀,俗名花木瓜,为辐鳍鱼纲鲀形目箱鲀科的一种。本鱼分布于印度太平洋区,包括东非、南非、马达加斯加、模里西斯、红海、塞席尔群岛、留尼旺、马尔地夫、印度、斯里兰卡、泰国、中国、韩国、日本、台湾、菲律宾、越南、泰国、马来西亚、印尼、新几内亚、澳洲、密克罗尼西亚、马里亚纳群岛、库克群岛、马绍尔群岛、新喀里多尼亚、萨摩亚群岛、东加、万那杜、法属波里尼西亚、加拉巴哥群岛、夏威夷群岛、墨西哥等海域。该物种的模式产地在太平洋南部。水深1至30公尺。本鱼体略呈正立方体,眼小口小,唇厚略突出。鳞片特化成骨质盾板的坚硬外壳。幼鱼似颗黑底白点的小圆球,随成长变为黑褐色;雄鱼体带蓝灰色,身上具有许多黄白点。背鳍软条9枚;臀鳍软条9枚,体长可达25公分。本鱼栖息于潟湖或珊瑚礁区,性情胆小迟钝,受惊吓时体会分泌出毒液,游速慢,属杂食性,以多毛类动物、海绵、软体动物、桡脚类与藻类等为食。产浮游性卵。观赏性鱼类,不供食用。","text2":"米点箱鲀不供食用,属什么性鱼类?","label":1} {"text1":"马来西亚皇家空军(;)的前身是成立于1936年的海峡殖民地-{志}-愿空军部队,在当时是英国皇家空军的辅助部队;二战期间,部队改名为马来亚后备空军部队。战后,空军部队曾一度被解散,但在1958年6月2日由国会通过空军法令恢复运作,重新运作的空军部队被名为马来亚皇家空军。1963年9月16日,空军部队又易名为马来西亚皇家空军。全马共有13个空军基地,马来半岛有11个,东马有2个。一共有2个师,西马为第1师,东马为第2师。共计15个中队,3个飞行训练中队。PASKAU (Pasukan Khas Udara),特种空勤部队。马来西亚皇家空军同样也拥有一支特种部队,主要的任务包括:马来西亚特种空勤部队是直接由英国特种空勤团(SAS)以及美国三角洲特种部队(Delta Force)训练的。","text2":"马来西亚皇家空军特种部队由谁训练?","label":1} {"text1":"德瑞莎·麦德罗(英语:Teresa Medeiros)是一位1960年代出生,美国女性罗曼史作家。目前出版23部作品,多次荣登纽约时报畅销书榜。作品题材多为历史罗曼史,亦有数本穿越时空、吸血鬼题材,等超自然罗曼史作品。德瑞莎·麦德罗出生于1960年代美国,处女作于21岁时出版。在成为知名小说家之前,职业是一名护士。截至2015年为止,已出版23 部作品,翻译至十七种语言,总印量超过一千万册。目前与她的丈夫和两只猫,住在美国肯塔基州。麦德罗作品多次荣登纽约时报畅销书榜,赢得PRISM奖两次,亦获得两次Waldenbooks Award,同时也是Romance Writers of America创始会员,亦为肯特州罗曼史写作家和小说家协会会员之一。作品题材多为历史罗曼史。有少数系列作品为超自然罗曼史,例如:有穿越时空剧情的女巫系列或吸血鬼题材的The Lord of Midnight Series等。作者亦有现代罗曼史、西部罗曼史作品各一,可谓多方尝试写作方向。德瑞莎·麦德罗在台湾的中文译作大多由林白\/果树出版社出版代理,早期作品出版代理较多。近期作品目前无台湾代理。最新一本中文译作,亦是唯一本由高宝集团代理的作品,为2004出版之《一夜绯闻》(One Night of Scandal)。该出版社将作者名称误值为Terest Medeiros,译名为泰瑞莎‧梅戴洛斯。","text2":"目前她和谁住在美国肯塔基州?","label":1} {"text1":"2008年至2009年英格兰联赛杯,或因赞助商而冠名为加宁杯(Carling Cup),是第49届的英格兰联赛杯赛事。胜出的队伍如果没获得欧洲赛参赛资格的话,可以获得2009年至2010年欧洲联赛第三圈外围赛资格。曼联在决赛互射十二码阶段击败热刺,成为今届联赛杯的冠军。第一圈的抽签于2008年6月13日举行,而赛事则于8月11日进行。72队参赛队伍分为南北两组,每组平均分为种籽队和非种籽队,而排名则根据上年成绩而定。12队英超联球队会加入角逐。第二圈抽签于2008年8月13日举行,赛事于2008年9月25日左右进行,除了曼城和白礼顿的赛事,此场于9月24日进行。8队要参加欧洲赛的队伍会加入角逐,由于今季有9队球队参加欧洲赛,所以起初不清楚哪8队会加入。直到2008年7月29日,赛会宣布阿士东维拉会在此赛圈加入。第三圈抽签于2008年8月30日举行,赛事于2008年9月24日左右进行,除了白礼顿和打比郡的赛事,此场于11月4日进行。第四圈抽签于2008年9月27日举行,赛事于2009年11月10日左右进行。第五圈抽签于2008年11月15日举行,赛事于2008年12月2日或3日进行。准决赛抽签于2008年12月6日举行,首回合于2009年1月6日或7日进行,次回合于1月20日或21日进行。\"曼联总比数赢4 – 3\"\"热刺总比数赢6 – 4\"今届决赛于2009年3月1日在温布莱球场举行。以下是今季联赛杯神射手名单︰","text2":"72队参赛队伍分为哪几组?","label":1} {"text1":"Internet Download Manager(简称IDM)是一个用于Windows系统的下载管理器,它是共享软件,免费试用期为30天,但是每月均有一段时间优惠。IDM可以让用户自动下载某些类型的文件,它可将文件划分为多个下载点以更快下载,并列出最近的下载,方便存取文件。相对于其它下载管理器,它具有独特的动态档案分割技术。它的出色之处在于,能分析下载地址中的实际地址,但若地址无效时则跳换成失效网页,当地址重新有效而无法恢复成原来的地址。它的不足之处在于,必须设置分段的临时文件存放的磁盘位置,下载完成后再自动合并成一个完整的文件,同时删除临时文件(这种做法不同于其他下载软件预先分配一整块磁盘空间)。所以,下载的如果是机密文件,就容易被数据恢复软件恢复,造成安全隐患。IDM可被调用于Internet Explorer、Safari、Google Chrome、Opera、Netscape Navigator以及Mozilla Firefox(火狐)浏览器中。","text2":"这个软件有什么不足之处?","label":1} {"text1":"「桥立」()是西日本旅客铁道(JR西日本)一列行驶于京都~天桥立之间的特别急行列车,沿途行经隶属于JR西日本的山阴本线与第三部门业者北近畿丹后铁道所属的宫福线、宫津线,是北近畿地区跨业者联营特急列车群北近畿大X网路()的组成份子之一。「北近畿大X网路」下属的诸列车全都是采跨社联运的方式运作,因此桥立号的车资实际上是由JR西日本与北近畿丹后铁道共同拆分,而与桥立号相对应、隶属于北近畿丹后铁道的特急列车是丹后发现号(),但不同的是丹后发现号在驶抵天桥立之后尚会继续行抵终点站丰冈才折返。桥立号的列车名源自位于终点站所在地的著名名胜、日本三景之一的天桥立。列车头牌上的代表色为红色,起源于耸立在宫福线沿线的大江山地区著名的「赤鬼」传说。这是桥立1・3・4・6・7・10号的编成这是桥立2・5・8・9号的编成桥立","text2":"列车头牌上的代表色为红色,起源于什么传说?","label":1} {"text1":"《大篷车》()是一部1971年的印度电影,导演是纳亚尔·胡赛因(Nasir Hussain),吉滕德拉(Jeetendra)和阿莎·帕雷克(Asha Parekh)主演,片长128分钟。上海电影译制厂1980年译有该片的中文版本。故事的主线是女主角苏妮塔躲避贪财邪恶的丈夫拉詹手下的追杀所进行的逃亡和自救的过程。拉詹为了获得苏妮塔父亲的巨额财产,伪装自己获得了苏妮塔父亲的信任在其公司担任要职,并成为他的养子。一次拉詹偷走了保险箱里的30万卢比,被苏妮塔父亲发现后将苏妮塔父亲推下楼致死,但是他没有发现苏妮塔父亲死前曾写下了一张给苏妮塔父亲的好友的一封信,里面包含了他错看拉詹的内容,后来这张信到了拉詹的情人莫妮卡手里。后来拉詹欺骗苏妮塔说她父亲自杀前曾对他说过要将苏妮塔嫁给他。于是两人很快举行了婚礼,不料新婚不久莫妮卡来到二人面前将信展示了,苏妮塔才知道父亲是被拉詹害死的。她在一个晚上偷偷跑了出来,准备去找位于班加罗尔的那位叔叔获得帮助,即信中父亲的好友。与此同时拉詹的几名手下也在抓紧追寻着她。在逃亡的过程中她由于身无分文,于是藏到了一辆负责运输到处流浪表演的吉卜赛艺人的大篷车里面,于是首先认识了莫汉,Johnny和Monto三人。后来又认识到了这个表演团的所有成员。在大家的关爱下,苏妮塔不仅学会了歌舞表演,更获得了莫汉的爱慕,当两人关系比较亲密的时候,遭到了这个团里最出众的歌舞女妮莎的反对,原来妮莎一直深爱着莫汉,也许是因为她个性泼辣的原因,莫汉一直躲避着她的爱。在莫汉等人的团结努力下,最后拉詹等一帮坏人得到了应有的下场。但是那位叔叔不仅被害,更遗憾的是一直深爱莫汉的妮莎也为救莫汉付出了生命的沉重代价。最终苏妮塔并没有选择过富贵小姐的生活,而是选择继续与大家同行,从此她代替妮莎成为了这个流浪表演团队里的一员。","text2":"《大篷车》是由谁导演的?","label":1} {"text1":"和广北,中银香港控股及中银香港前副董事长兼总裁,亦为南洋商业银行、南洋商业银行(中国)、集友银行及中银人寿前董事长。和广北为中银香港于香港银行公会之指定代表,并于2008年担任该会主席。和广北于1979年在北京第二外国语学院毕业,取得学士学位,并于1985年在美国德克萨斯州大学达拉斯分校取得国际管理学硕士学位。和广北毕业后加入中国银行,曾担任不同职位,并先后在纽约分行及巴黎分行工作。1999年至2004年担任中国银行常务董事;自2000年至2003年期间担任中国银行副行长。2015年3月6日中银香港公布,和广北因年龄原因,辞任该行副董事长、执行董事兼总裁等职位,","text2":"中银香港于香港银行公会之指定代表是谁?","label":1} {"text1":"王志刚(),河北安新县人,中华民国政治人物。学生时期大都在台北县淡水镇(今新北市淡水区)生活,直到考上台大全家才搬进台北市区。建国中学毕业,毕业于国立台湾大学农业经济系,获美国德州农工大学企业管理博士。学成归国后,王曾经于国立台湾大学、国立政治大学执教,并曾经担任台大国贸-{系系}-主任。执教鞭时期,郭台铭、严凯泰等知名企业家都是他的学生,更是台大有史以来第一个授课时完全没人翘课、座无虚席,连教室门边都有学生挤著听他讲学的兼任讲师。1996年,连战二度组阁,王出任中华民国经济部部长。2000年3月18日,总统大选陈水扁胜选不到十天,担任台糖武智基金会董事长的张有惠将基金会董监事改成自聘,不受政府监督,经济部长王志刚火速核准,超过两百亿的公有财产被侵吞为私产。2000年,自经济部长一职卸任后,一度担任中国国民党发言人,尔后前往元智大学教授EMBA课程,被评为明星教授之一。2008年起,以学、经历专长,出任中华民国对外贸易发展协会董事长与台湾创意设计中心董事长。王志刚育有一对儿女,都往新闻界发展,但长女之后回归家庭相夫教子;长子大学中文系毕业于某电视台当企划。王志刚是台湾外省人当中少数可以将闽南语(北部腔)说的一把罩的前内阁官员。","text2":"王志刚学生时期在哪里生活?","label":1} {"text1":"宗亲会是联络同姓宗族的组织,宗亲会是在中国传统伦理思想下衍生出的组织。源于中国人自古以来有慎终追远;饮水思源的传统礼教,重视家庭崇拜祖先观念。现今宗亲会组织常见于华人世界暨汉字文化圈,在日本称籓族在琉球多称之为门中会。当一个宗族世代繁衍,子孙往往散居各地,为了保持宗族的完整和团结,就成立宗亲会。宗亲会通常以同姓一族为组成团体,隐含彼此之间有著共同祖先血缘关系。在台湾及海外类似团体也有称之祭祀公业、祀产公业、祖公会、丁仔会等不同的组织称呼。名称虽不相同,但其主要的功能如:调解、互助、团结、联谊、承续维持宗族壮大之传统观念是相同的。而宗亲会除了平时的联络外,还有族谱的编制,宗族祖先的祭祀,所以亦可说宗亲会的设立就是为了维系伦理而产生的。","text2":"为什么会成立宗亲会?","label":1} {"text1":"鹿鸣宴,为科举制度中规定的一种宴会,始于唐代。唐代州县一级的考试称为解试,得中者称乡贡,亦称举人。鹿鸣宴就是为乡试后新科举人所设的宴会,因宴席中要唱吟《诗经·小雅》中的“鹿鸣”之诗:“呦呦鹿鸣,食野之苹。我有嘉宾,鼓瑟吹笙...”而取名为鹿鸣宴。此宴设于乡试放榜次日,由地方官吏主持,宴请之人除了新科举子之外,还有称为内外帘官等考场工作人员。明清两代沿袭唐例,鹿鸣宴同为地方长官宴请乡试中举的学子。清代鹿鸣宴更为隆重,宴会由省内巡抚所主持,即宴请新科举人,也同时招待考官,席间不仅唱《鹿鸣》诗,还跳魁星舞,规模宏大且场面热闹。在唐至清代的科举和教育体系中延续千年的鹿鸣宴,在主要是为得解举子饯行、励志的北宋,还相对普及;但在南宋则更多地强调鹿鸣宴古礼仪制和政教功能的恢复,缺少改造和更新,其繁文缛节又加重了地方的经济负担,使之几近废礼。","text2":"鹿鸣宴设于哪一天?","label":1} {"text1":"蕊蕊(,),原名陈蕊蕊,香港女歌手、演员及前舞蹈员。前无线电视舞蹈艺员。蕊蕊中学时就读乐善堂梁植伟纪念中学,其后毕业于EF国际语言学校(伦敦分校),主修语言为英语。2004年入读无线电视第9期舞蹈艺员训练班,并签约无线电视。至2009年因演出该台综艺节目《耳分高下》,且担任「Sing之天使」其一「柑桔」,使她渐渐受人注目。同年11月约满后,与唱片公司自由行娱乐(Freeway Entertainment)签约,经理人是汉洋。2011年2月,她发表首支派台歌曲《Make Up Baby》。2014年4月,蕊蕊与邓咏雪、陶枳樽和何丽娟组成女子歌唱组合「HunterZ」。2016年,她离开自由行娱乐和签约新公司。蕊蕊与台湾艺人金刚于2013年开始交往,但两人在2016年4月因性格不合而结束恋情。","text2":"组合「HunterZ」中有哪些成员?","label":1} {"text1":"延斯·克里斯蒂安·斯科(Jens Christian Skou,),丹麦化学家以及诺贝尔奖得主。斯科生于丹麦莱姆维的一个富有家庭。他父亲马格努斯·马丁努斯·斯科是一名木材和煤炭商,母亲安妮·玛格丽特·斯科在他父亲去世后接管了公司。15岁时斯科进入西兰岛海斯莱乌的一所寄宿学校,1944年从哥本哈根大学获得医学学位,随后1954年获得博士学位。1947年他开始在奥胡斯大学工作,并于1977年被聘为该校的生物物理学教授。斯科于1988年从奥胡斯大学退休,但仍保留了在学院的办公室。1997年,他同保罗·博耶、约翰·沃克一起发现了钠钾-ATPase(钠钾帮浦),获得诺贝尔化学奖。","text2":"斯科的父亲马格努斯·马丁努斯·斯科是做什么的?","label":1} {"text1":"归元寺,位于汉阳区翠微路和归元寺路交界,是中国湖北省武汉市的一座著名佛教曹洞宗寺院,武汉的佛教四大丛林(还包括宝通寺、莲溪寺、正觉寺)之一。湖北省佛教协会和武汉市佛教协会的所在地,湖北省文物保护单位。中国国务院首批公布的开展宗教活动的重点寺庙。寺名取佛经“归元性不二,方便有多门”之语意。藏经阁内供奉有释迦牟尼佛像。寺院占地17500多平方米(一说4.67公顷),分为中院、南院、北院三组,有藏经阁、大雄宝殿、五百罗汉堂三组主要建筑,殿舍200余间。现由宝通禅寺隆醒大师担任归元寺方丈。隆醒法师和原主持隆印皆为归元寺老主持昌明大师徒弟,昌明师最后的传法弟子隆智为昌明师晚年所收,俗世修行。乘坐武汉地铁4号线或6号线便可前往归元寺,需在钟家村站、汉阳火车站下车。","text2":"现由谁担任归元寺方丈?","label":1} {"text1":"蒙田大街(Avenue Montaigne)是法国巴黎第八区的一条街道。蒙田大街最初称为寡妇巷(allée des Veuves),因为妇女们在此聚集哀悼,但是自从18世纪初起这条街已经发生很大变化。这条街现在的名称得名于法国文艺复兴作家蒙田。在19世纪,这条街因周六夜晚五彩闪烁的Mabille球赢得了一些名声。蒙田大街拥有许多高级时装店,如克里斯汀·迪奥、香奈儿和华伦天奴,以及宝格丽等珠宝店,和雅典娜广场酒店(Plaza Athénée)等高档场所。到1980年代,蒙田大街被认为是法国高档时装及配件街道的“伟大的女性”(la grande dame),其重要性被认为超过了圣奥诺雷郊区街。一些服装设计师驻扎于此,例如路易威登。路易威登为这条街带来了投资和国际关注,设计师和公司,如席琳,洛伊,路易威登,Inès de la Fressange和以前的克里斯汀·拉克鲁瓦 ,都在这条街大量投资房地产。加拿大大使馆设在蒙田大街35号。蒙田大街15号坐落着香榭丽舍剧院(Théâtre des Champs-Élysées)。2008年12月4日,蒙田大街29号的Harry Winston精品店在打烊前遭到三四名武装劫匪抢劫,损失价值超过8000万欧元(约1亿美元)的“钻戒、项链和高档手表”,其中至少两名劫匪穿戴着假发和妇女的衣服。2007年10月这里也遭遇过类似的抢劫案,损失了2000万欧元。","text2":"在19世纪,这条街因何赢得了一些名声?","label":1} {"text1":"满洲国协和会为满洲国的官民一体国民组织,起初石原莞尔等人期许本组织能担任未来一党执政的重任,然而在之后则受到日本大政翼赞会旗下各组织影响。九一八事变后,倡导从中华民国独立并施行君主政治以推动新国家建设的自治指导部为协和会的起源。满洲国建国后,自治指导部则转型为官民一体化的满洲国协和会。协和会与满洲国政府共同宣称推动建国理想(建设王道乐土),并以此对国民进行宣传教育。协和会章程规定:“本会以谋求满洲国的建国精神和宣扬王道乐土政治为宗旨。” 1936年7月25日协和会成立4周年举行纪念会,改名为“满洲帝国协和会”。同年9月18日,日本关东军司令官植田谦吉发表《满洲帝国协和会之根本精神》的声明,阐述协和会的本质,就是以皇民化为目标的“民族协和”,实施“建国精神”,即“以实现日满一德一心、民族协和、王道乐土、道义世界为理想的天皇的圣意”。满洲帝国协和会中央本部机构庞大,设有总务部、指导部、实践部、训练部、文化部、青少年部、调查部等10多个部门。协和会基础单位为分会,与每个地区设立。并于各地方行政机关设立本部,负责统括分会。取代从未开议的立法院,以分会代表进行的连合协议会为实质之民意机关。石原莞尔等人期待协和会能取代关东军成为「未来的领导者」,拥有一党执政的地位。不过随著关东军对满洲国统治的深化,满洲国逐渐成为日本完全的属国,协和会失去其存在意义。满洲帝国协和会为唯一永久、举国一致的实践团体,与政府互成表里。实现以上建国理想,创建道义世界。满洲国协和会(1932年、大同元年)成立时的主要干部满洲国执政(后皇帝)任名誉总裁,国务总理大臣任会长,各省长、县长兼任所辖地域的本部长。满洲帝国协和会中央本部旧址位于今中央大街82号,人民广场南200米处。东南接儿童公园,北隔咸阳路与长春市委大楼后身相对,西邻人民大街。该旧址是一座砖石结构的组合建筑物,地面建筑为两层楼房,另有地下室,建筑面积7955平方米,占地面积8749平方米。1950年修复为长春市文化宫,1958年由沈阳军区接管,改为军人俱乐部,后又将部分房舍改建为吉林省军区第二招待所。旧址保存较为完好。","text2":"满洲国协和会在哪一次会议上改了名字?","label":1} {"text1":"在英国作家托尔金所创作的奇幻小说里,圣盔谷(Helm's Deep)是位于白色山脉西北方的一个大山谷。圣盔谷被一连串被称为圣盔渠的山丘阻挡著,山丘后方就是要塞号角堡,即是晶洞的入口。圣盔谷本来是一些刚铎人的聚居地,但在卡兰纳宏成为洛汗领土以后,圣盔谷就变成洛汗人的据点,艾辛河的守卫住在圣盔谷。在与以沃夫为首的登兰德人爆发战争期间,洛汗国王圣盔·锤手带领他的子民避难,他们在第三纪元2758年至2759年期间冬季在圣盔谷留驻。谷内一条蜿蜒的道路通往号角堡,谷内有马廐及兵器库,在后面还有一个大厅,那大厅是从山腰处挖成。号角堡的顶端由圣盔·锤手的号角组成。山谷被长长的围墙绕著,全都是坚固的岩石,除了一道暗渠能让深溪的河水进入,让在谷内进行长时期战争的战士提供水源补给,围墙高约二十尺,其厚度相当于四人紧贴著站在一起。谷内还有一条捷径通往号角堡后方的入口。在魔戒圣战时期,圣盔谷再次成为一些洛汗人的避难所,这时的洛汗国王是希优顿,这里也爆发了著名的圣盔谷之战。传说有人守备的圣盔谷永远不会被攻陷,甚至于没有敌人能突破围墙或在号角堡立足,但是在第三纪元3019年,萨鲁曼的强兽人军队几乎击败洛汗守军,他们用妖火爆破暗渠。没有人知道这些爆破物是甚么,但托尔金说这是「萨鲁曼的魔术」。在彼得·杰克逊导演的电影《魔戒三部曲》系列里,那些爆破物被描述为火药。","text2":"圣盔谷再次成为一些洛汗人的避难所是什么时候?","label":1} {"text1":"北港国际音乐文化艺术节 (Beigang International Music Festival)是由云林县北港爱乐协会主办,首次在2006年举行,受到政府单位及社会人士的支持与肯定。接著每年定期举办,至今成为云林县最大的国际文化艺术节。此音乐节提供一连串的音乐会,大部份是管乐(独奏、重奏及管乐团)并在陈家湖音乐学院开办音乐教学讲座,节目的设计是由来自世界各地的音乐家(由云林县北港爱乐协会邀请)共同参与,此艺术节的音乐艺术总监是钢琴家。北港是台湾开台400年来最古老的小镇,而北港朝天宫是全世界最有名的妈祖庙。除了宗教庆典及传统地方文化外,也需要文化艺术来美化生活。所以爱乐协会决定每年在北港举行国际文化艺术节,让北港小镇更活化,北港同时也是艺术节的重镇,在此有许多场音乐会的演出及教学讲座,也同时和青少年管乐团练习并演出。2005年举办北港乐团(成立于1925年)80周年庆暨北港爱乐季,2006云林县北港爱乐协会聘请陈学孟为音乐艺术总监,并把整个爱乐季提升到国际层面,从此整个爱乐季受到听众热烈的回响,媒体的争相报导更受到地方人士及政府机构的肯定。云林县县长苏治芬对此爱乐季相当重视,每年必亲自莅临开幕仪式。2007年德国代特莫尔德音乐大学校长Martin Christian Vogel是当年的贵宾。音乐会除了大部份在北港妈祖文化大楼举办外,也安排在新营及斗六两大城市来举行。节目包括古典音乐、通俗音乐,还有公园广场的文化晚会,在北港镇外的田园蜜语西餐厅的酒馆爵士乐,最后的闭幕高潮是北港青少年管乐团的演出。公家行政机关有云林县政府、行政院文建会、内政部、北港朝天宫、北港镇公所、云林县地方法院检察署、云林县观护志工协进会及台湾更生保护会云林分会。教育机构如各社区之中小学及大专院校等。乐器公司有Kawai、Jupiter等,另外北港公司行号,医生、老师及爱乐人士。2009获得芬兰西贝流士音乐大学(芬兰)的特别赞助。台湾各大报纸的地方版及电视三台皆有报导,另外《新台湾新闻周刊》2007\/08\/02也有专题采访。德国的Lippische Landeszeitung 对台湾德国的文化交流非常感兴趣,也在2007年的爱乐季有篇幅报导。该音乐节主办单位为云林县北港爱乐协会,其会员皆于无薪给下自愿工作。","text2":"为什么爱乐协会决定每年在北港举行国际文化艺术节?","label":1} {"text1":"林翠(英文名:Jeanette Lin Chui,),生于上海,籍贯广东中山,本名曾懿贞(Jeanette Tsang Yi Cheng),香港电影红星,著名粤语演员曾江之胞妹,歌手王馨平的母亲。她被香港影坛誉为「学生情人」、「NO小姐」,主演和参演的影片无数,亦正亦邪皆胜任。1949年随家人迁居香港,就读圣士提反女子中学。1953年考入由导演黄卓汉领导的自由影业,林黛正因《翠翠》(1953)名声鹊起。1954年,凭处女作《女儿心》大红,成为台柱明星。与自由影业三年片约期间,陆续主演《终身大事》(1955)、《马车伕之恋》(1956)、《山地姑娘》(1956)、《馥兰姐姐》(1956)、《蔷薇处处开》(1956)。同时外借给其他公司,如艺华影业的《化身姑娘》(1956) ,新天影业的《马路小天使》(1957)、〈流浪儿〉(1958)等。也曾为邵氏拍摄《夜来香》(1957)、《移花接木》(1957)、《千金小姐》(1959)等。1957年11月加盟国际电影懋业(电懋),开启其电影事业的黄金时期,主要作品包括《四千金》、《兰闺风云》(1959)、《豆腐西施》(1959)、《啼笑姻缘》(1964)、《空谷兰》(1966)、《苏小妹》(1967),期间也为邵氏拍摄《金菩萨》(1966)等。国泰电影出品的《游龙戏凤》(1968),是息影前的最后一部作品。她曾经历两次婚姻,第一任丈夫为名导演秦剑,两人于1959年结婚,婚后育有一子陈山河。1967年林翠与秦剑分居,当时香港规定要分居3年才可离婚。1968年8月,林翠为王羽生下王馨平。1969年6月秦剑自杀,同年年底林翠与王羽结婚。林翠与王羽育有王馨平、王加露、王美怡三女,可惜仍于1975年离婚。恢复单身后,创办真纳影业公司,出品唐书璇导演的《十三不搭》(1975) 及自导的《香港式离婚》(1976)。1977年移居美国旧金山,转而从事餐饮及租赁业。1980年末复出影坛,参与电影《海峡两岸》(1988)、《胭脂》(1991)及台湾电视剧《不了情》、《婆媳过招七十回》、《初恋三十年》演出,并有意转往幕后发展。1995年2月22日在台湾台北市突发气喘病殁于家中,享年59岁。","text2":"林翠息影前的最后一部作品是什么?","label":1} {"text1":"《花美男连锁恐怖事件》(),亦称花美男连环恐怖袭击事件或简称花美男,是一部2007年韩国搞笑神秘校园电影,是SM Pictures(SM Entertainment的子公司)制作的第一部电影 。出演电影的是Super Junior全员,但成员圭贤因2007年4月19日车祸受伤没有出演。2007年7月26日电影公映,并发布原声带。2月14日,一位学校的花美男遭到不寻常的袭击后,同样的袭击每个月的同一天都会发生在不同高中。传言说下一个受害者将是常青外语高中的学生。热心的学生起范决定调查这件事,将每月14日的恐怖袭击时间记录在博客中,使得时间成为学生群中的焦点话题。几次袭击事件过后,起范预言下一个受害者将是学生会主席{始源}、舞蹈社社长{希澈}、柔道队长{强仁}其中之一。电影的票房比预期的要好。参考网路,公司预计在销量22%左右,但销量超过41.19%. 电影得到了许多正面评论,证明电影相对偶像作用来说策划的功劳比较大。和其他花美男电影不同,这部电影能吸引人们的关注。尽管在公映一周时票房上取得了没有预料到的成功,电影只吸引了102,900左右的观众。 因为这是SM picture的第一部作品,该公司仍在努力,并成功地进入了电影界。 该公司将出席第12届釜山国际电影节,还将举办“SM之夜 ”。整个剩余的一年内,电影未能吸引超过10.26万观众,证明本片是SM Pictures一个巨大的损失。 然而,这两个版本的电影DVD在韩国和海外市场成为最畅销的DVD。这足以弥补生产的亏损八十五万韩元。这部电影在 中国、泰国、越南等其他亚洲国家的电影院上映。","text2":"《花美男连锁恐怖事件》的主演是?","label":1} {"text1":"罗马治世(拉丁语:Pax Romana),又称罗马和平,是指罗马帝国存在的五百多年间,前二百年比较兴盛的时期,亦即盛世。公元前30年,屋大维消灭埃及托勒密王朝,结束了罗马内战。一般将这一年视为罗马和平时期的开始,也有一个说法是从公元9年(罗马基本上停止侵略日耳曼地区)开始算起。公元161年,五贤帝中的最后一个─马尔库斯·奥列里乌斯即位,不久后遇上帕提亚入侵亚美尼亚,紧接著又发生瘟疫,他本人更在征伐日耳曼的途中病逝。期间,罗马帝国的财务状况恶化,日耳曼人也开始骚扰边境,帝国由盛转衰。从屋大维统一罗马至马尔库斯·奥列里乌斯过世(公元180年),长达200年左右的时间罗马大致富强稳定,没有较大的战乱,因此史称“罗马治世”。","text2":"为什么会有罗马治世的说法?","label":1} {"text1":"元羽(),字叔翻,河南郡洛阳县(今河南省洛阳市东)人,魏献文帝拓跋弘第四子,孟椒房所生。太和九年(485年)封广陵王,加侍中。元羽聪慧,擅长断狱。后为大理,主管京师狱讼。迁特进、尚书右仆射,又为太子太保、录尚书事、廷尉卿。他的大哥孝文帝元宏鼓励鲜卑与汉族通婚,元羽嫡妻降为妾室,再聘娶骠骑谘议参军荥阳郑平城的女儿郑始容。太和二十二年(498年)元羽随孝文帝征南齐,被裴叔业领兵击败,元羽的符节被齐军缴获。北魏宣武帝继位后,以他为司空、司州牧。元羽好色,与员外郎冯俊兴的妻子私通。景明二年(501年)五月的一个夜里,元羽去找冯俊兴的妻子寻欢,在路上被冯俊兴拦住打了一顿,冯俊兴把元羽藏了起来,五月十八日(501年6月19日),元羽去世,虚岁三十二,朝廷赠予使持节、侍中、骠骑大将军、司徒公、冀州刺史,给羽葆鼓吹、班剑四十人,谥号惠。元恭登基后,追尊元羽为先帝,生母王氏为先太妃。","text2":"为什么元羽嫡妻降为妾室?","label":1} {"text1":"林郁夫(,),原奥姆真理教的干部成员,出家名为。入教前是医生。奥姆真理教实行省厅制后担任「治疗省大臣」。是东京地下铁沙林毒气事件的其中一名行凶者,被判处无期徒刑。他和同样是教团干部的林泰男虽然同姓,但没有亲戚关系。林郁夫1947年出生于东京都品川的一个执业医生家庭。先后就读于庆应义塾中等部和庆应义塾高等学校。大学毕业于庆应义塾大学医学部,之后留学美国。归国后在茨城县东海村的医院工作。1989年开始相信奥姆真理教;1990年5月辞去工作带同妻子一起出家信教,很快成为麻原彰晃其中一个最信任的助手,他成为教团中的「师长」,以及东京都中野区教团附属医院院长。1994年成为奥姆真理教的「治疗省大臣」,主要负责研究「奥姆菜单」,管理「奥姆医院」,向信徒注射迷幻药LSD,「治疗」信徒中的动摇者,更有削去信徒指纹的行为。1995年轰动世界的东京地下铁沙林毒气事件,他是执行者之一。他携带两个用报纸包裹著的装有沙林毒气的容器进入千代田线后,在新御茶之水站擢穿容器后离去。他散播的毒气造成2人死亡,231人受伤。据他自己的回忆所称,擢穿容器之前,他曾经十分犹豫,自己行医多年就是为了不断拯救病人,但是如果擢穿容器的话,列车上的所有人就会在他面前倒下。他几次想放弃,但是最终还是相信这是为了崇高的目标而下手了。在行凶后,他没有躲避起来,而是回到奥姆真理教的一个分部。10多天后的4月8日,他在石川县穴水町的路上被逮捕,当时怀疑他正在偷窃放置在路上的自行车而以盗窃罪被捕。被捕初期,他拒不合作,在狱中进行绝食,还宣扬「警察和奥姆真理教的战争」。控方最初也因他曾犯有杀人、绑架、监禁等罪行而指控他为「杀人机器」,请求判处死刑。后来他全面交代犯罪事实,使地下铁沙林事件的具体经过得以公诸于众,还协助指证麻原彰晃。他本人也表示对自己的所作作为感到惭愧,令控方主动改为请求判处终身监禁。1998年5月,东京地裁判处他无期徒刑(其它4名与他一起执行地下铁沙林袭击任务的人都判处了死刑)。1998年,他的回忆录《奥姆与我》()在文艺春秋发表,描写了自己作为一个受过高等教育的人,帮助麻原彰晃从事肮脏行径的过去。回忆录的收益全部赠予受害者以及家属。","text2":"林郁夫原是什么教的成员?","label":1} {"text1":"在新闻媒体上,「门」常常作为一些丑闻事件的专称的词语后缀,形成许多“××门”的词语组合。汉语中作为语缀的“门”翻译自英语“-gate”。“-gate”这个英语后缀的出现,最早出自新闻媒体对美国总统尼克松的“水门事件”(watergate)的报道,用来专指尼克松所属共和党在大选中的丑行。(这个\"门\"是因为事件发生于美国民主党全国委员会所在地水门综合大厦,便衣特警出其不意地抓获了5个潜入民主党全国委员会总部安装窃听器和拍照文件,所以水门事件,\"水门“是指\"watergate\"),但是由于水门事件对美国历史以及国际新闻界都产生了很大影响,因此用来指重大的政治性丑闻事件,以及政界要人的丑闻。《麦克米伦高阶英语词典》将\"-gate\"收录作为英语的构词后缀,作为于其他名词连用构成一个新名词,用在新闻中专指对美国总统或政府有影响的政治丑闻。之后、语义逐渐广泛化,不仅仅局限在政治上,而是将任何新发生的,有轰动效应,能引起公众强烈关注的事件都称为“××门”,成为“丑闻”的代名词。但是、「罗生门」并不属于这种例子的范围,也与丑闻事件毫无关联。","text2":"语义逐渐广泛化,“××门”成为什么的代名词?","label":1} {"text1":"安提贞尼斯(希腊语:Aντιγένης;)是亚历山大大帝麾下将领之一。从腓力二世时就为马其顿王国效力,公元前340年时在围攻色雷斯的皮林塔斯时失去一只眼。当亚历山大大帝逝世后,第一次继业者之战爆发,安提贞尼斯跟随帝国摄政佩尔狄卡斯攻打埃及的托勒密,当佩尔狄卡斯抵达尼罗河三角洲最东部的城市佩鲁修姆(Pelusium)企图渡过尼罗河,遭受到失败。佩尔狄卡斯斯转道三角洲顶部的孟菲斯,但部分部队在渡河时又被河水冲走,部队士气低下,在佩尔狄卡斯的严厉惩处下,部队被激怒而发生哗变。银盾兵指挥官之一的安提贞尼斯与培松、塞琉古联手杀害了佩尔狄卡斯,并与托勒密谈和。在特里帕拉迪苏斯分封协议中,安提贞尼斯因此担任埃兰总督。在第二次继业者之战中,安提贞尼斯和其他东部行省总督加入欧迈尼斯一方来对抗安提柯,并在帕莱塔西奈战役、伽比埃奈战役两场会战中率领银盾兵都有良好表现,而当银盾兵在伽比埃奈战役后发生哗变,安提贞尼斯随著欧迈尼斯落入安提柯的手中,最终被活活烧死。","text2":"公元前340年时在围攻色雷斯的皮林塔斯时,安提贞尼斯失去了什么?","label":1} {"text1":"《蜜桃女孩》(日语:ピーチガール)是日本漫画家上田美和在1997年的爱情喜剧漫画作品,在讲谈社少女漫画杂志《别册FRIEND》连载,单行本全18集完结。在第23届讲谈社漫画赏是少女部门的得奖作品。现在《蜜桃女孩》换一个新的样貌连载(现在长期休载中)。在2005年1月8日到6月25日期间由日本东京电视台播放的电视动画(后一个节目是「魔女的考验」)。在2001年由台湾播放同名「蜜桃女孩〜Peach Girl〜」的电视剧全部有17集。在2007年11月3日到2008年1月5日由纬来日本台在下午两点与晚上八点播出蜜桃女孩电视动画。看起来很风骚的小桃,害怕被老是学自己的沙绘知道自己喜欢的人会被抢走,所以欺骗沙绘自己喜欢冈安浬,在一些巧合下。东寺森和小桃交往了。但是 接二连三遭遇到沙绘的阴谋阻挠。最后沙绘以相片威胁东寺,让东寺离开小桃。不知情的小桃伤心欲绝,就在这时,阿里安慰小桃,闯进了小桃的世界。就在小桃爱上阿浬的时候。却被告知有了最喜欢的人,要分手。小桃的一次次忍让,让阿浬了解自己最喜欢的人是小桃。但是因为误会小桃却又和东寺森交往。就在去旅行的时候,小桃经过一次生离死别,终于发现自己喜欢的还是阿浬,不能放弃他。最后,小桃和阿浬在一起了。2001年由台湾制作的蜜桃女孩电视剧里的角色由台湾艺人F4之一的吴建豪饰演。台湾由华视首播,日本从2006年1月19日到5月11日在BS日本播放。2016年3月29日公布将改编成真人电影,预计将在2017年5月20日上映。由女性时装女模特儿山本美月及男子偶像团体Hey! Say! JUMP成员伊野尾慧主演。导演将由神德幸治担任。","text2":"由台湾制作的蜜桃女孩电视剧里的角色有谁参演?","label":1} {"text1":"詹姆斯·切斯特(,),是英格兰出生的威尔斯职业足球运动员,司职中后卫,出身曼联青训系统,现时效力英冠球会阿士东维拉。切斯特8岁时就被曼联邀请到青训营接受训练,2005年7月正式签约。2007\/08年赛季,他是曼联预备队出场最多的球员和后防绝对主力,因此进入一线队。2009年1月20日,他在联赛杯半决赛对阵德比郡的比赛中替补出场,第一次代表曼联参加正式比赛。2月2日被外借到英甲球队彼德堡一个月,期间上阵5场。2009年9月15日切斯特再获外借到英冠球会普利茅夫3个月,但仅作赛3场便因伤提前被送返曼联。2010年8月3日切斯特获外借到英甲球会卡素尔直到年底才完结。期间表现出色,在各项赛事上阵23场及射入3球,于借用期满前表示希望继续留队,于2011年元旦主场对哈德斯菲尔德近门顶入为球队扳平2-2完场。赛后曼联召回切斯特,转会窗重开后动作多多的英冠球会侯城提出约30万英镑罗致旗下。","text2":"切斯特为什么能进入曼联的一线队?","label":1} {"text1":"突厥汗国(古代突厥如尼文:;)是原先在柔然统治下的阿史那氏部族于552年在中国以北地区建立的古代汗国,一度控制漠北、中亚等柔然故地。后分裂成东突厥汗国和西突厥汗国,7世纪时先后为唐朝所灭。而东突厥复国后形成的后突厥汗国亦在8世纪为回纥灭亡。有人认为阿史那部最先是生活在咸海边的塞种,后东走至叶尼塞河南方,受铁勒同化;阿史那部于6世纪初年游牧于金山一代(今阿尔泰山),归附于柔然,为其炼铁奴。柔然由于长期与铁勒(柔然人称其为高车人,因为他们高大的车轮而得名)战争而削弱;546年,阿史那部首领脱离柔然,并于550年在首领土门的带领下击败铁勒,552年又打败柔然,建立了政权,自称伊利可汗;同时期的中国将其音译为“突厥”。553年木杆可汗在都斤山(又作“郁都军山”、“乌德鞬山”,今蒙古国杭爱山)建立王庭。全盛时,其疆域东至大兴安岭,西抵西海(咸海),北越贝加尔湖,南接阿姆河南,建立了官制,有立法,有文字。583年,隋将长孙晟用离间之计,使突厥汗国分裂为东西两部,后在屡次与隋朝的战争中战败而走向衰落(参见隋与突厥之战)。唐太宗贞观四年(630年)与薛延陀攻灭东突厥,唐高宗显庆四年(658年)唐又灭西突厥,余部西迁中亚(参见唐灭东、西突厥之战)。在唐高宗末年(682年),再度建立后突厥汗国,最后在745年,后突厥帝国亡于回纥。","text2":"突厥汗国分裂成哪两个国家?","label":1} {"text1":"杨宁(),字彦谧,直隶歙县人。明朝政治人物。出自浙江余姚泗门双桥杨氏,为十一世。父杨升,考中洪武二十九年(1396年)乡试,官徽州府学教授,故入籍徽州。永乐丁酉(1417年)举人,宣德五年(1430年)庚戌进士,授刑部主事,善交权贵。正统初年,随尚书魏源宣抚大同等地。时王振把持朝政,正统四年(1439年)随都督吴亮征讨麓川(今云南省瑞丽市)土司思任发。正统六年(1441年)再随兵部尚书王骥征讨思任发,以功升刑部右侍郎。历任江西巡抚、礼部尚书。后以足疾改南京刑部尚书,正统七年为御史庄升所劾,明英宗复辟,被迫致仕。一年后去世。卒祀乡贤祠。同族同辈人有杨抚,正德十六年(1521年)进士,湖广提学副使。弟杨宜,正统十三年(1448)进士,广东按察司副使。后人有杨光先,明末清初时代,曾任清钦天监监正。","text2":"杨宁字什么?","label":1} {"text1":"《甜蜜小天使》()是日本漫画家赤冢不二夫的漫画作品,曾经三度由东映动画动画化。本作的漫画版初期投搞于集英社旗下的漫画杂志《Ribon》,后来转到讲谈社继续连载,亦有发行单行本与绘本。动画版则交由东映动画制作,1969年1月起开始播出,这是东映动画继《魔法使莎莉》(于1993年时成香港配音员梁少霞成名作)以后的另一部魔法少女动画作品。往后陆续制作第二作与第三作动画,但三作动画的制作与播出间隔了很长一段时间。原先「」的全名设定为「镜厚子」(),动画化后改为「加贺美厚子」,日语读音相同,但三作动画的日文表示不尽相同。厚子的形象亦曾在赤冢不二夫相关作品当中出现。由于当时华人圈对于日本动画的资讯较为缺乏,因此目前为止,本作没有正式或统一的中文译名(于香港播出时,将本作称为《-{小魔镜}-》)。中华电视公司(华视)于1991年5月6日至1992年6月22日播出时,将本作命名为《-{甜蜜小天使}-》,角色亦重命名,中文主题曲由汪石泉作词作曲、主唱不详、华视大乐队伴奏。本作香港版本主题曲由刘锡明主唱。2012年8月,与晨间节目《SUKKIRI!!》合作推出约1分钟的短篇Flash动画,为之后上映的真人电影版做宣传;厚子的形象也从小学生变成上班族,配音由平野绫担任。主角厚子是一名少女,透过镜子就能变身成任何模样,初期是设定为一面大镜子,只要对著镜子念出咒语就能变身,动画中则改为可随身携带,类似贝壳型状的变身盒。故事内容主要是围绕在主角与周遭的人们所发生的一切事件,透过变身能力解决他人的困难。动画版的故事设定上,三作各有不同,除了画风外,厚子的衣装、变身盒造型,与父母职业也有所不同。","text2":"本作的漫画版初期投搞于哪个杂志社?","label":1} {"text1":"希薇亚·普拉斯(英语:Sylvia Plath,),生于美国波士顿,儿童作家出身的美国天才诗人、小说家及短篇故事作家。希拉斯生于1932年10月27日的波士顿牙买加平原社区。母亲Aurelia Schober Plath (1906–1994)为奥地利裔, 父亲Otto Plath (1885–1940)为徳国血统,身为波士顿大学的生物学教授及昆虫学家。有弟弟Warren出生于1935年4月27日。除了较为著名的诗作外,普拉斯也以笔名Victoria Lucas创作了传记小说《》,书中主角,聪慧、有抱负的史密斯学院学生艾瑟·葛林伍德,便是普拉斯的化身,后于纽约时尚杂志实习期间,展开了一段精神崩溃的历程,正如同普拉斯在《Mademoiselle》杂志工作时,经历心理挣扎、自杀未遂的过程。普拉斯与女性文学天才安妮·塞克斯顿并称,被公认是自白诗重要的推动者之一。","text2":"普拉斯以什么笔名创作了传记小说?","label":1} {"text1":"短喙鼻鱼又称短吻鼻鱼,俗名剥皮仔、打铁婆、独角倒吊,为辐鳍鱼纲鲈形目刺尾鱼亚目刺尾鱼科的其中一个种。本鱼分布于印度太平洋海域,包括东非、红海、模里西斯、塞席尔、马尔地夫、斯里兰卡、安达曼群岛、日本、台湾、中国沿海、菲律宾、印尼、澳洲、新几内亚、马里亚纳群岛、马绍尔群岛、索罗门群岛、斐济群岛、万那杜、夏威夷群岛、法属玻里尼西亚、诺鲁、加拉巴哥群岛等海域。水深4至46公尺。本鱼体呈椭圆形而侧扁;体呈均匀的褐色,体侧在侧线下方有许多红褐色细横纹,尾鳍上有一橘红色弧形横带。头顶上的角状突起,超过吻长甚多。尾柄上有2枚骨质盾板,板中央有一突出颇高的锐脊。背鳍硬棘6枚、背鳍软条27至29枚、臀鳍硬棘2枚、臀鳍软条27至30枚。体长可达70公分。本鱼常成群在礁区水层中巡游,特别是在礁坡、礁外缘。幼鱼以藻类为食,成鱼则以浮游生物为食。食用鱼类,内脏及头部可能有毒性累积,故避免食用此部位。亦可供观赏。此鱼去腥味之方法乃以尖刀将鱼体侧面次下一、二刀后将鱼体弯曲以使血液流出,再将腹部内脏去除。尾柄棘会伤人,需注意。","text2":"短喙鼻鱼主要活动在什么深度的水域?","label":1} {"text1":"日产蓝鸟是日本日产汽车设计生产的小型车,开始在1957年生产,直到2001年结束。关于车名的由来,出自比利时作家莫里斯·梅特林克()所著小说《青鸟》()中带来希望的青鸟,象征该车型可替中小家庭带来无限的希望。目前,这款车在中国大陆地区,东风汽车曾制造U13型系,现已停产。不过深色的蓝鸟汽车至今依然可以在中国见到。另外,东风日产现售的名为蓝鸟的车型实为日产lannia,与01年即已停产的蓝鸟车系无任何联系。在台湾地区,这款车由裕隆汽车生产,先后由国产汽车与裕隆日产汽车销售。1960年起,裕隆汽车在台湾组装210型系(裕隆代号YLN-701),以「青鸟」为名,并随原厂改款导入310\/410\/510型系(裕隆代号YLN-702~YLN-706),之后停产数年。于1980年代初期先后导入910型系(裕隆代号YLN-911),以「吉利」为名。1984年导入U11型系(裕隆代号YLN-921\/YLN-923),以「吉利青鸟」为名。至1992年停产后,于1993年导入美制的U13,并以Altima为名于台湾销售,但成绩并不理想。2000年,裕隆汽车自行设计Sentra大改款(裕隆代号N16),豪华的外观与内装获日产采用做为Bluebird的后继车(日产代号G10)。2007年,裕隆再度引进G11,称为Bluebird Sylphy;至2013年由Super Sentra接棒销售。","text2":"日本蓝鸟的车名是怎么得来的?","label":1} {"text1":"小青,或称青青,是中国四大民间传说之一《白蛇传》中的主要角色之一,她是《白蛇传》主角白娘子(白素贞)的侍女。白素贞时常唤她为「青儿」或「青妹」,相貌美艳。后来白娘子被镇江金山寺法海禅师压制于雷峰塔下,小青逃出,白娘子被困于塔时,交代小青嫁给许仙为妾,以照顾许家。因说书人的个人喜好不同,小青的结局十分迥异。有些版本中小青与白娘子被同压于雷峰塔内,有些版本则让小青嫁给了许仙,甚至小青也与许仙生了一子,名曰许儒林,字梦龙;小许仕林两岁,与仕林同科进士。有的版本中小青潜心修道,终身未嫁,也有的版本小青嫁给了许仙的好友。不过,几乎所有版本,都是小青最后得道成为真仙。首次出现于冯梦龙小说集《警世通言》第二十八卷〈白娘子永镇雷峰塔〉中,为西湖内第三桥下一丈多长的青鱼精,《白娘子永镇雷峰塔》中描述小青身上穿著青衣服,头上一双角鬏,戴两条大红头须,插著两件首饰,手中捧著一个包儿,是个娇俏美貌的丫鬟。在最早的《白蛇传》戏曲「双蛇斗」中,小青则本是男儿身,名号为青峰山青峰宝洞中的“青蛇大仙”。由于偶遇白蛇,见她品貌端正,意欲与她结为伴侣,白不肯,两蛇相斗,青蛇败在白蛇手下,后来青蛇化作婢女,服侍白蛇。而后的文献中,她被改写为雌性蛇精,在今日流传的大多故事中,小青与白素贞皆为雌性蛇精,修道化成人形,白素贞为白蛇,小青则为青蛇,小青成为了白素贞的婢女,并伴著白素贞与许仙邂逅。《白蛇全传》中讲述小青是一条修炼了七百年的青蛇,而《雷峰塔奇传》中则讲述小青是一母青蛇精,在醉春楼中作巢,亦修行有八百余年,能飞腾变化。后来小青与白素贞遇上法海禅师,禅师将白蛇镇于雷峰塔中,永不得出,直到许仕林探母。有些版本则交代小青未被压在塔内,到了骊山到了向骊山老母学习雷法,再找法海报仇。还有些版本说小青受白娘子之托嫁给许仙为侧室,以照顾许仙与儿子,大多数版本中,小青没有子女;有些说书版本中,小青也与许仙生了一子,名曰许儒林,字梦龙,小白素贞之子许仕林两岁,与仕林同科进士。在《白蛇全传》中自白素贞被囚禁于雷峰塔后,小青则逃回北玄山黑风洞修炼20年的飞刀与三昧真火,发誓要烧毁雷峰塔,救出白娘子,并与法海决一死战。有些文献中则有描述小青持宝剑「龙泉青锋」,使用三昧真火或雷法烧塔的景象。","text2":"小青是哪部传说的主要角色之一?","label":1} {"text1":"《穿裘皮的维纳斯》()是奥地利作家利奥波德·范·萨克-马索克的代表作。这篇小说是马索克本人预想的史诗系列该隐的遗产中的一部分。《穿裘皮的维纳斯》属于该系列中的第一卷爱部分。本作品发表于1870年。小说的主题和人物灵感大量来自于马索克本人的生活。汪黛·冯·杜娜耶(小说的女主人公)是以一个新兴的文学作家Fanny Pistor为原型的。Pisor以一个虚构的名字女性男爵Bogdanoff和马索克来进行联系,马索克给出提高她作品的建议,使之适于出版,而Pistor取这样一个名字的目的是为了表达自己性格中奇妙的一面,而这方面对于马索克正是残暴和迷人的。小说描写了一个男人怀着对穿裘皮的维纳斯示爱的梦想的故事。开篇一个无名的叙述者将自己做的类似的梦讲述给朋友赛弗林,随后赛弗林将一份手稿\"一个超感觉论男人的忏悔\"交给他,并让他放弃对残暴女性的迷恋之梦。这份手稿讲述了一个叫做 Severin von Kusiemski 的故事,他十分着迷于 \"旺达·冯·杜娜耶\" ,自愿成为她的奴隶。他愿意受她的驱使,受她的惩罚,使自己成为她对之握有生杀予夺权利的财产。赛弗林将他经历的这种感觉称作\"超感觉\"。赛弗林和旺达旅行到意大利佛罗伦萨,在这次旅途当中,赛弗林作为旺达的仆人,更名为\"格列高\"。在佛罗伦萨的时候,旺达粗鲁地对待赛弗林,同时雇用了三个非洲女人以达到支配他的目的。当旺达遇到一个称为 Alexis Papadopolis 的英雄式的人物时,她与赛弗林之间的危机终于到来,在本书的结尾部分,赛弗林被旺达的新情人所侮辱,这时赛弗林放弃了从前的要求,反而开始宣称“在男女获得平等教育和公民权利时刻到来之前,男人必须处于统治女人的地位。”","text2":"小说结尾写了什么样的故事?","label":1} {"text1":"石中玉的父母石清和闵柔,年轻时曾和梅花拳掌门人梅芳姑有一段三角恋情,梅芳姑由爱生恨,在石中玉满周岁不久跟石中坚还没满月时,趁石清外出,到玄素庄攻击闵柔母子,闵柔防备不力痛失石中坚,因此开始溺爱石中玉,导致石中玉逐渐养成顽劣狡猾的性格。后来石清夫妇将他送上雪山派习武,拜风火神龙之称的封万里为师,封万里管教不善,让石中玉企图强奸掌门人白自在的孙女白阿绣。石中玉为躲避门规重罚,当日便逃下山远离雪山派追杀。石中玉偶遇长乐帮,因长乐帮上下无人出任帮主,接受赏善罚恶铜牌令赴侠客岛送死,于是拱他出任长乐帮帮主。石中玉不久也明白长乐帮的用意,于是不告而别(据长乐帮众所言,前帮主不愿独自赴会打算隐居,而石破天将其打倒夺得帮主,作威作福),躲在市井妓院间醉生梦死,逍遥快活。军师贝海石无奈,只得找了忠厚老实但相貌酷似石中玉的石破天来顶替他做长乐帮帮主。赏善罚恶使者查明长乐帮现任帮主石破天非石中玉,于是将石中玉找了出来,石中玉不愿被白万剑等人捉回雪山派,和情人丁珰密谋,骗取石破天的善良要他和自己对调。石中玉重获自由后,回长乐帮时被上门寻找石破天的谢烟客误以为是石破天,对他提出关于玄铁令的要求,石中玉命他歼灭雪山派。但是谢烟客在雪山派见到真正的石破天,得知自己被石中玉欺骗,原本打算将石中玉击毙。石破天不忍闵柔痛失爱子,决定要求谢烟客管教石中玉,同时告诉石中玉得好生伺候谢烟客的饮食起居,从此石中玉就在摩天崖和谢烟客一起生活。","text2":"最后,石中玉和谢烟客在哪里生活?","label":1} {"text1":"《明日·今天的未来》是一本由香港的财政司司长办公室制作的财政预算案咨询漫画。其作品由香港财政司长曾俊华提出、香港漫画家李志清执笔绘画、张正常担任编剧。制作费用约十万港元,并于2009年1月中旬出版。漫画内容在政府财政预算案的网站里连载,供市民自由观看。这也是香港政府第一次以漫画形式宣传和教育。在推出漫画后,曾俊华表示已收到近五千多份意见。漫画主要透过主角一段跨越时空的奇幻旅程,以令到市民明白未来建基于今日之上,当中的内容包含著各种在香港现时在发生的社会问题,例如是失业及金融海啸所带来的冲击等。而在剧情里的同时,也提供著一些经济数据,好让读者运用那些数据来思考一下怎样解决其问题。在漫画里,书中的主角阿日跟他的家人讨论金融海啸之下香港经济的何去何从。","text2":"书中的主角是谁?","label":1} {"text1":"入江俊郎(),日本东京都人,曾经历最高裁判所判事(最高法院法官)。1924年东京帝大法律系毕业。历任内阁法制局参事官,行政裁判所评定官,法制局部长、法制局次长、法制局长宫,贵族院议员,国立国会图书馆专门调查员,东京帝大、庆应大学讲师等职。1948年任众议院法制局长。1951年赴美国考察议会制度。1952年8月30日以51岁之龄成为史上最年轻的最高法院法官。1954年任务法省法制审议会委员及行政诉讼部会长。1971年1月9日,以退休年龄辞去官职。最高法院法官在任期间6707天是历代第1位(2008年2月现在)。辞去官职后就任了驹泽大学教授。亲美的右翼政客。爱好诗歌和谣曲。著有《宪法要论》、《地方自治法提义》等。其兄入江克己是关东日野柴油机公司总经理。","text2":"入江俊郎是何时辞去官职的?","label":1} {"text1":"尚健(伊江王子朝直,),琉球国摄政,童名思龟金。其名乘原为「朝忠」,后因避萨摩藩藩主岛津忠义之讳,改为「朝直」。尚健是尚灏王第五子,母为尚灏王的夫人小那霸阿护母志良礼。后因伊江御殿向世俊(伊江按司朝平)无嗣,被过继给向世俊为嗣子。1835年向世俊死后,成为伊江御殿第十一世家督。1859年,三司官马克承(小禄亲方良忠)、物奉行向汝霖(恩河亲方朝恒)、异国通事牧志朝忠被人告发有不当行为(牧志恩河事件),由尚健为法官断案,将他们革职流放或监禁。1868年作为明治维新的庆贺正使前往日本东京祝贺。1872年起出任琉球国摄政。1875年因受向有恒的牵连遭人弹劾,辞去摄政一职。1879年琉球国被日本兼并后,随尚泰王被迁往东京,被封为男爵。1896年病死于东京,享年79岁。冲绳政治家伊江朝雄是他的后代。|-style=\"text-align: center; background: #FFE4E1;\"","text2":"尚健从什么时候开始出任琉球国摄政?","label":1} {"text1":"59街是纽约市曼哈顿区的一条东西向街道,从约克大道到西侧高速公路,在第九大道\/哥伦布大道与第八大道\/中央公园西之间被时代华纳中心隔断。59街的大部分路段双向通行,但是在第九大道和第十大道单向通行,只允许西行。在第二大道,59街连接皇后大桥(Queensboro Bridge),此桥常被称为“59街桥”。59街构成曼哈顿中城与曼哈顿上城的边界。它将哥伦布圆环一分为二,在第五大道和中央公园西之间的路段构成中央公园的南部边界。沿中央公园的路段标为中央公园南。在59街以北,曼哈顿的街区在中央公园两侧分为上西城和上东城,在上西城,59街以北更名为阿姆斯特丹大道,第十一大道更名为West End Avenue.","text2":"59街的通行情况?","label":1} {"text1":"蕊蕊(,),原名陈蕊蕊,香港女歌手、演员及前舞蹈员。前无线电视舞蹈艺员。蕊蕊中学时就读乐善堂梁植伟纪念中学,其后毕业于EF国际语言学校(伦敦分校),主修语言为英语。2004年入读无线电视第9期舞蹈艺员训练班,并签约无线电视。至2009年因演出该台综艺节目《耳分高下》,且担任「Sing之天使」其一「柑桔」,使她渐渐受人注目。同年11月约满后,与唱片公司自由行娱乐(Freeway Entertainment)签约,经理人是汉洋。2011年2月,她发表首支派台歌曲《Make Up Baby》。2014年4月,蕊蕊与邓咏雪、陶枳樽和何丽娟组成女子歌唱组合「HunterZ」。2016年,她离开自由行娱乐和签约新公司。蕊蕊与台湾艺人金刚于2013年开始交往,但两人在2016年4月因性格不合而结束恋情。","text2":"蕊蕊的原名是什么?","label":1} {"text1":"渤海武王()讳大武艺,是渤海国第二代君主,在位期间719年至737年。大武艺弃唐朝正朔,建元为仁安。不过,在与唐朝交往时仍然用唐朝的开元年号。他在任期间,渤海国的疆域扩张东至绥芬河、及今俄罗斯沿海州、南至朝鲜半岛大同江、西至松花江流域。726年,他阻碍黑水靺鞨归附唐朝,他亲唐的弟弟大门艺逃至长安。727年,派遣高仁义出使至日本以遏制新罗, 728年,日本遣使向渤海答谢。 从此渤海国一直与日本维持着很好的外交和商贸关系。732年,大武艺派将军张文休率领水军跨渤海湾进攻唐朝登州(山东蓬莱),杀害登州刺史韦俊。葛福顺奉唐玄宗之命率兵征讨渤海国。733年,新罗奉唐朝之命从南方攻击渤海,由于大雪,士卒死者过半,无功而还。大武艺十分怨恨大门艺,秘密派刺客在洛阳天津桥南暗杀大门艺,但大门艺没有死;唐玄宗命令河南府搜捕大武艺派来的刺客,把他们全部杀死。737年,大武艺去世,谥号武王。《桓檀古记》称大武艺被尊为光宗、武皇帝。","text2":"哪一年日本遣使向渤海答谢?","label":1} {"text1":"律伦(,)是英国英格兰的郡、单一管理区。除了行政总部渥咸、小镇阿平厄姆外,郡内都是人口不多的小村庄。律伦既是名誉郡,又是单一管理区,无论把它看待成那种身分,它的人口和面积都分别是38,300、382平方公里。律伦伯爵、律伦公爵是英格兰贵族的头衔。首位律伦伯爵Edward of Norwich, 2nd Duke of York在1385年受封。首任律伦伯公爵约翰·曼纳斯在1703年受封,现任公爵大卫·曼纳斯在1999年受封。《1894年地方政府法案》生效后,律伦郡下辖3个乡区(Rural district):奥克姆区(Oakham Rural District)、阿平厄姆乡区(Uppingham Rural District)、凯顿乡区(Ketton Rural District)。1911年,奥克姆在奥克姆乡区分割出来,成为urban district。《1972年地方政府法案》在1974年4月1日生效,拉特兰成为莱斯特郡的非都市区。1992年成立的英格兰地方政府委员会(Local Government Commission for England)建议律伦成为单一管理区,不再是非都市区,在1998年4月1日生效至今。律伦郡在2001年人口普查中统计得人口34,560,相比起1991年人口普查统计得人口33,228,上升了4%。下图显示英格兰48个名誉郡的分布情况。律伦东北与林肯郡相邻,东与剑桥郡相邻,南与北安普敦郡相邻,西、西北与莱斯特郡相邻。","text2":"《1894年地方政府法案》生效后,律伦郡下辖哪三个乡区?","label":1} {"text1":"杜文辉(),中国足球运动员,司职前锋。1998年,杜文辉正式进入北京国安三队。1999年,杜文辉和邵佳一、崔威、王硕等四人去法兰克福培训半年。2002年,杜文辉进入北京国安一线队,杜文辉在热身赛中因骨折养伤3个月。2002年北京国安成绩非常好,所以伤愈后全年他一场联赛没打。2008年3月30日,中超首轮最后一场比赛北京国安主场迎战河南建业。比赛进行到第78分钟,北京国安发动反击,陶伟传球时被对手得到,然后河南建业的防守队员竟然出现致命失误,球被杜文辉断到,杜文辉随即劲射破网,将场上比分扩大为2-0。杜文辉这一进球也成为了中超联赛历史上第2000个进球。2011年,杜文辉与北京国安合同完结并正式转会江苏舜天,在当年实行自由转会后,成为第一位以自由球员身份加盟到其他球队的球员。2012赛季结束后,杜文辉离开了江苏舜天。2013年7月,中甲球队湖南湘涛正式宣布与杜文辉签约。","text2":"杜文辉和哪些人曾去法兰克福培训半年?","label":1} {"text1":"夏威夷秧鸡()是夏威夷岛一种神秘的秧鸡,但现已灭绝。牠们不懂得飞,生活在灌丛带。在普纳地区附近发现了几个夏威夷秧鸡的标本,包括一些较深色及一些较浅色而有斑点的标本。最先采集的夏威夷秧鸡标本是较为浅色的形态,共有五个标本,分别存放在莱顿的荷兰国家自然科学博物馆及纽约的美国自然历史博物馆。另外几个较深色形态的标本则存放在剑桥的大学自然历史博物馆、伦敦的自然历史博物馆、维也纳、纽约及檀香山的毕晓普博物馆。1778年绘画的一幅图画中是较浅色的夏威夷秧鸡,有可能就是莱顿的标本。近年亦有发现牠们的亚化石。由于夏威夷秧鸡有两种形态,造成了很多混淆。根据采集夏威夷秧鸡标本的过程,牠们最有可能生活在欧胡岛及考艾岛,而考艾岛亦曾有一种类似体型的动物出现。不过考艾岛发现的骨头却属于较大体型的,而夏威夷秧鸡的标本则较细小。在夏威夷岛发现了一种较细小的田鸡属亚化石,但其大小只如雷仙岛秧鸡。另外在凯卢阿-科纳亦发现一些骨头,约较夏威夷秧鸡大15%。一般接受较浅色的形态是未成年的夏威夷秧鸡,但却仍有待DNA研究证实。不论如何,浅色及深色的形态都被认为是同一物种,一些异名如下:最后五个异名是指深色的形态。当中\"Rallus obscurus\"在被描述时仍未有发现深色的形态,描述有可能是根据在莱斯特大宅的标本而造的。夏威夷秧鸡浅色形态的标本是于1778年在詹姆斯·库克(James Cook)的第三次旅程采集的,相信于当时并没有发现深色的形态。大部份或全部深色形态的标本都是于1860年至1864年采集的。最近证实见到夏威夷秧鸡是于1884年,另一个存疑的观察则是于1893年。1887年的搜寻未能发现牠们,不过却不足以确定牠们的灭绝。夏威夷秧鸡的灭绝可能是因入侵的大家鼠、猫及狗。狩猎亦可能是其中一种原因。","text2":"夏威夷岛发现的田鸡属亚化石有多大?","label":1} {"text1":"八云()是一个由西日本旅客铁道(JR西日本)营运,往返于与间的L特急列车班次,途经过山阳本线、伯备线与山阴本线。列车取名自岛根县东部旧国名出云国的化称呼——「」()。八云号列车于1972年3月15日开设,为山阴地区岛根县东部(出云市、松江市)和鸟取县西部(米子市、日野郡)等地乘客提供前往冈山站转乘山阳新干线的速达服务,为阴阳连络路线之一,当时八云号列车以行驶。1982年7月1日起,伯备线全线及山阴本线部分路段电气化,八云号改派新制的日本国铁381系电车行走。由于伯备线主要行走于山区之间,弯路较多,所以八云号381系列车特别配备列车倾斜装置,以提升过弯速度,节省行车时间,提升铁路与阴阳联络高速巴士之间的竞争力。时至今日,JR西日本仍然派出381系列车行走八云号。当年日本国有铁道原先计划于上越新干线开通之后,把来往上野站至新潟站间途经高崎线、上越线和信越本线的在来线特急朱鹭号列车的转到八云号继续服务,但由于以下原因计划被搁置,八云号改为新制381系电车行走。随著行走北陆本线和信越本线、来往金泽站至新潟站之间的特急北越号列车于2015年因北陆新干线开通而取消,八云号列车是JR集团唯一所有派车均于国铁年代制造的电车特急。以下为现时八云号的运行概况。八云号来往~间,距离为220.7公里,每天15往返,班次编号为1-30号,单号为下行往出云市方向,双号为上行往冈山方向, 所有列车皆为禁烟车。冈山站 - 仓敷站 - (总社站) - 备中高梁站 - 新见站 - (生山站) - (根雨站) - (伯耆大山站) - 米子站 - 安来站 - 松江站 - (玉造温泉站) - (宍道站) - 出云市站","text2":"为什么八云号381系列车特别配备列车倾斜装置?","label":1} {"text1":"虎胆神猫(又称克劳船长)是一个两维平台游戏,描述了主角克劳船长和他的冒险故事。此游戏融合了加勒比海盗的传奇和动画片里猫与狗永久纠纷的传统,制造出一系列生动有趣的人物与情节。主角纳撒尼尔· 约瑟夫·克劳船长是一位威震(动物)天下的海盗猫,已多次逃出美卡犬王国(注:该国在英语写成‘Cocker-Spaniards’,是‘Cocker Spaniel’(美卡犬)的双关语)所设的法网。但在游戏的开头,他终于一场激烈的海战中被国王的走狗勒·罗舍击败并逮捕。当他被关押在勒·罗舍的岩石堡时,他竟然发现一位前死犯所遗留的一封信和一张不完整的宝藏图。信中透露了一个稀奇珍宝:九命护身符。该宝物是由九颗宝石组成,据说能给予发现者长生不老的能力。读了此信后,克劳船长决心逃出岩石堡,以寻找那宝藏图遗漏的部分和那九颗宝石。游戏就在此正式开始。为了取得传说中的《九命护身符》,克劳船长得避开(或消灭)路途上遇到的冤家,并闯过十四个大关,而一关会比一关难闯。这十四关分别为:","text2":"该游戏的主人公是谁?","label":1} {"text1":"梅西尔72(也称为M72或NGC 6981)是在宝瓶座的依个球状星团,在1780年8月29日被皮埃尔·梅香发现。梅西尔随后也在10月4日和5日观测到,并将之编入星表中的第72号。当时这两位都认为他是一个暗淡的星云,而不是现在所认知的球状星团。即使使用10吋(25公分)口径的望远镜观察这个星团,依然只能看出模糊与微弱的影象,但使用20英吋的望远镜就能提高解像力。M72距离地球大约53,000光年,与银心也有一段的距离。但也有资料认为M72与地球的距离是62,000光年,直径为42光年。虽然一般的星团都有一些最老年的恒星,但M72拥有一些蓝巨星,因而被认为是一个年轻的星团。","text2":"M72是谁发现的?","label":1} {"text1":"粒唇鲻(学名:),俗名乌鱼、乌仔、乌仔鱼,为辐鳍鱼纲鲻形目鲻科粒唇鲻属其中一种。本鱼分布于印度太平洋区,包括红海、东非、南非、马达加斯加、模里西斯、塞席尔群岛、留尼旺、阿曼、叶门、马尔地夫、斯里兰卡、巴基斯坦、孟加拉、印度、日本、泰国、马来西亚、缅甸、俄罗斯、中国、台湾、柬埔寨、新加坡、越南、菲律宾、印尼、澳洲、新喀里多尼亚、萨摩亚群岛、纽埃、东加、帛琉、吉里巴斯、马绍尔群岛等海域。该物种的模式产地在红海。水深0至40公尺。本鱼体延长,前部近圆筒型,后部侧扁。上唇1\/3处具突起或形成瘤状物。上下颔无齿,仅舌头有齿。背鳍硬棘4至5枚、背鳍软条8至9枚;臀鳍硬棘3枚、臀鳍鳍条9枚。一纵列鳞片37至41枚。胸鳍黄色,基底上部具一暗蓝色点。体长可长60公分。本鱼栖息在浅海与河口区,但不溯游入河川,初孵仔鱼,以摄食浮游生物为主,随著体长的增长,幼鱼食性渐由动物性转为植物性,以吞食海底淤泥中的矽藻及有机碎屑为食。成长迅速,周年于即以成熟。属于高经济价值的食用鱼,但没有乌鱼那么饱满的卵,体型也较小。为养殖鱼类,红烧、煮汤均宜。","text2":"粒唇鲻随着体长的变化,还有什么会发生变化?","label":1} {"text1":"朝鲜马鲛(学名:),又称高丽马加䲠,俗名阔腹、白北,为辐鳍鱼纲鲈形目鲭亚目鲭科的其中一种。本鱼分布于印度西太平洋区,包括印度、巴基斯坦、孟加拉、斯里兰卡、中国、台湾、香港、日本、韩国、印尼、马来西亚、缅甸、越南、泰国等海域。该物种的模式产地在日本。水深0至50公尺。本鱼体形较短,横断面略侧扁,腹部甚宽,头高大于头长,尾鳍大而深分叉;第二背鳍、臀鳍及尾鳍发达,背部灰绿色,腹部银白色,体侧有4至5个纵列暗色斑,背鳍硬棘14至17没;背鳍软条20至24没;臀鳍硬棘0没;臀鳍软条20至24枚;脊椎骨46至47个,体长可达1.5公尺。本鱼为暖海域之洄游鱼类,常在表层海域追食沙丁鱼等鱼类;游速快,喜栖息于海流湍急的岛礁附近。为高经济价值鱼类,体型大富脂肪,适合各种烹饪方式食用。","text2":"朝鲜马鲛喜欢在哪里栖息?","label":1} {"text1":"距翅雁(\"Plectropterus gambensis\"),又名直翅鹅,是一种分布在撒哈拉以南非洲湿地的鸭科。距翅雁长75-115厘米,平均重4-6.8公斤,有时可以达10公斤,雄鸟较雌鸟大。牠们是非洲最大的水鸟,也是世界上平均最大的鹅。牠们主要是黑色,面部白色,翼上有很大的白色斑纹。其指名亚种\"P. g. gambensis\"的腹部及两侧有很大片白色,而另一个在赞比西河的亚种\"P. g. niger\",腹部只有小片的白色。雄鸟不单较雌鸟大,面上也有较大片的红色,上颌的基底有一个瘤。距翅雁的翼上有距,其用途不明。距翅雁会在水边的丛林中筑巢,有时也会利用树孔、其他洞穴及锤头鹳的巢。牠们是很寂静的鸟类,只会在飞行发出轻哨声。距翅雁的数量丰富,且是群居的。牠们会在地上觅食,日间主要会在水边休息。距翅雁是《非洲-欧亚大陆迁徙水鸟保护协定》所保护的物种之一。","text2":"距翅雁一般分布在什么地方?","label":1} {"text1":"杰里德·贝勒斯(Jerryd Bayless)1988年8月20日生于亚利桑那州凤凰城,美国职业篮球运动员。他在2008年NBA选秀中第一轮第11顺位被印第安纳步行者队选中,随即被交换到波特兰开拓者队。作为一年级新人的贝勒斯得到了球队最高的场均19.7分、2.7个篮板和4.0次助攻。 他的球队在那个赛季取得了一个18胜13负的成绩,在太平洋十校联盟中取得的8胜10负,球队在NCAA首轮比赛中被西弗吉尼亚大学所淘汰。 在完成大学一个赛季的球赛后,贝勒斯决定提前参加2008年NBA选秀。在2008年NBA选秀中,波特兰开拓者队在与印第安纳步行者队的5人大交换后得到了贝勒斯。这次交易完成于2008年6月26日,但是直到7月9日开拓者总经理凯文·普理查德才确认此事,他表示“贝勒斯将可以扩充球队后场的活力,他是一位得到证明的射手和有力的竞争者。”在同年7月20日,他取得了2008年NBA维加斯夏季联赛最有价值球员的荣誉,在夏季联赛中他得到了场均29.8分,带领开拓者取得了3胜2负的成绩。2010年10月23日,贝勒斯被交易到新奥尔良黄蜂换未来的第一轮选秀权。2010年11月20日,贝勒斯被交易到了多伦多猛龙以及斯托贾科维奇和一些钱,换来了杰瑞特·杰克、马库斯·班克斯和大卫·安德森。在他的首次为多伦多猛禽出赛,贝勒斯得到13分, 2个篮板和2次助攻。12月11日,2010,贝勒斯追平职业生涯最高的31分,要配5个篮板和7次助攻,帮助猛龙反弹从一个25分的劣势以120:116击败底特律活塞。2012年7月6日,多伦多暴龙退出了420万美元的报价合同,以杰里让他不受限制的自由球员2010年11月20日,贝勒斯签下了一份多年合约加盟孟菲斯灰熊队。2013年6月,​​他行使球员选项310万美元。2014年1月6日,波士顿凯尔特人与曼斐斯灰熊进行双方交易,考特尼·李被交易到灰熊队,杰里德·贝勒斯则是被交易到凯尔特人。2014年7月17日,贝勒斯将与公鹿签下两年600万美元合约。","text2":"在什么时候波特兰开拓者队在与印第安纳步行者队的5人大交换后得到了贝勒斯?","label":1} {"text1":"服务研习(SERVICE-LEARNING)为一种将「服务」与「学习」相互结合的一门课程,以事先规划的社会服务活动与结构化设计的反思过程,让学生运用课堂所学贡献社区;同时可透过服务的过程中得到启发及省思,学习课堂中学不到的知识与经验,服务与学习二者在课程中具有某种程度的平衡关系,它是一种「从做中学」(Learning by doing)的学习经历,强调服务与学习目标同等重要,对所有服务与被服务的人都能加强其完成目标,以达「互惠」之功效。据文献记载,「服务研习」一词最早是由美国南部地区教育董事会(Southern Regional Educational Board)于一九六七年首先提出,经过美国各级学校的推广发展后,衍伸出许多的解释和做法。而我国比较被广泛接受的定义为:「由学校与社区结合,共同协助学生应用所学知能去服务他人,并且在服务过程中不断的学习成长。所以服务-学习是学校教育的一环」(林胜义,2002)。","text2":"在我国一般是怎么定义服务研习的?","label":1} {"text1":"索科特拉机场(英文:Socotra Airport,阿拉伯文:مطار سقطرى الدولي),是也门索科特拉岛上的一座机场,是该岛唯一的商业机场。建于1999年。大部分班次都是往大陆的穆卡拉(里扬机场),大部分飞机都会在那里进行一个技术修理然后才到也门首都:萨那或南部海港亚丁。机场位于岛的北部,但主要旅游景点在岛的西部Qalansiyah海滩,由机场到岛西部需时约2小时。该处是一个无污染,无噪音和浪漫的海滩。机场没有任何定期的交通来往哈迪布,但可以等约15分钟乘任何车辆,大部分车都是向西走。旅客可以乘巴士,但需要比其他居民付多点车费,大约须30至50也门里亚尔(1美元formula_1 200也门里亚尔)。除非旅客懂操阿拉伯语或可以与私家车司机交易,否则如没有妥当安排,到达Hadibo是较困难的。值得注意的是,目前索科特拉岛仍然是一个非发达的岛屿,没有工业和农业,不能预期提供各种食品和当地食品,因此大量的收费项目,都是由也门内地进口的,连瓶装水也不例外。2015年3月,由于沙特阿拉伯的军事行动,索科特拉机场航线停飞。","text2":"由机场到岛西部需时多久?","label":1} {"text1":"孙奋(),字子扬,孙权的第五子、生母为仲姬,夫人为刘氏。三国时东吴皇子,后封齐王及章安侯。吴大帝太元二年(公元252年),孙奋被立为齐王,居于武昌。同年,吴大帝孙权逝世,太傅诸葛恪掌权,他不希望诸王居住在长江江边战略要地,于是要孙奋迁居到豫章。孙奋怒而不肯从命,又数次违反法令。诸葛恪上书劝谏孙奋,希望他有所节制,守法行事,尽忠东吴,以免日后与兄长鲁王孙霸一样遭到杀身之祸。(《诸葛恪劝孙奋书》)孙奋收到诸葛恪的上书后大惧,于是移居到南昌。孙奋又沉迷于游猎,官员和属下都苦不堪言。后来诸葛恪被孙峻诛杀,孙奋去到芜湖,要到建业看朝中事变。傅相谢慈等劝谏孙奋,被孙奋杀害。孙奋因擅杀官吏而被废为庶人,流放到章安县。太平三年(公元258年),又被封为章安侯。吴末帝建衡二年(公元270年),孙皓宠妃左夫人王氏逝世。孙皓十分悲伤,整天都在哭,多个月都没有出外露面,民间更有传闻说孙皓已死,于是有人讹称孙奋与上虞侯孙奉两人之中将会有一个要被立为皇帝。孙奋生母仲姬的陵墓在豫章,豫章太守张俊对孙奋会被拥立的传闻半信半疑,为仲姬的陵墓打扫。孙皓知道后,以车裂处决张俊及夷其三族,并且诛杀孙奋及他五个儿子,封国废除。仲姬,孙奋生母,葬在豫章,因为豫章太守张俊讨好孙奋,打扫仲姬墓,触怒孙皓,导致张俊及其三族,与孙奋和他五个儿子,全部被杀。陈寿评曰:「(孙)奋不遵轨度,固取危亡之道也。然奋之诛夷,横遇飞祸矣。」《三国志·吴书·吴主五子传》","text2":"孙奋是谁的儿子?","label":1} {"text1":"《小咩的管家》(),是日本漫画家宫城理子的一部爱情漫画,2006年于《玛格丽特》上连载,台湾中文版由东立出版社代理,单行本20卷已发售。从小是乌冬面店老板女儿的东云芽衣(荣仓奈奈饰演),某天父母因事故身亡只留下她一个人,却突然出现一名男子,自称是她的管家柴田理人(水嶋斐吕饰演),并告知芽衣是日本第一财团本乡家的继承人。为了把芽衣培养成真正的名门淑女,她被送进了著名的圣露丝亚女子学院。这里所有人也和芽衣一样,身边都跟著一个英俊的男管家。不同的是,大家都是真正的淑女名媛,无法适应上流社会生活的芽衣和所有人都格格不入。青梅竹马的玩伴柴田剑人(佐藤健饰演)为保护芽衣成为见习管家进入学院,再加上喜欢芽衣管家的人是学校里的老大——露丝亚,本乡诗织(山田优饰演),芽衣本来平凡简单的生活真实发生了翻天覆地的变化……2011年日本宝冢歌剧团改编成舞台剧,1~2月由宝冢歌剧团星组在宝冢Bow Hall和日本青年馆初演。","text2":"《小咩的管家》是谁的作品?","label":1} {"text1":"桂绍彬(),安徽省六安县人。中国人民解放军海军将领,开国少将。1931年参加红四方面军,历任独立第15团营部书记,红25军第73师第218团营部书记,第271团政治处秘书长,红31军第91师政治部秘书长等,并参加长征。抗战中任八路军第129师第386旅第771团司令部政治指导员,第772团政治处宣传股长,太岳军区政治部组织部部长,太岳军区政治部主任,晋冀鲁豫野战军8纵政治部主任,华北军区野战第一兵团8纵队政治部主任,陆军第60军政治部主任、副政委等。中华人民共和国成立后,他历任中国人民解放军海军航空学校政委,中南军区海军副政委兼政治部主任,南海舰队副政委、海军顾问,是中国海军航空兵的创建人之一。1955年,被授予海军少将军衔并荣获二级八一勋章、二级独立自由勋章、一级解放勋章。中共七大代表。2014年去世。","text2":"桂绍彬是哪里人?","label":1} {"text1":"塞斯·亚伦·罗根(,,)是一位加拿大男演员、编剧、喜剧演员、监制和导演。他在青少年时期曾经演出栋笃笑长达4年的时间,于16岁时获得了温哥华业余喜剧比赛(Vancouver Amateur Comedy Contest)第二名,因此开启了他的演艺生涯。在居住于加拿大期间,他曾经参加电视影集《怪胎们》(1999年-2000年)的试镜,并被录取演出一名配角。为了参与演出工作,他迁居至美国洛杉矶,《怪胎们》在播映一季后便停播。之后他取得了在现场喜剧《主修未定》(2001年–2002年)的演出,并成为该剧的编剧之一。在成为影集《Ali G个人秀》(2004年)的编剧之一后,他和该剧的其他编剧共同获得了艾美奖的提名,接著他在导演贾德·艾帕托的提携下开始在电影界发展。他曾经在艾帕托首次执导的电影《四十处男》(2005年)中演出主要的配角,并同时为该片的协同制作人。罗根的演出获得影评人的赞扬后,环球影业同意让他在艾帕托的下一部电影《好孕临门》(2007年)中成为主角。他与搭档伊凡·戈博合作多部喜剧,包括《男孩我最坏》(2007年)、《菠萝快递》(2008年)和《大明星世界末日》(2013年)。此外罗根也参与了2011年电影《青蜂侠》的演出。罗根也曾为《荷顿奇遇记》(2008年)、《功夫熊猫》(2008年)、《怪兽大战外星人》(2009年)和《我们撞到外星人》(2011年)中担任配音。塞斯·罗根的父亲Mark Rogen是美国人,在非营利组织工作。母亲Sandy是加拿大人,是一位社会工作者。父母年轻时在以色列的基布兹相遇。塞斯·罗根评价自己的父母为「激进的犹太社会主义者」。塞斯·罗根是俄罗斯犹太人后裔。13岁时塞斯·罗根接受了犹太教的男孩成年礼(Bar Mitzvah)仪式。小时候他参加了位于加拿大不列颠哥伦比亚省的犹太夏令营Camp Miriam(学习犹太历史和希伯来文,并探索犹太人身份的营地)。毕业于温哥华的灰点中学。","text2":"塞斯·亚伦·罗根为什么迁居至美国洛杉矶?","label":1} {"text1":"花龟(学名:)又称中华花龟、斑龟、台湾斑龟、长尾龟、珍珠龟、台湾龟、梅花龟,为龟科花龟属的爬行动物。分布于中国、越南、台湾等地,常生活于低海拔的水域,如池塘、运河,缓流的河流中。该物种的模式产地在中国。中华花龟的性格较温和,头部、颈部及四肢的皮肤上都长著亮绿色和黑色的条纹,幼体背甲呈浅灰绿色,有三条明显的脊棱。成年背甲会变为偏向棕色,背甲上的其中两条脊棱会渐渐消失。腹甲则偏向象牙色,每块甲亦带有黑斑。雄性的尾巴较为粗、长,排泄孔离甲壳较远,而成年的雌龟体型会大于雄龟。成年个体通常体长20至22厘米,最大体长可达25厘米。中华花龟为杂食性动物,会食鱼、 虾、水中植物等。幼年偏向肉食,成年偏向草食。在交配后,雌龟每窝可以下5-20颗蛋,孵化需时约60天可。中华花龟因巴西龟的入侵而令野生数量有所减少,现已被列入濒危野生动植物种国际贸易公约附录三中。而于香港、中国大陆及台湾亦为受欢迎的宠物龟只。","text2":"中华花龟以什么为食?","label":1} {"text1":"黄斑鲾(学名:),俗名碗米仔、金钱仔,为辐鳍鱼纲鲈形目鲾科的其中一个种。本鱼分布于印度太平洋区,包括东非、马达加斯加、模里西斯、塞席尔群岛、亚丁湾、红海、波斯湾、马尔地夫、巴基斯坦、印度、斯里兰卡、孟加拉湾、安达曼海、泰国、越南、马来西亚、柬埔寨、台湾、中国沿海、日本、韩国、菲律宾、印尼、澳洲、新几内亚、马里亚纳群岛、马绍尔群岛、帛琉、密克罗尼西亚、所罗门群岛、诺鲁、新喀里多尼亚等海域。水深10至100公尺。本鱼头部裸出无鳞。侧线仅达背鳍基底后端。腹缘的轮廓较背缘突出。标准体长为体高2倍,头长为眼径的3.5倍。背部淡黄银白,散布著许多虫纹状的暗色斑纹;腹部银白色。背鳍与臀鳍鳍条上具有橙黄色的小点。背鳍硬棘8枚、软条16枚;臀鳍硬棘3枚、软条14枚。体长可达15公分。本鱼属于热带沿海小型鱼,大都栖息于沙泥底沿岸。成群觅食于沙泥底海床。以底栖动物及浮游生物为主食。属于小型食用鱼,美味但多刺,适合煮清汤。","text2":"黄斑鲾有什么食用价值?","label":1} {"text1":"太的黄鲫(学名:\"\"),为辐鳍鱼纲鲱形目鲱亚目鳀科的其中一种。本鱼广泛分布印度西太平洋区,包括印度、孟加拉湾、斯里兰卡、缅甸、泰国、马来西亚、柬埔寨、越南、中国南海、台湾、印尼苏门答腊、婆罗洲、爪哇岛附近等海域。水深5至50公尺。本鱼体甚扁,背缘窄,腹缘有强锐的棱鳞。口大倾斜,口裂窄长。上颌稍长于下颌。上颌骨细长,不达鳃盖开口。胸鳍第一鳍条延长为丝状,向后达臀鳍起点。体背部青灰色,体侧和腹部银白带有黄色。吻部和头侧中部淡金黄色,鳃盖内面橘黄色。背鳍前方至头顶黑褐色。背鳍和尾鳍金黄色,背鳍末端、尾鳍上缘和后缘灰黑色。腹鳍白色,尖端黄色。体长可达20公分。本鱼近海洄游性鱼类,偶会进入岸边、港湾觅食。食用鱼,但体薄肉少,常晒成鱼干或是当下杂鱼处理。","text2":"太的黄作为食用鱼的特点是什么?","label":1} {"text1":"公差与配合是机械和建筑工程中涉及机械零件和建筑构件的制造、装配和互换性的一项重要的基础性标准化科目,属于强制性标准。它主要规范机械零部件等在制造过程中的误差大小和误差偏移量,以便机械零部件制造者和使用者根据最佳经济性的原则选择合适的误差等级(或公差等级),以达到其所希望的装配适合程度或互换性范围。注意,这里的公差([英] tolerance)概念与数字中的公差概念不同,它是机械和建筑工程行业基础标准中所规定的一系列数值范围,给出了与标定值相偏差的(法定)极限容许量。遵循公差与配合标准的实际意义是,使得由不同制造商提供的例如标定为同一规格的螺栓和螺母,可以不经再加工即可顺利完成装配。","text2":"公差与配合有什么作用?","label":1} {"text1":"板棍球(英语:Hurling;爱尔兰语:iománaíocht、iomáint),又称爱尔兰式曲棍球,是一种使用前端为平板的球棍运球与击球的团队球类运动,与盖尔式足球并称为爱尔兰两大运动。板棍球比赛双方各上场15名球员,比赛时间全国性70分钟,地区性60分钟,青少年组50分钟,分成上、下半场。比赛场地与盖尔式足球一样,为长方形草地,两端线中央各有一个「H」形球门。比赛的目的是将球用板棍打击射门得分,比赛时间终了时,以得分多者为胜队。持球员向前推进时,会结合握球、运球(将球置放于球棍前端板面上),然后用板棍打击传球给其他队友,或直接射门得分。板棍球与盖尔式足球一样,是由爱尔兰最大的运动组织-盖尔运动协会所主导。","text2":"板棍球比赛双方各上场多少人?","label":1} {"text1":"复线铁路,或双线铁路,是指在同一时间,两个相对的通行方向的列车互不干扰的铁路。有别于单线铁路,单线铁路通常每隔一段距离就设置一个会车时使用的车站,称为“会让站”,用于相对行驶的列车停车避让。复线铁路有点类似于“分道行驶”的公路,通过合理的设置,没有正面相撞的危险,可以大大提高运输的效率。但是复线铁路显然需要更宽的路基,在山区等地形困难的地段,有时不得不仍然只建设单线铁路,甚至部分仅供会车使用的车站不得不建在隧道中和桥梁上。复线铁路解决了单线铁路的会车的问题,但并未解决同方向不同速度的列车的超车问题。快速列车要超越慢速列车有两个方式:各国的繁忙铁路干线、高速铁路、城市轨道交通系统为了实现高密度和高效率,大部分都是复线铁路。平行复线是指两条铁路线路相距很近(中国铁路规定普速复线相距不小于4米,高速复线相距不小于4.4米),走向大致平行,沿途接轨的各车站相同。一般在地势简单或者一次新建成复线的情况下采用这种情况,具有占地节省、投资节省、共用车站的技术设备与行车设施等优点。所经过的桥梁、隧道往往都是复线桥、复线隧道。非平行复线是指两条线路线路虽然也是分为上下行固定行车方向,但是相距可能较远,线路走向可能不一致,沿途接轨的车站也可能不一致的情况。这是因为沿线地势复杂困难,难以布置双线;或者是既有单线运量饱和而新增建第二线改为复线行车,既有老线与第二线由于建设年代不同导致设计、施工技术差别巨大,往往是既有老线沿山麓河谷曲折展线,而新建第二线大量采用桥梁、隧道的捷径线。西康铁路的秦岭特长隧道就是因地势与施工限制,采取两条单线隧道。而丰沙铁路、陇海铁路的宝鸡——天水段,都是在既有单线技术条件下非常落后的情况下,新增二线采取了非常高的线路标准,桥隧比很大。双线铁路有复线与单线并列两种形式,差别在于列车行驶的安排,复线的情况列车在各别行车线只会向固定方向行驶,而单线并列则是各别行车线都可以双向运行。单线并列在台铁称为“双单线”,而台湾高铁称为“双线”。双线铁路并非指有两条铁轨的“双轨铁路”,而单线铁路也非指只有一条铁轨的“单轨铁路”。","text2":"什么是平行复线?","label":1} {"text1":"招重文(Chiu Chung Man,),生于香港,已退役香港足球运动员,司职中场,前香港奥运足球代表队及香港足球代表队队员,持有亚洲足协B级教练文凭​,现时担任香港超级联赛球队梦想FC技术总监。招重文生于香港,1987-88年度,年仅18岁的招重文,便随「升班马」美青初次在甲组亮相,并且表现相当不俗,在香港足球明星选举当选最佳年青球员。闯出名堂的招重文,翌年便「上山」加盟南华,并连续两届于香港足球明星选举当选最佳年青球员。惟在南华一直未能站稳正选位置,遂于1994年转投商业球队好易通,并成为球队中场主力。1997年夏天,香港足球总会仿效中超实施甲组联赛球员季前体测,招重文原本转投星岛,患有哮喘的他因未能通过体测,无法在甲组赛事上阵,被迫转投乙组联赛球队中福。1998-99年度球季,招重文重返甲组加盟二合,终凭努力成功通过体测,得以继续甲组球员生涯,并首次入选香港足球明星选举最佳11人。近年招重文仍有客串于乙丙组赛事上阵,2008-09年度球季,便协助骏升大中获得香港乙组足球联赛亚军,首次取得升班甲组资格。2015年10月15日,招重文代表「519香港传奇足球队」,在小西湾运动场举行在的「英超足球大师邀请赛」迎战「英超足球大师队」并取得一个入球最终以4–5不敌对手。招重文1989年入选新组成的第一届香港奥运队,参加过1992年巴塞隆拿奥运足球外围赛。香港足球代表队于1990年7月18日至8月10日前往欧洲集训,招重文首次入选。招重文退役后,曾担任南华青训教练。2008-09年度球季,曾执教过淦源,2011年夏天开始担任晨曦助教。2012年招重文升任教练,与陈发枝、谭兆伟共同执教。2014年开始担任中西区名校圣公会圣彼得小学足球队主教练,球队在多个赛事中表现出色。于2017年7月担任新成立的香港超级联赛球队梦想FC技术总监。","text2":"招重文出生在什么地方?","label":1} {"text1":"阿扎迪自由纪念塔(),位于伊朗首都德黑兰梅赫拉巴德国际机场附近,是德黑兰的地标,也是伊朗的象征。阿扎迪自由纪念塔原称作沙希亚德塔或国王纪念塔(),建成于1971年,为纪念波斯帝国建国2500周年献礼。1979年伊朗发生伊斯兰革命,巴列维王朝被推翻,伊朗伊斯兰共和国建立,国王纪念塔也改名为自由塔,象征伊朗自由神权时代的来临。塔高45米,塔基长63米,宽42米,呈灰白色,采用钢筋水泥和2500块产自伊斯法罕的大理石建成,象征波斯帝国建国的2500个岁月。阿扎迪塔附近设有一系列配套设施,包括博物馆和影像剧院。以阿扎迪塔为中心的阿扎迪广场有五万平方米大,广场有大型喷泉和绿地,每年伊朗国庆日,此处会举行盛大的阅兵仪式。","text2":"该塔建成于什么时间?","label":1} {"text1":"石谷站()是一个城市铁路东郊及伊拉瓦拉线的车站,位于悉尼南部的石谷,这里附近亦是石谷地方政府区域的总部所在地。车站跟随著路线在1884年10月开幕。每小时会有4班列车,繁忙时间会加设班次。由于往邦迪联运的列车会由2号或4号月台开出,因此乘客需在大堂的电子显示器上留意下一班列车会在那一个月台上开出;同一样地,往瀑布\/克罗努拉的列车会由3号或5号月台开出,亦需要在电子显示器上留意下一班列车会在那一个月台上开出。在2006年的时间表,南海岸线列车会在繁忙时间进入此站。1号月台\"(不常用)\"2号月台3号月台4号月台
5号月台
Stand A:Stand B:Stand C:Stand D:Stand E:Stand F:Stand G:Stand H:车站设有升降机并有职员在车站开放时间当值。","text2":"石谷站的班次情况是怎样的?","label":1} {"text1":"梅方,是一名中国足球运动员,场上位置是后卫,现在效力于中国足球超级联赛球队广州恒大。梅方出自武汉光谷青训体系,2008年进入武汉光谷一线队。当年8月,他和队友汪洋一起法乙球队斯特拉斯堡试训。 10月初,武汉光谷因不满中国足协的判罚而宣布退出中超联赛。梅方和汪洋再次到欧洲接受试训,11月前往德甲球队沃尔夫斯堡, 12月初又一起在比利时球队色格拉布鲁日试训。 梅方和汪洋都同时得到色格拉布鲁日的肯定并被邀请签约,但是两人回国后却因为湖北队将参加2009年的全运会而被湖北省足协大力挽留。最终梅方决定放弃留洋,留在武汉加盟了刚刚成立的湖北绿茵。梅方在2009赛季是湖北绿茵的主力球员,帮助球队杀入该赛季中乙联赛决赛,升级到中甲联赛。在决赛中,梅方射进球员生涯的首个联赛进球,不过他在点球决战中第一个出场并罚失点球,最终湖北绿茵不敌湖南湘涛屈居亚军。 2010年1月,他再次拒绝色格拉布鲁日的邀请,留队征战中甲联赛。 他在中甲联赛中继续作为球队的主力出战,并在2012年初和球队续约两年。 2012年3月,他被任命为球队的新任队长, 并在2012赛季率领武汉卓尔以联赛亚军的成绩升级到中超联赛。2013赛季,武汉卓尔在中超的成绩不佳,一直位于降级区。2013年8月18日,在联赛第21轮客场1比5惨败给广州富力后,武汉卓尔宣布剥夺梅方队长的职务。武汉卓尔以联赛倒数第一名的成绩降回中甲联赛后,合同即将到期的梅方一度传出有可能会转会到北京国安。 不过在2014年1月1日,他选择自由转会加盟中超卫冕冠军广州恒大。 2014年2月26日,梅方在2014年亚足联冠军联赛小组赛首轮广州恒大主场4比2逆转墨尔本胜利最后时刻替补亚历山德罗·迪亚曼蒂出场,首次代表广州恒大参加正式比赛。2014年6月18日,梅方在中国2比0击败马其顿的友谊赛首发出场,上演成年国家队首秀。2015年1月,梅方入选中国国家男子足球队2015年亚洲杯23人大名单,并成为球队主力后卫之一,可担任中后卫以及边后卫。","text2":"梅方哪一年进入武汉光谷一线队?","label":1} {"text1":"紫珠属(学名:\"\")是唇形科中的一个属,其中的植物主要是灌木和小型的树。根据不同的生物学家的定义它有40到150个种。大多数这些种生长在亚洲东部和东南部,其它生长在澳大利亚、北美洲的东南部和中美洲。紫珠属中生长在温带的种是落叶植物,生长在热带的种是常绿植物。叶简单,生长在茎的两侧,5至25厘米长。花成簇,白色至粉色。果实是浆果,直径2至5毫米,粉色至红紫色,带有一种非常不寻常的金属光泽。叶子落掉后它们在光秃秃的树干上成簇非常明显。浆果可以一直维持到冬天或者旱季,是鸟和其它动物的重要食物。不过动物只有在找不到其它事物时才会吃紫珠属的果实。紫珠属的果实味道非常不好,不适宜人吃。一些鳞翅目动物的幼虫吃紫珠属的果实。美洲紫珠是一种自然的驱虫剂。其中有四种物质被发现有驱虫效应,包括冰片。它也对传播黄热病、疟疾的蚊子和传播莱姆病的蜱有驱虫效应。美国农业部农业科研局把这个效应申报了专利。","text2":"紫珠属果实的外形有什么特点?","label":1} {"text1":"1960年美国总统选举,由民主党参议员约翰·肯尼迪以极微弱优势击败时任美国副总统理查德·尼克松成为总统。1960年,随著德怀特·艾森豪两任总统任期的即将结束。艾森豪的副总统理查德·尼克松成为共和党总统候选人,在电视辩论举行前,尼克森占尽优势。民主党提名马萨诸塞州参议员约翰·肯尼迪。他是参选的主要政党的第二位罗马天主教徒总统候选人(前一位是1928年的民主党人艾尔·史密斯)。在竞选的电视辩论中,肯尼迪指责在艾森豪和尼克松的共和党统治下,在冷战中,美国在军事上和经济上都落到了苏联后面,如果他当选总统,将“使美国再次运行起来”。尼克松作出回应说,如果他当选,将会继续艾森豪带给这个国家的“和平与繁荣”,国家正处于冷战之中,肯尼迪担当总统责任太年轻,缺少经验。最终肯尼迪在总统选举中以大约十万票(0.1%)的差距赢得选举。肯尼迪赢得303张选举人票,选举人票比1916年美国总统选举以来的任何一次总统选举都更接近,肯尼迪在普选中超出的票数也是美国历史上最接近的。而尼克森也成为首位赢得过半数州分,但输掉选举的总统候选人。一些历史学家对1960年选举是否有选举欺诈及舞弊提出争议,因为肯尼迪赢出的州中,很多州都不超过五个百份比,有数个州的选票十分接近,尼克森当时绝对可以提出要求重点选票,但他并没有这样做。只要重点选票后赢得其中两个州得克萨斯州和伊利诺州的选举人票,那么尼克森便成为总统了。这也是阿拉斯加州和夏威夷州首次参加总统选举,它们刚刚在前一年的1月3日和8月21日获得州的地位。这次选举中,夏威夷州支持民主党的肯尼迪,而阿拉斯加州支持共和党的尼克森,在此后的总统选举中大部分时间皆维持此格局,夏威夷州支持民主党,阿拉斯加州支持共和党。","text2":"夏威夷州支持哪个候选人?","label":1} {"text1":"古文化街是中国天津市南开区的一条由仿中国清代民间小式店铺组成的商业步行街,原为以天津天后宫为中心的“宫南大街”和“宫北大街”,是天津最早的文化、宗教和商贸聚集之地。目前,古文化街北起通北路,南至水阁大街,全长687米,宽5米。2011年,天津市规划局将这里纳入历史文化街区规划,定名古文化街历史文化街区。在漕运时代,天津老城城东门外、海河三岔河口西岸的宫南大街和宫北大街是繁盛的银号集中区,也是当时祭祀海神和船工聚会的娱乐之场所。在1900年八国联军入侵和1912年壬子兵变期间,宫南大街和宫北大街两度遭受洗劫,商业迁往天津租界,宫南大街和宫北大街从此衰落并逐步演变为普通民宅。1986年元旦,天津市人民政府对宫南大街和宫北大街进行改造翻新,新建百余栋仿清民间小楼,成为今天“津门十景”之一的古文化街。此后,这里逐渐成为天津比较大型的集市贸易和年货市场。其中,著名的天津皇会(娘娘诞辰吉日)也于每年春节期间在古文化街举行,成为天津春节习俗的一道风景。目前,天津古文化街街道两边设有百余家店铺,主要经营包括:古旧书籍、民俗用品、民族乐器、传统手工艺品等古董和文化用品。其中,较为著名的有杨柳青年画、泥人张、风筝魏、四宝堂、春在堂、萃文斋和刻砖刘等专卖店铺。2007年5月8日,天津古文化街旅游区经国家旅游局正式批准为国家5A级旅游景区。2002年年底,作为天津市海河综合开发改造工程的六大节点之一,天津市房产总公司投资人民币20亿元在原古文化街基础上又新建一个古文化街海河楼商贸区,新的古文化街海河楼商贸区东起张自忠路,西至东马路,南起水阁大街,北至通北路,总建筑面积为22万建筑平方米。此外,该商贸区还包括亲水平台和水阁客栈等综合商业项目。另外,古文化街的东北侧新建有文化小城、古文化街的西侧新建有古玩城、美博城等商业设施。古文化街目前全长580米,宽5米。沿途建有天津天后宫、通庆里、玉皇阁等建筑。目前,古文化街地区为天津市重要的商业街之一等。古文化街现在比较著名的建筑有:古文化街周边比较著名的建筑有:目前,与古文化街相交汇的主要道路有:","text2":"古文化街是哪里的商业步行街?","label":1} {"text1":"雾雨魔理沙()是日本同人游戏社团上海爱莉丝幻乐团所制作的弹幕射击游戏(STG)《东方Project》中的主角。喜欢搜集东西,努力追赶不用修行也很强的博丽灵梦。常与灵梦进行妖怪退治的竞争,虽然在退治妖怪方面十分勇敢,但是经常会惹麻烦,与博丽灵梦多数时间是友好的,常常与灵梦一起解决异变。居住于魔法森林的人类魔法使,常骑着扫把,「雾雨店」的独生女。会使用魔法森林的蘑菇调制火药,所使用的魔法几乎只有把东西破坏的效果,而且燃料需要长时间准备,但是其效果非常强力,在Spell Card规则成立下,纯粹的破坏力在人类当中属于最高等级。此外在幻想乡是出了名的小偷,帕秋莉的图书馆、香霖堂,如果不是古明地觉会读心术,甚至连地灵殿都会惨遭毒手。具有波浪式的金色长发,在头发的左边会绑辫子,与博丽灵梦年龄相似。经常佩戴黑色的帽子,穿着黑色的衣服和白色围裙,因衣服颜色而被昵称为“黑白”。战斗技巧以弹幕为主,其主要是魔炮和星星,名称大多跟星象有关。平时携带一把扫帚并在飞行时骑乘,但其实作为魔法使的她不用借助扫帚也能够飞行,只是由于她个人认为“不带扫帚就不像个魔法使”。自《封魔录》后,经常去博丽神社找灵梦。起初以“魔梨沙”在《封魔录》作为魅魔的支援出现,称魅魔为“(魅魔大人)”,但之后就以“魔理沙”出现在以后版本和官方设置文件中。所以“魔梨沙”应该为“魔理沙”的误字。即使在PC-98平台也有较大的差异,《封魔录》为红色头发,紫色魔女服装,《梦时空》则改为金色头发,《怪绮谈》则改为黑色魔女服,在Windows平台,其服装更像女仆装。使用魔法的能力。","text2":"雾雨魔理沙是哪个游戏的主角?","label":1} {"text1":"古大气层是在过去的地质时期的大气层,特别是地球大气层,在非特别指定的时刻。地球古大气层的构成储存在地质里,可以从现今对具有代理性的物质,例如氧化铁的丰度、木碳和树叶化石上气孔的密度,等的研究上推断出来。虽然今天大气层的成分是大约78%的氮和21%的氧,这在有生物前的大气被认为是高度缺乏的,实际上没有自由氧和氩,是由钾的同位素经由放射性衰变产生的,并且是以氮气、二氧化碳和甲烷为主。看得出来自由氧不是原本就存在的,是由细菌,像是蓝细菌或者蓝绿藻在距今大约20亿年前演变至能进行光合作用之后产生的。距今大约5亿年,氧浓度的增加使生物能发展出多细胞的动物性生物。从此之后,植物出现在土地上并且迅速的演变和放射状的散布至许多表面的土地上,大约在4亿5千万年前,在早石炭纪期间,氧的浓度超过了目前的数值,而大气中的数量降得比目前的值还要低。","text2":"氧的浓度超过了目前的数值是在什么时候?","label":1} {"text1":"Super A'can是台湾敦煌科技推出的家用电子游乐器,于1995年10月25日开始贩售,英文代号F-16,中文名敦煌。制造商为联华电子。Super A'Can主机和摇捍模仿了美版超级任天堂。机器核心是Sega Genesis/Mega Drive或SNK Neo Geo使用的Motorola 68000。因此A'can有世嘉MD的处理速度,和任天堂超任的色彩艳丽,堪称是16 BIT主机里最强。游戏采用全中文介面。1995年7月发布,定位对应超任等机型。对比当时的超任是旗鼓相当的机型,但由于市场操作失误及新一代主机的到来而走向失败。最后,A'can在亏损了超过六百万的美金下结束了它短暂的生命,联华电子则全面销毁所有相关开发和制作零件,而剩下的主机被当成垃圾卖给了美国。A'can是2000年以前联华电子旗下损失最严重的产品,据报道,联华为此损失6百万美元。Super A'Can的失败原因有几点。首先其本身机能虽可以媲美超级任天堂,但其上市已经是1995年,这时超级任天堂的机能已经落后。相较Sony的Playstation等大量应用3D技术的游戏机,Super A'Can就更加落伍了。游戏机本身上市价也太高,定位不明确。对于生产商方面,由于上游的晶片订单和任天堂的订单相抵触,受到任天堂方面的撤单压力。对开发者,只能使用次等开放软件开发,制作过程也过于紧凑。此外台湾以外的地区也没有为之配合的第三方软件开发商。在软件数量不足,市场惨淡的情况下,Super A'Can也就只能淡出市场。","text2":"Super A'can是什么?","label":1} {"text1":"卡路士·艾坚哈顿·艾华斯(,)是一名千里达足球员,担任右翼,现效力英冠球队米禾尔。卡路士·艾华斯出道自千里达球会Defence Force。他在2000年以25万镑加盟-{zh-hans:雷克瑟姆; zh-hk:域斯咸;}-。艾华斯在域斯咸度过五年,上阵165次。在2005年,他免费加盟英冠球会卢顿。他成了球迷的宠儿。在2007年1月2日日,艾华斯以150万镑加盟新特兰,签约三年半。他在新特兰的表现抢镜,接连在对锡菲联、高云地利及伯明翰城取得入球。在对修咸顿的比赛更射入致胜一球。在对般尼的比赛入球,更把新特兰带返英超。2008\/09年球季由于马白兰基加盟影响艾华斯的出场机会,于10月2日被外借到狼队3个月。11月20日仅为狼队出赛6场后被提早召回。2009年9月1日艾华斯转投英冠球会叶士域治,转会费135万英镑。2012年3月30日艾华斯与叶士域治续约留效多一季。卡路士·艾华斯曾代表千里达出战2006年德国世界杯。在对瑞典,他的表现令他成为全场最佳球员。2011年10月10日艾华斯在法庭承认于停牌期间驾驶,事发于9月26日,前一天他的妻子因怀孕并发症而入院,艾华斯送女儿上学,但未能截获的士而决定亲自驾驶。其妻于10月8日早产诞下只有两磅重的双胞胎。10月13日艾华斯被判入狱12星期,但缓刑12个月,同时罚款2,500英镑和需要进行200小时社会服务,他亦被所属球会处罚两星期薪金。","text2":"艾华斯因为什么原因导致被判入狱12星期,缓刑12个月,同时罚款2,500英镑和需要进行200小时社会服务?","label":1} {"text1":"《LOVE YOU 我的勇者公主》()是七月隆文撰写,三毛央()负责插画的轻小说作品。集英社Super Dash文库出版发行。中文版由青文出版社代理发行。电子角色扮演游戏「」,销售达到500万套,号称国民角色扮演游戏。著迷于「DRAGON BLESS」系列的主角神田俊,一边练等,一边幻想要是女主角罗莎莉能来到现实世界该有多好。就在这一瞬间,电视画面突然发出闪光,罗莎莉真的出现在现实世界。同时,全日本的「DRAGON BLESS III」里头的女主角全部都消失无踪,陷入无法继续游戏的怪异现象。来到现实世界、一时之间搞不清楚状况的罗莎莉,引发了袭击日本国会等各种骚动。另一方面,学生会长白金碧空告知主角「在这样下去游戏世界和现实世界两边都要毁灭了」……","text2":"《LOVE YOU 我的勇者公主》的女主角是谁?","label":1} {"text1":"是Xbox 360游戏机上的电子游戏,由Team Ninja开发。这是死或生系列《死或生 沙滩排球》的续作。同它的前作一样,《死或生 沙滩排球2》侧重于描绘海滩上的比基尼美女。由于增加了可游玩小游戏的数量而不再只有沙滩排球,所以将标题的“Beach Volleyball”去掉了。《死或生:沙滩排球2》采用了两个星期的休假模式,玩家可以尽情享受各种活动(如下所列)。每一天都分成三个部分(上午,下午和晚上),玩家每次可以执行其中一个活动。这个游戏十分强调友谊,玩家必须与其他女孩成为朋友,可以在岛上的商店给她们买她们喜欢的礼物,这样做将增加两角色之间的关系。如果两人之间的友谊在一个足够高的水平,对方可以被说服成为玩家的排球搭档。游戏以沙滩排球模式为主,另有多个小游戏包括水上摩托艇、踩浮板、沙滩抢旗、顶屁股、水上拔河、滑水、赌场等。游戏使用了《死或生4》所用引擎的增强版,新引擎支持自阴影和布料模拟等技术,而迷你游戏“水上摩托艇”使用了全新的物理引擎,虽然画面效果与其他小游戏类似。游戏中对乳摇系统进行了改进,以抑制乳房的异常摇晃,还增加了一个新的“棕黄色线系统”以防止泳装上出现棕黄色的分割线。《死或生:沙滩排球2》的游戏模式并不单一,其中包括若干个不同的子游戏。游戏最初设计了42个小游戏,最后只保留了7个(不包括沙滩排球)。在开发过程中,设计师板垣伴信表示他不会加入类似于蹦床比赛和湿T恤比赛的小游戏。他认为这是低级趣味并且“显示出妇女的负面形象”。在《死或生:沙滩排球2》中,玩家可以任意选择目前该系列中的九个女孩。游戏中每个女孩都有特定的物品和喜好,这些偏好会影响到她收到礼物时的反应。每个角色也都有各自最喜欢的颜色,这又会影响到他们对特定物品(和包装纸)的好恶。可供选择的角色有:需要注意的是,心并没有出现在《死或生:沙滩排球》中,并自此成为了可选的新增角色。
","text2":"为什么将标题的“Beach Volleyball”去掉?","label":1} {"text1":"益海嘉里投资有限公司(益海嘉里集团),是新加坡丰益国际有限公司在华投资的以粮油加工、仓储物流、内外贸易为主,集煤炭经营、清洁能源开发、房地产于一体的集团公司,也是中国国内最大的粮油加工集团之一。公司总部设在上海市浦东新区陆家嘴,属于新加坡独资企业。20世纪80年代,嘉里粮油在深圳蛇口设厂,并生产中国第一个小包装食用油品牌“金龙鱼”。2006年底,丰益国际以27亿美元收购了新加坡郭氏集团旗下的嘉里粮油,并将嘉里粮油与其在中国的子公司益海集团合并,成为益海嘉里投资有限公司。截止2008年9月,益海嘉里在中国的食用油粗炼和精炼厂共24家,特殊油脂生产厂5家,灌装油生产厂20家,大米厂2家,面粉厂5家,。丰益在内地共有 1500位销售人员、 4000个分销点及 380个货仓,并于全国 35个地点设有 130个生产设施;主要进口原料是黄豆及棕榈油。丰益正逐步拓展包装面粉、米及矿泉水业务;08年收入1,154亿港元,盈利46亿港元,分别增64%及68%,盈利率4%;09年上半年收入及盈利463亿及29.6亿元,盈利率6.4%。2009年7月22日,益海嘉里收购了西藏冰川矿泉水有限公司25%的股权。其后,益海嘉里退出西藏冰川矿泉水有限公司。","text2":"丰益在2008年的收入是多少?","label":1} {"text1":"迦拿的婚礼,天主教汉译作加纳婚宴,是《约翰福音》所记载而三卷《对观福音》都没有记载的一个事件。约翰记载说耶稣和他的门徒一同参加在迦拿举行的一场犹太婚礼。当主人的酒用尽时,耶稣的母亲马利亚(在约翰福音中没有提到名字)告诉耶稣说,“他们没有酒了。”耶稣回答说,“妇人,我与你有什么相干?我的时候还没有到。”耶稣的母亲对仆人说,“他告诉你们什么,你们就做什么。”()。耶稣命令仆人将空石缸都盛满水。他们照此办理,于是耶稣又告诉他们舀出一些水来,送给首席侍应。侍应品尝时,水已经变成了酒,他并不知道耶稣所做的。然后他对新郎说,新郎违背了宴席上先上好酒的风俗,把好酒留到最后才上()。这是耶稣所行的第一个神迹,用以展示他的荣耀,并且他的门徒就信入了他()。约翰福音提到耶稣的这个神迹,是紧接在耶稣告诉拿但业 “你将要看见比这更大的事”之后立即发生的。在约翰福音中,约翰使用古希腊词汇“semeion”(意为“记号”),或“ergon”(意为“工作”)来表示耶稣的神迹,而在对观福音中通常使用“dynamis”- 意为“能力的行动”来表示“神迹”一词 。这个事件是约翰福音用以证明耶稣神圣身位的7个神奇的“记号”中的第一个。约翰福音的这个事例可以被认为是旧约圣经中的预言的应验,例如和中预言,在弥赛亚的时代,酒将会特别丰富,而在也提到了弥赛亚的婚礼喜宴。这一事件对于基督教教牧神学的发展也相当重要,由于耶稣受邀请参加婚礼,并使用他神圣的能力将婚礼从灾难中挽救出来,被用作他赞成婚姻和世俗庆典的证据,与保罗书信中更为严厉的观点形成对比(例如,在哥林多前书7章)。这一事件也被用作反对基督教禁酒主义的一个论据。在罗马天主教中,迦拿的婚礼是玫瑰经的“光明五端”之一。","text2":"约翰记载说耶稣和他的门徒一同参加了在哪里举行的一场犹太婚礼?","label":1} {"text1":"北岳中学位于山西省大同市城区东部的铁牛里,是大同市重点中学 ,直属大同市教育局管辖 。学校创建于1986年9月10日,总占地面积50.5亩,建筑面积20000余平方米。北岳中学获得了市级文明单位,市先进基层党组织,市十佳青年示范群体创建单位,省级绿色学校等荣誉。根据大同市现行城市规划,北岳中学是御河西侧保留并扩建的中学之一,扩建工程已经在2010年5月30日开工。 根据计划,大同市将征收原校区东北方向的土地,扩大校区面积,将小学部搬迁到中学北部,并且将城区八小、城区三十小并入 。扩建后增加高中部。计划中,扩建后的校区占地135亩(含小学校区15亩,中学校区120亩);学校在校生总人数将由4000余人增至近7000人。扩建之后,小学部由4轨22个教学班增至6轨36个教学班,容纳在校生1800人;初中从12轨36个教学班增至18轨54个教学班,容纳在校生2970人;高中部设12轨36个教学班,容纳在校生1980人。新校区内计划建设教学楼、实验楼、科技图书楼、体育馆、行政楼、公寓食堂等。","text2":"扩建后的学校增设了哪个学段?","label":1} {"text1":"龙冈小学是江苏省盐城市盐都区龙冈镇的一所全日制小学,原名“龙冈市立第一国民学校”。龙冈小学创办于1904年(清朝光绪29年正月),1912年改名为市立第一高小学校,1915年(民国三年一月)又改名为第二国民高校。1928年改名为龙冈镇小学。1940年以前,在凤凰桥南首的放生庵、弥陀寺一度借用作为校舍。1941年下半年,日寇逼近龙冈,为适应形势,学校分散迁至农村坚持教学,校名为“龙冈镇小学裔家舍分校”及三里桥分校、祁家舍分校、大李庄分校等四所,校舍借用庙宇、祠堂或民房,课桌均是学生自备的板条搭搁。1945年9月,日本侵略军撤出龙冈,龙冈小学重新恢复设两初、三高5个班,学生近150人,校址在凤凰桥北首东侧的同仁堂。1946年,学校停办。1947年秋解放军攻占盐城县,龙冈小学重新建立,由镇长徐金堂为首组织起校董会,校址设在油坊巷西首,学生又干部子女带头入学。1950年,班级逐渐增多,又发展了西校。","text2":"龙冈小学改名为市立第一高小学校是在哪年?","label":1} {"text1":"映秀镇是中国四川省阿坝藏族羌族自治州汶川县的一个镇。映秀镇地处阿坝州南大门,是进出九寨沟、卧龙、四姑娘山的必经之地。映秀镇处于国道213线和省道303线交汇处,都汶高速公路直达映秀镇。映秀镇面积115.12平方公里。2004年末,映秀镇有农业人口4181人,8个行政村29个村民小组,因紫水工程移民,2005年有农业人口3387人,7个村,25个村民小组,7个行政村分别为:中滩堡村、老街村、枫香树村、黄家村、张家坪村、渔子溪村、黄家院村。2007年,映秀镇有耕地面积1800亩,已退耕地4950亩,常住人口6906人,其中农业人口4193人,素有“水电之乡”之称。映秀镇境内物种丰富,粮食作物主要以玉米、洋芋、红苕为主,经济作物以油菜籽以及各种蔬菜为主。2002年全镇人均收入1885元,粮食总产量658567公斤,农民人平有粮128公斤,乡镇企业总产值2027万元。","text2":"映秀镇处于哪两条线路的交汇处?","label":1} {"text1":"在数学中,三次互反律是关于模代数中两个对应的三次方程的可解性之间的关系的结论和定理。三次互反律最常使用艾森斯坦整数进行表述。艾森斯坦整数是指由形如 formula_1 的复数组成的环,记作 formula_2。其中 formula_3 和 formula_4 是整数,formula_5 为三次单位根:如果 formula_7 是formula_2中范数为 formula_9 的一个 素数。formula_10 与 formula_7 互素。定义三次剩余符号formula_12 为一个三次单位根,并满足再定义“原初”素数是模formula_14同余于formula_15的素数。由于每个素数在乘以formula_2中的一个单位元后都会成为“原初”素数,因此关于“原初”素数的定律仍具有普遍性。这时,三次互反律说明,对两个不同的“原初”素数 formula_7 和 formula_18,有此外有辅助定理:如果 formula_20 那么:由于因此可以计算任意艾森斯坦整数的三次剩余。","text2":"三次互反律在数学中指什么?","label":1} {"text1":"2008年888.com 斯诺克世界锦标赛是职业斯诺克积分排名赛,比赛时间为2008年4月19日至5月5日,地点为克鲁西布剧院。罗尼·奥沙利文在决赛中18比8战胜阿利斯特·卡特第3次夺冠。冠军:£250,000亚军:£125,0004强:£52,0008强:£22,00016强:£14,00032强:£10,60048强:£740064强:£4500预赛阶段单杆最高:£1000决赛阶段单杆最高:£10,000预赛阶段满分杆:£5000决赛阶段满分杆:£147,000共计£1,050,000资格赛于1月3日至5日进行。预赛于1月6日至11日以及3月7日至10日在世界斯诺克学院进行。","text2":"谁在2008年斯诺克世界锦标赛中夺冠?","label":1} {"text1":"《阴道独白》()是美国女作家伊芙·恩斯勒()所著的戏剧,1996年由作者本人首演于纽约外百老汇,获1997年奥比奖最佳剧本奖。该剧至今至少已被翻译成50种语言,在140个国家上演过。该剧至少已有5个不同的中文译本。《阴道独白》2004年曾在北京和上海被双双禁演;但几年来,该剧已在中国大陆的至少十所高校被未经授权的学生搬上过舞台。中国大陆的授权中文首演是2009年3月薪传实验剧团在北京9个剧场的5场公演,由新浪潮戏剧人王翀翻译并导演,至今已在北京、上海、深圳、长沙、杭州、天津演出30余场,全部提前售罄,引起凤凰卫视、《北京青年报》、《青年周末》、《时代周刊》、《侨报》、《南方都市报》、《南方周末》等中美媒体的关注。美国《时代周刊》将中国大陆的首演评价为“一系列的售罄演出,造成轰动。”磨铁图书有限公司曾经签下王翀译本的版权,但因为无法通过图书审查,至今未能出版。V日义演是《阴道独白》的作者恩斯勒发起的反对对妇女暴力的活动。这个全球性的活动于2010年已经进入了第12个年头,已经筹资超过7000万美元。《阴道独白》V日义演规则:2013年11月7日,北京外国语大学17名女生为了宣传《阴道独白》在校园演出,在人人网上发布一组持牌照片,牌子上写着“我的阴道说:我要自由”等以“我的阴道说”开头的文字,引起中国社交网站上许多负面评价。","text2":"《阴道独白》的作者是谁?","label":1} {"text1":"政府间海洋学委员会按联合国教科文组织大会第2.31条而成立。第一次会议在1961年10月27日于巴黎的联合国教育科学文化组织总部举行。共有40个国家成为委员会的创始成员国。至2004年委员会一共有129个成员国,设有会员大会,执行理事会和秘书处。秘书处设在法国巴黎。此外,委员会亦设有一些附属机构。国际太平洋地区海啸预警系统协调小组。40个成立会员国分别是:阿根廷,澳大利亚,比利时,巴西,加拿大,智利,中国,古巴,丹麦,多米尼加共和国,厄瓜多尔,芬兰,德意志联邦共和国,法国,加纳,印度,以色列,意大利,象牙海岸,日本,韩国,墨西哥,毛里塔尼亚,摩纳哥,摩洛哥,荷兰,挪威,巴基斯坦,波兰,罗马尼亚,西班牙,瑞士,泰国,突尼斯,苏维埃社会主义共和国联盟,阿拉伯联合共和国,联合王国,美利坚合众国,乌拉圭,越南。","text2":"委员会设有哪些部门?","label":1} {"text1":"李菁(1948年11月8日─),原名李国瑛,1960、70年代香港国语片演员。李菁一家于1949年中国大陆政权更替、国民政府撤退台湾后,随父母移居自香港。家中有五个哥哥、两个姊姊,李菁排行最小。她曾就读岭英中学。她自幼就对电影即有兴趣,就读宝血女子中学时,听闻邵氏所属的「南国实验剧团」公开招收第二期学员,不顾父母反对坚决投考。当年有超过两千人报名应试,李菁凭获清新的外表和表演天份幸运录取,与方盈、江青、郑佩佩、秦萍等40人成为同期同学。期间,曾客串演出《梁山伯与祝英台》(1963)与《玉堂春》(1964)。训练班结业后,与邵氏订下八年合约,初期被定位为古装型演员,她接连在《血手印》(1964)、《宝莲灯》(1965)、《文素臣》(1966)任配角,《鱼美人》是其担纲主演的第一部电影。1965年5月李菁以此片获得亚洲影展的影后宝座。此后「东家」邵氏有计划性的展开力捧,一连为她开拍新片,声势一路窜升,作品票房屡创新高。除了1969年时摔裂左腿骨休养半年,几乎一刻不停的拍片,陆续完成黄梅调电影。进入70年代,李菁已是『资深』的年轻巨星,不仅连续多年入选国语片十大明星,更是港台炙手可热的女星。1976年年底,李菁结束了与邵氏十三年的宾主关系,开始以自由演员的身分往来港台两地。1978年,李菁与导演罗马合组《长天公司》,在创业作『追』中担任主角。1979年相恋十年的男友雷觉华过世,使她大受打击,逐渐减少拍片量。1983年,长期陪伴在旁的母亲病故,遂宣布退出娱乐圈。据香港《壹周刊》描述,离开影坛后,她曾在股海开启事业第二春,未几却传出她投资失利的消息,最后只好脱产还债。90年代,李菁更销声匿迹,即使「邵氏」发行电影数位修复电影,也未见她现身。2012年3月该刊报道她拖欠跑马地名仕花园房租两年半却拒绝搬走,与房东数度对簿公堂。蛇形醉步(1980年)","text2":"李菁的原名叫做什么?","label":1} {"text1":"卦限是笛卡儿坐标系中,象限在三维空间的对应术语,用于空间解析几何的坐标系统。空间直角坐标系用于确定空间的任意一点的位置。先在指定空间内的任意一点取定并标记点 \"O\",作为坐标原点。经过点 \"O\",画出三条互相垂直的直线,把它们分别标记作 \"x\" 轴、\"y\" 轴和\"z\" 轴。用右手定则规定各轴线的正方向。每二条轴线确定出一个平面,作为坐标平面。由 \"x\" 轴和 \"y\" 轴确定的坐标平面称作 \"xy\" 平面;\"x\" 轴、\"z\" 轴确定 \"xz\" 平面;最后一对,\"y\"、\"z\" 二轴确定 \"yz\" 平面。按照传统,将 \"xy\" 平面配置在水平面上,\"z\" 轴置于铅直位置,而 \"xz\"、\"yz\" 二平面在图上垂直标示。这三个坐标平面将空间分为八个部分,这便是空间直角坐标系的8个卦限。八个卦限在几何图中通常以罗马数字“I、II、III、IV、V、VI、VII、VIII”标示。较为普遍的卦限数序均以 \"x\" 轴正半轴、\"y\" 轴正半轴和 \"z\" 轴正半轴确定的卦限为“第一卦限”,罗马数字标记为“I”。第二、三、四卦限的数序类似平面直角坐标中象限的数序。在 \"xy\" 平面上向逆时针方向增加数序。而后第五至七卦限在 \"xy\" 平面下同样以逆时针方向标记。因卦限相对象限较为罕见,世界各地的数学家乃至不同时代的数学印刷物都曾使用过不同的数序来标记各个卦限,所以为了避免混淆,可以采用另一种标记卦限的方式。直接地,明确指出某卦限范围内包含的 \"x\"、\"y\"、\"z\" 坐标的正负,来标记那个卦限。如图1中的第一卦限(I)标作“(+,+,+)”;第四卦限(IV)标作“(+,-,+)”;第七卦限(VII)标作“(-,-,-)”。","text2":"卦限是被用于哪里的?","label":1} {"text1":"重力位能或重力势能是指物体因为大质量物体的万有引力而具有的位能,其大小与其到大质量的距离有关。formula_1其中G为万有引力常数,M、m分别为两物体质量,r为两者距离。依据古典力学,在两个或更多的质量之间存在著重力位能。能量守恒要求这种重力位能永远是负值。证明令无穷远处为势能零点,则有formula_2且万有引力formula_3解得formula_4由于质点所减少的势能等于保守力对该质点所做的功,所以引力势能formula_5在广义相对论,重力位能被塑造成兰道-栗弗席兹赝张量 ,以允许古典力学的守恒定律能够获得保留。加上物质的应力-能量张量至兰道-栗弗席兹赝张量的结果是结合了物质和重力能赝张量导致散度为零的发散。有些人反对在基础上做如此的延伸,认为这样做在广义相对论中是不适当的,这是只是守恒律的需要所衍生的用途,在这样的情况下只是赝张量和真张量的结合。","text2":"为什么有些人反对在基础上做如此的延伸?","label":1} {"text1":"招重文(Chiu Chung Man,),生于香港,已退役香港足球运动员,司职中场,前香港奥运足球代表队及香港足球代表队队员,持有亚洲足协B级教练文凭​,现时担任香港超级联赛球队梦想FC技术总监。招重文生于香港,1987-88年度,年仅18岁的招重文,便随「升班马」美青初次在甲组亮相,并且表现相当不俗,在香港足球明星选举当选最佳年青球员。闯出名堂的招重文,翌年便「上山」加盟南华,并连续两届于香港足球明星选举当选最佳年青球员。惟在南华一直未能站稳正选位置,遂于1994年转投商业球队好易通,并成为球队中场主力。1997年夏天,香港足球总会仿效中超实施甲组联赛球员季前体测,招重文原本转投星岛,患有哮喘的他因未能通过体测,无法在甲组赛事上阵,被迫转投乙组联赛球队中福。1998-99年度球季,招重文重返甲组加盟二合,终凭努力成功通过体测,得以继续甲组球员生涯,并首次入选香港足球明星选举最佳11人。近年招重文仍有客串于乙丙组赛事上阵,2008-09年度球季,便协助骏升大中获得香港乙组足球联赛亚军,首次取得升班甲组资格。2015年10月15日,招重文代表「519香港传奇足球队」,在小西湾运动场举行在的「英超足球大师邀请赛」迎战「英超足球大师队」并取得一个入球最终以4–5不敌对手。招重文1989年入选新组成的第一届香港奥运队,参加过1992年巴塞隆拿奥运足球外围赛。香港足球代表队于1990年7月18日至8月10日前往欧洲集训,招重文首次入选。招重文退役后,曾担任南华青训教练。2008-09年度球季,曾执教过淦源,2011年夏天开始担任晨曦助教。2012年招重文升任教练,与陈发枝、谭兆伟共同执教。2014年开始担任中西区名校圣公会圣彼得小学足球队主教练,球队在多个赛事中表现出色。于2017年7月担任新成立的香港超级联赛球队梦想FC技术总监。","text2":"招重文转投星岛为何没有成功?","label":1} {"text1":"摩伽罗(Makara,玄奘新译为摩竭)印度神话中的海兽,恒河女神及伐楼拿的座骑。牠亦是印度教中代表爱与欲望的神祇伽摩的标志。传统上,摩伽罗被认为是海中异兽,有人认为牠的形象是源自鳄鱼,亦有人亦认为是鲸鱼、海豚、甚至是鱼身象头等的海兽。《大唐西域记》中记述一名为「摩竭」的大鱼。书中第八卷记载了漕矩咤国的一名大商人因为轻蔑佛法,一次在海上迷失方向,足足有三年之久,后来在濒死的时候看见一座大山,山势崇峻,船上的人都以为终于得救,但商人却知道那不过是「摩竭鱼」。书中描述摩竭鱼有如山一般大:「崇崖峻岭,须鬣也;两日联晖,眼光也。」而「摩竭」一词乃从梵文Makara音译而来。","text2":"《大唐西域记》中记述的大鱼叫什么?","label":1} {"text1":"《开宝本草》全称《开宝重定本草》,凡20卷,目录1卷,宋开宝六年尚药奉御刘翰、道士马志,翰林医官翟煦、张素、王从蕴、吴复圭、王光祐、陈昭遇、安自良等九人编著。宋开宝六年(973年)诏尚药奉御刘翰、道士马志等九人以《新修本草》、《蜀本草》为蓝本,“刊正别名,增益品目。”,《开宝新详定本草》20卷撰成。开宝七年(974年),宋太祖再次诏命刘翰、马志等人重新修订。卢多逊、李昉、王祐、扈蒙等又以陈藏器《本草拾遗》重校,再取《唐蜀本草》详校,称为《开宝重定本草》,共收载药物984种,新增的134种药物,许多新药如:乌药、蛤蚧、南烛、赤箭(天麻)、延胡素、没药、五灵脂、使君子、白豆蔻、山豆根等皆首见于本书。原书已佚,部分内容摘录自《证类本草》。","text2":"《开宝本草》的全称是什么?","label":1} {"text1":"计算机是由苹果公司为Mac OS X设计的一个基础计算器软件。它有三种模式:基础型、科学型和编程型。基础型包括一个数字输入板,四个分别用于加法,减法,乘法和除法的按钮以及记忆键。科学模式支持指数和三角函数,编程型则为使用者提供了更多计算机编程的操作菜单。计算机支持Reverse Polish notation,并且能够说出按下的键的名称和计算结果。计算机还包括一些基本的转换功能,转换单位分为下列几类:汇率将通过互联网更新。这个计算器软件具有悠久的历史,它可以追溯到最初的麦金塔平台,那时它还仅仅是一个简单的四函数计算器程序。尽管没有科学计算功能,并且只有第三方开发商提供升级服务。后来Mac OS 7.1.2版伴随首部PowerPC推出,苹果发布了名叫Graphing Calculator(又名NuCalc)的软件,这个软件成了Mac OS 9的一个标准组成部分。苹果现在推出了一个不同的程序叫Grapher。首次出现于1984年的Macintosh 128k,在Mac OS 9于2002年推出之前,它仅能进行基本的数学运算。在Mac OS X Tiger和以后的版本中的Dashboard Calculator widget。它只有它的桌面版的基本功能。","text2":"计算机还包括什么功能?","label":1} {"text1":"蒂芙尼公司(,)是美国一间钟表和珠宝和公司。1837年,蒂芙尼公司由和泰迪·杨在纽约市成立,开始时名称为蒂芙尼和杨(Tiffany & Young),商店设在下曼哈顿区,是一间专门销售时尚商品的精品店。当时公司由蒂芙尼、杨和艾利斯三人打理。1853年,查尔斯·蒂芙尼掌握了公司的控制权,将公司名称简化为蒂芙尼公司(Tiffany & Co),公司也从此确立了以珠宝业为经营重点。蒂芙尼逐渐在全球各大城市建立分店。蒂芙尼制订了一套自己的宝石、铂金标准,并被美国政府采纳为官方标准。时至今日,蒂芙尼已成为全球知名的精品公司之一。蒂芙尼蓝(Tiffany Blue)是蒂芙尼的标志色,在公司成立不久,蒂芙尼就采用了这种独特的颜色作为他们品质和工艺的标志。其蒂芙尼蓝色礼盒(Tiffany Blue Box)更成为美国洗练时尚独特风格的标志。","text2":"什么颜色是蒂芙尼的标志色?","label":1} {"text1":"Odyssey 是一个于1999年早期至2000年开发的Microsoft Windows的开发代号。Odyssey的目标是为以商业用户而设的NT核心的Windows 2000开发一个后续产品。Odyssey使用与Windows Neptune相同的版本, 6.0(版本编号「NT6.0」并不是产品的名称。而相同的版本编号于Windows Vista中被重新使用)而Windows Neptune亦曾经外流, 但于Windows 2000推出后Neptune 及Odyssey这两个开发团队则与Windows 2000的开发团队组合成Whistler专案, 即是于2001年推出的Windows XP。","text2":"Windows XP是什么时候推出的?","label":1} {"text1":"《老王同学会》()是2009年的一部台湾电视剧,故事讲述50岁的电台知名主持人老王和他的三位老朋友所面临的问题。此剧是中国电视公司(中视)与三立电视首次合作的电视剧,版权归三立电视持有。2009年3月6日,中视与三立电视合办《老王同学会》签约媒体茶会,中视总经理饶圣雄、三立电视总经理张荣华与该剧制作人王伟忠共同在象征物《三年中班同学录》上签名。以下时间以当地时间(台湾时间)为准依据〈中国电视公司观众意见反映处理情形公告:98年1月—3月〉,由于《老王同学会》的版权持有者为三立电视,中视已向三立电视购买该剧「无线台播映权」,故能在中视无线台播映该剧;而中华电信MOD收视户必须请求中华电信向三立电视购买该剧「网路播映权」,才可以在中视无线台收看该剧。中华电信及其频道商台湾互动电视公司始终未购买该剧播映权,故中华电信MOD收视户无法在中视无线台收看该剧。由著名的王伟忠制作,这出黑色喜剧围绕著老王的同学们,包含胆怯的老王、傻气的老林、心态年轻的老范与过世的老许。故事开始于老王某日祭拜老许时,遇到他的鬼魂……","text2":"中视无线台播映该剧为何能博播放该剧?","label":1} {"text1":"新白河车站()是一位于日本福岛县西白河郡西乡村字道南东,由东日本旅客铁道(JR东日本)与日本货物铁道(JR货物)所共用的铁路车站。新白河车站是JR东日本所属的东北新干线与在来线干线东北本线的交会车站,其实际位置位于西乡村与隔邻的白河市之交界附近(部分新干线月台则已经伸入白河市的范围内),是全日本唯一一个设址于村级地方自治体的新干线车站,但实际上却是白河市的门户站。新白河车站当初是日本国有铁道在1944年时设的一个号志站(),原名磐城西乡,1959年时升格为磐城西乡车站。但一直到1982年东北新干线通车后,才配合日本国铁的惯例,将车站以主要服务城市命名,而改为新白河。除了客运服务外,目前JR货物在新白河仅设有公路货运()服务的代收站,除此之外并无定期的货物列车在此站停靠,也没有相关的装卸设备。在1994年之前,新白河原本设有通往三菱制纸白河工厂的专用支线,主要是用在燃料用石油、化学药品与纸制品的输送,但1994年时工厂减产之后专用线也随之停用,目前纸制品主要是靠货柜或篷车(有盖货车)来运输。新干线为对向式月台2面2线的高架车站,并设有通过线2条。在来线为侧式月台1面1线与岛式月台1面2线,合计2面3线的地面车站。","text2":"1994年之前,新白河车站主要输送货物的类型是什么?","label":1} {"text1":"哈维尔·巴登 (西班牙语:Javier Bardem,),生于西班牙拉斯帕尔马斯,西班牙国宝级电影男演员,西班牙第一位奥斯卡金像奖得主,好莱坞星光大道入选者。巴登与潘妮洛普·克鲁兹是全世界唯一一对均获得过奥斯卡最佳配角奖的夫妻。巴登曾获得奥斯卡金像奖、金球奖、各雅电影奖、英国电影学院奖、影视演员协会奖等奖项肯定。以2000年电影《在夜幕降临时》的演出获得好评而打开知名度,并且成为历史上首位西班牙演员入围奥斯卡最佳男主角奖。2007年再度以《险路勿近》一片中的奇哥角色,获得奥斯卡金像奖、金球奖、英国电影学院奖、影视演员协会奖最佳男配角奖项肯定。他曾经分别以电影《在夜幕降临时》 (Before Night Falls)与《点燃生命之海》获得2000年及2004年两次威尼斯影展最佳男主角,2011年以《最后的美丽》获得坎城影展最佳男主角奖。","text2":"西班牙第一位奥斯卡金像奖得主是谁?","label":1} {"text1":"在各行政区设置市民运动中心是台北市长马英九主要政见之一,\"信义运动中心\"是台北市第12座开工市民运动中心,也是第八座完工的一座,其楼地板面积1,866.7275坪也是当中最小的。目前已经在2009年5月28日正式启用,承包经营单位是中国青年救国团。地址位在台北市信义区松勤街100号,就在信义路上HP大楼的后面(南边),信义国中后门对面的公园旁。\"信义运动中心\"一楼为阅览、棋奕区及幼儿游戏室,二楼为健身房、舞蹈教室、及社区教室,三楼为桌球、壁球及高尔夫球练习场,四楼为游泳池入口、淋浴更衣区,五楼为成人及儿童温水游泳池、水疗池、按摩池、三温暖区,六楼为多功能球场,七楼为攀岩场,顶层为三十公尺的射箭场。台北市体育处","text2":"该中心的承包经营单位是哪个?","label":1} {"text1":"利害关系人理论(Stakeholder theory)是一个组织管理和商业道德的理论,用于解决组织管理中的道德和价值问题。这一理论是1984年由在《Strategic Management: A Stakeholder Approach》一书中提出,他将利害关系人的概念与理论带入企业管理的领域内,首先他界定「利害关系人」是在一个组织中会影响组织目标或被组织影响的团体或个人,因此,他认为一位企业的管理者如果想要企业能永续的发展,那么这个企业的管理者必需制定一个能符合各种不同利害关系人的策略才行。在弗里曼之后,萨瓦奇及其同僚在1991年发展出从公司的董事会成员、雇员及社区代表等三方利害关系人的观点来提出公司的经营策略,因此从企业的观点来看,一个企业除了注重股东的权益外,必需同时关注员工、顾客、社区以及所有有与企业有关的个人或团体才行,类似以企业拥有者为核心的一个同心圆,层层向外扩散。","text2":"如果一个企业想要永续发展,应该怎么做?","label":1} {"text1":"梅嘉生(),江苏丹阳人。中国人民解放军军事将领。1938年参加新四军,历任新四军抗日义勇军挺进纵队支队长,团长,苏皖支队副司令员,南通保安旅副旅长,新四军一师三旅参谋长,四分区参谋长兼启海警卫团团长,二分区司令员等。日本投降后,梅嘉生历任苏中军区副参谋长,新四军一师参谋长,华东野战军四纵参谋长,陆军第23军副军长。参加过苏中七战七捷、涟水会战、枣庄会战、莱芜会战、孟良崮战役、鲁南战役、淮海战役、渡江战役和上海战役等。中华人民共和国成立后,梅嘉生赴越南担任军事顾问团参谋长、第一副团长,获越南二级军功勋章。1955年5月,调任海军航空兵部副司令员兼参谋长,是海军航空兵部队的主要创始人之一。1955年获海军少将军衔及二级独立自由勋章、一级解放勋章。1964年任东海舰队副司令员,参与指挥了击沉国民党海军“永昌号”炮舰、击伤“永泰号”猎潜舰的崇武以东海战。1975年任海军副司令员。","text2":"梅嘉生是哪一领域的主要创始人之一?","label":1} {"text1":"《老王同学会》()是2009年的一部台湾电视剧,故事讲述50岁的电台知名主持人老王和他的三位老朋友所面临的问题。此剧是中国电视公司(中视)与三立电视首次合作的电视剧,版权归三立电视持有。2009年3月6日,中视与三立电视合办《老王同学会》签约媒体茶会,中视总经理饶圣雄、三立电视总经理张荣华与该剧制作人王伟忠共同在象征物《三年中班同学录》上签名。以下时间以当地时间(台湾时间)为准依据〈中国电视公司观众意见反映处理情形公告:98年1月—3月〉,由于《老王同学会》的版权持有者为三立电视,中视已向三立电视购买该剧「无线台播映权」,故能在中视无线台播映该剧;而中华电信MOD收视户必须请求中华电信向三立电视购买该剧「网路播映权」,才可以在中视无线台收看该剧。中华电信及其频道商台湾互动电视公司始终未购买该剧播映权,故中华电信MOD收视户无法在中视无线台收看该剧。由著名的王伟忠制作,这出黑色喜剧围绕著老王的同学们,包含胆怯的老王、傻气的老林、心态年轻的老范与过世的老许。故事开始于老王某日祭拜老许时,遇到他的鬼魂……","text2":"《老王同学会》的版权归属于哪个公司?","label":1} {"text1":"三川口之战是北宋与西夏的一场战役,以宋败夏胜收场。北宋宝元二年(1040年)三月,西夏景宗元昊进攻宋朝,夏军从土门(今陕西安塞西北)突入,进攻金明寨(今陕西安塞南部)。金明寨位于延州(今陕西延安)北部,周围有36寨,由都监李士彬率蕃兵扼守。元昊一面率军佯攻北宋的金明寨(今陕西安塞南部),一面送信给宋朝延州(今陕西延安)知州范雍,表示愿意与宋和谈,制造假象,以麻痹范雍。范雍却信以为真,立即上书朝廷,对延州防御也松懈了。十八日晨,士兵解甲就寝,被反叛者擒获,诸寨不攻自破。同年七月,元昊派十万军包围了延州。宋朝大将刘平、石元孙奉命增援。宋军昼夜兼程,驰援延州。此役宋军处于明显劣势,郭遵建议先侦后进,刘平不听,恃勇轻敌,贸然轻进。二十二日夜,当他们到了三川口(今陕西延安西北)西10里驻营时,遭到西夏军队偷袭,遇到西夏军队重重包围。刘平下令郭遵和王信带骑兵半渡而击。李元昊军手下一名军人扬言要擒拿郭遵,郭遵「期必死,独出入行间」,挥铁杵击破其脑袋,两军皆大呼。宋军与夏军苦战,西夏军队损失十分惨重。但是因为寡不敌众,只好退守三川口附近的山坡。西夏又增援了大量军队。元昊多次写信劝降刘平,但刘平宁死不屈。郭遵三出三入敌营,杀敌数百人,其坐骑中枪,马踠仆地,郭遵被杀。刘平遣子刘宜孙求援于黄德和:“当勒兵还,并力拒贼,奈何先引去!”令其还击。黄德和却逃往甘泉(今属陕西)。最后,西夏军队猛攻宋军驻守的山坡。由于宋军人数太少,刘平、石元孙被俘。西夏军集兵延州城下,延州危在旦夕。时值大雪,夏军缺少御寒衣物,军纪始松弛。后来由于宋将许怀德偷袭元昊得手,西夏军队才被迫撤离宋朝境内,延州之围才得以缓解。战后,黄德和因临阵逃脱,被处以腰斩,枭首于延州城下。三川口之战中,虽然宋朝成功抵御西夏军队的入侵,但是损失太多,而且宋朝甘陕青宁边境的防御也处于被动地位。","text2":"三川口之战是哪两个国家的战役?","label":1} {"text1":"雷振邦(),满族,是中国著名的电影音乐作曲家、一级作曲、中国音乐家协会理事、中国电影音乐协会副会长、中国电影协会常务理事、第六届全国政协委员、中国民主同盟盟员。雷振邦出生于北京,从小就接触京戏。七八岁时,能唱京剧小段,还能用胡琴演奏。1928年,在沈阳奉天公立学校就读,后来升入南满中学。学会吹口琴,并加入学校口琴队,成为口琴队的指挥。1939年,赴日本求学,毕业于东京日本高等音乐学校作曲系。1943年,回国,先后在北平女子中学和惠中女子中学做音乐教员。抗日战争胜利后,他组织了一个业余交响乐团。1949年,到中国电影乐团工作,1955年,被调到长春电影制片厂任作曲。雷振邦善于吸收中国各少数民族的音乐元素作曲,创作了许多具有浓厚少数民族风格、脍炙人口的电影插曲,其中最著名的如为电影《刘三姐》创作的壮族风格的民歌,为电影《冰山上的来客》创作的塔吉克族风格歌曲《花儿为什么这样红》,为电影《五朵金花》创作的白族风格的歌曲《蝴蝶泉边》以及为电影《芦笙恋歌》创作的拉祜族歌曲《婚誓》等,至今仍然活跃在中国的音乐舞台上。此外尚有朝鲜族、景颇族、彝族等风格的电影歌曲。雷振邦的女儿雷蕾也是一位作曲家。","text2":"雷振邦有哪些头衔?","label":1} {"text1":"巴黎丽兹酒店(法语:)1898年开始营业,也是世界上最早的丽兹酒店。位于巴黎市中心旺多姆广场,是世界最著名和最豪华的宾馆之一。世界领先酒店组织()成员之一。该建筑于18世纪初开始建造,最早用于私人住宅,建筑物的外观是由朱尔斯·阿杜安·芒萨尔()设计,这位设计师也曾参与设计了著名的凡尔赛宫。1854年佩雷兄弟()将其改建成他们的信贷和金融机构总部。恺撒·丽兹()先生于1898年6月1日将其改造成酒店,和著名的烹饪专家奥古斯特·埃科菲()一起,让丽兹酒店成为美食与优质服务的代名词。酒店面对着著名的旺多姆广场,从客房中还可以俯瞰位于酒店后方的花园。酒店成为众多社会名流,富商聚集的场所。巴黎丽兹酒店的普通客房房价从730欧元至开始。套房的房价从3,000欧元开始,最高的房价甚至到达12,000欧元。酒店的餐厅\"L'Espadon\"被收录在2007年的米其林指南中。1997年8月31日戴安娜王妃乘坐巴黎丽兹酒店保安部副经理亨利·保㑩(Henri Paul)驾驶的梅赛德斯-奔驰轿车离开酒店,遭到狗仔队尾随,在经过阿尔玛桥附近的隧道时,所乘坐的汽车发生严重交通事故,王妃被送到医院后不久身亡。可可·香奈尔女士曾在巴黎丽兹酒店居住超过30年的时间,她把巴黎丽兹酒店作为他的家。1971年1月10日,她在巴黎丽兹酒店的私人套房中被发现死于心脏病。欧内斯特·海明威经常出现在酒店的一间酒吧,这间酒吧现在的名字叫海明威酒吧。这间酒吧还被财富杂志评为世界最好的酒吧。","text2":"巴黎丽兹酒店营业于哪一年?","label":1} {"text1":"美国马萨诸塞州一共有14个县,其中的8个县没有自己的政府,只有5个县拥有自己的独立政府,分别是巴恩斯特布尔县、布里斯托尔县、杜克斯县、诺福克县和普利茅斯县。而楠塔基特县则拥有一个县级和市级的混合政府。尽管如此,司法和执法区仍然以县之间的边界划分,且这些县在地理上仍被视为一个地理实体。在法律方面,不同县份的刑事诉讼案件是由不同区域律师负责。在埃塞克斯县,其刑事诉讼案件是由东区区域律师负责;而在米德尔塞克斯县,案件则是由北区区域律师负责。中区区域律师负责伍斯特县的案件;海角和海岛区区域律师负责杜克斯县、巴恩斯特布尔县和楠塔基特县的案件;而西北区区域律师则负责富兰克林县和汉普希尔县的案件。伯克希尔县、布里斯托尔县、汉普登县、诺福克县、普利茅斯县和萨福克县则拥有自己的区域律师。有11个马萨诸塞州县份已不复存在,而其中的大部份是因其领土被新罕布什尔州或缅因州占领而消失。马萨诸塞州现存最具历史的县份为埃塞克斯县、米德尔塞克斯县和萨福克县,它们是于1643年自马萨诸塞殖民地的诺福克县(马萨诸塞殖民地的诺福克县跟马萨诸塞州的诺福克县没有关系)独立。诺福克县在后来被新罕布什尔州占领。当这三个县成立时,它们仍然属于马萨诸塞殖民地。相反,最迟成立的县份是于1812年成立的汉普登县 。大部分马萨诸塞州的县份取名自英国地名,全因马萨诸塞州在早期是大英帝国的殖民地。郡镇(shire town)或县治(county seat)是指拥有县法院和行政办公室的城镇。一个县可以拥有多个郡镇。美国马萨诸塞州共有14个县。","text2":"马萨诸塞州现存最具历史的县有哪些?","label":1} {"text1":"但泽自由市()是1920年至1939年间的一个半自主城邦,包括波罗的海德意志港口但泽(今波兰格但斯克)及邻近地区的近200个城镇。第一次世界大战结束之后,根据1919年《凡尔赛条约》第100条(第三部第十一节)条款,该政权于1920年11月15日建立起来。但泽自由市包括但泽市及其附近的城镇、村庄和其他居民点,主要为德意志人居住,历史上属于德国。根据条约,这一地区将同一战后的德国(魏玛共和国)及新建立的波兰第二共和国(战间波兰)保持分立,但并非一个独立国家。自由市处于国际联盟的保护之下,与波兰为关税同盟关系。波兰拥有在但泽发展和维护交通、通讯和港务的全部权利。自由市的建立是为了使波兰能够获得一个足够规模的海港,市内大多数居民为德意志人,但亦有显著的波兰裔人口。德意志人口对于被迫同德国分隔感到相当不满,而在纳粹党当政的1935年至1936年尤其如此。由于波兰仍未完全控制这一港口(尤其是军事设施),1921年另一新港在格丁尼亚开始建设。1933年,自由市政府权力为当地纳粹党所取得。部份犹太人逃离了但泽。1939年德国收复但泽后,纳粹废除了自由市并将其并入新建的。他们将市内的波兰人和犹太人归为次等人,使其参与劳动或将其送往集中营。其中许多人被送往纳粹集中营,包括附近的(位于今波兰什图托沃)。1945年初苏联红军攻占但泽,大量居民逃亡或被杀。战后许多幸存的德意志人被驱逐,波兰人开始迁入。根据《波茨坦协定》,该市并入为波兰的一部分。","text2":"该市并入为波兰的一部分,是根据哪个条例来的?","label":1} {"text1":"世嘉(SEGA)NAOMI基板全称为New Arcade Operation Machine Idea,是继世嘉本公司研发的model3基本之后推出的街机基板。由于该基板采用与世嘉本公司研发的Dreamcast游戏机几乎相同的硬件结构,因此Dreamcast与Naomi游戏的跨平台移植十分容易,基板对3D图形的解析性能亦非常强大,另外,由于使用的储存介质为装取便捷的GD-ROM,可读取游戏数据容量增大的同时,进一步降低了制造成本。但坊间亦传出对使用GD-ROM做为储存介质基板稳定性的质疑声音。随后世嘉又推出了升级版基板NAOMI2,具有更强的3D性能,但基本结构与NAOMI相似。NAOMI基板上的名作有CVS,VR战士4,疯狂出租车等。","text2":"世嘉(SEGA)NAOMI基板的全称是什么?","label":1} {"text1":"三甲基氯硅烷(分子式:(CH)SiCl),也称氯化三甲基硅烷、氯代三甲基硅烷、三甲基一氯硅烷,室温下为无色液体,是卤代硅烷的一种,在有机合成中有很多用途。 它在无水情况下稳定,但遇水即分解,生成六甲基二硅氧烷和盐酸。由氯甲烷与硅粉在氯化亚铜存在下反应并精馏提纯获得,也可由亲核性甲基试剂如甲基锂与四氯化硅发生亲核取代反应制得。三甲基氯硅烷在有机化学中有很广泛的用途,既可作为三甲基硅基的来源,也可在无水情况下提供氯离子。醇、胺、羧酸和金属炔化物与三甲基氯硅烷迅速反应,分别生成三甲基硅醚、三甲基硅基胺类、三甲硅基酯和三甲硅基炔烃,可用于保护相应的官能团。用三甲基氯硅烷保护羟基的反应一般在三级胺中进行。保护后的化合物可通过与氟离子作用而脱去保护基。三甲硅基对酸是不稳定的,并且三甲硅基的引入会使化合物的挥发性增强,使得某些生化样品更易被质谱和气相色谱分析。三甲基氯硅烷与醇的反应会生成一分子的氯化氢,形成无水氯化氢的醇溶液。此溶液可以与酮和羧酸反应,用于温和合成相应的缩酮和酯。三乙胺和二异丙基氨基锂存在下,三甲基氯硅烷与可以烯醇化的醛、酮和酯反应,将其转化为三甲硅基烯醇醚。此类硅烯醇醚是很重要的烯醇负离子等价物,可以发生向山羟醛反应、麦克尔加成、Rubottom氧化反应(用mCPBA氧化为α-羟基酮)、烷基化等多种反应。用三甲基氯硅烷作原料,可以合成其他三甲基硅基化合物,包括:三甲基氟硅烷、三甲基溴硅烷、三甲基碘硅烷、三甲基氰硅烷、三甲基叠氮硅烷、三氟甲磺酸三甲硅酯(TMSOTf)等。其合成反应的通式如下:三甲基氯硅烷的其他反应:","text2":"三甲基氯硅烷与醇反应会产生什么?","label":1} {"text1":"川酒,指产于中国四川地区的白酒,因四川产酒量大和知名品牌众多而闻名国内外。在中国消费市场上形成了“川酒云烟”的说法。2007年四川白酒以86万吨的产量超过山东省在中国位居第一,此前山东已连续十五年是中国最大的产酒省份。四川也是全国白酒销售收入、利润和出口量最大的省份。同时,多年来全国多家大酒厂一直在四川采购原酒,形成了邛崃、崇州、大邑、绵竹、宜宾、泸州等生产原酒的中小企业群。虽然四川人口众多且居民喜饮酒,但是四川并不是全国白酒消费最大的省份,全国白酒消费最大的省份是广东、山东、江苏,这也是全国经济总量最大的省份。一般认为全国以饮酒著名的地区是东北、山东、西北、河南及河北。四川有获得历届中国名酒评比名酒称号的白酒六种,号称“六朵金花”,分别是五粮液、泸州老窖、剑南春、沱牌曲酒、全兴大曲、郎酒。其中郎酒属酱香型白酒,其余五种属于浓香型白酒。四川的名酒企业有四家国内上市公司,分别是五粮液(SHE:000858),泸州老窖(SHE:000568),沱牌曲酒(SHA:600702),水井坊(SHA:600779)。为丰富产品线,占领消费市场,四川各大名酒公司都开发有多个子品牌系列酒分别针对高端或低端市场,特别以五粮液和剑南春最为成功。中国酿酒历史久远,其中一些古代生产酒、储藏酒的酒坊、酒池、酒窖到现在已成为了文物。四川泸州市泸州大曲老窖池和绵竹市剑南春酒坊遗址均被列为全国重点文物保护单位。众多的川酒荣获中国商务部中华老字号称号。","text2":"川酒生产于什么地区?","label":1} {"text1":"李清楠(),台湾省彰化县人,柔道八段,合气道八段。他将合气道引进台湾并加以推广,其门下称他为大老师。他也是合气太乙拳的创始者。曾任中华民国合气道协会理事长、财团法人中华合气道总部道馆董事长、亚洲合气道总会会长、国际合气道联盟总会副会长、中华民国合气道协会永久会长。李清楠台湾省彰化县人,在中学时代开始学习柔道,毕业于台湾大学法学院。1959年,代表中华民国参加第三届亚洲运动会柔道比赛获得金牌。在日本东京参加比赛时,接触到合气道,对于它的护身法深为吸引,回国后即致力于合气道的学习及推广。经过友人介绍,认识了日本大阪合气会的富田诚治老师,并邀请他来台教授合气道。经日本大阪合气会会长田中万川师范的同意,田中万川的弟子富田诚治来到台湾,与李清楠在台北巿太平国小成立台湾第一个合气道道场。李清楠号召自己从前学习柔道的学生们,如游能松、李练达、黄显谟、叶武光、曾现财等人,一同加入合气道的学习,合气道因此得以在台湾推广扎根。民国62年(1973年)在台北市成立中华民国合气道协会,成为当时台湾合气道惟一的领导团体。民国80年〈1991年〉之后,合气道协会中许多人士为了追求理想,继续推展及维护道统,遂成立中华民国合气道推广训练协进会。民国85年〈1996年〉,他总结了一生习武的心得,发表了合气太乙拳,又名合气道球转入身转换术。民国97年(2008年)1月20日上午十时过世。享寿八十八岁。著有《合气道--基础篇》《合气道--应用篇》。","text2":"李清楠邀请谁来台教授合气道?","label":1} {"text1":"漂泊信天翁(学名:\"\")是一种大型的信天翁,生活在南冰洋附近。漂泊信天翁是体型最大的一种信天翁,也是众多鸟类中,翼展最长的一种鸟,平均达3.1m,最长可达3.7m。漂泊信天翁花很少气力便可飞行数千公里,其秘密在于牠的身体结构和飞行技巧。漂泊信天翁是现存鸟类中翼展最大的,它的平均翼展可达3.1米,被证实的最大的漂泊信天翁的翼展达到了3.7米,巨大的翼展赋予了漂泊信天翁良好的滑翔能力,它们可以在空中停留几个小时而不用挥动翅膀,同时,它每下降1米高度可以滑翔22米远。它的身长大约为1.35米,雌性的通常会比雄性的略小一些,体重一般在6-12公斤,有记录显示有些雏鸟在它们第一次飞行时的体重达到了16.1公斤。一般来说,羽毛的颜色和他们的年龄有关,成年漂泊信天翁的身体都是白色的,翅膀一般是黑色和白色。雌性的翅膀会比雄性的更白一些,尖端和翅膀后缘是黑色的。他们的头部侧边也会有一个不明显的桃子状的斑点。虽然不太明显,但当与其他几种从漂泊信天翁中分离出来的新种比较(如阿岛信天翁、特岛信天翁及安岛信天翁),漂泊信天翁的羽色是最白的。其他种类的繁殖年龄的信天翁在翅膀和身体上都有更多的黑色,很像是没有成熟的漂泊信天翁。漂泊信天翁的喙很大,喙和脚都是粉红色的。漂泊信天翁以乌贼,小鱼,和船只丢弃的废物为食,有的时候会吃得太饱而飞不起来,只好很无助地在水面上休息漂泊信天翁一次繁殖只生一个蛋,蛋是白色,上面有斑点,大约10厘米长。在繁殖季节,它们会在南冰洋的岛上(如奥克兰、新西兰、爱德华王子群岛等)占领一些松散的领地来筑巢并且繁育后代。它们的巢呈火山状,用植物建造,在底部宽约一米,顶部宽约半米。以前,水手们经常为了信天翁的骨头而捕捉信天翁,他们用它翅膀上的长骨头来做烟斗的管。在航海中,常常有信天翁跟随着船只,环绕着船只飞翔。早期来到南冰洋的探险家们常常为他们在孤独中和信天翁的结下的友谊而喝彩,在柯勒律治的《古舟子咏》中讲述了十字弓射下好运之鸟的恶人的命运。","text2":"漂泊信天翁羽毛的颜色跟什么有关?","label":1} {"text1":"椰子罐头(COCOCAN),原名水果村、诺亚方舟(Noah's Ark)是由游戏橘子开发的休闲网路游戏,台湾方面由于人数不断锐减,自2008年10月1日起停止营运。椰子罐头自2003年初在台湾推广以来,吸引不少网友的兴趣,玩家年龄层广,之后也在日本、香港等地掀起风潮。为了能让名称更贴近游戏的性质,游戏橘子决定将「诺亚方舟」改名为「椰子罐头」,由于本游戏是利用风向及仰角抛投各类武器来攻击对手,以获得等级的晋升和金币,而「COCOCAN」的台语谐音为「一直丢、一直丢」,再把「COCONUT」和「CAN」两个英文单字拼装起来,中文翻译即《椰子罐头》,用此作为正式名称。日本游戏橘子于2009年1月30日发布新闻表示,日版椰子罐头将于当地时间4月30日晚间五点起停止营运,届时椰子罐头将完全走入历史。台湾方面由于玩家人数不断锐减,游戏橘子于2008年7月1日发布即将停止营运公告,自10月1日起台湾版椰子罐头正式停止营运。香港版停止营运。在本游戏中,共设计出12个人物,但截至目前为止能选择的角色有9个。以下所列能力之均为初始值,创立角色另有7点基数(台湾版后期仅有2点)可调配至各属性中,另也可通过游戏内道具提升。注1:角色原名系指「水果村」及「诺亚方舟」时期的名称,栏位空白表示游戏更名「椰子罐头」后其角色名称无异动,标明\"-\"者代表该角色于「水果村」及「诺亚方舟」时期尚未登场。注2:官方所给予之中文名称仅有表单上所列出之四个角色,「酷猫」之名称为游戏更名为「椰子罐头」后同步更名。","text2":"椰子罐头是什么?","label":1} {"text1":"夏威夷秧鸡()是夏威夷岛一种神秘的秧鸡,但现已灭绝。牠们不懂得飞,生活在灌丛带。在普纳地区附近发现了几个夏威夷秧鸡的标本,包括一些较深色及一些较浅色而有斑点的标本。最先采集的夏威夷秧鸡标本是较为浅色的形态,共有五个标本,分别存放在莱顿的荷兰国家自然科学博物馆及纽约的美国自然历史博物馆。另外几个较深色形态的标本则存放在剑桥的大学自然历史博物馆、伦敦的自然历史博物馆、维也纳、纽约及檀香山的毕晓普博物馆。1778年绘画的一幅图画中是较浅色的夏威夷秧鸡,有可能就是莱顿的标本。近年亦有发现牠们的亚化石。由于夏威夷秧鸡有两种形态,造成了很多混淆。根据采集夏威夷秧鸡标本的过程,牠们最有可能生活在欧胡岛及考艾岛,而考艾岛亦曾有一种类似体型的动物出现。不过考艾岛发现的骨头却属于较大体型的,而夏威夷秧鸡的标本则较细小。在夏威夷岛发现了一种较细小的田鸡属亚化石,但其大小只如雷仙岛秧鸡。另外在凯卢阿-科纳亦发现一些骨头,约较夏威夷秧鸡大15%。一般接受较浅色的形态是未成年的夏威夷秧鸡,但却仍有待DNA研究证实。不论如何,浅色及深色的形态都被认为是同一物种,一些异名如下:最后五个异名是指深色的形态。当中\"Rallus obscurus\"在被描述时仍未有发现深色的形态,描述有可能是根据在莱斯特大宅的标本而造的。夏威夷秧鸡浅色形态的标本是于1778年在詹姆斯·库克(James Cook)的第三次旅程采集的,相信于当时并没有发现深色的形态。大部份或全部深色形态的标本都是于1860年至1864年采集的。最近证实见到夏威夷秧鸡是于1884年,另一个存疑的观察则是于1893年。1887年的搜寻未能发现牠们,不过却不足以确定牠们的灭绝。夏威夷秧鸡的灭绝可能是因入侵的大家鼠、猫及狗。狩猎亦可能是其中一种原因。","text2":"夏威夷秧鸡生活在什么地方?","label":1} {"text1":"芭芭拉·斯坦威克(Barbara Stanwyck;),本名露比·凯瑟琳·史蒂文斯(Ruby Catherine Stevens),出生于美国纽约州布鲁克林,为好莱坞知名女演员。三岁时,因为一场车祸,造成其母亲意外身亡,父亲也离家出走。13岁时辍学,在布鲁克林的一家百货公司当包装工。一心想跳舞的她从微簿的工资中赚取舞蹈课的学费。两年后,开始了演艺生涯,在齐格菲歌舞团担任跳舞女郎。1927年,改名为芭芭拉·斯坦威克,她在无声片《百老汇夜晚》中首次登上萤幕。1930年,斯坦威克参加了《闲花泪》的拍摄,导演法兰克·卡普拉发现了她的才华,让她在《群众》(\"Meet John Doe\")等影片中担任主角,从而使人们渐渐熟知了斯坦威克。1937年,斯坦威克在这部由金·维多执导的影片《慈母心》中,成功地试演了一位富于自我牺牲精神的坚忍母亲,因而赢得了奥斯卡最佳女主角提名。1940年代,她又分别以《郎才女貌》(又译《火球》)、《谋杀案》、《火车谋杀案》、《打错电话了》等影片,三次获德奥斯卡最佳女主角金像奖提名。1981年,林肯中心电影协会授予她电影成就奖。1982年,斯坦威克获颁奥斯卡终身成就奖。","text2":"1981年,林肯中心电影协会授予芭芭拉·斯坦威克什么奖项?","label":1} {"text1":"石越车站()是一由东日本旅客铁道(JR东日本)的所经营铁路车站,位于日本宫城县登米市石越町南乡字西门冲。石越是JR东北本线沿线的一个车站,由小牛田车站负责管理。根据JR东日本旗下各支社的管辖范围划分,石越车站是东北本线上与的界线,超过石越之后以北的诸站,都是由盛冈支社所管辖。除了JR东日本之外,石越车站原本也是第三部门铁路业者(,其前身为「栗原轨道」)的车站之一,也是其所属的之起点。其车站站房位于JR石越车站站前广场西北角的位置,但车站早已在2001年时就改为无人化经营、2007年3月随著栗原田原铁道线废线而停止营运。虽然在栗原田原铁道时代,该公司所属的路线与JR路线之间并没有实际的连结,但在之前的栗原电铁时代,由于货运列车的通过需要,曾设有连络线。在栗原电铁的最盛时期,甚至有过由仙台发车并直接由本站转入栗原电铁线的直通运行列车。东北新干线的栗驹高原车站是离本站最近的新干线车站,相距约9.5公里,有定期班次公车接驳。石越是一座设有侧式月台1面1线与岛式月台1面2线,合计2面3线的地面车站。各月台以跨线天桥连接。","text2":"超过石越之后以北的诸站,都是由谁所管辖?","label":1} {"text1":"达维德·罗泽纳尔(捷克语:,),是一名捷克足球运动员,司职中坚,于2011年夏季由德甲球会汉堡转投法甲球会里尔效力至今。罗泽纳尔出身于捷克国内球会奥洛穆茨西格玛,早于2003年已转到比利时效力当地球会布鲁日。在比利时期间,他随队赢得了2005年比甲联赛冠军。这使他获得法甲球会巴黎圣日耳曼青睐,于2005年6月加盟该队。在巴黎期间他获得球会2006-07年度球队最佳球员,但是由于巴黎圣日耳曼实力不济,长期都处于护级状态,使他在球队时从未赢过法甲联赛冠军。2007年6月,他以290万英镑转会费,被卖到英超联赛球会纽卡素。然而他在首半季未能适应英超比赛节奏,加上纽卡素内部出现人事变动,出现混乱,2008年1月他被外借到意甲球会拉素,租借期为半个赛季。至2008年夏季,罗泽纳尔以290万英镑的价格正式加盟拉齐奥。仅渡过一个球季,于2009年7月29日加盟德甲的汉堡,签约三年,身价达到490万欧元。2010年9月1日,被德甲汉堡放入可转会名单的罗泽纳尔以借用身份加入法甲球会里尔,借用期为一个球季。2011年6月30日,罗泽纳尔正式转会法甲里尔,合约为期三年。他早于2003年以前已是捷克青年军成员。2004年欧洲国家杯他被召入国家队,随队打入准决赛。但他在国家队的最佳成绩暂时仅止于此。2006年世界杯打了第一圈便出局,2008年欧洲国家杯他随队出战,也只能在小组赛结束后便打道回府。","text2":"达维德·罗泽纳尔为什么会获得巴黎圣日耳曼青睐?","label":1} {"text1":"《忘记你我做不到》,是香港歌手张学友的第九张国语专辑,该专辑于1996年6月由宝丽金唱片公司(台湾)发行。该张唱片是张学友巅峰时期的一张制作精良的国语唱片,唱片词曲多数由台湾本地音乐人创作。专辑中有多个歌曲在多年后仍被认为是经典国语歌曲的作品,亦有超过五首歌曲成为当年的音乐排行榜冠军歌曲,包括:《忘记你我做不到》、《情书》、《慢慢》、《我不愿意》及《左右为难》。《忘记你我做不到》在台湾,香港,新加坡,马来西亚等地都取得了非常炫目的销量。在发行此张唱片后,传媒亦有称张学友为华语乐坛的销量保证,乐坛前辈谭咏麟更是表示张学友是香港乐坛的良心。唱片不但打破多个销量记录以及获奖记录,单曲《左右为难》更是令香港实力派歌手郑中基一夜成名,并获得多个颁奖礼的最佳合唱歌曲。由于张学友在台湾取得了相当程度的成功,此年台湾亦有部分音乐人士建议改革金曲奖的最佳国语男歌手的国籍限制,可以令如张学友这样在台湾具有较高影响力的其他地区歌手公平的参与进来,并增加金曲奖的影响力。","text2":"《忘记你我做不到》中的哪些歌曲成为当年音乐排行榜冠军歌曲?","label":1} {"text1":"朴主永(,)。韩国足球运动员,司职前锋,现时效力经典K联赛球队FC首尔。速度是他最大的武器,曾在11秒内完成100米跑,而且是主射罚球的专家,有超强的破关能力,并且有令人惊叹的创造力,不时能够送出致命传球,协助队友取得入球进帐。2005年,朴主永在FC首尔出道,他19岁首次进入国家队,20岁即成为K联赛神射手,2005年被选为韩职最有价值球员。2006年,21岁的朴主永参加德国世界杯前,是公认的十大最值得期待的新星之一。职业生涯于2008年发生下滑之际,朴主永成功登陆法甲加盟摩纳哥并把握住命运,以出色的传球和自由球功夫坐稳摩纳哥主力前锋的位置。朴主永在摩纳哥効力了三个球季,其中在2010\/11年球季朴主永为球队在联赛出场33次射入12球,但该球季摩纳哥最终从法甲降班。2010年南非世界杯,朴主永身穿10号球衣,打满三场小组赛,带领韩国队获得小组第二杀入十六强,并在小组赛最后一场韩国队对尼日利亚的比赛中,射入当届世界杯第一个直接任意球破门。,最后韩国以2比2和尼日利亚打成平手。2011年8月30日,朴主永以400万镑转会费加盟英超球队阿森纳,签约2年,披上9号球衣,但朴主永转投阿仙奴后表现下滑,只为阿仙奴在英超上阵七分钟,即使被外借至西甲球队切尔达及英甲球队屈福特,仍未能重拾状态,2014年更被会方弃用。2014年10月1日,朴主永加盟沙特职业足球联赛球队艾沙巴布。2015年3月15日,朴主永回归成名地FC首尔,再续前缘,签约3年,借此解决球队火力疲弱的毛病。2016年11月6日,赛前同分、分列首、次名的全北现代汽车与FC首尔进行终极一战,59分钟,后备入替的朴主永接应尹日录的传送,在对手后卫力逼下仍能快一步起脚破网。FC首尔以1:0取胜,赛后累积70分封王。","text2":"2016年全北现代汽车与FC首尔终极一战中,朴主永做出了哪些贡献?","label":1} {"text1":"『死神的精确度』(しにがみのせいど、\"ACCURACY OF DEATH\")为伊坂幸太郎的日本小说作品,另外本作也被改编成广播剧和电影。本作为一个有点冷酷的死神,要以七天的日子来决定所调查的对象是否要死并以这些人构成六部短篇内容的故事。以下为本作的六篇短篇由NHK-FM频率的青春アドベンチャー加以广播剧化。初次播放日在2006年10月30日(月)〜11月3日(周五)、全5集。分别以书中的五个短篇「死神的精确度」「死神和藤田」「暴风雪中的死神」「恋爱中的死神」「死神对老婆婆」加以改编。《死神的精准度》(英语:Sweet Rain)「Sweet Rain~死神的精准度」为其电影标题,在2008年3月22日上映。取用「死神的精确度」、「死神和藤田」、「死神对老婆婆」三个短篇来构成电影版的内容。这也是金城武自从「回归者」以来,阔别六年而再度演出日本电影。本作能加以电影化,是作者伊坂提出工作人员中一定要由金城武主演的条件并获得承诺才加以实现。本超人气推理小说作家伊坂幸太郎(《家鸭与野鸭的投币式置物柜》)名作《死神的精确度》改编。千叶(金城武)是一名死神。酷酷的、不大理解人情事故、少根筋的死神。每当他出现,人间必定下雨。当人类碰到他的手,除了昏倒还会折寿。另外,他会依工作内容变化外型与年纪。他的工作是利用一个星期观察、接触特定的人类,最后再向高层提出报告,判断观察对象要「认可」(死亡)或「放行」(生)。今天,他又来人间执行任务,对象预定在七天后死亡的藤木一惠(小西真奈美)⋯⋯","text2":"『死神的精确度』说的什么故事?","label":1} {"text1":"中国钱包支付集团有限公司,(,),简称为中国钱包支付集团、中国钱包支付和中国钱包,是一间电脑软件公司,开发生物辨识及认证技术应用,成品如香港入境处的E道。前身为「宏霸数码集团(控股)有限公司」。宏霸数码集团成立于1999年,投资者包括龚如心和香港前风水师陈振聪。于2004年在伦敦证券交易所另类投资市场上市。头条网更称之为业内的龙头。2009年2月10日,宏霸数码集团于香港交易所上市,公司注册地在百慕达,前主席是朱伟民,核数师是国卫会计师事务所。根据上市文件指出,最大的单一股东是Veron International Limited,Veron International是己故富商龚如心其下的遗产之一。","text2":"2009年2月10日,宏霸数码集团于香港交易所上市,公司注册地哪里?","label":1} {"text1":"马来粽()是一种东南亚传统食物,在马来西亚、印尼、新加坡等马来族居多处欢庆伊斯兰开斋节常见这种食物,并且当成开斋节主要象征之一。马来粽由于制法与形状类似华人所扎的粽子,所以当地华人习惯上把它称作“马来粽”。通常所见的马来粽大抵可分两种:一种是采用羽状棕榈叶()包裹的,另一种却是用椰叶扎成。用来包扎马来粽的棕榈叶子与槟榔叶子大同而小异,长约二尺,每根羽叶拥有骨干三四支,张起来可有约三四寸的阔度,所以一粒粽子只须一根羽叶便足;绝不像华人所用的箬叶那么累赘。其次,由于棕榈叶子骨干细幼而韧,因而事先无须削去骨支也可如意包扎。按照一般的包扎方法,首先是将叶子摊开,打成活结样子的三角形粽囊,然后从空隙中塞入糯米。不过,这糯米并不是生米,而是事先调合适量椰浆煮过的半熟米。当空囊被塞满后,活结便加以缩紧,俾使叶子坚牢包扎住米粒;为了免得在烹煮时脱落起见,末端更须打上上个小结。扎妥后的粽子固是三角形,惟不似华人粽子的立方三角,而是略扁的等边三角形。(现代也常见方形马来粽。)在下锅之前,所有的粽子也分作三五只绑成一束,以便提携和计算,不过从头至尾无须任何绳子绑扎,因为那细长的叶子不但足以包扎,而且还有余叶可供绑束,这比起华人的粽子,无疑省却了不少功夫。专用来配食“沙嗲”的四角形粽子,也叫“都拔”,只用白米制成。包扎法是以椰子树的嫩叶,用编织草席的方法织成一只只四方形而略扁的小粽囊,仅留著上端开口,从中塞入洗净的米粒。编织多余叶子则用以绑扎成束。","text2":"马来粽这种东南亚的食物,一般用在什么地方?","label":1} {"text1":"《独家追辑》(),马来西亚ntv7电视台与制作公司Double Vision联合制作的一套时装电视剧,共20集。FMP杂志社因为两名资深记者突然辞职而陷入一片混乱,然而身为杂志社老板兼主编的何志光却置身事外,把所有的工作都交给了员工。杂志社采访主任吴皓泽和摄影记者杜子腾无意间卷入偷车事件,让保险员文沛晴无故被误会是共犯。在深入调查后,吴皓泽发觉文沛晴根本没有与偷车集团合谋,真正主谋竟然是广志强,一位经验老道的汽车销售员。在吴皓泽跟警界好友高勇合作下,广志强最后绳之于法。患有严重心脏病的孤儿李宛心在杂志社的大肆报道下找到了合适的心脏移植,无奈的是心脏出现排斥现象,最终不治身亡。可贵的是,李宛心的逝世唤醒了民众关于捐赠器官的意识。不过也有一些不法集团企图利用李宛心的事件募款基金,幸好最终叫颜慧妮等人给识破,及时阻止。何志光之幼妹何芷萱与上司闹翻,被逼从大报馆转到大哥的小杂志社任职。好胜的何芷萱与吴皓泽性格和工作方式迂回,两人因此成了斗气冤家。吴皓泽因一场交通事故,与文沛晴再次结缘,发展一段似有似无的感情。何志光和PINK姐前后陆续无故离奇失踪,何志光的长妹何芷杉从台湾赶回来撑局,好让何芷萱和吴皓泽可以追踪何志光失踪的新闻。经过一番查访之后,最后他们终于找到了何志光,原来何志光是遭到公司的员工小松绑架。小松因为涉及放火烧死一家五口案件而被何志光识破,发了疯的小松绑架何志光欲与他同归于尽,幸得吴皓泽等人时赶到,成功把何志光救出。另一方面,何芷萱因与吴皓泽一起工作又一起去追踪哥哥何志光失踪案件而渐渐喜欢上他,因而与吴皓泽、文沛晴陷入三角恋中。何芷萱与吴皓泽越来越合拍,引起文沛晴妒嫉,吴皓泽则面临选择,最终明白所爱的人是何芷萱,无奈何芷萱已离开大马。几年后,何芷杉放弃了台湾的事业留在大马发展;何志光与PINK姐有情人终成眷属,过着退休生活;颜慧妮和杜子腾各有各发展,唯独吴皓泽仍然独身,依然继续在新闻界冲刺。一日,吴皓泽外出跑新闻时突然被一辆回退的车子撞倒,下车理论的时候,发现司机竟然是刚刚从外国留学回来的何芷萱……主题曲:《记录》罗忆诗、李承运","text2":"《独家追辑》一共多少集?","label":1} {"text1":"枫树脚溪,又名南屯溪,旧称犁头店溪,位于台中盆地,属于乌溪水系,为土库溪的支流,流域分布于台中市乌日区、台中市中心及台中市潭子区,因流经枫树脚聚落(今南屯区枫树里)而得名。枫树脚溪本流发源西屯区何厝庄,向南流经南屯区沟仔墘、犁头店、下枫树脚,于南屯区、乌日区交界处汇入东侧流来之土库溪。依据经济部水利署公告,枫树脚溪注入土库溪,以土库溪为主流。旧南屯溪附近地区,早年经文史团体调查即发现有丰富的史前遗留,并将其命名为「麻糍埔遗址」。从台湾清治时期至民国70年代,南屯溪与南屯地区居民生活相互依存。旧南屯溪于清代台中盆地开垦史中,因邻近往来鹿港与葫芦墩街之贸易古道,逐渐形成犁头店街附近的重要农村聚落。随著台中市第二、三、四期扩大都市计划发布实施,南屯地区逐渐发展成住宅社区,以黎明路周边发展最为快速。80年代,配合第七期、第八期重划区开发,部分南屯溪河道纳入重划区排水,1995年(民国84年)南屯溪整治完成后,仅永顺路与永春东二路间第13期市地重划范围内尚存一小段天然河道,为南屯溪原本的自然河道,称为「旧南屯溪」,范围北自永顺路靠文心南七路段,沿枫乐巷南抵永春东二路和环中路交叉处附近。现今旧南屯溪之排水功能则已被新南屯溪的排水道取代。旧南屯溪拥有丰富生态且兼具疏洪功能,鱼类、水草与鸟类在此和谐共生,为都市中难能可贵的自然生态资源。此外,旧南屯溪部分河段保留水利设施,提供周边土地农作灌溉功能,象征旧南屯溪对传统农业社会的贡献。由于旧南屯溪有高度景观及人文意义,代表先民当年溯溪至台中开垦之历史意义,对南屯地区的发展具有历史、文化上之意义与价值,而且旧南屯溪沿线有优美绿带,展现旧南屯溪的自然生态与历史古迹,旧南屯溪畔有史前麻糍埔遗址,亦反映过去人类利用此溪水生活之意义。因此经过地方及文史专家争取,2013年10月台中市文化局正式将位于第13期重划区内的旧南屯溪、铁道等设施列为文化景观,成为台中市第1个河流的文化景观。","text2":"枫树脚溪在哪里?","label":1} {"text1":"分析机是由英国数学家查尔斯·巴贝奇设计的一种机械式通用计算机。从1837年首次提出这种机器的设计,一直到他去世的1871年,由于种种原因,这种机器并没有被真正的制造出来。但它本身的设计逻辑却十分先进,是大约100年后电子通用计算机的先驱。查尔斯·巴贝奇最初尝试的所谓差分机,可以通过求解差分来计算对数表和三角函数表,然后能近似计算多项式。由于巴贝奇与他的首席工程师起了争执,英国政府就撤回了这项项目的资金,差分机也因此没能完成。在这期间,巴贝奇意识到建造一种更加通用的机器(即所谓的分析机)是可行的,于是便于1833年开始了分析机的设计。分析机由蒸汽机驱动,大约有30米长、10米宽。它的输入由程序和数据组成,并使用打孔卡输入,这种输入方法被当时的织布机广泛采用。分析机通过一台打印机、一个弯曲的绘图仪和一个铃铛输出,也可以在纸上打孔以便日后读取。分析机采取普通的十进制定点计数法。它的“记忆体”大约可以存储1000个40位的十进制数(每个数约16.2kB)。有一个算术逻辑单元可以进行四则运算、比较和求平方根操作。刚开始研制的时候,分析机的外观被普遍认为和差分机相似 。1858年的图纸呈现了一个有规律的网格布局。与现代计算机的中央处理器(CPU)类似,其算术逻辑单元使用的微程序存储在插在被称为“桶”的滚筒上的支柱中,这为用户指定更加复杂的运算提供了便利。分析机使用的编程语言与今天的汇编语言类似,支持循环语句和条件分支,因此这门语言被认为是图灵完备的。分析机采用三种不同的打孔卡和读卡器来区分算术运算、数字常量和存储的指令,以此实现了数字在存储器和运算单元之间的加载和存储操作。巴比奇在1837至1840年间写下了24份程序,并在之后又写了一份。这些程序可以计算多项式、迭代公式、高斯消去法和伯努利数。","text2":"分析机是谁设计的?","label":1} {"text1":"米拉加亚龙属(学名:\"Miragaia\")是种剑龙下目恐龙,化石发现于葡萄牙,年代属于侏罗纪晚期。米拉加亚龙以牠们的长颈部而闻名,颈部具有至少17节颈椎。米拉加亚龙的正模标本(编号ML 433)发现于葡萄牙北部奥波多市的劳尔哈组(Lourinhã Formation),年代为侏罗纪晚期(启莫里阶晚期到提通阶晚期),约1亿5000万年前。这个标本是由一个部分颅骨,以及部分的身体前半段所构成;包含以下部位:大部分口鼻部、15节颈椎(缺少最前2节颈椎)、肩带、大部分前肢、13个背部骨板。米拉加亚龙的颅骨,同时也是欧洲所发现的第一个剑龙类颅骨。在正模标本的发现处附近,另外发现一个幼年个体标本(编号433-A),包含:一个部分骨盆、部分的脊柱,也被归类于米拉加亚龙。米拉加亚龙是由奥克塔维奥·马特乌斯(Octávio Mateus)等人在2009年叙述、命名。模式种是长颈米拉加亚龙(\"M. longicollum\");属名是以化石发现处的奥波多市的Miragaia堂区为名,种名则意为「长颈」。马特乌斯等人同时也提出一个系统发生学研究,认为米拉加亚龙与锐龙属于一个名为锐龙亚科(Dacentrurinae)的演化支;而锐龙亚科与剑龙属都属于剑龙科,两者互为姊妹分类单元。米拉加亚龙的明显特征,是其长于一般剑龙类的颈部,由至少17节颈椎所构成;与传统观念中,剑龙类的低矮步态、短颈部不同。米拉加亚龙的颈椎数量,甚至比大部分的蜥脚类恐龙还多(多介于12节到15节颈椎);只有盘足龙、马门溪龙、峨嵋龙等蜥脚类恐龙的颈椎数量,超过米拉加亚龙。马特乌斯等科学家推论,米拉加亚龙的长颈部可使牠们有更大的进食范围,或者是在求偶时具有视觉辨认的功能。蜥脚类恐龙的长颈部,是由部分背椎向前移动构成颈部脊椎、额外增加的颈椎、每个颈部脊椎的长度加长…等因素形成。科学家推测米拉加亚龙的长颈部,是由部分背椎向前移动构成颈部脊椎而形成;而非额外增加的颈椎。与剑龙属相比,米拉加亚龙的颈椎长度略长,但这可能是死后的化石化过程中,遭到外力变形的后果。与其他剑龙科相同,米拉加亚龙的口鼻部前端缺乏牙齿。前肢的尺骨\/桡骨长度比例,与剑龙属的比例相近。颈部肋骨与颈椎愈合。耻骨的末端大,与锐龙相同。背部骨板成三角形。","text2":"米拉加亚龙以牠们的什么而闻名?","label":1} {"text1":"DVD播放程式(DVD Player)是Mac OS X的默认DVD播放器。它支持所有的DVD标准功能,比如多重音频、视频及字幕轨道以及杜比音效、DVD@ccess URL和关闭字幕。在某些情况下,用户可以选择其中的VOB文件并将其打开。这个文件的地址是「\/Applications\/DVD Player.app」。只有当有一个DVD驱动器连接到计算机时,DVD播放程式才会被安装到OS中(例如,当一部电脑的引导盘磁盘映象发现计算机上只有CD-ROM而没有DVD-ROM驱动器,将不会启动DVD播放程式的安装)。DVD播放程式也完全兼容DVD Studio Pro和iDVD所撰写的DVD,包括用DVD Studio Pro撰写的HD DVD。DVD播放程式顺从大部分著作权法,并因此将执行最严格的限制DVD措施,如区域编码限制和限制用户行为(「停用行为」)。它甚至会令苹果的屏幕截取程序失效,直到退出DVD播放程式程序。这有效的防止了用户从屏幕上捕捉DVD内容(但用户也能通过使用Terminal command间接得到图片。当然,其它的一些软件也能达到此目的,比如VLC media player。另外,DVD区域编码限制也能通过MacTheRipper这个软件去除)。","text2":"DVD播放程式支持哪些DVD标准功能?","label":1} {"text1":"《科学战队炸药人》(原题:)是日本东映公司在1983年制作的「超级战队系列」第7部特摄作品。在日播放期间为1983年2月5日-1984年1月28日,全51集。远古时代,坠落于地球上的陨石中残存的生命物质,在地底下经过长年的进化成了有尾人一族-邪进化帝国。因尾巴的数量代表其身分的象征的特有文化,将利用它们超乎人类想像的科学力征服地上的人类,以让日本各地火山喷发为第一步,开始了侵略地上世界的行动。然而,及早察觉到这件事的科学家·梦野久太郎,集合了五名智慧与体力都相当优秀的年轻科学家到自己所营运的发明中心来,也正式宣布科学战队的成军。五人著装了梦野博士开发的战斗强化服后,成了炸药人并衔命全力阻止邪进化帝国侵略的邪恶野心。第9回后的播出时间皆缩短至25分钟。梦野久太郎于远古时代坠落到地球上的陨石中生命物质,长久以来寄居于地底进行著自主进化,并成为了能自由操纵火山的地底支配者「有尾人一族」们的帝国。属于卵生,故于蛋中出生,会将尾数加于名字,且尾数越多代表在帝国中越高。十分藐视没有尾巴的人类(地上人),并视为「下等生物」,在无法透过实验让人类长出尾巴后,便乘著有如怪物一般外观的巨大攻击要塞格兰吉兹摩,向地表上的人类展开侵略行动。亚顿卡亚洁诺比亚梅基度奇梅拉进化兽机械进化兽大爆炸光束小尾兵格兰基兹摩基兹摩斯奇托大木星 DyJupiter炸药马赫 Dyna Mach炸药战车 Dyna Mobile炸药拖车 Dyna Garry炸药猎鹰 Dyna Falcon炸药吉普车 Dyna Machine第一话登场。当炸药红喊出『合体! Grand Slam!』后,由炸药马赫、炸药战车、炸药拖车三体合而为一的巨大机器人。由梦野博士开发制造。于剧场版『199英雄大决战』中登场。武装招式必杀技以下时间以当地时间(日本时间)为准本作品的战队服装曾经被台湾自制特摄节目《太空战士》作为参考。片头曲片尾曲插曲","text2":"谁集合了五名智慧与体力都相当优秀的年轻科学家到自己所营运的发明中心来,也正式宣布科学战队的成军?","label":1} {"text1":"青花釉里红瓷仓,又称红釉堆塑楼阁式仓,是元代瓷器珍品,造型别致,属于随葬明器,青花釉里红瓷器物本已非常罕见,这种楼阁式瓷仓且有明确纪年的样式,迄今仅此一件。1974年出土于江西省景德镇,1979年9月,丰城县文化馆在江西省文物商店协助下征集得到,现藏江西省博物馆。瓷仓通高29.5cm,横宽20.5cm,进深10cm。仓顶庑殿重檐式,仓瓦为釉里红点彩串珠组成,中以隔墙分为前、后楼。楼阁高低相错,造型别致,充分体现了元代江南木结构建筑特色。插板式活动仓门两侧用青料书直行楷书七言句对联,右联为“禾黍丰而仓廪实”,左联为“子孙盛而福禄崇”,横披“南山宝象庄五谷之仓”。仓后两柱间空档有墓志,为青料直行正楷,共计12行,159字。根据墓铭,死者为“故景德镇长芗书院山长凌颖之孙女”,死于元惠宗至元四年戊寅(1338年),葬于南山。","text2":"青花釉里红瓷仓的珍贵性体现在哪里?","label":1} {"text1":"黑龙江省大兴安岭林业集团公司暨黑龙江大兴安岭林业管理局,是中国首批57家试点企业集团之一,隶属于中华人民共和国国家林业局。其上级不是黑龙江省。黑龙江大兴安岭林业管理局与黑龙江省大兴安岭地区行政公署为一套班子、两块牌子。林管局局长兼任地区行署专员、大兴安岭地委委员、地委副书记;副局长兼任副专员。1996年3月,国务院正式批准大兴安岭林业管理局成立大兴安岭林业集团公司。黑龙江省大兴安岭林业集团公司辖10个林业局,61个林场。  加格达奇林业局(驻加格达奇区)辖10个林场,即古里林场、白桦林场、东风林场、翠峰林场、跃进林场、多布库尔林场、达金林场、大黑山林场、大子扬山林场、罕诺河林场。  松岭林业局(驻小扬气镇)辖5个林场,即绿水林场、古源林场、大扬气林场、壮志林场、新天林场。  新林林业局(驻新林镇)辖8个林场,即塔源林场、宏图林场、新林林场、大乌苏林场、碧洲林场、翠岗林场、塔尔根林场、富林林场。  呼中林业局(驻呼中镇)辖6个林场,即雄关林场、呼源林场、苍山林场、宏伟林场、呼中林场、碧水林场。  塔河林业局(驻塔河镇)辖6个林场,即瓦拉干林场、蒙克山林场、塔林林场、绣峰林场、盘古林场、沿江林场。  阿木尔林业局(驻漠河县劲涛镇)辖6个林场,即长山林场、依林林场、龙河林场、兴安林场、红旗林场、青松林场。  图强林业局(驻漠河县图强镇)辖7个林场,即奋斗林场、二十八站林场、育英林场、壮林林场、潮河林场、卧龙河林场、潮满林场。  西林吉林业局(驻西林吉镇)辖5个林场,即河湾林场、前哨林场、河东林场、古莲林场、金沟林场。  十八站林业局(驻塔河县十八站乡)辖6个林场, 即十八站林场、查班河林场、小根河林场、白银纳林场、永庆林场、双河林场。  韩家园林业局(驻呼玛县韩家园镇)辖1个林场,即韩家园主伐林场。","text2":"黑龙江省大兴安岭林业集团公司隶属于哪个机构?","label":1} {"text1":"捷克爱乐乐团是一支捷克交响乐团。2006年一次由法国杂志《Le Monde de la Musique》的调查中,它被评为欧洲十大乐团第九位。乐团成立于1896年。它的原始成员来自布拉格国家歌剧院。1905年乐团独立,1945年被国有化。1946年,乐团出席第一届布拉格之春国际音乐节以庆祝其成立五十周年纪念。乐团的第一场音乐会由捷克音乐家德沃夏克指挥。而它与其他捷克音乐家的关系也非常密切,如斯美塔纳,扬那切克和马替奴。两战期间,乐团名声鹊起,当时领导乐团的是指挥家瓦茨拉夫·塔利赫()。随后是有名的捷克指挥家拉法埃尔·库贝里克(任期为1942-1948),卡罗尔·安切尔(Karel Ančerl,任期为1950-1968)和瓦茨拉夫·纽曼(任期为1968-1989)。吉里·贝洛拉维克曾两次出任首席指挥。乐团曾和很多有名的指挥家合作,如古斯塔夫·马勒,理查德·施特劳斯,保罗·欣德米特,赫伯特·冯·卡拉扬,卡尔·伯姆和祖宾·梅塔。","text2":"捷克爱乐乐团的原始成员来自哪里?","label":1} {"text1":"《阴道独白》()是美国女作家伊芙·恩斯勒()所著的戏剧,1996年由作者本人首演于纽约外百老汇,获1997年奥比奖最佳剧本奖。该剧至今至少已被翻译成50种语言,在140个国家上演过。该剧至少已有5个不同的中文译本。《阴道独白》2004年曾在北京和上海被双双禁演;但几年来,该剧已在中国大陆的至少十所高校被未经授权的学生搬上过舞台。中国大陆的授权中文首演是2009年3月薪传实验剧团在北京9个剧场的5场公演,由新浪潮戏剧人王翀翻译并导演,至今已在北京、上海、深圳、长沙、杭州、天津演出30余场,全部提前售罄,引起凤凰卫视、《北京青年报》、《青年周末》、《时代周刊》、《侨报》、《南方都市报》、《南方周末》等中美媒体的关注。美国《时代周刊》将中国大陆的首演评价为“一系列的售罄演出,造成轰动。”磨铁图书有限公司曾经签下王翀译本的版权,但因为无法通过图书审查,至今未能出版。V日义演是《阴道独白》的作者恩斯勒发起的反对对妇女暴力的活动。这个全球性的活动于2010年已经进入了第12个年头,已经筹资超过7000万美元。《阴道独白》V日义演规则:2013年11月7日,北京外国语大学17名女生为了宣传《阴道独白》在校园演出,在人人网上发布一组持牌照片,牌子上写着“我的阴道说:我要自由”等以“我的阴道说”开头的文字,引起中国社交网站上许多负面评价。","text2":"V日义演是什么?","label":1} {"text1":"《奥利瓦条约》(德语:'、波兰语:'、瑞典语:)也称《奥利瓦和约》,于1660年4月23日在波兰王国王家普鲁士省但泽附近的奥利瓦签署,签署方包括神圣罗马帝国皇帝利奥波德一世、勃兰登堡-普鲁士选帝侯腓特烈·威廉、瑞典国王卡尔十世和波兰国王约翰二世。条约中,约翰二世宣布放弃对瑞典王位的要求,同时放弃利沃尼亚和里加,上述地区自1620年代后被瑞典控制。条约解决了自齐格蒙特战争(1598年-1599年)、波兰-瑞典战争(1600年-1629年)、北方战争(1655年-1660年)以来瑞典和波兰之间一直持续的战争状态。条约还确认霍亨索伦家族拥有曾是波兰附庸的普鲁士公国的完全主权,但如果霍亨索伦家族被结束其统治,则普鲁士仍归波兰王室拥有。《奥利瓦条约》和同年《哥本哈根条约》的签署代表瑞典帝国的实力达到顶峰。","text2":"《奥利瓦条约》在哪里被签署?","label":1} {"text1":"科提斯·史东(,) 出生于澳大利亚 ,是一位厨师、烹饪书籍作者、与电视节目主持人。科提斯在澳洲的Penleigh and Essendon Grammar学校念书。他的父亲William是位会计师。他在五岁时就跟随奶奶下厨了。科提斯在对食物产生兴趣前就读商业学士学位,18岁时他开始从事厨艺事业,他到欧洲体验生活,并且学习义大利,法国,西班牙的美食。2008年,史东主持《帅哥厨师来我家》,并且著作了系列的食谱书,著作的\"Recipes to Put You in My Favorite Mood\"于2009年4月上市。2013年,主持《决战全球餐厅》。他在电视节目Martha中,宣称自己是单身。史东被People杂志列为最性感男性,几近追上乔治·克隆尼和Brad Pitt。他于2009年开始与女星 Lindsay Price 交往,2011年11月6号,他们的儿子Hudson出世,他们也于2012年11月宣布订婚。于2013年6月8号结婚。地点The Hilton's Sa Torre Mallorca Resort. 2014 年九月他们第二个儿子Emerson于加州出世","text2":"科提斯在对食物产生兴趣前就读的是什么专业?","label":1} {"text1":"这是曾对英国上下两院发表演说的人列表。由于每年举行国会开幕大典的时候,英国君主都会对两院议员致辞,所以本条目不包括每届国会开幕大典时的英国君主。1939年,法国总统阿尔贝·勒布伦成为第一位同时对英国上下两院发表演说的人士。除英王外,只有三位人士不止一次地登上两院讲台。其中一位是曼德拉,他先后于1993年以诺贝尔和平奖获得者身份和于1996年以南非总统身份两次登台讲演。另一位则是米哈伊尔·戈尔巴乔夫,他于1984和1993年分别以苏联外交代表和各国议会联盟代表的身份发表演说。此外,希蒙·佩雷斯于1986年和2008年分别以以色列总理和以色列总统身份发表演说。","text2":"第一位同时对英国上下两院发表演说的人士是谁?","label":1} {"text1":"台中工业区(Taichung Industrial Park),为台湾中部的一个大型工业区,隶属经济部工业局管理,位于台中市西郊大肚台地东侧,总面积580公顷,于1980年开发完成,北临东海大学与台湾大道,南邻台中市精密机械科技创新园区。工业区内目前计有921家工厂,从业员工数约40,000人,年产值约新台币3,299亿元。新进驻厂商大都属高科技产业,如:光电、电子与精密机械…等等,已逐渐从传统行业中转向高科技工业迈进。劳动部劳动力发展署中彰投分署与台中世贸中心亦设立于此。东距国道一号台中交流道与南屯交流道约1.5公里以及纵贯铁路台中车站9公里、西距国道三号龙井交流道3公里以及台中港15公里、南距高铁台中站5.5公里、北距台中国际机场8公里,交通非常便利。台中工业区周边往台中市区与国道的联外道路台湾大道、五权西路,一到通勤时间就会严重塞车。近年来大肚台地东侧陆续设立中部科学园区台中基地、台中精密机械园区,使得原本已严重塞车的周边道路(主要为台湾大道)雪上加霜,使得中央与地方政府加速开发交通建设已纾解此区的交通问题,陆续兴建南屯交流道、龙井交流道以及陆续完成跨越筏子溪的朝马路、福科路、中科路与特三号道路大型交通建设,并著手规划市政路的延伸案。","text2":"台中工业区目前有多少家工厂?","label":1} {"text1":"《大自然的魔兽巴奇》,是1984年8月19日,日本电视台《24小时电视「以爱拯救地球」》(第7次)的动画。制作单位是手冢制作。基因改造是21世纪的社会问题之一,而此动画正是批评日本政府于1984年批准重组DNA研究之举。在南美密林的深处,一个叫良介的日本猎人与一个土生土长的男孩捷高正等待著一只令人畏惧的野兽。然而,良介好像对这只野兽相当熟悉,在此时良介回忆起他的童年。当时还是暴走族的良介与摩托车帮会成员在公路狂飙时遇到一名奇怪的女人。一些帮会的成员与她搭讪,结果被「她」弄致重伤,帮会首领见状打算在隐藏处伏击「她」报仇,但是除了良介外所有帮会成员都被撕开。那个「女人」根本不是真正的「人」,「她」是一只基因改造的「猫女」,而且是被儿时的良介拯救和收养了的小猫-巴奇。但当「她」日渐长大,变得能用后腿走动甚而学会写「她」自己的名字和讲话时,巴奇突然离开良介,独自一个直到这次相聚。在良介与巴奇团聚后,他们决定合力找出巴奇诞生的真相,发现巴奇是良介母亲与研究员制作的「产品」,把人和美洲狮的脱氧核糖核酸重组而成的「生物」。其后良与巴奇跟随母亲到南美洲,阻止实验室人员与某国家的官员创造有可能摧毁人类的基因水稻,结果虽然摧毁水稻的基因,但良介的母亲就此牺牲,也令良介误会巴奇杀死自己的母亲,誓言报复。之后五年间,巴奇迅速回复「野性」,被人认为变得极度凶残,并袭击所有接近牠的人(其实袭击人的是另一头杀害捷高父亲的野兽)。当良介用手枪击中牠时,发现巴奇脖子上的小盒藏著母亲的遗书,知道误会了巴奇。第二天早上,巴奇的尸体不见了,但旁边有一道走向遥远山区的脚印。良介此刻希望巴奇能远离人类,渡过余生。","text2":"《大自然的魔兽巴奇》创作动机是什么?","label":1} {"text1":"《万有引力》()是村上真纪的漫画作品。在《你与我》连载。另外还有电视动画、OVA、广播剧CD等作品。第1部漫画全12卷、第2部漫画《万有引力EX》现在连载中,已出版单行卷2本、相关连载还有《新堂家的事情》(原名《新堂家の事情》、连载、7话完结、未出单行本。)跳脱出同志作品给人悲伤和不可告人的既定印象,作品主要是偏向喜剧的方式进行,而男主角愁一和瑛里的关系在本作也是公开的并获得其他配角的支持,这在同类型的作品当中算是非常少见的。主角新堂愁一是「BAD LUCK」的主唱。一天晚上,愁一在公园里与超美型人气小说家由贵瑛里命运地相遇。对于批评他写的歌词的由贵,愁一一直耿耿于怀。","text2":"另一个主角由贵瑛里是做什么的?","label":1} {"text1":"麦可·派翠克·金(,),美国电视节目导演、剧作家和制作人,曾赢得艾美奖。其知名的作品为HBO影集《欲望城市》,第二季后的结尾和开头都是他撰写的。金最近也担任了同名电影的导演。他也替HBO另一影集《喜开二度》(\"The Comeback\")撰写剧本,还有其他影集,如《威尔与格蕾丝》、《Cybill》和《风云女郎》(\"Murphy Brown\")。1980年代,金搬到了纽约,制作栋笃笑(stand-up comedy),并写剧本。最后他搬到了洛杉矶,写了《风云女郎》的剧本,因此获得艾美奖的提名。他是一位出柜的同性恋者,拥有自己的制作公司(Arcade Productions)。","text2":"金的性取向是什么?","label":1} {"text1":"麦可·派翠克·金(,),美国电视节目导演、剧作家和制作人,曾赢得艾美奖。其知名的作品为HBO影集《欲望城市》,第二季后的结尾和开头都是他撰写的。金最近也担任了同名电影的导演。他也替HBO另一影集《喜开二度》(\"The Comeback\")撰写剧本,还有其他影集,如《威尔与格蕾丝》、《Cybill》和《风云女郎》(\"Murphy Brown\")。1980年代,金搬到了纽约,制作栋笃笑(stand-up comedy),并写剧本。最后他搬到了洛杉矶,写了《风云女郎》的剧本,因此获得艾美奖的提名。他是一位出柜的同性恋者,拥有自己的制作公司(Arcade Productions)。","text2":"麦可·派翠克·金是什么人?","label":1} {"text1":"基隆拿国际机场(,)位于加拿大不列颠哥伦比亚省基隆拿市,为一单跑道机场,并提供前往温哥华、西雅图、维多利亚、卡加里、爱民顿、乔治王子城和多伦多的定期航班。季节性航点还包括坎昆、巴亚尔塔港、洛斯卡沃斯和拉斯维加斯。基隆拿国际机场的2014年总旅客量达1,602,899人次,为加拿大国内最繁忙机场之一。基隆拿市选民于1946年以466票对460票通过市政府方案,授权当局以2万加元购入艾利逊湖()以南约320英亩土地来兴建机场。艾利逊机场()于1947年开幕,当时草坪跑道全长。联邦交通部于1950年代出资让跑道改以碎石铺设,而随著加拿大太平洋航空于1958年开展每日以道格拉斯DC-3型号飞机来回温哥华的航班,市政府遂向联邦政府寻求拨款进一步改善跑道。跑道于1960年延长至英尺并改以沥青铺设。当局于1960年代在跑道南端一带增建新客运大楼,而新航空管制塔则于1970年代初落成。机场客运量于1997年突破80万人次。为了应付日益增长的航空交通,市政府从1998年起斥资2千万加元扩建机场设施,当中客运大楼楼面面积倍增至,机场可应付的客流量遂从每小时150人次增至450人次。","text2":"哪个机场是加拿大国内最繁忙的机场之一?","label":1} {"text1":"白翅栖鸭(\"Cairina scutulata\"),又名白翼木鸭,是一种鸭。白翅栖鸭属大型栖鸭,羽毛黑色,脚短,脚爪强而尖,双翼有白色的覆羽。头颈都是白色,满布黑点。雄鸭及雌鸭的体型及颜色相似,但雌鸭较雄鸭细小,色泽亦较暗。牠们日间会在大树的叶丛中休息,晚间会于森林中幽暗隐蔽及布满杂草的小潭和水流缓慢的溪流觅食。牠们在沼泽附近的树孔内筑巢,每次会生6-13颗蛋,孵化期为33至35天。白翅栖鸭是杂食性的,主要吃种子、水生植物、昆虫、蠕虫、软体动物、青蛙及细小的鱼类。白翅栖鸭是与疣鼻栖鸭一同分类在栖鸭属中。不过,根据粒线体DNA的细胞色素b及NADH脱氢酶亚基2的分析,并生物地理学的分布地模式显示,白翅栖鸭与疣鼻栖鸭的相似性正在减少。故此,有指应将牠们分类在单型的属中,是与疣鼻栖鸭无关,但与潜水鸭是近亲。白翅栖鸭以往广泛分布在印度东北部(阿萨姆邦,阿鲁纳恰尔邦)及孟加拉,经东南亚I至爪哇及苏门答腊。于2002年,牠们的数量只有800只,其中200只在老挝、泰国、越南及柬埔寨,150只在苏门答腊,450只在印度、孟加拉及缅甸。白翅鸭生活在茂密的热带常绿森林,河流和沼泽附近。由于失去栖息地、数量稀少及被猎杀,白翅栖鸭被世界自然保护联盟列为濒危物种,且受到《濒危野生动植物种国际贸易公约》附录一的保护。","text2":"白翅栖鸭还叫什么?","label":1} {"text1":"10号球,是一种新兴的花式撞球运动项目,其基本玩法与9号球类似,但多了一颗10号子球,而且击球前必须先指定球、指定袋,所以困难度提高很多,颇具发展潜力。参赛双方比球决定第一局谁先开球,此后各局采轮流开球制。排球时10颗子球紧密排成三角形,1号球在前端,并位于脚点上,10号球在三角形中间,其他各球位置不限。开球前应将母球放置于发球线后,并以球杆撞击母球使其先碰到1号球。开球时若无子球进袋,至少应有4颗子球触碰台边,否则即为犯规。开完球的第一杆可以做push out,要使用必须事先声明。所谓push out是指可以把母球推到任何一个位置,不受先碰到号码最小的球这条规则限制,你想把球打进也可以(但是10号打进要捡起来放回脚点)。若母球落袋一样算犯规,此时对手可以选择打或是不打。若开完球之后,无法打到目标球,选手通常会作push out,然后双方进入防守战。击球前必须先表明要将那一球打进那一袋,即所谓「指定球、指定袋」。若是要进的球很明确,则可省略指定球;但无论何种状况,均不可省略指定袋。至于子球碰撞颗星或与其他球的碰撞方式,均不必说明。每次击球时,母球必须先碰撞台面上号码最小的球,才算合法击球。当击球者将正确的球打进正确的袋后,始得以继续击球;否则换对手击球。若是指定球进错袋,或者进错球,也要换对手击球;但此时对手可以选择打或不打。比赛中先将10号球打进袋者赢得该局击球者若发生以下情形,将换由对手发自由球,亦即可将母球放置于台面上的任何位置再行击球。","text2":"10号球是什么?","label":1} {"text1":"荷兰中央统计局(荷兰文:Centraal Bureau voor de Statistiek, CBS;英文:Statistics Netherlands)又称荷兰统计局,缩写为CBS,成立于1899年,为专门收集荷兰统计资讯的政府部门。隶属于荷兰经济部,在海牙和希伦(Heerlen)设有办公室。自2004年1月3日起,中央统计局成为一个半官方机构。中央统计局所收集的统计资讯有:中央统计局执行计划须由中央统计委员会(Central Commission for Statistics)批准。根据1996年的法令,该独立委员会必须看守中央统计局的公正性、独立性、品质、相关性和连续性。","text2":"荷兰中央统计局什么时候成立的?","label":1} {"text1":"德国BR42型蒸汽机车是德国制造使用的一种货运蒸汽机车,也是所谓“战争机车”(Kriegslokomotiven)之一种。BR42型机车是从1943年开始制造的,是“战争机车”序列中重量排在第二的型号,仅次于最著名的产量冠军BR52型。该型机车适用于路况较好、能够承受更高轴重的线路,在这些线路上BR42型比BR52型更具经济性。该型机车在战争机车系列中的编号为KDL 3型(BR52是KDL1型,BR44是KDL2型)。BR42型机车总共生产了859辆,事实上德国本来也准备大批生产42型机车,但战争的结束也顺便终结了这一潜在的产量神话。当时还有一种以42型为蓝本并加以改进的机车设计,也因为二战的结束而永久地停留在了纸面上。战争结束时,仍有一些BR42型机车没有制造完成,停留在各前德占区的制造台上。战后,西德铁路完成了其中的16辆、波兰完成了126辆,连同遗留下来的3辆德国机车一同被划入了波兰铁路。另76辆未完成机车由奥地利制造完成,但造出后卖给了保加利亚和东德。此外,卢森堡从德国购买了20辆BR42。德国联邦铁路中的BR42型服役至1960年,而东德铁路的42型机车则使用到了1968年。","text2":"BR42型在战争机车系列中的编号是多少?","label":1} {"text1":"无限极广场(),原称维德广场(),是香港港岛中西区的一个甲级写字楼及商场,位于上环德辅道中、干诺道中和林士街交界。无限极广场由伍振民建筑师事务所(香港)有限公司设计,原由地铁公司及恒隆集团持有,乃1980年代恒隆与港铁公司合作发展的港岛线沿线上盖物业之一,至1987年入伙前出售予庄启程创办的维德集团,招租期间批给仲量行作独家租务代理。大厦原址前身为恒星戏院(旧称新世界戏院)及船政署大楼,因兴建上环站而拆卸,于1988年初重建完成。大厦楼高34层,包括30层甲级写字楼,著名租户为英国航空公司,基座4层为零售商铺,现在租户包括香港联合交易所、潮江春、Cedele餐厅、龙岛朱古力、Watson's Wine、Olivers Supersandwiches、康泰旅行社及大家乐等。2003年6月,摩根士丹利以8.4亿港元向维德集团购入当时的维德广场,再于2006年3月以26亿港元售给麦格理。2010年7月李锦记以43.475亿元向麦格理购入维德广场,并在2010年12月17日改名为「无限极广场」。2008年1月9日,明报报导,香港警务处商业罪案调查科讹骗组2B队拘捕五人,他们涉嫌于上环维德广场,冒充外国投资银行职员,以高投资回报为卖点,诈骗市民投资者房地产和证券期票。","text2":"设计无限极广场的是哪一家公司?","label":1} {"text1":"2020年宏愿()是由马来西亚第四任首相马哈迪于1991年第六个大马计划的会议上提出的政治方针,以“在2020年成为先进国”作为国家的奋斗目标。挑战1:共建一个拥有共同价值观与目标的团结马来西亚,马来西亚是一个和平的国家,不分地域与种族差异和睦共处,塑造一个忠于国家、肯于奉献的“马来西亚民族”。挑战2:创建一个自由解放、稳定与进步的马来西亚社会,国人拥有自力更生的自信心,对一切成就怀有优越感,并能够勇于面对任何困境。马来西亚社群将凭着自我努力赢取肯定,能充分了解自己的潜能所在,不会轻易屈服,并得到其他国家人民的敬仰。挑战3:培育和发展一个成熟民主的社会,打造一个增进相互了解,面向社会大众的马来西亚民主模式,使之成为其他发展中国家学习的榜样。挑战4:打造一个具备伦理与道德修养的社会,对宗教价值观与精神坚贞不移,并秉持最高道德观念。挑战5:创建一个开放与包容的社会,马来西亚各族能够自由奉行各自风俗习惯、文化与宗教信仰,并且对国家产生归属感。挑战6:创建一个科技进步的社会,一个具有创新和前瞻性的社群,不只成为科技的使用者,也是未来科技进步的贡献者。挑战7:创建一个富有爱心的社会与关怀文化,重视社会大体更胜于个人利益,人民的福祉不在围绕于国家或个人,而是在一个强而柔韧的家庭体系。挑战8:确保一个经济公正的社会,国家财富将更加公平与合理地分配,且所有投身经济发展者将获享共同的果实。挑战9:创建一个繁荣昌盛的社会,一个充满竞争、生气、活跃与强韧的经济体。","text2":"什么是2020年宏愿?","label":1} {"text1":"泰族()是越南的54个民族之一,又译傣族,人口132万8725(1999年统计)。是越南第二大少数民族,人数仅次于岱依族。主要分布在奠边省、莱州、山罗、和平、清化和乂安等省。越南的泰族和泰国的泰族、老挝的佬族、缅甸的掸族、中国的傣族都有密切的关系(他们认为祖先是来自西双版纳)。越南泰族的语言和前面提到的几个民族的语言都属于侗台语族台语支西南语群。他们不同于其他泰族,他们信仰佛教的比例较低。泰族在九世纪时从云南沿红河来到今天的越南这一带,因不愿受到越南人(京族)的压榨,被迫迁到华潘省一带,但后来这些地方又被越南人(京族)占领,泰族因而变成越南的少数民族。他们古代叫牛吼蛮。越南的泰族按照服饰的颜色分为以下几个部族:","text2":"越南泰族与其他泰族有何不同?","label":1} {"text1":",是1972年香港邵氏公司出品,由张彻及鲍学礼联合执导,陈观泰主演之武打电影。影片取材于清末拳师马永贞之事迹,并将年代向后推移自民国年间。而自此片起不但掀起黑帮片热潮,更开创了上海滩争霸的戏路,后以此为题材之影视作品更层出不穷。1990年代影星金城武重拍此片再演马永贞。山东青年马永贞(陈观泰饰)自幼练得一身好功夫,为谋求生活,只身来到十里洋场的大上海打拼。由社会低下阶层的苦力做起,因缘巧合结识黑帮头子谭四(姜大卫饰),两人惺惺相惜,马永贞更向谭四看齐,暗誓必要获得如其一般之地位。后马永贞于一壶春茶馆打去上海另一黑帮老大杨双(姜南饰)手下恶棍,夺得一壶春一带之地盘;另外又在杨双所招之擂台上打倒来自俄国的大力士。自此马永贞声名大噪,意气风发,为扩张地盘更夺去杨双一处赌馆,但此举却也使杨双对其怀恨在心。为夺取谭四地盘及利益,杨双与谭四手下勾结,使计将谭四杀害,其后杨双便将目标放诸马永贞。而马永贞在得知谭四为杨双杀害后,亦决心替谭四报仇,两人遂约于青莲阁相见。岂知杨双早命手下暗伏于青莲阁,马永贞不知遭袭,身负重伤,但凭一股劲力,终将杨双一伙尽数败于拳下,然而马永贞却也气衰力竭,倒于血泊之中。","text2":"1990年代由哪个影星再次饰演马永贞?","label":1} {"text1":"乔治·温斯顿(,),是一位美国钢琴家兼作曲家。乔治·温斯顿出生在密歇根州,并在蒙大拿州和密西西比州长大,他毕业于佛罗里达州戴兰德市的斯特森大学,居住在加利福尼亚州的旧金山 。他的大多数作品因为能使听众感到大自然的风景,并且反映了四季的精髓,给听众闲适、轻松、怡然、清新的感觉。乔治·温斯顿被誉为“新世纪音乐之父”,而他本人却回避这个说法,乔治·温斯顿称自己的音乐为“乡村田园钢琴曲”。乔治·温斯顿于1972年录制了自己的第一张专辑为《民谣与布鲁斯》(Ballads and Blues),当时并没有得到多大的反响。而直到1979年,威廉·阿克曼(William Ackerman)与他谈起为他的新唱片公司录制专辑的事情。起初,乔治·温斯顿只是用夏威夷吉他弹奏了一些简单的音乐作品,之后有为电台的夜间节目做背景音乐,这样也就成为了乔治·温斯顿的专辑《秋日》(Autumn)。在音乐中,他将秋日描述成为万圣节的颂歌。这张专辑《秋日》后来被听众大受好评。之后他的专辑《十二月》(December)和《辞冬》(Winter into Spring)都获得了白金唱片的称号,并在1996年的的格莱美奖中获得最佳新世纪音乐专辑。由于他对史努比花生漫画的喜爱,他在1996年制作了音乐专辑《Linus & Lucy-The Music of Vince Guaraldi》,这张专辑是献给格拉第(Vince Guaraldi)为花生漫画所做的音乐。在2002年他将19世纪60年代门户合唱团(The Doors)的音乐制作成专辑《Night Divides the Day - The Music of the Doors》,与其不同的就是音乐使用的是钢琴的独奏,并且音乐更加的恬静怡然。在乔治的音乐会上,乔治本人并不是一个特别爱表现自己的人,而是衣着十分简单,法兰绒衬衫和牛仔裤。秃顶、一脸的胡子,根本不像是在音乐会为听众表演的演员,反而经常被观众认为是为钢琴调音的技术人员。除了作曲和演出,在音乐会上乔治也会给观众演奏口琴和夏威夷吉他。他演奏这些乐器主要在音乐会上,并非在专辑中。在专辑《Remembrance - A Memorial Benefit》可以听得到这两种乐器的使用。他为自己的唱片公司Dancing Cat Records制作过夏威夷吉他的录音作品。","text2":"乔治·温斯顿是在哪里长大的?","label":1} {"text1":"二氧化三碳是一个无色刺激性气体,化学式为CO,分子中含有四个累积双键。它与CO、CO、CO等其他碳氧化物有重要联系。1873年,Brodie通过对一氧化碳放电,首次制得了二氧化三碳,Marcellin Berthelot创造了低氧化碳(carbon suboxide)这个名称, 而后来Otto Diels表示,更贴近有机化学的名称,如二羰基甲烷(dicarbonyl methane)与二氧代丙二烯(dioxallene)也是正确的名称。二氧化三碳可以通过加热干燥的五氧化二磷(分子式为PO)和丙二酸或丙二酸酯衍生物的混合物来制备。 所以二氧化三碳也可视为丙二酸的酸酐,为丙二酸酐以外的另一种丙二酸的酸酐,即丙二酸的\"第二酸酐\"。此外,丙二酸分子间脱一分子水生成的酸酐也是存在的。其他有关二氧化三碳的合成和反应信息可在Reyerson于1930年出版的综述中找到。二氧化三碳自发聚合生成红、黄或黑色的固体,结构类似于2-吡喃酮。1969年时,曾一度认为火星表面的颜色是由存在二氧化三碳导致的,但后来海盗号的研究推翻了这个说法。二氧化三碳可以作为丙二酸的制剂,同时它还可以增强皮革染料与皮革的亲和力。","text2":"关于火星表面的颜色问题,是谁推翻了关于二氧化三碳的说法?","label":1} {"text1":"美国教育部(United States Department of Education,缩写:ED)为美国内阁层级的联邦政府部门,由《教育部门组织条例》(Department of Education Organization Act, Public Law 96-88)成立,由吉米·卡特总统于1979年10月17日签署生效,并于次年5月4日开始运行。《教育部门组织条例》将原先的“美国卫生、教育及福利部”重组为“美国教育部”以及“美国卫生及公共服务部”,教育部由美国教育部长管理。教育部是目前最小的内阁层级联邦部门,只有约5000名雇员。教育部正式的英文缩写是ED,而不是DOE(美国能源部英文缩写)。不同于其他国家的教育部,美国教育部不干涉课程标准的设置(现今的「有教无类法案」,或称「不让任何孩子落后法案」,是个例外),地方及州政府掌管教育权并决定大部分课程。教育部主要的职权在于编列联邦补助方案,以及执行联邦关于民权及隐私的教育法案。","text2":"美国教育部是哪个层次的政府部门?","label":1} {"text1":"周以真(英文名Jeannette M. Wing,),美国计算机科学家。卡内基梅隆大学教授。美国国家自然基金会计算与信息科学工程部助理部长。ACM和IEEE会士。她的主要研究领域是形式化方法、可信计算、分布式系统、编程语言等。1993年她与图灵奖得主芭芭拉·利斯科夫合作,提出了著名的Liskov代换原则,是面向对象基本原则之一。1979年6月在麻省理工学院获得学士和硕士学位,导师中有图灵奖得主Ronald Rivest。1983年获得该校的博士学位。1983—1985年,在南加州大学任助理教授。1985年起,任教于卡内基梅隆大学。2004—2007年间,曾担任该校计算机系主任。2006年3月,在美国计算机权威期刊《Communications of the ACM》杂志上给出,并定义的计算思维(Computational Thinking)。认为:计算思维是运用计算机科学的基础概念进行问题求解、系统设计、以及人类行为理解等涵盖计算机科学之广度的一系列思维活动。周以真教授曾担任卡内基—梅隆大学计算机学院院长、美国国家科学院计算机科学与通讯部门主席、美国国防部高级研究计划署(DARPA)信息科学与技术委员会、美国国家科学基金科学顾问委员会以及Sloan 研究基金程序委员会的成员。2007年她出任美国国家科学基金会(NSF)计算机与信息科学与工程部(CISE)副部长,负责管理NSF在信息科学和计算机研究领域的基金。CISE每年的预算高达5.27亿美元,占整个美国联邦政府对计算机科学研究资助经费的86%。","text2":"CISE每年的预算高达多少美元?","label":1} {"text1":"中华人民共和国第十一届全运会跆拳道比赛于2009年9月9日至12日在枣庄滕州体育中心体育馆举行。赛事设有8个小项,每天进行两场决赛,共颁发8面金牌。所有运动员均要通过预赛取得参与十一运会的资格。为期4天的赛事中,两位奥运金牌得主—罗微与吴静钰最终均赢得冠军,至于在北京奥运中取得唯一一面男子组铜牌的朱国则在淘汰赛出局,无缘奖牌。总括而言,是届全运会之跆拳道项目奖牌分布极为平均,在8个小项中,共有7个代表团赢得金牌。本届全运会的跆拳道项目的预赛分为两个阶段。在首阶段中,每个级别的首12位运动员可取得参赛资格。而在第二阶段中,晋身半决赛的运动员可取得最后的名额。另外,赛事又设立如有运动员于预赛期间代表国家队出战世界锦标赛并取得前三名,均可取得参赛资格。本届全运会的跆拳道赛事采用国际跆拳道联会于2009年4月新修订的规则,包括比赛场地的缩少、击中对方头部由可取得两分升至三分等。另外,为求更加公平,本次赛事请来了8位分别来自加拿大、日本等的国际裁判执法,是全运会中首次以海外裁判负责执法跆拳道赛事。","text2":"中华人民共和国第十一届全运会跆拳道比赛的举办地点是哪里?","label":1} {"text1":"海上共和国(意大利语:)是中世纪意大利和达尔马提亚沿海地区一批繁荣的城市国家的统称。传统上,其定义特别是4个意大利城市:阿马尔菲共和国、比萨共和国、热那亚共和国和威尼斯共和国,它们的徽章出现在意大利海军的军旗上。这些城市在军事和商业上都彼此竞争,从第10到13世纪,它们建立起舰队,一方面用来保护自己,另一方面用来支持跨越地中海的广泛的贸易网络,并且在十字军东征中发挥重要作用。这些共和国在竞争中壮大自身,从事更换盟友和战争。上述四个是典型的意大利海上共和国,它们排列的顺序也反映了各自占优势的时间顺序。不过,意大利其他城镇也曾经拥有海上共和国的历史,尽管在历史上不是那么突出。它们是:加埃塔、安科纳、诺利(热那亚以西不远处独立的小城),以及达尔马提亚的杜布罗夫尼克(意大利语:拉古萨) 。在845年的奥斯提亚战役中,加埃塔提供了一支舰队,帮助教宗联盟击败了撒拉森人。这些海上共和国都是独立的城邦共和国,拥有自己的政府、货币和舰队。它们多数起源于曾经属于拜占庭帝国的疆域(主要的例外是热那亚和比萨)。所有这些城市在其保持独立的时期,都拥有相似的(但不完全相同)政府制度,商人阶层在其中获取了相当大的权力。这些海上共和国都深深地卷入了十字军东征,为其提供资助,但是更多的是从战争中得到政治利益和贸易机会。第四次十字军,名义上为了“解放”耶路撒冷,但是实际结果却是使威尼斯征服了扎达尔和君士坦丁堡。每个海上共和国都曾经统治过不同的海外领地,其中包括许多地中海岛屿,特别是撒丁岛和科西嘉岛 ,亚得里亚海,爱琴海和黑海(克里米亚)沿岸土地,和近东和北非的商业殖民地。在这些海上共和国中,最为重要的是热那亚共和国和威尼斯共和国。威尼斯在其领土最大时扩张到几乎整个的意大利东北部,西面达到距离米兰数公里处,而在东面占据了达尔马提亚、希腊伯罗奔尼撒的一部份,以及克里特和塞浦路斯两大岛屿。威尼斯的头号竞争对手是热那亚。威尼斯控制地中海东部的贸易,而热那亚不仅出现在地中海东部,而且垄断了地中海西部的贸易,其领土最大时扩张到科西嘉、撒丁岛、克里米亚和一些爱琴海岛屿。在塞奥芝亚战役中被威尼斯击败后,热那亚显著衰退。","text2":"第10到13世纪,海上共和国建立舰队的原因是什么?","label":1} {"text1":"宋云(生卒年不详),北魏时期敦煌人,曾和惠生(亦作慧生)同赴西域求经。为侍应太后的主衣子统。撰有《魏国以西十一国事》、《家记》等书。北魏明帝神龟元年(518年)十一月,受胡太后之命,与崇立寺沙门惠生、法力等出访天竺。至洛阳起,经吐谷浑、鄯善、左末(今新疆且末)、捍(媲摩,Phema)、于阗等地,进入西域。宋云等谒见哒王之后,经波斯、赊靡国、钵卢勒国,于神龟二年(519年)入乌场国,晋见乌场国王,为国王讲述孔子、老子、庄子学说。此后,宋云、惠生在天竺广礼佛迹,正光元年四月访问干陀罗、那迦罗阿国等地。正光三年(522年),携大乘经论一百七十部返回洛阳,宅居洛阳闻义里。","text2":"宋云是什么人?","label":1} {"text1":"鼻头角灯塔,位于中华民国新北市瑞芳区之鼻头角,西元1897年建造完成 ,为日治时期日本政府兴建的六角形铁塔,二战损毁,1971年改建为圆塔。灯塔乃海上航行重要的指标,而台湾最早的灯塔是澎湖西屿的石造灯塔, 清朝乾隆年间兴建 。日本统治台湾初期, 以生铁建造灯塔, 常在日本铸造好之后, 再运到台湾组立, 如富基角灯塔及鼻头角灯塔。 日治后期灯塔, 改用钢筋混凝土, 可塑性高造型丰富, 基座或与办公室合一, 如新北市的三貂角灯塔。鼻头角灯塔建于1896年(日本明治29年),西元1897年建造完成开始点火。1906年(日本明治39年)由于塔基岩盘出现裂痕,将原塔迁移至30公尺外的现址。旧塔原为日治时代兴建之六角形铁塔,二次大战遭美军攻击严重损毁,1971年改筑为钢筋混凝土之白色圆塔。","text2":"鼻头角灯塔是什么时候兴建的?","label":1} {"text1":"五马街是中国浙江省温州市最著名的商业街,位于该市中心地段,鹿城区五马街道,温州古城区的南部,东起解放南路,西接蝉街,全长424米。沿街各类商店鳞次栉比。五马街是温州传统的商业中心。1934年改称中山路,1949年以后又改回五马街。“文革”中曾改名“红卫路”,1982年又恢复五马街。1999年,被命名为国家级“百城万店无假货”示范街。2000年到2001年,五马街改建,保留了原有的中西合璧的建筑风格,以及第一百货商店、五味和副食品商场、金三益绸布店、老香山药店、温州酒家等一批老字号。魏晋时期的大书法家王羲之在永嘉郡(温州的古名)做郡守时,常常驾着五马并行的马车出行。五马街由此得名。明嘉靖《温州府志》:“王羲之,山阴人,为永嘉郡守,出乘五马,老幼仰慕,为立五马坊”。清光绪《永嘉县志》亦载:“王羲之守永嘉,庭列五马,绣鞍金勒,出则控之,故今有五马坊”。","text2":"五马街曾用过什么名字?","label":1} {"text1":"轧压均质装甲(英文:Rolled Homogeneous Armour;德文:gewalzte homogene Panzerung),缩写成「RHA」,又简称为均质装甲是为钢板之理论基础象征,RHA做为一种基数,用于对照军事装甲车之效能。直至第二次世界大战结束,大部份坦克的装甲形式以及其它装甲车辆均覆盖著钢板。若要增加车辆的防护,即意味著要增加更厚的钢板,而厚重的装甲相对的也减低了其机动性。从那时以来,一些形式的装甲开始发展出混入一些空气、陶瓷或是贫铀(depleted uranium,简称DU)于钢质之中。在现代化武器攻击下,更大的撞击力量与更高的温度切割冲击,传统的轧压均质装甲已不敷使用,遂由更高等级的装甲取代,而测量单位也跟著有所改变。更新的装甲估量数被称做「等效轧压均质装甲(Rolled Homogeneous Armour equivalency,简称RHAe),其做为一种粗略估计的单位,或是被炮弹穿透的能力,或是其装甲形式的保护能力,这些不一定是使用钢材。但是,因为各装甲间形式、质量、金属以及分工组装等不同,RHAe是否能有效的比对不同形式装甲,仍有许多争议性。当前美国陆军所使用,RHA钢板产品有军用标准MIL-A 12560,而最新标准则使用有MIL-A 46100。这些非常相似,但实际上在美国的钢板等级中,不同于标准高强度钢材合金4340,虽然其械制造与合金非常的相似。","text2":"轧压均质装甲的德文是什么?","label":1} {"text1":"《魔鬼出没的世界——科学,照亮黑暗的蜡烛》(,台湾版书名《魔鬼盘据的世界-萨根谈UFO、占星与灵异》)是一本由美国天文学家卡尔·萨根与妻子安·德鲁彦合写成的书。本书共有二十五章,其中第二十一章《自由之路》由萨根与德鲁彦合写,其他均为萨根独自完成。这本书从一个天文学家的角度,对当今社会盛行的伪科学做出了种种驳斥,并向公众宣扬科学方法。萨根在本书中,反复强调需要用怀疑的眼光去看待社会中的各种事物,凡事都要相信证据,而不是自己内心的感觉。作者在本书中“车库中的飞龙”一节里举了一个这样的例子:一个人宣称他的车库里住著一条无法被感知到的飞龙,时不时吐出没有热度的火焰。这条龙不能被看见,不发出任何声音,也不会留下任何其他形式的痕迹。正因如此,人们根本没有任何理由相信有这么一条龙存在。","text2":"《魔鬼出没的世界》一书作者是谁?","label":1} {"text1":"翼善冠,又称翼蝉冠。是冠的一种,是明代皇帝、藩王、亲王、郡王等所着之首服,也是朝鲜王朝国王及王世子、越南皇帝、明郑延平王及王世子与宁靖王、琉球国国王的首服。翼善冠发展自唐代的幞头,唐代男子兴头戴软脚幞头身穿圆领袍服,幞头专门用以缠裹头发及发髻。而唐代的幞头发展到宋代出现了官员公服的展脚幞头,外形方正。到了明代,朱元璋极力恢复唐宋时期汉人的服制。明代官员所着公服便沿袭宋代。而常服则是圆领袍搭配展翅的幞头,俗称乌纱帽,帽胎或竹或木,或纸,两片展开的帽翅则是薄黑纱。因而君与王所戴的则是两翅向上折的,故称“翼善冠”。明代的皇帝和如朝鲜国王等藩王所戴的翼善冠在外形的尺寸和颜色上有所区别,不得僭越。","text2":"翼善冠因什么而得名 ?","label":1} {"text1":"水教堂(Wasserkirche)是苏黎世利马特河中的小岛上的一座教堂,位于苏黎世的2座中世纪大教堂,苏黎世大教堂和苏黎世圣母大教堂之间。该地点在古代曾是异教集会之所,中心的那块石头现在教堂的地窖内,根据中世纪传说,这是圣徒Felix 和 Regula被处死的地方。第一座教堂建于10世纪,在各个阶段进行了重建,在1486年完成的重建达到了顶峰。在宗教改革过程中,水教堂被确定为偶像崇拜的地方,被世俗化,1634年成为苏黎世第一所公共图书馆,成为学习的中心,在19世纪极大地促成了苏黎世大学的创建。1839年,该岛与利马特河右岸连接,修筑了Limmatquai。1917年图书馆并入中央图书馆(Zentralbibliothek),建筑用作谷仓一段时间后,1940年开始重建和考古发","text2":"水教堂位于什么地方?","label":1} {"text1":"安德海(),一说名安得海,直隶南皮县(今河北省南皮县)人。宦官,清朝咸丰皇帝、慈禧太后的宠臣。安德海出生年份有多种说法:1837年,1842年,1844年。自幼自阉入宫,成为咸丰帝身边御前太监,人称「小安子」,颇知书,可以读《论语》、《孟子》。咸丰十一年(1861年)发生的辛酉政变中,安德海把遗诏密报慈禧太后那拉氏,深得慈禧太后的欢心,升为总管太监。同治八年(1869年)七月,安德海出京办货,乘上两艘太平船从京杭大运河一路南下,“招摇煽惑,声势赫然”,七月二十日,驶入山东境内,抵鲁北古城德州。山东巡抚丁宝桢以顺治帝祖训「宦竖非经差遣,不许擅出皇城」表示:“宦竖私出,非制。且大臣未闻有命,必诈无疑”为由,命泰安县知县何毓福将安德海与其随从逮捕,派人秘密向慈安太后请示,慈安命军机处回文,内称:“该太监擅离远出,并有种种不法情事,若不从严惩办,何以肃宫禁而儆效尤?著丁宝桢迅速派委干员于所属地方将六品蓝翎安姓太监严密查拿,令随从人等指证确实,毋庸审讯即行就地正法,不准任其狡饰。如该太监闻风折回直境,即著曾国藩饬属一体严拿正法。倘有疏纵,惟该督抚是问。其随从人等,有迹近匪类者,并著严拿,分别惩办,毋庸再行请旨。”八月七日,丁宝桢于济南西门外丁字街(今饮虎池街北段)斩首安德海,暴尸三日,随行二十余人,一律处死。安德海死后,李连英取代其地位。","text2":"谁逮捕了安德海?","label":1} {"text1":"beatmania是一款DJ模拟节奏打击游戏,由Konami公司于1997年发布第一作。很大程度上促成音乐游戏的发展。随着续作的发布,不仅有街机的续作,而且还转移到家用机和其他掌机,实现了100万套的销售。游戏的控制器由下方三颗白键、上方两颗黑键(模拟效果器按钮)和按键右方一个转盘(模拟打碟)组成,在街机机台上共有两组,可以进行双人游戏。屏幕上每个按键以及转盘各自对应一条“轨道”,玩家需要在随着音乐节奏沿轨道下落的音符记号与判定线重合时按下相应的按键或转动转盘以完成音乐的演奏,根据玩家按键或打碟的时机正确与否会产生JUST GREAT、GREAT、GOOD、BAD、POOR的判定,由此获得不同的分数。GOOD以上判定会令画面中央的能量槽增加,反之BAD和POOR判定则会减少,当一曲结束后能量槽到达指定的量则为过关,可以继续下一首歌曲的游戏,否则游戏结束。街机(业务用)作品","text2":"玩家怎样得到分数?","label":1} {"text1":"长吻仰口鲾(学名:\"Secutor insidiator\"),又称静仰口鲾,俗名金钱仔,为辐鳍鱼纲鲈形目鲾科的其中一个种。本鱼分布于印度太平洋区,包括东非、马达加斯加、模里西斯、塞席尔群岛、亚丁湾、马尔地夫、印度、斯里兰卡、孟加拉湾、安达曼海、泰国、越南、马来西亚、柬埔寨、台湾、中国沿海、日本、韩国、菲律宾、印尼、澳洲、新几内亚、马里亚纳群岛、马绍尔群岛、帛琉、密克罗尼西亚、所罗门群岛、诺鲁、新喀里多尼亚、法属玻里尼西亚、夏威夷群岛、吉里巴斯、萨摩亚群岛、东加、吐瓦鲁、加利福尼亚湾、加拉巴哥群岛、厄瓜多、巴拿马等海域。水深5至150公尺。本鱼与仰口鲾相似,但其体长卵形,标准体长为体高的2至3倍。头部无特殊斑点,体侧上方有蓝色小点,形成若干横带,体侧下方有小暗点,背鳍末端黑色。背鳍硬棘8枚、软条16枚;臀鳍硬棘3枚、软条14枚。体长可达15公分。本鱼在岸边至近海之沙泥底海床上皆可发现其踪影。常成群觅食小型无脊椎动物。小型食用鱼,较不具经济价值,多做下杂鱼处理。","text2":"长吻仰口鲾体长与体高有什么关系?","label":1} {"text1":"房屋检验师(Home inspector),简称验屋师或验房师,为具有专业工程素养及资深工程经验之房屋检验人员,提供购屋者在交屋验收阶段,标准化的专业验收流程及相关咨询服务。此举系于1976年率先由美国验屋师协会在北美推动,随后推广到世界各国。房屋检验师主要工作为协助买卖双方于交屋阶段,针对屋况检查室内空间、厨具及卫浴设备、公共设施、建材安全、给水及排水、机电设备等。近年来因交屋而产生之纠纷甚多,房屋检验师以专业角度让购屋者了解现有的屋况,如施工品质或是否使用违法建材等。随著消费者意识的抬头,有愈来愈多的购屋者更加重视房屋的品质,进而选择专业的房屋检验师代为验屋,以避免忽略房屋的瑕疵。专业的房屋检验师应具有资深营建经验及公共工程品管工程师且经由专业单位认证之资格,并可在营造施工中期、施工后期交屋过程中进行验证房屋的施工品质与公共设备安全验证,诸如电器等使用规范、消防安全、公共空间等是否按法规设置。","text2":"房屋检验师此举系于哪一年率先由美国验屋师协会在北美推动?","label":1} {"text1":"芝加哥哥伦布纪念博览会(World's Columbian Exposition),亦称芝加哥世界博览会,简称芝加哥博览会,是于1893年5月1日至10月3日在美国芝加哥举办的世界博览会,以纪念哥伦布发现新大陆400周年。共有19国参加,2750万人参观。因1871年的芝加哥大火而复兴重建的芝加哥市,于1890年由美国政府指定为博覧会的主办都市。在密西根湖畔,花了近3年兴建67万坪以上的公园,在展示的Court of Honor地区以及娱乐的Midway地区等广大会场建造了约200个建筑物。主会场Court of Honor地区,有美术馆、联邦政府馆、园艺馆、工艺馆、农业馆、机械馆、管理栋等等,为展示美国的繁荣建造了豪华的新古典主义建筑。以美国为中心,展示了各国来的工艺、美术、机械等等主题。除了以红砖造涂漆的美术馆外,其余建筑统一为白色,因夸示白人文明,而被称为「White City」。会场有由小乔治·华盛顿·盖尔·费里斯设计的机械式大摩天轮,直径75.5m,可乘坐2160人。大井浩二,《ホワイト・シティの幻影―シカゴ万国博覧会とアメリカ的想像力》,研究社出版,1993\/12。","text2":"芝加哥哥伦布纪念博览会为了纪念谁发现新大陆400周年?","label":1} {"text1":"巴林国际机场(,),是巴林王国全国唯一的民用机场,位于该国首都麦纳麦东北方向约7公里处的穆哈拉格岛上。它是海湾航空主要的枢纽机场,也是海湾地区第一座国际机场。巴林国际机场与阿布扎比国际机场、素万那普国际机场、多哈国际机场、迪拜国际机场、奥利弗·坦博国际机场、科威特国际机场、伦敦希思罗机场和马德里-巴拉哈斯机场一起被公司评为三星级机场。第一架预定抵达巴林的商务班机是1932年从伦敦飞往德里的名为“汉尼拔”的汉德利·佩季·H·P·42型飞机。这架飞机只搭载了24名乘客,从伦敦出发后用了好几天才抵达目的地,时速100英里。这条航线固定后,巴林建起了海湾地区第一座国际机场。","text2":"巴林国际机场位于哪个地方?","label":1} {"text1":"卡尔·菲利普王子(Prince Carl Philip, Carl Philip Edmund Bertil Bernadotte,)是瑞典国王卡尔十六世·古斯塔夫的第二个子女,也是唯一的一个儿子。出生时,由于瑞典宪法尚未修改,卡尔·菲利普王子得到王位第一继承权。同年11月修改宪法后,他的姐姐维多利亚取代他成为瑞典王储。外甥女艾丝黛拉公主是第二继承人。妹妹则是马德琳公主。王子在2010年开始和模特儿苏菲亚·赫尔克维斯交往,王室在同年八月证实了他们的关系。2014年6月27日,王室宣布王子和赫尔克维斯订婚的消息。10月23日,王室宣布两人将在2015年6月13日举行婚礼2016年4月19日,长子亚历山大王子在丹德吕德市丹德吕德医院出生。。最终在斯德哥尔摩完成婚礼。","text2":"宪法修改后菲利普王子的王位继承权被谁取代?","label":1} {"text1":"槻木车站()是一由东日本旅客铁道(JR东日本)与阿武隈急行所共用的铁路车站,位于日本宫城县柴田郡柴田町槻木新町一丁目。槻木车站是JR东日本东北本线与第三部门铁路线阿武隈急行线(原日本国铁丸森线)的交会车站,也是后者的终点站。由两家铁路公司所共用的槻木车站是由隔邻的船冈车站所代管,票务上由JR东日本委托子公司东北综合服务()经营,管辖上则属仙台分社的范围。除了现有的两家铁路公司之外,在1899年到1929年之间,曾一度是一家名为角田马车轨道(后来在蒸汽动力化之后改名为角田轨道)的路线起点站。除了客运业务之外,在1997年之前日本货物铁道(JR货物)曾在槻木设有货运车站,但在废站之前货运服务早已停止运作多年。侧式月台1面1线与岛式月台1面2线,合计2面3线的地面车站。","text2":"槻木车站位于日本哪里?","label":1} {"text1":"沃夫数(也称为国际黑子数、相对黑子数或苏黎世数)是用于测量太阳表面的太阳黑子和群组数目的数值。这个计算黑子数目的想法鲁道夫·沃夫于1849年在瑞士的苏黎世提出的,因此以他的名字(或地名)做为名称,使用黑子数和它们的群数组合,以补偿小黑子群对观测数量的变异。这个数值已经被研究人员记录和制成表格累积了近200年,并且发现黑子的活动每9.5至11年附近到达它的极大值(注:依据SIDC最近300年的数据和使用FFT作用于数据得到的最大值平均周期是10.4883年。),这个周期在1843年首度被施瓦贝注意到。相对数使用下列的公式来计算(每天收集一次黑子活动资料数值):此处\"R\"是黑子相对数,\"s\"是单独计算的黑子数目,\"g\"是黑子的群数,还有\"k\"是随地点和仪器改变的因素(也称为\"天文台因素\")。","text2":"这个计算黑子数目的想法是谁提出的?","label":1} {"text1":"波音720是波音公司所开发的一款四引擎窄体客机,衍生自波音707,于1959年11月23日首飞,1960年7月5日投入服务。中华民国的总统专机之一中美号即为此型客机。1957年7月,波音宣布将会在707-120的基础上开发新的707型号 ,新型号飞机所需的跑道长度较短。波音720最初的设计编号为707-020,当时联合航空对波音707-020产生极大兴趣,但联航之前已经选用道格拉斯DC-8而舍弃波音707,为免因重新选用波音707而带来负面形象,波音公司因此对飞机更名为波音720。波音公司在720的机翼上作出了改动,加装了翼套,减低机翼的展弦比,巡航速度提升0.02马赫,内侧前缘缝翼移到外翼,并装上克鲁格襟翼。发动机方面,波音720采用推力达12,500磅的普惠JT3C-7。机身方面,波音720减少了五片机架,使长度比707短2.54米,最大载客量为149人。此外最大起飞重量亦降低了。在发展过程中,波音曾把此计划称为717-020,与军用的KC-135同一编号,但MD-95最终才采用波音717名字。由于波音720与707的结构十分相似,因此无须生产原型机,相异的系统都在波音367-80测试。首飞于1959年11月23日进行,1960年6月30日取得适航认证,1960年7月5日交付给联合航空。美国航空的720亦在同年7月31日投入服务。波音720共生产65架。波音720B是波音720的子型号,采用JT3D发动机,推力达17,000磅,且耗油量较低;最大起飞重量增加至234,000磅。首飞于1960年10月6日进行,1961年3月交付给美国航空。波音720B共生产了89架,另外10架由720改装。整个波音720计划虽然只生产了154架,但研发成本低廉使它成为成功的机型。波音720最后一次飞行是在2010年9月29日,由加拿大普惠公司负责,之后普惠公司以波音747SP取代此飞机。来源:Boeing","text2":"波音720是什么?","label":1} {"text1":"B2,香港二人女子演唱团体,成员分别是谭嘉荃(Amy)及郭可颖(Sandy,郭可盈堂妹)。B2参加新城电台举办的「一级班少女歌唱大募集」分别获得亚军及季军,获得BMA唱片公司合约。2000年正式出道,她们的曲风以快歌为主。于2000年10月推出首张EP《B2 Attack》。她们于2000年「新城劲爆颁奖礼」获得最受欢迎组合的第三名。及后于2001年推出了《B2同名专辑》。她们的主打歌《型男索女》的歌词被认为粗俗,而被各大电台及电视台禁播。2002年1月,她们推出了首张大碟《娥世代》。及后她们于同年的6月在台湾推出了《娥世代》的台湾版。2003年,宣布退出乐坛。Amy于无线电视担任兼职,及为节目《娱乐大搜查》担任记者及写稿工作,Sandy则到外国进修。2006年,Amy离开了无线电视,转投「香港宽频电视」任主持。直到2012年,Amy加盟汉洋主理的自由行娱乐,正式重返乐坛。","text2":"B2是何时推出首张EP的?","label":1} {"text1":"《伊甸园之东》(,)是韩国MBC于2008年8月25日起播放的月火连续剧,由退伍之后接拍首部电视剧的宋承宪,加上延政勋、李多海、韩智慧、李沇熹、朴海镇等演出。但并没有收录插曲李承哲(이승철)的再没有这样的人(그런 사람 또 없습니다),此曲原为电影最悲伤的故事的插曲。2009年5月26日,持有《-{伊甸园之东}-》、《流星花园》、《我的亿万面包》等三部电视剧台湾版权的八大电视透过中视要求中华电信MOD频道商「台湾互动电视公司」(TITV),自该日起,必须自行以其他电视剧覆盖中视无线台播映此三部电视剧时的节目讯号;台湾互动电视公司宣称,该公司已致函国家通讯传播委员会(NCC),以寻求妥善处理。","text2":"再没有这样的原本是哪部电影的插曲?","label":1} {"text1":"《汉新月刊》(Sino Monthly New Jersey)创立于1991年九月,是美国新泽西州第一份华文月刊。汉新月刊的《汉》有汉族,汉语的意思;《新》则是指新泽西州,在新泽西州发迹,针对当地华人的议题为主要诉求的杂志。创办人李美伦(Meilun(Ivy) Lee)因感慨新泽西华人之众,却没有一份属于自己的刊物,于是创办该月刊,作为华人与华人之间,华人社区与美国主流社会之间联系的桥梁。汉新月刊每月一日发行,主要是读者订阅,也有部份透过华人经营的超商零售,每份1.25美元。目前发行以美国东部各州为主,但订阅户遍及全美国各州。每月出版的内容包括:专题, 社区, 教育, 财经, 休闲, 美食, 专栏, 小说, 散文, 新诗, 工商。该月刊每年并且出版四册地产专刊和一次年历手册。汉新月刊曾获得美国独立媒体协会(Indenpendent Press Association )新闻奖(ippies),包括了2004年获得特别报导的第三名;2005年获得移民报导第一名与深入报导第四名。汉新月刊在华人社区评价颇高,月刊也被许多美国图书馆所陈列与收藏,如普林斯顿大学的东亚图书馆、罗格斯大学的东亚图书馆、新泽西州孟莫斯郡图书馆总馆、东布朗士维克镇公立图书馆、李文斯顿镇公立图书馆……等。汉新月刊于1993年开办汉新文学奖征文比赛,是北美华人文学盛事,目前有短篇小说、散文与新诗三大类,鼓励与成就了不少华人作家,如闻人悦阅、青梅。","text2":"《汉新月刊》的创立人是?","label":1} {"text1":"桂樱(学名:\"\"),原产地在黑海靠近西南亚德一带,以及东南欧洲,从阿尔巴尼亚到保加利亚东部,从土耳其到高加索山脉,伊朗北部,甚至西欧,都有种植。在北美洲也称为“英格兰月桂”。常绿灌木,5-15米高,60厘米宽,树叶呈深绿色,兼任有光泽,10-25厘米宽,4-10厘米长,周围稍微成锯齿状。花蕾在春天出现,在夏天开放,7-15厘米的总状花序包含30-40个花朵,每个花朵1厘米宽。果实为一种的1-2厘米宽的浆果,在秋天变成黑色。桂樱作为观赏植物被全世界各地大规模地栽种,大多数生命力很强。桂樱全株有毒,食用会引起不适 ,果实种子含有糖苷和扁桃苷。桂樱水是由这种植物制造的蒸馏液,含有剧毒的氢氰酸成分,曾被认为有药用价值。","text2":"桂樱的果实是什么样子的?","label":1} {"text1":"捷克爱乐乐团是一支捷克交响乐团。2006年一次由法国杂志《Le Monde de la Musique》的调查中,它被评为欧洲十大乐团第九位。乐团成立于1896年。它的原始成员来自布拉格国家歌剧院。1905年乐团独立,1945年被国有化。1946年,乐团出席第一届布拉格之春国际音乐节以庆祝其成立五十周年纪念。乐团的第一场音乐会由捷克音乐家德沃夏克指挥。而它与其他捷克音乐家的关系也非常密切,如斯美塔纳,扬那切克和马替奴。两战期间,乐团名声鹊起,当时领导乐团的是指挥家瓦茨拉夫·塔利赫()。随后是有名的捷克指挥家拉法埃尔·库贝里克(任期为1942-1948),卡罗尔·安切尔(Karel Ančerl,任期为1950-1968)和瓦茨拉夫·纽曼(任期为1968-1989)。吉里·贝洛拉维克曾两次出任首席指挥。乐团曾和很多有名的指挥家合作,如古斯塔夫·马勒,理查德·施特劳斯,保罗·欣德米特,赫伯特·冯·卡拉扬,卡尔·伯姆和祖宾·梅塔。","text2":"乐团与其他哪些捷克音乐家关系密切?","label":1} {"text1":"《球阳》是琉球国三部官修编年史之一,由汉文写成。其名称「球阳」是对琉球的美称。《球阳》全称《球阳记事》,在第二尚氏王朝时期简称《记事》,《球阳》是琉球处分之后的通称。1743年(乾隆八年),唐通事郑秉哲(伊佐川亲方佑实)等奉尚敬王之命编纂,于清乾隆十年(1745年)初步完成;后由从中国留学而归的史官继续编写,直至1876年才被迫停止。球阳是琉球三部典籍中记载最为详尽的一部。原书共本卷22卷、附卷3卷;外卷(被命名为「遗老说传」)3卷、附卷1卷,今部分已亡佚。其内容包罗万象,涉及琉球王家系谱、国事、政治、经济、宗教、社会、文化、天文星象以及自然灾异等方方面面,是冲绳学研究的重要史料。目前已知的存世写本如下:此外,近代学者新垣义夫、宫里荣辉也对《球阳》进行整理校订。《球阳》各写本的目录存在著很大差异。","text2":"《球阳》全称什么?","label":1} {"text1":"裸狐鲣(学名:),又称裸䲠,俗名长翼,为辐鳍鱼纲鲈形目鲭亚目鲭科的一种。本鱼分布于印度太平洋区,包括南非、东非、红海、叶门、模里西斯、马达加斯加、葛摩、塞席尔群岛、留尼旺、马尔地夫、斯里兰卡、印度、缅甸、安达曼群岛、圣诞岛、可可群岛、中国、日本、韩国、台湾、菲律宾、越南、印尼、澳洲、斐济、新喀里多尼亚、东加、吐瓦鲁、密克罗尼西亚、萨摩亚群岛、法属波里尼西亚等海域。水深0至100公尺。本鱼体纺锤形,横切面近圆形,两背鳍几乎相连,在第二背鳍后方有6至7个离鳍,而在臀鳍后方有6个离鳍。尾鳍末缘呈弧形凹入,侧线完全,侧线后半部呈波浪状起伏,且在尾柄处和隆起的棱嵴相连。体背侧呈蓝紫色,腹部灰白色,身上无明显花纹或斑点。上下颔均具尖锐犬齿,背鳍硬棘13至15枚;背鳍软条12至14枚;臀鳍硬棘0枚;臀鳍软条12至13枚;脊椎骨38个,体长可达248公分。本鱼为沿岸中层洄游性鱼类,常一大群一起出现,属肉食性,主要以小鱼为食。为美味的食用鱼,属经济鱼种,可煮汤、盐烧或制成罐头,有雪卡鱼中毒之纪录。","text2":"裸狐鲣的尾鳍末缘呈什么形状凹入?","label":1} {"text1":"麟魁(),字梅谷,索绰罗氏,满洲镶白旗人,晚清大臣。道光六年(1826年)进士,选庶吉士,散馆改刑部主事,迁中允。历庶子、侍讲学士、詹事、通政使、左副都御史。二十二年,出署山东巡抚。二十三年,擢礼部尚书,管理太常寺、鸿胪寺。时黄河在中牟决口,督修河工,堤坝屡溃,被革职召还,降为三等侍卫。道光二十七年,召授礼部侍郎,调刑部。二十八年,复授礼部尚书,兼翰林院掌院学士。咸丰二年(1852年),在军机大臣上行走。七月,擢工部尚书。三年,调礼部,充总管内务府大臣,罢直军机。五年,迁刑部尚书。八年,复调礼部。十年,因谢恩奏折失检,被降调为刑部侍郎。同年秋天,英法联军内犯,被命署右翼总兵,充巡防大臣,主管京师西城治安。麟魁约束部下,组织民防,下令家家闭户,堆积柴薪,事急即放火与敌人同归于尽。十一年,迁左都御史,兼正白旗蒙古都统,不久升兵部尚书。同治元年(1862年),拜协办大学士。不久,奉命与沈兆霖赴甘肃平乱,行至兰州,数日遽卒,谥文端。《清史稿》有传。子恩寿,特赐举人,中同治十三年进士,官至陕西巡抚。","text2":"道光二十八年,麟魁官任何职?","label":1} {"text1":"DVD-Video是消费类视频格式,用于将数位视频存储在DVD光碟上。其最早于1996年11月在日本推出。至2003年止,DVD-Video格式已在亚洲、北美、欧洲和、及澳大利亚占有统治地位。使用DVD-Video规范的光碟需要带有MPEG-2解码器的DVD磁碟机才可播放(如DVD播放机或带有DVD播放软体的电脑DVD磁碟机)。商用DVD电影常使用MPEG-2压缩视频编码以及不同格式的音讯编码(通常为如下所述的多通道格式)。通常来讲,DVD电影的资讯速率从3 Mbit\/s到9.5 Mbit\/s不等,而比特率为自我调整速率。DVD-Video规范由DVD论坛创建,可以5,000美元的费用从DVD格式\/徽标授权公司(DVD Format\/Logo Licensing Corporation)中获得规范授权。DVD-Video规范不公开销售,且订户必须签署。DVD Book中的某些资讯为专有机密资讯。DVD-Video使用H.262(MPEG-2)压缩标准(最高码率为9.8 Mibit\/s)或MPEG-1压缩标准(最高码率为1.856 Mibit\/s)以录制移动图像。DVD-Video支援色彩深度为每色8位YCbCr与4:2:0色度抽样的视频。以下为H.262标准所支持的格式:以下为MPEG-1标准所支持的格式:所有视频模式下均支援4:3帧长宽比的视频,只有D1格式支援宽屏视频。MPEG-1不支援隔行扫描,而H.262均支持隔行扫描与逐行扫描。不率属于上述帧率的视频内容可使用技术编码为H.262,常见于编码每秒帧率为23.976的视频内容,使其以29.97的每秒帧率播放。","text2":"使用DVD-Video规范的光碟通常还需要什么配件才能播放?","label":1} {"text1":"刘玉婷(英文名:,),香港女演员。1986年从无线电视培训班出道,成为《欢乐今宵》中“欢乐小姐”一员。但很快因得罪高层遭到解雇,事后她向媒体哭诉经过,最终与无线不欢而散。后受橙天嘉禾赏识,成为嘉禾电影“五朵金花”之一。1987年至90年间,刘玉婷成为亚洲电视女艺人。在縯绎的角色中,以古装清丽女子形象给人印象最为深刻,代表作有《仙鹤神针》的玉箫,《九王夺位》的蓝如风。1996年,刘玉婷被台湾小说作家琼瑶相中,得以赴台主演电视剧《一帘幽梦》,并因此走红。此后留在台湾发展,一度成为台湾民视当家花旦。2005年,刘玉婷嫁给一名美国富商公子后淡出影视圈,移居美国纽约,婚后育有一儿。目前因儿子教育问题,已回到广州居住。2008年四川汶川大地震发生后,刘玉婷曾参加由广州市妇联、广州日报社、广州市卫生局联合主办的“广州妈妈爱心行动”,成为“爱心妈妈”一员。","text2":"刘玉婷参加了什么活动成为“爱心妈妈”的一员?","label":1} {"text1":"拖链,又名电线电缆保护拖链(Cable carrier),是一种束缚电缆或电线或空压管及油压管以方便其转动及运动的装置。1953年,德国的吉尔伯特·瓦宁格教授发明了世界上第一条钢制拖链。1954年成立于德国锡根市的佳宝来公司(KABELSCHLEPP)的持有人瓦尔德里奇认为拖链是一个全新的市场,可以创造巨大的需求,开始向市场推广第一条拖链。60多年前的一个概念创造了现在的庞大市场。现在第一条原创的钢制拖链样板已经发展为形形色色的钢制和塑料拖链,广泛应用于很多领域。KABELSCHLEPP佳宝来公司于1967发明了第一节由一个注塑成的塑料拖链,又成功的创造了更多的第一:轻便型拖链,3D拖链,无连接拖链。截至2007年,有超过7个全球制造商,包括FESMA,MURRPLASTIK,易格斯igus,椿本KABELSCHLEPP,Metreel,M Buttkereit,Gortrac和Cavotec.在2010年Tsubaki已将佳宝来公司合并,现Tsubaki Kabelschlepp公司。拖链分为钢制拖链,塑料拖链。钢制拖链由钢铝组合而成可以定做,塑料拖链又称工程塑料拖链,坦克链。拖链根据使用环境和使用要求的不同分为桥式拖链,全封闭拖链,半封闭拖链三种。 矩形金属软管,防护套,波纹管、金属软管也可以列入拖链防护及移动式供电产品。拖链应用于往复运动的场合,能够对内置的电缆、油管、气管、水管等起到牵引和保护的作用。拖链已被广泛应用于数控机床、电子设备、玻璃机械、注塑机机械手、搬运机械、塑料机械、起重设备、木工机械、汽车行业、产业车辆、金属加工机、机床、铸造机械、港口设备等行业。","text2":"现在第一条原创的钢制拖链样板已经发展为什么?","label":1} {"text1":"北海油田是世界著名的石油集中出产区,每日生产大约600万桶。位于大西洋的陆缘海——北海,它是介于欧洲大不列颠岛、挪威和欧洲大陆之间,所出产之石油为沿岸英国,挪威和荷兰等国所享有,也是布兰特原油指数主要标的。1970年代之前,以波斯湾为核心的亚洲中东地区,一直是西方工业国家石油能源的主要供应者。随着第二次世界大战后西方国家经济的恢复和繁荣,石油需求量逐年上增,使得亚洲石油国提高购价。这让各大石油巨头不愿购买,双方因此积怨。1973年第四次中东战争爆发,阿拉伯产油国以不满美国为首的西方国家支持以色列之立场为由,以能源为武器,宣布「石油禁运」,导致欧美油价暴涨,并相继波及各个经济领域,部分导致七十年代全球经济大衰退。因此,以英国为首的北海沿岸国家将目标转向沉寂多年的北海油田。在此之前,北海是欧洲航运要道,且海底地形复杂,海上气候恶劣,一直未有大规模地质勘探活动进行。而北海在1959年首先于荷兰近海发现格罗宁根气田(荷兰语:Groningen),此后更进入大规模开发阶段,先后于1969年发现埃克非思科油田(英语:Artsen non-Cisco),1971年发现布伦特油田(英语:Brent),大油田的相继发现,不仅缓解英国与挪威等西欧国家的能源短缺,而且更使挪威成为除加拿大和俄罗斯之外的第三大非OPEC石油出口国。北海石油的发掘,使得正处于疲软时期的英国、荷兰与丹麦等工业国家的经济获得帮助,也使得英国经济在「油荒」年代于西欧一枝独秀。北海原油因其品质高,产量稳定,所以迅速成为欧洲重要的能源供应地,因此欧洲原油交易市场多以每桶北海布伦特石油(约159升)作为市场参考价格。世界石油市场约6.4%的供货来源于此。","text2":"第四次中东战争爆发,对石油行业有什么影响?","label":1} {"text1":"保罗·朱利斯·路透(Paul Julius Freiherr von Reuter (Baron De Reuter), )英国犹太人,出身德意志卡塞尔市,世界新闻媒体业先驱,创办了著名的路透社 (路透电社)。他出生于德意志卡塞尔一个犹太家庭。1845年10月29日,他搬到了伦敦,在那里他自称约瑟夫。11月16日,他皈依了基督教,并改名为保罗·朱利斯·路透。一个星期后,11月23日,他和Ida Maria Elizabeth Clementine Magnus结婚。大革命失败后的1848年,他逃离德国,前往巴黎,并在夏尔·哈瓦斯新闻社工作(也就是现在的法新社AFP)。几经演变,路透首先成立路透社,在亚琛和布鲁塞尔之间使用鸽子传递新闻,甚至用它来连接柏林和巴黎。当火车速度远远超过了鸽后,路透于1851年让路透社在巴黎证券交易所上市。随后,电报取代了鸽子、火车等联系工具。电报穿过英吉利海峡连接了英国和欧洲大陆,这种联系延伸到西南海岸的爱尔兰。1863年,甚至有美国的船队直接订阅路透社的新闻。1851年,路透在伦敦设立办事处,在伦敦证券交易所上市交易。路透社成为世界上一个主要的金融新闻机构。1857年3月17日,路透在英国定居。1871年9月7日,路透被授予男爵称号。那天路透社的新闻标题是“维多利亚女王对路透授以英国贵族称号”。路透有三个儿子一个女儿。在1999年2月25日,在德国举办了纪念路透逝世100周年。路透家族的最后成员,路透的孙女于2009年去世,享年96岁。","text2":"保罗·朱利斯·路透在伦敦自称什么?","label":1} {"text1":"太平洋潜鸟(学名:\"\")是潜鸟目下的一种中型鸟种。有指太平洋潜鸟与黑喉潜鸟是同种。在阿拉斯卡、加拿大北部及至远东地区的巴芬岛及俄罗斯东面的勒拿河等地的冻土层的湖泊深处繁殖。与其他潜鸟不同的地方是太平洋潜鸟会成群地进行迁徙。如在冬天就集中在海洋上,特别是太平洋的海岸或大形的湖泊。当中包括中国、日本、北韩、南韩、美国及墨西哥等。偶尔也在格陵兰、香港及英国出现。繁殖期的太平洋潜鸟像一头光滑的普通潜鸟。牠们有灰色头、黑色喉、下腹白色及不规则的黑白相间的覆盖。非繁殖季节的羽毛较为沉褐,前颈部分为白色。喙为灰或白色,如锋利的匕首。侧羽上没有白色条纹图案是唯一将太平洋潜鸟与黑喉潜鸟分辨出来的方法。和所有的潜鸟一样,普通潜鸟也是捕鱼的好手,可以潜到深海中寻找食物。飞行时颈部向外延伸。","text2":"太平洋潜鸟冬天主要集中在哪里?","label":1} {"text1":"张孝骞(,),字慎斋,湖南省善化县南门外马厂巷人,内科医学家及医学教育家。幼年在私塾学习《四书》、《五经》及古文,1914年以第1名毕业于长沙长郡中学,并以第1名的成绩被录取进入湘雅医学院。1921年复以第1名毕业,并获得金牌及美国康涅狄格州政府授予的医学博士学位。毕业后留校专攻内科,于1924年1月到北京协和医学院(今中国协和医科大学)深造1年,随后正式留任于该校,陆续担任住院医师、总住院医师。1926年9月被选送到美国约翰·霍普金斯大学进行研究,1927年7月回国续任,1930年在协和医学院组建消化专业组,1932年晋升为副教授。1933年12月再次赴美国史丹福大学研究,1934年7月回国后,担任协和内科消化专业组主管。1937年芦沟桥事变后,他放弃协和医学院的研究工作,回长沙接任湘雅医学院院长职务。1938年夏季,战火逼近长沙,他不顾美国雅礼会的反对,迁校到贵阳,再于1944年迁到重庆。抗战胜利后,于1946年8月迁校回湖南。随后应美国国务院的邀请赴美国考察医学教育和讲学。1948年4月辞去院长职务,并于9回到北京协和医学院,担任内科学教授和内科主任。中华人民共和国成立后,他续任原职。1962年9月被任命为协和医学院副校长。1966年开始的文化大革命期间,他被打成「反动学术权威」、「特务」,备受侮辱和折磨。1978年以后担任中国医学科学院副院长。1987年8月8日因肺癌病逝于北京协和医院,享年90岁。张孝骞毕生致力于临床医学、医学科学研究和医学教育工作,对人体血容量、胃分泌功能、消化系溃疡、腹腔结核、阿米巴痢疾和溃疡性结肠炎等有较深入的研究,被喻为内科专家、医学教育家、中国消化病学的奠基人。。1948年获选为第一届中央研究院院士,1955年被推选为中国科学院生物学部委员(院士)。","text2":"张孝骞是哪里人?","label":1} {"text1":"奥亚吸蜜鸟(\"Moho braccatus\")是可爱岛特有及已灭绝的吸蜜鸟。牠们在20世纪初的亚热带森林甚为普遍,但自此数量就开始下降。1987年仍有听到牠们的歌声,但之后就被认为是已灭绝。灭绝的原因包括黑鼠、猪和带有鸟类疾病的蚊入侵,以及栖息地的破坏。奥亚吸蜜鸟是夏威夷最细小的吸蜜鸟之一,只有约20厘米长。牠们呈黑色或深褐色,脚上有稀疏的黄色羽毛,胸部及翼下有白带。牠们的喙尖锐及稍微弯曲,可以抽出花蜜。牠们特别喜欢山梗菜属及桃金娘的花蜜,也会吃细小的无脊椎动物及果实。奥亚吸蜜鸟在可爱岛峡谷的树孔中筑巢。牠们的近亲如夏威夷吸蜜鸟、监督吸蜜鸟及欧胡吸蜜鸟都已灭绝。对于奥亚吸蜜鸟所知有限。牠们的灭绝涉及复杂的问题,包括了蚊子散播疾病,令牠们退到较高的地方居住,但在较高的地方却缺乏树孔筑巢。另外在十年间曾发生两次飓风侵袭,将大部份有树孔的古树都破坏了,第二个飓风的到来亦阻碍了树木的生长,引致牠们于1987年灭绝。","text2":"奥亚吸蜜鸟体积有多大?","label":1} {"text1":"汤姆·麦克·阿波斯托(,)是一位美国解析数论学家,加州理工学院教授。阿波斯托1923年8月20日出生于犹他州。父母都是希腊人,父亲埃马努伊·阿波斯托(Εμμανουήλ Αποστολόπουλος,Emmanouil Apostolopoulos)1916年自希腊与其妻艾芙罗西妮·帕帕莎娜斯(Ευφροσύνη παπαθανασόπουλος,Efrosini Papathanasopoulos)移民至美国。其父亲在成为美国公民时将原名缩短为麦克·阿波斯托(Mike Apostol)。汤姆·麦克·阿波斯托则继承了父亲的名字。他在华盛顿大学获得化学工程学士与数学硕士。1948年则在加州大学柏克莱分校取得数学博士学位。之后,分别任教于加州大学柏克莱分校、麻省理工学院及加州理工学院。阿波斯托于2016年5月8日逝世,享寿92岁。阿波斯托著有多本数学教科书,相当有影响力。除了是有名的老师之外,也是「Project MATHEMATICS!」的开创者与计划指导者,这项计划是关于制作并探讨高中数学基本主题的影音作品。他曾协助改进马米孔·姆纳察卡尼扬发现的可视微积分(Visual Calculus),也曾和他合写过多篇文章。同时也为受欢迎的物理导论系列讲座—机械宇宙观提供学术内容。2001年2月21日,他被选举为雅典学院院士。此书被多所大学数学系采用,有高等微积分圣经级教科书封号,台湾译为高等微积分,中国大陆译为数学分析。","text2":"汤姆·麦克·阿波斯托所著被台湾译为高等微积分的教科书享有什么美誉?","label":1} {"text1":"Netlabel或称网络唱片厂牌,是指以在互联网上传播和散布以MP3及Ogg格式为载体的艺术家音乐性质的小型唱片厂牌。与传统唱片公司不同的是,Netlabel的音乐一般都以完全免费的性质传播,并大都基于一个通用的公共授权。(比如Creative Commons licenses, 中文译名为“创作共用”)。一般情况下,艺术家掌有自己音乐作品的版权。大多数的传播行为都采用游击营销的方式传播,少数Netlabel会有额外的资金供给于加盟的艺术家。在大多数情况下Netlabel都是由单独的个体维系厂牌的运作,这与传统唱片公司的性质截然不同。大多数作品都由艺术家本人独自掌握,处于一个唯一的身份维系他的作品使用权。在线音乐小组的始建要追溯到个人电脑应用初期,相关联于电视游戏和demoscene小组。早期的音乐小组通过发布基于DOS下的MOD格式数字文件,并俯着于通过C语言编写的视觉演示及创作小组自己绘制的材质,以软盘的性质免费传播。而Netlabel性质的确定则要推进至1990年MP3格式的发展及普及,大多数相关联的音乐作品都是电子及周边性质的音乐形式。时至今日Netlabel快速的发展和壮大于全世界。Netaudio音乐拥有传统商业模式所不具备的快速传播和适应于现代化的授权方式,让听众可以通过流媒体或免费下载收听到艺术家们的创作,这些因素构成了Netlabel的主要成分。","text2":"大多数的传播行为都采用什么方式传播?","label":1} {"text1":"英军服务团(英文:British Army Aid Group),是一支太平洋战争时期活跃于华南地区的英军情报部队,战时属于英军驻印度总部情报科,由军情九处管辖。英军服务团参与营救被日军关押于集中营内的盟军战俘、收集情报、进行策反等行动。香港重光后,英军服务团在1945年12月31日宣布解散。香港保衞战结束后,英国在香港的情报系统迅速瓦解,盟军难以掌握香港情况,只能从撤离香港的盟军,或因日军推行「归乡政策」逃难返到中国内地的华人得到零星的情报。1942年1月,香港大学医学院教授、香港义勇军陆军中校赖廉士在东江纵队成员协助下,由深水埗集中营逃至中国内地,并在1942年5月广东省曲江成立英军服务团,数月后迁往广西省桂林。英军服务团主要任务是协助营救逃亡内地的英国军民,故被编入负责战俘询问、英籍人员撤离与处理的军情九处,向驻重庆的英国大使馆汇报,由印度英军总司令管辖,而非受中国战区统帅的国民政府军事委员会支配。服务团大量起用华藉英军及华人英军雇员为骨干,部分人经训练后潜入香港及华南的日占区,向盟军提供情报、与集中营建立联系、绘制日军据点资料,并在战争结束前协助组建殖民地临时政府。英军服务团的特工均以特工编号作为身份识别,而与情报站沟通则以通讯代号,作为书信及无线电通讯加密,而架构上分为战地行动组及战地情报组,前者由从事破坏行动的英军地下特种行动处(Special Operations Executive)在华南地区活跃的Z组人员调配,后者则以华人特工为主。英军服务团在曲江、惠州、河源、清远、四会、沙坪、恩平等地设立前哨站,并一度在澳门设立情报站。1943年,英军服务团曾组派员参加美国军事顾问团在衡阳为国民政府军而设的伞兵训练营,英方一度计划以武力营救亚皆老街集中营的英藉军官,但最终因风险太高而搁置。1945年12月31日,英军服务团名义上纳入英国驻港的香港防卫军,架构正式解散。英国在1947年刊宪向多名英军服务团成员颁授大英帝国奖章(British Empire Medal)。据统计英军服务团约有80人阵亡,部分阵亡特工合葬于钻石山坟场的「抗日烈士墓」,而部分具军职人员则被埋葬在赤柱军人坟场。","text2":"英军服务团是什么时候解散的?","label":1} {"text1":"会城街道是江门市新会区政府所在地,是新会区政治、经济、文化、交通、通信、贸易的中心,总面积140.66平方公里,265975人。会城街道位于新会区北部,北靠圭峰山、云峰山与蓬江区的杜阮镇、白沙街道相连,东隔江门水道与江海区的礼乐街道、三江镇相望;南隔银洲湖与双水镇相望;西隔潭江及其支流与司前镇、大泽镇及罗坑镇相望。会城是新会城的简称。自隋朝开始,会城一直为新会地区的经济、文化和政治中心。自隋朝开皇十年废新会郡,建立新会县开始,县城就设在今会城街道。进入唐代,贞观十三年(639年)建冈州,州城也设置于会城,所以今日会城又名冈城。唐德宗贞元末,冈州撤销,新会改属广州,但县城仍位于会城街道。元至正二十一年(1361年),黄斌起义。县城遭攻陷,县尹暂时移往潮居都办公。明太祖洪武元年(1368年),征南将军廖永忠平定黄斌起义,重新置县治于会城。清代,新会县下辖龙溪、常德、寿宁及昆仑四乡、三坊、十二都、一百零四图,合共664条村庄。民国20年(1931年)3月1日,新会县政府迁往江门镇石湾庙。六年后,重新由石湾庙迁回。民国28年(1939年)4月2日,日军侵占县城,县政府迁往双水天亭豪山。直至1945年9月26日,县府随国民革命军64军光复县城。1949年10月24日,中国人民解放军进驻新会县城。翌日成立江会区军管会,县政府与军管会在江门办公。直至1950年4月末,新会县政府才迁回会城。1951年改称会城镇,1961年6月1日改为人民公社,1971年6月恢复会城镇,1992年和环城镇合并,2001年和七堡镇合并,2005年改为街道办事处至今。会城街道下辖以下地区:城际轨道巴士","text2":"今日的会城又名什么?","label":1} {"text1":"特伦特河畔斯托克(),又称史笃城、斯托克(),英国英格兰西米德兰兹区域的二级行政区(区级),拥有城市、单一管理区、自治市镇地位,有239,700人口,占地92.74平方公里,行政总部位于Stoke-upon-Trent。若视斯塔福德郡为名誉郡,则史笃城为斯塔福德郡一部分;若视斯塔福德郡为非都市郡,则史笃城不为斯塔福德郡一部分。史笃城之瓷器生产非常有名,号称「英国瓷都」,知名品牌包含 Mintons, Coalport, Davenport, Royal Crown Derby, Royal Doulton, Wedgwood and Worcester。2015年,在英国旅游局举办的#英国等你来命名#活动中,“万博陶瓷”一名被评选为“奢华购物”主题里“最受欢迎命名”之一。","text2":"被评选为“奢华购物”主题里“最受欢迎命名”之一的名字是什么?","label":1} {"text1":"杨元龙()祖籍江苏吴县(今苏州),香港知名爱国实业家,溢达集团创始人。先后获得上海圣约翰大学化学硕士学位和美国麻省洛厄尔纺织学院纺织化工硕士学位。杨元龙爱好养马,八十年代曾被冠以「马王」美誉。1986年卷入造马案,杨元龙在案件开审后承认控罪,但却以患上末期骨癌为由,并且得到多位医生联合证明他只余几星期寿命,以此为理由向法官求情免刑。结果,杨元龙被判监两年,缓刑两年,避过牢狱之苦,但罚款五百三十万元。杨元龙随即往美国医病,1989年返港,并经常神采飞扬地出席名流及商界等场合,并从事地产投资,杨元龙八十岁大寿时曾于港岛一间五星级酒店举行盛大豪门夜宴。1949年后,杨元龙和夫人杨蔡咏芳在香港从事纺织制衣业。1961年至1973年,连续12年担任香港棉业咨询委员会(后易名为纺织业咨询委员会)委员,代表香港政府参与“关税及贸易总协定”及“多种纤维协定”的制订;兼活跃于中华厂商联合会、香港工业总会、香港棉纺业同业公会、贸易发展局等行业组织。1978年创办溢达集团,一家纺织及制衣企业,全棉衬衫制造商和出口商。杨元龙名下兼创立侨美旅游事业有限公司,投资开办北京香山饭店。夫人杨蔡咏芳,长女杨敏德(溢达集团主席,曾任行政会议成员,现任全国政协委员、香港理工大学校董会主席),次女杨敏贤。","text2":"杨元龙获得哪两个硕士学位?","label":1} {"text1":"BA-6()是苏联在1930年代早期研制的装甲车。BA-6使用的45毫米炮(BT炮塔)和T-26坦克一样,另装备1挺7.62毫米DT机枪于车身中部,一挺DT同轴机枪于炮塔。BA-6重5.1吨,车长4.65米,宽2.1米,高2.2米,乘员4名。BA-6使用美国制GAZ-AAA底盘,这种底盘只适合在公路等良好路况下行驶。这情况后来在增加一对适合路轮时得到一点改善。BA-6外形接近BA-3,但BA-6取消了右后方一扇车门。BA-3装甲过于笨重,而BA-6装甲较薄(10毫米)且性能更好。BA-6最后被更佳的BA-10取代。在30年代早期,BA-6等装甲车可轻易击毁大部份同期装甲车,但其薄薄的装甲还是容易被小口径炮击破。BA-6参加了早期东线的战斗。由于其装甲不足以应付德军火力,所以其侦察角色被T-60坦克及T-70坦克等取代。","text2":"BA-6使用美国制GAZ-AAA底盘,这种底盘有什么特点?","label":1} {"text1":"草黄树鸭(学名:\"\")是分布在热带地区的树鸭属,包括中美洲、南美洲、撒哈拉以南非洲、印度次大陆及美国墨西哥湾沿岸地区。草黄树鸭长48-53厘米。牠们的喙是灰色的,头部及脚都很长,头部及上身呈浅黄色,两侧有些红色,冠深色,背部及双翼呈深灰色。尾巴及双翼上有栗褐色的斑纹,上尾有一白色的半月形,在飞时特别明显。雏鸟两侧对比较低,尾巴颜色也有不同。草黄树鸭很普遍。除了一些地区性的移动外,很多时都是留鸟,而在南欧也会有一些流浪者。牠们会在树枝上筑巢,有时也会在空心树中筑巢或利用其他鸟类的巢。牠们每次会生8-12只蛋。草黄树鸭喜欢栖息在淡水湖、稻田或水塘。牠们多在夜间觅食,主要吃种子及植物的其他部份。草黄树鸭是群居的,可以组成一大群。草黄树鸭是《非洲-欧亚大陆迁徙水鸟保护协定》所保护的物种之一。","text2":"草黄树鸭的体长是多少?","label":1} {"text1":"给水处理是通过水处理工艺去除水中的杂质,使水质符合使用要求或相应标准。传统的给水处理包括混凝、沉淀(或者气浮)、过滤和消毒等。随着水源水质的恶化尤其是有机污染物的增加,需要预处理和深度处理。预处理的主要对象是水中的有机污染物,同时也具有除味、除臭、除色的作用。常规水处理的目的主要是去除水中的悬浮固体、溶解固体及微生物。典型的常规处理包括混凝、沉淀(或者气浮)、过滤和消毒。混凝池混凝是指利用混凝剂将水中胶体粒子以及微小悬浮物聚集的过程,一般在反应池中进行。混凝后形成较大的矾花,利用沉淀或者气浮工艺将其去除。国内常用的混凝剂有氯化铁、聚合氯化铝等,助凝剂有聚丙烯酰胺(PAM)等。混凝机理有电性中和、吸附架桥、网捕和卷扫。","text2":"什么是给水处理?","label":1} {"text1":"造纸胡蜂(学名:\"\")是欧洲一种很普遍及著名的胡蜂。牠们在加拿大及美国是入侵物种。过冬后的造纸胡蜂蜂后会在春天的一个时间先行筑巢,并产子。最先出生的雌蜂会成为工蜂继续筑巢。雄蜂会在后来出生,一些雌蜂此时会与雄蜂交配,并离开蜂巢,在下一季成为新的蜂后。蜂群于夏天末就会消失,只余下雄蜂及未来的蜂后聚集在一起过冬。牠们很少会在原有的蜂巢过冬。在形态上,造纸胡蜂蜂后与其他负责繁殖的雌蜂没有分别。不过,由于在蜂群相互作用下不同的角色,牠们在行为上却有著分别。一般而言,蜂后是蜂群之首,牠会产大部份的卵,并且吃下参差的卵。蜂后会花大部份时间在蜂群的相互作用,而其他雌蜂则更多在觅食及照顾雏蜂。这种行为上的差别并非永恒:若蜂后离开了蜂巢,另一只雌蜂(第二大的)就会取代牠的位置。事实上,每只造纸胡蜂都会按牠们的个别的级别位置而变更其行为模式。一些研究显示蜂后会透过行为来压抑其他雌蜂卵巢的发育。蜂后相信是透过摆动腹部来发出支配的讯号,但有研究发现被去卵巢的蜂后失去了限制其他雌蜂繁殖的能力,但却仍维持其蜂后的地位。另外,雌蜂们的身份亦可以由其他因素所影响。若将可能成为蜂后的雌蜂放在蜂巢较早筑成的部份,这只雌蜂就会转而成为工蜂,相反亦然。这显示蜂群在阶级地位上有明显的灵活性。造纸胡蜂蜂后在外表上与工蜂有所分别,蜂后会以腹部打击蜂巢的表面,且会较其他下属多留在蜂巢中,故蜂巢的气味大部份是来自蜂后的。造纸胡蜂的分布地覆盖大部份的南欧及北非、亚洲的温带至中国。牠们亦被引入澳洲、北美洲及南美洲。造纸胡蜂的学名很多时会被误写为\"Polistes gallicus\",但这其实是柞蚕马蜂的学名。造纸胡蜂最初于1791年被命名为\"Vespa dominula\"。种小名的\"dominula\"意为「小妇人」,但一直以来很多学者都将之误并为\"dominulus\"。","text2":"蜂后在蜂群中的地位和作用是什么?","label":1} {"text1":"久里滨线()是一条连结日本神奈川县横须贺市堀之内站与同县三浦市三崎口站,属于京滨急行电铁(京急)的铁路线。车站编号使用的路线记号为KK。普通车只在平日早上繁忙时段行驶。现在普通车是不会行走京急久里滨以南的地方(1996年4月至1999年7月只在平日行驶)。特急在早上至早上繁忙时间、平日晚上至深夜,在该时段的列车全为快特。只有平日朝上运行,抵达横滨站7:30 - 8:30前往品川的最前的车卡是女性专用车。为方便前往成田山新胜寺参拜的乘客,在1969年至1972年间开办了来回三浦海岸 - 京成成田站的通宵列车。使用的车辆为1000形,停车站与京急线、都営线内特急一样,而在京成线内则以「临时特急」运行(京成津田沼站 - 京成成田间不停车,这个与押上站至京成津田沼间之间运行的特急有少许差异)。原定此线也会在1973年运行,但是由于东京都交通局就劳动争议问题而反对通宵行驶,最终亦停办此线。","text2":"女性专用车是指什么?","label":1} {"text1":"毛南语(或称毛难语)是侗台语族侗水语支(一说为\"澳斯特罗尼西亚语系\"或\"台-卡岱语系\")的一种语言,为毛南族人所使用,目前这种语言有大约3万人使用。主要分布在广西环江毛南族自治县的上南、中南和下南乡,邻近的几个县也有少量分布。毛南语和同族其他语言有很多共同点。和水语尤其接近。声母系统比较复杂,有四套塞音声母。2010年前该语言没有书写系统,用汉字音义拼记或以汉字为基础创造新字来记录毛南语的民歌和宗教诵本等,这种“土俗字”只有师公或少数学者认识。语言特点粗略来说句法属SVO类型(即主语+动词+宾语),如 Maenz(他/她)bai(去)her(集市)“他/她去赶集/赶墟”;中心语位于修饰语之前,如mus(猪)naem(黑)“黑色的猪”。2010年后才出现了以拉丁字母为基础的“毛南文字拼音方案”。现代政治经济词汇多为西南官话借词,老借词多为平话借词。语音方面,一般认为有8个能区别意义的声调(实际上只有6个能区别意义的调值);毛南语的声母系统(80多个声母)比汉语(约20个声母)、壮语(约30个声母)等复杂。毛南人多通壮语及汉语,通用汉文。毛南地区小学、中学采用汉语课本,用汉语讲授,但小学老师讨论教案多使用毛南语。毛南族聚居密度最大的下南乡政府日常语言一般是毛南语。环江毛南族自治县政府官方语言是汉语西南官话。毛南族的另外一个分支(约占人口1/3)聚居于贵州南部,通用“扬黄话”,所以也叫“扬黄人”(见陆天桥著《毛南语语法》. 美国佛罗里达: 环球出版社. 2008)。","text2":"毛南地区的中小学授课采用什么语言?","label":1} {"text1":"白眉丝刺莺(\"Sericornis frontalis\")是一种生活在澳洲沿岸的雀。牠们是吃昆虫的,并生活在下层丛林,在近城市的地方也可以发现牠们。牠们长11-14厘米,主要呈褐色,眼眉呈白色,三个亚种之间也很不同。牠们很少运动,并且会进行合作生殖。根据鸟类DNA分类系统,白眉丝刺莺被分类在斑啄果鸟科中,但已证实是错误的。现时将牠们分类在刺嘴莺科中。较大的褐色丝刺莺有时会被认为是牠们的亚种。白眉丝刺莺最初是由自然学家Nicholas Aylward Vigors及Thomas Horsfield于1827年描述。种小名是拉丁文「眉毛」的意思。牠以下有三个亚种:另外,在塔斯曼尼亚及巴斯海峡临岛的褐色丝刺莺有时被认为是白眉丝刺莺的一个亚种。白眉丝刺莺长约11.5厘米,上身呈橄榄褐色,瞳孔及白色的眉毛特别显眼。颈部呈白色,\"frontalis\"及\"laevigaster\"亚种有轻微的斑纹,而\"maculatus\"则有斑点。\"frontalis\"的耳羽呈灰色,\"laevigaster\"的是黑色,\"maculatus\"的则是褐色。喙幼而呈黑色。雌鸟整体较深色。白眉丝刺莺喜欢生活在森林或灌林地区。在悉尼未开垦的林地牠们很普遍。白眉丝刺莺主要是吃昆虫的。牠们很难被观察到,但经常唱歌,所以很易知道牠们的所在。牠们一般会以一群约6只一起生活,会进行合作生殖,即所有成员会协助喂养雏鸟。白眉丝刺莺于每年的6月或7月至11月或12月进行繁殖。牠们的巢圆拱形,以干草、叶子、树枝、树皮等来制造,多在地面上的密林处。牠们一胎会产二或三只蛋,蛋呈紫褐色至白色,约20 x 15毫米大。","text2":"白眉丝刺莺生活在什么地方?","label":1} {"text1":"北京景山学校是1960年代建立的一所集小学、初中、高中于一体的学校,位于中国北京市东城区,是北京市高中示范校。该校创办于1960年春,1977年成为全国重点学校,1978年成为北京市重点学校。1983年国庆节,邓小平为景山学校题词:「教育要面向现代化,面向世界,面向未来」。北京景山学校占地面积32亩,建筑面积33000平方米。拥有体育场、馆、游泳馆、形体训练馆、天象馆及实验室等。图书馆藏书8万册。北京景山学校的学制与其他学校不同,现行学制为小学5年,初中4年,高中3年。小学、初中实行九年一贯义务教育,学生小学毕业后可直接升入景山学校的初中部(不含高中部)。北京景山实验学校是北京景山学校的分部,成立于1996年,位于昌平区北七家镇王府街,是一所全日制寄宿学校,与北京景山学校实行同一学制,共享教学资源。","text2":"北京景山学校的学制与其他学校不同点在哪里?","label":1} {"text1":"原始印欧语数词已经被现代语言学家基于跨越所有印欧语系语言所找到的相似性而重构了。本文列出并讨论它们的假定形式。基数重构如下:其他重构典型的同 Beekes 和 Sihler 有轻微的差异(例如 Fortson)。元素 (在数20到90中)和 (在100中)是在假定这些数为 *deḱm̥(t)“10”的派生词的假定上重构的。Lehmann 相信大于10的数都是在方言群组中独立构造的而 最初意味著“一个大数”而非特指100。数3和4有带有后缀 的阴性形式,分别重构为 和 。特殊形式的数被用作前缀,通常用来形成 bahuvrihi 复合(比如英语中的“five-fingered”):序数是难于构造的,由于它们在后代语言中是各种各样的。下列重构是试探性的:结尾于音节主音鼻音的基数(7,9,10)在 thematic 元音前插入第二个鼻音,导致了后缀 和 。这些后缀和后缀 扩散到了临近的序数,例如吠陀梵语 “第八”和立陶宛语 “第九”。在下列语言中,用斜杠分隔的两个反映意味著:","text2":"哪些重构是试探性的?","label":1} {"text1":"胡燕泳(),香港新闻从业员。2001年胡在香港中文大学新闻与传播学院毕业,曾经从事公关工作。2003年转职香港有线电视新闻主播,并于同年转职到24小时亚视新闻台。2006年获奖学金,留学伦敦大学亚非学院,念国际研究与外交硕士。及后开始于免费频道本港台报道新闻,曾任担任本港台六点钟新闻、夜间新闻主播一职。2007年与黄珊、黄雅宇共同担任《主播天下》的主持及策划职务。2010年3月,她辞职并离开亚视新闻的播报工作,至2012年7月以兼职身分重返亚视新闻工作。2012年11月在报章撰文批评亚视高层干涉新闻部运作后,遭亚视解雇。2013年4月起,任无线新闻外电编辑,同年9月尾开始成为下午《新闻提要》主播。2013年12月7日起,担任《午间新闻》主播。2015年3月2日起,担任《环球新闻档案》旁白。2015年中,辞职并离开无线新闻,转职到香港经济日报。亚视受管理层连番操控,声称收视与TVB「四六开」惹来公众、广告业界人士和学者质疑,动员员工反对政府发出新免费电视牌照的举措也令全城非议。胡在2012年11月于《明报》发表题为《来自亚视的声音》的文章,力斥管理层王征和盛品儒于多次事件的处事手法都很有问题,批评王在亚洲电视误报江泽民逝世事件中没有任何承担;在政府总部举行的「大集会」王盛二人的言论及行为「着实超乎常人所能接受的尺度」,更严重的是「人们对亚视的鄙视,似乎并不规限于个别人士,而是整个亚视遭殃,新闻部也不能幸免」。另外,胡指出,过去她与同事制作《主播天下》时亚视给予新闻部很大自由度,但如今已今非昔比;原本《主播天下》的后续节目《ATV焦点》本来同样由新闻部记者制作,但最终已变质成为高层公器私用的平台;她和《ATV焦点》撰稿作者(前《大公报》执行总编辑雷竞斌)「在新闻部碰口碰脸」并不稔熟,但「因为他的个人大作,却引来四万宗投诉、网民的唾骂、前线记者在政总采访受滋扰;一人创作,新闻部百人当灾,我不甘心看到这境况」。2012年11月26日,亚视突然解雇七名员工,胡是其中之一;对她被辞退是否与批评公司有关,亚视公关部称不会回应。","text2":"胡燕泳是何时转职到香港经济日报的?","label":1} {"text1":"《坂上台风》()是戏画原预定于2008年9月26日发售日本成人游戏,后延期至同年11月28日发售。负责人物设计,、陆奥龙介负责剧本。日语「」的意思是「上坡的台风」。2011年7月28日由Alchemist发行PlayStation Portable版和2016年10月27日由ENTERGRAM发售PlayStation Vita版《》。在日本美少女游戏销售网站Getchu.com的2008年排名第15名。「因为这样会比较有趣」——主角皆川巧从小就以这个想法行动,把周围的人卷入闹剧中,因而被称之为「台风」。虽然在同学中受欢迎,却是个问题学生,父亲皆川吟二曾把他送入位于深山、校风严谨的神道学校巍叡山学园,但还是被踢出校。最后回到老家的武月学园,由学生会会长的青梅竹马守永由香里监视下入学。同日,巧在买饮料的途中遇见有点笨的绫濑奈都希。然后又在课室中遇见深咲凉,最初被回以冷淡的态度,突然变得非常开朗爽快,但知道巧「并非独自一人」后,又回到之前的冷淡。正式入学前日,由香里和巧约定如果巧上学迟到,就要服从一个要求。巧在当日真的迟了起床,跑到学园门口前遇到同学大泽柚,帮她越过门口后,已经迟到。而由香里的要求是要巧当班长,认为可以让他安份一些。渴望这个职位的奈都希不服一个转校生可以突然当上班长,对巧产生对抗心,协助由香里监视巧。而巧觉得凉对实现课室的意见箱的事项有兴趣,认为可以让凉变回原来开朗的性格,柚因为欠巧一个人情而不断帮助他作意见箱的工作。期后再出现「援军」,巧的朋友绮罗泰德被巧骗来武月学园,准备行动时又突然出现巧在以前深山学校时代认识的「朱丽叶」楠春……平凡的校园生活,正因这位问题学生而逐渐改变。","text2":"主角从小都有怎样的一个想法?","label":1} {"text1":"中央研究院历史语言研究所(简称史语所)是中华民国中央研究院负责中国近代著名的历史和语言学研究机构,1928年成立于广州。因陈寅恪、赵元任、李济、罗常培、李方桂、董作宾、梁思永、劳干、周法高、严耕望、石璋如、芮逸夫、全汉升等著名学者先后担任研究员,并取得了殷墟等处考古挖掘、内阁大库档案的整理研究、全国各省方言调查等重大学术成果,而成为世界学术界瞩目的重镇。傅斯年提出的“史学即史料学”、“动手动脚找东西”理念,对史语所影响至深。出版《历史语言研究所集刊》,素为世所重。1928年3月底,中华民国中央研究院筹备委员会一致通过创建该所,由傅斯年、顾颉刚和杨振声三人为常务筹备委员,筹备处在广州中山大学。1928年7月,筹备完成。1928年10月22日,迁入独立所址“柏园”,后以此日为所庆纪念日。1929年后,先后迁北京、上海、南京。对日抗战期间,历迁长沙、昆明和四川南溪县李庄。1946年,迁回南京。1948年冬,迁台湾。1954年,定居台北市南港区。最初设八个组。后合并为历史、语言和考古三组,由陈寅恪、赵元任、李济三人分任组长。1933年至1934年,曾一度与社会科学研究所合并。1934年增设人类学组。1958年,增设甲骨文研究室。1990年,甲骨文研究室改名为文字学组。1997年,语言学部分独立为语言学研究所。","text2":"史语所成立于什么时候?","label":1} {"text1":"2008年至2009年英格兰联赛杯,或因赞助商而冠名为加宁杯(Carling Cup),是第49届的英格兰联赛杯赛事。胜出的队伍如果没获得欧洲赛参赛资格的话,可以获得2009年至2010年欧洲联赛第三圈外围赛资格。曼联在决赛互射十二码阶段击败热刺,成为今届联赛杯的冠军。第一圈的抽签于2008年6月13日举行,而赛事则于8月11日进行。72队参赛队伍分为南北两组,每组平均分为种籽队和非种籽队,而排名则根据上年成绩而定。12队英超联球队会加入角逐。第二圈抽签于2008年8月13日举行,赛事于2008年9月25日左右进行,除了曼城和白礼顿的赛事,此场于9月24日进行。8队要参加欧洲赛的队伍会加入角逐,由于今季有9队球队参加欧洲赛,所以起初不清楚哪8队会加入。直到2008年7月29日,赛会宣布阿士东维拉会在此赛圈加入。第三圈抽签于2008年8月30日举行,赛事于2008年9月24日左右进行,除了白礼顿和打比郡的赛事,此场于11月4日进行。第四圈抽签于2008年9月27日举行,赛事于2009年11月10日左右进行。第五圈抽签于2008年11月15日举行,赛事于2008年12月2日或3日进行。准决赛抽签于2008年12月6日举行,首回合于2009年1月6日或7日进行,次回合于1月20日或21日进行。\"曼联总比数赢4 – 3\"\"热刺总比数赢6 – 4\"今届决赛于2009年3月1日在温布莱球场举行。以下是今季联赛杯神射手名单︰","text2":"2008年至2009年英格兰联赛杯胜出的队伍可以怎么样?","label":1} {"text1":"《第一季单车二势力》为2008年台湾年代MUCH台所制播的户外活动性节目,主持人为黑涩会美眉瑶瑶以及演艺圈新进MIA两位台湾艺人共同主持。该节目以主持人骑乘单车造访台湾各处知名景点,透过活泼风趣介绍当地有趣的人事物。《第二季单车二势力》全台湾走透透、吃透透、玩透透,此季单车二势力以校园周边美食为主轴,每集由主持人MIA及PARTY率领两所大专院校学生推荐的周边美食来PK,在介绍最好吃美食的同时,在节目中还加入了两校学生推荐PK的元素,让节目不只是提供吃喝玩乐讯息,还有竞赛性的趣味,增加内容张力。同时,用学生推荐美食也可以增加节目的互动性,拉近与一般观众的距离,节目也不将再是纯粹的单向介绍美食给观众。","text2":"用学生推荐美食有什么好处?","label":1} {"text1":"险礁屿,为台湾澎湖县白沙乡之岛屿,位在吉贝屿南方,乃是澎湖最大沙洲。因岛屿四周多暗礁而得名。岛上无人居住。全岛除了东半部有岩石构成的海蚀平台外,皆是砂粒、贝壳和珊瑚碎屑构成的白色砂滩。近年来因为中视偶像剧《原味的夏天》在岛上拍摄而爆红。2009年澎湖县政府举办消费券大摸彩特别奖——无人岛「险礁屿」岛主,一名4岁男童叶千维幸运得奖,但因为身分曝光引来关注;叶家人在不得己的情况下,决定上网拍卖一年无人岛岛主的权利。《原味的夏天》制作单位利用原有的石板屋搭建了一座两层楼的蓝顶小屋,取名为「八厘米蔚蓝」,成为岛上著名的观光景点。提供水、干粮、毛巾等物品。岛上唯一的民宿「海的美宿馆」,共有六间小木屋,分双人房和四人房。险礁屿无机场,须乘船舶至澎湖本岛转搭飞机(马公机场)往台北。由赤崁乘坐船艇前往约二十分钟即可抵达。岛上无公路。","text2":"险礁屿的地理位置?","label":1} {"text1":"沙丘之乱,或作沙丘宫变,是指战国时代赵国的一场兵变。赵主父(赵武灵王)与赵惠文王到沙丘(今河北省邢台市广宗县)游玩,安阳君公子章趁机发兵攻击赵惠文王,意图夺位。公子成和李兑率兵勤王,公子章逃跑,主父竟然接纳了公子章,公子成和李兑杀死公子章后,为了卸责,继续围困主父,主父因而饿死。赵武灵王长子公子章,母韩氏,原为太子,因武灵王宠幸吴娃,于是改立吴娃之子公子何为太子。前299年,赵武灵王攻破中山国都城灵寿,并吞了中山,迁中山王到肤施。武灵王见公子章失位,且侍奉弟弟,又不忍心,一度想将赵国瓜分,由二子各自为王,后为大臣们所阻。最后将之封于代郡(今河北蔚县),称安阳君,以田不礼为宰相。将军李兑知道公子章不能忍受这样的屈辱,日后必然谋反,并劝谏肥义自保,肥义虽然知道结果,但仍然决心保护公子何。肥义甚至还说:「如果有人要跟赵何相见的话,一定让我走在前面。」前298年,赵武灵王禅让予赵何,是为赵惠文王,武灵王自称主父(君主之父)。前295年主父和惠文王游于沙丘,公子章与田不礼诈称主父召见惠文王,打算将他谋杀,以篡位。肥义知道其中有诈,代替惠文王先去,被杀。公子成和李兑从邯郸赶来平乱,发四邑之兵,杀田不礼。公子章逃到主父宫中,主父因为父子之情,接纳了公子章。公子成和李兑率兵包围主父宫殿,终于杀了公子章。不过公子成和李兑害怕主父对他们两人报复,甚至灭族,只好一不做二不休,继续围困,对宫中的人说:「后出来的人灭族」,宫中人都逃了出来。主父逃不出来,苦无粮食,把树上的小鸟都掏来食用,过了三个月左右,在宫中饿死。谥为武灵王。","text2":"谁率兵包围主父宫殿,杀了公子章?","label":1} {"text1":"Boost C++ 函式库(Libraries)是一组扩充C++功能的经过同行评审(Peer-reviewed)且开放源码程式库。大多数的函式为了能够以开放源码、封闭专案的方式运作,而授权于Boost软体授权条款(Boost Software License)之下。许多Boost的开发人员是来自C++标准委员会,而部份的Boost函式库成为C++的TR1标准之一。为了要确保函式库的效率与弹性,Boost广泛的使用模板(template)功能。而它是针对各式领域的C++使用者与应用领域(Application Domain)上,包含的函式库类别从像smart_ptr 函式库这种类通用函式库,到像是档案系统的作业系统抽象层,甚至能够利用Boost来开发额外的函式库或是给进阶的C++使用者利用,像是MPL。现有的 Boost 包含大约150种不同的函数库,以下面几项做范例:Boost 包含了 uBLAS 线性代数函数库,能够借由基本函数库子函数(BLAS)来支持向量与矩阵形运算。Boost 也提供独立分布的模拟随机与 PRNG 独立性的机率分布,而这些能够具体的建立产生器。更详细的说明请参阅 Boost 随机数库。范例码演示建立执行绪:","text2":"现有的 Boost 包含多少种函数库?","label":1} {"text1":"按司(琉球语首里那霸方言:或;先岛方言:)是琉球国历史上的地方行政单位,后来成为琉球国贵族和地方豪族的一种称号。自第二尚氏王朝起只有王族成员才可出任。按司家族是国王家族的分家,相当于日本的宫家、中国的藩王府,在贵族中,其地位仅次于王子。女性王嗣(翁主)亦可成为按司,女按司称为按司加那志(アジカナシ或アンジカナシ)。《中山世谱》等琉球史书称,琉球人是天帝的子孙。天帝派遣女神阿摩美久(アマミキヨ)和男神志仁礼久(シネリキヨ)来到人间,生三男二女,为琉球人的祖先。其中长子是天孙,其后代世为国王;次子为按司(诸侯)的祖先;第三子为百姓(平民)的祖先。但事实上早期琉球没有国王这个概念,各地区的首领自称「按司」,有时也会被祝女授予「世之主」、「世主」之类的神号。根据冲绳学学者伊波普猷的说法,「按司」是琉球语中「主」(アルジ)的转讹。按司最早作为地方势力的首领出现于12世纪,并在各地建立了一些御城,彼此互相争战。后来尚巴志王统一了琉球。但按司仍在地方拥有一定势力,参与过琉球国内的政变。直到尚真王时期,琉球朝廷下令收缴各地兵器,召各按司赴首里城居住后,按司割据势力才被消弱。此后按司从地方豪强转型为都市贵族,派遣名叫「按司掟」(アジウッチ)的官员管理自己的领地。此后,在第二尚氏王朝的位阶制度确定之后,按司成为了琉球王族的一个位阶。没有即位的王子,其长子会被授予按司的位阶。因此按司一般姓向氏,名乘头世袭为「朝」字。只有国头按司家(国头御殿)例外:该家族姓马氏,名乘头世袭为「正」字。这是因为国头按司家的祖先马顺德(国头亲方正格)对琉球王室有重大功绩,曾救了尚元王一命。为了报恩,尚元王特别授予其子孙王族的待遇和按司的位阶。琉球王府征服奄美群岛和宫古群岛之后,这两个地区的酋长并没有被授予按司的称号。奄美群岛的酋长称「大亲」(ウフヌシ或ウフヤ),而宫古群岛的酋长则称「丰见亲」(トゥユミャ)。高级按司敬称为御殿(ウドゥン);低级按司的敬称为御前(ウメー),其子孙称按司前(アジメー)。一般笼统的称为御殿。按司的采地称按司地头。原则上,按司同亲方一样,会被授予一个间切的地头。受日本文化影响,有时也把领有一个间切的按司称为「大名」(デーミョー)。据《琉球藩臣家禄记》记载,琉球国末年共有25家按司。这25家分别是:","text2":"为什么只有国头按司家家族姓马氏,名乘头世袭为「正」字?","label":1} {"text1":"《中山世谱》是琉球国的汉文官修史书。康熙三十六年(1697年),摄政尚弘才等人奉尚贞王之命,以蔡铎为中心,对《中山世鉴》进行汉译和改订,于1701年完成初版《中山世谱》(被称为蔡铎本《中山世谱》),献给清朝康熙帝。蔡铎本《中山世谱》记载了琉球国自天孙王朝至尚益王代发生的事情。共正卷5卷、附卷1卷。雍正二年(1724年),由摄政尚彻主持,以蔡温为中心,对蔡铎本《中山世谱》进行加笔、修正,于翌年完成第二个版本的《中山世谱》,后世称之为蔡温本《中山世谱》。现存的《中山世谱》共有本卷14卷(主要是收录有关中国的事项)、附卷7卷(主要是收录有关萨摩藩的事项),蔡温本《中山世谱》的改订只到本卷九卷,其后是由史官继续修订,直到1876年才被迫停止。附卷的部分可能也是由蔡温所编修,后来又经唐荣都通事郑秉哲之手改订,成为现在的7卷。","text2":"蔡温本《中山世谱》到哪年才被迫停止?","label":1} {"text1":"莫德惠,满族,中华民国政治人物。字柳枕,吉林双城人(今属黑龙江)。莫德惠1883年4月16日生于新疆其父的军营中。其父明海,1870年从左宗棠平定陕甘回乱,参与收复新疆之役。以军功晋佐领。1875年举家迁入吉林双城。1906年考入天津北洋高等巡警学堂,毕业后分配吉林省警察厅任西局局员,旋即升任西局局长。1910年调任哈尔滨警务局滨江巡警局局长。1912年冬,莫德惠以社会贤达被举众议院议员,这是他跻身民国政坛的发端。1914年袁世凯解散国会,莫德惠受奉天都督张锡銮之邀出任双山县知事并兼理司法,成绩裴然。1918年出任清理吉林官产处处长,继为吉林永衡官钱银号监理官,清查土地局会办、财政部顾问,吉林省公署顾问,奉天省公署咨议,吉林省依兰道道尹。1921年与在吉林剿匪的张学良相结识,1923年促成粤皖奉联合反对曹锟贿选,得到张作霖倚重,出任奉天财政厅长代理省长、北洋政府农工部总长。1928年6月,随张作霖返奉专车遇险负轻伤。张学良主政东北后,出任东北保安、政务、外交等各委员会委员。东北易帜前夕,衔命赴日折服田中不干涉易帜成功,为张的密切合作者之一。中东路事件发生后,他被委派为东三省铁路公司理事长兼督办,以中国方面首席代表身份赴莫斯科参加中苏谈判。谈判取得成效,后因九一八事变,协议夭折。1937年1月7日,由北平抵南京之东北元老莫德惠、刘哲往见张学良,张认为西北战端将起,有违其「兵谏」之初衷;称中央军进入潼关系何应钦主谋,促莫、刘二人往见何应钦劝阻用兵;张并致函杨虎城称,「此事仍有转圜办法,切盼勿发生战事」。抗战时期,1937年任国民政府国民参政会主席团主席。1945年10月任东北宣慰使、东北救济会副会长。1946年任制宪国民大会代表。1947年,任国民大会代表。1948年任政府宪政督导委员会会长。1949年随政府转进台湾,同年3月21日起任行政院政务委员。1954年8月任考试院院长达12年。任内曾着手改革考试制度与技术,推行职位分类,倡导考用合一,创办、改进公务员保险制度等。1966年5月转任光复大陆设计研究委员会副主任委员。同年6月20日任总统府资政。张学良被幽禁大陆期间,莫德惠曾多次探望过张学良。张学良到台湾后,莫依然是张家的常客。1968年4月17日,莫德惠逝世于台北,终年85岁。编订有《双城莫德惠自订年谱》。","text2":"莫德惠是哪里人?","label":1} {"text1":"《十国春秋》,共114卷,清人吴任臣编撰纪传体取书。《十国春秋》,写十国君主之事迹,采自五代、两宋时的各种杂史、野史、地志、笔记等文献资料,计有吴十四卷,南唐二十卷,前蜀十三卷,后蜀十卷,南汉九卷,楚十卷,吴越十三卷,闽十卷,荆南四卷,北汉五卷,十国纪元表一卷,十国世系表一卷,十国地理表二卷,十国藩镇表一卷,十国百官表一卷。康熙八年(1669年)完成。洪亮吉《北江诗话》卷一载:“吴任臣撰《十国春秋》,搜采极博。”《越缦堂读书记》日记曰:“此书三过阅矣,丙辰(1856)读之尤细,甚薄其体载之疏;至壬申(1872)复阅,始叹其博不可及也。”《四库全书总目提要》评:“任臣以欧阳修作《五代史》,于十国仿《晋书》例为载记,每略而不详,乃采诸霸史、杂史以及小说家言,并证以正史,汇成是书。”吴任臣自序曰:“任臣以孤陋之学,思取十国人物事实而章著之,网罗典籍,爰勒一书,名曰《十国春秋》,为本纪二十,世家二十二,列传千二百八十二。人以国分,事以类属。又为《纪元》、《世系》、《地理》、《藩镇》、《百官》五表,总一百一十四卷。虽世远人湮,书册难考,乃鉴观诸邦,略得而论。……书成,聊著纂述之大指如此。康熙八年(1669年)己酉孟夏,仁和吴任臣撰。”又有周跋:“余校刊吴氏《十国春秋》,附刻《拾遗》、《备攷》二卷,锓板发十方后,复补录数条,亦未印行,年末采摭旧闻,则记载沿有阙。”乾隆五十三年(1788年)四月,周昂重刊《十国春秋》。一至十四卷十五至三十四卷三十五至四十七卷四十八至五十七卷五十八至六十六卷六十七至七十六卷七十七至八十九卷九十至九十九卷一百至一百三卷一百四至一百八卷一百九至一百十四卷","text2":"《十国春秋》一共多少卷?","label":1} {"text1":"高永是台湾的漫画家及作家。出生于台中县大甲镇(今台中市大甲区)。身高170cm,体重55kg,血型O型,星座巨蟹座。毕业于国立政治大学法律学系。代表作为《梵天变》、《星座刑事》、《隋唐英雄传》。1987年政大道南文学奖小说类首奖得主,1987年以《焚梦》出道于《欢乐漫画》,作品画风优美细腻、人物美形,作品常出现星座、占星、塔罗牌等奇幻神秘的主题,而《梵天变》则是以佛教为体材,在政大法律系之所学也展现在《星座刑事》中。喜欢的漫画家有萩尾望都、安彦良和、井上雄彦、池上辽一等。2005年任台北市漫画从业人员职业工会第三届理事长。漫画家冠良为其弟子,后期高永的作品大多与冠良合作创作。以创作《百无禁忌 Miss阿性》著名的漫画家李勉之(Amin)曾任高永的助手。从就读高中就开始进行漫画创作,1981年参加第三届小咪漫画新人奖以《罪与罚》获得第3名,之后数次参加并获得小咪漫画新人奖的奖项。","text2":"他本来的专业是什么?","label":1} {"text1":"《赖小子》是由中国导演韩杰编剧执导,贾樟柯监制的一部电影。2006年2月21日上映。影片讲述了小县城里几个混混(晋语称赖小子)的生活。喜平、流流、二宝,是几个不务正业的青年,他们开着拉煤的卡车,经常四处捣乱。流流因一个女孩的事情,挨了女孩的追求者,也是同学男孩的打,觉得气不过,而叫喜平跟二宝报复。那男孩倒在血泊之中,他们以为闹出了人命,于是离开家乡走上逃亡的路。在逃亡过程中,二宝说自己要当黑社会老大,有一天,他偷拿了他们所有的钱,自己跑了。喜平和流流逃往阳泉的路上,二人一起抢劫出租车时,喜平开枪却不慎打死了流流。影片的最后,喜平回到家乡,发现被他们打的男孩却其实并没有死。2006年鹿特丹电影节最佳影片金虎奖2006年第30届香港国际电影节数码竞赛单元最佳影片银奖2006年第8届台北电影节竞赛单元2006年第8届印度新德里国际电影节竞赛单元2006年意大利米兰电影节竞赛单元","text2":"影片讲述了什么?","label":1} {"text1":"《武者回归》(英语:Returner),2002年上映的日本电影。是使用了大量电脑特技的科幻电影、动作片。主演是金城武和铃木杏。2084年的地球被叫做「dagura(达古拉)」的外星人侵略,人类面临灭绝的危机。在最后的人类反抗军基地被攻破之刻,反抗军战士、少女美里通过时间机器,超越82年的时空,来到2002年10月15日,消灭地球上最初降临的达古拉。阴错阳差下,她出现在一个人口贩卖交易的现场,职业杀手宫本为复仇而刺杀黑社会头目沟口的枪战中,导致前者刺杀失败。宫本将美里当作偷渡客带回家中,美里将自己的身份告诉宫本,希望宫本能够帮助自己找到并消灭第一只降临的达古拉,阻止它将外星大军引到地球。宫本不相信美里的故事,美里用超科技在宫本颈部贴了炸弹,让宫本被迫跟美里一起行动。两人开始了寻找达古拉的行动。与此同时,沟口受黑社会大佬指派探查不明飞行物的事情,在科学研究所发现了配有超科技强大火力的外星战机和在生的外星人。沟口决定独占外星战机和外星人,雇佣了武装佣兵要将其劫出。而宫本和美里也查探到了外星人在研究所的线索。两人潜入研究所准备杀死外星人,美里却发现外星人并不是达古拉。这时沟口和佣兵也赶到,双方火拼后,美里和宫本用计逃出。沟口劫出外星战机和外星人后要将宫本和美里灭口。两人一面逃避沟口的追杀,一面思考如何阻止战争。从外星人给出的信息中,两人猜测可能是沟口杀死了外星人,而导致战争爆发。两人遂出发阻止沟口的行动,但反而被沟口识破。激战之后,美里被沟口劫持,宫本利用美里留下的超科技武器最终打败了沟口和他的手下,救回了外星人。最终,外星人派出飞船来接应同伴,原来达古拉就是穿上了拟态战斗服的外星人。两人最终阻止了战争爆发,宫本也成功复仇,杀死了沟口。然而这时,由于未来被改变,美里的身体慢慢消失……","text2":"《武者回归》是哪一年上映的日本电影?","label":1} {"text1":"美眉校探()是一部美国青春剧及侦探剧,编剧是罗伯·托马斯。该剧2004年9月22日首播,在UPN播放两年,并在2007年5月22日结束。该剧由克莉丝汀·贝尔主演。该剧主角是韦罗妮卡,一位高中毕业生,她在侦探父亲的指导下成为了一名私人侦探。编剧托马斯原本将该书写成一本年轻人的小说,将一位男性设定为主角。章节(剧集)呈一种特殊的结构:韦罗妮卡在解决一些毫不相同的较小案件的同时还在试图解决一个贯穿一季的谜团。该电视剧系列的前两季包含一个贯穿一季的谜团,在这一季的第一集中被详细介绍并最终在结尾被解开。第三季的编排与前两季相比有所不同,主要集中在一个延续几集的谜团上。广受好评的第一季(22集)在美国获得了每集250万人的收视反响。该剧曾多次登上各种最佳电视剧名单,并多次获得各种奖项及提名。在播映期间,它曾获得两项卫星奖提名,四项土星奖提名,五项青少年选择奖提名,并在2005年美国电影协会年度电视节目中成为重点。在CW电视台2007年度预告中,CW电视台董事长宣布该片停播。","text2":"美眉校探在播映期间获得了哪些提名?","label":1} {"text1":"尚真(;)是琉球国第二尚氏王朝第三代国王、第九代琉球国王,1477年(成化十三年)至1526年(嘉靖五年)在位。他是第二尚氏王朝第一代国王尚圆王之子。童名真加户樽金(),神号于义也嘉茂慧()。尚真未即位时被封为久米中城王子,1477年接受叔父尚宣威的禅位,并派遣长史梁应、使者吴是佳、通事梁德等至明朝,以尚圆王讣告,请求袭封为琉球国王。明宪宗遂在1479年(成化十五年)派兵科给事中董旻、行人司右司副张祥前往琉球,册封尚真为王。尚真王在位期间,不断进行对外扩张。1500年平定了八重山群岛的远弥计赤蜂之乱。1506年相继征服了久米岛按司势力和具志川按司势力,在1522年镇压「鬼虎之乱」,征服与那国岛。此后琉球的统治势力扩张至宫古、八重山,基本确定了琉球南部的版图。尚真王统治的50年被认为是琉球历史上最强盛的时期,确立了琉球的官员品秩、朝仪制度、神官制度、赋税制度、行政划分,扩建了首里城,废除了殉葬习俗,并于1526年召各按司赴首里居住,禁止私人拥有兵器,加强了中央集权,此后琉球进入稳定发展的时期。琉球群岛著名的建筑圆觉寺(1492年)、玉陵(1501年)、圆鉴池(1502年)、辩财天堂(1502年),都建于尚真王时期。用于国王处理政务、接待册封使的首里城北殿(1508年),也是在这一时期建造的。","text2":"尚真在位期间琉球势力扩张至什么地方?","label":1} {"text1":"李呈瑞,1912年出生。江西兴国人。1955年被授予海军少将军衔,荣获二级八一勋章、二级独立自由勋章和一级解放勋章。1930年参加红军,历任红军兴国模范师第三团排长,红三军团政治部直属政治处组织干事,红一军团4师12团连指导员,红一军团政治部直属政治处特派员,并参加长征。抗战中任抗大政治指导员、2团政治处主任,抗大总校政治部组织部干部科科长。第二次国共内战中任冀热辽军区冀中纵队政治部组织部部长,晋察冀军区3纵7旅副政委、炮兵旅政委,华北军区炮兵政委。建国后陆军第68军政治部主任、副政委兼政治部主任、政委,志愿军第68军政委、海军航空兵部首任政委,其部曾出现故事影片《打击侵略者》、《奇袭白虎团》等的题材。1953年任战俘解释团团长并参加板门店谈判。是中国海军航空兵的创建人之一。1967年逝世。","text2":"李呈瑞在抗战中担任过哪些职位?","label":1} {"text1":"澳门巴士2A路线是一条由澳门公共汽车有限公司经营,往返东方明珠和亚马喇前地的巴士路线。受到发夹弯弯位影响,巴士公司不可派出10米或以上巴士行走。2013年8月9日前:2013年8月9日起,本线可算是黑沙环一带营运较差路线,虽有辅助2路线功能,但其班次非常稀疏(与2路线和其他对手相比),而且竞争对手班次往往比本线频密,加上12路线比本线更快和更方便乘客,另外本线亦没有任何\"独市位\",令客量十分低迷,新福利时期更派出三菱Rosa以节省资源,离峰时段大部份班次都是吉车游街收场。部分乘客更于离峰时段情愿等12路线都不愿乘搭本线,尖峰时段则有较多乘客想有位坐,但只巧遇本线才上。","text2":"本线是否有任何\"独市位\"?","label":1} {"text1":"王夫人(),琅邪(国治今山东省临沂市北)人,吴大帝孙权妃嫔, 吴末帝孙皓父孙和之母。孙皓即位后,追尊王夫人为大懿皇后。《三国志》中称之为吴主权王夫人。王夫人被选入宫中,受到孙权宠幸,于黄武三年(224年)为孙权诞下三子孙和,孙权当时对王夫人的宠爱程度,仅次于步夫人。孙和也因为由王夫人所出的缘故在诸子中特别得到孙权喜爱。赤乌五年(242年),孙权立孙和为太子,并计划将王夫人立为皇后。但因孙权爱女全公主向来厌恶王夫人,屡次在孙权面前对王夫人加以毁谤。后来有一次,孙权生病,全公主散播谣言说王夫人见到孙权卧病后面露喜色,孙权因此发怒,最后王夫人忧患而死。后来,孙和在赤乌十三年(250年)被废黜,三年后孙和妻子张妃的舅舅诸葛恪被杀,孙和被认为有和诸葛恪串通复立的意图被孙峻赐死。元兴元年(264年)七月,孙和子孙皓被拥立为帝,后追尊祖母王夫人为大懿皇后,并将王夫人的三个弟弟封为列侯。王夫人为人有嫉妒之心,孙和被立为太子后,王夫人以孙和之母的身份母以子贵主持事务,其它被宠幸的姬妾,都被迁到外地居住,孙休的母亲吴主权王夫人也包括在内。","text2":"王夫人是什么地方的人?","label":1} {"text1":"台东间吸鳅(学名:)又称台东间爬岩鳅,为辐鳍鱼纲鲤形目爬鳅科间吸鳅属的鱼类,俗名台东间爬岩鳅。本鱼分布于台湾中央山脉以东之各河中上游等。该物种的模式产地在台湾。 被IUCN列为次级保育类动物。本鱼体纵扁延长呈圆筒状,腹部扁平。眼小,口下位。胸鳍颇为宽大,向两侧水平延伸,腹鳍亦呈水平延展,腹鳍内侧互相接近,基部具膜互相愈合,而其末端未完全相连。体呈青灰色或灰褐色,体背侧及头部具不规则虫蚀状淡色。偶鳍具有近圆形的白斑,背鳍具有3条灰黑色条纹,尾鳍具有3至4白色波状横带。体长可达10公分。本鱼为初级淡水鱼,栖息于溪流中、上游水势湍急的高溶氧区。为底栖性鱼类,平时平贴于石头上,属杂食性,以岩石附生藻类和水生昆虫为食。不具食用性。","text2":"台东间吸鳅属于什么种类?","label":1} {"text1":"皇家资产管理局 (Thai: สำนักงานทรัพย์สินส่วนพระมหากษัตริย์) 是 泰国 政府机关负责管理泰国国王及其家属财富的部门. 现任国王是普美蓬国王. 根据1936年皇家资产修正案第八款(พระราชบัญญัติ จัดระเบียบทรัพย์สิน ฝ่ายพระมหากษัตริย์), 皇家资产管理局(CPB)享受免税特别待遇. CBP在2004年的资产超过五十亿铢. 每年的CPB财务报告会单独提交给泰国国王. CPB目前由Chirayu Isarangkun Na Ayuthaya领导. CPB房地产有限公司和CPB资产管理有限公司都是该局的分支, 其领导是Michael David Selby. 在八十年代晚期, CPB有600余名雇员, 其中90%的人员从事该局不动产的管理.CPB根据宪法设立但独立于泰国政府. 普美蓬国王积极参与了该局的管理.","text2":"皇家资产管理局可以享受什么特殊待遇?","label":1} {"text1":"珍·哈露(,),活跃于30年代的美国著名女演员及被世人公认的性感女神。原名哈莉安·哈露·卡朋特·(Harlean Harlow Carpenter)。这名性感女星年仅26就去世了,不过即便在她去世后她的传奇依然在继续,包括玛丽莲·梦露在内的许多人将其视为自己的偶像。珍·哈露出生在密西西比州的堪萨斯市,是家中的独生子,她的童年生活非常奢侈。当她还是孩子时就加入了她母亲公司的社交圈,她的外表看上去非常成熟。1923年,当她12岁时母亲送她去了好莱坞的女子学校,希望她能成为一名女演员。她16岁的时候就嫁给了第一任丈夫,但她丈夫两年后就同她离婚了,并且留给了她一大笔分手费。结婚后她去了洛杉矶,在母亲的支持下,她同20世纪福克斯公司签下自己的第一份合同。珍·哈露出演第一部电影的时候每天可以赚取7美元的报酬,在经历过许多小角色之后她成为了一名明星。后来她加入了米高梅公司,在拍摄红头发女人(Red-Headed Woman)时她的片酬已经涨到了每周1250美元。在第三次婚姻结束后,珍·哈露认识了演员威廉·鲍威尔,两人的关系进展融洽,相处了2年时间几乎到了谈婚论嫁的地步。不过,因为女方想要孩子而男方不同意,两人最终没能步入婚姻的殿堂。哈露去世后,鲍威尔虽然没有丈夫的名分,他还是为女方支付了葬礼的账单。而且在埋藏哈露的教堂的地下室内,还为鲍威尔留下了一间空的墓室,只不过那个墓室始终没有投入使用。1937年,珍·哈露终于承认她长期遭受肾脏疾病的折磨,她的症状包括腹痛、呕吐、疲劳等等。不过,30年代的时候医生对肾病的认识不足,因而认为这些并不是非常严重的情况。1937年6月7日,在拍摄一部影片的时候她突然倒地,送往医院后不治身亡,这一年她只有26岁。","text2":"珍·哈露与威廉·鲍威尔为什么没有结婚?","label":1} {"text1":"里卡多·阿尔贝托·马蒂内利·贝罗卡尔(,),巴拿马政治家和企业家,前巴拿马总统。1952年3月11日,马蒂内利出生于巴拿马城。他曾获美国阿肯色州大学的工商管理学士学位和中美洲工商管理研究所的工商管理硕士学位。马蒂内利拥有大型连锁超市、银行、电视台、进出品公司及工厂等,是巴拿马知名的企业家。1998年,马蒂内利创立民主变革党。2004年,马蒂内利作为民主变革党的总统候选人参加总统选举,但败于民主革命党的总统候选人马丁·托里霍斯。2009年,马蒂内利再次参选总统,这次是代表民主变革党、巴拿马主义党等政党组成的联盟参选,在5月3日举行的总统选举中,马蒂内利击败执政党民主革命党总统候选人巴尔比娜·埃雷拉,当选为新一任巴拿马总统。作为中华民国在中南美洲重要友邦,但马丁内利任内的巴拿马被指政治立场摇摆不定。除了不时采取与中华民国相反的政治立场,在收取中华民国的经济援助之余,另一边又接受中华人民共和国的好处。","text2":"里卡多·阿尔贝托·马蒂内利·贝罗卡尔曾获什么学位?","label":1} {"text1":"瑞士经济是世界最为稳定的经济体之一。 其政策的长期性、安全的金融体系和银行的保密体制使瑞士成为避税投资者的安全避风港。瑞士是世界上最为富裕的国家之一,人均收入处在世界最高行列,同时有著很低的失业率和财政赤字。由于拥有发达的金融产业,服务业在瑞士经济中也占有日益重要的地位。瑞士是一些重要国际政经组织,如联合国、WTO、IMF、世界银行、OECD的会员,WEF「全球竞争力报告」,瑞士长期高居总排行世界第一。当附近欧洲国家失业率飙到20%以上,连德国失业率也达8%到10%间,瑞士失业率约3%,堪称欧洲最低。主要建立于精致工业品出口和观光业之上,但令人争议的秘密银行制度也让许多富人定期前往瑞士办理财产事务顺便进行高档旅游,人才政策采取全面开放,境内有多达124.9万名外籍工作人口,占整体就业人口超过1∕4,多数是高技术人士,并重视师徒制的技职教育体系,全国已发展出职训课程(VET)及产学专班(PET),通常是每周在校二天、工厂三天的师徒制训练。","text2":"为瑞士提供了大量工作岗位的产业主要是什么?","label":1} {"text1":"卢西亚诺·斯帕莱蒂(,),是意大利足球主教练。史巴列堤于1959年出生于意大利佛罗伦斯省,球员时代是在家乡球会出道,司职中场。。然而综观其球员生涯,史巴列堤都是在一些意丙球会效力,因而从未入选国家队。1991年他回到恩波利,并在该球会踢至退役。史巴列堤是于恩波利展开其教练生涯。起先是以球员身份执教,后转任青年军教头,再于1995年作为正式主教练。其间他带领了恩波里从意丙升上意甲。之后再辗转执教森多利亚、威尼斯、乌迪内斯等球会。其中在乌甸尼斯期间,史巴列堤还带领球会打入欧冠杯。2005年他接掌罗马帅印。这时正值罗马的混乱时期,曾先后起用四个教练,然皆下野告终。为使球队注入生气,史巴列堤乃改变战术,采用主攻踢法,但不设前锋位置。结果罗马终于回复生气,并于2005-06年以联赛第五名完成球季。史巴列堤执教罗马期间,意甲基本上都是由国际米兰垄断,然而史巴列堤也有交出成绩:2006-07年球季赢得意大利杯;翌季也蝉联成功。而在联赛罗马也经常夺得亚军而具有欧冠杯参赛资格。不过罗马始终财力有限,相比联赛前列分子,班底较为薄弱。2009年罗马出现财政危机,在意甲首两场联赛更遭逢两连败,结果史巴列堤主动提出辞职。2009年12月史巴列堤加盟俄罗斯球会辛尼特,签约三年。在其执掌教鞭的第一个球季,即率领辛尼特夺得联赛和杯赛的双冠王;接下来的第二个球季,又率球队夺得超级杯赛的冠军以及卫冕了俄超联赛冠军。直到他掌印的第三个球季,辛尼特花费巨资引进了浩克,维特塞尔等球星,希望借此在欧洲冠军联赛中大展宏图,但是该季球队反而在联赛中被莫斯科中央陆军反超仅夺得亚军。在该球季的欧冠联赛中亦在第一轮的淘汰赛中被德甲劲旅多特蒙德击败。2014年3月11日,辛尼特官方宣布与史巴列堤提前解除合约,其后史巴列堤没有执教任何球会。直至2016年1月,再次执教罗马,直至2016\/2017球季离开,执教国际米兰。","text2":"为使球队注入生气,史巴列堤用了什么方法?","label":1} {"text1":"尹福(),绰号「瘦尹」、「尹铁镯」(铁镯尹)。清代直隶省(今河北)冀县北漳淮村人,董海川之徒,清末八卦掌名家,开创尹氏八卦掌。尹福幼年因家乡盗匪为祸,流落至北京生活,到剪刀行当过学徒,后以卖油条、麻花卷度日。原习罗汉拳、弹腿等,师从著名镖师秦凤仪。在秦凤仪死后,拜肃王府太监董海川为师,因入门最早,为其大弟子。尹福天资聪颖,事师以礼,时时跟随在董海川身边,师兄弟皆对他十分佩服。艺成后以护院为生,曾任「崇外税务司」巡检、「善扑营」教习,后经满人佛音尼布(汉名叶诗梦)举荐供职皇宫担任侍卫,宫女、太监皆称其为老师。因为身形瘦长,外表斯文,而有「瘦尹」的绰号。他的掌力惊人,人一旦被他扣住就无法脱身,所以又被称为「铁镯子」、「尹铁镯」。他身处皇宫之中,行事低调,弟子多为皇宫侍卫,他也负责教导许多皇族成员武功。其师弟程廷华,在北京城中,与江湖人物来往密切,门徒遍满北京城,称为程派八卦掌,形成八卦掌的两大传承。当时又称尹福为宫内派,而程廷华为宫外派。尹派的特点主要是手掌平放,又称牛舌掌。掌势较少,较为朴实无华。另一个特色,则是以罗汉拳为主要入门。其长子尹世庭早逝,长女尹金玉嫁给其徒何金奎,三子尹成璋、四子尹玉璋皆承家学,擅长八卦掌。他的弟子有宫宝田、杨名山、马贵、居庆元、李永庆、马世清、门宝珍、钟声、金增启等人。","text2":"为什么尹福有“瘦尹”的绰号?","label":1} {"text1":"P. 1000 巨鼠重型巡航坦克(\"Landkreuzer P. 1000 Ratte\")是纳粹德国计划开发的一种重量达1000吨的超重型坦克,由德国的克虏伯公司研制,但计划在1943年被放弃,没有一辆P-1000被生产出来。此超重型坦克非常巨大,高度有11米,炮台也有两个平常人的高度。1942年初,德国武器局第5处计划在“大西洋防线自行280毫米海岸炮”计划的基础上开发P-1000超重型坦克。当时纳粹德国的军事发展以及力量已经达到了巅峰程度,阿道夫·希特勒本人在开始的时候对这种研发超重型坦克的项目也很感到兴趣,他觉得坦克愈大就愈强劲,并完全没有考虑到很多实际问题,就批准了由德国的克虏伯公司承担研发超重型坦克的工作。1942年6月23日,在克虏伯公司由Grote博士作主要指导,及Hacke博士协助,研发超重型坦克的研究开发工作正式开始。研发这一辆超重型坦克的研发代号为P-1000,称为「巨鼠」坦克(Ratte),如果研发完成后,设计重量可高达1000吨。同年该公司完成了P-1000「巨鼠」超重型坦克的设计书,并向希特勒汇报。他也很欣赏这个研发计划,并批准研发。在研发超重型坦克过程上,通常要考虑很多要点,例如在不同的地面或地形上,这种庞然大物是否会陷入沙地或泥地,地势、经费、运输、重量大小、零件可靠性、机动性、驾驶技术、攻击力以及研发的技术等等也没有考虑清楚。超重型坦克因为行动不方便,例如缺乏可以承托它们的桥梁,而它也会把公路压坏。而且P-1000的体积使它极其容易被轰炸机摧毁。最后,它的成本相当于80辆黑豹坦克,建造时间更是天文数字。P-1000超重型坦克的研发在1943年被军备和战时生产部长阿尔伯特·斯佩尔终止,没有一辆P-1000被生产出来。车体长度为35米,宽度是14米,高度是11米,为了承受本身的巨大重量,坦克每侧的履带有3.6米宽,由3条1.2米宽的分履带组合而成。P-1000超重型坦克动力系统是使用8台Daimler-Benz MB501型20缸的柴油发动机,一台柴油发动机马力为2000匹马力,输出动力可达到16000匹马力,就可以推动到这架超重型坦克,而行走最高速度是40公里/小时。P-1000超级坦克3D游戏观看影片(战地风云1942 模组)","text2":"P-1000巨鼠重型巡航坦克由哪个公司研制?","label":1} {"text1":"坦佩谷()是希腊色萨利大区北部一个峡谷的古名,它位于奥林帕斯山以南,俄萨山以北,被希腊诗人誉为“阿波罗和缪斯喜爱的去处”。山谷长10千米,狭窄处仅宽25米,两旁山崖高近500米,匹尼奥斯河于其间蜿蜒而过流向爱琴海。皮尼奥斯河右岸伫立着一座阿波罗神庙,周围月桂丛生,古时人们用它做成桂冠嘉奖皮西安竞技会的得胜者。坦佩谷也曾一度是阿波罗与库瑞涅之子阿里斯泰俄斯的居所,他也正是在此处追赶俄尔甫斯之妻欧律狄刻,以致她在逃走时被蛇咬死。公元13世纪山谷里建起了一座敬献给圣帕拉斯凯韦的教堂。坦佩关是希腊的战略关隘,因为它是从拉里萨穿山到达海岸的主要路线,也正是由于这一点,从古到今许多战役都在这里打响。如要避开此关,也可取道稍迂回的撒伦多普罗关。公元前480年,一万雅典人与斯巴达人试图在坦佩关阻止波斯国王泽克西斯一世的入侵,但波斯人取道撒伦多普罗从而绕开了希腊军队。公元前164年,在第三次马其顿战争期间,罗马人冲破马其顿国王珀尔修斯的防御,之后在彼得那战役中取得了胜利。公元前148年,安德里斯库斯的另一场斗争也在这里上演。无论在标志着罗马时代在希腊的终结的异族袭击中,还是在拜占庭和奥特曼时代,这里也不乏争战。今天对于大多数希腊人来说,坦佩也以其十分恶劣的路况和可怕的交通事故而臭名昭著。例如,在2003年,由于所乘客车与一辆满载木材的货车相撞,二十一名来自马克拉荷里的十一年级一个班的中学生全部死亡。尽管货车司机已尽力在几秒钟的时间内减速,但木材带着巨大冲力从货车后部飞出,从客车左部砸进,当场导致许多学生死亡。美国亚利桑那州的坦佩和澳大利亚新南威尔士的坦佩市都是由此命名。南非东开普省的一个农场也叫坦佩,还有着两个名为奥林帕斯和俄萨的农场邻居。另外,马来西亚槟城沿着一条小峡谷而设的坦佩谷路也是以它命名。","text2":"坦佩谷在什么地方?","label":1} {"text1":"《星期日泰晤士报》(),英国的一份于每周日出刊的报纸。《星期日泰晤士报》也有出版爱尔兰共和国的版本,不过其与《爱尔兰时报》没有直接关联。《星期日泰晤士报》由时报报业公司(Times Newspapers Ltd)出版,而该公司又是新闻集团的下属企业。《星期日泰晤士报》创刊于1821年,最初的名称是\"The New Observer\"(《新观察家》),不过这家报纸和《观察家报》(\"Observer\")没有任何关系,后更名为\"The Independent Observer\"(《独立观察家》)。1822年再更改为现在的名字。1981年,《星期日泰晤士报》被传媒大亨默多克的新闻集团收购。","text2":"哪一年《星期日泰晤士报》被收购?","label":1} {"text1":"黄宗仰(),俗名黄浩舜,别号乌目山僧,笔名黄中央。中国民主革命家、建筑设计师、佛教僧人。1865年,黄宗仰出生于中国江苏省苏州府常熟县商人家庭,1880年在常熟清凉寺出家,1884年在镇江江天寺受戒。1899年黄宗仰前往上海。1901年,受犹太富商哈同的华籍夫人罗迦陵聘请,设计建造爱俪园,并在其内讲授佛经。1902年4月,黄宗仰与章太炎、蔡元培等联合组建“中国教育会”,开始参与政治活动。1903年6月苏报案发,邹容和章太炎被上海公共租界工部局逮捕,黄宗仰多方营救章太炎未成后,逃亡日本横滨,在那里结识孙中山,并出资帮助他得以前往檀香山。1904年黄宗仰回上海。1914年,黄宗仰离开爱俪园,回到镇江任江天寺首座。1920年,黄宗仰募集巨款,使毁于太平天国战争的南京栖霞寺得以恢复。1921年7月,黄宗仰在栖霞寺圆寂,安葬于栖霞山,年56岁。","text2":"黄宗仰的身份是什么?","label":1} {"text1":"苏保罗(Paul O'Sullivan、),人称「阿苏」、「Paul苏」,是纽西兰出生的练马师,现时于香港从练。苏保罗1976年开始练马事业,其父是纽西兰传奇练马师苏礼云,胞弟苏利云为骑师,他与父亲共同名义训练马匹,其胜出的主要大赛有:1989年凭好利时赢得日本杯、1986年「Our Waverley Star」赢日本富士锦标。此外亦凭滑浪天堂赢得1991年觉士盾。曾胜出澳洲多项一级赛,而在纽西兰,差不多所有大赛都赢过,曾经训练出三匹纽西兰全年度马王以及十一匹不同组别的冠军赛驹。于纽西兰期间,总共有11年成为了练马师冠军。1998年,其父苏礼云光荣退休,苏保罗独立展开其练马事业,夺得2002-2003年度纽西兰冠军练马师。2004-2005年马季开始,苏保罗获得香港赛马会发给练马师牌照,展开于香港的练马事业,家族在纽西兰的事业,便交由胞弟苏利云接管,并于2004年10月30日,凭同欢畅团体名下的「领奖」取得在港从练首场头马。2006-2007年马季成为亚军练马师,2007年凭活力金刚赢得香港打吡大赛,赢得自来港开仓以来第一个分级赛冠军。2010-2011及2011-2012年度,马房成绩陷入低潮,之后成绩渐见起色。2013-2014年度马季,苏保罗训练的「友莹格」取得一季五连捷佳绩。2015年3月29日,苏保罗凭「友莹格」为香港夺得首次高松宫纪念赛冠军,扬威日本。5月17日,苏保罗凭「友莹格」为香港赛驹连续三届胜出KrisFlyer国际短途锦标冠军,扬威新加坡。苏保罗在2014-2015年马季于港夺得39场头马,并位列练马师榜的前六名。2015年11月14日,苏保罗凭「尚华盛甲」取得在港从练的第三百场头马。2015-2016马季成绩突出,取得50场头马位列练马师榜的第五名。","text2":"在纽西兰期间,有多少年获得练马师冠军?","label":1} {"text1":"民主共和党()是美国建国早期的一个政党。由美国开国元勋汤玛斯·杰佛逊和詹姆斯·麦迪逊在西元1789年创建。民主共和党的对立政党是联邦党(又叫做联邦同盟党)。与联邦党相比,民主共和党更强调各州的权力(States' rights),并且重视自耕农的权益,反对过份的联邦主义和君主主义。主要支持地区是中西部和南部。支持者大部份都是乡村地区的农民还有城市地区的工人,民主共和党可被视为今日美国两大党派中民主党的前身。1789年,美国开国元勋汤玛斯·杰佛逊和詹姆斯·麦迪逊创建美国民主共和党。他们因为反对联邦党的领袖、时任美国财政部长的亚历山大·汉密尔顿的经济政策,退出联邦政府,成为反对党。1796年的美国总统选举,民主共和党领袖汤玛斯·杰佛逊输给了联邦党候选人、时任美国副总统的约翰·亚当斯。但由于汤玛斯·杰佛逊是该次选举得票数第二多的候选人,因而成为副总统。但在随后宪法修正使总统和副总统分开改选,使总统的对手不会成为副总统。1800年的美国总统选举,汤玛斯·杰佛逊击败寻求连任的约翰·亚当斯,终于当选总统,并于西元1804年,获得连任。之后,民主共和党人詹姆斯·麦迪逊和詹姆斯·门罗赢得之后的四次美国总统选举,使得民主共和党连续执政长达24年,也使得主要竞争对手联邦党瓦解。1824年的美国总统选举,由于民主共和党未能推派单一候选人,四名民主共和党人同时参选,最终普选票和选举人票为四人中最多的安德鲁·杰克逊在众议院选举中未能当选,而是由亨利·克莱支持的约翰·昆西·亚当斯当选总统,而亨利·克莱则获约翰·昆西·亚当斯任命为国务卿,此一选举结果导致民主共和党瓦解。1828年和1832年的美国总统选举,分别由约翰·昆西·亚当斯、亨利·克莱与安德鲁·杰克逊对垒,约翰·昆西·亚当斯和亨利·克莱的支持阵营,被称为国民共和党,而安德鲁·杰克逊的支持阵营,以老共和党人(Old Republicans)为主,被称为民主党,今日的民主党以民主共和党的成立作为创党年份。","text2":"民主共和党的支持者主要集中在哪里?","label":1} {"text1":"黑鳍魣(学名:),又称黑鳍金梭鱼,俗名针梭、竹梭、巴拉库答 ,为辐鳍鱼纲鲈形目鲭亚目金梭鱼科的其中一种。本鱼分布于印度太平洋区,包括南非、东非、红海、塞席尔群岛、马尔地夫、台湾、日本、马来西亚、印尼、柬埔寨、澳洲、萨摩亚群岛、关岛、东加、马绍尔群岛、密克罗尼西亚、夏威夷群岛、新喀里多尼亚、法属波里尼西亚、墨西哥等海域。该物种的模式产地在日本长崎。水深40至70公尺。本鱼体延长呈鱼雷状,横切面几近圆柱形,侧线鳞列数100枚以上,无延长鳃耙数,腹鳍起点前于背鳍起点的垂直线;眼中等大,头长约为其6倍以上,胸鳍不具大黑斑,ㄍ形纹自体背到侧线脂略下方。腹鳍黑色,上颔骨未达眼前缘。背鳍硬棘6枚;背鳍软条9枚;臀鳍硬棘2枚;臀鳍软条7至9枚,体长可达170公分。本鱼栖息于潟湖与临海礁石区。成群活动,白天活动,游速快,属洄游性肉食鱼类,以鱼类、头足类等为食。为食用鱼,肉质鲜美,适合红烧、油煎或煮汤。","text2":"黑鳍魣作为食用鱼有什么优点?","label":1} {"text1":"中国耶稣教自立会是一个中国人自己创立的基督教组织,由中国牧师俞国桢创立于1906年。俞国桢原为浙江湖州新市镇长老会的中国牧师。1894年,美北长老会传教士范约翰休假回国,将其负责的上海汉壁礼路(今汉阳路)永祥里虹口长老会堂委托给俞国桢牧师。1903年,俞国桢在海宁路、克能海路(今康乐路)路口购地,次年建成沪北海宁路自立长老会堂,与差会断绝经济关系。1906年1月25日(农历正月初一),俞国桢发起成立中国耶稣教自立会。浙江平阳、湖北天门等地教会表示响应。于是在1910年,在上海成立中国耶稣教自立会总会,1915年,租用宝山路义品里258号为会所。1916年,在商务印书馆附近的闸北华界的宝通路340号新建较大的西式教堂,取名为闸北自立长老会堂。1920年,中国耶稣教自立会举行第一次全国代表大会,成立全国总会,包括中国16个省的189所礼拜堂,信徒2万余人,俞国桢被选为终身会长。1924年,俞国桢与长老会因闸北堂归属问题争执不下,最终俞辞职离开,闸北堂又回到长老会。1926年,俞国桢另外在江湾翔殷路建造永志堂作为自立会全国总会会所。1929年建成,1930年7月启用。随着中国民族主义运动的退潮,自立会的影响也有所下降。1932年一二八事变爆发,江湾为战区中心,自立会总会事务所和俞国桢住所均毁于战火,永志堂屋顶被炮弹击中,损失惨重。俞国桢只得临时将自立会总会事务所设在施高塔路(今山阴路)四达里林鸿斌牧师住所。同年11月16日,俞国桢在浙江温州讲道时在那里病逝。1934年,翔殷路永志堂修复,总会事务所迁回永志堂。1937年8月13日,规模更大的淞沪会战爆发,江湾再次沦为战区,这时永志堂完全被毁。抗日战争期间,自立会总会与各地分会基本失去联系。1939年1月,永志堂租用大西路(今延安西路)108号房屋,恢复礼拜,自立会全国总会亦在此处办公。1940年1月,总会临时办事处迁往海格路(今华山路)237弄1号谢永钦家中。1946年秋,谢永钦牧师捐出闸北宝通路539号土地,供重建永志堂之用,信徒也纷纷捐款。1947年,宝通路永志堂建成。1954年以后,在自立会全国总会动员下,散布于中国各地的数百处自立会陆续参加了三自爱国运动。1958年,上海宝通路永志堂参加闸北区联合礼拜,并入了闸北堂。此后自立会不再作为一个独立的教派而活动。","text2":"中国耶稣教自立会的创立者是谁?","label":1} {"text1":"《ICO》(中国大陆官方译为「古堡迷踪」,台湾又译「迷雾古城」)是由ICO小组制作的游戏。主创者上田文人承担了大量开发工作,开始在玩家群体中获得较高的声望。游戏原计划运行于初代PlayStation,后因受到机能限制转而成为PlayStation 2独占游戏。游戏开发历时四年,团队特别雇佣了一个“裁剪小组”,致力于简化游戏中不必要的元素,以增强游戏的代入感。《ICO》在剧情讲述和动画制作方面都有出色的表现:用极少的对话讲述了令人回味的剧情,用高光和关键格等技术制作了比同时代游戏先进的动画,影响了后来的很多游戏作品。《ICO》于商业上未成功,却因其独有的艺术元素(如充满古典氛围的古堡室内场景)成名,获得多个“年度游戏”和“游戏设计师选择”奖项或提名。《ICO》也常在许多游戏榜单中占据前席,被认为是一款出色的艺术品。2011年,《ICO》和《旺达与巨像》经过高清重制后于PlayStation 3平台发行。其中日本和香港地区有上述两个游戏的独立零售版发行,这种版本沿用了经典的原版封面。游戏的主角是名为ICO、头上长着两角的男孩;被视为不祥之兆的他被族人囚禁于一个古堡中。侥幸逃脱的他偶遇了白衣女孩Yorda,古堡女王的女儿。女王意图以Yorda为祭品,延长自己的寿命。因此ICO带领Yorda一同寻找古堡的出口,以逃离厄运。在游戏中,ICO需要在逃离古堡的旅途中极力保全白衣女子,与意图将她抓回囚笼的黑影怪对抗,协助她通过各种障碍(通过推箱子为Yorda提供垫脚石,是极重要的游戏内容)。作曲・编曲:大岛满/作词・对译:Lynne Hobday/演唱:Steven GeraghtyIco发售后即好评如潮,在Game Rankings上达到了90%的好评度,2001年位列第15位(PS2平台上的第10位).其他:汪达与巨像最后的守护者","text2":"2011年,《ICO》和什么经过高清重制后于PlayStation 3平台发行?","label":1} {"text1":"高升泰()是北宋年间云南大中国皇帝,滇东白蛮领袖。大理点苍山莲花峰芒涌溪人氏。1080年,杨义贞弑大理皇帝段廉义,自立为帝。时为鄯阐(昆明)侯的高升泰受其父高智升之命率滇东乌蛮三十七部兵马攻灭杨义贞,拥立段寿辉为大理皇帝。1081年,高智升与高升泰父子逼迫上明帝段寿辉退位出家,接著便拥立段正明继位。高升泰封为善阐侯。宋哲宗绍圣元年(1094年),一直以来都在专权的高升泰废段正明,自立为帝,国号“大中”,年号上治。分封诸子弟:其弟高升祥的势力在善阐府(昆明),幼子高泰贤的势力在永昌府(保山)、高泰运的势力在腾冲府(腾冲)、高泰慧的势力在鄯巨府(永胜)。1096年,高升泰病逝。由于云南诸部的反对,高升泰临终嘱咐其子高泰明还政段氏,于是高泰明拥立段正明的弟弟段正淳为大理皇帝,但其后大理国柄掌握在高氏手中,世称高国主。高升泰谥号富有圣德表正皇帝。在金庸小说《天龙八部》中,高升泰是段正明、段正淳的亲近臣子和好兄弟,段正明将皇位传给段誉。小说中高升泰、段正淳并没有当皇帝。","text2":"哪一年高智升与高升泰父子逼迫上明帝段寿辉退位出家?","label":1} {"text1":"田乞,妫姓,田氏,名乞,谥僖,史记作-{厘}-,又称为陈僖子、陈-{厘}-子、田-{厘}-子。是齐国田氏家族的首领之一,为田氏家族第七任首领,承袭兄长田开担任田氏家族首领。父亲田无宇。历仕齐景公、齐晏孺子、齐悼公三代。兄长田开无嗣子,田乞得立。田乞向百姓征收赋税时用小斗收进,借给百姓粮食时用大斗,向百姓施以恩德,晏婴多次向齐景公进谏,齐景公却不干涉。晏婴出使晋国,暗地对晋国羊舌大夫叔向说:「田家的先祖箕伯、直柄、虞遂、伯戏和先君陈胡公、大姬已经来到齐国,接受祭祀了。齐国将要成为田氏的了。」田氏的支系子孙司马穰苴因战功被任命为大司马,因田氏势力扩大而警惕的高张、国夏对齐景公说穰苴的坏话,司马穰苴被免官。田乞立志要除掉高氏和国氏。前490年,齐景公死后、年少的公子荼即位为齐晏孺子,高张、国夏专权。前489年六月,田乞联合鲍牧等大夫攻撃高张、国夏,十月晏孺子退位后被暗杀。田乞拥立晏孺子异母兄公子阳生为齐悼公。田乞担任齐国的宰相,田氏势力更加稳固。田乞死后,儿子成子田恒继位。","text2":"田乞死后由谁继位?","label":1} {"text1":"Group Sounds(或 Group Sound)是日本1960年代后半期以结他为主、由数人组成的摇滚乐队种类,简称 GS。自1966年披头四到日本公演以后,一边唱演唱一边亲自演奏电吉他等乐器的乐队在日本不断冒起,以年轻读者为主的艺能杂志《周刊明星》把这些乐队和音乐通称为「Group Sounds」或「Group Sound」,及后被广泛使用。1960年代末期,GS的热潮急速减退,踏入新摇滚音乐年代,部份GS乐队也开始解散,而The Spiders、The Tigers和The Tempters解散后,部份成员于1971年组成日本首个「PYG」。现在,一般提及的Group Sounds都是指1960年代后半期活跃于咖啡店的流行曲乐队和摇滚乐队。另外,乐队以和制英语命名都是GS的特色。\"(粗体字标记为队长)\"","text2":"在今天,一般提及的Group Sounds都指的是什么乐队?","label":1} {"text1":"陈黄穗(本名黄穗,),是香港消费者委员会前总干事,1998年获颁铜紫荆星章;2004年获香港中文大学颁授荣誉院士名衔;2007年获美国消费者权益委员会颁授荣誉大奖。陈黄穗的父亲黄福銮是崇基学院-{史地系}--{系主任}-,而其夫陈应保亦是崇基史地毕业。她于1968年毕业于香港中文大学社会学系,毕业后加入国际培幼会(Foster Parents Plan),两年后转投戒毒会,先后在妇女戒毒中心和石鼓洲戒毒所工作。1973年婚后转职到中大社工系任实习导师,过了两年,正式出任小童群益会的执行干事。1979年获选香港十大杰出青年,当年只有5人获选,其余4人包括林巨津、靳埭强、孔梁少根及潘宗光。陈黄穗自1985年5月1日出任消费者委员会总干事,直到2007年退休,由刘燕卿接任。2010年,陈黄穗获财政司司长委任为香港存款保障委员会的主席,任期两年。","text2":"陈黄穗在2004年获得了哪所大学颁授的荣誉院士名衔?","label":1} {"text1":"姬猪(学名:\"\")是一种在印度次大陆生活的小型猪,是猪科动物中最小的一种,曾经遍布印度、尼泊尔和不丹的高湿草原地带。 然而人类的活动已经大大破坏了姬猪的自然栖息地,目前只分布在阿萨姆邦,总野生种群估计少于150只。姬猪约55-71cm长,20-30cm高,尾巴约2.5 cm,体重6.6-11.8公斤。皮肤棕黑色。小猪出生时是粉灰色,然后变为棕色底,黄色条纹。它们头尖长,额头和脖子后面有较长的毛。成年雄性上口部的虎牙伸出唇外。野生姬猪挖出一个浅沟,垫上植物为窝,白天燥热时躲在窝里。主食为草树根,昆虫,小鼠类和小爬虫。它们寿命约为八年,1-2岁时性成熟。台风季之前交配,孕期为100天,一胎3-6头小猪。牠也是姬猪虱的特定寄主。","text2":"刚出生的姬猪是什么颜色?","label":1} {"text1":"白兰氏(英语:Brand's),是一家食品公司的品牌,著名于鸡精饮料,产品已经有170年历史以上。白兰氏品牌由食益补太平洋有限公司(英文名:Cerebos Pacific Limited)所有,而食益补是新加坡交易所主版上市公司,在1990年已经由日本三得利公司收购其股权。在1820年,白兰氏鸡精由英国白金汉宫御厨韩温白兰(H.W.Brand)发明,之后白兰先生于1835年创立白兰氏公司,在英国设厂生产白兰氏鸡精。2003年,白兰氏健康博物馆于台湾彰化县鹿港镇彰滨工业区开馆,除了企业相关主题外,其鸡精制造工厂以「空中走廊」的方式,开放一般民众参观。鸡精是白兰氏的主要产品,除传统鸡精外,亦开发添加冬虫夏草、花旗参等传统保健药材的鸡精。白兰氏另开发多种保健食品,饮品类包括红胶原青春饮、燕窝、蚬精及养参饮等。锭类包括维他命、深海鱼油、五味子芝麻锭。","text2":"白兰氏品牌有那个公司所有?","label":1} {"text1":"斑条魣(学名:),又称竹针鱼,俗名针梭、竹梭、巴拉库答,为辐鳍鱼纲鲈形目鲭亚目金梭鱼科得其中一种。本鱼分布于印度西太平洋区,包括南非、东非、红海、模里西斯、马达加斯加、马尔地夫、巴基斯坦、印度、斯里兰卡、缅甸、孟加拉、泰国、马来西亚、中国、台湾、香港、越南、柬埔寨、印尼、新几内亚、澳洲、斐济、马里亚纳群岛、万那杜、新喀里多尼亚、帛琉等海域。该物种的模式产地在Vizagapatum。水深20至200公尺。本鱼体延长呈鱼雷状,横切面几近圆柱形,侧线在第一背鳍前略向上弯曲;第一背鳍起点在胸鳍末端的正上方,在腹鳍起点的后方。鳃盖具2弱平棘,上颔骨未达眼前缘。背部暗绿色,约有20个浅但直纹到达侧线略下方,腹部银白色。背鳍硬棘6枚;背鳍软条9枚;臀鳍硬棘2枚;臀鳍软条7至9枚,体长可达150公分。本鱼栖息于潟湖、临海礁石区或海湾,白天活动,独居性,属于洄游性肉食鱼类,以鱼类、头足类等为食。为大型食用鱼,肉质鲜美,适合各种烹饪方法食用。","text2":"斑条魣的模式产地在哪里?","label":1} {"text1":"南洋美银汉鱼(学名:),为辐鳍鱼纲银汉鱼目银汉鱼科的其中一种。本鱼广泛分布于印度西太平洋区,包括红海、东非、马达加斯加、马尔地夫、印度、安达曼群岛、菲律宾、印尼、越南、泰国、马来西亚、台湾、中国沿海、日本、韩国、澳洲、密克罗尼西亚、马绍尔群岛、马里亚纳群岛、索罗门群岛、萨摩亚、夏威夷群岛、斐济等海域。水深0至4公尺。本鱼体延长而细,略呈圆柱状,头及眼适度大小,头部无小棘列。前上腭骨较长,末端超过瞳孔前缘;其前上突起及侧突起短而宽,且前上突起之高度与宽度相等。下腭骨上方略呈弧形,其后部不急速高耸;前鳃盖骨之后缘具一明显缺刻。背鳍2枚,第一背鳍始于体背中央之略后方,具4至6棘;第二背鳍位于位于第一背鳍之略后方,具1棘极8至11软条;臀鳍与第二背鳍对在而略前位,具1棘极12至17软条;腹鳍亚胸位;肛门较前位,开于腹鳍末端之腹缘;尾鳍开叉。无侧线,体侧鳞为圆鳞,鳞片大,具5纵列,一纵列有39至44枚鳞片。两腭齿细小,绒毛状;口盖骨及锄骨均有齿。体呈蓝绿色或灰色。体长可达14公分。为海水鱼,白天经常大量群聚于近岸。大部分于夜间索饵,其食物包括浮游性鱼卵、甲壳类、有孔虫及桡脚类。食用鱼,也可作为海鸟及其他鱼类的饵料鱼。","text2":"此鱼主要以什么为食?","label":1} {"text1":"静脉注射是一种医疗方法,即把血液、药液、营养液等液体物质直接注射到静脉中。静脉注射可分短暂性与连续性,短暂性的静脉注射多以针筒直接注入静脉,即一般常见的「打针」。连续性的静脉注射则以静脉滴注实施,连接软喉的药水樽会倒挂于架上,俗称「(打)点滴」、「(打)吊针」、「输液」、「(打)吊瓶」、「输水」(中国大陆俗称「挂水」)、香港俗称「吊盐水」,为将留置针插入静脉后固定,然后接上可更换或补充的瓶装或袋装医疗液体。如果短暂性静脉注射的液体多于50毫升的话也会使用点滴的方式注入。也有吸毒者使用静脉注射来吸食毒品。静脉注射也是死刑的行刑方式之一。静脉注射可以按照插入的静脉类型来细分。周边静脉注射是注射于(手、手臂、腿、脚上的静脉)上。这是最常见的静脉注射方式。中心静脉注射线上有大型静脉导管,例如上腔静脉、下腔静脉,甚至心脏的右心房。由于这些静脉有很大的直径血流速度快,所以中心静脉线一般用于管理能刺激血容线的物质,例如化学疗法。有两种类型的扩容:晶体和胶体。晶体一般可以做成无机盐的水溶液。胶体包含有较大的难以溶解的分子,例如明胶。血产品是任何带有血液的产品,血液可能来自捐献者。输血可能有时是为了抢救生命,例如因外伤导致的大量失血,也可以是手术造成的失血。输血有时也用于治疗因血液疾病导致的贫血或者。","text2":"连续性的静脉注射在中国大陆又被叫做什么?","label":1} {"text1":"董山(),明朝建州女真人,猛哥帖木儿之子。1433年猛哥帖木儿死后,建州左卫由他的弟弟、董山的叔叔凡察掌握。1437年明朝授予董山指挥使一职,董山和凡察从此开始争卫印。为避兀狄哈野人,正统五年(1440年),凡察、董山率残部逃往今新宾县烟筒山,投靠了李满住。1441年,董山升任都督佥事。1442年,为了解决董山和凡察之争,明朝政府又增设建州右卫,升两人为都督同知,分别掌握建州左卫和建州右卫。他曾杀掠边民多达10余万。明成化二年(1466年)十二月女真入犯,明朝总兵郑宏战败。明成化三年(1467年)正月,海西、建州女真复入鸦鹘关(今新宾苇子峪镇三道关)。明都指挥史邓佐在双岭中女真人的埋伏战死。三月复入连山关,掠开原、抚顺,窥铁岭、宁远、广宁。李满住也被朝鲜擒斩。他还曾接受朝鲜授于的官职:清史稿(时董山阴附朝鲜,朝鲜授以中枢密使。巡抚辽东都御史程信诇得其制书以闻,英宗使诘朝鲜及董山,皆习服,贡马谢。然后五夜至义州江,杀并江收禾民,掠男妇、牛马)。明朝成化曾对建州女真进行过打击,被称为“成化犁庭”,此次军事行动共擒斩女真人1700余口。明和朝鲜军(康纯,鱼有沼)历经一个月的围剿,共契寨四五百座,并释放女真人的包衣千余人。《清实录》有关于充善()的记载,可能和董山是同一个人。根据该记载,充善有三个儿子:妥罗、妥义谟、锡宝齐篇古。因此,充善是福满的祖父,觉昌安的曾祖父,塔克世的高祖父,清太祖努尔哈赤的五世祖。根据当代学者的研究,这种亲缘关系可能是虚构的,福满和锡宝齐篇古可能都是杜撰出来的人物。","text2":"1437年明朝授予董山什么职务?","label":1} {"text1":"米诺地尔()是一种钾通道开放药,这类药物在降压时常伴有反射性心动过速和心输出量的增加。对于血管扩张的作用具有选择性,见于冠状动脉,胃肠道血管和脑血管,而不扩张肾和皮肤血管。它还可以减低或停止掉发并且促进毛发再生。现在一般的门诊病人不需要处方就可以取得,用于。米诺地尔本来是用于控制高血压。后来偶然发现有一个有趣的副作用,就是扭转或减慢秃头过程。Upjohn推出了含2%的米诺地尔的外用药液,用来治疗秃头及脱发。该药液在美国及加拿大以Rogaine名称销售,在欧洲及亚太区则使用Regaine名称,在台湾译为「落建」。5%浓度的药液专供男性使用,而2%浓度的药液则适合女性使用。在台湾贩售的含5%浓度的米诺地尔成分生发药液产品如:落建生发系列的落健生发液、萌发生发液等。米诺地尔的专利保护期在1996年2月11日已经到期。5%泡沫配方的米诺地尔,跟液体配方一样有效。米诺地尔促进毛发生长的机转目前还未完全了解。此药物的结构式中有一氧化氮基团且可能做为一氧化氮的致效剂。它同样是,造成细胞膜的去极化。对于大面积的掉发,米诺地尔就比较没有效果。另外,对于年轻男性有五年以下的掉发困扰,它有非常显著的效果。目前,米诺地尔只用于的掉发。它同样也是个血管扩张剂。理论上,借由扩张血管及开通钾离子通道,使更多的氧气及养分到毛囊,会促使毛囊从休止期到生长期。常见的副作用有对眼睛的烧灼感及刺激性,治疗部位搔痒、红肿及刺激性,非预期位置的毛发生长。如果使用者有以下的严重副作用,立即寻求医疗照护:严重的过敏反应(红疹、荨麻疹、搔痒、呼吸困难、胸闷等);胸痛;晕眩;火烧心;立即且非预期的体重增加;手或脚的胀大。酒精和丙二醇可能会使头皮干燥,造成头皮屑和。有些配方改用奈米液滴为的就是减少因为酒精和丙二醇所造成的接触性皮肤炎。米诺地尔对猫来说是致命的。","text2":"米诺地尔对于血管扩张具有怎样的特性?","label":1} {"text1":"九龙巴士42M线是香港一条来往长宏及荃湾(愉景新城)的巴士路线。全程:$4.7乘客登上本线往荃湾后150分钟内以同一张八达通卡转乘43X\/43P、48X、49X、73X或278X\/278P线往新界东,或从43X\/43P、48X、49X、73X或278X\/278P线往荃湾登车后150分钟内以同一张八达通卡转乘本线往长宏,次程可获$4.2车资折扣优惠。现时本线使用9辆富豪超级奥林比安12米(3ASV),均属青衣车厂。长宏开经:寮肚路、青衣西路、枫树窝路、担杆山交汇处、青敬路、长安巴士总站,担杆山路、担杆山交汇处、青荃路、荃青交汇处、德士古道、杨屋道、大河道、青山公路—荃湾段、西楼角路及美环街。荃湾(愉景新城)开经:美环街、西楼角路、大河道、杨屋道、德士古道、荃青交汇处、青荃路、担杆山交汇处、青敬路、长安巴士总站、担杆山路、担杆山交汇处、枫树窝路、青衣西路及寮肚路。本线自开办至今,客量一直处于高水平,客源以街市买𩠌客和荃湾逛街客为主。由于本线途经杨屋道街市,因此每天都有大量乘客在街市买𩠌后乘搭本线回家,乘客在早上七时左右已经开始出现,一直到街市关闸为止。另外,近来荃湾区发展加快,有多个大型商场进驻(荃新天地、如心广场等),加上本线途经该地域,因此也吸引到这班乘客。当然,一般来往青衣和荃湾工作或逛街的乘客,为本线增加了不少客源。即使近来有新开办的特快小巴路线409和409K跟本线竞争,但本线早已深入民心,加上小巴的车资较本路线昂贵,服务地区又不太相同,本线的客量一直都持续高水平。与此同时,虽然九龙巴士41M线的服务范围跟本路线相若,但本线在青衣的服务范围较广,因此客量并未受太大影响。","text2":"九龙巴士42M线往返于哪两个站之间?","label":1} {"text1":"安达·云丹(,名字意为“母亲坚持(所立的人)”;),《旧唐书》作乞离胡,是吐蕃王朝末任赞普朗达玛的继任者。关于云丹的真实身份,《旧唐书》称是朗达玛王妃綝氏(那囊氏)的哥哥尚延力之子;而藏文文献《贤者喜宴》则称云丹是一名乞丐的儿子,被那囊氏收为养子。842年,朗达玛被佛教僧人拉隆·贝吉多杰刺杀后,那囊氏为了巩固自己的地位,诡称生下了一个“有牙的儿子”,将云丹拥立为赞普,自己摄政。但大臣们对这个刚出生就长了牙的孩子的真实身份十分怀疑,因此不少大臣反对云丹。大相韦·甲多热(结都那)不服,被那囊氏逮捕灭族。此后不服云丹统治的大臣退往山南地区,立次妃所生的遗腹子俄松,与云丹一系分庭抗礼数十年。从此,吐蕃分裂,吐蕃王国名存实亡。847年,云丹以伍如(逻些小昭寺一带)为据点,俄松以云如(雅隆昌珠寺一带)为据点,双方的支持者发生大规模交战。这场被后世称为“伍约之战”的内乱持续了23年,整个青藏高原地区甚至连吐蕃辖下的西域地区都卷入了战乱之中。869年爆发了臣民大反叛,赞普一族的威权完全丧失。根据《汉藏文书》记载,云丹死于36岁,无陵。而根据《雍仲本教目录》的记载,云丹是被仲康甫刺杀的。云丹死后,其子赤德贡宁嗣立。后来的拉萨王系是云丹的后代。","text2":"安达·云丹的父亲是谁?","label":1} {"text1":"干杉镇位于湖南省长沙县南部。地缘上,干杉北部与黄花镇接壤,东南部与江背镇为邻,西南部与黄兴镇交界,西北部与㮾梨镇相连,全乡总面积69.7平方公里,总人口22,097人(2000年人口普查);辖8个行政村、1个社区;乡政府驻干杉社区。今干杉乡1949年属龙潭乡,1956年为干杉乡;1958年属谷塘公社,1961年析干杉公社,1983年改为干杉乡。干杉乡在1995年长沙地区撤区并乡镇之前属㮾梨区,1995年㮾梨区撤销后直属县政府。2004年村级区划调整,调整之前为14个村,分别为百祥村、大岭村、石弓村、明月村、干杉村、建新村、大塘村、车马村、道渡村、纠田村、田坪村、斗塘村、长安村和石坡村;区划调整后为8个村、1个社区。","text2":"干杉镇位于什么地方?","label":1} {"text1":"鬼使神差(英文原名:《*batteries not included》;直译:不包含电池)是一套由环球影业及在1987年美国发行上映、加入了外星机械元素的家庭处境剧电影。剧本原本是被打算制作成电视剧并收录为《惊异传奇》的其中一集,但由于制作人斯蒂芬·斯皮尔伯格表示十分喜欢其故事因此最终将它摄制成一部独立的电影。故事发生在80年代末一个位于纽约曼哈顿东村的低层公寓大楼,受到大企业的摩天大厦发展计划影响,公寓的住客只走剩年迈的房东 Frank Riley 、其患有轻度老人痴呆症的妻子 Faye Riley 、穷画家 Mason Baylor 、被男朋友抛弃并怀有身孕的 Marisa Esteval 及已退休的黑人冠军拳击手 Harry Noble。受到企业雇来迫迁的不良分子骚扰,Frank Riley 几乎决定放弃的当晚,公寓忽然来了两个犹如迷你UFO的飞行生物,它们有将破烂修补完好的特殊能力,各自拥有独立的人格。虽然无法以言语沟通,这对机械生物就像民间传说中的家庭妖精和公寓的住客发展出互助共生的关系。","text2":"《鬼使神差》是哪一年上映的电影?","label":1} {"text1":"月球探险(法语: On a marché sur la Lune ;英语: Explorers on the Moon )是丁丁历险记的第17部作品。作者是比利时漫画家埃尔热。本作是奔向月球的续集。在本作中,搭乘火箭前往月球的有丁丁及其的宠物白雪、阿道克船长、卡尔库鲁斯教授、助手弗兰克和因为记错时间阴差阳错登上火箭的汤姆森汤普森。在图纳思教授的计划下,人类历史上第一艘载人火箭载著丁丁、米路、哈达克船长飞向了太空,正式开始了危险重重、充满未知之数的登月旅程。随后的考察中,危险和意外事故总与他们为伍,这是偶然?还是人为的蓄意破坏?他们当中谁是奸细?究竟丁丁能否粉碎这次阴谋,安全返回地球?还有最致命的问题:火箭上的氧气储备是否足够?","text2":"汤姆森汤普森为何会登上飞往月球的火箭?","label":1} {"text1":"保罗·卡拉斯(生于1967年8月13日)是以发现环绕著恒星的岩屑盘而著名的希腊裔的美国天文学家。卡拉斯的科学小组在距离地球25光年的北落师门观察到第一颗有著轨道运动的可见光影像的系外行星,这颗行星被命名为北落师门 b。卡拉斯因为乔治和玛丽亚卡维尔印斯由希腊克里特岛的赫拉克良移居到美国而出生在纽约。卡拉斯参加密西根底特律国家走读学校,并且在密西根大学的安纳堡分校学习天文学和物理学。他在夏威夷大学的天文学家大卫·杰维特的指导下于1996年获得天文学的博士学位。卡拉斯在德国海德堡的马克斯普朗克天文所、太空望远镜科学所和柏克莱加州大学进行博士后研究。在2006年,它成为柏克莱加州大学的兼任教授。卡拉斯和他的妻子Aspasia Gika和女儿Maria-Nikoleta、Natalia一起生活在柏克莱加州大学。卡拉斯使用哈伯太空望远镜上的日冕仪和夏威夷大学在夏威夷毛纳基山的2.2米望远镜发现了一些拱星盘。在1995年,他在绘架座β盘的光学影像中发现各种各样不对称的结构。他是研究第一张邻近的红矮星显微镜AU和明亮的北落师门光学影像岩屑盘的首席科学家。卡拉斯使用哈伯太空望远镜拍摄的影像揭露北落师门有一条由尘埃粒子构成的环带,类似我们太阳系的古柏带。然而,卡拉斯也发现这条环带是狭窄的并在几何上截止在15天文单位。这些特点被认为是北落师门的环带在形态学上受到重力雕刻的强有力证据。","text2":"卡拉斯的科学小组发现了什么行星?","label":1} {"text1":"一个 kb(kilobit,千比特) 表示 1,000 (10) 比特(精确些为 1,024 比特)。“kilobit”这一术语通常以缩略式“kbps”“kb\/s”或“kbit\/s”用于表示数据传输速率(码率),意为“千比特每秒”。例如,“56 kbit\/s 的 公共交换电话网”,“一条 512 kbps 的宽带连接”。注意kilobit(千比特)的缩写为kb,不要与术语kilobyte(千字节,缩写为kB或KB,其中的“B”是大写的)相混淆。尽管它的前缀“kilo-(千)”常和字节(Bytes)连用表示 1,024,但kb 的十进制定义(1kbps=1,000bps)在电信传送速率的相关文字中仍被统一使用。","text2":"kilobyte的缩写是什么?","label":1} {"text1":"「パパとKISS IN THE DARK」()是南原兼原作、桃季さえ插画的日本BL小说,白泉社发行,2005年发售OVA(2集)。白樱学院高中一年级生宗方实良的恋人是父亲,超受欢迎信息素演员宗方镜介。恩爱的二人周围有瞄准实良的学生会长的宇都宫贵之,打算保护实良不受贵之的危害的青梅竹马日野一树。还有受欢迎的女演员、贵之的母亲─宇都宫美月和镜介不断传出诽闻…每天心情困惑也复杂的实良…。受欢迎的男演员和他儿子的恋爱故事。宗方镜介的养子。常常被镜介耍得团团转。互相喜欢著对方。宗方实良的爸爸。当红的男演员。喜欢著实良却也经常捉弄他。和宇都宫美月是姐弟。学生会长。开学第一天帮助过实良,称实良小猫咪。与一树发生关系,称一树小鸡。宗方实良的青梅竹马,喜欢著实良。与贵之发生关系,后在一起。女演员。宗方镜介的诽闻对象,其实是姐弟关系。宗方实良的亲身母亲。地下乐团出道的歌手。和实良跟一树是青梅竹马。","text2":"和爸爸KISS IN THE DARK的插画作者是谁?","label":1} {"text1":"狮头社事件(内外狮头社事件),发生于1875年,为台湾清治时期的台湾原住民排湾族与清朝之战役,地点是屏东县狮子乡一带,被视为沈葆桢在台期间规模最大的战争。此事件发生的远因是长久以来存在的汉清朝对原住民政权的侵扰,近因则是同治13年秋狮头社人民被刺桐脚庄民杀害,双方交恶,且又临日军侵犯原住民,及狮头社逮捕入侵部落之庄民和王开俊麾下营夫,以致清朝将领王开俊率领官兵数千名进犯,但为排湾军所阻,后王开俊中伏而亡,且随后全军覆没,是以引战。清国钦差大臣沈葆桢在牡丹社事件后开始「开山抚番」的政策,计划打通往恒春和后山之路,遂于1875年(光绪元年)2月为强行开辟道路,借口保护庄民,发动淮军攻击内狮头社、外狮头社、竹坑社、本武社和草山社,并发生清军屠杀原住民之事,战事约持续3个月,出兵数万人,清军最后虽胜,但损失惨重,残兵所剩无几。狮头社事件后,沈葆桢于地图上,片面将中文当地之地名之竹坑社改永平社,本武社改永福社,草山社改永安社,内、外狮头社为改内、外永化社。就实际成效而言:番学大多荒废,招垦事务弊端多,表面上,番目与代管者即便协助官方,以维护部落治安,但事实上,当地生活习俗依旧,除了改变部落传统的经济结构外,并未真正动摇大龟艾社群的内在基础。是故,狮头社事件结束后,清廷在台湾南部番地所实施的开山抚番政策成效是有限的。在屏鹅公路南下狮子乡南世村前路旁的王太帅镇安宫,为祭祀王开俊,旁边还有一座三层白塔。匾额是前枋寮地区农会总干事李增隆1985年所立。","text2":"狮头社事件对当地有什么影响?","label":1} {"text1":"瑟琳娜·范·德·伍德森()又译莎琳娜·凡·德·活逊、韦倩莲,美国热门电视剧和小说《绯闻女孩》中的女主角。她有著迷人的美貌和惊人的领导能力,是布莱尔·沃尔多夫的好朋友。瑟琳娜是个高挑的金发女孩,康士坦次高中的女孩们因她的美貌而认为全天下没有什么是她得不到的。她并没有专业地训练过某种技能,但经常有人告诉她她的潜力无限。她的父亲经营著她高曾祖父在十八世纪创建的荷兰运输公司,母亲莉莉是一位社交名媛。她的父母出席了几乎所有的在城市中的名人或艺术组织发起的活动。范·德·伍德森家族居住在大都会博物馆和中央公园对面的公园大道994号公寓。瑟琳娜和她的家人买下半个顶楼的有14间居室的顶楼公寓。瑟琳娜神秘而美丽,是其他女孩的一个威胁。她有咬手指的习惯,在高中时几乎每次都是她先提出和男友分手。虽然对时尚有敏锐的意识和感觉,但她没有像布莱尔一样对自己的著装一定做到完美和一丝不苟。瑟琳娜是一个有魅力的、迷人的、天才的、风趣的、善良的、整天玩乐和生活悠闲的女孩,因此她吸引了非常多的男人。除了这些,纽约客的珍妮特·马尔科姆则形容她“光芒四射,善良,但有时会显得有些无聊。”Serena是莉莉和Keith van der Woodsen的大女儿;母亲是社交名媛及前芭蕾舞者;父亲是亿万富翁;她有个男同性恋的弟弟叫艾力(Eric)。Serena的父母在她小时就离婚了,Serena和Eric就和她母亲居住。在去寄宿学校前,Serena是上东城的“女王”,她离开后,她的好友胡贝雅(Blair)成为新的“女王”(Queen Bee)。另外,Serena;Blair及欧尼特(Nate)从小就是非常要好。在Sheppard家的婚礼上,Nate和Serena在酒吧台上发生性行为,令Blair受到很大的打击。","text2":"瑟琳娜为什么是其他女孩的威胁?","label":1} {"text1":"这个美国政党列表包含了以前和目前存在于美国政治的政党。这些政党在最近的总统竞选中提名了候选人,但在2004年的竞选中没有提名,他们也没有在足够多的州将候选人名单列入选票从而赢得2008年的总统竞选。美国第一党创立于2002年,由一群离开改革党的帕特·布坎南的支持者组成。这个党反对堕胎,反对枪支管制。旨在尽快结束种族配额和非法移民。布坎南从来没有宣称与这个党有联系,虽然这个党的首页上出售他的书。这个党吸引了一些人,如前任波士顿市长和驻梵蒂冈大使雷蒙德·弗林和前任约翰·伯奇协会(美国一极右组织)总裁约翰·麦克马纳斯。这些政党有些在以前的总统竞选中提名了候选人,但因为不同的原因没有在最近的总统竞选中提名候选人。其它的政党还没有提名过总统候选人。很少的一些政党在成立之初提名过总统候选人","text2":"美国第一党是什么时候创立的?","label":1} {"text1":"何尊,西周早期的一件青铜酒器,为贵族“何”作于周成王五年(前11世纪),是西周最早的有明确纪年的青铜器。1963年出土于陕西省宝鸡市贾村。何尊是中华人民共和国国家一级文物,2002年公布的第一批64件禁止出国(境)展览文物之一。现藏宝鸡青铜器博物院。其造型庄严厚重,纹饰精美,内底有铭文122字,为研究西周初年的历史提供了珍贵史料。何尊是“中国”一词已知的最早出处,“中国”在这里的含义,是指西周王朝的都城成周地区,即“天下”的中心——伊河、洛河流域,也就是今天的洛阳。何尊的出土地点在宝鸡市东北郊的贾村,这里常有西周文物出土。贾村镇西街的一户陈姓人家,常年在屋后三米高的断崖崖根取土,断崖上面未取土处逐渐突出。1963年的一天,陈家老二为防止上方土块掉落伤人,用镢头挖铲时意外发现了一件古铜器。陈家人不知其价值,在清理干净泥土之后,用于装盛粮食。1965年8月,因经济拮据,陈家将铜器卖给宝鸡市的一家废品收购站。同年9月,宝鸡市博物馆的职工在废品收购站将其发现并抢救运回博物馆。最初称为“饕餮纹青铜尊”。1975年在北京展出时,故宫博物院的唐兰(一说马承源)发现内胆底部有铭文122字,成为国宝级文物。何尊圆口方体,长颈,腹微鼓,高圈足,通高38.8厘米,口径28.8厘米,底部长20.2厘米,宽19.8厘米,总重14.6公斤。体侧装饰有四道镂空棱脊,以细雷纹为地,高浮雕,口沿下饰蕉叶纹,颈饰蚕纹图案,圈足处也饰有兽面纹。整体造型雄奇,工艺精美。何尊内底铸铭文12行、122字,因底部破孔,残损3字,现存119字。铭文记述成王继承武王遗志,营建东都成周之事,可与《尚书·召诰》、《逸周书·度邑》等古文献相互印证,具有非常重要的史料价值。铭文全文如下:大意是:成王五年四月,开始在成周营建都城洛邑,对武王行礼福之祭。丙戌日,成王在京宫大室对同族何进行训诰说:“你的先父公氏追随文王,后文王受天命,武王克商后告祭于天,以这里作为天下中心,治理民众。”成王将贝壳30朋赏赐给何,何因此作尊以纪念。","text2":"何尊一开始的名字是什么?","label":1} {"text1":"「パパとKISS IN THE DARK」()是南原兼原作、桃季さえ插画的日本BL小说,白泉社发行,2005年发售OVA(2集)。白樱学院高中一年级生宗方实良的恋人是父亲,超受欢迎信息素演员宗方镜介。恩爱的二人周围有瞄准实良的学生会长的宇都宫贵之,打算保护实良不受贵之的危害的青梅竹马日野一树。还有受欢迎的女演员、贵之的母亲─宇都宫美月和镜介不断传出诽闻…每天心情困惑也复杂的实良…。受欢迎的男演员和他儿子的恋爱故事。宗方镜介的养子。常常被镜介耍得团团转。互相喜欢著对方。宗方实良的爸爸。当红的男演员。喜欢著实良却也经常捉弄他。和宇都宫美月是姐弟。学生会长。开学第一天帮助过实良,称实良小猫咪。与一树发生关系,称一树小鸡。宗方实良的青梅竹马,喜欢著实良。与贵之发生关系,后在一起。女演员。宗方镜介的诽闻对象,其实是姐弟关系。宗方实良的亲身母亲。地下乐团出道的歌手。和实良跟一树是青梅竹马。","text2":"和爸爸KISS IN THE DARK的原作者是谁?","label":1} {"text1":"黑足猫(学名:\"\")是分布的南非、纳米比亚、博茨瓦纳及津巴布韦的一种野猫。牠们栖息在干旱的半沙漠及大草原,如南非干旱台地及部份高草原,但也分散在喀拉哈里沙漠。黑足猫平均重1.6公斤,是最小的猫科之一。雌猫平均重1.3公斤,雄猫重1.9公斤。体长36-52厘米,尾巴长13-20厘米,肩高25厘米。相对于身体来说,其头部很大。毛色是肉桂色至茶色或近白色,有黑色的斑点,肩上有黑色斑纹,脚及尾巴有黑环。牠们的脚底如其名是黑色的。黑足猫是独居及夜间活动的,所以很难见到牠们。日间牠们会躲在跳兔的巢内、岩石及丛林下及白蚁丘内。黑足猫主要猎食细小的猎物,如啮齿目及细小的鸟类,但也会猎食白翅鸨及草兔。昆虫及蜘蛛只占牠们猎物少于1%。黑足猫是很怕羞的动物,会尽量躲藏,但当被围困时就会激烈的保护自己。黑足猫每胎一般会产两只幼猫,有时介乎1-4只。在南半球春天、夏天及秋天时,雌猫可以产两胎。幼猫约于5个月大就可以独立,但仍会在母猫的领地生活。1岁大的雌性黑足猫会占有领地约10平方公里大,雄猫则占约22平方公里。成年雄猫的领地与1-4只雌猫的领地重叠。牠们会在夜间行8公里来觅食。牠们需要非常高的能量来生活,每晚需要250克的食物,即牠们体重的六分之一。黑足猫有两个可能亚种:这两个亚种的标本是在南非中部近金伯利的地方发现。一些学者指黑足猫在部分地区很普遍,不过大部份人都认为牠们很稀少,而世界自然保护联盟将牠们列为易危。","text2":"成年雄猫有什么特点?","label":1} {"text1":"兰大卫(,),暱称老兰医师,是一位出身苏格兰的医师及长老教会传教士,亦是彰化基督教医院创建人之一。长年在彰化地区行医,使当地产生一句俗谚:「南门妈祖宫,西门兰医生」。其子乃兰大弼医师。1870年8月2日,兰大卫出生于联合王国苏格兰。1895年,他毕业于爱丁堡大学医学院,之后在台展开长达40年的医疗传道工作,他经常和梅监雾牧师一起工作。 此外,兰大卫亦致力于培养台湾本土西医。1928年,一名13岁少年周金耀腿部溃疡,无法长皮,兰大卫将妻子连玛玉(Miss Marjorie Learner)四块腿皮移植予周金耀,后虽因排斥而手术失败,在施行自体植皮手术及悉心照顾下仍逐渐痊愈。1936年,兰大卫退休返回英国。1957年10月,兰氏因车祸去世。","text2":"1957年10月,兰氏因为何种原因去世?","label":1} {"text1":"59街是纽约市曼哈顿区的一条东西向街道,从约克大道到西侧高速公路,在第九大道\/哥伦布大道与第八大道\/中央公园西之间被时代华纳中心隔断。59街的大部分路段双向通行,但是在第九大道和第十大道单向通行,只允许西行。在第二大道,59街连接皇后大桥(Queensboro Bridge),此桥常被称为“59街桥”。59街构成曼哈顿中城与曼哈顿上城的边界。它将哥伦布圆环一分为二,在第五大道和中央公园西之间的路段构成中央公园的南部边界。沿中央公园的路段标为中央公园南。在59街以北,曼哈顿的街区在中央公园两侧分为上西城和上东城,在上西城,59街以北更名为阿姆斯特丹大道,第十一大道更名为West End Avenue.","text2":"59街位于哪里?","label":1} {"text1":"北海油田是世界著名的石油集中出产区,每日生产大约600万桶。位于大西洋的陆缘海——北海,它是介于欧洲大不列颠岛、挪威和欧洲大陆之间,所出产之石油为沿岸英国,挪威和荷兰等国所享有,也是布兰特原油指数主要标的。1970年代之前,以波斯湾为核心的亚洲中东地区,一直是西方工业国家石油能源的主要供应者。随着第二次世界大战后西方国家经济的恢复和繁荣,石油需求量逐年上增,使得亚洲石油国提高购价。这让各大石油巨头不愿购买,双方因此积怨。1973年第四次中东战争爆发,阿拉伯产油国以不满美国为首的西方国家支持以色列之立场为由,以能源为武器,宣布「石油禁运」,导致欧美油价暴涨,并相继波及各个经济领域,部分导致七十年代全球经济大衰退。因此,以英国为首的北海沿岸国家将目标转向沉寂多年的北海油田。在此之前,北海是欧洲航运要道,且海底地形复杂,海上气候恶劣,一直未有大规模地质勘探活动进行。而北海在1959年首先于荷兰近海发现格罗宁根气田(荷兰语:Groningen),此后更进入大规模开发阶段,先后于1969年发现埃克非思科油田(英语:Artsen non-Cisco),1971年发现布伦特油田(英语:Brent),大油田的相继发现,不仅缓解英国与挪威等西欧国家的能源短缺,而且更使挪威成为除加拿大和俄罗斯之外的第三大非OPEC石油出口国。北海石油的发掘,使得正处于疲软时期的英国、荷兰与丹麦等工业国家的经济获得帮助,也使得英国经济在「油荒」年代于西欧一枝独秀。北海原油因其品质高,产量稳定,所以迅速成为欧洲重要的能源供应地,因此欧洲原油交易市场多以每桶北海布伦特石油(约159升)作为市场参考价格。世界石油市场约6.4%的供货来源于此。","text2":"北海石油的发掘带来了什么样的影响?","label":1} {"text1":"相移电路(,RC电路),或称RC滤波器、RC网路,是一个包含利用电压源、电流源驱使电阻器、电容器运作的电路。一个最简单的RC电路是由一个电容器和一个电阻器组成的,称为一阶RC电路。三种基本线性类比积体电路元件包含: 电阻器 (R)、 电容器 (C) 和电感元件 (L)。 它可能借由下列四种复合性材料组合而成: RC电路、 RL电路、 LC电路 以及 RLC电路 。这些电路中, 大量的重要类型为许多类比积体电路的基础。 一致的是, 它们皆可借由 无源滤波器运作。若将电路视为一 分压器,其经由电容器的 电压 为:且经由电阻器的电压为:一个最简单的RC电路是由一个电阻器和一个电容器串联组成,而当一个电路只由一个充电的电容器和一个电阻器组成时,电容器会释出电流给电阻器。 根据克希荷夫电路定律我们可求得电流于电容器所耗时间内产生的变化,其结果经由 线性微分方程解求得时,其结果于指数衰变函数电路电流阻抗电容器电性阻抗的增加和存于电容器的电量有关。如果一个电容器的电压来源为交流电,此电容的电压会转变成交流电源,当交流电的频率越快时,因为蓄电的时间减少,使储存于电容器内的电压进而减少,同样也缩减了电容器的等效电阻。 这说明了电压器的等效电阻和电压源频率的反关系。电容器的阻抗或电阻 \"Z\"(欧姆)和 电容\"C\"(法拉第)为角频率 \"s\" 通常称为一复数,当其弦波稳态是一种特殊情况,这一情况下的输入电压由纯粹的正弦曲线(无指数衰减)组成。这样就得到结果而\"s\"估算为","text2":"一阶RC电路指什么?","label":1} {"text1":"黑金之国 (法语:Tintin au pays de l'or noir;英语:Land of Black Gold)是丁丁历险记的第15部作品。作者是比利时漫画家埃尔热。本作曾于1939年到1940年期间于二十世纪小伙伴上连载,内容与反法西斯有关,但由于比利时于二战被德国占领,连载被迫取消。二战后,埃尔热重新拾起了这部作品,并将其的故事主线更改。接二连三的汽车爆炸事件,触动了记者丁丁查根究底的神经,当他发现汽油是重要线索,便与好拍挡米路,在石油公司的安排下,远赴中东的石油产地追查。与此同时,杜邦德和杜邦特也接受了秘密任务,乔装上阵,他们与丁丁不约而同登上了石油公司的轮船斯俾多星号,目的地是海默德,而当地正值两大酋长争权,政局紧张。途中他们同遭插赃嫁祸,丁丁更被俘掳,无端卷入当地的政治角力中。他流落沙漠,迷途而且缺水缺粮,期间更遇上凶险的沙暴!千辛万苦抵达海默德,丁丁结识了本艾扎布酋长,又重遇了几个“老朋友”,却让他陷入了更大的危机!正巧酋长的小公子被绑架,丁丁身负寻人重任,营救行动波折重重。孤军作战的丁丁与恶势力周旋到底,他能否死里逃生,并救出人质?他又如何逐步解开汽车的爆炸之谜,从而揭破一桩国际的军事阴谋?","text2":"《黑金之国》是《丁丁历险记》系列的第几部?","label":1} {"text1":"教廷大使()是圣座的一个外交官衔,来自古拉丁语词汇\"Nuntius\",意为“envoy”(使节)。本文不仅有关该头衔,也说明其他衍生出的类似头衔,这些头衔都在罗马天主教会的组织结构中使用。教廷大使是圣座对一个国家或国际组织(例如,阿拉伯联盟)常驻的外交代表或外交使团团长,外交使节身分之职称,通常该大使也会有天主教教会的主教身分。根据1961年签订的《维也纳外交关系公约》,一个教廷大使拥有一般外交使节之身份。然而,《维也纳外交关系公约》也规定所驻国除了教廷大使之外,也可要求其他使节派驻,且也可派驻不论资格的大使于圣座。此外,教廷大使也兼任圣座和所驻国的全国或地区的罗马天主教教区主教团之间的联络者。准大使是专指派驻使节前往非天主教国家的大使,等同一般大使。","text2":"准大使指的是什么?","label":1} {"text1":"唐绍华(),安徽巢县人,中华民国剧作家、导演、诗人、学者。台湾影剧事业的开拓者之一。民国22年(1933年)国立中央大学毕业。曾任《中央日报》记者,并创办《中国人》旬刊(后改为《良心话》),主编《文化杂志》、《新世纪》及《国是》等刊物。抗战时在重庆编演话剧「碧血黄花」,轰动一时。抗战胜利后,在上海创办「中国第一电影企业公司」、「群星影艺公司」。民国38年任香港新华影业公司制片主任。民国40年到台,曾任台湾影业公司副总经理、嘉禾影业公司董事长,并在台湾艺术专科学校、政工干校、辅仁大学、中国文化大学任教。民国37年由他参与编剧的《花莲港》,是二战后台湾最早拍摄的电影之一。民国39年他导演的「春满人间」,带起了台湾民营乡土片的风潮。唐绍华所导的台语片《廖添丁传》(1956年)亦颇受瞩目。其他","text2":"唐绍华是哪里人?","label":1} {"text1":"青岛山是中国青岛市的“十大山头公园”之一,海拔高度128.5米。位于今青岛市南区中部偏北与市北区交界处,在贮水山与太平山之间。德国占领青岛期间,该山被命名为俾斯麦山,1899年,山上兴建了俾斯麦炮台。1914年日本攻占青岛,更名为万年山。1922年中国接收青岛主权,命名京山。中华人民共和国成立后,改名青岛山。1984年,青岛山辟为青岛市的开放公园,修建了数座亭台,可供登临观赏青岛海景与市容。青岛山炮台遗址于1984年被列为市重点文物保护单位。1997年市政府修复仍保存完好的规模庞大的德军要塞地下指挥部,并向公众开放。2016年,在青岛山炮台遗址东侧开始建设一战博物馆,预计2018年竣工开放。","text2":"青岛山曾经更改为什么名称?","label":1} {"text1":"阿尔维德·卡尔森(,),瑞典科学家。他最著名的成就是对神经递质多巴胺的研究以及该物质在帕金森氏症中的作用,他也因此成为2000年度诺贝尔生理学或医学奖的获奖者之一。此外,他也是沃尔夫医学奖的获奖者。卡尔森出生在瑞典的乌普萨拉,是隆德大学历史学教授Gottfrid Carlsson的儿子。1941年,卡尔森也进入隆德大学学习医学。1944年,他参与了检查被释放的纳粹集中营的犯人的工作,他们是被Folke Bernadotte带到瑞典来的。虽然瑞典是第二次世界大战的中立国,卡尔森的求学之路还是由于在军队中服役而被迫中断数年。1951年,卡尔森获得了医学和哲学博士学位,并随后成为隆德大学的教授。1959年,他又到哥德堡大学任教授。1957年,卡尔森提出多巴胺不仅仅是过去人们认为的去甲肾上腺素的前体,也是一种位于脑部的神经递质。在阿斯利康制药公司工作期间,卡尔森和他的同事们利用溴苯那敏合成了第一个投入市场的选择性5-羟色胺再吸收抑制剂——苯吡烯胺(zimelidine)。卡尔森发展出了一种测量脑组织中多巴胺含量的方法。根据这一方法,他发现基底核(控制运动的脑部区域)中多巴胺水平特别高。接着他给实验动物服用药物利血平(reserpine),结果导致多巴胺水平下降并引起运动失控。这一现象与帕金森氏症的症状相似。而通过在这些实验动物的饮食中添加左旋多巴(L-Dopa),多巴胺的前体,能够减轻相应症状。这些发现引起了其他医生的注意,他们尝试使用左旋多巴来治疗帕金森氏症患者,发现确实能够减轻疾病早期的一些症状。左旋多巴目前依然是治疗帕金森氏症的最普遍采用的方法的基础。卡尔森是一名饮用水氟化的反对者。他与其他1700多名卫生专业人士共同签署了一个停止氟化饮用水的请愿书。他声称他参与了在瑞典的辩论会,在会中他帮助说服国会,希望基于道德规范,将饮用水氟化定为非法。他相信氟化水侵犯了现代医学原理,即医学治疗应该根据个人情况来进行。","text2":"1957年,卡尔森提出什么论点?","label":1} {"text1":"桩(Stub \/ Method Stub)是指用来替换一部分功能的程序段。桩程序可以用来模拟已有程序的行为(比如一个远端机器的过程)或是对将要开发的代码的一种临时替代。因此,打桩技术在程序移植、分布式计算、通用软件开发和测试中用处很大。以下是桩程序的一个例子(伪码):上例中的伪码调用了 ThermometerRead函数,其返回一个温度。由于ThermometerRead需要去读取硬件设备,而这个函数现在还没能开发完成,不能正常工作。ThermometerRead只是简单的返回了一个合理的值,这样主程序就能正常调用这个函数,并继续接下来的开发了。可以注意到,虽然它接受了一个Source类型的参数,表明需要返回的温度是内部还是外部的,实际上并没有对这个参数进行任何使用。桩程序是一段并不执行任何实际功能的程序,只对接受的参数进行声明并返回一个合法值。这个返回值通常只是一个对于调用者来讲可接受的值即可。桩通常用在对一个已有接口的临时替换上,实际的接口程序在未来再对桩程序进行替换。在远程方法调用(RMI)中将客户辅助对象称之为Stub(桩);将服务辅助对象称之为skeleton(骨架)。RMI的过程是:客户对象一旦被调用,客户对象调用stub,stub调用网络远端的skeleton,而skeleton最终调用真正的服务对象。由此,在调用客户对象的时候,感觉上就是直接调用了真正的服务对象。","text2":"在远程方法调用(RMI)中将客户辅助对象称之为什么?","label":1} {"text1":"轧压均质装甲(英文:Rolled Homogeneous Armour;德文:gewalzte homogene Panzerung),缩写成「RHA」,又简称为均质装甲是为钢板之理论基础象征,RHA做为一种基数,用于对照军事装甲车之效能。直至第二次世界大战结束,大部份坦克的装甲形式以及其它装甲车辆均覆盖著钢板。若要增加车辆的防护,即意味著要增加更厚的钢板,而厚重的装甲相对的也减低了其机动性。从那时以来,一些形式的装甲开始发展出混入一些空气、陶瓷或是贫铀(depleted uranium,简称DU)于钢质之中。在现代化武器攻击下,更大的撞击力量与更高的温度切割冲击,传统的轧压均质装甲已不敷使用,遂由更高等级的装甲取代,而测量单位也跟著有所改变。更新的装甲估量数被称做「等效轧压均质装甲(Rolled Homogeneous Armour equivalency,简称RHAe),其做为一种粗略估计的单位,或是被炮弹穿透的能力,或是其装甲形式的保护能力,这些不一定是使用钢材。但是,因为各装甲间形式、质量、金属以及分工组装等不同,RHAe是否能有效的比对不同形式装甲,仍有许多争议性。当前美国陆军所使用,RHA钢板产品有军用标准MIL-A 12560,而最新标准则使用有MIL-A 46100。这些非常相似,但实际上在美国的钢板等级中,不同于标准高强度钢材合金4340,虽然其械制造与合金非常的相似。","text2":"轧压均质装甲的英文是什么?","label":1} {"text1":"《十国春秋》,共114卷,清人吴任臣编撰纪传体取书。《十国春秋》,写十国君主之事迹,采自五代、两宋时的各种杂史、野史、地志、笔记等文献资料,计有吴十四卷,南唐二十卷,前蜀十三卷,后蜀十卷,南汉九卷,楚十卷,吴越十三卷,闽十卷,荆南四卷,北汉五卷,十国纪元表一卷,十国世系表一卷,十国地理表二卷,十国藩镇表一卷,十国百官表一卷。康熙八年(1669年)完成。洪亮吉《北江诗话》卷一载:“吴任臣撰《十国春秋》,搜采极博。”《越缦堂读书记》日记曰:“此书三过阅矣,丙辰(1856)读之尤细,甚薄其体载之疏;至壬申(1872)复阅,始叹其博不可及也。”《四库全书总目提要》评:“任臣以欧阳修作《五代史》,于十国仿《晋书》例为载记,每略而不详,乃采诸霸史、杂史以及小说家言,并证以正史,汇成是书。”吴任臣自序曰:“任臣以孤陋之学,思取十国人物事实而章著之,网罗典籍,爰勒一书,名曰《十国春秋》,为本纪二十,世家二十二,列传千二百八十二。人以国分,事以类属。又为《纪元》、《世系》、《地理》、《藩镇》、《百官》五表,总一百一十四卷。虽世远人湮,书册难考,乃鉴观诸邦,略得而论。……书成,聊著纂述之大指如此。康熙八年(1669年)己酉孟夏,仁和吴任臣撰。”又有周跋:“余校刊吴氏《十国春秋》,附刻《拾遗》、《备攷》二卷,锓板发十方后,复补录数条,亦未印行,年末采摭旧闻,则记载沿有阙。”乾隆五十三年(1788年)四月,周昂重刊《十国春秋》。一至十四卷十五至三十四卷三十五至四十七卷四十八至五十七卷五十八至六十六卷六十七至七十六卷七十七至八十九卷九十至九十九卷一百至一百三卷一百四至一百八卷一百九至一百十四卷","text2":"《四库全书总目提要》中是怎样评价《十国春秋》的?","label":1} {"text1":"吴彦晟,是一名中国足球守门员,现效力于中国足球超级联赛球队上海申鑫。2001年,上海中远收购了吴彦晟所在的上海万申青年队。2004年,吴彦晟进入上海国际一线队名单。2005年9月7日,他在2005年中国足协杯四分之一决赛第二回合客场挑战山东鲁能泰山由于主力门将张晨突然生病而得到出场机会,上演职业生涯首秀。11月5日,他在联赛最后一轮上海国际1比3不敌重庆力帆的比赛首发出场,第一次亮相中超联赛。2006年,吴彦晟随球队迁到陕西。2007年,他在主力门将张晨转会上海申花后,逐渐坐稳了主力位置。2008年一度保持着当时联赛最少丢球的纪录,并且用自己稳健的发挥使陕西浐灞成为当赛季的半程冠军。不过在2010年,朱广沪与科萨先后担任球队的主教练后,吴彦晟淡出了球队的主力阵容。2011年,吴彦晟在整个赛季都无球可踢赋闲在家。2012年,吴彦晟加盟上海申鑫。","text2":"吴彦晟所在的上海万申青年队被谁所收购?","label":1} {"text1":"双斑长鳍天竺鲷(学名:),又称黑尾长鳍天竺鲷、暗体长鳍天竺鲷,俗名大面侧仔、大目侧仔,为辐鳍鱼纲鲈形目鲈亚目天竺鲷科的一个种。本鱼分布于西太平洋区,包括日本、印尼、巴布亚纽几内亚、菲律宾、澳洲、帛琉、马里亚纳群岛、斐济、关岛、萨摩亚群岛、索罗门群岛、万那杜、东加等海域。水深0-33公尺。本鱼体呈圆形侧扁,眼大。体呈淡红褐色,背部颜色略深,腹部银白色。被栉鳞且具红边缘,头部有一深褐色直条纹从眼睛延伸至鳃盖下缘,在塞概后上方靠近额部之地方有一黑色眼斑,尾柄部也具一黑色眼斑,尾鳍凹入,臀鳍延伸,背鳍硬棘7枚;背鳍软条9枚;臀鳍硬棘2枚;臀鳍软条16-17枚,体长可达11公分。本鱼栖息于海湾与潟湖,喜群游,属肉食性,以浮游生物和底栖性无脊椎动物为食。雄鱼与雌鱼交配后,雄鱼会将卵含在口中保护,孵化之。无任何经济价值。","text2":"它们一般会选择哪里作为栖息地?","label":1} {"text1":"环首刀,又称环首剑、汉刀、环头刀,是汉民族在西汉中期发展出的一种以单手或双手持用为主的短兵器;特点是单面开锋,厚脊薄刃,直脊直刃,刀柄首呈圆环形产生年代不详,由于社会、经济的发展与繁荣,生产力大增,汉人开始了全面以钢、铁器取代铜器的进程,亦用于兵器制作,钢或铁长剑迅速地取代青铜长剑。至汉武帝时期,始出现环首刀,并逐渐取代剑成为军队的基本制式装备,至东汉后期完全取代剑,剑从此退出军事舞台;魏晋南北朝时代环首刀作为短兵器的主角独领风骚三百多年。一般认为,环首刀的出现有二个主要原因;一、发展骑兵作战:由于匈奴的威胁,中国始发展大量骑兵,需要比长剑更适合劈砍的兵器,骑兵在马背上使用斩击远较刺击容易,且刺击可能发生兵器插在目标上不能拔出的状况;另刀拥有厚实背脊,重量较重,劈砍时产生的杀伤力与折断的机率均较剑优秀。二、刀比剑更易制造:单面开刃的工程比双面开刃简单省时,适合量产,为军队大量装备。
","text2":"环首刀哪个时期最早出现的?","label":1} {"text1":"广东潮剧院是中华人民共和国公营潮剧院团,于1958年12月25日组建。它整并了广东省潮剧团、源正潮剧团、怡梨潮剧团、玉梨潮剧团、赛宝潮剧团、三正潮剧团。在广东汕头市,设有「一团」、「二团」、「三团」3个演出团和舞台美术制作厂,并经营谢慧如潮剧艺术中心。中华人民共和国成立以来摄制的潮剧电影《火烧临江楼》、《告亲夫》、《闹开封》、《王茂生进酒》、《荔镜记》(朱石麟执导,彩色)、《苏六娘》(彩色)、《刘明珠》(彩色)、《张春郎削发》(彩色)、《烟花女与状元郎》(彩色)都是剧院主演。潮剧院也曾到泰国、马来西亚、新加坡、柬埔寨、越南、澳大利亚、法国等许多国家表演潮剧。","text2":"潮剧曾经去了哪些国家表演?","label":1} {"text1":"圣母无原罪主教座堂位于台湾台北市大同区,为天主教台北总教区的主教座堂,也是台北市首座天主教堂;由于座落于民生西路,亦名民生西路天主堂。教堂附设有一幼稚园,教堂建地并与同为天主教机构的静修女中比邻而居。清治时代末年,道明会来到台北传教,并于1889年在大稻埕建立第一座教堂。1905年,道明会林茂才神父来到大稻埕,后在教堂现址一带购入大量土地,并于1911年鸠工兴建哥德式教堂一座,于1914年落成,称为「蓬莱町大圣堂」,为日治时代台北市内规模最为宏伟的建筑之一。1916年,道明会又在教堂旁成立女学校,即今之静修女中。1945年5月31日,该教堂在二战末期发生的台北大空袭中遭盟军炸毁,造成在此避难的民众严重伤亡。第二次世界大战结束后,仅以木板在原址旁兴建简易教堂使用。1959年3月,吉朝芳神父就任该堂主任司铎后,即以重建大堂作为首要任务。经过多方奔走,1959年7月5日在郭若石总主教的主礼下,教堂的重建工程正式开工,1961年5月竣工,经时任台北署理主教田耕莘枢机指定为主教座堂,以始胎无染原罪圣母为主保。同年5月30日,由成世光辅理主教举行落成祝圣大典,隔日由时任教廷驻华公使高理耀蒙席主持首台弥撒圣祭。以下时间以当地时间(台湾时间)为准","text2":"圣母无原罪主教座堂位于何地?","label":1} {"text1":"卓以和(),籍贯北平,美国华裔物理学家,电器工程师,分子束外延(Molecular beam epitaxy)技术的鼻祖,量子级联激光器的共同发明人,对Ⅲ-V族化合物半导体、金属和绝缘体的异质外延和人工结构的量子阱、超晶格及调制掺杂微结构材料系统地开展了大量先驱性的研究工作。卓以和于1937年生于北京。他的祖父卓定谋(字君庸)是中华民国第一届国大代表。1949年全家移住香港,就读于培正中学。卓以和高中毕业后,留学美国,在伊利诺伊大学厄巴纳-香槟分校(University of Illinois at Urbana-Champaign)取得电机工程学士、硕士和博士学位。卓以和的专长为应用物理科学和工程科学。主要的贡献是发明分子束外延(Molecular beam epitaxy)技术和量子级联激光器(Quantum cascade laser)。卓以和目前担任美国Alcatel-Lucent贝尔实验室半导体研究所助理副总裁。目前并担任中华民国教育部、工研院、中研院物理所顾问。卓以和曾获选为获奖:卓以和夫妇育有一儿三女。","text2":"卓以和的祖父是谁?","label":1} {"text1":"施云奴(,),全名施维奥·文迪斯·甘波斯·祖利亚(Sylvio Mendes Campos Junior),出生于圣保罗,是一名巴西足球运动员。他是一名左路球员,常担任后卫或翼卫。他是一个进攻型的后卫,经常助攻予队友入球。他在哥连泰斯展开职业生涯,从1994年开始效力,直至1999年,他被英超豪门阿仙奴看中,成为首位巴西籍的阿仙奴球员。他担任偶像温特本的左后卫位置。但他只在阿仙奴逗了两年。在这短时间,他认识了很多朋友和追求者,亦曾在对锡周三和车路士的赛事射入「世界波」。离队后,他被艾殊利高尔代替。2001年,施云奴加盟切尔达,效力了3年。2004年,他以200万英镑转会费加盟巴塞隆拿,他为球会赢得2005和06年西甲联赛冠军和2006年欧联冠军。他和云邦贺斯、奥华马斯、法比加斯、美列达、希比、亨利一样,都是曾效力巴塞和阿仙奴的。2008年他在球会有好表现,使球会和他续约至2009年。施云奴于2009年与巴塞完成合约后以自由身加盟英超球会曼城,签约一年,仅为球队上阵10场后于季后被放弃。施云奴虽曾为巴西国家队效力6场,但他却有西班牙护照,居住西班牙3年后,使他成为「欧盟球员」,不受西甲「非欧盟球员政策」限制。","text2":"施云奴是如何成为首位巴西籍阿仙奴球员的?","label":1} {"text1":"《即刻毁灭》(\")是一部由科恩兄弟导演的黑色喜剧电影,于2008年9月12日在美国上映,由约翰·马克维奇、乔治·克隆尼、法兰西丝·麦朵曼、布莱德·彼特和蒂妲·丝云顿主演。本片曾于2008年8月27日在第65届威尼斯国际电影节首映 。根据导演科恩兄弟表示,剧情主题是关于美国中央情报局(CIA)、华盛顿特区的健身教练,以及网路交友约会。本片是这对兄弟导演再以《险路勿近》(\")赢得奥斯卡后的第一部新作,演员之一的蒂妲·丝云顿形容本片「...像是一种很夸张的荒谬喜剧。所有演员都很夸张,真的很夸张。电影充满了荒谬之处。比《险路勿近》要轻松多了。」奥斯朋·考克斯(,约翰·马克维奇 饰)是一名中央情报局分析师,他在因为酗酒问题被降职后辞去工作。他决定开始撰写在中情局期间的回忆录。他的妻子凯蒂·考克斯(,蒂妲·丝云顿 饰)想要和奥斯朋离婚,在与离婚律师开会时,她从奥斯朋的电脑中拷贝了所有的个人财务资料,同时也包括了他正撰写到一半的回忆录。存有这些资料的光碟辗转来到「硬汉健身房」,在此工作的查德·菲尔德海墨(,布莱德·彼特 饰)发现了这片光碟,并企图要勒索考克斯,","text2":"《即刻毁灭》是一部什么样类型的电影?","label":1} {"text1":"《我的快乐时代》是香港歌手陈奕迅的粤语音乐专辑,于1998年6月发行。
本专辑为陈奕迅加入华星唱片后推出的第三张广东专辑,收录了广受欢迎的歌曲《天下无双》和与专辑同名之作《我的快乐时代》,另外还收录一首日本歌手玉置浩二旧作品《Mr Lonely》的粤语改编之作《愈想愈无谓》。承继过往两张广东专辑的习惯,收录一些陈奕迅亲自作曲的作品,是次专辑收录了两首由他作曲、黄伟文填词的歌曲,分别是《新曲+精选》和《反高潮》,当中《新曲+精选》的歌词更是由以往的广东歌曲的歌名组合而成,甚为特别。多方面的选曲和尝试使这张专辑得到空前的成功,《天下无双》一曲不仅让陈奕迅获得多年来第一首十大中文金曲颁奖礼的「十大金曲奖」,还使他赢得首个叱咤乐坛流行榜颁奖典礼的「叱咤乐坛至尊唱片大奖」。","text2":"陈奕迅凭借哪首歌曲获得多年来第一首十大中文金曲颁奖礼的「十大金曲奖」?","label":1} {"text1":"崔真实(,),已故韩国女演员。20岁时以拍摄MBC剧集《朝鲜王朝500年-恨中录》出道,1995年凭借电影《杀妻秘笈》获得第33届韩国大钟奖最佳女主角奖及第31届韩国百想艺术大奖人气奖。2008年10月2日,因深受被指迫害安在焕烧炭自杀谣言困扰,在住所浴室利用绷带上吊自杀,享年岁。事后韩国政府拟于11月立法信息通信网法施行令修正案,防止匿名网民发放流言。曾经与日本职棒选手赵成珉于2000年结婚,因此息影,育有2名子女。之后在2002年10月因为家庭暴力,崔真实怀孕被推落楼梯,而在2004年9月离婚。2008年5月崔真实的两名子女依照当地的户籍法改随母姓。生前崔真实最后的日子与母亲同住。其弟崔真永亦为演员,2010年3月29日自杀身亡,得年39岁。2013年1月6日,其前夫赵成珉被发现在家自杀身亡。2013年11月27日,前经理人朴尚浩自杀。","text2":"她凭借电影《杀妻秘笈》获得哪些奖项?","label":1} {"text1":"杨清(,)是越南反抗唐朝的起义军首领。史书未记载生年。据吴士连的《大越史记全书》记载,杨清交州龙编人,其祖先「世为蛮酋」。唐朝开元年间,杨清被唐朝任命为驩州刺史。当时的安南都护李象古以「贪纵苛刻而失众心」,见杨清据守驩州,心中忌之,于819年召杨清至安南都护府任自己的牙将,令其带三千人兵前往镇压黄洞蛮。杨清见人心忿怒,暗与城中的杜士交、杨志烈勾结,率领所部乘夜袭破安南都护府,杀李象古和他的家属。据《旧唐书》记载,杨清起兵后,820年,唐朝任命唐州刺史桂仲武为新任安南都护,奉命讨伐,并许诺赦免杨清,封之为琼州刺史。杨清不从,据城反抗。桂仲武遣人招降杨清的属下。因杨清残暴不仁,部将纷纷献城投降。杨清与其子杨志贞被捕处决。杜士交拥立其子杨志烈为主,退往长州的凿溪(在今越南宁平省)继续抗唐。但不久即因势单力薄,率军投降唐朝。然而《大越史记全书》却与《旧唐书》的记载截然不同。《大越史记全书》称,杨清击败了唐朝桂仲武军,随后连结安南地区的各少数民族一起反唐,并连合南方的占城国攻击唐朝。杨清又于828年、841年两次败唐军的韩约、武浑部,此后《大越史记全书》便突然不见了关于杨清的记载。据《》:「(元和十五年三月)辛未,杨清伏诛。」以及《》:「(元和十五年三月)辛未,安南将士开城纳桂仲武,执杨清,斩之。」","text2":"杨清是谁?","label":1} {"text1":"岸本早未(),前日本女歌手。于京都府京都市出生、大阪府大阪市居住。所属唱片公司是GIZA Studio。10岁的时候,受到姐姐的影响而去学习舞蹈。在2002年10月,岸本早未在参加\"Giza studio presents DIG STAR audition\"后与Giza签约。在2003年6月25日生日当日正式出道(当时是16岁),并发布了第一首单曲\"迷Q!?-迷宫-MAKE★YOU-\"。最新发布的单曲是\"See you Darling\",于2007年2月28日发布。她的单曲曾被使用在一些动画中。\"迷Q!?-迷宫-MAKE★YOU-\"就作为片头曲用于TBS的侦探学园Q中。\"みえないストーリ\"和\"风に向かい歩くように\"就分别作为同剧的第三和第四片尾曲。北原爱子是她的好友,经常在节目Thursday Live中上镜。另外,她的服装亦受到安室奈美惠的影响。活动休止后目前从事流行服饰等相关工作。","text2":"最新发布的歌曲名称叫什么?","label":1} {"text1":"崖勤猎站()是城市铁路辖下的一个车站,位于国王十字地底。它是随着东郊线在1979年开幕,拥有一个岛式月台及一条回线,并且设有升降机。该车站位于东角商场的后面地底,而大堂则设有通道前往附近的新南头道和海洋道。但事实上车站并不是真的建在地底,而是车站的未端是建在地上的(尽管已被大堂和巴士转车站完全覆盖)。事实上,在东郊线的计划中崖勤猎站一直到榜上无名,直至1947年才改变现状。在1963年甚至提议将该站为终站,但4年就决定以终站改为邦迪联运站。最终车站在1979年6月23日开幕。尽管此站并不是城市铁路路线清理计划辖下的一项计划,但在崖勤猎站附近的工程亦和邦迪联运站和回线工程无不关系。包括一连串的讯号和机械的工程。使得崖勤猎站可以在邦迪联运站进行主要工程时或有重要事故时在不影响其余系统而利用该站作为终站。每天每小时会有6班列车,黄昏时和周未的每小时都会4班列车,在繁忙时间会有更多班次。1号月台2号月台","text2":"崖勤猎站有哪些部分组成?","label":1} {"text1":"龚鹏程,台湾教育家、作家,出生于台湾台北市,籍贯江西省吉安市,曾任《国文天地》总编辑、台湾学生书局总编辑,是台湾佛光大学与南华大学的创校校长、中华武侠文学学会会长,现任卢森堡欧亚大学马来西亚校区校长,游历中国大陆,任北京大学客座教授、南京大学客座教授、北京师范大学特聘教授、四川大学讲座教授,作育英才无数,著作七十余种。少年时代钟情中国武术,并由习武而博考文献,获得许多文献学的知识与方法。龚鹏程一直对于台湾的部分「本土派」学者的台湾文学研究论点与研究方法表达不满;1997年,他发表自著《台湾文学在台湾》,批评该类学者惯用的单线叙述法及其造成的简化历史等问题。2005年9月11日,玄奘大学中国语文学系兼任讲师杨宗翰说,他就读硕士班时改攻台湾文学,因此深知,囿于文化认同上的差异,台湾许多本土派学者都非常讨厌龚鹏程,却又没办法跳过或忽视龚鹏程所提出的诸多尖锐质疑。2003年12月11日,《台湾壹周刊》第133期报导,2000年10月佛光人文社会学院校长龚鹏程带诺贝尔文学奖得主高行健与该校十余名教职员去宜兰县礁溪乡富野酒店喝花酒,2001年2月龚鹏程再次带高行健喝花酒。2003年12月15日,高行健从法国传真声明稿给龚鹏程与该校教授马森,批评该文报导不实,请马森代他要求《台湾壹周刊》公开更正并回函道歉,否则提告;龚鹏程则说,没有一起喝花酒之事。《台湾壹周刊》总编辑裴伟表示,该文内容皆是采访「佛光山能参与事情的人」而得,采访过程皆有录音、记录,也向龚鹏程求证,并写入龚鹏程否认的话,已平衡报导;该刊将再做进一步求证,如有错误,将予更正。2003年12月18日,《台湾壹周刊》在《台湾苹果日报》刊载四分之一篇幅的道歉声明向高行健道歉;同日,高行健批评,该篇道歉声明毫无诚意,该刊必须发表有诚意的道歉声明向该文涉及的所有受害当事人公开、郑重道歉,并向读者保证将杜绝造谣、不实的报导;裴伟回应,该刊会在下期刊登道歉声明,但绝对不会对龚鹏程道歉,因为关于龚鹏程的部分有全部的证据。《台湾壹周刊》第135期刊登道歉声明,向高行健道歉。","text2":"《台湾壹周刊》为何不对龚鹏程道歉?","label":1} {"text1":"新切尔卡斯克(),是俄罗斯罗斯托夫州西部的一个城市,距首府罗斯托夫东北30公里。2002年人口170,822人。切尔卡斯克是由马特维普拉托夫,在1805年作为顿河Voisko土地,当了Cherkassk斯坦尼察的居民被迫离开的顿河银行就经常到其住处inundations行政中心。在苏维埃国内革命战争其间,新切尔卡斯克是顿河反革命心脏和下阿列克谢莫维奇卡莱丁指挥部。红军终于1920年1月7日击败了盘踞在此的白军并解放了这个镇。第二次世界大战期间,新切尔卡斯克一度与1942年7月24日及1943年2月13日被德国军队占领。切尔卡斯克曾是总主教看到希腊东正教大教堂,并有一个(1904年)中,对哥萨克艾泰曼宫殿,并马特维普拉托夫和叶尔马克季莫费耶维奇(Mikeshin,1904)的纪念碑。 2005年9月期间,另一古迹200周年庆祝活动,致力于白色和红色哥萨克和解,是揭开了罗曼诺夫家族的成员出席。1805年建城,1806年成为顿河州首府。闻名全苏联的诺沃切尔卡斯克电力机车厂就位于这座城市,其产品畅销欧亚,中国铁路8G型电力机车就是该厂产品。","text2":"新切尔卡斯克距首府罗斯托夫东北多少公里?","label":1} {"text1":"珍娜·荷兹(,)是一位美国电视色情演员,于2001年进入色情电影界。在2002年至2005年,她是吉儿·凯莉制作公司旗下女星。这段期间,荷兹大多只与女性表演,以表示对当时的摄影师男朋友的忠诚。她在2006年获得多项大奖的作品《Jenna Haze Darkside》中重新与男性演出。荷兹赢过多项成人产业大奖,包括2003年成人影带新闻奖的最佳新人和2009年的成人影带新闻奖的年度最佳女演出者。荷兹出生于加利福尼亚州富勒顿,成长于,也曾住在兰开斯特和明尼苏达州 。荷兹有两位姐姐,一位哥哥。 她有西班牙、德国和爱尔兰血统。她小时在学校的成绩出色,但是在初中时「发现了男孩和性」。于15岁时决定退学,开始在家自学,也找到了第一份工作。她做过一系列的低薪工作,像是速食经理及换油女郎等等。18岁时,荷兹尝试脱衣舞表现。 于19岁时,·荷兹在安纳罕一间她最爱的夜总会中,男朋友将他介绍给好友──知名色情男星彼得·诺斯和演员兼导演克雷文·穆尔黑德(Craven Moorehead),荷兹欣然接受穆尔黑德所提出的拍片邀请。两日后,她拍摄了第一部性爱场面。 在进入成人业之前,荷兹取好了艺名;珍娜此名是她在派饼餐厅工作时,为了与名为珍妮佛(Jennifer)的同事区分而所使用;荷兹是为了纪念当时的未婚夫和她最爱的歌曲:吉米·罕醉克斯的《Purple Haze》。","text2":"珍娜·荷兹拥有哪几国血统?","label":1} {"text1":"物理世界奇遇记(英文:The New World of Mr Tompkins)是美籍俄裔物理学家乔治·伽莫夫的一部科普小说。小说以主人公、银行职员汤普金斯先生(Mr Tompkins)不断的听讲座和在梦境中游览物理世界为线索,介绍了相对论和量子论的相关内容。该书的前身是于1940年出版的《汤普金斯先生身历奇境》。1944年,出版了续集《汤普金斯先生探索原子世界》。1965年,伽莫夫将两本书合并并进行了补充和改写,作成了《平装本里的汤普金斯先生》(\"Mr Tompkins in Paperback\"),即《物理世界奇遇记》。在伽莫夫去世后,英国科普作家罗素·斯坦纳德在1999年对这本书进行了修订,出版了《汤普金斯先生的新大陆》(\"The New World of Mr Tompkins\"),即《物理世界奇遇记》的最新版本。","text2":"《物理世界奇遇记》的作者是谁?","label":1} {"text1":"九龙巴士62X线是香港的一条繁忙时间巴士路线,来往屯门市中心及鲤鱼门邨。本线服务时间以外则由259D线绕经本线屯门市中心至海珠路一段。本线并不途经屯门公路转车站。全程:$16.6乘客登上本线后150分钟内以同一张八达通卡转乘以下路线,或从以下路线登车后150分钟内以同一张八达通卡转乘本线,次程可获车资折扣优惠:现时本线使用8辆丹尼士三叉戟12米(ATR)、2辆富豪超级奥林比安12米(3ASV)及1辆亚历山大丹尼士Enviro 500 MMC(ATENU)。屯门市中心开经:屯门乡事会路、海珠路、海皇路、皇珠路、屯门公路、青朗公路、青衣西北交汇处、长青公路、长青隧道、青葵公路、支路、呈祥道、龙翔道、蒲岗村道、彩虹道、彩虹通道、太子道东、观塘道、隧道、观塘道、鲤鱼门道、高超道及欣荣街。鲤鱼门邨开经:欣荣街、茶果岭道、高超道、油塘公共运输交汇处、高超道、鲤鱼门道、观塘道、隧道、观塘道、天桥、龙翔道、呈祥道、支路、青葵公路、长青隧道、长青公路、青衣西北交汇处、青朗公路、屯门公路、皇珠路、海皇路、海珠路、屯门乡事会路、屯兴路、屯喜路及屯汇街。","text2":"此线路是否途经屯门公路转车站?","label":1} {"text1":"熊津都督府(朝鲜语:웅진 도독부),是唐朝与新罗灭亡百济后,在百济故地建立的羁縻府,由百济遗民管理,后在新罗与唐的战争中,被新罗吞并。660年,百济被新罗和唐朝的联军灭亡。百济故土划分为熊津(今韩国忠清南道公州)、马韩(今韩国全罗北道益山)、东明(今韩国忠清南道扶余郡)、德安(今韩国忠清南道论山市恩津面)、金涟(不详)五个都督府。熊津都督府下辖熊津、东明、支浔、鲁山、古四、沙泮、带方、分嵯8州,治所在熊津州,都督、长史、司马、刺史都由百济人担任。从显庆五年(660年)至龙朔三年(663年),百济复国运动兴起,唐朝与新罗联合镇压,其间为了军事行动的需要,将五个都督府合为一个熊津都督府。由唐朝将领王文度担任熊津都督,王文度死后,由镇守百济故都泗沘的唐朝将领刘仁愿为熊津都督、带方州刺史。百济复国运动失败后,刘仁愿改任百济都护,唐高宗任命百济国末代国王义慈王的儿子扶余隆任熊津都督。但扶余隆害怕受到新罗国的侵略,未敢赴任,只好由唐朝将领刘仁轨检校熊津都督。高句丽灭亡后,刘仁轨、刘仁愿回国,熊津都督依然是未敢赴任的扶余隆,代行其事的是百济人熊津都督府长史难汗、熊津都督府司马祢军。后来百济国的国土大多被新罗国兼并。薛仁贵为鸡林道总管,协助熊津都督府,同新罗作战,最终失败。677年,熊津都督府被从泗沘迁至建安故城(今辽宁营口),与隶属安东都护府的建安州都督府合并。686年,新罗在熊津州也设置同名的都督府,757年,改称熊川。940年,高丽国建立后改熊川为公州、设置都督府。983年,升格公州府。","text2":"扶余隆为什么不敢上任?","label":1} {"text1":"水星音乐奖(\"Mercury Prize\",又称\"Mercury Music Prize\"、\"Nationwide Mercury Prize\"),是英国一年一度的音乐奖,表扬对象是英国或爱尔兰的年度最佳专辑。水星音乐奖由英国唱片业协会(\"BPI\")及英国唱片商公会(\"BARD\",\"the British Association of Record Dealers\")于1992年创办,作为全英音乐奖之外的音乐奖项选择。水星音乐奖原是由现已倒闭的水星电信公司赞助,1998年、2002年分别改由Technics、Panasonic赞助,2004年起则是由全英房屋抵押贷款协会(\"Nationwide Building Society\")赞助。评审团是由英国及爱尔兰的音乐家、音乐企业高层、记者等唱片工业人士所组成,并且通常于七月公布提名专辑、九月举行颁奖典礼。年度得主以粗体标示。","text2":"水星音乐奖的表扬对象是谁?","label":1} {"text1":"独立董事(,或称独立非执行董事、外部董事、独董),是指独立于公司股东且不在公司内部任职,并与公司或公司经营管理者没有重要的业务联系或专业联系,并对公司事务做出独立判断的董事。也有观点认为,独立董事应该界定为只在上市公司担任独立董事之外,不再担任该公司任何其他职务,并与上市公司及其大股东之间不存在妨碍其独立做出客观判断的利害关系的董事。通俗来讲,独立董事不在公司任职,不参与具体事务,没有公司股票,能为公司出谋划策略。独立董事主要义务是加强企业内部控制,利用角色上的独立性不受利益牵制,在股东会时能勇于持保留意见或反对意见并且留下会议记录。以下是《证券交易法》规定独立董事持保留或反对意见时必须在会议记录载明的事项:不过实务上由于刑责不够具体,因此公司治理出现问题时,独立董事通常不会有法律责任,只会承受短时间的舆论压力。","text2":"通俗来讲,独立董事平时在公司如何为公司的发展出谋略?","label":1} {"text1":"文森特·孔帕尼(Vincent Kompany,),是一名比利时足球运动员,司职中后卫。孔帕尼身材高大而且身体素质出众,是「欧洲最有天份的球员」之一。孔帕尼现时为英超球队曼城与比利时国家队双料队长。孔帕尼防守能力出众且组织后防能力极强,曼城有无孔帕尼的后防线强度天差地远。但是非常容易受伤,是标准玻璃人,也因此大大影响孔帕尼的总体评价。2008年奥运会,他入选了比利时国奥队,2008年8月12日,汉堡主教练祖尔在球会网页发表声明,要求甘宾尼在奥运分组赛第二场赛事后回到汉堡。最后,甘宾尼和队友费莱尼在首场对巴西国奥队的赛事中,两人均两黄一红被逐离场,结果二人均在第二场小组赛停赛。甘宾尼其后依循汉堡的指示回到球会,准备在8月15日的德甲首战对拜仁慕尼黑,成功助球队以2-2迫和拜仁。2008年8月21日,汉堡官方网页宣布已与曼城达成协议,甘宾尼将转投该英超球队。","text2":"孔帕尼是哪个国家的运动员?","label":1} {"text1":"裸狐鲣(学名:),又称裸䲠,俗名长翼,为辐鳍鱼纲鲈形目鲭亚目鲭科的一种。本鱼分布于印度太平洋区,包括南非、东非、红海、叶门、模里西斯、马达加斯加、葛摩、塞席尔群岛、留尼旺、马尔地夫、斯里兰卡、印度、缅甸、安达曼群岛、圣诞岛、可可群岛、中国、日本、韩国、台湾、菲律宾、越南、印尼、澳洲、斐济、新喀里多尼亚、东加、吐瓦鲁、密克罗尼西亚、萨摩亚群岛、法属波里尼西亚等海域。水深0至100公尺。本鱼体纺锤形,横切面近圆形,两背鳍几乎相连,在第二背鳍后方有6至7个离鳍,而在臀鳍后方有6个离鳍。尾鳍末缘呈弧形凹入,侧线完全,侧线后半部呈波浪状起伏,且在尾柄处和隆起的棱嵴相连。体背侧呈蓝紫色,腹部灰白色,身上无明显花纹或斑点。上下颔均具尖锐犬齿,背鳍硬棘13至15枚;背鳍软条12至14枚;臀鳍硬棘0枚;臀鳍软条12至13枚;脊椎骨38个,体长可达248公分。本鱼为沿岸中层洄游性鱼类,常一大群一起出现,属肉食性,主要以小鱼为食。为美味的食用鱼,属经济鱼种,可煮汤、盐烧或制成罐头,有雪卡鱼中毒之纪录。","text2":"裸狐鲣的俗名是什么?","label":1} {"text1":"塔武连特山国家公园()坐落于西班牙加那利群岛拉帕尔马岛,包括绵延宽阔的塔武连特山,该山为占据了岛的北部的一个大型火山。它于1954年成为国家公园。塔武连特山直径大约为10-{zh-hans:千米;zh-hk:千米;zh-tw:公里;}-。其最高点为穆查丘斯罗克,海拔2426-{zh-hans:米;zh-hk:米;zh-tw:公尺;}-,可以通过公路抵达。穆查丘斯罗克天文台的望远镜架设于最高点附近。15世纪,西班牙征服加那利群岛时期,这里是当地原住民关切人最后抵抗之处。入侵的西班牙人难以攻破此处,他们只好以举行和谈为借口将关切人首领诱出。公园里主要的植物为加那利松和濒危物种加那利刺柏。","text2":"公园里主要的植物有什么?","label":1} {"text1":"沙特雷站()是巴黎地铁1、4、7、11和14号线的换乘站,位于巴黎第一区和巴黎第四区的交界处。是巴黎最繁忙的地下铁路车站之一。沙特雷车站分为南北两部分。在南部,7号线月台位于约瑟夫河边街地底,11号线位于维多利亚街地底。在北部,1号线位于里沃利路地底,4号线位于大堂路地底,14号线在里沃利路和大堂路之间地底,呈对角线状。两部分通过一个长的滚梯通道相连,并且还与区域快铁大站沙特雷-大堂站通过另一个滚梯通道相通。从最南边的7号线月台到北边的沙特雷-大堂站月台要走大约750米的路程。除11号线设置了一岛一侧混合式月台之外,经过该车站的每条地铁线均有两个侧式月台,但仅有14号线月台安装月台闸门。里沃利路出口:里沃利路112号。维多利亚街出口:维多利亚街9号。沙特雷广场出口:夏特雷广场。圣奥宝图恩广场出口:圣奥宝图恩广场8号。拉方蒂耶路出口:拉方蒂耶路19号。圣德尼路出口:圣德尼路5号。费朗内里路出口:费朗内里路33号。贝当普瓦雷路出口:贝当普瓦雷路20号。圣马丹路出口:维多利亚街13号。城市剧场出口:维多利亚街15号。沙特雷剧场出口:维多利亚街17号。维多利亚街16号出口。巴黎大众运输公司巴士:21, 38, 47, 58, 67, 69, 70, 72, 74, 75, 76, 81, 85, 96.夜宵车:N11, N12, N13, N14, N15, N16, N21, N22, N23, N24, N120, N121, N122.Statistiques du STIF, pg 16","text2":"沙特雷站是巴黎地铁几号线的换乘站?","label":1} {"text1":"本条目为主要语言人口列表,罗列现时世上主要语言的语言人口,并依其人口排序。所谓语言人口,系指以该语言为母语的人口。由于不同统计之间的数据有所分别(有以母语为基础,或以第一语言为基础),本列表试图从现有的数据中整理出一个大概出来。不过,本列表列出的数据,可能会跟各种语言本身条目内的数据有出入。主要原因,是因为条目可能各自引用不同的数据来源,而这些不同的数据来源对“语言”及“方言”有不同的定义。举例说:阿拉伯语在《民族语》 (Ethnologue) 被归入一种语言内,但在美国中央情报局出版的《世界概况》里却被认为是一个语言集合。又例如:中文本身不论是语言及文字本身都有不同的种类。有观点把汉语看做一种语言,而官话、吴语、粤语、闽南语等作为汉语的方言来处理,而得出中文的人口有12亿人。亦有观点把这些方言看作10多种独立的语言,那么其中亦有5种语言(官话、吴语、粤语、闽南语、晋语)有资格列有全球25大语言人口内。相反地,印地-乌尔都语实际上只是一种语言,在下表中却被分开了。下表列出使用人口超过5000万人的语言,根据2017年版民族语。但使用人口总计可能不准确,基于不同时间或资料来源,数据存在差异。","text2":"阿拉伯语在美国中央情报局出版的《世界概况》里被认为是什么?","label":1} {"text1":"杜拉铝(英文:duralumin、duralumin、dural),又称硬铝,是最早的硬化铝合金,主要合金元素有铜、锰及镁。现在常用的合金类型是AA2024,其中有(重量百分比)4.4%铜、1.5%镁、0.6%锰。典型抗拉强度450Mpa,强度的差异主要取决于温度。杜拉铝因为比强度高,是飞机上常用的合金.但因为主材料铝在660度左右就会熔化,高速的喷射机会使用其他耐高温的合金,例如钛合金。杜拉铝由德国冶金工程师在Dürener Metallwerke Aktien Gesellschaft开发出来。在1903年Wilm发现含4%铜的铝合金淬火后会在室温下慢慢硬化。1909年进一步改善了杜拉铝的性质 。现在铝铜合金系统在国际合金命名系统(International Alloy Designation System)中被指定为2000系列铝合金,杜拉铝这个名字仅用于科普描述上。首次使用是用于飞艇的框架上。它的组成和热处理方式在战时是一个秘密,1930年代杜拉铝迅速在航空界传播,同时被引进制造飞机的单体技术。杜拉铝在精密工具也相当受到欢迎,质量轻也有强度。杜拉铝中加入了铜,虽然提高了强度,同时也使合金容易被腐蚀,在表面加上一层高纯度的铝板,抗腐蚀性将大大的提升。这些板料被称为,常用于航空业。锻造用铜铝合金常见名单:","text2":"杜拉铝是谁开发出来的?","label":1} {"text1":"Casey是一款西文无衬线体,是九广铁路公司为其铁路系统企业形象而设计的字体。Casey字体家族细分下可分成Casey Regular、Casey Medium和Casey Bold。同时Casey可以是一个人名,女性名字。举例 : Casey is Jason's girlfriend.据闻Casey这个名称是来自九铁英文简称KCR头两个字母的谐音(KC>Casey)。于网上亦有人制作Casey Replica字体以模仿Casey字体,都是由Myriad字体系列修改而成的。其与Casey的分别是前者间隔较小。实际上,Casey字体和其他相似字体例如Myriad和Segoe UI的分别只在于其字体显得较瘦,而数字则以Formata Cond加以修改。Casey是九广铁路公司委托Dalton Maag制作,于1996年开始率先使用在九铁标志英文\"KCR\"及柴油机车编号上。后来于东铁都城嘉慕电动列车翻新后,随后来港服务的SP1900型电动列车及轻铁在1998至2002年间翻新的第1、2期列车及分别在1997年购入的轻铁第3期列车均于列车编号及车内告示均用上此字体。九广西铁通车时,更广泛拓展到作为指示牌、车站名、宣传单张的官方字型。但个别机构如恒生Cash dollars的宣传品、领展商场的英文专用字型亦有做用类似Casey的字体。2008年起,长虹电视字体用上了Casey。但显示的Casey字体数字是等宽的,不是比例的。康文署辖下公园及运动场的部分指示牌亦采用Casey字体。","text2":"Casey字体和其他相似字体有什么区别?","label":1} {"text1":"《二十二史考异》又名《廿二史考异》,清代钱大昕撰,100卷,附三史拾遗五卷、诸史拾遗五卷。钱大昕少年专心史学,四十岁开始撰写《二十二史考异》,“岁有增益,卷帙滋多”,至五十五岁编定为一百卷。嘉庆二年(1797年)全书刻成。《二十二史考异》系统地考证二十二部正史,尤以《元史》费力极多。《二十二史考异》于二十四史中仅缺《旧五代史》和《明史》。与赵翼《廿二史劄记》、王鸣盛《十七史商榷》并称清代三大史学名著。梁启超认为此书成就可比王引之的《经义述闻》。林理炯称赞《廿二史考异》乃钱大昕一生精力所注之结晶。周振鹤曾比较钱大昕、赵翼、王鸣盛三人,并指出三人之学问有龙、虎、狗之别,钱大昕《廿二史考异》是“点石成金”,赵翼《廿二史劄记》是“披沙沥金”,至于王鸣盛《十七史商榷》最差,是脸上贴金之滥竽。吴绍烈以《廿二史考异》与《宋史》互校,指出钱大昕考证中的一些失误。","text2":"《廿二史考异》与哪两本并称清代三大史学名著?","label":1} {"text1":"强制接取管制(Forced-access regulation)是指地方强势的私人通讯业者必须让竞争者可以使用其网路来供企业营运之用的管制方法,也就是\"强制开放接取\"。一般对于强制接取管制的解释,认为对于小公司而言,由于高成本而无法建置新的网路设施,因此需利用此种方法来促进竞争。以美国而言,国会在1996年电信法中,强制地方电信公司必须在「管制的费率」之原则分享其线路给竞争者。电信法251条(3)(2)(B)写到:如果不能提供网路元件的接取,将会损害电信业者试图提供接取服务之能力与意愿。是否强制接取对于竞争环境有利,并且会降低价格目前还不能确定,因此在各方经济学者中呈现各说各话的局面。","text2":"美国国会在1996年的电信法中有什么规定?","label":1} {"text1":"和硕特汗国(),是1642年蒙古和硕特部首领固始汗在青藏高原建立的政权,从此确立了藏传佛教格鲁派(黄教)在西藏的统治。和硕特汗国始于1642年,于1717年亡于准噶尔汗国。1634年,格鲁派的四世班禅与五世达赖邀请蒙古和硕特部首领图鲁拜琥(即后来的顾实汗)入藏。1641年,顾实汗发兵攻打藏巴汗,包围了日喀则的藏巴汗府邸。1642年,藏巴汗兵败被俘,不久遇害。顾实汗继续西进,降服了后藏地区,并亲自驻守日喀则,命其长子达延汗驻守拉萨,分兵控制西藏各地,成为实际统治西藏的汗王和格鲁派的护教法王,被清朝顺治皇帝封为“遵行文义敏慧顾实汗”。顾实汗将八个儿子留在青海,护卫汗国,史称八台吉。1716年,准噶尔汗国出兵征服西藏,攻打拉萨。1717年,准噶尔军队攻入拉萨,杀死了和硕特汗国末任可汗拉藏汗,和硕特汗国灭亡。其后,西藏遂为清军控制。","text2":"和硕特汗国在哪里建立的政权?","label":1} {"text1":"张处瑾(),燕人,五代十国初期成德节度使留后张文礼的儿子。921年,张文礼发动兵变,杀死了赵王王镕全家,自立为留后。他既称臣于晋王李存勖又私通后梁,还勾结契丹耶律阿保机。李存勖以为王镕报仇为名,以王镕旧将符习为成德留后,又派相州刺史史建瑭、天平节度使阎宝,率军讨伐镇州。八月十一,晋军攻下了赵州,张文礼大惊腹疽发作而死。张处瑾秘不发丧,多次向李存勖谢罪未果,坚守镇州一年有余,晋军遭受重大打击,史建瑭阵亡,阎宝羞愤而死,随后的主帅昭义节度使李嗣昭、振武节度使李存进也相继阵亡。期间张处瑾又被弟张处球夺权。922年九月廿九,镇州粮尽,第四位晋军主帅李存审(符存审)破城,张处瑾及张处球、弟张处琪及张文礼妻(未知是否张处瑾母)被李存勖处死,张文礼的尸体在市上被车裂。","text2":"张处瑾是谁?","label":1} {"text1":"济南革命烈士陵园,也称英雄山革命烈士陵园,位于济南市南部英雄山(原名四里山)南麓、五里山西麓,建于1949年11月至1968年,为山东省规模最大的烈士陵园。其建设目的为埋葬1948济南战役中战死的华东野战军一方军官及士兵,并对济南市民免费公开开放,以达到“爱国主义”教育目的,与济南市市区内解放阁景观遥相呼应。该陵园建筑群体分三部分,北部为位于英雄山山巅的革命烈士纪念塔,通高约34米,塔身使用乳白色花岗岩砌成,顶部嵌有以红色花岗岩刻成的五角星,中部南北两面为“革命烈士纪念塔”七个馏金大字,由毛泽东题写,塔基为双层,南北两面镌刻有花圈图案浮雕;中部为建于五里山西麓的烈士骨灰堂及事迹陈列室(已扩建为济南战役纪念馆);南部为位于六里山西北的烈士墓区,中共一大代表、山东地方党组织创始人之一王尽美和早期中共山东省委书记刘谦初的墓也迁建于此。","text2":"济南革命烈士陵园是什么时间建立的?","label":1} {"text1":"第31届日本众议院议员总选举,在昭和42年(1967年)1月29日举行。此次大选,自民党虽然所得议席因为「黑雾事件」的影响略微减少,但是加上同盟的保守派无党籍议员,稳定取得绝对多数议席。社会党遭遇惨败,发表败北声明。而本届选举增加的定额议席和众议院解散时缺员的议席,基本都流入民社党和公明党,中间派政党势力取得跃进。此届选举令战后自民、社会两大党基本垄断政坛的时代结束,由于社会党的失败,日本走向多党制时代。自治省统计 - 比上届增加2.85%获得议席数:277议席获得议席数:140议席获得议席数:30议席获得议席数:25议席获得议席数:5议席9议席","text2":"第31届日本众议院议员总选举在哪一年举行?","label":1} {"text1":"政府间海洋学委员会按联合国教科文组织大会第2.31条而成立。第一次会议在1961年10月27日于巴黎的联合国教育科学文化组织总部举行。共有40个国家成为委员会的创始成员国。至2004年委员会一共有129个成员国,设有会员大会,执行理事会和秘书处。秘书处设在法国巴黎。此外,委员会亦设有一些附属机构。国际太平洋地区海啸预警系统协调小组。40个成立会员国分别是:阿根廷,澳大利亚,比利时,巴西,加拿大,智利,中国,古巴,丹麦,多米尼加共和国,厄瓜多尔,芬兰,德意志联邦共和国,法国,加纳,印度,以色列,意大利,象牙海岸,日本,韩国,墨西哥,毛里塔尼亚,摩纳哥,摩洛哥,荷兰,挪威,巴基斯坦,波兰,罗马尼亚,西班牙,瑞士,泰国,突尼斯,苏维埃社会主义共和国联盟,阿拉伯联合共和国,联合王国,美利坚合众国,乌拉圭,越南。","text2":"委员会亦设有什么附属机构?","label":1} {"text1":"马良弼(;)号汝舟,和名名护亲方良丰(),琉球国第二尚氏王朝时期大臣。童名太良金。他是马世荣(名护亲方良员)之子,马良诠(名护亲方良意)之孙。1551年生于首里城。1579年,为进贡及谢册封尚永王之恩事,以王舅的身份,与长史郑迵(谢名亲云上利山)一起出使明朝。在北京期间,萧崇业、谢-{杰}-赠送给马良弼「国佐元勲」四字匾额。归国后,因功升三司官座敷。1592年,其父马世荣退隐。因兄长马良辅(伊计亲方良真)早卒,马良弼遂以次子的身份继承家督之位。同年就任三司官,领名护间切总地头。当时日本逐渐强大,岛津义弘致书琉球,声称日本要出兵朝鲜,要求琉球提供粮草支援。马良弼与翁寄松(城间亲方盛久)都认为琉球弱小、日本强大,主张对日本和好,同主战派的郑迵、向里瑞(浦添亲方朝师)意见相对。1605年,翁寄松在与郑迵的政治斗争中失势,尚宁王转而倾向对日强硬的态度。1606年,册封使夏子阳、王士桢到达琉球,赠送给马良弼「世承天宠」四字匾额。1609年,萨摩藩派遣桦山久高、平田增宗率兵入侵琉球。尚宁王得知消息后,派马良弼、毛凤朝、菊隐、喜安等人前往北山。马良弼在此期间的行为,各史料是矛盾的。一说马良弼率一千名首里亲军前往北山防御,但中萨军伏兵之计,兵败被俘;马良弼仰天痛哭而不投降,因此受到萨军的尊敬。而另一种说法是马良弼前往北山的运天港,至萨摩军中请罪,并留在萨摩军中作人质,引导萨摩军开入那霸港。萨摩军包围首里城后,尚宁王向萨摩投降,并先至马良弼府邸暂时居住。尚宁王等人被掳至鹿儿岛时,马良弼同毛继祖(丰见城亲方盛续)被留下,一同管理首里城。1611年尚宁王归国后,马良弼继续担任三司官之职。1612年(万历四十一年),再以谢恩王舅的身份出使明朝,并就进贡之事同明朝交涉。翌年归国。马良弼于1614年致仕,由毛继祖继任其职。1615年,尚宁王向明廷报称马良弼因交涉进贡一事失败而遭到处决;事实上,根据《中山世谱》的记载,马良弼并未遭处死,翌年出使萨摩藩,告知赴明谢恩完竣之事。1617年逝世。","text2":"当时岛津义弘致书琉球,声称日本要出兵朝鲜,要求琉球提供什么支援?","label":1} {"text1":"可可·克里斯普(Covelli Loyce \"Coco\" Crisp,)出生于加利福尼亚州洛杉矶;是大联盟中克里夫兰印地安人的中外野手。他除了是一位左右开弓的打者之外也具备良好的防守技巧。他的速度也是他的特点之一。2008年11月19日,红袜队和皇家队进行一笔交易,皇家队用中继投手雷蒙·拉米雷斯(Ramon Ramirez)换来了克里斯普。2009年12月20日,克里斯普和奥克兰运动家签下1年500万美金加上2011年球队选择权的合约。2016年8月31日,克里夫兰印地安人队和奥克兰运动家队进行一笔交易,克里斯普时隔14年回到老东家,和球队总教练特里·弗兰克纳再次合作。","text2":"克里斯普和奥克兰运动家在2009年12月20日的合约中除约定1年500万美金外主要还有什么内容?","label":1} {"text1":"基础代谢率(,首字母缩写)是指在自然温度环境中,恒温动物(比如人)的身体在非剧烈活动的状态下,处于消化状态(肠胃充满食物,分解作用大于合成作用),维持生命所需消耗的最低能量。这些能量主要用于保持各器官的机能,如呼吸(肺)、心跳(心脏)、腺体分泌(脑及其他神经系统)、过滤排泄(肾脏)、解毒(肝脏)、肌肉活动等等。基础代谢率会随着年龄增加或体重减轻而降低,而随着肌肉增加而增加。疾病、进食、环境温度变化、承受压力水平变化都会改变人体的能量消耗,从而影响基础代谢率。基础代谢率的测量需要在严格的条件下进行,受测者必须处于清醒且完全静止状态,同时其交感神经系统需要保持非激活状态。另一种相关的条件但较宽松的测量是“静止代谢率”(单纯维持生理消耗,此数据用在描述变温动物居多)的测量基础代谢率和静止代谢率的测量可以采用量热法来进行气体分析而直接获得结果,或通过一个包含有年龄、性别、身高、体重的公式来进行间接估算。基础代谢率的单位为“千焦\/平方米\/小时”,表征每小时每平方米体表所散发的热量。","text2":"维持生命所需消耗的最低能量主要用于什么?","label":1} {"text1":"士徽(粤语:\"Si Fai\",越南语:\"Sĩ Huy\";)是东汉末年至三国时代交州政权的君主。为苍梧郡广信人,是士燮的第三子。226年,其父士燮病死,士徽继任统治交趾。孙权认为交趾路途险远,于是分合浦郡以北设立广州,并任命吕岱为广州刺史;交趾以南为交州,戴良为交州刺史。士徽被罢免交趾太守一职,改任安远将军领九真郡太守;取代士徽的则是孙权的亲信陈时。此事引起了士徽的不满,于是发兵阻止戴良和陈时赴任。有一个士徽的官吏叫做桓邻,叩头告诉士徽应该迎接戴良,却被士徽发怒笞杀。桓邻的哥哥的儿子桓发派亲族军队攻击,几个月没能攻下,于是双方约定和亲作罢。与此同时,孙权派吕岱前往诛杀士徽等人。吕岱透过士徽的堂弟士匡告诉士徽,欺骗他说,若他投降,孙权将予以赦免。士徽见守城无望,在兄长士祗的劝说下开城投降,兄弟等六人赤裸出降。吕岱另他们穿上衣服,第二天设宴招待士氏一族,当众宣读孙权的诏书,历数士徽的罪过。兵士就将士氏一族全部拉出斩首,并把首级送往武昌。士壹和士䵋被赦免,然而与士燮的儿子士廞都被免为庶人。过了几年之后,士壹和士䵋也因犯法被诛杀了。士廞因病身亡,没有留下子嗣。","text2":"孙权派谁前往诛杀士徽等人?","label":1} {"text1":"描述法是集合论(或者类的理论)中表示集合(或类)的一种方法。在一般情况下,只要给出一个关于元素的性质,就能依该性质构造出一个集合(或类),因此我们可以用描述该性质的方法来表示相应的集合(或类)。如用{x|x≥1.2}表示不小于1.2的全体实数,等等。在描述法中,对于客观问题,我们可以直接将性质描述在花括号中,这种方法称为“直接描述法”;对于数学问题,我们通常先用一个字母(或其元素的一般形式)代表其元素的“通项”,然后用数学表达式给出通项应满足的条件,这种方法称为“通项描述法”,“|”是通项描述法的特征符号。通常的描述法格式是{x|P(x)},它表示在论域内满足性质P的所有元素组成的集合(或类)。有时必须显式地给出全集(或论域),如{x|x>1且x∈N}表示大于1的全体自然数,这时的描述法格式为{x|P(x),x∈Ω}或{x∈Ω|P(x)}。因描述法的本质是给出了类的内涵,因此又称为内涵法。","text2":"描述法是集合论(或者类的理论)中表示什么的一种方法?","label":1} {"text1":"《星战》是香港歌手古巨基的录音室专辑,2005年10月20日由金牌娱乐发行。这张专辑共收录十一首新歌,前十首的歌名乍看之下,是七至九十年代的粤语经典流行歌曲-然而,这并不是一张翻唱专辑。为了向香港乐坛致敬,制作单位借用了这些经典歌曲的歌名,再重新作曲及填词。新曲中的部分歌词亦有和原曲相对应。专辑中第一首发表的歌曲为《Monica》,本是张国荣的经典舞曲,但在重新作曲后变为慢板情歌。MV中与香港小姐冠军曹敏莉饰演情侣。另一首主打歌为《天才与白痴》,MV邀得吴佩慈客串演出。2005年度亚洲游戏展,古巨基被委任为亚洲游戏展大使。专辑中的《明星》成为大会主题曲,MV中更加入PS2游戏侠盗银河的画面。为了配合大会主题,一间英国游戏公司更为他度身订造了「银河猎人」的造型,并成为本张专辑的封面。粗体表示四台冠军歌四大电子传媒其他媒介","text2":"《星战》是在哪年由哪个公司发布的?","label":1} {"text1":"黄喉拟水龟(学名:、)又称柴棺龟、石金钱龟,为泽龟科拟水龟属的爬行动物。分布于中国的安徽、云南、海南、河南、广东、贵州、江西、浙江、福建、广西、香港及越南北部,台湾,日本波照间岛、西表岛、石垣岛及与那国岛,生活于低海拔地区的河流、湖泊等。头部为绿色,有黄色带状纹,上颌橄榄色,下颌黄色;背甲长12~14公分;有三条脊棱,中央一条发达,两侧常不明显,后缘呈锯齿状,灰褐色背面,黄色腹面,散步四角形的黑斑,四肢为橄榄色,具有绿色纵走带状纹;趾间具蹼。材棺龟体格强健,杂食性但偏好肉食,水陆两栖,但较喜欢在水边活动,食物包括叶菜、蟋蟀、面包虫、,鱼虾等。生性胆怯,但略具攻击性,会因为争食而攻击同类或其他龟类,有夜行倾向。雄性尾部粗大,泄殖孔超过背甲边缘,雌龟则尾部细短,泄殖孔距躯体较近。雌龟大于雄龟,每年可产卵1-3次,每次下1-5颗蛋,约65-80天孵化。冬天温度低于20度以下时活动会逐渐减少,最后进入冬眠状态。常见于丘陵地带半山区的山间盆地或河流谷地的水域中,常于附近的小灌丛或草丛中。性情害羞,夜行性乌龟。该物种的模式产地在浙江舟山群岛。在台湾普遍称为「柴棺龟」。","text2":"黄喉拟水龟生活在什么样的地方?","label":1} {"text1":"拆白党()是20世纪20至40年代的上海俚语,泛指上海地区一群纠党并以色相行骗,白饮白食骗财骗色的青少年,多属男性。后来拆白党的声名大盛,连外埠都知道这个名称,凡属骗人财物的案件,国内皆称为拆白行为。「拆白」二字是「拆梢」与「白食」的简语。「拆」即朋友之间瓜分,「梢」即梢板。当时的上海流氓称钱财为梢板,他们会把骗来抢来的钱财瓜分。而因为当时流氓索取酒席白吃白喝也相当盛行,所以在「拆梢」外也加上「白食」,一般人均称这帮流氓为拆白党。后来这帮流氓开始变得有组织地进行行骗,多择富家女眷为行骗目标。另也有说,因为拆白党大半是翩翩少年,那时舶来品的雪花粉流行未久,党员人人乐用,皮肤擦得雪白。所以拆白,系指「擦白」,故拆白党亦称「雪花粉党」。奉行「三白主义」,用餐、看戏不付钱、诱奸女性;即是「吃白食、看白戏、(与异性)睡白觉」。男拆白党徒,有的扮成小滑头,也有伪装成文人,多是俊俏的青少年。党徒中有人专责情报,注意对象多是富家女眷。情报者暗中尾随了解目标的姓名、性情、出入特点、家庭背景后,一一记录向组织汇报,组织遂派合适者前去引诱目标。当目标对拆白党徒产生好感,党徒即以各种手段获利。而女拆白党徒手段与男的相似,但内幕没有太多组织结构,一般不过少数几个人结合在一起的小团体。","text2":"拆白党衍伸后代指什么行为?","label":1} {"text1":"罗马治世(拉丁语:Pax Romana),又称罗马和平,是指罗马帝国存在的五百多年间,前二百年比较兴盛的时期,亦即盛世。公元前30年,屋大维消灭埃及托勒密王朝,结束了罗马内战。一般将这一年视为罗马和平时期的开始,也有一个说法是从公元9年(罗马基本上停止侵略日耳曼地区)开始算起。公元161年,五贤帝中的最后一个─马尔库斯·奥列里乌斯即位,不久后遇上帕提亚入侵亚美尼亚,紧接著又发生瘟疫,他本人更在征伐日耳曼的途中病逝。期间,罗马帝国的财务状况恶化,日耳曼人也开始骚扰边境,帝国由盛转衰。从屋大维统一罗马至马尔库斯·奥列里乌斯过世(公元180年),长达200年左右的时间罗马大致富强稳定,没有较大的战乱,因此史称“罗马治世”。","text2":"五贤帝中的最后一个皇帝是谁?","label":1} {"text1":"罗彻斯特理工学院(Rochester Institute of Technology,缩写:RIT)是一所位于美国纽约州罗彻斯特的大学。罗彻斯特理工学院于1829年建校,1944年改为现在校名。它在2016年《美国新闻与世界报道》的美国全国大学排名中列于第107位。美国联邦政府的国立聋人理工大学(National Technical Institute for the Deaf)也设置其中。国立聋人理工大学在全美以聋哑人作为教学对象的高等教育机关中历史仅次于华盛顿特区的高立德大学,其手语课程在全美居首。接受计算机技术和经营学3年课程后可转入罗彻斯特理工学院其他院系。罗彻斯特理工学院在2016年的美国新闻与世界报道中排名美国全国性大学107名。其本科工程学科排名全国58名,商科本科排名全国77。研究生方面工科排名87,摄影学排名全国第4。","text2":"罗彻斯特理工学院是本科工程学科排名是多少?","label":1} {"text1":"陈开(),广东省三水(一说佛山)人,大成国首领。原为天地会领导人,受金田起义的影响,1854年与李文茂起义包围广州,两广总督叶名琛在英国香港总督宝宁援助下,坚守广州半年不失。陈开转移到广西省,联合西江水勇领袖梁培友,率战船一千艘,先克广西浔州(今桂平)。咸丰五年(1855年)八月,陈李等人入城,建立大成国,改元洪德,发行洪德通宝。改浔州为秀京,以浔州衙门为王府,并蓄发易服,颁发制度,分官设守,开炉铸钱。陈开自称平浔王(初称镇南王),并且分封诸王,李文茂为平靖王、梁培友为平东王、区润为平西王、梁昌为定北王。1858年,陈开率东路军从梧州出发,李文茂率西路军从柳州出发,进攻桂林,蒋益澧率湘军支援桂林。西路军战败,东路军撤退。其后,柳州、梧州失陷。1861年,清军大举进攻秀京。陈开水军全军覆没,退守秀京城内。秀京内地主起事,陈开逃出秀京,想投奔太平天国的石达开军队。途中,被俘杀害。","text2":"陈开为什么与李文茂起义包围广州?","label":1} {"text1":"并州刺史部,是汉朝十三州刺史部之一,州治在晋阳县(今山西省太原市西南)。西汉州域范围大致是今日的山西省大部,东汉时扩大至陕西省北部及内蒙古自治区中部一带。「并州」一词的出现始于春秋以后,后人在《尚书·禹贡》九州基础上,增加幽、-{并}-、营3州,大体指常山郡以北的区域。《汉书·地理志》载:「正北曰并州。:其山曰恒山,薮曰昭余祁,川曰虖池、呕夷,寖曰涞、易;其利布帛;民二男三女;畜宜五扰,谷宜五种。」西汉时,领太原郡、代郡、上党郡、云中郡、雁门郡、定襄郡6个郡国。东汉时,领太原郡、上党郡、上郡、西河郡、五原郡、云中郡、雁门郡、定襄郡、朔方郡9个郡国,东汉末置新兴郡、乐平郡2郡。","text2":"并州一词出现始于哪一年代?","label":1} {"text1":"中华民国空气污染指标(Pollutant Standards Index,PSI)是空气污染情况的一项指标,由中华民国行政院环境保护署于1993年扩充测站后推出,目标乃借由本测站系统监控全台湾所有的空气品质并加以通报改善。空气污染指标为依据监测资料将当日空气中悬浮微粒(PM10)、二氧化硫(SO)、二氧化氮(NO)、一氧化碳 (CO) 及臭氧 (O) 等5种空气污染物浓度数值,以其对人体健康的影响程度,分别换算出不同污染物之副指标值,再以当日各副指标值之最大值为该测站当日之空气污染指标值 (PSI) 。每监测区各取三个指标值最大的测站平均成为该监测区的指标值。转化为一个由0至500的单一数字,并按照指数高低而划分为良好、普通、不良、非常不良和有害五种级别。空气品质测站种类有一般空气品质监测站、工业空气品质监测站、交通空气品质监测站、国家公园空气品质监测站、背景空气品质监测站五种类别。有部分的测站会兼具两种类别。全台湾被分成七个空气品质监测区,北部地区(台北、新北、桃园、基隆)、新竹苗栗地区、中部地区(台中、彰化、南投)、云林嘉义台南地区、高雄屏东地区、宜兰地区、花莲台东地区。监测站位于台湾各地的学校和政府机关为主:台湾空气品质指数分为以下5级:","text2":"台湾有几个空气品质监测区?","label":1} {"text1":"天正之阵天正之阵」是天正13年(1585年)发生在伊予国地方的一系列战争的统称,包括丸山城之战、金子城之战及高尾城之战。1585年(天正13年)与羽柴秀吉(丰臣秀吉)联盟之毛利军派小早川隆景率军在四国的伊予国新居郡(现在的爱媛县新居滨市)登陆,金子城城主金子元宅率领地方势力准备跟有压倒性优势的小早川隆景决一死战。此战为秀吉四国征伐的其中最重要的一战。「天正之阵」的前哨战丸山城之战、丸山城城主黒川广隆因不敌毛利大军而降伏、投降后担任金子城・高尾城略攻的向导役。1585年7月,长宗我部元亲势的金子元宅集结了2000兵力与毛利辉元势小早川隆景的30000兵力战斗,而一些仰慕金子元宅的民众都一齐对抗毛利氏,元宅将金子城的全军指挥交给弟弟金子元春,自率200兵通过石川氏高峠城、驻扎在冰见(现在的西条市)的高尾城。金子元宅离开后,毛利军即大举进功金子城,战况激烈到神社、佛寺都被烧毁,不久,金子城陷落。毛利军随即进攻高尾城。7月12日,两军开始战斗,双方都有很大伤亡。7月17日,金子元宅放火烧掉高尾城,抱著以死殉国的决心在野野市原(野々市ヶ原)与毛利军进行最后决战。最后金子元宅在野野市原战死,600人全军覆没,而小早川隆景为了对金子元宅和他身边金子城、高尾城阵亡将士2000余名死士表示敬意,在野野市原修建千人冢。长宗我部元亲于此战失去了最大据点伊予,防线全崩,对羽柴军节节败退,不久就向秀吉投降。","text2":"什么是天正之阵?","label":1} {"text1":"亲核酰基取代反应(Nucleophilic acyl substitution),即发生在羧酸衍生物酰基碳上的亲核取代反应。反应的表达式如下:亲核酰基取代反应可在酸性或碱性的条件下进行。反应进程如下:首先亲核试剂进攻羰基碳发生亲核加成,形成四面体型中间体。然后消除一个负离子,取代反应结束。因为第一步反应是亲核加成,所以反应物中羰基碳的电正性越大,周围的空间位阻越小,反应越易进行。第二步反应是消除反应,取决于离去基团的性质,越易离去的基团越易发生反应。在羧酸衍生物中,基团离去能力的顺序为:反应进程如下:在酸作催化剂下,羰基氧首先质子化。质子化增加了羰基碳的电正性,第二步发生亲核加成,形成四面体型中间体。最后离去基团离去,反应结束。羧酸衍生物的酰基碳上的一个基团被亲核试剂所取代。如果亲核试剂是水,则产物是羧酸;如果亲核试剂是醇,则反应产物是酯;如果亲核试剂是氨/胺,则反应产物是酰胺。各种羧酸衍生物的反应性顺序如下:如果亲核试剂是有机金属试剂(如格氏试剂、有机锂试剂),则反应先生成酮,而生成的酮往往活性更强,继续与有机金属试剂发生亲核加成反应,生成醇。如果所用的羧酸衍生物是酰胺,则氮上如有活泼氢,便会与有机金属试剂反应,使有机金属试剂的消耗量增加,因此一般不用酰胺作原料进行合成。为了使反应停留在生成酮的第一步,可以用以下方法:","text2":"第一步反应是亲核加成会导致什么?","label":1} {"text1":"在数学上,超现实数系统是一种连续统,其中含有实数以及无穷量,即无穷大(小)量,其绝对值大(小)于任何正实数。超现实数与实数有许多共同性质,包括其全序关系「≤」以及通常的算术运算(加减乘除);也因此,它们构成了有序域。在严格的集合论意义下,超现实数是可能出现的有序域中最大的;其他的有序域,如有理数域、实数域、有理函数域、列维-奇维塔域(Levi-Civita field)、上超实数域和超实数域等,全都是超现实数域的子域。超现实数域也包含可达到的、在集合论里构造过的所有超限序数。超现实数是由约翰·何顿·康威(John Horton Conway)所定义和构造的。这个名称早在1974年便已由高德纳(Donald Knuth)在他的书《研究之美》中就被引进了。《研究之美》是一部中短篇数学小说,而值得一提的是,这种把新的数学概念在一部小说中提出来的情形是非常少有的。在这部由对话体写成的著作里,高德纳造了「surreal number」一词,用来指称康威起初只叫做「number」(数)的这个新概念。康威乐于采用新的名称,后来在他1976年的著作《论数字与博弈》(On Numbers and Games)中就描述了超现实数的概念并使用它来进行了一些博弈分析。康威使用递归构造了超现实数,其中每个数字都是两个数字集合构成的序对,记为{ \"L\" | \"R\" }。这两个集合要求 \"L\" 里的每个元素都严格小于每个 \"R\" 里的元素。不同的序对可能表达同样的数字:{ 1 | 3 } = { 3\/2 | 5\/2 } = 2.不妨来看几个例子。因此整数都是超现实数。(以上几行是定义而非等式!)至此我们可以通过超现实数定义二进分数(分母为2的幂次的分数)。","text2":"《研究之美》是一本怎样的小说?","label":1} {"text1":"松雪泰子,日本女演员,佐贺县鸟栖市出身。自小家境富裕,父亲松雪秀敏先纯是鸟栖瓦斯的社长。泰子就读于佐贺县立鸟栖商业高中,在学期间曾组乐团担任主唱,高中毕业即前往东京,加入演艺界,身高165公分的她在「MEN'S NONO」担任模特儿。1991年,参演电视连续剧《热血!新入社员宣言》,正式成为一名女演员。1993年以《白鸟丽子》一剧走红,随后又参演了《太阳不西沉》、《怪医黑杰克SP》和《真夜中的雨》,以及《砂之器》,原著小说《砂之器》中并没有松雪泰子饰演的「成濑麻美」这一个角色。松雪泰子2006年以《扶桑花女孩》的舞蹈老师平山圆香一角,获「2006年日刊体育电影大奖」最佳女主角奖。2008年,因演出《重金摇滚双面人》和《嫌疑犯X的献身》获日本电影金像奖优秀助演女优奖。2010年,以话题电视剧《Mother》获得第65回日剧学院赏最佳女主角奖。松雪泰子的弟弟为歌手。1998年,松雪泰子与吉他手Gaku结婚,2001年生下一子。2004年12月离婚,松雪泰子独自扶养儿子。","text2":"加入演艺圈之后,泰子一开始从事什么工作?","label":1} {"text1":"奥林匹克公园站 位于北京市朝阳区奥林匹克公园中心区景观路,是北京地铁8号线和15号线的一座换乘站。8号线车站开通时是作为服务于2008年北京奥运会的奥运支线的一座车站,曾经是第29届奥运会观众集散的主要车站。8号线车站位于大屯路南侧奥林匹克公园中轴广场地下,南北向布设。15号线车站在8号线下方,位于大屯路城市隧道下方,预留公交节点西侧,沿大屯路东西向布设。两线呈“L”形相交,通过通道换乘。8号线该站是一座地下两层的车站,岛式站台设计,。车站包含两部分,南半部分为地铁8号线停靠站台,北半部分是地铁与快速公交换乘的地下换乘站台(现在尚未开放)。乘客将可由地铁站台直接由通道到达公交站台换乘公交车。8号线站台吊顶以蓝色金属材料和灯具模仿水的自然形状,与地面附近水立方遥相呼应。车站屏蔽门印满了各种海洋动物的图案,与站台装饰风格相呼应。车站站台内有饮水机,供乘客免费使用。15号线该车站是地下两层三跨结构,岛式站台设计,位于大屯路隧道下方的车站主体为暗挖法施工,车站总长205.5m,标准段宽度为23.3m。8号线车站共有6个通向车站大堂的出入口,其中:东北口 (B)、东口 (C)、东南口 (D) 三个出入口与东侧“礼乐重门”地下广场连接,西北口 (A1)、西南出口 (E) 两个出入口通向西侧的商业步行街,北侧一个通向站内公交站台。现有5个出入口开放。15号线该车站设有3个出入口,换乘厅内另有一个独立存在的I口。","text2":"8号线站台吊顶是如何与地面附近水立方遥相呼应的?","label":1} {"text1":"净闲寺()是位于日本东京都荒川区南千住的净土宗寺院。在吉原游廓附近,因无依无靠的妓女死后「投入寺」知名。因为地处「三轮」,所以也叫做「三轮净闲寺」。1655年(明历元年)创建,比吉原游廓(1657年)早2年。据说于吉原妓院街病死的妓女被送到净闲寺并冠以「某某卖女」的戒名埋葬,事实上没有这样的文字记载,近年的研究表明只有「自杀」、「胡乱发誓」、「逃跑」、「私通」、「吸食鸦片」等坏了吉原规矩的妓女才被会冠以「某某卖女」的戒名。这种情况下,死人被剥去衣服用破草席包裹扔到净闲寺。因为怕死人作祟,所以被当作猫狗处理,使其灵魂落入畜生道。吉原380年的历史中共有25,000名妓女埋葬与此。净闲寺的网页上说,净闲寺是安政大地震(1855年)埋葬大量遇难妓女之后才被称为投入寺的。东京都荒川区南千住2-1-12","text2":"该寺庙什么时候创建?","label":1} {"text1":"《卡美拉3 邪神觉醒》(原题:)是1999年3月6日上映的日本电影,为卡美拉系列电影的第11部作品,也是大映「平成卡美拉三部曲」的完结篇。导演与前两部一样由金子修介担任。1995年时(即首部曲:卡美拉1时的东京),比良坂绫奈一家人因为加欧斯来袭而准备离开东京,此时卡美拉受到加欧斯的攻击而坠落,但在起身时却破坏了绫奈的家,绫奈的父母来不及逃出因而丧生。目睹这一切的绫奈,认为是卡美拉杀死了她的家人,从此开始憎恨卡美拉...1999年,长峰博士在赤道附近的调查发现了加欧斯的变异种,同一时间前后,也有科学家在日本西南方的深海中发现了大量卡美拉枯骨遗留下的一处神秘地点。父母丧生后,借住在乡下亲戚家的绫奈饱受同学欺负,有一天绫奈被霸凌者叫到供奉一个东西叫做\"柳星张\"的神社,并被要求将里面的东西给搬出来,此时世世代代守护著柳星张的守部家成员守部美雪见到此状,于是偷偷的去告知自己的哥哥龙成,龙成便到神社将绫奈的同学赶走,此时绫奈把东西搬出来,龙成便要求绫奈把东西给搬回去,但是在绫奈搬走东西的原处,两人发现了一个奇怪的物体,两人见状便吓得逃走,到了晚上,那个东西出现了变化,里面出现了一个东西...而在此同时,加欧斯出现在东京涩谷上空,没多久之后一阵爆炸声,加欧斯被击中,并且坠落在涩谷车站,此时卡美拉从空中降落...
","text2":"《卡美拉3 邪神觉醒》为卡美拉系列电影的第多少部作品?","label":1} {"text1":"伊藤坛(ITO Dan,),是一名日本足球运动员,于北海道出生,目前效力于东帝汶联赛的足球队。曾效力日本职业足球乙级联赛球队仙台维加泰,随后展开亚洲流浪之旅,先后在澳洲、新加坡、越南、汶莱、泰国、马来西亚、马尔代夫、香港、澳门、印度、缅甸、尼泊尔、柬埔寨、菲律宾、蒙古联赛落班,于2004年到香港加盟香港甲组足球联赛球队杰志两年,并曾入选港联;2008年重临香港短暂效力屯门普高,可司职中场多个位置,包括进攻中场、中场中。2009年到澳门落班加盟升班马名门世家加义,是澳门首名职业球员及日本外援。2009年5月底,于澳门联赛名门世家加义对皇朝一战,开波仅3.11秒即起脚笠射入网,当地电视台亦拍下整个入球过程,会方认为这个入球有机会创下最快入球纪录,已将资料上报亚洲足协等待确认。。","text2":"伊藤坛于澳门联赛名门世家加义对皇朝一战因为什么而出名?","label":1} {"text1":"丁比站()是雪梨城市铁路的东郊及伊拉瓦拉线的一个沿线车站,它服务悉尼的的一个地区—丁比。它位于先邓咸站和涡拉溪站和中间,更与涡拉溪站有一河之隔(即是谷治河(Cooks river))。该站设有1个岛式月台在中心,另2个侧式月台在旁,所有的月台都仍保留著原本的建筑物。在车站北面设有一这行人天桥连接各个月台,但随著东山线的列车不再停下此站,1和2号月台就不会再使用。此站是没有升降机及斜道等伤残人士设施的。。此站随著伊拉瓦拉线在1884年伸延到好市围站而建成。大部份时间每小时会在2班列车,星期一至五的繁忙时间会加设班次。在机场线开幕前,这里是东山线和伊拉瓦拉线分开的最后一个车站;但现时大部份的东山及机场线列车都只会途经机场,而途经先邓咸站的列车亦不会再停此站。1号月台2号月台3号月台4号月台该站不设升降机等伤残人士设施。车站在开放时间会在职员当值。","text2":"丁比站为什么不再使用1和2号月台?","label":1} {"text1":"《驱·逐》(),是2008年奇连·伊士活自导自演的美国电影,先在2008年12月12日在北美部分戏院上映,然后在次年1月9日全面公映。除了-{zh-hk:伊士活;zh-tw:伊斯威特;zh-cn:伊斯特伍德}-外,其他主演的演员有不少是苗族裔美国人。故事敍述一名新近丧偶的韩战退伍军人Walt Kowalski。此人脾气不好,与两个儿子相处不佳。他是福特汽车公司的退休员工,居住在密西根州底特律旁一个已沦为黑人区的小城。他意外的和苗族裔邻居年轻人建立关系并仗义勇为对抗地方不良份子与苗族裔中不良份子。苗族裔受恩涌泉以报,结果身患绝症的他牺牲自己以铲除苗族裔不良份子。片名Gran Torino是主角的爱车的型号,结果他将爱车遗留给苗族裔年轻人。","text2":"Walt Kowalski为什么牺牲了自己?","label":1} {"text1":"《明日·今天的未来》是一本由香港的财政司司长办公室制作的财政预算案咨询漫画。其作品由香港财政司长曾俊华提出、香港漫画家李志清执笔绘画、张正常担任编剧。制作费用约十万港元,并于2009年1月中旬出版。漫画内容在政府财政预算案的网站里连载,供市民自由观看。这也是香港政府第一次以漫画形式宣传和教育。在推出漫画后,曾俊华表示已收到近五千多份意见。漫画主要透过主角一段跨越时空的奇幻旅程,以令到市民明白未来建基于今日之上,当中的内容包含著各种在香港现时在发生的社会问题,例如是失业及金融海啸所带来的冲击等。而在剧情里的同时,也提供著一些经济数据,好让读者运用那些数据来思考一下怎样解决其问题。在漫画里,书中的主角阿日跟他的家人讨论金融海啸之下香港经济的何去何从。","text2":"这部漫画是由谁执笔绘画的?","label":1} {"text1":"亚砜是含有亚硫酰基(>S=O)官能团的一类有机化合物,可由硫醚氧化得到。常见的亚砜有氯化亚砜、二甲基亚砜、二苯基亚砜等。亚砜的通式为R-S-R',其中R和R'是有机基团。亚砜中的成键与氧化叔膦(RP=O)类似,由于电负性差异,S=O键中,硫显正价,氧显负价:硫原子为四面体结构,有一对孤对电子,类似于sp杂化的碳原子。当硫所连接的两个基团不相同时,就会产生手性,比如甲基苯基亚砜。有时构型转换所需的能量相当高,室温下的外消旋速率很慢,以至于对映体相对稳定。有些手性的亚砜在医药中有应用,比如埃索美拉唑和阿莫达非。此外,亚砜也被用作手性辅助剂。亚砜中硫的氧化态为0,处于硫醚(-2)和砜(+2)之间。因此氧化硫醚时,依次会得到亚砜、砜。例如氧化二甲基硫醚时可以得到二甲基亚砜和二甲基砜。很多有手性的亚砜可从非手性的硫醚为原料,在过渡金属和手性配体的存在下以不对称催化氧化反应合成。亚砜可由强碱如氢化钠去质子化,很多亚砜(如DMSO)都是很好的配体和烷化剂。","text2":"亚砜可以怎样得到?","label":1} {"text1":"贫齿鳄(学名:\"Edentosuchus\")是种已灭绝鳄类,属于鳄形超目的原鳄亚目,化石发现于中国新疆准噶尔盆地的吐谷鲁群,年代属于白垩纪早期。目前已经发现两个部分颅骨,以及数节颈椎;除此之外,一个呈天然状态的颅后骨骼,也可能属于贫齿鳄。贫齿鳄是在1973年由中国古生物学家杨钟健所命名,模式种是天山贫齿鳄(\"E. tienshanensis\")。杨钟健当年所发现的正模标本(编号IVPP-V-3236)是一个部分颅骨,与数节相连的颈椎。在2000年,洛杉矶郡立自然历史博物馆与中国地质博物馆组成的挖掘团队发现了第二个部分颅骨。杨钟健当时将贫齿鳄归类于原鳄亚目的贫齿鳄科(Edentosuchidae)。在2004年,迪亚戈·玻尔(Diego Pol)等科学家根据新发现的第二个颅骨,将贫齿鳄归类于原鳄亚目的原鳄科。贫齿鳄是种异齿型动物。贫齿鳄的前上颌骨牙齿呈圆椎状。上颌骨的前两颗牙齿有三个齿尖;接下来两颗具有更多的小型齿尖;而其余的上颌骨牙齿体积较大、呈球根状、有小型齿尖。贫齿鳄的下颌每边有九颗牙齿,第二颗牙齿相当长。贫齿鳄的颅骨只有数公分长,口鼻部短、颅骨的后段宽。贫齿鳄生存于炎热、潮湿的泛滥平原环境,该地当年为季节性气候。当地还发现数种乌龟、准噶尔翼龙科、兽脚类、蜥脚类、剑龙类、鹦鹉嘴龙科、鸟脚类恐龙的化石。","text2":"贫齿鳄的化石是属于哪一个年代的?","label":1} {"text1":"艺康D90是一款由日本相机制造厂尼康生产的数码单镜反光相机,于2008年8月27日发布,并预定当年9月开始销售,计划取代原本的进阶业余机种D80。尼康的D90机型推出后,在中阶(约1000美元)DSLR市场相当受到瞩目。主要由于它是第一款可以摄录影片的机型,大约同期间有佳能紧接著推出的高阶全画幅机型佳能 EOS 5D Mark II也支援类似功能。D90的录影模式最高可达1280x720解析度(HD 720p)、24fps,并支援相容的各种交换镜头特性,例如使用的镜头如果支援 -{zh-tw:VR防手振;zh-cn:VR防抖}- 功能,那影片拍摄就会支援。此录影功能在最高HD画质模式(1280x720解析度)下被设计成每段影片录影长度限制为5分钟,而SD画质模式(640x424解析度,320x216解析度)下则可录影20分钟。主要的销售版本包括内附-{zh-tw:VR防手震;zh-cn:VR防抖}-镜头的套装版(kit set,包括一支18-105mm F\/3.5-5.6G ED VR镜头)和不附镜头的单机版(body set)两种。一直以来,尼康D90都是中阶单镜反光相机中的热卖型号。日本的一份权威杂志做的一份调查显示,D90的两款套装于日本齐登十大畅销型号之一;而根据日本BCNranking网站于2010年上半年的统计,D90是最热卖的单镜反光相机。但毕竟D90早于2008年投入市场,加上1200万像数的影像感应器及不支援全高清录影功能,在今天的单镜反光相机的标准之中都显得比较逊色。因此网路上一直有传D90的后继机型的出现。随著拥有更高像数的尼康D3100于8月19日公布之后,2010年9月15日,尼康也终于正式公布D90的后继机型——尼康D7000,其功能及做工皆比D90大有进步。","text2":"尼康D90的后继机型是什么型号?","label":1} {"text1":"《摩斯拉对哥斯拉》(日文原名:)是1964年4月29日上映的日本电影,哥斯拉系列电影的第4部作品,日本观众人数达到720万人。第八号台风侵袭日本,把刚完工的仓田滨开发区给冲垮,每朝报社记者酒井和纯子前去采访,可是每朝报社的组长突然接到在静之浦出现不明巨蛋的消息,尽速招回二人转移阵地,正当三浦博士尚未解开谜底,快乐集团的熊山却以122万4560日圆的价额买下那巨蛋,而熊山和虎畑正准备要宣传展览巨蛋的活动时,听到守护巨蛋的小美人的警告,却不以为然,原来小美人是摩斯拉的守护者,她们对酒井、纯子和三浦博士说这是摩斯拉的,要求人类尽速归还,另一方面仓田滨突然出现哥吉拉,遂之向摩斯拉的巨蛋前去,酒井等人请求摩斯拉帮忙,一开始遭到拒绝,可最后摩斯拉答应前去营救。哥吉拉慢慢逼近摩斯拉巨蛋的展览区及虎畑的旅馆,因为要逃命的熊山企图夺走虎畑柜子里的现金,被虎畑枪毙了,不过虎畑最后还是被哥吉拉破坏建筑物时压死了,哥吉拉逼近巨蛋,突然摩斯拉及时赶到,双方展开大决战,混乱之中,摩斯拉中了热线攻击,不敌而亡。日本自卫队决定使用电击作战,结果两次皆都失败,另一方面小美人唤醒了巨蛋里的双胞胎摩斯拉幼虫,前去静之浦反击哥吉拉,哥吉拉被双胞胎摩斯拉双面夹击,被摩斯拉吐出的丝紧紧捆绕,跌落海里,终于哥吉拉被摩斯拉隔代子孙给打败了。","text2":"《摩斯拉对哥斯拉》是哥斯拉系列电影的第几部?","label":1} {"text1":"国民乐派,是广义古典音乐的一支,但没有明确的定义和共同的风格,经常被视为「浪漫主义音乐」或「现代音乐」的分支。大致就是指会在古典音乐的曲式中,使用了本国的民间音乐旋律和特别的乐器,并通常宣扬爱国主义或民族主义,主要在十九世纪中到二十世纪中活跃。一般认为十九世纪中前期,西方音乐的传统心脏的德语和义大利地区以外的欧洲和美洲国家,作曲家需要属于本地的音声。反之德语或义语的作曲家即使用民间音乐素材,仍然只是会被认为浪漫主义音乐,如小约翰·施特劳斯的圆舞曲原型虽然来自民间,却被当时的维也纳视为国际性的艺术。国民乐派起源众说纷纭,较流行有两说。一是十九世纪俄国作曲家格林卡的《爱国歌》,另一说是法国作曲家圣桑成立国家音乐社团组织,两人分别影响了本国的作曲家组织,交流了其对民间音乐的资料和心得,以及对于教育本国的音乐界作出重大的贡献。其他尚有肖邦和弗朗兹·李斯特之说,因为是他们首先采用故乡的民间音乐素材,但他们并未有组织本地作品家活动,也没有系统地使用和搜集民间音乐元素。在十九世纪后期其他欧洲国家也产生了自己国民乐派作曲家和作品,如英国的艾尔加的《威风凛凛进行曲》(Pomp and Circumstance Marches),捷克的斯美塔那的《我的祖国》。还有一些作曲家为自己尚末被认可为国家的故乡,创作鼓吹独立的作品,如芬兰的西贝柳斯的芬兰颂,挪威的格里格的培尔·金特组曲等。而美洲的作曲家也开始活跃起来,如美国的约翰·菲利普·苏萨的进行曲《星条旗永远飘扬》,而同期斯蒂芬·福斯特更有开创流行音乐的先河。到二十世纪初巴西的海托尔·维拉-罗伯斯,匈牙利的柯达伊,美国的查尔斯·艾夫斯等,新国民乐派作曲家开始向现代音乐的过渡。到二十世纪中后期起,因为流行音乐的兴起,也少有未被古典音乐使用的民歌,所以在欧美国民乐派渐趋于沈寂,但在古典乐传统地区外的亚洲,却有新型的国民乐派作品面世,如中国的冼星海的黄河大合唱,何占豪和陈钢的梁祝小提琴协奏曲等,代表了欧美以外的民族主义兴起。二十一世纪初,虽然因为再很少未独立的殖民地和国际主义盛行,政界不再需要常以音乐推广爱国思想,但国民乐派音乐并未因此而丧失了其地位,反而把这种音乐从政治服务中解放过来,受到本来创作国家外的欢迎。","text2":"在当时的亚洲,新型的国民乐派作品面世主要指哪些作品?","label":1} {"text1":"沙希德·哈希米内贾德国际机场()(波斯语:')俗称马什哈德国际机场(波斯语:'),是伊朗伊斯兰共和国第二大城市、礼萨呼罗珊省省会马什哈德的国际机场,距离马什哈德市区南郊约10公里。沙希德·哈希米内贾德国际机场是伊朗旅游航空、伊朗阿斯曼航空和塔班航空的枢纽机场,也是伊朗航空的重点机场。该机场有固定航班飞往伊朗国内各大城市,也有许多国际航班,主要飞往中亚和中东。马什哈德机场始建于公元1951年(伊朗历1330年)的巴列维王朝时期,最初只是提供小型飞机降落的军用机场。后经过多次扩建,改为军民两用机场,当时已经可以停降波音727、737、707等大型客机。伊朗伊斯兰革命胜利后,马什哈德国际机场改名“沙希德·哈希米内贾德国际机场”(“沙希德”是伊斯兰教中的殉教者),以纪念于1981年在当地遇害的伊斯兰革命领袖。","text2":"沙希德·哈希米内贾德国际机场是什么时候建立的?","label":1} {"text1":"小轮车(Bicycle Motocross,简称BMX)是一种车轮直径为20英寸的自行车,小轮车比赛分为在泥地赛道上的和以技巧为主的比赛。自由式小轮车包括五个不同的类型,分别是:街道,公园,U型道,泥地以及平地。不同的地形对自行车的技术要求也不相同。竞速小轮车与自由式越野单车有着显著的不同。自由式小轮车通常使用U型车闸,而竞速小轮车通常使用V型车闸,仅为竞速赛车在高速前进时需要保持稳定性。自由式越野单车的轴距更短以允许其更快的转弯。最近,小轮车运动的管理组织——国际自行车联盟开始赞助。在2008年北京奥运会上,小轮车竞速赛首次成为奥运会正式比赛项目 。1970年代,美国加利福尼亚州的孩子们骑着普通的自行车,在特意建造的越野跑道上行驶,并模仿他们所崇拜的越野摩托车手。越野单车运动便起源于此。拍摄于1971年的一部讲述摩托车竞速赛的纪录片《On Any Sunday》开场镜头便是一群骑着Schwinn公司的20吋Stringrays自行车的孩子们在越野飞驰。这部影片就是为了在全美鼓励这一运动而拍摄的。1970年代中期,厂家开始为这种运动设计生产专门的自行车。1977年,专门为日益壮大的小轮车运动而设立的成立。1981年4月,成立,并于次年举办了首次世界锦标赛。从1993年1月开始,小轮车运动正式进入国际自行车联盟。如今,小轮车已经成为美国一年一度的夏季极限运动大赛的主要项目之一。2008年的北京奥运会中,小轮车更是成为了正式比赛项目之一(亦为最先进入奥运的极限运动项目亦为较创新的项目,其他奥运项目由来已久)。","text2":"自由式小轮车有几种类型?","label":1} {"text1":"贾特拉帕蒂·希瓦吉国际机场(,,,)是印度共和国马哈拉施特拉邦首府孟买的国际机场,是该国乃至整个南亚地区最大、最繁忙的机场,以马拉地帝国开国君主贾特拉帕蒂·希瓦吉命名。第一航厦Santa Cruz营运国内线,第二航厦Sahar营运国际线。为印度航空和捷达航空的主要枢纽。机场亦有不经停直达班机前往香港及中国大陆的北京。圣塔克鲁兹皇家空军是在20世纪30年代在德里建成此机场,机场占地约1500英亩(610公顷)并设有三条跑道,面积比1942年至1947年第二次世界大战期间,几个皇家空军中队所在地朱胡机场更大。停机坪位于09\/27跑道南侧,该地区现时被称为“旧机场”,其中设施包括印度航空、印度航空工程和MIAL通用航空的飞机维修库。直至1946年,皇家空军开始将机场交给民航局开始营运民航航班时,皇家空军把两个旧有废弃机库转换成客运大楼,一个机库被用作处理国内航班,另一个用于处理国际航班,客运大楼两侧亦设有海关、移民局检查柜台和候机室。印度航空在位于两个机库附近当时拥有自己的客运大楼,并以其处理乘客。在开始营运的第一年,客运大楼每天约处理著六班民航客机。","text2":"机场停机坪的设施有哪些?","label":1} {"text1":"禾雀(学名:')又名爪哇禾雀、文鸟、黑芙蓉、白芙蓉、银芙蓉、浅黄褐芙蓉、爪哇雀,为雀形目梅花雀科禾雀属的鸟类,分布于印尼,一般生活于低地草原或具草甸、灌丛的空旷林地、也见于耕地、花园、城郊的村镇以及活动于稻田或甘蔗田、树林或灌木丛中。该物种的模式产地在亚洲及埃塞俄比亚。原分布于印尼的爪哇、峇里及巴韦安岛,现已引入到其他地方。可惜在原产地已濒临灭绝。体长13至14公分、体重24至30公克。原生地为印尼的爪哇与峇里岛地区,是知名的观赏鸟类。栖息于平原地带,主要食物为禾本科植物的种子。尽管禾雀在全世界被养殖作为宠鸟,但禾雀在原生地的野生族群因环境的人为限制而极需保护。于园林和农耕地结大群而栖,是具有高度社群性的鸟类。主要以禾本科和其他草类的种子为食。禾雀被饲养作宠物鸟已有几个世纪的历史,例如日本于江户时代已从中国引入禾雀作宠物。据说在400年前已有中国人饲养禾雀。因此禾雀经常被用作中国占卜术中灵雀占卜的「媒鸟」。相士首先把禾雀从小盒中放出来,然后禾雀会在占卜用的签筒抽一支签,相士此后便会根据签文向客人解释签文。这种占卜术在香港庙街极为盛行。由于栖息地开发,以及经常被设陷阱捕捉作宠物鸟类贸易,野生爪哇禾雀在原生地数目下降。《濒危野生动植物种国际贸易公约》(CITES)附录Ⅱ品种以及世界自然保育联盟(IUCN)中的易危物种。","text2":"禾雀的体长大约是多少?","label":1} {"text1":"深深爱过你是歌手薛之谦的第三张个人创作流行音乐专辑,于2008年11月26日正式发行。专辑有9首歌,新歌7首,另两首为:深深爱过你(现代版)和改编自上一张专辑你过得好吗的《梦开始的原点》。其中有两首风格迥异的韩式舞曲星河之役和Let You Go,因销量喜人唱片公司,推出了附加了“谦年传说”演唱会视频的改版专辑,预计于2009年4月9日发行。深深爱过你两个版本被拍摄为不同版本的两支MV,传说也被拍摄为MV。其余的两支MV:星河之役和我们的世界,均为演唱会LIVE版MV。1.传说 04:33(第一波燃情主打)2.深深爱过你(前世)04:38(第二波个人PK单曲)3.Let You Go 02:42(第三波热舞主打)4.给我的爱人 04:455.我们的世界 04:146.流星的眼泪 04:427.星河之役 03:438.深深爱过你(今生)04:17(第二波个人PK单曲)9.梦开始的原点 04:52","text2":"深深爱过你是谁的音乐专辑?","label":1} {"text1":"尚真(;)是琉球国第二尚氏王朝第三代国王、第九代琉球国王,1477年(成化十三年)至1526年(嘉靖五年)在位。他是第二尚氏王朝第一代国王尚圆王之子。童名真加户樽金(),神号于义也嘉茂慧()。尚真未即位时被封为久米中城王子,1477年接受叔父尚宣威的禅位,并派遣长史梁应、使者吴是佳、通事梁德等至明朝,以尚圆王讣告,请求袭封为琉球国王。明宪宗遂在1479年(成化十五年)派兵科给事中董旻、行人司右司副张祥前往琉球,册封尚真为王。尚真王在位期间,不断进行对外扩张。1500年平定了八重山群岛的远弥计赤蜂之乱。1506年相继征服了久米岛按司势力和具志川按司势力,在1522年镇压「鬼虎之乱」,征服与那国岛。此后琉球的统治势力扩张至宫古、八重山,基本确定了琉球南部的版图。尚真王统治的50年被认为是琉球历史上最强盛的时期,确立了琉球的官员品秩、朝仪制度、神官制度、赋税制度、行政划分,扩建了首里城,废除了殉葬习俗,并于1526年召各按司赴首里居住,禁止私人拥有兵器,加强了中央集权,此后琉球进入稳定发展的时期。琉球群岛著名的建筑圆觉寺(1492年)、玉陵(1501年)、圆鉴池(1502年)、辩财天堂(1502年),都建于尚真王时期。用于国王处理政务、接待册封使的首里城北殿(1508年),也是在这一时期建造的。","text2":"尚真在位时间是什么时候?","label":1} {"text1":"库拉湾海战(,1943年7月6日)是一场发生于科隆班加拉水域,在日本和美国舰队之间爆发的海战。1943年7月,所罗门群岛战事愈发激烈。盟军在新乔治亚群岛中的伦多瓦岛登陆后,开始准备下一轮对新乔治亚岛的进攻行动。为支援陆军的登陆行动,3艘轻巡洋舰和4艘驱逐舰组成第36.1特混编队,在沃尔登·安斯沃思少将的指挥下于7月4日对科隆班加拉岛和新乔治亚岛的日军设施进行了炮击。随后编队计划前往珊瑚海补给油料和弹药。然而在7月5日,美军接到情报,新的一支东京快车运输队正在经铁底湾南下。安斯沃思奉命北上,在新乔治亚岛附近拦截日本舰队。日本舰队由3艘驱逐舰和7艘运输驱逐舰组成,指挥官为秋山辉男少将,计划运载2,600名官兵增援科隆班加拉。7月6日凌晨1时06分,美军编队与日本舰队在科隆班加拉海域遭遇。1时57分,美军编队开火,在21分钟内共发射了612发炮弹,迅速击沉了日军旗舰新月号。秋山少将阵亡。2艘日本驱逐舰随后发射鱼雷。其中3枚鱼雷击中海伦娜号巡洋舰,造成海伦娜号大量进水并开始下沉。双方随后开始撤出战场,留下少数驱逐舰以营救落水舰员。5时左右,日本的天雾号和美国的尼克拉号驱逐舰(DD-449)再度交火。天雾号受创后退出战斗。7艘日本运输驱逐舰接到战斗报告后即停止卸载,立即返航。期间初雪号受损,长月号搁浅,运载的2,600名陆军官兵中仅有850人上岛。搁浅的长月号于次日被美国飞机击沉。美国驱逐舰拉德弗德号(DD-446)和尼克拉号共救起了超过750名海伦娜号的落水官兵,并因此获得了荣誉。","text2":"1时57分,美军编队开火迅速击沉了日方的哪一艘船舰?","label":1} {"text1":"《美国大白痴》()是美国流行庞克乐队Green Day的第七张专辑,于2004年9月21日发行。Green Day于2003年开始制作《美国大白痴》。由于当时正值伊拉克战争所以专辑主题是反战。专辑荣登美国告示牌专辑榜第一,封面以一个手持红色的心型手榴弹图案,将反战主义呈现出来,专辑获得第47届葛莱美最佳摇滚专辑奖。专辑在2006年MTV音乐录影带大奖MTV Video Music Awards,获得八项提名并带走七个奖项:年度最佳音乐录影带、最佳摇滚音乐录影带、最佳执导、最佳剪辑、最佳电影艺术、观众票选奖、最受喜爱专辑。专辑也获得2006年葛莱美奖的年度最佳专辑。除特别注明,所有歌曲均由Billie Joe Armstrong创作。","text2":"《美国大白痴》是哪个乐队的专辑?","label":1} {"text1":"元详(),字季豫,河南郡洛阳县(今河南省洛阳市东)人,魏献文帝拓跋弘幼子(第七子),椒房高氏所生。太和九年(485年)封北海王,加侍中。他的大哥孝文帝元宏鼓励鲜卑与汉族通婚,元详聘娶吏部郎中荥阳郑懿的女儿。元详任尚书仆射。太和二十三年(499年),元详作为孝文帝的顾命大臣之一,以司空辅佐新君宣武帝元恪。彭城王元勰等人以孝文帝的遗诏赐孝文幽皇后死,元详派长秋卿白整进去给冯皇后送毒药。元详想独揽大权,用阴谋将六哥元勰排挤,自己被皇帝任命为太傅,领司徒,侍中、录尚书事,贪财无厌,公私营贩,又抢夺良民宅第,威望很劣。元详妻子刘氏懦弱不嫉妒,元详则和堂叔安定王元燮的夫人高氏通奸,因为此关系,又与高氏的姐夫茹皓、刘胄、常季贤、陈扫静等往来稠密,专恣不法。高肇要掌握权力,首先要除掉诸王。宣武帝由于二叔咸阳王元禧谋反,对叔叔们都不信任,高肇诬告元详谋反,宣武帝深信不疑。正始元年(504年)五月,查办元详罪状,定为谋逆,废为庶人,囚禁在太府寺。元详的母亲高太妃这时才知道元详和元燮的王妃私通之事,大骂:“你的妻妾成群,个个年轻美貌,为何偏要私通那个高丽贱女人,以致陷罪到这个地步呢?”元详的妻子是宋王刘昶之女,宋文帝刘义隆的孙女,高太妃又骂刘氏不知嫉妒,没有管好丈夫。命人打了元详一百多板,刘氏几十板。正始元年六月十三日(504年7月10日),元详被囚禁不久就被元恪遣人秘密处决。详情一说是元详的几个家奴想把元详劫出来,秘密书写了名单,派侍婢交给元详。元详拿在手上刚要看,被门防主司(看守人员)发现了,突然跑进来把名单夺过来,上奏给宣武帝。元详大哭了几声,气绝而死。一说,元详被囚禁之后,母亲高氏、妻子刘氏时时前来相见,这天其母亲、妻子都不在,元详遂死在婢女的手里,守卫在外听见他大哭了几声就此毙命。元详以贪淫出了名,但确实没有谋反之意,被杀之后,远近叹息惊异。永平元年十月丁巳(508年11月15日),魏宣武帝诏令恢复元详原本的北海王爵位,谥号平王,以王的礼节下葬。","text2":"元详为什么用阴谋将六哥元勰排挤?","label":1} {"text1":"黄兆伦 () 别名幽,生于香港,Cosplay爱好者,是一名女性Cosplayer。在香港的Cosplay界十分出名,热爱玩Cosplay已有十多年,2005年创办Cosplay队伍BaLance。有参加香港动漫电玩节(前名:漫画节\/动漫节)的动漫Cosplay大赛(前名:Cosplay扮野大赛),至在2004年开始直到2006年的动漫Cosplay大赛,连续3届赢得Cosplay比赛冠军。并在香港动漫电玩节2008的动漫Cosplay大赛2008,扮演Monster Hunter的银火龙猎人,夺得冠军。她表示,参赛者愈来愈专业,她今次刻意往内地学习舞台技巧,为角色设计特别的舞台效果,结果自04至06年间得到冠军后,在08年的动漫Cosplay大赛再次夺得冠军。在香港动漫电玩节2009的动漫Cosplay大赛2009以扮演Final Fantasy VIIDirge of Cerberus的Genesis再度参战,但最后没有得奖!由她的队友区可儿夺得两项奖项,分别是分组冠军和全场总冠军。2011年至2013年香港动漫电玩节的动漫Cosplay大赛中均获得全场总冠军。","text2":"为什么黄兆伦前往大陆学习舞台技巧?","label":1} {"text1":"杨廷艺(,,部份文献写成「杨延艺」()),越南史上亦称为「杨正公」,交趾爱州人,原为静海节度使曲氏将领,当其主曲承美被中国南汉灭亡后,杨廷艺奋起抵抗,夺取了交趾的统治,因而被视为是越南自立建国的重要人物之一。杨廷艺是中国五代十国时期爱州人士,曾任交趾静海节度使曲颢的将领。当时,位于中国岭南的南汉与曲氏势成水火,曲颢之子曲承美继位后,于大有三年,930年(《大越史记全书·外纪全书·南北分争纪》作923年),南汉君主刘䶮出兵,擒获曲承美,占领交趾地区,并派骁将梁克正、交州刺史李进驻守,而杨廷艺则获南汉授以爵命,名义上成为了南汉的臣僚。据《安南志略》,刘䶮虽已得胜,但洞悉到「交趾民好乱,但可覊縻而已」,认为当地民众不会服从其统治。而杨廷艺亦为驱逐南汉而作好准备,《大越史记全书》提到,到了大有四年(931年)农历十二月,南汉官员李进向刘䶮告发「杨廷艺养假子三千人,图恢复」,廷艺随即先发制人,攻破李进所在的大罗城,李进逃归。适值南汉派遣程宝(《新五代史·南汉世家·刘隐》作「程宝」,《大越史记全书·外纪全书·南北分争纪》作「陈宝」)率兵赶来,包围杨廷艺,杨廷艺出战,击杀程宝。杨廷艺取得军事胜利后,便自称节度使,领州事。关于杨廷艺得胜后与南汉的关系,中国学者郭振铎、张笑梅作以下分析:「杨廷艺虽战败南汉军,领有交州。但他深恐南汉再次派大军来攻,为了本身的政权不受威胁,只好臣服南汉,服从南汉的诏谕,接受为交州节度使,刘䶮和杨廷艺之战宣告结束。」而在交趾方面,杨廷艺与本地的土豪结好,例如把女儿嫁给「世为贵族」的牙将吴权。南汉大有十年(937年)农历三月,杨廷艺被牙将矫公羡所杀。矫公羡行凶的原因,则是「恶廷艺所为,起所部攻杀廷艺,而代为节度。」矫公羡遂向南汉臣服,但其后亦被杨廷艺另一员牙将吴权所杀,并继续争取交趾政权的独立。","text2":"杨廷艺越南史上称他为什么?","label":1} {"text1":"《无双大蛇Z》()是光荣旗下ω-force开发的动作游戏,于2009年3月12日登陆索尼PlayStation 3,并于2009年11月27日推出Windows版。《无双大蛇Z》是真·三国无双系列和战国无双系列的融合作品——无双大蛇系列的第三作,将《无双大蛇》和《无双大蛇:魔王再临》的所有内容合并移植到最新的游戏及平台上,并添加了新的角色和关卡。得益于次世代游戏机的强大机能,该游戏在当时为无双系列中同屏幕人数最多的一款。该作的PlayStation 3日文版于2009年3月12日在日本和台湾地区发售,而PlayStation 3中文版则于2009年7月30日由台湾光荣推出,Windows中文版则于2009年12月24日开始发售。由于《无双大蛇Z》是先前两部作品的结合之作,因此游戏操作方式也一沿其旧:玩家选择三名角色组成小队,并在某一关卡中使用。不过,游戏不少某些细节得以加强。但是删减敌兵模组的状况依旧。PC版游戏画面较前一代有较大改观,且支持宽屏幕模式画面,同屏人数也明显增加。本作中,第二玩家可以选择有别于第一玩家的角色队伍,而不像前两作一样,两名玩家必须使用相同的角色。剧情模式也有部分改动,《无双大蛇Z》新增了12个关卡,使得剧情模式关卡的总数达到40个。而且剧情模式的关卡可以使用其他武将。游戏新增了以下的设置:在无双大蛇系列前两作92名角色的基础上,《无双大蛇Z》再度追加了两名新角色。此外,百百目鬼和牛鬼原本在《魔王再临》中只能在对战模式和生存模式中使用,而在《无双大蛇Z》中则可使用于故事模式和剧情模式,并且改变了他们的动作模组 。以下新增的角色:可在故事模式和剧情模式中使用的以下角色:加上这四位新增角色,在《无双大蛇Z》中共有96名角色登场,成为当时登场人数最多的无双系列游戏。尽管《无双大蛇Z》沿袭了光荣一贯的移植作风,整体并无太多新意,但在日本销量依然看好,上市首日便获得了售出7.9万套的好成绩。","text2":"《无双大蛇Z》在什么时候推出了Windows版?","label":1} {"text1":"天主教康布雷总教区(拉丁文:Archdiocesis Cameracensis)是罗马天主教在法国北部设立的一个总教区,范围相当于北部省的Avesnes-sur-Helpe、康布雷、杜埃、瓦伦谢讷专区。康布雷教区成立于第6世纪,管辖范围包括今比利时。1559年西班牙国王分出梅赫伦总教区和另外11个教区以抵抗宗教改革,康布雷教省的范围大为缩减,附属教区还有Saint Omer教区、图尔奈教区和那慕尔教区。1802年,康布雷失去所有附属教区,自身也附属于巴黎总教区。1817年,教宗和法国国王希望成立里尔教区,遭到康布雷主教Louis de Belmas的激烈反对。1841年,康布雷恢复总教区,附属教区为阿拉斯教区。现任总主教为2000年就任的 François Charles Garnier。2002年,康布雷失去教省地位,转而附属于天主教里尔总教区。","text2":"现任总主教是谁?","label":1} {"text1":"职业玩家通常指那些无其他工作,以进行电子游戏谋生或赚取收入的人。2000年以后,电子游戏随着PC和互联网的普及得到迅速发展。电子游戏产业带来了一定的供需要求,于是诞生了专门以进行电子游戏为职业的职业玩家。围绕电子游戏(尤其是网络游戏和竞技类电子游戏)发展出了众多的商业行为。一个职业玩家可能是通过销售网络游戏中的虚拟货币赚取金钱,也可能是通过进行电子竞技比赛得到奖金为生。随着近年网络游戏和宽带的发展,网络游戏玩家人数上升,网络游戏消费额和市场逐年扩大,也形成了除了对游戏运营官方点卡等的需求外的额外需求。于是有玩家通过网上或网下进行交易,用真实货币购买游戏中的虚拟道具从而使自己的虚拟角色更加强大。于是有个人或组织专门以进行虚拟道具交易或者账号交易等方式赚取利益,这部分人通常被称为网络游戏职业玩家。近年来随着像星际争霸此类电子竞技类游戏地发展,电子竞技已经成为新兴的竞技娱乐项目。由此产生了专门以进行电子竞技活动为职业的人。众多的电子竞技比赛和电子竞技战队可以为选手带来收入。中国:韩国:TOM网:由17岁民工窥视打钱工作室真实内幕17173:揭秘《魔兽》美服打钱工作室月赚十几万腾讯:电子竞技职业选手工资堪比白领","text2":"职业玩家是指什么样的人?","label":1} {"text1":"欧阳德勋(,),香港歌手、司仪、前唱片骑师,于1984年第三届新秀歌唱大赛季军出身。欧阳德勋在1972年就读新法书院(小学及中学部)太子道分校。他在1984年参加第三届新秀歌唱大赛,并获得季军。同年加入歌坛成为歌手。他在1984年至1989年签约成为华星娱乐有限公司旗下合约歌手。同时于1984年起至1995年签约成为无线电视艺员,并拍摄多个电视剧,如《城市故事》。期间,欧阳德勋于1989年至1993年在商业电台第一台担任全职唱片骑师。后来到了1997年至2010年,欧阳德勋转到香港电台第二台担任音乐节目监制及全职唱片骑师。2012年6月25日,他迎娶同为歌手的蔡立儿,结束长达16年的爱情长跑。圈中的好友如蒋丽萍、宝珮如、李国祥、周慧敏、蔡一智、李司棋、车婉婉、陈奂仁、许志安、苏永康、张衞健、梁汉文、吴国敬、张茜、黎芷珊和梁雨恩等都有出席其婚礼。欧阳德勋表示曾经与当时仍然是女朋友的蔡立儿分开达一年多的时间。其后,因为他重整个人生活且有信仰支持,所以他决定与蔡立儿破镜重圆。","text2":"欧阳德勋最初是通过什么方式出道的?","label":1} {"text1":"阿道夫·恩格勒(全名海因里希·古斯塔夫·阿道夫·恩格勒 )是一名德国植物学家,他在植物分类学和植物地理学上有重要的贡献。恩格勒出生于普鲁士王国的萨根(今属波兰),1866年在弗罗茨瓦夫大学获得博士学位,并留校教书,1871年,他成为慕尼黑植物协会的标本采集管理员,1878年,担任基尔大学教授,并当选为德国自然历史科学院院士,1884年他回到弗罗茨瓦夫,担任弗罗茨瓦夫大学教授和植物园长,1889年担任柏林大学教授和柏林植物园长,直到1921年退休,1930年在柏林逝世。恩格勒对植物分类学有很大的贡献,他制定的包括从藻类到有花植物的总的恩格勒分类法目前仍然广泛被植物标本分类中应用。他编写过大量关于植物分类法的书籍,并雇佣著名画家制作了描述有6 000种植物的33 000幅版画。1913年,他获得伦敦林奈学会颁发的林奈金质奖章,国际植物分类协会于1986年为纪念他设立了恩格勒奖章(参见奖章获得者名单)。柏林植物园出版的杂志《恩格勒拉》(Englera ISSN 0170-4818)也是以他命名的,有许多植物的属名也是为纪念他命名的,如沙穗属(\"Englerastrum\")、恩格勒豆属(\"Englerodendron\")等。恩格勒创办了《植物分类、发生学和地理学年鉴》(Botanische Jahrbücher für Systematik, Pflanzengeschichte und Pflanzengeographie ISSN 0006-8152),从1881年直到如今一直在莱比锡出版。其他著作有《植物自然分类》(\"Die Natürlichen Pflanzenfamilien\")、《植物界》(\"Das Pflanzenreich\")、 《地球植被》(\"Vegetation der Erde\")、《东非及其附近区域的植物界》(\"Die pflanzenwelt Ost-Afrikas und der nachbargebiete\")等()。","text2":"阿道夫·恩格勒在什么方面有重要的贡献?","label":1} {"text1":"甸恩·韦赫特(,)出生于牛津郡的阿宾顿(Abingdon),是一名英格兰足球运动员,现时效力英冠米杜士堡。他主要担任中场,但同时能以右闸上阵。韦赫特在家乡球队阿宾顿镇(Abingdon Town)的青年队出道,之后加盟当地联赛球队牛津联的青训系统。他在牛津联渡过了五个球季,成为了球队中一个有很好传球能力的中场捍将。在约满后,他拒绝续约并开始寻觅新球队。在2004年7月,韦赫特免费加盟当时英冠球队新特兰,经过仲裁,新特兰需支付牛津联15万英镑赔偿金,牛津联亦会在将来韦赫特转会收到25%的转会费分红。韦赫特很快在新特兰站稳阵脚,在2004\/05年球季为新特兰赢得英冠冠军。季后亦被队友选为「季度足球先生」。韦赫特在2005\/06年球季续约三年,并在该球季于作客对热刺的比赛射入一球三十码的罚球,但球队最终仍以2-3落败。新特兰在2006\/07年球季降级至英冠,韦赫特在中坚-{zh-hans:斯蒂文·考德维尔; zh-hk:卡达维;}-(Steven Caldwell)被卖至-{zh-hans:伯恩利; zh-hk:般尼;}-后再一次成为新特兰的队长。在2007\/08年英超球季,他在对韦根的比赛中受伤,令他在多场比赛中不能上阵,三个月后重返球场,但以右闸上阵时间居多,直至季尾才再次在中场指挥。2009年7月24日韦赫特离开光明球场加盟史笃城,签约四年,初步转会费为300万英镑,其后可增加刑500万英镑,旧球会牛津联在这单交易中可收取超过50万英镑的分红。他在8月15日对-{zh-hans:伯恩利;zh-hk:般尼}-的比赛中首次上阵。2013年6月6日,史笃城宣布旗下七位球员可以自由转会,韦赫特为其中之一。2013年7月2日,韦赫特以自由身加盟英冠米杜士堡并签约两年。","text2":"2013年7月2日,韦赫特签约了哪家足球俱乐部?","label":1} {"text1":"克里雍大饭店(Hôtel de Crillon)是巴黎的一座古老的五星级豪华酒店,位于协和广场10号(北侧),香榭丽舍大街东端。克里雍大饭店开始于蜂蜜色的大理石大堂,近旁是充满水晶吊灯和装饰有7种不同大理石的Les Ambassadeurs 餐厅。酒店的大厅充满了17和18世纪挂毯,吊灯,镀金和织锦家具,精致的雕塑,和路易十六的柜橱和椅子。其顶层的伦纳德·伯恩斯坦套房的阳台可以看到巴黎壮观的景色,也有一架Maestro的钢琴。克里雍大饭店有103间客房和44间套房,占据着被皇家路分开的两座相同的石头建筑中的一座,由路易十五委任建筑师路易斯·弗朗索瓦Trouard 兴建于1758年。起初两座建筑均为作为政府机构,东侧的建筑今天仍是法国海军部,然而,西侧的建筑建成了豪华酒店,很快,王后玛丽·安托瓦内特和她的精英朋友就经常光顾此处。她来到这家酒店学习钢琴课。","text2":"哪一侧的建筑今天仍是法国海军部?","label":1} {"text1":"贺万里( ),中国著名画家,美术理论家,中国装置艺术理论研究奠基人。生于安徽合肥,籍贯河北。以中国山水画泼彩山水泼墨闻名。南京艺术学院美术学博士毕业,后任职于扬州大学艺术学院 教授、硕士生导师。中国美术家协会会员。江苏省美术家协会会员。他兼任清代扬州画派研究会会长,以及石河子大学新疆大山水画研究所副所长,还有石河子大学西部绘画研究所研究员。主要研究领域涉及中国古代绘画理论、中西美术史、中国现代艺术、动漫理论等,尤以装置艺术理论见长。20世纪90年代以来,在《文艺研究》、《美术观察》、《美苑》、《艺术探索》、《中国画研究》、《宁夏大学学报》等公开出版的有影响的中文核心期刊或专业性刊物上发表学术论文50余篇。《装置艺术研究》 1999 中国文联出版社《光影与色彩》(译著) 2001 江苏美术出版社《永远的前卫:中国现代艺术的反思与批判》 2003 郑州大学出版社《谁在附庸风雅》2005 上海书画出版社《鹤鸣九天:中国古代绘画的功能论》(其博士毕业论文)《美术刀客:贺万里艺术论评集》2005 天津人民美术出版社《中国山水画通鉴:维扬异趣》 2006 上海书画出版社《中国当代装置艺术史(1979-2005)》2008 上海书画出版社其著作《装置艺术研究》详细,系统地首次向中国读者以及中国艺术家介绍阐明了 装置艺术 的主要内涵和特点,为中国的装置艺术研究奠定了理论基础。在2001年获得海峡两岸书画名家邀请展金奖。在2006年纪念孙中山诞辰140周年中国书画名家作品大展金奖。多幅作品被个人、企事业和美术馆收藏,在《美术报》、《中国书画报》、《美术大观》、《书与画》、《艺术当代》、《国画家》等刊物发表,并被专文推介。1.海峡两岸书画名家交流展,2000年,台北、南宁。一等奖2.纪念孙中山诞辰140周年中国书画名家邀请展,2001,台北、南宁。金奖3.当代著名书画家精品年展,2002,台湾阿里山4.贺万里泼彩山水画暨学术文献展,2006,扬州5.美术刀客----贺万里泼彩艺术精品展,2007,广州6.千岛湖名家名湖书画精品展,2007,杭州淳安7.首届中国美术教师艺术作品年度展,2007,沈阳。7.走近番禺·2008全国书画名家新春雅集,2008,广州8.周积寅教授从艺五十周年暨七十华诞师生书画展,2008,南京","text2":"南京艺术学院博士毕业后从事什么工作?","label":1} {"text1":"吉布提航空是一间吉布提的国家航空公司。服务由吉布提至非洲和中东,大部分飞机是湿租来的。航空公司基地位于吉布提国际机场。航空公司成立于1996年2月1日并于2月5日开始服务,由从前Puntavia主任穆萨里亚勒成立。成立初期以低票价服务使用Let L-410去航行德雷达瓦和哈尔格萨。现时的主席是Capt Moussa Waberi(97.3%)与其他投资者(2.7%),航空公司有46名员工(于2007年3月)。该航空公司所持有的《商业运输许可证》(\"AOC\")于2009年7月30日被撤销,正式结业。以下是吉布提航空的航线(2006年12月):吉布提航空有以下机队(2008年6月):","text2":"航空公司有多少名员工?","label":1} {"text1":"1918年教育法(\"Education Act 1918\"),又称为费舍教育法(\"Fisher Act\"),是英国国会通过的一项法案。草拟者为文教大臣赫伯特·费舍。1918年教育法将从学校毕业的年龄提高到14岁,并计划扩展高等教育。1918年教育法的另一特色是提供辅助设施(幼儿学校、体格检查和特殊需要儿童中心等)到1920年代,年幼儿童的教育日益成为政治家与教育家关注的问题。由于公众争论的升级,当时的政府委托哈多爵士负责进行教育调查,哈多委员会在1926年、1931年和1933年,发表了3个非常重要的报告。这些报告带来初等教育的重大改变。例如,导致了5-7岁儿童的幼儿学校和7-11岁的小学被分开实施。报告还建议班级规模不超过30人。这些建议标志进步主义教育思想胜过了传统教育思想,更多为政策制定者和教师们所喜爱。","text2":"1918年教育法提出的建议有什么意义?","label":1} {"text1":"克斯蒂·李·库克(英文:Kristy Lee Cook,),是一个美国乡村音乐歌手,曾以克斯蒂·李(Kristy Lee)发行过一张专辑《Devoted》。另外,她是《美国偶像》第七季的参赛者,获得第七名。克斯蒂在2008年6月与签约,个人第二张专辑《Why Wait》已于2008年9月16日发行。克斯蒂·李·库克在华盛顿州的西雅图出世,父母是Larry和Carlene Cook。她现时居住在俄亥俄州的塞勒姆。克斯蒂喜欢购买马匹,训练后并卖出。一点值得留意的是克斯蒂的姊夫是著名室内美式足球四分卫的John Dutton。在2001年,17岁的克斯蒂第一次与BMI签约。布兰妮·斯皮尔斯(Britney Spears)愿意为克斯蒂第一个音乐录影带作客串演出。 可是RCA其后把她和一些年轻的歌手冷冻,随后更与他们解约。之后克斯蒂搬往德克萨斯州的达拉斯,尝试继续进行她的演艺事业以及争取多点经验。克斯蒂开始担任模特儿并在一些广告里作演出。在2008年3月15日,与克斯蒂交过一年半、拥有一间园艺公司的男友Andy Dobner向她求婚。为了保持在美国偶像的注意力里,她决定保守这个秘密。当克斯蒂在2008年4月16日被淘汰时,她的男友再次向她求婚。根据克斯蒂的话,两人的婚礼可能会在2009年6月进行。 ,同时美国偶像第七季的参赛者,将会是她其中一个伴娘。克斯蒂参加了美国偶像在费城举行的海选,她唱了奇异恩典。三位评审都喜欢她的声音并一致通过海选,进入荷里活周。在荷里活周中,她第一次的演出并未能给评审极深的印象。克斯蒂第二次演出和最后一次都是唱了奇异恩典;结果她通过考验,成立半决赛中二十四位参赛者之一。在决赛周中,克斯蒂经常成为倒数三人之一;终于在4月16日被淘汰,克斯蒂是第一位在美国偶像 (第七季)曾是倒数三人的被淘汰者。","text2":"克斯蒂·李·库克的第一张专辑名是什么?","label":1} {"text1":"在数论中,雅可比符号是勒让德符号的一种推广,首先由普鲁士数学家卡尔·雅可比在1837年引进。雅可比符号在数论中的各个分支中都有应用,尤其是在计算数论的素性检验、大数分解以及密码学中有重要作用。勒让德符号formula_1是对于所有的正整数 formula_2 和所有的素数 formula_3 定义的。当formula_4 时,称formula_2 是模formula_3的二次剩余;当formula_7 时,称formula_2 是模formula_3的二次非剩余。运用勒让德符号计算时要将 formula_2 分解成标准形式,计算上十分麻烦,因此产生了雅可比符号:设 formula_11 是一个正奇数,其质因数分解式为 formula_12,并且正整数 formula_2 满足 formula_14 那么定义formula_15。","text2":"勒让德符号formula_1是对于那些符号而定义的?","label":1} {"text1":"萧贺硕(,),台湾创作歌手。毕业于台北医学大学医学检验暨生物技术学系(简称医技系),并考取执照并曾任医检师半年。2008年获得第19届金曲奖最佳新人奖。近期组成「萧贺硕与冷笑话乐团」(Shuo & Cool Humor),2014年以《Musicians》歌曲荣获第25届金曲奖最佳作曲人奖。2017年7月8日,萧贺硕与歌手巴赖应中华民国总统府邀请于总统府开放参观日上台演出,演出至第三首歌时拿出原住民族转型正义小教室标语毛巾「没有人是局外人」,被总统府人员要求撤下毛巾,甚至表演结束后想要参观总统府还被拒绝。萧贺硕说,他们表演结束要离开总统府,还被一群人盯著直到人行道,「急忙到连表演费都忘了付」。总统府回应,当时总统府工作人员并未阻止歌手挂出布条标语、并未要求歌手取下布条标语、并未中断演出,而是告知领团经纪人相关规定,提醒是否不要再有与表演无关的活动;随后经纪人在歌曲结束后上台,与歌手共同取下布条标语。","text2":"萧贺硕哪首歌荣获第25届金曲奖最佳作曲人奖?","label":1} {"text1":"克斯蒂·李·库克(英文:Kristy Lee Cook,),是一个美国乡村音乐歌手,曾以克斯蒂·李(Kristy Lee)发行过一张专辑《Devoted》。另外,她是《美国偶像》第七季的参赛者,获得第七名。克斯蒂在2008年6月与签约,个人第二张专辑《Why Wait》已于2008年9月16日发行。克斯蒂·李·库克在华盛顿州的西雅图出世,父母是Larry和Carlene Cook。她现时居住在俄亥俄州的塞勒姆。克斯蒂喜欢购买马匹,训练后并卖出。一点值得留意的是克斯蒂的姊夫是著名室内美式足球四分卫的John Dutton。在2001年,17岁的克斯蒂第一次与BMI签约。布兰妮·斯皮尔斯(Britney Spears)愿意为克斯蒂第一个音乐录影带作客串演出。 可是RCA其后把她和一些年轻的歌手冷冻,随后更与他们解约。之后克斯蒂搬往德克萨斯州的达拉斯,尝试继续进行她的演艺事业以及争取多点经验。克斯蒂开始担任模特儿并在一些广告里作演出。在2008年3月15日,与克斯蒂交过一年半、拥有一间园艺公司的男友Andy Dobner向她求婚。为了保持在美国偶像的注意力里,她决定保守这个秘密。当克斯蒂在2008年4月16日被淘汰时,她的男友再次向她求婚。根据克斯蒂的话,两人的婚礼可能会在2009年6月进行。 ,同时美国偶像第七季的参赛者,将会是她其中一个伴娘。克斯蒂参加了美国偶像在费城举行的海选,她唱了奇异恩典。三位评审都喜欢她的声音并一致通过海选,进入荷里活周。在荷里活周中,她第一次的演出并未能给评审极深的印象。克斯蒂第二次演出和最后一次都是唱了奇异恩典;结果她通过考验,成立半决赛中二十四位参赛者之一。在决赛周中,克斯蒂经常成为倒数三人之一;终于在4月16日被淘汰,克斯蒂是第一位在美国偶像 (第七季)曾是倒数三人的被淘汰者。","text2":"克斯蒂·李·库克是什么人?","label":1} {"text1":"是香港的监狱主题电影作品之一,也是前作《黑狱断肠歌之砌生猪肉》的后续作品。电影开头数分钟由1987年「香港十大杰出青年」陈慎芝和李兆基,讲解香港监狱制度的沿革和一些小故事。与前作一样,时间同设在以1960年代的香港,未有廉政公署,纪律部队贪污猖獗的时代。专帮黑社会小混混打官司的律师陈兆康,因袭警罪被判入狱九个月,其间结识了新入职的狱卒王志成,两人均对监狱的污烟瘴气、囚犯打架及狱卒庄Sir(下称食人鲳)、泰臣等滥用私刑事件看不过眼。阿成结识了阿康的姐姐和妹妹,并开始与康妹拍拖。另一方面,副狱长食人鲳垂涎康姐妹的美色,以康姐的床照要胁康姐听从他,又在康妹的汤中落春药,使食人鲳能迷奸她。康妹在被食人鲳强奸后上吊自杀身亡,阿成和阿康得悉消息后,义愤难平,而其他囚犯也因食人鲳以私刑把囚犯红孩儿打死的事感到气愤,决定发起一场骚乱,誓要把食人鲳杀死。在骚乱期间,成与康及其他囚犯把食人鲳迫进绞刑室,食人鲳最后被吊死在绞刑台上。事后阿成和阿康二人均被判无期徒刑。镇压监狱骚乱时,出动的水警轮「水警64」挂起香港特别行政区区旗,而非当时之香港旗。","text2":"水警64挂的是什么旗帜?","label":1} {"text1":"《少年犯之七人》是作家安部让二原作、漫画家柿崎正澄作画的日本青年漫画。自2001年起在小学馆发行的《周刊YOUNG SUNDAY》2001年51号上开始连载,2008年7月31日因连载杂志休刊,自第四部起移到《Big Comic Spirits》2009年29号(6月15日)上连载,至2010年5、6号(1月4日)连载完毕。单行本共发行22册,日文版皆已发行完毕,累计销量超过330万部。2006年获第51回小学馆漫画赏大赏。台湾繁体中文版由尖端出版代理,目前已发行至第20册(2009年10月14日出版)。香港繁体中文版由玉皇朝发行。原作者安部让二曾称之为「爱与勇气的故事」,这部作品亦是安部自身的经验所写成的。2010年4月,日本电视台将播映电视动画。昭和30年(1955年),6个16岁少年因犯罪而进入「湘南特别少教所」的二舍六房。他们对同屋里较年长的樱哥的教导铭记心中,出狱后,面对世间的痛苦、不合理,坚强地开始各自的人生。","text2":"《少年犯之七人》的完结日期是?","label":1} {"text1":"镰刀龙科(Therizinosauridae)意为「镰刀蜥蜴」是兽脚亚目镰刀龙超科的一科。镰刀龙科是群草食性或肉食性的动物,生存于白垩纪中到晚期,化石已在中国、蒙古、美国等地发现。镰刀龙科具有大型前肢,长指爪,身体宽广,脚部具有四个趾爪。牠们的头部小型,颈部长,嘴部前端缺乏牙齿,嘴部后段的牙齿平坦。前背部的脊椎具有高的神经弓,背肋宽,臀部结构类似鸟类。宽广的臀部显示,牠们可能具有长的消化系统,以用来消化植物。根据系统发生学的位置,以及在北票龙身上发现的类似羽毛的覆盖物,镰刀龙科(或镰刀龙超科)可能都具有类似的覆盖物。镰刀龙科是由叶甫根尼·马列夫(Evgeny Maleev)在1954年建立,以包含新发现的镰刀龙,但当时马列夫认为牠们是种类似乌龟的巨大动物。长期以来,镰刀龙科是认为是群奇特的兽脚类恐龙,可能是肉食性。随者更多镰刀龙科近亲的化石被发现,原本观念遭到质疑。这些新化石是草食性动物、具有许多奇异特征,外表介于鸟臀目与原蜥脚下目之间。这些新发现物种被归类于慢龙科(Segnosauridae),意指牠们的沉重身体、短后肢,可能采取类似树櫴的生活方式。后来的研究发现镰刀龙不属于鸟臀目、原蜥脚下目,是种奇特的兽脚类恐龙,属于草食性,其近亲是同样奇特的慢龙。由于镰刀龙科的建立早于慢龙科,因此具有优先权,而慢龙科成为镰刀龙科的次异名。在1998年,保罗·塞里诺(Paul Sereno)首次对镰刀龙科提出系统发生学定义,定义为:亲缘关系接近死神龙,而离似鸟龙较远的所有物种。随者更多大型镰刀龙类化石的发现,镰刀龙科与其他兽脚类恐龙的演化关系更为明确,镰刀龙科的范围更为狭窄。在2005年,保罗·塞里诺将镰刀龙科重新定义为:包含镰刀龙、懒爪龙、内蒙古龙在内的最小演化支。以下演化树是根据菲力·森特(Phil Senter)在2007年的研究:","text2":"镰刀龙科是谁建立的?","label":1} {"text1":"五山街道是中国广东省广州市天河区下辖的一个街道,始设于1953年。辖区总面积10.59平方公里,2005年总人口9.56万人,常住人口7.41万人,外来人口2.15万人。2010年,五山街道辖13个社区:茶山、岳洲、东莞庄、华工、华农、广外艺、瘦狗岭、五所、粤垦、农科院、白石岗、高胜、汇景。辖区内有茶山新村、五山花园、天一新村、嘉逸花园、半山翠庭、高胜花园、茶山小区、嵩山小区、凤凰新村、南秀村、南新村、西秀村、东秀村、春晖苑、半山雍景苑、南门苑、茶景苑、瀚景轩、汇景新城、丽晴轩、九洲文化家园等住宅小区。广州地铁3号线五山站岳洲路、五山路、粤汉路、汇景北路、东莞庄路、茶山路、华南东侧路(往 长兴街道 岑村 方向)、东莞庄一横路、长福路、广园快速路(瘦狗岭路)、粤垦路、凤阳路( 华农 校内道路)22路、20路、197路、78路、27路、138路、B10路(原234路)、41路、218路","text2":"五山街道始设于哪一年?","label":1} {"text1":"尚温(;)是琉球国第二尚氏王朝第十五代国王。1795年至1802年在位。童名思五郎金。他是尚穆王世子尚哲的次子。尚穆王死后由他继位。尚温王对儒学在琉球民间的普及做出了贡献。1798年,尚温王接受了国师蔡世昌(高岛亲方)的建议,下令在王宫以北开办国学(现首里高等学校),又开办了三个乡学(平等学校);同时废止了久米村对四个官生名额的垄断行为,规定官生中的二名应为首里人。这引起了不少久米村人的不满,引起了官生骚动。尚温王逮捕了首谋者金文和(松永亲云上)等人,镇压了这次骚乱。1800年(嘉庆五年),清嘉庆帝派修撰赵文楷、编修李鼎元为正副册封使,前往琉球国,册封尚温为王,并赐「海表恭藩」的御笔匾额。1801年,在乡民的请求下,尚温王又下令开办了一个乡学。1802年,琉球第一次向中国派遣首里城出身的官生。|-style=\"text-align: center; background: #FFE4E1;\"","text2":"尚温对什么学科在琉球民间的普及做出了贡献?","label":1} {"text1":"这是一个关于台湾历史上主要地震的列表,台湾位在地震活跃区环太平洋火山带中,菲律宾海板块和欧亚板块交界上。菲律宾海板块以每年平均82mm朝西北碰撞欧亚板块,因此地震频繁。台湾地震带主要有三:西部地震带、东部地震带及东北部地震带。西部地震带包括整个台湾西部地区,主要系因为板块碰撞前缘的断层作用而引发地震活动,震源深度较浅,由于人口稠密,因此大地震容易造成灾情。东部地震带系直接肇因于菲律宾海板块与欧亚板块碰撞所造成,地震频率高,震源深度较深。东北部地震带系受冲绳海槽扩张作用影响,多属浅层地震,并伴随有地热与火山活动现象。台湾历年来发生大地震均有地裂、山崩、断层等地壳变动之状况,如1906年3月17日嘉义大地震,产生梅山断层,长达13公里,水平变位最大为240公分,垂直变位最大为180公分,并有显著之地裂及喷泥等现象。1999年9月21日集集大地震,由车笼埔断层造成约100公里长之地表破裂,水平最大变位7公尺,垂直最大变位达4公尺,多处有喷沙、喷泥现象。然而也有地震起因于未破裂到地表上而无法用肉眼判断之盲断层所致,例如2013年3月27日于南投县仁爱乡发生之南投地震以及2016年2月6日于高雄市美浓区发生之台南地震即是。","text2":"嘉义大地震时产生了什么现象?","label":1} {"text1":"李希烈(),唐代燕州辽西人。唐德宗时为淮西节度使,是李忠臣的族侄,后叛唐自立,杀害来劝降的颜真卿,自己最终被部将毒杀。德宗建中二年(781年),李希烈奉诏讨伐成德节度使李宝臣之子李惟岳,并加封为南平郡王。六月,进驻随州,直捣襄阳,梁崇义兵败自杀。建中三年(782年),兼任平卢、淄青节度使,奉命征讨淄青李纳,李希烈反与河北藩镇朱滔、田悦等勾结,据许州(今河南许昌市),自称天下都元帅、建兴王,并铸行军都统印。建中四年正月,李希烈派部将李克诚袭击汝州(今河南临汝),生擒别驾李元平,又遣大将董待名等攻取尉氏(今河南尉氏),围攻郑州(今河南郑州),东都大震。建中四年(783年)十二月,攻入汴州(今河南开封),兴元元年(784年)正月以汴州为大梁府,称楚帝,设置百官,任命孙广、郑贲、李缓、李元平为宰相,年号武成。建中四年(783年)八月,李希烈发兵三万,围攻襄城(今河南襄县),九月,唐德宗为解襄城之围,诏令泾原(今甘肃泾原)等各道兵马援救襄城,不料爆发泾原兵变,泾原军人造反攻陷首都长安;唐德宗仓皇出逃至奉天(今陕西乾县),并发布《罪己诏》。李希烈率兵五万围宁陵(今河南宁陵),淮南节度使陈少游投降,李希烈引汴水灌城,濮州刺史刘昌和宁陵西城都知镇遏使高彦昭坚守,浙西节度使韩滉派遗王栖曜救援,栖曜择三千弩手善游者,夜渡汴水,希烈竟不知。一早,发弓射死登城者,希烈即被击退。兴元元年十月,李希烈又派部将翟崇晖袭取陈州(今河南淮阳)。宋亳节度使刘洽遣陇右幽州节度使曲环等联兵三万,于陈州城西击破之,十一月,生擒翟崇晖,斩首三万五千级。这时卢杞因嫉恨平原郡太守颜真卿,向德宗建议派颜真卿去安抚李希烈,又任命左龙武大将军哥舒曜为东都。颜真卿至汝州,李希烈反逼颜真卿投降,真卿不肯,被拘押。战火蔓延到河南,李希烈为刘玄佐(即刘洽)所败,逃归蔡州。贞元元年(785年)正月五日,颜真卿被李希烈缢杀于汝州。贞元二年(786年)初,李希烈连续进犯襄州、郑州,均被唐军击退。唐军收复汴州,希烈逃往蔡州(今河南汝南)。四月,李希烈吃变质的牛肉致病,被部将陈仙奇使医师陈山甫毒死,又杀其兄弟妻子七口,遂降唐,德宗命陈仙奇为节度使,吴少诚又杀陈仙奇。","text2":"李希烈怎么死的?","label":1} {"text1":"Apache Hadoop是一款支持数据密集型分布式应用程序并以Apache 2.0许可协议发布的开源软体框架。它支持在商品硬件构建的大型集群上运行的应用程序。Hadoop是根据谷歌公司发表的MapReduce和Google档案系统的论文自行实作而成。所有的Hadoop模块都有一个基本假设,即硬件故障是常见情况,应该由框架自动处理。Hadoop框架透明地为应用提供可靠性和数据移动。它实现了名为MapReduce的编程范式:应用程序被分割成许多小部分,而每个部分都能在集群中的任意节点上执行或重新执行。此外,Hadoop还提供了分布式文件系统,用以存储所有计算节点的数据,这为整个集群带来了非常高的带宽。MapReduce和分布式文件系统的设计,使得整个框架能够自动处理节点故障。它使应用程序与成千上万的独立计算的电脑和PB级的数据连接起来。现在普遍认为整个Apache Hadoop“平台”包括Hadoop内核、MapReduce、Hadoop分布式文件系统(HDFS)以及一些相关项目,有Apache Hive和Apache HBase等等。2008年2月19日,雅虎使用10,000个微处理器核心的Linux计算机集群运行一个Hadoop应用程式。其他知名用户包括:升阳电脑的Sun Grid Engine可以用来调度Hadoop Job。威斯康辛大学麦迪逊分校的Condor计算机集群软件也可以用作Hadoop Job的排程。","text2":"所有的Hadoop模块都有的一个基本假设是什么?","label":1} {"text1":"俺巴孩(;),《元史》作咸补海罕或罕布海,是蒙古诸部落统一前蒙兀国的“合罕”(即“汗”)之一。俺巴孩是成吉思汗曾祖父合不勒汗的堂兄弟和继承者。他原本是泰亦赤兀惕系的首领。《元朝秘史》和《拉施特书》将其尊称为“汗”。在合不勒汗时期,蒙古同塔塔儿因为一个巫医引发的事件发生冲突。或为平息冲突,俺巴孩登上汗位后,答应将其女儿嫁给一个塔塔儿的部落(主因人)首领,但在护送时和合不勒的长子斡勤巴儿合黑一起被塔塔儿人抓住,送与金朝献给金熙宗。俺巴孩被抓住后曾托人对合达安太子说:“今后以我为戒,你每将五个指甲磨尽,便坏了十个指头,也与我每报仇。”熙宗将俺巴孩钉在木驴上处死,这是“专惩治游牧叛人”的刑罚。之后成吉思汗在讨伐金国之前曾祭腾格里时曾说,讨伐金国是为了报其祖先俺巴孩等人的仇恨。据《元朝秘史》记载,俺巴孩死后,汗位由侄子忽图剌继承。","text2":"熙宗是怎样处死俺巴孩的?","label":1} {"text1":"唐彦谦(?),字茂业,号鹿门先生。并州晋阳(今山西太原)人。大中二年(848年)出生。才高负气,以博学多艺闻名乡里。能作诗,早年学温庭筠,唐彦谦师法众家,但李商隐对其影响最大,唐彦谦的《无题十首》,容易使人联想到李商隐的“无题诗”。杨亿称其“为诗纂慕玉溪,得其清峭感怆”。代表作有《采桑女》和《宿田家》。咸通末年上京考试,结果十余年不中。一说咸通二年登第。干符末年,兵乱,避地汉南。中和中期,王重荣镇守河中,聘为从事。历官兴元(今陕西汉中)节度副使,晋州、绛州、阆州(今四川阆中)、壁州(今四川通江)刺史。光启末,王重荣遇害,贬汉中掾曹。杨守亮镇守兴元(今陕西汉中市东)时,担任判官。晚年隐居鹿门山,专事著述。卒于汉中,卒年约在景福元年前后。著有《鹿门集》3卷,附拾遗1卷,续补遗1卷,并由吏部侍郎薛庭珪作序。","text2":"唐彦谦是哪里人?","label":1} {"text1":"爱晨径(英文:Oi Sen Path)是香港九龙中部的一条行人径,位于红磡以北,何文田以南,是十二号山的南坡,靠近港铁东铁线路轨和爱民邨。北端连接治民街;南端连接支路较多,可经行人天桥往香港理工大学育才道或经行人隧道往红磡温思劳街。此外,近温思劳街处可抄小径至忠孝街。此路早在20世纪初已建成,为当时唯一由红磡通往何文田或旺角的道路,但因道路多山坡,在雨季时容易爆发山泥倾泻,长而令道路封闭,加上汽车量大增,做成红磡通往何文田的不便。佛光街建成后,问题才得以解决,但亦令爱晨径的重要性大减。1990年代曾有匪徒藏于树上乘机把途人洗劫并把人绑于树上。由2012年12月21日起,因为沙田至中环线工程的展开,爱晨径暂时封闭,至今仍未有解封的时间表。香港街道映照-爱晨径","text2":"爱晨径是何时建成的?","label":1} {"text1":"本条目以编年体介绍香港政府各决策科(今决策局)于1973年设立至今的演变。1973年1974年1975年1976年1978年1979年1980年1981年1982年1983年1985年1988年1989年1990年1991年1993年1994年1997年1998年2000年2002年2007年2012年然而由于立法会拉布持续,特区政府未能于7月1日以前完成所有立法会程序,第四届特区政府最终以原有三司十二局上任。而五司十四局方案更因立法会换届而胎死腹中。(详情请参阅2012年香港立法会拉布攻防战、五司十四局及香港冗长辩论条目)2015年原商务局通讯及科技科更名为通讯及创意产业科,继续主管通讯及创意产业政策。2018年","text2":"特区政府为什么未能于7月1日以前完成所有立法会程序?","label":1} {"text1":"尚宣威(;)是琉球国第二尚氏王朝的第2代国王,1477年在位(在位仅六个月)。神号西之世主()。他是尚稷的次子,尚圆的胞弟。据《中山世谱》记载,尚宣威5岁时父母双亡,由其兄尚圆养育;9岁时随兄至首里。1453年(明景泰四年)升家来赤头。1463年(天顺七年)升黄冠。1470年(成化六年),因其兄尚圆成为琉球王,尚宣威以王弟的身份得到越来间切的领地,称越来王子。成化十二年(1476年)丙申,尚圆王薨。由于当时世子尚真还在冲龄,且第一尚氏王朝的支持者势力仍旧很大,群臣遂奉王弟尚宣威于次年丁酉登位。但先王之妃宇喜也嘉十分不满,勾结祝女集团,阴谋推翻他的统治。按照惯例,每当琉球新王登基,太阳神、君手摩神便会出现在首里城,附身于祝女,自殿内出,至奉神门后东面而立,祝贺新王,并授予其神号。成化十三年二月,太阳神、君手摩神如期附身于祝女,不过此次祝女却面向西面而立,群臣皆大惊失色。祝女托以神命,授予尚宣威王「西之世主」的神号;同时颂扬世子尚真,要求尚宣威禅位给世子。尚宣威的威严顿失,不得不遵照神命,禅位于尚真,并退隐于越来,在位仅六个月。尚宣威王与宇喜也嘉达成妥协,将女儿居仁嫁给尚真当王妃。同年八月初四病逝,寿四十八岁,谥号义忠。退位不久即,有人认为是死于宇喜也嘉的暗杀。他死后,他的后代世代领有越来间切之地。相传尚宣威王的坟墓位于冲绳市八重岛,其遗址在今冲绳县立美来工科高等学校(原冲绳县立中部工业高校)的西南侧,是冲绳市史迹、文化财之一。","text2":"尚宣威是尚稷的第几个儿子?","label":1} {"text1":"杰克·凯鲁亚克(Jack Kerouac,)是一位美国小说家、作家、艺术家与诗人,也是垮掉的一代中最有名的作家之一,与艾伦·金斯堡(Allen Ginsberg)、威廉·柏洛兹(William S. Burroughs)齐名。虽然他的作品相当受到欢迎,但是评论家并没有给予太多喝采。杰克·凯鲁亚克最知名的作品是《在路上》。1922年3月12日,凯鲁亚克出生于马萨诸塞州洛厄尔,父母为法裔美国人,他是家中幼子。他曾在当地天主教和公立学校就读,以橄榄球奖学金入纽约哥伦比亚大学,结识艾伦·金斯堡、威廉·柏洛兹和尼尔·卡萨迪等“垮掉的一代”。凯鲁亚克大学二年级退学从事文学创作,并辗转于美国海军和商用航运公司等处。1950年,第一部小说《乡镇和城市》出版。1957年的《在路上》问世后,他成为“垮掉的一代”的代言人,跻身二十世纪最有争议的著名作家行列。1969年10月20日,凯鲁亚克因肝硬化导致食道静脉曲张大量吐血而接受手术, 在接受手术后凯鲁亚克未再恢复意识, 于10月21日清晨5:15在佛罗里达圣彼得堡去世,享年47岁。《乡镇和城市》、《在路上》、《梦之书》、《达摩流浪者》、《地下人》、《孤独的旅人》和《孤独天使》等","text2":"杰克·凯鲁亚克与谁齐名?","label":1} {"text1":"古大气层是在过去的地质时期的大气层,特别是地球大气层,在非特别指定的时刻。地球古大气层的构成储存在地质里,可以从现今对具有代理性的物质,例如氧化铁的丰度、木碳和树叶化石上气孔的密度,等的研究上推断出来。虽然今天大气层的成分是大约78%的氮和21%的氧,这在有生物前的大气被认为是高度缺乏的,实际上没有自由氧和氩,是由钾的同位素经由放射性衰变产生的,并且是以氮气、二氧化碳和甲烷为主。看得出来自由氧不是原本就存在的,是由细菌,像是蓝细菌或者蓝绿藻在距今大约20亿年前演变至能进行光合作用之后产生的。距今大约5亿年,氧浓度的增加使生物能发展出多细胞的动物性生物。从此之后,植物出现在土地上并且迅速的演变和放射状的散布至许多表面的土地上,大约在4亿5千万年前,在早石炭纪期间,氧的浓度超过了目前的数值,而大气中的数量降得比目前的值还要低。","text2":"今天大气层的成分是什么?","label":1} {"text1":"杨复恭(),字子烙,本姓林,福建人,唐朝宦官。弘农郡公杨复光从兄。唐懿宗时,镇压庞勋有功,升枢密使。黄巢之乱,黄巢攻克长安(今陕西西安),僖宗出逃兴元,杨复恭代田令孜为左神策中尉。僖宗回长安后,任复恭为左神策护军中尉、六军十二卫观军容使,封魏国公。唐僖宗病危时,复恭趁命右神策护军中尉刘季述迎立拥立寿王李杰即帝位,是为昭宗,有功,专典禁兵,擅权跋扈。杨复恭暗中与河东节度使李克用勾结,反对唐昭宗讨伐李克用,并从中作梗,最终朝廷军大败于河东军。杨复恭的养子杨守贞被任为龙剑节度使,杨守忠被任为武定军节度使,都不纳贡,上表诋毁诽谤朝廷。昭宗想让舅舅王瓌为节度使,杨复恭不肯,王瓌就骂他,杨复恭同意任王瓌为黔南节度使,但王瓌行经杨复恭养堂侄山南西道节度使杨守亮地界时,被杨复恭指使杨守亮谋害。大顺二年(891年)八月,杨复恭被迫以上将军致仕。大顺二年(891年)冬十月,被告发谋反。十一月八日,昭宗令天威都将李顺节等人逮捕杨复恭。张绾、杨守信奋死抵抗,九日禁军刘崇望前兵增援,杨府渐感不支。杨复恭出逃至兴元依附杨守亮。景福元年(892年)七月,杨守亮欲投奔李克用,至华州,被韩建擒获。杨复恭素跟韩建不和,对其辱骂,韩建大怒,将杨守亮一行人押解至长安斩首。","text2":"杨复恭是哪里人?","label":1} {"text1":"新加坡仙姑殿成立于农历壬申年十月二十三日(1992年11月17日),也正是仙姑下凡普渡众生的日子。原址是在后港3巷大牌251的住家庙,当时只供奉八仙,主神为女仙何仙姑,其余的七仙是 - 铁拐李、吕洞宾、张果老、曹国舅、汉钟离、韩湘子、蓝采和。由于仙姑的名声大振,慕名而来的香客源源不断,常把怎个地方挤得水泄不通。在短短的四年,这个办事处已经不能容纳上门祈求和膜拜的香客。以至在农历丙子年(1996年),仙姑决定把仙姑殿搬迁到芽笼25巷一栋两层楼高的私人建筑以方便她办事和应付更多香客。平时上门膜拜的香客已经占据了里外的仙姑殿,每年到了八月初八的仙姑寿诞日“八仙瑶池大会”,更要在德福巷大广场举行庆祝活动,以便应付千人宴席。从仙姑殿成立以来的10年里,她拥有许多信徒,香火鼎盛,因得到香客们的鼎力相助和支持,仙姑殿终于在农历壬午年六月十八日(2002年7月27日)正是搬迁到坐落在淡滨尼路一栋仙姑自己所构思和设计的5层楼高大殿。这栋耗资800万的新建庙宇宏伟壮观,以莲花为焦点,殿前的三个2.5公尺高的金字“仙姑殿”更受瞩目。殿内供奉高达十尺的主神何仙姑,和高达八尺的其余七仙。此外还有五尺和三尺高的其他道教天神、阴神、佛祖、拿督公。到仙姑殿祈求,也不可不看看殿内的三宝,虎虎生威的虎爷公、千斤黄铜制成的玉皇大帝香炉和一块6吨重的花岗石所围城的许愿池。","text2":"主神何仙姑在殿内有多高?","label":1} {"text1":"海名轩(英文:The Harbourfront Landmark)位于香港九龙红磡,是由长江实业发展的一个高尚住宅项目,于2002年8月入伙住宅项目由3座楼高54-59层物业组成,高度为232.6米,提供324个单位,每层只有A和B两个单位。单位面积由1,891至2,551方呎,项目以大单位为主。 邻近九龙海逸君绰酒店及甲级商厦海滨广场,景观主要为维多利亚港海景。发展商将第3座约100个单位,用作长线收租。海名轩商业部份位于平台楼层,分别位于3楼、5楼及7楼,约77,021平方呎。泓富产业信托上市时,海名轩商业部份被注入为泓富产业信托上市的资产。2017年1月20日,泓富产业信托宣布以8.86亿港元向佳利亚洲(香港)有限公司出售海名轩商业部份。住客会所设于7楼,总面积逾13万平方呎(室内面积逾3万平方呎,户外逾10万平方呎),耗资$8,800万兴建。会所焦点落在多用途宴会厅上,面积逾千呎,足够摆放8围12人餐桌,住客可以预订,作为饮宴、派对或开会。宴会厅内采用的纽西兰原幅织制纯棉羊毛地毡,长实集团主席李嘉诚办公室同样使用,价值约$40万,另外装上4盏美国白宫御用的吊灯,天花铺上18K金箔,就连窗帘亦用上泰国真丝织制。会所另一卖点是辟出多个独立房间,包括理发室、按摩室、私人健身室、桌球室等,务求住户可以独享私人空间,内置先进音响、DVD机等视听影音设施。发展商同时斥资数十万,添置海草泥疗综合美容仓、色光热能震动按摩椅等,亦提供海景健身室及逾20米长室内泳池。","text2":"海名轩是一个什么项目?","label":1} {"text1":"天主教圣地亚哥-德孔波斯特拉总教区(拉丁语:Archidioecesis Compostellanus、西班牙语:Archidiócesis de Santiago de Compostela)是罗马天主教在西班牙西北部加利西亚设立的5个教区之一,圣地亚哥-德孔波斯特拉教省的中心。成立于1120年。其主教座堂是相传耶稣十二门徒之一的雅各伯安葬于此,是著名西班牙朝圣地圣地亚哥-德孔波斯特拉主教座堂。2011年,在当地1,299,880人口中,有教友1,191,800人,占辖区总人口91,7%、1,071个堂区、771名司铎、11名执事、254名修士、1,035名修女。现任总主教为Julián Barrio Barrio。","text2":"现任主教是谁","label":1} {"text1":"花煎()是一种朝鲜半岛传统的甜煎米饼,上面可食用的时花制成。韩语固有词又称为、、。在不少传统节日如上巳节、重阳节等都会食用。从高丽王朝起,朝鲜人在一些节日里有玩「花煎游戏」(,)的传统习俗,并会食用花煎。每逢上巳节,妇女就会带著叫燔铁()的一种厚煎板去郊游踏青,她们会采摘杜鹃花或其他盛开的花来做花煎,当中以杜鹃花煎(固有词为)最具代表性。花煎常会与加入同种花瓣的花菜(,一种把水果加入蜜糖水或五味子汁里制成的朝鲜传统甜品)一同食用,例如以杜鹃花煎与杜鹃花菜同吃。到了重阳节,人们也会举行花煎游戏,这时食用的花煎会以菊花瓣制作,亦即菊花煎(),配以菊花酒()或柚子花菜()。除了杜鹃花煎和菊花煎外,任何可吃的时令花卉都可用作制造花煎,例如春季常见以沙梨花制成的梨花煎()、樱花煎(韩语固有词:)、堇菜花煎(固有词:)。初夏时则多会食用蔷薇花煎()。鸡冠花煎(韩语固有词:)则多在秋季食用。当没有适合制作花煎的花时,会以水芹、艾草、石耳、红枣等做成花形装饰放在煎饼面团上。","text2":"没有合适的花做花煎一般会怎么做?","label":1} {"text1":"伦敦打吡(London Derbies)是以下同城打吡的统称:-{zh-hans:阿森纳;zh-hk:阿仙奴}-(北)、-{zh-hans:切尔西;zh-hk:车路士}-(西)、水晶宫(南)、-{zh-hans:托特纳姆热刺;zh-hk:热刺}-(北)、-{zh-hans:西汉姆联;zh-hk:韦斯咸}-(东)、-{zh-hans:布伦特福德;zh-hk:宾福特}-(南)、-{zh-hans:查尔顿竞技;zh-hk:查尔顿}-(南)、-{zh-hans:富勒姆;zh-hk:富咸}-(西)、-{zh-hans:女王公园巡游者;zh-hk:昆士柏流浪}-(西)、-{zh-hans:米尔沃尔;zh-hk:米禾尔}-(南)、-{zh-hans:AFC温布尔登;zh-hk:AFC温布顿}-(南)、-{zh-hans:巴尼特;zh-hk:班列特}-(北)、-{zh-hans:达根汉姆;zh-hk:杜根咸}-(东)、-{zh-hans:莱顿东方;zh-hk:奥连特}-(东)2015\/16年球季共有14支以大伦敦为基地的球队在英格兰超级足球联赛及各级英格兰足球联赛中角逐,同级球队每年在联赛最少有两次对碰的机会。以大伦敦为基地的前英格兰超级足球联赛及各级英格兰足球联赛俱乐部包括-{zh-hans:温布尔顿;zh-hk:温布顿}-(2003年移到白金汉郡的米尔顿凯恩斯)和-{zh-hans:泰晤士A.F.C.;zh-hk:泰晤士A.F.C.}-(1932年解散)。","text2":"14支以大伦敦为基地的球队在哪些比赛中角逐?","label":1} {"text1":"《异兽魔都》(ドロヘドロ)是林田球绘制的日本漫画,在小学馆的月刊IKKI上连载。异兽魔都的故事发生在并存的两个世界中,是普通人所居住的“洞穴”和魔法师居住的世界。魔法师常常通过“门”自由出入洞穴,用洞穴的居民做魔法练习。主人公开曼就是魔法的受害者之一,头部被变成蜥蜴,也完全记不起自己是谁。为了变回人脸和找回记忆,故事就从开曼和同伴二阶堂就从狩猎魔法师的行动开始了。人类,多数居民都成为魔法师练习下的牺牲品。快速变化的人体受不了了魔法,即使经过治疗大多数人会突然死亡。魔法师世界中最大的家族,拥有各种产业,也树立很多敌人,人员分布也非常广泛。目前因为十字眼帮老大的出现,家族被破坏殆尽,成员逃到洞穴当中。十字眼帮由一群不会使用魔法的人组成。在双眼都有著十字的刺青。","text2":"《异兽魔都》是哪个国家的漫画?","label":1} {"text1":"仰口鲾,又称鹿斑仰口鲾,俗名金钱仔,为辐鳍鱼纲鲈形目鲾科的鱼类。本鱼分布于印度西太平洋区,包括东非、马达加斯加、模里西斯、马尔地夫、印度、斯里兰卡、孟加拉湾、安达曼海、泰国、越南、马来西亚、柬埔寨、台湾、中国沿海、日本、菲律宾、印尼、澳洲、新几内亚、马里亚纳群岛、马绍尔群岛、帛琉、密克罗尼西亚、所罗门群岛、诺鲁等海域。水深5至20公尺。本鱼上下颌齿细小,刷毛状,各成一列,无犬齿。口极小,伸出时向下斜,闭合时下颌垂直位。标准体长为体高的1.5至1.8倍。由眼之前缘至颊部有一黑色线纹,鳃盖上有一黑斑。体侧上有细横纹5至6条,长度不超过体侧中心线。背鳍硬棘8枚、软条16枚;臀鳍硬棘3枚、软条14枚。体长可达8公分。属于小型热带沿岸肉食性鱼类,岸边至近海的沙泥地皆可发现其踪迹。游泳能力差,常聚集成群,缓慢的在海床上觅食。以浮游动物,如桡脚类、大型甲壳类的幼生期及稚鱼为食。可食用,为鱼体小,常当作下杂鱼。","text2":"仰口鲾主要分布在哪里?","label":1} {"text1":"曾其祥(Zeng Qixiang,),出生于广东省梅州市梅县区,中国的职业足球运动员,司职前锋,现时效力澳门甲组足球联赛球队名门世家加义。出生于广东省梅州市梅县区的曾其祥,2003年入选广东省全运队,参加2005年的第十届全运会,同年并入选广东队,参加过省港杯足球赛。2006年加盟宁波华奥,参加中国足球乙级联赛(南区)的比赛,2007年追随中国国家青年足球队教练翟飚,加盟FC四川。2008年四川大地震后,因FC四川内部情况混乱,曾其祥遂来港加盟香港甲组足球联赛地区球队天水围飞马。2009年1月,以借将身份加盟东方,同年7月转投中甲球队广东日之泉。2010年,加盟澳门甲组足球联赛劲旅名门世家加义。2008年10月8日晚,天水围飞马首次以元朗大球场作为主场,迎战南华。天水围飞马教练列卡度于61分钟派出曾其祥后备入替黄展鸿,由于当时天水围飞马场内已有六名外援球员,属于国援身份的曾其祥上阵后,令天水围飞马有多达七名非本地球员同时上阵,违反了足总该届香港甲组足球联赛非本地球员注八出六的规定,故此虽然天水围飞马于比赛中以3-2击败南华,最终被足总判输0-3","text2":"2003年曾其祥入选什么队?","label":1} {"text1":"詹姆士·维南·泰勒(James Vernon Taylor,),美国音乐人,吉他演奏家。泰勒出生于马萨诸塞州波士顿,在北卡罗莱纳州的卡波罗市(Carrboro)长大。泰勒的事业生涯起于1960年代中期,但直到70年代才逐渐受到欢迎。他多演唱感性柔和的歌曲。同时期蔚为风潮的自弹自唱者包含琼尼·米歇尔、Tom Rush、Cat Stevens、Carole King、John Denver、Jim Croce、Don McLean、Gordon Lightfoot及 Jackson Browne,同时还有 Carly Simon,后来成为泰勒的妻子。他在1976年出了张钻石级唱片红极一时,总共售出超过一千一百万张,他在90年代及00年早期也出过几张销售成绩斐然及获奖无数的歌曲,仍保有相当大量的听众。","text2":"詹姆士·维南·泰勒的身份是什么?","label":1} {"text1":"上海铁路博物馆位于上海市闸北区天目东路200号,自上海天目东路上海铁路局机关大院西行百米。上海铁路博物馆于2004年8月建成并对外开放。整个博物馆包括占地约1300平方米的室外广场展区和拥有3000余平方米建筑面积的博物馆主楼。博物馆主楼共4层,目前开放的是面积约1000平方米的底层展区,展品主要为图片、文献史料、实物等。上海铁路博物馆的整体布局有比较浓重的铁路往事般的历史氛围:室外的广场展区营造了一个早期铁路火车站的场景,笨重的蒸汽机车和木结构的月台雨棚显得饱经岁月沧桑。博物馆的4层主楼以80%的比例按照1909年建成的沪宁铁路上海站原样建设,再现了当年上海站的英式古典风格风貌。博物馆是上海市专题性科普场馆。","text2":"上海铁路博物馆的整体布局表现出了什么样的历史氛围?","label":1} {"text1":"冬季六边形也称为冬季大椭圆是一个看起来是六边形的星群,在六个顶点上分别是猎户座的参宿七、金牛座的毕宿五、御夫座的五车二、双子座的北河三(跳过北河二)、小犬座的南河三与大犬座的天狼星。虽然主要的天体都在北半球,但是几乎在地球上所有的陆地上都能看见(除了纽西兰的南岛,南美洲的智利和阿根廷最南边)这个星群在每年的12月至3月在天空中闪耀著。在热带和南半球(称呼这个星群为「夏季六边形」或「夏季大椭圆」)还可以延伸至更南边的老人星。更小但更规则的形状是「冬季大三角」,几乎是一个三边等长的正三角形,与大星群共用两个顶点(天狼星和南河三),第三个顶点是接近六边形中心位置的猎户座参宿四(Betelgeuse、α Ori)。这三颗星都是亮星表上的恒星,从地球上观察,也都在太阳系之外。参宿四是猎户座内很容易辨认的一颗1等星,位置在猎户的肩膀上,可以协助找到这个三角形。一旦确认了三角形,六边型就很容易发现了。使用下面各种不同的方法,通过猎户座内诸星不同方式连线的延伸,也能独立发现六边形顶点的每一颗星。六边形的星分属于六个不同的星座,以逆时钟方向旋转,从属于猎户座的参宿七开始,依序是金牛座、御夫座、双子座、小犬座和大犬座。","text2":"冬季六边形在六个顶点上分别是什么?","label":1} {"text1":"孙奋(),字子扬,孙权的第五子、生母为仲姬,夫人为刘氏。三国时东吴皇子,后封齐王及章安侯。吴大帝太元二年(公元252年),孙奋被立为齐王,居于武昌。同年,吴大帝孙权逝世,太傅诸葛恪掌权,他不希望诸王居住在长江江边战略要地,于是要孙奋迁居到豫章。孙奋怒而不肯从命,又数次违反法令。诸葛恪上书劝谏孙奋,希望他有所节制,守法行事,尽忠东吴,以免日后与兄长鲁王孙霸一样遭到杀身之祸。(《诸葛恪劝孙奋书》)孙奋收到诸葛恪的上书后大惧,于是移居到南昌。孙奋又沉迷于游猎,官员和属下都苦不堪言。后来诸葛恪被孙峻诛杀,孙奋去到芜湖,要到建业看朝中事变。傅相谢慈等劝谏孙奋,被孙奋杀害。孙奋因擅杀官吏而被废为庶人,流放到章安县。太平三年(公元258年),又被封为章安侯。吴末帝建衡二年(公元270年),孙皓宠妃左夫人王氏逝世。孙皓十分悲伤,整天都在哭,多个月都没有出外露面,民间更有传闻说孙皓已死,于是有人讹称孙奋与上虞侯孙奉两人之中将会有一个要被立为皇帝。孙奋生母仲姬的陵墓在豫章,豫章太守张俊对孙奋会被拥立的传闻半信半疑,为仲姬的陵墓打扫。孙皓知道后,以车裂处决张俊及夷其三族,并且诛杀孙奋及他五个儿子,封国废除。仲姬,孙奋生母,葬在豫章,因为豫章太守张俊讨好孙奋,打扫仲姬墓,触怒孙皓,导致张俊及其三族,与孙奋和他五个儿子,全部被杀。陈寿评曰:「(孙)奋不遵轨度,固取危亡之道也。然奋之诛夷,横遇飞祸矣。」《三国志·吴书·吴主五子传》","text2":"孙奋的封号是什么?","label":1} {"text1":"都城嘉慕威曼都城骑士(MCW Metrorider)是一款由都城嘉慕威曼(Metro-Cammell Weymann,缩写为MCW)于1986年研制的前置引擎小型单层巴士,主要针对英国本土市场使用,以合理售价,简易维修的优点而广受当地业界欢迎。都城嘉慕威曼曾经在1988年7月至11月把4部长阵版样板车送到香港(除中巴的CM2外,其余3部在新车投入服务时已装有Nippon Denso空调系统,CM2则后来加装三菱空调系统),但中巴的CM1和CM2-{只}-试用了7年便退役,1996年被拆卸;而九巴的AMR1和AMR2(原编号MR1及MR2)更只试用了5年,AMR1被必达巴士购入,辗转售往英国并重新登记车牌F737KGJ,AMR2则被首先运到广西服务,随后转到广州三汽公司使用,车牌为粤A33270。AMR2在加入九巴前曾于新加坡作样办车之用。Metrorider分有短阵及长阵版,前者可载25人,后者可载33人。短阵版当年售价为£26,000(英镑)。Metrorider不像其他一人控制巴士般把引擎放置在头轴前,而是把引擎放置在头轴上,就如半驾驶舱设计的巴士一般,令它的车身头幅设计成一个大幅度倾斜的平面,巧妙地令车厢内部空间更觉完整,亦令驾驶视线较为广阔。Metrorider的车身-底盘整合设计亦不像同厂Metrobus可让买家自由选择车身供应商。都城嘉慕威曼于1989年结业后将Metrorider的设计及生产权售予Optare。Optare为Metrorider作出少量改动,随即以Optare MetroRider的名义重新推出市场,亦广受当地业界欢迎。","text2":"都城嘉慕威曼都城骑士在1988年曾送样车到了什么地方?","label":1} {"text1":"大卫·沙诺夫(,)美国商业无线电和电视的先趋和企业家。被誉为美国广播通讯业之父。俄罗斯帝国(白俄罗斯)犹太裔移民。早年随父母移居美国。15岁到美国马可尼公司工作。17岁时成为公司的无线电报务员。1912年泰坦尼克号沉没事件使其一举成名,当海难发生时,萨尔诺夫正值班,此后三天三夜,他在电报室里将生还者名单接二连三发出,无形中所有人的希望都寄托在了他的身上。也正是此次意外事件使其名声大振。随后公司提升其为高级职员。1916年萨尔诺夫向公司递交一份备忘录,颇有预见性地建议公司研究开发他所说的「无线电乐盒」(Radio Music Box),这个构想实际上已经描绘了未来广播的雏形。但当时计划并未被采纳。创立国家广播公司(NBC)。1919年参与创立美国无线电公司(Radio Corporation of America,RCA),投注大部分心力,领导NBC诞生。1930年萨尔诺夫升任美国无线电公司总经理。1934年出任NBC的董事长。从此他的名字与NBC紧密的联系在一起,直到1970年退休。他一手掌控通讯和消费性电子帝国,即RCA和NBC二家公司。二战期间,他作为预备役军官,在艾森豪威尔的参谋部任新闻顾问,负责盟军在欧洲的广播,并领准将军衔,所以后来人们习惯称其为「萨尔诺夫将军」(General Sarnoff)。","text2":"被誉为美国广播通讯业之父的是谁?","label":1} {"text1":"陈豪文(Chan Ho Man,),香港职业足球员,司职前锋,前香港奥运足球代表队及香港足球代表队队员,现时效力于香港甲组足球联赛球会骏其天旭。出身于流浪,年仅16岁便获提升为甲组后备,被誉为香港球坛明日之星。1999年转投二合,逐渐踢出名堂,二合于2001年退出后,获香港甲组足球联赛老牌球会南华邀请加盟。陈豪文于2000-01年及2001-02年度球季,连续两届于香港足球明星选举中,当选最佳年青球员。2003年转投晨曦,直至2008年球季结束后,约满未获续约离队。陈豪文转投乙组球会福建。2009年初,加盟骏升大中,协助该队获得香港乙组足球联赛亚军,成功取得升班资格。2009年夏天,转投另一支乙组球队骏升花花,全季射入6球。2010-11年度球季,再转投香港丙组足球联赛球队喜利。2013年初,陈豪文转投香港丁组足球联赛球队天旭。陈豪文于1999年,首次入选香港奥运足球代表队,2000年更以20岁之龄入选香港足球代表队,迄今为香港队取得6个入球。","text2":"陈豪文为什么被称为香港球坛明日之星?","label":1} {"text1":"索科特拉机场(英文:Socotra Airport,阿拉伯文:مطار سقطرى الدولي),是也门索科特拉岛上的一座机场,是该岛唯一的商业机场。建于1999年。大部分班次都是往大陆的穆卡拉(里扬机场),大部分飞机都会在那里进行一个技术修理然后才到也门首都:萨那或南部海港亚丁。机场位于岛的北部,但主要旅游景点在岛的西部Qalansiyah海滩,由机场到岛西部需时约2小时。该处是一个无污染,无噪音和浪漫的海滩。机场没有任何定期的交通来往哈迪布,但可以等约15分钟乘任何车辆,大部分车都是向西走。旅客可以乘巴士,但需要比其他居民付多点车费,大约须30至50也门里亚尔(1美元formula_1 200也门里亚尔)。除非旅客懂操阿拉伯语或可以与私家车司机交易,否则如没有妥当安排,到达Hadibo是较困难的。值得注意的是,目前索科特拉岛仍然是一个非发达的岛屿,没有工业和农业,不能预期提供各种食品和当地食品,因此大量的收费项目,都是由也门内地进口的,连瓶装水也不例外。2015年3月,由于沙特阿拉伯的军事行动,索科特拉机场航线停飞。","text2":"什么是门索科特拉岛上唯一的商业机场?","label":1} {"text1":"曾淑娥(, ),是台湾女子足球运动员,她自2002年起代表中华台北女子足球代表队参加各项国际赛事。在2002年亚足联U-19女子足球锦标赛中,曾淑娥协助中华台北夺得亚军,并获得大会最佳球员及金靴奖。2008年8月,曾淑娥通过日本女子足球联赛的神户 Tasaki Perule的测试,随即因球队解散,无法成行。2009年1月,曾淑娥通过日本女子足球联赛的千叶 JEF的测试,但该队临时反悔,而无法加入。2009年3月,曾淑娥通过日本女子足球联赛的福冈 J. Anclas的测试,却因该队不提供签证事宜,而无法成行。同年9月,曾淑娥加入澳洲的坎培拉联队。2012年9月,曾淑娥加入法国女甲的罗亚尔圣艾蒂安女子足球俱乐部。2014年2月,曾淑娥加入奥地利SV Neulengbach,于3月23日代表SV Neulengbach先发出战瑞典Tyresö FF在欧洲女子冠军联赛的赛事,攻进一球,使曾淑娥成为台湾第一位在欧洲女子冠军联赛出赛和进球的球员。2014年9月,曾淑娥回到台湾加入台中蓝鲸女子足球队,10月5日首次出赛。2015年球季转队新竹FC,不久后加盟冰岛菲尔基尔足球俱乐部。2015年 到法国阿尔比队测试,成功争取到一季的合约。","text2":"谁成为台湾第一位在欧洲女子冠军联赛出赛和进球的球员?","label":1} {"text1":"徐阳,是已退役的中国足球运动员,曾经入选过国家队,退役后成为知名足球评论员。徐阳职业生涯的变故与中国足球转会体制有很大的关联。2001年初他以国脚身份从北京国安挂牌希望转会到重庆力帆,但是被山东鲁能中途“截牌”,最终以350万元的高价完成转会。不过加盟山东后徐阳却连续两年无法进入主力阵容。2003年,再次转会希望回到自己的母队八一队,但是八一成功摘牌后却最终支付不起山东鲁能索要的转会费,导致徐阳整年无球可踢,赛季结束后八一降级、解散,徐阳的所属权回归山东。2004赛季,徐阳仍然没有获得出场机会,赛季结束后最终选择了退役,当时刚刚年满30岁。退役后徐阳转型却颇为成功,成为国内较知名的足球评论员之一。","text2":"徐阳的退役年龄是多大?","label":1} {"text1":"中国文化大学华语中心(Mandarin Learning Center,缩写 MLC),隶属于中国文化大学推广教育部。每年有将近1千位外籍生来MLC研习中文。中国文化大学华语中心于1992年成立,提供外籍生华语课程,由华语教师设计适合各程度的课程,包含了从适合成人、青少年、儿童、上班族到从事华语教学的台湾人的训练。。位于文大推广部大夏馆大楼内,附近有国立台湾大学、国立台湾师范大学、国立台北教育大学等大专院校,邻近大安森林公园及台北市立图书馆。位于台北市大安区建国南路二段231号4楼。MLC华语中心设有普通教室,与多种数位学习设备,并有数位学习中心提供网路与电脑的教学使用。<\/ref>。2013,MLC使用注音符号和汉语拼音两种拼音系统教授中文,有部分的老师也具备双语能力。成人班课程为周一至周五每天3小时的训练,加上每周一次的线上TOP练习,做为老师评量学生进度与各班进度的统一标准。另外设计了HSK考前加强班和全修课程。MLC也提供多种课程长度,可配合学生在台居留期限做选择。MLC有使用师范大学出版的制式教材,也有设计不同的课程供学生选择。这些课程包含了电影、中文歌曲、旅游中文及HSK准备课程等。","text2":"中国文化大学华语中心隶属于哪个组织?","label":1} {"text1":"韦斯特曼纳群岛(英文:The Westman Islands;冰岛语:Vestmannaeyjar),位于冰岛南海岸外的火山群岛,由14个小岛屿组成,总面积约21平方公里。岛上四处岩石裸露,一片荒芜,唯一有居民的是最大之岛屿赫马岛(Heimaey)。岛上有韦斯特曼纳埃亚尔镇。1963年-1967年西南部因火山喷发而使小岛叙尔特塞突出海面,当时,全群岛覆盖了一层火山灰。冰岛外海的一座火山岛,也是冰岛的最南端。它是因海面下130公尺火山爆发而形成,于1963年11月14日突出海面。火山喷发一直持续到1967年6月5日,岛的面积也达到最大值2.7平方公里。之后,由于风和波浪的侵蚀,导致岛逐渐变小,到了2002年,其面积为1.4平方公里。","text2":"为什么小岛叙尔特塞能突出海面?","label":1} {"text1":"北港国际音乐文化艺术节 (Beigang International Music Festival)是由云林县北港爱乐协会主办,首次在2006年举行,受到政府单位及社会人士的支持与肯定。接著每年定期举办,至今成为云林县最大的国际文化艺术节。此音乐节提供一连串的音乐会,大部份是管乐(独奏、重奏及管乐团)并在陈家湖音乐学院开办音乐教学讲座,节目的设计是由来自世界各地的音乐家(由云林县北港爱乐协会邀请)共同参与,此艺术节的音乐艺术总监是钢琴家。北港是台湾开台400年来最古老的小镇,而北港朝天宫是全世界最有名的妈祖庙。除了宗教庆典及传统地方文化外,也需要文化艺术来美化生活。所以爱乐协会决定每年在北港举行国际文化艺术节,让北港小镇更活化,北港同时也是艺术节的重镇,在此有许多场音乐会的演出及教学讲座,也同时和青少年管乐团练习并演出。2005年举办北港乐团(成立于1925年)80周年庆暨北港爱乐季,2006云林县北港爱乐协会聘请陈学孟为音乐艺术总监,并把整个爱乐季提升到国际层面,从此整个爱乐季受到听众热烈的回响,媒体的争相报导更受到地方人士及政府机构的肯定。云林县县长苏治芬对此爱乐季相当重视,每年必亲自莅临开幕仪式。2007年德国代特莫尔德音乐大学校长Martin Christian Vogel是当年的贵宾。音乐会除了大部份在北港妈祖文化大楼举办外,也安排在新营及斗六两大城市来举行。节目包括古典音乐、通俗音乐,还有公园广场的文化晚会,在北港镇外的田园蜜语西餐厅的酒馆爵士乐,最后的闭幕高潮是北港青少年管乐团的演出。公家行政机关有云林县政府、行政院文建会、内政部、北港朝天宫、北港镇公所、云林县地方法院检察署、云林县观护志工协进会及台湾更生保护会云林分会。教育机构如各社区之中小学及大专院校等。乐器公司有Kawai、Jupiter等,另外北港公司行号,医生、老师及爱乐人士。2009获得芬兰西贝流士音乐大学(芬兰)的特别赞助。台湾各大报纸的地方版及电视三台皆有报导,另外《新台湾新闻周刊》2007\/08\/02也有专题采访。德国的Lippische Landeszeitung 对台湾德国的文化交流非常感兴趣,也在2007年的爱乐季有篇幅报导。该音乐节主办单位为云林县北港爱乐协会,其会员皆于无薪给下自愿工作。","text2":"北港国际音乐文化艺术节的主办方是?","label":1} {"text1":"重力位能或重力势能是指物体因为大质量物体的万有引力而具有的位能,其大小与其到大质量的距离有关。formula_1其中G为万有引力常数,M、m分别为两物体质量,r为两者距离。依据古典力学,在两个或更多的质量之间存在著重力位能。能量守恒要求这种重力位能永远是负值。证明令无穷远处为势能零点,则有formula_2且万有引力formula_3解得formula_4由于质点所减少的势能等于保守力对该质点所做的功,所以引力势能formula_5在广义相对论,重力位能被塑造成兰道-栗弗席兹赝张量 ,以允许古典力学的守恒定律能够获得保留。加上物质的应力-能量张量至兰道-栗弗席兹赝张量的结果是结合了物质和重力能赝张量导致散度为零的发散。有些人反对在基础上做如此的延伸,认为这样做在广义相对论中是不适当的,这是只是守恒律的需要所衍生的用途,在这样的情况下只是赝张量和真张量的结合。","text2":"重力位能或重力势能是什么?","label":1} {"text1":"乔氏猫(学名:'),又名美洲云豹或美洲渔豹,可能是南美洲最普遍的野生猫科。乔氏猫是以19世纪法国动物学家艾蒂安·若弗鲁瓦·圣伊莱尔(Étienne Geoffroy Saint-Hilaire)来命名的。他发现了乔氏猫与其他物种的不同。乔氏猫分布在安地斯山脉、潘帕斯平原及大厦谷。乔氏猫约长60厘米,高31厘米,大小如家猫,重2-4公斤,个别可以重达8公斤。牠们有黑色的斑点,毛色则会按地区而有所分别:在北部,一般都是呈黄褐色;在南部,一般都是呈灰色。无论是在野外或饲养底下,牠们都很易患上黑变病。乔氏猫主要猎食啮齿目、细小的蜥蜴、昆虫,且有时吃青蛙及鱼类。牠们是顶级掠食者,位于食物链的顶部。虽然牠们似乎数量很丰富,但一些保育人士认为牠们大量被猎杀而需要关注。怀孕的雌猫会很小心选择产子的地方。幼猫成长得很快,只需约6星期就可以完全活动。","text2":"乔氏猫的主要栖息地在哪?","label":1} {"text1":"买姓是一个罕见的汉姓,在台湾是西拉雅平埔族特有的罕见姓氏,在中国大陆则是回族姓氏。买姓在台湾是一个特殊罕见姓氏,其为台湾平埔族中的西拉雅族(或称之为西拉雅平埔族)的特有姓氏之一,原为「Takalomay」,汉化后改为「买姓」。在台湾对于被汉化的原住民统称平埔族,因此若往后看到买姓的族人可以了解其人为西拉雅平埔族的后代。在台湾的平埔族历史中,西拉雅族曾经是平埔族中人口最多且势力最强的族群。西拉雅的传统社会制度为母系制度,但是当年许多来自中国大陆的单身男子,因为与西拉雅族的女子通婚而取得了财产继承权,不少西拉雅族人的财产与土地就因为这样而流入汉人的手中。现在想要亲身从台湾的台南市地方上去体会西拉雅的全貌已经非常困难了,但从部分残存的村落、信仰、习俗、传统祭典等中,还是可以窥见西拉雅风俗之一二。(参见:西拉雅族)台湾的西拉雅族原本是分布在嘉南平原位于台南市(现佳里、麻豆、善化、新化、新市、七股等地方一带)。但因明朝末年与清朝在治理台湾期间,大批中国大陆的汉人移居台湾,这也迫使西拉雅族往台南和台东两地的山区移动(现大内、官田、玉井、东山等地一带),据当地户政机关统计,目前台湾买姓居民最多的地方为台南市左镇区。买郎宅是一个地方名称,位于台南市麻豆区,「买郎宅」即「姓买的平埔族人所垦拓的田园」的意思,此地亦为台湾特产麻豆文旦的发迹地。中国大陆的买姓为回族后裔,与台湾买姓源流不同。买姓、毒姓(注:汉人亦有此姓)、永姓","text2":"中国大陆的买姓是哪个族的后裔?","label":1} {"text1":"日野江城()坐落于肥前国高来郡有间庄(长崎县南岛原市北有马町戊谷川名)。也被记作日之江城、、火之江城、日之枝城。是国家级历史遗迹。位于岛原铁道北有马站北部的小山丘上。南边有有马川流过,东边有大手川流经。地界是在西北方配置三之丸、在东方配置二之丸以及在中央的山顶上配置了本丸的连郭式平山城。自平成7年(1995年)到平成12年(2000年)为止进行了4次发掘调查。其间出土了除了安土城以外看不到的直线台阶,结合了海外技术的石组,加上了金箔的瓦。能窥知其与先进的丰臣政权有着密切的关系。现在遗迹能确认石垣·空堀。镰仓时代前期的建保年间(1213年-1219年)统领高来郡的藤原经澄所筑。经澄在藤原北家以藤原纯友的子孙(等同于平氏)筑城的时候以有间为姓,后来改称有马。有马氏当初只不过是岛原半岛南部的一个势力,贵纯的时候压制了半岛内的其他势力而成长为战国大名。贵纯以日野江城的支城为由建造了原城。晴纯的时候有马氏的版图到达了最大,一直成长到领21万石为止。然后,作为后盾的大内氏灭亡了而由此变得受到龙造寺氏的压迫。第13代当主晴信成为了天主教大名而在城下建设(葡萄牙语:神学院,此为日本的历史用语),反过来破坏寺院和神社作为城的建筑木材。前面所说的发掘调查出的遗迹是晴信时代的东西。江户时代初期晴信领有4万石,日野江城成为了岛原藩的藩厅。可是,庆长17年(1612年)晴信因为冈本大八事件遭连坐罪在甲斐国切腹自杀。后嗣的嫡子直纯在庆长19年(1614年)被移封到日向国延冈城,有马氏告别了这座约400年的城。有马氏离开后直到元和2年(1616年)松仓重政入城前为止的时间里该城成为了幕府直辖领地。在此期间由锅岛氏、大村氏、松浦氏这些肥前的大名分担城的警戒。可是,松仓氏入封后,对日野江城觉得不便而新建岛原城,日野江城废城从此退出历史舞台。昭和57年(1982年)7月3日,被认定为国家级历史遗迹。自平成7年(1995年)到平成12年(2000年)进行了发掘调查。平成18年(2006年)初城地的约3成左右被损坏的事调查清楚了。这个是进行了合并前的北有马町打算把城地公园化为樱花名胜的时候损坏了,现在,还剩下对现状进行修复的课题。","text2":"日野江城也被记作什么?","label":1} {"text1":"刘定国(),本名刘玉云,中华民国(台湾)军人及政治人物,苗栗县人,乙未战争遗孤。少年于台湾受李钟萼抚养并学习汉文,15岁赴日本留学,加入由丘逢甲之子丘念台所创的同乡读书会,并随丘氏前往中国大陆,考入中央陆军官校(黄埔军校)就读。曾参加对日抗战,战后回台湾任职保安司令部,并担任新竹县防卫司令。1951年参选第一届苗栗县县长选举,因第一轮无人过半而由前两名进行第二轮选举。刘定国虽以63,627票击败黄运金的60,814票,但因仍具军人身分,随后被判决当选无效。1954年代表国民党参选第二届县长选举,获得当选。1957年当选连任。1960年当选为台湾省议会第二届议员,1972年底当选为第一次增额国民大会代表。刘定国是为苗栗县地方派系刘派创始人。","text2":"刘定国赴日本留学时加入了什么组织?","label":1} {"text1":"中华民国空气污染指标(Pollutant Standards Index,PSI)是空气污染情况的一项指标,由中华民国行政院环境保护署于1993年扩充测站后推出,目标乃借由本测站系统监控全台湾所有的空气品质并加以通报改善。空气污染指标为依据监测资料将当日空气中悬浮微粒(PM10)、二氧化硫(SO)、二氧化氮(NO)、一氧化碳 (CO) 及臭氧 (O) 等5种空气污染物浓度数值,以其对人体健康的影响程度,分别换算出不同污染物之副指标值,再以当日各副指标值之最大值为该测站当日之空气污染指标值 (PSI) 。每监测区各取三个指标值最大的测站平均成为该监测区的指标值。转化为一个由0至500的单一数字,并按照指数高低而划分为良好、普通、不良、非常不良和有害五种级别。空气品质测站种类有一般空气品质监测站、工业空气品质监测站、交通空气品质监测站、国家公园空气品质监测站、背景空气品质监测站五种类别。有部分的测站会兼具两种类别。全台湾被分成七个空气品质监测区,北部地区(台北、新北、桃园、基隆)、新竹苗栗地区、中部地区(台中、彰化、南投)、云林嘉义台南地区、高雄屏东地区、宜兰地区、花莲台东地区。监测站位于台湾各地的学校和政府机关为主:台湾空气品质指数分为以下5级:","text2":"空气污染指标主要是以哪五种空气污染物数值来进行换算?","label":1} {"text1":"广州白云足球队是中国广州市昔日的一支足球队,现时广州恒大足球俱乐部前身。其前身是广州市足球一队,于1961年4月组建,1967年停办。1977年10月26日广州市人民政府批准恢复广州市足球运动队,称广州市青年足球队,1978年8月复办。1979年6月国务院发文要求发展足球重点地区,广州列首批重点地区的首位。1980年广州青年队改名为广州市足球队,并参加丙级联赛,获分区赛第2名,晋升乙级,次年再获乙级联赛冠军,进入顶级联赛。1984年10月1日由白云山制药总厂出资赞助每年20万元,与广州市体委合办,改名为广州白云山制药厂体协足球队,成为中国大陆首支由体育部门与企业合办的运动队。1989年广州白云队扩大为足球俱乐部。1993年1月8日广州太阳神集团和广州市体委合作成立中国首家股份制职业足球俱乐部,改名为广州太阳神足球俱乐部。","text2":"广州白云山制药厂体协足球队是由广州市体委和哪个企业合办的?","label":1} {"text1":"入江俊郎(),日本东京都人,曾经历最高裁判所判事(最高法院法官)。1924年东京帝大法律系毕业。历任内阁法制局参事官,行政裁判所评定官,法制局部长、法制局次长、法制局长宫,贵族院议员,国立国会图书馆专门调查员,东京帝大、庆应大学讲师等职。1948年任众议院法制局长。1951年赴美国考察议会制度。1952年8月30日以51岁之龄成为史上最年轻的最高法院法官。1954年任务法省法制审议会委员及行政诉讼部会长。1971年1月9日,以退休年龄辞去官职。最高法院法官在任期间6707天是历代第1位(2008年2月现在)。辞去官职后就任了驹泽大学教授。亲美的右翼政客。爱好诗歌和谣曲。著有《宪法要论》、《地方自治法提义》等。其兄入江克己是关东日野柴油机公司总经理。","text2":"1948年,入江俊郎担任什么职务?","label":1} {"text1":"《美国爱国者法案》()是2001年10月26日由美国总统乔治·沃克·布什签署颁布的国会法案,正式的名称为「」,中文意义为「透过使用适当之手段来阻止或避免恐怖主义以团结并强化美国的法律」,取英文原名的首字缩写简称为「USA PATRIOT Act」,而「」也是英语中「爱国者」之意。2015年5月30日,美国参议院没有就延长本法案的决议达成一致意见,故本法案于6月1日起失效;6月2日,作为对爱国者法案的代替,修改了部分内容,尤其是第215段,终止了NSA的电话监控的《》在国会通过,效力至2019年。该法案英文名称为「Uniting and Strengthening America by Fulfilling Rights and Ending Eavesdropping, Dragnet-collection and Online Monitoring Act」,中文直译改为「透过完善人权与结束窃听、警网搜索和网路监视以团结并强化美国的法律」,也一样画葫芦取其字首组成「USA FREEDOM Act」,「freedom」即是英文的「自由」之意。这个法案以防止恐怖主义的目的,扩张了美国警察机关的权限。根据法案的内容,警察机关有权搜索电话、电子邮件通讯、医疗、财务和其他种类的记录;减少对于美国本土外国情报单位的限制;扩张美国财政部长的权限以控制、管理金融方面的流通活动,特别是针对与外国人士或政治体有关的金融活动;并加强警察和移民管理单位对于拘留、驱逐被怀疑与恐怖主义有关的外籍人士的权力。这个法案也延伸了恐怖主义的定义,包括国内恐怖主义,扩大了警察机关可管理的活动范围。美国前共和党众议员让·保罗曾在国会公开指责爱国者法案是布什总统的「社会主义政策」。","text2":"该法案的中文意义是什么?","label":1} {"text1":"德瑞莎·麦德罗(英语:Teresa Medeiros)是一位1960年代出生,美国女性罗曼史作家。目前出版23部作品,多次荣登纽约时报畅销书榜。作品题材多为历史罗曼史,亦有数本穿越时空、吸血鬼题材,等超自然罗曼史作品。德瑞莎·麦德罗出生于1960年代美国,处女作于21岁时出版。在成为知名小说家之前,职业是一名护士。截至2015年为止,已出版23 部作品,翻译至十七种语言,总印量超过一千万册。目前与她的丈夫和两只猫,住在美国肯塔基州。麦德罗作品多次荣登纽约时报畅销书榜,赢得PRISM奖两次,亦获得两次Waldenbooks Award,同时也是Romance Writers of America创始会员,亦为肯特州罗曼史写作家和小说家协会会员之一。作品题材多为历史罗曼史。有少数系列作品为超自然罗曼史,例如:有穿越时空剧情的女巫系列或吸血鬼题材的The Lord of Midnight Series等。作者亦有现代罗曼史、西部罗曼史作品各一,可谓多方尝试写作方向。德瑞莎·麦德罗在台湾的中文译作大多由林白\/果树出版社出版代理,早期作品出版代理较多。近期作品目前无台湾代理。最新一本中文译作,亦是唯一本由高宝集团代理的作品,为2004出版之《一夜绯闻》(One Night of Scandal)。该出版社将作者名称误值为Terest Medeiros,译名为泰瑞莎‧梅戴洛斯。","text2":"她的中文译本一般由谁推广?","label":1} {"text1":"荷兰农业自然及食品品质部(Ministerie van Landbouw, Natuurbeheer en Voedselkwaliteit)是荷兰掌管农业的行政部门。农业部的职责有四大领域:农业部首长为大臣,主办公室位于海牙市中心。农业部底下设有十个政策部门,由管理委员会统领。十个政策部门为:农业自然及食品品质部成立于1935年,当时名称为「农业及渔业部」(Ministerie van Landbouw en Visserij)。在这之前,农渔业政策原来是由内政及王国关系部管辖,之后移交给水利贸易及工业部(Ministerie van Waterstaat, Handel en Nijverheid)。二战后,农业部开始负责食物配给及农业重建。1946年至1982年,该部是个「客户」导向的部会,发展符合欧洲共同农业政策的农业。 1982年,农业部的职责增加了自然保育及露天休闲娱乐,开始专注于农业的永续发展。2003年,食物及消费性产品安全管理局(Voedsel en Waren Autoriteit)成为所属机关,同时农业部改名为农业自然及食品品质部。","text2":"1982年农业部增加了什么职责?","label":1} {"text1":"四季名花,即兰花、荷花、菊花、梅花四种名花,又称“中国四季名花”。兰花代表春季,荷花代表夏季,菊花代表秋季,梅花代表冬季,是花文化在群众中发展而形成的,从被人格化的花卉身上提炼出来的称号。春兰夏荷秋菊冬梅虽各开一季,但它们皆被中国人赋予一致的特征:高尚的品质、高洁的品性和不屈不挠的精神内涵。四季名花深受文人墨客喜爱,各有名句流传:兰花“如入芝兰之室,久而不闻其香”;荷花“出淤泥而不染”;菊花“此花开尽更无花”;梅花“暗香浮动月黄昏”,历史流传下来的关于四季名花的诗词数量之多,形式之丰,质量之高,不胜枚举。书画史上:画兰花者如郑板桥、八大山人、任伯年等;画荷花如张大千;画菊花如郑思肖;画梅花如王冕、汪士慎。在中国传统花艺中,将兰、荷、菊、梅同插一瓶,寓意“四季平安”。当代又有《中华字经》:“奇花异卉,艳丽荣秧,兰荷菊梅,四季芬芳。”","text2":"四季名花分别代表什么季节?","label":1} {"text1":"美国国家图书奖(又译美国国家书卷奖;)是美国文学界的最高荣誉之一,始于1950年,每年11月在纽约颁奖,设有小说、非小说、诗词、少年图书4个大奖,颁发给前一年出版的文学作品,并向做出卓越贡献的作家颁发终身成就奖。评奖宗旨是扩大美国文学的影响、吸引读者以及提高作品的文化价值。国家图书奖的起源最早可以追溯到1930年代,二战期间停止运作,战后由美国书商协会(American Booksellers Association)、美国图书出版理事会(American Book Publishers Council)和图书厂商协会(Book Manufacturers Institute)联合重新在业内发起评奖颁奖活动,1950年在纽约华尔道夫酒店举行颁奖仪式,引起大众的关注。1950年代开始,国家图书奖的评奖规则、种类不断变化,从1988年开始由全国图书基金会(National Book Foundation)负责评奖、颁奖事宜,每年11月在纽约举行颁奖典礼及晚宴。入选国家图书奖的原则是上一年度美国籍作者在美国出版的作品,设立4大类别,每个类别经专家评议后在9月份公布入选名单(共40名)、10月份公布最终入选名单,每个类别5名(共20名),11月在颁奖仪式上公布大奖获得者名单。最终入选名单者获得1000美元奖金及奖章、大奖获得者获得1万美元奖金及奖杯。除了4个大奖外,国家图书奖评委会每年还对做出重大贡献的人士颁发美国文学杰出贡献奖(Medal of Distinguished Contribution to American Letters)以及卓越服务奖(Literarian Award for Outstanding Service),以表彰这些人士在美国文学领域的终身成就。该列表为自1988年开始的获奖者名单。","text2":"美国国家图书奖的评奖宗旨是什么?","label":1} {"text1":",为香港1978年邵氏公司出品,张彻执导的一部武打电影。故事叙述天道庄庄主杜天道(陈观泰饰)因妻儿遭仇家天南三虎所害,自而性情大变,横霸乡里。其子杜常为天南三虎斩去双臂,使父子二人心中极不平衡,先后挖盲货郎陈顺(郭追饰)双眼、毒哑打聋铁匠韦打铁(罗莽饰)及断去青年胡阿贵(孙建饰)双腿,更重创为三人打抱不平之侠士王翼的脑部,使其疯颠失常。三人据王翼所携书信寻至王翼之师李正鹰(井淼饰)处,在听闻三人遭遇后,李正鹰允收三人为徒。加强陈顺之听力以补眼盲之憾;训练韦打铁之眼力以辅失聪不便;并替胡阿贵装上铁脚,三人身残心不残,在李正鹰教导下练就一身武艺,于是连同王翼一齐前往天道庄为民除害。在接连打倒杜天道师弟居高峰(杨熊饰)、天道庄万总管(王龙威饰)以及众武师庄众后,合四人之力,终将杜天道父子击败,除去镇上一大害。","text2":"电影残缺导演是谁?","label":1} {"text1":"山景巴士总站(英文:Shan King Bus Terminus)是香港新界屯门区一个巴士总站,位于菠萝山以东山脚下旺贤街,在山景邨旁,邻近轻铁石排站,现时有3条巴士路线以此处为总站。虽然城巴B3A线以「山景」为总站,但实际上并非停靠此站,而是在附近的山景邨景乐楼对外的一段屋邨通道。山景邨第1期于1983年起入伙,当时山景邨居民前往市区,需步行到石排头路搭乘66或66M。九巴于1985年1月31日开办新路线57M,当时山景邨总站是设于石排头路景丽楼外。当局预计山景邨第2及第3期于1986年8月入伙后,景丽楼外的总站将会于3至4年内不敷应用,于是在1985年成立一个小组研究兴建永久巴士总站的可行性,并于同年11月同意在2B路(即旺贤街)一幅土地兴建总站。新总站于1986年11月1日正式划为专营巴士专用,名为山景邨巴士总站(Shan King Estate Bus Terminus),九巴57M线却要到1987年2月19日才迁进山景总站。由九龙巴士营运的一条路线,提供屯门山景及屯门工业区往来荃湾、葵涌及荔景的巴士服务,于1985年1月31日起投入服务。于2011年9月19日投入服务,途经大兴邨、良景邨、田景邨、建生邨、屯门公路、龙翔道、黄大仙、新蒲岗、彩虹邨、九龙湾、牛头角及观塘市中心,为九龙巴士258D线的特别班次,只于平日上午繁忙时间开出两班(0710、0730)。提供屯门山景、屯门工业区、屯门市中心、安定及友爱往来港岛中区及湾仔的巴士服务,于1997年10月3日起投入服务。","text2":"山景邨第1期居民怎么到达市区?","label":1} {"text1":"汉生病疗养院是用来隔离检疫汉生病(痲疯病)患者的场所。中世纪时期,欧洲和印度的痲疯病疗养院的数量增加,这些痲疯病疗养院多兴建于修道院区附近。从历史上看来,人们非常恐惧痲疯病,因为患者的身体外观会有明显的伤残,且当时无治疗方法,被视为是一种不治之症。由基督教管理的疗养院通常被称为「Lazrus之屋」,是取自圣经中的一位痲疯患者拉撒路(Lazarus)之名。为了确保病患隔离,汉生病患的聚落常位于小岛上,或是偏远地区。有些疗养院有自己的交易货币,那是因为当时相信如果流通痲疯病人曾持有的金钱,会造成疾病传染。痲疯病也受到「奢侈禁止令」的管制,若痲疯病人要离开疗养院到其他地区,必须要佩戴摇铃以提醒一般人避开。1982年发展出汉生病的有效治疗方法,同时也发现汉生病为低度传染的疾病,约95%的人口有自然免疫,此后世界各地的汉生病疗养院开始慢慢减少。有些治疗方法尚不普及的地方,或是对汉生病仍有「不洁之病」刻板印象的地方,仍保有疗养院。世界上知名仍保存的汉生病疗养院位置列表如下:2001年日本公开对公立的汉生病疗养院进行司法审查,从而确定日本政府虐待病人,日本的地方法院下令政府必须要对之前的病患进行补偿。","text2":"痲疯病疗养院多兴建于什么地方附近?","label":1} {"text1":"旋风号运载火箭()苏联研制的几种非常不同的运载火箭的统称。被称为“旋风号”的火箭包括以下几种,请分别参见其主题条目:美国国防部对旋风-2的代号是“SL-11”,对旋风-3的代号是“SL-14”;美国国会的谢尔顿命名法称旋风-2A为“F-1-m”、旋风-2为“F-1-r”,称旋风-3为“F-2”。旋风-1因为计划取消而没有被美国方面赋予代号。在这些火箭中,旋风-1是在R-16洲际导弹(北约代号SS-7“鞍工”)的基础上设计的,后来研制计划被取消。旋风-2及旋风-3与旋风-1没有任何关系,它们是从R-36洲际导弹(北约代号SS-9“悬崖”)的基础上发展而来的。R-16和R-36都是苏联著名导弹专家扬格利领导设计的洲际弹道导弹。","text2":"旋风号运载火箭是哪个国家研制的运载火箭的统称?","label":1} {"text1":"澳门瑞吉金沙城中心酒店(St. Regis Macao, Cotai Central),位于路氹城塡海区金光大道地段,是路氹金光大道中一个五星级酒店发展项目.毗连澳门喜来登酒店、澳门金沙城中心假日酒店以及澳门金沙城中心康莱德酒店。酒店于2012年第四季开始动工,提供约400间客房及套房,附设有Iridium Spa水疗中心、The St.Regis Athletic Club健身中心、Outdoor Swimming Pool 室外游泳池及两间餐厅,未来将会增设酒店式公寓服务。酒店在2015年12月17日开幕。邻近金沙城中心的Conrad,Holiday Inn和Shereton酒店.对面就是Four Seasen和威尼斯人.澳门瑞吉酒店官方网站<\/div>","text2":"酒店从哪一年开始动工的?","label":1} {"text1":"丽纹龙蜥(学名:)为飞蜥科龙蜥属的一种。分布于中国大陆的河南、湖北、湖南、四川、贵州、云南、陕西、甘肃、西藏等地,主要生活于山区、常见于灌丛杂草间以及公路旁岩石上或碎石间。其生存的海拔范围约为380至2520公尺。该物种的模式产地在湖北宜昌附近。体长 9~25 公分,最大可达 40 公分,雄性体型较大。体侧各有一条由菱斑连贯(雌性连贯不明显)形成的草绿色(有些个体较偏黄绿色)纵带(雄性较鲜艳),雄性背部颜色呈橄榄色(雌性呈浅褐色且有深色斑块),头顶有许多草绿色的粗斑块(雌性呈黄绿色)。居住于潮湿的温带丛林,日行性,常在灌木丛间或岩石上活动,性格活跃,领域性强。多以小型无脊椎动物为食。养殖业者常称丽纹龙蜥为「丽纹」。丽纹龙蜥对环境的适应性强,适合饲养,注意事项如下:","text2":"丽纹龙蜥属于哪一类动物?","label":1} {"text1":"胡陈乡是浙江省宁海县下辖的一个乡,地处县境东部山区,西临力洋镇,南接长街镇,东北与象山县接壤。该乡由东仓、胡陈二乡合并而成,土地面积96.93平方公里,其中山林占67平方公里,耕地占11.3平方公里。2016年末人口22591人,共8038户,年人均收入约3500元。乡人民政府驻地为谐和路8号。乡域西南靠胡陈港,东、北、西三面均为沿海丘陵,西部有部分平原。中堡溪自茅芦岗发源,自西南方向注入胡陈港,沿溪亦分布有部分平原。北部大丹山是境内最高峰,也是宁海县境内较高的山峰。该乡辖有1个社区和18个村委会。特产洋芋、杨梅及水蜜桃。省道宁象线自西向东穿越乡境,县道胡陈–长街公路和门棚–张韩公路在乡境内与宁象线交会。戊己桥位于乡内西南部胡陈港尾的大麦塘。桥梁俗称“四十八洞桥”,建于清道光年间,为浙江省文物保护单位。上宅村为唐吏部尚书叶裕和南宋右丞相叶梦鼎故里,村口有为纪念叶梦鼎衣锦还乡的“归锦桥”,亦有鱼池、假山遗址。小学:胡陈乡中心小学初中:胡陈中学","text2":"胡陈乡境内最高峰是什么?","label":1} {"text1":"鉴湖原名镜湖,相传黄帝铸镜于此而得名,又称长湖、镜湖、庆湖,位于中国浙江省绍兴市西南1.5公里,古为大型农田水利工程之一,现为浙江名湖之一。东汉顺帝永和五年(140年),会稽太守马臻将山阴和会稽两地来水汇集成湖,是为古鉴湖,总面积约200平方公里,有“鉴湖八百里”之称。唐朝中叶之后,古鉴湖逐渐湮废;北宋中叶后大兴围垦,至南宋初已大部分成为耕地。今惟城西南尚有一段较宽的河道称为鉴湖,此外残留几个小湖。鉴湖面积约30.44平方公里,其主体部分狭长,东起亭山,西至湖塘,尽纳南山三十六源之水潴而成湖,长22.5公里,东西狭长形,形如一条宽窄相间的河道,镶嵌在宁绍平原之上。鉴湖一带是典型的江南水乡风光。王羲之有诗:“山阴道上行,如在镜中游”,描述了湖上桥堤相连,渔舟时现,青山隐隐,绿水迢迢这种景象。鉴湖水质极佳,驰名中外的绍兴黄酒就用鉴湖水酿制。湖滨有马臻墓、陆游故里、三山、快阁遗址等古迹。","text2":"鉴湖原名什么?","label":1} {"text1":"周宜辰,彰化县秀水乡人,台湾田径运动员,身高186公分,擅长项目标枪,曾经为台湾标枪(76.84公尺)全国纪录保持人。记录于2010年10月15日苗栗全国田径赛中由桃园农工郑兆村78.69米所破.周宜辰自1998年起接受标枪训练,就读于国立台湾师范大学(师大)运动竞技学系时接受蔡于儒教练指导,并于2005年在嘉义大专运动会第一次破全国纪录(76.18公尺),并于2006年于板桥台湾亚运决选赛再次破自己的纪录(76.84公尺)。2007-2008年间与张铭煌以及林家莹一起于德国柏林接受训练。周宜辰在柏林当地是由德国教练Winfried Heinicke指导。2008年底因运动伤害停止训练。周宜辰在2010年毕业于国立台湾师范大学运动竞技研究所硕士班。于台中大甲高中实习结束后,由台湾师范大学运动竞技学系聘为技术讲师,为师大田径队助理教练,亦在师大开设一般体育课程。","text2":"周宜辰的(76.84公尺)全国纪录最终被谁打破?","label":1} {"text1":"铸客铜鼎,又名“铸客大铜鼎”、“楚大鼎”或“大铸客鼎”。为战国时期楚国的青铜炊器,1933年出土于安徽省寿县朱家集(今属长丰县)李三孤堆楚王墓。是现存西周以来最重的鼎,位列64件禁止出国(境)展览文物之一,现藏安徽省博物馆。1933年,安徽省寿县朱家集的朱鸿初等纠集100多人盗掘李三孤堆(或称离散孤堆)的楚墓,获得大量文物,其中部分器物被安徽省政府收缴,保存在安庆安徽省立图书馆保存整理。1950年代初,转至合肥安徽省博物馆筹备处(今安徽省博物馆的前身)收藏。铸客铜鼎即在其中。铜鼎通高113厘米,口径87厘米,耳高36.5厘米,腹围290厘米,深52厘米,足高67厘米,重达400千克左右。形制为圆口方唇,鼓腹圆底,三蹄足。颈侧附双耳,耳上部略外展。鼎腹饰一周突起圆箍,双耳及颈部外壁饰有模印菱形几何纹,足根部饰有浮雕旋涡纹。鼎口平沿刻12字铭文:其开头即“铸客”二字,故依惯例得名“铸客铜鼎”;足和腹下刻有“安邦”二字吉语。","text2":"铸客铜鼎是在什么地方出土的?","label":1} {"text1":"李家昕(英文名:,)是新晋的体育记者,李家昕于2003年进入香港浸会大学修读本科课程,第一年修读社会科学学士的中国研究课程, 翌年转糸至体育系,修读体育及康乐管理文学士课程, 并于2007年毕业。毕业后先后投身香港有线电视及无线电视。 她于2008年北京奥运前获委派无线电视翡翠台的「电波圣火令」节目主持之一,走遍世界追纵圣火的传递,节目并于黄金时间于无线电视翡翠台播出,令她走红,广为市民所认识。 于奥运期间,她被派往青岛采访风帆赛事。2008北京奥运完结后,李家昕留任无线电视当体育主持,主持包括体育世界等体育节目。2009年,李家昕重返香港有线电视继续工作。幼稚园: 翠华幼稚园小学: 浸信会吕明才小学 (1990-1996)中学: 浸信会吕明才中学 (1996-2003)大学: 香港浸会大学 (2003-2007)保龄球, 曾晋身香港代表队羽毛球, 曾当选香港浸会大学校队香港电台 (2006)2006年世界杯客席主持香港有线电视 (2007-2008)担任体育记者,负责采访及报道体育消息。香港无线电视 (2008)担任体育记者及节目主持,包括:电波圣火令,2008年北京奥运会,体育世界...等香港有线电视 (2009)担任体育主持。CAPDASE Marketing and PR (2012)","text2":"2007-2008年,李家昕负责报道什么类型的新闻?","label":1} {"text1":"毛利秀元(、天正7年11月7日-庆安3年闰10月3日)是日本安土桃山时代武将,江户时代大名、长府藩第一代藩主。毛利元就四男穗井田元清长男。正室是丰臣秀长女儿大善院,继室是德川家康养女净德院。曾一度成为毛利辉元的养子。育有三子八女,长男宫松丸早逝、次男光广、三男元知。1579年(天正7年)11月7日生于备中猿挂城,幼名宫松丸,五岁成为辉元养子,直到1595年毛利辉元长子毛利秀就出生后才自立。早年参与文禄·庆长之役。其中在庆长之役最为活跃,代替患病的辉元率领3万大军为右军大将。与加藤清正、黑田长政、锅岛直茂合作包围黄石山城,攻占后占领全罗道、忠清道。之后在天安接到黑田长政遭明军突袭的消息,秀元立即救援,成功击退明军(稷山之战)。在冬季秀元准备撤退返回日本时,留在蔚山的实户元续、桂孙六及加藤清正等正输送兵粮、兵器,遭到明军和朝鲜军攻击,秀元立刻返回蔚山救援,成功击退朝鲜军(蔚山城之战)。秀元在1599年与辉元独立成为大名,在周防吉敷郡、长门两国及安艺国佐伯郡获得20万石领地。关原之战时,毛利军支持西军,总大将毛利辉元留守大坂城,秀元是毛利军的前线指挥。不过由于吉川广家已成为德川家康的内应,因为对于交战相当消极,结果在关原的战斗上,吉川广家军成为束缚了毛利家安国寺惠琼、长宗我部长束正家等军势的阻碍。其后随著西军阵势瓦解,毛利军在毫发无伤之下,亦宣告撤退。当毛利军返回大坂城时,毛利秀元及立花宗茂等将曾要求毛利辉元积极战斗,但是辉元最终撤离大坂城。关原之战后,成为了长府藩6万石藩主。与德川家康养女纳入继室,参加大坂之役,晚年在江户城居住成为了三代将军家光的御咄众。曾经引诱毛利就隆脱离长州藩独立,但失败而回。1650年在江户城病逝,享年72岁。墓地在泉岳寺、功山寺及丰功神社。法号为寺功山玄誉大居士。","text2":"毛利秀元所处的时代?","label":1} {"text1":"卫笑堂(),原名延桐,字梓生,山东栖霞县人,民国时期著名武术家,八步螳螂拳第三代传人,将八步螳螂拳传至台湾。卫笑堂在年青时即喜欢武术,其父卫稽云是地方上有名的武术家,他曾学习地趟拳,后随冯环义先生学习八步螳螂拳,尽得其妙。八步螳螂拳是民国初年时,由山东烟台三位武师-「通臂拳」师陈德善、「八卦拳」师王中庆、「螳螂拳」师姜化龙,三人所创,后由姜化龙传授给冯环义,而又传至卫笑堂。他在上海时曾至精武体育会,结识吴鉴泉先生,两人决定换艺,相互学习,因此学得吴氏太极拳。1949年(民国38年),因国共内战,只身赴韩国,因在韩友人与当地韩人发生冲突,为友人助拳,在混乱恶斗中击毙三个韩国小混混,重伤数人,隔天于当地报纸得知此消息后,卫笑堂得友人帮助复回山东。于1950年(民国39年)冬辗转到台湾,在台北开设饺子馆为生,闲暇之余在植物园教授拳法,弟子逾千人,遂使此拳在台湾及美加各地流传。但是八步螳螂拳在发源地山东反而因为文化大革命而消失了。民国73年(1984年),因心脏病发作过世。《实用螳螂拳》 卫笑堂著 首著序中,首次落款日期为民国56年秋天,民国57年九月由当时华联出版社刊印。民国69年6月再版。民国85年6月五洲出版社亦发行、民国93年亦再版。《实用螳螂拳续集》卫笑堂著 卫师父序里落款日期为民国60年10月10日,印行出版日期为民国61年5月,卫笑堂兼任发行人,福美彩色公司印刷。《实用螳螂拳秘笈》卫笑堂著 卫师父序里落款日期为民国65年仲冬,印刷出版日期为民国66年3月,卫校堂自力发行,福美彩色公司印刷。民国70年7月由当时任台湾大学国术会会长经由卫笑堂首肯,勘误前版部分内文后再版刊行,也由福美彩色公司印刷张老板印制。[1][1]卫笑堂老师著作资料参考《实用八步螳螂拳秘笈》p.337页 左显富‧杨清容校释,民国100年逸文出版社出版。知名弟子有、王秋雄、苏昱彰、林钧福、温送珍、陈国钦、吴而立、张家璜、郑荣贵、荀广龙、廖宏志、廖宏达、陈义华、左显富、朗咸平、颜锦福、张光宏、林松贤、彭韩萍、王杰、田贝康广等人。卫笑堂由中国大陆至台湾的这段历程,被台湾小说家张大春写进他的小说《城邦暴力团》中。","text2":"卫笑堂是什么拳法的第三代传人?","label":1} {"text1":"格拉斯曼定律是一个关于光学理论的经验法测,他说明了人类对色彩的感知(大约)是线性的。这个定律是由格拉斯曼所发现的。若两单色光组合成一测试色光,则观测者感知到的三原色数值为两单色光分别被单独观测的三原色数值之和。换句话说,如果光束一及光束二为单色光,而formula_1与formula_2分别为观测者对光束一及光束二的感知三原色数值,当此二光束合并时,观测者感知的三原色数值为formula_3,其中:更一般的来说,格拉斯曼定律说明了任一光束的三原色座标为formula_10为该光束对波长的强度分布;formula_11,formula_12,formula_13则分别为人眼中三种锥状细胞对不同波长的反应强度。","text2":"这个定律是谁发现的?","label":1} {"text1":"台北国际牛肉面节,为台湾一项以牛肉面为主轴的新兴节庆活动,主办单位为台北市政府产业发展局台北市商业处。该活动创立于2005年,当时名为「台北牛肉面节」。由于反应热烈,自第2年即扩大邀请国际美食家及牛肉面业者参与,并更名为「台北国际牛肉面节」。2011年台北国际牛肉面节已将台湾牛肉面的英译定为「New Row Mian」(仿日本寿司、韩国泡菜之英译,以当地语言直译,突显专有特色)2011年台北国际牛肉面节的主题是「牛香一世情」,谐音「留香一世情」,寓含尝过的人难以忘记它的香味;此外,另一层意涵则指「记忆里的留香」,主要活动项目包括:等。「2008年台北国际牛肉面节」主要活动项目如下:","text2":"2005年台北国际牛肉面节当时名为什么?","label":1} {"text1":"澳门巴士1路线是一条由澳门新时代公共汽车股份有限公司经营,往返妈阁和巴波沙大马路的巴士路线。本线在福利时代,主要采用从英国购入的二手Bristol L5G及Bristol LS5G行驶。后来因为本线客量较低,所以开始使用三菱Rosa,不过可能出现其他中类型的车特见(如平治O814(B车)等等),后来因为1A缩短总站和不经提督马路至妈阁一段路,和延长总站后,客量开始上升,用车亦开始使用苏州金龙10米\/12米(K06-K125)行走,间中有猛狮13.230(M车)在2011年7月31日前,本线主要使用车款如下:在2011年7月31日后,本线主要使用车款如下:2017年9月18日后:本线在开办至2008年间由于有1A、34竞争,由于1A、34跟本路线走线相同,加上班次频密,而且本线没有特别多\"独市位\",令本线客量偏低,用车更是载客量低的三菱Rosa。后来本线延长至至关闸总站,1A路线取消提督马路至妈阁一段路线,令该段路线成为本线\"独市位\",客量开始上升,本线亦不再派出三菱Rosa,改派更多载客量的车行走,除了处理下环区的挤密人流外,有时还要帮助清理在十六浦至提督马路上不到3路线的客量,经常都会有客满情况出现,所以被称为\"「线王」\"。虽然关闸总站附近有很多路线(如5、10、18等等)选择去妈阁总站,但在以下情况下本路线较有优势。由于该线经过的站数少(比其他线更方便),而且近年来(该线途经之处)有大量市民北上或回半岛南区(司打口至河边新街),加上它的最大竞争对手─5路线晚上在水坑尾经常塞车,基本上第二个站乘客已经因客满上不了车,令乘客流量不断增加","text2":"关闸总站附近还有哪些线路去妈阁总站?","label":1} {"text1":"列举法是集合论(或者类的理论)中表示集合(或类)的一种方法。如果已知集合(或类)的每一个元素,而且元素个数“相当有限”,我们可以通过“列举”其所有元素的方法来表示这个,如:{1,2,3}、{a}、{A,B,C,D,E}等。一对花括号“{ }”是集合(或类)表示法的特征符号。如果集合(或类)的元素有“很多”甚至“无限多”以至于很难或无法将其所有元素一一列出,但其元素又具有很明显的“规律”,可以用“…”略过规律性比较明显的大量元素,如用{1,2,…,100}表示其元素为从1到100的所有自然数、{A,B,…,Z}表示所有的大写英文字母、{3,4,5,…}表示从3开始的所有自然数,等等。列举法的实质是给出了集合(或类)的外延,因此又称为外延法。如果在列举法中列出了集合(或类)的所有元素,此时称为完全列举法,否则就称为部分列举法。","text2":"如果集合中的元素有很多甚至无限多且元素具有明显的规律性可以被怎么表示?","label":1} {"text1":"2001年中华民国县市长选举,即地方自治以来的第14届县市长选举,与第5届立法委员选举同时于2001年12月1日举行投票,选出当时中华民国自由地区23个县市(即台湾省各县市及福建省之金门县与连江县,不含直辖市台北市及高雄市)的行政首长。投票作业于当日上午7时至下午4时在中华民国自由地区各地投票所进行,其中台湾省选举人数为12,789,246人,出席投票者8,499,252人,投票率为66.46%;福建省选举人数为46,221人,出席投票者25,716人,投票率是65.76%。23位县市长席次中,民主进步党及中国国民党皆取得了9个县市的执政权,亲民党则取得台东县及连江县2席。此外,金门县县长选举则由新党籍的李炷烽胜出,而无党籍人士傅学鹏及陈丽贞则分别在苗栗县县长及嘉义市市长选举中胜出。各县市之县市长选举最高票及次高票之名单(粗体者为寻求连任者):政党轮替之县市:※备注:王建煊乃中国国民党、亲民党与新党共同提名,然由于中央选举委员会无法接受一位参选人由多个政党共同推荐,故最后在选举公报将其党籍栏留白,其他相关纪录则载为新党推荐。国民党在黄派的李雅景任满后,推派同属黄派翁重钧出马参选县长,林派陈明文遂宣布不再重新参与国民党党员重登记,退出国民党。2001年,加入民进党,并宣布参选嘉义县县长。他先在民进党内民调击败已获提名的前县长何嘉荣代表民进党参选。另有原民进党的李明宪,不满陈明文获党提名,逐自行参选,陈胜三则为李雅景政府之教育局长。","text2":"陈明文什么时候加入民进党?","label":1} {"text1":"乌马托鳞片属(\"Umaltolepis\")是一类已灭绝的银杏类植物,属于乌马托鳞片科(Umaltolepidiaceae)。生存于中侏罗世到早白垩世。乌马托鳞片属的雌性繁殖器官包括一个苞片和短柄。短柄基部具有鳞片叶。苞片顶生,可能包含两个裂片,胚珠可能生在其腋部。乌马托鳞片属营养叶的叶柄不明显,叶形呈线性、长舌形,部分呈假托勒利叶型。叶片中曾发现有树脂。乌马托鳞片属生存于中侏罗世到早白垩世;在西伯利亚的晚侏罗世和早白垩世地层,中国东北和华北的早白垩世地层,以及中国河南的中侏罗世地层中均有发现。乌马托鳞片属(\"Umaltolepis\")的模式种是发现于俄罗斯西伯利亚布列亚河(Bureya)盆地上侏罗统的瓦赫拉梅耶夫乌马托鳞片(\"Umaltolepis vachrameevii\" Krassilov,1972)。","text2":"乌马托鳞片属的营养叶有什么特征?","label":1} {"text1":"全球海平面观测系统(Global Sea Level Observing System, GLOSS)是一个政府间海洋学委员会的计划,其目的是测量海平面全球长期气候变化的研究。自2004年印度洋地震后,该计划的目的已经改变为收集海平面的实时数据。该项目目前正在升级超过290台观测站,以使他们能够通过卫星向新设立的国家海啸中心传送实时数据。他们还在组装太阳能电池板,令观测站即使在恶劣天气下仍能够继续运作。全球海平面观测系统与另一计划深海评估和报告海啸(Deep-ocean Assessment and Reporting of Tsunamis, DART)两者并非相互竞争,因为大部份GLOSS传感器都设于近岸地方,而DART传感器则设于深海地方。","text2":"自2004年印度洋地震后,该计划的目的已经改变成什么","label":1} {"text1":"彭晴(,),原名彭家敏,香港电台唱片骑师,兼任now宽频电视节目主持。彭晴曾就读圣公会圣玛利亚堂莫庆尧中学,14岁时移民加拿大,并在当地完成酒店管理大学学位课程,返港后曾任维珍航空空中服务员。彭晴于1998年加盟香港电台任职唱片骑师,于2008年3月转投now宽频电视任职now新闻台节目主持,现主要主持早上新闻节目《时事全方位》,于2008年11月起改任now宽频电视节目主持\"(即主持工作不再局限于now新闻台)\",并为now宽频电视举办的各大小记者会任主持人,近例为now香港台启播记者会,且盛传为将于2009年启播、现正筹备中的now娱乐新闻台作准备及策划工作,及为该台寻找合适的主持人(该频道即后来的now观星台)。虽然,2009年7月至10月期间彭晴没有正式在香港电台主持节目,但偶然也会客串在香港电台第二台的节目,暂代部份唱片骑师。如2009年7月5日上午八时至十时伙拍罗启新,主持《玩玩星期天》,暂代贵花田及龚小玲。同日晚上九时至午夜十二时,彭晴亦暂代区瑞强,主持现场音乐节目《2000靓歌再重聚》。2009年10月4日起,彭晴又再次于香港电台第二台主持节目,与曾志豪于星期日下午二时至四时主持节目中女宅男杀很大。","text2":"彭晴现主持的节目叫做什么?","label":1} {"text1":"米象或米象鼻虫(学名:),俗称米虫、谷牛,中国北方地区称为麦甲。在台湾、日本和世界其他地方均有分布。常生活在谷物中,繁殖速度快,为谷物中主要的害虫。米象每年约有8~9个世代,一世代约20~50天,在高温下繁殖较快,32℃时一世代只需25天。成虫平均寿命达3个月。成虫用口器将谷物啮成深孔,并产卵于孔内,通常一粒谷粒产一卵,数量依谷粒大小而异。幼虫孵化后以谷粒为食,将谷粒蛀穿成弯曲隧道,并逐渐囓成中空,虫粪则排于谷粒外。幼虫在米粒中结蛹羽化为成虫后具有飞翔能力,便会离开并开始交配。米象在代谢中产生水,一生不“饮水”;喜煤气味,会钻入煤气灶出气口结网;低温时进入假死状态,恢复正常体温后恢复活动。中国农村地区常用暴晒法除米象,实际这不能杀死米虫。正确的做法是将生虫大米置于阴凉通风处,让米象慢慢爬出,再筛;亦可采用磷他锌加有机酸熏杀。较为人熟知的同属物种如下:","text2":"除米象的正确做法是什么?","label":1} {"text1":"慕容延钊(),字化龙。太原人。出身将门,其父慕容章官至襄州马步军都校。延钊少年以勇敢干练著称,后汉时为枢密使郭威的部下,广顺元年(951年),补西头供奉官,历官尚食副使、铁骑都虞侯。柴荣即位后,为殿前散指挥使都校,显德五年(958年),以功迁殿前副都指挥使、为当淮南节度使。恭帝即位,改镇宁军节度。宋太祖即位,加殿前都点检。建隆二年(961年),改为山南东道节度、西南面兵马都部署。干德元年(963年)一月,宋太祖决定向荆湘地区发动进攻,以延钊为湖南道行营前军都部署、枢密副使李处耘为都监,以讨张文表,出兵湖南,顺便向高继冲借道荆南。不久宋军灭荆南,渡江南下,向潭州(今湖南长沙)进发。这时张文表已经被湖南周保权的属下杨师璠所杀,周保权拒绝宋军入境湖南。慕容延钊分兵两路,水陆并进,陆路出澧州(今湖南澧县),水师东趋岳州(今湖南岳阳)。一月底,水师占领江陵城,二月底,大破武平军于三江口(今岳阳北),斩首四千余级;李处耘由陆路至澧州,楚军望风而遁。三月,占领岳州(今湖南岳阳),三月,攻克朗州(今湖南常德),斩杀张崇富,将其首级悬挂于闹市高杆之上。当时慕容延钊抱病在身,太祖命他“肩舆即戎事。”李处耘与慕容延钊素不和,延钊的士兵一旦有错,李处耘直接杀伐决断,造成延钊一病不起。北宋干德元年(964年)卒。","text2":"慕容延钊是哪里人?","label":1} {"text1":"《大海》是台湾歌手张雨生发行第四张国语专辑。《大海》当时相当流行化的一张商业专辑,是张雨生歌唱生涯中商业成绩上最为成功的专辑,琅琅上口、激情澎湃的旋律以及细腻抒情的词曲意境,让《大海》成为张雨生在歌迷心目中非常杰出的作品,这张专辑总体制作水准很高,而音乐风格则相对趋于流行化。除了《大海》之外的其余几首主打曲可听性也都很不错(而其粤语版本由钟镇涛主唱),诸如《我是一棵秋天的树》、《I Don't Wanna Say Goodbye》以及《爱上你的一切》等,都成为张雨生流传广泛的作品,《心底的中国》这首歌是张雨生在军中写给父亲的作品,无论词曲都相当地令人动容。","text2":"对《大海》这张专辑,大众的评价是什么?","label":1} {"text1":"时惠环球控股有限公司(简称时惠环球)是香港的一间零售公司,现时为时富投资旗下公司之一,并曾经在香港交易所上市(编号:996)。时惠的主要业务为营运香港大型家庭用品连锁店实惠家居广场。时惠环球前身为实惠集团,于1986年成立。2001年3月,实惠集团被时富投资集团收购,其后于2005年7月易名为现称。2006年8月,商人田琬善向时富集团收购时惠环球股份,锐意将该公司转型为中国大陆的零售业务,并将原有的香港业务售回网融(中国)控股。2007年10月,时惠环球的大陆业务使用了时惠环球的上市地位,并命名为东方银座()。而时惠环球的名称仍使用于时富投资所控制的零售业务。","text2":"时惠环球的前身是什么?","label":1} {"text1":"Mac OS X v10.0「Cheetah」(「Cheetah」字面解作「猎豹」)是苹果为 Mac 产品所制作的作业系统Mac OS X的第一个版本。其早期版本相当慢,功能也不齐全,仅有少量来自独立开发商的应用软件。很多评论员认为它并不是一款成熟的大众化产品,但不失为一项有潜力的开发项目。苹果从1996年就开始重写Mac操作系统,对于翘首盼望多年的业界人士而言,推出Mac OS X v10.0本身就是一项了不起的成就。除去一些较小的补丁,Mac OS X的稳定性也颇具口碑,系统内核的错误比以往大大减少了。对它的批评主要是慢,与2000年9月的Mac OS X公开测试版相比,猎豹并没有跑得更快些。苹果声称Mac OS 10.0 Cheetah 有多种新特性,包括:","text2":"苹果从哪一年就开始重写Mac操作系统?","label":1} {"text1":"新濠锋酒店(),前称皇冠酒店(),位于氹仔广东大马路,是澳门第一家以六星级酒店标准所建造的五星级酒店,于2007年5月9日试业,并于2007年5月12日举行开幕典礼。新濠锋酒店原称皇冠酒店,于开幕期间邀请了国际著名影星周润发担任酒店代言人。新濠锋酒店是全氹仔最高之建筑物,酒店大楼高约160米,共36层,总建筑面积106,000平方米,设有216间豪华贵宾房,当中包括24间豪华贵宾套房及8间总统套房。酒店设有多项休闲及娱乐设施,包括新濠锋娱乐场、摩卡角子机娱乐场、各式餐厅及水疗中心等。其中新濠锋娱乐场总面积约17,000平方米,可容纳220张赌桌及500多台角子机。酒店原址为濠景花园第4期;地皮于2005年转售给新濠国际发展开展酒店建设计划,地盘面积约56,000平方呎。项目投资额15亿澳门元。新濠锋酒店设有四间殿堂级餐厅、一间特色餐厅及一间酒吧,为客人提供餐饮服务。酒店于开幕初期(皇冠酒店时期)还有多家酒吧和咖啡厅,但最终因生意不景,所以于2008年先后结业,名单如下:酒店设有不同种类的贵宾客房及套房,包括:新濠锋酒店设有3条穿梭巴士路线:新濠锋酒店 ↔ 关闸边检大楼新濠锋酒店 ↔ 氹仔市区 ↔ 新濠影汇新濠锋酒店 ↔ 新濠天地 ↔ 新濠影汇","text2":"新濠锋酒店位于什么地方?","label":1} {"text1":"黄贵妃(),本名、出身不详。宋光宗贵妃。黄氏在淳熙末年时为宋孝宗后宫嫔妃,封为和义郡夫人,黄氏姿色美丽,温和贤慧。宋光宗为皇太子时,因身边除了太子妃李凤娘以外,别无妾侍,孝宗便将夫人赐给太子为妾,得到太子专宠。太子在1189年即位后,便封黄氏为贵妃。绍熙二年十一月辛未(1191年12月14日),传出黄贵妃怀孕的消息,引起李凤娘嫉妒,决定将她害死,光宗因郊祭的关系前往斋宫居住,李凤娘便趁机将黄贵妃杀害,宣称贵妃暴卒。第二天在光宗祭祀的现场,风雨大作,祭坛上的蜡烛都被吹熄,无法继续进行祭典。光宗返宫以后,听闻贵妃暴卒,又因为祭典上风云变色之事,从此一病不起,不视朝政,自此政事多取决于李皇-{后}-。宋光宗另有侧室张贵妃、符婕妤,她们则都被送出宫外,嫁给寻常百姓。","text2":"宋光宗的其他嫔妃,后来都怎么样了?","label":1} {"text1":"武汉长江隧道位于湖北省武汉市长江一桥和长江二桥之间,设计为左右两条单洞式隧道,隧道为单向两车道。2008年12月28日通车,为中国首条穿越长江的隧道,也是武汉首条连接武昌和汉口的河底隧道。工程包括长江两岸明挖、暗挖隧道、江中盾构隧道、匝道、引道以及设备采购和安装等项目,涉及深坑、大断面泥水平衡盾构掘进(盾构直径为11.38m)浅埋大跨软弱围岩暗挖施工等工程技术。隧道总长5049.2单线米,其中左线长2550m,右线长2499.2m。包括过长江段2610m(左右线均为1305m)。盾构隧道工程左线2550m,右线2499m,在二条盾构隧道之间设置了5条联络通道,左右线隧道各在最低点设置一座江中泵房,盾构隧道穿越富含水的全新统新近沉积粉细砂、全新统沉积粉细砂、中粗砂、砂卵石层以及志留系砂岩泥岩不同的地层。该隧道由铁道第四勘察设计院设计,中铁隧道集团有限公司联合体施工。2004年11月28日,隧道正式动工建造。隧道于2008年4月19日双线贯通,并进入路面铺设阶段,2008年12月28日通车,该隧道是长江上第一条过江隧道。","text2":"二条盾构隧道之间设置了几条联络通道?","label":1} {"text1":"韩燕(),中国新疆人。学生时代在国内担任模特儿及车展女郎。后来就读海南大学油画系,并于当地参加过多项省内选美比赛,曾数度获奖。早期活跃于网路世界,以「雅漾韩」为网名,在国内网站上颇具知名度。2006年,韩燕当选海南博鳌形象小姐。同年参加由香港亚洲电视举办的亚洲小姐竞选,在中国内地赛事中以22号参选,夺得「最佳口才奖」、「最上镜小姐」与及赛区亚军。后来继续突围,顺利打入香港区总决赛,同届对手有陈瑀涵、舒夫真高、曾敏等。韩燕最终以4号佳丽的身份成功取得赛事季军及「亚洲亲善大使」的名衔,并签约成为亚洲电视合约艺员。其后,韩燕数次以泳装姿态为亚视主持综艺旅游节目,包括《碧水湾温泉养生之旅》、《开心清远温泉之旅》、《星级顺德自由行》、《番禺水上乐园斗多fun》、《阳光热浪嘉年华》等,均以其出众身材成为新闻焦点。2007年,韩燕参选北京奥运火炬手。经北京奥组委审核,于10月24日正式当选为2008年北京奥运国内火炬手之一,负责在5月4日至5月6日于海南三亚传递圣火,该站亦是北京奥运中国境内圣火传递路线的第一站。2010年韩燕回归亚洲电视。","text2":"韩燕就读于哪所大学?","label":1} {"text1":"毕耀明(Brian Butt Yiu-ming,)为现任北京泛美国际航空学校校长,前政府飞行服务队总监,前香港航空青年团总监。于1978年5月加入香港警务处,任职见习督察,1986年4月转往皇家香港辅助空军任职机师。1989年11月晋升为高级机师,1993年1月晋升为总机师,1996年8月出任政府飞行服务队总监,至2007年底提出请辞,共服务政府29年。 总机师陈志培于2008年3月27日起接任政府飞行服务队总监一职。毕耀明1954年出生于香港,在香港接受中小学教育,1977年在加拿大取得科学学士学位,1995年取得香港大学工商管理硕士学位,已婚,育有2名儿子。毕耀明大学毕业后返回香港,投考国泰航空机师失败,其后加入香港警务处,同时也是皇家香港辅助空军的志愿飞行员。他曾被派往商业罪案调查科,是首批受训的伪钞鉴别专家。1986年离开警队(当时职级为总督察),加入皇家香港辅助空军担任全职直升机机师,是部队唯一的华人机师,拥有定翼机及直升机驾驶执照,具直升机飞行教练资格。其后晋升高级机师及总机师,后于1996年8月1日出任政府飞行服务队总监,年仅42岁,成为最年轻的华人总监。2003年8月26日晚上,一辆政府飞行服务队海豚直升机由总部起飞,前往长洲接载伤者途中,于大屿山伯公坳附近的山头坠毁,两名机员罹难。这次意外,是政府飞行服务队于1993年成立以来,首次有队员坠机导致殉职的事故。民航处的调查报告指直升机师疏忽,并提出多项改善建议<\/ref> <\/ref> ,政府飞行服务队全数接纳。","text2":"毕耀明离开警队后去了哪里工作?","label":1} {"text1":"所罗门行动()是指1991年以色列政府把埃塞俄比亚犹太人空运回以色列的行动。早在海尔·马里亚姆·门格斯图成为总统之前,1984至1985年的摩西行动和约书亚行动中已有约8000名埃塞俄比亚犹太人送回以色列。在1990年,以色列政府和军方已注意到门格斯图政权不稳,1991年5月,衣索比亚的门格斯图政权倒台,厄利垂亚和提格雷族的反政府武装部队取得政权,以色列政府和犹太组织决定把在衣索比亚的犹太人空运回以色列,以助他们躲避内乱及重返以色列。5月24日行动正式开始,该行动在36小时内,利用以色列空军的C-130和以色列航空的波音747货机,把14324名犹太籍衣索比亚人运回以色列。5月24日,一架以色列航空的波音747货机则一次运载1122人(已登录的只有1087人,不少未登录的都是在母亲怀内的小童),其中有5人是在该航班上出生的。然而该行动并没有把所有犹太籍衣索比亚空运回以色列,。","text2":"以色列政府和犹太组织为什么决定把在衣索比亚的犹太人空运回以色列?","label":1} {"text1":"1918年教育法(\"Education Act 1918\"),又称为费舍教育法(\"Fisher Act\"),是英国国会通过的一项法案。草拟者为文教大臣赫伯特·费舍。1918年教育法将从学校毕业的年龄提高到14岁,并计划扩展高等教育。1918年教育法的另一特色是提供辅助设施(幼儿学校、体格检查和特殊需要儿童中心等)到1920年代,年幼儿童的教育日益成为政治家与教育家关注的问题。由于公众争论的升级,当时的政府委托哈多爵士负责进行教育调查,哈多委员会在1926年、1931年和1933年,发表了3个非常重要的报告。这些报告带来初等教育的重大改变。例如,导致了5-7岁儿童的幼儿学校和7-11岁的小学被分开实施。报告还建议班级规模不超过30人。这些建议标志进步主义教育思想胜过了传统教育思想,更多为政策制定者和教师们所喜爱。","text2":"1918年教育法又被称为什么?","label":1} {"text1":"乌巢之战是曹操与袁绍两军相争其中一部分,是官渡之战中最重要的一个环节。事件导致袁绍大军无粮食而溃败,曹操实力再为增强。汉献帝建安五年(200年),曹操得到天子,迁都许昌,于政治上取得优势,并平定多个势力,遂与河北大族袁绍于官渡(今河南中牟东北)决斗。曹操采用奇袭、声东击西之计、诱敌之计、霹雳车,并派出大将关羽,所以多次击败袁军。虽然如此,但袁绍军兵多粮足,曹操粮少,曹操不能支持下去,正巧袁绍手下谋士许攸来降,许攸献上奇袭一计,曹操言听计从,将袁军粮草全部烧毁,情况自此改变。曹操已经杀死吕布、平定袁术、降服张绣、夺回徐州、并击走刘备,收容关羽。而袁绍则将势力强大的公孙瓒消灭,将要与曹操决斗,争夺天下。200年,袁绍派陈琳写下檄文,指曹操祖父乃宦官,父亲是乞丐,靠金钱做得高官,曹操则把弄朝中大权,要将曹操讨伐。袁绍之后先后派颜良、文丑两员大将分别到白马、延津,皆被击败,两将双双被杀。许攸是袁绍谋士,因为贪财,所以被袁绍赶走,之后投奔曹操。曹操闻讯,说:「子远来了,大事可成!」再请许攸入座相谈。许攸问到:「贵军军粮可以用多久?」曹操答曰:「尚可支持一年。」许攸再说:「哪有这么多?说真的吧!」曹操再答:「还可以支持半年。」许攸说:「难道你不想打败袁绍吗?为何不说真话?」曹操说:「跟你开玩笑而已,其实军粮只剩此月的份量。」许攸献计说:「今孟德孤军独守,既无援军,亦无粮食,此乃危急存亡。现在袁军有粮食存于乌巢,虽然有士兵,但无防备,只要派轻兵急袭乌巢,烧其粮草,不过三天,袁军自己败亡!」曹操听计后大喜,选精兵假扮袁军,马含衔枚,士兵拿著柴草向乌巢出发,遇上其他人问话时,皆回答:「袁绍怕曹操奇袭,派我们把守。」袁军不疑有诈,放其通行。到达乌巢后,曹军放火,营中大乱,大破袁军,粮草尽烧,斩领将眭元进、韩莒子、吕威璜、赵叡等首级,割下淳于琼的鼻,杀士卒千余人,将他们所有鼻割下,连同牛、马的舌头一同送往袁军,袁军将士大惊。曹操问败将淳于琼:「你今天弄成这样,是甚么缘故?」淳于琼答:「胜负乃天所控制的,问我干甚么?」曹操欲想留下淳于琼性命,许攸指,淳于琼日后看到自己的样子必然痛恨曹操,曹操斩淳于琼。","text2":"曹操后来为什么支持不下去了?","label":1} {"text1":"飞黄腾达5为美国真人秀《飞黄腾达》的第五辑,Donald Trump为执行制作人和主持人。学徒的应聘者可以在线申请(与既往的各辑相同) 。该辑于2006年2月27日开始在美国NBC频道播出。在本辑的终集,Sean Yazbeck 是最后的状元,成为Donald Trump的新学徒。 Lee Bienstock作为榜眼,被证明是非常有实力的,在本辑结束的数月后也被招聘。在以往的各辑中,本辑是第一个未能进入Nielsen排名前50的,其观众少于一千万。本辑排名51名,观众973万。飞黄腾达5是由George H. Ross和Carolyn Kepcher作为会议室评议员的最后一辑,也是概念上在完全纽约市录制的最后一辑。随后的名人版飞黄腾达回到过曼哈顿,评议员主要是Donald Trump的子女。参赛者不仅包括美国人,还包括了来自阿塞拜疆、英国和加拿大的应聘者。同以往一样,Trump亲自选定的应聘者。但他第一次直接指定了起始的项目经理,而没有按照性别或者教育背景确定分组。随后由经理选择队员建队。与以往不同,在本辑中获胜队的经理没有解聘豁免权,因而所有人都必须非常努力,以避免被裁掉。下表为本季度的参赛者,以其原队伍作出排列。队伍最初以当初的分组方法:一组队伍名为淘金者(Gold Rush),另一组的队伍名为协力者(Synergy)。","text2":"名人版飞黄腾达的参赛者中除美国人外还包括哪些国家的应聘者?","label":1} {"text1":"深深爱过你是歌手薛之谦的第三张个人创作流行音乐专辑,于2008年11月26日正式发行。专辑有9首歌,新歌7首,另两首为:深深爱过你(现代版)和改编自上一张专辑你过得好吗的《梦开始的原点》。其中有两首风格迥异的韩式舞曲星河之役和Let You Go,因销量喜人唱片公司,推出了附加了“谦年传说”演唱会视频的改版专辑,预计于2009年4月9日发行。深深爱过你两个版本被拍摄为不同版本的两支MV,传说也被拍摄为MV。其余的两支MV:星河之役和我们的世界,均为演唱会LIVE版MV。1.传说 04:33(第一波燃情主打)2.深深爱过你(前世)04:38(第二波个人PK单曲)3.Let You Go 02:42(第三波热舞主打)4.给我的爱人 04:455.我们的世界 04:146.流星的眼泪 04:427.星河之役 03:438.深深爱过你(今生)04:17(第二波个人PK单曲)9.梦开始的原点 04:52","text2":"专辑其中有两首风格迥异的韩式舞曲,是哪两首?","label":1} {"text1":"大眼魣(学名:),又称大眼金梭鱼,为辐鳍鱼纲鲈形目鲭亚目金梭鱼科的其中一种,俗名吹鱼、细鳞。本鱼分布于印度太平洋区,包括东非、红海、塞席尔群岛、阿曼、马尔地夫、伊朗、巴基斯坦、印度、日本、台湾、中国、越南、柬埔寨、菲律宾、印尼、澳洲、新几内亚、帛琉、库克群岛、马绍尔群岛、马里亚纳群岛、新喀里多尼亚、纽埃、索罗门群岛、东加、吐瓦鲁、斐济、关岛、萨摩亚群岛、夏威夷群岛、法属波里尼西亚等海域。该物种的模式产地在Society群岛。水深0至300公尺。本鱼体延长呈鱼雷状,横切面几近圆柱形,侧线在第一背鳍前略向上弯曲,第一背鳍起点在胸鳍末端正上方,在腹鳍起点之后;鳃盖具2弱平棘;上颔骨末端达或接近眼前缘,下颔骨突出。体浅白,背部蓝绿或棕色。体侧不具角形纹,眼大,鳃弓呈刚毛状及胸鳍下具一大黑斑。背鳍硬棘6枚;背鳍软条9枚;臀鳍硬棘2枚;臀鳍软条9枚,体长可达75公分。本鱼白天常成群在礁湖内礁块的上面水层或在礁坡外徘徊,属肉食性,以鱼类为食。为食用鱼,适合各种烹饪方式。","text2":"大眼魣的模式产地是哪里?","label":1} {"text1":"本列表是无线电视剧集收视列表的子列表,列出2000年代香港无线电视剧集的收视统计,排名根据全剧平均收视及观众人数顺序列出。跨年度剧集于播放大结局的年份列出。本列表只会列出无线电视于黄金时段首播的自制剧集之收视,外购剧集、重播剧集、于非黄金时段首播的海外首发剧集及香港电台剧集均不会列出。以下列出2000年代十大最高电视收视点的香港无线电视剧集:以下为2000年香港无线电视剧集于翡翠台之收视列表:以下为2001年香港无线电视剧集于翡翠台之收视列表:以下为2002年香港无线电视剧集于翡翠台之收视列表:以下为2003年香港无线电视剧集于翡翠台之收视列表:以下为2004年香港无线电视剧集于翡翠台之收视列表:以下为2005年香港无线电视剧集于翡翠台之收视列表:以下为2006年香港无线电视剧集于翡翠台之收视列表:以下为2007年香港无线电视剧集于翡翠台之收视列表:以下为2008年香港无线电视剧集于翡翠台之收视列表:以下为2009年香港无线电视剧集于翡翠台及高清翡翠台之收视列表:","text2":"无线电视剧集收视列表 (2000年代)的排名是根据什么列出的?","label":1} {"text1":"纽西兰鹌鹑(学名:\"\")是自1875年就已灭绝的鹌鹑。雄鹑与雌鹑相似,但雌鹑较浅色。纽西兰鹌鹑最初是由约瑟夫·班克斯(Sir Joseph Banks)在跟随詹姆斯·库克(James Cook)第一次旅程中描述的。第一个标本是由Jean René Constant Quoy及Joseph Paul Gaimard在跟随迪蒙·迪尔维尔(Dumont D'Urville)的旅程中,于1827年所采集的。纽西兰鹌鹑有时被认为与澳洲鹌鹑是同一物种。梅西大学的研究发现在蒂利蒂利马唐伊岛的鹌鹑并非纽西兰鹌鹑的余种,更不是纽西兰鹌鹑及褐鹌鹑的混种。研究亦发现纽西兰鹌鹑与澳洲鹌鹑有亲密的系统发生学关系,但明显是独立的物种。","text2":"梅西大学的研究发现了什么?","label":1} {"text1":"花煎()是一种朝鲜半岛传统的甜煎米饼,上面可食用的时花制成。韩语固有词又称为、、。在不少传统节日如上巳节、重阳节等都会食用。从高丽王朝起,朝鲜人在一些节日里有玩「花煎游戏」(,)的传统习俗,并会食用花煎。每逢上巳节,妇女就会带著叫燔铁()的一种厚煎板去郊游踏青,她们会采摘杜鹃花或其他盛开的花来做花煎,当中以杜鹃花煎(固有词为)最具代表性。花煎常会与加入同种花瓣的花菜(,一种把水果加入蜜糖水或五味子汁里制成的朝鲜传统甜品)一同食用,例如以杜鹃花煎与杜鹃花菜同吃。到了重阳节,人们也会举行花煎游戏,这时食用的花煎会以菊花瓣制作,亦即菊花煎(),配以菊花酒()或柚子花菜()。除了杜鹃花煎和菊花煎外,任何可吃的时令花卉都可用作制造花煎,例如春季常见以沙梨花制成的梨花煎()、樱花煎(韩语固有词:)、堇菜花煎(固有词:)。初夏时则多会食用蔷薇花煎()。鸡冠花煎(韩语固有词:)则多在秋季食用。当没有适合制作花煎的花时,会以水芹、艾草、石耳、红枣等做成花形装饰放在煎饼面团上。","text2":"花煎是什么?","label":1} {"text1":"《甜蜜小天使》()是日本漫画家赤冢不二夫的漫画作品,曾经三度由东映动画动画化。本作的漫画版初期投搞于集英社旗下的漫画杂志《Ribon》,后来转到讲谈社继续连载,亦有发行单行本与绘本。动画版则交由东映动画制作,1969年1月起开始播出,这是东映动画继《魔法使莎莉》(于1993年时成香港配音员梁少霞成名作)以后的另一部魔法少女动画作品。往后陆续制作第二作与第三作动画,但三作动画的制作与播出间隔了很长一段时间。原先「」的全名设定为「镜厚子」(),动画化后改为「加贺美厚子」,日语读音相同,但三作动画的日文表示不尽相同。厚子的形象亦曾在赤冢不二夫相关作品当中出现。由于当时华人圈对于日本动画的资讯较为缺乏,因此目前为止,本作没有正式或统一的中文译名(于香港播出时,将本作称为《-{小魔镜}-》)。中华电视公司(华视)于1991年5月6日至1992年6月22日播出时,将本作命名为《-{甜蜜小天使}-》,角色亦重命名,中文主题曲由汪石泉作词作曲、主唱不详、华视大乐队伴奏。本作香港版本主题曲由刘锡明主唱。2012年8月,与晨间节目《SUKKIRI!!》合作推出约1分钟的短篇Flash动画,为之后上映的真人电影版做宣传;厚子的形象也从小学生变成上班族,配音由平野绫担任。主角厚子是一名少女,透过镜子就能变身成任何模样,初期是设定为一面大镜子,只要对著镜子念出咒语就能变身,动画中则改为可随身携带,类似贝壳型状的变身盒。故事内容主要是围绕在主角与周遭的人们所发生的一切事件,透过变身能力解决他人的困难。动画版的故事设定上,三作各有不同,除了画风外,厚子的衣装、变身盒造型,与父母职业也有所不同。","text2":"《甜蜜小天使》是谁的漫画作品?","label":1} {"text1":"北疆风铃草()为桔梗科风铃草属多年生草本植物。茎直立,高0.2-1.3米。基生叶具长柄,卵状披针形至卵形,6-15×2-7厘米,基部心形,顶端急尖;茎生叶下部的具长柄,上部的无柄,椭圆形、长卵形至卵状披针形, 4-10×0.7-4厘米,基部截形、圆形或楔形,顶端急尖或渐尖;全部叶边缘有尖锯齿。花数朵集成头状花序,无总梗,亦无花梗,在茎顶集成复头状花序,有时在茎中上部叶腋间也有单生的头状花序;总苞片卵圆状三角形,每朵花下还有一枚大小不等的苞片,在头状花序中间的花先开,其苞片也最小。花萼裂片钻形至线状三角形,7-13×2-3毫米;花冠紫色、蓝紫色或蓝色(稀白色),管状钟形,长1.5-2.5厘米,分裂至中部。蒴果近球形至倒卵状圆锥形,长约4.5毫米。种子长矩圆状,扁,长1-1.5毫米。花期7-9月,果期9-10月。2n=30,34,90。生于海拔2600米以下的草地、亚高山草甸、灌丛、山坡草地。产黑龙江、吉林、辽宁、内蒙古、新疆。日本九州、朝鲜半岛、蒙古、俄罗斯、哈萨克斯坦、西亚、欧洲也有分布。广泛栽培供观赏,在北美归化。接受亚种15个, 中国有3个。聚花风铃草()和名为,韩名为(紫花棒)。","text2":"北疆风铃草的花萼裂片有什么特点?","label":1} {"text1":"锦丰园(Kam Fung Garden)是香港新界荃湾荃景围的一个私人屋苑,发展商为南丰发展,管理公司为民亮物业管理。锦丰园共有两座住宅,在1989年入伙,屋苑本身设有公园、游泳池、球场、停车场等基本设施,并仅限住户使用。锦丰园在荃景围各屋苑中,可谓最能够自成一角,因为屋苑面前是沙咀道天桥,而另一边则是山头,跟荃湾中心尚有一段距离。除此之外,屋苑严禁闲杂人等进入,住户需要输入密码才可进入屋苑的公园或住宅,而访客亦需登记,因此保安方面都是严密的。另一方面,锦丰园位处近青山公路的位置,而且屋苑设有私家通道连接青山公路「荃景围天桥」之巴士站,相比起荃威花园、千里台等住宅较多选择,而不需只局限使用五条荃景围巴士路线。而且,该巴士站亦设有小巴95A连接港铁荃湾站,令住户不用前往荃湾中心乘搭小巴95或95M。然而,相比起荃湾中心,锦丰园为纯住宅区,不如荃湾中心商场般方便,本身没有商场,购物须前往对面马路荃德花园的茶餐厅、便利店、士多等,相对比较宁静及不会有闲杂人等出入。","text2":"锦丰园为什么不如荃湾中心商场方便?","label":1} {"text1":"福井舞()是出身于京都府京都市的日本女创作歌手,23岁时出道,血型A型,所属唱片公司为J-more。2004年,与WADAGAKI、SHINO组合地下音乐队Poplar,发表了两张专辑,天照和梦死物语。在2006年时退出,2007年10月加入了Avex独立发展,2008年2月下旬自京都上东京。2010年8月20日,以单曲「爱之歌」初次亮相,同年9月同曲的原唱铃声下载数目超过50万。11月19日,作为第二张的单曲发表「Lucky」成为了初次亮相的契机的。使用小时学钢琴的琴进行作曲,据说作词是先以英语写完,之后转换成日语完成的,作词及作曲都是以「Maifukui」的名义发表。","text2":"福井舞所属哪家唱片公司?","label":1} {"text1":"印度-阿拉伯数字系统,或称印度数字系统,是一系列的十进制进位制的记数系统,起源于9世纪的印度。此系统像一种语系,当代的很多文字系统里的不同记数符号都是起源于此系统。印度-阿拉伯数字起源于印度的婆罗米数字,在中世纪时传入中东和西方。各个地区根据当地的文字系统改造了其数字字符。现在还在使用的三大分支是:印度-阿拉伯数字系统对西方中世纪的科学发展起了重要的作用。它合并了进位制和十进制系统,让数字的纪录更加简便,也简化了小数和循环小数的记载。此系统只需要13个符号就可以表示所有有理数:10个数字符号、小数点、负号和一种分数符号(分号或循环小数符号)。印度-阿拉伯数字系统也巩固了“〇”在西方世界的概念。列奥纳多斐波那契把这个系统带到欧洲,并把阿拉伯文的翻译文本带到拉丁美洲,称它为“liber abaci”。从12世纪开始,它被使用于欧洲的数学,并从十五世纪开始被广泛使用。在中国,瞿昙悉达于718年引进了印度数字与零,但中国的数学家觉得它们没有用处,因为他们已经有算筹(苏州花码的前身)。19世纪,中国和日本接受了西方的印度-阿拉伯数字系统。","text2":"印度-阿拉伯数字系统起源于几世纪?","label":1} {"text1":"光华体育会(Kwong Wah Athletic Association,简称光华),成立于1936年,是香港一支老牌足球队,曾在香港甲组足球联赛角逐,现时于香港乙组足球联赛作赛。光华是香港一间老牌足球会,于1936年,由中华电力公司文职人员组成,当中有华人及外籍人士,包括后来成为球星的侯榕生。光华成立后,随即于1936–37年加入香港丙组足球联赛,当年球队经费由中电负责。翌年,获升上乙组联赛角逐,并赢得初级银牌冠军。成立第三年,光华便首次升上香港甲组足球联赛角逐。1950年代初,光华获得许文奎等上海来港商人支持,以一班原上海青白足球队球员为主力,当时效力过光华的上海名将包括有:张金海、吴祺祥、陈明哲、罗寿福、严士鑫等。光华成绩一直只是平平,在1962–63年赢得香港足球高级银牌赛冠军,是该会在甲组唯一的重要锦标。1965–66年,光华在甲组联赛12支球队中位列包尾,宣告护级失败,降落乙组作赛。1972–73年,光华获得乙组联赛亚军,得以重返甲组作赛,并且邀得华探长颜雄入阁,招兵买马增强实力,包括著名守门员何容兴。1976–77年,光华在甲组联赛名列尾二,与另一支老牌球会九巴一同降班,自此未能再重返甲组。1977–78年,光华在香港乙组足球联赛12支球队名列第 11,连续两年都要降班,宣告降落丙组作赛。1990年夏天,因乙组联赛扩充队数,光华获得升上乙组,惟于1990–91年,光华在乙组 10 支球队中名列第 9,与第 10 名的流浪一同降回丙组作赛。2008–09年球季,光华虽然邀得前香港足球代表队前锋巴贝利助阵,但在香港丙组(甲)联赛 19 战 5 胜 1 和 13 负得 16 分,位列第 18 名。2009–10年球季,光华以 19 战 2 胜 6 和 11 负成绩,在香港丙组(甲)联赛名列第 18 位。在2010–11年球季,光华邀得前甲组球员吴圳聪加盟,加强球队后防。香港足球总会于2012–13年球季,重组丙组联赛,并复办丁组联赛。光华在上一届香港丙组(甲)联赛名列第18名,故需降落丁组联赛角逐,光华邀得前香港足球代表队前锋罗伦士及前甲组球员凌汉宗加盟,增强实力争取回升丙组,开季取得六连胜佳绩,至2012年10月14日第七轮联赛才与西贡朋友赛和 2–2,本季首度失分。","text2":"光华的成绩一直平平,在1962–63年取得什么辉煌成绩?","label":1} {"text1":"树皮是木质植物,例如树的茎和根最外面的部分。狭义的树皮包括三层:木栓、木栓形成层和栓内层,以及外部的各种死组织,广义的树皮还包括韧皮部。有的植物的树皮中含有各种生物碱、单宁、染料和香料等,可以提炼各种药材、毒品、毒药、树脂等,也可以用某些种类的树皮做软木、绳索、织布、造树皮船、绘制树皮画等,或直接用树皮作装饰。由外向内,树皮可分为外表皮、由木栓、木栓形成层和栓内层组成的周皮以及内里的韧皮部。外表皮是树木最外部的死组织,由角质化的细胞组成。周皮是韧皮部和外表皮之间的部分,包括木栓、木栓形成层和栓内层的总称,周皮形成后,表皮即脱落。木栓是树皮外层的主要成分,能隔绝水分和气体通过,对树有保护作用。木栓形成层通常只有一层或两层细胞,是分生生长木栓的组织,向外生长成木栓层,向内形成栓内层,不过在根部的木栓形成层是由中柱鞘转变的。栓内层是木栓形成层向内部分化出的一层细胞。韧皮部在木质部的树干和周皮之间,是树皮内部输送营养的部分。随着树皮逐渐生长加厚,外层组织逐渐死亡,狭义的树皮只包括木栓和外部的死组织。树皮的各个部分都有不同的用途,木栓的质地轻、富有弹性、不透水,木栓发达的树种如栓皮栎的树皮,可以用来制作瓶塞、救生圈、隔音板等,在北美洲和中国东北用桦树皮制作小舟和器物。有些树皮甚至可以食用,最有商业价值的树种是金鸡纳树和肉桂树,可以提炼药物奎宁和香料, 阿司匹林是从柳树皮中提炼的;夏栎(\"Quercus robur\")树皮是揉革用的单宁酸的主要原料;在园艺中,经常用树皮碎屑培育兰花等不能在土壤中生长的附生植物花卉。许多种类的昆虫、真菌和苔藓,附生在树皮上 。不同树种的树皮形状、色泽、瘢痕及脱落情况都不相同,树皮可以作为鉴定树木种类、年龄的重要依据。树木在树皮受伤后,可以自己生长出多余的木栓来修复。","text2":"周皮形成后,表皮如何变化?","label":1} {"text1":"墨西哥蝴蝶鱼(\"Ameca splendens\"),又称阿迈喀鳉,为辐鳍鱼纲鲤齿目幸鳉科的其中一种。本鱼以往分布在墨西哥阿美加河,但现已野外灭绝的一种鱼。牠们曾是很受欢迎的观赏鱼,但近年数量却大幅下降。牠们有可能在美国内华达州东南部再次在野外生存。 牠们的模式产地是在墨西哥哈利斯科州塔栖兰附近的塔栖兰河。墨西哥蝴蝶鱼外观吸引,而且容易饲养。雄鱼有很大及呈黑色的背鳍,尾鳍也是呈黑色,在末端有一黄纹。身体呈赭色,两侧及背部分别呈银色及褐色,而雄鱼有灿烂金属色的鳞片,雌鱼两侧却有黑点及赭色鱼鳍。雄鱼在兴奋时鱼鳍颜色会加深,两侧随心情而显示深浅色的黑纹。雄鱼与雌鱼的另一个分别在于牠们的肛门鳍,其肛门鳍前端分叉,方便交配。雄鱼较为细小,只有7至8厘米长,而雌鱼可以长达10厘米。墨西哥蝴蝶鱼适合生活在清洁及充氧的环境,水温介乎20至25℃,pH值中性,硬度在5至10之间。牠们不能忍受低pH值及过软的水,且不适合与雨林物种一同饲养。牠们优于游泳,且是群居生活,喜欢3至5条雄鱼及3至7条雌鱼一同生活。牠们不会吃植物,但亦会咬叶子。牠们可以减低藻类的生长及清理碎石,对植物的生长有帮助。一些浮动的植物如水蕨属或金鱼藻属可以替鱼苗提供保障。墨西哥蝴蝶鱼会吃冷藏、冻干、薄片或片状鱼饵。牠们喜欢吃一星期大的孔雀鱼,但亦需要吃如藻类的植物。牠们与慈鲷科都是水族箱中专吃藻类的鱼类。若藻类不足够,牠们会吃有机蔬菜,如生菜、菠菜或豌豆。饲养墨西哥蝴蝶鱼需有强烈的照明来帮助藻类的生长,最好的是日光直接照射。夏天,可以将牠们放在室外的池塘或水族箱中,但须小心鸟类、猫及其他猎食者。墨西哥蝴蝶鱼在水族箱中经常会繁殖。每6至10个星期,雌鱼就会产卵。鱼苗可以长达20毫米。鱼苗未出生前,雌鱼会用像人类脐带的肠外带状物来哺育牠们。","text2":"墨西哥蝴蝶鱼是哪个科目中的一种生物?","label":1} {"text1":"虚拟电路(,缩写为 VC),又称为虚电路、虚连接或虚通道,在分组交换的电脑网路上,交换资料的传输方式之一。它是一种预接式(connection-oriented),或线路交换式(circuit-switched)的资料传输方法,在两个终端系统(End system)间,建立一条连线,来进行资料交换。在使用虚拟电路之前,必须先在两个节点或软体应用程式间建立连线。在建立连线之后,两个节点之间,就可以进行资料串流的交换。概念来自于电路交换,其运作方式就如同在两个端点间,建立起专用的实体层线路连线一般,因此又称为虚拟连线(virtual connection)或虚拟通道(virtual channel)。在分组交换中,虚拟电路与资料包是两种主要传输方式。在通信和网络中,虚电路是由分组交换通信所提供的面向连接的通信服务。在两个节点或应用进程之间建立起一个逻辑上的连接或虚电路后,就可以在两个节点之间依次发送每一个分组,接受端收到分组的顺序必然与发送端的发送顺序一致,因此接受端无须负责在收集分组后重新进行排序。虚电路协议向高层协议隐藏了将数据分割成段,包或帧的过程。虚电路通信与电路交换类似,两者都是面向连接的,即数据按照正确的顺序发送,并且在连接建立阶段都需要额外开销。但是,电路交换提供稳定的比特率和延迟时间,而虚电路服务的比特率和延迟时间要取决一下因素:许多虚电路协议通过数据重传,包括检错纠错和自动重传请求(ARQ),提供可靠的通信服务。","text2":"虚拟电路是什么传输方式?","label":1} {"text1":"弄蛇术指东南亚地区(尤指印度)的一种街头表演艺术,表演者会向著放在竹篓里的蛇类吹奏管乐乐器,蛇就会从竹篓中探出头来,做出各种摆弄身姿的动作,但并不攻击耍蛇人,好像被催眠了一样,以取悦路人。弄蛇表演者多数以眼镜蛇或蝰蛇为其操控对象,他们本身会养育这些毒蛇,有些会把蛇的尖牙或毒囊拔除掉,甚至会暂时封著其嘴巴,以策安全。有时候,表演者也会直接接触蛇类,透过挟弄长蛇以做出更多高难度的危险动作;或者安排毒蛇与獴相斗,这些表演甚至会以动物的生死来作胜负准则。弄蛇术在在印度最为著名,另外巴基斯坦、孟加拉国、斯里兰卡、泰国、马来西亚,以至北非的埃及、摩洛哥及突尼西亚也有相关的表演技艺。弄蛇术曾经是古埃及的本土技艺之一,现在则以印度为发展中心,在东南亚、中东地区及北非地区扩展。表演者的表演场地多为街道边或市场,是颇有地方特色的街头表演,但自20世纪以后,有些地区的政府已经视弄蛇表演视为存有危险性的行为,加上在1972年印度禁止一般市民私人拥有蛇类,弄蛇表演业开始式微。在弄蛇表演中,弄蛇者会带著一个放有蛇类的篮子,透过吹奏乐器(笛)而令蛇类作出各种特殊的反应。其实蛇类是没有外耳的,尽管牠们长有内耳,但牠们并不会受到音乐的影响。有研究者指出,许多弄蛇者实际上是以双手的姿势向蛇类发出讯号,蛇类其实是根据笛子的左右摆动与及弄蛇者跺脚的震动才会作出多种扭动身体的动作。而弄蛇者的这些技巧都不会被在场的观众容易看得出来,因为他们是厉害的掩眼法高手。弄蛇者很少会直接用手捉著他们的蛇,他们的蛇亦大多数是无毒的或是被剥下牙齿的眼镜蛇。另外,有部分弄蛇者会表演眼镜蛇与獴只相斗的环节,不过这种环节并不普遍,因为不管是蛇类还是獴只都很容易会在相斗的过程中受伤,甚至死亡。现在印度政府为了保护野生蛇类,决定停止国内的街头弄蛇表演业。在一些印度地区,耍蛇业是非法的。","text2":"弄蛇术现在都是以哪里作为发展的重点?","label":1} {"text1":"瓦尔迪斯· 东布洛夫斯基斯(Valdis Dombrovskis,),出生于里加,是拉脱维亚新时代党成员,欧洲议会议员,2009年3月至2014年1月任拉脱维亚总理。于略5年 任期.他毕业于拉脱维亚大学物理与数学学院。1995年获得了里加工业大学工程方面的经济学学士学位,1996年获得拉脱维亚大学物理学硕士学位。他1995年至1996年在德国美因茨大学物理研究所做实验室助理,1997年在拉脱维亚大学固体物理学研究所做助理,1998年在美国马里兰大学电气工程系做助理研究员。从2002年以来,他一直是拉脱维亚新时代党领导成员。2002年至2004年他担任财政部长。2003-2004年担任欧洲联盟理事会观察员。作为欧洲议会议员,他是三个欧洲议会委员会的成员:预算委员会,欧洲与非洲、加勒比、太平洋联合委员会,欧洲与拉丁美洲联合委员会。他也是一个负责预算控制的经济和货币事务委员会委员会成员,也是欧盟与哈萨克斯坦、吉尔吉斯斯坦、乌兹别克斯坦、塔吉克斯坦、土库曼斯坦和蒙古议会合作委员会的成员。他作为欧洲联盟的6人欧洲议会议员监督了2007年10月举行的多哥议会选举。以戈德马尼斯为总理的政府于2009年2月10日宣布辞职,拉脱维亚总统扎特莱尔斯2月26日授权东布罗夫斯基斯组建新一届政府。他领导的人民党、新时代党、绿党和农民联盟、祖国与联盟党及公民联盟五党联合政府3月12日获得议会批准,2013年因为超市坍塌事件辞职。","text2":"2002年至2004年瓦尔迪斯· 东布洛夫斯基斯担任什么职务?","label":1} {"text1":"系统管理汇流排(, 缩写为SMBus或SMB)是一种两条讯号所组成源自于I2C的一种汇流排,其设计应用于轻量级的通讯。最常于主机板的电源开关指令的通讯中发现其存在(例如笔记型电脑中,重复充电的子系统),其他的元件,例如温度、风扇或电压的感测器的通讯中也可以看到其踪影。SMBus 由 Intel 于 1995年所定义. 包含有 clock, data, 以及基于 Philips' I²C serial bus 协定的指令。其时钟频率范围在 10 kHz 到 100 kHz.(PMBus 可延伸至 400 kHz)FreeBSD, OpenBSD, NetBSD, DragonFly BSD, Linux, MS-Windows 2000, MS-Windows XP 和 MS-Windows Vista 皆可支援 SMBus devices, 但MS-Windows 98 以及其更早版本不支援。","text2":"它的钟频率在什么范围?","label":1} {"text1":"《丁丁与字母艺术》(法语: Tintin et l'alph-art ;英语: Tintin and Alph-Art ),又译《丁丁与阿尔发艺术》,是《丁丁历险记》的第24部作品,也是最后一部作品,作者是比利时漫画家埃尔热。故事的主要题材是现代派艺术,埃尔热在1983年执笔这部作品过程中不幸逝世,导致这部作品成为未完成的作品。为了逃避毕安卡的爱情之访,哈达克意外地结认新派黑人艺术家拉莫·纳许(Ramó Nash),并替丁丁约定艺术专家弗卡特(Fourcart)向他「爆料」,然而不久就意外身亡。在几次逃过被枪毙的险境下,丁丁不但揭发弗卡特是被杀,并发现了新兴邪教的教主恩达迪涅·阿卡斯(Endaddine Akass),就是支助纳许的艺术赝品贩售家。不久,毕安卡邀请丁丁及哈达克前往阿卡斯位于意大利的豪宅,阿卡斯却把丁丁囚禁,准备把他用热塑胶活活灌死,造成凯撒扩展式的艺术品...收到米卢求救纸条的哈达克及时救回丁丁,然而他们却无法脱离阿卡斯的魔爪,其后阿卡斯向二人表明自己就是拉普洛斯。庆幸机警的纳许及时报警,并制止拉普洛斯把丁丁及哈达克吊死,然而拉普洛斯却因而失足跌下山坡至死。另外,在埃尔热的原稿出现的艺廊职员Martine Vandezande,郤在Rodier同人版的结尾向丁丁表示好感,引起丁丁迷一点点争议。","text2":"这部作品为什么成为了未完成的作品?","label":1} {"text1":"泡泡浴()()直译为肥皂乐园,亦有译为泡泡澡,是存在于日本的一项性服务,曾经被命名为土耳其浴(),后来由于抗议改为现名,内容类似泰国浴。具体上来说,就是由一名性工作者向嫖客提供性服务的一种方式。在很多知名的成人电影作品当中,泡泡浴通常有性暗示的意味,即妓女通常会帮客人先抹上肥皂,而且方式与一般方式(鸳鸯浴)的有所差异,妓女会以身体的任何部位(如大腿、胸部甚至阴部)透过身体互相摩擦的方式抹上肥皂,借由全身的力量达到高级情趣的意味。在日本风俗店中,泡泡浴属于店舖型性风俗营业1号。根据统计,日本全国的泡泡浴店舖,目前有1,249家(平成20年,2008年)。另外,某些泡泡浴店仅限日本人前往消费而谢绝外国人。泡泡浴最早被日本叫做土耳其浴(日语:),原意是指中东奥斯曼帝国的一种公共浴室,与伊斯兰教的教义规定祈祷之前必先沐浴有关。土耳其浴巧妙的被日本人理解运用,后因土耳其留学生向厚生省诉求。1984年12月19日,「东京都特殊浴场协会」公开寻求改名,借用英文Soapland改为泡泡浴(ソープランド)的现名只对外国人开放的日本第一而且唯一的泡泡浴在2017夏天开放了","text2":"在很多知名的成人电影作品当中,泡泡浴通常有什么暗示意味?","label":1} {"text1":"醉猿(\"Dionysopithecus\")是一属史前的灵长目,生存在中新世的中国及巴基斯坦。醉猿的化石最初是由李传夔于1978年在中国江苏的双沟发现,化石包括了一个左上颌骨连同几颗牙齿。由于当时李传夔是住在双沟的一间酒厂内,故将这些化石以希腊神话的酒神狄俄倪索斯来命名,并以双沟为种小名。李传夔同时间亦发现了宽齿猿的化石。后来于1990年发现了另一些醉猿化石,并名为东方醉猿。在巴基斯坦亦有发现一些醉猿的遗骸。醉猿最初被认为是属于原康修尔猿科,但更多的化石发现显示醉猿的牙齿更像树猿,故被分类在上猿科中。醉猿的化石属于中新世早期,比原康修尔猿的更早。不过学者一般都不认为醉猿就是现今长臂猿的祖先,而是介乎猿与人类之间的分支。有些学者认为醉猿的祖先是生活在非洲,并迁徙至欧洲演化为上猿。而上猿亦进而扩散至亚洲,成为醉猿及宽齿猿。","text2":"1990年发现的化石被命名为什么?","label":1} {"text1":"天主教桑斯总教区是罗马天主教在法国中部设立的一个总教区,相当于勃艮第大区的约讷省。该教区成立于第1世纪,第3世纪升格为总教区,曾有一个时期桑斯总主教被冠以“高卢和日耳曼主教长”头衔。1622年以前,桑斯总教区由7个附属教区:天主教沙特尔教区、天主教欧塞尔教区、天主教莫城教区、天主教巴黎教区、天主教奥尔良教区、天主教讷韦尔教区和天主教特鲁瓦教区。1622年,巴黎教区升格为总教区,沙特尔教区、奥尔良教区和莫城教区划归巴黎教省。1920年以前,桑斯总主教住在桑斯,此后居住在欧塞尔。2002年,桑斯总教区失去教省中心地位,勃艮第大区成立第戎教省。法国革命时期,桑斯总主教Loménie de Brienne枢机发誓拥护教士公民组织法,但拒绝成为第一批拥护宪法的主教,将枢机主教的帽子还给教宗,拒绝成为拥护宪法的图卢兹主教,两次被桑斯的雅各宾党人投入监狱,最后因中风死在狱中。","text2":"天主教桑斯总教区相当于什么?","label":1} {"text1":"《武尊少林》(),是香港无线电视1993年拍摄的电视连续剧,全剧共20集,监制杨锦泉。主要演员有温兆伦、黎姿、张兆辉、梁小冰、郭蔼明、罗乐林、白彪等,剧集特别邀请体操王子李宁饰演「普照」。主题曲由徐嘉良作曲,黄霑填词,温兆伦主唱。少林寺两本宝经「易筋经」与「洗髓经」被盗,高僧普照(李宁饰)乔装查访,途中与清廷特务组织六扇门的成员阿雪(郭蔼明饰)渐生情愫。经一番争夺,普照夺回「易筋经」,但因误杀无辜,甘愿成十八铜人,而阿雪虽得到「洗髓经」,却被主人连坤(白彪饰)所杀。端木家与白家两个武林世家经常发生争斗,白家年青公子白世芬(温兆伦饰)与端木家的娇娇女端木娇(黎姿饰)更是斗气冤家,在不知不觉间端木娇更爱上了世芬。端木娇虽生性刁蛮,但富正义感,对同母异父,被受端木家冷落的兄长孔提(张兆辉饰)格外关怀。两家的一次决战中,孔提以偷学的端木家功夫救了世芬,因而被逐出门。世芬与孔提上少林学武,并在少林寺结识了尚悟真,原来悟真是四皇子胤禛,混入少林偷取秘笈,即为日后之雍正(罗乐林饰)。世芬被派下山找回「洗髓经」,途中救了被追捕的少女楚梦色(梁小冰饰),世芬对梦色一见钟情。原来梦色本为连坤之妻,因苦练从连坤手上偷走的「洗髓经」而回复青春容貌。世芬最终虽完成任务,将「洗髓经」送回少林寺,然而雍正为求长生不老,竟要求少林交出经书,否则下令火烧少林,究竟少林最后能否避过这场大灾劫呢?","text2":"《武尊少林》的主题曲由谁作曲?","label":1} {"text1":"五所川原车站()是一位于日本青森县五所川原市大町,由东日本旅客铁道(JR东日本)与津轻铁道所共用的铁路车站。虽然两家公司是共用车站设施,但为了区隔所属系统与路线的差异,津轻铁道的部分特别以津轻五所川原车站()命名。五所川原是所在地五所川原市的主车站,也是JR东日本五能线与津轻铁道线的交会点,一些等级较高的快速或临时快速列车皆有在本站停靠。2002年(平成14年)入选东北车站百选。岛式月台1面2线的地面车站。站舍与月台以跨线天桥连络。2号月台是并行的留置线,作为此站到发的临时列车留置之用。侧式月台1面1线的地面车站。与JR共用通道及跨线天桥。但是站舍与闸口不同。※临时快速「」的相邻停车站参见「」的条目。","text2":"JR东日本五能线与津轻铁道线的交会点是什么?","label":1} {"text1":"江背镇位于湖南省长沙县东南部,为长沙县第二大乡镇,镇域北、东、南为浏阳市地域。地缘上,江背镇北与浏阳市永安镇、洞阳镇接壤,西与葛家乡、镇头镇为邻,南连柏加镇,西与黄花镇、干杉乡交界,全镇总面积175平方公里,总人口53,997人(2000年人口普查);辖13个村、3个社区;政府驻江背社区。今江背镇在1995年长沙地区撤区并乡镇之前属江背区(由五美区改名)江背镇、梅花乡和五美乡地域,1995年,三乡镇合并组建江背镇。2004年村级区划调整,调整之前为37个村、2个社区,分别为坳头村、板桥村、抄冲村、赤霞村、东庄村、洞井村、福冲村、好布村、河田村、洪江村、江背村、金田村、金洲村、开福村、立新村、联丰村、麻塘村、梅花村、美新村、楠木村、漆桥村、齐心村、清塘村、石背村、铁锁村、万古村、乌川村、五福村、五美村、湘阴村、小埠村、肖桥村、阳雀村、印山村、钟桥村、朱桥村和砖田村,2个社区;区划调整后为13个村、3个社区。","text2":"江背镇位于什么地方?","label":1} {"text1":"安雅·罗素法(,),来自俄罗斯圣彼得堡的模特儿。她是《全美超级模特儿新秀大赛》第十季的亚军。2008年,安雅宣布改回出生时的名字:安雅·罗素法(Anya Rozova),在此之前是使用安雅·冈()。安雅于俄罗斯出生,后来被一个居住在美国夏威夷群岛欧胡岛檀香山的家庭领养。安雅十七岁时曾参与香奈儿、路易·威登及芬迪(Fendi)等品牌的非正式时装秀。2007年,她于瓦伊帕胡高级中学毕业。毕业后,她当了一名售货员。她曾为Russell Tanoue拍摄照片,Russell Tanoue称赞她是「有前途的新面孔」。安雅在半准决赛面试时说她对模特儿行业充满热诚,所以参加全美超级模特儿新秀大赛。她于比赛中表现出色,曾五次首名入围,平均入围顺序更拿下历届以来最优异的成绩(2.64),另外胜出三次小挑战,分别获得与评判尼祖·百克拍照、为柠檬味道的七喜拍摄广告的机会及十万美元、和盖马蒂洛(Gai Mattiolo)设计的晚装。在最后两强中,安雅与另一名参赛者惠妮·汤姆森为范思哲走秀,但评判认为她在台上不够惠妮突出,所以选了惠妮当冠军,安雅屈居亚军(但就整体表现来说,部份网友认为安雅才是第十季名副其实的冠军。)安雅在比赛拿五次第一,也胜出多次小挑战。安雅赛后再次与Russell Tanoue合作,为2008年4月30日出版的MidWeek杂志拍摄封面及内页照。其后她参加了V杂志与Supreme模特儿公司合办的模特儿选拔赛2008。她其后更与Elite签约。最近她与香港的模特儿公司 Style International Management 签约,并在香港发展其模特儿事业。她曾在很多香港的时装杂志中任模特儿,《Jet》、《东方日报》、《Elle》等。","text2":"安雅·罗素法参加了什么比赛获得了亚军?","label":1} {"text1":"绵阳站,位于四川省绵阳市涪城区临园路西段,为成都铁路局管辖的一等站,距成都站115公里,距宝鸡站554公里。目前主要办理宝成铁路的客运列车到发作业和西成客运专线的长途动车组及成绵乐城际动车组到发作业。绵阳站目前建有5座候车站台,站线共12道,并建有有天桥1座、地下通道1条。2008年5月12日发生的汶川大地震使绵阳站受损严重,候车大厅完全停用,在车站广场另建临时候车区使用。2009年10月1日,绵阳站作为汶川地震灾后重建的重点项目正式开工重建,总投资2.42亿元,总建筑面积达15000平方米,按最高候车人数3000人设计,抗震设防烈度为8度。2010年10月1日,绵阳站重建工程竣工投入使用。2014年12月20日,成绵乐客运专线全线开通,绵阳站所有站房、站台投入使用。","text2":"绵阳站目前主要办理什么?","label":1} {"text1":"轧压均质装甲(英文:Rolled Homogeneous Armour;德文:gewalzte homogene Panzerung),缩写成「RHA」,又简称为均质装甲是为钢板之理论基础象征,RHA做为一种基数,用于对照军事装甲车之效能。直至第二次世界大战结束,大部份坦克的装甲形式以及其它装甲车辆均覆盖著钢板。若要增加车辆的防护,即意味著要增加更厚的钢板,而厚重的装甲相对的也减低了其机动性。从那时以来,一些形式的装甲开始发展出混入一些空气、陶瓷或是贫铀(depleted uranium,简称DU)于钢质之中。在现代化武器攻击下,更大的撞击力量与更高的温度切割冲击,传统的轧压均质装甲已不敷使用,遂由更高等级的装甲取代,而测量单位也跟著有所改变。更新的装甲估量数被称做「等效轧压均质装甲(Rolled Homogeneous Armour equivalency,简称RHAe),其做为一种粗略估计的单位,或是被炮弹穿透的能力,或是其装甲形式的保护能力,这些不一定是使用钢材。但是,因为各装甲间形式、质量、金属以及分工组装等不同,RHAe是否能有效的比对不同形式装甲,仍有许多争议性。当前美国陆军所使用,RHA钢板产品有军用标准MIL-A 12560,而最新标准则使用有MIL-A 46100。这些非常相似,但实际上在美国的钢板等级中,不同于标准高强度钢材合金4340,虽然其械制造与合金非常的相似。","text2":"当前美国所使用的RHA钢板产品标准是什么?","label":1} {"text1":"椰子罐头(COCOCAN),原名水果村、诺亚方舟(Noah's Ark)是由游戏橘子开发的休闲网路游戏,台湾方面由于人数不断锐减,自2008年10月1日起停止营运。椰子罐头自2003年初在台湾推广以来,吸引不少网友的兴趣,玩家年龄层广,之后也在日本、香港等地掀起风潮。为了能让名称更贴近游戏的性质,游戏橘子决定将「诺亚方舟」改名为「椰子罐头」,由于本游戏是利用风向及仰角抛投各类武器来攻击对手,以获得等级的晋升和金币,而「COCOCAN」的台语谐音为「一直丢、一直丢」,再把「COCONUT」和「CAN」两个英文单字拼装起来,中文翻译即《椰子罐头》,用此作为正式名称。日本游戏橘子于2009年1月30日发布新闻表示,日版椰子罐头将于当地时间4月30日晚间五点起停止营运,届时椰子罐头将完全走入历史。台湾方面由于玩家人数不断锐减,游戏橘子于2008年7月1日发布即将停止营运公告,自10月1日起台湾版椰子罐头正式停止营运。香港版停止营运。在本游戏中,共设计出12个人物,但截至目前为止能选择的角色有9个。以下所列能力之均为初始值,创立角色另有7点基数(台湾版后期仅有2点)可调配至各属性中,另也可通过游戏内道具提升。注1:角色原名系指「水果村」及「诺亚方舟」时期的名称,栏位空白表示游戏更名「椰子罐头」后其角色名称无异动,标明\"-\"者代表该角色于「水果村」及「诺亚方舟」时期尚未登场。注2:官方所给予之中文名称仅有表单上所列出之四个角色,「酷猫」之名称为游戏更名为「椰子罐头」后同步更名。","text2":"本游戏中可供选择的游戏人物有多少个?","label":1} {"text1":"在数学中,单李群是不含非平凡的连通正规李子群的连通李群。另一个等价的定义是:单李群是对应到单李代数的连通李群。单李群是李群理论中的基本构件,依照其李代数的复化,可以分成三族典型群,与有限个例外李代数。前者在几何学与数论中的应用有悠久历史,而后者则涉及数学中的某些特殊配置与当代理论物理学。在应用上,我们通常会考虑更一般的半单李群或约化群。约化群的表示是当前数学的热点之一。单群的分类法是先考虑其李代数的复化,并分类相应的根系。为了从复数域回到实数域,下一步是分类复李代数的实形式,这可藉 Vogan 图完成。最后,李代数一一对应到单连通李群,为了从李代数层次回到李群层次,还须要计算单连通单李群的中心。复单李代数的分类如下,以下的 formula_1 代表邓肯图的顶点个数:","text2":"单李群的另一个等价定义是什么?","label":1} {"text1":"安卡拉战役(Battle of Ancyra)发生约在前239年,也可能在前240年,是塞琉古帝国内战-兄弟战争的一场决定性战役。当塞琉古国王塞琉古二世与托勒密王朝结束第三次叙利亚战争,一场内战紧接著在塞琉古帝国爆发,塞琉古二世的母亲劳迪丝要求他给予弟弟安条克·伊厄拉斯共同摄政权以及塞琉古领土中安那托利亚的管治权,安条克企图掌握整个帝国,便向塞琉古二世宣战,爆发兄弟战争。占据小亚细亚的安条克向附近的本都王国国王米特里达梯二世、卡帕多细亚王国国王阿里阿拉特三世和加拉太人同盟。对于安条克·伊厄拉斯的叛乱,塞琉古二世立即入侵安那托利亚,并在初期的军事行动中都有进展,并把矛头转向米特里达梯二世,而安条克率领一支包含许多加拉太人的大军来对抗,两军在Ancyra(安卡拉旧称)附近交战。战役结果塞琉古二世大败,损失了20,000名兵马且仅仅勉强逃脱。之后,战争仍维持几年,双方似乎签定和约,塞琉古二世失去了帝国安那托利亚部分和塔尔索西部。","text2":"安卡拉战役是发生在哪个国家的内战?","label":1} {"text1":"围棋棋手师承关系,是指在围棋棋手中的师承关系。大部分围棋棋手都有自己的老师,但围棋的师承关系不像传统艺术那么明显。一般认为,日韩的围棋师承关系较为严格,甚至达到徒弟必须住在师傅家的“内弟子”程度,典型的例子是曹薰铉和李昌镐的师徒关系;而由于中国棋院的类似国家队的制度,中国的围棋师徒关系相对比较松散,达到“内弟子”程度的只有罗建文与罗洗河。不过聂卫平与他的弟子们有签有合同,并且规定了奖金分成制度,因此受到争议。木谷门是世界围棋最大的门派,段位合计超过五百段,旗下的“六大超一流”曾横扫世界棋坛,木谷实也被视为一代围棋宗师。在中国,聂卫平的徒弟最多,实力最强,收徒最正规, 聂也被视为中国的木谷实。","text2":"在中国,达到“内弟子”程度的师徒是谁?","label":1} {"text1":"中国政法大学弑师案,是指2008年10月28日晚发生在中国政法大学的一场由于师生恋而引发的血腥惨案。该校法学院教授程春明在该校端升楼201教室内,被该校男学生付成励用菜刀当众袭击身亡。此案引发巨大的社会轰动和深刻的道德反思。2009年10月20日,北京市第一中级人民法院一审宣判,以故意杀人罪判处23岁的付成励死刑,缓期两年执行。2007年4月,付成励结识了中国政法大学女研究生陈某,交往了几个月后,2007年8月,付成励与初恋女友陈某正式确立了恋人关系。在后来二人去北戴河游玩期间,陈某把自己曾和中国政法大学教师程春明发生过一年多性关系的事告诉付成励。知道这件事后,付成励的情绪受到沉重打击,遂对程春明动了杀机。","text2":"北京市中级人民法院一审宣判的结果是什么?","label":1} {"text1":"都兰山位于台湾台东县东河乡,为卑南族人心目中的圣山,同时也是目前台湾正向联合国申请世界文化遗产的项目之一。都兰山又名「台东富士山」,因山形似富士山,日治时期日人称之为「台东富士」而来。台湾目前保存最完整的新石器时代遗址,也就是卑南遗址,一直都与都兰山有著密不可分的关系,在卑南遗址的村落往北边望去,就是村人们心中的圣山都兰山。而考古挖掘出土的所有石棺所葬的方向也都朝北,表现出族人们希望逝者灵魂归往圣山的信仰。都兰山目前居住为阿美族人的都兰部落,同时许多台东的阿美族部落也视都兰山为圣山,在当地县政府与社区居民的推动下,都兰山也发展著文化观光产业,希望借由传统文化与山海美景来吸引更多游客。","text2":"都兰山又被称为什么?","label":1} {"text1":"星点东方鲀(学名:)又名黑点多纪鲀、星点多纪鲀,俗名星点河鲀、龟鱼,是辐鳍鱼纲鲀形目四齿鲀科东方鲀属的一种鱼类。星点东方鲀分布于西北太平洋区,包括日本、韩国、台湾、香港、越南,以及中国南海、东海、渤海、黄海等海域,该物种的模式产地在东京、Misaki、Wakanoura、Tsuruga、长崎。5-100公尺。星点东方鲀体呈圆筒形,被覆由鳞片特化的细棘;口小。背部呈褐红色或暗绿色,散布许多淡色圆斑点;腹部白色。体侧近胸鳍部位有一黑色胸斑,背鳍基底也有一黑斑,尾鳍截形,各鳍黄色,尾鳍后缘橙黄色。背鳍软条12-14枚;臀鳍软条10-12枚,体长可达15公分。星点东方鲀常生活于近海底层,喜栖息于沿岸海草丛生的海域和河口附近岩礁区。游动缓慢,受惊吓时会吸入大量空气或水,将鱼体鼓胀成圆球状,以吓退掠食者,属肉食性,以甲壳类、软体动物等为食。繁殖期时,成鱼会聚集在沿岸砾石区产卵受精。食用鱼,但皮肤、精巢和内脏均有河豚毒素。","text2":"星点东方鲀分布于西北太平洋区,包括哪些海域?","label":1} {"text1":"慧卡·芝玛曼(,),德裔巴西模特儿,经过一阵精灵模特儿的热潮后。2007年1月起,慧卡取代了吉玛·沃德(Gemma Ward)的位置,被模特儿权威网站model.com封-{后}-,2010年2月更跻身殿堂级超模。芝玛曼14岁时就进入的家乡的模特儿学校,开始了她的模特儿生涯。今年岁的Raquel在模特儿圈已可称之为前辈,前阵子流行精灵系model,模特儿界几乎被吉玛·沃德、莉莉·蔻儿(Lily Cole)等占据。但现在精灵系风潮已不在,Raquel于是得以出线,凭借她苗窕的身材、德巴混血野性中带优雅的气质,成了近来各大品牌的爱用名模,Fendi设计师卡尔·拉格斐(Karl Lagerfeld)便钦点她为该名牌拍摄多季的平面广告。","text2":"什么时候,慧卡取代了吉玛·沃德(Gemma Ward)的位置?","label":1} {"text1":"冈山绿雉(拉丁字:Fagiano Okayama F.C.)是日本一家足球会,位于冈山市内,现于日本职业足球联赛乙级(J2)比赛。\"Fagiano\" 在意大利文中解作雉。冈山绿雉成立于1975年,是自川崎制铁水岛迁离神户市后(即现今神户胜利船)才开始运作。部分原属旧球会的球员,自己另组一队新球队,命为「江河自由踢球者」(River Free Kickers)。数年后,球队于分县联赛中比赛。在2003年,球队改名为「冈山绿雉」,自此成绩便一直进步。在2005年,冈山绿雉升上了中国地方联赛。2007年7月,绿雉成为第一队在低于日本足球联赛组别比赛的队伍,得到日职联会员身分(J. League Associate Membership)。同年12月2日,绿雉赢得地区联赛附加赛第一名,并且获得升上日本足球联赛。在2008年赛季中,球队成功以第4名完成联赛,获得升上日本职业足球乙级联赛(J2)的资格。因此当升班制度在12月1日正式法定后,冈山绿雉会在2009年参与J2比赛。","text2":"冈山绿雉的地理位置在哪里?","label":1} {"text1":"玩转姻玄路(Love Match Turnaround),为香港有线电视的一个玄学爱情电视节目,于2009年3月7日起在有线娱乐台逢星期六22:00-23:00播放,并于星期日在有线新知台21:00-22:00重播,每集大约60分钟(连广告),主持为麦玲玲、黄翠如、区焯文。节目每集会选出一位主角与另外三位追求者,由玄学家麦玲玲以面相、手相、身相、八字命理、姓名学等不同算命方式来预计三位追求者与其主角是否匹配。除此以外,节目会与三位追求者进行问答,透过答案协助主角选择哪一位追求者。而且,每集均会邀请一位嘉宾进行有关爱情的访问。而节目类型于香港算作少数。其节目是欢迎公众报名参与,但有指,主角与三位追求者为有线电视的媒子,为了做节目而邀请的。节目首15分钟为访问嘉宾,期后会播放主角介绍,继而介绍三位追求者,出场时追求者会戴上明星面具,例如:周杰伦、刘德华等。然后麦玲玲会以面相推断追求者的性格,并会作表演吸引主角,其后追求者会与主持及嘉宾进行问答,而主角全程会坐于一个蛋形中,于最后选择追求者后才会见其样貌。","text2":"玩转姻玄路节目的播放时间及主持人是?","label":1} {"text1":"《ADULT VIDEO》(,\"ADULT VIDEO\")是东京事变的第四张DVD,跟《ADULT VIDEO Original Sound Track》,一起于2006年3月23日发行。发行当周即卖出1.6万张,总计销售额3.2万张,名列2006年年度销售榜第52位。初回限定为「Digipak纸盒仕样」。本音乐录影带是东京事变第二张映像集,共收录六首音乐录影带,包括两首来自单曲《修罗场》内的曲目,四首来自第二张专辑《大人》的曲目。首先是取材自大人的团员介绍作品「歌舞伎」,然后是改编专辑版本的「秘密 FOR DJ」,再来是第三张单曲,同样是改编版本的「恋爱幻梦 FOR MUSICIAN」,与以白色为基调的「修罗场」,然后是鼓手刄田缀色大秀舞技的喧哗上等,最后则是改编专辑版本的「黄昏泣 FOR MOTHER」做为结尾,总计六首。早于3月6日起,即在MTV JAPAN拨出「秘密 FOR DJ」音乐录影带,同一天,「秘密 FOR DJ」也开始接受歌迷线上下载。等到3月20日,官方网站特地开设「特设网站」。日本则于3月23日发行DVD,台湾则是等到4月4日才发行日版DVD才发行进口盘。《ADULT VIDEO Original Sound Track》(,\"\"ADULT VIDEO\" Original Sound Track\")是东京事变的第二张黑胶唱片,跟《ADULT VIDEO》DVD,一起于2006年3月23日发行。总计销售额1,065张。这张黑胶唱片为东京事变第二张黑胶唱片,主要是将《ADULT VIDEO》收录的曲目音源化的一张黑胶唱片,因此也可以说这是《ADULT VIDEO》的原声带,采完全限定生产。早于3月6日起,「秘密 FOR DJ」开始接受歌迷线上下载。日本则于3月23日发行黑胶唱片,台湾则是完全没有进口。","text2":"发行当周售卖出唱片多少张?","label":1} {"text1":"赵允让(),宋朝皇族,字益之,宋太宗第四子商王赵元份的第三子。濮安懿王。史称他天资浑厚,外庄内宽,喜愠不见于色。赵元份在世时,赵允让为右千将军。宋真宗的长子周王赵祐去世后,真宗以绿车旄节迎赵允让到宫中抚养。皇子赵祯(宋仁宗)出生后,用箫韶部乐送还府邸,官居卫州刺史。宋仁宗即位,授汝州防御使、宁江军节度使。后来又让他知大宗正寺。他勉励好学的宗室子弟,劝戒不上进的,故人莫不畏服。庆历四年(1044年),封汝南郡王,拜同平章事,改判大宗正司。嘉祐四年(1059年)去世,年六十五,赠太尉、中书令,追封濮王,谥号安懿。仁宗在位久无子,以赵允让第十三子赵宗实为皇子,改名赵曙。仁宗崩,皇子赵曙即位,是为宋英宗。宋英宗即位后,围绕赵允让是作为皇伯还是皇父的争议,被称为濮议。","text2":"围绕赵允让是作为皇伯还是皇父的争议被称为什么?","label":1} {"text1":"扁头豹猫(学名:'),或简称扁头猫,是一种体型较小的野外猫科动物,分布在泰国南部、马来西亚、汶莱及印尼的森林。牠们因失去栖息地及污染而濒危。只有两只扁头猫受到饲养,都是在马来西亚的动物园。像很多细小的猫科,牠们都曾被分类在猫属之中。扁头豹猫体长41-50厘米,尾巴很短,只长13-15厘米,重1.5-2.5公斤。牠们一般呈深赤褐色,头部更红,腹部较白。除了面部上淡淡的斑纹,牠们可谓没有斑纹。牠们四肢较短,耳朵短而圆。趾间的蹼可以帮助牠们在泥泞及水中活动。牠们的头颅骨稍长,头顶扁平。扁头豹猫栖息在次生林及原生林,很多报告都指是在近水区见到牠们。牠们主要猎食青蛙、鱼类及甲壳类,也会捕捉大家鼠及鸡。牠们的前臼齿较长,爪不能伸缩。这与半水中生活的鼬科相似,随时可以进到水中。整体而言,牠们野外的生活资料不详。妊娠期约56天,每胎产1-2只幼猫。饲养的扁头猫寿命可以超过14岁。一般相信牠们是夜间活动的,但观察显示牠们是黄昏及破晓出没的。扁头豹猫被世界自然保护联盟列为濒危,且列在《濒危野生动植物种国际贸易公约》附录一中。牠们的成年数量相信少于一万只,其下的亚群没有一个有多于1000只成年。牠们受到失去栖息地及水污染的威胁。牠们除了在在汶莱外都受到完全的保护。在野外很少会见到牠们。","text2":"扁头豹猫主要食物来源是什么?","label":1} {"text1":"《银河骑士》(The Adventures of the Galaxy Rangers)是美国与日本东京ムービー于1986年所合作,自1986年9月14日~12月11日之间,共播映65集(每周一~周五带状播出)的动画作品。台湾于1987年至1988年间于每周五下午5:30~6:00时段由台视引进播出。公元2086年两位外星人和平使节来到地球寻求人类的合作与协助,也为了报答人类的对于合作的好意,他们慷慨捐出一项非常珍贵的礼物——『超驱动能(hyperdrive)』,使人类首次克服时空之限制,能够更轻易跨入大宇宙中的银河,展开了外太空探索的崭新一页。在地球本身方面,为了地球自身的生存安全与大宇宙的命运前途,立即组成了『宇宙防卫总部』,担负起打击恶魔的神圣使命,以便共同对付一帮存心在大宇宙中破坏和平安宁的恶徒。每位队员都有配戴藏有能源传继系统的『第五系列太空警徽(Series Five Implant)』,只要当负担打击来敌时按动所佩带的警徽,立刻就可以使自己的「能」力因扩增而达于极点,拥有风驰雷掣难有匹敌者之力。","text2":"第五系列太空警徽的作用是什么?","label":1} {"text1":"吉林省实验中学始建于1948年,校址在吉林市北山脚下,时名为吉林市联合初级中学。1950年,学校更名为吉林省立第一中学,随后,学校于1955年随吉林省政府迁至长春,座落于美丽的南湖之滨,名为吉林省实验中学。吉林省实验中学经常被称为“实验园”,是吉林省教育厅直属、省首批办好的重点中学,是吉林省首批示范性高中,与东北师范大学附属中学(附中)、长春市第十一高中(十一高)、长春市实验中学(市实验)并称为“吉林省四大名校”。实验中学还是国家教育部指定的基础教育实验基地校,同样也是清华大学、北京大学等全国名牌重点大学生源基地校。吉林省实验中学始建于1948年,是省教育厅直属的首批重点中学。半个世纪以来,学校在教育教学实践中形成了“笃学、践行、求是、创新”的校风;“严谨、多思、善诱、精深”的教风;“刻苦、灵活、互助、进取”的学风,赢得了社会各界的广泛认可。吉林省实验中学占地面积近10万平方米(160多亩),建筑面积8万平方米。有理、化、生实验室9个;学生阅览室2个;语音室2个;计算机室5个;600台电脑。多媒体教室25个;教师电子备课室1个;校园电视台1个;图书储藏室2个。有400米跑道的运动场和600平方米的风雨篮、排球场,体育艺术中心设有游泳池、篮排球馆、各类器乐排练室等体育艺术活动场所。现有79个教学班,近5000名学生。教职员工248人,有228名任课教师,其中特级教师8名,高级教师84名,中级教师59名,研究生以上学的教师有80名,有国家级骨干教师8人,省级骨干教师25人,市级学科带头人16人,有省级科研型名教师4人。有46名同志在全国各级各类学术团体中担任理事长、理事等职务。张熙恒","text2":"吉林省实验中学始建于哪一年?","label":1} {"text1":"《西斯廷圣母》(Sistine Madonna),亦称《希斯汀圣母》,为义大利画家拉斐尔圣母像中的代表作,装饰于为纪念教宗西斯笃一世而重建的皮亚琴察圣西斯托教堂,最初它被放在教堂的神龛上,至1574年为止,一直保存在圣西斯托教堂,故得此名。现为德国德累斯顿的历代大师画廊(萨克森州立艺术博物馆的一部分)收藏。画中表现圣母抱著圣子从云端降下,两边帷幕旁画有一男一女,身穿金色锦袍的教宗西斯笃一世向圣母、圣子做出欢迎的姿态。而稍作跪状的圣芭芭拉,她虔心垂目,侧脸低头,微露羞怯,表示了对圣母、圣子的崇敬和恭顺。位于中心的圣母体态丰满优美,面部表情端庄安详,秀丽文静,在更高的起点上塑造了一位人类的救世主形象。扒在下方的两个小天使睁著大眼仰望圣母的降临,稚气童心跃然画上。","text2":"《西斯廷圣母》名字的由来?","label":1} {"text1":"甘必大大街(Avenue Gambetta)是巴黎二十区的一条街道,得名于法国政治家莱昂·甘必大(1838年至1882年)。甘必大大街是一条林荫大道,始于奥古斯特梅蒂维埃广场(place Auguste-Métivier,海拔54米),然后向东北方,沿着square Champlain,前往 place Martin-Nadaud.然后折向东,到达甘必大广场(place Gambetta,87米)。然后厉害地折向东北方,经过德农医院(Hôpital Tenon),到达西涅克广场(place Signac,99米)和圣法高广场(Saint-Fargeau,108米)。经过 Tourelles行政中心的后面,法国国外情报局(DGSE)总部,以及佐治谷泳池(piscine Georges-Vallerey) ,结束于丁香门(porte des Lilas,116米)。","text2":"甘必大大街的折向是什么方向?","label":1} {"text1":"陶鹰鼎是仰韶文化陶器,出土于陕西省华县(现渭南市华州区)太平庄。2002年列为中华人民共和国禁止出国(境)展览文物之一,现藏中国国家博物馆。1993年,中国第一次申办奥运,当时在瑞士的洛桑市的奥林匹克博物馆正举行申奥国家文化展,当时国际奥委会主席萨马兰奇亲自于中国北京及西安等地挑选参展文物,最终陶鹰尊与青铜大面具、错金银龙凤青铜方案、金缕玉衣、兵马俑、鎏金银盘、凤冠和皇帝龙袍共七件文物成为参展品。陶鹰鼎通高36厘米,整体结构质朴,周身光洁,未加纹饰。鼎上雕有一只两眼圆睁的猫头鹰,以鹰的前胸为鼎腹,双翅后收构成鼎之中后部,鹰背开有鼎口,鹰足和尾部为鼎足。鹰的双目圆睁,喙部呈勾状,整体造型颇具气势,十分生动。","text2":"陶鹰鼎是属于什么陶器?","label":1} {"text1":"拳击道是一个由传统韩国武术的风格及泰拳的混合的技击运动;在某程度上拳击道被称为韩国版本的拳击。在某些城市圈里亦有被称为「Muay Kwon」(泰道)及「Kwon Ho」。「拳击道」一词有几个意思,分别是「拳击及攻击的艺术」、「第一个攻击」及「拳击的艺术」。「拳击道」中的「拳击」代表著拳打及拳击,而「道」即是其艺术或是修练。现在其总部位于韩国的斧山。该武术是由钟道武(音译,其谚文罗马化为「Jung Do-Mo」)看到一个擅长跆拳道的人跟泰拳练习者砌磋后所创立的。创始人表示这个混合体比起单独使用跆拳道或是泰拳更有破坏性。拳击道的左右铭是「To attack offensively, never to surrender」(意译为「永不放弃狠狠的攻击」)。其教义为自信、实践、忠诚、尊敬及责任。这个武术的基本技术是跆拳道的基本踢击技巧,再加上一些泰拳的技巧,例如是一些防卫踢击(类似散打)及一些使用膝盖及手肘的攻击。另外,一些泰拳中部分步法也被删除掉,就好像是泰拳中的步进(Stepping)改为跆拳道里的滑步(Slides)及快步(Quick steps)。对于拳击的闪避(类似美式风格对于拳击的急忙低头避开)已加进这个技击运动以代替泰拳里有缺点的「Back up」,这令到使用者可以闪避及还击得快点。现在,这种风格的混合武术已经进步到站在战斗的角度其中一个最有实力的武术之一。拳击道比较少有类似掌(即是做出一个张开手掌的动作)的技术;练习者认为,这是因为在对手用力的防护或者是击打的这个情况下,手指会很容易夭断或脱臼。这个武术的足部技巧是混合自跆拳道及Kickboxing(有氧搏击操)的动作。跆拳道的基础能够在其混合武术中看到,例如是前踢(因为踢出时比其他的踢击都要快)、后踢、侧踢及勾踢等等。另外一个被混合至拳击道的是泰式的旋踢,但在跆拳道里该踢击是被禁止的,这是因为这种踢击有著惊人的速度。尽管在泰国比较少有拳击道的习训者用其武术来对抗泰拳的练习者,只不过他们的技术也有一定的水平。不幸地拳击道还有很多的进步及发展空间,不能广泛被大众公认为这是一种武术;有见及此,一些人认为这种武术在不少人知道其生存的时候,已经走到去灭绝的边缘。","text2":"拳击道的座右铭是什么?","label":1} {"text1":"开角龙亚科(Chasmosaurinae)又名角龙亚科(Ceratopsinae),是角龙下目角龙科的一个亚科,角龙科的另一个亚科是尖角龙亚科。开角龙亚科是群大型四足动物,前肢略弯,短于后肢。牠们的体型壮硕,四肢粗壮。头骨后方的头盾大,由顶骨、鳞状骨构成,头盾长度可相当于头骨本身。喙骨与前齿骨构成长、狭窄的喙状嘴。牠们具有多列的齿系,会不断地生长、取代磨损的牙齿。开角龙亚科与尖角龙亚科的最明显差异在于头盾与角的形状。与尖角龙亚科相比,开角龙亚科的鼻角较短、额角较长、头盾较长、头盾具有大的洞孔。但是,三角龙的头盾短,头盾没有洞孔。五角龙与牛角龙的头盾可达2公尺长,是已知具有最大头部的动物。头盾周围的颈盾缘骨突(Epoccipital)短小、或没有,与尖角龙亚科不同。如同其他角龙科,开角龙亚科生存于白垩纪晚期的北美洲西部。化石发现于坎潘阶与马斯垂克阶地层,年代为8,000万到6,500万年前。开角龙亚科是在奥塞内尔·查利斯·马什(Othniel Charles Marsh)在1888年建立。其名称来自于角龙属,但角龙的化石过于稀少,目前状态为疑名。因此劳伦斯·赖博(Lawrence Lambe)在1915年建立开角龙亚科(Chasmosaurinae)以取代角龙科。三角龙的头骨较短,产生分类学上的争议,较晚期才被归类于角龙亚科。","text2":"开角龙亚科生存在哪个时代?","label":1} {"text1":"老教堂(Oude Kerk)是阿姆斯特丹最古老的本堂区教堂,1306年由乌德勒支主教祝圣。它耸立在德瓦伦(De Wallen),现在阿姆斯特丹的主要红灯区。教堂占地面积约三千三百平方米,其基础是一个人工土堆,被认为是这个沼泽地带最坚固的地基。这座教堂的屋顶是欧洲最大的中世纪木制拱顶,其爱沙尼亚木板追溯到1390年,拥有欧洲顶级的声学效果。许多音乐会在这里进行,包括BBC Singers和Academy of St. Martin in the Fields。地面完全用墓碑铺砌,其原因是这座教堂兴建在一个公墓上。直到1865年,当地居民继续埋葬在教堂范围内。在老教堂有2500个坟墓,埋葬了10,000名阿姆斯特丹市民。在1578年宗教改革以前,老教堂是一座天主教堂。威廉一世 (奥兰治)打败西班牙之后,教会接受了新教加尔文教派。在16世纪的战争中,教堂多次遭到抢劫和污损,只有天花板上的绘画得以幸免。","text2":"老教堂是由哪个主教祝圣的?","label":1} {"text1":"《丁丁与丛林战士》(法语:\" Tintin et les Picaros \")是《丁丁历险记》的第23部作品,也是《丁丁历险记》中最后一部完整作品。作者是比利时漫画家埃尔热。于1976年初版。这部作品与《丁丁在苏联》和《丁丁在刚果》并为丁丁历险记中最受争议的作品。但与前两部不同,这次的争议主要是在艺术上而非政治上。为营救身陷囹圄的毕安卡等好友,丁丁一行流落森林,遇上阿卡扎尔将军。丁丁决定冒险一拼──协助阿卡扎尔和他的丛林战士,推翻塔比奥卡政权。不料丛林战士却中了塔比奥卡的奸计,溃不成军,弄得阿卡扎尔一筹莫展。丁丁凭著图纳思的新发明,戏剧性地令军队重振士气、直捣黄龙。","text2":"《丁丁与丛林战士》的作者是谁?","label":1} {"text1":"《人在边缘》()是香港电视广播有限公司制作的时装恩仇电视剧,全剧共30集,监制张华标。主要演员有黎明、刘青云、林文龙、罗慧娟、陈法蓉、黎美娴、方刚等。主题曲由卢东尼作曲,陈少琪填词,黎明主唱。此剧是1990年香港年度收视冠军,全剧平均收视为42点,亦是张华标唯一一部监制的作品,黎明亦凭此剧走红。本剧曾于2006年在广东电视台珠江频道及于2010年7月5日在TVB星河频道重播,于2009年8月在无线收费电视经典台《我们的…黎明》每周六至日连续播映四集,特别调动此剧于2009年10月3日-4日周六至日连续播映六集。2015年6月及2017年9月6日在TVB经典台重播。2016年6月1日起在翡翠台重播,并附加上中文字幕。林雪娥早年为救家人,诬陷郑细凤杀人,细凤不忿被冤枉,越押逃走。十八年后,雪娥长子刘志光长大成人,任职惩教署。细凤之子林奕龙误入歧途,被判入劳役中心,重遇志光。奕龙故意为难志光,但志光却细心引导奕龙返回正途。经历重重波折,二人成为了好友。此时,细凤回港寻找奕龙,雪娥幼子志基利欲薰心,错手杀了细凤养子郑伟,更冒认是细凤之子,期间,志基因恐雪娥泄露真相而错手将她杀死。志光见其弟丧心病狂,决大义灭亲,与奕龙联手对付志基……","text2":"2015年6月及2017年9月6日,该剧在哪个台重播?","label":1} {"text1":"黄额闭壳龟(学名:)又称梅花箱龟、梅花盒龟,为龟科闭壳龟属的爬行动物,又称梅花盒龟、花背盒龟、海南闭壳龟等。分布于北部湾,尤其是越南北部以及中国大陆的海南,亦见于广西及寮国等地,但尚未确定是否存在于柬埔寨东北部。一般生活于山区的溪流中,但海南岛的黄额闭壳龟是生活于低地的。因陆栖性强,有部分学者认为应将黄额闭壳龟和黄缘盒龟列入Cistoclemmys属。黄额闭壳龟是杂食性的,圈养个体每次产一至三枚卵。黄额闭壳龟虽被列为极危物种,并是华盛顿公约附录2、越南红皮书物种,但仍面对野生个体被捕捉食用的威胁。中国是黄额闭壳龟主要的消费国,多用作食物和医药。除此之外,分布局限、生境的破坏、不易繁殖和捕捉作宠物等问题也令黄额闭壳龟数量锐减。黄额闭壳龟的人工饲养十分困难,因此龟种适应环境能力欠佳,且十分神经质难以开食,提高了饲养的难度。日本第一个成功驯养和繁殖该物种的动物园是札幌市丸山动物园。而香港动植物公园亦曾饲养黄额闭壳龟。","text2":"日本第一个成功驯养和繁殖该物种的动物园是哪一个?","label":1} {"text1":"王季思(),是一位生于浙江温州永嘉梧埏上田村(今属瓯海区)的戏曲研究者。王季思为温州望族王澈之后。他自幼饱读经书,热爱观赏戏曲、小说,小学未毕业即考入浙江省立第十中学。1925年,他考入国立东南大学文学系,受业于曲学大师吴梅门下,从事戏曲学习和研究并深受赏识;吴梅在王季思的一次作业后批道:“自万里(赵万里)、雨亭(孙雨亭)、维钊(陆维钊)去后,复得斯才,我心喜极!”。大学毕业后,他先后执教于浙江省立第十中学、江苏省立松江女子中学。八·一三战争爆发后,他回乡投入抗日工作。后来,他先后在贵州和杭州浙江大学任教。其后,他长期执教于广州中山大学直至去世。王季思在批林批孔运动中被视为“反动学术权威”,并因此遭到迫害。其子为同样任教于中山大学的王则柯。《西厢五剧注》、《集评校注西厢记》、《桃花扇注》、《中国十大古典悲剧集》、《中国十大古典喜剧集》、《元杂剧选》、《元散曲选》、《中国戏曲选》、《全元曲选》、《王轮轩戏曲新论》、《王季思学术论著自选集》等。","text2":"1925年,王季思考入国立东南大学文学系,受业于曲学大师吴梅门下学习什么什么?","label":1} {"text1":"安达·俄松(,,藏语中意思是“光护”),又译为微松、欧松、沃松等。他是吐蕃王朝末任赞普朗达玛的儿子。俄松是朗达玛的遗腹子。842年,朗达玛被佛教僧人拉隆·贝吉多杰刺杀后,王妃綝氏(那囊氏)拥立哥哥尚延力三岁的儿子(一说是乞丐的儿子)乞离胡为赞普,王妃摄政。綝氏杀害了反对让云丹嗣位的大相韦·甲多热(结都那)等人,试图巩固云丹的赞普地位。俄松出生后,其生母为了避免遭到那囊氏的迫害,点灯日夜守护着他。后来不服云丹统治的大臣退往山南地区,立次妃所生的遗腹子俄松,与云丹分庭抗礼。847年,云丹以伍如(逻些小昭寺一带)为据点,俄松以云如(雅隆昌珠寺一带)为据点,双方的支持者发生大规模交战。这场被后世称为”伍约之战“的内乱持续了23年,整个青藏高原地区甚至连吐蕃辖下的西域地区都卷入了战乱之中。869年爆发了臣民大反叛。他们挖掘赞普的陵墓,杀害贵族和赞普的后裔,从此赞普一族的威权完全丧失。从此,吐蕃四分五裂,吐蕃王国名存实亡。在起义军的追赶下,俄松的后代逃往各地。后来的阿里王系(即古格王朝)、拉达克王系、雅隆觉阿王系、普兰王系、亚泽王朝都是俄松的后代。","text2":"安达·俄松的父亲是谁?","label":1} {"text1":"《黄金B段班》()是台湾中天娱乐台于2008年7月21日至2009年2月19日,每星期一至星期五(2008年11月7日起改为每星期一至星期四)晚间九点播出的谈话性综艺节目。该节目找来最当红的「B咖」主持,以艺人角度为出发点,探讨艺人辛苦在演艺圈中奋斗且不为人知的一面。「B咖」是在台湾演艺圈最近兴起的新名词,「咖」字在台语中与「脚」同音,与英文的B字组合起来后,意思就是二线演员、配角。「黄金B段班」在意义上具有黄金等级的B咖所开的班级。2008年7月21日,《黄金B段班》开播,主持人小钟、小马、沈玉琳、小甜甜、佩甄及韦汝开始推辞上9点谈话节目;模仿康康出名的黄镫辉,则要在《黄金B段班》中扮演综艺B咖中的B咖,立志作个有质感的谐星,要与陈汉典抢饭碗!《黄金B段班》企图开创另类谈话节目型态,加入类戏剧元素,给6位主持人合乎自己性格的角色,节目暂定八人挑梁主持,除小钟、小马、沈玉琳、小甜甜、佩甄及韦汝6人确定外,还有两名人选难产中。制作单位透露,会由戏剧界去寻找新面孔,缺乏一个美艳的谐星,还有可爱的傻大姊。 佩甄对新节目收视率有信心,「你看小钟负责客家族群,我负责带入婆婆妈妈收看,韦汝吸引年轻人,小马也有非洲支持者,8个人一个人只要负责0.25(%)的收视率,轻轻松松就能破2(%)。」有时候会以三男\/三女主持,会称另外三位为黄金三宝,加了口号『黄金B段班! 旺旺!』,每集片尾由沈玉琳独挑大梁的「沈语录」是该节目特色之一。





","text2":"《黄金B段班》特色之一是什么?","label":1} {"text1":"红天竺鲷(学名:),又称小凑天竺鲷,俗名大目侧仔,为辐鳍鱼纲鲈形目鲈亚目天竺鲷科的其中种。本鱼分布于印度太平洋区,包括红海、东非、马达加斯加、留尼旺、塞席尔群岛、模里西斯、葛摩、马尔地夫、圣诞岛、印度、泰国、越南、日本、台湾、澳洲、萨摩亚群岛、斐济、夏威夷群岛、法属波里尼西亚等海域。该物种的模式产地在马萨瓦、红海。水深8至35公尺。本鱼体延长而侧扁,眼大,口大略下位。鱼体略透明呈淡红色,侧线明显,尾鳍凹入,背鳍硬棘7枚;背鳍软条9枚;臀鳍硬棘2枚;臀鳍软条8枚,体长可达6公分。本鱼栖息于岩礁区,白天躲藏于岩架下或岩洞中,夜间出来觅食,属肉食性,以多毛类或其它底栖甲壳类为食。繁殖期时,雄鱼具有口孵习性,卵约7日化成仔鱼,由雄鱼吐出,具短暂的仔鱼飘浮期。可做为观赏鱼。","text2":"该鱼在繁殖期有什么习惯?","label":1} {"text1":"铁电性()是某些材料存在自发的电极化,并在外加电场的作用下可以被反转的特性。该术语被用于类比铁磁性,其中,材料表现出永久磁矩。当铁电性于1920年被Valasek在酒石酸钾钠中发现时,铁磁性就已经被知道。其英文术语的前缀\"ferro\",意思是铁,被用来描述属性尽管大多数铁电材料不含有铁。多数材料的极化是与外加电场成线性正比的,非线性效应是不显著的。这种极化叫做介电极化。有些称作顺电体的材料,其线性之极化效应更加显著。于是与极化曲线斜率相对应的介电常数为一个外加电场之函数。除了非线性效应外,铁电材料中还存在自发极化。铁电材料的不同之处在于它的自发极化可以在外加电场作用下被反转,产生一个电滞曲线。一般来说,材料的铁电性只存在于某一相应温度以下,称为居里温度。在这个温度以上,材料变为顺电体。铁电材料的非线性性质可以用来制造电容可调的电容器。一个铁电电容器的典型结构是两个电极夹一层铁电材料。铁电材料的介电常数不仅可以调节,而且在相变温度附近值非常大。这使得铁电电容器与其他电容器相比体积非常小。带有滞归特性的自发极化的铁电材料可以用来制造存储器。在实际应用中,铁电材料可以用来制造电脑和RFID卡。这些应用通常是基于铁电薄膜,这样用一个不太大的电压就可以产生一个强大的矫顽场。铁电材料内部的电偶极子与材料的晶格密切相关,于是材料晶格的变化将导致材料自发极化的变化。自发极化的变化将产生一个表面电荷。由此,在铁电电容器当中,即使没有外加电压,电流也会产生。改变晶格的两个因素是力和温度。外加的机械应力可以产生表面电荷的性质称作压电性,温度的变化导致自发极化的变化的性质称作焦电性。","text2":"什么是焦电性?","label":1} {"text1":"Syfy(原名Sci-Fi 频道)是美国国家广播公司(NBC)的一个有线电视频道,于1992年9月24日开播。Syfy专门播放科幻、奇幻、惊悚、超自然等电视影集,制作播放的著名影集包括《异形庇护所》(\"Sanctuary\")、《第十三号仓库》(\"Warehouse 13\")、2004年版本的《太空堡垒卡拉狄加》(\"Battlestar Galactica\")、《星际之门》(\"Stargate\")系列影集等等,亦有播出世界摔角娱乐所属节目SmackDown。Syfy一名于2009年7月7日开始采用。1989年,佛罗里达州博卡拉顿,沟通律师Mitchell Rubenstein和他的妻子Laurie Silvers想出了科幻频道的概念,并计划在1990年12月开始广播,但缺乏资源来实施。1992年3月,这个概念被收购,当时是派拉蒙电影与环球影业之间的合资公司。该频道被视为与经典电影和电视剧天生配合,这此作品在其两个电影制作公司的保险库中,包括环球影业的《'》、《科学怪人》和的电视剧《'》,以及派拉蒙电影的《星际争霸战》。《星际争霸战》的创作者金·罗丹贝利和作家以撒·艾西莫夫是最初的咨询委员会成员之一,但是在1992年9月24日终于启播的时候,他们都已经去世了。Rubenstein回忆说:「在放映上做的第一件事是『献给纪念以撒·艾西莫夫和金·罗丹贝利』。」伦纳德·尼莫伊是在曼哈顿的海登天文馆(Hayden Planetarium)举行该频道启播派对上的仪式主持。艾西莫夫的遗孀和罗丹贝利的遗孀都双双出席。电视网上广播的第一个节目是电影《星球大战》(1977年)。1994年,派拉蒙电影被出售给,然后在下年公司收购了松下电器产业株式会社所控制的股份公司(其中环球影业是一家子公司)。1997年,Viacom将其在USA电视台的股份出售给环球影业,环球影业在下一年便将所有的电视资产剥离至。三年后,迪勒将这些资产卖回环球影业, 当时是威望迪公司的子公司(当时被称为威望迪环球)。威望迪公司的电影和电视制作和有线电视资产随后在2004年与通用电气的全国广播公司(NBC)合并成NBC环球。频道的高清版本于2007年10月3日在DirecTV上推出。在2013年,Syfy被授予的。","text2":"1997年,Viacom做了什么举动?","label":1} {"text1":"陆军幼年学校,为旧日本陆军为了从幼年培养干部将校候补所设立的全住宿制教育机构。模仿普鲁士的军事学校()设立,毕业生拥有旧制中学校二年级毕业生的程度。1870年(明治3年)横滨语学研究所编入大阪兵学寮便是日本陆军幼年学校的开始。1871年(明治4年)大阪兵学寮被分开为陆军兵学寮与海军兵学寮,同年迁移到东京。1872年(明治5年)伴随著陆军兵学令的更改,陆军兵学寮幼年学舍改名为幼年学校。1874年陆军士官学校从兵学寮独立后,隔年幼年学校也改名为陆军幼年学校并从兵学寮独立。1877年并入日本陆军士官学校而暂时消灭。至1887年,随著陆军士官学校官制、陆军幼年学校官制的制定而再度设立。1897年时,因应军备扩张政策而增加培育人才数量,废除了陆军幼年学校官制,取而代之的是中央幼年学校条例与陆军地方幼年学校条例。东京为陆军中央幼年学校,另外于仙台、名古屋、大阪、广岛、熊本设立陆军地方幼年学校。每校各有约50名学生,中央幼年学校为14岁开始的2年,地方幼年学校为13岁开始的3年就读。陆海军士官的子嗣可享学费半价,战死者遗腹子学费全免。因为制服的领口有金星标志,也被称为「星的学生」。于校内学习军事学与普通学问,毕业生约占陆军将校的三分之一。被当时的人认为是陆军的旧制中学校。不过在当时陆军认为出身自文部省管辖的中学校,接受自由教育的人才不能信任,要想在陆军里出人头地除了日本陆军大学校毕业外,陆军幼年学校毕业也是重点。1918年(大正7年)制定陆军幼年学校令,陆军中央幼年学校与陆军地方幼年学校都改名为陆军幼年学校。1922年,由于华盛顿海军条约世界各国均开始缩减军备,同年大阪校废校。1923年名古屋校、1924年仙台校、1927年熊本校、1928年广岛校依序废除后,仅剩中央幼年学校。广岛校废除该年,中央幼年学校改编为日本陆军士官学校预科。1936年中国战线扩大所致,先是广岛幼年学校复校。之后1937年仙台幼年学校、1939年熊本幼年学校、1940年大阪幼年学校、名古屋幼年学校依次复校。大阪幼年学校移往离楠木正成居城很近的千代田村(现在的河内长野市),1940年4月1日时共150名44期生入学。太平洋战争结束后,日本陆军幼年学校与日本陆军士官学校一起遭到废除。","text2":"陆军幼年学校对哪些人有优待?","label":1} {"text1":"尼古拉·伊万诺维奇·叶若夫(,)苏联政治人物,秘密警察首脑。是斯大林大清洗计划的主要执行者之一,后在政治运动中失势被处决。叶若夫早年并不出众,30多岁时,是苏联中央组织分配处一个小事务官,但此后5年,他的仕途如电梯般上升。真正使其在历史上留名的,是其担任秘密警察组织——内务人民委员部(克格勃前身)领导时参与大清洗运动的经历。苏共十七大,叶若夫成为中央委员。两年后成为中央书记处书记、检查委员会主席,后受斯大林赏识进入内务部。从1936年到1937年在大清洗运动中,叶若夫主持下内务部逮捕了150万人,并处决了其中的半数。1938年1月9日,苏共党中央作出《关于错误地解雇因反革命罪被捕者家属这一事实的决定》,接着,全会一致呼吁“不要无充分理由就告人,要把犯错误的人和破坏分子区分开”。大清洗的方式开始由全面清洗转为定点清除。4月,叶若夫兼任水运人民委员,内务部实际由贝利亚负责。11月,叶若夫给斯大林写了封很长的忏悔信,请求宽恕。半年后,叶若夫“招供”了自己的罪行:“德国间谍、托洛茨基阴谋集团成员,谋杀斯大林”。1940年2月4日被处决,当时6岁的养女娜塔莉亚活了下来。苏联解体后他的养女娜塔莉亚要求为叶若夫平反,但在1998年6月4日遭到驳回。叶若夫在秘密警察机构待的时间和影响和其后来者贝利亚相比并不算长,但在克格勃(当时为内务部)领导中,叶若夫常被史学界拿来和其前任雅戈达相比,他们有着异常相似的出身、性格、仕途轨迹、罪恶甚至戏剧性的命运。在苏联人的印象中,他们都是残暴无天的人物。因叶若夫身高不足1米60,有个绰号“血腥的侏儒”。","text2":"1938年1月9日,苏共党中央的大清洗的方式有什么变化?","label":1} {"text1":"劝业会场始建于清光绪三十三年(1907年)五月,是中国天津历史上一座系统陈列工业工艺产品的场所,位于今天的河北区中山路中段东南侧,现为中山公园,原劝业会场大部分建筑已不复存在。清光绪二十八年(1902年)七月十二日,新任直隶总督兼北洋大臣袁世凯从天津都统衙门手中接回天津统治权并在海防公所(今金钢公园一带)建立直隶总督新署。清光绪三十一年(1905年)五月,袁世凯开始在北起大经路(今河北区中山路),南至金钟河(现今已被填平),东临昆纬路,西靠近北洋造币总厂的土地上建设一座占地两百余亩的综合性园林游乐场并将其命名为“劝业会场”。劝业会场作为当时中国最早系统陈列工业工艺产品的场所经常展览书画名家作品,清光绪三十一年(1905年)10月7日至10月14日,劝业会场举办了首届天津劝业展览会,到场观众约有15万人次,交易金额约为3万余元。1912年,劝业会场更名为“天津公园”,后又改称为“河北公园”。1920年冬,北京中国画学研究会在劝业会场的商业会议厅举办“中国南北方重要画家大展”,徐世昌、陈师曾、齐白石、吴昌硕、任伯年以及渡边晨亩、山田敬中、胜田焦琴、荒木十亩等的作品也同时展出。1928年北伐胜利后,为纪念孙中山,又更名为“中山公园”,今天天津中山公园的占地面积只为当年劝业会场占地面积的三分之一。劝业会场分为三个部分,分为商务区、公园区。其中,商务区设有前门两侧的平房店铺,公园区位于过街钟楼后面,公园区后侧还建有商品陈列馆、省立图书馆和咨议局大楼。劝业会场内的游乐设施有荷花池、月牙池、鹤亭、鹿亭、八角音乐亭和抛球房等,商业和办公设施建有劝工陈列所、教育品制造所、学办处、办公洋楼和茶房等。","text2":"劝业会场具体指什么?","label":1} {"text1":"仁义潭水库位于嘉义县番路乡八掌溪上游,属离槽水库,总容量达2,782万立方公尺,集水面积 3.66平方公里,坝顶长度1535公尺,宽度9公尺,有效蓄水量2506万立方公尺。年供水量为五千二百万公吨。 它是嘉义市及嘉义县民雄地区公共给水及工业用水的主要来源,并具有调节兰潭水库贮水量的功能。由于兰潭水库容量已不敷需求,因此台湾省政府于1979年度开始编列预算予以兴建,该水库于1987年8月完工正式蓄水,水源以引用八掌溪水,经沈淀池后再导入水库,所以是离槽式的水库,其取水口在台3线吴凤桥下游约50公尺处,设一拦水坝,经过1.5公里之引水道导入仁义潭水库,与兰潭水库串联运用,以供应大嘉义地区的民生用水。","text2":"仁义潭水库的集水面积是多少?","label":1} {"text1":"XEvil 为横向卷轴动作游戏,由Steve Hardt在麻省理工学院开发,属于自由软体。游戏模式以free-for-all对战为主,玩家互相对抗,或是对抗电脑。本游戏被评为\"有史以来最生动暴力的电脑游戏之一\",其暴力成份引发了不少争议。本游戏首建于1994年,现已存在多种不同平台的版本可供下载。The game is available for anyone, and leaves a large pool of characters to pick from. 如同多数的线上游戏,其故事线并不完整,而就其属的类别来说,其实也非必要。The game has been reviewed and被认为是暴力且具成瘾性,本质上却极为简单。\"XEvil\" 为第三人称二维横向卷轴游戏。随机生成关卡,玩家也用随机角色攻击敌人但是也可以通过梯子以及电梯过关。作为自由软件,能在世界各地各种不同的作业系统上进行游戏。游戏中还包含了\"Kill-Kill-Kill\"-模式,这个选项会让游戏变得更加困难。游戏的唯一目标是生存,可以单人或多人模式进行。\"XEvil\"背后的故事是,你是死后的灵魂,正被送往地狱。。。为了决定你在地狱中的位阶,你必须在死亡对战中竞争,来决定你的地位。玩家的角色为随机决定,各有各的优势跟劣势。角色有忍者,步行机器人,英雄,异型,螺旋桨人,杀手(由螺旋桨人制造),火恶魔,狗,雪怪,龙等等。\"XEvil\" been called by the online community \"fast action\"and \"is a must-have for anyone interested in straightforward mayhem.\"Finally the game \"features an incredibly addictive gameplay.\"GamezWorld, who gave the game a 7 out of 10, commented \"If you like little shooting orgys, you may like XEvil.\"重要开源游戏列表","text2":"什么是XEvil?","label":1} {"text1":"伯根-贝尔森集中营 (,又称Bergen-Belsen或Belsen) ,是纳粹德国在德国西北部下萨克森建立的一座集中营,该地毗邻伯根和策勒。1943年到1945年,统计约有50,000名苏联战俘和超过50,000名犯人死于其中。他们中超过35,000人死于1945年初流行的斑疹伤寒。该集中营于1945年4月15日被英国第十一装甲师解放。当时约有60,000名犯人尚在其中,绝大多数都已身患重病,另外还有13,000具未处理的尸体。〈由于二战末期纳粹陆续把欧洲各地集中营的犯人迁移到德国境内的集中营,导致各集中营极度拥挤,再加上当时斑疹伤寒极为严重,集中营内死亡人数增加迅速,党卫军来不及处理尸体,才导致尸体任意堆叠在营区内〉。BBC的随军记者理查德·丁布尔比这样形容当时的景象:英国准将看到该集中营的惨状时相当生气,拿元帅杖殴打投降的德国空军元帅艾尔哈德·米尔希,元帅杖也被打断。在伯根-贝尔森集中营里死去的15岁犹太女孩安妮·法兰克,她的日记成为二次大战期间纳粹消灭犹太人的最佳见证,日记中展现了惊人的勇气与毅力。","text2":"该集中营被哪国军队解放?","label":1} {"text1":"棘眦海蛇属(学名:),是蛇亚目海蛇科下的一种有毒单型蛇属,属下只有棘眦海蛇(\"Acalyptophis peronii\")一种海蛇,主要分布于南中国海、澳洲等海域。棘眦海蛇主要分布于泰国湾、越南、南中国海、印尼、菲律宾、新畿内亚、澳洲(包括新南威尔士、北领地、昆士兰、西澳州)等地区的海域及海岸。该物种的模式产地在澳大利亚。Duméril, ANDRÉ MARIE CONSTANT 1853. Prodrome de la classifcation des reptiles ophidiens. Mém. Acad. Sci., Paris, 23: 399-536","text2":"棘眦海蛇属属于哪一科?","label":1} {"text1":"本条目以编年体介绍香港政府各决策科(今决策局)于1973年设立至今的演变。1973年1974年1975年1976年1978年1979年1980年1981年1982年1983年1985年1988年1989年1990年1991年1993年1994年1997年1998年2000年2002年2007年2012年然而由于立法会拉布持续,特区政府未能于7月1日以前完成所有立法会程序,第四届特区政府最终以原有三司十二局上任。而五司十四局方案更因立法会换届而胎死腹中。(详情请参阅2012年香港立法会拉布攻防战、五司十四局及香港冗长辩论条目)2015年原商务局通讯及科技科更名为通讯及创意产业科,继续主管通讯及创意产业政策。2018年","text2":"2015年原商务局通讯及科技科更名为?","label":1} {"text1":"藏霞精舍(Chong Har Ching Ser)位于香港新界粉岭的黄岗山(现粉岭市中心),属广东省清远市的先天道派之藏霞洞的分支,由朱翰亭先生所创立,为一所道观,以苦练修行为办道宗旨,供奉观音、吕洞宾、北帝、关羽、三清道祖、黄龙真人及孔子。每年之三元节及盂兰节为主要庆典。道观旁边的祥华邨原来计划以所在地命名为「黄岗山邨」,后有意见指此名容易让人联想起1911年的黄花岗起义,房屋署改为以藏霞精舍「藏霞」二字的谐音,为该新建屋邨取名「祥华邨」。在1920年创立,现在的建筑属1950年代。从外面看,建筑应为小型的二进四合院格局。与广东省清远市的藏霞洞道观,佛山市的飞霞洞道观及南洋的先天道有紧密联系,交流互访。","text2":"藏霞精舍还与哪些道观有联系?","label":1} {"text1":"第三次靳云鹏内阁是第二次靳云鹏内阁总辞之后靳云鹏重新组织的内阁,成立于民国10年(1921年)5月14日,结束于同年12月24日。靳云鹏的此次内阁排斥了旧交通系的势力,但是他安排的财政总长继任人李士伟由于曾经在日本洋行做过买办,招致各方的强烈反对而未能赴任,一开始就由次长潘复代理,后来更正式免去李的职务,任命高凌霨为财政总长;而留任的教育总长范源濂以学潮为由也没有到任,由马邻翼代理。由于安福国会已被解散,此时中国没有国会,所以此届内阁的国务总理和阁员的任命都没有经过国会的批准,所以徐世昌大总统命令内阁署理行之。内阁成员一律带有「特任」的字样。正因此此届内阁的合法性受到质疑。靳云鹏在第二次内阁时得罪了旧交通系的领袖梁士诒,梁士诒于是决意扳倒靳云鹏,他从张作霖和徐世昌两方面分化总统府、政府、军阀的关系。靳云鹏因为未能满足张作霖安插亲信的要求而和张作霖闹翻,后来又因为冷言讽刺徐世昌总统和北京税收问题而和徐世昌对立。最终,靳云鹏被迫辞职,梁士诒在张作霖推荐下继任总理。","text2":"为什么此届内阁的合法性受到质疑?","label":1} {"text1":"巨颏虎(\"Megantereon\"),又名巨剑剑齿虎或巨剑齿虎,是古代的剑齿虎,斯剑虎的祖先。巨颏虎的化石碎片是在非洲、欧亚大陆及北美洲发现。巨颏虎唯一完整的骨骼是在法国发现。估计巨颏虎是生存于300-90万年前。巨颏虎的体型像现今的美洲豹,大小如大型的豹。牠们的前肢粗壮,而下前肢有如狮的大小。牠们的颈部肌肉强壮,一咬可以致命。巨颏虎并非以其剑齿来咬死猎物,因为其剑齿并不够坚硬。牠有可能会咬伤猎物,待猎流血致死。不过这样牠需要保护猎物,免受其他掠食者抢夺,但其保护策略就不得面知。现时一般认为巨颏虎会像其他剑齿虎般利用剑齿来割破猎物颈部大部份神经及血管。但是当猎物挣扎时,巨颏虎的剑齿仍有破损的风险。巨颏虎的物种数量并不清楚,因为大部份的化石都只是碎片。一些学者指巨颏虎真正的物种数量是较以下的少:.","text2":"巨颏虎唯一完整的骨骼是在哪发现的?","label":1} {"text1":"吉冈秀隆(),日本男演员、创作歌手。出生于埼玉县蕨市。血型为A型。亚细亚大学经济学部二年级肄业。身高172公分,体重55公斤。5岁进入若草剧团,75年参与东京电视台时代剧《大江戸捜查网》的演出。1977年出演野村芳太郎监督的『八つ墓村,饰主人公・寺田辰弥的幼年时代。之后在1980年公映的山田洋次监督的『远山的呼唤』中演出女主角的儿子“武志”一角,继而成为山田洋次的长年御用演员之一。1981年开始出演山田洋次监督的长篇系列电影『男人真命苦』),从第27部开始一直到最终章出演主人公寅次郎的外甥“诹访満男”一角,并从第42部开始,因为主演渥美清的身体欠佳,情节作以调整,寅次郎的外甥满男和满男的女朋友阿泉也成为故事的主角,和寅次郎展开了双线的情节。直至1996年渥美清病逝,『男人真命苦』也于第48部完结。在电视剧方面,从1981年开始,在富士电视台的长寿经典名作『来自北国』(日文名“北の国から”,仓本聪脚本)中饰主角之一的黑板纯,之后20余年中,一直持续拍摄特别篇,直至2002年『遗言篇』终于完结。遂后也与『遗言篇』中饰演黑板纯女友的女优内田有纪共结连理,并在『来自北国』所在的外景地北海道富良野市举行婚礼,遂成一段佳话。但由于婚后两人聚少离多,终究于2005年12月21日协议离婚。2003年首次主演民放台趋势剧,富士电视台的小孤岛大医生,得到了极高的视聴率与口碑,并在接下来相继推出了2004的特别篇与2006年的续篇,也都获得了极高的视聴率。而在2005年主演的电影「永远的三丁目的夕阳」中,因良好口碑与票房也于2007年续拍,而他也凭精湛的演技创下以同影片同角色两度拿下日本电影金像奖最优秀主演男优赏的记录。从童星时代出道至今,除了演员发展,音乐才华亦受瞩目,94年配合同名电影推出第一张自己做词作曲的单曲『ラストソング』(Last Song),于当年畅销47万张,其后他所推出的几张专辑几乎都一人包办了词曲的创作。","text2":"吉冈秀隆在1980年公映的山田洋次监督的『远山的呼唤』中演什么角色?","label":1} {"text1":"第三次靳云鹏内阁是第二次靳云鹏内阁总辞之后靳云鹏重新组织的内阁,成立于民国10年(1921年)5月14日,结束于同年12月24日。靳云鹏的此次内阁排斥了旧交通系的势力,但是他安排的财政总长继任人李士伟由于曾经在日本洋行做过买办,招致各方的强烈反对而未能赴任,一开始就由次长潘复代理,后来更正式免去李的职务,任命高凌霨为财政总长;而留任的教育总长范源濂以学潮为由也没有到任,由马邻翼代理。由于安福国会已被解散,此时中国没有国会,所以此届内阁的国务总理和阁员的任命都没有经过国会的批准,所以徐世昌大总统命令内阁署理行之。内阁成员一律带有「特任」的字样。正因此此届内阁的合法性受到质疑。靳云鹏在第二次内阁时得罪了旧交通系的领袖梁士诒,梁士诒于是决意扳倒靳云鹏,他从张作霖和徐世昌两方面分化总统府、政府、军阀的关系。靳云鹏因为未能满足张作霖安插亲信的要求而和张作霖闹翻,后来又因为冷言讽刺徐世昌总统和北京税收问题而和徐世昌对立。最终,靳云鹏被迫辞职,梁士诒在张作霖推荐下继任总理。","text2":"第三次靳云鹏内阁成立和结束于哪一年?","label":1} {"text1":"陈衣凡(),又名家贵,中华民国空军将领。1912年10月18日出生于奉天海城(今辽宁鞍山)。中学期间适值九一八事变而决定投身军旅,1933年进入黄埔军校10期工兵科。后受孙逸仙“无空防即无国防”之主张,于1934年以绩优选送中央航空学校第五期甲班轰炸科。1936年以优异成绩毕业,后成为空军第13中队飞行员。1938年任第八大队第14中队分队长,5月19日23时半中国空军远征日本本土散发传单,陈衣凡驾中国空军仅剩最后一架He111A轰炸机担任后勤支援运输。以后复入空军参谋大学。1945年赴美国恩尼德航校、柏格斯川航校、同盟国三军联合计划作为学校、美国海军高级军官两栖作战班。1946年回国,入实践研究院联战班及三军大学战争学院将官班,钻研中西军事理论,遂任空军第2大队副大队长。1948年4月16日任第1大队副大队长,接受加拿大乔治·司徒华(George E. Stewart)教官的带飞训练,成为第一位放单飞的蚊式机飞行员。以后任大队长。到台湾后,任台中基地指挥官。1952年任空军总部第2处处长。1953年派任驻菲律宾空军武官。1957年任空军训练司令部参谋长。1959年任空军作战署署长。1964年任空军总部人事署长暨国防部人力司司长、空军作战司令部司令。1968年3月任空军总部参谋长、1969年7月任空军副总司令。1970年6月29日为中将空军第六任总司令,7月3日晋升空军二级上将。1975年6月任总统府战略顾问,驻约旦大使。2008年2月6日3时20分因肺积水、发炎与器官衰竭病逝于台北市内湖三军总医院,享寿96岁。3月9日10时在三军总医院万安怀德生命纪念馆隆重举行公祭,由空军司令彭胜竹上将代表国防部长蔡明宪前往致哀,由总统府战略顾问陈燊龄一级上将担任主祭官,前空军总司令乌钺上将率治丧委员会,前行政院长郝柏村、前退辅会主委许历农、前国防部长李天羽以及立法委员蒋孝严等党政军要员,前往现场吊唁。由赵知远、林文礼、黄显荣、夏瀛洲等四位退役上将为灵柩覆盖党旗,由总统府战略顾问沈国祯上将、空军司令彭胜竹上将、联勤司令金乃杰上将、副参谋总长彭进明上将担任国旗覆旗官,以表彰其一生为空军军事训练及中华民国军事外交的卓越贡献。告别式后发引安厝于台北碧潭空军烈士公墓。夫人朱以苹,朱佛定长女。","text2":"为什么陈衣凡会投身军旅?","label":1} {"text1":"《恶女花魁》()是安野梦洋子于2001年起连载的漫画,现已停止连载。于2007年由蜷川实花拍摄成电影,演员一字排开土屋安娜、椎名桔平、成宫宽贵、木村佳乃、菅野美穗、小泉今日子、安藤政信、永濑正敏各个都是顶顶有名的大人物,再加上苹果歌姬-椎名林檎的配乐之下,成功的刻划出江户时期恶女花魁的传奇。「不知怎的,我就是会,我就是懂得怎么看男人才会使他们兴奋。」—— 清叶清叶在八岁(幼时小池彩梦饰)时,就被卖到游廓。起初是在花魁粧妃(菅野美穗饰)旁服侍,并学习如何成为花魁,而在花魁粧妃出嫁后,便把发簪象征性的传承给清叶。长大后的清叶(土屋安娜饰)不负粧妃的期待成了花街众所瞩目的焦点。但是自己却被迷恋许久的恩客抛弃,让她对爱情彻底失望,并发誓再也不要爱上男人,转而全心全意的投入工作,把服务男人当成一种技术,不久后她当上了花街上的首席-花魁。但是在她遇上武士-仓之助(椎名桔平饰)后,生活渐渐起了变化,而仓之助也趁势向她求婚,让清叶又惊又喜,但是却也害怕再次受到伤害,因此她说:「当花街下起樱花雨,我就嫁给你!」隔天,她被妈妈桑的声音吵醒,打开窗户,却见到令她永生难忘的美景-樱花雨。原来是仓之助将他处的樱花树全移到花街前,让这条街下起了缤纷的樱花雨。正当所有人都为了眼前的美景暂歇脚步之时,清叶在窗边流下了眼泪,不仅是感伤幼时离开家乡后就再没见过樱花雨,更重要的是也被仓之助所做的一切所感动。看来清叶非嫁不可了!游廓里的鸨母准备好华丽的嫁妆和衣裳,准备让她快乐的出嫁成为武士夫人。但,就在出嫁前一晚,她找到了她生命中的真命天子……原声带在电影上映前就已经发行了,而这张专辑也是椎名林檎暌违四年后所发行的专辑。","text2":"清叶在几岁被卖到游廓?","label":1} {"text1":"树皮是木质植物,例如树的茎和根最外面的部分。狭义的树皮包括三层:木栓、木栓形成层和栓内层,以及外部的各种死组织,广义的树皮还包括韧皮部。有的植物的树皮中含有各种生物碱、单宁、染料和香料等,可以提炼各种药材、毒品、毒药、树脂等,也可以用某些种类的树皮做软木、绳索、织布、造树皮船、绘制树皮画等,或直接用树皮作装饰。由外向内,树皮可分为外表皮、由木栓、木栓形成层和栓内层组成的周皮以及内里的韧皮部。外表皮是树木最外部的死组织,由角质化的细胞组成。周皮是韧皮部和外表皮之间的部分,包括木栓、木栓形成层和栓内层的总称,周皮形成后,表皮即脱落。木栓是树皮外层的主要成分,能隔绝水分和气体通过,对树有保护作用。木栓形成层通常只有一层或两层细胞,是分生生长木栓的组织,向外生长成木栓层,向内形成栓内层,不过在根部的木栓形成层是由中柱鞘转变的。栓内层是木栓形成层向内部分化出的一层细胞。韧皮部在木质部的树干和周皮之间,是树皮内部输送营养的部分。随着树皮逐渐生长加厚,外层组织逐渐死亡,狭义的树皮只包括木栓和外部的死组织。树皮的各个部分都有不同的用途,木栓的质地轻、富有弹性、不透水,木栓发达的树种如栓皮栎的树皮,可以用来制作瓶塞、救生圈、隔音板等,在北美洲和中国东北用桦树皮制作小舟和器物。有些树皮甚至可以食用,最有商业价值的树种是金鸡纳树和肉桂树,可以提炼药物奎宁和香料, 阿司匹林是从柳树皮中提炼的;夏栎(\"Quercus robur\")树皮是揉革用的单宁酸的主要原料;在园艺中,经常用树皮碎屑培育兰花等不能在土壤中生长的附生植物花卉。许多种类的昆虫、真菌和苔藓,附生在树皮上 。不同树种的树皮形状、色泽、瘢痕及脱落情况都不相同,树皮可以作为鉴定树木种类、年龄的重要依据。树木在树皮受伤后,可以自己生长出多余的木栓来修复。","text2":"狭义的树皮指的是什么?","label":1} {"text1":"金刚(King Kong)是电影中的虚构怪兽,一种巨大的猩猩,最早出现在1933年的电影《金刚》之中,之后1976年及2005年各有一部同名的重拍电影,日本和香港邵氏的电影公司也拍过相关题材的作品。目前「金刚」的角色使用权在环球影业。「金刚」最早由美国电影制片创造,在1933年的原作中,这个生物的名字其实是「刚」(Kong),是骷髅岛上原住民对牠的称呼,「金」(King,王者、国王之意)是片中人物将牠捕获、并带到纽约展示时加上的头衔,然而Kong就是丹麦语的国王之意,与King相同字源。<\/nowiki>身长:30m体重:未知登场电影:2017年《金刚:骷髅岛》\/2020年《金刚大战哥吉拉》","text2":"目前'金刚“这个角色的使用权在哪家公司?","label":1} {"text1":"李嗣昭(),本名韩进通,字益光。汾州大谷县人。后唐太祖李克用的养子。本是农民的儿子。李克用出猎,落脚韩家,发现周围树林之中竟郁郁有气,非常奇怪,便叫来主人询问。回答说家中刚生下一子。李克用以为此子有富贵气象,就买下婴儿,让其弟李克柔养以为子,短小精悍,年轻时好饮酒,克用命其戒酒,遂终身不饮。为衙内指挥使。光化三年(900年)出兵山东,攻取洺州,朱温亲自率军讨战,嗣昭退兵。天祐三年(906年)李嗣昭同周德威与后梁军队大战于潞州,双方坚持一年有余,德威战死,唐庄宗率银枪军快速登上无石山,以迷惑后梁军,李嗣昭则从侧面发动快攻,大破后梁军。天祐十九年(922年),唐庄宗在望都被契丹包围,晋军出入多次,尚不能解,李嗣昭率三百骑兵突围救出庄宗,秃馁的部队退走。晋军奋力攻击,追到易州,获毡裘、毳幕、羊马不可胜计。四月二十四日,嗣昭代替阎宝进攻后梁镇州张文礼时,头部中箭,嗣昭忍痛拔出,再以该箭射死敌兵,返回营地后,因伤口血流不止,当夜即去世。庄宗即位后,追赠太师、陇西郡王。有子八人,长曰李继俦,泽州刺史;次李继韬、李继忠、随使马军都指挥使李继达、相州刺史李继能、潞府左司马李继袭、随使马步军都虞候李继远(小名定哥)、潞府右司马李继镕,除李继达、李继镕外,皆夫人杨氏所生。杨氏善于经商,致家财百万。后晋年间,追封李嗣昭为韩王。","text2":"李嗣昭年轻时喜好做什么?","label":1} {"text1":"家铉翁(1213年-?)号则堂,眉山(今属四川)人。先祖自唐末迁居入蜀,是眉州望族。宋理宗时以荫补官,官常州(今江苏武进县)知县,政誉翕然,迁浙东提点刑狱,入京担任大理少卿。咸淳八年(1272年),权知绍兴府、浙东安抚提举司事。德祐二年(1276年),赐进士出身。是年二月,元军攻临安,丞相吴坚、贾余庆发文命各地献城投降。铉翁独不降。后奉诏出使元朝,被强留馆中。闻知宋亡,日夕哭泣,不仕,自号则堂,被羁縻于河间(今属河北省),以《春秋》教授弟子,前后长达十七年之久。元成宗即位(1294年),赐衣服并放还,年纪已八十多岁。卒于家。著有《说易》、《春秋序例》、《孝经解义》等书皆失佚。另有《则堂集》二十卷,今存《永乐大典》本六卷。","text2":"家铉翁是哪里人?","label":1} {"text1":"开角龙亚科(Chasmosaurinae)又名角龙亚科(Ceratopsinae),是角龙下目角龙科的一个亚科,角龙科的另一个亚科是尖角龙亚科。开角龙亚科是群大型四足动物,前肢略弯,短于后肢。牠们的体型壮硕,四肢粗壮。头骨后方的头盾大,由顶骨、鳞状骨构成,头盾长度可相当于头骨本身。喙骨与前齿骨构成长、狭窄的喙状嘴。牠们具有多列的齿系,会不断地生长、取代磨损的牙齿。开角龙亚科与尖角龙亚科的最明显差异在于头盾与角的形状。与尖角龙亚科相比,开角龙亚科的鼻角较短、额角较长、头盾较长、头盾具有大的洞孔。但是,三角龙的头盾短,头盾没有洞孔。五角龙与牛角龙的头盾可达2公尺长,是已知具有最大头部的动物。头盾周围的颈盾缘骨突(Epoccipital)短小、或没有,与尖角龙亚科不同。如同其他角龙科,开角龙亚科生存于白垩纪晚期的北美洲西部。化石发现于坎潘阶与马斯垂克阶地层,年代为8,000万到6,500万年前。开角龙亚科是在奥塞内尔·查利斯·马什(Othniel Charles Marsh)在1888年建立。其名称来自于角龙属,但角龙的化石过于稀少,目前状态为疑名。因此劳伦斯·赖博(Lawrence Lambe)在1915年建立开角龙亚科(Chasmosaurinae)以取代角龙科。三角龙的头骨较短,产生分类学上的争议,较晚期才被归类于角龙亚科。","text2":"开角龙亚科在外形上有什么特征?","label":1} {"text1":"《霎时感动》(英文:),是香港电视广播有限公司的一个励志资讯节目,为了该台2009年香港力量行动而设的一个节目,第一辑于2009年2月2日至2010年3月19日逢星期一至五晚上在翡翠台及高清翡翠台播映,共222集。第二辑于2011年2月14日至2011年3月25日逢星期一至五晚上在翡翠台及高清翡翠台播映,共30集。第三辑于2013年5月13日至2013年10月11日逢星期一至五晚上在翡翠台播映,共59集。节目同时于tvb.com提供足本重温。每集约4分钟,每集献上一个饶有寓意、发人深省的励志性小故事,带出人生哲理,由艺能界、商界、政界主持分享内心体会和感受。遗忘城市生活急促步伐,走过营营役役的一天,找回片刻属于自己的感动,从感动中释怀。在2009年12月4日举行的《万千星辉颁奖典礼》中,此节目获得最具欣赏价值大奖。节目亦以原名出版成书。","text2":"《霎时感动》是为什么而设的一个节目?","label":1} {"text1":"马良弼(;)号汝舟,和名名护亲方良丰(),琉球国第二尚氏王朝时期大臣。童名太良金。他是马世荣(名护亲方良员)之子,马良诠(名护亲方良意)之孙。1551年生于首里城。1579年,为进贡及谢册封尚永王之恩事,以王舅的身份,与长史郑迵(谢名亲云上利山)一起出使明朝。在北京期间,萧崇业、谢-{杰}-赠送给马良弼「国佐元勲」四字匾额。归国后,因功升三司官座敷。1592年,其父马世荣退隐。因兄长马良辅(伊计亲方良真)早卒,马良弼遂以次子的身份继承家督之位。同年就任三司官,领名护间切总地头。当时日本逐渐强大,岛津义弘致书琉球,声称日本要出兵朝鲜,要求琉球提供粮草支援。马良弼与翁寄松(城间亲方盛久)都认为琉球弱小、日本强大,主张对日本和好,同主战派的郑迵、向里瑞(浦添亲方朝师)意见相对。1605年,翁寄松在与郑迵的政治斗争中失势,尚宁王转而倾向对日强硬的态度。1606年,册封使夏子阳、王士桢到达琉球,赠送给马良弼「世承天宠」四字匾额。1609年,萨摩藩派遣桦山久高、平田增宗率兵入侵琉球。尚宁王得知消息后,派马良弼、毛凤朝、菊隐、喜安等人前往北山。马良弼在此期间的行为,各史料是矛盾的。一说马良弼率一千名首里亲军前往北山防御,但中萨军伏兵之计,兵败被俘;马良弼仰天痛哭而不投降,因此受到萨军的尊敬。而另一种说法是马良弼前往北山的运天港,至萨摩军中请罪,并留在萨摩军中作人质,引导萨摩军开入那霸港。萨摩军包围首里城后,尚宁王向萨摩投降,并先至马良弼府邸暂时居住。尚宁王等人被掳至鹿儿岛时,马良弼同毛继祖(丰见城亲方盛续)被留下,一同管理首里城。1611年尚宁王归国后,马良弼继续担任三司官之职。1612年(万历四十一年),再以谢恩王舅的身份出使明朝,并就进贡之事同明朝交涉。翌年归国。马良弼于1614年致仕,由毛继祖继任其职。1615年,尚宁王向明廷报称马良弼因交涉进贡一事失败而遭到处决;事实上,根据《中山世谱》的记载,马良弼并未遭处死,翌年出使萨摩藩,告知赴明谢恩完竣之事。1617年逝世。","text2":"1579年为进贡及谢册封尚永王之恩事,马良弼以什么身份与长史郑迵(谢名亲云上利山)一起出使明朝?","label":1} {"text1":"企业定位安天是一家反病毒引擎和解决方案厂商,提供检测PC恶意代码和移动恶意代码的顶级反病毒引擎和下一代反病毒服务。安天成立于2000年。在过去的15年,安天始终专注于反病毒领域,致力于向合作伙伴提供最好的反病毒引擎和解决方案,对用户需求有着深刻的理解,并积累了丰富的经验和技术。安天还对重大安全威胁作出高效的应急响应,并积极参与学术界和产业界活动。核心产品安天的主要产品是名为AVL SDK的反病毒引擎中间件,可以用于检测PC平台和移动平台的恶意代码、广告件和间谍件。AVL SDK的用户可以轻松地将它集成到自己的网络设备产品、软件或移动应用中,立即获得顶级的反病毒能力。AVL SDK可以被移植到不同的硬件平台,并适应不同的网络环境和计算能力;它对恶意代码的检测能力也已经得到权威测试和学术研究的验证。目前,AVL SDK已经被美国、日本和中国的多家安全企业采用,运行在数万台网络设备和近千万台移动设备之中。安天还提供下一代反病毒服务,包括开放的恶意代码云检测、恶意代码知识百科、后端自动分析系统、按需人工分析和响应等,从而协助引擎的用户建立应对恶意代码相关威胁的综合能力。技术研究安天坚持在反病毒和信息安全领域进行探索和研究。在过去的十余年,安天在恶意代码检测等方向发布了大量技术文献,参与了多个信息安全开源项目,并在各类学术和产业会议上做技术报告。安天加入了AVAR、MUTE等组织和MAPP等计划,并与全球的多所高校和研究机构建立了合作关系。应急响应安天的CERT团队为公众提供重大恶意代码和安全事件的应急响应服务。在历史上,安天率先发现了红色代码II、口令蠕虫等恶意代码,对冲击波、魔波、震荡波、震网、火焰、Adrd、CarrierIQ等恶意代码或PUA也提供了深度的分析报告和有效的应对方案。地域分布安天大本营位于中国哈尔滨,在北京、武汉、深圳等多地设有研发中心,在全国多个城市有分支机构。团队文化安天团队以工程师文化为主导,以“创造就是我们的脚步”为信念,以带给用户“信息安全每一天”为使命,崇尚安全工程师的责任感和荣誉感,崇尚彪悍、协作、民主、快乐的团队风格。","text2":"安天实验室坚持什么样的理念?","label":1} {"text1":"野利仁荣()为西夏开国重臣,是西夏景宗李元昊皇后野利氏的疏族。相传为西夏文创始人,于大庆元年(1036年)秉承景宗旨意创造蕃书(西夏文),成十二卷,行体方整,但笔划繁冗。(《宋史》卷四百八十六 列传第二百四十四记载:“元昊自制蕃书,命野利仁荣演绎之,成十二卷,字形体方整类八分,而画颇重复。”)天授礼法延祚二年(1039年)建蕃学,野利仁荣受任主其事。(《宋史》卷四百八十六 列传第二百四十四记载:“野利仁荣主蕃学”)野利仁荣死于延祚五年(1042年),赠富平侯。天盛十四年(1162年)西夏仁宗李仁孝为表其封制蕃字之功,追赠为广惠王。(《宋史》卷四百八十六 列传第二百四十五记载宋绍兴三十二年(1162年)西夏国仁宗“始封制蕃宇师野利仁荣为广惠王”。)。","text2":"野利仁荣是什么人?","label":1} {"text1":"凹间(床之间)是日本住宅里叠席房间(和室)的一种装饰。在房间的一个角落做出一个内凹的小空间,主要由床柱、床框所构成。通常在其中会以挂轴、生花或盆景装饰。凹间和其中的摆饰是传统日本住宅内部必备的要素。凹间最早出现在安土桃山时代(14至16世纪)。一开始可能起源于禅宗僧侣用的佛坛,用来放置供奉物品,现代在日式房舍中的用途则为陈列插花与画轴,以营造茶道的氛围,或是烘托其他艺术品。。凹间前方的座位被视为「上座」,因此正确的礼节是必须安排最重要的宾客背对凹间而坐。这样的礼仪是出于谦虚,主人不应该被认为要向宾客炫耀或展现凹间中的装饰,因此绝不可安排宾客面向凹间而坐。而凹间也是严格禁止踏入的。在江户时代,一般平民建造凹间被认定是禁止的奢侈行为,而明治时代之后在和室设置凹间就变得十分普遍。到了近代由于悬挂字画挂轴的习惯减少,有许多的和室并没有建造凹间。就算是原本设置有凹间的和室,也有许多被改建为柜子使用,甚至是摆放小型佛堂或祖先牌位用。一些日式旅馆的凹间也被作为放置电视或保险箱的地方。","text2":"按照正确的礼节,最重要的宾客的座位通常被如何安排?","label":1} {"text1":"玉珊瑚又称为圣诞果科别:茄科学名:Solanum pseudo-capsicum L别名:珊瑚豆,红珊瑚,珊瑚樱,野辣茄,耶路撒冷樱桃原产地:欧亚大陆和热带及亚热带地区,巴西马得拉岛花期:全年均能开花,但以春季较开,约2至5月左右。花色:白色玉珊瑚又叫"樱桃椒",茄科长绿灌木,因为果实酷似樱桃而得名。由于玉珊瑚果实通常在圣诞节时由绿转红,故又称圣诞果!珊瑚的植株多直力生长,具有很多分枝。幼体深绿色;老枝灰褐色。叶子矩形,边缘长有疏松的锯齿。每年7~8月间开花,花色白色。果实为球形的浆果,直径2CM左右,9~12月成熟,成熟时由绿色变为橙红色,可以长时间地停留在枝条上。玉珊瑚由于花期长,因此主要用为观赏植物(果实从结果到成熟落果可持久达三个月以上),玉珊瑚的果实浑圆晶莹,玲珑可爱,为观果之珍品,适合庭植缘栽,盆栽,果枝作花材。台湾省平地至低海拔山区普遍栽植,部分山区或荒地甚且有归化现象。","text2":"玉珊瑚为什么主要用为观赏植物?","label":1} {"text1":"埃及圣鹮(学名:),又名埃及圣朱鹭、埃及圣鹭、圣鹮,是撒哈拉以南非洲、伊拉克东南部及以往埃及的一种朱鹭。牠们在埃及备受尊敬,经常被制成木乃伊当做托特的象征。牠们也被引入到法国、义大利、西班牙及美国。现在,在台湾西部滨海地区也可看到牠们。保守估计全台湾约有1200至1500只的族群量。埃及圣鹮成鸟长68厘米,身体全白,后羽黑色。头部及颈部都是秃及黑色的,喙厚而弯,脚也是黑色的。飞行时双翼张开后边有一道黑色羽毛。雄鸟及雌鸟相似,雏鸟呈灰白色,喙较少,颈上有一些羽毛。埃及圣鹮在树上筑巢,会与其他水鸟(如鹭科)一同生活。牠们很多时会在猴面包树上以树枝筑巢,每次产2-3颗蛋。埃及圣鹮在沼泽湿地及泥泞出没,有时会到访农地及垃圾站。牠们主要吃鱼类、青蛙及其他水中生物,包括昆虫与螺类。埃及圣鹮很静,但有时也会发出一些哇声。引入到南欧的埃及圣鹮急速破坏了燕鸥等动物的栖息地,且与牛背鹭及白鹭竞争筑巢的地方。牠们很能适应环境,在冬天的时候会吃垃圾来补充牠们的食物。埃及圣鹮是《非洲-欧亚大陆迁徙水鸟保护协定》中所保护的物种之一。在古埃及,埃及圣鹮倍受尊崇,且会被制成木乃伊,成为托特的象征。希罗多德及老普林尼都指埃及圣鹮可以对抗蛇。另亦有指只要将埃及圣鹮作为祭牲,就可以杀死带来瘟疫的苍蝇。","text2":"埃及圣鹮是什么文件中所保护的物种之一?","label":1} {"text1":"矢幅车站()是一由东日本旅客铁道(JR东日本)所经营的铁路车站,位于日本岩手县紫波郡矢巾町又兵卫新田。矢幅是JR东日本东北本线沿线的一个车站,管辖上位于JR东日本盛冈支社的范围之内,是个由盛冈车站管理,并委由JR东日本子公司Jaster()所经营的业务委托车站,设有绿窗口()。矢幅车站虽然不是新干线的停车站,但站体却是与东北新干线的高架路线整合在一起,其车站东口就位于新干线的高架桥之下。目前的跨线式站体(桥上站舍)是在2008年时全新启用的。矢幅车站是距离岩手县知名的律令时代城栅遗迹、德丹城遗迹最近的车站,已被日本指定为国家史迹的该遗迹,目前是被作为矢巾町历史民俗资料馆用途。侧式月台1面1线与岛式月台1面2线,合计2面3线的地面车站,设有跨站式站房。","text2":"矢幅车站位于何处?","label":1} {"text1":"书香文化节为澳门大型书展活动之一。由澳门出版协会主办,从2005年起,每年分春、秋两季举办两次书展。在2005年以前,澳门主要有三间书店每年举办大型书展,包括星光书店的阅读文化节书展、澳门文化广场的两岸四地(澳门)书展及一书斋的书市嘉年华,当时阅读文化节书展及两岸四地(澳门)书展皆不定期、举办地点亦各有不同,只能靠宣传才能知道在甚么时候举办。直到现在的主办单位澳门出版协会成立以后,才由2005年开始定期举办「书香文化节──两岸四地(澳门)书展2005」,每年书展始分为春、秋两季进行。春季书展于每年4月举行,由星光书店统筹承办,于澳门理工学院体育馆举行;而秋季书展则于每年10月举行(早年由于展场问题延于12月举行),由澳门文化广场统筹承办,于澳门塔石体育馆举行。2016年秋季开始,变更名称由「书香文化节──两岸四地(澳门)书展2016」改为「书香文化节──2016澳门书展」,以后沿用新名称。书香文化节为澳门春、秋两季澳门的大型书展。因澳门的书展并非由政府全资主办,而是由个别非牟利组织申请支助或个别书店主办,而且澳门的出版社数量也较少,因此即使是澳门的大型书展,规模上与周边地区的书展相距甚远。展出的书籍则是集合本土及周边各地区的中、英、葡文书籍,因此两岸四地同时会有出版商参展,尤以香港出版商居多,为周边地区的出版商之间提供了一个业务交流的机会。不少澳门出版书籍都会选择在书展时首发,主办单位有时还会邀请这些首发书籍的作者出席签名会、亦会邀请两岸四地知名作家及知名人士出席签名会。由于是免费入场,而且购书亦可获优惠折扣(一般是八折或九折)或特价优惠,因此每年都会吸引大批书迷到场选购书籍。据主办单位表示,过往书展约有四至五万人次参与。弘扬中华文化;推动本澳阅读风气,鼓励持续进修、终身学习,从而达至提升本澳居民的文化素质;促进两岸四地图书出版事业的交流互动。书香文化节主要由主办方面提出各项主题活动,吸引各界文化爱好者到场参观及交流。如有:每届书展均会请来新书首发人士发表作品相关的介绍及写作分享,以及邀请文化界及知名人士与读者对话。","text2":"澳门有哪三间书店每年都举办大型书展?","label":1} {"text1":"陈襄(1017年-1080年)字述古,福州侯官人,世称古灵先生。生于宋真宗天禧元年(1017年),与陈烈、周希孟、郑穆友好,人称「海滨四先生」。庆历二年(1042年)进士,初任浦城县(今属福建省)主簿,历知仙居、河阳、蒙阳等县,因反对新法,曾请求神宗贬斥王安石、吕惠卿以谢天下。神宗时,出知明州。庆历八年(1048年)到仙居任县令,“民尚朴野,罕知读书”,“民穷多变,监狱患满”,又逢县境大水,著手赈济灾民。卒于神宗元丰三年(1080年)。摸钟辨盗电视剧《包公》里有这样一幕:包公让嫌疑人在黑暗中去摸一口大钟、从而找出真正的盗贼,其实这个故事就是嫁接自陈襄。事见,苏洵写了《钟能辨盗》,而在沈括写的《梦溪笔谈》中也记载了这个故事。陈襄任建州浦城知县时,有一家遭到了盗窃,抓到几个嫌疑人,但不知道谁确实是小偷。于是陈襄哄骗他们说:“某庙里有一口很灵验的钟,能分辨谁是小偷。”他派人把那口钟抬到官署后阁,祭祀起来,又带领这一群囚犯站到钟前,宣布说:“没有偷东西的人,摸这口钟,它不会响;如果偷了东西的人去摸,钟就会发出声响。” 祭祀完毕后,用帐子把大钟围起来,再暗地里让人用墨汁涂钟。之后,让嫌疑人一个个把手伸进帷帐里去摸钟,出来就检验他们的手,发现都有墨汁,只有一人手上无墨。陈襄对这个人进行审讯,此人不得不承认自己是盗贼。原来他是害怕钟响,没有敢去摸。","text2":"陈襄被世人称为什么?","label":1} {"text1":"张孝骞(,),字慎斋,湖南省善化县南门外马厂巷人,内科医学家及医学教育家。幼年在私塾学习《四书》、《五经》及古文,1914年以第1名毕业于长沙长郡中学,并以第1名的成绩被录取进入湘雅医学院。1921年复以第1名毕业,并获得金牌及美国康涅狄格州政府授予的医学博士学位。毕业后留校专攻内科,于1924年1月到北京协和医学院(今中国协和医科大学)深造1年,随后正式留任于该校,陆续担任住院医师、总住院医师。1926年9月被选送到美国约翰·霍普金斯大学进行研究,1927年7月回国续任,1930年在协和医学院组建消化专业组,1932年晋升为副教授。1933年12月再次赴美国史丹福大学研究,1934年7月回国后,担任协和内科消化专业组主管。1937年芦沟桥事变后,他放弃协和医学院的研究工作,回长沙接任湘雅医学院院长职务。1938年夏季,战火逼近长沙,他不顾美国雅礼会的反对,迁校到贵阳,再于1944年迁到重庆。抗战胜利后,于1946年8月迁校回湖南。随后应美国国务院的邀请赴美国考察医学教育和讲学。1948年4月辞去院长职务,并于9回到北京协和医学院,担任内科学教授和内科主任。中华人民共和国成立后,他续任原职。1962年9月被任命为协和医学院副校长。1966年开始的文化大革命期间,他被打成「反动学术权威」、「特务」,备受侮辱和折磨。1978年以后担任中国医学科学院副院长。1987年8月8日因肺癌病逝于北京协和医院,享年90岁。张孝骞毕生致力于临床医学、医学科学研究和医学教育工作,对人体血容量、胃分泌功能、消化系溃疡、腹腔结核、阿米巴痢疾和溃疡性结肠炎等有较深入的研究,被喻为内科专家、医学教育家、中国消化病学的奠基人。。1948年获选为第一届中央研究院院士,1955年被推选为中国科学院生物学部委员(院士)。","text2":"张孝骞幼年是在哪里学习知识的?","label":1} {"text1":"青花釉里红瓷仓,又称红釉堆塑楼阁式仓,是元代瓷器珍品,造型别致,属于随葬明器,青花釉里红瓷器物本已非常罕见,这种楼阁式瓷仓且有明确纪年的样式,迄今仅此一件。1974年出土于江西省景德镇,1979年9月,丰城县文化馆在江西省文物商店协助下征集得到,现藏江西省博物馆。瓷仓通高29.5cm,横宽20.5cm,进深10cm。仓顶庑殿重檐式,仓瓦为釉里红点彩串珠组成,中以隔墙分为前、后楼。楼阁高低相错,造型别致,充分体现了元代江南木结构建筑特色。插板式活动仓门两侧用青料书直行楷书七言句对联,右联为“禾黍丰而仓廪实”,左联为“子孙盛而福禄崇”,横披“南山宝象庄五谷之仓”。仓后两柱间空档有墓志,为青料直行正楷,共计12行,159字。根据墓铭,死者为“故景德镇长芗书院山长凌颖之孙女”,死于元惠宗至元四年戊寅(1338年),葬于南山。","text2":"青花釉里红瓷仓是哪个朝代的瓷器?","label":1} {"text1":"自催化是指一个化学反应所生成之产物为该反应之催化剂。若一组化学反应中一部分反应的产物足以催化其他反应,使得整组化学反应可自我供应能量和『食物分子』,则此组反应可称作集体自催化(collectively autocatalytic)(参见)二级自催化反应速率方程formula_1 是 formula_2A和B的浓度随时间依据下式变化formula_3 与formula_4典型的自催化反应等式图形是:反应起初进行地很慢,因为几乎没有催化剂存在。反应速率随著反应进行使催化剂量增加而加快,之后随著浓度减少而减慢。若实验的反应物或产物浓度遵守双弯曲曲线,则此反应可能是自催化。例如:5 CO + 2 MnO+16 H →10 CO + 2 Mn+ 8 HO其反应式右侧的生成物Mn为该反应的催化剂,反应会随著Mn浓度的增加,逐渐加快。1995年提出生命最初起源形式是自催化化学网状系统。英国 动物行为学家理察·道金斯在他2004年著作《祖先的故事》写道自催化是无生源论的可能解释。书中引用与同事在加州完成的实验。该实验中他们以自催化氨基腺苷三酸酯(autocatalyst amino adenosine triacid ester (AATE))合成氨基腺苷(amino adenosine)与五氟苯酯(pentafluorophenyl ester),其中一个来自实验的系统包含了多种能自行催化自身合成的AATE。此实验证明了自催化可能可以造成族群内以遗传进行竞争,可解释为早期形式的天择二位研究者,Robert Ulanowicz 与 Stuart Kauffman 提出自催化反应在生命演化中扮演重要角色,且继续构成生命结构的基本元素。自催化反应与级数创建()","text2":"怎样才可以被称为集体自催化?","label":1} {"text1":"《写给西藏的歌——和平的艺术》(英语:)是一张由来自美国、英国、加拿大和南非等国的全球十多名著名歌手在2008年录制的慈善音乐合辑。合辑中的音乐艺术家包括斯汀、苏珊·薇格和艾拉妮丝·莫莉塞特等歌星。本专辑宣称是为了支援西藏,促进西藏地区的和平和基本人权,包括言论自由和宗教自由,以及表示对第十四世达赖喇嘛·丹增嘉措的支持。专辑同时也是设在华盛顿特区的和平艺术基金会(Art of Peace Foundation)的实践计划的一部分,基金会的负责人Michael Wohl是专辑的出版人,Rupert Hine为监制。专辑在北京奥运会开幕的当日,即2008年8月8日公开发行。8月5日,先在苹果公司iTunes上供人购买下载,当天和平艺术基金会也发布了专辑的MV,仅仅一天,本专辑就成为iTunes全球流行音乐销售量第一位。专辑的CD也在8月19日公开发售。","text2":"写给西藏的歌这张专辑的目的是什么?","label":1} {"text1":"《东京战争》(TOKYO WARS)是一款由南梦宫发行的街机游戏,可由一至四名玩家控制各自的坦克作战。可供玩家选择的战场有日本市区以及海湾码头,选择时间为20秒,之后游戏将会自动锁定当前选择。坦克的阵营由颜色区分,分为绿色(代表绿色哥斯拉)和白色(代表白色猛犸象)。豪华版座舱的主炮在射击时,有一个空气动力的后坐力模拟效果。比起没有后坐力的常规版的座舱,豪华版有着更强的真实感。而在这20部坦克厮杀的战场上,屏幕顶部的箭头可以清晰指示敌友信息。每场游戏持续15分钟,或者在全部敌方坦克被击毁后结束。游戏采用了南梦宫的超级系统22主板以及自制座舱。玩家必须坐在座舱中进行游戏。游戏模式可以选择加入不同阵营与其他玩家对抗,或者加入同一阵营合作对抗AI控制的对手。与大部分街机游戏不同的是,游戏不记录最高得分,也不会使用首字母区分一号玩家和二号玩家。玩家必须观看自己的监视器以确认是否击毁对手。虽然同一时间只有两名玩家可以同时进行游戏,使用4台游戏机链接可以使最多8名玩家进行淘汰制的比赛。由于玩法简单,大部分机器只需要一枚代币即可进行游戏(豪华座舱版需要两枚或更多)。但每名玩家需各投一枚代币,且每个代币需要投入各自的投币槽中,类似于其他的街机视频游戏,与弹珠游戏不同。","text2":"游戏模式该如何选择?","label":1} {"text1":"梳邦()是马来西亚雪兰莪州八打灵县的一个市郊,行政上隶属于莎亚南市政厅(MBAS)。其范围包括梳邦新村和。苏丹阿都阿兹沙机场--吉隆坡旧有的国际机场枢纽就坐落于此,主要提供东南亚区域的航班。梳邦区内有许多高尔夫球场如格林玛丽高尔夫球场、和 主要住宅区种类有高级公寓、半独立式、洋房等。梳邦也是马来西亚的一个国会下议院远区,选民背景多元。目前该区议员为自2008大选至今,来自希望联盟人民公正党的西华拉沙。截止2004年的选区划分,梳邦下有武吉兰樟、 哥打白沙罗和巴也加拉斯三个州议会选区。马航和飞萤航空皆在 梳邦苏丹阿都阿兹沙机场设立总部。而 和拉亚航空也在这设有总部。梳邦柏兰岭商业中心是梳邦天空花园航站楼最近的商业枢纽。吉隆坡日本学校位于梳邦的,是马来西亚除槟城以外唯一一间日本学校。梳邦是马来西亚最多雷暴的地区,其中:梳邦交通完善,可通往临近城镇如吉隆坡、梳邦再也、莎亚南和灵市市郊如双威镇、、哥打白沙罗、万达镇、和珍珠白沙罗。主要道路有 联邦大道、 (KESAS)、新巴生河流域大道 (NKVE),、 (NPE)、,、和白沙罗-蒲种大道。","text2":"梳邦位于哪里?","label":1} {"text1":"本多忠高(1526年/1528年-1549年4月16日)是日本战国时代于三河国的武将。家臣。父亲是本多忠丰。在大永6年(1526年)(一说享禄元年(1528年))出生,家中长男。与父亲忠丰同様,仕于松平清康和松平广忠。天文16年(1547年),在松平一族的松平信孝为夺取家督而发起反乱时,跟随广忠出阵攻打。翌年(1548年),在尾张国的织田信秀侵攻三河,今川氏的太原雪斋和广忠在小豆坂迎撃时,随军与织田军战斗并战胜(小豆坂之战)。天文18年(1549年)3月,主君广忠在冈崎城内被岩松八弥暗杀,令松平氏和三河国人众极之动摇。今川氏当主今川义元为阻止三河国人会投向织田氏,于是派太原雪斋进攻织田氏在三河的据点(第三次)。因为守将织田信广成功坚守城池,忠高与大久保忠俊等人一同进行夜袭并取得战果,但是在翌日的战斗中被敌人的箭矢射中而战死。墓所在爱知县冈崎市大和町的妙源寺。戒名是泰元院殿行誉忠高大居士。战死地安城市安城町赤冢(旁的大乘寺域内)亦有建立墓碑。","text2":"本多忠高什么时候出生的?","label":1} {"text1":"里卡多·阿尔贝托·马蒂内利·贝罗卡尔(,),巴拿马政治家和企业家,前巴拿马总统。1952年3月11日,马蒂内利出生于巴拿马城。他曾获美国阿肯色州大学的工商管理学士学位和中美洲工商管理研究所的工商管理硕士学位。马蒂内利拥有大型连锁超市、银行、电视台、进出品公司及工厂等,是巴拿马知名的企业家。1998年,马蒂内利创立民主变革党。2004年,马蒂内利作为民主变革党的总统候选人参加总统选举,但败于民主革命党的总统候选人马丁·托里霍斯。2009年,马蒂内利再次参选总统,这次是代表民主变革党、巴拿马主义党等政党组成的联盟参选,在5月3日举行的总统选举中,马蒂内利击败执政党民主革命党总统候选人巴尔比娜·埃雷拉,当选为新一任巴拿马总统。作为中华民国在中南美洲重要友邦,但马丁内利任内的巴拿马被指政治立场摇摆不定。除了不时采取与中华民国相反的政治立场,在收取中华民国的经济援助之余,另一边又接受中华人民共和国的好处。","text2":"2009年,马蒂内利再次参选总统,这次是代表什么参选?","label":1} {"text1":"魏绛,中国春秋时代晋国的武将、政治家,魏氏第四代领袖。姬姓,魏氏,名绛、谥号昭、庄。魏犨之孙,魏悼子之子(系本称魏绛是魏犨的幼子)。历仕晋悼公、晋平公。史称魏昭子、又称魏庄子。前573年,晋悼公即位时,魏绛因为勇敢被提拔为中军司马。前570年,晋悼公会盟诸侯,悼公的弟弟扬干搅乱队列,魏绛杀了替扬干驾车的人。悼公大怒:“合诸侯以为荣,今辱吾弟!”将诛杀魏绛。士鲂、张老劝说悼公,悼公读魏绛辩解之书:“日君乏使,使臣斯司马。臣闻师众以顺为武,军事有死无犯为敬。君合诸侯,臣敢不敬?君师不武,执事不敬,罪莫大焉。臣惧其死,以及扬干,无所逃罪。不能致训,至于用钺。臣之罪重,敢有不従,以怒君心,请归死于司寇。”悼公体会到了魏绛忠于职守,重赏了魏绛、士鲂、张老。前569年,魏绛升格为新军佐。同年冬,戎狄与晋讲和,魏绛使戎、翟亲附,晋国再于诸侯中称霸。晋悼公表彰魏绛的功绩,赐安邑(今山西省夏县),安邑成为魏氏发展的中心。前559年,悼公去世,继续辅佐晋平公。前555年,参与讨伐齐国,死后,儿子魏舒继任魏氏领袖。","text2":"晋悼公即位时,魏绛被提拔为什么?","label":1} {"text1":"科罗拉多大学波德分校(University of Colorado at Boulder,官方:CU-Boulder, UCB;非官方:Colorado和CU)是科罗拉多大学系统的旗舰。它成立于1876年,比科罗拉多加入联邦的时间还早5个月。是美国新闻与世界报道所评全美最佳公立大学之一。科罗拉多大学共有三个校区:博尔德校区、丹佛校区,以及科罗拉多泉校区。博尔德校区是科罗拉多大学主校区,该校区位于洛叽山麓的波德市。整个学校由9个学院系统组成,提供超过150个学术项目,有近3万名学生,在2010年授予了6781个学位。该大学物理系先后有4人获得诺贝尔奖,光学、原子方向实力超群,在美国排名第一。工程领域最具特色的是航天工程(Aerospace Engineering Sciences),该专业在美国新闻与世界报道上2011年的排名全美第十二。该系诞生十几位宇航员,并且从2009年起聘期退役宇航员担任某实验类课程指导。航天科学与工程方向下的LASP实验室专门从事航天领域的研究,具备从卫星的设计、在轨运行以及返回数据处理、科学研究的全程研究能力。据统计美国现役的所有卫星都有科罗拉多大学设计、制造的仪器设备。大气与海洋科学系成立的时间不长,但目前已跃升至全美国前五名,并与位于博尔德附近的美国大气研究中心有着密切合作。历史上,科罗拉多大学的学生,研究人员或者学术人员中,总共诞生了11个诺贝尔奖获得者,7个麦克阿瑟奖获得者,以及众多航天员。曾经登陆太空的太空人,有11位是该大学的毕业同学。","text2":"科罗拉多大学物理系先后有几人获得诺贝尔奖?","label":1} {"text1":"康斯坦丁·萨米埃尔·拉菲内克-施马尔茨(,),欧洲19世纪的通才。他在动物学、植物学、气象学、地质学、人类学、语言学等方面作出很大的贡献,他通晓多种语言,在写作、翻译方面也有许多成就,他基本是自学成才的,但行为古怪,有人认为他几乎是精神病,在他生前不受人们理解,但去世后逐渐被人们认为他的成就远远超过他所在的时代。他出生于君士坦丁堡郊区的加拉塔,父亲是来自马赛的法国商人,母亲是出生于伊斯坦布尔的德国人后裔,他童年是在马赛度过的,12岁时已经掌握了植物学用的拉丁语,并开始收集植物标本。1802年,他19岁时前往美国,认识许多植物学家,1805年,回到欧洲,定居在西西里岛的巴勒莫经商,25岁时已经挣到足够他后半生从事研究的钱。1815年,他的儿子去世,他离开妻子,前往美国,但运他行李的船触礁沉没,他丧失了50箱书籍和超过6万件贝类标本。在纽约,他成为1818年成立的《自然历史论坛》的创始人之一,他又收集了超过250件植物和动物标本。1819年,他被聘为特兰西瓦尼亚大学的植物学教授,同时业余教授法语和意大利语。1817年,他出版了《路易斯安那州植物》(\"\"),结果受到当时许多植物学家的批评,1826年,由于和校长争吵,他离开大学。他到了费城,完全依靠自己出资作公开演讲和出版书籍,1828年-1830年,出版了《药用植物,美国和北美药用植物手册》,1833年,出版了《拉方斯克植物标本》,描述了许多植物新种,并且为黑尾土拨鼠(\"Cynomys ludovicianus\")、美国白足鼠(\"Peromyscus leucopus\")和黑尾鹿(\"Odocoileus hemionus\")确定了拉丁种名。从1836年至1838年,他确认了几百个植物新属和几千个新种,但绝大部分并没有被科学界认可。他当时认识到物种可能会进化形成新种,但没有能发展成一个系统的进化理论,他还研究了北美洲的一些印第安人部落的传说、语言和史前遗迹等。他在费城由于胃癌逝世,由他朋友把他安葬了,他生前收集的标本都被当成废物处理掉了。1841年,托马斯·纳托尔为纪念他,将菊科的雪苣属(\"Rafinesquia\" Nutt.)以他命名,他去世后,他的著作越来越被科学界重视。","text2":"康斯坦丁·萨米埃尔·拉菲内克-施马尔茨的童年是在哪儿度过的?","label":1} {"text1":"是香港其中一个公共屋邨,位于东区柴湾,分为兴华(一)邨和兴华(二)邨两部份。前身为徙置区,名为兴华徙置区,俗称「兴华新区」,于1971年落成。在1974年,房委会将兴华(一)邨所有楼宇重新命名,1994年拆卸。重建后的项目编号为HK16,全邨有3幢38层高的和谐一型大厦,名为美华楼、卓华楼及兴翠楼(前称雍华楼及兴翠苑)及1幢8层高停车场大楼,其中美华楼基座则为2层高商场「兴华广场」,并设有天桥及通道连接港铁柴湾站。房委会透过「自选单位计划」把美华楼及卓华楼单位编配给受柴湾邨13座及北角邨重建影响的租户,1999年入伙,而原本房委会于1998年打算把雍华楼改为居屋出售,名为兴翠苑,而邨内的楼宇外墙名牌及指示牌亦已更改作兴翠苑之际,政府突然宣布停售居屋政策,房委会遂决定将兴翠苑改回作公屋用途,房委会把所有楼宇名牌的「苑」字拆除,并改为「楼」字,并透过「自选单位计划」把兴翠楼全幢单位编配给受柴湾邨14及15座重建及北角邨重建影响的租户,让他们自行拣选合适的居住单位,期间空置了一年,最后推迟至2000年才开始入伙。兴华(二)邨于1976年落成,由前工务局设计,由前徙置事务处及香港房屋委员会按前徙置事务处规格及标准兴建,所以富有前徙置事务处屋邨的建筑设计特色。当年2月1日由于爱秩序湾艇户大火,有接近3,000人受灾。政府为了尽早安置灾民,裕兴楼及丰兴楼均提早六周落成。有5幢建于山上,另外2座大厦(裕兴楼及丰兴楼)则依山而建,设有大型升降机、架空走廊及天台通道,分别连接环翠道及柴湾道,方便居民出入,建筑设计上与同期的葵盛西邨十分相似。兴华广场(前称兴华商场)的提升工程于2009年4月开展,并于2010年第一季完成。除了鲜活街市外,亦有面包店、水果店、超级市场、快餐店等。其后街市被改建为商舖,而超级市场亦搬到前街市位置。","text2":"兴华(二)邨什么时候落成的?","label":1} {"text1":"马来闭壳龟(学名:)又名安布闭壳龟,为龟科闭壳龟属的爬行动物。分布于印度尼科巴群岛、孟加拉、缅甸、泰国、寮国、柬埔寨、越南中部及南部、马来西亚西部、新加坡、菲律宾及印尼,并可能分布至中国广东及广西,常见于平原地区的沼泽、湿地、池塘、河流中的水荡以及水稻田等水流缓慢、底质松软的水域。该物种的模式产地在印度尼西亚。幼体几乎完全水栖,成体偏好陆栖。草食性为主,吃水生植物和真菌,也吃一些昆虫。如要以马来闭壳龟作为宠物龟,饲养者要深入研究饲养方式,并要做好一般护理和照顾。考虑以马来闭壳龟作为宠物时,要了解它是一种热带、非冬眠的物种,比许多其他箱龟需要更温暖的水(约82华氏度)。完全成熟的成龟通常需要半水陆的饲养环境。闭壳龟马来闭壳龟比其他箱龟更能适应在水中生活。不幸的是,这往往导致被错误地放在陆地中饲养。虽然他们是拙劣的游泳者和比其他半水栖龟需要较浅水,但他们往往会比较喜欢在水中。在野生环境中,他们往往会住在几乎完全在水晒太阳和产卵,他们甚至可以在水中交配。","text2":"该物种的模式产地在哪个国家?","label":1} {"text1":"四线天竺鲷(学名:),为辐鳍鱼纲鲈形目鲈亚目天竺鲷科天竺鲷属的一种。本鱼分布于印度西太平洋区,包括马达加斯加、留尼旺、塞席尔群岛、斯里兰卡、台湾、泰国、越南、印度、日本、马来西亚、印度尼西亚、菲律宾以及中国南海及东海、澳洲、斐济等海域。该物种的模式产地在印度Pondicherry。水深0至80公尺。本鱼体延长而侧扁,眼大,口大略下位。鱼体黄褐色,腹部银白色,体侧具4条深色细纵带,其中一条从吻部穿过眼睛延伸至尾鳍底。背鳍硬棘8枚;背鳍软条9枚;臀鳍硬棘2枚;臀鳍软条8枚,体长可达13公分。本鱼栖息于珊瑚礁或岩礁区,白天躲藏于岩架下或岩洞中,夜间出来觅食。繁殖期时,雄鱼具有口孵习性,卵约7日化成仔鱼,由雄鱼吐出,具短暂的仔鱼飘浮期。不具经济价值。","text2":"四线天竺鲷的模式产地是哪里?","label":1} {"text1":"《过敏世界》是香港歌手张学友的第十六张粤语专辑,唱片制作人欧丁玉,宝丽金唱片公司制作发行。《过敏世界》在推出后广受好评,并受到市场的极度认可,坐拥五首冠军歌曲,而其中三首为四台冠军歌,是香港流行音乐历史上的一张重要唱片。过敏世界专辑中的同名主打歌曲《过敏世界》充斥了对香港狗仔队跟踪名人等一些不检点行为的批评,其中歌曲由张学友亲自作曲。由于争议性很大,虽然被定为第四主打歌,但并未在香港的电台、电视中播放,被之前定位为非主打歌的《这么近(那么远)》所取代。在录制此张唱片前,张学友曾经奔赴美国进修音乐,而唱片在发行后体现了他的进步非常明显,特别是由他亲自谱曲的两首歌曲《这么近(那么远)》,以及《过敏世界》被专业人士认为具有良好的音乐质素。另外,他用高亢的嗓音演绎了歌曲《我哭了》、《无心恋》以及《这么近(那么远)》,得到乐迷一致好评。歌曲《屈到病》讽刺当时香港的一些家庭主妇妻管严风格。第三主打歌《离开以后》更是包揽了多个当年香港音乐颁奖礼的奖项。值得注意的是一首开创香港流行音乐串唱先河的一首歌曲《友情歌》,他将自己于1985年至1993年的一些经典歌曲会聚成为极其押韵的一首歌曲,歌曲长达13分钟半,编曲人亦特别出色的结合了各个歌曲不同的曲调,形成为一种独特的风格,香港著名填词人黄霑亦有在当时表示,《友情歌》是香港流行音乐一种新的尝试和突破。进入2000年后,歌手古巨基等亦有演唱类似于友情歌的《劲歌金曲》等歌曲,但大多数歌曲都是改变来自他人演唱过的旧歌。","text2":"《过敏世界》的唱片制作人是谁?","label":1} {"text1":"张名振(),字侯服,应天江宁(治今江苏南京)人。少伉爽,有大略,太监曹化淳延为上宾。崇祯十六年(1643年),担任台州府石浦游击。1645年清军南下,南京、杭州俱陷,南明弘光朝覆灭。十一月,张名振等人拥鲁王于绍兴担任监国,以隔年为鲁监国元年。张名振受封定西侯、富平将军。1646年,绍兴陷落,张名振等随扈鲁王逃亡海上,辗转前往厦门,投靠郑芝龙的从兄弟郑彩。1648年,张名振迎鲁王回到浙东。隔年(1649年),张名振联合阮进,攻击奉隆武年号之舟山总兵黄斌卿,将之击杀于海上,取得舟山岛。随后,张名振迎鲁王驻舟山,并筹划以舟山为根据地,顺治六年,驻跸南田。顺治八年(1651年)九月二日,清军攻破舟山,大学士张肯堂自杀死,张名振护卫鲁王三度航于海上,辗转于浙江、福建海上;最后与鲁王前往厦门,投靠郑成功。永历八年(1654年),张名振向郑成功请师北伐,女眷亦随军出征,入长江,师抵南京燕子矶。永历十年(1656年),张名振联合张煌言再次北伐,但不幸死于战阵;其遗嘱由张煌言总领兵马,但多数的军队最终都被郑成功所收编。","text2":"张名振怎么取得舟山岛的?","label":1} {"text1":"水头镇是浙江省南部温州市平阳县下属的一个镇。国家级风景区南雁荡山的东大门,是平阳县经济副中心,县域北港地区经济、文化、交通中心和物资集散地,素以山水秀丽,辖地广阔和经济活力强劲而闻名。全镇总面积36.3平方公里,耕地面积15672亩,下辖5个办事处,45个行政村和14个居民区,2000年总人口85734人,外来员工2万多人。水头镇周边地区,是平阳县闽南人的主要聚居地之一。镇政府驻剧院路,邮编:325405南湖曾为海角浪汤汤,嬗作明湖接大江。那岁泥平新水退,旧时鱼宅已栽桑。【注】[1]汤汤(shāngshāng):浩茫貌。[2]新水退:犹“水新退”,意为湖水刚刚退去。江屿分杈江夹好河山,簇落人家涘甸间。到埠天天两潮水,运输忙煞板车班。【注】分杈江:指鳌江干流中游“水头江”和支流“腾蛟江”,二流形如树之分杈,江屿则处其所夹也。麻步北港通航门户宽,江街人货往来欢。乡村近远多麻产,船埠新增课税官。","text2":"水头镇的地理位置有什么优势?","label":1} {"text1":"龚鹏程,台湾教育家、作家,出生于台湾台北市,籍贯江西省吉安市,曾任《国文天地》总编辑、台湾学生书局总编辑,是台湾佛光大学与南华大学的创校校长、中华武侠文学学会会长,现任卢森堡欧亚大学马来西亚校区校长,游历中国大陆,任北京大学客座教授、南京大学客座教授、北京师范大学特聘教授、四川大学讲座教授,作育英才无数,著作七十余种。少年时代钟情中国武术,并由习武而博考文献,获得许多文献学的知识与方法。龚鹏程一直对于台湾的部分「本土派」学者的台湾文学研究论点与研究方法表达不满;1997年,他发表自著《台湾文学在台湾》,批评该类学者惯用的单线叙述法及其造成的简化历史等问题。2005年9月11日,玄奘大学中国语文学系兼任讲师杨宗翰说,他就读硕士班时改攻台湾文学,因此深知,囿于文化认同上的差异,台湾许多本土派学者都非常讨厌龚鹏程,却又没办法跳过或忽视龚鹏程所提出的诸多尖锐质疑。2003年12月11日,《台湾壹周刊》第133期报导,2000年10月佛光人文社会学院校长龚鹏程带诺贝尔文学奖得主高行健与该校十余名教职员去宜兰县礁溪乡富野酒店喝花酒,2001年2月龚鹏程再次带高行健喝花酒。2003年12月15日,高行健从法国传真声明稿给龚鹏程与该校教授马森,批评该文报导不实,请马森代他要求《台湾壹周刊》公开更正并回函道歉,否则提告;龚鹏程则说,没有一起喝花酒之事。《台湾壹周刊》总编辑裴伟表示,该文内容皆是采访「佛光山能参与事情的人」而得,采访过程皆有录音、记录,也向龚鹏程求证,并写入龚鹏程否认的话,已平衡报导;该刊将再做进一步求证,如有错误,将予更正。2003年12月18日,《台湾壹周刊》在《台湾苹果日报》刊载四分之一篇幅的道歉声明向高行健道歉;同日,高行健批评,该篇道歉声明毫无诚意,该刊必须发表有诚意的道歉声明向该文涉及的所有受害当事人公开、郑重道歉,并向读者保证将杜绝造谣、不实的报导;裴伟回应,该刊会在下期刊登道歉声明,但绝对不会对龚鹏程道歉,因为关于龚鹏程的部分有全部的证据。《台湾壹周刊》第135期刊登道歉声明,向高行健道歉。","text2":"龚鹏程是哪里人?","label":1} {"text1":"大连金州体育场,座落于辽宁省大连市金州区,是一座可承办足球、田径等运动项目的综合性体育场。总占地面积为7.9万平方米,其中,主体及内场占地3万平方米。现在该体育场可容纳观众30,776人。以下内容中新金州体育场将被称作金州体育场。1958年,老金州体育场竣工。当时这座体育场仅能容纳观众12,000人。以现今的标准来看,老金州体育场难以达到目前体育比赛,尤其是足球比赛的要求。随着中国体育赛事随着改革开放的深入而日渐增多,老金州体育场已经很难满足现代比赛的需求。20世纪90年代,大连市政府决定重建金州体育场。1996年10月26日,新金州体育场动土兴建;1997年6月18日竣工。而在之后,2007年,大连市政府又出资重新整修体育场,包括修缮电子屏幕等设施,使体育场达到了现今的面貌。该体育场承办的最重量级赛事就是1998年世界杯预选赛亚洲区十强赛,1997年10月31日中国队主场迎战卡塔尔队的比赛。在这场关乎出线的关键大战中中国队以2:3败北。而最终中国队在本次比赛中以3胜2平3负的战绩名列小组第三,未能冲入1998年世界杯决赛圈,也从此留下“金州没有眼泪”的说法。后来,该体育场成为了中国足球超级联赛大连实德足球俱乐部的主场。在这座体育场中,大连实德曾经多次获得甲A、中超以及中国足协杯的桂冠;这里也成为了大连足球甚至大连体育的象征之一。如同中国其他大型体育场,金州体育场也经常承办演唱会、大型文艺晚会等演出活动。","text2":"新金州体育场兴建和竣工时间分别是什么时候?","label":1} {"text1":"朱庇特神庙(拉丁文:Aedes Iovis Optimi Maximi Capitolini)位于罗马的卡比托利欧山,是古罗马最伟大的宗教庙宇。对于第一座朱庇特神庙的了解,大多来自罗马的传说,据说完成于公元前509年(罗马共和国成立的年份)9月13日,供奉朱庇特、朱诺和弥涅耳瓦。卢修斯·塔克文·布里斯库在与萨宾人作战时来到神庙发誓,似乎表明已经有一定基础,不过大部分工作由卢修斯·塔克文·苏佩布完成。传说还提到在兴建神庙之前,这个地点已经有其他神祠,当占卜师执行仪式寻求许可迁移神像时,据信只有护界神Terminus 和青春女神Iuventas拒绝迁移,因此他们的神祠被纳入新的建筑。由于Terminus是护界神,他的拒绝迁移被解释为对未来罗马的国运是有利的预兆。原来的神庙实测为大约60 x 60米,是罗马全国最重要的宗教建筑。三位神各有单独的cella,朱诺在左侧,弥涅耳瓦在右侧,朱庇特在中间。神庙装饰以许多赤陶雕塑,其中最著名的是朱庇特驾着四马战车,安置在三角墙的顶部。这个雕塑,以及主cella 的朱庇特神像,据称是伊特鲁里亚工匠 Vulca of Veii的作品。第一神庙在卢基乌斯·科尔内利乌斯·苏拉独裁期间的战争中遭到焚毁,收藏的预言书也在大火中失落。布鲁特斯和其他刺客在谋杀恺撒后将自己锁在里面。奥古斯都重建了神庙。第二神庙在公元69年四帝之年韦帕芗与进城称帝者的战斗中被焚毁。皇帝韦帕芗、提图斯和图密善重建神庙。根据古代文献,图密善使用至少一万二千塔兰同黄金为屋顶铜瓦镀金。为了保持原有的版本,复杂的雕塑替代品装饰着三角墙。卢浮宫内收藏的一幅文艺复兴时期复原画显示,在三角墙的最高点是一辆四马战车和一辆二马战车。两尊雕像充当中央柱脚,檐角上有战神马尔斯和女神维纳斯雕像。在三角墙上,朱庇特两侧是朱诺和弥涅耳瓦,坐在宝座上,下面是展开翅膀的老鹰。他们的两侧分别是太阳神和月亮驾着二轮战车。图密善完成的神庙保持基本完好无损超过400年,直到5世纪遭到劫掠。今天,只在保守宫(Palazzo dei Conservatori)后面花园的展览区内有少量遗迹,其废墟是卡比托利欧博物馆的一部分。","text2":"第二神庙什么时候被毁的?","label":1} {"text1":"文化区(Cultural area)是指文化较接近的地区。此概念与另一个类似的概念「文化圈」(德文:)的意思并不尽相同,后者现时在日本非常流行。「文化区」与「文化圈」,两者都是与文化相关的地域名词,两者同样是在第二次世界大战之前提出的,意思亦有少许类似,但并不尽相同。首先,「文化区」是由美国的Clark Wissler和阿弗烈·克鲁伯,两位人类学家提出的文化人类学概念;「文化圈」则是由德国的Fritz Graebner和奥地利的Wilhelm Schmidt,两位民族学家提出的。另外,「区(Area)」的意思非常清晰,就是指地域。「圈(Circle)」的意思却很模糊,例如「演艺圈」的圈字就有「界」的意思,「华人圈」的圈字则是指社群,而「文化圈」按字面并没有地域的意思,但现时流行于日本的「文化圈」概念,事实上却是一个地域名词。不过因为日本用「圈」字代替了「区」字,使其地域性质变得不明显,一般人便不容易察觉其背后所隐含的地域性质。这类概念亦最容易被利用和变质,最明显的例子就是「大东亚共荣圈」。至于「文化区」则没有此问题,因为意思本来就很清晰,很难被利用。最后,「文化区」所著重的是地理,很少会著重历史。但现时流行于日本的「文化圈」概念,所著重的却是「历史发展」与「地域统合」。","text2":"什么是文化区?","label":1} {"text1":"陆小凤,是古龙小说的代表人物之一。古龙亲笔的陆小凤故事有七篇:大金鹏王(原名陆小凤传奇)、绣花大盗(又名凤凰东南飞)、决战前后、银钩赌坊、幽灵山庄、凤舞九天、剑神一笑。他总能遇到十分稀奇的事,但总能以高超的智慧逢凶化吉,后人评曰:“翩翩人中凤,遨游九重天,纵无灵犀指,眉毛亦堪豪,尊其为“侠探”。陆小凤脸上最明显的特征,就是有四条的眉毛。所谓的四条眉毛,指的是陆小凤的嘴上的两撇胡须,修的和眉毛一样整齐漂亮,所以武林人士都说陆小凤有四条眉毛。陆小凤最擅长的武功,名为灵犀一指,指的就是陆小凤的两根手指头,可以夹住普天下大部份高手向他攻击的兵器,包括刀、枪、剑、戟、斧、钺、钩、叉、镗、棍、槊、棒、鞭、锏、锤、抓、拐子、流星锤等。陆小凤多次遭受敌人袭击,靠的几乎都是灵犀一指来脱困。其中最为江湖流传的是用灵犀一指粉碎了叶孤城的天外飞仙。而陆小凤的轻功「双飞彩翼」也是一绝,虽然古龙并没有特别详加描述,但是在小说当中,陆小凤的轻功可以排上武林头几位。陆小凤是一个很妙的人,古龙喜欢写妙人,从名字到性格,古龙笔下有很多妙人,陆小凤是一个很典型的代表。陆小凤的名字听起来很温柔,这一点很妙,人也如其名,很温柔(对女性),这一点也很妙,外貌四条眉毛也很妙,最妙的就是武功,陆小凤主要武功来自于李商隐的诗“身无彩凤双飞翼,心有灵犀一点通”。可以说陆小凤是古龙小说中最妙的几个人之一。陆小凤在小说当中的好朋友有很多,甚至连当朝皇帝都将陆小凤视为知心好友。陆小凤也有很多的红粉佳人,如欧阳情(出现在大金鹏王朝、决战前后)、薛冰。据《白玉老虎》一书提及,陆小凤唯一传人为花满天,但未有明确指出其与花满楼之间的关系。","text2":"为什么武林人士都说陆小凤有四条眉毛?","label":1} {"text1":"李呈瑞,1912年出生。江西兴国人。1955年被授予海军少将军衔,荣获二级八一勋章、二级独立自由勋章和一级解放勋章。1930年参加红军,历任红军兴国模范师第三团排长,红三军团政治部直属政治处组织干事,红一军团4师12团连指导员,红一军团政治部直属政治处特派员,并参加长征。抗战中任抗大政治指导员、2团政治处主任,抗大总校政治部组织部干部科科长。第二次国共内战中任冀热辽军区冀中纵队政治部组织部部长,晋察冀军区3纵7旅副政委、炮兵旅政委,华北军区炮兵政委。建国后陆军第68军政治部主任、副政委兼政治部主任、政委,志愿军第68军政委、海军航空兵部首任政委,其部曾出现故事影片《打击侵略者》、《奇袭白虎团》等的题材。1953年任战俘解释团团长并参加板门店谈判。是中国海军航空兵的创建人之一。1967年逝世。","text2":"李呈瑞于哪一年参加红军?","label":1} {"text1":"尚清王(;)是琉球国第二尚氏王朝的第四代国王,为第三代国王尚真王的第五王子。1527年至1555年在位。神号天续之按司添()。尚清童名真仁尧樽金(),原为中城王子,是尚真王的庶子,华后所生。华后为使尚清成为王世子,不断向尚真王进谗言,诬称世子尚维衡对她欲行非礼。尚真王大怒,废尚维衡,立尚清为世子。1526年12月11日,父王尚真薨,他于翌年即位,同时派正议大夫郑绳往明朝,请求袭封。但郑绳在途中遭遇台风溺死。尚清重新派出长史蔡瀚、马吾喇等人赴明。1534年(嘉靖十三年),明嘉靖帝派吏科左给事中陈侃、行人司行人高澄为正副册封使,前往琉球国,册封尚清为王。1537年,第四代尚清王率军北伐,攻取奄美群岛。琉球王国终于将势力扩张到整个琉球列岛,确定了北起喜界岛、奄美大岛,南至宫古、八重山群岛的疆界,即琉球史书中所称「三省并三十六岛」。同年奄美大岛酋长向尚清王报告,诬称与湾大亲欲发动叛乱。尚清王发兵前往镇压。与湾大亲自缢身亡,尚清王掳其子糠中城而归。1542年,琉球商船与中国海船发生争执并发生斗殴,明廷命令琉球勿轻易中国商船贸易。1547年,修建大美御殿。1553年,尚清王在那霸港附近修建屋良座森城,以加强了对倭寇的防备。1555年6月29日,尚清王薨去,享年59岁。临终前遗命法司毛龙吟、和为美、葛可昌三人辅佐尚元。但他死后,和为美、葛可昌突然变心,欲拥立尚鉴心为新王。最后,在毛龙吟的帮助下,尚清王生前指定的继承人尚元在斗争中胜利,并在翌年继位。|-style=\"text-align: center; background: #FFE4E1;\"","text2":"1553年,尚清王在那霸港附近修建屋良座森城,有什么用处?","label":1} {"text1":"Internet Download Manager(简称IDM)是一个用于Windows系统的下载管理器,它是共享软件,免费试用期为30天,但是每月均有一段时间优惠。IDM可以让用户自动下载某些类型的文件,它可将文件划分为多个下载点以更快下载,并列出最近的下载,方便存取文件。相对于其它下载管理器,它具有独特的动态档案分割技术。它的出色之处在于,能分析下载地址中的实际地址,但若地址无效时则跳换成失效网页,当地址重新有效而无法恢复成原来的地址。它的不足之处在于,必须设置分段的临时文件存放的磁盘位置,下载完成后再自动合并成一个完整的文件,同时删除临时文件(这种做法不同于其他下载软件预先分配一整块磁盘空间)。所以,下载的如果是机密文件,就容易被数据恢复软件恢复,造成安全隐患。IDM可被调用于Internet Explorer、Safari、Google Chrome、Opera、Netscape Navigator以及Mozilla Firefox(火狐)浏览器中。","text2":"这个软件有什么作用?","label":1} {"text1":"微指令的编译方法是决定微指令格式的主要因素.考虑到速度,成本等原因,在设计计算机时采用不同的编译法 .因此微指令的格式大体分成两类:水平型微指令和垂直型微指令。一次能定义并执行多个并行操作微命令的微指令,叫做水平型微指令.水平型微指令的一般格式如下:控制字段 判别测试字段 下地址字段按照控制字段的编码方法不同,水平型微指令又分为三种:一种是全水平型(不译法)微指令,第二种是字段译码法水平型微指令,第三种是直接和译码相混合的水平型微指令。微指令中设置微操作码字段,采用微操作码编译法,由微操作码规定微指令的功能,称为垂直型微指令。垂直型微指令的结构类似于机器指令的结构.它有操作码,在一条微指令中只有l-2个微操作命令,每条微指令的功能简单,因此,实现一条机器指令的微程序要比水平型微指令编写的微程序长得多 .它是采用较长的微程序结构去换取较短的微指令结构。","text2":"什么是垂直型微指令?","label":1} {"text1":"吴嘉丽,台湾化学学者、女权工作者,曾任考试院考试委员、台湾女科技人学会理事长。她专精于苔藓植物化学研究,关注科学普及教育、妇女权益、推动性别平等教育等。1969年台湾大学化学系毕业。1976年获美国华盛顿大学博士学位。1976年回台后在淡江大学教书,1983年升正教授。2012 年退休。1986年到1988年以教授兼任淡江大学化学系主任兼研究所所长。2002年9月1日出任考试院考试委员。2008年创办《台湾女科技人电子报》2011年底成立台湾女科技人学会。2014年 ~ 2017 年担任国际女科技人联络网理事。《化学、医药与社会》(1999年)《化学、食品与社会》(2004年)","text2":"吴嘉丽毕业于哪个大学?","label":1} {"text1":"乙酸铜(乙酸铜(II)),是化学式为Cu(CHCOO)的化合物,其中CHCOO指乙酸根CHCOO。Cu(CHCOO)是深绿色晶体,一水合物Cu(CHCOO)(HO)略带蓝绿色。乙酸铜在古代被用作杀菌剂和绿色颜料,目前多用作无机合成中铜(II)的来源,也可在有机合成中作为催化剂或氧化剂。和所有铜化合物一样,乙酸铜的焰色反应为蓝绿色。乙酸是食物发酵的副产物,而乙酸铜最早也是在葡萄园中获得的。当时铜片夹在生产葡萄酒剩余的葡萄皮和残渣中,人们发现不久后就有蓝色物质在铜片表面生成。刮下少许后发现它可溶于水。该固体被用作颜料。它与三氧化二砷混合后生成醋酸亚砷酸铜,作为杀虫剂和杀菌剂使用,称作巴黎绿。乙酸铜的主要用途是在有机合成中作为催化剂或氧化剂使用。例如,Cu(CHCOO)可以催化两个末端炔烃的偶联,生成1,3-二炔:反应的中间体包括乙炔亚铜等,再经乙酸铜氧化,得到炔基自由基。此外,用乙酸铜来合成炔胺(含有氨基的末端炔烃)也涉及乙炔亚铜中间产物。将无水Cu(CHCOO)和金属铜一起加热会得到无色易挥发的乙酸亚铜:乙酸铜(II)用水合肼也能还原成乙酸亚铜。乙酸铜在发现后的几个世纪内都是通过以上方法制取的。但这种方法制得的乙酸铜杂质较多。现在实验室中的制备方法分为三步,总反应为:二水合物会在100°C真空失水:也可以用碳酸铜和乙酸反应制备乙酸铜:Cu(CHCOO)(HO),以及类似的Rh(II)、Cr(II)乙酸盐二聚体都采取“中国灯笼”式的结构。 每个铜原子都为四个氧原子所围绕,乙酸根的每一个氧原子都与一个铜原子键连,键长1.97Å(197pm)。两个水分子配体占上下,Cu-O键长为2.20Å(220pm)。两个五配位的铜原子之间的距离为2.65Å(265pm),与金属铜中Cu—Cu距离(255pm)相近。两个铜原子存在很弱的共价作用,导致乙酸铜的磁矩随着温度降低而减小,比如一水合乙酸铜室温下的磁矩为1.40 B.M.,而在90K时仅为0.36 B.M.。由于自旋方向相反抵消,Cu(CHCOO)(HO)实质上是反磁性的。它对推动现代反铁磁体耦合理论发展有很重要的贡献。","text2":"乙酸铜最早从哪里获得的?","label":1} {"text1":"棕树蛙(学名:\"\")是树蟾科雨滨蛙属下的一种,原生于新南威尔斯南部,北至阿勒达拉(Ulladulla),但有零星报告指在更北的地区也有其踪迹。棕树蛙分布广泛,在维多利亚州南部大部分地区及南澳州东部均有分布,整个塔斯曼尼亚,并包括巴斯海峡内的海岛上。在纽西兰的种群为引入种,并不原生于此地。体长约45毫米,背部表面为淡至深褐色,有一宽黑线纹由眼后一直延伸至整个背部,另有一黑带由鼻孔延伸至眼睛、鼓膜而至肩膀,淡白色条状在其下,但由口部延伸至前臂。大腿背部为红色,而不是黑色大理石条纹,由此可分辨出其近亲维氏雨滨蛙(\"L. verreauxii\")。腹部为奶油色,某些在西维多利亚州或南澳州东南部的个体会部分或全身变绿。此种能在不同生境下生活,包括森林、农地、半干旱地区、高山地区、甚至是近郊地方。在阿德莱德、墨尔本及霍巴特的近郊地区十分普遍,原因是牠们常因追逐昆虫而停靠在当地的玻璃窗外。雄性会在水坝、沟渠、池塘及河滨池塘的水源处发出听起来如口哨般的 weep-weep-weep 叫声,全年均会呜叫,特别是在雨后。棕树蛙的卵很易被发现,常在草丛内的根部、水生植物内成串缠绕。","text2":"棕树蛙的体长是多少?","label":1} {"text1":"2008年至2009年英格兰超级联赛的英格兰超级联赛于2008年8月16日开始,2009年5月24日结束。揭幕战由阿仙奴主场迎战升班球队西布朗,结果阿仙奴凭新将拿斯利的入球以 1–0 战胜对手,全取 3 分。虽然曼联在第三十七轮与阿仙奴的「褪色大战」中仅能以 0–0 战平对手,但仍然无阻他们历来第 11 次夺得英超冠军;这亦是曼联继1998年至2001年后,第 2 次于英超实现三连霸。曼联也藉这次冠军,成功追平利物浦 18 次夺得英格兰顶级联赛锦标的纪录。另外于本季赛事中,每场比赛之后备名额会由以往的 5 个,放宽至 7 个名额,与欧洲各大联赛相符。本季共有20支队伍参加,升班球队有冠军联赛冠军西布朗,亚军史笃城及附加赛冠军侯城。今季新升班球队有3队,来自上季英格兰足球冠军联赛:今季离开超级联赛球队有3队,降级到英格兰足球冠军联赛:","text2":"2008年至2009年英格兰超级联赛起讫于何时?","label":1} {"text1":"胡子鲇(学名:)为辐鳍鱼纲鲶形目胡子鲇科胡子鲇属的其中一种鱼类,又名胡鲶、塘虱、土虱、土杀鱼、塘角鱼、过山鳅。本鱼体长约20至24.5厘米,灰褐色或深灰色,腹部呈白色与灰色,无鳞。有须4对,背鳍和臀鳍均延长,背鳍软条59-63枚,臀鳍软条42-46枚,胸鳍有一毒棘,圆形尾鳍。鳃腔上方具有一树枝状辅呼吸器,能呼吸空气。可作为食用鱼及养殖鱼类。胡子鲇分布于亚洲台湾、中国、香港、柬埔寨、越南、寮国及菲律宾的淡水流域。在中国,分布于南自海南岛北至长江中下游、西自云南、东至台湾等,属于亚热带小型底栖鱼类。其常栖息于水草丛生的江河,生活于热带、池塘、沟渠以及沼泽的稻 田的洞穴内或暗处。该物种的模式产地在亚洲、非洲。本种亦被引入日本、菲律宾以及夏威夷。","text2":"胡子鲇在中国一般分布在什么地方?","label":1} {"text1":"曾健明(Leo Tsang,)歌影视三栖艺人,现为香港无线电视部头合约艺员。曾独得市政局年度全港舞台剧《最优异演员奖》,出道于1977年丽的电视第三期艺员训练班。1978年《丽的艺训班》毕业,成为全职艺员。1980年代开始在TVB拍电视剧,最擅长演骗子的角色,可以说骗人骗到出神入化,由于拍骗人的戏实在太深入民心,导致现实生活中,很多人都分不清他说的是真话还是假话,骗人骗到真假难辨,绝对称得上是TVB的骗子王,于综艺节目《Sunday好戏王》获颁「另类好戏王」,演技大受肯定。2016年更得到《骗子王》奖项,被誉为《最有价值演员首位》。
及后随知名歌唱家,如秦燕老师,韦秀娴老师,叶惠康教授,程路禹教授等等,钻研乐理及歌唱技巧。近年更拜梁月玲老师门下习艺,唱功更是突飞猛进、一日千里。1990年代曾为香港电台电视制作部拍摄教育电视,当时以扮演医生、成展权的父亲居多,又经常参演《警讯》及《绳之于法》,做尽一切坏人及骗徒。1994年『永盛影业』剧本赛,得《最佳故事银奖》。曾知天份得天独厚,乃半由人力半由天,遂以义工身份,到各大院舍,献唱怀旧金曲,以娱一众长者老友记。拿手曲目包括:《风流梦》、《卖花曲》、《热带情歌》、《盼知音》、《百花亭之恋》、《相思泪》、《秋风吹谢了春红》、《相逢恨晚》、《怀旧》、《彩蝶双飞》、《一心想玉人》等等,不计其数。","text2":"曾健明是怎样出道的?","label":1} {"text1":"亚砜是含有亚硫酰基(>S=O)官能团的一类有机化合物,可由硫醚氧化得到。常见的亚砜有氯化亚砜、二甲基亚砜、二苯基亚砜等。亚砜的通式为R-S-R',其中R和R'是有机基团。亚砜中的成键与氧化叔膦(RP=O)类似,由于电负性差异,S=O键中,硫显正价,氧显负价:硫原子为四面体结构,有一对孤对电子,类似于sp杂化的碳原子。当硫所连接的两个基团不相同时,就会产生手性,比如甲基苯基亚砜。有时构型转换所需的能量相当高,室温下的外消旋速率很慢,以至于对映体相对稳定。有些手性的亚砜在医药中有应用,比如埃索美拉唑和阿莫达非。此外,亚砜也被用作手性辅助剂。亚砜中硫的氧化态为0,处于硫醚(-2)和砜(+2)之间。因此氧化硫醚时,依次会得到亚砜、砜。例如氧化二甲基硫醚时可以得到二甲基亚砜和二甲基砜。很多有手性的亚砜可从非手性的硫醚为原料,在过渡金属和手性配体的存在下以不对称催化氧化反应合成。亚砜可由强碱如氢化钠去质子化,很多亚砜(如DMSO)都是很好的配体和烷化剂。","text2":"在医药中应用的亚砜有哪些?","label":1} {"text1":"法比安·恩斯特(Fabian Ernst,),出生于德国汉诺威,是德国足球运动员,司职防守中场,效力土耳其联赛班霸比锡达斯。出身于德国小型球会汉诺威96,曾于1998-2000年间效力汉堡。效力汉堡时共上阵48场,没有进球。恩斯特终其球员生涯,效力最大的球会乃云达不莱梅。这也是恩斯特最成功之时。他于2000-05年间效力,一共出赛152场德甲,打入11球。这段期间恩斯特随球会赢得2004年德甲和德国杯双料冠军,并于翌年打入欧洲联赛冠军杯。2005年他转会到沙尔克04,可是却只能取得两届德甲亚军(2005、2007)。2009年,恩斯特征战海外,转至土耳其班霸比锡达斯。恩斯特乃于2002年首次进入德国国家队。惟迄今仅参加过2004年欧洲国家杯一项大型赛事。截至2007年,他代表国家队出场24次,打入1球。","text2":"恩斯特效力于哪支球队?","label":1} {"text1":"阿加德兹大区(Région de Agadez)是尼日尔的一个大区,位于该国北部。东邻查德,西邻阿尔及利亚,北邻利比亚。面积634,209平方公里,是非洲面积最大的行政区,该地区占尼日尔全国面积约52%,但因处于撒哈拉沙漠腹地而人烟稀少,2012年人口487,620人。气候极其干旱,该地没有任何河流和湖泊。首府阿加德兹。下分3县。该地曾是黑非洲和北非交易的通道,在20世纪90年代后期,旅游业成为当地的大产业,又发现了铀矿,这为尼日尔提供了约20%的外汇。但自上世纪70年代起当地就有图阿雷格人的叛乱,反对尼日尔政府.原因是干旱的气候造成多次人道主义危机,铀矿收入并没惠及当地人。近年来基地组织和博科圣地也开始活跃,广阔的撒哈拉沙漠成为各种极端势力盘踞的窝点。阿加德兹大区分为1个市,3个省。","text2":"为何该地区的人口十分稀少呢?","label":1} {"text1":"马贵(),字世卿,河北省涞水县人,为民国初年武术名家,尹派八卦掌代表人物之一。马贵家中在北京开设永业木器行,因此外号「木马」,与其师叔,绰号「煤马」的马维祺,合称二马。擅长绘画,常绘螃蟹送人,所以又被叫做「螃蟹马」。马贵18岁时拜尹福为师,又得到其师祖董海川的传授,因此八卦掌功力极高。其走掌时身势低矮而发劲火爆,精通点穴,好与人比试,曾经以一记切腕将对手手臂打断,而名闻北京。又精通判官笔、七星杆、子午鸳鸯钺等兵器。民国元年(1911年),受聘为总统府武职教官。民国19年(1920年),任宪兵学校武术教官。民国17年(1928年),许兰州创立河北国术馆,聘为顾问。马贵受到尹福及董海川的指导,在八卦掌上有很深的心得,但吝于传授,虽然弟子极多,但少人得其真传。因其不擅理财,不事生产,晚年经济状况不佳,生活孤苦,皆靠弟子奉养及朋友接济。弟子有王培生、李少庵、刘万川、李保华等人。","text2":"马贵为什么又被称之为“螃蟹马”?","label":1} {"text1":"第十一次伊松佐河之役(Eleventh Battle of the Isonzo),是第一次世界大战期间,伊松佐河战役中的一部分。1917年8月18日至9月15日,意大利陆军总司令、陆军元帅卡多尔纳集结51个师的兵力、5,300门火炮,发起第十一次战役。奥军统帅部急调11个师的兵力增援第5集团军。意大利第2集团军对巴因西扎高地实施主要突击,第3集团军在戈里齐亚至海滨之间实施辅助突击。8月19日,意第2集团军所属第24、第27军强渡伊松佐河,突破奥军三道防线,推进10公里,占领具有战略价值的巴因西扎高地,后因伤亡太大,炮兵和后勤补给困难,被迫于8月29日停止进攻。意第3集团军虽有协约国和联合舰队的舰炮支援,但被奥军左翼所阻。9月4日,奥军发起反击未果。此战共造成意大利军损失16.7万人,奥军伤亡也惨重,仅被俘即超过3万人,最后被迫求援于德国。","text2":"十一次战役是谁发起的?","label":1} {"text1":"陈妍希(,),本名陈玫璇,中华民国女演员、女歌手,出生于中华民国台北市,高中时赴美求学。2006年返回台湾演出偶像剧出道。2011年演出九把刀电影《那些年,我们一起追的女孩》中的女主角沈佳宜而走红,获第48届金马奖最佳女主角提名。2014年演出神雕侠侣中的小龙女,获得第17届华鼎奖-中国古代题材电视剧最佳女演员奖。2016年以悬疑惊悚片夏威夷之恋(Pali Road)(原名:未知人生),荣获第32届洛杉矶亚太电影节「剧情类最佳女主角奖」,首度挑战全英文对白。2016年7月21日与陈晓结婚,同年12月20日于台北产下一子。陈妍希高中时即赴美念书,也在美国念大学。2006年自美国南加州大学毕业后返回台湾,隔年签给可米瑞智文化传播事业有限公司董事长兼总经理柴智屏,随后参与三部台湾偶像剧的演出。2009年,陈妍希参与她第一部电影的演出,在《听说》中饰演听障游泳选手一角,也因此片获得金马奖的提名。2011年,因饰演电影《那些年,我们一起追的女孩》中的女主角沈佳宜而走红,被封为国民女神、气质女神。2013年推出第一张音乐专辑《Me, Myself, and I》。同年出演中国大陆湖南卫视的电视剧《神雕侠侣》中的小龙女一角,以此作品获得第17届华鼎奖-中国古代题材电视剧最佳女演员奖。2015年8月26日,承认与男星陈晓交往。当天双方经纪人证实此消息经陈晓和陈妍希在8月27日在各自微博上以爱心公布恋情,也是继李铭顺、范文芳后第二对「神雕侠侣」。二人于2016年7月5日领取结婚证,同日宣布怀孕。2016年7月19日,二人婚礼在北京雁栖湖举办,共有300多名嘉宾出席,之后将于7月21日在台湾台北举行归宁宴。","text2":"陈妍希荣获第17届华鼎奖什么奖项?","label":1} {"text1":"渤海武王()讳大武艺,是渤海国第二代君主,在位期间719年至737年。大武艺弃唐朝正朔,建元为仁安。不过,在与唐朝交往时仍然用唐朝的开元年号。他在任期间,渤海国的疆域扩张东至绥芬河、及今俄罗斯沿海州、南至朝鲜半岛大同江、西至松花江流域。726年,他阻碍黑水靺鞨归附唐朝,他亲唐的弟弟大门艺逃至长安。727年,派遣高仁义出使至日本以遏制新罗, 728年,日本遣使向渤海答谢。 从此渤海国一直与日本维持着很好的外交和商贸关系。732年,大武艺派将军张文休率领水军跨渤海湾进攻唐朝登州(山东蓬莱),杀害登州刺史韦俊。葛福顺奉唐玄宗之命率兵征讨渤海国。733年,新罗奉唐朝之命从南方攻击渤海,由于大雪,士卒死者过半,无功而还。大武艺十分怨恨大门艺,秘密派刺客在洛阳天津桥南暗杀大门艺,但大门艺没有死;唐玄宗命令河南府搜捕大武艺派来的刺客,把他们全部杀死。737年,大武艺去世,谥号武王。《桓檀古记》称大武艺被尊为光宗、武皇帝。","text2":"渤海武王在任期渤海国的疆域有多大?","label":1} {"text1":"板桥体育场,由台湾新北市政府负责管理,是新北市板桥区的体育运动场区,邻近新北市政府及板桥车站。整个场区又可细分成板桥第一运动场区(新北市板桥区汉生东路278号)及板桥第二运动场区(新北市板桥区民权路117号)。新北市政府在市内另设有新庄体育场(新庄区)及树林体育场(树林区)两个体育运动场区。地址:22066新北市板桥区汉生东路278号民国七十六年(1987年)因应当时台北县承办台湾区运动会时而兴建此馆,在民国九十二年(2003年)又因为当时台北县举办中华民国全国运动会时整修场馆完工,并在台北2009年夏季听障奥林匹克运动会中,做为足球比赛预备场地。平时提供各项田径赛事(包括田赛项目:跳高、撑竿跳高、跳远、三级跳,铅球、铁饼、标枪、链球;径赛项目:短跑、中跑、长跑、接力跑、跨栏和障碍跑等等各项目训练及比赛场地,或作为各项活动租借用场地使用,夜间时段并有照明设备提供一般民众运动使用。民国六十五年(1976年)兴建,民国八十九年(2000年)九月整建完成。地址:22066新北市板桥区民权路117号一、面积:32,633平方公尺。二、设施:三、特色:民国六十八年(1979年)兴建,民国八十四年(1995年)四月整建完成。地址:新北市板桥区中正路8号一、面积:9,183平方公尺。二、设施:三、特色:是一座可供室内排球、篮球、羽球、手球、桌球、柔道、跆拳道及旱冰曲棍球等多功能室内体育馆。为大型演讲及演唱会等各项活动之场所。包含板信公园、社后公园与三角公园。一、面积:二、设施:三、特色:","text2":"板桥体育场由哪个政府负责管理?","label":1} {"text1":"格兰特·列比达(,)是一名英格兰职业足球员。他主要担任中场,但同时能以右翼上阵。他现时效力英冠球队米杜士堡。列比达自小是新特兰球迷。他在2003年加入新特兰青年军。他在2003年9月首次代表新特兰一队在英格兰联赛杯后备出战哈德斯菲尔德。在2005年9月,列比达被外借至洛达咸,自此取得更多上阵机会。在新领队-{zh-hans:罗伊·基恩; zh-hk:坚尼;}-的带领下,列比达出战了所有2006\/07年球季新特兰的比赛,射入7球,为球队第三入球最多球员。在2008年6月,列比达续约至2011年。2009年9月1日列比达转投英冠球会叶士域治,转会费265万英镑,签约三年。2012年5月29日列比达于约满叶士域治后,以自由身转投另一支英冠球队米杜士堡,签约三年。列比达首次代表英格兰U21在2009年欧洲U-21足球锦标赛外围赛出战保加利亚U21,入替-{zh-hans:阿格邦拉霍;zh-hk:艾邦拉荷}-。球队赢2-0。","text2":"格兰特·列比达在哪一年加入了新特兰青年军?","label":1} {"text1":"达则错(),又名达克次湖、达格济错,藏语意为虎顶湖,位于西藏那曲地区尼玛县的一个断陷盆地内,北纬31度49分~31度59分、东经87度25分~81度39分,羌塘高原北部。达则错长21.1公里,最大宽16.9公里,平均宽11.60公里,面积244.7平方公里。湖面海拔4459米,水深31.7米,湖水湛蓝,湖心透明度达7.2米。东西两岸地势平缓开阔,南北岸则山体陡峭。湖岸上分布着多条非常明显的同心古岸堤(见卫星影像图),最高的一条高于现湖面90米,显示出达则错在冰河时期无论面积、水深均远甚于当前。达则错地处干旱、寒冷的北羌塘,年均气温0~2.0度,年降水量仅200毫米,湖水补给主要依赖波仓藏布(又名莫昌藏布)、那若曲。波仓藏布发源于藏北高原中部的雪山,长约200公里。","text2":"达则错在哪里?","label":1} {"text1":"第四次马其顿战争(前150年-前148年)是罗马与马其顿的最后一仗。事缘公元前168年,第三次马其顿战争结束,罗马人废黜马其顿最后的国王珀尔修斯,但马其顿人都不甘心成为罗马的附庸,直至自称是珀尔修斯的亲生子的安德里斯库斯于前150年夺回马其顿王座企图复辟,激怒了罗马政府。尽管安德里斯库斯取得过早期的胜利,在前148年成功控制了色萨利,并和迦太基结盟,但同年即被罗马将军昆图斯·凯西里乌斯·梅特卢斯在第二次皮德纳战争中击败。安德里斯库斯出逃色雷斯,但已不再获得支持,反而被送到罗马,标志着动乱的结束。安德里斯库斯在马其顿短暂的统治,被定性为残暴的、压榨的。马其顿最后成为罗马的一个行省。","text2":"第四次马其顿战争是哪个国家的战争?","label":1} {"text1":"《武者回归》(英语:Returner),2002年上映的日本电影。是使用了大量电脑特技的科幻电影、动作片。主演是金城武和铃木杏。2084年的地球被叫做「dagura(达古拉)」的外星人侵略,人类面临灭绝的危机。在最后的人类反抗军基地被攻破之刻,反抗军战士、少女美里通过时间机器,超越82年的时空,来到2002年10月15日,消灭地球上最初降临的达古拉。阴错阳差下,她出现在一个人口贩卖交易的现场,职业杀手宫本为复仇而刺杀黑社会头目沟口的枪战中,导致前者刺杀失败。宫本将美里当作偷渡客带回家中,美里将自己的身份告诉宫本,希望宫本能够帮助自己找到并消灭第一只降临的达古拉,阻止它将外星大军引到地球。宫本不相信美里的故事,美里用超科技在宫本颈部贴了炸弹,让宫本被迫跟美里一起行动。两人开始了寻找达古拉的行动。与此同时,沟口受黑社会大佬指派探查不明飞行物的事情,在科学研究所发现了配有超科技强大火力的外星战机和在生的外星人。沟口决定独占外星战机和外星人,雇佣了武装佣兵要将其劫出。而宫本和美里也查探到了外星人在研究所的线索。两人潜入研究所准备杀死外星人,美里却发现外星人并不是达古拉。这时沟口和佣兵也赶到,双方火拼后,美里和宫本用计逃出。沟口劫出外星战机和外星人后要将宫本和美里灭口。两人一面逃避沟口的追杀,一面思考如何阻止战争。从外星人给出的信息中,两人猜测可能是沟口杀死了外星人,而导致战争爆发。两人遂出发阻止沟口的行动,但反而被沟口识破。激战之后,美里被沟口劫持,宫本利用美里留下的超科技武器最终打败了沟口和他的手下,救回了外星人。最终,外星人派出飞船来接应同伴,原来达古拉就是穿上了拟态战斗服的外星人。两人最终阻止了战争爆发,宫本也成功复仇,杀死了沟口。然而这时,由于未来被改变,美里的身体慢慢消失……","text2":"2004年的地球被叫做什么的外星人侵略?","label":1} {"text1":"穴醚是一类人工合成的,可以与阳离子发生配位的双环和多环多齿配体。“穴醚(\"cryptand\")”一词是指该配体形如空穴,将底物分子容纳在里面。整个分子是一个三维的结构。因此与单环的冠醚相比,穴醚配合物更加稳定,对底物分子的选择性也更强。形成的复合物具有脂溶性。唐纳德·克拉姆、让-马里·莱恩和查尔斯·佩特森通过对穴醚和冠醚进行研究,开创了超分子化学的先例,并因此获得了1987年的诺贝尔化学奖。最为常见且最为重要的穴醚是N[CHCHOCHCHOCHCH]N(右图),IUPAC名称为1,10-二氮杂-4,7,13,16,21,24-六氧杂双环[8.8.8]二十六碳烷,俗称[2.2.2]<\/nowiki>-穴醚。方括号内的数字表示在两个氮桥头之间每个桥上的氧原子个数。全胺穴醚对碱金属阳离子具有极高的亲和力,通过穴醚与碱金属作用,可以成功得到含K等碱金属阴离子的盐类。穴醚的三维内部空腔可以和外来离子紧密结合,形成的复合物被称为穴状化合物。结合能力最强的是较硬的阳离子,包括NH(铵离子)和镧系元素、碱金属、碱土金属的阳离子。穴醚利用分子中的氮和氧与这些离子配位,由于不同的离子与不同三维结构的穴醚结合能力不同,通过选取适当的穴醚,可以将不同的碱金属阳离子区分或分离出来。与冠醚类似,大环穴醚一般也是利用胺和卤代烃的缩合反应制备的。但由于环系更为复杂,穴醚的产率通常不高。穴醚的制备较为困难,且价格昂贵;但是,与冠醚之类的其他配位剂相比,穴醚与碱金属离子结合更紧密,选择性更强。 通过配位,穴醚可以将一般情况下不溶于有机溶剂的盐类溶于另一相中,用作相转移催化剂,加快化学反应的速率; 也可以稳定碱金属负离子,使碱化物和电子盐得以合成。另外,穴醚还可以帮助Sn之类津特耳离子(Zintl ion)的结晶。在制作核医学需要的显像用药物氟代脱氧葡萄糖(简称 FDG)时,会使用[2.2.2]穴醚来络合反应物KF中的钾离子,提高放射性F离子的亲核性,以便将F连接到脱氧葡萄糖中。","text2":"通过穴醚与碱金属的作用可以得到什么?","label":1} {"text1":"诺夫哥罗德-谢韦尔斯基(乌克兰语:Новгород-Сіверський;俄语:Новгород-Северский)乌克兰切尔尼戈夫州的一座古城,是诺夫哥罗德-谢韦尔斯基县的首府。诺夫哥罗德-谢韦尔斯基位于杰斯纳河河畔,距基辅约330千米,距乌俄边境约45千米。当前人口约为15000人。诺夫哥罗德-谢韦尔斯基于1044年第一次见载于史册。在古罗斯时代,该地是一个重要的公国。所有诺夫哥罗德-谢韦尔斯基王公都属于留里克王朝。著名的史诗《伊戈尔远征记》的主人公伊戈尔·斯维亚托斯拉维奇就是诺夫哥罗德-谢韦尔斯基的公爵。1239年,西征的蒙古人摧毁了诺夫哥罗德-谢韦尔斯基。后来它被向东扩张的立陶宛大公国吞并。1500年,诺夫哥罗德-谢韦尔斯基王公瓦西里·伊万诺维奇·舍米亚契奇携封地归顺莫斯科大公伊凡三世,使该城重归俄国人所有。但在留里克王朝绝嗣后的混乱时期(俄国历史上著名的空位时期),该城再度被波兰(当时与立陶宛组成王朝联合)占领。罗曼诺夫王朝建立初期,因国力低迷也没能夺回该城。直到沙皇阿列克谢·米哈伊洛维奇在位时期,俄国在俄波战争中获胜,才最终收复了诺夫哥罗德-谢韦尔斯基(同时夺取了第聂伯河左岸乌克兰)。","text2":"诺夫哥罗德-谢韦尔斯基当前的人口约是多少人?","label":1} {"text1":"包天笑()是包公毅的笔名。20世纪中国通俗文学作家,鸳鸯蝴蝶派作家。包天笑是江苏省苏州府吴县人。1894年考取秀才,此前2年已经开私塾教书为生。1900年与友人合资在苏州开设东来书庄并发行《励学译编》,经销中国留学生在日本出版的《浙江潮》等期刊,以及日文书刊。1901年,包天笑与其表兄尤志选(号子青)在苏州护龙街砂皮巷口创办大众化报纸《苏州白话报》。1906年,包天笑定居上海爱文义路胜业里,任《时报》外埠新闻和副刊《余兴》编辑,并编辑《小说时报》和《妇女时报》。1912年,包天笑去商务印书馆兼职,参加编写国文教科书。此后先后出任文明书局刊物《小说大观》和《小说画报》,大东书局《星期》周刊,以及《立报》《花果山》副刊主编。包天笑的翻译作品有《空谷兰》、《梅花落》、《迦因小传》、《天方夜谈》等。后来著有不少白话小说。他本人也兼任明星影片公司编剧,将其中一些作品改编为电影。包天笑参加过几个文人社团。1909年加入南社。1922年加入青社。1947年,包天笑前往台湾,1949年后定居香港,写作《钏影楼回忆录》。1973年10月30日,包天笑在香港病逝,享年97岁。","text2":"包天笑属于中国近代小说的哪个流派?","label":1} {"text1":"红尾蝴蝶鱼,又称黄蝴蝶鱼,俗名黄网蝶,为辐鳍鱼纲鲈形目蝴蝶鱼科的其中一种。本鱼分布于西太平洋区,包括菲律宾、印尼、中国南海、东海、日本、台湾、越南、马来西亚、澳洲北部、新几内亚、所罗门群岛、诺鲁、斐济群岛、马里亚纳群岛、马绍尔群岛、密克罗尼西亚、帛琉等海域。该物种的模式产地在安汶岛。水深4至20公尺。本鱼鱼体背面为淡褐色,腹面为青灰色,上有许多平行的棕色横线,及斜向臀鳍方向的斜线互相交错,造成许多近于菱形的格子,其越往身体后方越小。而约在背鳍鳍条后半部处有块橘色区域自背鳍延伸至臀鳍处,其前缘略呈弧形。尾鳍上亦有块弧形橘色带。具黑色眼带,在额部亦有一黑斑。背鳍硬棘13枚、软条22枚;臀鳍硬棘3枚、软条16至17枚。体长可达14公分。本鱼栖息在珊瑚礁区或港湾的外缘珊瑚礁较茂密处,属肉食性,以底栖小动物为食。色彩鲜艳的观赏性鱼类,不供食用。","text2":"黄蝴蝶鱼的栖息地在哪里?","label":1} {"text1":"《蜘蛛人》(原题:)是1978年(昭和53年)5月17日~1979年(昭和54年)3月14日期间,在东京电视台播出的特摄剧集,全41集(首播时时追加重播了两集)。当时东映是向美国惊奇漫画申请了角色的使用版权。本作品使用了惊奇漫画版蜘蛛人的角色,加上东映原创的故事和设定,并坐上巨大机器人「雷奥柏顿」战斗。从此以后「英雄坐上机器人,和巨大化后的怪人战斗」这一公式剧情,被继承到往后的超级战队系列中。东映与惊奇漫画签定了「3年内可使用对方的角色」的协议,而产生了此作品。本来是打算做成以日本武尊为主角,蜘蛛人为配角的作品,但最后的成品蜘蛛人是主角。山城宇宙考古学的山城博士、赛车手拓也、妹妹新子、弟弟拓次一家和平地生活著。有一天,拓也听到了不思议的声音「我的兄弟啊!来吧兄弟!我的兄弟啊…」。原来那是400年前,被率领铁十字团的怪物教授灭族,而执意复仇的蜘蛛星王子加利亚的声音。听得到这心电感应的,全宇宙只有一人。加利亚帮被暗算的拓也注射了蜘蛛提取液,让他变成了蜘蛛人,和怪物教授和铁十字团战斗。山城拓也穿上「蜘蛛防护服(Spider Protector)」后就会变成蜘蛛人,拥有蜘蛛的能力,可以在墙上、天花板爬行。并能用「蜘蛛感应」察知敌人。弱点是寒冷。另外一大特色是登场时的对白。本作在蜘蛛人登场时会在敌人前摆姿势,并说出特殊对白。这时会播放主题曲的前奏作背景音乐。而当蜘蛛人在做出场宣告说自己是“Spider-Man”时。于漫画蜘蛛宇宙里大量来自不同平行时空的蜘蛛人遭到被拥有非凡力量的继承者(Inheritors)家族猎杀捕食,发现此事的究极蜘蛛人(八爪博士)四处召集各平行时空里尚为生存的平行宇宙蜘蛛人们与继承者家族对抗。在宇宙队长的能量被继承者长老吸收后山城拓也驾驶Leopardon及时出现支援,但Leopardon终究不敌吸收宇宙能量能力大为提升的长老而遭击坠,事后拓也生存了下来并加入了究极蜘蛛人的团队。","text2":"《蜘蛛人》何时被播出?","label":1} {"text1":"自由软体铸造场(官方英语名称:),隶属于中研院资讯科技创新研究中心,乃一负责研究推广自由软体/开放原始码软体的学术单位。于2003年,获得中华民国经济部工业局的专案补助而成立。2015年,因中华民国科技部政策改变,决定不再补助,专案计划于年底停止。一言以蔽之:推广自由软体,OSSF Supports Software Freedom。铸造场的目标为培育台湾自由软体社群与人才。其实际工作为:在自由软体铸造场建置专案,可设定源码授权,图文授权,相关叙述以及是否隐藏专案。关于申请资格,必须在网站上注册之后,填写相关资料,再经由人工审核,约需耗费最长72小时的时间。","text2":"关于申请资格约需耗费多久?","label":1} {"text1":"阿部定吉(、1505年(永正2年) - 1549年(天文18年)),战国时代武将,通称大藏。父亲为阿部定时。阿部四郎兵卫定次是其弟。阿部定吉是仕奉德川家康的祖父·松平清康、父亲·松平广忠的家臣。1535年,松平家的七代当主松平清康,遭到阿部定吉之子阿部正丰暗杀,后阿部定吉成为松平广忠的家臣。请参阅守山崩条目。清康叔父·樱井松平信定,乘清康之死,和清康之弟·藏人信孝在合战中占据了冈崎城。信定认为松平广忠的存在是个危险要素,于是准备加害于他。于是广忠逃到伊势国依附于吉良持广。持广死后,广忠逃到骏河国依附于今川义元以寻求新的靠山,并且从今川氏成功借到了兵马。广忠和清康之弟2人·信孝(与信定不和)、康孝还得到大久保忠俊的协力,夺回了冈崎城。清康死后,广忠在外足足漂流了5年。此后,表面上和广忠合作的信孝,觊觎着康孝的遗留领地。广忠和阿部定吉商议后,趁信孝前往骏河今川义元处时,攻下信孝的三木城,把他放逐了。其他方面,大久保忠俊也暂时离开了广忠。虽然自己本身没有罪,但是对杀害了清康的自己儿子的罪却深深地自责着。阿部定吉没有立继承人,使血统断绝了。此后,同族阿部正胜一脉势力开始强盛,江户时代多人就任老中。","text2":"阿部定吉生活在什么时代?","label":1} {"text1":"丙烯酸,又称压克力酸,是化学式为CHO的有机化合物,是最简单的不饱和羧酸,由一个乙烯基和一个羧基组成。纯的丙烯酸是无色澄清液体,带有特征的刺激性气味。它可与水、醇、醚和氯仿互溶,是由从炼油厂得到的丙烯制备的。丙烯酸可发生羧酸的特征反应,与醇反应也可得到相应的酯类。最常见的丙烯酸酯包括丙烯酸甲酯、丙烯酸丁酯、丙烯酸乙酯和丙烯酸-2-乙基己酯。丙烯酸及其酯类自身或与其他单体混合后,会发生聚合反应生成均聚物或共聚物。通常可与丙烯酸共聚的单体包括酰胺类、丙烯腈、含乙烯基类、苯乙烯和丁二烯等。这类聚合物可用于生产各式塑料、涂层、粘合剂、弹性体、地板擦光剂及涂料。丙烯酸有很强的刺激性辛辣气味,对皮肤有强烈刺激性及腐蚀性。眼部接触可能会造成无法治愈的角膜烧伤,吸入相当量的蒸汽可能会对呼吸系统造成刺激,导致昏睡或头痛。少量接触危害不大,但高浓度的接触可能会引起肺水肿。","text2":"丙烯酸有什么外部特征?","label":1} {"text1":"加西特王朝,是西亚两河流域古代王朝。公元前16世纪由加喜特人,克喜特人,喀西特人(Kassites)建立。又称加喜特巴比伦、巴比伦第三王朝。加喜特人可能原居于札格罗斯山脉中部,其语言由于语料缺乏而仍未归类,但其部份统治者有印欧人名字。公元前16世纪初加西特人占据巴比伦,建加喜特王朝。此时约当阿古姆二世时期。布尔纳布里亚什二世至卡什提里亚什四世期间(约前14世纪中~前13世纪后半),为其繁荣时期,与埃及新王国、赫梯帝国、亚述同为并立的大国。公元前13世纪后半期,亚述人和埃兰人入侵。约公元前1157年,加喜特王恩利尔纳丁·阿基被埃兰人拐走,加喜特王朝灭亡。加喜特人在入主两河流域后,将侵占的土地分配给本族的朝臣、贵族和官吏,形成大地产私有主。其政治体制基本上是贵族政治,后期地方贵族势力日趋强大,王权削弱。加喜特人恢复了两河流域的秩序、和平和统一,特别是在其统治的中后期,社会经济有所发展,巴比伦、尼普尔、西巴尔等城市经济相当繁荣。人们在两河流域推广用于牵引的马和战车,并在建筑物上以砖刻浮雕代替石雕。这一风格为以后的新巴比伦王国和阿契美尼德王朝所继承。加喜特人全面接受两河流域固有的文化、宗教和阿卡德语言文字,迅速塞姆化。其国王重建和修饰尼普尔、拉尔沙、乌尔、乌鲁克等地的神庙。这一时期的重要遗物“界碑”,实际是国王所授地产的凭证。史称此时为界碑时代,即得名于此。界碑的右面或上部刻神像或神的象征,如以圆盘象征太阳神沙马什,以月牙象征月神欣,以锄头象征马尔杜克等;左面或下部的铭文多为王授土地情况。","text2":"哪个王朝又称加喜特巴比伦、巴比伦第三王朝?","label":1} {"text1":"律伦(,)是英国英格兰的郡、单一管理区。除了行政总部渥咸、小镇阿平厄姆外,郡内都是人口不多的小村庄。律伦既是名誉郡,又是单一管理区,无论把它看待成那种身分,它的人口和面积都分别是38,300、382平方公里。律伦伯爵、律伦公爵是英格兰贵族的头衔。首位律伦伯爵Edward of Norwich, 2nd Duke of York在1385年受封。首任律伦伯公爵约翰·曼纳斯在1703年受封,现任公爵大卫·曼纳斯在1999年受封。《1894年地方政府法案》生效后,律伦郡下辖3个乡区(Rural district):奥克姆区(Oakham Rural District)、阿平厄姆乡区(Uppingham Rural District)、凯顿乡区(Ketton Rural District)。1911年,奥克姆在奥克姆乡区分割出来,成为urban district。《1972年地方政府法案》在1974年4月1日生效,拉特兰成为莱斯特郡的非都市区。1992年成立的英格兰地方政府委员会(Local Government Commission for England)建议律伦成为单一管理区,不再是非都市区,在1998年4月1日生效至今。律伦郡在2001年人口普查中统计得人口34,560,相比起1991年人口普查统计得人口33,228,上升了4%。下图显示英格兰48个名誉郡的分布情况。律伦东北与林肯郡相邻,东与剑桥郡相邻,南与北安普敦郡相邻,西、西北与莱斯特郡相邻。","text2":"律伦的人口和面积是多少?","label":1} {"text1":"Ubuntu Studio 是一个Ubuntu官方认可的Ubuntu作业系统衍生版本 。这个版本主要是为一般的多媒体制作而设计。原始的版本是基于Ubuntu 7.04,在2007年5月10日发布。Ubuntu Studio 8.04包含的即时内核(real-time kernel)是专门为繁重的影音、绘图作业修改的。排程器使得应用程式可以取得立即的CPU时间,大大地减少音频延时 。 8.10版的发布则没有这个即时内核。Ubuntu Studio同时包含了一个艺术性主题,黑底蓝按键的主题与ubuntu本身的棕桔色形成了鲜明的对比。Ubuntu Studio目前已有Live CD版本,因此使用者可以透过图形化安装介面来进行安装。此外,映像档的大小约是2.7GB,一般的CD容纳不下,因此通常是由DVD安装。Ubuntu Studio也可以透过APT从网路安装。","text2":"原始的版本是基于什么?","label":1} {"text1":"毕耀明(Brian Butt Yiu-ming,)为现任北京泛美国际航空学校校长,前政府飞行服务队总监,前香港航空青年团总监。于1978年5月加入香港警务处,任职见习督察,1986年4月转往皇家香港辅助空军任职机师。1989年11月晋升为高级机师,1993年1月晋升为总机师,1996年8月出任政府飞行服务队总监,至2007年底提出请辞,共服务政府29年。 总机师陈志培于2008年3月27日起接任政府飞行服务队总监一职。毕耀明1954年出生于香港,在香港接受中小学教育,1977年在加拿大取得科学学士学位,1995年取得香港大学工商管理硕士学位,已婚,育有2名儿子。毕耀明大学毕业后返回香港,投考国泰航空机师失败,其后加入香港警务处,同时也是皇家香港辅助空军的志愿飞行员。他曾被派往商业罪案调查科,是首批受训的伪钞鉴别专家。1986年离开警队(当时职级为总督察),加入皇家香港辅助空军担任全职直升机机师,是部队唯一的华人机师,拥有定翼机及直升机驾驶执照,具直升机飞行教练资格。其后晋升高级机师及总机师,后于1996年8月1日出任政府飞行服务队总监,年仅42岁,成为最年轻的华人总监。2003年8月26日晚上,一辆政府飞行服务队海豚直升机由总部起飞,前往长洲接载伤者途中,于大屿山伯公坳附近的山头坠毁,两名机员罹难。这次意外,是政府飞行服务队于1993年成立以来,首次有队员坠机导致殉职的事故。民航处的调查报告指直升机师疏忽,并提出多项改善建议<\/ref> <\/ref> ,政府飞行服务队全数接纳。","text2":"毕耀明共服务香港政府多少年?","label":1} {"text1":"熊津()是百济中期的首都名字,它的位置在今天韩国公州市,位于首都首尔以南150公里。百济语发音为古莫那罗(),韩语中意为熊,意为国、国家。475年高句丽长寿王南扩占领百济旧都慰礼城(今首尔)。百济文周王再次迁都熊津。西元538年,百济圣王将都城自熊津(今公州市)迁移至泗沘。660年 唐朝灭亡百济,并在百济故地建立设置了5个都督府:“熊津、马韩、东明、金涟、德安五都督府”,纳入唐王朝直接管理。但随着后来的百济复国运动,唐朝在665年把五个都督府统一合并为熊津都督府。罗唐战争后,唐朝最终退出朝鲜半岛。熊津被统一新罗并入版图之中,新罗在熊津州设置都督府。757年 - 改称熊川。高丽时改称公州。详见「公州市」。","text2":"熊津的位置在今天什么地方?","label":1} {"text1":"纽约大街(Avenue de New-York)是巴黎十六区的一条街道。原名quai Debilly,1918年以日本首都东京更名东京大街(avenue de Tokio)。1945年2月26日,东京大街更名为纽约大街。路名的这些变化取决于政治局势。在第一次世界大战时,日本是法国的盟友,攻击德国在远东占有的港口青岛。在第二次世界大战中,日本是纳粹德国的盟友,因此在战后就以解放法国的美国的最大城市纽约命名这条路。此后在1964年,其下游的延伸线,塞纳河畔的帕西滨河路(quai de Passy),也更名为肯尼迪总统大街(avenue du Président-Kennedy)。东京宫(Palais de Tokyo)为1937年展览而建,得名于东京大街;1945年以后仍然保持了原有的名称。","text2":"东京大街在什么时间更名为纽约大街?","label":1} {"text1":"阿玛格斯战役(Battle of Amorgos),发生在公元前322年七月,是马其顿军队与雅典人所领导的希腊城邦联军在拉米亚战争中的最后一场海战,此战终结了雅典海军。当希腊城邦得知亚历山大大帝在公元前323年6月逝世后,爆发了对马其顿霸权的挑战。此时,克拉特鲁斯命令克利图斯作为舰队指挥官,在阿卑多斯战役击败雅典舰队,使安提帕特获得一些增援,但雅典舰队并未被毁灭,反而更进一步强化舰队力量,使雅典海军达到200艘战舰之多,使克拉特鲁斯的大军无法进一步对安提帕特提供援助,公元前322年七月两军舰队于萨摩斯岛以西60英哩处的阿玛格斯海域交战,克利图斯所率领的马其顿舰队再一次打败雅典将领伊厄泰翁(Euetion)的雅典舰队,此战使雅典海军遭到毁灭打击,从此雅典海军就此一蹶不振。对于失去制海权的希腊城邦来说,此战也注定拉米亚战争的失败,现在克拉特鲁斯的大军可以安全横过海峡,与安提帕特会军。","text2":"什么是马其顿军队与雅典人所领导的希腊城邦联军在拉米亚战争中的最后一场海战?","label":1} {"text1":"盘龙江又名滇池河,是云南省昆明市的一条河流,发源于嵩明县梁王山西麓的白沙坡,上游也叫牧羊河,在盘龙区松华街道注入松华坝水库,后由北向南流经昆明市区,最终在官渡区六甲街道洪家大村附近流入滇池,被昆明市民称为昆明的母亲河。盘龙江全长108公里,流域面积847平方公里,平均流量7.17立方米\/秒,年径流量2.75亿立方米,松华坝水库以上河段平均宽8米,以下平均宽35-40米,松华坝水库以下河段由人工改直。2005年昆明市区四个区(五华区、盘龙区、官渡区、西山区)重新划分时,盘龙江被作为东西分界线,得胜桥是昆明市主城四区的交界点。盘龙江上共有51座桥梁,从源头到河口依次为:龙川桥、幸福桥、神龙桥、大花桥、鱼龙桥、翠龙桥、巨龙桥、北仓桥、飞龙桥、蟠龙桥、升龙桥、霖雨路桥、霖雨桥、吴上村老桥、吴上村新桥、金色大道、跃龙桥、马村桥、官房桥、环北桥、敷润桥、圆通桥、桃源捌角桥、博润桥、南太桥、宝尚桥、得胜桥、小人桥、双龙桥、环南桥、永安桥、永平桥、官南立交桥、南坝桥、\"新建未命名\"、锦苑桥、乘龙桥、谷昌桥、博汇桥、向日葵忠字桥(陈家营)、广福路跨江桥、向日葵忠字桥(张家庙)、小钢桥(张家庙)、向日葵忠字桥(叶家村)、向日葵忠字桥(严家村)、向日葵忠字桥(金家社区梁家村)、小钢桥(梁家村)、向日葵桥(金家村)、环湖路桥、洪闸桥。元朝初期,赛典赤·赡思丁曾率人整治盘龙江,盘龙江上最早的龙川桥建于公元1280年前,相传为瞻思丁修建。","text2":"盘龙江全长多少?","label":1} {"text1":"时惠环球控股有限公司(简称时惠环球)是香港的一间零售公司,现时为时富投资旗下公司之一,并曾经在香港交易所上市(编号:996)。时惠的主要业务为营运香港大型家庭用品连锁店实惠家居广场。时惠环球前身为实惠集团,于1986年成立。2001年3月,实惠集团被时富投资集团收购,其后于2005年7月易名为现称。2006年8月,商人田琬善向时富集团收购时惠环球股份,锐意将该公司转型为中国大陆的零售业务,并将原有的香港业务售回网融(中国)控股。2007年10月,时惠环球的大陆业务使用了时惠环球的上市地位,并命名为东方银座()。而时惠环球的名称仍使用于时富投资所控制的零售业务。","text2":"商人田琬善向时富集团收购时惠环球股份,做了哪些改变?","label":1} {"text1":"YAWL(发音为`yorl')是Yet Another Workflow Language的简写。它是一个以对工作流模式的研究为基础而定义的工作流语言,旨在提供对绝大部分工作流模式的直接支持(传统工作流系统则提供最多六成左右的支持)。YAWL系统(通常也简称YAWL)是一个基于JAVA的开源工作流系统。它是对YAWL语言的一个软件化实现。整个系统主要由工作流引擎,流程定义工具,任务执行序列处理模块,工作流执行资源服务等组成,其面向服务的体系结构使系统具有较强的扩展性及(与其它系统的)交互操作性。YAWL工作流语言及系统最初是由荷兰爱因霍温科技大学(Technische Universiteit Eindhoven)与澳大利亚昆士兰科技大学(Queensland University of Technology)的学者们共同定义并开发的。此后,不断有一些集团组织如 InterContinental Hotel Group 和 first:telecom 等陆续参与此项目的开发并作出贡献。","text2":"YAWL是指什么?","label":1} {"text1":"韦斯特曼纳群岛(英文:The Westman Islands;冰岛语:Vestmannaeyjar),位于冰岛南海岸外的火山群岛,由14个小岛屿组成,总面积约21平方公里。岛上四处岩石裸露,一片荒芜,唯一有居民的是最大之岛屿赫马岛(Heimaey)。岛上有韦斯特曼纳埃亚尔镇。1963年-1967年西南部因火山喷发而使小岛叙尔特塞突出海面,当时,全群岛覆盖了一层火山灰。冰岛外海的一座火山岛,也是冰岛的最南端。它是因海面下130公尺火山爆发而形成,于1963年11月14日突出海面。火山喷发一直持续到1967年6月5日,岛的面积也达到最大值2.7平方公里。之后,由于风和波浪的侵蚀,导致岛逐渐变小,到了2002年,其面积为1.4平方公里。","text2":"是什么导致岛逐渐变小?","label":1} {"text1":"《欢迎来到东莫村》(,)是一部2005年的韩国电影,导演朴光贤首次导演电影,在韩国超过800万人次观看。电影以朝鲜战争为背景,表现了反战的主题,和对朝鲜半岛和平统一的思考。朝鲜战争期间,在汉白山下有个村子叫东莫村,村子的人不知道外界发生了战争,连枪械都不知道何物的一个如同“桃花源”之地。可是安宁的村庄外来的客人们打破宁静,联合国军的史密斯驾驶的战机坠落于村子的附近、三名朝鲜人民军的士兵和军官,以及两名韩国国军的官兵先后来到村子里,他们在这个村庄相遇,原本彼此相互存在著冲突,气氛紧张,可是当他们发现村民是如此地淳朴,而决定不再发生冲突而伤害村民。于是他们一起合作,共同抵御外来的入侵。","text2":"《欢迎来到东莫村》的导演是谁?","label":1} {"text1":"气腔龙属(学名:\"\")是种兽脚亚目坚尾龙类恐龙,生存于白垩纪晚期的阿根廷。气腔龙的化石是在1996年发现于门多萨省的Anacleto组地层,地质年代约8400万年前,相当于桑托阶。气腔龙的化石具有类似鸟类的呼吸系统。气腔龙目前只有唯一种,里约科罗拉多气腔龙(\"A. riocoloradensis\") 。属名在希腊文意为 \"aeros\"(空气)与 \"osteon\"(骨头);种名则意为科罗拉多河,意指发现化石的科罗拉多组。气腔龙身长9公尺,是种二足、肉食性恐龙,生存于白垩纪桑托阶,接近8400万年前。气腔龙的化石包含:一颗牙齿、一些头盖骨、数节完整或部分的颈椎、背椎、荐椎、数根颈部与胸部肋骨、腹肋、叉骨、左肩胛喙状骨、左肠骨、左右耻骨。从化石显示,该个体并非完全成熟体。气腔龙不属于该时期南方各大陆的任何大型兽脚类恐龙,包含:阿贝力龙科、鲨齿龙科、棘龙科。气腔龙可能代表一个新的族系,或是存活到很晚的基础坚尾龙类。保罗·塞里诺(Paul Sereno)认为气腔龙可能与异特龙超科有关联,是这群侏罗纪动物辐射演化的结果。根据2009年的新研究,有一群异特龙超科恐龙生存至白垩纪时期,体型修长、指爪类似棘龙科,这群动物被建立为新猎龙科,气腔龙被归类于新猎龙科的大盗龙类演化支。气腔龙的某些骨头具有中空空间,包含:叉骨、肠骨、腹肋,显示气腔龙可能具类似现代鸟类的气囊呼吸系统。气囊系统可协助空气进入或排出身体内,增加肺脏的呼吸效能。保罗·塞里诺推测,气腔龙骨头内的空间可协助体温循环,并增进呼吸效能。在2008年9月,保罗·塞里诺(Paul Sereno)等人将研究公布于网路期刊《PLoS ONE》。但当时《国际动物命名法规》并不承认网路期刊所发表的学名具有有效性,必需透过纸本发行的期刊发行,才会承认是有效的命名。在2009年5月,《PLoS ONE》杂志主编发行该研究的纸本补遗版本。因此气腔龙虽然是在2008年公布于网路期刊,但直到2009年才具有有效性。","text2":"气腔龙属生存于哪里?","label":1} {"text1":"三棱龙属(学名:\"Trilophosaurus\")是种已灭绝双孔亚纲爬行动物,生存于三叠纪晚期,外表类似蜥蜴,跟主龙类有接近亲缘关系。三棱龙是种草食性动物,身长可达2.5公尺。头骨短而厚重,有宽广平坦的颊齿,表面锐利,可切断坚硬的植物。前上颌骨与下颌前部缺乏牙齿,牠们生前可能有角质覆盖的喙状嘴。三棱龙的头骨也很独特,因为牠们的下颞孔消失了,使牠们看起来像是阔孔类爬行动物的头骨,所以三棱龙类最初跟楯齿龙目、鳍龙超目一起被归类于阔孔亚纲。在1988年,Robert L. Carroll提出下颞孔消失是为了使头骨强化。到目前为止,三棱龙的化石发现于北美与欧洲的晚三叠纪地层。","text2":"三棱龙属和哪些生物一起被归类于阔孔亚纲?","label":1} {"text1":"树皮是木质植物,例如树的茎和根最外面的部分。狭义的树皮包括三层:木栓、木栓形成层和栓内层,以及外部的各种死组织,广义的树皮还包括韧皮部。有的植物的树皮中含有各种生物碱、单宁、染料和香料等,可以提炼各种药材、毒品、毒药、树脂等,也可以用某些种类的树皮做软木、绳索、织布、造树皮船、绘制树皮画等,或直接用树皮作装饰。由外向内,树皮可分为外表皮、由木栓、木栓形成层和栓内层组成的周皮以及内里的韧皮部。外表皮是树木最外部的死组织,由角质化的细胞组成。周皮是韧皮部和外表皮之间的部分,包括木栓、木栓形成层和栓内层的总称,周皮形成后,表皮即脱落。木栓是树皮外层的主要成分,能隔绝水分和气体通过,对树有保护作用。木栓形成层通常只有一层或两层细胞,是分生生长木栓的组织,向外生长成木栓层,向内形成栓内层,不过在根部的木栓形成层是由中柱鞘转变的。栓内层是木栓形成层向内部分化出的一层细胞。韧皮部在木质部的树干和周皮之间,是树皮内部输送营养的部分。随着树皮逐渐生长加厚,外层组织逐渐死亡,狭义的树皮只包括木栓和外部的死组织。树皮的各个部分都有不同的用途,木栓的质地轻、富有弹性、不透水,木栓发达的树种如栓皮栎的树皮,可以用来制作瓶塞、救生圈、隔音板等,在北美洲和中国东北用桦树皮制作小舟和器物。有些树皮甚至可以食用,最有商业价值的树种是金鸡纳树和肉桂树,可以提炼药物奎宁和香料, 阿司匹林是从柳树皮中提炼的;夏栎(\"Quercus robur\")树皮是揉革用的单宁酸的主要原料;在园艺中,经常用树皮碎屑培育兰花等不能在土壤中生长的附生植物花卉。许多种类的昆虫、真菌和苔藓,附生在树皮上 。不同树种的树皮形状、色泽、瘢痕及脱落情况都不相同,树皮可以作为鉴定树木种类、年龄的重要依据。树木在树皮受伤后,可以自己生长出多余的木栓来修复。","text2":"阿司匹林是从哪种树皮中提炼的","label":1} {"text1":"溪头龙蜥(学名:),又名牧氏攀蜥,俗称竹虎,旧称牧茂氏攀蜥,为飞蜥科龙蜥属下的物种。仅分布于台湾,正模标本采集自南投县的针叶林中,由日本动物学家牧茂市郎采集。其生存的海拔范围为1500至1500米。体长 10~25 公分,最大可达 27 公分。头部有一条粗黑的过眼线,嘴部外缘与腹部体色较浅。雌雄体色差异甚大,体色会随环境而小幅度改变;雄性最显著特征在于身上有许多粗而黑的斑块(斑块颜色有个体差异),鬣鳞明显;雌性体色会因个体差异而有所不同,虽然大多以绿色为主(有些个体也有与雄性相似的粗斑块,但颜色较接近绿色)但有些个体除了斑块外,自头部经背脊中央至尾部均为暗褐色,形成一条宽纵带,称之为「棕背型」。为树栖性生物,日行性,白天常在森林边缘空旷处活动,且常在树丛面向光的叶面。领域性强,雄蜥遇惊扰时喉部会扩张,并做出类似伏地挺身的姿势,多以小型无脊椎动物为食","text2":"雄性有什么显著特征?","label":1} {"text1":"张家树()原名张端六,圣名类思,是天主教上海教区的第一任自选自圣的主教,任期为1960年至1988年。1893年(清光绪十九年),张家树出生于江苏省松江府南汇县西八灶(今上海市浦东新区康桥镇沿南村)的一个世代信仰天主教的家庭,这是一个历史悠久的天主教徒聚居村,村中有一所规模宏大的哥特式天主教堂(张家痛苦圣母堂)。后来就读于耶稣会在上海徐家汇创办的徐汇公学,1910年立志修道。1911年,张家树赴英国留学。1918年回国,1920年再度赴英,1923年在英国晋铎,先在法国巴黎、里昂等地的华侨教友中工作,至1925回国,任浦东傅家玫瑰圣母堂副本堂神父。次年任徐汇公学教导主任,1937年8月升为校长。1955年9月8日,上海教区主教龚品梅和一批神父被打成“龚品梅反革命集团”,被捕入狱。1960年4月23日,上海教区一批神父教友推举67岁的张家树为上海教区正权主教,4月27日,在徐家汇大堂由沈阳总教区总主教皮漱石主礼祝圣。上海教区主教座堂也迁到徐家汇圣依纳爵主教座堂。张家树总结这一时期说:“我60年圣主教后,政府只叫我做两件事:写材料,交代自己,揭发他人。”1966年“文化大革命”开始,蒯大富率领北京清华大学红卫兵“井冈山兵团”南下,以“破四旧”为名毁坏徐家汇圣依纳爵主教座堂,批斗神父和修女,并强迫他们污辱圣像和十字架。张家树对着圣像祈祷长跪不起。此后张家树被下放劳动,修伞、洗玻璃瓶子。1979年,张家树得到平反。1980年5月,中国天主教第三届代表会议召开,张家树当选为新成立的中国天主教主教团团长和中国天主教教务委员会主任。1982年10月开办佘山修院。1985年,年已93岁的张家树祝圣金鲁贤为自己的助理主教。1988年2月中旬,张家树在上海去世,享年95岁,其骨灰被安放在徐家汇圣依纳爵主教座堂。","text2":"张家树出生在什么地方?","label":1} {"text1":"昃臣道()是香港香港岛中西区的一条街道,位于中环香港会及遮打花园以西,旧最高法院大楼及皇后像广场以东,爱丁堡广场及干诺道中以南,德辅道中以北。在历史上,昃臣道是填海所得的地皮,平坦路是一条行车马路,两旁是行人路。昃臣道的南段是露天停车场,有权泊车的车牌号码是AM、LC等字头的香港政府及立法会议员车辆。同时该处也经常是指定示威区,见报率不少。昃臣道的名字来自香港19世纪末著名银行家及议员汤玛士·昃臣爵士。2008年夏季奥林匹克运动会香港区火炬接力曾经途经此道,火炬手是范徐丽泰。香港电影《英雄本色》第二场,周润发在香港会所大厦门前进食街头小食猪肠粉,就是在昃臣道拍外景。","text2":"2008年经过昃臣道的奥林匹克运动会火炬接力手是谁?","label":1} {"text1":"王宠惠内阁,以王宠惠为阁揆,成立于民国11年(1922年)9月19日,结束于同年11月29日。「法统重光」后,由于直系保派津派处处给黎元洪大总统设置障碍,先前两届内阁(周自齐、颜惠庆)都很快下台。之后,吴佩孚主张内阁成员应该由参加过华盛顿会议的代表和「好政府主义」的支持者组成,以改善政府形象。于是老同盟会成员、与胡适一起提倡「好政府」的王宠惠成为国务总理的人选。1922年9月王阁成立后,阁员大多公共形象良好,包括华盛顿会议的代表王宠惠和顾维钧、提倡「好政府主义」的罗文干和汤尔和,因此一开始被社会舆论寄予较大期望,时人称为「好人内阁」。胡适等人建议政府实现宪政、整顿国务、公开政务。但是这些期盼最终也没有得到实现,王宠惠政府在军阀的指挥下只能做「挡债借钱的事」,为直系筹备军饷。由于王宠惠惟吴佩孚之命是从,使曹锟、吴景濂为之不满,他们积极倒阁。11月18日,众议院议长吴景濂和副议长张伯烈以财政总长罗文干在签订「华义借款」合同时,没有经过国会和总理的批准即签字,涉嫌受贿,要求黎元洪将其逮捕(即「罗文干案」)。曹锟稍后也通电支持国会严查此案,得到各地不满吴佩孚的都督的相应。吴佩孚在压力下最终让步。11月29日,王宠惠内阁请辞。","text2":"胡适等人建议政府如何做?","label":1} {"text1":"欧阳玄(),字元功,号圭斋,因避清讳,其名又作欧阳元,其先家庐陵,为欧阳修之后,后迁居浏阳,故为浏阳人。生于是元世祖至元二十年(1283年),祖籍江西,后迁居湖南浏阳。其父欧阳龙生,入元后仕至道州路教授。欧阳玄自幼聪明,八岁能成诵,年十四下笔辄成章。元仁宗延祐二年(1315年),中式第三名进士。曾任翰林待制,兼国史院编修官,元统六年(1333年),任翰林院直学士,奉诏编修《泰定帝实录》、《明宗实录》、《文宗实录》和《宁宗实录》。官至翰林学士承旨。编有《辽史》、《金史》、《宋史》三史。又编有《太平经国》、《至正条格》、《经世大典》。至正十七年(1357年)病逝大都。赠大司徒、柱国,封楚国公,谥曰“文”。有《圭斋文集》15卷、《睽东记》传世。","text2":"欧阳玄有哪些作品传世?","label":1} {"text1":"齿䲠(学名:),又名东方狐鲣、烟仔虎、掠齿烟、乌鰡串,为狐鲣属的一种。本鱼分布于印度太平洋区,包括南非、东非、葛摩、马达加斯加、斯里兰卡、印度、马来西亚、韩国、中国、日本、台湾、越南、印尼、新几内亚、澳洲、夏威夷群岛、墨西哥、宏都拉斯、萨尔瓦多、瓜地马拉、哥伦比亚、厄瓜多、哥斯大黎加、尼加拉瓜、秘鲁等海域。该物种的模式产地在日本。水深0-80公尺。本鱼体呈纺锤型,横切面近圆形,两背鳍距离近,第二背鳍后方有7-9枚离鳍,臀鳍有6-7枚离鳍,尾鳍深分叉。头部无鳞,胸部鳞片大,形成胸甲,胸甲小,其后端不超过胸鳍。体背侧蓝灰色,具数条几乎平形的深蓝色纵带,其下方数条则断断续续,腹面银白色,背鳍硬棘17-19枚;臀鳍硬棘0枚; 臀鳍软条14-16枚;脊椎骨44-45个,体长可达102公分。本鱼栖息在大洋的中上层,为远洋鱼类,游泳速度很快,属肉食性,以摄食鱼类为主。本鱼为鲜美的食用鱼,尤以秋冬的味道更好,肉质细嫩,新鲜时可做成生鱼片、罐头、煮汤或盐烧。","text2":"齿䲠分布在哪个大洋区?","label":1} {"text1":"阎振兴(,,字光夏,河南汝南人,中华民国政治人物、水利学家。曾任国立台湾大学校长,在其任内,发生台大哲学系事件。在美国爱荷华大学取得工学博士学位,曾在黄河流域做过河川水利工作,1947年出任河南大学教授兼工学院院长。1949年随政府到台湾,在高雄港务局做总工程师,1957年出任台湾省立成功大学教授兼校长,1962年转任台湾省政府教育厅厅长,担任教育部部长期间宣布推动九年国教。曾任行政院青年辅导委员会主任委员、行政院国家科学委员会副主任委员、行政院原子能委员会主任委员、中山科学研究院院长、国立清华大学校长、国立台湾大学校长、总统府国策顾问、总统府资政。行政院原子能委员会主任委员任内发生辐射屋事件。1977年4月10日,阎振兴与第二任妻子严淑莲结婚。2005年1月7日,阎振兴因呼吸衰竭在台大医院病逝,享年94岁。24日,阎振兴追思会在台大医院景福会馆举行,总统陈水扁及副总统吕秀莲到场颁挽额「学渊绩懋」及「硕望清徽」;前后任台大校长如陈维昭、虞兆中,前财政部长陆润康也到场致辞。台大交响乐团同学彭靖如、盛维安、王年恺献奏送别与骊歌表达哀思。1970年5月7日,行政院会通过清华大学校长阎振兴调任台湾大学校长,6月1日11时举行交接典礼,接任台大校长。阎振兴于上任后在硬体方面设立了工业研究中心(1975);制度方面于夜间部增设了历史系(1971)、中文系(1972),成立台大校讯社、侨生及外籍学生辅导室(1973),实施电子计算机注册选课(1976)以及校园规画委员会的设置(1978)。阎振兴任内开创与中研院合作的管道。阎振兴于台大哲学系事件时为了熄灭学运风潮而消极处理,危害校园学术自由的传统。台大哲学系事件调查小组召集人柯庆明教授表示:「……当时的台大校长阎振兴在处理过程中,知道许多人是被陷害,但却为灭学运而任由当时代理系主任孙智燊整肃无辜的教师,对大学自主与学术尊严也是一大伤害。」1981年8月1日,阎振兴卸任,由担任过工学院院长的虞兆中接任。","text2":"1947年,阎振兴担任什么职务?","label":1} {"text1":"拜耳张力学说(Baeyer张力学说,或张力学说)是由阿道夫·冯·拜尔于1885年用以解释不同环烷烃的稳定性而提出的一个理论。这个学说认为,所有环状化合物都具有环平面结构,由于键角(即多边形内角)与\"sp\"杂化轨道正常键角(109°28')有差别,因此所有环系都存在角张力。这个偏转角可以用(\"sp\"杂化轨道正常键角 - 多边形内角)÷ 2 来计算。各常见环烷烃的偏转角可以依此计算出来,见下表。根据这些数据,可以认定大环化合物与小环化合物一样,环系越偏出五元环,偏转角越大,张力越大。由于张力越大,分子能量越高,分子越不稳定,故小环的环丙烷环系容易开环。这便是拜耳张力学说对不同环烷烃稳定性的解释。事实上,大环化合物是稳定的。除三元环和芳香环具有平面结构外,其他环都不是真正的平面结构,因此自然也就不存在所谓“偏转角”,拜耳张力学说是错误的。但它所提出的当分子内键角偏离正常键角时会产生张力的现象,却是存在的。这种张力称为角张力。","text2":"拜耳张力学说所认为的是什么?","label":1} {"text1":"卦限是笛卡儿坐标系中,象限在三维空间的对应术语,用于空间解析几何的坐标系统。空间直角坐标系用于确定空间的任意一点的位置。先在指定空间内的任意一点取定并标记点 \"O\",作为坐标原点。经过点 \"O\",画出三条互相垂直的直线,把它们分别标记作 \"x\" 轴、\"y\" 轴和\"z\" 轴。用右手定则规定各轴线的正方向。每二条轴线确定出一个平面,作为坐标平面。由 \"x\" 轴和 \"y\" 轴确定的坐标平面称作 \"xy\" 平面;\"x\" 轴、\"z\" 轴确定 \"xz\" 平面;最后一对,\"y\"、\"z\" 二轴确定 \"yz\" 平面。按照传统,将 \"xy\" 平面配置在水平面上,\"z\" 轴置于铅直位置,而 \"xz\"、\"yz\" 二平面在图上垂直标示。这三个坐标平面将空间分为八个部分,这便是空间直角坐标系的8个卦限。八个卦限在几何图中通常以罗马数字“I、II、III、IV、V、VI、VII、VIII”标示。较为普遍的卦限数序均以 \"x\" 轴正半轴、\"y\" 轴正半轴和 \"z\" 轴正半轴确定的卦限为“第一卦限”,罗马数字标记为“I”。第二、三、四卦限的数序类似平面直角坐标中象限的数序。在 \"xy\" 平面上向逆时针方向增加数序。而后第五至七卦限在 \"xy\" 平面下同样以逆时针方向标记。因卦限相对象限较为罕见,世界各地的数学家乃至不同时代的数学印刷物都曾使用过不同的数序来标记各个卦限,所以为了避免混淆,可以采用另一种标记卦限的方式。直接地,明确指出某卦限范围内包含的 \"x\"、\"y\"、\"z\" 坐标的正负,来标记那个卦限。如图1中的第一卦限(I)标作“(+,+,+)”;第四卦限(IV)标作“(+,-,+)”;第七卦限(VII)标作“(-,-,-)”。","text2":"笛卡尔坐标系通常用的罗马数字是什么?","label":1} {"text1":"Astro AOD是马来西亚Astro拥有的最新TVB剧集频道,该频道是由香港电视广播(国际)有限公司(TVBI)与马来西亚Astro合作开办,该频道全天候24小时播出。Astro On Demand剧集首映于2007年7月16日正式启播,该频道的标语是「港剧潮流,顶级享受」,该频道与香港同步播出最新、最热播的香港TVB剧集。此外Astro On Demand HD于2013年6月3日正式启播,该频道备有粤语及华语双声道服务,同时也备有中文及马来语字幕服务。Astro On Demand剧集首映于2017年5月,正式改名为「Astro AOD」。Astro AOD HD - 频道350 (HD)Astro AOD - 频道351 - 353,频道361 - 363 (SD)从2010年起,Astro AOD和MY FM(主办单位)开始与TVB协办“MY AOD我的最爱颁奖典礼”以取代现已停办的Astro华丽台电视剧大奖。此颁奖礼依据前一年11月至当年10月间在Astro AOD播出的剧集,让剧迷们上网投选心目中最爱的艺人和电视剧集,并送出丰富奖品,尤其回馈玉龙配套用户。此颁奖在2013年起改由TVB娱乐新闻台主办、Astro协办,并更名为“TVB 马来西亚星光荟萃颁奖典礼”。下表列出未曾在Astro AOD播出的TVB剧集。","text2":"Astro AOD是由谁开办的?","label":1} {"text1":"亨利·普赛尔(,),巴洛克时期的英格兰作曲家,吸收法国与义大利音乐的特点,创作出独特的英国巴洛克音乐风格。他被认为是英国最伟大的作曲家之一,独霸乐坛两百年,一直到20世纪初爱德华·埃尔加成名之前,没有一个本土生长的英国作曲家达到他的成就。生於伦敦的西敏地区,父亲曾任查理二世的宫廷乐师。少年时是王室小礼拜堂合唱团团员,嗓子受伤后任乐器保管员。曾从约翰·布洛学习。1680年成为西敏寺的管风琴师,1682年又兼任王室小礼拜堂管风琴师。1695年初,替病逝的玛丽女王创作葬礼歌曲——《\"\"》;7月时,他为王位的推定继承人格洛斯特公爵威廉王子谱写六岁的生日歌曲;他在11月猝逝,年仅36岁。据《音乐家的罗曼史》一书,有一天,普赛尔因夜归而被妻子拒谙门外,因感冒而冻死,葬于西敏寺内他的管风琴附近。普赛尔在短暂的一生中创作了大量的器乐、歌曲、话剧配乐及少数歌剧,在英国古典音乐历史上有重要的地位。他的歌剧《狄朵与埃涅阿斯》是英国歌剧名作。而他的戏剧配乐《摩尔人的复仇》中的一个主题更被20世纪作曲家布里顿用进《青少年管弦乐队指南》。","text2":"《摩尔人的复仇》中的一个主题被布里顿用进什么中?","label":1} {"text1":"秦泰山刻石,又称封泰山碑,秦始皇二十八年(前219年)东巡泰山时所立。碑高四尺五寸、宽一尺四寸,字型工整、笔画圆健,是秦始皇统一文字后的小篆,相传为宰相李斯所书。刻石北、东、西三面为秦始皇诏书,共144个字,南面为秦二世元年(前209年)所刻二世诏书78个字。刻石原立于岱顶,至明只存29字。清乾隆五年(1740年)遭火,刻石遂失;嘉庆二十年(1815年)在山顶玉女池中搜得残石2块,尚存10字,移至岱庙;现在位于泰山岱庙东御座的露台前西侧,仅存二世诏书中的10个残字:斯、臣、去、疾、昧、死、臣、请、矣、臣。最著名的拓本为宋拓百六十五字本,收165字,曾由明朝大收藏家安国收藏,现藏于日本东京台东区立书道博物馆。兹录刻石全文如下:","text2":"现在位于泰山岱庙东御座的露台前西侧,仅存二世诏书中的哪10个残字?","label":1} {"text1":"《大海》是台湾歌手张雨生发行第四张国语专辑。《大海》当时相当流行化的一张商业专辑,是张雨生歌唱生涯中商业成绩上最为成功的专辑,琅琅上口、激情澎湃的旋律以及细腻抒情的词曲意境,让《大海》成为张雨生在歌迷心目中非常杰出的作品,这张专辑总体制作水准很高,而音乐风格则相对趋于流行化。除了《大海》之外的其余几首主打曲可听性也都很不错(而其粤语版本由钟镇涛主唱),诸如《我是一棵秋天的树》、《I Don't Wanna Say Goodbye》以及《爱上你的一切》等,都成为张雨生流传广泛的作品,《心底的中国》这首歌是张雨生在军中写给父亲的作品,无论词曲都相当地令人动容。","text2":"哪首歌是张雨生写给父亲的歌?","label":1} {"text1":"革命法庭(Revolutionary Tribunal),于法国大革命期间,国民公会为了审判政治犯而在巴黎成立的法庭,为实行恐怖统治的强力机构。由一个陪审团、一个检察长和两个代检察长组成,均由国民公会任命,其判决定案后则不得再上诉。1794年6月10日,在罗伯斯庇尔鼓动下,颁布了《牧月22日法令》,禁止囚犯雇用律师为自己辩护,并规定死刑为唯一刑罚。在此以前,革命法庭于13个月内曾宣判1,220人死刑;在通过法令和罗伯斯比垮台之间的49天中,有1,376人被判处死刑,其中多为无辜之民众。送交法庭的犯人名单由民众委员会草拟,修订后由公安委员会和救国委员会联合签署。罗伯斯比是向法庭提供犯人的主要人物。革命法庭于1795年5月31日被撤销。","text2":"在通过法令和罗伯斯比垮台之间的49天中,有多少人被判处死刑?","label":1} {"text1":"明清音系指行用于明朝和清朝的汉语官话音系,是现代普通话的前身。和之前历朝一样,明朝和清朝都编有官方的韵书,如《洪武正韵》、《佩文诗韵》等。但这些韵书都承袭平水韵的音系,不能作为分析当时口语的依据。所幸的是,有一些私修的描述声韵的册子,不同程度地反映了当时的实际语音,如明兰茂作《韵略易通》,清无名氏作《圆音正考》。从明季开始,陆续有西方人到中国,他们用拉丁字母标注汉字的读音,为语音的实际音值留下了宝贵的材料。如明朝有利玛窦作《西字奇迹》,金尼阁作《西儒耳目资》。除此之外,还有一些对音材料作为补充。《韵略易通》的创作约当十五世纪,上有一首早梅诗,归纳了当时的声母系统,一共四句二十字,每个字代表一个声母。原诗为:东风破早梅,向暖一枝开。
冰雪无人见,春从天上来。由此得到明初声母如下表:一百多年后,意大利传教士罗明坚和利玛窦留下的手稿和书籍用拉丁字母记录了当时的官话语音,他们记录的声母系统比早梅诗多了和,但这套语音系统对中古汉语的疑母的划分十分混乱,部分疑母字的确有声母脱落的现象,但同时有大量影母的字被派分了疑母的声母。《西字奇迹》是用拉丁字母为汉字注音写的几篇文章,《西儒耳目资》是用拉丁字母注汉字音的汇集,两者之间有传承关系,使用了统一体系的拉丁拼音方案。从这套拉丁拼音系统可以归纳出明朝的韵母系统。继塞音韵尾消失之后,鼻音韵尾-m至此也消失了,因此只剩下元音韵尾-i、-u和鼻音韵尾-n、-ŋ。同时发展出卷舌元音。《西字奇迹》等书用元音字母上的标号表示声调,一共有5种标号,分表代表阴平、阳平、上、去、入5调。早梅诗的音系中古汉语见、溪、群、晓、匣等母的字仍然保持舌根音、、,精、清、从、心、邪等母的字也仍然是平舌音、、。但到了清朝,这种局面改变了。清朝后期的《圆音正考》针对当时尖团混淆的现象,要求严格区别尖团音。其序言说:“试取三十六母字审之,隶见溪郡晓匣五母者属团,隶精清从心邪五母者属尖,判若泾渭。”可见原作、、的细音字,在这之前已经变为舌面音(团音)、、了。而且该书写作的当时,、、的细音也有混入舌面音的迹象。至此,现代普通话的声母格局就完全形成了。","text2":"哪些韵书反映了当时的实际语音?","label":1} {"text1":"欧阳德勋(,),香港歌手、司仪、前唱片骑师,于1984年第三届新秀歌唱大赛季军出身。欧阳德勋在1972年就读新法书院(小学及中学部)太子道分校。他在1984年参加第三届新秀歌唱大赛,并获得季军。同年加入歌坛成为歌手。他在1984年至1989年签约成为华星娱乐有限公司旗下合约歌手。同时于1984年起至1995年签约成为无线电视艺员,并拍摄多个电视剧,如《城市故事》。期间,欧阳德勋于1989年至1993年在商业电台第一台担任全职唱片骑师。后来到了1997年至2010年,欧阳德勋转到香港电台第二台担任音乐节目监制及全职唱片骑师。2012年6月25日,他迎娶同为歌手的蔡立儿,结束长达16年的爱情长跑。圈中的好友如蒋丽萍、宝珮如、李国祥、周慧敏、蔡一智、李司棋、车婉婉、陈奂仁、许志安、苏永康、张衞健、梁汉文、吴国敬、张茜、黎芷珊和梁雨恩等都有出席其婚礼。欧阳德勋表示曾经与当时仍然是女朋友的蔡立儿分开达一年多的时间。其后,因为他重整个人生活且有信仰支持,所以他决定与蔡立儿破镜重圆。","text2":"欧阳德勋在商业电台第一台做什么工作?","label":1} {"text1":"布龙齐诺 Bronzino()全名阿纽洛·迪·科西莫\"Agnolo di Cosimo\",也称尼奥洛·布龙齐诺。是意大利佛罗伦萨的风格主义画家。布龙齐诺这个昵称的来历已不可考,但是应该和他黝黑的肤色或者他常用的肖像画主题有关。现代医学有研究证明他的肤色可能是来自爱迪生氏病,因为肾上腺功能不足导致皮肤上的色素沉积。1503生于佛罗伦萨。根据他同时代的乔尔乔·瓦萨里,布龙齐诺是蓬托莫的学徒,并且他的画风受了他老师的很多影响。布龙齐诺也是一位出色的肖像画家,在大部分职业生涯里,充任美第奇家族的宫廷画家。1563年参与创建绘画学院。布龙齐诺出生于佛罗伦萨的一个屠户家。按照他同时代的乔尔乔·瓦萨里的说法,布龙齐诺最先从师于Raffaellino del Garbo, 14岁以后跟着蓬托莫学习绘画。蓬托莫曾把年幼的布龙齐诺画入他的一系列\"旧约圣经中的约瑟\"(Joseph (Hebrew Bible)|Joseph in Egypt) 肖像画作品中,这些作品现今珍藏在伦敦国家美术馆 。他的画风受了他老师的很多影响,对他老师的一生而言,两人一直相互合作。在佛罗伦萨阿诺河老桥边的Santa Felicita教堂的小礼堂里,我们时常可以看到布龙齐诺的早期手迹。礼堂内室、祭坛大作 Deposition from the Cross 和侧墙湿壁画\"Annunciation\"是由蓬托莫完成的。很显然,布龙齐诺被安排了创作穹顶湿壁画的任务,令人惋惜的是,这些作品并没有幸存下来。据乔尔乔·瓦萨里,其中四幅圆形福音绘画是由布龙齐诺完成的。然而由于他的技巧和他导师的是如此相似,至今学者们仍在为此争辩不止。在他生命的最后阶段,布龙齐诺投身于佛罗伦萨Accademia delle Arti del Disegno的诸多活动中,他是这家学院的创办人之一。画家Alessandro Allori 是他最喜爱的徒弟,公元1572年布龙齐诺过世年间,他是和Allori一家住在一起的(Alessandro是Cristofano Allori的父亲) 。布龙齐诺一生大部分时光是在佛罗伦萨度过的。","text2":"布龙齐诺 Bronzino是什么风格主义画家?","label":1} {"text1":"第八大道(Eighth Avenue)是美国纽约市曼哈顿西部的一条南北走向道路,单向北行,是曼哈顿最长的街道之一。它开始于西村和Bleecker街十字路口的Abingdon广场,衔接哈德孙街,向北经过44个街区,经过切尔西、Garment区、Hell's Kitchen的东端,中城和百老汇剧院区,最后在58街进入哥伦比亚圆环。再向北,这条路变成沿着中央公园的中央公园西,到110街以北,又变成Frederick Douglass Boulevard.在非正式场合,Frederick Douglass Boulevard 有时仍被称作第八大道。 Fredrick Douglass Boulevard 最后结束于哈莱姆河西159街附近的Harlem River Drive。虽然这条大道在曼哈顿不同地段更换了几个名称,但事实上却是一条连续的道路。自从1990年代起,第八大道穿过格林尼治村和切尔西的一段成了纽约市同性恋社区的中心,有迎合同性恋者的酒吧和餐馆。事实上,纽约市每年的骄傲游行就沿着格林尼治村的这一段第八大道举行。除了时报广场以外,第八大道从42街到50街的一段在1970年代、1980年代和1990年代初也是非正式的红灯区,此后被有争议地改建为适宜家庭居住的环境。","text2":"第八大道在110街以北叫什么名字?","label":1} {"text1":"《惊悚401号房》是MTV制播的电视节目,由杰瑞·帕达里基主持。每个影片中的演员,以各种不可思议的魔术手法,让旁人感到目瞪口呆,绝对不敢相信自己的眼睛。命名为「《惊悚401号房》」,主要是名魔术师哈利·胡迪尼于1926年,在底特律慈恩医院(Detroit's Grace Hospital)的401号病房去世。因此,节目就以“\"Room 401\"”为之。美国地区是从2007年7月17日起播送,直至同年8月21日为止。台湾地区则是从2008年8月5日起至同年的8月9日为止,于台湾时间之0时30分起,与《爆笑监狱兔》、《偶们最风流》(Fur TV)共同播出,此段时期,《惊悚401号房》以每集的第一部份做试播5分钟。于同年8月11日起,正式从第一集起播出完整节目,8月22日重播第一集,至8月29日播出第八集作为结束。于9月7日起每周日晚间十时,再重播节目。所有的表演,即使发生危险状况,也不会作马赛克处理,保证一刀未剪;同时节目呈现的画面,绝对没有幕后特效,绝对真实呈现。例如第二集第一部份的〈一分为二〉(Ice Carving)部分,即使电锯将另外一位同伴的身体一分为二,电视画面也完整播出,让一旁不知情的路人感到无比的恐惧与不可思议。而魔术表演的同时,影片内的主角绝对不会受到流血或残废的伤害,但这些主角看到如此不可思议又可怕的画面,大部分会身心受创或受到惊吓,甚至还会有哭泣的时候。例如第三集第三部份〈飞行恐惧〉,就让女主角惊吓到哭泣。下列列出第一季第一到第八集的大意。由于节目均属特技效果,节目播出之前,均会播出一段警语,请观众不可模仿,以免造成重大伤害。部分表演者是由Magic X团体所演出。","text2":"《惊悚401号房》是谁主持的?","label":1} {"text1":"《战神》(')是由索尼电脑娱乐发行的动作游戏,由索尼电脑娱乐旗下公司圣塔莫尼卡工作室开发。游戏是根据希腊神话改编。按情节来看是《战神》系列第三作。2007年,被游戏网站IGN评价为PlayStation 2最优秀游戏第4名。SCEA于2009年11月17日推出重制《战神》与《战神II》战神合辑版,并针对PS3硬体加以强化移植,解析度提升为720p,也支援PS3奖杯系统。《战神III》于2010年3月推出,及后于2015年7月以高清重制方式在Playstation 4平台上推出。最新作《战神》计划于2018年年初推出,故事将由希腊神话转至北欧神话。游戏主角克雷多斯原本是一名斯巴达将军。克雷多斯在一次与野蛮人巴比伦王展开战争之时不敌,在接近被杀而命悬一线时,克雷多斯以自身灵魂作为代价,成功呼召了初代战神阿瑞斯降临人世,阿瑞斯赐予克雷多斯由冥府火焰所打造而成的混沌剑,得到混沌剑的克雷多斯轻易击杀巴比伦王。自此克雷多斯成为阿瑞斯在人间的忠实奴仆及追随者同时也是人间的一名完美战士,在之后的战争中战无不胜。好景不常,阿瑞斯为了让克雷多斯放弃人间的所有拖累,精心安排地设局令克雷多斯杀了自己的妻女。自此克雷多斯受到诅咒,他妻女的骨灰嵌入了克雷多斯的身体并每晚受到恶梦的折磨,自此克雷多斯化身为斯巴达的鬼魂并展开了他向阿瑞斯的复仇之旅。","text2":"《战神》这个游戏是根据什么神话改编?","label":1} {"text1":"上田车站()是一位于日本长野县上田市天神1丁目,由东日本旅客铁道(JR东日本)、信浓铁道()与上田电铁所共用的铁路车站。上田车站是JR东日本所经营的北陆新干线与两条地方铁路线信浓铁道线()与上田电铁别所线之交会车站。配合全面高架化的新干线路线,JR东日本所属的月台区是位于车站二楼(但检票口位于一楼),至于信浓铁道与上田电铁的部分,检票口皆位于二楼的桥上站屋内,但月台与铁路线则分别设置于二楼(信浓铁道线)与一楼(别所线)处。对向式月台2面2线的高架车站。。是北陆新干线轻井泽至长野段唯一的高架车站。由于不设待避线,因此设有月台闸门。起初计划为岛式2面4线,但根据通车后的需求预测轻井泽与长野之间不需要待避设备,成为现时对向式月台2面2线。对向式月台(下行本线)与岛式月台(中线与上行本线)2面3线的地面车站。侧式月台1面1线的高架车站。1998年3月29日由地面车站改建为高架车站。","text2":"上田车站在日本的哪个地方?","label":1} {"text1":"孙达志(Andreas Schutz、),旧译薛达志,人称「光头志」、「老孙」,是德国练马师及前业余骑师。由2014年4月10日起,他向香港赛马会要求,将其中文名字改为「孙达志」。孙达志的父亲孙德思(旧译薛德思)是练马师,他是德国业余骑师,曾经成为业余骑师冠军。后来与父亲合作成为练马师。1998年正式从父亲接手马房,立即取得好成绩。在德国曾经赢得五次德国打吡、德国橡树大赛。主要马匹包括西单路、思乐骥、易博来等等,其余比赛亦包括巴登大赛、新加坡航空国际杯、罗马大赛等等。2006\/07年马季,孙达志获香港赛马会发练马师牌照,于2006年9月24日凭「时代明星」取得在港从练首场头马,亦凭好爸爸胜出主席锦标,该驹其后在翌季赢得香港一哩锦标,在港赢得第一个一级赛,并成为2007-2008年度香港马王。2011年6月22日,孙达志凭「好球」取得在港从练的第一百场头马。2011\/2012年度马季,孙达志单季取得28场头马,为在港个人从练生涯最高纪录。孙达志从2006\/07赛季起在港练马,至2015\/16年马季结束,累积所赢头马共181场。2009\/10、2014\/15及2015\/16累积三个马季头马低于下限,而不获马会续牌。随后,孙达志在法国重新开始练马师生涯。","text2":"孙达志凭借取得在什么港从练的第一百场头马?","label":1} {"text1":"方顺吉(),台湾高雄市凤山区人,台湾男歌手、男演员、主持人。方顺吉是小童星起家,中华艺术学校毕业。在校期间留级,霸凌同学。1992年参加台视五灯奖(与当时的冠军打成平手)。1993年参加三立《二十一世纪新人歌唱排行榜》儿童组歌唱比赛,闯关成功。1994年10月举辨「方强强滚反毒演唱会」,创国内年纪最小开三万人演唱会之歌手。1994年6月15日出版第一张个人专辑《翘脚髯嘴须》(与方婉真(方宥心)、萧玉芬两人),创国内年纪最小专辑破一百万张之歌手。1994年9月24日出版第二张个专辑《强强滚》,专辑销售破50万张。1994年11月14日与同比赛出身的童星歌手钟旎菱、邬兆邦、萧玉芬、张俊豪、慈袖静等人出合辑《美梦成真》,销售破25万张。其后又在方婉真(方宥心)《彩虹》专辑中与其合唱《彩虹》一曲。1995年1月26日出版第三张个人比赛歌曲专辑,销售破30万张。1995年6月29日出版第四张人专辑《彩色精灵》专辑,销售破30万张。1996年1月31日参与《让爱无所不在》合辑,其中收录强强滚,翘脚髯嘴须。2000年大旗唱片出版《大旗2000超级音乐快递》合辑,收录了《让爱无所不在》与大人变声版的《鼓励》两首歌曲。2002年12月参与中视闽南语连续剧《漂浪之女》记者招待会,于剧中饰演男主角「蔡顺吉」。2003年2月22日服食摇头丸被查缉,勒戒24天。2003年5月20日在高雄县燕巢乡看守所进行勒戒,6月12日完成勒戒。2003年8月19日贴钱发专辑《鼓励》,举行发片记者会,并发誓不再碰毒。2003年9月6日接拍大爱戏剧《后山姐妹》。2004年2月出版第6张个人专辑《异乡情梦》。2004年7月演出民视《蓝色水玲珑》借种大丈夫。2004年8月演出民视《亲戚不计较》。2004年8月演出台视《兄弟姊妹》。2006年卷入华视《快乐星期天》评审包小柏被殴事件,带嫌犯进摄影棚,被疑教唆打人,同年携大麻被捕,因未吸食以5万元交保。2012年1月加入天帝海国际股份有限公司顺瑛堂生技电视主持人。","text2":"2004年,方顺吉有哪些活动?","label":1} {"text1":"Photoshop移动版(Photoshop Mobile)是Adobe公司正在展开的一项移动服务,不过它并未提供图片编辑功能。它暂时只提供英文版本。Photoshop Mobile提供了一种图片存储和浏览的服务,令手机可以直接与 Photoshop.com 紧密连结,无缝地随身使用网上图片分享及展示服务,同时用家可即时将手机所拍摄的相片上载更新相片集。另外,它可以为每个Photoshop注册的用户提供2GB的在线存储空间,而高级的会员(需收费)更可以得到20GB的存储空间。暂时只限于使用Windows Mobile的智能手机被支援,包括摩托罗拉Music 9M,摩托罗拉9H,Palm 750w,Plam 700w,三星Blackjack1 和三星Blackjack2等。随著时间的推移,将会逐渐支持更多的手机。根据官网的消息,以下手机将会被支援,包括苹果公司的iPhone,黑莓的珍珠,摩托罗拉的RAZR2,诺基亚 5310 及 诺基亚的6310等。","text2":"Photoshop Mobile提供了一种图片存储和浏览的服务有什么优点?","label":1} {"text1":"凉宫春日的忧郁 角色CD是从2006年7月5日到2007年2月21日由Lantis发行的一连串单曲。在2009年再发行凉宫春日的忧郁 新角色CD,从2009年9月30日到2009年12月9日由Lantis发行。2006年7月5日发行。由平野绫主唱,凉宫春日(平野绫)版本的晴天愉快。2006年7月5日发行。由茅原实里主唱,长门有希(茅原实里)版本的晴天愉快。2006年7月5日发行。由后藤邑子主唱,朝比奈实玖瑠(后藤邑子)版本的晴天愉快。2006年12月6日发行。由松冈由贵主唱,鹤屋学姊(松冈由贵)版本的晴天愉快。2006年12月6日发行。由桑谷夏子主唱,朝仓凉子(桑谷夏子)版本的晴天愉快。2007年1月24日发行。由青木沙耶香主唱,虚妹(青木沙耶香)版本的晴天愉快。2007年1月24日发行。由白鸟由里主唱,喜绿江美里(白鸟由里)版本的晴天愉快。2007年2月21日发行。由小野大辅主唱,古泉一树(小野大辅)版本的晴天愉快。2007年2月21日发行。由杉田智和主唱,阿虚(杉田智和)版本的晴天愉快。2009年9月30日发行。由平野绫主唱。2009年9月30日发行。由茅原实里主唱。2009年9月30日发行。由后藤邑子主唱。2009年11月18日发行。由小野大辅主唱。2009年12月9日发行。由杉田智和主唱。2009年12月9日发行。由松冈由贵主唱。2009年12月9日发行。由白石稔主唱。","text2":"凉宫春日的忧郁 角色CD具体是指什么?","label":1} {"text1":"澳门巴士1路线是一条由澳门新时代公共汽车股份有限公司经营,往返妈阁和巴波沙大马路的巴士路线。本线在福利时代,主要采用从英国购入的二手Bristol L5G及Bristol LS5G行驶。后来因为本线客量较低,所以开始使用三菱Rosa,不过可能出现其他中类型的车特见(如平治O814(B车)等等),后来因为1A缩短总站和不经提督马路至妈阁一段路,和延长总站后,客量开始上升,用车亦开始使用苏州金龙10米\/12米(K06-K125)行走,间中有猛狮13.230(M车)在2011年7月31日前,本线主要使用车款如下:在2011年7月31日后,本线主要使用车款如下:2017年9月18日后:本线在开办至2008年间由于有1A、34竞争,由于1A、34跟本路线走线相同,加上班次频密,而且本线没有特别多\"独市位\",令本线客量偏低,用车更是载客量低的三菱Rosa。后来本线延长至至关闸总站,1A路线取消提督马路至妈阁一段路线,令该段路线成为本线\"独市位\",客量开始上升,本线亦不再派出三菱Rosa,改派更多载客量的车行走,除了处理下环区的挤密人流外,有时还要帮助清理在十六浦至提督马路上不到3路线的客量,经常都会有客满情况出现,所以被称为\"「线王」\"。虽然关闸总站附近有很多路线(如5、10、18等等)选择去妈阁总站,但在以下情况下本路线较有优势。由于该线经过的站数少(比其他线更方便),而且近年来(该线途经之处)有大量市民北上或回半岛南区(司打口至河边新街),加上它的最大竞争对手─5路线晚上在水坑尾经常塞车,基本上第二个站乘客已经因客满上不了车,令乘客流量不断增加","text2":"为何后来客量又会上升了?","label":1} {"text1":"陶街,是中国广州市越秀区一条著名的街道。位于解放中路以西,朝天路以东,六榕路侧,因街内及附近的电器市场及二手物品市场而在珠三角地区驰名,被誉为广州“男人的天堂”。陶街之名,是为了纪念明代的陶成、陶鲁父子。因陶成、陶鲁父子有战绩,明朝嘉靖四十年,于忠贤坊后街(今陶街)建“忠烈祠”以纪念。自20世纪90年代起,陶街逐渐成为了周末的新旧电器交易的跳蚤市场。后来规模扩至附近街道,如粤华街、解放中路、将军东路、六榕路等地,而贩卖的货品亦不限于新旧电器及配件,军事藏品、黑胶唱片、音响设备、通讯设备等也吸引不少爱好者。除了在陶街之外,不少商铺开入了后来开张的六榕路陶街电器城、将军东电器城和解放中路的新陶街电器城。因此,当谈及电器及二手市场时,广义的“陶街”还包括以上的地点。","text2":"陶街是从何时开始变成了跳骚市场?","label":1} {"text1":"神秘的流星(法语:L'Étoile mystérieuse;英语:The Shooting Star)是丁丁历险记的第十部作品。作者是比利时漫画家埃尔热。本作于1941年开始以黑白两色在报纸上连载。1942年以彩色形式出版。这也是丁丁历险记中第一部直接以彩色形式出版画册的作品。故事的主线是丁丁与他的小狗白雪前往北冰洋对一颗流星进行探险的故事。流星擦过地球,陨石坠落北冰洋!科学家发现陨石蕴含未知的新金属,这个惊天大发现,吸引了欧洲顶尖的科学家联同丁丁和哈达克船长组成考察队,乘坐\"曙光号\"到北冰洋寻找陨石,取出样本做研究。当消息传到以利字当头的商人耳中,商人一心借此发大财,出动\"培里号\"与\"曙光号\"在北冰洋争分夺秒,誓要成为第一个发现陨石的人。陨石最终花落谁家?","text2":"《神秘的流星》的作者是谁?","label":1} {"text1":"布里亚特人( \"Buriad\",俄罗斯布里亚特语:,)为蒙古族的一支,分布在鄂温克自治旗、西伯利亚贝加尔湖以东、以北、色楞格河一带以及东方盟,人口436000人,是东北亚较大的少数民族,有自己的自治共和国,首都是乌兰乌德。他们是分布最北的蒙古人。布里亚特人分前后贝加尔共有十个部落,后贝加尔包括色楞格人、豁里人、库达拉人、巴尔古津人。前贝加尔包括阿拉尔人、巴拉干人、韦尔霍连斯克人、通卡人。布里亚特人最早出现在蒙古秘史,是被术赤降服的林中百姓。在元代,蒙古人把他们纳入草原封建帝国的政治系统内。归岭北行省管理,元代的大可汗把黄金家族中的小汗派到那里行政,并传授先进的草原封建社会或中原的生产技术,使森林中部落人口不减反增,成为一个新的蒙古人口殖民区成为蒙古共同文化圈的一部分,已经具有行政单位的性质,北元时代,他们加入了四卫拉特(后来离开)。清代在恰克图条约归俄罗斯,是西伯利亚较俄化的部落。后来成为泛蒙古主义的支持者之一。中国大陆的布里亚特人分布在内蒙古自治区,人数不多。在西藏有一位布里亚特喇嘛阿格旺多杰是达赖手下(俄罗斯派去西藏探险礼佛的也是他们)。他们在十七世纪信仰藏传佛教,首领是班智达堪布由沙皇任命。1846年有庙34,喇嘛4594人,信徒122529人。他们所信仰的藏传佛教是噶尔丹入侵喀尔喀时喀尔喀人带来。1853年尼古拉一世批准穆拉维约夫起草的东西伯利亚喇嘛教则例作为规则管理他们。 布利亚特民族与蒙古族有许多相同的特征和习俗,但两族的历史和语言各异,从而布利亚特有资格成为一个独特的民族群体。居住在中国的布利亚特族人是在1917年从俄罗斯移民过来的。 与蒙古族其它的少数民族团体不同,布利亚特族并没有受到喇嘛教太大的影响。大多数的布利亚特族人崇拜自然和神灵。在布利亚特族人的村子里有许多巫师。他们是新蒙古人种代表形态之一。","text2":"为什么布利亚特族并没有受到喇嘛教太大的影响?","label":1} {"text1":"张涵予(),中国大陆演员,北京人,华谊兄弟旗下艺人,2007年主演冯小刚电影《集结号》而成名,之后主演多部卖座电影包括《风声》、《智取威虎山3D》、《湄公河行动》,成功塑造大萤幕的硬汉形象。1988年毕业于中央戏剧学院表演系。张涵予以配音出道,期间帮《鲨鱼黑帮》 、《魔戒》、《特洛伊》 等电影配过音,亦参与电视剧《贻笑大方》、《贫嘴张大民的幸福生活》、《梦开始的地方》、《与青春有关的日子》等,及一连串冯小刚电影《没完没了》 、《手机》 、《大腕》 、《天下无贼》的演出,号称“冯小刚御用龙套”,得到冯小刚赏识,在其2007贺岁档《集结号》担任主角,这部电影在无大牌演员的情况下创下中国华语电影2.6亿人民币的票房纪录,张涵予也凭借此片迅速窜红,成功跻身于中国一线影星行列,并在2008年以该片获得第29届大众电影百花奖、第45届金马奖最佳男主角奖,被称为大器晚成的百花影帝。2009年,亦以同片获得第13届华表奖优秀男演员奖。2014年他在徐克执导的《智取威虎山3D》中饰演其多年来梦寐以求想演的侦查英雄杨子荣,成为其又一部代表作品,并获得第30届金鸡奖最佳男主角。他在林超贤执导的2016年警匪动作片《湄公河行动》饰演英勇的缉毒队长高刚,取得好口碑与11.8亿元的高票房。2016年他在吴宇森执导的、翻拍自1976年日本电影《追捕》的电影中担纲主演,预计将于2018年春天上映。","text2":"张涵予凭借什么电影获得了30届金鸡奖最佳男主角?","label":1} {"text1":"底特律第一公理会教堂(First Congregational Church)位于美国密歇根州底特律森林东路33号,伍德沃德大道(Woodward Avenue)路口。底特律第一公理会成立于1844年12月25日。曾经在底特律河附近建造过2座教堂,第三座教堂于1891年兴建于今址,由建筑师约翰法克森(John Lyman Faxon)设计。增建部分称为天使之翼(Angel's Wing),1921年由建筑师阿尔贝特卡恩(Albert Kahn)设计。教堂为理查森风格,使用粗凿暖红色石灰岩。正立面有五湾凉廊,和一道前山墙胸墙。其上方是带窗格的圆形窗,侧面是圆形拱。这座教堂还拥有一个120英尺的钟楼,顶部是8英尺高的天使乌利尔铜像这座教堂参照了威尼斯和拉文纳的教堂。圣所有木雕、天花板肖像、玫瑰窗和华丽的色彩。教会提供教堂历史和建筑方面的展览。游客可以自助游览历史性设施和建筑物。 教会还举办地下铁道生活博物馆,模拟过去的地下铁路的故事。","text2":"教会还举办什么活动?","label":1} {"text1":"光华体育会(Kwong Wah Athletic Association,简称光华),成立于1936年,是香港一支老牌足球队,曾在香港甲组足球联赛角逐,现时于香港乙组足球联赛作赛。光华是香港一间老牌足球会,于1936年,由中华电力公司文职人员组成,当中有华人及外籍人士,包括后来成为球星的侯榕生。光华成立后,随即于1936–37年加入香港丙组足球联赛,当年球队经费由中电负责。翌年,获升上乙组联赛角逐,并赢得初级银牌冠军。成立第三年,光华便首次升上香港甲组足球联赛角逐。1950年代初,光华获得许文奎等上海来港商人支持,以一班原上海青白足球队球员为主力,当时效力过光华的上海名将包括有:张金海、吴祺祥、陈明哲、罗寿福、严士鑫等。光华成绩一直只是平平,在1962–63年赢得香港足球高级银牌赛冠军,是该会在甲组唯一的重要锦标。1965–66年,光华在甲组联赛12支球队中位列包尾,宣告护级失败,降落乙组作赛。1972–73年,光华获得乙组联赛亚军,得以重返甲组作赛,并且邀得华探长颜雄入阁,招兵买马增强实力,包括著名守门员何容兴。1976–77年,光华在甲组联赛名列尾二,与另一支老牌球会九巴一同降班,自此未能再重返甲组。1977–78年,光华在香港乙组足球联赛12支球队名列第 11,连续两年都要降班,宣告降落丙组作赛。1990年夏天,因乙组联赛扩充队数,光华获得升上乙组,惟于1990–91年,光华在乙组 10 支球队中名列第 9,与第 10 名的流浪一同降回丙组作赛。2008–09年球季,光华虽然邀得前香港足球代表队前锋巴贝利助阵,但在香港丙组(甲)联赛 19 战 5 胜 1 和 13 负得 16 分,位列第 18 名。2009–10年球季,光华以 19 战 2 胜 6 和 11 负成绩,在香港丙组(甲)联赛名列第 18 位。在2010–11年球季,光华邀得前甲组球员吴圳聪加盟,加强球队后防。香港足球总会于2012–13年球季,重组丙组联赛,并复办丁组联赛。光华在上一届香港丙组(甲)联赛名列第18名,故需降落丁组联赛角逐,光华邀得前香港足球代表队前锋罗伦士及前甲组球员凌汉宗加盟,增强实力争取回升丙组,开季取得六连胜佳绩,至2012年10月14日第七轮联赛才与西贡朋友赛和 2–2,本季首度失分。","text2":"光华成立后,参加了什么比赛?","label":1} {"text1":"伍蕃(1890年-1959年),字少裴,中华民国将领,善书法。顺德人。广东黄埔陆军小学堂第二期,南京陆军第四中学,(1914年)保定陆军军官学校第二期骑兵科毕业,历任国民革命军第七军第五旅参谋长。1927年底,任第十五军第三师(后改称第五十二师)参谋长。广西省保安处长。1929年11月任广西省政府委员,广西省保安司令部中将司令。1932年任第四集团军总司令部高级参议,第五师中将师长。1936年(民25年)二月授陆军少将。抗日战争时,任第四战区军官训练团副教育长,第七战区挺进第三纵队司令。 1946年2月退役。1936年秋,广东还政中央,余汉谋任第四路军总司令。缪培南将军为参谋长,「七七」抗战后缪兼广州警备司令。「七七」全面抗日战争爆发,两广成立第四战区,以张向公为司令长官,余汉谋为副长官,战区以第十二集团军余汉谋部守北江,第九集团吴奇伟部守东江,第三十五集团邓龙光部守西江,南区以第十六集团军夏威部负责。其后战局转移至桂南,四战区部移桂,广东方面乃成立第七战区,以余汉谋为司令长官。广东沦陷后,临时省会在曲江,伍蕃时任广东绥靖公署军务处长而广州市公安局侦缉队长为陈植云,一九四一年,在曲江顺德韶旅同乡会内成立“顺德青云儿童教养院,周之贞任院长,廖平子任副院,伍蕃任校董,先后抢救沦陷区的难童八百余人。第七战区成立挺进第三纵队(简称挺三),运用游击战在敌后活动,配合国军作战。活动地区包括中山、顺德、鹤山、新会等地。挺三直属于第七战区司令长官部,配属于六十四军。挺三的司令部下设参谋室、副官室、军需室、无线电台、有线电通讯队,中山指挥所、顺德指挥所。战时,在贵州省中部的息烽设有国民党部及军官训练学校并有集中营。","text2":"伍蕃是从哪些学校毕业的?","label":1} {"text1":"安森美半导体(ON Semiconductor,)是一家财富美国1000强半导体供应商。公司的产品系列包括电源和信号管理、逻辑、分立及定制器件,帮助客户解决他们在汽车、通信、计算机、消费电子、工业、LED照明、医疗、军事\/航空及电源应用的独特设计挑战,既快速又符合高性价比。公司在北美、欧洲和亚太地区之关键市场运营包括制造厂、销售办事处及设计中心在内的世界一流、增值型供应链和网络。总部位于美国亚利桑那州的凤凰城,2016年销售收入达39.07亿美元,位列在全球20大半导体公司之一。安森美半导体前身是摩托罗拉集团的半导体元件部门,于1999年独立上市,继续生产摩托罗拉的分立晶体管,标准模拟和标准逻辑等器件。安森美半导体制造以下的各种产品:安森美半导体的各个产品部门:","text2":"安森美的产品主要有哪些?","label":1} {"text1":"乌马托鳞片属(\"Umaltolepis\")是一类已灭绝的银杏类植物,属于乌马托鳞片科(Umaltolepidiaceae)。生存于中侏罗世到早白垩世。乌马托鳞片属的雌性繁殖器官包括一个苞片和短柄。短柄基部具有鳞片叶。苞片顶生,可能包含两个裂片,胚珠可能生在其腋部。乌马托鳞片属营养叶的叶柄不明显,叶形呈线性、长舌形,部分呈假托勒利叶型。叶片中曾发现有树脂。乌马托鳞片属生存于中侏罗世到早白垩世;在西伯利亚的晚侏罗世和早白垩世地层,中国东北和华北的早白垩世地层,以及中国河南的中侏罗世地层中均有发现。乌马托鳞片属(\"Umaltolepis\")的模式种是发现于俄罗斯西伯利亚布列亚河(Bureya)盆地上侏罗统的瓦赫拉梅耶夫乌马托鳞片(\"Umaltolepis vachrameevii\" Krassilov,1972)。","text2":"乌马托鳞片属是一类怎样的植物?","label":1} {"text1":"天弁二(δ Sct\/盾牌座δ)是一个在盾牌座的巨星。视星等4.72,是这个小且模糊不清的星座的第五亮星。根据依巴谷卫星的视差测量,该星距离地球大约202光年。天弁二是盾牌座δ型变星的原型。盾牌座 δ是一颗黄白色的F-型巨星,以4.65小时的周期规律的胀缩,造成光度在+4.60至+4.79之间变化。该星化学元素的丰度异常类似于Am星。在1900年,利克天文台的天文学家发现该星的视向速度会变化。这个0.19377天的周期和0.2等的光度变化在1935年被提出,认为该星的变化是本质的,而不是光谱联星造成的。1938年发现了第二个周期,并且对该变化提出了胀缩理论。之后的观测显示它以多个径向和非径向的模式变化。它以离心率0.11的轨道绕行银河系,距离银心最近22,310光年,最远27,590光年。如果盾牌座δ保持目前的动向和亮度,它将会在10光年内的距离掠过太阳系,在公元1,150,000年到1,330,000成为全天最亮的恒星。它将达到-1.84等,比目前天狼星的-1.46等更亮。该星有2个光学伴星。第一个视星等12.2,与盾牌座δ相距15.2弧秒。第二个视星等9.2,距离53弧秒。佛兰斯蒂德把盾牌座的很多恒星归为天鹰座。盾牌座δ被编号为天鹰座2。拜耳命名法的δ是由古德命名的,并不是拜耳。","text2":"1938年发现了第二个周期,并且对该变化提出了什么?","label":1} {"text1":"武汉公交1路是中国湖北省武汉市的第一条公交线路,运营区间为双墩站至芦沟桥路(南)站。由武汉公交集团公司第三营运公司经营。1路于1929年2月10日使用“雪弗兰”卡车改造而成的客车开线,从汉口六合路开往硚口,成为当时华中地区第一趟公共汽车。为了方便车辆运行,当时的汉口市政府曾发行债券改造道路。1路汽车先后于1931年、1943年、1946年因为水灾和亏损等原因停止行驶,其后又数度开线。1949年7月20日,1路重新开线,由四官殿开往车站路。后来数次调整线路,至2005年11月改为双墩至山海关路至今。共14站。双墩→新合村→太平洋→仁寿路→宝丰路→同济医院体育馆→中山公园→六渡桥→江汉路→北京路→车站路→三阳路→六合路→抚顺路→芦沟桥路(南)共12站。芦沟桥路(南)→五福路→三阳路→大智路→江汉路→友谊路→中山公园→同济医院体育馆→宝丰路→仁寿路→太平洋→新合村→双墩","text2":"武汉公交1路的运营区间是什么?","label":1} {"text1":"Google数据中心()是Google用来提供服务的计算机软件和大型硬件资源等技术基础设施。美国有9间;欧洲有3间;亚洲有2间;南美洲有1间。Google构建称为反向索引的数据结构来索引文档,通过查询词获得文档的列表,索引由硬盘驱动器提供,通过增加每个分片的副本数量和服务器数量以处理不断增加的查询量,有足够的服务器来保存整个索引的副本在主存储器。2010年6月,Google推出「Caffeine」的索引和服务系统,可以持续抓取和更新搜索索引,使用MapReduce作业批量更新了其搜索索引,期后更推出「Percolator」的分布式数据处理系统。Google网页伺服器是Google基于Apache HTTP Server开发的Web服务器软件。亚洲美国南美洲欧洲","text2":"Google数据中心有什么用处?","label":1} {"text1":"《碧血剑》()是香港电视广播有限公司与中国中央电视台旗下的中国国际电视总公司合作摄制的古装武侠电视剧,由林家栋、江华、佘诗曼及吴美珩领衔主演,以及由李添胜担任监制。此剧根据金庸所著的武侠小说《碧血剑》为基础,再把小说改编剧集,然后进行拍摄制作。此剧由无线电视继1985年版《碧血剑》之后,无线电视为拍摄制作第二部同名剧集。此剧在无线电视翡翠台首播前的电视预告片及宣传片中,为无线电视所选播的33周年台庆剧。电视广播(国际)有限公司授权台湾弘音多媒体科技股份有限公司于2008年推出发行了《碧血剑》DVD影碟零售版本,此影碟收录全套35集,共7只碟,设有粤语及国语发音版本并配上非隐藏繁体中文字幕。","text2":"《碧血剑》是由哪两家公司合作拍摄的?","label":1} {"text1":"宪法广场()是希腊首都雅典的主要广场,位于希腊国会之前,得名于1843年9月3日奥托国王在起义后批准的希腊宪法。宪法广场北到Vassileos Georgiou A'街,南面是Othonos 街,西到Filellinon 街,东到阿马利亚大街。广场的东部比西部高,有一些大理石台阶通到阿马利亚大街。其下方是地铁车站。附近多户外咖啡馆,是受欢迎的市中心聚会地点。广场的南北有两片绿地,种植有遮荫树木,中心是一个大型喷泉,与雅典人一同度过炎夏。广场经常举行政治游行。东侧的阿马利亚大街对面,就是议会大厦及其周围的国家公园。每个小时在广场与议会之间的无名烈士墓前举行总统卫兵换岗仪式。在星期日和官方假日,大部分卫兵都参加换岗仪式,并有军乐队伴奏。宪法广场还是雅典多种公共交通形式的枢纽,地铁在此设站,公共汽车和电车可通达城市各处。宪法广场与机场之间开行机场专线和地铁线泥路。雅典市在广场提供高速(4 Mbit\/s)无线上网。宪法广场附近有许多名胜古迹,例如雅典卫城、酒神剧场、Areopagus、雅典古市场与哈德良图书馆、罗马市场的风塔、Choragic Monument of Lysicrates、哈德良拱门、奥林匹亚宙斯神庙、Pnyx、菲洛帕波斯纪念碑 on Nymphs山、Kerameikos公墓、无名烈士墓和吕卡维多斯山。古老的教堂也散布在该区,有些可追溯到中世纪。","text2":"宪法广场得名于哪里?","label":1} {"text1":"菲腊牙科医院()是香港一家牙科医院,根据香港法例第1081章《菲腊牙科医院条例》成立,并以英女王伊利沙伯二世夫婿爱丁堡公爵菲腊亲王命名。牙科医院位于香港岛西营盘医院道与东边街交界,毗邻香港佐治五世纪念公园。香港政府于1979年4月批出9700万港元的建造合约动工兴建11层高的医院大楼,整个牙科医院项目涉资2亿港元。1981年3月24日,爱丁堡公爵菲腊亲王出席牙科医院的开幕礼,并主持揭幕仪式,而牙科医院也以他的称号名为「菲腊牙科医院」。菲腊牙科医院的运作经费由香港政府提供,并作为香港大学牙医学院牙医学生的校舍和训练中心。牙科医院并不提供公共牙科诊疗服务,但会通过转介及遴选接收病人,供牙科学生进行训练及实习,以及进行牙科医疗相关研究。","text2":"整个牙科医院项目投入了多少资金?","label":1} {"text1":"舵(英语:Rudder),为航行设备上用于改变或保持航行方向的一种装置。最早用于船舶,是控制船舶航向的设备。通俗的来说,舵就是“和轮船外壳相连接,控制其运动方向的装置”,各类船桨也具有相类似的功能。古代的船只往往靠人力来推动,有些把船桨用双耳圆环固定在船舷的两侧,有些则安装在船尾,其中一部分船桨甚至设计为两者皆可通用。在地中海地区的文献中,两侧的船桨往往被称为“四分之一船桨”,其得名于此类船桨安装的位置在船身后方的四分之一处。而位于船尾的桨与现代轮船的转向原理相类似,一般为船尾中心线的位置。其结构设计非常多样化,古代欧洲往往通过铁制的螺栓和铰链将其和船身连接,阿拉伯人的设计是在船尾安装一个圆环,把船舵插在其中。中国古代的舵和阿拉伯比较类似,但是是通过滑轮驱动的。古埃及人在西元前3100年美尼斯时期就已发明。 但那比较像控制方向的船尾船桨而非现代的船舵,不需用人力一直控制的转向舵,最早出现在西元一世纪的中国,至于欧洲可能到12世纪才有这类船舵。1955年在广州近郊出土的东汉陶制船模,船尾就设有一支舵,这是中国最早的舵。从出土的汉朝文物和文献可以说明,舵在汉朝已经有。1978年在天津市静海县宋朝古船上发现了平衡舵。1999年在安徽省淮北市唐朝木船上发现拖舵。","text2":"哪一年在安徽淮北市发现了拖舵?","label":1} {"text1":"雨果大街(Avenue Victor-Hugo)是法国巴黎十六区的一条街道,开始于戴高乐广场,终止于亨利马丁大街。这是通往凯旋门的12条大街之一,长度仅次于最长的香榭丽舍大街。其位置介于福煦大街和克勒贝尔大街之间。雨果大街穿过非常别致的十六区的北部,长约2千米,是巴黎最有声望的街道之一。其确切的长度是1825米,宽36米(第一段,戴高乐广场与雨果广场之间,从雨果广场到亨利马丁大街稍窄)。这条林荫大道,在亨利马丁大街路口装饰着维克多·雨果雕像。它跨越了雨果广场。这条大街曾经称为Eylau大街和圣克卢大街(avenue de Saint-Cloud)。1881年2月28日,在法国诗人维克多·雨果79岁诞辰的第三天,这条大街命名为雨果大街。大街中段有地铁2号线的维克多·雨果站。雨果大街124号:维克多·雨果晚年的住处,1907年加高。","text2":"雨果大街是什么时候被如此命名的?","label":1} {"text1":"鳢鱼(学名:)又称鲖鱼、斑鳢、南鳢、鳢、台湾鳢鱼,俗称雷鱼、乌鱼或南方蛇头鱼。鳢鱼在鳢科中算是较大型的种类,一般成年鳢鱼体长可达10至40公分之间,而最大纪录也有60公分,与泰国鳢()体型差不多大。鳢鱼的身体修长,呈圆筒型或棒型,头顶平,身体后方侧扁,口部大,并较其他鳢科鱼类来地短而钝,下颚稍微突出,瞳孔稍小,尾鳍以及胸鳍呈圆形,背鳍基部长,侧边腹鳍短小,身体大部分区域覆盖著中小型鳞片,近又头部部份覆盖著大型鳞片。鳢鱼的体色通常为暗灰色或者为灰黑色,侧边无侧线,但有2纵列黑色斑纹,两边约各有10至12个,腹部则呈现灰黑色,各鱼鳍颜色均较为淡。鳢鱼为次级淡水鱼,喜爱栖息于河川中下游流域、水库、湖泊、池塘、沼泽、沟渠等水流和缓与水生植物繁多的浅水区,并喜在水底游动,躲在树枝底下或水草丛中。鳢鱼可以爬上岸,能在缺氧或淤泥中生活,但无法长期脱离水中,且在陆上不太灵活,可能是鳢鱼的身体过于粗壮及脊椎数过少导致的。鳢鱼分布不广,原产于中国、日本、菲律宾等亚洲国家。原本被引入美国当做宠物鱼或观赏鱼的鳢鱼,现今被一些饲主非法弃养,流入美国东部沿岸河流、密西根湖及西海岸加州的银木湖(Silverwood Lake)等地。鳢鱼的性情凶猛,掠食性极为强,对食物并不挑剔,主要食物为小鱼、两栖类以及一些甲壳纲动物(如:螃蟹、虾类等)。鳢鱼被人们视为一种十分高级的食用鱼,在大部分餐厅都能见到鳢鱼。此外,鳢鱼汤也是术后进补的好食材。","text2":"成年鳢鱼体长最大纪录有多少公分?","label":1} {"text1":"农劲荪,名竹,字劲荪(亦有一说认为「农劲荪」可能是假名),原籍湖北。曾赴日本留学,并结识孙文。他在日本时加入了同盟会,并在回到中国后,为激起国人忧国之心而努力。离开日本后,他回到天津经营怀庆药栈(一说「怀兴药行」),博学好友的他在此时与霍元甲结识,并请霍进他的药行帮忙。农劲荪是位真实存在的历史人物,身为传奇人物霍元甲的挚友,此号人物也总是出现在以霍元甲为中心人物的后代各改编作品中。就如同霍元甲一般,农劲荪的故事也有许多不同的版本。农劲荪爱国情操的产生,一说是因为自幼师从太平天国志士学习,一说则是因为在日本受到孙文的感动。不管原因如何,霍击退外国武师,农大加宣传并提供资金。二人的事迹之中,其中之一是击败俄国大力士奥皮音(事实上对方听到是霍元甲就先认输了),而最重要的成就,便是创立中国精武体操会(上海精武体育总会前身)。","text2":"农劲荪与霍元甲最重要的成就是什么?","label":1} {"text1":"威廉·亨利·“斯马什”·帕克(William Henry \"Smush\" Parker,),绰号“死神”,美国职业篮球运动员,身高1.93米,场上位置组织后卫。帕克出身于洛克公园,是著名的街头篮球明星,2002年弃学成为职业球员,但在当年的NBA选秀中落选。克里夫兰骑士队为其提供了一年合同,但在次年将其放弃。帕克之后前往希腊打球。之后重返NBA,先后短暂效力于底特律活塞队和菲尼克斯太阳队。2005年赛季开始前,帕克意外地得到了洛杉矶湖人队传奇主教练菲尔·杰克逊的垂青,不但获得了湖人队的合同,还一举成为其主力组织后卫,同科比·布莱恩特成为后场搭档。但是在2007年季后赛开始前,杰克逊开始培养新人乔丹·法玛尔,帕克丢失了主力位置。不满于忽然的失宠,帕克在新赛季开始前转投迈阿密热火队,但是在热火队帕克并无出色表现,在2008年3月10日被热火队裁员。其后洛杉矶快船队和丹佛掘金队都对其表示过兴趣,但最终都放弃了。之后帕克栖身于NBDL。2009年1月,帕克加盟CBA的广东东莞银行队,并帮助广东华南虎队夺得了连续两个赛季的CBA总冠军。2010年9月,帕克与俄罗斯球队签约一年。","text2":"威廉·亨利·“斯马什”·帕克的绰号是什么?","label":1} {"text1":"陈衡哲(),中国女作家,原名陈𪆒,字乙睇,笔名莎菲。她祖籍湖南衡山,出生于江苏省武进县。1914年,她考取清华大学在上海留美奖学金,卜美国瓦萨学院(Vassar College)获文学学士后入芝加哥大学(University of Chicago)攻读西洋史获硕士. 1920年9月27日,与任鸿隽结婚。在1917年1月胡适的《文学改良刍议》出现之后,陈衡哲的白话小说《一日》于1917年5月发表在《留美学生季报》上。因此引起了中国文学史上第一篇白话小说是否是《狂人日记》的争议。。1920年,获硕士学位的陈衡哲,受蔡元培之邀回到北京大学任教,开讲西洋史和英文课,成为中国第一位女教授。一年之后,她怀孕生女,辞职养育孩子的同时,写作了“女性视角”的《西洋史》。她认为,女性应该尽力发展双重角色:母职和独立人格。陈衡哲是当期之才子也。病逝于上海。她与丈夫任鸿隽都是胡适多年的好友,1920年时胡适还曾在《新青年》上作诗《我们三个朋友》赠予他们夫妇二人。胡适的女儿素斐的名字则来源于陈衡哲的英文名Sophia。","text2":"陈衡哲回北大教授什么课程?","label":1} {"text1":"国会法案(Act of Congress)是由美国宪法授权美国政府所制定颁布的成文法。当议案在国会两院以简单多数的得票通过,接著再由总统签署后即完成立法,并正式成为联邦法律。在颁布成为联邦法律之前,议案必须通过参众两院的半数投票同意后,再经总统签署。经国会参众两院认可的成文法或决议文(resolution)必须经过以下任一程序才可成为法律:经由前两种程序通过的法案将由总统「颁布」(promulgation)实行。若法案是经由第三种方式通过,最后再审期间国会的监票委员负责颁布法律。在美国宪法的规定中,若总统在时限内未对议案或决议文表示意见并交还国会,议案将自动成为法律;然而,若国会在这段期间内休会,那议案便会被废弃且无法发起再审(参见:口袋否决)。此外,若总统在国会休会期间否决了议案或决议文,再审成功的条件需要获得国会两院三分之二以上的投票支持。成为法律的国会法案会列在《美国联邦法律大全》(\"United States Statutes at Large\")中。和民间私人出版的《美国法典》相较,几乎所有的法案都有做出小幅的文字修正。所有国会法案都不得违反宪法,也不得超越宪法赋予国会的权力。否则美国最高法院将能够宣布法案违宪。","text2":"总统要签署议案,必须满足什么条件?","label":1} {"text1":"1918年教育法(\"Education Act 1918\"),又称为费舍教育法(\"Fisher Act\"),是英国国会通过的一项法案。草拟者为文教大臣赫伯特·费舍。1918年教育法将从学校毕业的年龄提高到14岁,并计划扩展高等教育。1918年教育法的另一特色是提供辅助设施(幼儿学校、体格检查和特殊需要儿童中心等)到1920年代,年幼儿童的教育日益成为政治家与教育家关注的问题。由于公众争论的升级,当时的政府委托哈多爵士负责进行教育调查,哈多委员会在1926年、1931年和1933年,发表了3个非常重要的报告。这些报告带来初等教育的重大改变。例如,导致了5-7岁儿童的幼儿学校和7-11岁的小学被分开实施。报告还建议班级规模不超过30人。这些建议标志进步主义教育思想胜过了传统教育思想,更多为政策制定者和教师们所喜爱。","text2":"1918年教育法的草拟者是谁?","label":1} {"text1":"曼哈顿下城(,或称)是美国纽约市曼哈顿最南端的部分,其最常见的边界是北到14街,西到哈德孙河,东至东河,南到纽约港(也称为上纽约湾)。当提到曼哈顿下城商务区时,通常指北到14街以南约1.5英里,和该岛最南端以北一英里,从哈德孙河附近的钱伯斯街(Chambers Street),东到布鲁克林大桥出入口和立交桥。其他两个主干道有时也被确定为曼哈顿下城的北部边界:钱伯斯街以北大约半英里的运河街(Canal Street),和14街以北大约半英里的23街。曼哈顿下城商务区构成钱伯斯街以南区域的核心,包括金融区—通常称为华尔街,和世界贸易中心遗址。在该岛最南端是炮台公园(Battery Park),在西侧,TriBeCa区横跨钱伯斯街,在街的东端是庞大的曼哈顿市政大楼。纽约市政厅就位于金融区以北。钱伯斯街南面还有炮台公园城规划区域和南街海港历史区域。钱伯斯街和布鲁克林大桥以北、运河街以南是纽约最早的唐人街,许多法院大楼和其他政府办公楼也位于这一区域。下东城跨运河街。运河街以北、14街以南有SoHo、肉类加工区(Meatpacking District)、西村、格林尼治村、小意大利、Nolita和东村。介于14街和23街之间是下切尔西、联合广场、熨斗区(Flatiron District)、Gramercy和大型住宅项目Peter Cooper Village—Stuyvesant Town。曼哈顿下城商务区是全美第四大中央商务区,仅次于曼哈顿中城、芝加哥的卢普区和华盛顿哥伦比亚特区。此前曾经是美国第三大商务区,之所以下降到第四位是因为九一一事件爆发后,该地损失了世界贸易中心16,000,000平方英尺的办公大楼楼板面积,相当于辛辛那提全市办公大楼的总和。但是兴建自由塔兴建高盛财务公司的新总部后,预计该地区将恢复其排名第三位。","text2":"美国另外几个比较大的商务区叫什么名字?","label":1} {"text1":"满洲国协和会为满洲国的官民一体国民组织,起初石原莞尔等人期许本组织能担任未来一党执政的重任,然而在之后则受到日本大政翼赞会旗下各组织影响。九一八事变后,倡导从中华民国独立并施行君主政治以推动新国家建设的自治指导部为协和会的起源。满洲国建国后,自治指导部则转型为官民一体化的满洲国协和会。协和会与满洲国政府共同宣称推动建国理想(建设王道乐土),并以此对国民进行宣传教育。协和会章程规定:“本会以谋求满洲国的建国精神和宣扬王道乐土政治为宗旨。” 1936年7月25日协和会成立4周年举行纪念会,改名为“满洲帝国协和会”。同年9月18日,日本关东军司令官植田谦吉发表《满洲帝国协和会之根本精神》的声明,阐述协和会的本质,就是以皇民化为目标的“民族协和”,实施“建国精神”,即“以实现日满一德一心、民族协和、王道乐土、道义世界为理想的天皇的圣意”。满洲帝国协和会中央本部机构庞大,设有总务部、指导部、实践部、训练部、文化部、青少年部、调查部等10多个部门。协和会基础单位为分会,与每个地区设立。并于各地方行政机关设立本部,负责统括分会。取代从未开议的立法院,以分会代表进行的连合协议会为实质之民意机关。石原莞尔等人期待协和会能取代关东军成为「未来的领导者」,拥有一党执政的地位。不过随著关东军对满洲国统治的深化,满洲国逐渐成为日本完全的属国,协和会失去其存在意义。满洲帝国协和会为唯一永久、举国一致的实践团体,与政府互成表里。实现以上建国理想,创建道义世界。满洲国协和会(1932年、大同元年)成立时的主要干部满洲国执政(后皇帝)任名誉总裁,国务总理大臣任会长,各省长、县长兼任所辖地域的本部长。满洲帝国协和会中央本部旧址位于今中央大街82号,人民广场南200米处。东南接儿童公园,北隔咸阳路与长春市委大楼后身相对,西邻人民大街。该旧址是一座砖石结构的组合建筑物,地面建筑为两层楼房,另有地下室,建筑面积7955平方米,占地面积8749平方米。1950年修复为长春市文化宫,1958年由沈阳军区接管,改为军人俱乐部,后又将部分房舍改建为吉林省军区第二招待所。旧址保存较为完好。","text2":"满洲国协和会是一个什么协会?","label":1} {"text1":"蓝灰扁尾海蛇(学名:\"Laticauda colubrina\"),又名黄唇青斑海蛇,是蛇亚目眼镜蛇科扁尾海蛇属下的一种有毒海蛇,俗名灰海蛇、火烧蛇。主要分布于印度洋至太平洋一带的海域,孟加拉湾到马来半岛沿海、巴布亚新几内亚、澳大利亚、菲律宾、斐济、汤加、日本沿海以及台湾沿海等地。目前未有任何亚种被确认。蓝灰扁尾海蛇的腹部扁平宽大,鼻端及吻部同样扁平,吻鳞为单片大鳞;身上有21至25排不规则分布的鳞片。其身体主要呈稍微被压扁的近管状的形状,吻部高翘,前额有一片大鳞片(亦有部分蛇类可能没有这块鳞片)。有一片眶前鳞及两片眶后鳞,7至8片上唇鳞,第三至四片上唇鳞贴近双眼;有五片下唇鳞,接触颏部。腹部有213至245片鳞片,雄蛇有37至47片尾鳞,雌蛇则有29至35片尾鳞。雄性的蓝灰扁尾海蛇约长9公分,雌蛇则可长达14公分。体色方面,牠们的背部多以蓝色及灰色为主,腹部则以黄色为主,身上满布黑色的横向条纹。唇部呈黄色,头部则主要为黑色。蓝灰扁尾海蛇栖息于海洋之中,多于夜间出没沿岸沙滩、岩礁间。日间时候,牠们多躲藏于海岸边的岩缝间休眠。以海蛇科而言,蓝灰扁尾海蛇拥有较强的陆行本领,即使离海有数十米之遥,仍有可能找到他们的踪影。捕食方面以一般鱼类为主。蓝灰扁尾海蛇属卵生蛇类,雌蛇每年均于海岸岩石间产卵,每次约能诞下5至6枚蛇卵。蓝灰扁尾海蛇拥有强烈的神经毒素,但由于牠们的口部细小,攻击性较低,因此对人类的威胁并不特别严重。","text2":"蓝灰扁尾海蛇日间都在做什么?","label":1} {"text1":"工研院产业学院(简称产业学院,英文简写:ITRI College)位于台湾新竹县竹东镇,是工业技术研究院衔接研究发展与研发服务的新模式。在新竹市、台北市、台中市、原台南县、原高雄县等地皆设有学习中心。工研院产业学院主要是以教学计划协助产业升级与转型,同时也协助工研院内各所和中心的产业教育和对于产业界的服务,此外亦协助政府拟订国家人才发展策略,并建立完备的人才发展制度,使企业得以更容易孕育专业与跨领域人才。执行工研院员工训练,厚植人才资本,使工研院成为世界级的研发组织;并依据产业人才职能需求,以工研院四十年来蓄积深厚的研发能量为基础,汇聚工研院研发成果、领域专家,及先进的机仪设备与环境等各项资源,依据市场与职能需求,发展多元学习模式设计基础并办理各类科技专业学程,为产业升级与企业转型建立核心人才。另一方面产业学院持续研究先进国家人才发展策略,不仅协助政府拟定新兴产业人才政策与培训计划,亦协助推动我国职能基准、人才认证及发展的机制, 期望为台湾人才竞争力建立坚实的基础与优势。成为「培育新世纪产业创新人才」的标竿机构","text2":"工研院产业学院还在哪些地方设有学习中心?","label":1} {"text1":"宝永山()是位于日本静冈县御殿场市的一座火山,在宝永4年(1707年)的宝永大喷火中诞生的,为富士山最大的侧火山(寄生火山)。标高2,693米。因其登山路线已经得到整备,可进行攀登。由于在宝永喷火后富士山迄今还没有再喷发,因此宝永山成为了最近的一座因喷发而产生的寄生火山。与攀登富士山相比,攀登宝永山的难度较低。宝永火口是因宝永大喷火而形成的3个相连的火口。若能登上宝永山顶峰,则可近距离的观看到火口。自富士宫口五合目开始到新6合目的这段路上,有前往富士山顶的登山道和前往宝永山的登山道分岔标识。自此继续向前走后,可看到富士宫口与须山口的分岐点。自该处向前走后不久就可到宝永山山顶。","text2":"宝永火口是怎么形成的?","label":1} {"text1":"2009年印度的选举是印度独立以来的第15次。分为五个阶段。有7亿多可以投票的合格选民。预计两个主要政党国大党和印度人民党都不会在下院543个席位中赢得足以独立组建政府的席位。这使左翼和地区性政党在安得拉邦等关键邦内的选举舞台上扮演更重要的角色。250多万安全人员在印度举行选举的15个邦和两个领地内执勤。在4月16日开始的第一阶段的投票过程中至少有18人遇害。在几个邦里,对投票站和安全部队的袭击扰乱了投票的进行。在中部和东部地区的几个邦内,安全戒备份外严密。在那些地区,4月16日的时候,被称为纳萨尔派的毛派叛乱分子在贾坎德邦,引爆了一颗地雷,杀害了一些选举官员和政府军人,并且纵火焚烧了一些投票站,扰乱了投票进程。全国选举副专员巴拉克里西南说,暴力等破坏行动影响了86个投票站,在投票站总数中只占极小部分。国大党取得206席,其所属的「团结进步联盟」合计取得262席。印度人民党取得116席,其所属的「全国民主联盟」合计取得158席。「第三阵线」合共取得76席。「第四阵线」合共取得27席。","text2":"2009年印度议会选举是印度独立以来的第几次选举?","label":1} {"text1":"Apache Hadoop是一款支持数据密集型分布式应用程序并以Apache 2.0许可协议发布的开源软体框架。它支持在商品硬件构建的大型集群上运行的应用程序。Hadoop是根据谷歌公司发表的MapReduce和Google档案系统的论文自行实作而成。所有的Hadoop模块都有一个基本假设,即硬件故障是常见情况,应该由框架自动处理。Hadoop框架透明地为应用提供可靠性和数据移动。它实现了名为MapReduce的编程范式:应用程序被分割成许多小部分,而每个部分都能在集群中的任意节点上执行或重新执行。此外,Hadoop还提供了分布式文件系统,用以存储所有计算节点的数据,这为整个集群带来了非常高的带宽。MapReduce和分布式文件系统的设计,使得整个框架能够自动处理节点故障。它使应用程序与成千上万的独立计算的电脑和PB级的数据连接起来。现在普遍认为整个Apache Hadoop“平台”包括Hadoop内核、MapReduce、Hadoop分布式文件系统(HDFS)以及一些相关项目,有Apache Hive和Apache HBase等等。2008年2月19日,雅虎使用10,000个微处理器核心的Linux计算机集群运行一个Hadoop应用程式。其他知名用户包括:升阳电脑的Sun Grid Engine可以用来调度Hadoop Job。威斯康辛大学麦迪逊分校的Condor计算机集群软件也可以用作Hadoop Job的排程。","text2":"Hadoop框架为应用提供什么?","label":1} {"text1":"《1918年人民代表法令》(英语:Representation of the People Act 1918),亦称为“《第四部改革法令》”,是英国在1918年进一步扩大下议院选民基础的法案。根据1884年改革法案,凡是每年付十磅以上租金的租户或者是拥有自己物业的城镇男性居民,或者是农村里每年支付十二磅以上地租的男性农户和土地年产值超过五磅的地主都有权参与下议院的选举。虽然选举权的财产限制已经很低了,但是仍然会有一部分人因为财产限制而无法参与选举。伴随着第一次世界大战的结束,成千上万的复员军人发现自己不惜牺牲生命保护了英国宪政制度,到头来却没有投票权。因此,政治家们也开始考虑要进一步扩大选举权。在本次改革中,彻底取消了下议院选举权的财产限制。规定但凡年满21周岁的成年男性以及年满30周岁的成年女性,均有下议院的选举权。经过本次改革,英国符合选举条件的选民人数从七百七十万左右一跃变成两千一百万左右。并且在1918年举行了真正意义上的普选。由于本次的改革法案中对女性选民的年龄限制高过男性,因此被认为还是不公平。而这一状况直到1928年改革法案才得以改善。另外,由于在之前的英国选举中,存在着违背当代一人一票(one person, one vote)准则的情况:有将近7%的选民拥有复数选票,比如拥有复数物业的人有权在其物业所在选区以及自己所居住选区都进行投票,而1918年改革法案并没有对这一情况进行改善。这一状况直到1948年改革法案才予以改善。","text2":"经过本次改革,英国符合选举条件的选民人数有何变化?","label":1} {"text1":"黑边鲾(学名:),又称台湾鲾,俗名碗米仔、金钱仔,为辐鳍鱼纲鲈形目鲾科的其中一个种。本鱼分布于印度太平洋区,包括东非、马达加斯加、模里西斯、塞席尔群岛、亚丁湾、红海、马尔地夫、巴基斯坦、印度、斯里兰卡、孟加拉湾、安达曼海、泰国、越南、马来西亚、柬埔寨、台湾、中国沿海、日本、韩国、菲律宾、印尼、澳洲、新几内亚、马里亚纳群岛、马绍尔群岛、帛琉、密克罗尼西亚、所罗门群岛、诺鲁、新喀里多尼亚等海域。水深10至100公尺。本鱼全身银白色,体呈椭圆而侧扁,体高甚大。背鳍硬棘部之边缘具黑色,吻部密布黑斑。全身被小型鳞片,深埋于皮中,不易脱落。侧线完整而高位,口能伸缩自如,在水中取食时,会发出「波波」的声音。眼前有小棘2枚。背鳍硬棘8枚、软条16枚;臀鳍硬棘3枚、软条14枚。体长可达14公分。本鱼属于小型热带沿岸肉食性鱼类,喜栖息在沿岸沙泥地、河口及内湾,常与其他种类聚集成群一起觅食,以底栖生物及浮游生物为食。小型食用鱼,经济价值较差,通常当做下杂鱼处理。","text2":"黑边鲾主要分布在哪里?","label":1} {"text1":"英国专辑排行榜()是按英国唱片销售量排名的排行榜,由英国官方排行榜公司(\"The Official UK Charts Company\",简称OCC)编辑,并发表在音乐周刊(前75名)、OCC官网(前100名),完整的前200名列表则只刊登在排行榜+。『英国专辑排行榜』只是个统称,一般是指流行百大专辑榜(Official Albums Chart Top 100),专辑要拥有登上该榜的资格,必须要有确定的曲目、定价,并至少有三首歌曲或总共二十分钟长,而且定价不能过于低廉(价格不能低于4.24英镑)。但自从1989年1月份起,该榜将不接受多位歌手合作的合辑作品,该类作品将被归类在英国合辑排行榜(Official Compilations Chart Top 100)内,例如1992年的《终极保镳电影原声带》()就被归类为合辑只能进入此榜。详细的入选资格可参见OCC官网。虽然专辑销售量渐渐成了判断歌手成功与否的依据,但专辑排行榜一直不比单曲排行榜(UK Singles Chart)来得受瞩目。然而事实上,近年来专辑排行榜仍维持著不错的状况,即使音乐数位化革命正威胁著传统唱片市场,但整体而言仍算稳定,2005年甚至有专辑在英国达到1.262亿张的销售量。","text2":"专辑要拥有登上该榜的资格,必须要有什么?","label":1} {"text1":"曾涣钊 Arthur Tseng(),台湾管理顾问、作家、企业家。现任于安瑟管理顾问公司创办人总经理、限制管理(TOC)知识发展中心主持人。曾任职外商公司、担任生产管理经理、物料管理主任、经理及处长等职,有二十余年实务经验。从事企业管理顾问工作二十余年,为台湾和中国数百家知名企业授课和辅导。专长领域在限制理论(TOC)之系统思考(Thinking Process)与管理技能(Management Skills)、关键链专案管理(CCPM)、企业资源规划(ERP)及供应链管理(SCM)等,美国营运管理协会APICS在台代表。经历中国生产力中心(CPC)、工研院、经济部中小企业处、资策会、交通大学推广教育等机构讲师、中华采购管理协会(SMIT)理监事\/讲师、民营企业顾问、经济部资讯专业人员鉴定计划命题\/审题委员等。发表过管理文章数十篇,散见于经济日报、资讯与电脑杂志、管理杂志、EMBA杂志、采购与供应双月刊等。","text2":"曾涣钊现在在哪个公司工作?","label":1} {"text1":"康氏马鲛(学名:)为鲭科马鲛属的鱼类,中国大陆俗名马鲛(康氏马鲛的简称),闽南语称马加鱼、𩵚魠鱼(土脱鱼、土托鱼、土魠鱼、头魠鱼)。分布于印度洋北部沿岸、红海、东至澳大利亚、北至日本、朝鲜以及中国大陆南海、台湾海峡、东海南部等海域,属于近海暖水性中上层鱼类。本鱼无鳔,第二背鳍下方侧线突然向下弯曲,幼鱼身体上具椭圆形的大黑斑,中间三分之一第一背鳍白色,其余鳍黑色,背鳍硬棘15-18枚;背鳍软条15-20枚;臀鳍软条16-21枚;脊椎骨42-46枚,体长可达240公分,栖息在沿岸水质混浊的海域,会进行回游,属肉食性,以鱼类、头足类、甲壳类等为食。台湾海域所产的马加鱼有六种:䲠鱼肉美刺少,含高蛋白质、低脂肪、矿物质、钙、磷、铁等,是闽南、台湾地区喜食的鱼类,尤其土托鱼羹更是有名的小吃。","text2":"康氏马鲛属于什么食性鱼类?","label":1} {"text1":"内大臣府是日本曾经设立以辅佐天皇、负责宫廷之文书等事务之政府机关。创立于1885年,1945年废止。其最高长官为内大臣,并不等同于日本古代律令制度下的同名太政官职位。明治政府在1885年将太政官制改为内阁制度。作为内阁成员之一,内阁总理大臣职务从国务大臣原负责之职务分离,内大臣一职则负责内阁权限之外宫廷事务而再度设立。明治政府下所设立之内大臣平常在宫中辅佐天皇,负责玉玺保管和诏令与其他宫廷文书之事务。内大臣负责执行天皇之决定,而非直接对民众负责,其职务与权力的范围不甚明朗,缺乏准确之定义,通常是天皇非常信赖、并通常有特殊关系之人。当初太政官制废止之时,前太政大臣三条实美出任内大臣。内大臣空缺之紧急情况由枢密院之议长临时代理内大臣之职务。这始自二二六事件之后枢密院议长一木喜德郎紧急代理内大臣(虽然当日即辞去职务)之先例。另外、还有身为皇族之贞爱亲王出任内大臣(1912年 - 1915年)之先例。当初之内大臣在昭和时代由元老取代,在政府中有非常大的权力。太平洋战争结束后,驻日盟军总司令(SCAP)于1945年11月24日命令废除内大臣府。1907年以降,内大臣府由11人名职员构成。至废止前由以下的人员构成。※括号内为任期","text2":"内大臣府是日本曾经用来做什么的机关?","label":1} {"text1":"新发田长敦(しばた ながあつ,1538年(天文7年)─1580年(天正8年)。新发田家的祖先是在源平合战中,讨伐越后城资盛的总指挥御家人佐佐木盛纲。1530年,上杉家一族的上条定宪和守护代长尾家相互对立,发生史称的上条之乱。五十公野城城主新发田纲贞(新发田长敦之父)与本庄房长、鲇川清长、水原政家、黑川清实、五十公野景家、加地春纲、竹俣昌纲等等下越国人(即扬北众)一起站到上条定宪一方,与长尾为景交战,在历经一番战战合合后,最终都为上杉谦信的勇武所折服,成为其麾下势力。上杉谦信平定越后之际,新发田纲贞的嫡子新发田长敦继承了家名,成为新发田城城主。新发田长敦侍奉上杉谦信后逐渐头角显露,以优秀的外交手腕著称,担任春日山城门的班职,并在各地的战役中活跃,因此受上杉谦信的重用,列为越后七手组大将之一。第四次川中岛之战中,新发田部队击溃了武田军诸角虎定部,并讨取老将诸角虎定,立下了大功。1568年,武田军进入信州长沼,新发田家受到上杉谦信动员的命令,新发田长敦率同一族的五十公野家出阵信州饭山,这以后他又先后担当了上杉家内政和充分发挥了其优秀的外交手腕,在与武田胜赖的和平谈判中出尽全力,直至1580年病死为止。作为其一族的竹俣庆纲和入继了五十公野家并担任外交方面要职的弟弟五十公野治长(新发田重家),也是上杉谦信政权中占有重要地位的家臣。1578年谦信病殁,他在之后的御馆之乱中支持上杉景胜一方,表现十分活跃。新发田重家是扬北众中的重镇,御馆之乱后,因赏赐问题而产生谋反之心,『越后治乱记』中谈到:上杉景胜对新发田重家的战功给予了封赏,可是新发田重家对得到的赏赐数量却表示不满,一方面他装做满足于继承新发田家,而另一方面却在不知不觉中走向了谋反的道路。1580年,新发田长敦的弟弟新发田重家从娘家五十公野家返回继承了新发田家。1582年,新发田重家最后决定谋反,并侵占新潟津的冲之口,并与织田信长取得了联系及协助织田军攻打上杉家,而且逼的兵败如山倒的上杉景胜差点自尽,不过却因为发生本能寺之变而令正在进攻上杉家的织田军返回近畿,同时亦因新发田重家的谋反而令新发田家走向了灭亡。","text2":"新发田长敦在世年份?","label":1} {"text1":"勉方言是勉语的一种,属于苗瑶语系瑶语支。说勉方言的人自称“勉”或“优勉”等,共有100万人,其中中国55万,越南35万,其余分布在老挝、泰国、美国等地。中国一般以广西金秀瑶族自治县长垌乡镇冲村的勉语为代表腔。1984年中国和美国统一的瑶文方案所拼写的正是勉方言。毛宗武(2004)把国内的勉方言划分为四个土语过去曾把勉方言和金门方言合称勉金方言。勉文有30个声母。《瑶文方案》中声母的排列顺序是b p mb m hm f w, z c nz s, d t nd n hn l hl, j q nj ny hny y, g k nq ng hng h。镇冲勉语中还有声母,在勉文中作为声母h在介音前的变体。有零韵头、i 、u 、ui 四种。和声母的配合有一定限制,比如韵头iu只能接在软腭声母之后。也可以把韵头分析成声母的颚化和唇化。其中aa、ae、or为长音,er很罕见。因为a必须和韵尾连用,所以aa后无韵尾的时候简写作a。借用汉语的舌尖元音拼写为ir。韵尾有元音韵尾(i、u)、鼻音韵尾(m、n、ng)和塞音韵尾(p、t、k、q )三种。其中k韵尾多数已经脱落变为q。有八个声调。塞音韵尾只能配合两个入声调,其他韵尾只能配合舒声调。入声调原来用单独调号q和r,现在用相近的舒声调的调号(v和c)。韵腹元音的长短对入声调调值稍有影响。长元音第七调调值为55,第八调调值为12。","text2":"勉方言属于什么语序语支?","label":1} {"text1":"天主圣三桥(意大利语:Ponte Santa Trinità)是意大利佛罗伦萨市中心一座跨越阿诺河的石灰岩桥梁,兴建于文艺复兴时期。天主圣三桥是世界上最古老的椭圆型拱桥,长32米。相邻的两座桥梁是东侧的老桥和西侧的卡瑞拉桥(Ponte alla Carraia)。该桥由佛罗伦萨建筑师Bartolomeo Ammanati兴建于1567年至1569年。坐落在老桥下游的显著位置,是佛罗伦萨中世纪街道规划的重要环节。1252年兴建的木桥被7年后的洪水冲走,重建的石桥又被1333年的洪水摧毁。Taddeo Gaddi的五拱石桥毁于1557年的洪水,于是兴建Ammanati的石桥。1608年,桥上增建了四季雕塑,以庆祝科西莫二世·德·美第奇与玛利亚·马格达莱纳的婚礼:《春》(Pietro Francavilla),《夏》和《秋》(Giovanni Caccini),《冬》(Taddeo Landini)。1944年8月8日,该桥被撤退的德国军队摧毁,1958年重建,使用从阿诺河打捞的原来的石头,或者同一采石场的的石料。","text2":"1608年,桥上为什么增建了四季雕塑?","label":1} {"text1":"华阳路街道是中国上海市长宁区下辖的一个街道办事处,位于长宁区东部,东到长宁路、安西路、武夷路接邻江苏路街道,北到苏州河接邻普陀区。面积2.04平方公里,户籍人口7.04万人(2008年),下辖21个居委会。华阳路街道的主要街道长宁路和定西路,构成繁华的中山公园商圈。辖区内的圣约翰大学旧址(今华东政法大学)、中山公园,是愚园路历史文化风貌区的重要组成部分。上海市轨道交通二号线、三号线、四号线以该街道辖区的中山公园站为换乘枢纽。华阳镇下辖华阳路一居委会、华阳路二居委会、华阳路三居委会、华阳路四居委会、华阳路五居委会、华院居委会、秀水居委会、长宁路一居委会、长宁支路二居委会、长宁路二居委会、苏一居委会、潘东居委会、潘西居委会、陶家宅居委会、徐家宅居委会、姚家角居委会、建宁居委会、潘中居委会、天诚居委会、飞乐居委会、西一居委会。","text2":"华阳路街道下辖多少个居委会?","label":1} {"text1":"曼西人(),旧称“沃古尔人”(),是西伯利亚西部,鄂毕河流域的原住民族。曼西人分布在乌拉尔山以东的鄂毕河流域,即俄罗斯联邦秋明州的汉特-曼西自治区及其周边地区,也分布在秋明州。根据俄联邦2002年的人口统计,总人口数量为11432。各地曼西人的人口数据如下:曼西人和汉特人在族源、语言、历史和文化等方面有诸多共同点。其族源可追溯到1世纪中叶的额尔齐斯河流域的草原乌戈尔部落。后来一些部落迁至鄂毕河下游,逐渐形成新的民族,以捕鱼和狩猎为生,生活方式为半定居半游牧。他们是15-16世纪的西伯利亚汗国臣民,但也有自己酋长国。16世纪末期,曼西人被沙俄哥萨克叶尔马克·齐莫菲叶维奇征服(是第一个)。与汉特人相比,曼西人受俄罗斯同化程度更深。1930年,前苏联建立奥斯恰克-沃古尔民族区,1940年改称汉特-曼西民族区。1960年代以来,该地区成为重要的石油、天然气产地。曼西语和汉特语相近,也与匈牙利语有关,这些语言共同组成乌拉尔语系的乌戈尔语支。曼西语、汉特语和俄语都是汉特-曼西自治区的官方语言。","text2":"曼西人与汉特人有什么差别?","label":1} {"text1":"梅方,是一名中国足球运动员,场上位置是后卫,现在效力于中国足球超级联赛球队广州恒大。梅方出自武汉光谷青训体系,2008年进入武汉光谷一线队。当年8月,他和队友汪洋一起法乙球队斯特拉斯堡试训。 10月初,武汉光谷因不满中国足协的判罚而宣布退出中超联赛。梅方和汪洋再次到欧洲接受试训,11月前往德甲球队沃尔夫斯堡, 12月初又一起在比利时球队色格拉布鲁日试训。 梅方和汪洋都同时得到色格拉布鲁日的肯定并被邀请签约,但是两人回国后却因为湖北队将参加2009年的全运会而被湖北省足协大力挽留。最终梅方决定放弃留洋,留在武汉加盟了刚刚成立的湖北绿茵。梅方在2009赛季是湖北绿茵的主力球员,帮助球队杀入该赛季中乙联赛决赛,升级到中甲联赛。在决赛中,梅方射进球员生涯的首个联赛进球,不过他在点球决战中第一个出场并罚失点球,最终湖北绿茵不敌湖南湘涛屈居亚军。 2010年1月,他再次拒绝色格拉布鲁日的邀请,留队征战中甲联赛。 他在中甲联赛中继续作为球队的主力出战,并在2012年初和球队续约两年。 2012年3月,他被任命为球队的新任队长, 并在2012赛季率领武汉卓尔以联赛亚军的成绩升级到中超联赛。2013赛季,武汉卓尔在中超的成绩不佳,一直位于降级区。2013年8月18日,在联赛第21轮客场1比5惨败给广州富力后,武汉卓尔宣布剥夺梅方队长的职务。武汉卓尔以联赛倒数第一名的成绩降回中甲联赛后,合同即将到期的梅方一度传出有可能会转会到北京国安。 不过在2014年1月1日,他选择自由转会加盟中超卫冕冠军广州恒大。 2014年2月26日,梅方在2014年亚足联冠军联赛小组赛首轮广州恒大主场4比2逆转墨尔本胜利最后时刻替补亚历山德罗·迪亚曼蒂出场,首次代表广州恒大参加正式比赛。2014年6月18日,梅方在中国2比0击败马其顿的友谊赛首发出场,上演成年国家队首秀。2015年1月,梅方入选中国国家男子足球队2015年亚洲杯23人大名单,并成为球队主力后卫之一,可担任中后卫以及边后卫。","text2":"梅方的职业是什么?","label":1} {"text1":",为1978年香港邵氏公司出品,张彻执导的一部武打电影。满清入主中原后,有讳于汉族民众以武术造反,严禁民间习武。少林本也应列禁例之内,但有鉴于寺中弟子皆武艺高强,清廷于是命少林选派弟子,教授八旗官兵武艺。然而有一将军(王龙威饰)深觉少林终乃清廷心腹大患,又不满于其隐匿寺中武僧,仅指派俗家弟子前往授艺,终生挑拨南北少林两家,令双方自相残杀以剿少林之念。将军请得北少林弟子包山雄(鹿峰饰)、徐方(孙建饰)及杨仲飞(江生饰)三人至府中任教头之职,岂知三人抵达之后才发现将军府早已有三位南少林教头(屠龙、孙新祥、余太平饰),双方在将军耸动下比武以分高低,由北少林三人胜出,得任将军府教头之位。南少林三弟子心灰意冷,欲离府而去,却教将军私下杀害,并嫁祸于北少林弟子。南少林三人之师麦奇(詹森饰)得知此事后,遂命其长子麦远(杨熊饰)和另二弟子(萧玉龙、谭镇渡饰)前往将军府向北少林三弟子挑战,北少林弟子因不慎误伤三人,而更加深了南北少林间的误会。在门下众弟子皆伤于北少林弟子下,麦奇悲愤难当,招回游历在外的弟子何英武(郭追饰)及朱赞成(罗莽饰),伙同次子麦风(韦白饰)分别投于南少林名宿门下修练武艺,以和北少林弟子对抗,为同门师兄弟报仇。何英武等三人武艺大成后,虽敬重于北少林三弟子,疑事有蹊跷,但碍于同门弟子实乃伤在三人手下,不得不找上将军府与三人一决生死……。","text2":"电影中被将军杀害的三位南少林武僧由谁饰演的?","label":1} {"text1":"观察家报(The Observer)是英国的一份报纸。于每周周日发行。观察家报实际上是周一到周六发行的卫报的周日版。政治立场偏向自由主义和社会民主主义。观察家报创刊于1791年12月4日,是世界第一份在礼拜日发行的报纸。在1911年,William Waldorf Astor收购了观察家报,之后该报在政治立场上较为倾向保守党。1942年,该报公开宣言了无党派倾向的编辑方针,这在当时是较为少有的。1977年,因为经营状况欠佳,观察家报被卖给了Atlantic Richfield公司(现在的ARCO)。之后又被Lonrho plc收购。1993年起,观察家报成为卫报媒体集团旗下的一份报纸。","text2":"哪一年报纸公开宣言了无党派倾向的编辑方针?","label":1} {"text1":"中国政府为了奖励在科技进步活动中作出突出贡献的公民,推动中国科技事业的发展,国务院设立五个国家科学技术奖。包括国家最高科学技术奖、国家自然科学奖、国家技术发明奖、国家科学技术进步奖和中华人民共和国国际科学技术合作奖。奖项中规格最高的国家最高科学技术奖每年评审一次,每次选出不超过两名科技成就卓著、社会贡献巨大的公民,由国家主席亲自签署并颁发荣誉证书和高额奖金。每位获奖者的奖金总额均为人民币500万元,其中50万元直接授予个人,另外450万元作为科学研究经费由获奖人全权管理具体用途。国家自然科学奖是授予在数学、物理学、化学、天文学、地球科学、生命科学等基础研究和信息、材料、工程技术等领域的应用基础研究中,阐明自然现象、特征和规律、做出重大科学发现的中国公民的奖项。国家自然科学奖不授予组织。国家自然科学奖设一、二等两个奖励等级。国家自然科学一等奖是自然科学领域的国家最高奖。国家技术发明奖授予在产品、工艺、材料及其系统等有重大技术发明的中国公民。国家技术发明奖不授予组织。国家技术发明奖设一、二等两个奖励等级。国家技术发明奖表彰的是当年中国国民经济中新的、先进的、效益好的新技术。国家科学技术进步奖授予在技术研究、技术开发、技术创新、推广应用先进科学技术成果、促进高新技术产业化,以及完成重大科学技术工程、计划等过程中做出创造性贡献的中国公民和组织。每年国家科学技术进步奖总数不超过400项,分为特等奖,一等奖, 二等奖3个等级。中华人民共和国国际科学技术合作奖是由中国国务院设立的国家科学技术奖励,以奖励对中国科技事业作出贡献的外国人或组织。其前身为国家科学技术委员会设立的中国国际科技合作奖,根据1993年7月经全国人大常委会八届二次会议通过的《中华人民共和国科学技术进步法》的规定,国务院设立“中华人民共和国国际科学技术合作奖”(简称国际科技合作奖)。国家科学技术奖励委员会是对奖励进行评选的机构,设立主任委员,副主任委员,秘书长,委员等职位,一般主任委员,秘书长各一名,副主任委员两名,另外有委员若干名。委员会成员主要是中国科技部,中国科学院,中国工程院及各知名大学的专家学者。","text2":"中华人民共和国国际科学技术合作奖的前身是什么?","label":1} {"text1":"《猪哥亮俱乐部》是巨登育乐于1990年后期至1994年贩售、出租的综艺节目录影带(录影秀),制作人是杨登魁。巨登育乐是八大电视的前身。《猪哥亮俱乐部》是巨登育乐第一部综艺节目、也是八大电视开基原祖,主打游览车、台湾中部、台湾南部小家庭市场,与三立影视《猪哥亮歌厅秀》形式相似,但多了一些灯光效果,舞台设计与《猪哥亮歌厅秀》不同。也在这同时,三立影视推出《三立五虎将 金牌点唱秀》;而「三立五虎将」其中两员,一个是猪哥亮昔日徒弟贺一航,一个是猪哥亮昔日老战友-{余天}-,形成「师徒对打」的景况,十分有趣。该节目曾于台湾艺术台重播。因巨登育乐要把《猪哥亮俱乐部》改组,与白冰冰联合主持,才停制《猪哥亮俱乐部》。","text2":"为什么《猪哥亮俱乐部》会停制?","label":1} {"text1":"瓮盘狸藻节(学名:Utricularia\" sect. \"Calpidisca)为狸藻属下的一节。该节的10个物种皆为小型的陆生食虫植物。其分布于非洲,其中1个物种分布延伸到墨西哥以及亚洲印度。1916年,约翰·亨德利·巴尔哈特将该组作为一个独立的属进行了最初的描述。经过小宫定志在其1973年的分类修订中将其降为狸藻属下的一组。彼得·泰勒在其1986年发表的狸藻属分类学专著《狸藻属——分类学专著》中,将该组继续置于狸藻亚属下。更多现今的种系发生学资料和修订使双瓣狸藻亚属恢复且让本节置于它的范围内。经过进一步的系统发生学数据的支持,在恢复双瓣狸藻亚属的同时,也将瓮盘狸藻组置于其下。","text2":"瓮盘狸藻节分布在哪个地区?","label":1} {"text1":"DVD播放程式(DVD Player)是Mac OS X的默认DVD播放器。它支持所有的DVD标准功能,比如多重音频、视频及字幕轨道以及杜比音效、DVD@ccess URL和关闭字幕。在某些情况下,用户可以选择其中的VOB文件并将其打开。这个文件的地址是「\/Applications\/DVD Player.app」。只有当有一个DVD驱动器连接到计算机时,DVD播放程式才会被安装到OS中(例如,当一部电脑的引导盘磁盘映象发现计算机上只有CD-ROM而没有DVD-ROM驱动器,将不会启动DVD播放程式的安装)。DVD播放程式也完全兼容DVD Studio Pro和iDVD所撰写的DVD,包括用DVD Studio Pro撰写的HD DVD。DVD播放程式顺从大部分著作权法,并因此将执行最严格的限制DVD措施,如区域编码限制和限制用户行为(「停用行为」)。它甚至会令苹果的屏幕截取程序失效,直到退出DVD播放程式程序。这有效的防止了用户从屏幕上捕捉DVD内容(但用户也能通过使用Terminal command间接得到图片。当然,其它的一些软件也能达到此目的,比如VLC media player。另外,DVD区域编码限制也能通过MacTheRipper这个软件去除)。","text2":"什么情况下DVD播放程式才会被安装到OS中?","label":1} {"text1":"王崇愚(),满族,物理学家,中国科学院院士。主要研究领域为计算凝聚态物理及计算材料物理,在金属合金领域作出重大贡献,是中国多尺度研究领域的开拓者。王崇愚1950年秋—1952年夏在北洋大学读书,因为高校院系调整,他1952年夏—1953年秋在清华大学读书,1953年秋—1954年夏,转入北京钢铁学院(今北京科技大学)读书。王崇愚长期在冶金部钢铁研究总院工作,在金属电子缺陷领域作出重要贡献,1993年他当选中国科学院学部委员。1999年进入清华大学物理系担任教授。 是中科院咨询委员会委员,中科院技术科学部常委,中科院国际材料物理中心学术委员会委员,中国金属学会理事。","text2":"王崇愚的主要研究领域是什么?","label":1} {"text1":"狐狸和山鹰是伊索寓言中一个著名的故事。从前有一只狐狸和一只山鹰。他们一个住树下的洞;一个在树枝上筑巢,久而久之便成了好友。某日狐狸去猎食,请山鹰帮忙照顾小狐狸,山鹰虽然答应了,但及后却因为非常肚饿,竟然将三只小狐狸全捉回巢中与小鹰一起吃。狐狸回来后,发现有小狐狸的骨在鹰巢上跌下,知道原来是山鹰背信弃义吃了小狐狸。狐狸虽既悲且愤,但自己不会飞,哪能对付山鹰?唯有先忍下这口气,稍后再打算。几日后,有人烧羊肠来祭神,山鹰嗅到香味,便打算去抢羊肠回来给小鹰吃。当山鹰抢到羊肠后,便将羊肠带回巢中。但原来羊肠上还有点死灰,而且大风一吹便令死灰复燃,很快便火借风威地烧著了鹰巢。四只小鹰因为羽翼未丰,被烧个皮开肉烂,一只一只飞堕地上。狐狸便当著山鹰的面把四只小鹰全部吃了。山鹰见状羞愤难当,亦一头撞向悬崖自尽死了。背信弃义的人一定有报应。","text2":"狐狸为什么没有马上报仇?","label":1} {"text1":"是日本十八禁游戏制作公司NEXTON的旗下品牌BaseSon开发的PC美少女游戏及其改编的电视动画。BaseSon的第五部作品。PC版游戏的正式日文标题是《》(恋姬†无双 ~心跳☆满是少女的三国演义~),PlayStation 2移植版游戏和漫画版的标题是。《Web恋姫†梦想》()是游戏橘子日本分公司研发的网页游戏,发表于2010年6月2日。原订名称是,在2010年9月14日更改成现在的这个名称。2012年6月25日游戏橘子宣布将推出台湾版,2012年8月29日台湾版封测,2012年9月5日台湾版公测,2013年3月28日结束营运。《恋姬†无双》获得萌系游戏大赏2007的、。《ファミ通》1172号的クロスレビュー对PlayStation 2版给予27\/40分数。","text2":"PC版游戏的正式日文标题是什么?","label":1} {"text1":"尹福(),绰号「瘦尹」、「尹铁镯」(铁镯尹)。清代直隶省(今河北)冀县北漳淮村人,董海川之徒,清末八卦掌名家,开创尹氏八卦掌。尹福幼年因家乡盗匪为祸,流落至北京生活,到剪刀行当过学徒,后以卖油条、麻花卷度日。原习罗汉拳、弹腿等,师从著名镖师秦凤仪。在秦凤仪死后,拜肃王府太监董海川为师,因入门最早,为其大弟子。尹福天资聪颖,事师以礼,时时跟随在董海川身边,师兄弟皆对他十分佩服。艺成后以护院为生,曾任「崇外税务司」巡检、「善扑营」教习,后经满人佛音尼布(汉名叶诗梦)举荐供职皇宫担任侍卫,宫女、太监皆称其为老师。因为身形瘦长,外表斯文,而有「瘦尹」的绰号。他的掌力惊人,人一旦被他扣住就无法脱身,所以又被称为「铁镯子」、「尹铁镯」。他身处皇宫之中,行事低调,弟子多为皇宫侍卫,他也负责教导许多皇族成员武功。其师弟程廷华,在北京城中,与江湖人物来往密切,门徒遍满北京城,称为程派八卦掌,形成八卦掌的两大传承。当时又称尹福为宫内派,而程廷华为宫外派。尹派的特点主要是手掌平放,又称牛舌掌。掌势较少,较为朴实无华。另一个特色,则是以罗汉拳为主要入门。其长子尹世庭早逝,长女尹金玉嫁给其徒何金奎,三子尹成璋、四子尹玉璋皆承家学,擅长八卦掌。他的弟子有宫宝田、杨名山、马贵、居庆元、李永庆、马世清、门宝珍、钟声、金增启等人。","text2":"为什么尹福是拜肃王府太监董海川的大弟子?","label":1} {"text1":"希望号(,,意为“希望”)是伊朗首颗自行研制的人造卫星。伊朗国家电视台报道说2009年2月2日成功发射了一颗用于科研和通讯的数据处理人造卫星。伊朗自行研制的 “信使号 2”(Safir)运载火箭发射升空后,卫星已经被送入低地轨道。本次发射恰逄伊朗革命的30周年庆典,总统内贾德出席了发射仪式,并开启卫星传送信号“Allāhu Akbar”(真主至大)。他说发射这颗卫星是为了向全世界传播“一神论,和平和公正”。伊朗外交部长马努切赫尔·穆塔基说发射这颗卫星完全是为了“满足国家需要”,“纯粹是为了和平的目的”。希望号是伊朗的第二颗在轨卫星。2005年俄罗斯为伊朗制造并发射了一颗名为Sina-1的卫星。希望号卫星装载有卫星控制装置及动力供应系统,可用于收集情报和对设备进行检验。但很多功能还没全部激活。伊朗表示发射卫星是为了改进通信技术和互联网应用技术和监测自然灾害等用途,但是西方分析家则认为此举将等于为该国增加了发射洲际弹道导弹的能力。2008年2月4日,在新航天中心揭幕仪式上内贾德总统宣布,希望号将会在不久的将来发射升空。该卫星为了避免飞跃邻国采用的发射方向是东南方,越过印度洋,轨道倾角为55.5度。轨道高度据报道为246──377公里, 轨道周期90.76分钟。","text2":"伊朗自行研制的什么发射升空后,卫星已经被送入低地轨道?","label":1} {"text1":"显存全称显示记忆体,亦称帧缓存,它是用来存储显示芯片处理过或者即将读取的渲染数据。如同计算机的内存一样,显存是用来存储图形数据的硬件。在显示器上显示出的画面是由一个个的像素点构成的,而每个像素点都以4至64位的数据来控制它的亮度和色彩,这些点构成一帧的图形画面。为了保持画面流畅,要输出和要处理的多幅帧的像素数据必须通过显存来保存,达到缓冲效果,再交由显示芯片和中央处理器调配,最后把运算结果转化为图形输出到显示器上。从上公式可预知使用高分辨率和开启全屏抗锯齿、各向异性过滤、物理效果模拟、多边形顶点数据运算、海量三维函数运算等高级画面效果后,计算速度较慢的显示芯片需要更多的显存空间才能流畅显示。早期显卡的显存容量只具有512KB、1MB、2MB等极小容量时,显存容量成为最大分辨率的一个瓶颈;但目前主流显卡已经淘汰512MB的显存容量,主流中低端级显卡的显存容量是1GB或2GB,某些高端显卡甚至已经具有4GB或6GB的显存,在这样的情况下,显存容量早已经不再是影响最大分辨率的因素。现在显卡性能越来越强,这当然不能忽略显示芯片的发展速度快的事实,但显存的频率、位宽与带宽也制约着显卡的总体性能。高级的显存能让显示芯片全力工作,令输出的画面更流畅。显示芯片性能的日益提高,其数据处理能力变得更强,使得显存数据传输量和传输率的要求变得更高,显卡对显存的要求也更高,现时大部分显卡使用的是GDDR2至5代规格的显存。虽然各个类型的显存因厂家与规格而不同,而GDDR5规格的显存频率、位宽与带宽都普遍比其他规格高,少部分显卡使用更高频宽和位宽的HBM,HBM为现时最高价规格。","text2":"高级的显存能让显示芯片全力工作,会有什么好处?","label":1} {"text1":"威赛克斯白肩猪是家猪的一个品种,起源于英国的西郡(威赛克斯),特别是威尔特郡以及汉普郡的新福里斯特地区。威赛克斯白肩猪为黑色,身体的前部有一条白带。白带从一只前脚穿过肩部并延伸到另一只脚,形状类似马鞍。威赛克斯白肩猪传统上为树林放牧。这个品种是英国少数没有受到远东地区来的“那不勒斯”猪杂交影响的品种之一。威赛克斯白肩猪品种协会于1918年成立于英国。20世纪中叶以来,养猪业开始向集约化发展并影响到威赛克斯白肩猪的发展。同时,另一个相同颜色但稍有区别的品种Essex也面临着同样的问题。1967年,这两个品种为了防止灭绝而进行融合。此外产生了一个杂交品种(“威赛克斯白肩猪”也经常被用于称呼英国白肩猪)。很少的Essex猪在英国以纯种的形式保留了下来并有人试图恢复这个品种。但威赛克斯猪现在一般认为在原产国已经灭绝。但是有一些威赛克斯白肩猪在品种融合前已经被出口到世界上的其它地方,在澳大利亚、新西兰及其它地区仍有该品种存活。2008年,澳大利亚注册的种母猪不到100头并被列为濒危动物。2006年,威赛克斯白肩猪的胚胎和精液被进口到英国并重新组群。19世纪初,汉普夏地区一些相似的猪被出口到北美,并初步形成汉普夏猪。","text2":"威赛克斯白肩猪起源于哪里?","label":1} {"text1":"波兰是位于中欧的一个国家,地处德国以东。波兰地势平坦,国土大部都是平原。波兰波罗的海沿岸地区除格但斯克外,缺乏天然良港。位于西北部的什切青亦是波兰重要的港口。波兰东北部的湖区林木茂密,人口较少。在南部,喀尔巴阡山脉构成了波兰同捷克、斯洛伐克和乌克兰的国界。波兰国土的平均标高仅有173米。最高峰是位于波兰同斯洛伐克边境上塔特拉山脉中的雷瑟峰,标高2,499米。格但斯克湾沿岸约60平方公里的陆地在海平面以下。波兰最大的两条河流是维斯瓦河和奥得河。维斯瓦河的流域面积占去波兰一半的国土。波兰全国的年均降水量大约在600毫米左右。但一些山区中的观测站测得的年降水量可达1300毫米。总的来说,南部的山区地带降水量较北部平原为高。面积比较:略小于新墨西哥州陆地边界:\"总计:\" 2,888 km\"接壤邻国:\"白俄罗斯416 km,捷克790 km,德国467 km,立陶宛103 km,俄罗斯(加里宁格勒州)210 km,斯洛伐克541 km,乌克兰528 km海岸线:770 km\"最低点:\"Raczki Elblaskie -1,8 m\"最高点:\"雷瑟峰 2,499 m自然资源:煤、硫磺、铜、天然气、铁、锌、铅、盐、耕地","text2":"波兰是哪里的一个国家?","label":1} {"text1":"西里伯斯鹿豚(学名:)又名西里伯斯野猪、北方苏来威斯鹿豚或苏拉威西鹿豚,是印尼苏拉威西岛北部及邻近群岛的鹿豚。牠们有两对獠牙,颚骨的獠牙穿过鼻端顶,向后弯曲至前额。牠们是濒危物种。西里伯斯鹿豚起初被认为是巴比如萨猪鹿的亚种,但近期的研究显示牠们其实是独立的物种,在地理分布、体型、雄鹿豚的上犬齿形状也有不同。在新的分类下,真正的巴比如萨猪鹿只限于布鲁岛及苏拉群岛。西里伯斯鹿豚栖息在热带森林的草丛及藤丛、河岸及湖边。牠们大体都没有毛,呈灰褐色,可以提供了某程度的伪装。西里伯斯鹿豚有两对獠牙,是特大的上下犬齿,向后弯曲指向头部。雄鹿豚的上犬齿甚至穿过鼻端。西里伯斯鹿豚受到印尼法律的保护,但仍面对非法猎杀的威胁。饲养的西里伯斯鹿豚是近亲繁殖的。在布朗克斯动物园(Bronx Zoo)的西里伯斯鹿豚繁殖力很强,但并非很多动物园也有饲养牠们。","text2":"很多动物园都饲养西里伯斯鹿豚吗?","label":1} {"text1":"美国与土耳其之间的关系可追溯到1947年实行的、旨在加强土耳其经济及军事自立的杜鲁门主义。多年来,美土两国通过密切的双边外交关系以及作为北约盟国中的两支最大军队之间的伙伴关系,使这一目标从冷战时期一直发展至今。2003年,由于土耳其议会决定拒绝让美国为首的盟军把土耳其作为进入邻国伊拉克的基地,美土关系出现紧张,但是两国政府后来发起,为加强战略合作建立了框架。2009年,在奥巴马宣布将分期从伊拉克撤出作战部队后,土耳其表示愿意为美国军事设备提供过境便利。同时有媒体报道,土耳其有可能成为伊朗恢复与美国关系的斡旋国。2011年叙利亚爆发内战后,由于美国支援叙利亚库尔德族民主联盟党武装力量对抗伊斯兰国,引致美土关系恶化。土耳其政府认为民主联盟党属于库尔德工人党分支,视之为恐怖组织。2016年土耳其政变后,土耳其政府指控流亡美国的土耳其教士法图拉·居连是政变幕后黑手,要求美国把居连遣返土耳其,美国政府则要求土耳其提供实质证据,否则不会遣返居连。2017年3月初,美国派遣少量美军进驻叙利亚北部曼比季,阻止土耳其攻击当地的叙利亚民主力量部队。","text2":"美土关系第一次出现紧张是在什么情况下?","label":1} {"text1":"都督福满(),爱新觉罗氏,明朝建州左卫人。觉昌安的父亲,塔克世的祖父,努尔哈赤的曾祖父。1636年5月16日(清崇德元年四月乙亥朔十二日丙戌),皇太极追尊他为庆王。1648年12月18日(清顺治五年十一月辛酉朔五日乙丑),顺治帝追谥他为直皇帝,庙号兴祖。同时受追尊得还有努尔哈赤的高祖父清肇祖,也就是福满之父。但是据后来修订的《满州实录》记载,福满之父锡宝齐篇古是清肇祖都督孟特穆的孙子。根据当代学者的研究,福满和锡宝齐篇古可能是虚构的人物。福满有六子,德世库,刘阐,索长阿,觉昌安,包朗阿,宝实。都居住在赫图阿拉的觉罗地方,‘环卫而居,彼此相距近者5里,远者不过20里,是为六工,又称为宁古塔贝勒,亦通称努尔哈赤六祖’。皇太极天聪九年(1635),为区分直系与旁支的世系关系,皇太极诏令觉昌安四子塔克世的后裔称宗室子孙称为宗室,系黄带。觉昌安之外五祖子孙称觉罗,系红带,以别于宗室,后裔子孙载入觉罗族谱。","text2":"福满之父锡宝齐篇古是清肇祖都督孟特穆的孙子,是在哪里记载的?","label":1} {"text1":"「天神小子」(),是日本快乐快乐月刊从2007年1月号到2010年9月号连载的的作品。台湾快乐快乐亦从2007年10月号开始连载。是作者预告「能够超越『可乐小子』的作品」。主角伽利略以成为天神为目标,挑战高第天神学校的入学考试。人物名多是以历史上出名的人物为范本(夹注号内为受取材的历史人物)。在4班待遇最差,是低于标准的放牛班。可是由于大家被期待的标准大多相同,所以是天神学校中十分有趣的班级。G技:G铁拳,W.G铁拳、神之G铁拳D技:D铁拳GD技:GOD铁拳G技:女神的吐息,女神的喷嚏G技:岩窟王G技:光之翼G技:毕沙罗的墙壁G技:热焰制敌拳G技:神之手,凤凰箭雨G技:枭枭烟雾,突刺之烟,G.烟怒,G.烟怒陨石G技:荆棘暴龙、荆棘暴龙猛冲、荆棘暴龙大雷疱G技:人间大砲、人间大砲MAXG技:绝对指挥棒、雨中的镇魂曲、艾西莫夫最终乐章G技:极限巨影G技:弹簧式G技:乙女的束缚G技:分身术G技:G技:石油危机G技:动物变化G技:超速射线G技:LA.SHA-BORZ现任天神。天神学校的教师。1班的G技教师。G技:注意我无天神大人的宠物。D技:千斩鬼牌、恶魔之手恶魔王,杀死了伽力略的双亲,以笛福的身份在恶魔学校最强5人之中生活。D技:暗黑流星","text2":"「天神小子」是作者预告「能够超越什么的作品」?","label":1} {"text1":"微指令的编译方法是决定微指令格式的主要因素.考虑到速度,成本等原因,在设计计算机时采用不同的编译法 .因此微指令的格式大体分成两类:水平型微指令和垂直型微指令。一次能定义并执行多个并行操作微命令的微指令,叫做水平型微指令.水平型微指令的一般格式如下:控制字段 判别测试字段 下地址字段按照控制字段的编码方法不同,水平型微指令又分为三种:一种是全水平型(不译法)微指令,第二种是字段译码法水平型微指令,第三种是直接和译码相混合的水平型微指令。微指令中设置微操作码字段,采用微操作码编译法,由微操作码规定微指令的功能,称为垂直型微指令。垂直型微指令的结构类似于机器指令的结构.它有操作码,在一条微指令中只有l-2个微操作命令,每条微指令的功能简单,因此,实现一条机器指令的微程序要比水平型微指令编写的微程序长得多 .它是采用较长的微程序结构去换取较短的微指令结构。","text2":"微指令的格式大概能分成哪两类?","label":1} {"text1":"马沙沟位于台湾台南市将军区的长沙-{里}-、平沙-{里}-,将军溪出海口南侧。有长沙与平沙两村落,拥有两座渔港及一座滨海游憩区。因近年举办海滩派对而逐渐知名。旧南瀛八景之一「绿汕帆影」之所在。将军区长沙-{里}-、平沙-{里}-的名称中的「沙」字,即是来自于马沙沟。而将军区的马沙沟与顶山子脚、下山子脚、北埔四地因为同是吴姓开垦之地,故合称「四埔」,吴姓家族对外自称为「四埔吴」。根据清朝方志的记载,马沙沟与北门屿斜对,南隔青鲲鯓沙线六里,原本应为滨外沙洲,后来才与陆地连在一起。故过去有人认为马沙沟的地名与当地原为滨外沙洲有关。但学者黄文博认为,该地名是因为「马加、鲨鱼多在水浊的沙沟底始易捕获」而来,并加上开庄者陈马洲名字中的「马」字而来。而学者杨森富则认为马沙沟之名来自西拉雅语中MASAKAU之音译,「MA」是接头语,而「SAKAU」同「SAKAN」,为渔港之意,而「AU」则为地名语尾。台61线西滨快速公路将军交流道
","text2":"马沙沟为什么变得有名?","label":1} {"text1":"吞武里王朝(1769年 - 1782年)是泰国历史上的一个王朝。其建立者是中泰混血儿达信(中文名郑信)。1767年,缅甸侵略暹罗,灭大城王朝。郑信在东南沿海一带组织了一支军队,反抗缅甸的统治。不久,清缅战争爆发,缅甸在大城的大部分驻军被迫回国抵抗清军,郑信趁机收复了大城。然而,大城因战争残破不堪,郑信没有将都城定在那里,而是建立在了吞武里。郑信在位期间消灭了各地割据势力,又击退缅甸的再次进攻,收复了清迈等地。他还征服了兰纳、万象、瑯勃拉邦、占巴塞,并同越南的阮主争夺真腊(今柬埔寨)。此外,泰国的教育和宗教学习得到了推进,并积极同中国、英国与荷兰展开贸易活动。1782年,达信被通銮·却克里所弑,一说是达信在政变中被杀,却克里援救不及。却克里即位后,成立却克里王朝,自称拉玛一世,迁都拉达那哥欣岛,史称曼谷王朝,吞武里王朝亦因此结束其短短十三年的统治。","text2":"它的建立者是谁?","label":1} {"text1":"韩燕(),中国新疆人。学生时代在国内担任模特儿及车展女郎。后来就读海南大学油画系,并于当地参加过多项省内选美比赛,曾数度获奖。早期活跃于网路世界,以「雅漾韩」为网名,在国内网站上颇具知名度。2006年,韩燕当选海南博鳌形象小姐。同年参加由香港亚洲电视举办的亚洲小姐竞选,在中国内地赛事中以22号参选,夺得「最佳口才奖」、「最上镜小姐」与及赛区亚军。后来继续突围,顺利打入香港区总决赛,同届对手有陈瑀涵、舒夫真高、曾敏等。韩燕最终以4号佳丽的身份成功取得赛事季军及「亚洲亲善大使」的名衔,并签约成为亚洲电视合约艺员。其后,韩燕数次以泳装姿态为亚视主持综艺旅游节目,包括《碧水湾温泉养生之旅》、《开心清远温泉之旅》、《星级顺德自由行》、《番禺水上乐园斗多fun》、《阳光热浪嘉年华》等,均以其出众身材成为新闻焦点。2007年,韩燕参选北京奥运火炬手。经北京奥组委审核,于10月24日正式当选为2008年北京奥运国内火炬手之一,负责在5月4日至5月6日于海南三亚传递圣火,该站亦是北京奥运中国境内圣火传递路线的第一站。2010年韩燕回归亚洲电视。","text2":"韩燕主持过哪些节目?","label":1} {"text1":"Mac OS X v10.1「Puma」(「Puma」字面解作「美洲狮」)是苹果为 Mac 产品所制作的作业系统Mac OS X的第二个版本。这个操作系统最初于2001年9月25日发布,增加了系统的效能和新的功能,像是 DVD 播放。由于 v10.0 的口碑不好,苹果为 v10.0 的使用者推出免费的升级 CD,此为对于只有运行 Mac OS 9 的人推出 $129 美元的盒装版本。有人发现升级的 CD 其实就是完整的安装光碟,只要移除一个特定的档案就能够在 Mac OS 9 系统使用;苹果随后重新推出该 CD,实际上真的是把不必要的缩减掉,使得无法在这样的系统上安装。苹果声称Mac OS v10.1 Puma 有多种新特性,包括:","text2":"什么是Mac OS X v10.1「Puma」?","label":1} {"text1":"雨果大街(Avenue Victor-Hugo)是法国巴黎十六区的一条街道,开始于戴高乐广场,终止于亨利马丁大街。这是通往凯旋门的12条大街之一,长度仅次于最长的香榭丽舍大街。其位置介于福煦大街和克勒贝尔大街之间。雨果大街穿过非常别致的十六区的北部,长约2千米,是巴黎最有声望的街道之一。其确切的长度是1825米,宽36米(第一段,戴高乐广场与雨果广场之间,从雨果广场到亨利马丁大街稍窄)。这条林荫大道,在亨利马丁大街路口装饰着维克多·雨果雕像。它跨越了雨果广场。这条大街曾经称为Eylau大街和圣克卢大街(avenue de Saint-Cloud)。1881年2月28日,在法国诗人维克多·雨果79岁诞辰的第三天,这条大街命名为雨果大街。大街中段有地铁2号线的维克多·雨果站。雨果大街124号:维克多·雨果晚年的住处,1907年加高。","text2":"雨果大街的确切长度有多长?","label":1} {"text1":"南挪威(书面挪威语、新挪威语:Sørlandet),挪威地理上的南部,斯卡格拉克海峡沿岸的两个郡,包括东阿格德尔()与西阿格德尔(),总面积达,依2008年7月所公告统计结果,人口共有273,818人。南挪威大致上是历史上阿格德尔()王国的领地,1902年,由当地作家威尔海姆·克拉格()提出南挪威这个名称与观念,在这之前,这个区域被视为是西挪威的一部份。有时候,挪威人会把邻近罗加兰与泰勒马克两个郡的部份地方,视为南挪威的一部份。依文化地理学的观点,南挪威可被定义为斯卡格拉克海岸带,与北方山谷有所区别。南挪威内八成人口分布在气候温和的沿岸地区,接近渔获丰富的渔场,而且是斯卡格拉克海峡海上航线所经之处,著名的城镇从西到东有:","text2":"在南挪威这个名称提出之前此地归属于哪里?","label":1} {"text1":"美国营运管理协会(,缩写为 ),是一个非营利性质的国际教育机构,其提供专业证照课程、训练工具及网络联系机会来提升专业人士职场的绩效表现。APICS最早成立于1957年名为美国生产和存货控制协会。目前在全球已有超过43,000的个人会员及10,000间以上的企业会员。2008年4月,此机构的全球总部移至美国伊利诺州的芝加哥。APICS对“营运管理”的定义是“一个研究的领域,主要针对一个以制造或服务的机构如何做有效的规划﹑排程﹑使用和控制,研究的范围包括影响作业的各个功能,例如设计工程﹑工业工程﹑管理资讯系统(MIS)﹑品质管理﹑生产管理﹑存货管理﹑会计及其他。”APICS提供两项专业的认证:APICS CPIM(鉴定产业管理师)及APICS CSCP(国际供应链管理师)。APICS CFPIM(鉴定产业杰出管理师)是授给在营运管理专业令人推崇的领导,全球已有超过1,200位专业人士获颁此项证照。CIRM(Certified in Integrated Resource Management,鉴定资源整合管理师认证):根据美国营运管理协会(APICS)的资料,目前全世界已有数千人取得CIRM的认证。APICS认定此认证在全球企业的活动与过程中为高阶的专业知识。然而,APICS已于2008年停止了这认证计划。对于拥有此认证的专业人士,APICS将继续承认此认证。同时,APICS在2008年发行了APICS营运管理相关知识体系架构(OMBOK)。OMBOK架构定义专业的范围及提供有效的知识给营运管理专业人士。","text2":"美国营运管理协会现有多少会员?","label":1} {"text1":"南艾蒿(学名:)为菊科蒿属的植物。分布于尼泊尔、朝鲜、柬埔寨、印度尼西亚、亚热带、温带、老挝、欧洲、小亚细亚、印度、热带、台湾岛、马来西亚、斯里兰卡、日本、泰国、越南、非洲以及中国大陆的河南、广西、辽宁、广东、云南、陕西、福建、浙江、湖南、湖北、安徽、黑龙江、甘肃、内蒙古、江苏、江西、贵州、山东、山西、四川、吉林、河北等地,多年生草本,植株有香气。主根稍明显,侧根多;根状茎短,常具匍匐茎,并有营养枝。茎单生或少数,高50—100厘米。生长于海拔1,000米的地区,多生长在路旁、低海拔至中海拔地区的山坡以及田边等地,目前尚未由人工引种栽培。白蒿、大青蒿、苦蒿、紫蒿(四川),红陈艾(广西),刘寄奴(贵州)","text2":"南艾蒿的根有什么特点?","label":1} {"text1":"昃臣道()是香港香港岛中西区的一条街道,位于中环香港会及遮打花园以西,旧最高法院大楼及皇后像广场以东,爱丁堡广场及干诺道中以南,德辅道中以北。在历史上,昃臣道是填海所得的地皮,平坦路是一条行车马路,两旁是行人路。昃臣道的南段是露天停车场,有权泊车的车牌号码是AM、LC等字头的香港政府及立法会议员车辆。同时该处也经常是指定示威区,见报率不少。昃臣道的名字来自香港19世纪末著名银行家及议员汤玛士·昃臣爵士。2008年夏季奥林匹克运动会香港区火炬接力曾经途经此道,火炬手是范徐丽泰。香港电影《英雄本色》第二场,周润发在香港会所大厦门前进食街头小食猪肠粉,就是在昃臣道拍外景。","text2":"昃臣道的地理位置在哪里?","label":1} {"text1":"'(;英语简称为')是在日本、香港、台湾、韩国等地举办的同人志即卖会,由日本SE株式会社(,今Deleter株式会社)开办,并在日本以外地区委托当地同人团体举办。主要在中小型都市举办。但在2008年8月3日举行的「Comic World in 横滨 49」后,现已全面停办。Comic World 香港(,简称CWHK),是香港最早举办的大型同人志即卖会,每年举行两次。台湾同人志贩售会(,简称为CWT),由台湾同人志科技股份有限公司主办,每半年举行一次,分为CWT(台大场)、CWT-T(台中场)与CWT-K(高雄场),现今多以台北市台大综合体育馆、台中市逢甲大学体育馆、高雄市社教馆综合体育馆为举办场地。其观展族群多以女性为主,参与社团的作品也多为女性向。另外,也在2010年10月17日于香港举办一届「Comic World Taiwan - Hong Kong」(简称 CWT-HK),与CWHK不同。竞争对手为开拓动漫祭。Comic World从1999年开始,在首尔与釜山巡回举办。每回活动均以「(○回)(都市名)Comic World」命名。","text2":"香港最早举办的大型同人志即卖会是什么?","label":1} {"text1":"B2,香港二人女子演唱团体,成员分别是谭嘉荃(Amy)及郭可颖(Sandy,郭可盈堂妹)。B2参加新城电台举办的「一级班少女歌唱大募集」分别获得亚军及季军,获得BMA唱片公司合约。2000年正式出道,她们的曲风以快歌为主。于2000年10月推出首张EP《B2 Attack》。她们于2000年「新城劲爆颁奖礼」获得最受欢迎组合的第三名。及后于2001年推出了《B2同名专辑》。她们的主打歌《型男索女》的歌词被认为粗俗,而被各大电台及电视台禁播。2002年1月,她们推出了首张大碟《娥世代》。及后她们于同年的6月在台湾推出了《娥世代》的台湾版。2003年,宣布退出乐坛。Amy于无线电视担任兼职,及为节目《娱乐大搜查》担任记者及写稿工作,Sandy则到外国进修。2006年,Amy离开了无线电视,转投「香港宽频电视」任主持。直到2012年,Amy加盟汉洋主理的自由行娱乐,正式重返乐坛。","text2":"B2是何时正式出道的?","label":1} {"text1":"地热谷是一处硫气及温泉的出口,位在台湾北投中山路的阳明山山谷洼地,当地人又称「磺水头」或「鬼湖」 ,因长年蒸气弥漫、热气腾腾,使人联想成恐怖的地狱。早年亦曾有游客失足跌入高温泉水中,被活活烫死,因而另有别名「地狱谷」。在日治时期,地热谷被评为台湾八景十二胜之一,有「磺泉玉雾」美誉。原因是日出晨光照射在温泉冒起的蒸气,幻化出变化多端的光影,如身处如梦似幻的仙境当中。泉水清澈微绿似玉,所以有人称为「玉泉谷」。地热谷的泉水属酸性硫磺泉,水温高达摄氏90度,水质清澈且呈微绿似玉的色泽,俗称青磺泉。水中酸碱度达PH值1.4~1.6,腐蚀性极高,因此不宜久泡。虽然如此,地热谷温泉亦为北投温泉的源头之一,原因是泉中含微量放射性元素镭,对人体有保健的功效,主要是供应为新北投公园一带的温泉浴室或旅馆,包括历史悠久的泷乃汤。早期当地民众常携带鸡蛋到地热谷煮「温泉蛋」,后来发生烫伤及环境保护考量封闭。维护管理的台北自来水事业处也表示地热谷温泉属于硫酸盐泉、硫酸盐氯化物泉,含有重金属,有危害人体健康之虞,不建议引水煮蛋。现在政府重整地热谷景观,导引泉流,蓄水成湖,并以步道、扶栏环绕四周,步道一旁设有赏景凉亭,周边还有以石子与水泥筑成亲水沟渠,游客可免费将双脚泡在温泉中,市政府已将新北投公园连同地热谷规划为「北投温泉亲水公园」。","text2":"地热谷被当地人称为什么?","label":1} {"text1":"太平洋群岛托管地(),是联合国委托美国于西太平洋密克罗尼西亚群岛的一个托管地带,美国对该地区实际控制,直至1994年10月帛琉宣布独立为止。此区原属于大日本帝国在1920年接受国际联盟之托,所设置的南洋厅。1944年太平洋战争期间被美军攻占。战后联合国交由美国治理。1986年10月21日,美国结束对马绍尔群岛的管理,而马里亚纳群岛及加罗林群岛的雅蒲、楚克、波纳佩和科斯雷四个区域(已同意组成密克罗尼西亚联邦)也在同年11月3日终止。联合国则于1990年12月22日正式终止以上地区的托管。而帛琉则于1994年5月24日结束,并于10月1日与美方达成协议,正式宣布独立。目前这个地带可分为以下四个领域:","text2":"哪个国家实际管理太平洋群岛托管地?","label":1} {"text1":"台西镇 (),中国青岛市旧区划名,位于胶州湾东岸现市南区台西半岛中部,铁路青岛站以西。台西镇是德国胶州湾租借地要塞防线内界规划建设4个区域之一,也是2个华人劳工居住区之一。德租时期的台西镇的范围大致是台西一路、磁山路、台西四路和贵州路合围成的四边形区域。关于“台东”“台西”之名的来源,习惯性说法是:贮水山因明代建有烽火台而曾被称为“烽台岭(亦称风台岭)”,而台东、台西因地处烽台岭之东西两侧而得名。然而根据德租时期的手绘中文地图来看,“风台岭”被标注在今青岛山(“炮台山”)与贮水山之间的一个山丘上,而该山丘(根据德文地图高度在75-80米之间)的位置在青岛山北侧,胶宁高架、广饶路、登州路与青岛山山体的合围区域内,而非贮水山。至于“以烽台岭为界”“山上有明代烽火台”的说法,至今未找到可靠记载。清帝国时代该地区原有一村庄,名为“小泥洼”,德租初期音译作,后又转译为“小梅坞”。1901年,德属胶澳总督府高价收购了原村庄,开始建设台西镇,后逐渐形成了中国人居住的市镇。1910年,德属胶澳总督府设置台西镇为德属胶澳保护领青岛市的四区之一。1929年9月,中华民国青岛市政府撤销台西镇,成立第一区。1935年5月,青岛市政府重划市乡区域并改定名称,撤销第一区,成立台西区。","text2":"1935年,政府对台西镇做了什么改变?","label":1} {"text1":"穆罕默德·乌尔德·阿卜杜勒-阿齐兹( \"Mohamed Ould Abdel Aziz\";),毛里塔尼亚现任总统。1956年,生于因希里省首府阿克茹特。2005年8月参与政变,废黜毛里塔尼亚总统马维亚·乌尔德·西德·艾哈迈德·塔亚。2008年8月,发动政变,推翻总统西迪·乌尔德·谢赫·阿卜杜拉希和总理叶海亚·乌尔德·艾哈迈德·瓦格夫。此后,阿卜杜勒·阿齐兹成为国务委员会主席,行使国家元首的权力。阿齐兹1977年加入毛里塔尼亚军队,并在摩洛哥的梅克内斯皇家军事学院学习,毕业以后在毛军队获得迅速上升。他创建了精锐的总统卫队,并且平息了2003年6月的未遂政变,以及2004年8月的军事叛乱,因此获得了国家最高军事奖。2005年8月,国家安全局长埃利·乌尔德·穆罕默德·瓦勒等人发动政变,成立“争取公正与民主军事委员会”接管政权,作为总统卫队指挥官的阿齐兹是政变的参与者和执行者。2007年3月25日举行大选,独立候选人阿卜杜拉希当选总统。2008年8月6日,以总统府特别参谋长阿齐兹为首的军人,发动政变,扣押阿卜杜拉希总统,成立“国务委员会”,接管政权。2009年4月15日辞去「国务委员会主席」职务,竞选总统。参议院院长巴·马马杜(Ba Mamadou Mbaré)将履行临时国家元首职责,负责处理过渡时期日常国务。2009年7月18日,穆罕默德·乌尔德·阿卜杜勒-阿齐兹总统选举中击败前国家元首瓦勒、民主力量联盟主席达达赫和现任国民议会议长布勒海尔等9位候选人,获得52.58%有效选票,并于8月5日在努瓦克肖特奥林匹克体育场宣誓就职。2014年6月,在反对派强烈抵制选举下,阿齐兹以82%得票率再度连任茅利塔尼亚总统。","text2":"穆罕默德·乌尔德·阿卜杜勒-阿齐兹生于哪年?","label":1} {"text1":"米高(\"迈克\")·安东尼奥·鲍威尔(,)是一位美国的田径运动员。目前为男子跳远(8米95)的世界纪录保持者。鲍威尔出生于宾夕法尼亚州的费城。在1991年世界田径锦标赛上,鲍威尔以5cm的优势打破了由鲍勃·比蒙保持23年之久的男子跳远世界纪录,成绩为8.95m。目前,该项纪录仍在保持,使得鲍威尔成为1900年以来保持世界纪录超过20年的第四个人。鲍威尔在1988年汉城奥运会和1992年巴塞罗那奥运会上均获得银牌,在1993年世界田径锦标赛和1995年世界田径锦标赛中分别获得金牌和铜牌。鲍威尔于1996年亚特兰大奥运会后退役,之后于2001年复出,并希望能够参加2004年雅典奥运会,但是没有如愿。","text2":"鲍威尔在1988年汉城奥运会上成绩如何?","label":1} {"text1":"巴塞罗那-埃尔普拉特机场(,;),多简称为埃尔普拉特机场(),是西班牙第二大城市巴塞罗那的联外国际机场,乃西班牙仅次于马德里-巴拉哈斯机场的第二大机场,也是加泰隆尼亚最大的机场,由()经营,以提供西班牙国内,以及欧洲、北非的国际航线为主。埃尔普拉特机场位于巴塞罗那市区西南方,开航于1918年,是巴塞隆纳第二座启用的机场。巴塞隆纳至马德里间的穿梭航班(Air shuttle)服务,被称为「天桥」(、),在马德里-巴塞隆纳高速铁路于2008年开通前,是欧洲最繁忙的航线。在2010年,超过292万人次使用巴塞隆纳机场,较2009年成长6.5%,使其跻身欧洲第十大繁忙机场。","text2":"在2010年,超过多少万人次使用巴塞隆纳机场?","label":1} {"text1":"西里伯斯鹿豚(学名:)又名西里伯斯野猪、北方苏来威斯鹿豚或苏拉威西鹿豚,是印尼苏拉威西岛北部及邻近群岛的鹿豚。牠们有两对獠牙,颚骨的獠牙穿过鼻端顶,向后弯曲至前额。牠们是濒危物种。西里伯斯鹿豚起初被认为是巴比如萨猪鹿的亚种,但近期的研究显示牠们其实是独立的物种,在地理分布、体型、雄鹿豚的上犬齿形状也有不同。在新的分类下,真正的巴比如萨猪鹿只限于布鲁岛及苏拉群岛。西里伯斯鹿豚栖息在热带森林的草丛及藤丛、河岸及湖边。牠们大体都没有毛,呈灰褐色,可以提供了某程度的伪装。西里伯斯鹿豚有两对獠牙,是特大的上下犬齿,向后弯曲指向头部。雄鹿豚的上犬齿甚至穿过鼻端。西里伯斯鹿豚受到印尼法律的保护,但仍面对非法猎杀的威胁。饲养的西里伯斯鹿豚是近亲繁殖的。在布朗克斯动物园(Bronx Zoo)的西里伯斯鹿豚繁殖力很强,但并非很多动物园也有饲养牠们。","text2":"西里伯斯鹿豚栖息在哪里?","label":1} {"text1":"铁路节是中华民国为纪念铁路事业发展而订的节日,日期订于6月9日,系因中国第一条自建铁路唐胥铁路特地订于「铁路之父」乔治·史蒂芬生的生日1781年6月9日开工。铁路传入中国是始于清同治四年(1865年)英国商人在北平宣武门外筑一小铁路约长里许,不久拆毁。光绪二年(1876年)英国怡和洋行筑淞沪铁路,地方人士反对,由政府出资拆毁。光绪五年(1879年),开平煤矿在唐山至胥各庄筑运煤铁路(即唐胥铁路),于同年6月9日开工,历时两年通车,此为中国自建铁路之始。民国54年(1965年),应铁路界人士要求,中华民国内政部与交通部核定以唐胥铁路开工日为铁路节,自此延续迄今。虽然6月9日为中国大陆铁路事业起始之日,与同样起始于清末的台湾铁路事业毫无关系,但仍有部分资料讹传该日为清代台湾铁路开工之日,造成外界对于6月9日即为台湾铁路管理局局庆的误解。","text2":"铁路什么时候传进中国?","label":1} {"text1":"伯奇还原反应(Birch还原)是指用钠和醇在液氨中将芳香环还原成1,4-环己二烯的有机还原反应。此反应最早由澳大利亚化学家Arthur John Birch (1915–1995)在1944年发表。 Birch还原的重要性在于:尽管剩下的双键(非芳香性)更为活泼,该反应却能停留在环己双烯上,而不继续还原。反应中的钠也可以用锂或钾取代,使用的醇通常是甲醇或叔丁醇。使用Birch还原的一个例子是还原萘:其他人也发表了很多篇关于此反应的综述。钠溶于液氨中会形成一个电子盐的亮蓝色溶液,化学式为[Na(NH)] e。溶剂化电子会与芳香环加成,形成一个自由基负离子。溶液中的醇此时作为质子试剂提供一个氢原子。对大多数反应物来说,氨上的氢酸性还不够。如果是取代芳香化合物,当取代基是羧基等吸电子基时,能够稳定碳负离子并生成最少取代的烯烃; 当取代基是供电子基时,则生成取代最多的烯烃。 热力学不稳定的非共轭1,4-加成产物往往产率超过热力学稳定的1,3-加成产物,这是由于共轭的戊二烯负离子中间体HOMO的最大轨道系数是在中间那个碳原子上,导致生成的1,4-环己双烯没有办法经过平衡移动而生成更加热力学稳定的产物,因此,生成的是动力学稳定产物。在卤代烃的存在下,上文提到的碳负离子也可以发生亲核取代反应生成新的碳-碳键。如下图所示,在Birch还原中生成的负离子中间体可以被一个合适的亲电试剂捕获,例如卤代烃:根据逆合成分析,前者即是后者的合成子。在下图所示反应中,1,4-二溴丁烷被加入到苯甲酸叔丁酯中,最后生成烷基化的1,4-环己双烯产物。","text2":"什么是伯奇还原反应?","label":1} {"text1":"莫里斯·欧内斯特·吉布( ,),英国歌手、词曲作者、多乐器演奏家及唱片监制,出生于英国皇家属地曼岛的首府道格拉斯,双亲都是英国人。他吉布斯家族五姐弟排行第四(在1969年,因为罗宾跟巴里和经理人吵架而单飞,姐姐莱斯利成为代唱),是双胞胎哥哥罗宾的弟弟。他与罗宾和大哥巴里·吉布在澳洲组成的合唱团比吉斯闻名世界,是20世纪摇滚乐和迪斯科高峰时间的代表乐团,开创了多种新颖的演唱方式,影响十分深远,成为史上最成功的其中一支乐队。莫里斯在乐队中专司乐器演奏和作曲。2003年初,莫里斯因为肠扭曲病发而在佛罗里达州的迈阿密海滩接受手术,但是手术后状况不见好转,发生并发症,莫里斯不幸于2003年1月12日逝世。他两个哥哥因此十分悲痛,并且表示因为莫里斯的离去,以后他们也不会再以Bee Gees的名义演出。2012年,当罗宾离世后,他的女儿Samantha 成为巴里个人演唱会嘉宾。","text2":"莫里斯在乐队中扮演什么角色?","label":1} {"text1":"六原车站()是一由东日本旅客铁道(JR东日本)与日本货物铁道(JR货物)所共用的铁路车站,位于日本岩手县胆泽郡金崎町()大字三尻()字丹藏堰。六原是JR东日本东北本线沿线的一个小车站,属于JR东日本盛冈支社的管辖范围内,是个由北上车站管理,委托由JR东日本子公司Jaster()代为经营的业务委托车站。除了客运业务外,JR货物在六原车站站区内也设货柜装卸场,用以装卸由专用线或车携货运()运来的货柜。其中专用线的部分,六原车站内有一条通往北上高科技制纸(,原三菱制纸北上工厂)、长约0.8公里的专用线,主要是用以将纸制品运往外地,或自酒田港车站将液态氯运往造纸厂。至于在六原车站停靠的货物列车,则包括有自陆前山王发车、驶往八户货运车站的临时专用货物列车,会在六原停车进行车厢的解连。侧式月台1面1线与岛式月台1面2线,合计2面3线的地面车站。※2号月台截至2014年1月为止没有定期旅客列车使用。","text2":"除了客运业务外,六原车站还有什么业务?","label":1} {"text1":"是香港其中一个公共屋邨,位于东区柴湾,分为兴华(一)邨和兴华(二)邨两部份。前身为徙置区,名为兴华徙置区,俗称「兴华新区」,于1971年落成。在1974年,房委会将兴华(一)邨所有楼宇重新命名,1994年拆卸。重建后的项目编号为HK16,全邨有3幢38层高的和谐一型大厦,名为美华楼、卓华楼及兴翠楼(前称雍华楼及兴翠苑)及1幢8层高停车场大楼,其中美华楼基座则为2层高商场「兴华广场」,并设有天桥及通道连接港铁柴湾站。房委会透过「自选单位计划」把美华楼及卓华楼单位编配给受柴湾邨13座及北角邨重建影响的租户,1999年入伙,而原本房委会于1998年打算把雍华楼改为居屋出售,名为兴翠苑,而邨内的楼宇外墙名牌及指示牌亦已更改作兴翠苑之际,政府突然宣布停售居屋政策,房委会遂决定将兴翠苑改回作公屋用途,房委会把所有楼宇名牌的「苑」字拆除,并改为「楼」字,并透过「自选单位计划」把兴翠楼全幢单位编配给受柴湾邨14及15座重建及北角邨重建影响的租户,让他们自行拣选合适的居住单位,期间空置了一年,最后推迟至2000年才开始入伙。兴华(二)邨于1976年落成,由前工务局设计,由前徙置事务处及香港房屋委员会按前徙置事务处规格及标准兴建,所以富有前徙置事务处屋邨的建筑设计特色。当年2月1日由于爱秩序湾艇户大火,有接近3,000人受灾。政府为了尽早安置灾民,裕兴楼及丰兴楼均提早六周落成。有5幢建于山上,另外2座大厦(裕兴楼及丰兴楼)则依山而建,设有大型升降机、架空走廊及天台通道,分别连接环翠道及柴湾道,方便居民出入,建筑设计上与同期的葵盛西邨十分相似。兴华广场(前称兴华商场)的提升工程于2009年4月开展,并于2010年第一季完成。除了鲜活街市外,亦有面包店、水果店、超级市场、快餐店等。其后街市被改建为商舖,而超级市场亦搬到前街市位置。","text2":"为什么房委会将兴翠苑改回作公屋用途?","label":1} {"text1":"性倾向和兵役在世界各国军事中有著多种的政策态度因应面对。多数的西方国家军队废除了排斥同志从军的政策,参与北约组织的26个成员国家中,超过20个国家允许男同性恋、女同性恋和双性恋在军中服务。以色列是中东地区唯一允许男同性恋与女同性恋公开在军中服务的国家。国际间各国对于军中同性恋人事的政策和态度显得相当多样。一些国家确保男同性恋与女同性恋能公开身份在军中服务,并给予他们同等权利及保障他们的同性恋基本权利。多数国家则是对此无特别禁止但也没有平权措施,,只有少数国家继续完全禁止任何同性恋者在军中服役。而像美国这样军中特定性倾向曾被高度政治化的国家,不必然是多数国家的情况。总得来说,性取向在这些文化更被当成一个人身份认同的一部分。","text2":"参与北约组织的国家中有多少成员国废除了排斥同志从军的政策?","label":1} {"text1":"吕奇()生于广东台山市,原名汤觉民。香港粤语片演员、编剧、导演。1949年来香港,是圣保罗书院校友。吕是香港电影颇具影响力的人物。与影迷公主陈宝珠被称为银幕情侣。他是香港女歌手吕珊的世叔伯。配偶为曾经在他电影中演出的女演员凌黛,育有两儿。吕奇1958年考入中联演员训练班,翌年加入邵氏粤语片组,第一部主演电影为《恋爱与贞操》。1959年7月与参加邵氏粤语片组。首部主演作品《公子多情》轰动一时。陈宝珠更是他银幕上的最佳拍档。1965年,吕奇与邵氏电影的合约到期后自组廿一世纪公司,初时兼任编剧,后来掌执导之职。吕奇自编自演电影《情人的眼泪》取得巨大成功。《蔓莉蔓莉我爱你》为他自编自导自演的第一部电影。他执导的电影约三十部,较著名的有《娘惹之恋》、《偷心贼》等。1974年自组金禾公司,为邵氏包拍影片,专门制作情色电影。 多部经典艳情片,如《丹麦娇娃》、《财子、名花、星妈》等皆出自其公司。他的公司在八十年代连续出品的多部色情片在香港三级片发展史中可谓举足轻重。其电影多以色情片格局的讽剌社会,并取得票房成功,其中《财子、名花、星妈》更是香港首出“露毛”色情片。1987年完成《命带桃花》后,吕奇结束他在香港的事业转向台湾经商。副导演(1部)导演(29部)编剧(32部)执行监制(1部)演员(96部)吕奇导演的粤语片,喜欢分析社会问题,例如《色情与道德》,主题上强调「女性的堕落」不是因为班女仔道德上有问题,实情是被社会逼害,例如父母贪钱、给黑社会操控、男友又骗财骗色。吕奇某个时期的电影,给学者分类为港产色情片,这批电影对白上是清教徒的说教,影像上出现的是青春玉女的胴体。吕奇退休之后,和太太移民到台湾,专心经营股票生意,加上物业收租,生活惬意。曾有传闻指,吕奇目前在台湾当的士司机。但他本人在2003年接受新城电台长途电话访问时否认此事。","text2":"吕奇第一部主演的电影是什么?","label":1} {"text1":"李煦,满洲正白旗人,祖籍山东莱州府。清康熙、雍正年间政治人物。父李士桢,本姓姜,崇祯十五年(1642年)被清兵掳去,过继给满洲正白旗佐领李西泉为子,遂改姓李。李家为内务府的包衣,接近皇室,顺治四年,八旗抡才,士桢以第十六名中选,成为清皇室的近臣。获得康熙帝赏识。李煦历官内阁中书、韶州知府、宁波知府、畅春园总管。康熙三十二年(1693年)出任苏州织造。李煦与妹夫曹寅(任江宁织造三十年),不断向康熙帝呈递密折,奏报江南地方民情。八次兼任巡视两淮盐课监察御史,千方百计搜集各方面的情报。李煦为人宽怀仁慈,颇得当地的民心,有「李佛」称号,但是“公子性奢华,好串戏,延名师,以教习梨园”,康熙四次南巡,李煦、曹寅等大事铺张,导致亏空高达数十万两。雍正二年,因亏空公家白银三十八万两,被抄家籍产。雍正五年(1727年)二月又因曾为雍正政敌胤禩等诸皇子(即阿其那、塞思黑)买过苏州女子,定为“奸党”,发配往吉林(打牲)乌拉(黑龙江布特哈旗)苦寒之地,“两年来仅与佣工二人相依为命,敝衣破帽,恒终日不得食”,最后因冻饿死于当地。临死之时,“囊中无一钱……亲识无一人在侧。”。有二子,李鼎、李鼐。","text2":"为什么会被抄家?","label":1} {"text1":"美国冰球联盟(英语:,缩写:AHL)是北美的一个职业冰球联盟。全部AHL的29支队伍都和NHL的30支球队有着密切的合作。 AHL既为NHL联盟的小联盟, 有着相当高的比赛水平, 比赛激烈程度并不亚于NHL联赛。联盟办事处设在斯普林菲尔德,马萨诸塞州。现任总裁是大卫安德鲁斯。每年季后赛冠军授予以NHL第一任总裁弗兰克 考尔德( 1917年至1943年)命名的考尔德杯。AHL赛季分为常规赛和季后赛两个阶段。常规赛季从每年10月开始到下一年度4月中旬。每个队有80场常规赛,40个主场,40个客场。每年的1月中旬举行AHL全明星赛。常规赛结束后两个联盟成绩前8名进入七战四胜制季后赛,争夺联盟冠军。最后两个联盟冠军夺考尔德杯。AHL的队伍发展到现在有30支队伍分布在美国和加拿大。分为东部联盟和西部联盟。东部联盟又分为大西洋区(七支球队)东部分区(七支球队)。西部联盟又分为北部分区(八支球队),西部分区(八支球队)。注解:深色队名为本赛季参赛球队","text2":"该联盟的办事处在什么地方?","label":1} {"text1":"龙舌兰酒效应原意指1994年墨西哥所爆发的金融危机。1990年代开始,金融市场迈向全球化趋势之下,当某一地区或国家产生严重金融震荡甚至是危机时,会对临近地区而至于全球造成影响,媒体为增加报导吸引力,于是会从此地区或国家取一代表物名并直接冠于其上。而当1994年墨西哥爆发严重的金融危机时,媒体便用当地著名的代表物-龙舌兰酒(Tequila)-冠上名号,从此金融学家与媒体便普遍的将此次金融危机称做「龙舌兰酒效应」(Tequila Effect)。造成此次墨西哥金融危机的原因:一般金融界将「龙舌兰酒效应」指为金融债券不稳甚至大量倒债造成开发中国家整体市场的大波动。之后部分媒体引申将「龙舌兰酒效应」扩大为:遭到与墨西哥相关的事物(国家、比赛队伍……)所击溃,如同被龙舌兰酒狂灌而失去应有的水准,","text2":"1994年墨西哥发生了什么?","label":1} {"text1":"云端储存是一种网路线上储存()的模式,即把资料存放在通常由第三方代管的多台虚拟伺服器,而非专属的伺服器上。代管(hosting)公司营运大型的资料中心,需要资料储存代管的人,则透过向其购买或租赁储存空间的方式,来满足资料储存的需求。资料中心营运商根据客户的需求,在后端准备储存虚拟化的资源,并将其以储存资源池(storage pool)的方式提供,客户便可自行使用此储存资源池来存放档案或物件。实际上,这些资源可能被分布在众多的-{zh-hans:服务器主机;zh-hant:伺服主机;}-上。云端储存这项服务乃透过Web服务应用程式介面(API), 或是透过Web化的使用者介面来存取。云端储存闸道可以视需求安装在客户端,好让云端储存的服务在使用上,就像是在本地端储存一样。云端储存闸道是透过以网路设备或伺服器,将云端储存的 SOAP 或 REST 应用程式介面 (API) 转换成基于区块 (block-based) 的储存协定,如 iSCSI 与光纤通道 (Fibre Channel),或是基于档案 (file-based) 的网路储存协定,如 NFS 或 CIFS (SMB)。根据 2010 年所做的一项研究调查,公用的云端储存服务正在迅速成为对企业更具吸引力的选择。在 2010 年,包括 Nirvanix, Nasuni, CTERA Networks 以及 Petaera 等服务提供商,都开始使用新一代物件导向的储存技术,让超大量资料的储存成本比起传统的企业储存系统更为大幅缩减。这些服务提供商提供云端储存闸道给企业客户,协助将传统的档案储存协定转换成为快取式物件导向储存。于此同时,由欧盟赞助的「远见云端计划」(VISION Cloud EU Project) 也建立了以云为主的底层基础架构,利用开放规格和新技术,提供可扩展、高弹性、且可靠的架构,强化大量资料的储存服务。IDC 储存分析师 Brad Nisbet 指出,这种「成本效益高、使用方式友善」解决方案服务的提供,让市场中有越来越多的中小企业乐于采用云端储存。","text2":"资料中心营运商根据客户的需求在后端准备了什么?","label":1} {"text1":"墨西哥土拨鼠(\"Cynomys mexicanus\"),又名墨西哥草原松鼠或墨西哥草原犬鼠,是原住于墨西哥的一种啮齿目。牠们是日间活动的。由于牠们被看为害虫,估其数量下降至濒危水平。墨西哥土拨鼠栖息在海拔1600-2200米的平原上。牠们分布在墨西哥的圣路易斯波托西州北部及科阿韦拉州。牠们主要吃草,并透过食物来吸收水份。牠们有时也会吃昆虫。牠们的天敌有郊狼、短尾猫、鹰、獾及鼬。墨西哥土拨鼠是会冬眠的,其繁殖季节也较短,一般只在1月至4月。妊娠期为1个月,雌鼠每年只会产一胎,一胎平均有四子。幼鼠出生时眼睛闭合,会先以尾巴作为辅助,直至出生后40日才能看见。于5月至6月会断奶,到了1岁就会离开巢穴。冬天前幼鼠就会离开母鼠。幼鼠之间会互咬、嘶叫及扭住来玩耍。牠们1岁后就达至性成熟,寿命约3-5年。成年重约1公斤及长14-17吋,雄性较雌性大只。牠们呈黄色,耳朵较深色,腹部较浅色。墨西哥土拨鼠的语言最为复杂,能奔跑达每小时55公里。所以当受到威胁时,牠们会大叫作为警报,并且高速逃走。墨西哥土拨鼠的巢穴是挖掘出来的。巢穴的入口像漏斗,通道长达100呎,两侧有空间储存食物及休息。巢穴可以多达几百只墨西哥土拨鼠,但一般少于50只,群族有一只雄性的领袖。牠们有时会与斑点黄鼠及穴鸮分享他们的洞穴。于1956年,墨西哥土拨鼠曾在科阿韦拉州、新莱昂州及圣路易斯波托西州出没。到了1980年代,牠们从新莱昂州消失,其分布地少于800平方米。由于牠们被认为是害虫,故经常被毒杀,到了1994年到达濒危的状况。","text2":"墨西哥土拨鼠主要分布在哪些地区?","label":1} {"text1":"阿德姆·利亚伊奇(,),是一名塞尔维亚职业足球运动员,司职进攻中场,也能担任翼锋。目前效力意甲球队-{zh-hans:都灵;zh-hk:拖连奴;zh-tw:杜里诺;}-。拿积于2005年,他14岁的时候加入柏迪逊。他首次在一队上阵是在2008年7月29日,2008\/09赛季欧洲联赛冠军杯第二轮资格赛第一回合,他以后备身份上阵。他取得的第一个入球是在2008年11月23日联赛对阵OFK贝尔格莱德的赛事。2009年1月,英超球队曼联宣布利亚伊奇与队友苏兰·托锡一起转会曼联,苏兰·托锡将直接进入曼联阵容,而拿积则将以租借形式留在游击队效力。2009年12月3日曼联在网站宣布球队虽然有收购权,但由于中场人材过剩及财政问题,决定取消交易。2010年1月13日,意甲球队费伦天拿宣布签下拿积。通过体检后,拿积签订了为期5年的合约,转会费为750万欧元。拿积首次为费伦天拿上阵是在2010年1月31日联赛作客2-2战平卡利亚里的赛事,他在82分钟后备上阵。2012年5月2日,在一场意甲联赛中,费伦天拿时任主教练德里奥·罗西在把拿积换下后,拿积在鼓掌后被德里奥·罗西认为是在讽刺他,德里奥·罗西立即痛殴在后备席的拿积。比赛结束后,德里奥·罗西马上被俱乐部解雇。事后,拿积的队友认为拿积并没有冒犯德里奥·罗西的意思。2012-13赛季,拿积在主场对阵国际米兰时与队友祖维迪各自梅开二度,以4-1击败对手。拿积首次为塞尔维亚U-21上阵是在2008年9月7日,对匈牙利U-21的赛事。","text2":"利亚伊奇取得的第一个入球是在哪场赛事上?","label":1} {"text1":"胡燕泳(),香港新闻从业员。2001年胡在香港中文大学新闻与传播学院毕业,曾经从事公关工作。2003年转职香港有线电视新闻主播,并于同年转职到24小时亚视新闻台。2006年获奖学金,留学伦敦大学亚非学院,念国际研究与外交硕士。及后开始于免费频道本港台报道新闻,曾任担任本港台六点钟新闻、夜间新闻主播一职。2007年与黄珊、黄雅宇共同担任《主播天下》的主持及策划职务。2010年3月,她辞职并离开亚视新闻的播报工作,至2012年7月以兼职身分重返亚视新闻工作。2012年11月在报章撰文批评亚视高层干涉新闻部运作后,遭亚视解雇。2013年4月起,任无线新闻外电编辑,同年9月尾开始成为下午《新闻提要》主播。2013年12月7日起,担任《午间新闻》主播。2015年3月2日起,担任《环球新闻档案》旁白。2015年中,辞职并离开无线新闻,转职到香港经济日报。亚视受管理层连番操控,声称收视与TVB「四六开」惹来公众、广告业界人士和学者质疑,动员员工反对政府发出新免费电视牌照的举措也令全城非议。胡在2012年11月于《明报》发表题为《来自亚视的声音》的文章,力斥管理层王征和盛品儒于多次事件的处事手法都很有问题,批评王在亚洲电视误报江泽民逝世事件中没有任何承担;在政府总部举行的「大集会」王盛二人的言论及行为「着实超乎常人所能接受的尺度」,更严重的是「人们对亚视的鄙视,似乎并不规限于个别人士,而是整个亚视遭殃,新闻部也不能幸免」。另外,胡指出,过去她与同事制作《主播天下》时亚视给予新闻部很大自由度,但如今已今非昔比;原本《主播天下》的后续节目《ATV焦点》本来同样由新闻部记者制作,但最终已变质成为高层公器私用的平台;她和《ATV焦点》撰稿作者(前《大公报》执行总编辑雷竞斌)「在新闻部碰口碰脸」并不稔熟,但「因为他的个人大作,却引来四万宗投诉、网民的唾骂、前线记者在政总采访受滋扰;一人创作,新闻部百人当灾,我不甘心看到这境况」。2012年11月26日,亚视突然解雇七名员工,胡是其中之一;对她被辞退是否与批评公司有关,亚视公关部称不会回应。","text2":"胡燕泳是何时被解雇的?","label":1} {"text1":"圣克鲁斯-德特内里费(西班牙语:)位于西班牙加那利群岛特内里费岛上,是加那利自治区的首府之一,也是圣克鲁斯-德特内里费省的首府。圣克鲁斯-德特内里费狂欢节为当地知名的街头嘉年华会。根据考古的发现,圣克鲁斯-德特内里费地区约两千年前关切人开始就有人类居住的痕迹。原住民关切人称这块地方为\"阿尼亚措\"()。后来这地方成为大西洋和加那利群岛的重要港口之一,这种状况持续至今。在18世纪火山爆发摧毁了加拉奇科港口之后这个原本是渔村的地方突发重要,并成为岛上的主要港口。它后来从拉拉古纳独立出来,并在19世纪国王费尔南多七世的谕旨下成为加那利群岛的首府。在1833年至1927年之间圣克鲁斯-德特内里费一直是加那利群岛的唯一的首府。1927年在皇家法令下和大加那利岛的拉斯帕尔马斯共同分享加那利群岛首府的地位。这种安排延续至今。圣克鲁斯-德特内里费名字的意思是“特内里费岛上的圣十字架”,以纪念建城时埋在市中心的的一个十字架。圣克鲁斯-德特内里费狂欢节为当地知名的街头嘉年华会。此外还有各种舞蹈、戏剧和音乐节。全市共有5条快速公路连接市内其他地区:圣克鲁斯-德特内里费港是西班牙的主要港口之一,连接欧洲、非洲和美洲。特内里费岛上有两个机场。位于北部的北特内里费机场距市区只有10公里。更为繁忙的特内里费南部机场位于岛上稍远的南部。","text2":"圣克鲁斯-德特内里费港是西班牙的主要港口之一,连接了哪几个洲?","label":1} {"text1":"汪大燮内阁成立于民国11年(1922年)11月29日,结束于民国11年(1922年)12月11日。汪大燮成为国务总理,是因为黎元洪大总统利用总统特权提出;汪大燮内阁的阁员也是黎元洪提名的,而内阁名单一出台,就惹来曹锟、吴景濂、张伯烈等人的反对,因为汪大燮、许世英、高恩洪等都与罗文干案关系密切。曹锟的反对声明再一次得到各地都督的相应,使汪大燮进退两难,内阁阁员纷纷托辞不肯赴任,最后只有两名阁员出席,其它各部均由次长代理。吴佩孚也再一次为罗案而通电认错。12月5日,众议院通过决议案查办高恩洪、罗文干涉嫌舞弊渎职受贿一案;黎元洪也通电解释组织汪大燮内阁是为了让接收青岛时可以有国务总理的副署,并且表示同意由津保派的张绍曾组阁。12月11日,汪大燮请辞。他本来只和黎元洪约定10日的总理期限,由于短短10日内受到各方不断的非难,届满之后,汪不肯延期,他敦请王正廷代理总理职务。","text2":"汪大燮届满之后敦请谁代理总理职务?","label":1} {"text1":"克里米亚鞑靼人( 或 , 或 , ,)或称“克里米亚人”(),是原定居于克里米亚半岛的突厥语民族,他们的历史最早可以追溯至斯基泰人、希腊人、哥德人、东斯拉夫人、罗马尼亚人与切尔克斯人时期。后来,威尼斯人及热那亚人与突厥特别是匈人、阿瓦尔人、保加尔人、可萨人、佩切涅格人及钦察人混血,形成克里米亚鞑靼人。他们是最早宣传泛突厥主义的民族。与伏尔加鞑靼人不同,他们受到乌古斯人影响比较大,在金帐汗国时代已经被伊斯兰化。虽然克里米亚鞑靼人被称为鞑靼人,但其语言实际上与伏尔加鞑靼人的语言相差较远。他们分布在克里米亚半岛、土耳其、罗马尼亚、保加利亚、波兰、立陶宛、白俄罗斯与北美、巴西及西欧,也有分布在北欧(芬兰)。他们以前是奥斯曼帝国贩奴活动的中间人,诺盖人捕奴后交给他们,送到奥斯曼帝国(直到18世纪初,克里米亚鞑靼人在某些时期几乎每年到俄罗斯和乌克兰进行频繁的毁灭性袭击,称为草原民族收成,有些研究人员估计,15至18世纪合共超过300万斯拉夫人出口土耳其)。他们有自己的汗国(以克里米亚为中心,以北的草原至北高加索,他们好战,土耳其人有事也找他们帮忙),先是土耳其人的臣民,他们的汗是拔都的兄弟秃花帖木儿的后人,家族名是格来。克里米亚鞑靼人主要生计就是靠袭击别国。18世纪被俄罗斯帝国吞并,他们在第二次世界大战被指与纳粹德国勾结,被苏联领导人斯大林集体流放中亚和西伯利亚,直到1950至1990年代后才陆续返回,但地方已被俄罗斯人与乌克兰人填补(他们曾经过二次流亡,第一次是克里米亚汗国亡国流亡土耳其,第二次是史达林硬送他们到乌兹别克与哈萨克)。2014年克里米亚危机爆发时,许多克里米亚鞑靼人发起声援基辅政府的示威游行,并且拒绝参加克里米亚举行的归属公投。克里米亚宣布入俄后,克里米亚鞑靼人在巴赫奇萨赖召开临时代表大会,要求建立克里米亚鞑靼族民族区域自治的政治和法律程序,不久,立陶宛发起联合国安理会非正式会议讨论克里米亚鞑靼人的人权问题,但遭到俄罗斯的抵制。俄罗斯总统普京在接见俄罗斯联邦鞑靼斯坦共和国总统鲁斯塔姆·明尼哈诺夫时声称他将派人研究为克里米亚鞑靼人恢复名誉的问题。2016年,克里米亚鞑靼人的最高代表机关「克里米亚鞑靼人民族议会」被俄罗斯禁止。","text2":"克里米亚鞑靼人是原定居于克里米亚半岛的哪个民族?","label":1} {"text1":"国民年金保险,简称国民年金、国保,法源依据《国民年金法》,于2007年7月20日经立法院三读通过,是中华民国于2008年10月1日开始实施的一项社会保险制度,主要的纳保对象为未参加军保、公教保、劳保、农保的25岁以上未满65岁中华民国国民。国民年金主管机关为卫生福利部(2013年7月23日卫生福利部成立以前为内政部),并委托劳动部劳工保险局为保险人。国民年金的法源依据《国民年金法》,于2007年7月20日经立法院三读通过,2007年8月8日总统令公布,2008年10月1日施行。国民年金目的在保障25岁以上、未满65岁,且未参加军人保险(军保)、公教人员保险(公保)、劳工保险(劳保)、农民健康保险(农保)之国民,于发生老年、身心障碍、生育及死亡事故时,本人及遗属之基本经济生活。国保之保险事故分为老年、生育、身心障碍及死亡4种,保险给付项目计有:老年年金给付、生育给付、身心障碍年金给付、丧葬给付及遗属年金给付等5种。具津贴性质的年金项目为:身心障碍基本保证年金、老年基本保证年金、原住民给付。自得请领之日起,应于5年请求权时效内提出申请。以下对于国民年金的各项给付的条件做说明。被保险人在国民年金保险加保期间,遭受伤害或疾病,达重度以上领有身心障碍手册(非重大伤病卡),经治疗终止症状固定或是再行治疗不能期待复原,需经身心障碍医疗机构评定无工作能力。如同时符合劳、公教保、军保、农保的身障给付,仅能择一请领。身心障碍基本保证年金是被保险人在参加国保前,已领有重度以上身心障碍手册(非重大伤病卡)或证明,经身心障碍鉴定医疗机构评定为无工作能力,且在国内居住,每年居住超过满183天。无下列任何情形之一者,得申请每月4,872元(105年1月起由4,700元调整至4,872元)的身心障碍基本保证年金。","text2":"国民年金的法源依据是什么?","label":1} {"text1":"朴属(学名:)是蔷薇目大麻科的一属,约60-70种落叶乔木,广泛分布在北半球暖温带地区,以及中非和南美洲部分地区。本属植物一般为10-25米高的乔木,稀有40米高,单叶互生,叶有3-15厘米长,有齿;花小,早春开花,雄花长,雌花圆;果实为直径6-10毫米的核果,类似枣,大部分种类可以食用。树皮可用来造纸。1981年的克朗奎斯特分类法将朴属列入榆科,1998年根据基因亲缘关系分类的APG 分类法单独分出一个科,列入蔷薇目,2003年经过修订的APG II 分类法不承认这个科,将本属放到大麻科中。本属与大麻科其它属的关系如下:Biodiversity\" 4(2): 127–132. PDF fulltext","text2":"朴属是怎么归类为大麻科的?","label":1} {"text1":"人体探险队(韩文:)是SBS的“星期天真好”的一部分,每周日晚5:30在SBS播出。节目的目的是要回答大家在人的身体方面好奇的问题,并通过Super Junior用自己的身体做实验来证实。由于它的娱乐和科普性,这个节目吸引了广大的观众。前一个系列——《X-MAN》——结束放映,SBS的GOOD SUNDAY决定一个新的教育计划,对观众介绍人体。申东烨被选为主持人,与Super Junior成员共同出演节目。人体探险队在SBS播出的GOOD SUNDAY三个节目中收视率是最高的。这也是第一个和目前唯一的Super Junior每一个成员都参加出演了的综艺节目。每一集有一个主要议题,探讨关于人体的方面。议题不尽然是物理方面的,也有围绕平衡感(第六集)和眼泪(第八集)的。韩国播出日期从2007年11月11日到2008年2月3日每集的主题如下所列:由于Super Junior的繁忙,人体探险队2008年2月3日结束放映。原本预计是暂时性的中断,但是Super Junior有诸多通告(包括准备亚洲巡回演唱会),无法排开档期,只得宣布就此结束。但有计划称,若情况允许,不排除有第二季的可能。","text2":"人体探险队播出的频道是什么?","label":1} {"text1":"李抱玉(),河西武威(甘肃武威)人。本名安重璋,唐肃宗赐姓李氏,唐玄宗为其改名抱玉。出自昭武九姓之一安国。唐朝初年开国功臣安兴贵的后裔。李抱玉家族世代居住于河西,以善养名马而闻名。李抱玉在西州长大,爱好骑射,熟识军事。安史之乱时,唐肃宗在灵武即位,至德二年(757年)五月赐姓李氏,乾元初年(758年),投靠太尉李光弼,屡建战功,由右羽林大将军升为持节郑州(今属河南)诸军事兼郑州刺史。唐玄宗以其战河西有功,为其改名抱玉。李光弼固守河阳(今河南孟县东南),光弼问李抱玉:「将军能为我守南城二日乎?」李抱玉果不负使命,贼帅周挚撤退。以坚守河阳,收复怀州,皆功居第一,迁泽州刺史、兼御史中丞,封栾城县公。唐代宗即位,擢为泽潞节度使、潞州大都督府长史、兼御史大夫,加领陈、郑二州,迁任兵部尚书,封武威郡王。抱玉多次上书恳辞王爵,代宗只好改封其为凉国公,拜为司徒。广德元年(763年),吐蕃攻陷京师长安,唐代宗逃往陕州,李抱玉兼任凤翔节度使,负责围剿溃卒与乡村亡徒组成的土匪军,旬日内平定,以功迁司空。李抱玉为唐朝镇守西部要害,抵御吐蕃入侵,深受唐代宗的恩宠,官至同中书门下平章事,又兼山南西道节度使、河西陇右山南西道副元帅、判梁州事,连统三道节制,兼领凤翔、潞、梁三大府,秩处三公。李抱玉以任位崇重,请求辞去司空及山南西道节度、判梁州事的职务,并退授兵部尚书。他镇守凤翔十余年,虽无破虏之功,但境内比较安定,颇为当时所称赞。大历十二年(777年),卒于河西陇右副元帅、同中书门下平章事任上,年七十四。代宗罢朝三日,追赠太保,谥号昭武。子李自正,字尚贞,少府监,袭凉国公。工正书,其父李抱玉纪功碑阴记为其所书。","text2":"李抱玉镇守凤翔十余年表现如何?","label":1} {"text1":"都督福满(),爱新觉罗氏,明朝建州左卫人。觉昌安的父亲,塔克世的祖父,努尔哈赤的曾祖父。1636年5月16日(清崇德元年四月乙亥朔十二日丙戌),皇太极追尊他为庆王。1648年12月18日(清顺治五年十一月辛酉朔五日乙丑),顺治帝追谥他为直皇帝,庙号兴祖。同时受追尊得还有努尔哈赤的高祖父清肇祖,也就是福满之父。但是据后来修订的《满州实录》记载,福满之父锡宝齐篇古是清肇祖都督孟特穆的孙子。根据当代学者的研究,福满和锡宝齐篇古可能是虚构的人物。福满有六子,德世库,刘阐,索长阿,觉昌安,包朗阿,宝实。都居住在赫图阿拉的觉罗地方,‘环卫而居,彼此相距近者5里,远者不过20里,是为六工,又称为宁古塔贝勒,亦通称努尔哈赤六祖’。皇太极天聪九年(1635),为区分直系与旁支的世系关系,皇太极诏令觉昌安四子塔克世的后裔称宗室子孙称为宗室,系黄带。觉昌安之外五祖子孙称觉罗,系红带,以别于宗室,后裔子孙载入觉罗族谱。","text2":"都督福满是谁的父亲?","label":1} {"text1":"杜范(),字成之,号立斋,黄岩杜家村人。少年师从杜知仁,是朱熹的再传弟子。南宋嘉定元年(1208年)进士,历官金坛县尉,婺州司法参军,安吉司理参军,《宋史》称“杜范在下僚已有公辅之望”。端平二年(1235年)十二月,任监察御史,劾尚书右丞郑清之,理宗不敢得罪郑党,贬杜范为起居郎。次年任太常少卿。嘉熙二年冬,任江西宁国知州。嘉熙四年六月,任吏部侍郎兼中书舍人。淳祐二年(1242年)六月,任端明殿学士、同签枢密院事。淳祐四年(1244年)十二月,官右丞相兼枢密使,临安市民“欢呼载道”,逐史嵩之党人刘晋之、郑起潜等,次年上疏五事:“正治本,肃宫闱,择人才,惜名器,节财用。”。淳祐五年(1245年)四月二十一日,病逝临安,宋理宗谥清献。葬于黄岩县西70里靖化乡黄杜岭(今牌门)。著有《古律诗歌》5卷。黄震在《戊辰修史丞相杜范传》中说:“端平大坏之余,方得正人如杜公。”车若水《恸立斋先生》称:“元元含望久,及是事方新。四海看更化,皇天忍误人。”有侄杜浒于宋亡后以身殉国。","text2":"杜范官右丞相兼枢密使期间曾经上疏五事,是哪五事?","label":1} {"text1":"毕耀明(Brian Butt Yiu-ming,)为现任北京泛美国际航空学校校长,前政府飞行服务队总监,前香港航空青年团总监。于1978年5月加入香港警务处,任职见习督察,1986年4月转往皇家香港辅助空军任职机师。1989年11月晋升为高级机师,1993年1月晋升为总机师,1996年8月出任政府飞行服务队总监,至2007年底提出请辞,共服务政府29年。 总机师陈志培于2008年3月27日起接任政府飞行服务队总监一职。毕耀明1954年出生于香港,在香港接受中小学教育,1977年在加拿大取得科学学士学位,1995年取得香港大学工商管理硕士学位,已婚,育有2名儿子。毕耀明大学毕业后返回香港,投考国泰航空机师失败,其后加入香港警务处,同时也是皇家香港辅助空军的志愿飞行员。他曾被派往商业罪案调查科,是首批受训的伪钞鉴别专家。1986年离开警队(当时职级为总督察),加入皇家香港辅助空军担任全职直升机机师,是部队唯一的华人机师,拥有定翼机及直升机驾驶执照,具直升机飞行教练资格。其后晋升高级机师及总机师,后于1996年8月1日出任政府飞行服务队总监,年仅42岁,成为最年轻的华人总监。2003年8月26日晚上,一辆政府飞行服务队海豚直升机由总部起飞,前往长洲接载伤者途中,于大屿山伯公坳附近的山头坠毁,两名机员罹难。这次意外,是政府飞行服务队于1993年成立以来,首次有队员坠机导致殉职的事故。民航处的调查报告指直升机师疏忽,并提出多项改善建议<\/ref> <\/ref> ,政府飞行服务队全数接纳。","text2":"毕耀明在哪所大学取得了硕士学位?","label":1} {"text1":"德班国际机场(;),前称路易博塔机场,位于南非德班。这个机场是南非的国际机场(共3个)中最细的机场。共有一条跑道。机场附近有两条公路:N2公路和M4公路。德班国际机场服务国际航线多年来都是下降。有一段短时间,新加坡航空曾有航线服务德班市。现在,大多数国际航空公司都只飞往南非的约翰内斯堡。德班相对会有较低的国际客运量,而且机场只有一条短跑道,不能让波音747降落。在2007年度(2007年4月至2008年3月),机场服务了近4.4万人次。于1970年代和2007年,曾计划画把机场迁移到德班以北30公里的香格里拉慈悲,但多次被搁置。最后在2007年批准建造沙卡国王国际机场(KSIA) ,并在2007年9月动工,并于2010年世界杯足球赛前完成。当KSIA当行业务时,现有的机场即将关闭。德班国际机场坐落于一个大型和重要的德班南工业盆地,这已经有大部分重工业。鉴于德班的一般都是很丘陵地形,如此大和平板包裹是理想是发育未来工业,扩建港口,或两者兼施。","text2":"于1970年代和2007年,曾计划画把机场迁移到哪里?","label":1} {"text1":"国民乐派,是广义古典音乐的一支,但没有明确的定义和共同的风格,经常被视为「浪漫主义音乐」或「现代音乐」的分支。大致就是指会在古典音乐的曲式中,使用了本国的民间音乐旋律和特别的乐器,并通常宣扬爱国主义或民族主义,主要在十九世纪中到二十世纪中活跃。一般认为十九世纪中前期,西方音乐的传统心脏的德语和义大利地区以外的欧洲和美洲国家,作曲家需要属于本地的音声。反之德语或义语的作曲家即使用民间音乐素材,仍然只是会被认为浪漫主义音乐,如小约翰·施特劳斯的圆舞曲原型虽然来自民间,却被当时的维也纳视为国际性的艺术。国民乐派起源众说纷纭,较流行有两说。一是十九世纪俄国作曲家格林卡的《爱国歌》,另一说是法国作曲家圣桑成立国家音乐社团组织,两人分别影响了本国的作曲家组织,交流了其对民间音乐的资料和心得,以及对于教育本国的音乐界作出重大的贡献。其他尚有肖邦和弗朗兹·李斯特之说,因为是他们首先采用故乡的民间音乐素材,但他们并未有组织本地作品家活动,也没有系统地使用和搜集民间音乐元素。在十九世纪后期其他欧洲国家也产生了自己国民乐派作曲家和作品,如英国的艾尔加的《威风凛凛进行曲》(Pomp and Circumstance Marches),捷克的斯美塔那的《我的祖国》。还有一些作曲家为自己尚末被认可为国家的故乡,创作鼓吹独立的作品,如芬兰的西贝柳斯的芬兰颂,挪威的格里格的培尔·金特组曲等。而美洲的作曲家也开始活跃起来,如美国的约翰·菲利普·苏萨的进行曲《星条旗永远飘扬》,而同期斯蒂芬·福斯特更有开创流行音乐的先河。到二十世纪初巴西的海托尔·维拉-罗伯斯,匈牙利的柯达伊,美国的查尔斯·艾夫斯等,新国民乐派作曲家开始向现代音乐的过渡。到二十世纪中后期起,因为流行音乐的兴起,也少有未被古典音乐使用的民歌,所以在欧美国民乐派渐趋于沈寂,但在古典乐传统地区外的亚洲,却有新型的国民乐派作品面世,如中国的冼星海的黄河大合唱,何占豪和陈钢的梁祝小提琴协奏曲等,代表了欧美以外的民族主义兴起。二十一世纪初,虽然因为再很少未独立的殖民地和国际主义盛行,政界不再需要常以音乐推广爱国思想,但国民乐派音乐并未因此而丧失了其地位,反而把这种音乐从政治服务中解放过来,受到本来创作国家外的欢迎。","text2":"二十世纪初,新国民乐派作曲家开始向什么方向发展?","label":1} {"text1":"板棍球(英语:Hurling;爱尔兰语:iománaíocht、iomáint),又称爱尔兰式曲棍球,是一种使用前端为平板的球棍运球与击球的团队球类运动,与盖尔式足球并称为爱尔兰两大运动。板棍球比赛双方各上场15名球员,比赛时间全国性70分钟,地区性60分钟,青少年组50分钟,分成上、下半场。比赛场地与盖尔式足球一样,为长方形草地,两端线中央各有一个「H」形球门。比赛的目的是将球用板棍打击射门得分,比赛时间终了时,以得分多者为胜队。持球员向前推进时,会结合握球、运球(将球置放于球棍前端板面上),然后用板棍打击传球给其他队友,或直接射门得分。板棍球与盖尔式足球一样,是由爱尔兰最大的运动组织-盖尔运动协会所主导。","text2":"比赛的目的是什么?","label":1} {"text1":"雅典的废墟(,Op.113),是乐圣贝多芬的一部歌唱剧作品,创作于1811年,于1812年在布达佩斯首演。这部作品最有名的部分是其序曲和第四乐章土耳其进行曲,甚至非古典乐迷也能认出土耳其进行曲的主题。因此,序曲与土耳其进行曲都经常被单独取出来演奏,而作品的其它部分演奏机会较少。贝多芬的另一部作品,D大调六个变奏的钢琴变奏曲(Op.76),主题便是来自土耳其进行曲。智慧女神密涅瓦因为嫉妒苏格拉底而没有在法庭上为他辩护,因为这个罪,宙斯让她沉睡2000年。在两千年的沉睡结束的时候,密涅瓦醒了,被墨丘利带到了雅典。她深爱的雅典变成了废墟,当看到在土耳其统治下雅典时,她惊呆了。后来得知罗马也和雅典一样成为了废墟。听墨丘利说,缪斯他们逃到了匈牙利的佩斯。于是,密涅瓦和墨丘利踏上了去匈牙利的旅途,在那里他们看到了人对神和缪斯的无比忠诚。佩斯作为新的雅典开始繁荣复苏。","text2":"雅典的废墟是谁的作品?","label":1} {"text1":"赵允让(),宋朝皇族,字益之,宋太宗第四子商王赵元份的第三子。濮安懿王。史称他天资浑厚,外庄内宽,喜愠不见于色。赵元份在世时,赵允让为右千将军。宋真宗的长子周王赵祐去世后,真宗以绿车旄节迎赵允让到宫中抚养。皇子赵祯(宋仁宗)出生后,用箫韶部乐送还府邸,官居卫州刺史。宋仁宗即位,授汝州防御使、宁江军节度使。后来又让他知大宗正寺。他勉励好学的宗室子弟,劝戒不上进的,故人莫不畏服。庆历四年(1044年),封汝南郡王,拜同平章事,改判大宗正司。嘉祐四年(1059年)去世,年六十五,赠太尉、中书令,追封濮王,谥号安懿。仁宗在位久无子,以赵允让第十三子赵宗实为皇子,改名赵曙。仁宗崩,皇子赵曙即位,是为宋英宗。宋英宗即位后,围绕赵允让是作为皇伯还是皇父的争议,被称为濮议。","text2":"谁是赵允让?","label":1} {"text1":"黑监督吸蜜鸟(\"Drepanis funerea\")是一种已灭绝的雀。牠们是于1893年在莫洛凯岛发现,曾一度生活在山顶上的树林中。在茂宜岛亦曾发现牠们的化石。由于牠们的外表美丽,很多时会被捕猎来作为标本。黑监督吸蜜鸟的体型较大,由尾巴至喙尖长约8吋,但却不及夏威夷监督吸蜜鸟。牠们呈浅黑色,有一些白色的主羽。牠们的喙长而且较为弯曲,雄鸟的喙较雌鸟的长。牠们有时会接近人群及飞得很低,从未飞高过12呎以上。牠们的栖息地受到牛及鹿的破坏。原本用来捉大家鼠的红颊獴,却掠食牠们。最后观察到黑监督吸蜜鸟是于1907年,当时有三只鸟被射,虽然最后一只逃走了,但牠们肯定已经灭绝。于1936年曾进行了一次大规模的搜寻,但却没有发现牠们的踪影。现时的黑监委吸蜜鸟标本分别存放在不莱梅、波士顿、檀香山、伦敦及纽约。","text2":"黑监督吸蜜鸟的栖息地受到了什么破坏?","label":1} {"text1":"《人在边缘》()是香港电视广播有限公司制作的时装恩仇电视剧,全剧共30集,监制张华标。主要演员有黎明、刘青云、林文龙、罗慧娟、陈法蓉、黎美娴、方刚等。主题曲由卢东尼作曲,陈少琪填词,黎明主唱。此剧是1990年香港年度收视冠军,全剧平均收视为42点,亦是张华标唯一一部监制的作品,黎明亦凭此剧走红。本剧曾于2006年在广东电视台珠江频道及于2010年7月5日在TVB星河频道重播,于2009年8月在无线收费电视经典台《我们的…黎明》每周六至日连续播映四集,特别调动此剧于2009年10月3日-4日周六至日连续播映六集。2015年6月及2017年9月6日在TVB经典台重播。2016年6月1日起在翡翠台重播,并附加上中文字幕。林雪娥早年为救家人,诬陷郑细凤杀人,细凤不忿被冤枉,越押逃走。十八年后,雪娥长子刘志光长大成人,任职惩教署。细凤之子林奕龙误入歧途,被判入劳役中心,重遇志光。奕龙故意为难志光,但志光却细心引导奕龙返回正途。经历重重波折,二人成为了好友。此时,细凤回港寻找奕龙,雪娥幼子志基利欲薰心,错手杀了细凤养子郑伟,更冒认是细凤之子,期间,志基因恐雪娥泄露真相而错手将她杀死。志光见其弟丧心病狂,决大义灭亲,与奕龙联手对付志基……","text2":"《人在边缘》的主题曲由谁主唱?","label":1} {"text1":"布鲁诺·爱德华多·雷古费·阿尔维斯(葡萄牙语:,,),葡萄牙足球运动员,司职后卫,现效力于苏超联赛流浪者。这名球员有巴西人血统,因其父乃巴西人,其母则是葡萄牙人。出身于波图,早年多次被外借至国内其他小型球会,至2005年起才由波图收回。他以其身材和制空能力见称,其间他与巴西人-{zh-hans:佩佩;zh-hk:比比;zh-tw:佩佩;}-组成后防中路组合,协助球会赢得三届葡超联赛冠军(2005-06、2006-07、2007-08)。他于2007年夏季获选入葡萄牙国家队。2008年欧洲国家杯仅于分组赛一场无关痛痒的赛事中上阵。
2010年以后,布鲁诺·阿尔维斯逐渐成为国家队主力球员,在2012年欧洲足球锦标赛上他成为首发中后卫,随球队一路闯进半决赛,但是在半决赛中与西班牙的点球大战中,阿尔维斯先是与队友沟通不利弄错了出场顺序,随后又罚失了点球,导致葡萄牙点球2-4不敌对手无缘决赛。
2014年,阿尔维斯随队参加了在巴西进行的世界杯,作为首发中后卫参加了所有的三场小组赛比赛。2016年,他再次入选欧洲杯葡萄牙队的大名单中,虽然他的主力位置已经被若泽·丰特夺走,但是在与威尔士的半决赛中,阿尔维斯临危受命,顶替停赛的佩佩出战,最终帮助葡萄牙成功战胜对手,并且随后随队伍获得了本届欧洲杯的冠军。
2017年,布鲁诺·阿尔维斯再度随葡萄牙队参加了在俄罗斯进行的联合会杯,在本届比赛中发挥稳定,帮助球队获得了赛事的季军。","text2":"布鲁诺·阿尔维斯以什么能力见称?","label":1} {"text1":"ORDO经济与社会秩序年鉴(德语:\"ORDO - Jahrbuch für die Ordnung von Wirtschaft und Gesellschaft\"),专业年刊,1948年由德国经济学家Walter Eucken和Franz Böhm首次发行,曾被弗赖堡学校(Freiburger Schule)用作其学术讨论中心。“秩序自由主义”(Ordoliberalismus)一词与刊物名称相互呼应。此外,社会市场经济 (西德在第二次世界大战之后建立的经济制度)也几乎全部由ORDO年刊发展而来。ORDO杂志着眼于掌管现代社会的经济和政治机构,致力于为不同学科(比如经济、法律、政治科学、社会学和哲学)的学者提供一个交流与辩论的平台。一份ORDO杂志的页数在400至500页之间。其中既有德文也有英文文章,平均长度为17页。交到杂志社的手稿都须通过客观的双盲审核才能得到发表(同行评审)。斯图加特Lucius & Lucius出版社自1996年起负责ORDO杂志的发行。该杂志还设有电子版,读者可在网上进行有偿阅读,其中的文章小结与英语节选等内容是免费的。在ORDO发表文章的有诺贝尔获奖者詹姆斯·M·布坎南 (James M. Buchanan), 米尔顿·弗里德曼 (Milton Friedman), 弗里德里克·哈耶克 (Friedrich von Hayek)和乔治·斯蒂格勒(George J. Stigler),以及卡尔·波普尔(Karl R. Popper)和穆瑞·罗斯巴德(Murray Rothbard)","text2":"ORDO杂志设立的目的是什么?","label":1} {"text1":"北海是一列日本国有铁道(国铁)于1967年3月1日至1986年11月1日之间,行驶于函馆~旭川间的函馆本线特急列车。日本国有铁道为了使青函连络船的旅客能经由更多途径连络道内,在函馆本线上设立了该列车,“北海”也是目前为止唯一经由「山线」(函馆本线长万部~小樽间路段)的特急列车。最初投入运行时使用日本国铁Kiha80系柴联车行驶,运行初期为7节车厢编成、一天仅一往返。1973年10月2日,因运行成果佳,故将编成调为十节一组,也有经由钏网本线延驶至网走站的班次,此时的特急北海达到了全盛期。1981年10月1日,班次调为一天二往返,因Kiha80系逐渐老化而将北海3、4号改用为新制的Kiha183系车辆。以Kiha80系行驶的班次调为九节车厢一组,Kiha183系行驶的班次则是十辆编组。1984年2月1日,编组再度缩为六节,偶尔才会增结为七节,而且只使用Kiha80系行驶,直到翌年才恢复Kiha183系行驶,编组也都固定为六节。1986年11月1日,特急「北海」废止,特急列车从此自山线上消失。因平坦的室兰本线(海线)逐渐被重用,「山线」已有没落的趋势。虽然经由室兰本线来往函馆与札幌的里程数较山线长,但因为弯道比函馆山线少的多,所费时间也较短。「山线」一路上都是急转弯和陡坡,马力低的车辆要通过是十分困难的。因此后来由道南前往道央的优等列车全部改走室兰本线。「北斗」即是「北海」的海线版本。「北海」停驶后,其做为道南连络道央用列车的重要性,改由「北斗」取代。在「北海」行驶的同时,山线上也有急行列车「二世谷」()行驶,但在北海号废除的同时,二世谷号调整为仅有夏季行驶的临时列车。现在「山线」上仅有定期行驶的快速列车二世谷快车()、普通车和临时特急列车二世谷滑雪特快()、SL二世谷号行驶。","text2":"现在「山线」上仅有定期行驶的哪些列车?","label":1} {"text1":"赫蒙族(Hmong)是苗族的一个分支,在越南被划为分53个法定少数民族之一。现在分布在中国、越南、老挝、泰国,甚至美国(见苗族裔美国人)等西方国家。在18世纪时,由于政治动荡以及为了寻找更多的耕地,大量的赫蒙族人开始大规模南迁。在越战结束后以及老挝人民民主共和国建立后,许多赫蒙族人受到越南和老挝两国的严重的民族宗教迫害,通过泰国难民营进入美国。最早一批是在1975年,只有3,466人获准庇护身份进入美国。2011年越南西北部边境地区的奠边省发生罕见大型骚乱,多达7000越南的赫蒙族人(苗族)发动和平抗争,要求越南当局消除对少数民族的压迫、给予更多宗教自由及自主权。越南当局调动军队镇压,据称最少28死、数百人失踪。越南当局禁止外国传媒到当地采访,又截断当地电源及通讯。德新社引述当地官员指,示威民众架设了路障,并建立了防御设施,当局已有1300名警察及为数不明的士兵派到当地。","text2":"赫蒙族分布在哪里?","label":1} {"text1":"颜信辉为台湾会计师及会计学家,现任淡江大学教授与会计长,曾多次参与制定中华民国的财务会计准则与参予国际财务报告准则(IFRS)翻译搞的审阅,多年来持续宣导堆动中华民国全面直接使用国际会计准则(T-IFRS),放弃中华民国过去数十年来自行制定并参杂美国会计准则(FASB)的财务会计准则(T-GAAP),以原则基础(Principle-based)会计取代规则基础(Rule-based)会计。颜信辉,拥有十分完整的会计专业背景,1986年(民国75)于淡江大学会计系毕业,1988年获政治大学会计所硕士学位,他回到淡江担任讲师一职,为了让自己的视野更宽广,他选择留职停薪至美国伊利诺大学厄巴纳-香槟分校校区攻读会计硕士,回国后继续在本校担任教职,并同时攻读台大会计博士,1996年获台湾大学会计学博士学位。曾任中华民国财团法人会计研究发展基金会财务会计委员会顾问,委员,中华民国考试院典试委员等职务,现为淡江大学会计系教授兼会计室主任,以及会计研究发展基金会的财务会计准则委员会委员。其妻为金管会证期局第六组组长詹静秋。","text2":"颜信辉是谁?","label":1} {"text1":"南洋美银汉鱼(学名:),为辐鳍鱼纲银汉鱼目银汉鱼科的其中一种。本鱼广泛分布于印度西太平洋区,包括红海、东非、马达加斯加、马尔地夫、印度、安达曼群岛、菲律宾、印尼、越南、泰国、马来西亚、台湾、中国沿海、日本、韩国、澳洲、密克罗尼西亚、马绍尔群岛、马里亚纳群岛、索罗门群岛、萨摩亚、夏威夷群岛、斐济等海域。水深0至4公尺。本鱼体延长而细,略呈圆柱状,头及眼适度大小,头部无小棘列。前上腭骨较长,末端超过瞳孔前缘;其前上突起及侧突起短而宽,且前上突起之高度与宽度相等。下腭骨上方略呈弧形,其后部不急速高耸;前鳃盖骨之后缘具一明显缺刻。背鳍2枚,第一背鳍始于体背中央之略后方,具4至6棘;第二背鳍位于位于第一背鳍之略后方,具1棘极8至11软条;臀鳍与第二背鳍对在而略前位,具1棘极12至17软条;腹鳍亚胸位;肛门较前位,开于腹鳍末端之腹缘;尾鳍开叉。无侧线,体侧鳞为圆鳞,鳞片大,具5纵列,一纵列有39至44枚鳞片。两腭齿细小,绒毛状;口盖骨及锄骨均有齿。体呈蓝绿色或灰色。体长可达14公分。为海水鱼,白天经常大量群聚于近岸。大部分于夜间索饵,其食物包括浮游性鱼卵、甲壳类、有孔虫及桡脚类。食用鱼,也可作为海鸟及其他鱼类的饵料鱼。","text2":"此鱼在夜间,会有何动作?","label":1} {"text1":"莙荙菜(\"Beta vulgaris\" subsp. \"cicla\"),即叶用甜菜,俗语又叫牛皮菜、厚皮菜、猪乸菜,苋科菾属的耐寒性一年生或二年生草本叶菜,是甜菜的一个变种。常见于地中海料理。莙荙菜原产欧洲南部,公元5世纪由阿拉伯传入中国。虽然菜叶常绿,但其叶柄颜色多变,有绿色、红色等多种颜色。莙荙菜的营养价值非常高,而且经过裁培后,菜的营养都聚集到叶片,而不是像甜菜那样聚在根茎。因此,莙荙菜被誉为是其中一种最健康的菜,常用于健康餐单里。莙荙菜虽然味道微苦,但在世界各地方的料理均有出现。在北美及欧洲亦有用来做西式馅饼的馅料;华南地区一般都是灼熟、用蒜头煮或煮汤,用以去掉苦味,口感也媲美烫菠菜。莙荙菜的幼苗可以用来拌色拉。","text2":"莙荙菜有什么属性?","label":1} {"text1":"三带立旗鲷,又称金口马夫鱼,俗名南洋关刀,为辐鳍鱼纲鲈形目蝴蝶鱼科的一个种。本鱼分布于印度太平洋区,包括东非、亚丁湾、马尔地夫、葛摩、模里西斯、塞席尔群岛、印度、斯里兰卡、马来西亚、安达曼海、泰国、菲律宾、印尼、中国南海、东海、日本、台湾、越南、新几内亚、所罗门群岛、澳洲、马里亚纳群岛、马绍尔群岛、密克罗尼西亚、帛琉、诺鲁、斐济群岛、东加、吉里巴斯、吐瓦鲁、万纳杜、法属波里尼西亚、墨西哥、加拉巴哥群岛、厄瓜多等海域。水深2~20公尺。本鱼乃因其头和体侧有3条黑色斜横带,且背鳍第四条鳍棘延长如丝,故其名。体白或略黄,第一条黑带在背鳍起点前往下延伸,涵盖眼部而至腹鳍末梢。第二条则起自背鳍第四鳍棘,几乎和第一条平行。第三条则在背鳍鳍条部的基底部。幼鱼则在臀鳍处有枚黑眼斑,且背鳍第四鳍棘较成鱼细长。背鳍硬棘11~12枚、软条21~22枚;臀鳍硬棘3枚、软条17~18枚。体长可达18公分。本鱼喜欢栖息在亚潮带珊瑚密生的平台和礁坡上,属肉食性,以珊瑚虫为主食。为高价值观赏鱼,不供食用。台湾鱼类资料库","text2":"三带立旗鲷以什么为主食?","label":1} {"text1":"圣马丁驻军教堂(Garnisonkirche St. Martin)是德累斯顿的阿尔贝特城(Albertstadt)的一座驻军教堂,位于一片军营建筑群的中心。此教堂按建筑师罗索(Lossow)和费维格(Viehweger)]的设计,为当时在阿尔贝特城的驻军而建,成于1895到1900年间。这是一座双重教堂(Doppelkirche)。它包括新教和天主教两个部分,1945年以后只有天主教部分仍在使用,现在是德累斯顿新城圣方济各沙勿略牧区;新教部分因沦为森柏歌剧院(die Semperoper)的服装仓库而失去其宗教用途,亦不供参观。自从1900年10月28日教堂落成,天主教厅的礼拜者就源源不断。第二次世界大战结束前她一直被用作驻军教堂;1945年6月起,她成为圣•弗朗西斯科•塞维尔(圣方济各沙勿略)(St. Franziskus Xaverius)牧区教堂(之前该牧区的牧区教堂位于阿尔贝特广场(Albertplatz),毁于1945年的大轰炸),直至今日。近年来附近的军官学校的士兵也来参与礼拜和宗教服务。前方中央是一个圣坛(Hochaltar)(新罗马式教堂的核心结构),上面的马赛克画述说着耶稣乃救世主。圣坛前方有是诵经台(Ambo)和祭台(Zelebrationsaltar),其支撑部分则分别是象征着基督的阿尔法A和欧米伽Ω。后殿(Apsis)上方的彩色玻璃小圆窗尤为奇美,它们描绘着圣伊丽莎白公主、圣嘉禄•鲍荣茂主教、阿尔贝特国王、乔治骑士和玛蒂尔达女王。在大玻璃窗上画着圣母玛利亚像和教堂的守护者圣•马丁的割袍像。东南侧是拼嵌有创世图案的玫瑰窗(Schöpfungsfenster),下面的马赛克装饰画是大天使米迦勒。教堂后方的管风琴是耶穆里希三兄弟于1900年所造,分列于其左右玻璃窗上的是殉道者圣•莫里斯和圣•维克多。两侧回廊下面有15幅现代风格的耶稣受难图,及1幅老卢卡斯•克拉纳赫(Lucas Cranach der Ältere)的朝圣画圣母之佑的摹本。中殿(Mittelschiff)和两侧的祈祷室(Seitenkapellen)里各有美轮美奂的彩绘,它们在2005至2007年的修复工程耗资巨大。祈祷室的老吊灯(Leuchter)亦值得一观;中殿悬挂的巨型吊灯则为2006年新制。","text2":"圣马丁驻军教堂是哪里的驻军教堂?","label":1} {"text1":"汇豪山()位于香港九龙慈云山,是一52层高的单幢式住宅,于2008年5月落成,发展商为爪哇控股。汇豪山前身为香岛中学慈云山分校,亦是慈云山十年来的新屋苑。由于建在山丘上,一些单位可以侧面观赏到港岛东之景色。汇豪山的名字源自美国纽约著名的传统豪宅区「」,爪哇集团将这元素注入汇豪山,希望以此制造出高级住宅的形象。汇豪山地址为九龙慈云山蒲岗村道99号,同时邻近慈云山中心等建筑。汇豪山亦是位于45区小学校网中。汇豪山为一52层建筑(不设4、13、14、24、34、44字楼),合共304个单位(一梯八伙),每伙面积少于1,000平方呎 ,单位间隔分别为2房、3房(3房分2款,其中1款为套房)。而位于51层的单位则超过1,000平方呎,发展商称为「特色单位」,另52顶层为3层复式单位连天台。汇豪山设有25,000平方呎名为「Forest Place」「汇豪坊」的商场,商场共有两层,佳宝食品市场曾租用地下舖位,而上层则由食肆租用,但商场各商户已于2014年12月迁出。现今佳宝食品超级市场再次迁入,还打算扩充至第二层。","text2":"爪哇集团将这元素注入汇豪山,希望以此制造出怎样的形象?","label":1} {"text1":"丁丁在西藏(法语: Tintin au Tibet ;英语: Tintin in Tibet )是丁丁历险记的第20部作品。作者是比利时漫画家埃尔热。从1958年9月到1959年11月每周连载于丁丁杂志,并在1960年出版书籍。据埃尔热表示,这是他最喜欢的一部作品。《丁丁在西藏》已被翻译成32种语言,一方面被高度批评,另一方面被赞扬,被达赖喇嘛授予真理之光奖。丁丁最重要、也最珍视的中国朋友——张充仁,在乘坐飞机时失事,丁丁梦到张在雪地中满身是血向他求救,出发前往失事地点西藏,决定在冰天雪地中找到他深信还活着的朋友。尽管所有的人都认为这是不可能的事,也无法改变丁丁的决心。1957年10月,埃尔热将绘制好的《红海鲨鱼》的封面寄给了卡斯特曼出版社,并在之后的几天时间构思他的下一部作品。2006年6月1日,在布鲁塞尔,国际声援西藏运动组织达赖授予埃尔热的遗孀范妮·罗德威尔此书真理之光奖。《丁丁在西藏》被改编成电视 、广播 、纪录片、戏剧 和游戏。《丁丁历险记:独角兽号的秘密》(英语:The Adventures of Tintin)是一部2011年上映的美国动画影片,改编自著名比利时漫画家埃尔热的同名漫画作品。本片由史蒂芬·斯皮尔伯格执导,彼得·杰克逊担任制片人。本片获得重大的成功,上画后不久便在夺下法国全年总票房的第四名宝座,更得一致的好评。过了不久更得到第39届安妮奖等一共43项提名,其中包括“最佳动画”。其后更在第69届金球奖获得“最佳动画”名誉。本片被外界传媒认定为“本世纪最成功的真人动画”。本片是史蒂芬·斯皮尔伯格的第一套动画。埃尔热. 中国少年儿童出版社. 译者: 张敏. 2002-4-1. ISBN 9787500760825.","text2":"《丁丁在西藏》已经被翻译成多少种语言?","label":1} {"text1":"澎湖玄武岩自然保留区位于台湾澎湖县,包含锭钩屿、鸡善屿及小白沙屿3个无人岛,主要的保育对象为当地特殊的玄武岩地形景观。澎湖群岛是台湾三大火山群之一,除了花屿之外,各岛大多由玄武岩组成。火山熔岩流在冷却时体积收缩,形成玄武岩的柱状节理,呈现六角柱或多角柱结构。随后由于海蚀及其他地形作用的影响,形成许多高低起伏、变化多端的柱状玄武岩。保留区位于黑潮支流、南海季风流、及潮汐流交会处,邻近海域蕴藏丰富的渔类资源。鸟类大多为冬季候鸟和过境鸟,常见鸟种包括黄足鹬、大苇莺和家燕等。夏季候鸟以苍燕鸥和白眉燕鸥较多,其次为小燕鸥和红燕鸥。澎湖县野鸟学会2014年6月在保留区内的鸡善屿发现有黑嘴端凤头燕鸥在此繁殖,是马祖列岛、韭山列岛后,确认的第三个繁殖地。「世界遗产」登录工作有许多前瞻性的保存观念,为使国人保存观念与国际同步;2002年初,文化部(原:行政院文化建设委员会)陆续征询国内专家及函请县市政府与地方文史工作室提报、推荐具「世界遗产」潜力点名单;其后于2002年召开评选会选出11处(2009年经会勘后,增加至17处)台湾世界遗产潜力点,澎湖玄武岩自然保留区便是其中之一。澎湖玄武岩自然保留区是由地底流出的火山熔岩冷却形成后,形成各式的柱状玄武岩。由玄武岩组成的岛屿受到海蚀作用形成海崖、海蚀洞、海蚀柱、海蚀沟等天然美景,在亚洲地区群岛中更是少见,正符合世界遗产登录标准的第七项。澎湖玄武岩自然保留的地质年代是台湾海峡火山熔岩最活跃的年代,至今仍保留非常独特与优美的玄武岩地景,其雄伟柱状节理及丰富的地形变化符合世界遗产登录标准的第八项。澎湖玄武岩自然保留位处偏远,海流湍急、岩壁陡峭,人迹罕至,因此,每年4月至9月已成为保育类珍贵稀有鸟类的繁殖天堂,2002年更发现濒临绝种的海洋野生动物—绿蠵龟上岸产卵,极具研究与保育价值,符合世界遗产登录标准的第十项。","text2":"澎湖玄武岩自然保留区是怎么形成的?","label":1} {"text1":"古川车站()是一位于日本宫城县大崎市古川车站前大通1丁目,由东日本旅客铁道(JR东日本)所经营的铁路车站。古川车站是JR东日本所属的东北新干线与地方交通线、陆羽东线的交会车站,因此是邻近地区非常重要的交通枢纽。两条路线在古川这里以垂直方向交会,其中陆羽东线的在来线月台位于地面上,新干线月台则是高架设计。在2002年之前,JR货物原本在古川车站内也设有货运专用车站,但在货运站废站之后改以古川货柜中心()取而代之,并在2006年时根据新的政策,将货柜中心改名为古川线外货运站(,或简写为「古川ORS」)。东北新干线与陆羽东线的轨道在本站是以接近垂直的角度立体交差。其中,在来线的陆羽东线为岛式月台1面2线的地面车站,由于月台的容量是根据1980年客运设施迁至目前位置时的需求决定,因此只有约8节车厢的长度。至于新干线月台的部分,是设置于3楼高度的高架车站。虽然构造上采用的是正中央夹著作为通过线用的本线之2面3线构造,但因13号月台并没有铺设轨道,因此实际上只有2面2线配置。","text2":"2006年,货柜中心改名为什么?","label":1} {"text1":"大成食品(亚洲)公司,是一家东亚地区的大型鸡肉供应商。该公司成立于1995年,其总部位在香港,母公司为大成长城企业。该企业现为肯德基在中国大陆最主要的鸡肉供应商,其主要竞争对手为卜蜂。1957年,韩浩然与牟清善在台湾台南县开设黄豆榨油工厂。1960年,投标购入长城面粉公司,从事面粉生产业。1971年,出品沙拉油。1974年,大成集团在台南成立。1978年,大成集团在台湾证券交易所上市,属于台湾农畜食品股票上市公司(,1978年5月20日上市)。1982年,统一企业、泰华油脂、大成长城、三菱商事等企业在台湾台南官田合资成立榨油厂公司「大统益」(,1996年2月9日上市);该厂产出之油品由投资企业各自以其品牌通路销售。1989年,在印尼泗水开办水产工厂,又在中国大陆辽宁沈阳设立饲料厂,广东深圳蛇口设面粉厂。1995年,与丸红株式会社合资在辽宁大连成立「大成食品(大连)公司」,于大连瓦房店市砲台经济开发区从事种鸡饲养、鸡雏孵化、饲料产销、肉鸡屠宰及深加工一条龙产业化经营。2007年,大成食品(亚洲)在香港交易所上市。","text2":"大成食品(亚洲)公司的母公司是哪家公司?","label":1} {"text1":"溪头龙蜥(学名:),又名牧氏攀蜥,俗称竹虎,旧称牧茂氏攀蜥,为飞蜥科龙蜥属下的物种。仅分布于台湾,正模标本采集自南投县的针叶林中,由日本动物学家牧茂市郎采集。其生存的海拔范围为1500至1500米。体长 10~25 公分,最大可达 27 公分。头部有一条粗黑的过眼线,嘴部外缘与腹部体色较浅。雌雄体色差异甚大,体色会随环境而小幅度改变;雄性最显著特征在于身上有许多粗而黑的斑块(斑块颜色有个体差异),鬣鳞明显;雌性体色会因个体差异而有所不同,虽然大多以绿色为主(有些个体也有与雄性相似的粗斑块,但颜色较接近绿色)但有些个体除了斑块外,自头部经背脊中央至尾部均为暗褐色,形成一条宽纵带,称之为「棕背型」。为树栖性生物,日行性,白天常在森林边缘空旷处活动,且常在树丛面向光的叶面。领域性强,雄蜥遇惊扰时喉部会扩张,并做出类似伏地挺身的姿势,多以小型无脊椎动物为食","text2":"溪头龙蜥属于什么类的动物?","label":1} {"text1":"桥杭岩是位于和歌山县东牟娄郡串本町的旅游名胜,吉野熊野国立公园的一部分。是由对著纪伊大岛的海岸西南一列大概850米长,共约40块大小不一的岩石接连所组成。地质学的解释认为,桥杭岩是石英斑岩因火成活动侵入泥岩层中,日后因侵蚀差别效应,较为柔软的泥岩很快就消蚀殆尽,而残存坚硬的柱状石英斑岩所致。日本民俗传承,弘法大师(空海上人)与天邪鬼打赌,看谁能在天亮之前完成从串本到纪伊大岛的石桥,弘法大师役使法力,很快就完成大部分的桥墩(桥杭)。天邪鬼眼见不敌,于是学鸡鸣,弘法大师也以为黎明将至,转身离去,只留下桥墩在当地至今。从桥杭岩远望日出之美夙有定评,被认定是日本朝日百景之一;同时也被日本政府指定为名胜与天然记念物。","text2":"桥杭岩是由多大的岩石拼接而成?","label":1} {"text1":"《伊斯坦布尔:一座城市的记忆》(土耳其文:İstanbul: Hatıralar ve Şehir)大体上是奥尔罕·帕慕克一部忧伤的自传回忆录,于2005年被译成英文,同年获得德国书业和平奖。该书是一段撼动了整个土耳其的巨大文化变迁的记录——现代化与不断退却的传统之间的斗争,它也是一逝去家庭传统的挽歌,更多的,它还是一本博斯普鲁斯,以及伊斯坦布尔和这海峡之间历史的书。动手写此书时,帕慕克处于抑郁症爆发的边缘。在一次访谈中,他说:“当时我的生活,因为很多事情,处在一场危机之中;细节就不多说了:离婚,父亲去世,职业上的问题,这个问题,那个问题,所有的事都很糟糕。如果我软弱的话,一定会得抑郁症的,但每天我起来洗个冷水澡,坐下来,回忆然后写作,永远记得要把这本书写得美。”他的家庭对这本书很是不满,特别是他的哥哥。帕慕克说因为这本书他失去了他的哥哥,他还承认这本书也伤害了他母亲的感情。书中作者个人的回忆和与伊斯坦布尔紧密相关的作家、艺术家交织在一起。书中有一整章献给了十九世纪制作过君士坦丁堡版画的西方艺术家梅林。帕慕克本人最喜欢的伊斯坦布尔作家,他灵感的来源,雅哈雅、科丘、希萨尔和坦皮纳都被他写入了书中。他最钟爱的西方行旅作家对他而言,就像是他个人的奈瓦尔、歌德和福楼拜。本书所采用的插图大部分是由古勒拍摄的,由帕慕克亲自挑选的。帕慕克认为古勒的照片有一种忧伤的韵味。此外,帕慕克还加入了一些家庭照片。","text2":"《伊斯坦布尔:一座城市的记忆》获得了什么奖项?","label":1} {"text1":"李琰(),出生于辽宁省大连市,女子短道速滑运动员、教练员,基督徒。李琰生于大连,两岁时随当时为现役军人的父亲来到黑龙江省的宝泉岭农场,在那里开始学滑冰。1981年,李琰开始在佳木斯市的合江体校接受滑冰专业训练。1982年获得全国少年比赛第二名,被调入牡丹江市体校训练,1987年进入中国国家短道速滑队。1988年,李琰在卡尔加里冬奥会上的短道速滑表演项目中获得女子1000米金牌和500米、1500米铜牌。1992年,李琰又在阿尔贝维尔冬奥会上获得500米银牌。阿尔贝维尔冬奥会结束后,李琰退役,进入东北财经大学国际金融专业学习,毕业后在大连市税务局工作并结婚。2001年,李琰被派往斯洛伐克援教,在两年时间内带领斯洛伐克队成为欧洲强队。2003年,李琰被聘为美国国家短道速滑队主教练,其弟子阿波罗·安东·奥赫诺获得了都灵冬奥会男子500米金牌。2006年都灵冬奥会结束后,李琰拒绝了美国的续约要求,回国担任中国短道速滑队主教练。2010年温哥华冬奥会上,李琰带领王蒙夺得短道速滑500米冠军,使王蒙成为中国第一个在冬奥会上成功卫冕的冠军。在王蒙赛完之后,她跪倒在冰面向教练李琰表示感谢. 周洋夺得短道速滑1500米冠军。打破了韩国短道速滑在这个项目上的垄断,成为首位在冬奥会该项目上夺得金牌的非韩国选手。王蒙,周洋,张会和孙琳琳组成的中国女队以4分06秒610的成绩打破女子3000米接力的世界纪录、勇夺冬奥会桂冠,为中国队夺得本届冬奥会第四枚金牌。这枚金牌不但打破了韩国队接力项目连续四届冬奥会的垄断,同时也是中国代表团在冬奥会历史上首枚团体金牌。王蒙,周洋在女子1000米决赛中角逐冠军。最终,王蒙获得冠军,中国女子短道速滑队包揽了4个所有项目的金牌,创造了冬奥会的记录。王蒙也成为冬奥会史上三次夺冠的第一人。2010年5月,李琰续约,将执教中国国家短道速滑队至2014索契冬奥会。","text2":"第一个在冬奥会上成功卫冕冠军的中国人是谁?","label":1} {"text1":"二氧化三碳是一个无色刺激性气体,化学式为CO,分子中含有四个累积双键。它与CO、CO、CO等其他碳氧化物有重要联系。1873年,Brodie通过对一氧化碳放电,首次制得了二氧化三碳,Marcellin Berthelot创造了低氧化碳(carbon suboxide)这个名称, 而后来Otto Diels表示,更贴近有机化学的名称,如二羰基甲烷(dicarbonyl methane)与二氧代丙二烯(dioxallene)也是正确的名称。二氧化三碳可以通过加热干燥的五氧化二磷(分子式为PO)和丙二酸或丙二酸酯衍生物的混合物来制备。 所以二氧化三碳也可视为丙二酸的酸酐,为丙二酸酐以外的另一种丙二酸的酸酐,即丙二酸的\"第二酸酐\"。此外,丙二酸分子间脱一分子水生成的酸酐也是存在的。其他有关二氧化三碳的合成和反应信息可在Reyerson于1930年出版的综述中找到。二氧化三碳自发聚合生成红、黄或黑色的固体,结构类似于2-吡喃酮。1969年时,曾一度认为火星表面的颜色是由存在二氧化三碳导致的,但后来海盗号的研究推翻了这个说法。二氧化三碳可以作为丙二酸的制剂,同时它还可以增强皮革染料与皮革的亲和力。","text2":"二氧化三碳可以如何制备?","label":1} {"text1":"托定咸(Tottenham,)是位于英格兰大伦敦北部哈林盖伦敦自治市的一个区域。位于查令十字东北6.6英里(10.6公里)处。托定咸的历史最早可追溯至中世纪,当时住了七十户人家。随著伦敦市发展,到1894年,托定咸被列入伦敦市区一部分。现在的托定咸居住著来自世界各地的民族,是一个多元文化热点地区。这里有规模较大的非洲-加勒比人、西非裔、库尔德人、土族塞浦路斯人、土耳其人、爱尔兰人和葡萄牙人社区,总共流行达300种语言。托定咸是英超俱乐部托定咸热刺的主场所在地。热刺曾经赢得过两次联赛、八次足总杯、两次联盟杯、一次欧洲优胜者杯和四次联赛杯冠军。热刺的主场白鹿巷球场就位于该处。白鹿巷球场附近有铁路通过,站名即为「白鹿巷」。距离白鹿巷球场最近的地铁车站则是七姐妹车站。","text2":"托定咸是一个怎样的区域?","label":1} {"text1":"太保,中国古代职官。从周朝开始设置,负责监护和辅佐年幼的国君。召公是第一个太保,《大戴礼记》说:“召公为太保,周公为太傅,太公为太师。”武王去世,成王年少,召公任太保,以长老身份监护。周公东征胜利,建东都成周(今河南洛阳),成王至成周亲政,召公为此作长篇教导,即《尚书·召诰》。周公与召公分陕而治“自陕以西,召公主之,自陕而东,周公主之。” 后召公子孙以太保为氏。春秋后废,汉复置,位次太傅。历代沿置,多为大官之加衔,并无实职。《明史》载“太师、太傅、太保为三公,正一品...掌佐天子,理阴阳,经邦弘化,其职至重。” 在周朝,太保与太师、太傅合称“三公”。朝鲜高丽时代亦设有太保,朝鲜王朝改称大保。","text2":"谁是第一个太保?","label":1} {"text1":"粗鳞鮻(学名:,又称绿背鮻、白鮻为为辐鳍鱼纲鲻形目鲻科鮻属的其中一种。俗名西鱼、白鲻、鲻鱼。本鱼分布于印度太平洋区,包括南非、阿曼、斯里兰卡、巴基斯坦、孟加拉、印度、泰国、日本、缅甸、马来西亚、台湾、新加坡、菲律宾、印尼、澳洲、萨摩亚群岛、万那杜、法属玻里尼西亚等海域。该物种的模式产地在孟买。水深0至40公尺。本鱼体延长,前部圆筒型,后部侧扁。体高大鱼头长,头中大,前端平扁,后部侧扁。口小,亚腹位,口裂呈「A」形。脂性眼睑不甚发达。背面呈深绿色,腹侧白色的,具有3至6条不明显又深色的斑纹沿著上面的列鳞片。背鳍略呈灰色,尾鳍蓝色的有黑色的边缘。背鳍硬棘4至5枚、背鳍软条8至9枚;臀鳍硬棘3枚、臀鳍鳍条9枚。一纵列鳞片28至29枚。胸鳍无色,基部亦无色素斑。体长可长40公分。为近海暖水性鱼类。栖息在浅海或河口咸淡水交汇处。吞食底层淤泥,以摄食其中之矽藻即有机碎屑。食用鱼,但没有乌鱼那么饱满的卵,体型也较小。红烧、煮汤均宜。","text2":"粗鳞鮻一般栖息在什么地方?","label":1} {"text1":"纹腹叉鼻鲀(学名:),俗名白点河鲀、乌规、花规、绵规,为辐鳍鱼纲鲀形目四齿鲀科的一种。本鱼分布于印度太平洋区,包括东非、南非、马达加斯加、模里西斯、红海、塞席尔群岛、留尼旺、马尔地夫、印度、斯里兰卡、泰国、柬埔寨、中国、韩国、日本、台湾、菲律宾、越南、泰国、马来西亚、印尼、新几内亚、澳洲、密克罗尼西亚、库克群岛、马里亚纳群岛、关岛、夏威夷群岛、帛琉、新喀里多尼亚、萨摩亚群岛、东加、万那杜、加拉巴哥群岛、法属波里尼西亚、中美洲、厄瓜多等海域。该物种的模式产地在印度。水深3至50公尺。本鱼体呈圆筒型,被覆由鳞片特化成的细棘;口小,鱼体背部深灰色或黑褐色,散布许多白色圆点;腹部乳白,具数条深褐色细纵带,尾鳍圆形,背鳍软条10至11枚;臀鳍软条10至11枚,体长可达50公分。本鱼属于广盐性的鱼类,幼鱼偏好在河口区活动,游动缓慢,受惊吓时会吸入大量的水和空气,将身体涨大成圆球状,以吓退掠食者。晚上就地而眠,很少躲入洞中。属肉食性,以小型底栖动物为食。本鱼禁止食用,其卵巢和肝脏有河豚毒素,皮肤和肠也有毒。多做为观赏鱼。","text2":"纹腹叉鼻鲀鱼体呈什么型?","label":1} {"text1":"C-Mag弹鼓(Beta C-Mag、Century magazine,意为「世纪弹鼓」,简称C-Mag)是由吉姆·苏利文(Jim Sullivan)设计的100发双室型弹鼓,主要提供给5.56×45毫米北约、7.62×51毫米北约和9×19毫米口径的枪械使用。C-Mag大部份部件以塑料制造,双室左右对称排列的设计,中间转为直排弹匣适配器作连接,依靠旋转弹簧及假弹链作推力将子弹推进至供弹口,比一般的直排弹匣具有更多的装弹量。早期型C-Mag采用全哑黑色外壳,而后期型改用了透明的后部外壳以便于射手快速观看弹药剩余量,亦有其他颜色的外壳提供。以M16突击步枪作举例,一个对应M16、装有100发5.56x45 NATO子弹的C-Mag约重2.1公斤,100发子弹就需要四个30发弹匣作供弹容器,虽然四个共装100发弹匣仍比装有100发的C-Mag为轻,但C-Mag可连续发射而30发弹匣却需更换四次。由美国陆军士兵在阿富汗的试验中发现:C-Mag在的模拟战之中是不可靠的,而且经常有供弹的问题。这问题在充满尘埃和沙土的环境中更加严重。C-Mag还有弹匣释放压力的问题,因为正如上述所指,四个共装100发弹匣仍比装有100发的C-Mag为轻。美国陆军TACOM发出了「地面防范文件」(GPM-02-017),并警告在行动期间其使用。这个GPM在伊拉克自由行动前已经启动。至于接受测试的C-Mag是不是在美国军队内部广泛使用,并没有一个回答。5.56×45毫米北约9×19毫米7.62×51毫米北约目前各国军方对C-Mag的购买意欲不太大,一般的直排弹匣生产便宜、通用性高,运输及配套更简易,C-Mag只装备于一些特种部队和个别士兵自行购入。另外亦有一些采用弹匣供弹的轻机枪以C-Mag作预设供弹具,如德国联邦国防军装备的MG36。1994年9月13日,美国联邦暴力犯罪控制和执法法案通过,只允许平民使用弹匣供弹而且弹数必须等于或少于10发。在这法案通过以前,民间购买和拥有的C-Mag是合法的,但由于数量有限,在当时的C-Mag比较稀有和昂贵。这项禁令在2004年9月13日开始过期,Beta公司已经恢复了民间市场的C-Mag的生产。但是在一些国家、地区之中,已经制定了其对「大容量」弹匣的法律。","text2":"C-Mag大部份部件是用什么制造的?","label":1} {"text1":"欧阳德勋(,),香港歌手、司仪、前唱片骑师,于1984年第三届新秀歌唱大赛季军出身。欧阳德勋在1972年就读新法书院(小学及中学部)太子道分校。他在1984年参加第三届新秀歌唱大赛,并获得季军。同年加入歌坛成为歌手。他在1984年至1989年签约成为华星娱乐有限公司旗下合约歌手。同时于1984年起至1995年签约成为无线电视艺员,并拍摄多个电视剧,如《城市故事》。期间,欧阳德勋于1989年至1993年在商业电台第一台担任全职唱片骑师。后来到了1997年至2010年,欧阳德勋转到香港电台第二台担任音乐节目监制及全职唱片骑师。2012年6月25日,他迎娶同为歌手的蔡立儿,结束长达16年的爱情长跑。圈中的好友如蒋丽萍、宝珮如、李国祥、周慧敏、蔡一智、李司棋、车婉婉、陈奂仁、许志安、苏永康、张衞健、梁汉文、吴国敬、张茜、黎芷珊和梁雨恩等都有出席其婚礼。欧阳德勋表示曾经与当时仍然是女朋友的蔡立儿分开达一年多的时间。其后,因为他重整个人生活且有信仰支持,所以他决定与蔡立儿破镜重圆。","text2":"欧阳德勋的妻子是谁?","label":1} {"text1":"电脑支援协作学习是的一个用电脑辅助协作学习的研究主题。它跟电脑支援协作工作相关。其范围横跨心理学、电脑科学与教育。CSCL 是一个主要的方法,结合协作学习、合作学习的优点,经由网路、电脑,使用远距学习或同地点学习,在网路上或是电子化教室进行学习。CSCL 的目的是:支援学生有效率的共同学习。CSCL 支援学习者之间,意见与资讯的沟通;在学习活动中,协同存取资讯、文件以及授课者、同侪的回馈。CSCL 也支援、促进群组进行、动态活动,可以达到面对面沟通做不到的事情。(比如学习者可以对他们的沟通贴标签。)因为网路授课的需求剧增,教育者、授课设计者了解CSCL 的利益与限制是很重要的。就像很多教育活动,CSCL 活动也很难去评估它的效率、效果。早期的努力著重在电脑中介沟通可能有害的效果,但忽略了电脑中介沟通潜在的利益。历史上,科技创新在传统课堂教室上,缺少足够的证据,可以证明科技创新可以增强学习。CSCL 研究主要的特色在于它多元的方法论:CSCL 研究者采用实验法、逼进法、谈话分析、个案研究等。","text2":"电脑支援协作学习怎么操作?","label":1} {"text1":"阿兹特克人的历史观是由五个不同的「太阳」(也就是世界)所组成,每个太阳纪代表一个时期。世界最初只有水,后来诸神的创造者──奥梅特奥特尔创造了自己。祂是一个双性体,同时有男性面相的奥梅特奎特利,以及女性面相的奥梅西瓦特尔。这两位神生有四子──四个不同颜色的特斯卡特利波卡:特斯卡特利波卡、克特萨尔科瓦特尔、维齐洛波奇特利、以及希佩托特克。这四位神祇各据一方,后来更创造了天地诸神。刚开始,北方由黑色的特斯卡特利波卡统治;东方由白色的特斯卡特利波卡,也就是克特萨尔科瓦特尔统治;南方由蓝色的特斯卡特利波卡维齐洛波奇特利统治;西方由红色的特斯卡特利波卡希佩托特克统治。之后,特斯卡特利波卡付出一只脚为诱饵,四神合力击败水怪西帕克特利,创造了世界。Nahui Ocelotl,意为「太阳豹」。那一期的居民是巨人类,后来,后来世界以居民为美洲豹所噬而结束。Nahui Ehecatl,意为「太阳风」。那一期居民毁于暴风。Nahui Quiahuitl,意为「太阳雨」那一期居民毁于火雨。Nahui Atl,意为「太阳水」。那一期居民毁于洪水,并全变成鱼。一对夫妇幸存了下来。Nahui Ollin,意为「地震阳」。亦即现世。居民会毁于多次地震(或一次超强地震)。","text2":"在初始时,东西南北四个方向分别是由谁统治的?","label":1} {"text1":"杨晨(),名保定,字定孚,又字蓉初,号定夫,晚年号月河渔隐,浙江省台州府黄岩县(台州路桥区)人。同治四年,乡试中举。光绪三年,登进士。光绪三年五月,改翰林院庶吉士。光绪六年四月,散馆后,授翰林院编修。光绪十一年,任顺天乡试同考官。光绪十四年,任山东道监察御史。光绪十九年,任江南道监察御史、四川道监察御史。光绪二十一年,任会试同考官、刑科给事中。杨晨著述甚多,以《三国会要》最为称世,原为二卷,其孙杨绍翰续编为六卷本。治理鉴洋湖有方,后人作《山水记》云:“鉴洋湖,纵一里,横五里。为东南巨浸,中有沙洲芦荻。水多银鱼,长寸许,如小薤叶,色白如银,味最美。击楫中流,恍如剡中风味。”创立诗社月河吟社,生活状态可从其所作《湖墅》诗看出:","text2":"杨晨晚年号什么?","label":1} {"text1":"中欧自由贸易协定(Central European Free Trade Agreement,简称:CEFTA)是中欧到东南欧的一个自由贸易区。中欧自由贸易协定是在冷战结束后,于1992年12月21日在波兰的克拉科夫签订,由维谢格拉德集团(波兰、捷克斯洛伐克、匈牙利)成员国组成。之后在1996年,斯洛维尼亚加入;1997年,罗马尼亚加入;1998年,保加利亚加入;2002年,克罗埃西亚加入;2006年,马其顿加入。其后于1995年9月11日在捷克的布尔诺,2003年7月4日在斯洛维尼亚的布莱德对条文内容进行了修订。2005年以前的条件现在的条件CEFTA的旧成员国最终全部加入了欧盟,因此实际上CEFTA成为了正式加入欧盟之前的一个阶段。目前的塞尔维亚、黑山、阿尔巴尼亚和马其顿也是欧盟加盟候选国。","text2":"中欧自由贸易协定具体指什么的?","label":1} {"text1":"反独裁民主联盟(、),简称反独联或「」(),原名「」或「」,是泰国一个反对人民民主联盟,并支持塔信·钦那瓦的政治团体。媒体一般俗称「红衫军」(),因为他们行动时穿著红色衬衣以示识别。反独裁民主联盟在2006年前总理塔信被政变推翻后成立,支持塔信。在一年的军政府统治时期,反独联组织了多次大型反政府示威。2007年底,亲塔信的沙马·顺达卫经选举上台。但是不久,泰国政治危机再起,沙马和其继任者颂猜·旺沙瓦相继下台,人民民主联盟的势力、泰国民主党在小党支持下重新执政,反独联于是再一次掀起街头抗争运动,要求总理阿披实辞职,举行大选。红衫军于2010年4月开始集结于曼谷百货公司、高级饭店聚集的商业区(Rama I 路周边)及金融区(Silom路周边)附近,百货公司及饭店停止营业超过一个月,曼谷高架电车及地铁亦停止营运,公车因此停驶或改道,先前的冲突迫使部份国家关闭大使馆并发出旅游警示,造成以观光业为主的泰国经济蒙受重大损失,同时也加大阿披实政府的压力,演变成更严重的流血冲突。政府军警在5月流血清场后,红衫军激进派在曼谷市区重要建筑纵火,使曼谷许多重要建筑全毁或半毁,更加深两方对立,表面上示威活动已在曼谷结束,但长期贫富不均、城乡矛盾所造成的社会问题仍旧没有解决,亦对泰国政治埋下不安的种子,而阿披实政府管治的合法性亦受到质疑。2011年7月3日,获他信支持的为泰党在泰国国会选举胜出,成为泰国当时执政党。","text2":"2011年泰国的执政党是哪个党?","label":1} {"text1":"长吻仰口鲾(学名:\"Secutor insidiator\"),又称静仰口鲾,俗名金钱仔,为辐鳍鱼纲鲈形目鲾科的其中一个种。本鱼分布于印度太平洋区,包括东非、马达加斯加、模里西斯、塞席尔群岛、亚丁湾、马尔地夫、印度、斯里兰卡、孟加拉湾、安达曼海、泰国、越南、马来西亚、柬埔寨、台湾、中国沿海、日本、韩国、菲律宾、印尼、澳洲、新几内亚、马里亚纳群岛、马绍尔群岛、帛琉、密克罗尼西亚、所罗门群岛、诺鲁、新喀里多尼亚、法属玻里尼西亚、夏威夷群岛、吉里巴斯、萨摩亚群岛、东加、吐瓦鲁、加利福尼亚湾、加拉巴哥群岛、厄瓜多、巴拿马等海域。水深5至150公尺。本鱼与仰口鲾相似,但其体长卵形,标准体长为体高的2至3倍。头部无特殊斑点,体侧上方有蓝色小点,形成若干横带,体侧下方有小暗点,背鳍末端黑色。背鳍硬棘8枚、软条16枚;臀鳍硬棘3枚、软条14枚。体长可达15公分。本鱼在岸边至近海之沙泥底海床上皆可发现其踪影。常成群觅食小型无脊椎动物。小型食用鱼,较不具经济价值,多做下杂鱼处理。","text2":"长吻仰口鲾的俗名是什么?","label":1} {"text1":"马自达Zoom-Zoom广岛球场(),又常简称为马自达球场(),是一个位于日本广岛县广岛市南区南蟹屋的公营棒球场。该球场是日本职棒的比赛场地之一,也是隶属于中央联盟的广岛东洋鲤鱼队之主球场,于2009年开幕,拥有33,000人的容量。马自达球场的前身,是曾经存在于广岛市中区的第一代,1957年启用的该球场在2009年时结束职棒比赛场地的用途后,被作为业余棒球比赛场地短暂使用至2010年后封馆,并于2012年起进行拆除工程。作为取代,于南区修筑的新球场在落成时原本暂订命名为新广岛市民球场,但在实际启用前,以广岛作为根据地的当地汽车制造大厂马自达以每五年3亿日圆的金额取得球场的,并根据该车厂的广告标语「ZOOM-ZOOM-变得更想乘坐」()而将球场命名为马自达Zoom-Zoom广岛球场。马自达球场的主要公共运输门户为JR西日本所经营的,球场距离车站南口约10分钟的步行时间。","text2":"马自达Zoom-Zoom广岛球场简称是什么?","label":1} {"text1":"北海油田是世界著名的石油集中出产区,每日生产大约600万桶。位于大西洋的陆缘海——北海,它是介于欧洲大不列颠岛、挪威和欧洲大陆之间,所出产之石油为沿岸英国,挪威和荷兰等国所享有,也是布兰特原油指数主要标的。1970年代之前,以波斯湾为核心的亚洲中东地区,一直是西方工业国家石油能源的主要供应者。随着第二次世界大战后西方国家经济的恢复和繁荣,石油需求量逐年上增,使得亚洲石油国提高购价。这让各大石油巨头不愿购买,双方因此积怨。1973年第四次中东战争爆发,阿拉伯产油国以不满美国为首的西方国家支持以色列之立场为由,以能源为武器,宣布「石油禁运」,导致欧美油价暴涨,并相继波及各个经济领域,部分导致七十年代全球经济大衰退。因此,以英国为首的北海沿岸国家将目标转向沉寂多年的北海油田。在此之前,北海是欧洲航运要道,且海底地形复杂,海上气候恶劣,一直未有大规模地质勘探活动进行。而北海在1959年首先于荷兰近海发现格罗宁根气田(荷兰语:Groningen),此后更进入大规模开发阶段,先后于1969年发现埃克非思科油田(英语:Artsen non-Cisco),1971年发现布伦特油田(英语:Brent),大油田的相继发现,不仅缓解英国与挪威等西欧国家的能源短缺,而且更使挪威成为除加拿大和俄罗斯之外的第三大非OPEC石油出口国。北海石油的发掘,使得正处于疲软时期的英国、荷兰与丹麦等工业国家的经济获得帮助,也使得英国经济在「油荒」年代于西欧一枝独秀。北海原油因其品质高,产量稳定,所以迅速成为欧洲重要的能源供应地,因此欧洲原油交易市场多以每桶北海布伦特石油(约159升)作为市场参考价格。世界石油市场约6.4%的供货来源于此。","text2":"北海油田日生产石油多少桶?","label":1} {"text1":"艾莉雅·沙伯(,),于美国纽约市出生,是美国神童之一,也是全球最年轻大学教授。9岁时,沙伯已经获得跆拳道黑带。11岁,跟美国洛克兰管弦交响乐团(Rockland Symphony Orchestra)合作演出,并曾进入朱利亚德学院研习。10岁时,沙伯进入纽约州立大学石溪分校就读,并于14岁时,以第一名成绩取得应用数学学士学位。毕业后,进入了宾州德瑞索大学(Drexel University)的材料科学暨工程学博士班。2008年,18岁的沙伯获聘为韩国首尔的建国大学(Konkuk University)先进科技融合系细胞科学的专任教授。2008年4月21日,被金氏世界纪录誉为世界史上最年轻的教授,打破300年来的纪录。英国科学家牛顿的门生麦克劳林()在1717年,仅以19岁年龄开始担任教授。","text2":"2008年,沙伯被聘为什么职务?","label":1} {"text1":"穆罕默德·纳希德(迪维希语:މުހައްމަދު ނަޝީދު ,拉丁转写:Mohamed Nasheed,),马尔代夫共和国前任总统。纳希德1989年毕业于英国利物浦约翰摩尔斯大学,1991年因批评当局而遭逮捕下狱,次年被判处三年有期徒刑,1993年出狱后又多次遭到当局逮捕。2000年,纳希德曾一度代表马累而当选为马尔代夫国会议员,但六个月后因被指控曾偷盗过国家文件而再次接受当局的处罚。2003年,纳希德流亡海外,积极组建马尔代夫民主党。2004年,纳希德获得英国政治避难权,次年回到马累。他的到来使得马尔代夫民主党人气大增,因此在2005年,他又再一次被下狱,被认定为恐怖分子。2008年,在总统大选中一举击败加尧姆,当选马尔代夫第三任民选总统。2012年2月7日,在国内爆发警察叛变及持续数周的群众示威后,纳希德在电视记者会上宣布辞去马尔代夫总统,并将总统职权移交给副总统穆罕默德·瓦希德·哈桑。但辞职之后不久,纳希德突然公开宣称“辞职是被人用枪顶着头的强迫辞职”。","text2":"穆罕默德·纳希德为何又一次下狱?","label":1} {"text1":"贾特拉帕蒂·希瓦吉国际机场(,,,)是印度共和国马哈拉施特拉邦首府孟买的国际机场,是该国乃至整个南亚地区最大、最繁忙的机场,以马拉地帝国开国君主贾特拉帕蒂·希瓦吉命名。第一航厦Santa Cruz营运国内线,第二航厦Sahar营运国际线。为印度航空和捷达航空的主要枢纽。机场亦有不经停直达班机前往香港及中国大陆的北京。圣塔克鲁兹皇家空军是在20世纪30年代在德里建成此机场,机场占地约1500英亩(610公顷)并设有三条跑道,面积比1942年至1947年第二次世界大战期间,几个皇家空军中队所在地朱胡机场更大。停机坪位于09\/27跑道南侧,该地区现时被称为“旧机场”,其中设施包括印度航空、印度航空工程和MIAL通用航空的飞机维修库。直至1946年,皇家空军开始将机场交给民航局开始营运民航航班时,皇家空军把两个旧有废弃机库转换成客运大楼,一个机库被用作处理国内航班,另一个用于处理国际航班,客运大楼两侧亦设有海关、移民局检查柜台和候机室。印度航空在位于两个机库附近当时拥有自己的客运大楼,并以其处理乘客。在开始营运的第一年,客运大楼每天约处理著六班民航客机。","text2":"1946年,皇家空军对机场管理作了什么改变?","label":1} {"text1":"孔平仲,字义甫,一作毅父。新喻(江西省新余市)人。孔子四十七代孙,属于“临江派”。北宋诗人。生卒年不详。治平二年(1065年)进士,授分宁〔今江西省九江市修水县)主薄。熙宁三年(1070年),任密州(今山东省潍坊市诸城市)教授,曾任秘书丞、集贤校理,又提点江浙铸钱、京西刑狱。六年四月充任秘书阁校理、朝奉大夫。绍圣年间贬为惠州(今广东省惠州市惠阳区)别驾,流放英州(今广东省清远市英德市)。元符三年(1100年)七月,授朝奉大夫。崇宁元年(1103年)八月,管勾衮州太极观,不久卒。著有《续世说》、《孔氏谈苑》 、《珩璜新论》、《释稗》等。兄弟孔文仲、孔武仲皆有文名。","text2":"著有哪些著作?","label":1} {"text1":"德国BR45型蒸汽机车是一种货运机车,亦是德国“标准蒸汽机车”之一。45型机车是德国有史以来制造过的功率最大的蒸汽机车。该型机车由汉寿尔工厂于1936到37年之间生产,在1940年之前全部入役。本来还有103辆45型机车的订单,但这些订货于1941年取消。这是由于二战爆发后德国机车工业集中生产力生产类似于BR52这样的简化版“战争机车”。45型机车共有28辆,编号从45 001 -到45 028。战后,BR45型机车开始频繁出现由于超压而造成的锅炉损害。为此,德国联邦铁路从1950年起为45 010, 45 016, 45 019, 45 021以及45 023 等机车安装了有燃烧室的下火箱以及下给加煤机。东德也对45型进行了改造,不过他们的方法是提高锅炉压力。然而这种改造没有收到预期的效果,并且使这些机车在1959年就早早退役(东德有很多蒸汽机车都使用到了7、80年代)。45型机车其实从设计上就有问题,主要是锅炉结构十分脆弱,就像之前的06型。在锅炉和主要部件更换以后,该型机车的真正实力才显现出来。在45型使用的最后阶段,其主要任务就是在44型难以担当的重型货运中发挥重要作用。到1968年,德国联邦铁路仅有3辆45型仍在使用(45 023 、45 010、 45 019),这些机车编为045型。45型退役后只有45 010号保存了下来,被收藏于纽伦堡交通博物馆。2005年,该馆发生严重火灾,这唯一的45型在大火中严重损毁。目前,有关方面正计划对该车进行修缮。","text2":"该型机车是在哪里被生产的?","label":1} {"text1":"奥威尔斯合成(Auwers合成)是苯并呋喃与苯甲醛发生缩合、溴化生成2-溴-2-(α-溴苄基)苯并呋喃酮,然后经醇碱处理重排,转变为黄酮醇的反应。由卡尔·冯·奥威尔斯于1908年发现。由于这个反应对反应物和产物的结构要求都比较高,并且反应物二溴化物比较难制备,产率也不是很高(70%),因此它在有机合成中的应用并不十分广泛。一个应用的例子见下图:一个可能的机理如下图所示。氧原子上的孤对电子使溴离子离去,生成被氧原子稳定的碳正离子,然后氢氧根进攻,得到一个具有半缩酮结构的中间体。此中间体发生分子内E1cB反应,氢离子被夺去,接着氧负对不饱和酮进行麦克尔加成,新生成的烯醇负离子再将剩下的一个溴离子挤出去,得到最终产物。当原料苯并呋喃环中的氧原子有间位的取代基(如甲基、甲氧基)时,产物不是黄酮醇,而是2-苯甲酰基苯并呋喃酮的衍生物。","text2":"奥威尔斯合成是由谁发现的?","label":1} {"text1":"路姜(),中国足球运动员,司职中场。1990-1996年路姜在北京什刹海体校学习,1996年加入北京国安青年队。路姜在2000年进入北京国安一线队,曾入选国青队、国奥队,曾经是一名问题球员,2002年因泡吧事件与国安队友张帅一同被开除出国奥队,但同年路姜还入选了中国国家足球队在与叙利亚的友赛中为国家队上场比赛。在2008年中超联赛第18轮北京国安主场与武汉光谷比赛的第93分钟,武汉队员李玮峰准备解围,结果皮球打在路姜身上弹出,李玮峰收不住脚,左脚踩在了已经倒地的路姜的胸口上,而就在李玮峰转身后,右脚又有个蹬踏的动作踩到了路姜,路姜极为恼火,立即起身掐住李玮峰的脖子将其放倒,这个动作被当值主裁判看到,并出示红牌将路姜直接罚出场外。事后,两人都得到足协8场停赛,罚款8000的处罚。这处罚也令武汉光谷不满而退岀中超联赛。2009年,路姜帮助北京国安获得中超联赛冠军,是球队首座顶级职业联赛冠军。2012年,路姜加盟中甲联赛球队湖南湘涛。2013年,转投另一支中甲联赛球队北京八喜。足协重罚京汉战冲突当事人 李玮峰路姜各禁赛八场","text2":"路姜在哪一年加入北京国安青年队?","label":1} {"text1":"杨安儿()金末“红袄军”起义首领。山东益都(今山东青州)人。原名杨安国,以卖鞍材为业,人称杨安儿。其妹杨妙真,号李姑姑。大安年间,聚众起义,后投降金国,担任刺使,官至防御使、副都统。大安三年(1211年)回到山东,成为红袄军首领,贞祐二年(1214年)克莱州(今山东掖县),刺史徐汝贤献城投降,又进攻登州(今蓬莱),登州刺史耿格交纳州印,杨安儿建立政权,置官属,改元“天顺”。贞祐二年(宋嘉定七年,1214年)金人与蒙古乞和,集中兵力镇压山东,仆散安贞率猛将完颜霆(李二措)、黄掴阿鲁答「花帽军」镇压,安贞击败杨安儿于益都城东,杨安儿转至莱阳,所占州县相继失陷。年底与汲政乘舟入海,欲走岠嵎山(今栖霞东北),以图再起,为舟人曲成陷害,堕水而死。","text2":"杨安儿的原名是什么?","label":1} {"text1":"石守谦(),艺术史研究者,曾任国立台湾大学艺术史研究所教授、兼所长,国立故宫博物院副院长、院长。现在是中央研究院历史语言研究所特聘研究员。2012年当选为第29届中央研究院院士。石守谦自文化史的角度,反思中国绘画史的发展历程,特别关注画史上风格转变的文化脉络、区域性的画史发展,及画史上「雅」、「俗」观念的互动与区别。其研究成果曾二度获得国科会杰出研究奖 (1994-96 及 1996-98)。著有专书《风格与世变》(台北:允晨文化,1996年)与论文多篇,在中国艺术史界享有极高声誉。近期出版《从风格到画意──反思中国绘画史》(台北:石头出版社,2010年),总结近十多年来的研究成果。2007年被控涉入故宫改建扩建工程弊案,士林地检署求处15年有期徒刑,全案在2009年4月30日一审结束,石守谦获判无罪。另外,在故宫南院招标案中,石守谦等人被指控涉嫌违法修改招标规格、内定特定厂商、泄漏招标金额。经其他投标厂商向行政院公共工程委员会提出申诉后,公共工程委员会撤销标案,使得故宫遭得标厂商诉请法院求偿,让故宫损失3900多万元。士林地检署侦办故宫南院弊案,于2009年8月25日侦查终结,石守谦等人被依贪污等罪起诉,求处重刑。2011年9月1日,士林地方法院针对上述故宫南院招标案,一审宣判石守谦等五人无罪,判决书中指出该案议价、签约程序合乎流程,查无不法,五人均无犯罪事实。士林地院并于判决书中多次点出检方起诉时的诸多违误、无稽与偏颇之处。2011年11月29日,针对故宫扩建案,台湾高等法院于二审时,仍采信原审见解,判决石守谦等人无罪。高等法院审理认定,例如在小标绑大标部分,故宫决议以规划、设计、监造三合一方式办理招标,乃九十年三月第一次评选委员的共识和决议,并无不法,也无操纵评选结果等事证,因此仍判石守谦等人无罪。2012年9月20日,台湾高等法院对故宫南院案进行二审宣判,仍判决石守谦等5名被告无罪。","text2":"在故宫南院招标案中,石守谦最终判决是什么?","label":1} {"text1":"服务研习(SERVICE-LEARNING)为一种将「服务」与「学习」相互结合的一门课程,以事先规划的社会服务活动与结构化设计的反思过程,让学生运用课堂所学贡献社区;同时可透过服务的过程中得到启发及省思,学习课堂中学不到的知识与经验,服务与学习二者在课程中具有某种程度的平衡关系,它是一种「从做中学」(Learning by doing)的学习经历,强调服务与学习目标同等重要,对所有服务与被服务的人都能加强其完成目标,以达「互惠」之功效。据文献记载,「服务研习」一词最早是由美国南部地区教育董事会(Southern Regional Educational Board)于一九六七年首先提出,经过美国各级学校的推广发展后,衍伸出许多的解释和做法。而我国比较被广泛接受的定义为:「由学校与社区结合,共同协助学生应用所学知能去服务他人,并且在服务过程中不断的学习成长。所以服务-学习是学校教育的一环」(林胜义,2002)。","text2":"服务研习主要强调什么?","label":1} {"text1":"北京师范大学天文系,创建于1960年,由原属物理系的部分师资力量和学生组成,是中国大陆北方的第一个天文学院系。天文系系办位于北师大北太平庄主校区教十楼2层东侧办公区。大部分教研室和教室办公室位于教十楼二层、三层和四层,研究生实验室位于教十楼三层和教九楼五层。校内有三座天文台。现任系主任是朱宗宏教授,系党总支书记是张保州教授。目前天文系有教职员工18人。其中教授7人,副教授、高级工程师8人。教师中具有博士学位的10人。天文系本科每学年招生20人左右,学制4年。由于中途转系调整等因素,每学年获理学学士学位的毕业生15-18人左右。硕士生每学年招生15人左右,学制3年。博士生每学年招生少于10人,学制3年。本科生第四年和硕士生第2年可以根据个人成绩申请硕博连读,硕博连读的整体学制为5年。天文系有良好的科研氛围,本科生二年级开始可以申请“本科科研基金”跟随系内老师从事天文学科研工作。研究生按照负责的导师进行课题化培养。天文系学生团体包括学生会和天文学社。学生会组织全系学生的日常活动,天文学社是天文学爱好者社团,在北师大全校并联合北京市各高校开展天文和科技普及活动。","text2":"博士生的学制是几年?","label":1} {"text1":"花煎()是一种朝鲜半岛传统的甜煎米饼,上面可食用的时花制成。韩语固有词又称为、、。在不少传统节日如上巳节、重阳节等都会食用。从高丽王朝起,朝鲜人在一些节日里有玩「花煎游戏」(,)的传统习俗,并会食用花煎。每逢上巳节,妇女就会带著叫燔铁()的一种厚煎板去郊游踏青,她们会采摘杜鹃花或其他盛开的花来做花煎,当中以杜鹃花煎(固有词为)最具代表性。花煎常会与加入同种花瓣的花菜(,一种把水果加入蜜糖水或五味子汁里制成的朝鲜传统甜品)一同食用,例如以杜鹃花煎与杜鹃花菜同吃。到了重阳节,人们也会举行花煎游戏,这时食用的花煎会以菊花瓣制作,亦即菊花煎(),配以菊花酒()或柚子花菜()。除了杜鹃花煎和菊花煎外,任何可吃的时令花卉都可用作制造花煎,例如春季常见以沙梨花制成的梨花煎()、樱花煎(韩语固有词:)、堇菜花煎(固有词:)。初夏时则多会食用蔷薇花煎()。鸡冠花煎(韩语固有词:)则多在秋季食用。当没有适合制作花煎的花时,会以水芹、艾草、石耳、红枣等做成花形装饰放在煎饼面团上。","text2":"花煎都有自己的时令,初夏多食什么花煎?","label":1} {"text1":"白鹤洞街道是中国广东省广州市荔湾区下辖的一个街道(旧属芳村区),位于荔湾区南部(广州市西南部)。辖区总面积3.1平方公里,下辖9个社区,常住人口3.4万人。白鹤洞得名是由于清朝时年,该地土地肥沃,水草繁盛,引来白鹤觅食以及群居,清廷亦因此曾于该地建筑招鹤亭。后来逐步有人迁徙而至。白鹤洞有很多美丽的传说,例如白鹤仙等。白鹤洞区域内有两所名校紧依在一起,是为广州市培英中学(校本部)以及广州市真光中学,均于十九世纪七十年代所建立。此外还有广船中学等多所学校。此外,广州钢铁厂在此也曾有广钢中学、广钢小学,但随着钢铁厂的迁移以及关闭等,广钢中学则由真光中学接收,成为其附属的小学以及初中部,并更名为真光实验学校。而广钢小学则物归原主交由基督教女青年会接收(民国时期为女青年会所拥有土地)。广州钢铁集团有限公司前身广州钢铁厂也曾于此建立生产基地。2008年经由国家发改委审批,获准归入上海宝钢。白鹤洞邻近鹤洞大桥以及珠江隧道,交通便利。白鹤洞地区房价较同为广州南部的海珠区房价便宜。也因此,目前有很多海珠区人士过江购房居住。","text2":"白鹤洞区有哪两所名校?","label":1} {"text1":"柳林镇位于中国山东省菏泽市巨野县西南部,地处巨野、成武、定陶、牡丹区三县一区交界处,共有人口6.2万人,辖49个行政村,总面积123平方公里。万福河流经其南部。邮编:274922。1958年9月,根据中共中央、毛泽东主席关于在农村建立人民公社的号召和指示,巨野县于8日~12日制定了人民公社区划和调整方案,合并23个乡,成立11处人民公社。柳林公社为其中之一。1983年4月29日,经山东省政府批准,公社改乡。1995年10月23日,撤销巨野县柳林乡,设立柳林镇。2001年2月6日,撤销张表乡,其行政区域并入柳林镇。现任镇党委书记:孙学兵,镇长:国清强。主产传统作物小麦、棉花,近年大蒜、桑蚕业发展迅速。柳林镇为菏泽市养牛示范镇。鲁西黄牛,号称中华国宝,曾经呈现村村建场、户户养牛的局面。现有柳林镇畜牧园区、五一养殖小区、鲁西特种动物养殖场等示范畜牧园区。2005年被菏泽市评为全市畜牧生产先进乡镇。林产品加工业成为该镇的主导产业。全镇现有以食品加工、酿造等为主的企业超过百家。详见企业列表。巨野煤田的规划区内。巨野煤田共七对矿井,其中位于柳林镇的万福煤矿目前已开工建井。万福煤矿地质储量5亿吨,工业储量3.26亿吨,可采储量2亿吨,井型180万吨/年,服务年限70年。镇上有初级中学柳林镇第一中学、柳林镇第二中学,其中柳林镇第一中学为巨野县规范化学校。部分村庄设小学,镇上有中心小学。根据公元1840年(清道光20年)《巨野县志》记载:
1933年《中国建设》刊登的“查勘报告”中载述:
万福河形成于宋朝。北宋期间,黄河屡决滑县,东行经东明、菏泽,直趋“菏泽湖”。然后分为两股:一股经济水故道,东北出大清河入海;一股经古菏水,东南流经鱼台入泗而注淮。在此基础上,进一步演变为清代的柳林河和后来的万福河。下属49个行政村:张表村(又名张表集)、柳林村(又名柳林集,为镇政府驻地)、徐堂、刘楼、前王庄、徐庄、义和集、张楼、葛集、崔庄、吕坑、孟庄、毛胡同、王西庄、马楼、东葛集、西葛集、前陈楼、后陈楼、张庄、火头刘、西蒋海、东蒋海、张堂、大李楼、杜庄、戴楼等。其中,张表村原为张表乡政府驻地。辖区人口最多的姓氏为王姓。按姓名、村庄、职务顺序。(资料来源:《巨野县志》)","text2":"柳林镇的主要农作物是什么?","label":1} {"text1":"《丁丁与字母艺术》(法语: Tintin et l'alph-art ;英语: Tintin and Alph-Art ),又译《丁丁与阿尔发艺术》,是《丁丁历险记》的第24部作品,也是最后一部作品,作者是比利时漫画家埃尔热。故事的主要题材是现代派艺术,埃尔热在1983年执笔这部作品过程中不幸逝世,导致这部作品成为未完成的作品。为了逃避毕安卡的爱情之访,哈达克意外地结认新派黑人艺术家拉莫·纳许(Ramó Nash),并替丁丁约定艺术专家弗卡特(Fourcart)向他「爆料」,然而不久就意外身亡。在几次逃过被枪毙的险境下,丁丁不但揭发弗卡特是被杀,并发现了新兴邪教的教主恩达迪涅·阿卡斯(Endaddine Akass),就是支助纳许的艺术赝品贩售家。不久,毕安卡邀请丁丁及哈达克前往阿卡斯位于意大利的豪宅,阿卡斯却把丁丁囚禁,准备把他用热塑胶活活灌死,造成凯撒扩展式的艺术品...收到米卢求救纸条的哈达克及时救回丁丁,然而他们却无法脱离阿卡斯的魔爪,其后阿卡斯向二人表明自己就是拉普洛斯。庆幸机警的纳许及时报警,并制止拉普洛斯把丁丁及哈达克吊死,然而拉普洛斯却因而失足跌下山坡至死。另外,在埃尔热的原稿出现的艺廊职员Martine Vandezande,郤在Rodier同人版的结尾向丁丁表示好感,引起丁丁迷一点点争议。","text2":"这部作品的结尾为什么会受到丁丁迷一点点争议?","label":1} {"text1":"是日本科乐美集团下游戏部门所开发的电玩游戏,于1986年9月26日在FC磁碟机平台上发售。本作后来的故事骨干后来延伸到其他主机上,比较著名的是大型电玩板的《闹鬼之城》(\"Haunted Castle\")、《超级恶魔城IV》(\"Super Castlevania IV\",此为超任版同名作品之英译)、日本X68000电脑上出的同名作品,及后来出在PS上、以X68000版为基础的《恶魔城年代记》(\"Castlevania Chronicles\")。不管上述的作品为何,通通都是讲西蒙‧贝尔蒙多挑战德古拉的名战传奇。西元1691年,在中世纪欧洲的一个和平小国『特兰西瓦尼亚』地区,德古拉在上次被打倒的一百年后,被心智邪恶的邪教徒借由黑弥撒仪式(x68000版像是捏爆一颗从人取下来的新鲜心脏,将血洒到德古拉的棺材)而复活,身为伟大的灭魔名族之后,西蒙·贝尔蒙多(,Simon Belmont)再次拿起家族传承的圣鞭「吸血鬼杀手」(,Vampire Killer),顶著跟先祖、祖父一样的强健身体,以及一票副武器,闯进待兵以发的恶魔城讨伐德古拉,最后他也成功了,德古拉再次被打倒,但是德古拉却在被太阳光照到化为灰之前,对西蒙下了一个诅咒……主角西蒙携带家传祖鞭,依靠魔法力量消耗品——爱心来施展圣器攻击,一路破关斩将","text2":"恶魔城是哪家集团开发的游戏?","label":1} {"text1":"是一部2008年上映的香港惊吓片电影,由彭氏兄弟监制,钱江汉执导。电影分为三个部分,分别由演员李璨琛、江若琳及阮民安主演。电影在2008年4月3日上映。根据《电影检查条例》,本片列为青少年及儿童不宜(IIB)。本电影分为三部分,三个故事均与不详数字13有关。Au Cun是一名病态赌徒,终日醉心于麻雀。他不但将怀孕的太太置之不理,还被炒掉了工作,负债累累。他的赌友黄泉给了他一个会带来好运的古钱币,只要将钱币含在口中,打麻雀时“13幺<\/nowiki>精灵”便会庇佑他,令他赢足13日。可是,他每天只可在赢一铺大牌,而且不能退出,必须在13天里赢足13局。Au Cun含着钱币打麻雀,幸运果然一直眷顾他。然而,他很快便发现,输家一个接一个死去。结果他被精灵缠身……究竟“13幺<\/nowiki>精灵”是否要用人命来换取好运呢?陆颖(江若琳饰)在梦中醒来,便发觉自己处身于一个食人族世界!而她的男友Sunny(阮民安饰)正被人屠宰烹煮,她母亲在肢解她的父亲,然后将他煮成一道佳肴,颖跑到街上,却被一群丧尸包围,她被吓至昏厥……当她再次神智清醒后,赫然发现自己的身体已经失去了一些东西……陈吉隆(阮民安饰)是一个双失青年,没有学历,一事无成。他被13万美元奖金吸引,决定参加一个名为《吓死你》的电视真人SHOW。谁有勇气在鬼屋里待上13日就是最终胜利者。可是当游戏开始后,他才发现游戏的真正恐怖超乎想像。","text2":"此电影三个部分的共同之处是什么?","label":1} {"text1":"远藤实(),二战后日本歌谣界最具代表性的作曲家之一,曾写下五千多首作品,包括《北国之春》、《高三学生》等传世名曲。由著名演歌手千昌夫原唱的《北国之春》,以及这首歌被蒋大为和余天等翻唱的中文版受到很多华人的喜爱。出生于日本东京府东京市向岛区(现在是东京都墨田区向岛),第二次世界大战期间被疏散到新潟县西蒲原郡内野町(现在是新潟市西区内野)居住生活。在新潟一带被称作“”(Kadozuke)的演艺风格对其后作为作曲家的远藤实的人生给予了很大的影响。 所谓“”就是在别人家门口前进行的诸如越后狮子等街头演出,以收取金钱和物品的卖艺表演。1949年,十七岁时的他怀着成为歌手的梦想来到东京,一边在街头吉它卖艺,一边自学作曲。1956年, 在日本水星唱片(、Nippon Mercury Record)公司为歌手藤岛桓夫创作了热卖歌曲「月娘晚安」(「お月さん今晩わ」),这同时也是他作曲家生涯的第一首歌曲。远藤实一生共谱写五千余首作品(大部分是演歌),其作品孕育了日本歌谣界非常出名的诸如舟木一夫,千昌夫,森昌子等演歌歌手。1988年,在夏威夷接受心脏搭桥手术。1994年,远藤实获得日本唱片大奖功劳奖,并设立了远藤实歌谣音乐振兴财团。此外他还在新潟市开设了音乐资料馆“实唱馆”2003年,当选为第一位因推广大众音乐获选为日本政府认定的“文化功劳者”。2003年,在朝鲜绑架日本人问题上,为鼓励遭到绑架的受害者及其家属,远藤实作词作曲写了「寒冬过后必是春天」(「春の来ない冬はない」),并灌录成唱片赠送给他们。2008年12月6日10时54分,因急性心肌梗塞在东京都内的一家医院去世,享年七十六岁。日本政府为表彰远藤实经由大量的乐曲而对大众音乐发展所作出的功绩, 于2008年12月6日封其正四位及授予旭日重光章。更于2008年12月26日及2009年1月23日举行的关于授予国民荣誉奖的内阁会议上正式决定将国民荣誉奖颁发给著名作曲家远藤实。他是日本国民荣誉奖的第十六位获奖者,也是第四位获得该奖项的作曲家(前三位作曲家分别是古贺政男,服部良一,吉田正)。日本的国民荣誉奖通常授予“广受国民爱戴,为社会带来希望业绩的卓著”人物。上一次颁奖是前日本首相森喜朗将奖项授予2000年悉尼奥运会女子马拉松冠军高桥尚子。","text2":"十七岁时远藤实怀着梦想来到哪儿?","label":1} {"text1":"高雄展览馆站位于台湾高雄市苓雅区,为高雄捷运环状轻轨的捷运车站。车站代码为C8。站区位于新光码头公园内。设置二侧式月台。此站原计划站名「新光路站」,行政院已于2008年3月20日通过环状轻轨兴建案,2009年5月11日公告招商流标,2010年1月21日二次公告招商再度流标。之后高雄市政府决定采OT模式办理,并将环状轻轨与水岸轻轨两案整并,此站计划名称改名「新光公园站」。后于环状轻轨第一阶段兴建时,高雄市政府捷运工程局举行征名活动,活动第一阶段共有「市立图书馆站」、「高雄展览馆站」、「新光码头站」等三个建议站名,第二阶段则以「高雄展览馆站」、「市立图书馆总馆站」、「新光码头站」三名称进行票选,最终于2015年3月29日公告以「高雄展览馆站」为确定名称。于2016年6月26日启用。","text2":"环状轻轨第一阶段共有哪些建议站名?","label":1} {"text1":"《扶桑花女孩》是一部真实事件改编的日本电影,改编自日本常磐市Hawaiian Center(现为Spa Resort Hawaiian)的成立过程,由日本导演李相日执导。故事背景是在昭和40年(1965年),日本福岛县常磐以煤矿维生的小镇逐渐没落,而镇长打算兴建「夏威夷度假中心」来拯救小镇的失业危机。度假中心的负责人吉本(岸部一德 饰)特地从东京聘请一位舞蹈老师平山圆香(松雪泰子 饰)教导矿工的女儿们练习草裙舞,但此事受到当地居民的大力阻挠。谷川纪美子(苍井优 饰)与木村早苗等人却努力跟著舞蹈老师学舞,不久早苗的父亲被解雇,早苗因为家庭因素不得不放弃自己的梦想。其后随着大家的努力,扶桑花女孩逐渐打开知名度,纪美子的母亲看到自己的女儿练舞的神情而产生了怜惜。她终于了解到,练习跳舞也是一种努力的付出。为了借用暖炉,她替渡假村四处奔走,表演当天还在舞台前面忘情的为自己的女儿加油。第30届日本电影金像奖:2006年蓝丝带奖:电影旬报十佳奖","text2":"这部电影获得什么奖项?","label":1} {"text1":"冰青是日本女性声优、歌手。所属经纪公司为贤Production。千叶县出身。日本大学艺术学部演剧科毕业。2004年春天开始以歌手身分活动之际,把艺名改成「冰青」。在这之前都以本名横手久美子活动。2009年开始作为昭和歌谣组合milleluci的一员活动中。1998年1999年2000年2001年2002年2003年2004年2005年2006年2007年2008年2009年2010年2011年2012年2013年2008年1997年1998年2000年2001年2002年2003年2004年2005年2006年2007年2008年2009年2010年2011年2012年2016年ドラマCD2011年1.君のもとへ1.冬花火(2010.4.21)","text2":"冰青的本名叫什么?","label":1} {"text1":"山原猫(学名:'),又名南美山猫或安第斯山猫,是一种细小的野生猫科。牠们是传统上两种没有亚种的猫科之一。牠们的数量估计少于2500只,但却只有有限的保育工作。山原猫的外观像雪豹,栖息在海拔3500-4800米的地方,远远高于林木线。牠们约有家猫的大小,但尾巴较长,毛色呈银灰色,有斑纹及斑点。牠们体长约60厘米,尾巴长42厘米,肩高36厘米,体重5.5公斤。山原猫是所知最少及最为稀少的猫科之一,对牠们的认识都是来自仅有的观察及一些标本。牠们未被驯养。估计牠们只生活在秘鲁、玻利维亚、智利及阿根廷安地斯山脉的高山上。山原猫喜欢栖息在高山环境,其分布亦受当地的猎物情况所影响。总有效群体大小可能是低于2500只,因失去猎物及栖息地、被猎杀及没有多于250只的亚群等缘故而有下降的趋势。由于牠们只居于高山上,人类居住的山谷成为了屏障,分隔了牠们的群族。牠们在玻利维亚及智利因祭祀用途而受到猎杀。牠们的数量稀少,较同样生活在高海拔的南美草原猫更为稀疏。在其分布地,牠们的遗传多样性非常低。","text2":"山原猫又被称为?","label":1} {"text1":"17173网是一家位于中国大陆,旨在提供电子游戏及电子游戏产业类新闻与资讯的网站。同时17173网还提供供游戏玩家交流的论坛、视频分享及下载服务等内容。2001年3月,17173网成立。最初网站名为“网游天下”,且为个人网站。2001年5月,17173网被网龙公司收购,收购金额不详。2003年10月,搜狐斥资2050万美元收购17173网。2004年12月,17173网推出首届17173游戏动漫嘉年华活动。2005年5月,17173网标志和吉祥物诞生。2007年3月,17173网发起首届世界网游日。2008年5月,17173网举行首届中国网游竞技大赛。2011年12月,畅游以1.625亿美元收购17173.COM。自畅游收购17173网后,有分析师认为17173网在游戏厂商广告兼具双重身份或将导致广告主的放弃, 同时也引发业内对于17173网未来公正性和客观性的质疑。2013年5月15日,17173员工猝死曾引发过有关“过劳死”的讨论,但事后证明员工猝死与17173无关。2006年期间,17173曾因旗下产品涉嫌侵权而被天空游戏网诉至法庭,但并无下文。","text2":"17173创建之初的名字叫什么?","label":1} {"text1":"《夺命捉迷藏》(),日本小说家山田悠介的出道恐怖小说。2008年改编成真人版电影在日本上映。杀光全国500万姓「佐藤」的人!西元3000年某日,国王突然决定进行「要像捉迷藏一样,捉住全国姓佐藤的人」为期七日的游戏。全国100万人作为「鬼」,配备「佐藤探知风镜」的特殊的风镜,探索附近姓佐藤的人,进行追捕。被鬼捉住的人,将被带去王国的绝密收容所杀死。住在横滨市内的大学生·佐藤翼,看著父亲和朋友在自己眼前被杀,发誓要存活下来。身为田径选手的翼,为了找出年幼时离散的妹妹,在死的赛道上疾驰。「夺命捉迷藏」1周期间,翼能从鬼中平安逃脱吗?与原作相比设定上有很大的变更,比如平行世界理论的设定。制作费约1亿日圆。为2008年电影版的半年后的故事,其中有人物换角,饰演佐藤爱的演员为吉永淳,而饰演佐藤洋的演员为三浦翔平。此年度分别于5月12日、5月19日、5月26日接连上映《夺命捉迷藏 3》、《夺命捉迷藏 4》及《夺命捉迷藏 5》,由UNITED CINEMAS制作。2015年7月11日上映,由特林德尔·玲奈、筿田麻里子、真野惠里菜共同担任女主角,园子温执导。此版本与之前设定皆不同,为追杀「全国女高中生」为主题。正式上映后,才发表由斋藤工饰演片中的白发老爷爷。2013年4月9日,于埼玉电视台、神奈川电视台、千叶电视台等电视台,以「夺命捉迷藏 THE ORIGIN」的标题预定播出。","text2":"2013年4月9日有哪些电视台以「夺命捉迷藏 THE ORIGIN」的标题预定播出?","label":1} {"text1":"律伦(,)是英国英格兰的郡、单一管理区。除了行政总部渥咸、小镇阿平厄姆外,郡内都是人口不多的小村庄。律伦既是名誉郡,又是单一管理区,无论把它看待成那种身分,它的人口和面积都分别是38,300、382平方公里。律伦伯爵、律伦公爵是英格兰贵族的头衔。首位律伦伯爵Edward of Norwich, 2nd Duke of York在1385年受封。首任律伦伯公爵约翰·曼纳斯在1703年受封,现任公爵大卫·曼纳斯在1999年受封。《1894年地方政府法案》生效后,律伦郡下辖3个乡区(Rural district):奥克姆区(Oakham Rural District)、阿平厄姆乡区(Uppingham Rural District)、凯顿乡区(Ketton Rural District)。1911年,奥克姆在奥克姆乡区分割出来,成为urban district。《1972年地方政府法案》在1974年4月1日生效,拉特兰成为莱斯特郡的非都市区。1992年成立的英格兰地方政府委员会(Local Government Commission for England)建议律伦成为单一管理区,不再是非都市区,在1998年4月1日生效至今。律伦郡在2001年人口普查中统计得人口34,560,相比起1991年人口普查统计得人口33,228,上升了4%。下图显示英格兰48个名誉郡的分布情况。律伦东北与林肯郡相邻,东与剑桥郡相邻,南与北安普敦郡相邻,西、西北与莱斯特郡相邻。","text2":"英格兰贵族的头衔有哪些?","label":1} {"text1":"在物理学中,费米气体(Fermi gas),又称为自由电子气体(free electron gas)、费米原子气体,是一个量子统计力学中的理想模型,指的是一群不相互作用的费米子。费米气体是理想气体的量子力学版本。在金属内的电子、在半导体内的电子或在中子星里的中子,都可以被视为近似于费米气体。处于热力平衡的费米气体里,费米子的能量分布,是由它们的数目密度(number density)、温度、与尚存在能量量子态集合,依照费米-狄拉克统计的方程式而表征。泡利不相容原理阐明,不允许两个或两个以上的费米子占用同一个量子态。因此,在绝对零度,费米气体的总能量大于费米子数量与单独粒子基态能量的乘积,并且,费米气体的压力,称为「简并压力」,不等于零。这与经典理想气体的现象有很明显的不同。简并压力使得中子星或白矮星能够抵抗万有引力的压缩,因而得到稳定平衡,不致向内爆塌。在低温下,玻色原子气体可以形成玻色-爱因斯坦凝聚(Bose-Einstein condensation, BEC),这是由爱因斯坦在1925年的理论而预言的。费米子由于泡利不相容原理,不能形成BEC。但可通过Feshbach共振,利用磁场调节费米原子间的相互作用,使费米子配对转变成玻色型粒子而形成BEC。由于前述定义忽略了粒子与粒子之间的相互作用,费米气体问题约化为研究一群独立的费米子的物理行为的问题。这问题本身相当容易解析。一些更深奥,更进阶,更精密的理论,牵涉到粒子与粒子之间的互相作用的理论(像费米液体理论或相互作用的微扰理论),时常会以费米气体问题为研究的开端。","text2":"什么是费米气体?","label":1} {"text1":"鳃棘鲈(学名:)为鮨科鳃棘鲈属的鱼类,俗名:豹纹鳃棘鲈、花斑刺鳃鮨、豹纹豹脍、东星斑、七星斑。分布于印度洋到太平洋中部、北至日本南部的菲律宾、台湾岛以及南海诸岛的海南岛等,属于暖水性沿岸鱼类。其一般生活于珊瑚礁。颜色鲜艳,是很凶猛的「掠食者」,会吞掉任何它能吞掉的鱼。也会吃掉观赏用的甲壳类动物;不会骚扰它们吞不下去的鱼。食物包括各种海水鱼、鱿鱼、贝类及虾。需要经常提供一些活食,足够的营养会保持它的色彩。水深3至100公尺。虽然曾是美味的食用鱼,鱼肉可作生鱼片。在市场上均切成小块出售,头可煮清汤、背肉宜清蒸,腹肉红烧最美,但过度滥捕,数量已开始下降,已列为受保护的鱼类,政府严厉禁止渔民抓捕,因此已经成为稀有品种的鱼类。 IUCN已评级为近危。","text2":"这种鱼的数量下降的原因是什么?","label":1} {"text1":"布兰卡契小堂(意大利语:Capella dei Brancacci)是佛罗伦萨卡尔米内圣母大殿内的一座天主教小堂,由于其绘画时代有时被称为“文艺复兴初期的西斯廷小堂”。布兰卡契小堂的设计者是彼得·布兰卡奇,始建于1386年。公众进入需要通过邻近的修道院(由伯鲁乃列斯基设计)。小堂和大殿分别单独向外开放,因此有不同的开放时间,要从小堂进入大殿其余部分参观非常困难。图案装饰的赞助人是菲利斯·布兰卡奇,彼得的后裔,曾担任佛罗伦萨驻开罗使节到1423年。当他回到佛罗伦萨,他雇用Masolino da Panicale画他的小堂。Masolino的助手,21岁的马萨乔,比Masolino 年轻18岁,协助他,但在Masolino离开前往匈牙利担任国王的画师期间,于是由马萨乔负责小堂的建造。这时Masolino回来,向他的优秀学生学习。然而,马萨乔在完成小堂之前被召往罗马,27岁时在罗马去世。后来由菲利皮诺·里皮完成小堂的其余部分。不幸的,在巴洛克时期,一些绘画作品被看作是不合时宜,在前面安置了一个坟墓。","text2":"布兰卡契小堂后来由谁完成其余的部分?","label":1} {"text1":"日耳曼尼亚(拉丁语:';德语:')是古代欧洲的一处地名,位于莱茵河以东、多瑙河以北,同时也包括被古罗马控制的莱茵河以西地区。地名来自高卢语,由罗马共和国统帅尤利乌斯·凯撒最先使用,以指代莱茵河以东的居民,意为“邻居” 。日耳曼尼亚地区分布着多个部落,以日耳曼人为主,同时还包括凯尔特人、波罗的人、斯基泰人和古斯拉夫人。在随后的数个世纪中,该地区的部落组成受民族融合和民族迁徙的影响而几经变化。公元前2世纪之前,人们对居住在北欧的部落情况知之甚少。约公元前320年,来自马西利亚的探险家毕提亚斯航行至不列颠和北欧沿海,成为最早能够区别凯尔特人和日耳曼人的地中海人。尤利乌斯·凯撒对日耳曼人、罗马人和高卢人之间的文化差异进行了描述:高卢人虽然好战,但还算文明,而日耳曼人则是野蛮未开化的民族,对罗马人和高卢人是严重的威胁。有关日耳曼尼亚最详尽的资料是罗马帝国历史学家塔西佗的《日耳曼尼亚志》。古罗马人将日耳曼尼亚分为两个部分,即莱茵河以西及以南、由罗马人占领的小日耳曼尼亚和莱茵河以东的大日耳曼尼亚。小日耳曼尼亚又被分为两个行省:下日耳曼尼亚省(相当于今天低地国家的南部,首府科隆)和上日耳曼尼亚省(相当于今天的瑞士和法国的阿尔萨斯大区,首府美因茨)。公元前12年,罗马帝国皇帝奥古斯都开始征战大日耳曼尼亚,并委任日耳曼尼库斯和提比略管理该地区。至公元6年,罗马人已控制易北河以西的日耳曼尼亚地区,计划继续征服整个地区并将其并入罗马帝国疆域。公元9年,日耳曼人在条顿堡森林战役中击败罗马人,使罗马人的计划破灭。随后,奥古斯都以莱茵河和多瑙河为依托建立起帝国的边界。英语中“德国(Germany)”的即来自于日耳曼尼亚,而德国人自己称自己的国家为“Deutschland”。其它现代语言如希伯来语、保加利亚语、意大利语、希腊语、罗马尼亚语、维吾尔语和俄语中对德国的称谓同样来自日耳曼尼亚。","text2":"日耳曼尼亚在哪里?","label":1} {"text1":"超异能冒险(Race to Witch Mountain)是部2009年奇幻冒险电影,翻拍自1975年的《巫山大逃亡》(Escape to Witch Mountain),根据 Alexander Key 1968年的同名原著改编,由安迪·菲克曼(Andy Fickman)执导,巨石强森、安娜苏菲亚·罗伯与亚历山大·路德维格(Alexander Ludwig)主演。2008年三月在洛杉矶进行拍摄制作,2009年3月13日北美首映,台湾于4月10日上映。倒楣的的士司机杰克·布鲁诺(巨石强森饰)碰上一对正在逃亡的少年兄妹莎拉(安娜苏菲亚洛饰)及薛夫(阿历山大卢域饰)。由他们上车的一刻开始,亡命旅程立即展开。原来莎拉及薛夫是身怀异能的外星来客,更掌握拯救地球未来的秘密关键;因此成为政府特工及地下组织恶势力的猎物,更有其他外星战队向他们展开追杀。杰克为了救己救人,必须在限时之前,把两名外星少年送回他们在地球的太空基地…金·理查兹和艾克·艾森曼是1975年旧版的男女主角,当年演出都还是孩童青少年,两人皆在新版中客串演出。2007年七月华特迪士尼影业找导演安迪·菲克曼执导,与 Matt Lopez 共同编剧,新版将1975年《巫山大逃亡》改编为「现代重拍版」,与旧版故事是大相异庭。八月巨石强森确选为主角,电影计划于2008年三月开拍。导演表示并非纯粹旧版重拍,确切地说是制作出一个全新章节的大魔域(Witch Mountain,早期译为巫山)世界。原著小说内容充斥著「黑暗惊悚」题材,导演企图将1975年旧版电影当时删减的要素加进新版电影里。新版中加入UFO专家、军队、CIA等各方人马于电影中,更是把外星生物概念加进影片作为主要核心。电影正式定名为《超异能冒险》(Race to Witch Mountain),在洛杉矶开拍。影评家给予《超异能冒险》普遍中等的评价。电影评论网站烂番茄,基于74篇评论中有38%的影评家给予正面的评价。Metacritic网站基于23篇评论,平均分数100分只拿到50分。而IMDb分数投票10分得到5.7分。","text2":"超异能冒险翻拍自哪部电影?","label":1} {"text1":"京岘山位于江苏省镇江市市区东南郊,山高300米左右,受爆破采石以及市区拓宽影响,现已被铲平为新区的一部分。山上靠近陈家湾方向原有宋朝著名将领宗泽和夫人陈氏合葬冢,在市区拓宽时被移至今天的纪念碑处。宗泽安葬夫人时曾赋诗“一对龙湖青眼开,乾坤倚剑独徘徊,白云是处堪埋骨,京岘山头梦未回。”京岘山的来历据说是在周朝,有风水先生看到这里云雾缭绕,紫气升腾,就断定这一带要出王侯将相。到了公元前221年,秦始皇最统一中国。为保其一统江山万古千秋,秦始皇传旨,急驱3000多名赭衣囚徒,星夜赶往京岘山,名为修筑驰道,实是劈山削岭,断其 “龙脉”,败其“风水”,并煞费苦心地将谷阳更名为“丹徒”,顾名思义,即赤衣囚徒之地。对此,清朝乾隆帝在下江南时曾经写《京岘山》诗:“凿京岘命赭衣徒,名象虽存迹久芜,未及临洮东属海,长城筑罢竟亡胡。” 对秦始皇不会引导使用人才进行嘲讽。","text2":"京岘山位于何地?","label":1} {"text1":"科誉(香港)有限公司(英文名:POSH Office Systems(Hong Kong)Ltd.),POSH命名源于: Project Office de Sun Hing,简称 POSH, 科誉的前身是一间香港公司新兴钢具厂有限公司 (Sun Hing Steel Furniture Factory Ltd.),创立于1959年。科誉在1992年于香港成立有限公司,并以POSH为注册商标,销售、研发及设计时尚办公室家具。科誉的创办人、首任主席、行政总裁兼首席执行官是严志明。在1990年代起与美国上市公司合作,互相协助开发大中华市场分销网络。现时科誉(中国)在中国大陆的产品陈列室包括在北京、南京、上海、杭州、西安、宁波、成都、东莞、昆明、深圳、广州等城市。科誉的香港总部及旗舰展示中心设于黄竹坑黄竹坑道33-35号17楼和湾仔合和中心,而香港陈列室包括在湾仔轩尼诗道、中环香港贸易中心及中环前荷李活道已婚警察宿舍元创方。科誉的香港仓库及物流中心位于鸭脷洲海湾工业中心,而生产基地在中国大陆东莞及中国大陆宁波。2011年3月11日美国正式收购科誉。2014年6月1日科誉宣布即日起严志明将退任科誉主席、行政总裁及首席执行官职务,只保留首席设计师一职,科誉行政总裁职务由美国亚太区副总裁Mr. Jeremy Hocking暂代兼任。科誉营业总监郑敏坚擢升为商务总监,管理科誉所有销售工作,科誉其他主要部门,包括﹕行政、营运、市场策划及产品管理等均由美国的亚太区所属范畴总监兼任主管。在香港电影《无间道》中,香港警察写字楼所采用的家具为POSH出品。尤其男主角之一的刘健明所坐的大班椅。","text2":"科誉的创办人、首任主席、行政总裁兼首席执行官是谁?","label":1} {"text1":"卡路士·艾坚哈顿·艾华斯(,)是一名千里达足球员,担任右翼,现效力英冠球队米禾尔。卡路士·艾华斯出道自千里达球会Defence Force。他在2000年以25万镑加盟-{zh-hans:雷克瑟姆; zh-hk:域斯咸;}-。艾华斯在域斯咸度过五年,上阵165次。在2005年,他免费加盟英冠球会卢顿。他成了球迷的宠儿。在2007年1月2日日,艾华斯以150万镑加盟新特兰,签约三年半。他在新特兰的表现抢镜,接连在对锡菲联、高云地利及伯明翰城取得入球。在对修咸顿的比赛更射入致胜一球。在对般尼的比赛入球,更把新特兰带返英超。2008\/09年球季由于马白兰基加盟影响艾华斯的出场机会,于10月2日被外借到狼队3个月。11月20日仅为狼队出赛6场后被提早召回。2009年9月1日艾华斯转投英冠球会叶士域治,转会费135万英镑。2012年3月30日艾华斯与叶士域治续约留效多一季。卡路士·艾华斯曾代表千里达出战2006年德国世界杯。在对瑞典,他的表现令他成为全场最佳球员。2011年10月10日艾华斯在法庭承认于停牌期间驾驶,事发于9月26日,前一天他的妻子因怀孕并发症而入院,艾华斯送女儿上学,但未能截获的士而决定亲自驾驶。其妻于10月8日早产诞下只有两磅重的双胞胎。10月13日艾华斯被判入狱12星期,但缓刑12个月,同时罚款2,500英镑和需要进行200小时社会服务,他亦被所属球会处罚两星期薪金。","text2":"艾华斯在域斯咸的五年时间里共上阵了多少次比赛?","label":1} {"text1":"高永是台湾的漫画家及作家。出生于台中县大甲镇(今台中市大甲区)。身高170cm,体重55kg,血型O型,星座巨蟹座。毕业于国立政治大学法律学系。代表作为《梵天变》、《星座刑事》、《隋唐英雄传》。1987年政大道南文学奖小说类首奖得主,1987年以《焚梦》出道于《欢乐漫画》,作品画风优美细腻、人物美形,作品常出现星座、占星、塔罗牌等奇幻神秘的主题,而《梵天变》则是以佛教为体材,在政大法律系之所学也展现在《星座刑事》中。喜欢的漫画家有萩尾望都、安彦良和、井上雄彦、池上辽一等。2005年任台北市漫画从业人员职业工会第三届理事长。漫画家冠良为其弟子,后期高永的作品大多与冠良合作创作。以创作《百无禁忌 Miss阿性》著名的漫画家李勉之(Amin)曾任高永的助手。从就读高中就开始进行漫画创作,1981年参加第三届小咪漫画新人奖以《罪与罚》获得第3名,之后数次参加并获得小咪漫画新人奖的奖项。","text2":"他的作品风格如何?","label":1} {"text1":"鼻头角灯塔,位于中华民国新北市瑞芳区之鼻头角,西元1897年建造完成 ,为日治时期日本政府兴建的六角形铁塔,二战损毁,1971年改建为圆塔。灯塔乃海上航行重要的指标,而台湾最早的灯塔是澎湖西屿的石造灯塔, 清朝乾隆年间兴建 。日本统治台湾初期, 以生铁建造灯塔, 常在日本铸造好之后, 再运到台湾组立, 如富基角灯塔及鼻头角灯塔。 日治后期灯塔, 改用钢筋混凝土, 可塑性高造型丰富, 基座或与办公室合一, 如新北市的三貂角灯塔。鼻头角灯塔建于1896年(日本明治29年),西元1897年建造完成开始点火。1906年(日本明治39年)由于塔基岩盘出现裂痕,将原塔迁移至30公尺外的现址。旧塔原为日治时代兴建之六角形铁塔,二次大战遭美军攻击严重损毁,1971年改筑为钢筋混凝土之白色圆塔。","text2":"鼻头角灯塔建于什么时期?","label":1} {"text1":"银彭纳石首鱼(学名:Pennahia argentata),又称白姑鱼,俗名白口、帕头、黄顺,为辐鳍鱼纲鲈形目石首鱼科的其中一个种。本鱼分布在西北太平洋区,包括泰国、日本、韩国、台湾、马来西亚、中国东海、南海、香港、越南等海域。水深3至40公尺。本鱼头呈钝尖形,吻不突出,口裂大,端位,倾斜,上颌约等长。上颌外列齿扩大为犬齿,内列细小绒毛状齿。下颌齿2列,内列齿较大。体侧上半为褐紫色,下半部为银白色,背鳍软条部中间有一白色纵线,硬棘部有黑边。鳃腔黑色,口腔白色。背鳍硬棘11枚、背鳍软条25至28枚;臀鳍硬棘2枚、臀鳍软条7至8枚。体长可达28公分。本鱼为近海中下层鱼类,主要栖息在沙泥底质海域。每年4至6月,成群向沿岸回游产卵。肉食性,以甲壳类、海胆、小鱼为食。食用鱼,肉质佳,适合油炸和清蒸食用。","text2":"银彭纳石首鱼的体长可以达到多长?","label":1} {"text1":"信贷挂钩票据(Credit-linked note),是一种连动债和结构性产品。作为一种特别的金融衍生工具,虽然名称为债券,事实上是一种高风险的金融衍生产品。这类产品的发行商一般利用来自投资者的款项买入抵押品,然后作出若干“掉期”财务安排,从而令发行商得以将产品的潜在整体回报提升至高于传统银行存款的回报水平。这些“掉期”安排通常与若干拥有高信贷评级的参考机构有关连。在香港、台湾及新加坡等地曾有名为和等一般被认为属于信贷挂钩票据的产品发售,但认为发行的迷你债券在其三层结构内隐藏了不同于一般连结资产的债务抵押债券的,因此并不应被算作信贷挂钩票据。迷你债券以期权和套利为基础,即是以保险金的原理,赚取中间的差价。虽被称为「债券」,但实际上只是一堆信贷挂钩票据。它先成立一间空壳公司作为迷你债券的发行人,然后再向投资者销售一大堆信贷挂钩票据,包括一些债务抵押债券(),当中有许多高风险的信贷违约掉期,再混入一些较稳建的蓝筹股,以杠杆方式抵消其风险。发行人利用客户的资金投资,赚取这些证券定期产生的现金流,客户则可收取比一般银行存款较高的利息。","text2":"「债券」的实际上是什么?","label":1} {"text1":"港铁巴士K68线是由港铁巴士营运的西铁线及轻铁接驳巴士,属循环线,行走元朗工业邨及元朗公园之间,途经朗屏邨、元朗市中心等地区。本路线于2004年1月18日起配合656线分拆为两条路线而投入服务,方便往来元朗工业邨至元朗市及元朗公园的乘客。最初路线往元朗公园方向是以顺时针方向行走,但是绕经朗屏站公共运输交汇处迂回,加上在妈横路或元朗安乐路已能够接驳西铁,因此于2004年4月25日便不再绕经朗屏站公共运输交汇处,往元朗公园方向也改以逆时针方向行走。配合元朗南进一步发展,由2009年10月12日起,往元朗公园方向绕经十八乡路(翘翠峰、南元朗官立小学)及榄口村路,成为唯一一条全日服务该带的公共交通服务。此外,本路线是现时唯一一条只于元朗新市镇内(包括元朗工业邨和元朗南)服务的路线。本线初期使用富豪奥林比安(2xx)、富豪B10M(4xx)单层巴士和丹尼士三叉戟三型(7xx)行走,但后来引入亚历山大丹尼士Enviro 200 Dart单层巴士,把富豪B10M(4xx)调走,单层用车改派亚历山大丹尼士Enviro 200 Dart单层巴士(9xx),但现时本线用车大多以亚历山大丹尼士Enviro 500 MMC(5xx)及富豪B9TL(3xx)为主,亚历山大丹尼士Enviro 200 Dart数目渐渐减少,令本线用车都是双层巴士,而丹尼士三叉戟三型(6xx)、Enviro 400(14x)、亚历山大丹尼士Enviro 500 11.3米(825-833)间中会行走本线。由于本路线两边总站位置都是偏远的,加上并非驶经元朗区心脏地带,除繁忙时间外客量只属一般,不过由于沿途没有其他巴士路线竞争,接近独市生意,所以仍有一定需求。现时本线主要客源是出入元朗工业邨、横洲杨屋村、朗屏邨、赵聿修纪念中学、元朗公立中学以及公园北路一带住宅的乘客。","text2":"港铁巴士K68线是何时被拆分成两条线路的?","label":1} {"text1":"面波,又称表面波(),是沿不同介质(常常是两密度不同的流体)界面传播的波。折射率梯度波导下的电磁波也可以成为表面波。地波(沿地面传播的无线电波)也是一种表面波。在地震中,面波也是地震波之一,在地球表面扩散,因此测量面波震级也是除测量矩震级外的另一种估计地震规模的方法。对于浅源地震,中国习惯使用面波震级,美国则习惯使用体波震级或矩震级,这三个震级无法进行对比,但可以通过经验公式换算。在无线电学中,有表面波特征的电磁波也被称为地波,长波即是其中之一,它可以沿着地球表面传播。由于地球表面和大气的折射率不同以及大气中的折射率渐变,低频的电磁波(如长波)会发生衍射,造成了“电磁波没有沿直线传播”的假象。另外,电离层也能反射长波。","text2":"表面波可以怎样形成?","label":1} {"text1":"安森美半导体(ON Semiconductor,)是一家财富美国1000强半导体供应商。公司的产品系列包括电源和信号管理、逻辑、分立及定制器件,帮助客户解决他们在汽车、通信、计算机、消费电子、工业、LED照明、医疗、军事\/航空及电源应用的独特设计挑战,既快速又符合高性价比。公司在北美、欧洲和亚太地区之关键市场运营包括制造厂、销售办事处及设计中心在内的世界一流、增值型供应链和网络。总部位于美国亚利桑那州的凤凰城,2016年销售收入达39.07亿美元,位列在全球20大半导体公司之一。安森美半导体前身是摩托罗拉集团的半导体元件部门,于1999年独立上市,继续生产摩托罗拉的分立晶体管,标准模拟和标准逻辑等器件。安森美半导体制造以下的各种产品:安森美半导体的各个产品部门:","text2":"安森美的总部在什么地方?","label":1} {"text1":"贺万里( ),中国著名画家,美术理论家,中国装置艺术理论研究奠基人。生于安徽合肥,籍贯河北。以中国山水画泼彩山水泼墨闻名。南京艺术学院美术学博士毕业,后任职于扬州大学艺术学院 教授、硕士生导师。中国美术家协会会员。江苏省美术家协会会员。他兼任清代扬州画派研究会会长,以及石河子大学新疆大山水画研究所副所长,还有石河子大学西部绘画研究所研究员。主要研究领域涉及中国古代绘画理论、中西美术史、中国现代艺术、动漫理论等,尤以装置艺术理论见长。20世纪90年代以来,在《文艺研究》、《美术观察》、《美苑》、《艺术探索》、《中国画研究》、《宁夏大学学报》等公开出版的有影响的中文核心期刊或专业性刊物上发表学术论文50余篇。《装置艺术研究》 1999 中国文联出版社《光影与色彩》(译著) 2001 江苏美术出版社《永远的前卫:中国现代艺术的反思与批判》 2003 郑州大学出版社《谁在附庸风雅》2005 上海书画出版社《鹤鸣九天:中国古代绘画的功能论》(其博士毕业论文)《美术刀客:贺万里艺术论评集》2005 天津人民美术出版社《中国山水画通鉴:维扬异趣》 2006 上海书画出版社《中国当代装置艺术史(1979-2005)》2008 上海书画出版社其著作《装置艺术研究》详细,系统地首次向中国读者以及中国艺术家介绍阐明了 装置艺术 的主要内涵和特点,为中国的装置艺术研究奠定了理论基础。在2001年获得海峡两岸书画名家邀请展金奖。在2006年纪念孙中山诞辰140周年中国书画名家作品大展金奖。多幅作品被个人、企事业和美术馆收藏,在《美术报》、《中国书画报》、《美术大观》、《书与画》、《艺术当代》、《国画家》等刊物发表,并被专文推介。1.海峡两岸书画名家交流展,2000年,台北、南宁。一等奖2.纪念孙中山诞辰140周年中国书画名家邀请展,2001,台北、南宁。金奖3.当代著名书画家精品年展,2002,台湾阿里山4.贺万里泼彩山水画暨学术文献展,2006,扬州5.美术刀客----贺万里泼彩艺术精品展,2007,广州6.千岛湖名家名湖书画精品展,2007,杭州淳安7.首届中国美术教师艺术作品年度展,2007,沈阳。7.走近番禺·2008全国书画名家新春雅集,2008,广州8.周积寅教授从艺五十周年暨七十华诞师生书画展,2008,南京","text2":"《装置艺术研究》一书的出版有哪些重要的意义?","label":1} {"text1":"和广北,中银香港控股及中银香港前副董事长兼总裁,亦为南洋商业银行、南洋商业银行(中国)、集友银行及中银人寿前董事长。和广北为中银香港于香港银行公会之指定代表,并于2008年担任该会主席。和广北于1979年在北京第二外国语学院毕业,取得学士学位,并于1985年在美国德克萨斯州大学达拉斯分校取得国际管理学硕士学位。和广北毕业后加入中国银行,曾担任不同职位,并先后在纽约分行及巴黎分行工作。1999年至2004年担任中国银行常务董事;自2000年至2003年期间担任中国银行副行长。2015年3月6日中银香港公布,和广北因年龄原因,辞任该行副董事长、执行董事兼总裁等职位,","text2":"和广北取得了哪个学校的学士学位?","label":1} {"text1":"百想艺术大奖(;)是由主办的韩国代表性综合艺术奖项,与韩国电影青龙奖和韩国电影大钟奖并称韩国三大影视奖,从1965年举办第1届,至2017年为第53届。大奖名称源于1954年韩国日报主创者的好人百想命名。大众文化艺术的颁奖典礼,是由前年度颁奖典礼后一个月起,至当年颁奖典礼前一个月播放或上映的大众文化作品中,分为电视及电影部门作品及演出者进行审查。从1965年第1届到1973年只有电影及演剧部门,1974年设立电视部门,2002年起删除演剧部门,只剩电影和电视大奖。百想艺术大奖曾数次更名,1983年第19届为止为「韩国演剧电影艺术奖」,1985年第21届为止为「韩国演剧电影TV艺术奖」,1986年第22届为止为「韩国百想艺术大奖」。1987年第23届起更名为「百想艺术大奖」,沿用至今。2006年主办方由韩国日报的中央日报收购,改由中央日报文化事业主管。审查委员分为电视及电影两部分,每部门选出委员各7名。这些委员是由主办方选定或委托,部分亦同时兼任执行委员。此外,各部门需选出一名审查委员长,然后再从其中选出一名全体(电视及电影)审查委员长。在审查委员长的主持下,决定公开的大奖以及各部门奖。各部门大奖会在颁奖典礼当天决定以及发表。《第1届韩国戏剧电影艺术奖》(1965年)","text2":"百想艺术大奖的审查委员分为哪两部分?","label":1} {"text1":"鳃棘鲈(学名:)为鮨科鳃棘鲈属的鱼类,俗名:豹纹鳃棘鲈、花斑刺鳃鮨、豹纹豹脍、东星斑、七星斑。分布于印度洋到太平洋中部、北至日本南部的菲律宾、台湾岛以及南海诸岛的海南岛等,属于暖水性沿岸鱼类。其一般生活于珊瑚礁。颜色鲜艳,是很凶猛的「掠食者」,会吞掉任何它能吞掉的鱼。也会吃掉观赏用的甲壳类动物;不会骚扰它们吞不下去的鱼。食物包括各种海水鱼、鱿鱼、贝类及虾。需要经常提供一些活食,足够的营养会保持它的色彩。水深3至100公尺。虽然曾是美味的食用鱼,鱼肉可作生鱼片。在市场上均切成小块出售,头可煮清汤、背肉宜清蒸,腹肉红烧最美,但过度滥捕,数量已开始下降,已列为受保护的鱼类,政府严厉禁止渔民抓捕,因此已经成为稀有品种的鱼类。 IUCN已评级为近危。","text2":"鳃棘鲈喜欢生活在什么地方?","label":1} {"text1":"西藏南路隧道,是位于上海黄浦江上的一座公路隧道,供2010年上海世博会专用,2011年2月1日22时起,隧道正式向社会车辆开放。西藏南路隧道为南北走向,北岸位于黄浦区,南岸位于浦东新区,全长2.67公里,设双向四车道,设计时速40公里。主线北侧起于黄浦区西藏南路中山南路口,南侧起于浦东新区高科西路云莲路口。西藏南路隧道在浦西设有龙华西路匝道,在浦东设有雪野路、浦东南路等两组匝道。2009年7月11日凌晨开始,因应打浦路隧道全封闭大修,西藏南路隧道先期建成的东线临时对公交车辆开放双向通行,供原行驶于打浦路隧道的数条公交线路行驶。2010年1月4日,公交车恢复打浦路隧道通行。在世博期间,隧道只供世博园区内车辆通行,不对社会车辆开放。2011年2月1日22时起,隧道正式向社会车辆开放。2月2日起,新辟中国馆班车3线行走隧道,以方便游客参观世博会中国馆续展。9月25日起,又新辟中国馆班车4线行走隧道,连接世博浦东和浦西原区,以方便游客参观月亮船(原世博会沙特馆)和上海世博会纪念展","text2":"西藏南路隧道的南岸位于哪里?","label":1} {"text1":"斑鱵(学名:)为辐鳍鱼纲鹤鱵目鱵科的一种。本鱼分布于印度西太平洋区,包括东非、留尼旺、塞席尔群岛、阿曼、印度、泰国、越南、马来西亚、模里西斯、中国、日本、台湾、印尼、澳洲、密克罗尼西亚、新喀里多尼亚、帛琉、万那杜、菲律宾、东加、萨摩亚群岛等海域。其主要栖息于水的中上层。该物种的模式产地在红海。水深0-30公尺。本鱼体略呈长枪状,稍侧扁。吻细长且上颌短于下颔,眼大,体背部青黑色,腹部银白,鳞片大但易脱落。侧线上具5-6个明显黑斑,下颔尖端为红色。尾鳍深叉,体长可达45公分。本鱼栖息于亚热带海域的水表层,常栖息在水流平静的内湾,受惊时会越出水面。属杂食性,以藻类及浮游生物为主。繁殖期至河口产卵。属高级食用鱼,适合油煎食用。2.FishBase3.台湾鱼类资料库","text2":"斑鱵主要分布在什么地方?","label":1} {"text1":"突厥汗国(古代突厥如尼文:;)是原先在柔然统治下的阿史那氏部族于552年在中国以北地区建立的古代汗国,一度控制漠北、中亚等柔然故地。后分裂成东突厥汗国和西突厥汗国,7世纪时先后为唐朝所灭。而东突厥复国后形成的后突厥汗国亦在8世纪为回纥灭亡。有人认为阿史那部最先是生活在咸海边的塞种,后东走至叶尼塞河南方,受铁勒同化;阿史那部于6世纪初年游牧于金山一代(今阿尔泰山),归附于柔然,为其炼铁奴。柔然由于长期与铁勒(柔然人称其为高车人,因为他们高大的车轮而得名)战争而削弱;546年,阿史那部首领脱离柔然,并于550年在首领土门的带领下击败铁勒,552年又打败柔然,建立了政权,自称伊利可汗;同时期的中国将其音译为“突厥”。553年木杆可汗在都斤山(又作“郁都军山”、“乌德鞬山”,今蒙古国杭爱山)建立王庭。全盛时,其疆域东至大兴安岭,西抵西海(咸海),北越贝加尔湖,南接阿姆河南,建立了官制,有立法,有文字。583年,隋将长孙晟用离间之计,使突厥汗国分裂为东西两部,后在屡次与隋朝的战争中战败而走向衰落(参见隋与突厥之战)。唐太宗贞观四年(630年)与薛延陀攻灭东突厥,唐高宗显庆四年(658年)唐又灭西突厥,余部西迁中亚(参见唐灭东、西突厥之战)。在唐高宗末年(682年),再度建立后突厥汗国,最后在745年,后突厥帝国亡于回纥。","text2":"隋将长孙晟使用了什么伎俩使得突厥汗国分裂?","label":1} {"text1":"栢慧豪园(Central Park Towers)位于香港新界西元朗天水围新市镇第24B区,为天水围3个私人住宅屋苑之一,也是天水围的豪宅屋苑,保安严密。栢慧豪园的住户收入远高于天水围其他地方居民,可以说是富豪及中产集中地,所以与其他天水围屋苑形成强列对比。物业发展商为长江实业地产,屋苑一共分为2期,共提供2,960个单位。目前该屋苑第一期已落成并已于2008年第一季入伙,而第2期命名为栢慧豪廷,于2010年11月开始入伙。物业第一期由5座住宅大厦组成,楼高58至59层,合共提供1,892个单位,于2008年第一季入伙。单位间隔备有两房、三房、三房连主人套房及五房连主人套房间隔。全部单位客饭厅均为方形间隔,另外配备区内独有的弧形线条露台。栢慧豪园第二期命名为「栢慧豪廷」,由3座住宅大厦组成,楼高58层,合共提供1,068个单位,屋苑已于2010年11月开始入伙。每户设38呎露台(连工作平台)。物业邻近置富嘉湖、天水围公园及湿地公园。发展商亦声称斥资1.6亿港元兴建占地210,000方呎的住客会所大楼,共有逾70项动静及室内、外设施。包括70米瑞士琉森湖泊式园林泳池、4个家庭式水疗按摩浴池、首创的Thai-boxing泰式纤体擂台、幻彩保龄球场及多用途室内球场等。不过会所大楼属第二期工程部份,会所大楼在2010年才开幕。除港岛东区、南区、西贡区和大埔区未有全日服务之巴士路线外,本屋苑的对外交通基本可达全港各处以及深圳湾口岸。","text2":"栢慧豪园在哪里?","label":1} {"text1":"少牙斑鲆(学名:),俗名贫齿扁鱼、地鱼、稀齿斑鲆、铁斧,为辐鳍鱼纲鲽形目牙鲆科的其中一种。本鱼分布于西北太平洋区,包括日本南部、中国、台湾、越南、菲律宾、泰国等海域。该物种的模式产地在长崎。水深30至40公尺。本鱼体极扁平,略呈卵圆形;口大、眼大,双眼同位于左侧且间距极小。有眼侧为深褐色,鳃盖缘具2个暗色斑,尾鳍楔形,背鳍软条78至82;臀鳍软条61至65枚;脊椎骨38个,体长可达30公分。本鱼仔稚鱼时期行浮游生活,两眼左右对称,成长后渐移至左侧,且改行底栖生活。平时大多停栖于海底,并将鱼体埋于沙中,能随环境改变体色,很少游动,属肉食性,以小型底栖动物为食。食用鱼,肉质佳,适合清蒸或红烧。","text2":"少牙斑鲆的鳃盖缘具多少个暗色斑?","label":1} {"text1":"张露(原名张秀英,英文名:,),生于江苏苏州,是1940年代至1950年代上海的国语女歌手,有「中国歌后」的美誉,其代表作是《你真美丽》及经典代表作《给我一个吻》。张露4岁时随家人移居上海居住,但及后丧父而家境清贫;1945年的一天,在家中的张露趁弟弟睡觉时哼歌,无意中被任职电台的邻居听到,发觉她歌声惊为天人,在其引荐下瞒著母亲到上海电台客串唱歌,后转为正职歌手,及后改艺名为张露。在暪著母亲的情况下,张露不想公开其真实名字,而电台以「张大小姐」来介绍她;上海话「大」的发音为「du」,而「大」跟「露」字在上海话发音相近,所以她在电台越唱越红后就以张露作为其艺名。张露其后被获邀到夜总会演出,并与百代唱片签约为旗下歌手。张露在1946年推出首张个人唱片《小巷春》及《关不住》;1948年首次在电影《柳浪细鸯》演出并演唱《淑女窈窕》一曲;同年因歌曲《你真美丽》而红遍上海。1952年,因政局问题而迁到香港定居,后被各大夜总会争邀献唱,张露并以百变及奔放形象奠定了在乐坛的地位;她灌录的国语歌曲《迎春花》、《小小羊儿要回家》、《不许他回家》成为她的代表作品,而她的代表经典名曲《给我一个吻》(1954年)的「给我一个吻,可以不可以?吻在我的心上,让我想念你」唱到家传户晓,除红遍中港外,更享有「中国歌后」的美誉,也是她个人演艺事业的巅峰时期;期间在电影《那个不多情》及《爱的俘虏》演出。虽然张露在1975年退出歌坛,但偶然也会献唱。并在1980年移民加拿大,但于1985年为了照顾留在香港发展的杜德伟而举家回流香港居住。约在2008年11月,张露和杜德伟到上海探亲期间举行一个歌迷聚会,张露上台献唱《给我一个吻》。2009年1月25日,张露因大脑中风昏迷而被紧急送院医治,但延至翌日下午四时,因器官衰竭于香港律敦治医院病逝,终年77岁。根据其生前意愿,遗体先在香港被火化于上海市郊青浦区福寿园安葬。1957年,张露和于新加坡演出时认识的西班牙裔乐队领班及鼓手奥利()结婚;并育有杜德智及杜德伟两子,分别为香港歌手。","text2":"张露是怎样走上歌手之路的?","label":1} {"text1":"闵浦大桥,是位于中国上海黄浦江上的一座公路双层斜拉桥,是跨越黄浦江的第八座大桥,上层S32公路段于2009年12月31日建成通车,下层(普通公路层)则于2010年4月20日通车。建成时为世界上跨径最大的双塔双索面双层公路斜拉桥。闵浦大桥西岸位于闵行区吴泾镇放鹤路,东岸位于闵行区浦江镇沈杜公路,大桥位于徐浦大桥和奉浦大桥之间,全长约3.61公里。主桥全长1212米,主跨长708米。通航能力为2万吨级。于2005年9月1日开工建设,2009年7月19日开始合龙,2009年7月22日大桥主桥提前合龙。闵浦大桥上层设双向八车道及紧急停车道,是S32申嘉湖高速公路的跨黄浦江设施,下层为普通公路,设双向六车道。","text2":"闵浦大桥是跨越黄浦江的第几座大桥?","label":1} {"text1":"田大益,字或损,号博真,四川省重庆府定远县人,明朝政治人物。历任钟祥知县、升兵科给事中,太常少卿。田大益早年在县城(今中心镇)之天印山南的九洞里,与张一鲲、田大年、张登仕等人一起苦读,后多人中进士。他亦授钟祥知县,兵科给事中。为官期间,多书奏折,批评时弊。概时帝倦勤,上章着虽千万言,大率屏置勿阅故也。时冯应京武昌被逮之事,他上书言:“陛下驱率狼虎,飞而食人,使天下之人,剥肤而吸髓,重足而累息,以致天灾地坼,山崩川竭。衅自上开,愤由怨积,奈何欲涂民耳目,以自解释,谩曰权宜哉!今楚人以奉故,沈使者不返矣,且欲甘心巡抚大臣矣。中朝使臣不敢入境侦缓急,逾两月矣。四方观听,惟在楚人。臣意陛下必且旷然易虑,立罢矿税,以靖四方,奈何犹恋恋不能自割也!夫天下至贵,而金玉珠宝至贱也。积金玉珠宝若泰山,不可市天下尺寸地;而失天下,又何用金玉珠宝为哉!今四方万姓,见陛下遇楚事而无变志,知祸必不解,必且群起为变。此时即尽戮诸珰以谢天下,宁有济耶?”","text2":"武昌爆发骚动的原因是?","label":1} {"text1":"三色龙胆(学名:)为龙胆科龙胆属的植物,是中国的特有植物。分布于中国大陆的青海、甘肃等地,生长于海拔2,200米至3,200米的地区,多生在湖边漫滩草地、河滩草地、沼泽化草甸、林下以及路边,目前尚未由人工引种栽培特征一年生草本,高3-5厘米,茎紫红色,下部光滑,上部具细乳突,在基部多分枝,枝铺散,斜上升。基生叶大,在花期枯萎,宿存,倒卵圆形,长7-11毫米,宽5-6.5毫米,先端钝圆,具短小尖头,边缘光滑,软骨质,两面光滑,中脉软骨质,在下面突起,叶柄膜质,光滑,长2-3毫米,连合成长0.5毫米的筒;茎生叶对折,疏离,长于或等于节间,矩圆披针形至线状披针形,长4-7毫米,宽1.5-2.5毫米,愈向茎上部叶愈长,先端钝至渐尖,边缘膜质,狭窄,平滑,两面平滑,中脉在下面呈脊状突起,叶柄背面有细乳突或光滑,连合成长2-2.5毫米的筒。花数朵,单生于小枝顶端;花梗紫红色,具细乳突,长2-5.5毫米,藏于上部叶中;花萼倒锥形,长7-8毫米,光滑,裂片披针形或三角形,长2.5-3毫米,先端急尖,边缘膜质,狭窄,平滑,两面光滑,中脉绿色或白色,在背面呈脊状突起,并向萼筒下延,弯缺窄,截形;花冠常闭合,上部淡蓝色或蓝色,下部黄绿色,外面具绿色宽条纹,筒形,长11-13毫米,裂片卵形或卵状椭圆形,长1.2-2毫米,先端钝,具短小尖头,褶卵形,长1.5-2毫米,先端钝,具极短小尖头,全缘或有不整齐细齿;雄蕊着生于冠筒中部,整齐,花丝丝状钻形,长4-4.5毫米,花药直立,稀微弯曲,矩圆形,长约1毫米;子房狭椭圆形,长2.5-3.5毫米,两端渐尖,柄粗壮,长2-2.5毫米,花柱线形,连柱头长约2毫米,柱头2裂,裂片矩圆形。蒴果外露,倒卵状矩圆形,长5.5-6.5毫米,先端钝圆,具宽翅,两侧边缘具狭翅,基部渐狭成柄,柄粗壮,长至15毫米;种子黑褐色,矩圆形,长1.1-1.4毫米,表面具细网纹。花果期6-8月。","text2":"三色龙胆分布在哪些地区?","label":1} {"text1":"拉图约瑟法·伊洛伊洛瓦图·乌卢伊武达,CF,MBE,MSD,KStJ(Ratu Josefa Iloilovatu Uluivuda,),通称约瑟法·伊洛伊洛(Josefa Iloilo),斐济政治家,斐济总统。他曾经是世界上年龄最大的国家元首。伊洛伊洛生于斐济一个酋长家庭。接受师范教育,曾担任教师、校长。1977至1982年当选众议院议员。1978年成为大酋长委员会成员。1992年4至6月担任看守政府林业部长,同年5月被任命为参议员。1996年6月当选参议长。1997年11月当选为斐济维提岛西区武达省大酋长。1999年1月出任副总统。2000年5月斐发生政变,马拉总统辞职,大酋长委员会于7月任命伊为总统。斐政局趋于稳定后,恢复宪政,大酋长委员会于2001年3月再次任命伊为总统。2006年3月,经大酋长委员会推选续任总统,任期5年。2006年12月5日,在军政矛盾激化的情况下,当时的斐济军方领导人弗兰克·姆拜尼马拉马宣布由军队接管斐济政权并解散议会,推翻了由民选总理莱塞尼亚·恩加拉塞领导的斐济政府,并于2007年1月5日宣誓就任过渡政府总理。2007年4月,姆拜尼马拉马恢复了伊洛伊洛的总统职务。2009年4月9日斐济上诉法院宣布,弗兰克·姆拜尼马拉马领导的斐济过渡政府不符合宪法,建议总统伊洛伊洛重新任命一名看守政府总理,解散议会并尽快举行大选。上诉法院还宣布2006年12月时任斐济武装部队司令的姆拜尼马拉马推翻恩加拉塞领导的民选政府和解散议会均没有法律依据。伊洛伊洛随后宣布废除宪法,解除所有法官的职务,并于4月11日再次任命姆拜尼马拉马为斐济新一届过渡政府总理。伊洛伊洛在2009年7月30日退休,晚年患帕金森症,经常要前往澳大利亚接受治疗。他在2011年2月6日病逝于斐济,终年90岁。","text2":"约瑟法·伊洛伊洛晚年患上了什么病?","label":1} {"text1":"饭盛女()又叫饭卖女()、宿场女郎(),是江户时代在宿场以奉公人名义提供性服务的私娼。「饭盛女」是俗称、幕府法规文书使用「食卖女」。从名字上来说是在宿场给客人盛、卖饭的女人。事实上为来往的男性旅客提供性服务。也并非所有的饭盛女都卖淫,单纯从事类似现在仲居那样工作的饭盛女也是有的。17世纪随着五街道和宿场的兴建设置、伴随着日本国内的交通量大增,旅笼屋产业发达起来。宿场有服侍旅客的下女,其中一部分就演变为饭盛女。为了规避江户幕府禁止在宿场卖淫的规定就起了饭盛女这个名字、旅笼之间为了获取旅客纷纷提供了这种服务。当时无偿的公役使宿场经营困难,为了得到客源宿场再三要求幕府公开允许饭盛女的存在。幕府在权衡之后不得不默认,但是对宿屋中饭盛女的人数作了规定(一间旅笼最多两名),以防无限制扩大。饭盛女通常是因父母欠债无法偿还或从别处拐卖而来,生活相当悲惨。据历史学家五十岚富夫所著《饭盛女》一书记载,藤泽宿(位于今 神奈川县藤泽市)的饭盛女平均寿命只有21.3岁,41年间共有39名饭盛女死去。","text2":"饭盛女在法规上叫什么?","label":1} {"text1":"关中书院,是一所位于中国陕西省西安市的书院,始建于明朝万历三十七年(1609年),天启六年(1626年)一度被禁毁,后于清朝康熙三年(1664年)重建,一度成为全国四大著名书院之一,也是西北四大书院之执牛耳者。1903年,关中书院更名为陕西省第一师范学堂,1912年改为陕西省第一师范学校,1982年定名为陕西省西安师范学校。关中书院现在位于书院门,书院建筑规模宏大,中间讲堂6间曰“允执堂”,左右南屋4间,东西号房各6间,讲堂后边有假山,“三峰耸翠”,“宛若一小华岳”,讲堂前半亩方塘,竖亭于中,砌石为桥。书院有门两重,大门二楹,二门四楹,郡丞刘孟直书“八景诗”以壮其观,学者王大智书隶字为书院题名。","text2":"关中书院,位于哪里?","label":1} {"text1":"胡仙博士,OBE,JP(Sally Aw Sian),祖籍福建省永定县,是东南亚企业家,前全国政协委员,胡文虎的养女,曾经拥有香港大坑道及新加坡的虎豹别墅及7份报章,当年人称「报业女王」。由于兄长胡好早逝,胡仙成为胡文虎家族的继承人。1954年胡文虎病逝后,胡仙接掌星岛报业,并在1972年令星岛报业在香港上市。胡仙一度拥有7份报章,包括《星岛日报》、《星岛晚报》、《英文虎报》、《快报》、《天天日报》、《华南经济新闻》及与大陆合资的《深星时报》。1972年7月胡仙在香港创立胡文虎基金会。基金会是以非牟利慈善信托基金的形式注册,所有善款全部来自胡仙个人。1998年,受到亚洲金融风暴影响,胡仙于地产及股票投资严重亏蚀,而且惹上官非(后称「胡仙案」)。同年出售《天天日报》,次年再出售星岛日报予 Lazard Asia。根据律政司前刑事检控专员江乐士披露「胡仙案」的内情,当年时任律政司长的梁爱诗袒护胡仙而在有足够证据起诉胡仙的情况下仍然选择放弃捡控,然而梁爱诗对此否认。","text2":"江乐士披露「胡仙案」的内情是怎样的?","label":1} {"text1":"费尔南多·加戈(Fernando Gago,),是一名阿根廷足球运动员,主要司职后腰,目前效力于小保加,也是阿根廷国家足球队的成员。加高于2004年为小保加上阵,很快于成为主力成员,吸引多间欧洲球会向他招手。2006年11月15日,皇家马德里主席证实了加高正式转会皇家马德里。这次转会费用为2040万欧元。由于他司职防守中场,因此很多人把他跟前阿根廷“5号”费兰度·列当度比较,更有传媒称他为\"新列当度\"。加高加盟后首季穿上16号球衣,起初很多人认为他是极有才华的中场球员。但是,由于表现欠佳,在季初都得不到重用。逐渐适应西甲的比赛节奏后,他的表现开始改善。在2007-08,加高的球衣号码由16改为8号。在他的第二个赛季,加高展示他真正的能力,代替了穆罕穆德·迪亚拉成为正选,协助球队连续第二次羸得联赛冠军。在2008-09赛季由于受伤,加高季初只打了两场比赛,在整个联赛他只有11次西甲联赛上阵,但在本赛季,加戈取得了加盟皇马后的首个入球。在2009-10赛季,主席佩雷斯打造一支全新的银河舰队,随著巴西球星卡卡加盟,他让出8号,改穿5号球衣,这号码跟他在阿根廷国家队一样。由于沙比·阿朗素及拉辛拿·迪亚拉表现出色,加高差劣的表现完全被比下去,令他只能成为后备,甚至排在穆罕穆德·迪亚拉之后成为第四防中,更多次被排除在比赛的大名单之外。和同期加盟的冈萨洛·伊瓜因和马些路相比,加高原地踏步,令部分皇马球迷质疑其能力与身价。加高平庸的表现与同样被誉为新列当度的班尼加(效力于华伦西亚)出色的表现形成强烈对比。由于在2010-11赛季仍然不受重用,在2011年8月31日,皇马同意让加高借用转回罗马。加高曾经为阿根廷U-20赢得,但他并没有入选2006年德国世界杯名单。他入选了2008年夏季奥林匹克运动会男子足球比赛,此后他一直成为阿根廷国家足球队的成员。在新教练的马勒当拿上任后,他跟马斯查兰奴一直在首发阵容。但随后因在2010年世界杯外围赛表现差劣而饱受批评,被指责不应再入选国家队。","text2":"费尔南多·加戈为什么从8号球衣改成5号球衣?","label":1} {"text1":"丰田 实(日文:とよ田 みのる,本名:豊田 実(发音相同)。男性、)是日本的漫画家。出生于东京都大岛町、居住在江户川区。从东京造形大学造形学部毕业。代表作是《来恋爱吧》。在25岁才立志要当一名真正的漫画家。一直持续著投稿作品,在2000年以『レオニズ』得到afternoon四季赏夏之竞赛的佳作(没有连载)。2002年以『来恋爱吧』得到同奖的春之四季大赏而初次亮相。从2003年到2005年在月刊afternoon连载《来恋爱吧》,这部描写高中情侣新鲜恋爱的故事,特色在于男主角的夸张老实个性,有趣的故事内容获得不分男女老幼的广大支持。作品的特征是人物线条在背景下会显得特别鲜明、特别粗重。最初受到ますむらひろし的影响而多描绘SF、奇幻等作品,但因为每次投搞都落选而决定以『来恋爱吧』做为路线的改变。","text2":"在月刊afternoon连载《来恋爱吧》的时间是?","label":1} {"text1":"《战斗司书》()系列是山形石雄的出道作,前嶋重机插画的轻小说。第1集「战斗司书与恋爱爆弹」是日本集英社第4届Super Dash小说新人奖大赏得奖作品。所有续作均以「战斗司书」为名,日本、台湾均已出版完结,全10集。2008年3月漫画化,由筿原九负责作画,在《Ultra Jump Egg》上连载,以第1集《战斗司书与恋爱爆弹》为漫画名连载。动画《战斗司书 The book of Bantorra》于2009年10月2日开始播放,于2010年4月2日结束,全27话。以有著制造无限蚁群、预测未来、时间静止等特殊能力的武装司书,与拥有同样特殊能力的神溺教团战士们的战斗为主轴,发展故事。自远古开始,人死后的灵魂会被「过去神」邦特拉封在『书』中,收藏在地下的邦特拉图书馆。神离开人类后,管理图书馆和挖掘『书』的工作落在武装司书身上。不少犯罪者亦会为自私的目的搜集、私藏及滥用『书』,其中最主要的是神溺教团。神溺教团为了他们的教义,无视别人的生命,夺取他人的『书』,而不断与武装司书对抗。战斗司书系列中的部分角色名称以及追忆战器名称的出处。","text2":"《战斗司书》的动画一共有多少话?","label":1} {"text1":"剑齿虎是指多种不同的物种,主要是分类在猫科下的剑齿虎亚科、鬣齿兽科及猎猫科,且包括有袋下纲的两个科,生存于新生代的不同时期,各自演化出像剑的犬齿。牠们的上犬齿最长可达20厘米,纵然口合起来,其犬齿仍然清晰可见。剑齿虎的身形一般都较现今的猫科强壮和庞大,仿佛像熊一般。剑齿虎最先出现于约始新世中期,最后消失的则约在几千年前。以下是各剑齿虎的属,并其分布地及出现时期:所有剑齿虎都是生存于3370万年前至9000年前,但牠们的演化支却在更早的时间就已经开始分支。袋剑虎的分支是最先分开的,约在白垩纪晚期。牠们是有袋类,较为接近袋鼠及负鼠而多于猫科。肉齿目则接著分支,再为猎猫科,而最终是猫科的剑齿虎。","text2":"剑齿虎是一个物种吗?","label":1} {"text1":"《恶女花魁》()是安野梦洋子于2001年起连载的漫画,现已停止连载。于2007年由蜷川实花拍摄成电影,演员一字排开土屋安娜、椎名桔平、成宫宽贵、木村佳乃、菅野美穗、小泉今日子、安藤政信、永濑正敏各个都是顶顶有名的大人物,再加上苹果歌姬-椎名林檎的配乐之下,成功的刻划出江户时期恶女花魁的传奇。「不知怎的,我就是会,我就是懂得怎么看男人才会使他们兴奋。」—— 清叶清叶在八岁(幼时小池彩梦饰)时,就被卖到游廓。起初是在花魁粧妃(菅野美穗饰)旁服侍,并学习如何成为花魁,而在花魁粧妃出嫁后,便把发簪象征性的传承给清叶。长大后的清叶(土屋安娜饰)不负粧妃的期待成了花街众所瞩目的焦点。但是自己却被迷恋许久的恩客抛弃,让她对爱情彻底失望,并发誓再也不要爱上男人,转而全心全意的投入工作,把服务男人当成一种技术,不久后她当上了花街上的首席-花魁。但是在她遇上武士-仓之助(椎名桔平饰)后,生活渐渐起了变化,而仓之助也趁势向她求婚,让清叶又惊又喜,但是却也害怕再次受到伤害,因此她说:「当花街下起樱花雨,我就嫁给你!」隔天,她被妈妈桑的声音吵醒,打开窗户,却见到令她永生难忘的美景-樱花雨。原来是仓之助将他处的樱花树全移到花街前,让这条街下起了缤纷的樱花雨。正当所有人都为了眼前的美景暂歇脚步之时,清叶在窗边流下了眼泪,不仅是感伤幼时离开家乡后就再没见过樱花雨,更重要的是也被仓之助所做的一切所感动。看来清叶非嫁不可了!游廓里的鸨母准备好华丽的嫁妆和衣裳,准备让她快乐的出嫁成为武士夫人。但,就在出嫁前一晚,她找到了她生命中的真命天子……原声带在电影上映前就已经发行了,而这张专辑也是椎名林檎暌违四年后所发行的专辑。","text2":"原声带是由谁发行的?","label":1} {"text1":"卡拉瓦亚龙属(学名:\"Callawayasaurus\")是种海生爬行动物,属于蛇颈龙亚目薄板龙科。卡拉瓦亚龙的化石最初由Samuel Paul Welles在1962年发现,原本被归类为\"Alzadasaurus\"的一个种,在1999年由肯尼思·卡彭特(Kenneth Carpenter)建立为新属。卡拉瓦亚龙的属名是以古生物学家Jack M. Callaway为名,这位古生物学家曾对史前海生爬行动物作出重大的研究。。种名则是以化石发现地哥伦比亚为名。卡拉瓦亚龙的头骨长35公分身长估计可达8公尺长。卡拉瓦亚龙的鼻孔长,位于上颌骨上方。上颌骨具有3到5颗牙齿。颈部有56节颈椎,与其他薄板龙科相比,相当短。除了正模标本以外,还发现另一个几乎完整的标本,较正模标本粗壮。某些科学家认为这是两性异形的特征。卡拉瓦亚龙的化石发现于哥伦比亚博亚卡省的Villa de Leyva。卡拉瓦亚龙的生存年代为白垩纪早期(阿普第阶),约1亿2500万到1亿1200万年前。","text2":"卡拉瓦亚龙属于什么科?","label":1} {"text1":"黑嘴树鸭(\"Dendrocygna arborea\")是加勒比海的一种树鸭属的鸭。黑嘴树鸭是树鸭属中最大及最深色的,长48-58厘米。牠们的喙是黑色的,头部及脚都很长。牠们的前颈淡色,面部呈浅褐色。冠、背部、胸部及双翼呈深褐色至黑色,身体的其他部份是白色的,有很密的黑色斑纹。雄鸟及雌鸟相似,雏鸟的则较深色,腹部颜色也较不明确。黑嘴树鸭散布在西印度群岛,最大的群族是在巴哈马群岛,在古巴、开曼群岛、安提瓜和巴布达及牙买加也有少数的群族。虽然牠们大部份时间都站著不动,但迁徙就可以达100公里以上。牠们会在树孔、树枝、丛林、地上及其他茂密的地方筑巢。牠们每次会生10-16只蛋。牠们习惯在树上生活。黑嘴树鸭是夜间活动的,在沼泽及红树林出没。牠们主要吃植物的部份,包括王棕属的果实。黑嘴树鸭受到大量捕猎,其蛋亦被采集。牠们所居住的湿地因开发而逐渐消失,而余下的大部份都受到破坏。牠们正受到《濒危野生动植物种国际贸易公约》附录二及《迁移物种公约》附录二的保护","text2":"黑嘴树鸭的生活习性是怎样的?","label":1} {"text1":"科罗拉多大学波德分校(University of Colorado at Boulder,官方:CU-Boulder, UCB;非官方:Colorado和CU)是科罗拉多大学系统的旗舰。它成立于1876年,比科罗拉多加入联邦的时间还早5个月。是美国新闻与世界报道所评全美最佳公立大学之一。科罗拉多大学共有三个校区:博尔德校区、丹佛校区,以及科罗拉多泉校区。博尔德校区是科罗拉多大学主校区,该校区位于洛叽山麓的波德市。整个学校由9个学院系统组成,提供超过150个学术项目,有近3万名学生,在2010年授予了6781个学位。该大学物理系先后有4人获得诺贝尔奖,光学、原子方向实力超群,在美国排名第一。工程领域最具特色的是航天工程(Aerospace Engineering Sciences),该专业在美国新闻与世界报道上2011年的排名全美第十二。该系诞生十几位宇航员,并且从2009年起聘期退役宇航员担任某实验类课程指导。航天科学与工程方向下的LASP实验室专门从事航天领域的研究,具备从卫星的设计、在轨运行以及返回数据处理、科学研究的全程研究能力。据统计美国现役的所有卫星都有科罗拉多大学设计、制造的仪器设备。大气与海洋科学系成立的时间不长,但目前已跃升至全美国前五名,并与位于博尔德附近的美国大气研究中心有着密切合作。历史上,科罗拉多大学的学生,研究人员或者学术人员中,总共诞生了11个诺贝尔奖获得者,7个麦克阿瑟奖获得者,以及众多航天员。曾经登陆太空的太空人,有11位是该大学的毕业同学。","text2":"博尔德校区位于哪个城市?","label":1} {"text1":"《热天午后》(Dog Day Afternoon)是美国导演薛尼·卢梅的代表作之一,在1975年首映,由艾尔·帕西诺及约翰·卡佐尔(John Cazale)等所主演。《热天午后》根据1972年真实发生的一单银行抢劫案所改编,并获得奥斯卡奖与金球奖的提名,编剧弗兰克·皮尔森赢得奥斯卡最佳原创剧本奖。香港导演尔冬升亦受本片启发并于1987年改编本片并拍摄电影《人民英雄》。1972年8月22日炎热的下午,纽约市布鲁克林的一家银行遭到桑尼和萨尔两个人的持枪抢劫。由于银行里的大批金钱刚被运走,保险柜里只剩下1100元。不久后,银行被大批警察包围,在该行里工作的几位女性和一位上年纪的负责人穆尔范尼成为劫匪的人质,于是桑尼与负责该案的探长莫雷蒂展开周旋和谈判。桑尼在谈判中要求警方为其准备一辆开往机场的客车,计划乘飞机飞往北非的阿尔及利亚,待其和萨尔上飞机后再将人质释放。尽管桑尼是一个抢劫犯,但是他在银行外面与警方的谈话与大声演讲内容却获得大量围观民众的呐喊支持。桑尼还让警方将其同性伴侣李昂叫来,在两人的电话通话中,得知桑尼之所以要抢劫银行,目的是为李昂筹集变性所需费用。后来FBI探员雪尔顿代替莫雷蒂继续进行谈判,时间逐渐到了夜晚,通过安排,桑尼与自己的前妻安姬通了电话,桑尼与她育有一子一女两个孩子,安姬变胖后桑尼与李昂产生感情。为了迫使桑尼向警方投降,警方还将桑尼的老妈叫来,她劝儿子尽快投降以获得宽大处理,但是桑尼监持原定的逃脱计划不为所动。桑尼在银行里还安排写下了一份遗嘱,记载著对李昂和两个子女的未来照顾计划。最后在警车的护送下,桑尼与萨尔与一众人质乘坐一辆车前往机场。就在桑尼以为目标快要达成之际,司机墨菲探员与雪尔顿探员巧妙合作,导致萨尔被墨菲击毙,桑尼最终被捕,后来判处20年的监禁,李昂也成功变性成为一名女人。该片获得了极大的好评,在烂番茄网站,它获得了97%的新鲜度;知名影评人Vincent Canby评论道“它是Sidney Lumet最精确,最耀眼的关于纽约的电影”,并对全部演员赞扬道“他们都塑造了杰出的人物”(brilliant characterizations);Roger Ebert评价说Sonny是“最有趣的一个现代电影角色”,并给予影片三星半(满分四星)的打分。","text2":"《热天午后》首映与哪一年?","label":1} {"text1":"嵊山镇是浙江省舟山市嵊泗县下属的一个镇。曾为嵊泗县和舟山渔场驻地。嵊山镇位于嵊泗县东部,境内的海礁(童岛)既是浙江省的最东端,也是中国大陆领海基点之一。全镇陆地总面积为7.13平方公里,海域面积广达2900平方公里。2000年人口普查时嵊山镇有人口10807人,壁下乡有739人,合计11546人,其中3\/4为渔业人口。常年有流动人口2000人,多为到此地抲鱼停泊的苏浙闽沿海各省渔民。2010年人口普查时总人口0.85万。2001年壁下乡撤消,并入嵊山镇。目前共设4个渔农村新型社区和7个行政村,即:渔业是嵊山镇的传统产业,嵊山曾为舟山渔场场部所在地。嵊山渔港为舟山渔场的重要渔港,浙江省最大的鲜活海水产品出口基地,也是国家一级渔港和二类开放口岸。","text2":"全镇陆地总面积多大?","label":1} {"text1":"菜豆(学名:)通称包括云藊豆、白肾豆、架豆、芸豆、刀豆、玉豆、去豆、四季豆等,一年生豆科植物,是餐桌上的常见蔬菜之一。油豆角(\"Phaseolus vulgaris\" var. \"chinensis\" Hort.)是蔓生菜豆在中国东北的优质软荚变种。以荚内油分多得名,有500年的栽培历史。 油豆角主要是以食荚为主,烹煮后豆荚软面,纤维少,豆香味浓,蛋白质含量较高。矮菜豆,或者矮生菜豆(\"Phaseolus vulgaris\" L. var. \"humilis\" Alef.)是菜豆种的一个变种,就是广州常见的龙牙豆。中南美原产。一般认为16世纪末经由欧洲传到中国,17世纪传到了日本。四季豆的多个品种均含植物血凝素(phytohaemagglutinin,PHA),当中以红芸豆的含量最高,白芸豆的植物血凝素只有红芸豆的三分之一,而扁豆(\"Dolichos lablab\")的植物血凝素含量则只有红芸豆的5%到10%。所以在煮食四季豆时,要在100度的沸水中煮至少10分钟,以去除豆内的植物血凝素,然后才可继续烹煮。至于干豆,根据美国食物及药品管理局(FDA)的建议,在食用前应把干豆浸在水里至少五小时,然后把浸过豆的水倒走。假若四季豆没有在沸水里煮过,植物血凝素的毒性不单不能消除,还可能会增加。FDA指四季豆在摄氏80度的水里煮,植物血凝素会增加至未煮前的五倍。斑豆()是一种有斑点的菜豆,是等北美洲南部最常见的豆类,在美国及墨西哥西北部均有生长。斑豆的烹调方法通常都是用来煮肉汤或制成斑豆泥;而煮好的斑豆或斑豆泥,均是卷饼的饀料。斑豆的幼嫩豆荚亦可吃,有时会被当作青豆或四季豆的一种。","text2":"在煮食四季豆时,有什么注意事项?","label":1} {"text1":"在结构力学上的自由度(Degrees of freedom),或称动不定度(Degrees of kinematic indeterminacy),意指分析结构系统时,有效的结构节点上的未知节点变位数。其中称之为「有效」是因为结构构件上的任一点,都应有机会具有自由度,我们只选择其中对分析整体结构有用的节点变位来讨论,而称为「未知」则因为为求解容易,我们通常尽可能减少自由度的数量,因此扣除已知的变位。自由度大致有两种型式:在平面中,只有三个自由度,一者为面旋转,二者为前后及左右两个移动。在立体中,有六个自由度,三个为前后、上下及左右三个移动和前后、上下及左右三面旋转。简单来说就是沿三个坐标轴的移动和绕三个坐标轴的转动把构建相对于参考系具有独立运动参数的数目称为构件的自由度自由度作为结构力学中的重要概念,是描述一个结构基本情况的基本参数。在结构分析中,将自由度作为主要未知数,基本求解方法有两种:利用变形谐合条件求解的方法,称为力法,此法的应用范围是未知的自由度较少的情况。利用力平衡条件求解的方法,称为位移法,此法应用较为广泛,尤其在求解高阶超静定结构的情况下较力法容易,适合利用线性代数(矩阵)的方式配合程式撰写来求得欲知的自由度。","text2":"为什么称之为未知?","label":1} {"text1":"杜氏棱鳀(学名:),俗名突鼻仔、含西、西姑鱼,为辐鳍鱼纲鲱形目鲱亚目鳀科的其中一种。本鱼广泛分布印度西太平洋区,包括巴基斯坦、斯里兰卡、印度、孟加拉湾、安达曼海、泰国、越南、柬埔寨、马来西亚、中国南海、台湾、菲律宾巴拉望岛吕宋岛、印尼苏门答腊、婆罗洲、爪哇岛附近等海域。水深1至20公尺。本鱼体窄长侧扁。口大,口裂微倾斜。上颌长于下颌。上颌骨延长,向后延伸至胸鳍末端。体背青绿色,头顶、吻部及鳃盖上部黄色,头部背面后方具一暗青色斑。体侧和腹部银白色。背鳍青黄色,后方有鞍状绿色斑。尾鳍浅黄色,后缘黑色。胸鳍、腹鳍、臀鳍淡色。体长可达15公分。本鱼栖息于沿岸,常出现在沙泥质海域及河口区。食用鱼,但鱼体薄肉少,通常晒成鱼干,或是当下杂鱼。","text2":"杜氏棱鳀外型特点?","label":1} {"text1":"亨利·普赛尔(,),巴洛克时期的英格兰作曲家,吸收法国与义大利音乐的特点,创作出独特的英国巴洛克音乐风格。他被认为是英国最伟大的作曲家之一,独霸乐坛两百年,一直到20世纪初爱德华·埃尔加成名之前,没有一个本土生长的英国作曲家达到他的成就。生於伦敦的西敏地区,父亲曾任查理二世的宫廷乐师。少年时是王室小礼拜堂合唱团团员,嗓子受伤后任乐器保管员。曾从约翰·布洛学习。1680年成为西敏寺的管风琴师,1682年又兼任王室小礼拜堂管风琴师。1695年初,替病逝的玛丽女王创作葬礼歌曲——《\"\"》;7月时,他为王位的推定继承人格洛斯特公爵威廉王子谱写六岁的生日歌曲;他在11月猝逝,年仅36岁。据《音乐家的罗曼史》一书,有一天,普赛尔因夜归而被妻子拒谙门外,因感冒而冻死,葬于西敏寺内他的管风琴附近。普赛尔在短暂的一生中创作了大量的器乐、歌曲、话剧配乐及少数歌剧,在英国古典音乐历史上有重要的地位。他的歌剧《狄朵与埃涅阿斯》是英国歌剧名作。而他的戏剧配乐《摩尔人的复仇》中的一个主题更被20世纪作曲家布里顿用进《青少年管弦乐队指南》。","text2":"亨利·普赛尔所处的时期是什么?","label":1} {"text1":"托马斯·纳托尔(,是英国的植物学家和动物学家。纳托尔出生于约克郡的一个小村庄,早期到英格兰工作,作为一个印刷学徒工,后来他到美国访问,在费城遇到了美国植物学家本杰明·史密斯·巴顿,巴顿鼓励他对研究自然历史的兴趣。1810年,他参加了一个由富翁阿斯特发起赞助的,沿密苏里河的考察团,后来离开考察团,和一位为利物浦植物园采集植物的英国植物学家一起,沿和南达科他州的印第安人做皮毛交易的商人路线继续考察,收集的大量的未知植物标本,这次考察路线虽然以前刘易斯与克拉克远征曾经走过,但他们收集的植物标本仍然有许多遗漏。纳托尔考察后,由于当时英国和美国开战,他只得经由新奥尔良回到伦敦,回来后继续研究他收集的标本。1815年,他又回到美国继续采集和研究工作,1818年,出版了《北美植物分属》一书,以后两年中,他在阿肯色州的红河谷考察,回到费城后又出版了《1819年在阿肯色州旅行记实》。1825年,他被任命为哈佛大学植物园的园长,1832年到1834年,他出版了《美国和加拿大的鸟类学手册》。1834年,他辞职,跟随一个考察团前往堪萨斯州、怀俄明州和犹他州考察,12月,他横跨太平洋,到夏威夷州,1835年春天,回到大陆,在太平洋西北海岸度过一年,并到过圣迭戈。从1836年到1841年,他在费称的自然科学学院工作,这期间,他为亚萨·格雷和约翰·托雷编著的《北美植物》提供了许多资料。后来了他的叔夫去世,为他留下遗产,但根据其遗嘱,他每年必须有9个月待在英国,1841年12月,他离开美国,走以前出版了《北美洲森林:没有被弗朗索瓦·安德列·米歇描述的树》,是第一部北美洲树种的大全。他在兰开夏郡的圣海伦去世,安葬在一个乡村教堂墓地。有几种鸟类和植物都是以他的名字命名的。","text2":"纳托尔为什么经由新奥尔良回到伦敦?","label":1} {"text1":"《杀狗记》,全名《杨德贤妇杀狗劝夫》,元末南戏。徐仲由作。写孙华夫妇和其弟孙荣的失和与团圆。对「酒肉朋友」之间种种卑劣势利行为的暴露比较充分,所写生活场面颇吸引人。《杀狗记》现存明代汲古阁刊本。元杂剧有《杀狗劝夫》,与本剧为同一题材,无名氏作。孙华结交了柳隆卿、胡-{}-子传两名酒肉朋友,日日挥霍,驱逐了弟弟孙荣,孙华之妾杨玉贞苦劝不得,持刀杀死一只狗,把狗尸扮成人形,丢弃家中,孙华回家之后,见尸体大惊,于是向柳隆卿、胡-{}-子传求救,两人不理会。孙华只好请求弟弟孙荣相助,孙荣慨然应允。而后柳隆卿、胡-{}-子传居然向孙氏兄弟恐吓金钱,最后杨玉贞才说出经过,原来只是一具狗尸,水落石出,从此孙氏兄弟一家和睦。","text2":"《杀狗记》主要说了什么?","label":1} {"text1":"陈襄(1017年-1080年)字述古,福州侯官人,世称古灵先生。生于宋真宗天禧元年(1017年),与陈烈、周希孟、郑穆友好,人称「海滨四先生」。庆历二年(1042年)进士,初任浦城县(今属福建省)主簿,历知仙居、河阳、蒙阳等县,因反对新法,曾请求神宗贬斥王安石、吕惠卿以谢天下。神宗时,出知明州。庆历八年(1048年)到仙居任县令,“民尚朴野,罕知读书”,“民穷多变,监狱患满”,又逢县境大水,著手赈济灾民。卒于神宗元丰三年(1080年)。摸钟辨盗电视剧《包公》里有这样一幕:包公让嫌疑人在黑暗中去摸一口大钟、从而找出真正的盗贼,其实这个故事就是嫁接自陈襄。事见,苏洵写了《钟能辨盗》,而在沈括写的《梦溪笔谈》中也记载了这个故事。陈襄任建州浦城知县时,有一家遭到了盗窃,抓到几个嫌疑人,但不知道谁确实是小偷。于是陈襄哄骗他们说:“某庙里有一口很灵验的钟,能分辨谁是小偷。”他派人把那口钟抬到官署后阁,祭祀起来,又带领这一群囚犯站到钟前,宣布说:“没有偷东西的人,摸这口钟,它不会响;如果偷了东西的人去摸,钟就会发出声响。” 祭祀完毕后,用帐子把大钟围起来,再暗地里让人用墨汁涂钟。之后,让嫌疑人一个个把手伸进帷帐里去摸钟,出来就检验他们的手,发现都有墨汁,只有一人手上无墨。陈襄对这个人进行审讯,此人不得不承认自己是盗贼。原来他是害怕钟响,没有敢去摸。","text2":"《钟能辨盗》的作者是谁?","label":1} {"text1":"土地伦理是环境伦理的视角之一,是由奥尔多·利奥波德()在他的《沙郡年纪》(\"\")一书中首次倡导的。其中他写道,需要一种“新的伦理”,“一种处理人与土地,以及人与在土地上生长的动物和植物之间的伦理观”。在他那个时代,(USFS)的主流观念从创立者吉福德·平肖()开始,就是追求经济利益的和功利主义的,而利奥波德则主张一种“生态学 ”的态度(这个词由芝加哥大学的亨利·钱德勒·考尔斯()于20世纪初在对印地安那沙丘的研究中提出,利奥波德是该术语最早的推广者之一)。资源保护主义在更偏向人类中心主义的范式中获得了它的首要地位,而与此同时,利奥波德的著作和启发与约翰·缪尔一起引发了环境主义的发展。利奥波德认为,伦理演变的下一步,是扩展为包含生物共同体(概括起来是“土地”)的非人类成员在内的伦理观。利奥波德将他的土地伦理的基本原理论述为:“当一个事物有助于保护生物共同体的和谐、稳定和美丽的时候,它就是正确的,当它走向反面时,就是错误的。 ”他还将该观点描述为:“土地伦理只是扩大了这个共同体的界限,它包括土壤、水、植物和动物,或者把它们概括起来:土地。……土地伦理是要把人类在共同体中以征服者的面目出现的角色,变成这个共同体中的平等的一员和公民。它暗含着对每个成员的尊敬,也包括对这个共同体本身的尊敬。”","text2":"利奥波德认为,伦理演变的下一步是什么?","label":1} {"text1":"江背镇位于湖南省长沙县东南部,为长沙县第二大乡镇,镇域北、东、南为浏阳市地域。地缘上,江背镇北与浏阳市永安镇、洞阳镇接壤,西与葛家乡、镇头镇为邻,南连柏加镇,西与黄花镇、干杉乡交界,全镇总面积175平方公里,总人口53,997人(2000年人口普查);辖13个村、3个社区;政府驻江背社区。今江背镇在1995年长沙地区撤区并乡镇之前属江背区(由五美区改名)江背镇、梅花乡和五美乡地域,1995年,三乡镇合并组建江背镇。2004年村级区划调整,调整之前为37个村、2个社区,分别为坳头村、板桥村、抄冲村、赤霞村、东庄村、洞井村、福冲村、好布村、河田村、洪江村、江背村、金田村、金洲村、开福村、立新村、联丰村、麻塘村、梅花村、美新村、楠木村、漆桥村、齐心村、清塘村、石背村、铁锁村、万古村、乌川村、五福村、五美村、湘阴村、小埠村、肖桥村、阳雀村、印山村、钟桥村、朱桥村和砖田村,2个社区;区划调整后为13个村、3个社区。","text2":"什么时候三乡镇合并组建江背镇?","label":1} {"text1":"重力位能或重力势能是指物体因为大质量物体的万有引力而具有的位能,其大小与其到大质量的距离有关。formula_1其中G为万有引力常数,M、m分别为两物体质量,r为两者距离。依据古典力学,在两个或更多的质量之间存在著重力位能。能量守恒要求这种重力位能永远是负值。证明令无穷远处为势能零点,则有formula_2且万有引力formula_3解得formula_4由于质点所减少的势能等于保守力对该质点所做的功,所以引力势能formula_5在广义相对论,重力位能被塑造成兰道-栗弗席兹赝张量 ,以允许古典力学的守恒定律能够获得保留。加上物质的应力-能量张量至兰道-栗弗席兹赝张量的结果是结合了物质和重力能赝张量导致散度为零的发散。有些人反对在基础上做如此的延伸,认为这样做在广义相对论中是不适当的,这是只是守恒律的需要所衍生的用途,在这样的情况下只是赝张量和真张量的结合。","text2":"能量守恒对这种重力位能的要求是什么?","label":1} {"text1":"兰渝铁路是中国一条连接甘肃省兰州市与重庆市的铁路。全长820公里,其中甘肃境内段长435公里,川渝段385公里。另外修建南充经广安至高兴支线89公里。全线为新建双线电气化铁路,支线部分为单线,线路等级为国铁Ⅰ级,兰渝段客车时速为200-250公里\/小时,南高支线时速为160公里\/小时。规划输送能力:客车50对\/日、货运5000万吨\/年。兰渝铁路项目估算总投资774亿元,其中甘肃境内段投资约430亿元,由铁道部与甘肃省、四川省、重庆市合资建设,建设工期为6年。兰渝铁路建成后,兰州与重庆的铁路运输距离将由1466公里缩短至820公里,动车组最快运行时间缩短为5.5小时,普通客车缩短为7小时。兰渝铁路西起甘肃省兰州市经榆中县、渭源县、岷县、宕昌县、陇南市武都区至四川省境内,再经广元市、苍溪县、阆中市、南部县、南充市最后至重庆市合川区、北碚区接入重庆枢纽。正线:兰州、兰州东、夏官营、渭源、岷县、哈达铺、陇南西、陇南、姚渡、羊木、广元、太公、苍溪、阆中、南部、龙桂、南充北、李渡、武胜、渭沱、合川站、北碚、重庆北南高支线:南充东、岳池、广安南、高兴","text2":"兰渝铁路西起于何地?","label":1} {"text1":"三斑天竺鲷,为辐鳍鱼纲鲈形目鲈亚目天竺鲷科的其中一种,俗名三线天竺鲷、大面侧仔。本鱼分布于西太平洋区,包括可可群岛、泰国、香港、日本、越南、马来西亚、新加坡、澳洲、菲律宾、印尼、巴布亚纽几内亚、马绍尔群岛、马里亚纳群岛、密克罗尼西亚、索罗门群岛、帛琉、东加、萨摩亚群岛等海域。该物种的模式产地在Buro。水深1-34公尺。本鱼体延长而侧扁,眼大,口大略下位。鱼体粉红色或橙色,体具褐色不规则斑纹,被栉鳞,鳞片具深色边缘,侧线明显,尾鳍截形,背鳍硬棘7枚;背鳍软条9枚;臀鳍硬棘2枚; 臀鳍软条8枚,体长可达14.2公分。本鱼栖息于近海珊瑚礁区,白天躲藏洞穴中,夜间出来觅食,属肉食性。繁殖期时,雄鱼具有口孵习性,卵约7日化成仔鱼,由雄鱼吐出,具短暂的仔鱼飘浮期。不具任何经济价值。2.FishBase","text2":"三斑天竺鲷是否具有经济价值?","label":1} {"text1":"拳击道是一个由传统韩国武术的风格及泰拳的混合的技击运动;在某程度上拳击道被称为韩国版本的拳击。在某些城市圈里亦有被称为「Muay Kwon」(泰道)及「Kwon Ho」。「拳击道」一词有几个意思,分别是「拳击及攻击的艺术」、「第一个攻击」及「拳击的艺术」。「拳击道」中的「拳击」代表著拳打及拳击,而「道」即是其艺术或是修练。现在其总部位于韩国的斧山。该武术是由钟道武(音译,其谚文罗马化为「Jung Do-Mo」)看到一个擅长跆拳道的人跟泰拳练习者砌磋后所创立的。创始人表示这个混合体比起单独使用跆拳道或是泰拳更有破坏性。拳击道的左右铭是「To attack offensively, never to surrender」(意译为「永不放弃狠狠的攻击」)。其教义为自信、实践、忠诚、尊敬及责任。这个武术的基本技术是跆拳道的基本踢击技巧,再加上一些泰拳的技巧,例如是一些防卫踢击(类似散打)及一些使用膝盖及手肘的攻击。另外,一些泰拳中部分步法也被删除掉,就好像是泰拳中的步进(Stepping)改为跆拳道里的滑步(Slides)及快步(Quick steps)。对于拳击的闪避(类似美式风格对于拳击的急忙低头避开)已加进这个技击运动以代替泰拳里有缺点的「Back up」,这令到使用者可以闪避及还击得快点。现在,这种风格的混合武术已经进步到站在战斗的角度其中一个最有实力的武术之一。拳击道比较少有类似掌(即是做出一个张开手掌的动作)的技术;练习者认为,这是因为在对手用力的防护或者是击打的这个情况下,手指会很容易夭断或脱臼。这个武术的足部技巧是混合自跆拳道及Kickboxing(有氧搏击操)的动作。跆拳道的基础能够在其混合武术中看到,例如是前踢(因为踢出时比其他的踢击都要快)、后踢、侧踢及勾踢等等。另外一个被混合至拳击道的是泰式的旋踢,但在跆拳道里该踢击是被禁止的,这是因为这种踢击有著惊人的速度。尽管在泰国比较少有拳击道的习训者用其武术来对抗泰拳的练习者,只不过他们的技术也有一定的水平。不幸地拳击道还有很多的进步及发展空间,不能广泛被大众公认为这是一种武术;有见及此,一些人认为这种武术在不少人知道其生存的时候,已经走到去灭绝的边缘。","text2":"拳击道的总部在哪?","label":1} {"text1":"我们看菊花去是白先勇1959年发表的短篇小说,一个弟弟送患上精神病的姊姊去医院的故事,也是白先勇的一篇自传。故事中的男主角「我」受父亲之托,为了骗患有精神病的姐姐去医院治疗,就跟姐姐说要去看菊花。一路上,作者交代了姐姐精神失常的线索。到了医院时,姐姐看苗头不对,便问“弟弟,我们不是去看菊花吗?来这里——”,弟弟告诉她“我们先去看一位朋友马上就去看菊花”。故事到最终是林大夫把姐姐关到铁栅里面。「我」一个人身在游人零落的新公园(今二二八和平纪念公园)看菊花。(我们看菊花去)是白先勇写自己的三姐白先明因患「精神分裂症」从美国回到台湾的一个故事,姐弟二人从小一起“在桂林上小学”,感情深厚。","text2":"白先勇的三姐得了什么病?","label":1} {"text1":"王俊生是一名中国体育界、足球界的知名人物,曾经担任中国足协专职副主席,是中国足球职业化的重要奠基人。王俊生年轻时为足球守门员,不到16岁就被选入北京队,后来成为北京队主力门将,1973年,时任国家队主教练年维泗曾将其选进国家队。1981年退役后,王俊生进入北京体育学院学习,获得大专学历,之后进入北京市体委工作。1991年进入中国足协工作后,王俊生于1992年至2000年出任足球运动管理中心主任、党委书记和中国足协专职副主席、秘书长。2002年7月,他离开中国足协,出任中体产业公司监事会主席。担任中国足协专职副主席、国家足球运动管理中心主任的八年时间是王俊生最受关注的一段时间,也是中国足球发生巨大变化的一段时间,在此期间中国足球进行了一系列全新的尝试。1992年,中国足协聘请德国人克劳斯·施拉普纳担任国家队主教练。施拉普纳由此成为中国国家足球队历史上第一个外国籍教练。1993年,中国足协经过挑选,派遣一批少年球员到巴西训练。由于受健力寳赞助,冠名为健力寳青年队。后来,在1999年参加奥运会预选赛的那一批国奥队的很多队员都来自健力寳青年队。1993年底,中国足协开始推广俱乐部制,1994年全新的全国足球甲级A组联赛开战,成为中国足球职业化的开端。","text2":"中国足球职业化的开端是什么?","label":1} {"text1":"希望号(,,意为“希望”)是伊朗首颗自行研制的人造卫星。伊朗国家电视台报道说2009年2月2日成功发射了一颗用于科研和通讯的数据处理人造卫星。伊朗自行研制的 “信使号 2”(Safir)运载火箭发射升空后,卫星已经被送入低地轨道。本次发射恰逄伊朗革命的30周年庆典,总统内贾德出席了发射仪式,并开启卫星传送信号“Allāhu Akbar”(真主至大)。他说发射这颗卫星是为了向全世界传播“一神论,和平和公正”。伊朗外交部长马努切赫尔·穆塔基说发射这颗卫星完全是为了“满足国家需要”,“纯粹是为了和平的目的”。希望号是伊朗的第二颗在轨卫星。2005年俄罗斯为伊朗制造并发射了一颗名为Sina-1的卫星。希望号卫星装载有卫星控制装置及动力供应系统,可用于收集情报和对设备进行检验。但很多功能还没全部激活。伊朗表示发射卫星是为了改进通信技术和互联网应用技术和监测自然灾害等用途,但是西方分析家则认为此举将等于为该国增加了发射洲际弹道导弹的能力。2008年2月4日,在新航天中心揭幕仪式上内贾德总统宣布,希望号将会在不久的将来发射升空。该卫星为了避免飞跃邻国采用的发射方向是东南方,越过印度洋,轨道倾角为55.5度。轨道高度据报道为246──377公里, 轨道周期90.76分钟。","text2":"伊朗表示发射卫星是为了什么?","label":1} {"text1":"黑监督吸蜜鸟(\"Drepanis funerea\")是一种已灭绝的雀。牠们是于1893年在莫洛凯岛发现,曾一度生活在山顶上的树林中。在茂宜岛亦曾发现牠们的化石。由于牠们的外表美丽,很多时会被捕猎来作为标本。黑监督吸蜜鸟的体型较大,由尾巴至喙尖长约8吋,但却不及夏威夷监督吸蜜鸟。牠们呈浅黑色,有一些白色的主羽。牠们的喙长而且较为弯曲,雄鸟的喙较雌鸟的长。牠们有时会接近人群及飞得很低,从未飞高过12呎以上。牠们的栖息地受到牛及鹿的破坏。原本用来捉大家鼠的红颊獴,却掠食牠们。最后观察到黑监督吸蜜鸟是于1907年,当时有三只鸟被射,虽然最后一只逃走了,但牠们肯定已经灭绝。于1936年曾进行了一次大规模的搜寻,但却没有发现牠们的踪影。现时的黑监委吸蜜鸟标本分别存放在不莱梅、波士顿、檀香山、伦敦及纽约。","text2":"黑监督吸蜜鸟在什么地方被发现过?","label":1} {"text1":"吕克·蒙塔尼耶(,) ,法国病毒学家,2008年诺贝尔生理学或医学奖获得者。吕克·蒙塔尼耶是世界艾滋病研究及预防基金会(World Foundation for AIDS Research)的共同奠基人(co-founder)和国际病毒联盟(Program for International Viral Collaboration)的总监(co-directs)。他得到超过20项大奖,包括第三等法国荣誉军团勋章,来思克奖(Lasker Award,1986年,佳德纳基金会国际奖(Gairdner Foundation International Award;Gairdner Award,1987年),和2008年的诺贝尔生理学或医学奖。蒙塔尼耶的会计师父亲喜欢在家里地下室做科学实验,这使蒙塔尼耶从小就对科学产生了兴趣。由于祖父患结肠癌,他决定投身医学。其研究工作主要致力于寻找艾滋病疫苗和疗法。2010年11月,蒙塔尼耶受聘上海交通大学全职教授,并在上海交通大学生命科学技术学院创建Montagnier研究所,专攻艾滋病研究。","text2":"蒙塔尼耶为什么从小就对科学产生了兴趣?","label":1} {"text1":"Mac OS X v10.1「Puma」(「Puma」字面解作「美洲狮」)是苹果为 Mac 产品所制作的作业系统Mac OS X的第二个版本。这个操作系统最初于2001年9月25日发布,增加了系统的效能和新的功能,像是 DVD 播放。由于 v10.0 的口碑不好,苹果为 v10.0 的使用者推出免费的升级 CD,此为对于只有运行 Mac OS 9 的人推出 $129 美元的盒装版本。有人发现升级的 CD 其实就是完整的安装光碟,只要移除一个特定的档案就能够在 Mac OS 9 系统使用;苹果随后重新推出该 CD,实际上真的是把不必要的缩减掉,使得无法在这样的系统上安装。苹果声称Mac OS v10.1 Puma 有多种新特性,包括:","text2":"首次发布的时候增加了什么新的功能?","label":1} {"text1":"梅嘉生(),江苏丹阳人。中国人民解放军军事将领。1938年参加新四军,历任新四军抗日义勇军挺进纵队支队长,团长,苏皖支队副司令员,南通保安旅副旅长,新四军一师三旅参谋长,四分区参谋长兼启海警卫团团长,二分区司令员等。日本投降后,梅嘉生历任苏中军区副参谋长,新四军一师参谋长,华东野战军四纵参谋长,陆军第23军副军长。参加过苏中七战七捷、涟水会战、枣庄会战、莱芜会战、孟良崮战役、鲁南战役、淮海战役、渡江战役和上海战役等。中华人民共和国成立后,梅嘉生赴越南担任军事顾问团参谋长、第一副团长,获越南二级军功勋章。1955年5月,调任海军航空兵部副司令员兼参谋长,是海军航空兵部队的主要创始人之一。1955年获海军少将军衔及二级独立自由勋章、一级解放勋章。1964年任东海舰队副司令员,参与指挥了击沉国民党海军“永昌号”炮舰、击伤“永泰号”猎潜舰的崇武以东海战。1975年任海军副司令员。","text2":"梅嘉生是哪里人?","label":1} {"text1":"维甲酸综合症()是“全反式维甲酸()综合症”的简称,是使用维甲酸治疗急性早幼粒细胞白血病()时发生的最严重并发症,多见于单用全反式维甲酸诱导治疗的过程中。维甲酸综合症的发生率为3%~30%,死亡率为5%~29%。在ATRA诱导治疗的第1~3天早期进行化疗,其发生率为6%~15%,死亡率为6%。初诊时白细胞计数较高及治疗后白细胞迅速上升的患者易发生。该病临床表现有发热、体重增加、肌肉骨骼疼痛、呼吸窘迫、肺间质浸润、心包积液、胸腔积液、皮肤水肿、低血压、急性肾功能衰竭等,严重者可导致死亡。全反式维甲酸引起的的其他不良反应还包括头痛、颅内压增高、肝肾功能损害、皮肤口唇干燥、阴囊皮炎溃疡等。维甲酸综合症的发病机制尚未充分阐明,可能与细胞因子(IL-1、IL-6、TNFα)的大量释放和粘附因子(CD116、CDw65、VLA-4、CD11a\/CD54)的表达增加有关。须暂时停用全反式维甲酸,吸氧、利尿,每日2次静脉注射地塞米松10mg,以及白细胞单采清除和化疗等。在治疗早幼粒细胞白血病合并出血的患者时,除服用维甲酸外,还需输注新鲜冰冻血浆和血小板。维甲酸与其他方法的联合治疗不仅可以提高完全缓解率(CR)和无病生存期(DRS),也可降低维甲酸综合症的发病率和死亡率。","text2":"除停用维甲酸之外还得静脉注射什么物质以及白细胞单采清除和化疗等?","label":1} {"text1":"2010年亚洲运动会棒球比赛,于2010年11月13日至11月19日,在广州奥林匹克体育中心运棒球场举行。参赛队伍有日本、南韩、中华台北、中国、中国香港、泰国、蒙古、巴基斯坦等八队。比赛需要打9局,如打成平局,继续比赛,第10局开始采用突破僵局制,一律从无人出局一、二垒有人时开始进行,直至决出胜负。如双方比分相差10分及以上时,7局可结束比赛;双方比分相差15分及以上时,5局可结束比赛。下列所有的比赛时间皆是北京时间(UTC+8)11月13日 12:0011月13日 13:0011月18日 12:0011月18日 18:0011月19日 12:0011月19日 18:00日本面对蒙古和泰国的比赛中,出现了「无安打比赛」,但是两者都因为比分差距在15分以上而在五局提前结束,而且都是投手接力完成。","text2":"2010年亚洲运动会棒球比赛在哪里举行的?","label":1} {"text1":"凯特·维多莉亚·“凯蒂”·彤丝朵(\"Kate Victoria \"KT\" Tunstall\",),是一位苏格兰的创作歌手。她以在英国BBC电视台的音乐节目「裘斯·荷兰的晚间秀」(\"Later ... with Jools Holland\")上一首现场独唱的自创曲黑马与樱桃树(\"Black Horse and the Cherry Tree\")首度在观众面前亮相。凯蒂·彤丝朵至今在评论界享有不错的评价,并曾获全英音乐奖及葛莱美奖提名。她的外祖母是华人。凯蒂选择以同音的「KT」代替本名「Katie」作为她的名字。她说,「『Kate』总让我联想到一个娇媚的小姑娘,在农田里为她的男人烤著面包。我对那没有任何偏见,我只是觉得,那不是我所想像作为一位摇滚明星的样子。」同时,也与歌手凯蒂·玛露(\"Katie Melua\")做个区隔。2005年,彤丝朵在一些粉丝把她与创作歌手蒂朵(\"Dido\")的音乐做比较后,因公然抨击蒂朵「她X的根本不会唱歌」而惹了一些争议。凯蒂后来道了歉,并表示她不想因此卷入公开的争执。彤丝朵并否认关于她是同性恋的传言,同时澄清她首张专辑封面上的彩虹吊袜带并不是代表同志的身分,她当时对此象征并不知情。2007年4月,彤丝朵动了外科手术,来矫正她因童年感染,而尺寸较一般人小的肾脏的位置。2003年,彤丝朵开始与她乐团里的鼓手路克·布伦交往。2007年的圣诞节,布伦在彤丝朵位于苏格兰圣安德鲁斯的老家向她求婚,两人并于2008年9月6日于苏格兰斯凯岛的弗洛地格瑞乡村别墅旅馆结婚。","text2":"凯蒂·彤丝朵至今在评论界享有不错的评价,并曾获什么奖?","label":1} {"text1":"保加利亚是一个位于欧洲东南部巴尔干半岛上的国家,毗邻罗马尼亚、塞尔维亚、马其顿共和国、希腊、土耳其和黑海。多瑙河构成保加利亚同罗马尼亚之间的边界(直到锡利斯特拉为止)。保加利亚国土面积110,550平方公里,略大于冰岛或美国的田纳西州。 保加利亚位于黑海西海岸。北与罗马尼亚,西同塞尔维亚和马其顿,南与希腊和土耳其接壤。虽然保加利亚的国土面积不大,但地形却十分多样。横贯保加利亚国土中央的巴尔干山脉将保加利亚的河流分为两大水系。以北的河流多注入多瑙河,以南的则多注入爱琴海。面积:\"总计:\"110,910 km²\"陆地:\"110,550 km²\"水域:\"360 km²面积比较:略大于田纳西州。陆地边界\"总计:\"1,808 km\"接壤国家:\"希腊 494 km;马其顿共和国 148 km;罗马尼亚 608 km;塞尔维亚 318 km;土耳其 240 km海岸线:354 km最低处:黑海 0 m最高处:穆萨拉峰 2,925 m","text2":"多瑙河对于保加利亚而言有着怎样的地理意义?","label":1} {"text1":"协和桥(Pont de la Concorde)是法国巴黎一座跨越塞纳河的拱桥,介于协和广场的堤道(quai des Tuileries)(右岸)和奥赛堤道(quai d'Orsay)(左岸)之间。它在过去曾称为路易十六桥(pont Louis XVI)、革命桥(pont de la Révolution)、协和桥,波旁复辟时期(1814年)复称路易十六桥,1830年再度恢复协和桥名称,直至今日。附近有地铁的国民议会站和协和站。1787年,建筑师Jean-Rodolphe Perronet受命建造新桥。计划开始于1755年,当时开始兴建路易十五广场(今协和广场),以取代此处的渡口。在动荡的法国革命中工程继续进行,利用了拆卸巴士底狱的石材建造,完成于1791年。1810年,拿破仑一世在桥的置于两旁安放法兰西第一帝国8位阵亡将军的雕像。波旁复辟时期改为12座巨大的大理石雕像,其中包括4名大臣Suger、苏利、黎塞留、柯尔贝尔,4名皇家将军(杜·盖克兰、贝亚德、孔代, 蒂雷纳子爵)和4名航海家勒内杜高依、迪凯纳、Suffren、图尔维尔.然而,这些雕像对于桥梁太过沉重,路易-菲利普一世将它们转移到了凡尔赛宫。过桥交通变得十分拥挤,1930年到1932年,该桥加宽到原来的2倍。工程师德瓦尔和马利特仔细维护新古典主义建筑原貌。1983年最后一次修复。今天,这座桥是巴黎的道路交通要冲(环城大道的桥梁除外)。","text2":"1810年,拿破仑一世在桥的置于两旁安放什么?","label":1} {"text1":"萧规曹随,汉语中的一个成语,形容继任人沿用前任定下的规则、做法等等,不作更改。萧何与曹参两人少年担任秦朝小官吏时,便已是好友,后来西汉建国,萧何身为宰相,曹参身为大将,地位不凡的两人却反而有了嫌隙。萧何担任相国时,参考前朝文献制订典章及制度。萧死前,推荐曹参继任。曹参上任后,认为萧何订下的法令已很完备,所以继续沿用而不作改动。曹参就任汉相国期间,整日饮酒食肉,政治上清静无为,继续执行萧何留下的政策,不予变动。汉惠帝认为自己被曹参轻视,于是命其子御史大夫曹窋劝谏,曹参把曹窋鞭笞了两百下并赶出门外,汉惠帝于是亲自责问曹参。曹参摘帽,向皇帝俯首谢罪:「陛下您认为,您与先帝相比,谁较为英明神武?」皇帝回道:「我怎敢与先帝比?」曹参又问说:「我跟萧何比,谁较贤能?」皇帝说道:「您好像不太比得上他。」曹参接著说:「陛下说得对,且高祖跟萧何平定了天下,法令都健全具备。陛下只要垂拱而治,我们这些官吏坚守岗位,遵守他们的法令而不犯过失,不是很适当吗?」时人歌颂:「萧何制定法律,调和整齐如一;曹参继任相国,遵法而不犯过失。施载清净无为的政策,人民因而安宁统一。」史称「萧规曹随」。据前汉书记载,曹参与萧何于沛县时为挚友,但萧何担任相国时反生嫌隙。不过曹参为汉相国时,却对萧何担任相国时之制度一无变更。现今中文使用中,世人所熟知的「萧规曹随」有率由旧章、依样葫芦、一成不变、沿袭旧规等等意涵,然而曹参为汉相国时,却不是此成语所指称之无所事事,反而是选拔贤才、注重吏治,各官吏均选任老成持重之人,若有好名之人,即使其去职。可见「萧规曹随」的传统见解对曹参为汉相国的治术略有误解。曹参担任相国时,对于下属有极小过错,反而为其隐忍不公开。原典内容如下:司马迁著《史记·卷五十四·曹相国世家》:「参代何为汉相国,举事无所变更,一遵萧何约束。……惠帝怪相国不治事,以为『岂少朕与』?……参免冠谢曰:『陛下自察圣武孰与高帝?』上曰:『朕乃安敢望先帝乎!』曰:『陛下观臣能孰与萧何贤?』上曰:『君似不及也』。参曰:『陛下言之是也。且高帝与萧何定天下,法令既明,今陛下垂拱,参等守职,遵而勿失,不亦可乎?』惠帝曰:『善。君休矣!』……百姓歌之曰:『萧何为法,顜若画一;曹参代之,守而勿失。载其清净,民以宁一。』」扬雄著《法言》·〈渊骞第十一〉:「萧也规,曹也随。」","text2":"曹参上任后,是怎么做的?","label":1} {"text1":"《过敏世界》是香港歌手张学友的第十六张粤语专辑,唱片制作人欧丁玉,宝丽金唱片公司制作发行。《过敏世界》在推出后广受好评,并受到市场的极度认可,坐拥五首冠军歌曲,而其中三首为四台冠军歌,是香港流行音乐历史上的一张重要唱片。过敏世界专辑中的同名主打歌曲《过敏世界》充斥了对香港狗仔队跟踪名人等一些不检点行为的批评,其中歌曲由张学友亲自作曲。由于争议性很大,虽然被定为第四主打歌,但并未在香港的电台、电视中播放,被之前定位为非主打歌的《这么近(那么远)》所取代。在录制此张唱片前,张学友曾经奔赴美国进修音乐,而唱片在发行后体现了他的进步非常明显,特别是由他亲自谱曲的两首歌曲《这么近(那么远)》,以及《过敏世界》被专业人士认为具有良好的音乐质素。另外,他用高亢的嗓音演绎了歌曲《我哭了》、《无心恋》以及《这么近(那么远)》,得到乐迷一致好评。歌曲《屈到病》讽刺当时香港的一些家庭主妇妻管严风格。第三主打歌《离开以后》更是包揽了多个当年香港音乐颁奖礼的奖项。值得注意的是一首开创香港流行音乐串唱先河的一首歌曲《友情歌》,他将自己于1985年至1993年的一些经典歌曲会聚成为极其押韵的一首歌曲,歌曲长达13分钟半,编曲人亦特别出色的结合了各个歌曲不同的曲调,形成为一种独特的风格,香港著名填词人黄霑亦有在当时表示,《友情歌》是香港流行音乐一种新的尝试和突破。进入2000年后,歌手古巨基等亦有演唱类似于友情歌的《劲歌金曲》等歌曲,但大多数歌曲都是改变来自他人演唱过的旧歌。","text2":"香港歌手张学友的第十六张粤语专辑是什么?","label":1} {"text1":"乔治·斐兹杰惹(,)又译乔治·菲茨-{杰拉德}-,全名乔治·弗朗西斯·斐兹杰惹(George Francis FitzGerald),大不列颠与爱尔兰联合王国的爱尔兰裔物理学家,都柏林三一学院教授。乔治·斐兹杰惹于1851年8月3日在大不列颠与爱尔兰联合王国爱尔兰岛都柏林出生,父亲威廉·斐兹杰惹(William FitzGerald)是基拉卢的主教,母亲叫安妮·弗朗西斯·斯托尼(Anne Francis Stoney),舅父乔治·约翰斯顿·斯托尼(George Johnstone Stoney)是提出「电子」这术语的物理学家。1883年,乔治·斐兹杰惹依据马克士威方程组提出制造急速振荡电流即可产生电磁波的仪器。1888年,海因里希·赫兹证明电磁波的存在。1892年,乔治·斐兹杰惹尝试解释迈克生-莫立实验:「如果物质是由带电荷的粒子组成,一根相对于以太静止的量杆的长度,将完全由量杆粒子间取得的静电平衡决定,而量杆相对于以太在运动时,量杆就会缩短,因为组成量杆的带电粒子将会产生磁场,从而改变这些粒子之间的间隔平衡。这一来,迈克耳孙-莫雷实验所使用的仪器,当它指向地球运动的方向时就会缩短,而缩短的程度正好抵消光速的减慢。」有些人曾经试行测量乔治·斐兹杰惹提出的缩短值,但都没有成功。这类实验表明乔治·斐兹杰惹提出的缩短,在一个运动体系内是不能被处在这个运动体系内的观察者测量到的,所以他们无法判断他们体系内的绝对速度,光学的定律和各种电磁现象是不受绝对速度的影响的。再者,动系中的短缩,乃是所有物体皆短缩,而动系中的人,是无法测量到自己短缩值的。1904年,荷兰物理学家亨德里克·洛伦兹提出了著名的洛伦兹变换,成功解释迈克耳孙-莫雷实验的结果。乔治·斐兹杰惹受消化系统问题长期困扰。1901年2月21日,在家中因溃疡而死,终年49岁。","text2":"最终成功解释迈克生-莫立实验的是谁?","label":1} {"text1":"杨清(,)是越南反抗唐朝的起义军首领。史书未记载生年。据吴士连的《大越史记全书》记载,杨清交州龙编人,其祖先「世为蛮酋」。唐朝开元年间,杨清被唐朝任命为驩州刺史。当时的安南都护李象古以「贪纵苛刻而失众心」,见杨清据守驩州,心中忌之,于819年召杨清至安南都护府任自己的牙将,令其带三千人兵前往镇压黄洞蛮。杨清见人心忿怒,暗与城中的杜士交、杨志烈勾结,率领所部乘夜袭破安南都护府,杀李象古和他的家属。据《旧唐书》记载,杨清起兵后,820年,唐朝任命唐州刺史桂仲武为新任安南都护,奉命讨伐,并许诺赦免杨清,封之为琼州刺史。杨清不从,据城反抗。桂仲武遣人招降杨清的属下。因杨清残暴不仁,部将纷纷献城投降。杨清与其子杨志贞被捕处决。杜士交拥立其子杨志烈为主,退往长州的凿溪(在今越南宁平省)继续抗唐。但不久即因势单力薄,率军投降唐朝。然而《大越史记全书》却与《旧唐书》的记载截然不同。《大越史记全书》称,杨清击败了唐朝桂仲武军,随后连结安南地区的各少数民族一起反唐,并连合南方的占城国攻击唐朝。杨清又于828年、841年两次败唐军的韩约、武浑部,此后《大越史记全书》便突然不见了关于杨清的记载。据《》:「(元和十五年三月)辛未,杨清伏诛。」以及《》:「(元和十五年三月)辛未,安南将士开城纳桂仲武,执杨清,斩之。」","text2":"为什么杨清的部将纷纷献城投降?","label":1} {"text1":"让-弗朗索瓦·科佩(,),法国政治家,现任人民运动联盟主席。1964年5月5日出生于巴黎西南郊区布洛捏—比昂固赫(Boulogne-Billancourt)市的一个来自罗马尼亚的犹太家庭,毕业于法国国立行政学院(ENA)自由、平等、博爱届(1987-1989),自1995年起任塞纳-马恩省地区首府牟市(Meaux)市长,1997-2002年在巴黎第八大学教经济和金融。2002-2007年出任国务秘书、部长级代表及让-皮埃尔·拉法兰和多米尼克·德维尔潘两届政府的发言人。2007年,尼古拉·萨科齐担任总统后,未进入政府,以其国民议会议员身份竞选为国民议会人民运动联盟(UMP)党派主席。2010年11月17日出任人民运动联盟(UMP)书记。在尼古拉·萨科齐连任总统失败后,他宣布辞去人民运动联盟党主席一职。2012年11月18日的党主席选举中,以微弱优势战胜竞选对手前总理弗朗索瓦·菲永,但后者也宣称以微弱优势战胜对手。次日党内选举活动组织与检查委员会(COCOE)宣布科佩以50.03%的优势获胜,但菲永阵营仍然不承认这一结果。在经过一周的激烈论战与互相攻击后,其党内特况处理国家委员会(CONARE)宣布科佩以 50,28 %的选票胜选。但菲永阵营始终拒绝承认对方获胜的选举结果。12月末他们两个最终达成协议,科佩接受在2013年进行新一轮选举并修改选举章程,菲永则解散他在国民议会成立的党内独立小组,人民运动联盟党政治局也重申了这一决定。","text2":"让-弗朗索瓦·科佩现任什么职务?","label":1} {"text1":"斐迪南一世(,)是霍亨索伦亲王利奥波德和葡萄牙公主安东妮亚的长子。于1914年10月14日到1927年7月20日期间在位。在第一次世界大战中,罗马尼亚王国加入了协约国一方,同德意志帝国作战。但斐迪南本人却是德国贵族霍亨索伦家族的成员。第一次世界大战结束后,罗马尼亚获得了大片的土地。斐迪南于1927年7月20日逝世,他的继任者是米哈伊一世。生于德意志西南方的锡格马林根,头衔为霍亨索伦-锡格马林根亲王斐迪南·维克多·亚伯特·麦因拉特。过去的霍亨索伦-锡格马林根亲王一直统治著霍亨索伦-西格马林根侯国,直到1850年普鲁士并吞了霍亨索伦-西格马林根侯国。","text2":"斐迪南一世是谁?","label":1} {"text1":"化学物质毒性数据库(RTECS,Registry of Toxic Effects of Chemical Substances)是一个记录化学物质毒性资料的数据库。它的资料均来源于公开的科学文献,不过并没有指出那些研究报道是否足够可靠。在2001年之前这个数据库是美国国家职业安全卫生研究所()的一个免费提供的出版物。如今RTECS属于硅谷高科技公司(),并且只能通过收费订阅方式获得。RTECS中主要包括以下六大类化学物质的毒性数据:其中记录有该化学物质的数值毒性值,如半数致死量(LD50或LC50),最低中毒剂量(TDLo),最低中毒浓度(TCLo)等,以及实验所使用的物种和给药途径。除此之外,所有的数据也都列出了其文献来源,然而并没有对其有任何的评估。RTECS根据美国于1970年颁布的职业安全和健康法(PL 91-596)第20条(a)(6)款建立,是一项美国国会授权的活动。它原先的版本,有毒物质列表(Toxic Substances List),于1971年6月28日发布,包括了约5,000种化学物质的毒理学资料。不久,其名称就变为了现在的化学物质毒性数据库(Registry of Toxic Effects of Chemical Substances)。截至2001年1月,该数据库已包含有152,970种化学物质。2001年12月,RTECS被NIOSH转让给了爱思唯尔MDL(Elsevier MDL),一家私营公司。随后,硅谷高科技公司于2007年收购了爱思唯尔公司的MDL,RTECS也在收购内容中。如今这个数据库只能通过每年的收费订阅来获得。RTECS由加拿大职业健康安全中心(Canadian Centre for Occupational Health and Safety)提供,有英语,法语和西班牙语三个语言版本。用户可通过互联网、企业内部局域网或光盘的方式订阅获得,也可以在美国国家信息服务公司(NISC,National Information Services Corporation)及专家出版公司(Expert Publishing, LLC)上找到。","text2":"在2001年之前这个数据库是哪个公司提供的出版物?","label":1} {"text1":"丁谓(),字谓之,后更字公言,北宋时期苏州长州(今江苏苏州)人。善言谈,喜欢作诗,于图书、博奕、音律无一不精。出自寇准门下,太宗淳化三年(992年)进士,授大理寺评事、通判饶州事。真宗咸平初除三司户部判官,大中祥符初,权三司使。咸平五年(1012年)任户部侍郎。官至参政知事。当政后极力排斥寇准,干兴元年(1022年)二月,再贬寇准为雷州司户参军。丁谓同党雷允恭因先帝陵寝工程事故,坐“擅移皇堂”罪,丁谓受牵连,贬为太子太保。后以“丁谓前后欺罔”罪,被贬崖州(今海南省琼山县)司户参军。以秘书省监致仕归里。景祐四年(1037年)病卒。著有《丁谓集》8卷、《虎丘集》50卷、《刀笔集》2卷、《青衿集》3卷、《知命集》1卷,皆佚。大中祥符年间,皇宫内失火,丁谓奉命修缮被烧毁的宫殿,但是苦于取土的地方很远。丁谓竟然命令工人挖大街上的土,没几天大街就成了深深的水沟,丁谓又命令工人把汴水引入沟中,再用很多竹筏、木筏与船运送建材,顺著水沟,运到宫中。修完宫殿后,把废弃的瓦砾泥土填到沟里,又填成街道。一举三得,省下了亿万文钱。丁谓机敏有智谋,然有才无德,寇准喝汤时弄污胡须,丁谓立刻帮寇擦胡子,寇准调侃他说:“参政是国家的大臣,竟然会帮长官擦胡子啊!”丁谓怀恨,拜相后排挤寇准,“只要跟寇准要好的,一律贬斥。”丁谓被贬崖州,路过雷州时,寇准派人送一蒸羊在路迎他。寇准还怕自己的仆人找丁寻仇。。有僧妙海者,昔出入丁谓门下,丁谓移住光州时,妙海往见之,丁谓与之相别曰「吾不死,五年当复旧位。」后五年,元昊反,边事起,朝廷更用大臣,丁谓沐浴衣冠,卧佛堂中而薨。尚书省左仆射、资政殿大学士,判郓州事王曾闻丁谓卒,语人曰「斯人智数不可测,在海外犹用诈得还。若不死,数年未必不复用。斯人复用,则天下之不幸,可胜道哉!吾非幸其死也。」","text2":"丁谓擅长喜欢些什么?","label":1} {"text1":"哈卡斯人(Khakas) 亦作Khakas或Hakas,又称“米努辛斯克鞑靼人”、阿巴坎鞑靼人,“叶尼塞鞑靼人”、叶尼塞土耳其人等,属蒙古人种西伯利亚类型。使用哈卡斯语,分萨盖、卡钦、克孜尔3种方言,属阿尔泰语系突厥语族。哈卡斯人主要分布在俄罗斯哈卡斯共和国、部分分布在克拉斯诺亚尔斯克等地,另外还有一部份分布在中国黑龙江省齐齐哈尔市富裕县,现在约有1500人,被称为富裕柯尔克孜人,目前被划分为柯尔克孜族。哈卡斯人原分五部,即“卡钦”、“萨盖”、“克孜尔”、“别尔季尔”和“科伊巴尔”。其起源有争议,有学者认为哈卡斯人是在17、18世纪时,叶尼塞吉尔吉斯人、凯特人和萨莫耶德语族等逐渐融合而形成的。他们曾经建立一帝国,后受准噶尔部攻击,俄罗斯用很大气力征服他们,要他们交毛皮税。人们认为他们与柯尔克孜族同族,也是原欧洲人。黑龙冮富裕县柯尔克孜族与他们同族。哈卡斯人使用哈卡斯语,属突厥语族北突厥语群,分萨盖、卡钦、克孜尔三种方言。1926年创制了以西里尔字母为基础基础的拼音文字,1929年一度改用拉丁字母,1939年又恢复使用西里尔字母。哈卡斯人传统生活方式是半游牧半狩猎,也种植小麦、燕麦等。哈卡斯人本信仰萨满教,19世纪以来,被沙俄征服后,一些民众在形式上皈依东正教,但仍然保留一些萨满教传统。分布在中国黑龙江省的则信仰藏传佛教。哈卡斯人会玩一种近似土耳其跳棋的棋类,称为哈卡斯跳棋。","text2":"哈卡斯人会玩一种近似土耳其跳棋的棋类,称为什么?","label":1} {"text1":"霍弋(,卒于271年或以前),字绍先。荆州南郡枝江人,三国时期蜀汉及西晋将领。父亲为霍峻,同样是蜀汉将领。蜀汉昭烈帝末年即223年,为太子舍人。刘禅登位后,除谒者。任在公元226年,随诸葛亮北伐,诸葛亮任他为记室,让他和儿子(实为侄子)诸葛乔相处。诸葛亮死后,任黄门侍郎。刘禅立刘璿为太子,以霍弋为太子中庶子。刘璿好骑射,出入无度,霍弋援引古书进行规劝,表现很得体。后来参军,任庲降都督「副贰都督」,转任「护军」,统事如前。霍弋曾领永昌太守讨伐贼军,后迁监军翊军将军,兼任建宁太守,统治南中诸郡。263年,升为安南将军。同年,霍弋闻魏兵入蜀(魏灭蜀之战),欲赴成都,刘禅以备敌既定,不听霍弋言。霍弋闻成都失守、蜀汉亡国后,素服望西大哭三日。后降于魏,司马昭拜为南中都督,与巴东太守罗宪守备蜀地。霍弋后来遣兵救援吕兴,吕兴被其手下功曹杀害,霍弋表奏建宁爨谷为交趾太守,又率牙门将董元、毛炅、孟干、孟通、爨能、李松、王素等人出征,平定交趾、日南、九真三郡,封为列侯,进号崇赏焉。《资治通鉴》上说:霍弋自宁州(今云南)遗杨稷等人经略交、广二州,采取水、陆二路并进。交州平定后,霍弋令杨稷、毛炅等戌守交趾,并与他们起誓说:「如果贼兵围城不满百日而投降,你们的家属就要受诛;但如果超过百日而救兵不至,以至于城池陷落,就由我霍弋担起责任。」271年,交州又被吴军陶璜夺回,杨稷、毛炅粮尽援绝,守城百日后投降而就死。在三国演义称霍戈,内容与正史无大差别,主要描述尽在三国演义第119回。","text2":"霍弋的父亲是谁?","label":1} {"text1":"肖特330(又名SD3-30)是一款由肖特兄弟公司研发,以Skyvan为基础的小飞机,载客量达30人,投入服务时以相对低廉的价格和较低的运营成本而著称。肖特330是一款由贝尔法斯特的肖特兄弟公司研发,以Skyvan为基础的短距起落飞机,机身横切面与Skyvan一样,采用高单翼设计,双尾翼,但拥有比Skyvan更长的翼展和机身,机身延长了3.78米,最大载客量达30人,载油量比Skyvan多出60%,起落架可收缩。首架肖特330原型机于1974年8月22日首飞,。肖特330有两款货运型号,第一款是Short 330-UTT(Utility Tactical Transport),是一款强化了地板和可跳伞舱门的军用机,,此型号产量不多,主要用家是泰国,购了4架。另一款是雪尔帕人型(Sherpa),拥有更宽的机尾货门,首飞于1982年8月22日进行,首个用家是于1983年订了18架的美国空军,主要用于在欧洲的美军基地间运输备用零件。肖特330是一款短程或通勤用客机,计设符合美国通勤运输机规条,载客量达30人,用于取代旧款的毕琪99和DHC-6。肖特330于1976年8月投入服务,用家是加拿大的时间航空。尽管它的外型肥胖(有一家区域航空公司称它为「小屋」),但它很快便证明自己是一款可靠且低成本的飞机。肖特330比其竞争对手稍慢一些,但它有着安静、舒适和坚固的声誉。肖特330所采用的普惠PT6A-65R发动机因为高效齿轮箱而变得安静。机舱内部配置和装饰是与波音工程师共同参考大型客机开发和设计的,机身结构比较强,支外翼和方型机身有利于维护和服务。肖特330于1992年9月停产,共生产了136架(包括军机和货机),至1998年约有35架仍然运作中。肖特330的设计成为肖特360的设计蓝本。","text2":"雪尔帕人型首个用家是谁?","label":1} {"text1":"大刺鳅(学名:)为辐鳍鱼纲合鳃目刺鳅科刺鳅属的一种,俗名锯齿泥鳅、带刀鱼、猪妈锯、刀鳅。本鱼分布于亚洲地区的河川,包括中国南部及西南部、越南、寮国、泰国、柬埔寨、缅甸、印度、不丹、尼泊尔、印尼、巴基斯坦、马来西亚、斯里兰卡等。本鱼体修长,上部分为金黄色,下半部为深褐色,腹面有两排椭圆形斑块。臀鳍、背鳍及尾鳍,在鱼体后部连接成一片。背鳍硬棘33至40枚;背鳍软条67至82枚;臀鳍软条67至83枚;脊椎骨87至98枚,体长可达90公分。本鱼多生活于有石块的江河底层、或岸边有水草处以及在乱石缝隙中。属肉食性,通常于夜间活动,具侵略性,以底栖昆虫、蠕虫等为食。在原产地为食用鱼,另外也具观赏的价值。","text2":"大刺鳅的上部分为什么颜色?","label":1} {"text1":"阿卑多斯战役(Battle of Abydos),发生在公元前322年春天,为希腊化时代一场较不显著的海战,是马其顿军队与雅典人所领导的希腊城邦联军在拉米亚战争中的第一场海战。当雅典得知亚历山大大帝在公元前323年6月逝世后,他们决定反抗马其顿在希腊的霸权,并招募一支强大的雇佣军和重建一支强大的海军,许多希腊城邦也纷纷加入。此时,因为马其顿在东方的征服行动,马其顿的欧洲总督安提帕特仅拥有少量的部队,安提帕特因此撤退至拉米亚,并要求亚洲迅速增援。此时,克拉特鲁斯正在奇里乞亚修建舰队,命令克利图斯作为舰队指挥官,先增援一部分军队给安提帕特,而雅典海军试图控制爱琴海制海权,来阻止亚洲对拉米亚增援,两方舰队于阿卑多斯附近发生战斗,雅典海军遭到马其顿海军击败,部分船只遭到俘虏,但雅典还保有一定实力。","text2":"阿卑多斯战役是谁领导的?","label":1} {"text1":"曼努乔(Manucho,全名Mateus Alberto Contreiras Gonçalves,),是一名安哥拉足球运动员,司识前锋、左翼,现时效力西甲球会华拉度列。出生于安哥拉首都罗安达的曼努乔,早期一直都是在自己国家之联赛打滚。2007年12月英超球会曼联到安哥拉看中了曼努乔。时曼联锋线人手紧张,曼联未作长时间考虑,就决定把曼努乔签下。2008年1月曼努乔加盟曼联,加盟初期由于未获工作许可,而被转借到希腊帕纳辛纳克斯。2008年8月他凭借之前在非洲杯上代表国家队的出色表现,成功获得工作许可,并且在9月23日的联赛杯第三圈中第一次代表曼联出场,后补入替出战米杜士堡。11月15日第一次在联赛中亮相,主场对史笃城于74分钟入替卡洛斯·特维斯,不过这也成为他曼联生涯的唯一一次联赛出场。综观在曼联时曼努乔都未能争取得正选,2009年1月16日,曼努乔答应被租借至侯城直到球季结束。7月17日转投西甲球会华拉度列,签约五年。文奴祖代表安哥拉国家足球队参加国际赛。在2008年非洲国家杯表现出色,虽然安哥拉最终在八强止步,但文奴祖也能入选2008年非洲国家杯最住阵容,更以4球成为四名非国杯神射手亚军之一,仅次于5球的伊度奥。","text2":"曼努乔是哪国人?","label":1} {"text1":"李冲(),原名思冲,字思顺,魏孝文帝替他改名冲,陇西郡狄道县(今甘肃省定西市临洮县)人,北魏镇北将军、敦煌宣公李宝的幼子,北魏官员,陇西李氏仆射房始祖。李冲好交游,北魏孝文帝初年,任秘书中散、内秘书令、南部给事中。迁中书令,加散骑常侍,仍兼给事中。随后转南部尚书,赐爵顺阳侯。李冲提出均田制、三长制,孝文帝采纳,以五家为一邻,五邻为一里,五里为一党,各设一长。北魏国力大增。改置百司,开建五等,以冲参定典式,封荥阳郡开国侯,食邑八百户,拜廷尉卿。寻迁侍中、吏部尚书、咸阳王师。立太子后,拜为太子少傅。李冲受文明太后所宠幸,每月赏赐多达数十万钱,进爵陇西公,太后又密赠珍宝、御物到其府第,外人不得而知。李冲原本家境清贫,自此成为富室;然而李冲谦虚自牧,慷慨散财,从族人姻亲到乡里邻人,都分及财物。493年,北魏迁都洛阳,任镇南将军、侍中、少傅,负责营建新都,封阳平郡开国侯。迁尚书仆射、仍领少傅,改封清渊县开国侯。太子元恂被废时,李冲罢领少傅。李冲年四十余岁时,鬓发班白,姿貌丰美,未有衰老之状。李冲因自己提拔的李彪跟自己利益相背,而跟其他贵族一起攻讦李彪。李冲个性素来温柔,但一旦暴怒时,便发病狂悸,言语错乱,且会扼腕叫骂,称李彪为小人。医药无法治疗,医师或有诊断李冲为肝藏受伤破裂的。十余日后卒,卒年四十九,赠司空,谥曰文穆。葬于覆舟山。杜预早生李冲150多年,二人并称李杜。六世孙李元亨,安邑县令。李元亨生李昌庭,唐朝棣王友。李昌庭孙朝议郎、滑州酸枣县令李宙。李宙夫人卢氏之母郑氏为郑善果来孙。","text2":"他为何攻讦李彪?","label":1} {"text1":"盾蟒(学名:),又称头黑锦蛇,是脊索动物门爬虫纲有鳞目蛇亚目蟒科盾蟒属的生物,无毒性。主要分布在澳洲的北半部,是当地的特有蛇种。在澳洲原住民的梦世纪(Dreamtime)文化里,盾蟒的形象经常出现于许多传说故事之中。纪录中最长的盾蟒大概有300厘米长,以蟒科动物而言,盾蟒的体型并不算巨大。身体普遍呈淡黄色,有黑色或红色的间条,特色在于其头颈部位多呈深黑色。盾蟒学名中的\"melanocephalus\"在英语中就有「黑色的头」的含义。身体颜色根据地域差异亦会有所分别。盾蟒适应干燥的环境,因此主要生长于沙漠、荒野、热带草原、森林等地带。具备夜行性,多于夜间活动,日间则会躲藏于由其它动物所掘的洞穴或茂密的林间栖息。与一般蟒科动物一样,盾蟒是肉食性生物,主要进食爬虫类、鸟类及小型哺乳类的动物。以卵生繁殖,雌蛇每次能生产3至10枚蛇卵,怀孕期大致相隔两至三个月。发育不良的幼小盾蟒将会成为同伴的食物,天敌为人类及澳洲野犬。由于澳洲积极开拓北部地区,直接使盾蟒的栖息范围锐减,盾蟒的数字亦因而有所下滑,目前已受到动物保育组织的关注。在日本有饲养盾蟒的情况,不过由于澳洲当地的盾蟒栖息环境受到影响,政府下令禁止输出野生品种的盾蟒,因此日本国内作饲养用途的盾蟒,多数是以往从澳洲运往日本用作研究及展览的少量盾蟒的繁殖种。这些繁殖种数量极少,令日本的盾蟒价格高涨。野生的盾蟒多数以捕食爬虫类动物为主,而受饲育的盾蟒主要食粮则是老鼠之类的小型哺乳动物。","text2":"盾蟒有毒吗?","label":1} {"text1":"H-α,在天文学和物理学上是氢的一条具体可见的红色发射谱线,波长为6562.8 Å。依据原子的波耳模型,电子是存在于量子化能阶的轨道上绕著原子的原子核。这些能阶以主量子数 \"n\" = 1、2、3、... .来描述,电子只能存在于这些状态中,并且也只能在这些状态中转移。这一组从 \"n\" ≥ 3 转换至 \"n\" = 2 的谱线称为巴耳末系,并以连续的希腊字母依序为成员命名:在来曼系,命名的惯例是:H-α的波长是656.281 奈米,是在可见电磁频谱的红色部分,并且是天文学家追踪气体云气中被电离的氢含量最容易的方法。因为将氢原子的电子从 n = 1激发到n = 3,与将他游离的能量几乎相同,因此电子被激发到n = 3而不被游离的机率是非常小的。反而是,在被电离之后的氢核再与新的电子再结合成氢原子时,在新的原子,电子可以先存在于任何一个能阶上,然后再落至基态(n=1)并辐射出光子来转换。几乎有一半的时间,这些能阶会包括n=3至n=2的转换,因此原子将辐射出H-α。所以,H-α发生在氢被电离的区域内。因为氢是星云的主要部份,相对的H-α很容易自吸收而饱和,因此他可能显示云气的形状和范围,但不能用来确定云气的质量。替代的,二氧化碳、一氧化碳、甲醛、氨或甲基氰化物是典型的用于测定云气的质量。H-α滤镜是以H-α的波长为中心设计的一种窄频带宽的光学滤镜,它们以能通过滤镜的波长带宽来描述其特性。这些滤镜由多层(~50)的真空喷涂来镀镜,这些层次都选择能导致干涉效应以过滤掉除了需要的波长以外的所有波长。另一种选择是使用光标准具(etalon)做为窄频滤镜(以阻拦过滤器或能量阻绝过滤器),以H-α发射线的波长为中心只让很窄的波长通过(<1 Å)。光标准具和二向色性干涉滤波器的物理在本质上是相同的(依靠在表面之间的建设性或破坏性干涉反射),但执行的方法不同(干扰滤波器依靠内部反射的干涉)。由于可见的H-α特征有时会与高速度结合(像是高速移动的日珥和喷发物),太阳H-α光标准具通常都能调整(经由倾斜或改变温度)来应付伴生的都卜勒效应。","text2":"H-α是什么?","label":1} {"text1":"'(;英语简称为')是在日本、香港、台湾、韩国等地举办的同人志即卖会,由日本SE株式会社(,今Deleter株式会社)开办,并在日本以外地区委托当地同人团体举办。主要在中小型都市举办。但在2008年8月3日举行的「Comic World in 横滨 49」后,现已全面停办。Comic World 香港(,简称CWHK),是香港最早举办的大型同人志即卖会,每年举行两次。台湾同人志贩售会(,简称为CWT),由台湾同人志科技股份有限公司主办,每半年举行一次,分为CWT(台大场)、CWT-T(台中场)与CWT-K(高雄场),现今多以台北市台大综合体育馆、台中市逢甲大学体育馆、高雄市社教馆综合体育馆为举办场地。其观展族群多以女性为主,参与社团的作品也多为女性向。另外,也在2010年10月17日于香港举办一届「Comic World Taiwan - Hong Kong」(简称 CWT-HK),与CWHK不同。竞争对手为开拓动漫祭。Comic World从1999年开始,在首尔与釜山巡回举办。每回活动均以「(○回)(都市名)Comic World」命名。","text2":"Comic World在哪里举办?","label":1} {"text1":"世界羽毛球联合会(),原羽毛球组织,于1978年2月成立,1981年5月26日与国际羽毛球联合会(现称羽毛球世界联合会,BWF)合并后停止运作。成立于20世纪30年代的国际羽毛球联合会(国际羽联,IBF)一直是管理国际羽毛球的机构。几个国家对国际羽联的数项政策不满:国际羽联接受中华民国为国家组织;拒绝因南非政府的种族隔离政策而中断南非的会员资格及拒绝修改国际羽联章程中的不合理条款。这几个国家在非IBF成员中华人民共和国的带领下,成立世界羽毛球联合会。于1977年9月召开由亚洲羽球联盟由发起的会议,提出了成立世界羽毛球联合会。同年11月,在世界羽联筹委会第二次会议上,非洲羽毛球联合会宣布退出国际羽联。1978年2月在香港召开的第3届会议上正式宣布成立世界羽毛球联合会。几乎全部来自亚洲和非洲的二十二个国家和地区加入世界羽联:孟加拉国、文莱、中国、加纳、圭亚那、香港、印度、伊朗、肯尼亚、马来西亚、毛里裘斯、尼泊尔、尼日利亚、朝鲜、巴基斯坦、菲律宾、新加坡、韩国、斯里兰卡、坦桑尼亚、泰国和赞比亚。 奥地利、法国、墨西哥(代表泛美羽毛球联合会)、瑞典、联邦德国和南斯拉夫派出没有投票权的观察员列席会议。之后世界羽毛球联合会组织了两届自己的世界锦标赛,一届于1978年在曼谷举行,另一届于1979年在杭州举行。在中华民国主管组织更改名称为「Chinese Taipei Association(-{中华台北羽球协会}-)」及国际羽协禁止南非羽毛球协会参加汤姆斯杯、尤伯杯和世界羽毛球锦标赛后,两个羽毛球联合会决定合并。1981年3月举行了特别会议,投票结果57对4支持合并。1981年5月26日,世界羽毛球联合会正式与国际羽毛球联合会正式合并。","text2":"世界羽毛球联合会最终与哪一组织合并?","label":1} {"text1":"深沪镇是位于中华人民共和国福建省泉州市晋江市东南部的一个镇。由于靠近台湾海峡,渔业资源丰富。是渔业养殖基地和贸易港口。并且有许多的特色小吃,如深沪鱼丸,马加羹,目鱼球都是闽南地区较为有名的小吃,目前其主要支柱产业为纺织业,并已其为主导引领周边产业发展。深沪镇镇政府驻狮峰村。深沪镇共辖7个社区、12个行政村,分别是:狮峰社区、金屿社区、东埯社区、后山社区、璧山社区、南春社区、港阜社区、华山村、首峰村、群峰村、华峰村、东华村、浔光村、华海村、坑边村、东山村、柳山村、运伙村、科任村。深沪镇的地标是烽火台。这个烽火台有百几年的历史,现在成了旅游观光景点。晋江深沪湾国家地质公园为中国国家地质公园之一。","text2":"深沪镇镇政府具体位于哪个村?","label":1} {"text1":"威赛克斯白肩猪是家猪的一个品种,起源于英国的西郡(威赛克斯),特别是威尔特郡以及汉普郡的新福里斯特地区。威赛克斯白肩猪为黑色,身体的前部有一条白带。白带从一只前脚穿过肩部并延伸到另一只脚,形状类似马鞍。威赛克斯白肩猪传统上为树林放牧。这个品种是英国少数没有受到远东地区来的“那不勒斯”猪杂交影响的品种之一。威赛克斯白肩猪品种协会于1918年成立于英国。20世纪中叶以来,养猪业开始向集约化发展并影响到威赛克斯白肩猪的发展。同时,另一个相同颜色但稍有区别的品种Essex也面临着同样的问题。1967年,这两个品种为了防止灭绝而进行融合。此外产生了一个杂交品种(“威赛克斯白肩猪”也经常被用于称呼英国白肩猪)。很少的Essex猪在英国以纯种的形式保留了下来并有人试图恢复这个品种。但威赛克斯猪现在一般认为在原产国已经灭绝。但是有一些威赛克斯白肩猪在品种融合前已经被出口到世界上的其它地方,在澳大利亚、新西兰及其它地区仍有该品种存活。2008年,澳大利亚注册的种母猪不到100头并被列为濒危动物。2006年,威赛克斯白肩猪的胚胎和精液被进口到英国并重新组群。19世纪初,汉普夏地区一些相似的猪被出口到北美,并初步形成汉普夏猪。","text2":"威赛克斯白肩猪的外形特征有哪些?","label":1} {"text1":"Joseph Saddler(约瑟 萨德勒)(1958年1月1日生于巴贝多桥镇)最为人所知的是他的艺名Grandmaster Flash(闪耀大师),他是美国嘻哈音乐的先驱,是历史上第一位DJ,同时也是第一个入选摇滚名人堂的嘻哈音乐家。他与Kool Herc(库哈克)和嘻哈教父Afrika Bambaataa(阿菲卡巴莫巴塔)并称DJ三大鼻祖。Grandmaster Flash童年时与家人从西印度群岛移民到美国曼哈顿的布朗克斯Grandmaster Flash早期曾与著名歌手Kurtis Blow(克杰斯 布洛)和Lovebug Starski(爱情虫斯塔尔斯基)合作。他在1970年末亦成立了自己的团体,开始有Cowboy(牛仔男孩,名叫Keith Wiggins),Melle Mel(梅勒 梅尔名叫Melvin Glover)和Kid Creole(基德 克利奥尔名叫Nathaniel Glover)三位成员,后来加入了Rahiem(拉海姆名叫Guy Todd Williams)与Scorpio(亦称纳斯先生名叫Eddie Morris)组成了Grandmaster Flash and the Furious Five(闪耀大师与狂暴五人)。Melle Mel(梅勒 梅尔)还持有一个二线服装品牌\"Sedgwick & Cedar\"。2006年11月他发表了一部儿童有声读物\"Portal In The Park\"。在2007年1月30日发表了他的首张个人专辑\"Muscles(肌肉)\"他的首个单曲与音乐录影带\"M3(The New Message)(新讯息)\",是纪念著名单曲\"The Message(讯息)\"发表25周年。","text2":"美国嘻哈音乐的先驱是哪一位?","label":1} {"text1":"深圳体育场,座落于深圳市福田区笔架山下,东邻上步北路,北接泥岗西路,南靠笋岗路,落成于1993年6月,总面积达到24892平方米,总投资1.41亿元人民币,场地包含一个主体育场及1个副场。设有12个主看台,可容纳32500名观众,是一座全飘棚式的体育场,场内拥有设施有会议室、运动员休息室、贵宾厅及新闻发布室等。此场地承办2011年夏季世界大学生运动会的足球比赛。深圳体育场是深圳足球队的主场。体育场内有一个标准的国际足球比赛场地,其草坪面积达7992平方米。另外还有一块面积达16900平方米的标准国际田径比赛场地,这块场地的面积在全中国比较,属于较大面积。深圳体育场一共有4层,每层分别设有运动员、裁判员、其他工作人员和观众的入口,这样不会造成混乱和人员交叉。比赛区内有24个呈环状分布的观众坐席区,另外还有三间贵宾包厢,每间可容纳22人。内有4间运动员更衣室和4间裁判员更衣室,这些更衣室设施齐全,配备有洗手间、沐浴间、更衣处等,另外还有许多办公用地。比赛场地内配备了一部电梯,还配备了许多看台观众疏散通道。在场馆的东西两侧还有两座天桥,分别连接上步路和泥岗路。在一楼有可供观众撤离以及消防车通过的五个疏散通道口。另外场地内还有可供三百人举行会议的会议室、比赛配套用房及设施、运动员休息室和设备操作室。深圳巴士集团股份有限公司运营之路线深圳体育场从2007年正式从事业单位专制为企业,正式成为深圳市国资委全资控股的国有企业","text2":"深圳体育场位于什么位置?","label":1} {"text1":"乙酸酐是由乙酸衍生出来的酸酐,分子式为(CHCO)O,常缩写为AcO。在有机合成中常用它作乙酰化试剂或失水剂。在室温下乙酸酐为无色液体,与空气中的水分反应,从而散发出乙酸的强烈味道。乙酸酐可由乙酸甲酯的孟山都法羰基化制得,常以铑和锂的碘化物作催化剂:该反应中,乙酸甲酯先被转化为碘甲烷和一个乙酸盐。然后碘甲烷的羰基化得到乙酰碘,和乙酸盐或乙酸反应生成产物。由于乙酸酐在水中不稳定,该反应需在无水环境中进行。相比之下,孟山都乙酸合成法虽然也涉及含铑催化剂催化羰基化碘甲烷,但起码有部分反应是在水溶液中的。1922年,随着由乙酸酐合成的醋酸纤维需求量大大增加,瓦克(Wacker Chemie)发明了一种新的制取乙酸酐的方法,即以乙烯酮和乙酸反应来制取。该反应中的乙烯酮可由乙酸高温失水获得。乙酸酐价格很便宜,在实验室中通常不需制取,直接购买使用。AcO主要用来乙酰化纤维素制取醋酸纤维,应用在胶片和其他领域中。一般醇类和胺类可被乙酰化。例如乙酸酐与乙醇的反应为:通常以碱,如吡啶作为催化剂。具有路易斯酸性的钪盐也可作为催化剂。阿司匹林(乙酰水杨酸),是用乙酸酐对水杨酸乙酰化来制取的。乙酸酐与吗啡反应可用于合成海洛因,因此它被美国司法部缉毒署列为第二类易制毒化学品。乙酸酐在水中的溶解度大约是2.6%(以质量)。但是和其他酸酐一样,其水溶液不稳定,容易发生水解反应生成乙酸:乙酸酐具有刺激性而且可燃。由于它可与水反应,灭火时常选用二氧化碳或醇泡沫。乙酸酐蒸汽对人有害。","text2":"乙酸酐的分子式是什么?","label":1} {"text1":"六原车站()是一由东日本旅客铁道(JR东日本)与日本货物铁道(JR货物)所共用的铁路车站,位于日本岩手县胆泽郡金崎町()大字三尻()字丹藏堰。六原是JR东日本东北本线沿线的一个小车站,属于JR东日本盛冈支社的管辖范围内,是个由北上车站管理,委托由JR东日本子公司Jaster()代为经营的业务委托车站。除了客运业务外,JR货物在六原车站站区内也设货柜装卸场,用以装卸由专用线或车携货运()运来的货柜。其中专用线的部分,六原车站内有一条通往北上高科技制纸(,原三菱制纸北上工厂)、长约0.8公里的专用线,主要是用以将纸制品运往外地,或自酒田港车站将液态氯运往造纸厂。至于在六原车站停靠的货物列车,则包括有自陆前山王发车、驶往八户货运车站的临时专用货物列车,会在六原停车进行车厢的解连。侧式月台1面1线与岛式月台1面2线,合计2面3线的地面车站。※2号月台截至2014年1月为止没有定期旅客列车使用。","text2":"在六原车站停靠的货物列车有哪些?","label":1} {"text1":"本尼迪克特·理查德·奥格曼·安德森(,),生于中国昆明,是美国著名的学者,专门研究民族主义和国际关系。为康乃尔大学荣休教授。著有《想像的共同体》(\"Imagined Communities\"),提出民族是想像的共同体。安德森的父亲名叫詹姆斯·卡鲁·奥格曼·安德森(James Carew O'Gorman Anderson)是盎格鲁爱尔兰人,其母为英格兰人。其弟为历史学家佩里·安德森。其祖母法兰西斯·安德森(Frances Anderson),来自一个长期支持爱尔兰独立运动的家庭,外曾祖父为梅杰·普赛尔·奥格曼(Major Purcell O'Gorman)。其祖父来自于爱尔兰沃特福德郡,其家族为苏格兰安德森家族的后裔,在1700年代移居到爱尔兰。其祖父是大英帝国武官,奉派至英国海峡殖民地槟榔屿。其父在槟榔屿出生,是中国海关总税务司中的官员。安德森在中国昆明出生,在5岁时,其父亲带全家移民美国加州,安德森在此长大。9岁时,其父去世,安德森全家移居爱尔兰。安德森中学进入英国伊顿公学,大学就读剑桥大学,在此取得学士。研究所就读于美国康乃尔大学,在此取得博士学位。安德森于2015年12月13日在印尼巴图过世于睡眠中。","text2":"安德森是何时去世的?","label":1} {"text1":"杨宁(),字彦谧,直隶歙县人。明朝政治人物。出自浙江余姚泗门双桥杨氏,为十一世。父杨升,考中洪武二十九年(1396年)乡试,官徽州府学教授,故入籍徽州。永乐丁酉(1417年)举人,宣德五年(1430年)庚戌进士,授刑部主事,善交权贵。正统初年,随尚书魏源宣抚大同等地。时王振把持朝政,正统四年(1439年)随都督吴亮征讨麓川(今云南省瑞丽市)土司思任发。正统六年(1441年)再随兵部尚书王骥征讨思任发,以功升刑部右侍郎。历任江西巡抚、礼部尚书。后以足疾改南京刑部尚书,正统七年为御史庄升所劾,明英宗复辟,被迫致仕。一年后去世。卒祀乡贤祠。同族同辈人有杨抚,正德十六年(1521年)进士,湖广提学副使。弟杨宜,正统十三年(1448)进士,广东按察司副使。后人有杨光先,明末清初时代,曾任清钦天监监正。","text2":"杨宁在哪一年中举人?","label":1} {"text1":"尚温(;)是琉球国第二尚氏王朝第十五代国王。1795年至1802年在位。童名思五郎金。他是尚穆王世子尚哲的次子。尚穆王死后由他继位。尚温王对儒学在琉球民间的普及做出了贡献。1798年,尚温王接受了国师蔡世昌(高岛亲方)的建议,下令在王宫以北开办国学(现首里高等学校),又开办了三个乡学(平等学校);同时废止了久米村对四个官生名额的垄断行为,规定官生中的二名应为首里人。这引起了不少久米村人的不满,引起了官生骚动。尚温王逮捕了首谋者金文和(松永亲云上)等人,镇压了这次骚乱。1800年(嘉庆五年),清嘉庆帝派修撰赵文楷、编修李鼎元为正副册封使,前往琉球国,册封尚温为王,并赐「海表恭藩」的御笔匾额。1801年,在乡民的请求下,尚温王又下令开办了一个乡学。1802年,琉球第一次向中国派遣首里城出身的官生。|-style=\"text-align: center; background: #FFE4E1;\"","text2":"尚温的童名是什么?","label":1} {"text1":"保罗·克洛岱尔(,),全名保罗-路易-夏尔-马里·克洛岱尔(Paul-Louis-Charles-Marie Claudel),法国诗人、剧作家、散文家、外交官,长姊卡米耶·克洛岱尔是雕塑家。1895-1909年到中国(清朝)担任领事保罗·克洛岱尔于1868年8月6日生于法国皮卡第大区埃纳省的费尔河畔维勒讷沃。父亲Louis-Prosper是政府公务员,处理抵押和银行交易事务;母亲Louise Cerveaux来自香槟行省,家庭成员都是农夫或牧师。1881年,他一家移居巴黎,他在当地的路易大帝高中读高中,后于巴黎政治学院读大学。他本来没有信仰,但于1888年(18岁)圣诞节在巴黎圣母院听到天主教的晚祷后,就信了天主教。保罗·克洛岱尔在1893-1936年期间当上法国的外交人员。他在中国(清朝)逗留14年,是逗留时间最长的国家。1895年末,保罗·克洛岱尔在上海写信给法国诗人马拉美,信中这样说:之后,他在《巴黎杂志》写了很多散文诗,讲述中国风情。保罗·克洛岱尔在巴西里约热内卢时正值第一次世界大战,他负责监督由南美洲运往法国的食物供应。当时他的秘书达吕斯·米约后来成为著名作曲家。保罗·克洛岱尔受法国象征主义诗人阿蒂尔·兰波的影响很深远。","text2":"他在中国逗留了多久?","label":1} {"text1":"沈昌珉(、,),艺名最强昌珉(,),是韩国组合东方神起的成员中年纪最轻的一位。中学时,在学校打羽毛球,被SM娱乐的星探发现,并进入韩国的SM娱乐。曾取得第六届SMBest大奖及歌唱奖冠军。此外,昌珉为能更快以东方神起的名义活动,2015年7月初通过了义务警察合格,已于2017年8月18日退伍。(因收视率问题而下车)(因昌珉当时以东方神起成员身份在忙韩文七辑而忍痛下车)(但由于昌珉在拍摄时与日本巡回演唱会同时进行,所以未能参与这次印度之采访之旅,下一次旅行地点还没定。)2004年年末各大颁奖典礼获奖部分2005年年末各大颁奖典礼获奖部分2006年年末各大颁奖典礼获奖部分2007年各大颁奖典礼获奖部分还有第一次在日本Oricon榜第1位2008年各大颁奖典礼获奖部分2009年各大颁奖典礼获奖部分置,并刷新了东方神起自己保持的外国艺人获公信榜周冠军次数最多的记录)2010年各大颁奖典礼获奖部分2011年各大颁奖典礼获奖部分2013年各大颁奖典礼获奖部分","text2":"昌珉是韩国哪个组合的成员?","label":1} {"text1":"舵(英语:Rudder),为航行设备上用于改变或保持航行方向的一种装置。最早用于船舶,是控制船舶航向的设备。通俗的来说,舵就是“和轮船外壳相连接,控制其运动方向的装置”,各类船桨也具有相类似的功能。古代的船只往往靠人力来推动,有些把船桨用双耳圆环固定在船舷的两侧,有些则安装在船尾,其中一部分船桨甚至设计为两者皆可通用。在地中海地区的文献中,两侧的船桨往往被称为“四分之一船桨”,其得名于此类船桨安装的位置在船身后方的四分之一处。而位于船尾的桨与现代轮船的转向原理相类似,一般为船尾中心线的位置。其结构设计非常多样化,古代欧洲往往通过铁制的螺栓和铰链将其和船身连接,阿拉伯人的设计是在船尾安装一个圆环,把船舵插在其中。中国古代的舵和阿拉伯比较类似,但是是通过滑轮驱动的。古埃及人在西元前3100年美尼斯时期就已发明。 但那比较像控制方向的船尾船桨而非现代的船舵,不需用人力一直控制的转向舵,最早出现在西元一世纪的中国,至于欧洲可能到12世纪才有这类船舵。1955年在广州近郊出土的东汉陶制船模,船尾就设有一支舵,这是中国最早的舵。从出土的汉朝文物和文献可以说明,舵在汉朝已经有。1978年在天津市静海县宋朝古船上发现了平衡舵。1999年在安徽省淮北市唐朝木船上发现拖舵。","text2":"古代的船是怎样被推动的?","label":1} {"text1":"汪嵩(),中国足球运动员,司职前锋,现效力于江苏苏宁。在2007年中国足球甲级联赛中,汪嵩为成都谢菲联攻入17粒入球,在射手榜上排列第二位,帮助球队冲超成功。凭借自己出色的表现,汪嵩入选了中国国家足球队,这也使得他成为成都谢菲联队历史上第一名入选国家队的队员。2008年1月12日,中国国家队在迪拜与汉堡足球俱乐部进行比赛时,汪嵩登场亮相24分钟,这是他第一次为国家队出场。1月20日中国国家队与黎巴嫩进行比赛,汪嵩登场20分钟。不过,在这之后的世界杯预选赛中他却未能获得出场机会。2010年2月9日,汪嵩以接近300万元的身价加盟杭州绿城。2014年12月10日,杭州绿城通过官方微博宣布不再与汪嵩续约。2014年12月15日,广州富力宣布汪嵩自由转会加盟球队。2017年2月28日,江苏苏宁宣布汪嵩转会加盟球队,签约三年。","text2":"中国国家队与黎巴嫩进行比赛,汪嵩登场多长时间?","label":1} {"text1":"树皮是木质植物,例如树的茎和根最外面的部分。狭义的树皮包括三层:木栓、木栓形成层和栓内层,以及外部的各种死组织,广义的树皮还包括韧皮部。有的植物的树皮中含有各种生物碱、单宁、染料和香料等,可以提炼各种药材、毒品、毒药、树脂等,也可以用某些种类的树皮做软木、绳索、织布、造树皮船、绘制树皮画等,或直接用树皮作装饰。由外向内,树皮可分为外表皮、由木栓、木栓形成层和栓内层组成的周皮以及内里的韧皮部。外表皮是树木最外部的死组织,由角质化的细胞组成。周皮是韧皮部和外表皮之间的部分,包括木栓、木栓形成层和栓内层的总称,周皮形成后,表皮即脱落。木栓是树皮外层的主要成分,能隔绝水分和气体通过,对树有保护作用。木栓形成层通常只有一层或两层细胞,是分生生长木栓的组织,向外生长成木栓层,向内形成栓内层,不过在根部的木栓形成层是由中柱鞘转变的。栓内层是木栓形成层向内部分化出的一层细胞。韧皮部在木质部的树干和周皮之间,是树皮内部输送营养的部分。随着树皮逐渐生长加厚,外层组织逐渐死亡,狭义的树皮只包括木栓和外部的死组织。树皮的各个部分都有不同的用途,木栓的质地轻、富有弹性、不透水,木栓发达的树种如栓皮栎的树皮,可以用来制作瓶塞、救生圈、隔音板等,在北美洲和中国东北用桦树皮制作小舟和器物。有些树皮甚至可以食用,最有商业价值的树种是金鸡纳树和肉桂树,可以提炼药物奎宁和香料, 阿司匹林是从柳树皮中提炼的;夏栎(\"Quercus robur\")树皮是揉革用的单宁酸的主要原料;在园艺中,经常用树皮碎屑培育兰花等不能在土壤中生长的附生植物花卉。许多种类的昆虫、真菌和苔藓,附生在树皮上 。不同树种的树皮形状、色泽、瘢痕及脱落情况都不相同,树皮可以作为鉴定树木种类、年龄的重要依据。树木在树皮受伤后,可以自己生长出多余的木栓来修复。","text2":"由外向内,树皮可分为什么?","label":1} {"text1":"红翅旋壁雀(学名:),俗名爬树鸟、石花儿、爬岩树,是鸣禽亚目的一种小型鸟类。旧属䴓科,今独立出来成为旋壁雀科,之下只有旋壁雀属一个属。分布于欧洲、西南亚地区以及中国大陆的新疆、西藏、青海、甘肃、宁夏、内蒙古、四川、中国东北、河北、北京、河南、陕西、湖北、江西、安徽、江苏、云南、福建?、广东等地,多生活于非树栖高山型、栖息在悬崖和陡坡壁上以及或栖于亚热带常绿阔叶林和针阔混交林带中的山坡壁上。该物种的模式产地在欧洲南部。有关本物种的分类,鸟类学家有些少争议:在鸟类传统分类系统中,林奈最初把本物种归入旋木雀科(Certhiidae)。现时基本上确定本属物种是一个单系群,不过并所有人认同Clements (2007)把本属独立成科的主张。Dickinson (2003)认为本属充其量只能成为䴓科之下的一个亚科(Tichodromadinae)。本物种有两个亚种,基本上有地域分别:","text2":"红翅旋壁雀分布于什么地形?","label":1} {"text1":"安德烈·马特斯('、',)。13岁时,组了他人生第一个乐团-Viper,当时在巴西造成轰动。之后因为跟团员要走的音乐风格不同,所以就离团了。在1991年,Andre Matos创立了新古典金属乐团Angra(安格拉),1993年的首张专辑「Angels Cry」便一举拿下日本与欧洲。之后也是因为跟团员要走的音乐风格不同,所以也离开了Angra。在2001年,Andre Matos组了第三个乐团-Shaman(魔法巫医乐团),这个团的乐风明显的加入很多民俗音乐的风格,当然也是获得了许多好评。2006年,Matos决定单飞,并于2007年发行了首张个人作品-Time To Be Free。2009年将推出个人第二张作品-Mentalize。","text2":"安德烈·马特斯组建的第一个乐团叫什么名字?","label":1} {"text1":"杰里米·布罗基(Jeremy Brockie,),是一名新西兰足球运动员,现效力于新西兰足球锦标赛的豪克斯湾联队。之前效力于澳大利亚足球甲级联赛的新西兰骑士和悉尼FC。过去人们通过他那金黄色的长发髻一眼就能辨认出他来。布罗基在新西兰足球锦标赛2004-05赛季效力于坎特伯雷联。2005-06赛季他是新西兰骑士在澳大利亚甲级联赛的第一个赛季比赛中的不多见的亮点之一,9次首发,共攻入4球。他在澳大利亚甲级联赛中的首场比赛就在北港体育场()与纽卡斯尔喷气机对阵,这场比赛他就攻入2球。布罗基在另一场对阵悉尼FC的比赛中最后一刻攻入1球,为骑士队夺取1分。他的第4粒进球是在墨尔本的奥林匹克公园对阵墨尔本胜仗时获得的。2006年3月17日,新西兰骑士宣布布罗基已经决定离开本俱乐部,并加盟悉尼FC开始一份为期2年的合同。2007年3月6日因为个人原因布罗基提前解除了合同。2007年7月,杰里米签约豪克斯湾联足球俱乐部。布罗基代表新西兰队参加2008年北京奥运男子足球比赛。新西兰被编在C组,同组球队有巴西队、比利时队、中国队。在8月7日沈阳奥体中心的首场赛事,布罗基首发出场,并在53分钟打入1球,比赛最终战成1:1平。8月10日,在沈阳奥体中心,布罗基首发出场并打满全场,但最终新西兰队以0:5落败。8月13日,在上海体育场同比利时队的比赛中,布罗基因前两场比赛身负2张黄牌,无缘比赛,新西兰队最终以0:1败给比利时队。新西兰队最终积1分,C组排名垫底出局。","text2":"8月13日在上海体育场,布罗基战况如何?","label":1} {"text1":"本条目以编年体介绍香港政府各决策科(今决策局)于1973年设立至今的演变。1973年1974年1975年1976年1978年1979年1980年1981年1982年1983年1985年1988年1989年1990年1991年1993年1994年1997年1998年2000年2002年2007年2012年然而由于立法会拉布持续,特区政府未能于7月1日以前完成所有立法会程序,第四届特区政府最终以原有三司十二局上任。而五司十四局方案更因立法会换届而胎死腹中。(详情请参阅2012年香港立法会拉布攻防战、五司十四局及香港冗长辩论条目)2015年原商务局通讯及科技科更名为通讯及创意产业科,继续主管通讯及创意产业政策。2018年","text2":"本条目以何方式介绍香港政府各决策科?","label":1} {"text1":"电玩巴士是中国大陆一家专注于游戏行业的综合性游戏门户网站,成立于2004年9月,先后吸收整合了国内著名的掌机之王(gbgba.com),“psps2.com”,“xboxsky.com”和“17game8.com”。现已成中国大陆著名的游戏网站之一。网站口号是“简单游戏,快乐生活!”2013年被完美世界以2.55亿人民币收购。游戏频道中包括“PSP、PSV、NDS、3DS、PC、GBA、PS3、X360、Wii、Wii U、安卓、iPhone4S、苹果、iPad、Nokia、腾讯游戏及网络游戏”等专题主页的链接。2004年,电玩巴士首先在北京、上海、广州、深圳、武汉、南京、成都等一线城市开设了数十家电玩巴士品牌授权店,给玩家提供了一个无忧购物、游戏体验、信息交流、维修服务四位一体的休闲空间。现已在全国130余家实体连锁。2009年2月25日,电玩巴士网站突然无法访问。同日,万网作出对电玩巴士无法访问作出回应:“中国万网于二月二十日接到上级主管部门的通知,要求对部分网站停止域名解析。万网在接到该通知后,对列明的网站,包括tgbus.com(电玩巴士)进行了停止域名解析操作。万网作为专业的域名注册服务机构,将及时执行政府通知,除此情况外,在域名指向的站点中不存在违法信息的前提下,万网不会对用户的域名解析和指向做任何操作。同时,万网呼吁广大用户及时检查网站内容的合法性。”2月26日,巴士开放临时域名“www.5068.com”。但其后也被封停。2009年4月27日,电玩巴士网站重新开通2009年7月22日,电玩巴士的PSP论坛上出现了一条名为“友情提醒 各位用户、各位版主 关于PSP游戏下载 Sony公告”的帖子,一名特殊的版主,来自索尼电脑娱乐(SCE)的工作人员“SCEIIP”,将与电玩巴士一起为规范国内PSP游戏市场而作出努力。另外,国内部分PSP游戏资源下载的网站相继收到了来自索尼电脑娱乐的律师函,要求相关网站停止提供侵权游戏拷贝的服务。有业内人士称这是索尼公司为行货游戏机进入大陆市场而努力的一种表现。但是也有另一种声音,认为这次事件其实是电玩巴士借助索尼来炒作自己。而此事至今已无下文。","text2":"电玩巴士首先在哪些地方开设了数十家电玩巴士品牌授权店?","label":1} {"text1":"沙希德·阿亚图拉·达斯特盖卜国际机场()(波斯语:')是伊朗伊斯兰共和国法尔斯省省会设拉子的国际机场,本名设拉子国际机场(波斯语:'),后为了纪念在该城出生并死于爆炸(1981年)的宗教领袖阿亚图拉而改现名(“沙希德”是伊斯兰教中的殉教者)。沙希德·阿亚图拉·达斯特盖卜国际机场于2005年完成了修缮和扩建,已经成为仅次于首都德黑兰伊玛目霍梅尼国际机场的全国第二大机场。沙希德·阿亚图拉·达斯特盖卜国际机场由四个大楼组成,其中两个供国际航班的乘客使用,另外两个供国内航班的乘客使用。另外,亦有一座供国际航班乘客使用的大楼正在兴建,并预计于2020年建成,建成后,现原有的客运大楼会改为朝觐 (伊斯兰教)航班之用。","text2":"沙希德·阿亚图拉·达斯特盖卜国际机场的四个大楼分别作何使用?","label":1} {"text1":"铃木重秀(、1546年-1586年)是日本战国时代的铁炮佣兵习团杂贺众的头领,人称杂贺孙巿、孙一。为铃木佐大夫的儿子。石山合战时,杂贺众与本愿寺派均有多数人员参战,导致织田信长陷入苦战。重秀与下间赖廉并称为「大坂之左右大将」。在「本愿寺文书」有「铃木孙一于石山合战非常活跃」的文字记录。之后孙一为协助毛利军将军粮补给送进本愿寺,而前往播磨国,并于织田信长讨伐杂贺城时,率领铁砲僧兵队数千人据守,导致信长久攻不下。荒木村重反叛信长并据守有冈城时,也加入协防。后来其父死于藤堂高虎的计谋(一说),石山合战讲和交涉的阶段,「三下间」(赖廉、仲之、赖龙)的起请文中,也出现「杂贺一向宗」四宿老与「杂贺众」七人(包括重秀)等人连署的文字记录。在织田与本愿寺双方的文书中,对于重秀的军事才能都有很高的评价,并受到显如重用。后来本愿寺显如与信长讲和退出石山后,重秀投降信长。天正9年(1581年),信长与杂贺庄的土桥守重渐渐产生冲突,隔年天正10年(1582年)1月23日重秀派刺客将守重暗杀,同年2月8日取得完全胜利。天正12年(1584年)小牧长久手之战时,根来众・杂贺众等纪州势力多数跟随织田信雄、德川家康,但重秀为羽柴秀吉方,担任铁砲首领,领兵200。天正13年(1585年)跟随秀吉进攻纪州,3月25日担任使者前往太田城劝降,战后将儿子送至秀吉处当人质。此后史料不见重秀的踪迹,据考证可能死于大坂,儿子后来也担任铁砲首领,名为「铃木孙一郎」。","text2":"重秀为什么要投降信长?","label":1} {"text1":"花斑拟鳞鲀,又称圆斑拟鳞鲀、花斑皮剥鲀,俗名小丑砲弹、花斑拟板机鲀,是辐鳍鱼纲鲀形目鳞鲀亚目单棘鲀科的其中一种。本鱼分布于非洲东海岸到太平洋中部波利尼西亚、北至日本南部以及西沙群岛海域和台湾海峡等,属于暖水性鱼类。水深10至60公尺。本鱼体侧扁,呈卵形,鱼体呈灰褐色,吻黄色,齿白色,具缺刻,自眼后到第二背鳍之前的身体上半部有一黄色鞍状斑,成鱼黄斑有散布许多褐斑,幼鱼则否。成鱼身体下半部有许多大白斑,且尾柄上有条黄色宽横带。幼鱼则全身皆布满大白斑,尾鳍无色,背鳍两个,第一背鳍黑色,第二背鳍、胸鳍及臀鳍呈白色,第一背鳍第一棘粗大,尾鳍圆呈黄色,基部及鳍缘则为黑色,背鳍及臀鳍软条截平。全身披骨质鳞片。背鳍硬棘3枚;背鳍软条25至27枚;臀鳍软条21至27枚;脊椎骨18枚。体长可达30公分。本鱼喜栖息在珊瑚礁外缘峭壁处,幼鱼则在超过20公尺的深峭壁洞穴或岩脊附近活动。主要以海胆、甲壳类及软体动物为食。肉有毒,非食用鱼。由于色彩鲜艳,为具高价值的观赏鱼。","text2":"花斑拟鳞鲀属于什么温度性质的鱼类?","label":1} {"text1":"芭芭拉·利斯科夫(,),本名Barbara Jane Huberman。美国计算机科学家,2008年图灵奖得主,2004年约翰·冯诺依曼奖得主。现任麻省理工学院电子电气与计算机科学系教授。1961年在加州大学伯克利分校获得数学学士学位。1968年在斯坦福大学获得博士学位,她是美国第一个计算机科学女博士。导师为1971年图灵奖得主约翰·麦卡锡教授,论文题目是程序。利斯科夫领导了许多重要的项目,包括小型低成本交互式的分时操作系统Venus,第一个支持数据抽象的面向对象编程语言的CLU的设计与实现(1970年代),第一个支持分布式程序实现的高级语言(1980年代),面向对象数据库系统Thor,还有最近的Byzantine分布式容错系统。其中,CLU语言对现代主流语言如C++\/Java\/Python\/Ruby\/C#都有比较深远的影响。而她从这些实际项目中提炼出来的数据抽象思想,已经成为软件工程中最重要的精髓之一。她另外一个在程序设计中有广泛应用的成就,是与周以真(Jeannette Wing)合作发展出子类型定义,并且一起提出的Liskov代换原则,这是面向对象程序设计中最重要的几大原则之一。利斯科夫是美国工程院院士,美国艺术与科学院院士,美国计算机协会(ACM)会士。2002年,她被公认是在麻省理工学院顶级女性教师之一,在美国科学领域中的排名前50位大学教学人员之中。\"Program Development in Java\",Addison-Wesley, 2001","text2":"芭芭拉·利斯科夫是哪一年的获图灵奖得主?","label":1} {"text1":"库氏天竺鲷(学名:),俗名大目侧仔,为辐鳍鱼纲鲈形目鲈亚目天竺鲷科的其中一种。本鱼分布于印度洋-西太平洋区,包括东非、南非、红海、台湾、香港、日本、韩国、越南、印尼、菲律宾、巴布亚纽几内亚、澳洲、新喀里多尼亚等海域。该物种的模式产地在澳大利亚昆士兰。水深0-15公尺。本鱼体延长而侧扁,口大而略下位。体侧约有5条暗褐色宽纵带,第三条纵带约位于中央,延伸至尾柄末端具一圆形暗斑,各鳍皆透明无色,背鳍硬棘7枚;背期软条9枚;臀鳍硬棘2枚;臀鳍软条8枚,体长可达10公分。本鱼白天躲于石块下的礁洞中,夜间则于附近觅食底栖性甲壳类动物,属肉食性,于黄昏时交配,雄鱼具有口孵习性,卵约7日化成仔鱼,由雄鱼吐出,具短暂的仔鱼飘浮期。可食用,通常以下杂鱼处理。","text2":"它的体长可达到多少?","label":1} {"text1":"保荐代表人是中国股票发行市场的负有保荐责任的个人。在中国首次公开发行股票、上市公司发行新股、可转换公司债券及证监会认定的其他情形,均需要2个保荐代表人在项目文件上签字,才有资格向证监会报送发行申报文件。保荐制度是证监会从香港证券市场借鉴而来的制度,旨在细化和具体证券发行中的中介责任,监管机构减少对具体项目的实质性判断,是使证券发行走向注册制的过程之一。中国证监会对保荐机构和保荐代表人进行监管。中国证券业协会负责保荐代表人资格考试管理。由于监管机构对资格考试采取严格限制通过率的措施,且需在境内证券发行项目中担任过项目协办人方能注册。证券公司必须拥有4名以上保荐代表人方可从事保荐业务,使得保荐代表人成为中国证券市场的稀缺人群,刚推出保荐代表人制度的前几年其年薪均在人民币100万以上,远高于证券公司一般员工。但随着监管机构对保荐代表人考试由每年一次考试变为两次考试,且符合特定条件的保荐代表人可以“双签”规定的出台,保荐代表人的稀缺性已经越来越小,其平均薪酬也有下降的趋势。","text2":"监管机构对于保荐代表人考试有何变化?","label":1} {"text1":"卡路士·艾坚哈顿·艾华斯(,)是一名千里达足球员,担任右翼,现效力英冠球队米禾尔。卡路士·艾华斯出道自千里达球会Defence Force。他在2000年以25万镑加盟-{zh-hans:雷克瑟姆; zh-hk:域斯咸;}-。艾华斯在域斯咸度过五年,上阵165次。在2005年,他免费加盟英冠球会卢顿。他成了球迷的宠儿。在2007年1月2日日,艾华斯以150万镑加盟新特兰,签约三年半。他在新特兰的表现抢镜,接连在对锡菲联、高云地利及伯明翰城取得入球。在对修咸顿的比赛更射入致胜一球。在对般尼的比赛入球,更把新特兰带返英超。2008\/09年球季由于马白兰基加盟影响艾华斯的出场机会,于10月2日被外借到狼队3个月。11月20日仅为狼队出赛6场后被提早召回。2009年9月1日艾华斯转投英冠球会叶士域治,转会费135万英镑。2012年3月30日艾华斯与叶士域治续约留效多一季。卡路士·艾华斯曾代表千里达出战2006年德国世界杯。在对瑞典,他的表现令他成为全场最佳球员。2011年10月10日艾华斯在法庭承认于停牌期间驾驶,事发于9月26日,前一天他的妻子因怀孕并发症而入院,艾华斯送女儿上学,但未能截获的士而决定亲自驾驶。其妻于10月8日早产诞下只有两磅重的双胞胎。10月13日艾华斯被判入狱12星期,但缓刑12个月,同时罚款2,500英镑和需要进行200小时社会服务,他亦被所属球会处罚两星期薪金。","text2":"卡路士·艾坚哈顿·艾华斯现效力于哪个球队?","label":1} {"text1":"斑林狸(\"Prionodon pardicolor\"),又名斑灵狸或东方蓑猫,是生活在喜玛拉雅山中部及东部的麝猫。牠们身体短小,呈浅色,头部尖长,四肢细小。由于牠们会咬著猎物的腹部来拖行猎物,故有时被误会为蟒蛇或其他大型的有毒蛇。牠们主要吃昆虫、鸟类及细小的哺乳动物。分布于锡金、不丹、印度、尼泊尔以及中国大陆的贵州、广东、云南、四川、广西(西南部)等地,多见于海拔2000米以下的阔叶林林缘灌丛以及亚热带稀树灌丛或高草丛附近。该物种的模式产地在尼泊尔。斑林狸身体细长,四肢短小,尾巴很长。体表的颜色范围从深褐色到淡黄色。两个长条纹从耳朵后方一直延伸超越肩膀,和两个短条纹沿着颈部。前腿的爪和后腿的跗关节都有斑点。圆柱尾有八或九个大的黑圈,有狭窄的白色环隔开。长有五个脚指头,爪能缩回前爪的爪鞘。斑林狸重约0.45kg,长约36–38厘米(不含尾),尾则长达30–33厘米。它的高度约为13–14厘米,其胸围14.6厘米,头部长约7.6厘米。","text2":"斑林狸又名什么?","label":1} {"text1":"是一间于1993年至1999年参加日本职业足球联赛的日本球队。球队原称全日本空输足球队,并且参加了日本足球联赛。球队名称「飞翼」是取自德文 ,可解作「翼」。尽管球队从未取得过日本足球联赛或日本职业足球联赛冠军,飞翼却赢得多项的重要锦标,包括天皇杯、亚洲杯赛冠军杯和亚洲超级杯。1998年,由于有球队投资者撤走资金,球队最大的赞助商全日本空输与日产汽车商讨成立新横滨水手,把两间同样位于横滨的横滨水手和横滨飞翼合并,飞翼的球员亦成为了新水手队的球员。虽然在新水手队的名字中加入了 \"F\",新球队的名字 \"Yokohama F. Marinos\" 的 \"F\" 的代表飞翼。可是在横滨飞翼和横滨水手合并后的第二年,一些横滨飞翼的球迷不认为自己应该支持合并后的横滨水手,因为实际上飞翼已经解散,和横滨水手是飞翼的同市死敌。所以,他们向公众募捐,与国际管理集团结盟,和集合横滨飞翼的球迷,成立了横滨Fulie体育会(Yokohama Fulie Sports Club)。然后他们仿效巴塞罗拿采用的社会学模式,在Fulie体育会下成立了FC横滨,并是日本首间由球迷拥有和经营的球会。1999年1月1日,飞翼在最后一场比赛中击败清水心跳,夺得天皇杯冠军。天皇杯 (2): 1993, 1998亚洲杯赛冠军杯 (1): 1994-95亚洲超级杯 (1): 1994-95","text2":"横滨飞翼为什么这么出名?","label":1} {"text1":"纹腹叉鼻鲀(学名:),俗名白点河鲀、乌规、花规、绵规,为辐鳍鱼纲鲀形目四齿鲀科的一种。本鱼分布于印度太平洋区,包括东非、南非、马达加斯加、模里西斯、红海、塞席尔群岛、留尼旺、马尔地夫、印度、斯里兰卡、泰国、柬埔寨、中国、韩国、日本、台湾、菲律宾、越南、泰国、马来西亚、印尼、新几内亚、澳洲、密克罗尼西亚、库克群岛、马里亚纳群岛、关岛、夏威夷群岛、帛琉、新喀里多尼亚、萨摩亚群岛、东加、万那杜、加拉巴哥群岛、法属波里尼西亚、中美洲、厄瓜多等海域。该物种的模式产地在印度。水深3至50公尺。本鱼体呈圆筒型,被覆由鳞片特化成的细棘;口小,鱼体背部深灰色或黑褐色,散布许多白色圆点;腹部乳白,具数条深褐色细纵带,尾鳍圆形,背鳍软条10至11枚;臀鳍软条10至11枚,体长可达50公分。本鱼属于广盐性的鱼类,幼鱼偏好在河口区活动,游动缓慢,受惊吓时会吸入大量的水和空气,将身体涨大成圆球状,以吓退掠食者。晚上就地而眠,很少躲入洞中。属肉食性,以小型底栖动物为食。本鱼禁止食用,其卵巢和肝脏有河豚毒素,皮肤和肠也有毒。多做为观赏鱼。","text2":"请问纹腹叉鼻鲀受惊吓时会吸入大量的什么东西,将身体涨大成圆球状,以吓退掠食者?","label":1} {"text1":"C-Mag弹鼓(Beta C-Mag、Century magazine,意为「世纪弹鼓」,简称C-Mag)是由吉姆·苏利文(Jim Sullivan)设计的100发双室型弹鼓,主要提供给5.56×45毫米北约、7.62×51毫米北约和9×19毫米口径的枪械使用。C-Mag大部份部件以塑料制造,双室左右对称排列的设计,中间转为直排弹匣适配器作连接,依靠旋转弹簧及假弹链作推力将子弹推进至供弹口,比一般的直排弹匣具有更多的装弹量。早期型C-Mag采用全哑黑色外壳,而后期型改用了透明的后部外壳以便于射手快速观看弹药剩余量,亦有其他颜色的外壳提供。以M16突击步枪作举例,一个对应M16、装有100发5.56x45 NATO子弹的C-Mag约重2.1公斤,100发子弹就需要四个30发弹匣作供弹容器,虽然四个共装100发弹匣仍比装有100发的C-Mag为轻,但C-Mag可连续发射而30发弹匣却需更换四次。由美国陆军士兵在阿富汗的试验中发现:C-Mag在的模拟战之中是不可靠的,而且经常有供弹的问题。这问题在充满尘埃和沙土的环境中更加严重。C-Mag还有弹匣释放压力的问题,因为正如上述所指,四个共装100发弹匣仍比装有100发的C-Mag为轻。美国陆军TACOM发出了「地面防范文件」(GPM-02-017),并警告在行动期间其使用。这个GPM在伊拉克自由行动前已经启动。至于接受测试的C-Mag是不是在美国军队内部广泛使用,并没有一个回答。5.56×45毫米北约9×19毫米7.62×51毫米北约目前各国军方对C-Mag的购买意欲不太大,一般的直排弹匣生产便宜、通用性高,运输及配套更简易,C-Mag只装备于一些特种部队和个别士兵自行购入。另外亦有一些采用弹匣供弹的轻机枪以C-Mag作预设供弹具,如德国联邦国防军装备的MG36。1994年9月13日,美国联邦暴力犯罪控制和执法法案通过,只允许平民使用弹匣供弹而且弹数必须等于或少于10发。在这法案通过以前,民间购买和拥有的C-Mag是合法的,但由于数量有限,在当时的C-Mag比较稀有和昂贵。这项禁令在2004年9月13日开始过期,Beta公司已经恢复了民间市场的C-Mag的生产。但是在一些国家、地区之中,已经制定了其对「大容量」弹匣的法律。","text2":"100发子弹就需要几个30发弹匣作供弹容器?","label":1} {"text1":"黄花镇位于湖南省长沙县中南部,为长沙县第三大乡镇。地缘上,黄花镇北与安沙镇、果园镇接壤,西与星沙镇、榔梨镇为邻,南连干杉乡,东与浏阳市永安镇交界,全镇总面积169平方公里,总人口74,219人(2000年人口普查);辖20个村、3个社区;政府驻黄花路社区(黄花市)。今黄花镇在1995年长沙地区撤区并乡镇之前属黄花区黄花镇、回龙乡和谷塘乡地域,1995年,三乡镇合并组建黄花镇。2004年村级区划调整,调整之前为44个村、3个社区,分别为黄花村、黄垅村、曹坪村、合心村、排头村、观山村、龙井村、长界村、南田村、新湘村、茶塘村、黄峰村、雷鸣村、石丰村、长湖村、蛟塘村、联盟村、罗塘村、大路村、胜利村、木马村、东培村、高岸村、建设村、火炬村、凤凰村、谷塘村、金甲村、岐山村、丰收村、东风村、华兴村、回龙村、龙家滩村、大湖村、兴湖村、崩坎村、银龙村、望江村、新塘村、荷庄村、石岭村、大兴村和梁坪村,大黄花路、黄谷路、机场口3个社区;区划调整后为20个村、3个社区。位于长沙县的中部,离星沙只有十多公里比较发达,各村通了两车道的水泥路有三一中成公司在此,生产泵送产品的配件,年产值5亿左右。在此工作很多时间,所以知道","text2":"全镇面积多大?","label":1} {"text1":"日本工业大学(日文:日本工业大学, 平假名:にっぽんこうぎょうだいがく,英文:Nippon Institute of Technology),简称日本工大,是本部设在日本埼玉县宫代町的私立大学。日本工业大学建校于明治40(1907)年,前身为东京工科学校。位于紧邻东京都的琦玉县,至东京做电车只需30分钟。交通便利。在接受中国留学生方面,该校定期接受华中科技大学的国费留学生,深造本科,硕士,博士课程。并且和浙江大学和中国海洋大学也有留学生交流。该校的工业技术博物馆收藏了日本工业化以来的所有工业设备模型,见证了日本工业化的发展步伐。被日本文部科学省认定为日本国家工业技术遗产。该工业技术博物馆实行免费开放制度。该校实行的\"工房\"教育被日本文部科学省认定为具有特色的大学教育计划而给与特别经费支援。该校特点是纯理工系大学,并且有从初中,高中,短期大学,专门学校,大学,大学研究生院,社会人研究生院这一系列完备的附属学校体系。毕业校友超过30000人,现在活跃在理工学科的各个领域。2007年,日本工业大学建校100周年。日本工业大学(博士前期课程(修士) 博士后期课程)日本工业大学大学院的留学生一年的学费总共65万日元左右,在理工科大学中属于中下水平,因此日本工业大学的大学院性价比显得尤为突出。另外大学院提供TA和RA的研究机会,并提供各种奖学金机会。","text2":"日本工业大学的特点有哪些?","label":1} {"text1":"金泛(,),韩国男演员,本名金尚泛()。2006年出演MBC《不可阻挡的High Kick》出道。2009年参演《花样男子》而走红。身高181cm,体重66kg的标准身材,加上阳光的笑容和青春的外形,有微笑王子之称。金泛也曾经到澳大利亚留学了一段时间,但是最后还是因为外国的环境差异而回到韩国。金泛在17岁的时候就出演了MBC日日剧《不可阻挡的High Kick》,在面临合约纠纷时还是轻松的继续演戏,以他的资历在学生时代当然也很受欢迎。金泛也曾经到澳大利亚留学了一段时间,但是最后还是因为外国的环境差异而回到韩国。金泛19岁就已经有了六段恋爱,但并未因此而放弃学业,根据金泛在「知己知彼」栏目中透露,高中时期考试最低排过班级第8名。2013年10月,与文瑾莹因拍摄《火之女神井儿》而发展为恋人,交往7个月后和平分手。","text2":"金泛于哪一年走红?","label":1} {"text1":"《莎华丽雅》(;)该部影由环球影业2007年9月出品,严格来说是第一部产于宝莱坞的美国影片。山杰·李拉·班莎里导演的意图是向大众推介两名新人兰比尔·卡浦尔和松楠·卡浦尔。他们分别是印度著名导演拉吉·卡浦尔的孙子和孙女。电影推出半年后,兰比尔获得印度电影观众奖——最佳新人奖。影片音乐优美,主题曲得到传媒的广泛好评,明星拉妮·穆科吉在影片中演出一支舞曲。沙莱曼·罕也出演一个仅有十句对白的配角。一个年轻人兰比尔,某晚来到某城市红灯区的RK酒吧卖唱。他先遇到一个好心的妓女,后来同一个晚上在小桥上遇到一个美丽而伤心的姑娘。他想和姑娘搭讪,姑娘却跑得远远的。兰比尔根据妓女的提示去寻住处。他找到一所房子里的老太太,好不容易说服老太太让他住下来。当他听说老太太的儿子三十七年前当兵阵亡,他给予老人一点安慰。第二天,他又在同一条小桥上遇到同一个姑娘。渐渐地他们相互信任,姑娘告诉他自己叫莎金娜,是织地毯的。有一天,兰比尔邀请莎金娜到酒吧向她表白对她的爱意,莎金娜却同时向她道出来自己伤心的原因是她去年曾经爱上茵曼,他们相约一年以后在小桥上会面,茵曼却没有出现。她要求兰比尔为她写一封信并交给茵曼。兰比尔不情愿地写下信,随即仍进了火炉,他骗姑娘说寄出了信,也去很多地方但是没有找到茵曼,姑娘渐渐也死心了,和兰比尔发展友谊。就在几天后,茵曼突然出现在小桥上……","text2":"《莎华丽雅》是哪个公司出品的?","label":1} {"text1":"陈氏天竺鲷(学名:),俗名大目侧仔,为辐鳍鱼纲鲈形目鲈亚目天竺鲷科的其中一种。本鱼分布于西北太平洋区,包括台湾及日本琉球群岛海域。该物种的模式产地在琉球群岛。水深70至100公尺。本鱼体延长而侧扁,眼大,口大略下位。体呈淡红色,各鳍为深红色; 体侧具2条暗色又狭窄的纵纹。第一背鳍的上半部黑色的; 在第二背鳍棘与胸鳍基底之间的一个小的黑色斑点。尾柄中央具有一个黑色的斑点与瞳孔一样大或略大。前鳃盖骨具微弱地锯齿状的边缘。背鳍硬棘8枚;背期软条9枚;臀鳍硬棘2枚;臀鳍软条8枚;脊椎骨24个,体长可达13公分。本鱼栖息于向海的岩礁区,白天躲藏于岩盘下,夜间出来觅食,属肉食性,以多毛类或其它底栖甲壳类为食。繁殖期时,雄鱼具有口孵习性,卵约7日化成仔鱼,由雄鱼吐出,具短暂的仔鱼飘浮期。可食用,但多做为下杂鱼处理。","text2":"陈氏天竺鲷主要分布在什么地方?","label":1} {"text1":"圣马丁驻军教堂(Garnisonkirche St. Martin)是德累斯顿的阿尔贝特城(Albertstadt)的一座驻军教堂,位于一片军营建筑群的中心。此教堂按建筑师罗索(Lossow)和费维格(Viehweger)]的设计,为当时在阿尔贝特城的驻军而建,成于1895到1900年间。这是一座双重教堂(Doppelkirche)。它包括新教和天主教两个部分,1945年以后只有天主教部分仍在使用,现在是德累斯顿新城圣方济各沙勿略牧区;新教部分因沦为森柏歌剧院(die Semperoper)的服装仓库而失去其宗教用途,亦不供参观。自从1900年10月28日教堂落成,天主教厅的礼拜者就源源不断。第二次世界大战结束前她一直被用作驻军教堂;1945年6月起,她成为圣•弗朗西斯科•塞维尔(圣方济各沙勿略)(St. Franziskus Xaverius)牧区教堂(之前该牧区的牧区教堂位于阿尔贝特广场(Albertplatz),毁于1945年的大轰炸),直至今日。近年来附近的军官学校的士兵也来参与礼拜和宗教服务。前方中央是一个圣坛(Hochaltar)(新罗马式教堂的核心结构),上面的马赛克画述说着耶稣乃救世主。圣坛前方有是诵经台(Ambo)和祭台(Zelebrationsaltar),其支撑部分则分别是象征着基督的阿尔法A和欧米伽Ω。后殿(Apsis)上方的彩色玻璃小圆窗尤为奇美,它们描绘着圣伊丽莎白公主、圣嘉禄•鲍荣茂主教、阿尔贝特国王、乔治骑士和玛蒂尔达女王。在大玻璃窗上画着圣母玛利亚像和教堂的守护者圣•马丁的割袍像。东南侧是拼嵌有创世图案的玫瑰窗(Schöpfungsfenster),下面的马赛克装饰画是大天使米迦勒。教堂后方的管风琴是耶穆里希三兄弟于1900年所造,分列于其左右玻璃窗上的是殉道者圣•莫里斯和圣•维克多。两侧回廊下面有15幅现代风格的耶稣受难图,及1幅老卢卡斯•克拉纳赫(Lucas Cranach der Ältere)的朝圣画圣母之佑的摹本。中殿(Mittelschiff)和两侧的祈祷室(Seitenkapellen)里各有美轮美奂的彩绘,它们在2005至2007年的修复工程耗资巨大。祈祷室的老吊灯(Leuchter)亦值得一观;中殿悬挂的巨型吊灯则为2006年新制。","text2":"圣马丁驻军教堂成立于什么时间?","label":1} {"text1":"这个美国政党列表包含了以前和目前存在于美国政治的政党。这些政党在最近的总统竞选中提名了候选人,但在2004年的竞选中没有提名,他们也没有在足够多的州将候选人名单列入选票从而赢得2008年的总统竞选。美国第一党创立于2002年,由一群离开改革党的帕特·布坎南的支持者组成。这个党反对堕胎,反对枪支管制。旨在尽快结束种族配额和非法移民。布坎南从来没有宣称与这个党有联系,虽然这个党的首页上出售他的书。这个党吸引了一些人,如前任波士顿市长和驻梵蒂冈大使雷蒙德·弗林和前任约翰·伯奇协会(美国一极右组织)总裁约翰·麦克马纳斯。这些政党有些在以前的总统竞选中提名了候选人,但因为不同的原因没有在最近的总统竞选中提名候选人。其它的政党还没有提名过总统候选人。很少的一些政党在成立之初提名过总统候选人","text2":"美国第一党反对哪些社会现象或者政策?","label":1} {"text1":"说明文是按实际情形加以说明。用客观的角度来解释事物、说明事理的文章就称为说明文。方位说明:用方位词表示方位。分类说明:根据事物的性质、功能等一定标准分门别类,逐一说明。好处是能把复杂的事物条理分明地解说清楚。数据说明:用数据精确地说明对象。好处是运用准确的数据,能准确和科学地显示事物的特点,增加可信性及说服力。举例说明:举出具有代表性的例子来说明事物的本质、特征。好处是通过具体事例来说明事物的特点,把抽象、复杂的事物或事理说得具体做诠释:从侧面对事物的某一特点做解释摹状貌:具体地描述说明对象的现象、状态或变化。这方法能增强吸引力,使事物呈现在读者眼前。比较说明:作比较是将两种类别相同或不同的事物、现象加以比较来说明事物特征的说明方法。好处:突出强调说明对象的特点。打比方:以人们熟悉的事物来比喻说明的对象。这方法能把抽象的事物或事理解说得具体易明。引用说明:引用名人的名言或文学作品来说明,这方法能让读者更相信作者的文章。常见的说明顺序如下,适当地安排说明顺序,有助读者逐步掌握说明内容。时间顺序:按照事物发展的先后顺序来作说明 : 由早到晚、由春到冬、由古至今等。一般来说会先把说明对象按具体情况以时段为序划分成若干段落,逐段说明。说明时并非像流水赈般记录整件事情的始末,而是抽取最能突出说明对象特征的内容来写。空间顺序:按照空间转移的顺序来作说明:由内至外、由上至下、由前到后、由左至右、由东到西、或倒过来由外至内、由下至上等。运用空间顺序说明时,必须清楚交代观察的转移,文章的脉络才会清晰。逻辑顺序:按照事理的逻辑关系来作说明 :由因到果,由浅到深,由主到次,由一个类别到另一类别,由一般到个别等。使用这种说明顺序,不但便于说清个别事物的特质,还有利于揭示各种事物之间的互相关系。说明的层次一般分为总述、分述、总结,适当地安排说明层次,使文章条理井然,读者更容易全面掌握说明对象的特点。","text2":"什么是说明文?","label":1} {"text1":"ZiL-2906(;后期车款型号为ZiL-29061),是一款由苏联货车及重型装备生产商ZiL于1975年至1991年间生产的。其主要用途为搜索从太空返回地球的太空船和船员。苏联在1970年代频繁地进行载人太空飞行。为了更有效地搜索返回地球的太空人,ZiL受官方命令开发代号为「蓝鸟」()的救援车辆。该系列车辆包括六轮的ZiL-4906型;以及两侧配有呈螺钉形的横向长轮作为推进器的Zil-2906组成。每次执行搜救任务时,当局会出动三辆ZiL-4906,其中一辆负责搭载一部ZiL-2906。如果搜救队需通过难以穿越的地形(如泥泞、雪地、水域或冰面),搜救者会卸下ZiL-2906并以之进行搜索。由1980年起生产的车辆经过改良,故其编号加上后缀(即ZiL-29061型),以资识别。现时所有ZiL-2906及ZiL-29061型救援车皆隶属俄罗斯国防部辖下的联邦航空搜救局()。ZiL-2906长、阔、高,车身与一艘马达快艇形状相似。其车厢两侧各装配一个长、直径的螺旋装置。每个螺旋装置各由一副生产的引擎驱动,并配有三档的变速器。车上有两个容量为67.5公升的,可支持车辆行走大约。车辆可携带大约420公斤的货物。1980年代生产的ZiL-29061则配有两副由VAZ生产的引擎,以及四档的变速器。此外,ZiL-29061长、阔、高,略大于ZiL-2906。ZiL-2906及ZiL-29061皆以铝为主要用料,后者亦有部份组件以碳纤维制成。","text2":"ZiL-2906是由谁生产的?","label":1} {"text1":"宝田总站(英文:Po Tin Bus Terminus)位于香港屯门震寰路宝田邨,邻近轻铁田景站,现时有2条巴士路线和2条特别班次路线以此处为总站。由九龙巴士营运的一条路线,提供屯门宝田、良景、田景、大兴及山景往来九龙东部的巴士服务。于1993年3月15日起投入服务。2004年5月8日提升至全日服务,并由良景巴士总站迁往此处为总站。由九龙巴士营运的一条路线,屯门宝田、建生、良景、田景、新墟及屯门市中心往来佐敦及尖沙咀的特快巴士服务。于2000年8月27日起投入服务。(于上午07:35及上午08:05设有特别班次由大兴往红磡站。)由九龙巴士营运的一条路线,58M线的特别班次。提供屯门宝田、建生及屯门市中心往来荃湾及葵涌的巴士服务。平日早上于宝田开出6班(0640、0700、0720、0740、0800、0820)。","text2":"宝田总站位于什么地方?","label":1} {"text1":"佛罗伦斯圣母领报大殿(Basilica della Santissima Annunziata)是意大利佛罗伦萨的一座罗马天主教次级圣殿,圣母忠仆会的总会,位于同名的圣母领报广场东北侧。这座教堂由圣母忠仆会的7位发起人创立于1250年,一位修士开始创作《圣母领报》图,但是后来放弃了,因为他认为自己无法创作足够美丽的图画,应该是天使在他睡觉时完成。这幅画被安放在教堂里,受人尊敬,1444年曼图亚的贡扎加家族出自捐建了一个特别祭坛。最初委托米开罗佐建造,但是由于贡扎加的Ludovico三世特别钦佩莱昂·巴蒂斯塔·阿尔伯蒂,阿尔伯蒂在1469年获得委任。但是由于已经建成的基础,他的眼光受到了限制。工程完成于1481年,在阿尔伯蒂去世之后。虽然教堂在17世纪增加了巴洛克装饰,但是仍然可以看见穹顶下面、祭坛周围圆形空间的基本形状。教堂的立面是在1601年由建筑师乔瓦尼·巴蒂斯塔·卡奇尼增建,仿照了伯鲁乃列斯基的孤儿院的立面。育婴堂对面的建筑,由安东尼奥·达·罗桑加罗设计,在1520年代也赋予了伯鲁乃列斯基立面。这座教堂内部充满了从14世纪到19世纪的艺术杰作管风琴(1628)是佛罗伦萨最古老的,也是全意大利第二古老的管风琴。","text2":"《圣母领报》图被安放在哪里?","label":1} {"text1":"科克大学(直译为科克大学学院,,)是爱尔兰国立大学的一部分。大学建于1845年,原名“皇后学院”(Queen's College, Cork);1908年爱尔兰大学法案出台后,学校更名为“科克大学学院”(University College, Cork)。1997年大学法案将学校更名为'“爱尔兰国立大学科克”(National University of Ireland, Cork);1998年教育部将大学更名为“科克大学”(University College Cork - National University of Ireland, Cork)。科克大学被星期日泰晤士报选为2003-2004年度爱尔兰大学,2005-2006年和2011-2012年再度当选年度爱尔兰大学。2007年,科克大学在泰晤士高等教育世界大学排名286位,2008年226位,2009年207位。科克女王学院(Queen's College, Cork)乃根据维多利亚女王发布的\"为发展爱尔兰的高等教育\"法案建立。根据该法案,三所学院于1845年10月30日分别在贝尔法斯特(贝尔法斯特女王大学)科克和戈尔韦(爱尔兰国立大学戈尔韦)建立。科克学院1849年正式办学,当时有23位教授及181位学生,一年后便加入爱尔兰女王大学。医学楼建于1860年至1880年,医学院也迅速因其毕业生而建立了名声。爱尔兰最早的两名女医学学生于1898年医学院毕业(20年后牛津大学才开始招收女医学学生)。在随后的一个世纪中,爱尔兰大学法案(1908)成立了爱尔兰国立大学,包括都柏林、科克和戈尔韦的三所成员学院。三所学校也有大学学院的地位,例如“大学学院,科克”。1997年大学法案使大学学院成为爱尔兰国立大学的成员大学,也使成员大学成了全方位的独立大学,但是学位和文凭仍由爱尔兰国立大学颁发。科克大学是爱尔兰的研究型大学,其政府科研经费全国最高。现有艺术、凯尔特研究与社会科学学院、商学与法学院、医学与健康学院、科学、工程和食品科学学院。科克大学还是爱尔兰中国研究院的所在地。学生在中国研究院能够通过艺术和商业学习中国语言和文化。中国研究院最近荣获2008年欧洲语言奖。","text2":"科克大学原名是什么?","label":1} {"text1":"文礼阁(Man Lai Court),是位于新界沙田区大围的一个私人屋苑,位处港铁东铁线南面和城门河东面,由九广铁路公司及长江实业联合发展。整个文礼阁共有4座共分两期发展,第一期于1990年完成第1至3座;而第二期的第4座则于1993年落成。文礼阁兴建前原址为九广铁路大围职员宿舍,而原有的九广铁路职员宿舍则迁往大埔滘,名为策诚轩,同为九广铁路公司及长江实业发展。文礼阁及策诚轩均由兴业建筑师有限公司负责设计。文礼阁共有4座住宅楼宇,每座有20层,每层有8个单位,单位有736平方呎和750平方呎两款,实用率高达八成五。屋苑内有一教会基督教宣道会证恩堂,设于第4座地下1-5号舖。文礼阁初期由高衞物业管理负责保安及物业管理工作,曾2003年起由新昌管理服务取代,于2015年起由佳定物业管理有限公司取代。每当大雨之后,文礼阁对开一段城门河,也会堆积著经由雨水渠冲入河中的路边垃圾。这些垃圾堆积在河里数日后,便会传出阵阵臭味。","text2":"文礼阁位于哪里?","label":1} {"text1":"迪士尼乐园巴士R42线是香港一条来往香港迪士尼乐园及大围铁路站的巴士路线,由龙运巴士经营。本线在香港迪士尼乐园未建成前已经规划,最初编号为D42线。2005年6月22日政府公布香港迪士尼乐园公共交通安排后,才正名为R42线。不设分段收费及八达通转乘优惠。美田路、车公庙路、狮子山隧道公路、沙田正街、沙田市中心巴士总站、沙田正街、横壆街、源禾路、沙田乡事会路、大埔公路、城门隧道公路、城门隧道、象鼻山路、大河道北、大河道、杨屋道、德士古道、青荃路、青衣北岸公路、青衣西北交汇处、青屿干线、北大屿山公路、竹篙湾交汇处、竹篙湾公路、神奇道及幻想道。幻想道、神奇道、竹篙湾公路、竹篙湾交汇处、北大屿山公路、青屿干线、青衣西北交汇处、青衣北岸公路、青荃路、德士古道、杨屋道、大河道、大河道北、象鼻山路、城门隧道、城门隧道公路、大埔公路、沙田乡事会路、大埔公路、沙田市中心巴士总站、沙田正街、白鹤汀街、沙田正街、狮子山隧道公路、车公庙路及车公庙路。虽然本线是龙运巴士营运,但实际所用的是九龙巴士48X线的车辆。2012年9月30日,大部分班次改由于同日起减少假日班次的龙运巴士E42线抽调车辆行走,但回程班次若于晚上八时开出,仍会由九巴抽调车辆行走(通常为亚历山大丹尼士Enviro 500 )。虽然本路线在沙田区受到港铁的竞争较小,但由沙田区往来迪士尼的乘客一直不多,而且乘客多会由E42转乘R8,因总票价($19.9)比起本路线($22.3)为便宜,因此乘搭本路线的乘客不多。更甚的是,规划时本路线不经荃湾,在荃湾区议会要求下才改经大河道北及杨屋道,但几乎无人上落,因为在荃湾区的乘客多会乘坐港铁往来。此外,自从2009年港铁西铁线九龙南线通车后,港铁转线次数减少,乘客在南昌站可经西铁线转乘东涌线,虽然走线迂回却更为方便,引致更多沙田区乘客弃用巴士,改为用港铁往来,故由2018年起,本路线改为只于假日才行走。2014年11月29日:有人在讨论区贴文指该晚唯一的回程班次(Enviro500 8527/RK3419)严重超载,并批评龙运当晚拒绝加班的处理手法。。","text2":"迪士尼乐园巴士R42线是怎样正名为R42线的?","label":1} {"text1":"玛黑区(法语:Le Marais,意为沼泽,又译为\"玛黑区\")是法国巴黎的一个区域,横跨巴黎右岸的第三区和第四区,传统的布尔乔亚区域。19世纪末和20世纪上半叶,蔷薇路(Rue des Rosiers)周边地区成为许多来自东欧犹太人的家园。因此在法国沦陷期间,玛黑区也成为纳粹攻击的目标。1944年以后,该区一度衰落。1969年,安德烈·马尔罗使玛黑成为第一个保护区(\"secteur sauvegardé\"),该区拥有许多博物馆,美术馆和历史遗迹,大量文学家、艺术家居住在这里,玛黑区的艺术、文化氛围因此浓厚。不同于巴黎其他地方,这里原有的商店出售改业后,门面却不更换,这种保护措施导致奇怪的结果,如“面包店”销售时尚物品,或变成了豪华酒店。直至2013年,法国老佛爷百货集团公司旗下的巴诗威百货公司(LE BHV MARAIS)正式启动一项企业文化更新计划,使玛黑区又一次成为了巴黎时尚最前沿的街区,同时也成为了巴黎周末人气最旺的地方。近年来,随着国际游客到巴黎旅游的人数增加,玛黑区也开始成为文化、艺术旅游、巴黎式购物以及年轻化生活方式的震中。","text2":"玛黑区的艺术、文化氛围为什么浓厚?","label":1} {"text1":"五所川原车站()是一位于日本青森县五所川原市大町,由东日本旅客铁道(JR东日本)与津轻铁道所共用的铁路车站。虽然两家公司是共用车站设施,但为了区隔所属系统与路线的差异,津轻铁道的部分特别以津轻五所川原车站()命名。五所川原是所在地五所川原市的主车站,也是JR东日本五能线与津轻铁道线的交会点,一些等级较高的快速或临时快速列车皆有在本站停靠。2002年(平成14年)入选东北车站百选。岛式月台1面2线的地面车站。站舍与月台以跨线天桥连络。2号月台是并行的留置线,作为此站到发的临时列车留置之用。侧式月台1面1线的地面车站。与JR共用通道及跨线天桥。但是站舍与闸口不同。※临时快速「」的相邻停车站参见「」的条目。","text2":"津轻铁道的部分为什么以津轻五所川原车站命名?","label":1} {"text1":"国立基隆高级商工职业学校,简称基隆商工、基商(National Keelung Commercial & Industrial Vocational Senior High School,KLCIVS),位于基隆市七堵区长兴里,邻近台铁七堵车站、长兴地下道、明德国中及七堵地下道,基隆一所国立技术型高级中学暨综合型高级中学并附设进修学校,为全国最早参与试办综合高中学制课程的十八所高中职之一。目前综高部及高职部均设有商业与管理群、设计群、电机与电子群电机类、电机与电子群电子类四个类群,七个科系及学程,综高部另设有学术社会及自然学程。1977年台湾省政府教育厅于台北商专成立附设空中商业专科进修补习学校,并设置基隆教学辅导处于基隆商工,由校长兼任教学辅导处主任。目前科系设有专科部(二专)与学院部(二技)共五个科系二十五班,另设兼任副主任一人及干事二人与专任工友一人,面授教师聘请有讲师资格以上的教师或相关学者担任,目前共有四十九人。诚信勤毅刘韵章作曲 周道隆、冯国旭作词基隆商工篮球队是高中篮球联赛(HBL)球队,培育出许多超级篮球联赛(SBL)好手,例如「黑人」陈建州、陈孝荣、蔡文诚、李家瑞。100学年度基隆商工靠著「天龙阵」发挥拿下校史最好成绩第三名。以下是基隆商工的策略联盟学校列表:","text2":"基隆商工位于什么地方?","label":1} {"text1":"白云鄂博铁矿是位于中国内蒙古自治区中部的一座大型露天铁-稀土矿床,1927年由著名地质学家丁道衡发现。白云鄂博位于乌兰察布草原上,北距蒙古国106公里,南距包头市市区149公里,属包头市白云鄂博矿区管辖,周围由达尔罕茂明安联合旗环绕。白云鄂博是一座海拔1783米的圆形山。白云鄂博在蒙古语中是“富饶的祭祀地”的意思。“鄂博”就是敖包的另一种汉语音译。当地蒙古人称它为“白音博格德”,意思为“富饶的神山”。白云鄂博铁矿1957年2月27日建矿,年产矿石1100万吨,职工总数6700人。白云鄂博有世界上最大稀土矿藏。二十一世纪以来该矿区陆续发现有张培善石、杨主明云母、何作霖矿、氟四面体铁金云母和氟木下云母等五种新矿物。前三者先后已获国际矿物学协会的新矿物、命名和分类委员会投票通过,批准为新矿物,以中国科学家命名。","text2":"白云鄂博在蒙语中是什么意思?","label":1} {"text1":"高丽忠穆王(;),是高丽王朝第29任君主(1344年—1348年在位),讳王昕(),蒙古名字八思麻朵儿只()忠惠王长子,为德宁公主亦怜真班所生。王昕曾经长期在元大都充任元朝皇帝的宿卫。1343年,忠惠王因为无道而被绑架到元大都问罪。资政院使高龙普(高丽人)抱王昕朝见元顺帝。元顺帝问他是以自己的父亲还是母亲为榜样。王昕选择了母亲。元顺帝高兴地称赞他明事理,并立他为高丽君主,是为忠穆王。忠穆王继位后由德宁公主摄政,一改先王的弊政,将先王宠信的佞臣全部罢免。同时任命了一些贤臣,并将忠惠王建造的新宫拆毁,改建为崇文馆。对禄科田的制度的弊端进行改革,设立赈济都监救济百姓。闵渍等增修《编年纲目》,下令编纂忠烈王、忠宣王、忠肃王三代的实录。1346年又整治都监。由于元朝奇皇后的族弟奇三万非法夺占他人田地,被下狱死。元朝因此遣使前来调查,罢免了一些官员,并任命王煦为判事,下令整治都监。1347年,忠穆王向元朝朝廷请求先王的谥号,但被元廷拒绝了。于是又遣金那海前往元朝,请求改正先王之罪。不久忠穆王就逝世了,葬于明陵,谥显孝大王(),元朝赐谥忠穆。庶弟忠定王继位。","text2":"1347年,忠穆王向元朝朝廷请求先王的谥号,结果如何?","label":1} {"text1":"诺夫哥罗德-谢韦尔斯基(乌克兰语:Новгород-Сіверський;俄语:Новгород-Северский)乌克兰切尔尼戈夫州的一座古城,是诺夫哥罗德-谢韦尔斯基县的首府。诺夫哥罗德-谢韦尔斯基位于杰斯纳河河畔,距基辅约330千米,距乌俄边境约45千米。当前人口约为15000人。诺夫哥罗德-谢韦尔斯基于1044年第一次见载于史册。在古罗斯时代,该地是一个重要的公国。所有诺夫哥罗德-谢韦尔斯基王公都属于留里克王朝。著名的史诗《伊戈尔远征记》的主人公伊戈尔·斯维亚托斯拉维奇就是诺夫哥罗德-谢韦尔斯基的公爵。1239年,西征的蒙古人摧毁了诺夫哥罗德-谢韦尔斯基。后来它被向东扩张的立陶宛大公国吞并。1500年,诺夫哥罗德-谢韦尔斯基王公瓦西里·伊万诺维奇·舍米亚契奇携封地归顺莫斯科大公伊凡三世,使该城重归俄国人所有。但在留里克王朝绝嗣后的混乱时期(俄国历史上著名的空位时期),该城再度被波兰(当时与立陶宛组成王朝联合)占领。罗曼诺夫王朝建立初期,因国力低迷也没能夺回该城。直到沙皇阿列克谢·米哈伊洛维奇在位时期,俄国在俄波战争中获胜,才最终收复了诺夫哥罗德-谢韦尔斯基(同时夺取了第聂伯河左岸乌克兰)。","text2":"诺夫哥罗德-谢韦尔斯基的乌克兰语是什么?","label":1} {"text1":"曾其祥(Zeng Qixiang,),出生于广东省梅州市梅县区,中国的职业足球运动员,司职前锋,现时效力澳门甲组足球联赛球队名门世家加义。出生于广东省梅州市梅县区的曾其祥,2003年入选广东省全运队,参加2005年的第十届全运会,同年并入选广东队,参加过省港杯足球赛。2006年加盟宁波华奥,参加中国足球乙级联赛(南区)的比赛,2007年追随中国国家青年足球队教练翟飚,加盟FC四川。2008年四川大地震后,因FC四川内部情况混乱,曾其祥遂来港加盟香港甲组足球联赛地区球队天水围飞马。2009年1月,以借将身份加盟东方,同年7月转投中甲球队广东日之泉。2010年,加盟澳门甲组足球联赛劲旅名门世家加义。2008年10月8日晚,天水围飞马首次以元朗大球场作为主场,迎战南华。天水围飞马教练列卡度于61分钟派出曾其祥后备入替黄展鸿,由于当时天水围飞马场内已有六名外援球员,属于国援身份的曾其祥上阵后,令天水围飞马有多达七名非本地球员同时上阵,违反了足总该届香港甲组足球联赛非本地球员注八出六的规定,故此虽然天水围飞马于比赛中以3-2击败南华,最终被足总判输0-3","text2":"曾其祥以什么为职业?","label":1} {"text1":"阿尔维德·卡尔森(,),瑞典科学家。他最著名的成就是对神经递质多巴胺的研究以及该物质在帕金森氏症中的作用,他也因此成为2000年度诺贝尔生理学或医学奖的获奖者之一。此外,他也是沃尔夫医学奖的获奖者。卡尔森出生在瑞典的乌普萨拉,是隆德大学历史学教授Gottfrid Carlsson的儿子。1941年,卡尔森也进入隆德大学学习医学。1944年,他参与了检查被释放的纳粹集中营的犯人的工作,他们是被Folke Bernadotte带到瑞典来的。虽然瑞典是第二次世界大战的中立国,卡尔森的求学之路还是由于在军队中服役而被迫中断数年。1951年,卡尔森获得了医学和哲学博士学位,并随后成为隆德大学的教授。1959年,他又到哥德堡大学任教授。1957年,卡尔森提出多巴胺不仅仅是过去人们认为的去甲肾上腺素的前体,也是一种位于脑部的神经递质。在阿斯利康制药公司工作期间,卡尔森和他的同事们利用溴苯那敏合成了第一个投入市场的选择性5-羟色胺再吸收抑制剂——苯吡烯胺(zimelidine)。卡尔森发展出了一种测量脑组织中多巴胺含量的方法。根据这一方法,他发现基底核(控制运动的脑部区域)中多巴胺水平特别高。接着他给实验动物服用药物利血平(reserpine),结果导致多巴胺水平下降并引起运动失控。这一现象与帕金森氏症的症状相似。而通过在这些实验动物的饮食中添加左旋多巴(L-Dopa),多巴胺的前体,能够减轻相应症状。这些发现引起了其他医生的注意,他们尝试使用左旋多巴来治疗帕金森氏症患者,发现确实能够减轻疾病早期的一些症状。左旋多巴目前依然是治疗帕金森氏症的最普遍采用的方法的基础。卡尔森是一名饮用水氟化的反对者。他与其他1700多名卫生专业人士共同签署了一个停止氟化饮用水的请愿书。他声称他参与了在瑞典的辩论会,在会中他帮助说服国会,希望基于道德规范,将饮用水氟化定为非法。他相信氟化水侵犯了现代医学原理,即医学治疗应该根据个人情况来进行。","text2":"阿尔维德·卡尔森获得过什么奖项?","label":1} {"text1":"肾萎缩(atrophy of kidney),是根据肾脏体积与人的年龄、性别、身高、体重相互比较后,相对偏小或明显缩小的一种异常现象。长时间的肾脏疾病,导致肾小球受损,使整个肾脏出现体积缩小。肾萎缩和肾功能改变是相辅相成的,当患者出现肾萎缩时已经或者必然会出现肾功能不全及至肾功能衰竭(尿毒症)。肾萎缩可见于先天性肾发育不全及有胶原和钙质沉著或某些急性病变,如:急性肾小球性疾病、糖尿病性肾硬变、肾移植排异、慢性肾小球肾炎、肾皮质坏死、Alport综合症、急性肾小管坏死、高血压性肾硬化等。局灶性或弥漫性肾实质解剖的破坏见于任何肿块性病变(如:囊肿、肿瘤、脓肿和血肿),局灶性肾盂肾炎,实质瘢痕(如肾梗死或萎缩性肾盂肾炎),婴儿型多囊肾,慢性肾盂肾炎或损伤等病变。","text2":"肾萎缩一般在在什么病中可见?","label":1} {"text1":"武胜县万善职业中学,简称万善职中,是一所位于四川省武胜县万善镇的职业中学,它也是广安市重点职业中学。学校为万善中学改制而成,在之前亦为万善职中,其间多反复。学校现在占地面积34500平方米,建筑面积22500平方米,教职工120人(其中省级骨干教师1人,市级教学名师1人,县级教学名师5人,高级教师25人,“双师型”教师20人),在校生1800人。学校教学、生活设施设备齐全、齐备,建有校园计算机网络系统,远程教育信息网络系统、闭路电视网络系统,无线电广播系统。拥有多媒体微机教室、多媒体网络教室,多媒体语音教室等。教育、教学、财务、图书、食堂均实现了计算机管理。","text2":"武胜县万善职业中学的简称是什么?","label":1} {"text1":"塔武连特山国家公园()坐落于西班牙加那利群岛拉帕尔马岛,包括绵延宽阔的塔武连特山,该山为占据了岛的北部的一个大型火山。它于1954年成为国家公园。塔武连特山直径大约为10-{zh-hans:千米;zh-hk:千米;zh-tw:公里;}-。其最高点为穆查丘斯罗克,海拔2426-{zh-hans:米;zh-hk:米;zh-tw:公尺;}-,可以通过公路抵达。穆查丘斯罗克天文台的望远镜架设于最高点附近。15世纪,西班牙征服加那利群岛时期,这里是当地原住民关切人最后抵抗之处。入侵的西班牙人难以攻破此处,他们只好以举行和谈为借口将关切人首领诱出。公园里主要的植物为加那利松和濒危物种加那利刺柏。","text2":"塔武连特山的直径大约有多少?","label":1} {"text1":"中国女子足球超级联赛是由中国足球协会组织的,中国女子职业足球俱乐部参加的原全国最高水平的足球职业联赛,该联赛开始于1997年,脱胎自原中国女子足球联赛。第一届只有4只球队参加,是中国女子足球联赛首次采取主客场制进行比赛。因中国女子足球市场化程度不够,受关注度较低,女超联赛赛制经常变化,在初期为主客场循环制,自2003年起,采取南北分区加决赛的赛制,仍为主客场双循环,2005赛季起采取分赛区的赛会制比赛形式,2007年起和中国女子足球联赛合并,采取南北分赛区和总决赛的赛制,为赛会制比赛形式。因中国女足俱乐部队较少,女足联赛没有成熟的升降级制度。2011年联赛改制为全国女子足球联赛。2015年,女超联赛重新回归,2014年全国女子足球联赛的前八名参加新女超联赛,后八名参加中国女子足球甲级联赛,升降级名额1.5个,第八名降级,第七名与女甲联赛亚军进行升降级附加赛。","text2":"这个联赛是什么时候开始的?","label":1} {"text1":"檀萃,字岂田,号默斋,安徽望江(安庆)人。乾隆二十六年(1761年)进士,选贵州清溪县知县。后任云南禄劝县知县,在黔、粤当官期间,好旅行,凡足迹所至处,辄随手劄录,乾隆四十九年(1784年),见北京皮黄演出的盛况,曾咏下「丝弦竟发杂敲梆,西曲二黄纷乱忙。酒馆旗亭都走遍,更无人肯听昆腔」的诗句。乾隆四十七年奉命运解滇铜赴京,途中翻船,生铜六万余斤沉入水里,又管理铜厂亏缺铜斤一万余斤。为巡抚谭尚忠所参劾,革职查办,流放云南,遂遍历滇中。后受聘于昆明育材书院及黑井万春书院。著有《黔囊》、《蒙岳记》、《农部琐录》、《滇海虞衡志》十三卷、《楚庭稗珠录》六卷、《穆天子传·注疏》六卷等。","text2":"檀萃的字号是什么?","label":1} {"text1":"苏丹航空()是一家苏丹的国家航空公司,总部设在苏丹首都喀土穆。该航空公司以喀土穆国际机场为枢纽,主要经营国内航线以及北非、中东区域内的国际航线。苏丹航空是阿拉伯航空运输组织的成员。苏丹航空成立于1947年,首批飞机是4架哈维兰鸽8座飞机。1952年,苏丹航空引进7架28座的道格拉斯DC-3。这些飞机在飞往亚丁、阿斯马拉、贝鲁特、开罗、吉达等地的国际航线上服务。1959年,苏丹航空加入国际航空运输协会。后来,苏丹航空成为非洲最早引进福克F27的航空公司。1970年代,苏丹航空开始引进波音707和波音737;1980年代引进福克F50;1990年代引进空中客车A310。2007年,苏丹政府对苏丹航空进行私有化改革。改革之后,政府仅持有苏丹航空30%的股份。新的投资者正在加强苏丹航空的预订系统、电子客票、电子值机等方面的技术升级。2008年6月23日,苏丹民航管理局宣布对苏丹航空进行停飞,理由是违反民航安全要求。民航管理局表示这项决定与6月10日发生的苏丹航空109号航班空难无关。但数小时后,苏丹航空排除问题、申诉成功,获准恢复飞行。截至2009年5月1日,苏丹航空共有8个国内目的地和11个国际目的地。截至2017年11月苏丹航空的机队现有以下飞机:","text2":"苏丹航空主要经营哪些地区的的国际航线?","label":1} {"text1":"叶定仕故居位于香港沙头角莲麻坑,建于约1907年,由莲麻坑村叶氏第八代叶定仕出资兴建。叶定仕于19世纪90年代初加入同盟会,积极支持孙中山的革命活动,因此叶氏建屋时仿照孙中山翠亨村的故居兴建。叶定仕故居的历史背景及建筑风格甚具价值,叶定仕故居曾被香港古物古迹办事处评为二级历史建筑,而政府于2009年11月刊宪,将故居提升为香港法定古迹。叶定仕生于1882年(有说1879年),乃莲麻坑叶氏的第8世祖。叶定仕年少时曾到泰国当裁缝,承包军服制作,并累积了一定财富。1908年孙中山到访泰国寻求海外华侨支援其革命事业,叶定仕因此成为同盟会的活跃份子,主持客家人加盟工作,并曾参与数次反清起义,辛亥革命后也曾参与反军阀运动。1936年,叶定仕于泰国的物业在当地的排华运动中被土匪霸占,叶定仕遂与家人回港定居于莲麻坑的住所。1943年,叶定仕在日本占领香港期间逝世。叶定仕故居混合中西建筑风格,使用木材及青砖建造。房屋外观为意大利式建筑风格,设计简约对称。整栋建筑楼高两层,每层各有3个房间,正立面设有门廊,并以方柱支撑著上层的阳台,而阳台的栏杆则以琉璃花瓶作装饰。屋顶采用中式金字瓦顶,而外墙则绘上吉祥图案的壁画。故居自1967年后丢空,屋内物品被人搜掠后大多已散失,连叶定仕的个人资料也消失无踪,包括他当年参与同盟会的文件。现时大屋日久失修,屋外杂草丛生,露台走廊木板破烂不堪,连外墙也需要木条固定支撑,以免倒塌。政府于2008年发表「边境禁区土地规划研究」,建议将莲麻坑剔出禁区范围之外,并活化区内包括叶定仕故居的古迹,冀以点、线、面方式,把具文化遗产价值的建筑物串连发展生态旅游,鼓励非政府组织把空置屋宇及已荒废学校发展作度假营,并设立文物径,将附近的古迹连接起来,促进旅游。","text2":"政府于哪一年发表「边境禁区土地规划研究」?","label":1} {"text1":"米拉加亚龙属(学名:\"Miragaia\")是种剑龙下目恐龙,化石发现于葡萄牙,年代属于侏罗纪晚期。米拉加亚龙以牠们的长颈部而闻名,颈部具有至少17节颈椎。米拉加亚龙的正模标本(编号ML 433)发现于葡萄牙北部奥波多市的劳尔哈组(Lourinhã Formation),年代为侏罗纪晚期(启莫里阶晚期到提通阶晚期),约1亿5000万年前。这个标本是由一个部分颅骨,以及部分的身体前半段所构成;包含以下部位:大部分口鼻部、15节颈椎(缺少最前2节颈椎)、肩带、大部分前肢、13个背部骨板。米拉加亚龙的颅骨,同时也是欧洲所发现的第一个剑龙类颅骨。在正模标本的发现处附近,另外发现一个幼年个体标本(编号433-A),包含:一个部分骨盆、部分的脊柱,也被归类于米拉加亚龙。米拉加亚龙是由奥克塔维奥·马特乌斯(Octávio Mateus)等人在2009年叙述、命名。模式种是长颈米拉加亚龙(\"M. longicollum\");属名是以化石发现处的奥波多市的Miragaia堂区为名,种名则意为「长颈」。马特乌斯等人同时也提出一个系统发生学研究,认为米拉加亚龙与锐龙属于一个名为锐龙亚科(Dacentrurinae)的演化支;而锐龙亚科与剑龙属都属于剑龙科,两者互为姊妹分类单元。米拉加亚龙的明显特征,是其长于一般剑龙类的颈部,由至少17节颈椎所构成;与传统观念中,剑龙类的低矮步态、短颈部不同。米拉加亚龙的颈椎数量,甚至比大部分的蜥脚类恐龙还多(多介于12节到15节颈椎);只有盘足龙、马门溪龙、峨嵋龙等蜥脚类恐龙的颈椎数量,超过米拉加亚龙。马特乌斯等科学家推论,米拉加亚龙的长颈部可使牠们有更大的进食范围,或者是在求偶时具有视觉辨认的功能。蜥脚类恐龙的长颈部,是由部分背椎向前移动构成颈部脊椎、额外增加的颈椎、每个颈部脊椎的长度加长…等因素形成。科学家推测米拉加亚龙的长颈部,是由部分背椎向前移动构成颈部脊椎而形成;而非额外增加的颈椎。与剑龙属相比,米拉加亚龙的颈椎长度略长,但这可能是死后的化石化过程中,遭到外力变形的后果。与其他剑龙科相同,米拉加亚龙的口鼻部前端缺乏牙齿。前肢的尺骨\/桡骨长度比例,与剑龙属的比例相近。颈部肋骨与颈椎愈合。耻骨的末端大,与锐龙相同。背部骨板成三角形。","text2":"米拉加亚龙属化石发现于哪个国家?","label":1} {"text1":"托定咸(Tottenham,)是位于英格兰大伦敦北部哈林盖伦敦自治市的一个区域。位于查令十字东北6.6英里(10.6公里)处。托定咸的历史最早可追溯至中世纪,当时住了七十户人家。随著伦敦市发展,到1894年,托定咸被列入伦敦市区一部分。现在的托定咸居住著来自世界各地的民族,是一个多元文化热点地区。这里有规模较大的非洲-加勒比人、西非裔、库尔德人、土族塞浦路斯人、土耳其人、爱尔兰人和葡萄牙人社区,总共流行达300种语言。托定咸是英超俱乐部托定咸热刺的主场所在地。热刺曾经赢得过两次联赛、八次足总杯、两次联盟杯、一次欧洲优胜者杯和四次联赛杯冠军。热刺的主场白鹿巷球场就位于该处。白鹿巷球场附近有铁路通过,站名即为「白鹿巷」。距离白鹿巷球场最近的地铁车站则是七姐妹车站。","text2":"托定咸位于哪里?","label":1} {"text1":"复线铁路,或双线铁路,是指在同一时间,两个相对的通行方向的列车互不干扰的铁路。有别于单线铁路,单线铁路通常每隔一段距离就设置一个会车时使用的车站,称为“会让站”,用于相对行驶的列车停车避让。复线铁路有点类似于“分道行驶”的公路,通过合理的设置,没有正面相撞的危险,可以大大提高运输的效率。但是复线铁路显然需要更宽的路基,在山区等地形困难的地段,有时不得不仍然只建设单线铁路,甚至部分仅供会车使用的车站不得不建在隧道中和桥梁上。复线铁路解决了单线铁路的会车的问题,但并未解决同方向不同速度的列车的超车问题。快速列车要超越慢速列车有两个方式:各国的繁忙铁路干线、高速铁路、城市轨道交通系统为了实现高密度和高效率,大部分都是复线铁路。平行复线是指两条铁路线路相距很近(中国铁路规定普速复线相距不小于4米,高速复线相距不小于4.4米),走向大致平行,沿途接轨的各车站相同。一般在地势简单或者一次新建成复线的情况下采用这种情况,具有占地节省、投资节省、共用车站的技术设备与行车设施等优点。所经过的桥梁、隧道往往都是复线桥、复线隧道。非平行复线是指两条线路线路虽然也是分为上下行固定行车方向,但是相距可能较远,线路走向可能不一致,沿途接轨的车站也可能不一致的情况。这是因为沿线地势复杂困难,难以布置双线;或者是既有单线运量饱和而新增建第二线改为复线行车,既有老线与第二线由于建设年代不同导致设计、施工技术差别巨大,往往是既有老线沿山麓河谷曲折展线,而新建第二线大量采用桥梁、隧道的捷径线。西康铁路的秦岭特长隧道就是因地势与施工限制,采取两条单线隧道。而丰沙铁路、陇海铁路的宝鸡——天水段,都是在既有单线技术条件下非常落后的情况下,新增二线采取了非常高的线路标准,桥隧比很大。双线铁路有复线与单线并列两种形式,差别在于列车行驶的安排,复线的情况列车在各别行车线只会向固定方向行驶,而单线并列则是各别行车线都可以双向运行。单线并列在台铁称为“双单线”,而台湾高铁称为“双线”。双线铁路并非指有两条铁轨的“双轨铁路”,而单线铁路也非指只有一条铁轨的“单轨铁路”。","text2":"复线铁路没有解决什么问题?","label":1} {"text1":"Miro(原名为Democracy Player及DTV)是由开发的集媒体播放器、网络电视于一身的开源软件,自身可播放多种文件格式,也可通过下载功能下载互联网上的影片并且加以管理,支持Microsoft Windows、Mac OS X及Linux作业系统。Miro是遵循GNU通用公共许可证发布的开源软件。Miro可播放如Quicktime、WMV、MPEG、AVI、XVID等在内的多种格式。Miro可从YouTube、Google影片、Blip以及其他网站下载影片,也可以使用搜索功能在互联网上寻找自己需要的影片。Miro能自动从使用RSS的「频道」下载影片,并进行管理和播放。通过YouTube的高清影片功能,可以获得高质量的影片。","text2":"Miro可以使用搜索功能在什么地方寻找自己需要的影片?","label":1} {"text1":"假想粒子,理论物理学家提出的物理模型中假想的一些粒子,基本没有切实的实验依据,很有可能宇宙中根本就不存在这些粒子。这些粒子的提可能只是为了给某些物理现象作一种可能的解释,或者是因为这种粒子如果存在也不会破坏现有的物理定律,因此没有理由相信它们一定不存在。暗物质相关假说中的假想粒子有大质量弱相互作用粒子(WIMP)、惰性中微子(sterile neutrino)、加速子(Acceleron)。超光速理论研究中的运动速度远超光速的快子(tachyon,也叫速子、迅子)。弦理论中一些仅带有北极或南极单一磁极的磁单极子,以及X玻色子和Y玻色子。作为标准模型的补充提出的超对称理论认为,每一种基本粒子都匹配一种被称为超对称伙伴(Superpartner)的粒子,玻色子的伙伴如重力微子(gravitino)、光微子(photino)、胶微子(gluino),而费米子的伙伴叫超粒子(sparticle),就是在每种费米子前加一个s,如超电子(selectron)、超夸克(squark)、超中微子(neutrilino)。此外还有其它一些理论提出的轴子、引力子等等。在标准模型理论基础上还有一些可能的新强子态复合粒子被提出,由2个胶子或3个胶子组成的胶球(glueball),由胶子和夸克组成的混杂态(hybrid state),由4个夸克组成的四夸克态,由5个夸克组成的五夸克态,由6个夸克组成的六夸克态(双重子态)等等。","text2":"理论物理学家假想粒子的目的是什么?","label":1} {"text1":"萨克森-迈宁根(Sachsen-Meiningen)是韦廷家族在图林根的一个邦国。1680年由萨克森-哥达分裂产生。1871年加入德意志帝国。1918年德国君主制覆亡,1920年成为魏玛共和国图林根州的一部分。萨克森-哥达公爵兼萨克森-阿尔滕堡公爵恩斯特一世有12个儿子,其中有七人活到成年。1675年,恩斯特一世去世,他活着的最年长的儿子弗里德里希继承了他的领地。但遭到其他儿子的反对。1680年,弗里德里希一世与六个弟弟通过协议瓜分了父亲的领土。其中恩斯特一世的第六子博恩哈德得到了迈宁根,并于次年称萨克森-迈宁根公爵。随着1699年萨克森-科堡和1710年萨克森-勒姆希尔德的绝嗣,萨克森-迈宁根扩大了国土。1735年,萨克森-科堡的松讷贝格归于萨克森-迈宁根,1753年,勒姆希尔德的三分之二归于萨克森-迈宁根。19世纪初,萨克森-迈宁根先后加入了莱茵邦联和德意志邦联。1825年,萨克森-哥达-阿尔滕堡绝嗣,归于萨克森-科堡-萨尔费尔德和萨克森-希尔德布尔格豪森。1826年,恩斯廷系诸邦国领地重新划分。萨克森-迈宁根得到了希尔德布尔格豪森、萨尔费尔德、坎堡、克拉尼希费尔德等领地,领土大大增加。1829年,萨克森-迈宁根公国实施宪法。1833年,图林根诸邦加入德意志关税同盟。1866年的德意志内战中,萨克森-迈宁根与奥地利帝国和巴伐利亚王国结盟,与普鲁士王国作战。结果是普鲁士王国获胜,萨克森-迈宁根公爵博恩哈德二世被迫逊位于其子格奥尔格二世。萨克森-迈宁根成为北德意志邦联的一个邦。1871年,萨克森-迈宁根加入德意志帝国。1918年德国革命推翻了君主制,萨克森-迈宁根公国成为萨克森-迈宁根自由州,并于1920年并入图林根自由州。1826年后的萨克森-迈宁根公国分为四个县:","text2":"萨克森-哥达公爵有多少个儿子?","label":1} {"text1":"陈苇绫(),改名前为陈春如,绰号超人,台南市中西区仙草里人,台湾女子举重运动员。2004年夏季奥林匹克运动会第一次代表台湾出赛,而成绩还不如台湾另一名同一量级参赛选手陈涵彤,只获得第11名。2008年北京奥运会女子举重48公斤级比赛中代表中华台北获得金牌,总合196公斤。亦为该队于京奥获得的第一枚奖牌。2009年世界运动会健力赛轻量级,陈苇绫以蹲举,硬举,以及总合三破世界纪录夺得金牌。陈苇绫以总合177公斤在2011年彰化全国运动会中完成女子举重3连霸,赛后宣布退休。陈苇绫在2016年里约奥运女子举重48公斤级获得7名,总合181公斤。2016年6月,2008年夏季奥林匹克运动会举重女子48公斤级银牌得主西贝尔·厄兹坎遭到国际举重总会公布药检呈阳性反应。2016年8月,2008年夏季奥林匹克运动会举重女子48公斤级金牌得主陈燮霞遭到国际举重总会公布药检呈阳性反应。2017年3月,陈苇绫确定递补金牌。","text2":"2009年世界运动会健力赛轻量级,陈苇绫破了哪些世界纪录?","label":1} {"text1":"雅典埃莱夫塞里奥斯·韦尼泽洛斯国际机场(,),是位于希腊斯帕塔的一座民用机场,2001年3月28日开通,为雅典及阿提卡大区服务。1995年,希腊政府与德国建筑施工公司豪赫蒂夫集团(HOCHTIEF)签订了一项合作发展计划,在斯帕塔地区修建一座国际机场。雅典国际机场是一家私营公司,并根据合作发展计划以及希腊法律,在30年内拥有对全部设施的“设计、融资、建造、完善、试营、维护、运作、管理以及发展”的特许权。希腊政府持有机场的55%的股份,其余的45%由三家私人股东持有。机场建筑工程总投资达到2.2亿欧元,费时四年完成。2001年3月28日,雅典国际机场正式投入营运。雅典国际机场位于雅典东南,离市区20公里,离市中心45公里。它占地17.5平方公里,一共有一座航站楼、两条跑道,89个停机位,24个旅客登机桥以及48个登机口。","text2":"机场建筑工程多少年完成?","label":1} {"text1":"李玉,字玄玉,号苏门啸侣,又号一笠庵主人。江苏吴县人。清初著名戏曲作家。约生于明万历末,卒于康熙二十年(1681)前后。平生好奇学古,管花肠篆。吴伟业《北词广正谱序》称“其才足以上下千载,其学足以囊括艺林”,但命运数奇不偶,连厄于有司。遭官府压制。焦循《剧说》卷四谓:元玉系申时行相国家人,为申府孙公子所抑,不得应科举。晚年参加乡试,仅得副榜。甲申变后绝意仕进,故一生未曾为官。在苏州戏曲氛围中,李玉于词曲独有癖好。他既富才情,又娴音律,发为声歌,每借韵人韵事,谱之宫商,以抒垒块之气。李玉是明清以来创作最宏富的传奇作家,所作传奇题材广泛,结构精炼,适合舞台表演,以致“每一纸落,鸡林好事者争被管弦,如达夫、昌龄,声高当代”(钱谦益《眉山秀题词》),于剧坛独树一帜,为苏州派作家的主要代表。存世作品佚作评点曲谱参与曲谱编撰","text2":"“其才足以上下千载,其学足以囊括艺林”这句话出自哪位诗人的哪本著作?","label":1} {"text1":"克伦特罗(Clenbuterol),是一种β2-肾上腺素受体促效剂(β2-adrenergic agonist),类似麻黄素(Ephedrine)作用,临床上经常用来治疗慢性阻塞性肺疾(COPD),亦被作为缓和气喘急性发作时的支气管扩张剂(Bronchodilator)用药。20世纪80年代初,美国Cyanamid公司意外发现其有明显的促进生长、提高瘦肉率及减少脂肪的效果,于是其被畜牧业作为瘦肉精使用。但由于其副作用,欧共体于1988年1月1日起禁止盐酸克伦特罗物质当饲料添加剂使用。1991年被FDA禁止。1997年,中华人民共和国农业部下文严禁β-肾上腺素类激素在饲料和畜牧生产中使用,盐酸克伦特罗列为第一位。欧盟限制克伦特罗使用于用作食用的动物上。在欧盟以外的国家,克伦特罗被当作马匹的气管扩张剂,在治疗上的商品名为Ventipulmin®,可以口服和静脉注射的方式使用,也被用于牛只分娩时,帮助子宫扩张。。克伦特罗也是一种非固醇类的合成和新陈代谢加速剂,作用机制未明。它能够增加肌肉对脂肪的比例,提高动物的瘦肉量,但其毒性高于具有相同功能的莱克多巴胺,若使用于食用性畜产动物,则为非法行为。2006年9月,上海市通报有超过330人因食用了含克伦特罗的猪肉而中毒,在美国也发生类似的案例,这些案例致使美国政府立下使用规定,限制克伦特罗的用途限于马匹医疗。2009年2月,广东省至少有70人因为食用了猪的内脏后,发生食物中毒现象,这起食物中毒被认为与克伦特罗残留所引起。这些患者表示在食用了于当地市场购入的猪内脏后,发生腹痛和腹泻的现象。2010年环法自行车赛选手阿尔伯托·康塔多被验出体内含有克伦特罗,在他的检体中验出的克伦特罗浓度为每毫升50皮克(5 × 10 g),他表示这是因为吃了含克伦特罗的食品所致。但他已因此遭西班牙自由车联盟处罚禁赛一年。","text2":"克伦特罗有什么用途?","label":1} {"text1":"司马模(),字-{}-元表,西晋高密王司马泰的第四子。少好学,与琅琊王司马睿及范阳王司马虓在宗室中有贤名。初封平昌公,晋惠帝末年,拜冗从仆射,累迁太子庶子、员外散骑常侍。成都王司马颖奔长安,东海王司马越以弟弟司马模为北中郎将,镇守邺城。永兴初年,成都王司马颖原来的部下公师籓、楼权、郝昌等攻邺城,广平太守丁邵率众来救,范阳王司马虓也遣兖州刺史苟晞救援,公师籓逃散。之后,迁镇东大将军,镇守许昌,进爵南阳王。永嘉初年,转征西大将军、开府、都督秦雍梁益诸军事,代河间王司马颙镇守关中。东海王司马越征司马模为司空,遣中书监傅祗代替镇守关中。司马模谋臣淳于定建议司马模继续留守长安,和哥哥司马越一个在内、一个在外,遣世子司马保为西中郎将、东羌校尉,镇守上邽,司马越进司马模进位太尉、大都督。311年,永嘉之乱,镇守蒲坂的牙门赵染,求冯翊太守不得,率众投降刘聪。刘聪派其子刘粲和赵染攻长安,司马模派淳于定抵御,被赵染击败。司马模投降赵染。赵染坐在那里数落司马模的罪行,送到刘粲处,刘粲杀了他,以司马模的妃子刘氏赐给胡张本为妻,子司马保继立为南阳王。","text2":"司马模是谁?","label":1} {"text1":"时惠环球控股有限公司(简称时惠环球)是香港的一间零售公司,现时为时富投资旗下公司之一,并曾经在香港交易所上市(编号:996)。时惠的主要业务为营运香港大型家庭用品连锁店实惠家居广场。时惠环球前身为实惠集团,于1986年成立。2001年3月,实惠集团被时富投资集团收购,其后于2005年7月易名为现称。2006年8月,商人田琬善向时富集团收购时惠环球股份,锐意将该公司转型为中国大陆的零售业务,并将原有的香港业务售回网融(中国)控股。2007年10月,时惠环球的大陆业务使用了时惠环球的上市地位,并命名为东方银座()。而时惠环球的名称仍使用于时富投资所控制的零售业务。","text2":"时惠环球控股有限公司的主要业务是什么?","label":1} {"text1":"Dabbler 由 Sketcher 发展而来,是一套面向儿童的电脑绘画软件,由 Fractal Design 最初研制,并配合压感笔和数位板一同使用。为著名绘画软件 Fractal Design Painter 的简化版本。Dabbler 2.0.4 曾在中国大陆很受欢迎。Dabbler 和 Painter 后来经 MetaCreations 最终转给了 Corel。Fractal Design 和 MetaTools 合并为 MetaCreations 后,MetaCreations 发布了 Art Dabbler,Corel 在收购 MetaCreations 后仍发布了一段时间的 Art Dabbler,但后来取消了这一计划,转而开发 Painter Essential。","text2":"Dabbler是什么?","label":1} {"text1":"超异能冒险(Race to Witch Mountain)是部2009年奇幻冒险电影,翻拍自1975年的《巫山大逃亡》(Escape to Witch Mountain),根据 Alexander Key 1968年的同名原著改编,由安迪·菲克曼(Andy Fickman)执导,巨石强森、安娜苏菲亚·罗伯与亚历山大·路德维格(Alexander Ludwig)主演。2008年三月在洛杉矶进行拍摄制作,2009年3月13日北美首映,台湾于4月10日上映。倒楣的的士司机杰克·布鲁诺(巨石强森饰)碰上一对正在逃亡的少年兄妹莎拉(安娜苏菲亚洛饰)及薛夫(阿历山大卢域饰)。由他们上车的一刻开始,亡命旅程立即展开。原来莎拉及薛夫是身怀异能的外星来客,更掌握拯救地球未来的秘密关键;因此成为政府特工及地下组织恶势力的猎物,更有其他外星战队向他们展开追杀。杰克为了救己救人,必须在限时之前,把两名外星少年送回他们在地球的太空基地…金·理查兹和艾克·艾森曼是1975年旧版的男女主角,当年演出都还是孩童青少年,两人皆在新版中客串演出。2007年七月华特迪士尼影业找导演安迪·菲克曼执导,与 Matt Lopez 共同编剧,新版将1975年《巫山大逃亡》改编为「现代重拍版」,与旧版故事是大相异庭。八月巨石强森确选为主角,电影计划于2008年三月开拍。导演表示并非纯粹旧版重拍,确切地说是制作出一个全新章节的大魔域(Witch Mountain,早期译为巫山)世界。原著小说内容充斥著「黑暗惊悚」题材,导演企图将1975年旧版电影当时删减的要素加进新版电影里。新版中加入UFO专家、军队、CIA等各方人马于电影中,更是把外星生物概念加进影片作为主要核心。电影正式定名为《超异能冒险》(Race to Witch Mountain),在洛杉矶开拍。影评家给予《超异能冒险》普遍中等的评价。电影评论网站烂番茄,基于74篇评论中有38%的影评家给予正面的评价。Metacritic网站基于23篇评论,平均分数100分只拿到50分。而IMDb分数投票10分得到5.7分。","text2":"影评家普遍给予《超异能冒险》什么样的评价?","label":1} {"text1":"日本古伦美亚(;)是日本第一家唱片公司,成立于1910年(明治43年)10月1日。长于演歌、歌谣、儿童节目的动画歌曲、传统艺能与学校音乐教材方面,1970年代中期跨足流行歌曲领域。其商号中的「古伦美亚」(,)来自早期的合作对象,美国的。自创业当初至1960年代为止,一直持有英国EMI的与哥伦比亚唱片在日本的发行权利;而前者在1962年转移至当时的东芝音乐工业(之后的东芝EMI,现在的EMI Music Japan),后者在1969年和Sony成立合并公司「CBS Sony」(现今的日本索尼音乐),结果使该公司变成空有「Columbia」的商标留存。早在留声机时代,日本蓄音器商会1929年与古伦美亚唱片合并前,便在台湾发行唱片。台湾第一张唱片是日本蓄音器商会在1914年的发行《大开门》,内容为客家八音。下列依照日语五十音顺序收录。","text2":"台湾的第一张唱片是什么?","label":1} {"text1":"黄尾魣(学名:),又称黄尾金梭鱼,俗名针梭、竹梭,为辐鳍鱼纲鲈形目鲭亚目金梭鱼科的其中一种。本鱼分布于印度西太平洋区,包括东非、红海、模里西斯、阿曼、印度、泰国、圣诞岛、可可群岛、马来西亚、日本、台湾、越南、印尼、菲律宾、巴布亚纽几内亚、香港、澳洲、马里亚纳群岛、新喀里多尼亚、关岛、萨摩亚群岛、东加等海域。该物种的模式产地在红海。水深3至50公尺。本鱼体延长呈鱼雷状,横切面几近圆柱形,侧线直而不弯曲;尾鳍深分叉,胸鳍末端达第一背鳍起点,腹鳍起点远前于第一背鳍起点。鳃盖具一尖棘,上颔骨远前于眼前缘。背部暗绿色,腹部银白色,尾鳍镶黑边,在侧线下具一暗色纵带,背鳍硬棘6枚;背鳍软条9枚;臀鳍硬棘2枚;臀鳍软条9枚,体长可达60公分。本鱼常成群地在礁湖或向海礁区的上方游走,属肉食性,以鱼类为食。为食用鱼,适合各种烹调方式。","text2":"黄尾魣主要分布在什么地方?","label":1} {"text1":"曾淑娥(, ),是台湾女子足球运动员,她自2002年起代表中华台北女子足球代表队参加各项国际赛事。在2002年亚足联U-19女子足球锦标赛中,曾淑娥协助中华台北夺得亚军,并获得大会最佳球员及金靴奖。2008年8月,曾淑娥通过日本女子足球联赛的神户 Tasaki Perule的测试,随即因球队解散,无法成行。2009年1月,曾淑娥通过日本女子足球联赛的千叶 JEF的测试,但该队临时反悔,而无法加入。2009年3月,曾淑娥通过日本女子足球联赛的福冈 J. Anclas的测试,却因该队不提供签证事宜,而无法成行。同年9月,曾淑娥加入澳洲的坎培拉联队。2012年9月,曾淑娥加入法国女甲的罗亚尔圣艾蒂安女子足球俱乐部。2014年2月,曾淑娥加入奥地利SV Neulengbach,于3月23日代表SV Neulengbach先发出战瑞典Tyresö FF在欧洲女子冠军联赛的赛事,攻进一球,使曾淑娥成为台湾第一位在欧洲女子冠军联赛出赛和进球的球员。2014年9月,曾淑娥回到台湾加入台中蓝鲸女子足球队,10月5日首次出赛。2015年球季转队新竹FC,不久后加盟冰岛菲尔基尔足球俱乐部。2015年 到法国阿尔比队测试,成功争取到一季的合约。","text2":"曾淑娥是哪儿的女子足球运动员?","label":1} {"text1":"成都二十中全名成都市第二十中学校,在成都本地又常简称为“二十中”,现有三个校区,分别为:茶店子高中部、黄忠初中部和花照学校。成都二十中原名成都私立大同中学,由李则民于民国19年 (1930年)秋创建,原校址在成都市内的学道街14号,抗日战争期间曾一度迁址双流,1949年后迁至现址,并更名为成都第二初级中学。1956年起开始招收高中学生并再更名为成都二十中。1979年被命名为金牛区重点中学;1990年3月9日被命名为省级重点中学,于1994年获验收;1997年被评为“四川省校风示范校”;2002年7月,学校被正式命名为“四川省国家级示范性普通高中”。学校现有占地约120亩,有高、初中两个校区,并另拥有“全国艺术教育先进学校”、“四川省德育先进集体”、“成都市文明单位”、“实验教学示范学校”、“体育传统项目先进学校”、“心理健康教育示范学校”等多项称号。自2003年起,学校 (高中)每年面向全省招收理科实验班学生。办学指导思想:改革、创新、发展。教育理念:陶冶学生性灵,启迪学生心智,关注学生未来。学校设有总务处、教务处、医务室、实验室、图书室、学校办公室等处室和语文、数学、英语、政治、体育、物理、化学、历史、地理、生物、信息技术等教研组。从1997年起,学校于每年新年期间举办新年音乐会,并且要求全体师生参加,这在成都市是独有的。学校办有校刊《大同梦》(前身《热土》)和《大榕树》,分别由教师和学生主办。学校对外交往广泛:在国内,作为全国高级中学校长委员会成员,学校与全国近60所知名中学有经常性的资料和教师交流;国际上,每年都与美、德、日等国学校开展一系列文化交流活动。学校与日本广岛加计高中于2001年建立了友好关系,2010年与英国麦德赫德语言学校结为友好学校。
","text2":"成都二十中三个校区,分别为哪几个?","label":1} {"text1":"银斑密棘杜父鱼,是辐鳍鱼纲鲉形目杜父鱼亚目绒杜父鱼科的其中一种。本鱼分布北太平洋区,包括日本、白令海、中国黄海、鄂霍次克海、阿拉斯加至加利福尼亚海岸。水深0至120公尺。本鱼体延长而侧扁,头中大;头背具凹槽。口中大,端位;上颔末端延伸至瞳孔之后缘下方。上下颔、锄骨和腭骨均具细齿。吻部及下颔具多对须。鳃盖膜相连但与喉峡部分离。尾柄细长而侧扁。体无鳞而被小棘。背鳍分离,第一背鳍6至9枚硬棘,第二背鳍20至25枚软条;第一背鳍第5与第6硬棘间具一深刻;第二背鳍比第一背鳍鳍条高。体背橄榄色或绿色,体腹白色至黄铜色;体背另具黑色斑块;侧线下方具白色斑点;各鳍黑色或褐色且具大型淡色斑块,但腹鳍除外,最大体长可达20公分。为沿岸浅水域底栖鱼类,喜栖息在石砾底水域。以小鱼及甲壳类为食。可食用,但数量不多,较不具经济价值。","text2":"银斑密棘杜父鱼栖息在多深的水下?","label":1} {"text1":"夏威夷监督吸蜜鸟(\"Drepanis pacifica\")是夏威夷特有的雀,因栖息地消失或过份捕猎的灭绝。夏威夷监督吸蜜鸟是所有已灭绝的鸟类中最美丽的一种。牠们呈黑色,脚上、主羽及近尾巴位置有橙黄色的羽毛。眼睛细小及呈黑色,喙呈黑黄色及稍微弯曲,长达3吋。牠们生活在森林冠层,主要吃山梗菜属的花蜜。夏威夷监督吸蜜鸟是夏威夷其中一种最有荣誉的鸟类。夏威夷原住民将牠们橙黄色的羽毛用在皇室的装饰上,这亦是令牠们灭绝的其中一个原因。穿上牠们橙黄色羽毛的装饰物代表了个人的权位。卡美哈梅哈大帝著名的黄色长衣估计就用了8万只夏威夷监督吸蜜鸟的羽毛来制成。夏威夷原住民会将檀香及面饱树的树汁,加上水造成黏土。他们会将这块黏土放在山梗菜属花朵的附近,等待夏威夷监督吸蜜鸟自投罗网。饥饿的夏威夷监督吸蜜鸟会在花朵内吃花蜜,而脚则会被黏土黏住。他们回来会将夏威夷监督吸蜜鸟的橙色羽毛逐一拔出。期后有指会将夏威夷监督吸蜜鸟当为家禽饲养、煮来吃掉、或是放生,各有不同的说法。不过这几种情况都会造成夏威夷监督吸蜜鸟的减少。夏威夷监督吸蜜鸟所居住的地方改变成为农耕的地方,破坏了牠们的食物。虽然很早就已经发现了夏威夷监督吸蜜鸟,但却很快就消失。在美国及欧洲有很多牠们的标本,都是大量捕捉所获得的。牠们似乎于1899年灭绝。","text2":"夏威夷原住民为什么喜欢将鸟的羽毛作为装饰?","label":1} {"text1":"新人王战是日本日本棋院和关西棋院针对未满25歳、棋力七段以下职业棋士举行的职业围棋比赛。创办于1976年。主办单位为:《赤旗报》和日本两大棋院。优胜奖金为200万日圆。最初参赛资格为未满30岁和棋力七段以下,2006年第31届起,将参赛年纪改为25歳以下,2008年第33届起再修正棋力限制为六段以下,同时增加规定曾获得「新人王」优胜头衔者,即从新人王战中「毕业」,未来不得再参加新人王战。具有参赛资格的职业棋士中,日本棋院奖金排名前17名和关西棋院4名可直接进入本战,其余棋士经由预选再产生日本棋院八名、关西棋院四名,合计33人共同进行本战,以单败淘汰赛产生优胜,其中决赛以以三战两胜决定胜负。","text2":"新人王战是什么比赛?","label":1} {"text1":"饭盛女()又叫饭卖女()、宿场女郎(),是江户时代在宿场以奉公人名义提供性服务的私娼。「饭盛女」是俗称、幕府法规文书使用「食卖女」。从名字上来说是在宿场给客人盛、卖饭的女人。事实上为来往的男性旅客提供性服务。也并非所有的饭盛女都卖淫,单纯从事类似现在仲居那样工作的饭盛女也是有的。17世纪随着五街道和宿场的兴建设置、伴随着日本国内的交通量大增,旅笼屋产业发达起来。宿场有服侍旅客的下女,其中一部分就演变为饭盛女。为了规避江户幕府禁止在宿场卖淫的规定就起了饭盛女这个名字、旅笼之间为了获取旅客纷纷提供了这种服务。当时无偿的公役使宿场经营困难,为了得到客源宿场再三要求幕府公开允许饭盛女的存在。幕府在权衡之后不得不默认,但是对宿屋中饭盛女的人数作了规定(一间旅笼最多两名),以防无限制扩大。饭盛女通常是因父母欠债无法偿还或从别处拐卖而来,生活相当悲惨。据历史学家五十岚富夫所著《饭盛女》一书记载,藤泽宿(位于今 神奈川县藤泽市)的饭盛女平均寿命只有21.3岁,41年间共有39名饭盛女死去。","text2":"饭盛女的出生如何?","label":1} {"text1":"萧规曹随,汉语中的一个成语,形容继任人沿用前任定下的规则、做法等等,不作更改。萧何与曹参两人少年担任秦朝小官吏时,便已是好友,后来西汉建国,萧何身为宰相,曹参身为大将,地位不凡的两人却反而有了嫌隙。萧何担任相国时,参考前朝文献制订典章及制度。萧死前,推荐曹参继任。曹参上任后,认为萧何订下的法令已很完备,所以继续沿用而不作改动。曹参就任汉相国期间,整日饮酒食肉,政治上清静无为,继续执行萧何留下的政策,不予变动。汉惠帝认为自己被曹参轻视,于是命其子御史大夫曹窋劝谏,曹参把曹窋鞭笞了两百下并赶出门外,汉惠帝于是亲自责问曹参。曹参摘帽,向皇帝俯首谢罪:「陛下您认为,您与先帝相比,谁较为英明神武?」皇帝回道:「我怎敢与先帝比?」曹参又问说:「我跟萧何比,谁较贤能?」皇帝说道:「您好像不太比得上他。」曹参接著说:「陛下说得对,且高祖跟萧何平定了天下,法令都健全具备。陛下只要垂拱而治,我们这些官吏坚守岗位,遵守他们的法令而不犯过失,不是很适当吗?」时人歌颂:「萧何制定法律,调和整齐如一;曹参继任相国,遵法而不犯过失。施载清净无为的政策,人民因而安宁统一。」史称「萧规曹随」。据前汉书记载,曹参与萧何于沛县时为挚友,但萧何担任相国时反生嫌隙。不过曹参为汉相国时,却对萧何担任相国时之制度一无变更。现今中文使用中,世人所熟知的「萧规曹随」有率由旧章、依样葫芦、一成不变、沿袭旧规等等意涵,然而曹参为汉相国时,却不是此成语所指称之无所事事,反而是选拔贤才、注重吏治,各官吏均选任老成持重之人,若有好名之人,即使其去职。可见「萧规曹随」的传统见解对曹参为汉相国的治术略有误解。曹参担任相国时,对于下属有极小过错,反而为其隐忍不公开。原典内容如下:司马迁著《史记·卷五十四·曹相国世家》:「参代何为汉相国,举事无所变更,一遵萧何约束。……惠帝怪相国不治事,以为『岂少朕与』?……参免冠谢曰:『陛下自察圣武孰与高帝?』上曰:『朕乃安敢望先帝乎!』曰:『陛下观臣能孰与萧何贤?』上曰:『君似不及也』。参曰:『陛下言之是也。且高帝与萧何定天下,法令既明,今陛下垂拱,参等守职,遵而勿失,不亦可乎?』惠帝曰:『善。君休矣!』……百姓歌之曰:『萧何为法,顜若画一;曹参代之,守而勿失。载其清净,民以宁一。』」扬雄著《法言》·〈渊骞第十一〉:「萧也规,曹也随。」","text2":"萧规曹随是什么意思?","label":1} {"text1":"在数学中,三次互反律是关于模代数中两个对应的三次方程的可解性之间的关系的结论和定理。三次互反律最常使用艾森斯坦整数进行表述。艾森斯坦整数是指由形如 formula_1 的复数组成的环,记作 formula_2。其中 formula_3 和 formula_4 是整数,formula_5 为三次单位根:如果 formula_7 是formula_2中范数为 formula_9 的一个 素数。formula_10 与 formula_7 互素。定义三次剩余符号formula_12 为一个三次单位根,并满足再定义“原初”素数是模formula_14同余于formula_15的素数。由于每个素数在乘以formula_2中的一个单位元后都会成为“原初”素数,因此关于“原初”素数的定律仍具有普遍性。这时,三次互反律说明,对两个不同的“原初”素数 formula_7 和 formula_18,有此外有辅助定理:如果 formula_20 那么:由于因此可以计算任意艾森斯坦整数的三次剩余。","text2":"艾森斯坦整数是指什么?","label":1} {"text1":"国际乒乓球联合会(简称国际乒联,,ITTF)是一个国际性乒乓球管理组织。1926年1月,在德国柏林,德国、奥地利、匈牙利、瑞典等国的代表决定成立国际乒联。同年12月,国际乒联正式成立,并在伦敦举行第一届世界乒乓球锦标赛。国际乒联的总部设在瑞士洛桑,至2016年共有222个成员协会,现任主席是德国籍。惯例: MT\/WT: 男团\/女团; MS\/WS: 男单\/女单; MD\/WD: 男双\/女双; XD: 混双不像其他比较受欢迎的运动组织那样,国际桌球总会倾向承认一些在国际上不被普遍承认或领土有争议性的政府组织,例如:国际桌球总会目前就承认科索沃联邦桌球队,即便科索沃在大部分的国际热门运动都被排斥在外,他也在1970年承认当时不被国际承认的中华人民共和国,并且因而引发后续美国总统尼克森到中国访华的事件,史称乒乓外交。","text2":"国际乒乓球联合会的简称是什么?","label":1} {"text1":"马良弼(;)号汝舟,和名名护亲方良丰(),琉球国第二尚氏王朝时期大臣。童名太良金。他是马世荣(名护亲方良员)之子,马良诠(名护亲方良意)之孙。1551年生于首里城。1579年,为进贡及谢册封尚永王之恩事,以王舅的身份,与长史郑迵(谢名亲云上利山)一起出使明朝。在北京期间,萧崇业、谢-{杰}-赠送给马良弼「国佐元勲」四字匾额。归国后,因功升三司官座敷。1592年,其父马世荣退隐。因兄长马良辅(伊计亲方良真)早卒,马良弼遂以次子的身份继承家督之位。同年就任三司官,领名护间切总地头。当时日本逐渐强大,岛津义弘致书琉球,声称日本要出兵朝鲜,要求琉球提供粮草支援。马良弼与翁寄松(城间亲方盛久)都认为琉球弱小、日本强大,主张对日本和好,同主战派的郑迵、向里瑞(浦添亲方朝师)意见相对。1605年,翁寄松在与郑迵的政治斗争中失势,尚宁王转而倾向对日强硬的态度。1606年,册封使夏子阳、王士桢到达琉球,赠送给马良弼「世承天宠」四字匾额。1609年,萨摩藩派遣桦山久高、平田增宗率兵入侵琉球。尚宁王得知消息后,派马良弼、毛凤朝、菊隐、喜安等人前往北山。马良弼在此期间的行为,各史料是矛盾的。一说马良弼率一千名首里亲军前往北山防御,但中萨军伏兵之计,兵败被俘;马良弼仰天痛哭而不投降,因此受到萨军的尊敬。而另一种说法是马良弼前往北山的运天港,至萨摩军中请罪,并留在萨摩军中作人质,引导萨摩军开入那霸港。萨摩军包围首里城后,尚宁王向萨摩投降,并先至马良弼府邸暂时居住。尚宁王等人被掳至鹿儿岛时,马良弼同毛继祖(丰见城亲方盛续)被留下,一同管理首里城。1611年尚宁王归国后,马良弼继续担任三司官之职。1612年(万历四十一年),再以谢恩王舅的身份出使明朝,并就进贡之事同明朝交涉。翌年归国。马良弼于1614年致仕,由毛继祖继任其职。1615年,尚宁王向明廷报称马良弼因交涉进贡一事失败而遭到处决;事实上,根据《中山世谱》的记载,马良弼并未遭处死,翌年出使萨摩藩,告知赴明谢恩完竣之事。1617年逝世。","text2":"马良弼是哪一国的第二尚氏王朝时期大臣?","label":1} {"text1":"澳门巴士1路线是一条由澳门新时代公共汽车股份有限公司经营,往返妈阁和巴波沙大马路的巴士路线。本线在福利时代,主要采用从英国购入的二手Bristol L5G及Bristol LS5G行驶。后来因为本线客量较低,所以开始使用三菱Rosa,不过可能出现其他中类型的车特见(如平治O814(B车)等等),后来因为1A缩短总站和不经提督马路至妈阁一段路,和延长总站后,客量开始上升,用车亦开始使用苏州金龙10米\/12米(K06-K125)行走,间中有猛狮13.230(M车)在2011年7月31日前,本线主要使用车款如下:在2011年7月31日后,本线主要使用车款如下:2017年9月18日后:本线在开办至2008年间由于有1A、34竞争,由于1A、34跟本路线走线相同,加上班次频密,而且本线没有特别多\"独市位\",令本线客量偏低,用车更是载客量低的三菱Rosa。后来本线延长至至关闸总站,1A路线取消提督马路至妈阁一段路线,令该段路线成为本线\"独市位\",客量开始上升,本线亦不再派出三菱Rosa,改派更多载客量的车行走,除了处理下环区的挤密人流外,有时还要帮助清理在十六浦至提督马路上不到3路线的客量,经常都会有客满情况出现,所以被称为\"「线王」\"。虽然关闸总站附近有很多路线(如5、10、18等等)选择去妈阁总站,但在以下情况下本路线较有优势。由于该线经过的站数少(比其他线更方便),而且近年来(该线途经之处)有大量市民北上或回半岛南区(司打口至河边新街),加上它的最大竞争对手─5路线晚上在水坑尾经常塞车,基本上第二个站乘客已经因客满上不了车,令乘客流量不断增加","text2":"后来为何换成了三菱Rosa?","label":1} {"text1":"内大臣府是日本曾经设立以辅佐天皇、负责宫廷之文书等事务之政府机关。创立于1885年,1945年废止。其最高长官为内大臣,并不等同于日本古代律令制度下的同名太政官职位。明治政府在1885年将太政官制改为内阁制度。作为内阁成员之一,内阁总理大臣职务从国务大臣原负责之职务分离,内大臣一职则负责内阁权限之外宫廷事务而再度设立。明治政府下所设立之内大臣平常在宫中辅佐天皇,负责玉玺保管和诏令与其他宫廷文书之事务。内大臣负责执行天皇之决定,而非直接对民众负责,其职务与权力的范围不甚明朗,缺乏准确之定义,通常是天皇非常信赖、并通常有特殊关系之人。当初太政官制废止之时,前太政大臣三条实美出任内大臣。内大臣空缺之紧急情况由枢密院之议长临时代理内大臣之职务。这始自二二六事件之后枢密院议长一木喜德郎紧急代理内大臣(虽然当日即辞去职务)之先例。另外、还有身为皇族之贞爱亲王出任内大臣(1912年 - 1915年)之先例。当初之内大臣在昭和时代由元老取代,在政府中有非常大的权力。太平洋战争结束后,驻日盟军总司令(SCAP)于1945年11月24日命令废除内大臣府。1907年以降,内大臣府由11人名职员构成。至废止前由以下的人员构成。※括号内为任期","text2":"内大臣一职为何再次设立?","label":1} {"text1":"曼努埃尔·弗里德里希(,),是一位德国足球运动员,在场上司职后卫。弗里德里希在巴特克罗伊茨纳赫出生,于2013年11月自由转会至德甲班霸多蒙特。这名球员出身于于SG Guldental 07俱乐部。1995年他转会到缅恩斯05。其一直都不是一线球员,大部份时间都是在德乙里度过。2002年夏季转会往德甲的云达不莱梅,惟大部分时间都被投置闲散,甚至只能打二队。2004年他再重返缅恩斯05时。缅恩斯05终于升上德甲,他效力了三季上阵逾百场联赛。不过缅恩斯05始终实力不足,2007年只能获德甲第16名要降级至德乙。于是在2007年7月,弗里德里希转会到德甲的勒沃库森。他于2006年3月首次入选德国国家队,是有史以来首位缅恩斯05球员入选正式国家队。不过在正式比赛中他甚少出场,没有入选2006年世界杯以至2008年欧洲国家杯阵容。","text2":"弗里德里希在哪里出生的?","label":1} {"text1":"乌巢之战是曹操与袁绍两军相争其中一部分,是官渡之战中最重要的一个环节。事件导致袁绍大军无粮食而溃败,曹操实力再为增强。汉献帝建安五年(200年),曹操得到天子,迁都许昌,于政治上取得优势,并平定多个势力,遂与河北大族袁绍于官渡(今河南中牟东北)决斗。曹操采用奇袭、声东击西之计、诱敌之计、霹雳车,并派出大将关羽,所以多次击败袁军。虽然如此,但袁绍军兵多粮足,曹操粮少,曹操不能支持下去,正巧袁绍手下谋士许攸来降,许攸献上奇袭一计,曹操言听计从,将袁军粮草全部烧毁,情况自此改变。曹操已经杀死吕布、平定袁术、降服张绣、夺回徐州、并击走刘备,收容关羽。而袁绍则将势力强大的公孙瓒消灭,将要与曹操决斗,争夺天下。200年,袁绍派陈琳写下檄文,指曹操祖父乃宦官,父亲是乞丐,靠金钱做得高官,曹操则把弄朝中大权,要将曹操讨伐。袁绍之后先后派颜良、文丑两员大将分别到白马、延津,皆被击败,两将双双被杀。许攸是袁绍谋士,因为贪财,所以被袁绍赶走,之后投奔曹操。曹操闻讯,说:「子远来了,大事可成!」再请许攸入座相谈。许攸问到:「贵军军粮可以用多久?」曹操答曰:「尚可支持一年。」许攸再说:「哪有这么多?说真的吧!」曹操再答:「还可以支持半年。」许攸说:「难道你不想打败袁绍吗?为何不说真话?」曹操说:「跟你开玩笑而已,其实军粮只剩此月的份量。」许攸献计说:「今孟德孤军独守,既无援军,亦无粮食,此乃危急存亡。现在袁军有粮食存于乌巢,虽然有士兵,但无防备,只要派轻兵急袭乌巢,烧其粮草,不过三天,袁军自己败亡!」曹操听计后大喜,选精兵假扮袁军,马含衔枚,士兵拿著柴草向乌巢出发,遇上其他人问话时,皆回答:「袁绍怕曹操奇袭,派我们把守。」袁军不疑有诈,放其通行。到达乌巢后,曹军放火,营中大乱,大破袁军,粮草尽烧,斩领将眭元进、韩莒子、吕威璜、赵叡等首级,割下淳于琼的鼻,杀士卒千余人,将他们所有鼻割下,连同牛、马的舌头一同送往袁军,袁军将士大惊。曹操问败将淳于琼:「你今天弄成这样,是甚么缘故?」淳于琼答:「胜负乃天所控制的,问我干甚么?」曹操欲想留下淳于琼性命,许攸指,淳于琼日后看到自己的样子必然痛恨曹操,曹操斩淳于琼。","text2":"曹操都采用哪些计策来打败袁绍?","label":1} {"text1":"周瘦鹃(),中国当代作家、园艺家,属于“鸳鸯蝴蝶派”代表人物之一。周瘦鹃原名周祖福,字国贤,系江苏省苏州府吴县人,1895年6月30日(清光绪二十一年闰五月初八)出生于上海,父亲是一名职员。6岁时父亲因病去世,靠母亲为人缝补维持家计。先后就读于上海储实两等小学和老西门民立中学。1912年中学毕业前患病,毛发脱光,得校长器重,留校任教,不久改行为职业作家。他在中学时代即开始从事写作,用笔名“泣红”,在商务印书馆出版的《小说月报》上发表了自己的处女作,话剧《爱之花》。1915年,周瘦鹃参加南社。此后任职于中华书局,翻译出版《福尔摩斯侦探案全集》和《欧美名家短篇小说丛刊》(其中包括高尔基作品中最早的中译),获得稿费后,得以和胡凤君结婚。自1920年起,周瘦鹃任《申报》副刊《自由谈》编辑,直到1932年底。同时,他还主编或与人合编《礼拜六》周刊、大东书局《半月》杂志(后改名《紫罗兰》和《新家庭》)、《紫兰花片》、《良友画报》等多种报刊。1931年,周瘦鹃迁居苏州,在王长河头辟紫兰小筑,人称周家花园。他往来于苏州与上海之间,继续负责申报的《春秋》副刊。1937年淞沪会战爆发,周瘦鹃与程小青两家经浙江南浔避居安徽黟县山区。次年《申报》在租界复刊,周瘦鹃也回到上海复职,居愚园路田庄,并在海格路售卖盆景维持生计。经同学蒋保厘介绍,周瘦鹃加入上海中西莳花会,在比赛中曾两度夺魁,获得彼得葛兰奖杯。第三届仅获次奖,遂愤而退出。至太平洋战争爆发,日军占领租界,周瘦鹃辞去《申报》副刊编务。1946年起,周瘦鹃再度隐居苏州,闭门研究盆景。1950年代以后,陈毅、周恩来、叶剑英等都前去拜访。此后曾发表有关花草园艺、游记的散文集《行云集》、《花花草草》、《花前琐记》、《花前续记》等。在文化大革命中,周瘦鹃受到张春桥的点名批判。1968年8月11日(农历七月十八日)投井身亡,年73岁。1973年获苏州有关方面平反并落实政策。当年被抄家没收的二百多个盆景未予实物归还,由有关部门折价补偿,合每盆二元五角人民币。被抄走的字画包括明代画家沈周的一幅中堂,后折价补偿七百元人民币。","text2":"1946年起周瘦鹃再度隐居开始研究什么?","label":1} {"text1":"绿宝石失窃案 (法语: Les Bijoux de la Castafiore ;英语: The Castafiore Emerald )是丁丁历险记的第21部作品。作者是比利时漫画家埃尔热。本作与之前的丁丁历险记有著很大的不同,丁丁首次进行没有离开自己家的冒险,同时故事中没有明显的反派角色,充满了喜剧色彩。丁丁和船长原本在城堡悠闲度假,却因歌后突然造访而弄得鸡飞狗跳;媒体对歌后的行踪极度关注,穷追猛打;歌后一颗珍贵的绿宝石又突然失踪,引起了一波接一波的疑团,究竟谁的嫌疑最大?是船长刚刚收留的一伙吉卜赛人?是偷偷混入记者群中的神秘男子?是歌后的贴身女仆?还是行迹鬼祟的钢琴师?","text2":"《绿宝石失窃案》的作者是谁?","label":1} {"text1":"拜耳张力学说(Baeyer张力学说,或张力学说)是由阿道夫·冯·拜尔于1885年用以解释不同环烷烃的稳定性而提出的一个理论。这个学说认为,所有环状化合物都具有环平面结构,由于键角(即多边形内角)与\"sp\"杂化轨道正常键角(109°28')有差别,因此所有环系都存在角张力。这个偏转角可以用(\"sp\"杂化轨道正常键角 - 多边形内角)÷ 2 来计算。各常见环烷烃的偏转角可以依此计算出来,见下表。根据这些数据,可以认定大环化合物与小环化合物一样,环系越偏出五元环,偏转角越大,张力越大。由于张力越大,分子能量越高,分子越不稳定,故小环的环丙烷环系容易开环。这便是拜耳张力学说对不同环烷烃稳定性的解释。事实上,大环化合物是稳定的。除三元环和芳香环具有平面结构外,其他环都不是真正的平面结构,因此自然也就不存在所谓“偏转角”,拜耳张力学说是错误的。但它所提出的当分子内键角偏离正常键角时会产生张力的现象,却是存在的。这种张力称为角张力。","text2":"拜耳张力学说是谁提出来的?","label":1} {"text1":"亚马留将军(,,又译亚马喇、亚马勒)是葡萄牙著名的海军将领,有「独臂将军」之称;是澳门第79任总督。亚马留是法西斯古·亚马留()以及安娜·文多沙()所生的第一胎;亚马留分别有一个弟弟及妹妹分别是左齐任·亚马留()及法兰西亚·亚马留()。亚马留和玛利亚()结婚;玛利亚于1843年6月11日为他诞下法西斯古·法里尼亚·亚马留()。亚马留于1803年3月4日在葡萄牙里斯本亚加打那()出生;1821年,身为上校的亚马留随葡萄牙皇家海军舰队到巴西展开殖民战争中被大炮打中,及后在不用麻醉的情况下截肢而失去右臂;当他截肢后把其右臂掷开并高呼「葡萄牙万岁!(!)」;葡萄牙人因此称他为「独臂将军」。1839年,亚马留被获封为骑士。亚马留于1846年4月到达澳门就任总督,上任后第二个月起征收土地税、人头税等多项税务、开始扩张葡萄牙在澳门的版图以及把统治权扩至华籍居民;1847年开放澳门的赌禁禁令;翌年强行在关闸一带拆屋及挖坟以开辟一条由水坑尾至关闸的马路;1849年3月13日派兵拆毁于位关前街的清廷粤海关澳门关部行台及在市政厅大楼内刻有《澳夷善后事宜条议》的石碑。亚马留1849年致其好友的信称:“昨天有人来偷偷告诉我,华人悬赏2万葡币取我的首级,若如此,澳门有经费可以维系4个月。我的头,他们出2万,那我的全身价值多少?”1849年8月22日,香山县龙田村村民沈志亮、郭金堂等人在距离关闸约100米处(即莲峰庙西北角,因当时关闸位于目前位置以南)假装向亚马留告状,后被沈志亮举起刈草刀劈中下颔及斩首而身亡,导致后来之关闸事件。站在澳葡政府角度,亚马留被视为民族英雄,因而该政府在东望洋街、亚马喇马路、亚马留圆形地、亚马喇土腰都以亚马留命名以作纪念。澳葡政府在1940年在澳门南湾亚马留圆形地竖立一座亚马留骑马的铜像,但铜像在中华人民共和国国务院港澳事务办公室主任鲁平要求下于1992年10月28日卸下运回葡萄牙首都里斯本。","text2":"亚马留的出生日期是多少?","label":1} {"text1":"丁一宇(,),韩国男演员。常见错误译名为郑日宇。他是家中长子,有一位大他五岁的姐姐,父亲是一位商人,妈妈是国民大学古代织物研究所所长。2006年因出演MBC长篇家庭喜剧《不可阻挡的High Kick》受到观众喜爱而走红,并凭该剧获得2007年MBC演艺大赏最佳新人奖。2008年,凭借电影《我的爱》获得第44届百想艺术大赏男子最佳新人赏提名,并于同年成为北京奥运会韩国地区的火炬手。2009年,在出演电视剧《一枝梅归来》和《拜托小姐》之后重回校园专心学业,直至2011年以SBS电视剧《49天》作为回归作品。2016年11月14日,出演中国首档明星创业真人秀节目《星星来啦》在上海举行发布会,在各大平台造成热烈回响,点播率已经达到250万。2016年12月8日入伍,由于其于2006年曾遭遇车祸,手腕、骨盆等多处受伤,因而被允许以公益勤务要员身份服役2年。","text2":"丁一宇的妈妈从事了什么职业?","label":1} {"text1":"达则错(),又名达克次湖、达格济错,藏语意为虎顶湖,位于西藏那曲地区尼玛县的一个断陷盆地内,北纬31度49分~31度59分、东经87度25分~81度39分,羌塘高原北部。达则错长21.1公里,最大宽16.9公里,平均宽11.60公里,面积244.7平方公里。湖面海拔4459米,水深31.7米,湖水湛蓝,湖心透明度达7.2米。东西两岸地势平缓开阔,南北岸则山体陡峭。湖岸上分布着多条非常明显的同心古岸堤(见卫星影像图),最高的一条高于现湖面90米,显示出达则错在冰河时期无论面积、水深均远甚于当前。达则错地处干旱、寒冷的北羌塘,年均气温0~2.0度,年降水量仅200毫米,湖水补给主要依赖波仓藏布(又名莫昌藏布)、那若曲。波仓藏布发源于藏北高原中部的雪山,长约200公里。","text2":"达则错湖边地势怎么样?","label":1} {"text1":"《星战》是香港歌手古巨基的录音室专辑,2005年10月20日由金牌娱乐发行。这张专辑共收录十一首新歌,前十首的歌名乍看之下,是七至九十年代的粤语经典流行歌曲-然而,这并不是一张翻唱专辑。为了向香港乐坛致敬,制作单位借用了这些经典歌曲的歌名,再重新作曲及填词。新曲中的部分歌词亦有和原曲相对应。专辑中第一首发表的歌曲为《Monica》,本是张国荣的经典舞曲,但在重新作曲后变为慢板情歌。MV中与香港小姐冠军曹敏莉饰演情侣。另一首主打歌为《天才与白痴》,MV邀得吴佩慈客串演出。2005年度亚洲游戏展,古巨基被委任为亚洲游戏展大使。专辑中的《明星》成为大会主题曲,MV中更加入PS2游戏侠盗银河的画面。为了配合大会主题,一间英国游戏公司更为他度身订造了「银河猎人」的造型,并成为本张专辑的封面。粗体表示四台冠军歌四大电子传媒其他媒介","text2":"专辑中的歌曲《Monica》原唱是谁?","label":1} {"text1":"胡仙博士,OBE,JP(Sally Aw Sian),祖籍福建省永定县,是东南亚企业家,前全国政协委员,胡文虎的养女,曾经拥有香港大坑道及新加坡的虎豹别墅及7份报章,当年人称「报业女王」。由于兄长胡好早逝,胡仙成为胡文虎家族的继承人。1954年胡文虎病逝后,胡仙接掌星岛报业,并在1972年令星岛报业在香港上市。胡仙一度拥有7份报章,包括《星岛日报》、《星岛晚报》、《英文虎报》、《快报》、《天天日报》、《华南经济新闻》及与大陆合资的《深星时报》。1972年7月胡仙在香港创立胡文虎基金会。基金会是以非牟利慈善信托基金的形式注册,所有善款全部来自胡仙个人。1998年,受到亚洲金融风暴影响,胡仙于地产及股票投资严重亏蚀,而且惹上官非(后称「胡仙案」)。同年出售《天天日报》,次年再出售星岛日报予 Lazard Asia。根据律政司前刑事检控专员江乐士披露「胡仙案」的内情,当年时任律政司长的梁爱诗袒护胡仙而在有足够证据起诉胡仙的情况下仍然选择放弃捡控,然而梁爱诗对此否认。","text2":"胡仙什么时候开始接掌星岛报业?","label":1} {"text1":"山原猫(学名:'),又名南美山猫或安第斯山猫,是一种细小的野生猫科。牠们是传统上两种没有亚种的猫科之一。牠们的数量估计少于2500只,但却只有有限的保育工作。山原猫的外观像雪豹,栖息在海拔3500-4800米的地方,远远高于林木线。牠们约有家猫的大小,但尾巴较长,毛色呈银灰色,有斑纹及斑点。牠们体长约60厘米,尾巴长42厘米,肩高36厘米,体重5.5公斤。山原猫是所知最少及最为稀少的猫科之一,对牠们的认识都是来自仅有的观察及一些标本。牠们未被驯养。估计牠们只生活在秘鲁、玻利维亚、智利及阿根廷安地斯山脉的高山上。山原猫喜欢栖息在高山环境,其分布亦受当地的猎物情况所影响。总有效群体大小可能是低于2500只,因失去猎物及栖息地、被猎杀及没有多于250只的亚群等缘故而有下降的趋势。由于牠们只居于高山上,人类居住的山谷成为了屏障,分隔了牠们的群族。牠们在玻利维亚及智利因祭祀用途而受到猎杀。牠们的数量稀少,较同样生活在高海拔的南美草原猫更为稀疏。在其分布地,牠们的遗传多样性非常低。","text2":"为什么山原猫的数量越来越少?","label":1} {"text1":"尚温(;)是琉球国第二尚氏王朝第十五代国王。1795年至1802年在位。童名思五郎金。他是尚穆王世子尚哲的次子。尚穆王死后由他继位。尚温王对儒学在琉球民间的普及做出了贡献。1798年,尚温王接受了国师蔡世昌(高岛亲方)的建议,下令在王宫以北开办国学(现首里高等学校),又开办了三个乡学(平等学校);同时废止了久米村对四个官生名额的垄断行为,规定官生中的二名应为首里人。这引起了不少久米村人的不满,引起了官生骚动。尚温王逮捕了首谋者金文和(松永亲云上)等人,镇压了这次骚乱。1800年(嘉庆五年),清嘉庆帝派修撰赵文楷、编修李鼎元为正副册封使,前往琉球国,册封尚温为王,并赐「海表恭藩」的御笔匾额。1801年,在乡民的请求下,尚温王又下令开办了一个乡学。1802年,琉球第一次向中国派遣首里城出身的官生。|-style=\"text-align: center; background: #FFE4E1;\"","text2":"尚温是哪个国家的国王?","label":1} {"text1":"布袋线,为位于台湾台南市新营区与嘉义县布袋镇间,由台湾糖业股份有限公司新营总厂经营之轻便铁路,今既废止。此线为糖业铁路客运化之始,开办于1909年。纵贯线铁路在选线时,在嘉义曾文溪间即有经过盐水港(今盐水)或新营庄(今新营)之议。详见纵贯线 (南段)。当时虽曾研议另建新营经由盐水至北门屿(今北门区)之官线铁道支线弥补不足,唯此案未成真。新营=盐水=布袋间铁道运输稍后由制糖会社完成,属于盐水港制糖兼办之客运业务。据铁道部之资料,新营庄=盐水港间五哩三分于1909年(明治42年)5月20日开始营业,亦为台湾首条糖业铁路定期营业线。然而,根据同年3月4日《台湾日日新报》资料,在官方核准开办营业线之前,当时新营-{庄}-=岸内-{庄}-间即对外办理客运(一日4往复),车资内地人(即日本人)15钱、本岛人(台湾人)10钱;1913年(大正2年)3月8日,营业区间延至布袋嘴(今布袋)。战后布袋线曾进行数次改动。包括糖铁新营车站(原址位于台铁车站旁,今已成停车场)因破烂不堪,1950年迁移百余米至今址。另外,布袋车站因位于市区之外,曾应民众要求,利用盐业铁路(台盐公司所有)延伸营业间区750m至贴近市区的半路店、但仅维持数年。沿线各车站亦有重修,大部份皆非日治时期原貌。布袋线为762mm狭轨铁路,但在新营=岸内间,另有一条并行之新岸线,为762mm及1067mm轨距之三线轨道。后者可允许台铁货车驶至岸内。新营 - 厂前 - (南信号所) - 工作站前 - 东太子宫 - 太子宫 - 南门 - 盐水 - 岸内 - 义竹 - 埤子头 - 安溪寮 - 前东港 - 振寮 - 布袋 (- 半路店)支线:东子宫 - 纸浆厂","text2":"糖铁新营车站当初迁移的原因是什么?","label":1} {"text1":"奥兹玛计划是康乃尔大学的天文学家法兰克·德雷克,于1960年在美国国家无线电天文台使用位于西维吉尼亚的绿堤电波望远镜所从事的早期搜寻地外文明计划(SETI),实验的目的是通过无线电波搜寻邻近太阳系的生物标志信号。这个计划后来以虚构的奥兹国统治者奥兹玛女王来命名,灵感则来自无线电广播李曼·法兰克·鲍姆出版绿野仙踪这本书中虚构的翡翠城 。德雷克使用直径85英尺(26公尺)的电波望远镜,以频率1.420G赫兹的电波观察天苑四和天仓五 ,这两颗都是在太阳系附近,并且似乎是有适于生物居住的行星。扫描的频宽是40万赫兹,而每一台接收机单频宽是100赫兹,获得的资料以磁带保存以供事后的分析。在四个月断断续续的观测,累积了超过150小时的讯息,但没有发现可供辨识的讯号。在1960年4月8日曾经侦测到一个假信号,事后被确认是来自在高空飞过的飞机.。接收器的波长调整在21公分的附近,这是星际空间中的氢自然发射出的波长;这种波长被认为是试图跨越星际进行无线电通讯的生物所熟悉的,并被作为一种宇宙通讯的标准。第二次的实验称为奥兹玛二代计划,由班杰明·萨克曼和派翠克·帕默共同执行的,在同一个天文台,花了四年(1973-1976)的时间,断断续续的观测了650颗恒星。","text2":"第二次实验由班杰明·萨克曼和谁共同执行?","label":1} {"text1":"五山街道是中国广东省广州市天河区下辖的一个街道,始设于1953年。辖区总面积10.59平方公里,2005年总人口9.56万人,常住人口7.41万人,外来人口2.15万人。2010年,五山街道辖13个社区:茶山、岳洲、东莞庄、华工、华农、广外艺、瘦狗岭、五所、粤垦、农科院、白石岗、高胜、汇景。辖区内有茶山新村、五山花园、天一新村、嘉逸花园、半山翠庭、高胜花园、茶山小区、嵩山小区、凤凰新村、南秀村、南新村、西秀村、东秀村、春晖苑、半山雍景苑、南门苑、茶景苑、瀚景轩、汇景新城、丽晴轩、九洲文化家园等住宅小区。广州地铁3号线五山站岳洲路、五山路、粤汉路、汇景北路、东莞庄路、茶山路、华南东侧路(往 长兴街道 岑村 方向)、东莞庄一横路、长福路、广园快速路(瘦狗岭路)、粤垦路、凤阳路( 华农 校内道路)22路、20路、197路、78路、27路、138路、B10路(原234路)、41路、218路","text2":"五山街道具体位于什么地区?","label":1} {"text1":"守护代是日本中世武家体制下,令制国「守护」的代理职()。在战国时代「下克上」(幕府权威衰落、武家上层体制丧失约束力)的背景下,许多守护大名家的重臣得以守护代名义揽权,实质掌控领国。在镰仓、室町两时代,各国守护常居于武家政权核心的幕府所在地(即镰仓或京都室町),领国内的事务则委任代官执行,故有守护代一职。守护代多由谱代等世袭重臣担任,他们可通过代行守护权限,将领国内的武士组织化,成为本身的权力基础。经过室町时代中期应仁之乱等武家上层内斗后,守护代实质掌控领国的现象尤为普遍;进入15、16世纪之交的战国乱世后,一些势力稳固的守护代甚至转型成为「战国大名」。守护代出身的战国大名包括:","text2":"在战国时代的背景下,守护实际上拥有什么权力?","label":1} {"text1":"李抱玉(),河西武威(甘肃武威)人。本名安重璋,唐肃宗赐姓李氏,唐玄宗为其改名抱玉。出自昭武九姓之一安国。唐朝初年开国功臣安兴贵的后裔。李抱玉家族世代居住于河西,以善养名马而闻名。李抱玉在西州长大,爱好骑射,熟识军事。安史之乱时,唐肃宗在灵武即位,至德二年(757年)五月赐姓李氏,乾元初年(758年),投靠太尉李光弼,屡建战功,由右羽林大将军升为持节郑州(今属河南)诸军事兼郑州刺史。唐玄宗以其战河西有功,为其改名抱玉。李光弼固守河阳(今河南孟县东南),光弼问李抱玉:「将军能为我守南城二日乎?」李抱玉果不负使命,贼帅周挚撤退。以坚守河阳,收复怀州,皆功居第一,迁泽州刺史、兼御史中丞,封栾城县公。唐代宗即位,擢为泽潞节度使、潞州大都督府长史、兼御史大夫,加领陈、郑二州,迁任兵部尚书,封武威郡王。抱玉多次上书恳辞王爵,代宗只好改封其为凉国公,拜为司徒。广德元年(763年),吐蕃攻陷京师长安,唐代宗逃往陕州,李抱玉兼任凤翔节度使,负责围剿溃卒与乡村亡徒组成的土匪军,旬日内平定,以功迁司空。李抱玉为唐朝镇守西部要害,抵御吐蕃入侵,深受唐代宗的恩宠,官至同中书门下平章事,又兼山南西道节度使、河西陇右山南西道副元帅、判梁州事,连统三道节制,兼领凤翔、潞、梁三大府,秩处三公。李抱玉以任位崇重,请求辞去司空及山南西道节度、判梁州事的职务,并退授兵部尚书。他镇守凤翔十余年,虽无破虏之功,但境内比较安定,颇为当时所称赞。大历十二年(777年),卒于河西陇右副元帅、同中书门下平章事任上,年七十四。代宗罢朝三日,追赠太保,谥号昭武。子李自正,字尚贞,少府监,袭凉国公。工正书,其父李抱玉纪功碑阴记为其所书。","text2":"李抱玉名字是怎么得来的?","label":1} {"text1":"蔡守训(),中华民国法官,现任士林地方法院庭长。曾审判小灯泡案、国务机要费案、特别费案等重大案件。因为不畏压力将卸任元首陈水扁判刑,创下中华民国宪政史纪录,与吴定亚和徐千惠被称为「司法悍将」。年幼时父亲由于忙于做生意,母亲则是农务补贴家用,可说是由祖父母从小带大,小时常在祖父的杂货店玩,特别听祖父的话,蔡守训虽不是特别聪颖,但非常用功。本来想考明星高中,但祖父说读师专后即有工作,蔡守训就去考台北师专当老师,师专毕业后于小学担任老师,后来考上中兴大学法律系,1995年开始担任法官。曾审理过台糖弊案、邱小妹事件、陈水扁家庭密帐案等,以特别费案、国务机要费案、小灯泡案最受瞩目。由于国务机要费案和扁案具有关联性,因此台北地方法院决定要协调并案,启动协商机制。审判长蔡守训和周占春都认为两案有相牵连关系,为避免裁判歧异,都主张并案给对方。审核小组五位庭长最终的决议并案由蔡守训负责,双方都表示:「充分尊重审核小组做成的决议,分案机制就是如此」;蔡并说:「平常心,但今后可能会忙一点了」,大法官也解释不违宪。拉法叶案、力霸案、新瑞都案等也都是后案并前案,因此并非特例。2008年12月30日,认定陈水扁涉及重罪,且有逃亡及勾串证人、湮灭证据之虞,裁定收押,全案正式进入审判程序。2009年9月11日,重判陈水扁无期徒刑,禠夺公权终身;吴淑珍也判处无期徒刑,全案还可以上诉。扁案审理期间,承审的法官承受莫大压力,却换来不同立场的严厉批判,因为住宅都遭到独派包围抗议过,网路上也有人留言恐吓下达追杀令,甚至遭到跟拍,人身安全受到威胁,警方出动特勤人员陪同上下班,并在其住宅周边设警戒及加强巡逻保护家属。徐千惠只能选择搬家,行事低调的蔡守训出门需要变装,也都为了扁案几乎牺牲了自己的家庭生活。判决换来的评价两极,有人说他是现代包公,也有人批评他是司法杀手。面对舆论,蔡守训始终都只有淡淡地说「我只是尽法官的本分」。","text2":"蔡守训和谁一起被称为「司法悍将」?","label":1} {"text1":"盾蟒(学名:),又称头黑锦蛇,是脊索动物门爬虫纲有鳞目蛇亚目蟒科盾蟒属的生物,无毒性。主要分布在澳洲的北半部,是当地的特有蛇种。在澳洲原住民的梦世纪(Dreamtime)文化里,盾蟒的形象经常出现于许多传说故事之中。纪录中最长的盾蟒大概有300厘米长,以蟒科动物而言,盾蟒的体型并不算巨大。身体普遍呈淡黄色,有黑色或红色的间条,特色在于其头颈部位多呈深黑色。盾蟒学名中的\"melanocephalus\"在英语中就有「黑色的头」的含义。身体颜色根据地域差异亦会有所分别。盾蟒适应干燥的环境,因此主要生长于沙漠、荒野、热带草原、森林等地带。具备夜行性,多于夜间活动,日间则会躲藏于由其它动物所掘的洞穴或茂密的林间栖息。与一般蟒科动物一样,盾蟒是肉食性生物,主要进食爬虫类、鸟类及小型哺乳类的动物。以卵生繁殖,雌蛇每次能生产3至10枚蛇卵,怀孕期大致相隔两至三个月。发育不良的幼小盾蟒将会成为同伴的食物,天敌为人类及澳洲野犬。由于澳洲积极开拓北部地区,直接使盾蟒的栖息范围锐减,盾蟒的数字亦因而有所下滑,目前已受到动物保育组织的关注。在日本有饲养盾蟒的情况,不过由于澳洲当地的盾蟒栖息环境受到影响,政府下令禁止输出野生品种的盾蟒,因此日本国内作饲养用途的盾蟒,多数是以往从澳洲运往日本用作研究及展览的少量盾蟒的繁殖种。这些繁殖种数量极少,令日本的盾蟒价格高涨。野生的盾蟒多数以捕食爬虫类动物为主,而受饲育的盾蟒主要食粮则是老鼠之类的小型哺乳动物。","text2":"盾蟒主要分布在哪?","label":1} {"text1":"米象或米象鼻虫(学名:),俗称米虫、谷牛,中国北方地区称为麦甲。在台湾、日本和世界其他地方均有分布。常生活在谷物中,繁殖速度快,为谷物中主要的害虫。米象每年约有8~9个世代,一世代约20~50天,在高温下繁殖较快,32℃时一世代只需25天。成虫平均寿命达3个月。成虫用口器将谷物啮成深孔,并产卵于孔内,通常一粒谷粒产一卵,数量依谷粒大小而异。幼虫孵化后以谷粒为食,将谷粒蛀穿成弯曲隧道,并逐渐囓成中空,虫粪则排于谷粒外。幼虫在米粒中结蛹羽化为成虫后具有飞翔能力,便会离开并开始交配。米象在代谢中产生水,一生不“饮水”;喜煤气味,会钻入煤气灶出气口结网;低温时进入假死状态,恢复正常体温后恢复活动。中国农村地区常用暴晒法除米象,实际这不能杀死米虫。正确的做法是将生虫大米置于阴凉通风处,让米象慢慢爬出,再筛;亦可采用磷他锌加有机酸熏杀。较为人熟知的同属物种如下:","text2":"米象一年约能产生多少世代?","label":1} {"text1":"天主教格拉纳达总教区(拉丁语:Archidioecesis Granatensis、西班牙语:Archidiócesis de Granada)是罗马天主教在西班牙南部设立的一个总教区,主教座堂设在格拉纳达的布尔戈斯圣母主教座堂。3世纪成立教区,1492年升格为总教区,附属教区有天主教阿尔梅里亚教区、天主教卡塔赫纳教区、天主教瓜迪克斯教区、天主教哈恩教区和天主教马拉加教区。2011年,在当地793,400人口中,有教友764,300人,占辖区总人口96,3%、267个堂区、385名司铎、2名执事、197名修士、669名修女。现任总主教为Francisco Javier Martínez Fernández。","text2":"现任总主教是谁?","label":1} {"text1":"燧发枪(flintlock)是一类存在于17-19世纪的枪械,包括燧发滑膛枪、燧发手枪、燧发线膛枪(来复枪)。燧发枪是由法国于17世纪发明的,燧发枪出现后,很快就取代了之前的火绳枪、簧轮枪以及较原始的燧发枪。燧发枪统治了枪械界长达两个世纪,直到19世纪才被采用击发式点火的枪械逐步取代。原始的燧发击发装置,如(doglock)、(snaphance)、等早在16世纪就出现了,而真正意义上的燧发枪则是由法国于17世纪早期发明,而具体的发明者和发明年份则不得而知。目前比较流行的说法认为燧发枪是法国诺曼底艺术家、枪匠、琴师以及发明家(Marin le Bourgeoys)于1620年发明的。马哈·雷·布吉艾斯设计的燧发击发装置成为了后来燧发枪的标准,很快就取代了之前的击发装置。燧发枪之后统治了枪械界整整两个世纪,直到19世纪早期才被新出现的雷管枪(使用击发式点火)逐步取代。","text2":"燧发枪的种类有哪些?","label":1} {"text1":"张处瑾(),燕人,五代十国初期成德节度使留后张文礼的儿子。921年,张文礼发动兵变,杀死了赵王王镕全家,自立为留后。他既称臣于晋王李存勖又私通后梁,还勾结契丹耶律阿保机。李存勖以为王镕报仇为名,以王镕旧将符习为成德留后,又派相州刺史史建瑭、天平节度使阎宝,率军讨伐镇州。八月十一,晋军攻下了赵州,张文礼大惊腹疽发作而死。张处瑾秘不发丧,多次向李存勖谢罪未果,坚守镇州一年有余,晋军遭受重大打击,史建瑭阵亡,阎宝羞愤而死,随后的主帅昭义节度使李嗣昭、振武节度使李存进也相继阵亡。期间张处瑾又被弟张处球夺权。922年九月廿九,镇州粮尽,第四位晋军主帅李存审(符存审)破城,张处瑾及张处球、弟张处琪及张文礼妻(未知是否张处瑾母)被李存勖处死,张文礼的尸体在市上被车裂。","text2":"张处瑾被谁处死了?","label":1} {"text1":"《王菀之Ivana首张国语创作专辑》为王菀之在2007年推出的首张全国语创作专辑,台湾版发行日期为2007年12月6日,香港版发行日期为2007年12月18日,第二版发行于2008年2月15日。第一版包括一只CD,共收录了十一首新歌及两首Bonus Track。除了四首歌曲为全新创作外,其余歌曲均从王菀之旧作改编。香港版特别收录全新广东歌《奇异恩典》;第二版包括一只CD及一只DVD,DVD中收录了三首新歌的MV。全碟歌曲均由王菀之作曲(《奇异恩典》除外)。全碟由王菀之作曲,除了《奇异恩典》由William Walker作曲及王菀之附加旋律。备注:","text2":"《奇异恩典》的作曲者是谁?","label":1} {"text1":"大唐威龙,是香港漫画《天子传奇》系列的第四部作品,讲述的故事是唐太宗李世民如何成为天子的故事。大唐威龙是黄玉郎作品当中,评价相当高的一部、甚至于有人认为是最好的一部。故事以奇情开场,描述李世民其实是双胞胎,大孖命格富贵,但是短命。细孖虽然只晚一刻钟出生,但有帝王之相。于是极乐正宗的宗主摩诃叶,便将命带帝王相的细孖掳走,秘密训练李世民武功六神诀,准备扶持李世民登上皇位,自己高居太上皇。李世民受命将孪生大哥杀死(应了短命的命数)后,混入李家开始江湖历练。此时唯一目击杀人顶替过程的正一道道士李淳风,则被摩诃叶下了格杀令四处遁逃。故事遂采双线并行。到了杨玄感觉醒霸王转世宿命后,又转为三位主角的三分线。","text2":"大唐威龙,是香港漫画哪一个系列?","label":1} {"text1":"突厥汗国(古代突厥如尼文:;)是原先在柔然统治下的阿史那氏部族于552年在中国以北地区建立的古代汗国,一度控制漠北、中亚等柔然故地。后分裂成东突厥汗国和西突厥汗国,7世纪时先后为唐朝所灭。而东突厥复国后形成的后突厥汗国亦在8世纪为回纥灭亡。有人认为阿史那部最先是生活在咸海边的塞种,后东走至叶尼塞河南方,受铁勒同化;阿史那部于6世纪初年游牧于金山一代(今阿尔泰山),归附于柔然,为其炼铁奴。柔然由于长期与铁勒(柔然人称其为高车人,因为他们高大的车轮而得名)战争而削弱;546年,阿史那部首领脱离柔然,并于550年在首领土门的带领下击败铁勒,552年又打败柔然,建立了政权,自称伊利可汗;同时期的中国将其音译为“突厥”。553年木杆可汗在都斤山(又作“郁都军山”、“乌德鞬山”,今蒙古国杭爱山)建立王庭。全盛时,其疆域东至大兴安岭,西抵西海(咸海),北越贝加尔湖,南接阿姆河南,建立了官制,有立法,有文字。583年,隋将长孙晟用离间之计,使突厥汗国分裂为东西两部,后在屡次与隋朝的战争中战败而走向衰落(参见隋与突厥之战)。唐太宗贞观四年(630年)与薛延陀攻灭东突厥,唐高宗显庆四年(658年)唐又灭西突厥,余部西迁中亚(参见唐灭东、西突厥之战)。在唐高宗末年(682年),再度建立后突厥汗国,最后在745年,后突厥帝国亡于回纥。","text2":"哪一年再度创立后突厥汗国?","label":1} {"text1":"原驼(\"Lama guanicoe\")是原住于南美洲的骆驼科。牠们肩高1.07-1.22米,重90公斤。毛色变化很少,由浅褐色至深肉桂色,腹部渐变成白色。牠们的面部呈灰色,耳朵细小而直立。牠们的眼睛很大,呈褐色,体型流线型,步履活跃。原驼原住于南美洲干旱山区。牠们分布在玻利维亚、秘鲁、厄瓜多尔、哥伦比亚、智利及阿根廷的阿尔蒂普拉诺高原。在智利及阿根廷牠们主要分布在巴塔哥尼亚,但在如百内国家公园及大火地岛,牠们要与家畜竞争,故数量受到限制。玻利维亚印第安人会饲养原驼,这可以帮助保存牠们的数量。原驼一般寿命约20-25岁。现时估计牠们的数量有40-60万匹。原驼是以群族居住,包含了一只主雄驼、几匹雌驼及牠们的幼驼。成年的雄驼会自行再组织群族。当感到威胁时,牠们会吐口水及发出高音的叫声来通知群族逃难。雄驼一般会殿后来保护群族。牠们可以奔跑达每小时56公里,也是游泳能手。牠们只会从仙人掌中吸取养份。原驼是南美洲最大的野生哺乳动物之一。牠们只有一种天敌,就是美洲狮。原驼为了保护颈部,发展出特厚的皮肤,在羊驼、大羊驼及小羊驼都有这种特征。玻利维亚人会用牠们的皮肤来制造鞋。原驼的繁殖期是在11月至2月间,期间雄驼会很激烈的打斗来展示其突出及权力。妊娠期为11个月,每胎会产一匹幼驼。幼驼一出生就可以行走。雄幼驼会在1岁前都会跟著群族。原驼可以栖息在高达海拔3962米的地方。为了在这种低氧气的环境中生存,牠们一茶匙的血液就约有6800万个红血球,是人类的4倍。原驼是纤维很柔软及温暖,在很多高价纤维中都可见到。牠们的羊毛仅次于小羊驼的。原驼的毛皮,尤其是幼驼的,有时会用作代替赤狐的毛皮,因它们的质感极为相似。原驼像大羊驼般是有两层毛皮的:粗糙的护毛及柔软的绒毛,直径约为16-18微米,比最高质的开士米还要纤幼。","text2":"原驼的天敌是谁?","label":1} {"text1":"雪印乳业株式会社(Snow Brand Milk Products Co., Ltd.),是一个日本制造、贩售乳制品的公司。1925年创业。过去是以奶油、起士、牛奶为主力产品的日本最大乳制品公司,雪印集团为年营业额达1兆日圆的食品集团。在丑闻案爆发之后,乳食品事业以外的各部门分离成子公司,现在以制造奶油、起士、玛琪琳为事业重心。。营业本部位于东京都新宿区本塩町13番地,东京本社地图 (MapFan Web)。公司登记地址为北海道札幌市东区苗穂町六丁目1番1号,札幌本社地图 (MapFan Web)。(用地内有雪印乳业史料馆、日本ミルクコミュニティ札幌工场。)2000年6月29日起,以近畿地方为主,发生雪印乳业的乳制品造成的集体食物中毒事件(雪印集体食物中毒事件)。食物中毒原因是由于北海道广尾郡大树町的大树工场疏于清洁管理,导致在混合低脂奶粉及生乳的设备里滋生金黄色葡萄球菌,并使得大阪工场的管理制度缺失与窜改品质管理资料的问题浮出台面。其后于2002年1月,制造火腿、香肠等肉制品的子公司雪印食品被揭发雪印伪造牛肉产地证明事件(将进口澳洲牛肉申报成日本产牛肉,诈领狂牛症对策用的政府补助经费)。雪印企业集团面临存亡危机,事业整体遭到强制重组。甚且,在雪印食品发生的问题不单只是1间食品厂商的丑闻,而是暴露出了日本农畜产业内部隐蔽实情的企业体质。","text2":"2002年,子公司发生了什么影响社会威信的事件?","label":1} {"text1":"副剑齿虎(学名:Paramachairodus)是一种已灭绝的剑齿虎亚科剑齿虎。分布于1500万至900万年前中新世末期的欧洲和亚洲。副剑齿虎是已知最古老的剑齿虎,大量副剑齿虎化石发现于西班牙的马德里附近的一个中新世晚期化石点Cerro Batallones。有两具豹形化石比较出名,一具是大约处在瓦西里期到Turolian期的“Paramachairodus ogygia”,另一具则是完全处在Turolian期“Paramachairodus orientalis”。大多数学者认为发现的第三个物种“Paramachairodus maximiliani”可能和第一具化石是同种的。副剑齿虎肩高约58厘米,与美洲豹相似,但体型更加轻盈。四肢形状显示牠们是灵活的攀爬者,且可以猎杀相对大型的猎物。","text2":"副剑齿虎主要分布于哪个时期?","label":1} {"text1":"中国政府为了奖励在科技进步活动中作出突出贡献的公民,推动中国科技事业的发展,国务院设立五个国家科学技术奖。包括国家最高科学技术奖、国家自然科学奖、国家技术发明奖、国家科学技术进步奖和中华人民共和国国际科学技术合作奖。奖项中规格最高的国家最高科学技术奖每年评审一次,每次选出不超过两名科技成就卓著、社会贡献巨大的公民,由国家主席亲自签署并颁发荣誉证书和高额奖金。每位获奖者的奖金总额均为人民币500万元,其中50万元直接授予个人,另外450万元作为科学研究经费由获奖人全权管理具体用途。国家自然科学奖是授予在数学、物理学、化学、天文学、地球科学、生命科学等基础研究和信息、材料、工程技术等领域的应用基础研究中,阐明自然现象、特征和规律、做出重大科学发现的中国公民的奖项。国家自然科学奖不授予组织。国家自然科学奖设一、二等两个奖励等级。国家自然科学一等奖是自然科学领域的国家最高奖。国家技术发明奖授予在产品、工艺、材料及其系统等有重大技术发明的中国公民。国家技术发明奖不授予组织。国家技术发明奖设一、二等两个奖励等级。国家技术发明奖表彰的是当年中国国民经济中新的、先进的、效益好的新技术。国家科学技术进步奖授予在技术研究、技术开发、技术创新、推广应用先进科学技术成果、促进高新技术产业化,以及完成重大科学技术工程、计划等过程中做出创造性贡献的中国公民和组织。每年国家科学技术进步奖总数不超过400项,分为特等奖,一等奖, 二等奖3个等级。中华人民共和国国际科学技术合作奖是由中国国务院设立的国家科学技术奖励,以奖励对中国科技事业作出贡献的外国人或组织。其前身为国家科学技术委员会设立的中国国际科技合作奖,根据1993年7月经全国人大常委会八届二次会议通过的《中华人民共和国科学技术进步法》的规定,国务院设立“中华人民共和国国际科学技术合作奖”(简称国际科技合作奖)。国家科学技术奖励委员会是对奖励进行评选的机构,设立主任委员,副主任委员,秘书长,委员等职位,一般主任委员,秘书长各一名,副主任委员两名,另外有委员若干名。委员会成员主要是中国科技部,中国科学院,中国工程院及各知名大学的专家学者。","text2":"国家技术发明奖授予哪些人?","label":1} {"text1":"布兰卡契小堂(意大利语:Capella dei Brancacci)是佛罗伦萨卡尔米内圣母大殿内的一座天主教小堂,由于其绘画时代有时被称为“文艺复兴初期的西斯廷小堂”。布兰卡契小堂的设计者是彼得·布兰卡奇,始建于1386年。公众进入需要通过邻近的修道院(由伯鲁乃列斯基设计)。小堂和大殿分别单独向外开放,因此有不同的开放时间,要从小堂进入大殿其余部分参观非常困难。图案装饰的赞助人是菲利斯·布兰卡奇,彼得的后裔,曾担任佛罗伦萨驻开罗使节到1423年。当他回到佛罗伦萨,他雇用Masolino da Panicale画他的小堂。Masolino的助手,21岁的马萨乔,比Masolino 年轻18岁,协助他,但在Masolino离开前往匈牙利担任国王的画师期间,于是由马萨乔负责小堂的建造。这时Masolino回来,向他的优秀学生学习。然而,马萨乔在完成小堂之前被召往罗马,27岁时在罗马去世。后来由菲利皮诺·里皮完成小堂的其余部分。不幸的,在巴洛克时期,一些绘画作品被看作是不合时宜,在前面安置了一个坟墓。","text2":"公众进入布兰卡契小堂需要通过哪里?","label":1} {"text1":"武兹反应(Wurtz reaction),是以法国化学家查尔斯-阿道夫·武尔茨为名。这是一种与有机化学、有机金属化学与最近的无机主族元素聚合物相关的偶联反应,即利用两种卤代烃与钠反应产生新的 C-C 键,以合成更长的碳链:另外其他的金属被发现也能参与反应,比如铁、银、锌。由铟、铜催化或者是锰和氯化铜催化.与卤代芳香烃合成相关的反应则称为武兹-费提希反应。参与反应的自由基为R•。钠的一个电子转移到卤素,产生一个卤化钠和一个烷基。烷基从另一个钠原子接受一个电子变成带负电的烷基阴离子,而钠则形成阳离子。最后,烷基阴离子在S2反应中取代卤素,形成一个新的 C-C 共价键。将碘甲烷和氯乙烷与钠金属粉末放置于无水醚中反应。反应后的烷烃的碳链会比反应前加倍,例如碘甲烷形成乙烷,氯乙烷形成正丁烷。反应使用的溶剂,包括上述的例子,都必需是无水的,因为烷基阴离子是碱性的(烷基质子的pKa是48–50),因此,他们会倾向将水去质子化形成氢氧离子(OH),而产生醇,如此会降低预期烷烃的产出量。因为很多限制(如下),这个反应非常少的用于合成, 尤其是当合成的目标烷烃在天然来源很容易获得的时候。比如说原油,或是很多能够从脂肪酸来转化过来的条件。 然而,Wurtz偶连在关很多小分子环的时候相当有用,特别是三元环。 比如二环丁烷就是通过1-溴-3-氯环丁烷和钠来合成的,并且可以得到95%的高收率。武兹反应只限制于对称烷烃的合成。如果将两种不同的卤烷放在一起反应,结果将会产生两种不同的烷链,并混合在一起,通常将它们分离出来是很难的。当反应包含自由基时,副反应也会产生烯烃。当卤烷与卤素相连的碳原子取代太多,这个副反应便显得很重要。这是因为在第二阶段中S2反应所需的能量会变的很高,所以这个消除反应更容易发生。科里-豪斯合成在烷烃合成反应中具有可选择路径(alternative route),因此排除了武兹反应的一些限制。武兹反应还导致了另一个极有用的试剂的发明,即格林纳试剂。法国人巴比尔指导研究生格林尼亚研究用镁来代替钠做武兹反应,结果反应没有成功,却导致了格林纳试剂的发明。","text2":"武兹反应是以谁的名字命名的?","label":1} {"text1":"2008年至2009年英格兰超级联赛的英格兰超级联赛于2008年8月16日开始,2009年5月24日结束。揭幕战由阿仙奴主场迎战升班球队西布朗,结果阿仙奴凭新将拿斯利的入球以 1–0 战胜对手,全取 3 分。虽然曼联在第三十七轮与阿仙奴的「褪色大战」中仅能以 0–0 战平对手,但仍然无阻他们历来第 11 次夺得英超冠军;这亦是曼联继1998年至2001年后,第 2 次于英超实现三连霸。曼联也藉这次冠军,成功追平利物浦 18 次夺得英格兰顶级联赛锦标的纪录。另外于本季赛事中,每场比赛之后备名额会由以往的 5 个,放宽至 7 个名额,与欧洲各大联赛相符。本季共有20支队伍参加,升班球队有冠军联赛冠军西布朗,亚军史笃城及附加赛冠军侯城。今季新升班球队有3队,来自上季英格兰足球冠军联赛:今季离开超级联赛球队有3队,降级到英格兰足球冠军联赛:","text2":"本次赛季谁获得了英超冠军?","label":1} {"text1":"深圳体育场,座落于深圳市福田区笔架山下,东邻上步北路,北接泥岗西路,南靠笋岗路,落成于1993年6月,总面积达到24892平方米,总投资1.41亿元人民币,场地包含一个主体育场及1个副场。设有12个主看台,可容纳32500名观众,是一座全飘棚式的体育场,场内拥有设施有会议室、运动员休息室、贵宾厅及新闻发布室等。此场地承办2011年夏季世界大学生运动会的足球比赛。深圳体育场是深圳足球队的主场。体育场内有一个标准的国际足球比赛场地,其草坪面积达7992平方米。另外还有一块面积达16900平方米的标准国际田径比赛场地,这块场地的面积在全中国比较,属于较大面积。深圳体育场一共有4层,每层分别设有运动员、裁判员、其他工作人员和观众的入口,这样不会造成混乱和人员交叉。比赛区内有24个呈环状分布的观众坐席区,另外还有三间贵宾包厢,每间可容纳22人。内有4间运动员更衣室和4间裁判员更衣室,这些更衣室设施齐全,配备有洗手间、沐浴间、更衣处等,另外还有许多办公用地。比赛场地内配备了一部电梯,还配备了许多看台观众疏散通道。在场馆的东西两侧还有两座天桥,分别连接上步路和泥岗路。在一楼有可供观众撤离以及消防车通过的五个疏散通道口。另外场地内还有可供三百人举行会议的会议室、比赛配套用房及设施、运动员休息室和设备操作室。深圳巴士集团股份有限公司运营之路线深圳体育场从2007年正式从事业单位专制为企业,正式成为深圳市国资委全资控股的国有企业","text2":"深圳体育场可以容纳多少名观众?","label":1} {"text1":"日野江城()坐落于肥前国高来郡有间庄(长崎县南岛原市北有马町戊谷川名)。也被记作日之江城、、火之江城、日之枝城。是国家级历史遗迹。位于岛原铁道北有马站北部的小山丘上。南边有有马川流过,东边有大手川流经。地界是在西北方配置三之丸、在东方配置二之丸以及在中央的山顶上配置了本丸的连郭式平山城。自平成7年(1995年)到平成12年(2000年)为止进行了4次发掘调查。其间出土了除了安土城以外看不到的直线台阶,结合了海外技术的石组,加上了金箔的瓦。能窥知其与先进的丰臣政权有着密切的关系。现在遗迹能确认石垣·空堀。镰仓时代前期的建保年间(1213年-1219年)统领高来郡的藤原经澄所筑。经澄在藤原北家以藤原纯友的子孙(等同于平氏)筑城的时候以有间为姓,后来改称有马。有马氏当初只不过是岛原半岛南部的一个势力,贵纯的时候压制了半岛内的其他势力而成长为战国大名。贵纯以日野江城的支城为由建造了原城。晴纯的时候有马氏的版图到达了最大,一直成长到领21万石为止。然后,作为后盾的大内氏灭亡了而由此变得受到龙造寺氏的压迫。第13代当主晴信成为了天主教大名而在城下建设(葡萄牙语:神学院,此为日本的历史用语),反过来破坏寺院和神社作为城的建筑木材。前面所说的发掘调查出的遗迹是晴信时代的东西。江户时代初期晴信领有4万石,日野江城成为了岛原藩的藩厅。可是,庆长17年(1612年)晴信因为冈本大八事件遭连坐罪在甲斐国切腹自杀。后嗣的嫡子直纯在庆长19年(1614年)被移封到日向国延冈城,有马氏告别了这座约400年的城。有马氏离开后直到元和2年(1616年)松仓重政入城前为止的时间里该城成为了幕府直辖领地。在此期间由锅岛氏、大村氏、松浦氏这些肥前的大名分担城的警戒。可是,松仓氏入封后,对日野江城觉得不便而新建岛原城,日野江城废城从此退出历史舞台。昭和57年(1982年)7月3日,被认定为国家级历史遗迹。自平成7年(1995年)到平成12年(2000年)进行了发掘调查。平成18年(2006年)初城地的约3成左右被损坏的事调查清楚了。这个是进行了合并前的北有马町打算把城地公园化为樱花名胜的时候损坏了,现在,还剩下对现状进行修复的课题。","text2":"为什么日野江城废城会退出历史舞台?","label":1} {"text1":"紫峰大厦(;也叫绿地中心·紫峰大厦()或绿地广场·紫峰大厦();曾用官方名称南京绿地金融中心())是一座位于中国江苏省省会南京市鼓楼区鼓楼广场的摩天大楼。该大厦开发商为上海绿地集团及南京国资集团,由美国芝加哥SOM建筑设计事务所的前任设计合伙人设计,于2008年9月26日封顶,正式高度为450米,共有楼层89层,目前是世界第十高、中国大陆第三高的摩天大楼,也是江苏省最高的摩天大楼。大厦建筑结构,主楼内部由钢筋混凝土的“巨型核心筒”剪力墙系统构建,外围为成型宽翼缘钢、钢筋混凝土构成的复合柱,此二结构之间为钢结构伸臂桁架及带状桁架。紫峰大厦电梯配置列表主楼办公区:酒店部份:商业区:宴会厅:主楼消防电梯:","text2":"是谁设计了紫峰大厦?","label":1} {"text1":"《星战》是香港歌手古巨基的录音室专辑,2005年10月20日由金牌娱乐发行。这张专辑共收录十一首新歌,前十首的歌名乍看之下,是七至九十年代的粤语经典流行歌曲-然而,这并不是一张翻唱专辑。为了向香港乐坛致敬,制作单位借用了这些经典歌曲的歌名,再重新作曲及填词。新曲中的部分歌词亦有和原曲相对应。专辑中第一首发表的歌曲为《Monica》,本是张国荣的经典舞曲,但在重新作曲后变为慢板情歌。MV中与香港小姐冠军曹敏莉饰演情侣。另一首主打歌为《天才与白痴》,MV邀得吴佩慈客串演出。2005年度亚洲游戏展,古巨基被委任为亚洲游戏展大使。专辑中的《明星》成为大会主题曲,MV中更加入PS2游戏侠盗银河的画面。为了配合大会主题,一间英国游戏公司更为他度身订造了「银河猎人」的造型,并成为本张专辑的封面。粗体表示四台冠军歌四大电子传媒其他媒介","text2":"专辑中的歌曲《明星》成为什么主题曲?","label":1} {"text1":"广州国际女子网球公开赛是一项由女子网球联合会(WTA)在中国广州举行的室外硬地网球巡回赛事。该项赛事为WTA三级赛,从2004年起每年举办1次,总奖金为22万5千美元。参赛选手根据比赛成绩计算积分,累计至女子网球联合会的排名系统。赛事的首个单打冠军是中国运动员李娜于2004年夺得,这同时也是中国网球运动员获得的首个单打冠军头衔。广州国际女子网球公开赛采用《ITF网球比赛规则》和《WTA赛事规则》,为单打正选赛32人,双打正选赛16队的单淘汰赛。每场比赛采用三盘两胜制,每盘比赛在局数比分是6比6时,采用决胜局制(七分先胜利);双打第三盘按决胜盘制(十分先胜利)。","text2":"广州国际女子网球公开赛采用什么赛制?","label":1} {"text1":"巴士系数是软件开发中关于软件项目成员之间信息集中及共享度的一个衡量指标。一个项目至少失去若干关键成员的参与(“被巴士撞了”,指代职业和生活方式变动、婚育、意外伤亡等任意导致缺席的缘由)即导致项目陷入混乱、瘫痪而无法存续时,这些成员的数量即为巴士系数。对关键成员的诠释为“对项目不可或缺”,即其掌握的项目信息(例如设计、源码、知识)并不被众多其他成员所熟悉、共享。在一个巴士系数很高的项目中,就算某个人突然不参与工作,也会有很多其他个人掌握相关的项目信息而足以接替他的位置。“巴士系数”作为术语在1998年起就已经在工商管理领域较为常见,Debian项目文件中也有正式提及。曾有讨论称Linux内核的巴士系数低至1——创始人暨首席架构师Linus Torvalds。","text2":"对关键成员是如何诠释的?","label":1} {"text1":"参加2008年北京夏季奥林匹克运动会的中国体操队一行共计27人,其中男运动员6人,女运动员6人,官员与工作人员15人;领队高健,副领队黄玉斌。中国为体操领域的主要强队之一,体操也是中国大陆自1984年参加洛杉矶奥运会以来的每年参赛项目。首次参加的就获得5金4银2铜共计11枚奖牌的好成绩,其中被誉为体操王子的李宁一人独得3金1银1铜共计5枚奖牌。官员与工作人员(15人):领队:高健,副领队:黄玉斌;教练(10人):陈雄、王国庆、王红卫、陆善真、刘桂成、刘群琳、金卫国、熊景斌、莫建邦和黄志基;管理:张红亮,医生:张佩文和刘舒。运动员(12人):男运动员(6人):杨威、李小鹏、陈一冰、肖钦、黄旭和邹凯;女运动员(6人):程菲、杨伊琳、江钰源、何可欣、李珊珊和邓琳琳。","text2":"中国体操队有多少人参加2008年北京奥运会?","label":1} {"text1":"特雷维索足球俱乐部(Associazione Sportiva Dilettantistica Treviso 2009)是意大利特雷维索的一家足球俱乐部,成立于1909年,1993年重组,2009年再次重组。特雷维索队目前在意大利地区联赛“Eccellenza Veneto”中参赛。球队的颜色为浅蓝和白。2005年之前,特雷维索足球俱乐部从未获得过意大利足球甲级联赛的参赛资格,大部分年代都在意大利乙级到丁级之间的联赛中参赛。这期间,俱乐部的最好成绩是意乙的第6名,是由主教练内雷奥·罗科在1950-51赛季带队获得的。1993年,俱乐部由于财政问题破产重组,并降入丁级。在1994至1997三年间,主教练吉乌塞佩·皮隆率领球队连升三级,时隔40多年后重新返回意乙。2001年,球队降入丙1联赛,但很快在2003年返回乙级。2005年,皮隆回到特雷维索执教并率队在意乙中获得了创纪录的第5名,获得了升级附加赛资格,但输给了佩鲁贾。随后,升级的热那亚和都灵被取消资格,特雷维索和阿斯科利顶替以上两队升入甲级。因此,2005-06赛季是特雷维索队历史第一次也是迄今为止唯一一次参加意大利足球甲级联赛。球队战绩不佳,并在联赛中途更换了主教练,在2006年4月9日1比3输给墨西拿之后就以提前降级,最终仅获得联赛倒数第一。“电话门”案终审后,佛罗伦萨和拉齐奥未被判降级,尤文图斯被判为第20名,特雷维索以第19名的成绩降入乙级。2009年夏天,俱乐部最终因为破产而被意大利足协勒令降级。俱乐部重组并使用了新名称——A.S.D. Treviso 2009,以代替原来的Treviso F.B.C. 1993,而球队则参加意大利地区联赛“Eccellenza Veneto”。2013年夏天,俱乐部以\"Associazione Calcio Dilettanti Treviso 2013\"名称重建,降入意大利第6级别联赛 (Promozione)。","text2":"2009年夏天,俱乐部为什么勒令降级?","label":1} {"text1":"高永是台湾的漫画家及作家。出生于台中县大甲镇(今台中市大甲区)。身高170cm,体重55kg,血型O型,星座巨蟹座。毕业于国立政治大学法律学系。代表作为《梵天变》、《星座刑事》、《隋唐英雄传》。1987年政大道南文学奖小说类首奖得主,1987年以《焚梦》出道于《欢乐漫画》,作品画风优美细腻、人物美形,作品常出现星座、占星、塔罗牌等奇幻神秘的主题,而《梵天变》则是以佛教为体材,在政大法律系之所学也展现在《星座刑事》中。喜欢的漫画家有萩尾望都、安彦良和、井上雄彦、池上辽一等。2005年任台北市漫画从业人员职业工会第三届理事长。漫画家冠良为其弟子,后期高永的作品大多与冠良合作创作。以创作《百无禁忌 Miss阿性》著名的漫画家李勉之(Amin)曾任高永的助手。从就读高中就开始进行漫画创作,1981年参加第三届小咪漫画新人奖以《罪与罚》获得第3名,之后数次参加并获得小咪漫画新人奖的奖项。","text2":"高永是谁?","label":1} {"text1":"国际乒乓球联合会(简称国际乒联,,ITTF)是一个国际性乒乓球管理组织。1926年1月,在德国柏林,德国、奥地利、匈牙利、瑞典等国的代表决定成立国际乒联。同年12月,国际乒联正式成立,并在伦敦举行第一届世界乒乓球锦标赛。国际乒联的总部设在瑞士洛桑,至2016年共有222个成员协会,现任主席是德国籍。惯例: MT\/WT: 男团\/女团; MS\/WS: 男单\/女单; MD\/WD: 男双\/女双; XD: 混双不像其他比较受欢迎的运动组织那样,国际桌球总会倾向承认一些在国际上不被普遍承认或领土有争议性的政府组织,例如:国际桌球总会目前就承认科索沃联邦桌球队,即便科索沃在大部分的国际热门运动都被排斥在外,他也在1970年承认当时不被国际承认的中华人民共和国,并且因而引发后续美国总统尼克森到中国访华的事件,史称乒乓外交。","text2":"国际乒联是什么时候成立的?","label":1} {"text1":"路易斯-恩里克·马丁内斯·加西亚(,),是一名已经退役的著名西班牙足球运动员,以场上位置多变而闻名,职业生涯中曾经出任过除了守门员与中后卫之外的所有位置。目前为足球教练,曾执教西甲球队巴塞隆拿。1996年,路易斯·恩里克从皇家马德里自由转会至巴塞罗那,在双方球迷中都引起轩然大波。不过他最终凭借出色表现征服了巴塞罗那的球迷,之后更加成为俱乐部的队长。2008年,他开始担任俱乐部二队的主教练,并带领巴塞B队连续2年升班及取得西乙季军。2011年安历基确认不与巴塞B队续约,并于夏季正式担任意甲球会罗马主教练。他仅执教一季后离任,在2013年起担任西甲球会切尔达主教练,担任一季后离任,在2014年5月重返巴塞隆拿担任主教练,并且于2015年获得「五冠王」的成绩。2017年3月1日在带队6-1战胜希洪竞技后的新闻发布会上,宣布将不会续约并不会担任下一赛季巴塞罗那队的主教练。球员时期与名帅瓜迪奥拉是巴萨及西班牙国家队队友。","text2":"路易斯·恩里克是哪个国家的?","label":1} {"text1":"第三十三届夏季奥林匹克运动会(,),又称2024年巴黎奥运会,将于2024年8月2日至8月18日在法国巴黎举行。共有两座城市参与最终角逐,分别是巴黎以及洛杉矶。美国波士顿原本也提出申办,但于2015年7月27日宣布退出,后美国奥委会决定由洛杉矶顶替。德国汉堡原本也有意申办,但于2015年11月29日宣布由于在全民公投中有超过半数的市民投票反对申办而宣布退出,而最先正式提出申办的罗马在经过多次会议后,于2016年9月21日由罗马市长维吉尼亚·拉吉宣布退出这届奥运会的申办,撤销申办报告。布达佩斯则在2017年2月22日由匈牙利政府撤回申办。另外不论是哪个城市举办,都是该城市第一次举行残奥会。残奥会从1960年开始,1984年夏季在纽约\/曼德维尔举行,1988年起与奥运同城。2017年7月11日,国际奥委会决定以投票同时选出2024年和2028年奥运会的举办城市,两座城市同时获得奥运会举办权,打破以往在每届的7年前才选出的惯例。洛杉矶、巴黎和国际奥委会将在全会结束后立刻进行三方谈判,寻求敲定一份“三方协议”,通过谈判来决定举办奥运会的先后顺序(但仍可能改变,例如其中一方仅保证进入来次决选)。2017年7月31日,经过洛杉矶与巴黎协商决定:由巴黎举办2024年夏季奥林匹克运动会,洛杉矶举办2028年夏季奥林匹克运动会。这使得巴黎在时隔100年后第三次举办奥运会。之后国际奥委会于2017年9月13日在秘鲁利马举行的第131届国际奥林匹克委员会会议上正式确认这一协商。2015年9月16日,国际奥会公布了五个候选城市;然而汉堡在2015年11月29日全民投票后撤回申办,罗马在2016年9月21日由罗马市长撤回申办。布达佩斯在2017年2月22日由匈牙利政府撤回申办。2017年7月31日,经过洛杉矶与巴黎协商决定:由巴黎举办2024年夏季奥林匹克运动会,洛杉矶举办2028年夏季奥林匹克运动会。加拿大奥林匹克委员会(COC)主席奥布特(Marcel Aubut)26日表示,他将推动多伦多争取主办2024年奥运。民调机构Mainstreet Research表示,近1\/2的多伦多市民反对举办奥运。","text2":"哪两座城市参与了第三十三届夏季奥林匹克运动会的最终角逐?","label":1} {"text1":"体循环(又称为大循环)是心血管循环系统中,携带充氧血离开心脏,进入身体各部位进行气体交换及运输养分后,将缺氧血带回心脏的部分。相对于体循环的另一种血液循环则称为肺循环(又称为小循环)。其循环式如下:左心室→主动脉→小动脉→组织微血管→小静脉→大静脉(上、下腔静脉)→右心房先由左心室将从肺静脉送回心脏充满营养和氧气的充氧血从大(主)动脉输出至身体各部位组织的微血管进行养分的运输以及气体的交换。由大动脉渐分支出小动脉,再分支出为微血管。在微血管中,血液中的养分以及氧气分子会送至组织细胞中,组织细胞中的二氧化碳分子以及废物则会送至血液中。接下来再将完成交换及运输的减氧血经由上下大静脉送回右心房,而继续进行肺循环。","text2":"相对于体循环的另一种血液循环又叫什么?","label":1} {"text1":"体循环(又称为大循环)是心血管循环系统中,携带充氧血离开心脏,进入身体各部位进行气体交换及运输养分后,将缺氧血带回心脏的部分。相对于体循环的另一种血液循环则称为肺循环(又称为小循环)。其循环式如下:左心室→主动脉→小动脉→组织微血管→小静脉→大静脉(上、下腔静脉)→右心房先由左心室将从肺静脉送回心脏充满营养和氧气的充氧血从大(主)动脉输出至身体各部位组织的微血管进行养分的运输以及气体的交换。由大动脉渐分支出小动脉,再分支出为微血管。在微血管中,血液中的养分以及氧气分子会送至组织细胞中,组织细胞中的二氧化碳分子以及废物则会送至血液中。接下来再将完成交换及运输的减氧血经由上下大静脉送回右心房,而继续进行肺循环。","text2":"大动脉会分支出什么?","label":1} {"text1":"多线天竺鲷(学名:),又称黄体天竺鲷,俗名大目侧仔,为辐鳍鱼纲鲈形目鲈亚目天竺鲷科的其中一种。本鱼分布于印度西太平洋区,包括菲律宾、澳洲西北部、印尼、越南、台湾及新几内亚等海域。该物种的模式产地在雅加达、爪哇。水深5至40公尺。本鱼体延长而侧扁,眼大,口大略下位。鱼体呈淡黄色,稚鱼期体侧具有排列紧密的深色纵纹,成鱼期则渐渐变模糊,尾鳍凹形,各鳍也呈淡黄色,体长可达10公分。本鱼栖息于水质清澈的珊瑚礁、岩礁,白天躲藏于岩架下或岩洞中,夜间出来觅食,属肉食性,以多毛类或其它底栖甲壳类为食。繁殖期时,雄鱼具有口孵习性,卵约7日化成仔鱼,由雄鱼吐出,具短暂的仔鱼飘浮期。可食用,但多做为下杂鱼处理。","text2":"多线天竺鲷一般栖息在什么地方?","label":1} {"text1":"李重进(),沧州(今河北省沧州市)人,生于太原,五代时后周禁军统帅之一,太祖郭威第四姊福庆长公主之子。李重进在后晋天福中,入仕为殿直。后汉初年,随舅舅郭威征讨于河中。后周建立的广顺元年(951年),太祖郭威以李重进为内殿直都知、领泗州刺史,女婿张永德为内殿直小底四班都知;又升李重进为小底都指挥使,而以张永德接任内殿直都知。翌年(952年),郭威以李重进为大内都点检兼马步都军头,张永德为小底第一军都指挥使;后又以李重进为殿前都指挥使,张永德为殿前都虞候,掌管殿前亲军。两年后,郭威病危,传位于发妻柴氏的侄子、养子柴荣,临终前特命李重进向柴荣行君臣之礼,以免其觊觎皇位。显德元年(954年),世宗柴荣即位,以姑表兄李重进为侍卫亲军马步军都虞侯,妹夫张永德接任殿前都指挥使,分掌侍卫亲军和殿前亲军。李重进、张永德本以姻亲之故,在数年间不次擢升,但后来都在战争中展现出过人的军事才能。在决定后周生死存亡的高平之战后,李重进以战功加使相衔,升侍卫亲军都指挥使,母福庆长公主追封燕国大长公主;而张永德以战功加检校太傅,授义成军节度使,妻寿安公主进封晋国长公主。北宋建隆元年(960年),宋太祖赵匡胤即位,加李重进为中书令,命令韩令坤代替李重进,将重进移镇至青州(治所在今山东省青州市),李重进拒绝调动,派遣幕僚翟守珣说服李筠起兵抗命,翟守珣却将此事泄露给宋太祖,于是太祖要求翟守珣拖延李重进出兵,以防止李重进与李筠南北呼应。翟守珣回去后,向重进诋毁李筠不足与谋事,重进果然中计,错失良机。李筠四月起兵反宋,六月兵败,自焚死。同年九月李重进起兵,十月,太祖亲征,带领石守信、王审琦、李处耘平叛,十一月,到达扬州城下,即日入城,李重进举家自焚。","text2":"广顺元年,太祖郭威封李重进什么官职?","label":1} {"text1":"彩荣路(),是一条位于香港九龙观塘区平山的两线双向行车道路。该路由彩兴路与彩荣路交界开始,经过彩霞道交界,后向彩福邨方向伸展,终点尽头位于彩福邨彩欢楼、彩荣路公园前。彩荣路最首次于1999年5月28日出版的「牛头角及九龙湾分区计划大纲图S\/K13\/13」出现,该计划大纲图已可见彩荣路的雏型。于彩云道及佐敦谷毗邻的发展计划中出现的编号E1、E2及G1道路,则与1999年规划的有所出入,相信已作出修改。E1道路由彩兴路与彩荣路交界为起点,终点尽头于彩霞道与彩荣路交界;E2道路则指由彩霞道与彩荣路交界起至彩荣路与彩荣里交界为止的路段;G1道路指由彩荣路与彩荣里交界起至彩荣路回旋处止的路段。彩荣路首段已于2008年7月11日刊宪,其后于2010年7月16日将其余一段彩荣路刊宪。现在彩荣路分别设有5个巴士站,其中在彩福邨附近设置了两个巴士站。彩荣里(),是香港牛头角佐敦谷内的一条道路。该路主要连接彩云道海水配水库及彩云道食水配水库。该路的路旁设至一条通往佐敦谷登山径的入口。","text2":"彩荣路最首次出现在哪里?","label":1} {"text1":"洪龙浩(,;),是天主教平壤教区原主教。出生于朝鲜半岛平壤,1933年5月25日(基督升天日)晋升司铎。1944年3月24日被教宗庇护十二世任命为平壤宗座代牧区宗座代牧(vicar apostolic),领衔奥济亚,并于同年圣伯多禄及圣保禄宗徒节(6月29日)被担任过平壤宗座署理的德源自治会院区院牧波尼法爵‧苏尔主教祝圣为主教。1962年3月10日,当天主教平壤教区成立后,被教宗若望二十三世任命为该教区首任主教。他于1949年被金日成政府囚禁后失踪。根据平壤教区的前任宗座署理、首尔总主教郑镇奭枢机称:没有消息证明有司铎在1940年代末的迫害中存活下来,当时166名司铎以及修士被杀或被绑架。教廷年鉴仍然将当时的平壤主教,主教洪龙浩,列为失踪,他若活到现在已经是100岁了。这是圣座的姿态,以指明朝鲜教会经受并仍在发生的悲剧。2013年公布的《宗座年鉴》,洪主教被宣布辞世,教区主教之位随即从缺。韩国主教团已请求梵蒂冈册封圣人部发出「无异议」许可,从而为洪主教及其他八十位殉道者启动宣福程序。","text2":"2013年公布的什么文件中,洪主教被宣布辞世?","label":1} {"text1":"托纳蒂赫(Tonatiuh,或作Ollin Tonatiuh)是阿兹特克的太阳神,其名字的意思为「日之运行」。阿兹特克人认为他是天堂托兰(Tollan)之主。由于阿兹特克人相信他取代第四个太阳纪元中的太阳神,成为第五个纪元的个太阳神。根据他们的宇宙观,每一个太阳纪都有一个的太阳,而阿兹特克人认为他们仍然身处于托纳蒂赫的太阳纪。众神在特奥蒂瓦坎创造第五太阳时,纳纳瓦特辛(Nanahuatzin)与特库西斯特卡特尔(Tecuciztecatl)自愿跳入篝火,争逐成为第五纪元的太阳,后来特库西斯特卡特尔抵受不了烈火,相反,纳纳瓦特辛则坚持到底,最后纳纳瓦特辛成为太阳,而特库西斯特卡特尔则成了月亮,而纳纳瓦特辛由一个满身溃伤的神祇变成强大的太阳神托纳蒂赫;不过,托纳蒂赫成为太阳以后,竟向诸神索命,诸神于是献出心脏。阿兹特克人因此认为要借由活人献祭,以确保太阳会继续运行。根据阿兹特克的创世神话,托纳蒂赫要求阿兹特克人以活人向他献祭,否则他拒绝于天上运行。有说每年有20,000人因托纳蒂赫与其他神祇而牺牲,尽管有人认为这个数字是阿兹特克人用来唬吓敌人或遭西班牙人恶意中伤而被夸大的。阿兹特克人深受太阳吸引,并创立太阳历,准确度仅次于马雅人的太阳历。现今保留下来的阿兹特克遗址亦与太阳有密切关系。阿兹特克的活人献祭是非常普遍的,他们把活人的心脏和鲜血奉献给托纳蒂赫,而托纳蒂赫则凭借他的热力和人类奉献给他的鲜血,赐予战士力量和勇气。在阿兹特克的神话中,托纳蒂赫每晚会与黑暗战斗,他会被大地妖怪特拉尔特奎特利吞下,而他们接纳在战斗中被杀的战士,护送他们上天堂。在阿兹特克历法法中,托纳蒂赫是「13天周期」(trecena)中「第一天(死亡)至第十三天(火石)」之主宰。在托纳蒂赫之前神祇的是查尔奇赫特里奎,而紧随托纳蒂赫的则是特拉洛克。","text2":"阿兹特克人决定进行活人献祭的原因是什么?","label":1} {"text1":"鲍里索夫BATE足球俱乐部(,发音:[baˈtɛ];,\"FK BATE Borisov\"),也称波里索夫贝特,是一家设在波里索夫的白俄罗斯足球俱乐部。2008年,俱乐部成为白俄罗斯足球史上第一支杀入欧洲冠军联赛小组赛阶段的球队。队名巴特(BATE)是\"Borisov Works of Automobile and Tractor Electric Equipment\"(鲍里索夫汽车与拖拉机电子设备厂)的缩写。球队最早成立于1973年,但是无法在苏联足球中获得真正的地位。1996年,俱乐部重新建立,参加白俄罗斯联赛,获得了5个联赛冠军和1个杯赛冠军。1996年至2014年间,俱乐部的主场是。2014年,球队迁入新建的鲍里索夫竞技场。","text2":"1996年,重新建立的俱乐部参加白俄罗斯联赛取得了什么成绩?","label":1} {"text1":"上海科学技术出版社位于上海市钦州南路71号,始建于1955年12月初,现设有科学编辑部、工业编辑部、农业编辑部、医学编辑部、科普编辑部、科教编辑部、国际部、合作出版编辑室、声像部等二十几个编辑部、室。建社至今,已出版各类图书11000种,累计印数达5.7亿册。该社于1955年12月初由中国科学图书公司编审处、大东书局、大中国图片出版社、新亚书店等单位联合建成,之后兼并了交流无线电出版社、北京书店上海分店、电世界出版社、世界舆地社、永祥印书馆编辑部、电工图书出版社和上海机电图书出版社,初名科学技术出版社。1957年4月,上海科学普及出版社并入;8月,又与上海卫生出版社合并,改名为“上海科技卫生出版社”,1959年3月,恢复现名。文革时期该社一度被停摆,后并入上海人民出版社。1978年1月恢复建制。","text2":"上海科学技术出版社现设有哪些编辑部?","label":1} {"text1":"美国国家女子足球队(,简称USWNT)是由美国足球联盟()运营,代表美国出战国际比赛的女子足球队。美国国家女子足球队目前位于国际足联的排名的第一位。美国国家女子足球队赢得了三个世界杯冠军(1991、1999和2015)、4枚奥运会金牌(1996、2004、 2008 和2012)和6次()冠军(2000、2003、2004、2005、2007和2008)。19岁以下美国国家女子足球队赢得了2002年19岁以下世界女子足球锦标赛冠军;20岁以下美国国家女子足球队赢得了2008年20岁以下世界女子足球锦标赛冠军。美国国家女子足球队的第一场比赛发生在1985年8月18日,当时的主教练是麦克·莱恩(Mike Ryan)。2004年3月,美国国家女子足球队的两名著名球员米娅·哈姆和米歇尔·阿科尔斯()入选FIFA 100名单。该名单内是为了庆祝国际足联成立100周年,由贝利拟定的包括了125名在世的最优秀的球员的名单。在1997年和1999年,美国国家女子足球队被美国奥委会选为当年的最佳团队。《运动画刊》将美国国家女子足球队选为1999年年度最佳运动员或团队()。1999年女子世界杯足球赛上,美国队通过点球以5比4战胜中国队,赢得世界杯冠军。这被认为是最有影响和最值得回忆的胜利。由于这次胜利,美国队在世界上崭露头角,并引起了媒体对女足运动和女足运动员的极大关注。1999年7月10日,超过9万名观众(女子体育项目上的最高纪录)进入玫瑰碗球场来观看美国队和中国队的比赛。在经过90分钟比赛以及加时赛之后,两队战成0比0。()射入最后一个点球,帮助美国队以5比4战胜中国队。1999年女足世界杯上的胜利让美国队成为了国际明星。她们的胜利是女子足球历史上的里程碑。它让许多年轻女足运动员在女足运动上走得更远。但是,该队伍还是会有失利的时候,例如2016年夏季奥林匹克运动会时在八强赛时与后来银牌得主的瑞典踢和后以PK大战3-4在八强出局。并且只有挪威、日本、德国、巴西曾经胜过该队伍。又因为德国在该届奥运获得金牌间接使美国失去第一位置,名次也被德国取代。","text2":"美国国家女子足球队一共赢得几个世界杯冠军?","label":1} {"text1":"Beuchat国际,简称为Beuchat,于1934年在法国马赛成立,一直致力于潜水器材的设计、生产及推广,现已成为享负盛名的国际品牌。Beuchat业务分三个主要范围公司始创于1934年,由瑞士制表名家的后人 Georges Beuchat成立。 Georges Beuchat 不单是一代潜水先驱,并是于1948年建立的法国水下协会之创办人之一。过去的75年来,公司曾使用不同名称,例如:“Pêche Sport”, “Beuchat”, “Beuchat Sub” 及 “Beuchat International”。Georges Beuchat 于1982年将业务售予Alvarez de Toledo 家族,另于2002年开始由 Margnat 家族接管。Beuchat是一间国际公司,最初由Georges Beuchat率先带产品跨出法国,行销全球。于七十年代,他更剑鱼设计成Beuchat商标 ,时至今天,每件产品都可发现踪影。过去七十五年来,Beuchat一直透过设计、创意及销售新产品带动水底活动潮流。自成立以来,Beuchat一直在生产 水中狩猎 器材方面处于领导位置,并曾为多位比赛者如Pedro Carbonell, Sylvain Pioch, Pierre Roy, Ghislain Guillou 及 Vladimir Dokuchajev赢得多项国家及世界殊荣。Georges Beuchat于1961年荣获Exportation Award。Scubapro的“S”商标乃源自Beuchat 的“Souplair” 呼吸调节器。","text2":"Beuchat三个主要范围公司是谁创办的?","label":1} {"text1":"荷西·马力亚·西列斯(José-Maria Siles),为西班牙记者以及欧洲及国际问题专家。经历巴黎、波恩、拉巴特、柏林、纽约及布鲁塞尔等地担任驻外记者。曾报道过2006年以黎冲突、最后一次黎巴嫩-以色列战争、海地人民起义及刚果民主共和国民主主义运动。现为布鲁塞尔独立信息网络经理、欧洲委员会对外发言人及欧洲小组与欧洲问题独立专家网的成员,巴塞罗那自治大学新闻信息科学专业研究生硕士。就读期间,制作了关于弗朗哥法案下的审查制度的纪录片。西列斯的国际记者生涯开,始于「法国国际广播」新闻办公室,亦是西班牙报纸驻巴黎通讯员。1986年,被西班牙国营频道(TVE)委任为驻德国总编辑。其报道了德国冷战期间最后两年的情况。1989年11月9日夜晚,他是在查理检查站下报道拆除柏林围墙经过的记者之一。在东西德统一后,西列斯至拉巴特开设了西班牙电视台的办事处,报道北非洲上升中的极端伊斯兰教主义,以及西撒哈拉非殖民地化开始的进程。作为一个战地记者,从1992年开始,他见证了戏剧性的萨拉热窝事件和代顿协议后城市的统一。随后,他跟随斯洛博丹·米洛塞维奇前往海牙,并关注了前南斯拉夫战犯法庭。1994年,他被南方电视台、安达卢西亚(西班牙)公共广播公司指定为经理。在改组新闻处以后,远赴西班牙电视台美国分部,成为纽约和华盛顿的通讯员,为期两年。作为一名熟知欧洲问题的通讯员,他报到了欧洲的货币统一,欧盟扩大以及北大西洋公约组织新的军事行动。","text2":"从1992年开始,他见证过什么戏剧性的事件?","label":1} {"text1":"田乞,妫姓,田氏,名乞,谥僖,史记作-{厘}-,又称为陈僖子、陈-{厘}-子、田-{厘}-子。是齐国田氏家族的首领之一,为田氏家族第七任首领,承袭兄长田开担任田氏家族首领。父亲田无宇。历仕齐景公、齐晏孺子、齐悼公三代。兄长田开无嗣子,田乞得立。田乞向百姓征收赋税时用小斗收进,借给百姓粮食时用大斗,向百姓施以恩德,晏婴多次向齐景公进谏,齐景公却不干涉。晏婴出使晋国,暗地对晋国羊舌大夫叔向说:「田家的先祖箕伯、直柄、虞遂、伯戏和先君陈胡公、大姬已经来到齐国,接受祭祀了。齐国将要成为田氏的了。」田氏的支系子孙司马穰苴因战功被任命为大司马,因田氏势力扩大而警惕的高张、国夏对齐景公说穰苴的坏话,司马穰苴被免官。田乞立志要除掉高氏和国氏。前490年,齐景公死后、年少的公子荼即位为齐晏孺子,高张、国夏专权。前489年六月,田乞联合鲍牧等大夫攻撃高张、国夏,十月晏孺子退位后被暗杀。田乞拥立晏孺子异母兄公子阳生为齐悼公。田乞担任齐国的宰相,田氏势力更加稳固。田乞死后,儿子成子田恒继位。","text2":"田乞在齐国的地位是?","label":1} {"text1":"迷鳄属(学名:\"Mystriosuchus\")又译须鳄,意为「汤匙鳄鱼」,是种已灭绝植龙目,生存于三叠纪晚期(诺利阶中期)的欧洲。迷鳄是由埃伯哈德·弗拉士(Eberhard Fraas)在1896年命名,包含两个种:模式种\"M. planirostris\"、\"M. westphali\"。模式种的身长约4公尺,化石为一完整化石,发现于1895年。其颅后骨骼显示,迷鳄是最习惯水生生活的植龙类,而头骨显示迷鳄主要以鱼类为食。迷鳄过去曾归类于个别的亚科-迷鳄亚科(Mystriosuchinae),近年则改归类于伪帕拉鳄亚科(Pseudopalatinae),但迷鳄与该亚科的大部分属有特征上的不同。迷鳄曾被认为是种水生动物,近年在义大利北部发现的一个标本,显示某些迷鳄是完全海生动物。","text2":"据发现,迷鳄以什么为食物?","label":1} {"text1":"小行星8990,即同情小行星(8990 Compassion),是太阳系的小行星之一。其公转轨道位于火星和木星之间。小行星8990于1980年8月6日由捷克的克列特天文台(Kleť Observatory)发现。2001年10月9日,国际天文联合会(IAU)下属的小天体命名委员会(Committee on Small Body Nomenclature)发布公告,将小行星8990、小行星8991和小行星8992分别命名为同情(Compassion)、团结(Solidarity)和宽容(Magnanimity),以悼念2001年9月11日在美国纽约和华盛顿发生的九一一恐怖袭击事件中的罹难者和受害者。小行星8990被命名为同情(Compassion)的原因是希望世界人民同情九一一恐怖袭击中罹难者的朋友和家属,并希望罹难者的朋友和家属能克服悲伤和哀痛。","text2":"小行星8990又叫什么?","label":1} {"text1":"欧洲足协年度最佳阵容(UEFA Team of the Year)是由欧洲足联颁发的一个足球奖项,是通过网上问卷调查评选出来的。该奖项始于2001年。欧足联官方网站的注册用户可以根据欧洲俱乐部的球员当年的表现和欧洲俱乐部或欧洲国家队的教练当年所率领球队的战绩来选择自己心目中的11人名单和教练。2007年,欧足联收到了接近1200万份问卷。近年来,欧足联官方网站允许访问者在参加问卷调查的同时为阿富汗的地雷受害者捐款。欧足联的工作人员根据当年情况拟定了一份60人的大名单。欧足联官方网站的注册用户可以通过访问欧足联官方网站参与调查,然后提交自己心目中的11人名单(1名门将,4名后卫,4名中场,2名前锋)和教练。经过问卷调查和统计数据之后,欧足联会将结果公布于欧足联的官方网站上。卡莱斯·普约尔6次进入最佳阵容;蒂埃里·亨利、伊克尔·卡西利亚斯和克里斯蒂亚诺·罗纳尔多5次进入最佳阵容;亚历山德罗·内斯塔、史蒂芬·杰拉德、利昂内尔·梅西和约翰·特里4次进入最佳阵容;齐内丁·齐达内、罗纳尔迪尼奥、帕维尔·内德维德、吉安路易吉·布冯、哈维·埃尔南德斯和卡卡3次进入最佳阵容。蒂埃里·亨利(2001-2004)、史蒂芬·杰拉德(2004-2007)、利昂内尔·梅西(2008-2011)分别连续4年进入最佳阵容。而克里斯蒂亚诺·罗纳尔多(2007-2011)和伊克尔·卡西利亚斯(2007-2011)则连续5年进入最佳阵容。巴塞罗那的球员26次进入最佳阵容;AC米兰的球员16次进入最佳阵容;皇家马德里的球员16次进入最佳阵容。主教练方面,何塞·穆里尼奥4次进入最佳阵容;亚历克斯·弗格森2次进入最佳阵容。主教练:热拉尔·霍利尔,利物浦足球俱乐部主教练: 塞诺尔·居内什 土耳其国家队主教练: 何塞·穆里尼奥 波尔图主教练: 何塞·穆里尼奥 波尔图\/ 切尔西主教练: 何塞·穆里尼奥 切尔西主教练: 马尔切洛·里皮 意大利国家队主教练: 亚历克斯·弗格森 曼联主教练: 亚历克斯·弗格森 曼联主教练: 何塞普·瓜迪奥拉 巴塞罗那主教练: 何塞·穆里尼奥 国际米兰主教练: 何塞普·瓜迪奥拉 巴塞罗那","text2":"近年来,欧足联官方网站允许访问者在参加问卷调查的同时还可以干什么?","label":1} {"text1":"马里亚纳群岛及帛琉战事是在太平洋战争期间美国军队在1944年6月至11月攻击在马里亚纳群岛及帛琉的日本帝国军队,美军的攻势在切斯特·威廉·尼米兹的指挥下,紧跟随吉尔伯特及马绍尔群岛战事而展开,目的是攻占日军在中太平洋的军事基地,以支援同盟国重返菲律宾群岛,及提供基地对日本本土实施战略轰炸行动。在攻势开始时,美国海军陆战队和美国陆军在美国海军的支援下,于1944年6月在塞班岛实施登陆,作为回应,日本帝国海军联合舰队对支援登陆的美国海军舰队实施打击,在6月19日–6月20日的菲律宾海海战航空母舰战役中,日本海军被打败,舰载机及岸基飞机遭到惨重损失。之后,美军于1944年7月分别在关岛及天宁岛实施登陆,经过激烈的战斗后,美军在7月攻占塞班岛及在8月占领关岛和天宁岛,美军在塞班岛及天宁岛修建机场,至第二次世界大战结束前作为B-29超级堡垒轰炸机对日本本土实施战略轰炸的基地,包括原子弹轰炸广岛及长崎。与此同时,为了保障美军攻击在菲律宾日军时的侧翼安全,1944年9月美国海军陆战队和美国陆军在帛琉的贝里琉及安加尔实施登陆,在贝里琉经过激烈的战斗后,该岛在1944年11月终于被美军占领。经过了在马里亚纳群岛及帛琉的登陆战役后,盟军继而成功地在1944年10月于菲律宾群岛和1945年1月在硫磺岛及硫球群岛实施登陆","text2":"美军在哪儿修建了机场?","label":1} {"text1":"老巴拉克·侯赛因·欧巴马(Barack Hussein Obama, Sr.,),肯尼亚经济学家。卢欧族人。奥巴马的父亲奥尼安戈先生年轻时曾广泛游历,加入英国殖民地军队,旅游欧洲、印度和桑给巴尔。在那里,奥尼安戈先生从罗马天主教改信伊斯兰教,并改了名字侯赛因。在内罗毕他成为一名传教士的厨师和当地的草药师。奥巴马在1959年获得奖学金,到美国的夏威夷大学留学。1961年与美国女子斯坦利·安·邓纳姆在夏威夷结婚,同年儿子巴拉克·奥巴马出生。1962年,奥巴马获得哈佛大学的奖学金,进入该校修读研究生课程。1964年奥巴马与安·邓纳姆离婚,女方取得巴拉克·奥巴马的抚养权。奥巴马未能在哈佛完成博士课程,只取得经济学硕士学位。奥巴马回国后,成为运输部的经济顾问、财政部的高级经济顾问。期间奥巴马发表文章批评政府的计划经济政策。在与乔莫·肯雅塔总统的斗争中失败退出政坛。1982年,他在内罗毕一场车祸中丧生。他的儿子巴拉克·奥巴马是美国第44任总统,也是美国第一位拥有非裔血统(50%)的总统。","text2":"巴拉克·奥巴马是美国的第几任总统?","label":1} {"text1":"檀萃,字岂田,号默斋,安徽望江(安庆)人。乾隆二十六年(1761年)进士,选贵州清溪县知县。后任云南禄劝县知县,在黔、粤当官期间,好旅行,凡足迹所至处,辄随手劄录,乾隆四十九年(1784年),见北京皮黄演出的盛况,曾咏下「丝弦竟发杂敲梆,西曲二黄纷乱忙。酒馆旗亭都走遍,更无人肯听昆腔」的诗句。乾隆四十七年奉命运解滇铜赴京,途中翻船,生铜六万余斤沉入水里,又管理铜厂亏缺铜斤一万余斤。为巡抚谭尚忠所参劾,革职查办,流放云南,遂遍历滇中。后受聘于昆明育材书院及黑井万春书院。著有《黔囊》、《蒙岳记》、《农部琐录》、《滇海虞衡志》十三卷、《楚庭稗珠录》六卷、《穆天子传·注疏》六卷等。","text2":"檀萃曾受聘于哪里?","label":1} {"text1":"西里龙属(学名:\"Silesaurus\")是种恐龙形类动物,生存于三叠纪晚期卡尼阶的波兰,接近2亿3000万年前。西里龙的化石发现于波兰西里西亚奥波莱的Keuper黏土层,西里龙的属名与种名即以发现地西里西亚与奥波莱为名。模式种是奥波莱西里龙(\"S. opolensis\"),是由Jerzy Dzik在2003年命名。西里龙已发现20个化石,使牠们成为研究最深入的恐龙早期祖先。西里龙的身长接近2.3公尺,在机能上可以二足行走。西里龙体型轻,适合奔跑。西里龙是草食性动物,牙齿小、圆锥状、带有锯齿。齿骨前端没有牙齿,某些古动物学家因此认为西里龙具有喙状嘴。一些科学家认为西里龙不是恐龙,但属于恐龙形类(Dinosauriformes)动物。Mikoluszko指出,西里龙的肱骨缺乏延长的三角嵴、颈椎缺乏骨骺…等恐龙的特征。但是,西里龙具有以下恐龙特征:有些研究根据以上特征,将西里龙归类到基础鸟臀目恐龙,或是基础鸟臀目恐龙的近亲。某些科学家认为西里龙是原蜥脚下目与鸟臀目的连接。以下根据M.D. Ezcurra在2006年的研究:以下演化树则是根据斯特林·内斯比特(Sterling Nesbitt)的2011年早期主龙类研究:","text2":"它的化石是在哪里发现的?","label":1} {"text1":"后劲溪是位在台湾高雄市的河川,长13公里,流经楠梓区、仁武区、大社区等地,为高雄地区1,600多公顷的农田的灌溉水源。因后劲(今属楠梓区)聚落得名。凤山县采访册记载:「后劲溪,在仁寿、半屏里交界,源受楠梓坑溪,西南行递纳竹仔门陂,中圳、月眉两圳,旁注大戆、向望仔、海坪三圳,本支过右冲溪,湾中港,汇万丹仔港,合流入海,长十二里。」楠梓坑溪则载:「在观音里楠梓坑街大桥下,源受帮陷、大陂、林仔边三沟,西南行二里许,下授后劲溪。」一来自高雄地区的曹公圳,曹公圳在仁武区的八卦寮与从东蜿蜒而来的狮龙溪汇合。另一支流,则为源自大社区的楠梓坑溪。后劲溪全长约13公里,流域面积广达73.45平方公里。养殖渔业有虱目鱼、鲈鱼、白虾的鱼塭约50公顷。农业有包含一期稻作在内的农田约940公顷。不过后劲地区附近目前有中油高雄炼油厂(含第五轻油裂解厂)、台塑仁武厂、仁大工业区与加工出口区高雄园区。1987年时后劲居民因反对第五轻油裂解厂建立的反五轻运动,有一系列环保抗争运动,后来也有学生组成后劲溪工作队,是社运和环保工作结合的范例之一 。水利会为了解决污染水源灌溉问题,惯用「稀释法」解决灌溉水污染问题。由高雄农田水利会楠梓工作站在2007年的高雄市教师会生态教育中心座谈会提到,因应后劲溪的污染,水利会在曹公圳每天有3台350马力抽水机24小时在供应水量,以稀释灌溉的水源。","text2":"后劲溪长多少公里?","label":1} {"text1":"六原车站()是一由东日本旅客铁道(JR东日本)与日本货物铁道(JR货物)所共用的铁路车站,位于日本岩手县胆泽郡金崎町()大字三尻()字丹藏堰。六原是JR东日本东北本线沿线的一个小车站,属于JR东日本盛冈支社的管辖范围内,是个由北上车站管理,委托由JR东日本子公司Jaster()代为经营的业务委托车站。除了客运业务外,JR货物在六原车站站区内也设货柜装卸场,用以装卸由专用线或车携货运()运来的货柜。其中专用线的部分,六原车站内有一条通往北上高科技制纸(,原三菱制纸北上工厂)、长约0.8公里的专用线,主要是用以将纸制品运往外地,或自酒田港车站将液态氯运往造纸厂。至于在六原车站停靠的货物列车,则包括有自陆前山王发车、驶往八户货运车站的临时专用货物列车,会在六原停车进行车厢的解连。侧式月台1面1线与岛式月台1面2线,合计2面3线的地面车站。※2号月台截至2014年1月为止没有定期旅客列车使用。","text2":"六原车站坐落在什么地方?","label":1} {"text1":"奥林匹克休战(Olympic Truce)最早是指古希腊各个城邦之间,在四年一度的古代奥林匹克运动会前后停止战争、允许运动员参加比赛和观众前往观看的休战协议。在1992年巴塞隆纳奥运时,挪威提议恢复奥林匹克休战的精神,在奥运开幕前一周起至闭幕后一周之间的期间内停战,经与会169国确认。南斯拉夫内战因1994年冬季奥林匹克运动会而暂时休战,是现代第一个因奥林匹克休战而暂时休战的战争。之后每届夏季奥运及冬季奥运的主办国都会要求参与国签署休战协议。但仍有在奥运期间未休战的情形,例如在2008年北京奥运期间的2008年南奥塞提亚战争,而2012年时英国也表示在奥运期间不会停止阿富汗的军事行动。联合国在1993年10月25日通过的第48\/11号决议,呼吁会员国在奥运开幕前七天到闭幕后七天遵守奥林匹克休战。在中也再度在「和平、安全与裁军」一节中提及:「我们促请会员国从今以后个别及集体遵守奥林匹克休战,并支持国际奥林匹克委员会努力通过体育和奥林匹克理想促进和平及人与人之间的相互谅解。」","text2":"恢复后德奥林匹克的休战精神指什么?","label":1} {"text1":"六横镇是浙江省舟山市普陀区最南端的一个海岛镇。以主岛六横岛为名。六横镇总面积113.04平方公里,包括主岛六横岛及附近的佛渡岛、元山岛、凉潭岛等。西隔佛渡水道与宁波市北仑区相望,东南为磨盘洋人口主要位于主岛六横岛上。根据第五次人口普查,当时六横地区各乡镇的人口分别为峧头镇23833人,台门镇23527人,双塘乡9096人,佛渡乡2095人,合计58551人。2005年底六横镇(包括原峧头镇、台门镇、双塘乡)人口5.65万,佛渡乡人口0.21万,合计5,86万。现六横镇镇政府驻西安路54号原峧头镇政府驻地,并与六横岛全国海岛资源综合开发试验区管理委员会合署办公。为舟山市3个享受副县级待遇的乡镇之一(另两个为衢山镇和金塘镇)。下设包括9个渔农村新型社区和45个行政村。其中9个渔农村新型社区基本和1992年前的9个小乡范围相同。","text2":"2005年底六横镇人口有多少?","label":1} {"text1":"多音字是指有多个读音的字,在各种语言普遍都有。一字二音或一字多音往往影响该字的意思。古代用字少,因此产生一字多意。为了区别不同意思,说话时以变音来分别意思,称为破音字。汉语在传播过程中,与其他语言融合或自身发展时,会演生出许多读音,诸如懒音、文白异读、语音脱落等。日本汉字最为明显,日本汉字的读法一般有二个以上,是因不同时期、不同地方传入的结果。在汉语中亦间有取自游牧民族等其他民族语言、梵语、其他汉语语言或方言的发音属于此类汉字简化的「同音合并」主要以北方话为基础,不少在北方话同音但在其他语言如粤语并不同音也被迫合并,如「-{只}-」为「-{只}-」、「-{只}-」的合并字,「-{松}-」为「-{松}-」、「-{松}-」的合并字,间接变成多音字。有些被合并的字甚至普通话也不同音。将字源和意义不相关、甚至读音也不同的字合并,这样一来就容易误读,例如:唐德宗-{李适}-的“-{适}-”(kuò)就很可能被误读为shì。「粤语正音运动」中,部分高校推行何文汇等人主张的「正音」,导致社会上同时存在「正读」和「俗读」多种发音,有专家批评这是「人为制造语音混乱」","text2":"汉字简化的「同音合并」主要以什么为基础?","label":1} {"text1":"乔治·肖(,)是一名英国的植物学家及动物学家。肖生于白金汉郡的一个村落内,于牛津大学莫德林学院(Magdalen College)受教育,于1772年取得其硕士学位,并成为执业医生。1786年起在牛津大学内担任植物学的助理讲师。1788年成为了伦敦林奈学会的共同创办人之一,1789年出任皇家学会的会员。1791年出任大英博物馆自然史的助理馆长一职直至1806年。他发现大部分由汉斯·斯隆(Hans Sloane)捐出的收藏品的状况并不佳,医学及解剖学上可用的都捐到伦敦皇家外科学院(Royal College of Surgeons)的博物馆里去,但更多的藏品如填塞了的动物及鸟类标本等却腐败得只有焚毁一途。由于当年博物馆的薪金极少,因此肖只有花更多时间去靠写作为生,而忽略了去处理这些藏品。肖出版了其中一本最早期介绍澳大利亚动物的英文著作——《新荷兰动物学》(Zoology of New Holland, 1794)。此外,他也是最早期检验鸭嘴兽标本的自然史学家,并将有关描述于1799年出版的《博文学家文集》(The Naturalist's Miscellany)一书中发表。","text2":"肖出版了最早期介绍澳大利亚动物的英文著作是什么?","label":1} {"text1":"丹后发现号(,Tango Discovery)是一列由日本第三部门铁路公司北近畿丹后铁道所拥有,并与西日本旅客铁道(JR西日本)一同联营的特别急行列车,亦是KTR8000形柴油车的暱称。列车名称中的「丹后」是指列车所服务的京都府北部地区在令制国时代的旧名,丹后国。丹后发现号是北近畿丹后铁道与西日本旅客铁道联合经营、服务范围涵盖京都、大阪与丹后等地区的特急列车路网——北近畿大X网路()的成员列车之一。总体的运转系统是由西日本旅客铁道京都站(JR西日本)经由山阴本线、舞鹤线的东舞鹤站和宫福线・宫津线丰冈的直通运转系统,于天桥立站连络特急「桥立」和「文殊」。丹后发现号全部由KTR8000形柴油车担任。在京都直通系统上,为2往复运转,其中1号、2号、4号京都~丰冈间的列车和京都~东舞鹤站间的列车在绫部站会分割并合。又1号由久美滨至丰冈间、2号由丰冈至久美滨间皆是快速列车,3号只行驶京都~福知山、东舞鹤间而不经由北近畿丹后铁道线。丰冈发著在宫津站、和东舞鹤发著在绫部会变更进行方向。营业上最高速度为80km\/h。京都~绫部、福知山间和电车特急「城崎」、「丹波」、「舞鹤」相比所要时间较多。北近畿丹后铁道线内的系统为天桥立~丰冈间设定为2往复。在京都直通系统运行开始以前,以2编成运用,主要接驳搭乘「桥立」、「文殊」特急列车的旅客。除了联络绫部与京都方面的特急外,也行驶绫部~天桥立、久美滨间(绫部~西舞鹤间快速),担任非电化区间舞鹤线的特急连络,现在只有运用1编成。在设定为2往复后,西舞鹤~天桥立间变更列车编号,普通列车(64号和72号为星期六和假日运行的「丹后悠游号」)。以下为连络的特急列车:","text2":"丹后发现号是谁的暱称?","label":1} {"text1":"杰克·凯鲁亚克(Jack Kerouac,)是一位美国小说家、作家、艺术家与诗人,也是垮掉的一代中最有名的作家之一,与艾伦·金斯堡(Allen Ginsberg)、威廉·柏洛兹(William S. Burroughs)齐名。虽然他的作品相当受到欢迎,但是评论家并没有给予太多喝采。杰克·凯鲁亚克最知名的作品是《在路上》。1922年3月12日,凯鲁亚克出生于马萨诸塞州洛厄尔,父母为法裔美国人,他是家中幼子。他曾在当地天主教和公立学校就读,以橄榄球奖学金入纽约哥伦比亚大学,结识艾伦·金斯堡、威廉·柏洛兹和尼尔·卡萨迪等“垮掉的一代”。凯鲁亚克大学二年级退学从事文学创作,并辗转于美国海军和商用航运公司等处。1950年,第一部小说《乡镇和城市》出版。1957年的《在路上》问世后,他成为“垮掉的一代”的代言人,跻身二十世纪最有争议的著名作家行列。1969年10月20日,凯鲁亚克因肝硬化导致食道静脉曲张大量吐血而接受手术, 在接受手术后凯鲁亚克未再恢复意识, 于10月21日清晨5:15在佛罗里达圣彼得堡去世,享年47岁。《乡镇和城市》、《在路上》、《梦之书》、《达摩流浪者》、《地下人》、《孤独的旅人》和《孤独天使》等","text2":"1969年10月20日,凯鲁亚克因为什么而接受手术?","label":1} {"text1":"《十国春秋》,共114卷,清人吴任臣编撰纪传体取书。《十国春秋》,写十国君主之事迹,采自五代、两宋时的各种杂史、野史、地志、笔记等文献资料,计有吴十四卷,南唐二十卷,前蜀十三卷,后蜀十卷,南汉九卷,楚十卷,吴越十三卷,闽十卷,荆南四卷,北汉五卷,十国纪元表一卷,十国世系表一卷,十国地理表二卷,十国藩镇表一卷,十国百官表一卷。康熙八年(1669年)完成。洪亮吉《北江诗话》卷一载:“吴任臣撰《十国春秋》,搜采极博。”《越缦堂读书记》日记曰:“此书三过阅矣,丙辰(1856)读之尤细,甚薄其体载之疏;至壬申(1872)复阅,始叹其博不可及也。”《四库全书总目提要》评:“任臣以欧阳修作《五代史》,于十国仿《晋书》例为载记,每略而不详,乃采诸霸史、杂史以及小说家言,并证以正史,汇成是书。”吴任臣自序曰:“任臣以孤陋之学,思取十国人物事实而章著之,网罗典籍,爰勒一书,名曰《十国春秋》,为本纪二十,世家二十二,列传千二百八十二。人以国分,事以类属。又为《纪元》、《世系》、《地理》、《藩镇》、《百官》五表,总一百一十四卷。虽世远人湮,书册难考,乃鉴观诸邦,略得而论。……书成,聊著纂述之大指如此。康熙八年(1669年)己酉孟夏,仁和吴任臣撰。”又有周跋:“余校刊吴氏《十国春秋》,附刻《拾遗》、《备攷》二卷,锓板发十方后,复补录数条,亦未印行,年末采摭旧闻,则记载沿有阙。”乾隆五十三年(1788年)四月,周昂重刊《十国春秋》。一至十四卷十五至三十四卷三十五至四十七卷四十八至五十七卷五十八至六十六卷六十七至七十六卷七十七至八十九卷九十至九十九卷一百至一百三卷一百四至一百八卷一百九至一百十四卷","text2":"《十国春秋》中的五表是哪五表?","label":1} {"text1":"甘必大大街(Avenue Gambetta)是巴黎二十区的一条街道,得名于法国政治家莱昂·甘必大(1838年至1882年)。甘必大大街是一条林荫大道,始于奥古斯特梅蒂维埃广场(place Auguste-Métivier,海拔54米),然后向东北方,沿着square Champlain,前往 place Martin-Nadaud.然后折向东,到达甘必大广场(place Gambetta,87米)。然后厉害地折向东北方,经过德农医院(Hôpital Tenon),到达西涅克广场(place Signac,99米)和圣法高广场(Saint-Fargeau,108米)。经过 Tourelles行政中心的后面,法国国外情报局(DGSE)总部,以及佐治谷泳池(piscine Georges-Vallerey) ,结束于丁香门(porte des Lilas,116米)。","text2":"甘必大大街是哪个区的一条街道?","label":1} {"text1":"一个 kb(kilobit,千比特) 表示 1,000 (10) 比特(精确些为 1,024 比特)。“kilobit”这一术语通常以缩略式“kbps”“kb\/s”或“kbit\/s”用于表示数据传输速率(码率),意为“千比特每秒”。例如,“56 kbit\/s 的 公共交换电话网”,“一条 512 kbps 的宽带连接”。注意kilobit(千比特)的缩写为kb,不要与术语kilobyte(千字节,缩写为kB或KB,其中的“B”是大写的)相混淆。尽管它的前缀“kilo-(千)”常和字节(Bytes)连用表示 1,024,但kb 的十进制定义(1kbps=1,000bps)在电信传送速率的相关文字中仍被统一使用。","text2":"“kilobit”用于表示什么?","label":1} {"text1":"幸福豹篮球队(简称「幸福豹」;)是由台湾幸福水泥董事长陈两传于1990年代创建的男子篮球队。该队先参加社会甲组篮球联赛(甲组联赛),赢得冠军后,于1994年与裕隆、宏国、泰瑞等三队合组中华职业篮球联盟(中华职篮),成为职业球队;最初英文队名为「Lucky Leopards」,之后重新包装球队标志,将两个英文单字融合成为自创字「LUCKIPar」。幸福豹在中华职篮四个完整球季均打进季后赛,但始终未能晋级决赛。在中华职篮结束营运后不久,幸福豹于1999年解散,部分本土队员为改参加甲组联赛的前中华职篮球队或其他甲组球队吸收。交大机械BBS站篮球看板转录的职篮六年选秀名单","text2":"幸福豹篮球队在什么时候解散?","label":1} {"text1":"唐绍华(),安徽巢县人,中华民国剧作家、导演、诗人、学者。台湾影剧事业的开拓者之一。民国22年(1933年)国立中央大学毕业。曾任《中央日报》记者,并创办《中国人》旬刊(后改为《良心话》),主编《文化杂志》、《新世纪》及《国是》等刊物。抗战时在重庆编演话剧「碧血黄花」,轰动一时。抗战胜利后,在上海创办「中国第一电影企业公司」、「群星影艺公司」。民国38年任香港新华影业公司制片主任。民国40年到台,曾任台湾影业公司副总经理、嘉禾影业公司董事长,并在台湾艺术专科学校、政工干校、辅仁大学、中国文化大学任教。民国37年由他参与编剧的《花莲港》,是二战后台湾最早拍摄的电影之一。民国39年他导演的「春满人间」,带起了台湾民营乡土片的风潮。唐绍华所导的台语片《廖添丁传》(1956年)亦颇受瞩目。其他","text2":"民国37年唐绍华参与编剧了哪部剧?","label":1} {"text1":"《老王同学会》()是2009年的一部台湾电视剧,故事讲述50岁的电台知名主持人老王和他的三位老朋友所面临的问题。此剧是中国电视公司(中视)与三立电视首次合作的电视剧,版权归三立电视持有。2009年3月6日,中视与三立电视合办《老王同学会》签约媒体茶会,中视总经理饶圣雄、三立电视总经理张荣华与该剧制作人王伟忠共同在象征物《三年中班同学录》上签名。以下时间以当地时间(台湾时间)为准依据〈中国电视公司观众意见反映处理情形公告:98年1月—3月〉,由于《老王同学会》的版权持有者为三立电视,中视已向三立电视购买该剧「无线台播映权」,故能在中视无线台播映该剧;而中华电信MOD收视户必须请求中华电信向三立电视购买该剧「网路播映权」,才可以在中视无线台收看该剧。中华电信及其频道商台湾互动电视公司始终未购买该剧播映权,故中华电信MOD收视户无法在中视无线台收看该剧。由著名的王伟忠制作,这出黑色喜剧围绕著老王的同学们,包含胆怯的老王、傻气的老林、心态年轻的老范与过世的老许。故事开始于老王某日祭拜老许时,遇到他的鬼魂……","text2":"《老王同学会》的故事开端是什么?","label":1} {"text1":"心理咨商是协助人们增进心理健康的一门专业,而心理咨商能发挥效果最关键的因素乃是咨商关系。咨商关系的目标在促进当事人心理健康状态的提升转换,关系的表现是科学也是艺术。科学部份表现在咨商心理师如何于咨商关系中运用专业理论与研究来促进咨商关系的内涵,而艺术则表现在真诚的人际关系直觉与感情互动。咨商关系包括当事人与咨商心理师两个主体,是一种刻意营造的人际关系,不同于其他种类的人际关系,咨商关系的相互性系以当事人的最佳权益为优先的考量。咨商关系的良窳涉及咨商气氛,营造咨商气氛的重要决定因素包括:当事人的人格特质和心理准备。以及咨商心理师的态度和感情。这些咨商气氛是从咨商心理师的立场所提出来的,从当事人的立场来说,也许从头就未注意到这些,但是经由这些特质,当事人得以对自己的冲动或行为、挫折或选择,过去的情境与现在的问题加以坦率的检视。","text2":"咨商关系的目标是什么?","label":1} {"text1":"马克·雅各布斯(,)生于纽约,是位美国时尚设计师。马克是同名品牌以及副牌的领衔设计师。2013年10月2日LVMH集团主席Bernard Arnault证实与LV合作长达十六年的Marc Jacobs10月约满后将离任,专注发展其个人品牌。雅各布斯的祖母在他幼时教导他如何缝制衣物,并成了影响雅各斯一生最大的人。15岁时,雅各布斯到路易威登在纽约的前卫精品店「胡闹」担任采购员,并认识了佩瑞·艾里斯,雅各布斯曾表示艾里斯「让我觉得很酷、给了我很多希望。」1981年,雅各布斯于纽约艺术与设计高级中学毕业、前往帕森设计学院就读,并于1984年获得佩瑞·艾里斯金环奖、切斯特温伯格金环奖、帕森设计学院的年度设计学生等最高荣誉。2007年,雅各布斯被OUT杂志评价为「美国五十大同性恋男女」。同年3月12日,雅各布斯因毒品及酗酒而接受治疗。雅各布斯是公开的同性恋者,现与巴西广告经理罗伦佐·马尔同(Lorenzo Martone)交往。2009年3月,女性时装日报报导两人将在交往一年后订婚。MARC BY MARC JACOBS主要走青春时尚路线,深受时下青少年欢迎。另外,MARC BY MARC JACOBS在香港亦设有特卖场。","text2":"马克·雅各布斯因为什么原因而被接受治疗?","label":1} {"text1":"法医昆虫学(),为应用昆虫及其他自然科学的理论与技术,研究并解决司法实践中有关昆虫问题的一门科学。在自然界中,昆虫不仅取食动物的尸体,还在其尸体中活动,促使大量微生物进入尸体,使之加速崩溃。为法医昆虫学一切工作的基础。由宋朝法医学家宋慈在淳祐七年(1247年)成书的法医学案例著作《洗冤集录》含括了已知最古老的法医昆虫学案件。该谋杀案于端平二年(1235年)发生,一名村民被人用刀刺死。仵作们认为死者的创伤是由镰刀造成的,而镰刀是种在收获季节用于割稻的工具,这一事实使他们怀疑同伴农工涉有重嫌。地方县令将村民都聚集在衙门广场,以便暂时收缴镰刀。几分钟内,一大群苍蝇循著肉眼无法识别的血迹气味而来,围著一把镰刀,对其他的不屑一顾。很显然地,该镰刀的所有者便是罪魁祸首,后来该农工被捕快拿下后请求宽恕。","text2":"地方县令为什么将村民都聚集在衙门广场?","label":1} {"text1":"三司官,亦称法司,是琉球国朝廷的最高执政机构,也是这个机构所有官员的官职名称。琉球共设三司官三人,称号为「某某亲方」。三位三司官分别监督「用意方」(管理国家财产和山川的治理)、「给地方」(管理给与役人俸禄和旅费)、「所带方」(管理租税和国库的出纳)三个物奉行所。三司官的官阶由正一品至从二品,这是琉球士族中最高的官阶,相当于中国和日本的三公或三师。虽然在三司官之上还有摄政,但摄政一般不参与政治活动,因此三司官是琉球国实质上的最高级别官员。三司官由王族、上级士族共约二百余名投票选举产生。王族成员只有选举权,没有被选举权。所有三司官的候选人必须是出生在首里城的上层阶级,拥有较高的道德修养,而且精通文学、伦理、经济、治国的儒家学者。这与中国古代的科举制度有些相像,但三司官的选举没有科举考试那样严格。第二尚氏王朝时期,翁氏、马氏、毛氏池城家和毛氏丰见城家四家三司官辈出,时人将他们与王族的分家向氏并称为「五大姓」、「五大名门」。据《中山世谱》记载,三司官一职早已有之,原称「阿司多部」,至明末,改称「法司」。但三司官拥有较高的政治权力始于第二尚氏王朝时期。1556年,年幼的尚元王登上王位。因尚元王是哑巴,群臣推举德高望重的大臣毛龙唫等三人代理尚元王管理国家。从此以后,三司官成为执政之臣。1571年尚元王死去,但三司官这一统治机构却被保留了下来。1609年萨摩入侵琉球之后,萨摩藩派兵监督琉球国政,琉球王的权力被完全架空。1611年,萨摩与琉球签订了《掟十五条》(),规定三司官一职只能由倾萨摩派人士出任。此后,三司官名义上是独立琉球国的高级官员,实际上已沦为萨摩藩统治琉球的代理人。1879年琉球处分,琉球被日本灭掉,改为冲绳县。三司官的权力被完全剥夺,但明治政府同意琉球国上层阶级维持他们的特权。华族令颁布后,原琉球的三司官都被列入日本华族。","text2":"三司官的官阶有多大?","label":1} {"text1":"《美好的一年》()是一部2006年的浪漫喜剧电影,由罗素·高尔、亚伯特·芬尼及玛莉安·歌迪雅等主演,雷利·史考特执导。本片内容改编自一本2004年的同名小说,作者是彼得‧梅尔()。年轻时的麦斯·史金纳(Max Skinner,费迪·夏摩亚饰)在他叔叔亨利(亚伯特·芬尼饰)位于法国东南方普罗旺斯的葡萄园里过暑假时,从叔叔身上学习该如何体验生活中更美好的事物。麦斯长大后(罗素·高尔饰),在伦敦的证券交易所工作,勤勉且雄心勃勃,赚取金钱的方式游走在犯罪及法律边缘。当他收到他叔叔死讯以及得知他是庄园的唯一继承人后,他飞往普罗旺斯,准备将葡萄园快速卖掉。他到达不久后,政府开始调查他一笔可疑的投资交易,公司高层亦叫他立即回伦敦作解释。机缘巧合下,他未能及时赶回伦敦,最后被公司停职一个礼拜。他只好留在法国,和庄园的酿酒人法兰西斯·杜夫(Francis Duflot),一起为庄园做必要的修复。期间,麦斯发觉有不少人其实不希望庄园被卖掉,包括害怕失去心爱葡萄树的杜夫与当地的一个咖啡店老板芬妮(Fanny Chenal,玛莉安·歌迪雅饰)。在他犹豫不决之际,一个来自加州纳帕县的品酒师克莉斯·劳勃(Christie Roberts)突然出现,并宣称她是亨利的私生女。","text2":"麦斯未能及时赶回伦敦,后来发生了什么?","label":1} {"text1":"沧龙亚科(Mosasaurinae)是有鳞目沧龙科的一科,生存于白垩纪晚期。在1967年,戴尔·罗素(Dale Russell)将沧龙亚科定义为具有以下特征:前上颌骨前方的喙小,或是没有、齿骨与上颌骨具有14颗以上的牙齿。第五、六、七对脑神经穿越耳后的两个孔、枕骨底部或基蝶骨没有基底动脉的血管沟、方骨的上镫骨突末端扩张、至少有31节荐前椎,多为42到45节、荐前的脊椎总长,长于荐后的脊椎总长、后端尾椎的神经棘长,形成尾鳍、四肢关节的表面平顺、距骨与腕骨高度骨化。在1997年,G. L. Jr. Bell出新的沧龙超科的系统发生学研究,仍将沧龙亚科保持为一个演化支的地位,但将倾齿龙族改归类于沧龙亚科,并新建圆齿龙族。沧龙亚科的化石可发现于各大洲,除了澳洲、南美洲以外。沧龙亚科首次出现于土仑阶,并持续生存到马斯垂克阶末期。沧龙亚科的体型差距大,\"Carinodens\"的身长约3到3.5公尺,硬椎龙的体长约7公尺,而霍氏沧龙的身长可达17公尺,是最长的沧龙类之一。许多沧龙亚科是鱼食性动物,或者猎食对象广泛,以鱼类或其他海生爬行动物为食。其中的圆齿龙族演化出适合压碎的牙齿,可咬碎菊石或海生乌龟的硬壳。","text2":"沧龙亚科生存于什么时代?","label":1} {"text1":"阿加德兹大区(Région de Agadez)是尼日尔的一个大区,位于该国北部。东邻查德,西邻阿尔及利亚,北邻利比亚。面积634,209平方公里,是非洲面积最大的行政区,该地区占尼日尔全国面积约52%,但因处于撒哈拉沙漠腹地而人烟稀少,2012年人口487,620人。气候极其干旱,该地没有任何河流和湖泊。首府阿加德兹。下分3县。该地曾是黑非洲和北非交易的通道,在20世纪90年代后期,旅游业成为当地的大产业,又发现了铀矿,这为尼日尔提供了约20%的外汇。但自上世纪70年代起当地就有图阿雷格人的叛乱,反对尼日尔政府.原因是干旱的气候造成多次人道主义危机,铀矿收入并没惠及当地人。近年来基地组织和博科圣地也开始活跃,广阔的撒哈拉沙漠成为各种极端势力盘踞的窝点。阿加德兹大区分为1个市,3个省。","text2":"阿加德兹大区面积有多大?","label":1} {"text1":"张可昀(),艺名小甜甜,出生于台湾彰化县,台湾女艺人。原名「张倞耘」,曾用名「张巧宜」、「张芳奕」(一度过继母系,改称「陈芳奕」),2013年10月由为好友蝴蝶姐姐改名的命理老师,改本名为张可昀,从小被外祖父母和舅父养大,只身到台北后,汪建民和宝妈成为她的干父母,而他们都是「仨个口演艺事业有限公司」旗下艺人。自私立宜宁高中应用外语科(英文组)毕业后,到友松传播上班。在友松传播制作的纬来综合台综艺节目《电视笑话冠军》参赛取得冠军并转任该节目评审,其后则以谐星角色积极的在萤幕前曝光,连2004年当时《神出鬼没》都接下通告,2007年成为《大明星运动会》和《娱乐百分百》的固定来宾,也在蔡康永主持的《两代电力公司》以不计形象的装疯卖傻来吸引观众,2011年与好友蝴蝶姐姐、小优成立「孝女时代」。2012年《球爱天空》女配角。2014年凭《雨后骄阳》入围第49届金钟奖最佳戏剧节目女配角奖。","text2":"张可昀在2007年成为哪两档节目的固定来宾?","label":1} {"text1":"委内瑞拉蚁鸫(学名:'),又名塔奇拉鸫或委内瑞拉蚁八色鸫,是一种神秘的雀。牠们暂时被分类在蚁鸫科内,有待审定。牠们曾只于1955年至1956年间被见到及采集,被怀疑可能已经灭绝。直至2016年,科学家在埃尔塔马国家公园重新发现了委内瑞拉蚁鸫。委内瑞拉蚁鸫约长17厘米。上身呈褐色,冠及颈部呈灰色。展翼有黑色斑纹,喉咙及耳朵呈褐色,颊上有白色斑条。下身呈白色,两侧及胸部有灰色的斑纹。于1955年至1956年间,在委内瑞拉塔奇拉州西南部共采集了4个委内瑞拉蚁鸫标本。此后60年里它们都没有被再次发现,被怀疑可能已经因丧失栖息地而灭绝。2016年,一组考察队于埃尔塔马国家公园重新发现了它们,这也是第一次记录下委内瑞拉蚁鸫的声音和拍摄到它们的活体照片。","text2":"委内瑞拉蚁鸫什么时间被采集过?","label":1} {"text1":"金牛街,曾位于昆明市中心,盘龙江边。早在昆明城建城时就已存在。南起南太桥,北至珠玑街。由于街头立有一尊铜制水牛而得名。因铜制水牛闪闪发光,故也称为金牛。金牛腹中空,脊背上有一大孔。传说金牛是用来镇水只用。虽然这个传说现今已经无从考证,但是由于金牛的特殊结构,并且早前金牛位置位于江水之上,当洪水高速从其腹下流过,便会产生特殊的声学效果,嗡嗡作响。也算是有预测洪水之用。街上主要以居民为主,建筑多为老式院落,基础设置较差。在90年代昆明市旧城改造过程中大量拆除,只留下明代初建立的具有历史意义的金牛街清真寺。如今金牛街已不复存在,取而代之的是沿江边小道扩建而成的金井街,以及南太桥围绕原金牛附近的街心花园。","text2":"金牛街的名字得名于什么?","label":1} {"text1":"大眼魣(学名:),又称大眼金梭鱼,为辐鳍鱼纲鲈形目鲭亚目金梭鱼科的其中一种,俗名吹鱼、细鳞。本鱼分布于印度太平洋区,包括东非、红海、塞席尔群岛、阿曼、马尔地夫、伊朗、巴基斯坦、印度、日本、台湾、中国、越南、柬埔寨、菲律宾、印尼、澳洲、新几内亚、帛琉、库克群岛、马绍尔群岛、马里亚纳群岛、新喀里多尼亚、纽埃、索罗门群岛、东加、吐瓦鲁、斐济、关岛、萨摩亚群岛、夏威夷群岛、法属波里尼西亚等海域。该物种的模式产地在Society群岛。水深0至300公尺。本鱼体延长呈鱼雷状,横切面几近圆柱形,侧线在第一背鳍前略向上弯曲,第一背鳍起点在胸鳍末端正上方,在腹鳍起点之后;鳃盖具2弱平棘;上颔骨末端达或接近眼前缘,下颔骨突出。体浅白,背部蓝绿或棕色。体侧不具角形纹,眼大,鳃弓呈刚毛状及胸鳍下具一大黑斑。背鳍硬棘6枚;背鳍软条9枚;臀鳍硬棘2枚;臀鳍软条9枚,体长可达75公分。本鱼白天常成群在礁湖内礁块的上面水层或在礁坡外徘徊,属肉食性,以鱼类为食。为食用鱼,适合各种烹饪方式。","text2":"大眼魣的体长可达到多少?","label":1} {"text1":"花旗参蜜,一种以为花旗参及蜂蜜调合而成的饮品,可热饮或冷饮。花旗参:具有二十多种「人参皂」的主要活性成份,对中枢神经系统有镇静和新陈代谢的作用,防止抑压性溃疡、增强胃肠蠕动、抗疲劳、影响内分泌系、及加快糖类、脂肪和蛋白的新陈代谢及合成。并能够滋补中气、增强活力、补肺消热、宁神安睡、开胃健脾、 生津、止渴。蜜糖:一种高浓度的糖液,能产生很高热量,成分包括是单糖、果糖和葡萄糖、有机酸、人体必需微量元素及氨基酸等,并富含多种维他命,不但有助身体排毒,更能补虚润肺燥、滋润喉咙。李时珍在《本草纲目》,指蜜糖能清热、补中、解毒、润燥及止痛五大功效。参蜜早已在香港的餐饮业占了一定的地位。在一般的茶餐厅加少许钱已能尝到;在快餐店、酒楼等亦不难找到参蜜的踪影;甚至在部份cafe及自助餐中也有参蜜提供;而因为参蜜有润喉的功能,在近年出现的烧烤场及卡拉OK场所中参蜜是必然之选来的。一般餐厅都会用蜜糖开水再加几片花旗参冲制参蜜,因此,餐厅中提供的大多是热参蜜。部份餐厅会用平价、较少的花旗参,令冲制出来的参蜜出现没有参味、过甜甚至淡而无味等问题。随著时代发展,近年开始出现参蜜与其他饮品的配搭如柠檬参蜜。","text2":"花旗参蜜的饮用方式有什么?","label":1} {"text1":"雁门乡是位于中国四川省阿坝藏族羌族自治州汶川县的一个乡,素有汶川县“东大门”之称,是冉尤大道的重要关口。雁门乡因“负山临江,两岸壁立,中通一线只有鸿雁可以飞越”而得名。乡境东连茂县南新乡,南面与彭州市相连,西邻威州镇和西北的克枯乡。全乡幅员面积144.56平方公里,耕地面积5287亩,人均有地0.85亩,现有农户1305户,人口6197人,2002年全乡工业总产值817万元,人均纯收入702元。雁门乡共有9个行政村,21个村民小组。全乡羌、藏、回、汉等多种民族,其中羌族占97%,是羌民族的聚居地。雁门乡属岷江上游典型的干旱河谷地带,西北低走势,境内村寨最高海拔2080米,最低海拔1350米。雁门乡资源丰富,境内有多种野生动植物及矿产资源。岷江河穿境而过,雁门沟内水量丰富。","text2":"雁门乡的耕地面积有多大?","label":1} {"text1":"归元寺,位于汉阳区翠微路和归元寺路交界,是中国湖北省武汉市的一座著名佛教曹洞宗寺院,武汉的佛教四大丛林(还包括宝通寺、莲溪寺、正觉寺)之一。湖北省佛教协会和武汉市佛教协会的所在地,湖北省文物保护单位。中国国务院首批公布的开展宗教活动的重点寺庙。寺名取佛经“归元性不二,方便有多门”之语意。藏经阁内供奉有释迦牟尼佛像。寺院占地17500多平方米(一说4.67公顷),分为中院、南院、北院三组,有藏经阁、大雄宝殿、五百罗汉堂三组主要建筑,殿舍200余间。现由宝通禅寺隆醒大师担任归元寺方丈。隆醒法师和原主持隆印皆为归元寺老主持昌明大师徒弟,昌明师最后的传法弟子隆智为昌明师晚年所收,俗世修行。乘坐武汉地铁4号线或6号线便可前往归元寺,需在钟家村站、汉阳火车站下车。","text2":"归元寺属于湖北省的哪个城市?","label":1} {"text1":"旧字体是指日本于1946年公布《当用汉字表》以前所惯用的汉字,与《康熙字典》中的字体几乎完全相同,故也称作康熙字体;与现今使用的繁体字也大同小异,当中有些字的笔画甚至比现今使用的繁体字还多。而《当用汉字表》颁布以后所包含的简化的汉字,称为日本新字体。旧字体为日本汉字的传统形式,其简略化形式称为「新字体」,这些字在很久以前就已经存在了,在中国和日本的民间常用。二战以后,中国大陆和日本分别进行汉字简化。1947年「当用汉字」发布以前,旧字体称为「正字」()或「正字体」()。在颁布当用汉字后的1950年代,旧字体仍常用于印刷或书写,原因是打印排版更新迟缓、部分人仍习惯用旧字。日本人名中,日本政府曾发布《人名用汉字许容字体表》来处理人名中的旧字体和异体字,后来该表于2004年合并到人名用汉字后停用。目前日本人名可以使用的旧字体和异体字收录于「户籍法施行规则第六十条第一号」中,其中包括旧字体「」等、异体字「」等。不过也有旧字体未收录于人名用汉字,如「」。现代日本社会用字以新字体为主,不过由于历史、文化上的种种原因,部分汉字的旧字体甚至比新字体(所谓「常用汉字字体」)还常用。例如「」是「」的旧字体,但「」的使用频率占压倒性的多数。与中国大陆不同的是,目前中国大陆的繁体字在绝大多数场合,包括人名、地名用字都需改用简化字,而日本政府并未在颁布新字体后宣布禁止使用旧字体,基于此原则,目前日本的出版物仍然可以见到混用旧字体和新字体的情况。由于日本自二战战败以来从未修宪,1946年公布的《日本国宪法》至今仍是旧字旧假名的(尽管民间常见的是现代常用的新字新假名版)。有些地方的地名也包含旧字体,例如奈良县「」中,「」是「」的旧字体。尽管该市也曾广泛使用「」的表记,但今大多已改正。ACG作品中也常出现旧字体,例如:《》(「」为「」的旧字体)、《圣斗士星矢》主角之一(「」为「」的旧字体)、《十二生肖守护神》反派之一(「」为「」的旧字体,不过「」并未收录于人名用汉字中,故不能用于户籍登记)。然而另一方面,虽然新字体政策并不涉及过往的作品,但实际上由于学校只教授新字体,受现代教育的日本人无法阅读旧字体,或至少读起来有困难,因此出版社出于营利目的,常以便于年轻人阅读为由,将过去的书籍转换成新字新假名出版印刷。","text2":"在颁布当用汉字后的1950年代,为什么旧字体仍常用于印刷或书写?","label":1} {"text1":"虚拟电路(,缩写为 VC),又称为虚电路、虚连接或虚通道,在分组交换的电脑网路上,交换资料的传输方式之一。它是一种预接式(connection-oriented),或线路交换式(circuit-switched)的资料传输方法,在两个终端系统(End system)间,建立一条连线,来进行资料交换。在使用虚拟电路之前,必须先在两个节点或软体应用程式间建立连线。在建立连线之后,两个节点之间,就可以进行资料串流的交换。概念来自于电路交换,其运作方式就如同在两个端点间,建立起专用的实体层线路连线一般,因此又称为虚拟连线(virtual connection)或虚拟通道(virtual channel)。在分组交换中,虚拟电路与资料包是两种主要传输方式。在通信和网络中,虚电路是由分组交换通信所提供的面向连接的通信服务。在两个节点或应用进程之间建立起一个逻辑上的连接或虚电路后,就可以在两个节点之间依次发送每一个分组,接受端收到分组的顺序必然与发送端的发送顺序一致,因此接受端无须负责在收集分组后重新进行排序。虚电路协议向高层协议隐藏了将数据分割成段,包或帧的过程。虚电路通信与电路交换类似,两者都是面向连接的,即数据按照正确的顺序发送,并且在连接建立阶段都需要额外开销。但是,电路交换提供稳定的比特率和延迟时间,而虚电路服务的比特率和延迟时间要取决一下因素:许多虚电路协议通过数据重传,包括检错纠错和自动重传请求(ARQ),提供可靠的通信服务。","text2":"虚拟电路概念来源于什么?","label":1} {"text1":"在数学中,三次互反律是关于模代数中两个对应的三次方程的可解性之间的关系的结论和定理。三次互反律最常使用艾森斯坦整数进行表述。艾森斯坦整数是指由形如 formula_1 的复数组成的环,记作 formula_2。其中 formula_3 和 formula_4 是整数,formula_5 为三次单位根:如果 formula_7 是formula_2中范数为 formula_9 的一个 素数。formula_10 与 formula_7 互素。定义三次剩余符号formula_12 为一个三次单位根,并满足再定义“原初”素数是模formula_14同余于formula_15的素数。由于每个素数在乘以formula_2中的一个单位元后都会成为“原初”素数,因此关于“原初”素数的定律仍具有普遍性。这时,三次互反律说明,对两个不同的“原初”素数 formula_7 和 formula_18,有此外有辅助定理:如果 formula_20 那么:由于因此可以计算任意艾森斯坦整数的三次剩余。","text2":"三次互反律的辅助定理是什么?","label":1} {"text1":"佛罗里达山狮(学名:'),又名佛罗里达美洲狮或美洲狮佛罗里达亚种\"'。过去被认为是美洲狮的一个亚种,近年支持将其合并在北美山狮内(\"Puma concolor couguar\")。栖息在美国佛罗里达州南部的松林、针叶林及沼泽。雄狮重约150磅,栖息在大落羽衫国家自然保护区(Big Cypress National Preserve)、大沼泽地国家公园及美洲狮国家野生动物保护区(Florida Panther National Wildlife Refuge)。牠们是美国东部唯一的美洲狮,只占有以往牠们分布地的5%。牠们现存的数量估计只有80-100只。佛罗里达山狮一直都被认为是美洲狮的亚种,学名为\"Puma concolor coryi\"。于1967年牠们被美国渔业与野生动物局(United States Fish and Wildlife Service)列为濒危,且是一直最为受到保护的猫科群族。根据美洲狮线粒体DNA的研究显示,佛罗里达山狮与很多美洲狮亚种都很相似,建议将牠们合并重新分类为北美山狮(\"Puma concolor couguar\")。自此,一些经典的文献都不将佛罗里达山狮看为独立的亚种,而是与其他亚种合并为北美山狮。不过在研究方面,尤其是有关佛罗里达山狮的保育工作,仍然会将牠们列为独立的亚种。然而重新分类的建议却仍未有公认的做法。在美国佛罗里达州已进行保育工作来挽救余下的佛罗里达山狮。不过,这项工作因佛罗里达山狮所需的生活面积而变得不容易:每一个繁殖单位,即1只雄狮及2-5只雌狮就需要约200平方公里的栖息地。一群240只佛罗里达山狮就需要8000平方公里的栖息地,并足够的遗传多样性来避免近亲繁殖。从德克萨斯州引入的8只雌性美洲狮近亲似乎能成功地减低近亲繁殖的问题。佛罗里达州南部是一个快速发展的地区,天然环境的开发影响著佛罗里达山狮的生存。牠们的死亡率最多是来自交通意外及为地盘而自相残杀,这都是因失去栖息地、栖息地的退化及分裂。在近那不勒斯开发的圣母镇(Ave Maria)有指是牠们的主要栖息地。不过,佛罗里达山狮的分布地却被受争议,这引发影响地产商及环保组织的争议。由于以往美国渔业与野生动物局使用了错误的数据,误会了佛罗里达山狮的分布地,致使批核了一些发展项目。","text2":"佛罗里达山狮又名什么?","label":1} {"text1":"周志华(),台湾配乐家,现任 华亚国际音乐有限公司 音乐总监。台湾澎湖人,从小自学吉他。1981年到台北,在朋友家中见到了甫发明不久的MIDI器材,产生了极大的兴趣。1987年进功学社销售YAMAHA MIDI乐器,1989年度获选为最佳业务员 晚上在钢琴酒吧弹电BASS。1992年转至利达数位影音科技股份有限公司,开始制作广告配乐的职业生涯。上海,台北 声动录音室 特约作曲、日本YAMAHA流行音乐学校MIDI讲师、经济部工业局数位内容学院课程评审,专业广告、戏剧、电玩、动画、流行音乐唱片之作曲、编曲、配乐。作品有1000支电视广告、100部电视剧、6部日本NHK纪录片及数十张唱片。","text2":"周志华现任哪个公司的音乐总监?","label":1} {"text1":"芬兰战争是在1808年2月至1809年9月期间于芬兰地区爆发的一场战争,参战国家为瑞典及俄罗斯。最后瑞典战败,其东部的三成领土被割让,成为附庸于俄罗斯帝国的芬兰大公国。俄罗斯亚历山大与拿破仑签署了《提尔西特条约》后,他建议瑞典国王古斯塔夫四世应加入大陆封锁。与此同时,英国皇家海军袭击了哥本哈根,展开了英俄战争。英国照1780年和1800年的条约,要求古斯塔夫关闭波罗的海所有外国军港。但是瑞典国王回答说,只要法国人控制著波罗的海主要港口之前,他们就不可能达到目的。1808年2月21日,在弗里德里·威廉·冯·布克霍夫登的带领下24 000名俄军士兵越过边界,夺取了塔瓦斯胡什。瑞典对这次袭击毫无准备,直到四月才宣战。约有19000名瑞典军驻扎在芬兰的各个堡垒中,而其余的军队由于担心遭到丹麦的袭击而无法离开瑞典南部。3月,俄罗斯人攻陷了库奥皮奥,鞑靼风琴,雅各布斯塔德,,赫尔辛夫,汉科与哥德兰和奥兰群岛。4月,俄罗斯人受到更多的挫败,来自哥德兰和奥兰群岛的居民支持的瑞典,迫使俄国的维希上校及其驻军投降。 5月26日,英国舰队进入哥德堡,但与瑞典国王的交恶。俄罗斯人获得了大量的增援后,他们的人数增至55,000人,而瑞军则达36,000人。 8月14日,俄军发起新的进攻。俄国的11000强军队于9月1日在库尔丹(9月1日),(9月2日)和(9月14日)取得了更重要的胜利。在芬兰东部,芬兰游击队运动逐渐消失。当时俄罗斯军队已经占领了芬兰全境。 11月19日,签署《弗里德里克斯哈姆条约》,瑞典军队被迫离开芬兰。","text2":"瑞典军队离开芬兰时,签下了什么条约?","label":1} {"text1":"翠池是台湾雪山山脉的高山湖泊,位于雪山主峰西方约一公里,雪山七号圈谷之下,雪霸国家公园境内,地属苗栗县泰安乡,为大安溪之源。翠池所在地海拔高度3520公尺,是台湾高度最高的天然湖泊。翠池在泰雅语中,名为「Siron Hagai」,意为石头水池。翠池池形呈椭圆状,长约40公尺,宽约20公尺,受降雨影响,水量不定;丰水期时池水可深达1公尺,若逢枯水,则只有中央的洼地存有积蓄,但终年不涸,冬季时亦不结冰。池底铺满板岩碎块,周遭则是台湾面积最大的玉山圆柏纯林,林木高大,有别于其他高山地区矮小匍匐的丛灌景观。翠池北方有一处开口,池水聚流而下,是为雪山溪;溪水与大雪溪、马达拉溪等相汇集后,注入大安溪。东北处,有两道往西北方延伸的凸提,各长约50、60公尺,高3公尺,雪山溪穿过两道凸提,形成小峡谷。翠池南方则是碎石坡,为雪山七号圈谷,涵养地下泾流,是翠池水的来源。由于翠池南有圈谷坡,北有凸提,因此推断翠池可能是由冰河作用侵蚀所形成冰斗湖。翠池之上、下各有上翠池、下翠池二池,上翠池为季节性小沼,干季无水。","text2":"翠池在泰雅语中,意思是什么?","label":1} {"text1":"拃是人张开的手从拇指尖到小指(或中指)尖之间的距离,长约五寸。同时拃也是一个动词,表示用这样的方式量长度。英语中类似的说法是 span,是从拇指尖到小指尖之间的距离。在古代西方,1 span 等于 1\/2 腕尺(cubit),参见英制。英制中,1 span在斯拉夫语言中,类似的说法是 piad(复数 \"piędź\",俄语 \"пядь\"),是从拇指尖到食指尖的宽度,参见“俄制单位”。1 piad := 4 vershok在斯瓦希里语中,拃等价的说法是 \"futuri\" 。在印地-乌尔都语以及印度、巴基斯坦其它北方语言中,拃一般用作非正式度量称为 \"bālisht\"(乌尔都:بالشت,印地语:बालिश्त)。","text2":"在印地-乌尔都语以及印度、巴基斯坦其它北方语言中,拃一般用作非正式度量被称为什么?","label":1} {"text1":"管海马(学名:)为辐鳍鱼纲棘背鱼目海龙科海马属的鱼类,俗名库达海马。其繁殖过程独特,交配时雌性将卵子转移到雄性管海马的育儿袋中。受精作用何时进行仍然不明,但经受精后的合子至少会留在受儿袋中四至五星期,直至雄性管海马将它们喷出为止。分布于日本、印度洋、新加坡、菲律宾、夏威夷群岛、澳洲及非洲东部、台湾、香港以及渤海、东海、南海等海域。该物种的模式产地在新加坡。本鱼体延长,口小,吻突出呈管状,头大。鳞片特化成骨板,颈前板上有冠状突起。体色多变,从黄色、褐色、黑色甚至橘红色都有,无尾鳍。本鱼栖息于藻类或海草丛茂盛的海域,适应力强,略能忍受盐度的变化,常以尾部缠绕海草、珊瑚或石块上,游泳能力弱,体色随环境变化。属肉食性,以无脊椎动物为食,繁殖期为春夏季,雌鱼将卵产在雄鱼的孵卵囊中,由雄鱼负责照顾。古希腊和古罗马相信海马是波塞冬或海王星的海神属性,被认为是象征力量和权力。 欧洲人相信海马携带死去水手的灵魂到地府,给他们安全通道和保护,直到他们达到自己的灵魂的目的地。观赏鱼,可做为中药材。","text2":"管海马有什么实用价值?","label":1} {"text1":"《初恋的回忆》()是一部2002年的美国爱情电影,由执导,Karen Janszen编剧。改编自尼可拉斯·史派克的1999年同名小说《\"\"》。主演是夏恩·威斯特、曼蒂·摩儿、和戴露·汉娜。Landon Carter是镇上的一个著名小混混,终日不学术,与一群猪朋狗友胡混。在一次捉弄新来的学生的时候,致使新生受伤,全班人由于巡警的发现而急速离开,只有Landon仍然照顾他,在他要离开时却因汽车出了事,而被巡警当场逮住。然而警方或者任何一方都没有要追究Landon Carter,不过学校却要他一方面做义工,并且演出春季汇演剧,以及打扫园。在他与母亲去做礼拜时,女主角Jamie Suvillian出场。Landon最初对她不理不睬,甚至有点讨厌,不过除了在学校,春季汇演剧、义工处却都能碰见她。而Landon在一次乘搭Jamie的便车时,女主角说了一句“42”令Landon开始感到兴趣。并追问什么是第一。Jamie回答:“I tell you then I have to kill you”。后来他为了演好汇演剧而希望Jamie帮他后,他又因为顾全面子而令Jamie感到失望使得他只可一人练习台词。到正式汇演时,Jamie脱下斗篷之时令到在场诸人包括Landon都为之惊艳。Jamie也都唱出了汇演剧中的歌曲《Only Hope》。歌曲完后,Landon意外地吻了一下Jamie(剧情中并没有)在Jamie的影响下,Landon远离了原来的糜烂生活,甚至与他们翻脸。而父母均反对他们在一起,Landon母亲认为他只是玩玩,而Jamie父亲则认为Landon是朽木不可雕也而拒绝女儿和他交往。然而更深层的原因却在一次Jamie与Landon在夜间拍拖时被揭出:Jamie患上了白血病,并且只有半年命。Jamie希望他远离自己,避免受到伤害,但Landon却更加义无反顾地与她在一起。就在他为她制作一个更大的望远镜时,Jamie因开心激动而晕倒。在Jamie在家休养时期,望远镜制作完成后,Landon向Jamie求婚,而Jamie也都答应。两人最后在Jamie父亲的祝福下成了婚。该剧于曾于当时位列全美票房第三。也是一部十分赚人热泪的作品。评价都倾向于优秀。电影公司也发行了电影原声带。","text2":"《初恋的回忆》改编自哪一本小说?","label":1} {"text1":"《丁丁与字母艺术》(法语: Tintin et l'alph-art ;英语: Tintin and Alph-Art ),又译《丁丁与阿尔发艺术》,是《丁丁历险记》的第24部作品,也是最后一部作品,作者是比利时漫画家埃尔热。故事的主要题材是现代派艺术,埃尔热在1983年执笔这部作品过程中不幸逝世,导致这部作品成为未完成的作品。为了逃避毕安卡的爱情之访,哈达克意外地结认新派黑人艺术家拉莫·纳许(Ramó Nash),并替丁丁约定艺术专家弗卡特(Fourcart)向他「爆料」,然而不久就意外身亡。在几次逃过被枪毙的险境下,丁丁不但揭发弗卡特是被杀,并发现了新兴邪教的教主恩达迪涅·阿卡斯(Endaddine Akass),就是支助纳许的艺术赝品贩售家。不久,毕安卡邀请丁丁及哈达克前往阿卡斯位于意大利的豪宅,阿卡斯却把丁丁囚禁,准备把他用热塑胶活活灌死,造成凯撒扩展式的艺术品...收到米卢求救纸条的哈达克及时救回丁丁,然而他们却无法脱离阿卡斯的魔爪,其后阿卡斯向二人表明自己就是拉普洛斯。庆幸机警的纳许及时报警,并制止拉普洛斯把丁丁及哈达克吊死,然而拉普洛斯却因而失足跌下山坡至死。另外,在埃尔热的原稿出现的艺廊职员Martine Vandezande,郤在Rodier同人版的结尾向丁丁表示好感,引起丁丁迷一点点争议。","text2":"《丁丁与字母艺术》的主要题材是什么?","label":1} {"text1":"琵琶湖特快()是西日本旅客铁道(JR西日本)营运的一个特别急行列车(特急)的名称。列车在东海道本线(琵琶湖线、JR京都线)行驶,来往米原和大阪。琵琶湖特快的前身是1988年开办的专线列车「琵琶湖Liner」()。当时以485系在平日提供服务。2003年起改用新型的681系或683系后,升格为特急班次。2007年3月17日,东日本旅客铁道(JR东日本)和东海旅客铁道(JR东海)营运的东海道本线特急列车「东海」停止服务后,琵琶湖特快成为东海道本线内唯一在日间行驶的特急班次。米原站 - 彦根站 - 近江八幡站 - 野洲站 - 守山站 - 草津站 - 石山站 - 大津站 - 山科站 - 京都站 - 新大阪站 - 大阪站京都列车区","text2":"琵琶湖特快的前身是什么?","label":1} {"text1":"安乐影片有限公司 (),香港电影公司,主要业务为包括电影制作、电影和影碟发行、广告制作,同时经营有12间戏院的百老汇院线,由现时公司执行董事之一江志强的父亲江祖贻于1950年成立,为全港历史最悠久的电影发行公司,过去已发行逾100套电影。目前安乐影片员工有超过200人,年度营业额介乎10,000,000-49,999,999港元之间。安乐影片早已与索尼影业和环球影业签订了香港戏院发行协定,负责把它们的电影发行至全香港各大戏院。另一方面,安乐影片亦有自行买片以作发行。江志强近年来亦参与不少华语片的投资和制作,例如《卧虎藏龙》、《英雄》、《色,戒》和《寒战》等。2012年3月15日,now TV与华谊兄弟、安乐影片及陈可辛我们制作共同筹组的now爆谷台于香港启播。","text2":"安乐影片的年度营业额是多少?","label":1} {"text1":"苏黎世圣彼得教堂是瑞士苏黎世老城的4座主要教堂之一,另外三座是苏黎世大教堂, 苏黎世圣母大教堂和Predigerkirche。教堂毗邻Lindenhof 山,昔日罗马城堡的地点,兴建在朱庇特神庙的遗址上. 8或者9世纪的早期教堂为10 乘 7 米大小,大约1000年前后改建为罗曼式教堂,1230年改为后期罗曼式,1460年重建为哥特式教堂。在瑞士宗教改革之前,圣彼得教堂是该市唯一的本堂区教堂,其余的都附属于修道院。现存建筑奉献于1706年,是新教主宰时期的第一座教堂。修复工作完成于1970-1975年。尖塔的钟面直径8.7米,是欧洲最大的教堂钟面。教堂的尖塔属于苏黎世市,而中殿属于瑞士归正会的圣彼得堂区。","text2":"苏黎世圣彼得教堂的修复工作是什么时候完成的?","label":1} {"text1":"孙揆(),字圣圭。唐朝昭义军节度使。刑部侍郎孙逖五世从孙,进士出身。早年担任户部巡官。历官中书舍人、刑部侍郎、京兆尹。曾监斩一度称帝后来败亡的秦宗权。大顺元年(890年)八月,唐昭宗讨伐河东节度使李克用,任孙揆为副招讨使、昭义军节度使,李存孝率三百骑兵埋伏在长子谷,孙揆一战即溃,被俘虏,押往太原。李克用劝降孙揆,许以河东副节度。孙揆拒绝投降,大骂李克用:“我天子臣也,怎么能事你这个沙陀小儿!”李克用大怒,将他处以锯刑。锯不行,孙揆骂道:「死狗奴,锯人当用板夹,汝岂知邪?」最后刽子手找来木板锯死孙揆。孙揆至死,骂不绝口。赠左仆射。清朝皇帝乾隆曾言:「夫揆将兵趣潞中,路遇伏遭擒,是乃出其不意。至被执不屈,骂贼捐躯,大节卓然可纪。」著有《灵应传》。","text2":"清朝皇帝乾隆是如何评价孙揆的?","label":1} {"text1":"丹拿山邨是原位于香港北角丹拿道的香港房屋协会公共房屋,现已拆卸。丹拿山邨前称「书局街廉租屋宇」,由于当时屋宇建设委员会的北角邨亦是位于书局街,房协避免公众误会,加上政府将该地方命名为丹拿山区,所以房协亦将此屋邨改名为丹拿山邨。丹拿山邨(Tanner Hill Estate)位于北角丹拿道(Tanner Road),由周耀年李礼之建筑师工程师事务所设计。该屋邨共有两座大厦,分别名为黄鹤楼(Yellow Stork House,丹拿道2号)及松树楼(Pine Tree House,丹拿道4号),于1960年及1961年落成,属香港缺水时代兴建的咸水楼。1996年底房协计划拆卸丹拿山邨,当时估计受丹拿山邨重建计划影响的家庭为563个,住户人数共2 260名。房协在港岛区物色到最少五个租住公共屋邨供安置受影响家庭之用,其中包括健康邨第一及第三期、励德邨、明华大厦及渔光邨,及后更有居民被安置到非房协管理的北角邨。此外,房协亦让受影响家庭优先选购住宅发售计划的单位,因此有很多住户在经济能力许可下都购置健康邨第二期的单位。2000年为配合北角站扩建工程兴建通风大楼而拆卸,但拆卸后的土地一直丢空;直到2008年房协才决定将该地重建成三幢高尚住宅式长者房屋,门牌改为丹拿道3号,并于2009年3月24日动工兴建,现为北角丹拿山长者屋-隽悦,预计在2015年初落成。丹拿山邨未拆卸时,可以斜路或石级路直达。斜路可以人车并行,约为三条行车线的阔度,由现时丹拿花园正门右面向上行。石级路则有二百多级,由七姊妹道与书局街交界处上(即现时有路牌上香花径的上山路,不过从前是一条无盖石屎石梯)。沿石梯上,中段设有两个凉亭,而乘凉内亦设有椅子。至后段有分岔路转上宝马山,由于以前宝马山学校区交通不便,曾是众多学生回校回家的必经之地。丹拿山邨松树楼大堂位置,设有一士多名为「松园士多」,除有粮食出售外,更有石油气及火水送货服务,利便当时居民。","text2":"书局街廉租屋宇为什么改名为丹拿山邨?","label":1} {"text1":"苏黎世圣彼得教堂是瑞士苏黎世老城的4座主要教堂之一,另外三座是苏黎世大教堂, 苏黎世圣母大教堂和Predigerkirche。教堂毗邻Lindenhof 山,昔日罗马城堡的地点,兴建在朱庇特神庙的遗址上. 8或者9世纪的早期教堂为10 乘 7 米大小,大约1000年前后改建为罗曼式教堂,1230年改为后期罗曼式,1460年重建为哥特式教堂。在瑞士宗教改革之前,圣彼得教堂是该市唯一的本堂区教堂,其余的都附属于修道院。现存建筑奉献于1706年,是新教主宰时期的第一座教堂。修复工作完成于1970-1975年。尖塔的钟面直径8.7米,是欧洲最大的教堂钟面。教堂的尖塔属于苏黎世市,而中殿属于瑞士归正会的圣彼得堂区。","text2":"苏黎世圣彼得教堂什么时候改建为罗曼式教堂?","label":1} {"text1":"三斑天竺鲷,为辐鳍鱼纲鲈形目鲈亚目天竺鲷科的其中一种,俗名三线天竺鲷、大面侧仔。本鱼分布于西太平洋区,包括可可群岛、泰国、香港、日本、越南、马来西亚、新加坡、澳洲、菲律宾、印尼、巴布亚纽几内亚、马绍尔群岛、马里亚纳群岛、密克罗尼西亚、索罗门群岛、帛琉、东加、萨摩亚群岛等海域。该物种的模式产地在Buro。水深1-34公尺。本鱼体延长而侧扁,眼大,口大略下位。鱼体粉红色或橙色,体具褐色不规则斑纹,被栉鳞,鳞片具深色边缘,侧线明显,尾鳍截形,背鳍硬棘7枚;背鳍软条9枚;臀鳍硬棘2枚; 臀鳍软条8枚,体长可达14.2公分。本鱼栖息于近海珊瑚礁区,白天躲藏洞穴中,夜间出来觅食,属肉食性。繁殖期时,雄鱼具有口孵习性,卵约7日化成仔鱼,由雄鱼吐出,具短暂的仔鱼飘浮期。不具任何经济价值。2.FishBase","text2":"它们会选在栖息在什么地方?","label":1} {"text1":"单角鼻鱼,又称长吻鼻鱼(学名:),俗名剥皮仔、打铁婆、独角倒吊,为辐鳍鱼纲鲈形目刺尾鱼亚目刺尾鱼科鼻鱼属的鱼类。单角鼻鱼分布于印度太平洋海域,包括东非、红海、模里西斯、塞席尔、马尔地夫、留尼旺、马达加斯加、斯里兰卡、安达曼群岛、日本、台湾以及西沙群岛、中沙群岛、舟山群岛等中国沿海、菲律宾、印尼、新几内亚、新喀里多尼亚、澳洲、新几内亚、马里亚纳群岛、马绍尔群岛、密克罗尼西亚、帛琉、索罗门群岛、斐济群岛、万那杜、夏威夷群岛、法属玻里尼西亚、诺鲁、吉里巴斯、吐瓦鲁等海域。水深1至80公尺。本鱼体呈椭圆形而侧扁;成鱼头顶有角状突起,其长度与吻长略同,吻背朝后上方倾斜,直到角突处为止。体色为蓝灰色,腹侧则为黄褐色,尾柄上的骨质板为青黑色。雄鱼除角突外,尾鳍截形,上下叶延长成丝状,上叶较长。背鳍硬棘6枚、背鳍软条27至30枚、臀鳍硬棘2枚、臀鳍软条27至30枚。体长可达70公分。本鱼多半在礁沟,礁坡或有涌浪处成群活动,交配时成对出现,属杂食性,幼鱼以藻类为食,成鱼则以浮游生物为食。大鱼可为观赏用。亦可食用,此鱼不新鲜时,腥味极重,故一捕获,即需去除内脏,尾柄有硬棘,须注意。多煮汤,尤其煮味噌汤,味美。","text2":"单角鼻鱼为什么一捕获,即需去除内脏?","label":1} {"text1":"《热天午后》(Dog Day Afternoon)是美国导演薛尼·卢梅的代表作之一,在1975年首映,由艾尔·帕西诺及约翰·卡佐尔(John Cazale)等所主演。《热天午后》根据1972年真实发生的一单银行抢劫案所改编,并获得奥斯卡奖与金球奖的提名,编剧弗兰克·皮尔森赢得奥斯卡最佳原创剧本奖。香港导演尔冬升亦受本片启发并于1987年改编本片并拍摄电影《人民英雄》。1972年8月22日炎热的下午,纽约市布鲁克林的一家银行遭到桑尼和萨尔两个人的持枪抢劫。由于银行里的大批金钱刚被运走,保险柜里只剩下1100元。不久后,银行被大批警察包围,在该行里工作的几位女性和一位上年纪的负责人穆尔范尼成为劫匪的人质,于是桑尼与负责该案的探长莫雷蒂展开周旋和谈判。桑尼在谈判中要求警方为其准备一辆开往机场的客车,计划乘飞机飞往北非的阿尔及利亚,待其和萨尔上飞机后再将人质释放。尽管桑尼是一个抢劫犯,但是他在银行外面与警方的谈话与大声演讲内容却获得大量围观民众的呐喊支持。桑尼还让警方将其同性伴侣李昂叫来,在两人的电话通话中,得知桑尼之所以要抢劫银行,目的是为李昂筹集变性所需费用。后来FBI探员雪尔顿代替莫雷蒂继续进行谈判,时间逐渐到了夜晚,通过安排,桑尼与自己的前妻安姬通了电话,桑尼与她育有一子一女两个孩子,安姬变胖后桑尼与李昂产生感情。为了迫使桑尼向警方投降,警方还将桑尼的老妈叫来,她劝儿子尽快投降以获得宽大处理,但是桑尼监持原定的逃脱计划不为所动。桑尼在银行里还安排写下了一份遗嘱,记载著对李昂和两个子女的未来照顾计划。最后在警车的护送下,桑尼与萨尔与一众人质乘坐一辆车前往机场。就在桑尼以为目标快要达成之际,司机墨菲探员与雪尔顿探员巧妙合作,导致萨尔被墨菲击毙,桑尼最终被捕,后来判处20年的监禁,李昂也成功变性成为一名女人。该片获得了极大的好评,在烂番茄网站,它获得了97%的新鲜度;知名影评人Vincent Canby评论道“它是Sidney Lumet最精确,最耀眼的关于纽约的电影”,并对全部演员赞扬道“他们都塑造了杰出的人物”(brilliant characterizations);Roger Ebert评价说Sonny是“最有趣的一个现代电影角色”,并给予影片三星半(满分四星)的打分。","text2":"哪一部香港电影是由这部电影改编的?","label":1} {"text1":"槲鸫(学名:\"\")属于鸫科鸫属,是一种常见雀鸟,广泛分布于欧洲、非洲北部、北亚中部、中亚及中国西部的疏林和耕地,其中中国境内仅有新疆西南部、西北部天山分布有\"T. v. bonapartei\"。此物种是由林奈在其1758年著作《自然系统》(\"Systema naturae\")被首次描述,使用了目前的学名。槲鸫的中文名反映了其以槲寄生为食的特点,而学名的种加词\"viscivorus\"的意思也是“食槲寄生者”。2007年的分子生物学研究表明槲鸫与羽色相近的欧歌鸫(\"T. philomelos\")和宝兴歌鸫(\"T. mupinensis\")有较近的亲缘关系,这三种鸫从遍布世界的鸫中分化出形成的分支较早,因此槲鸫与欧洲其他的鸫比如乌鸫(\"T. merula\")亲缘较远。体长平均为27 cm,体型比欧歌鸫大,雄鸟和雌鸟外形相似。虹膜为褐色,喙黑色,基部黄色,背部深灰褐色,胸腹黄或白色,比欧歌鸫浅很多,并密布有黑色斑点,尾羽外侧尖端、翼下及覆羽边缘白色,脚为粉褐色。槲鸫为候鸟,冬天会从北方到南方越冬,迁徙时会形成小型鸟群。杂食动物,以多种昆虫、蚯蚓、蜗牛、蛞蝓和浆果为食,尤喜食槲寄生的果实,故因此而得名。冬季时槲鸫会占据果树,并且拒绝其他种类的鸫前来觅食。生性胆小谨慎,站立时体态正直。树上筑巢,在以草叶、草根和泥土编织,内铺有干草的杯状巢中产卵,一窝4-5枚卵,卵白色,上有灰褐或红褐色斑点。叫声为干涩颤音“zerrrrr”及凄郁的下降笛音,似乌鸫,不如欧歌鸫的鸣声婉转。天气恶劣或在夜间,雄性会在树木、屋顶或高高的栖木上发出响亮婉转的鸣叫,一般是在多雨的早春。","text2":"槲鸫的叫声如何?","label":1} {"text1":"超文本咖啡壶控制协议(英文:Hyper Text Coffee Pot Control Protocol,HTCPCP)是一种被IETF虚构的协议。这个协议在文档\"RFC 2324\"(发布于1998年4月1日)中被定义,是一个恶搞RFC。该协议被设计为一个类似HTTP的协议,可以用于控制、监测和诊断咖啡壶,后来也被拓展到茶壶;该协议有时也可以视作一种早期的物联网实验。尽管互联网国际标准机构经常在4月1日发布一些恶搞之作,他们还是让这个协议看上去像是一个真的、非虚构的协议(比如HTTP)。2014年4月1日发布的恶搞RFC 7168是此协议的扩展,正式支持茶壶。HTCPCP是HTTP协议的扩展。HTCPCP请求通过URI架构codice_1来引用,并且还包含了若干种HTTP请求:这个协议还定义了两种错误答复:","text2":"超文本咖啡壶控制协议具体是由什么定义的?","label":1} {"text1":"深圳体育场,座落于深圳市福田区笔架山下,东邻上步北路,北接泥岗西路,南靠笋岗路,落成于1993年6月,总面积达到24892平方米,总投资1.41亿元人民币,场地包含一个主体育场及1个副场。设有12个主看台,可容纳32500名观众,是一座全飘棚式的体育场,场内拥有设施有会议室、运动员休息室、贵宾厅及新闻发布室等。此场地承办2011年夏季世界大学生运动会的足球比赛。深圳体育场是深圳足球队的主场。体育场内有一个标准的国际足球比赛场地,其草坪面积达7992平方米。另外还有一块面积达16900平方米的标准国际田径比赛场地,这块场地的面积在全中国比较,属于较大面积。深圳体育场一共有4层,每层分别设有运动员、裁判员、其他工作人员和观众的入口,这样不会造成混乱和人员交叉。比赛区内有24个呈环状分布的观众坐席区,另外还有三间贵宾包厢,每间可容纳22人。内有4间运动员更衣室和4间裁判员更衣室,这些更衣室设施齐全,配备有洗手间、沐浴间、更衣处等,另外还有许多办公用地。比赛场地内配备了一部电梯,还配备了许多看台观众疏散通道。在场馆的东西两侧还有两座天桥,分别连接上步路和泥岗路。在一楼有可供观众撤离以及消防车通过的五个疏散通道口。另外场地内还有可供三百人举行会议的会议室、比赛配套用房及设施、运动员休息室和设备操作室。深圳巴士集团股份有限公司运营之路线深圳体育场从2007年正式从事业单位专制为企业,正式成为深圳市国资委全资控股的国有企业","text2":"深圳体育场自2007年起称为哪个单位全资控股的国有企业?","label":1} {"text1":"冒鹤亭(),名广生,字鹤亭,号疚翁,别署疚斋,以字行,中国江苏如皋人,蒙古族。冒氏为元世祖九子脱欢后裔,冒襄裔孙。冒鹤亭生于广州,清朝光绪二十年(1894年)中举人,后从俞樾学习。辛亥革命后任北洋政府财政部顾问。抗战时期任上海太炎文学院教授。1947年任南京国史馆纂修,与赛金花有交往。著有《小三吾亨诗》、《小三吾亨词》、《小三吾亨文甲集》、《后山诗注补笺》、《四声钩沉》、《宋曲章句》、《疚斋词论》等。有子冒孝鲁。1944年5月的《古今文史》半月刊,刊登周黎庵《清乾隆帝的出生》一文中回忆冒鹤亭曾考证出:“乾隆生母李佳氏,盖汉人也。”庄练在《中国历史上最具特色的皇帝》一书中说:“冒鹤亭因为曾在热河都统署中做幕宾之故,得闻热河行宫中所传述之乾隆出生秘事如此,实在大可以发正史之隐讳。”","text2":"冒鹤亭的儿子是谁?","label":1} {"text1":"花卷车站()是一由东日本旅客铁道(JR东日本)所经营的铁路车站,位于日本岩手县花卷市驿前大通()1丁目。花卷是JR东日本所属的主力干线东北本线与地方交通线釜石线的交会车站,也是后者的起点站,管辖上位于JR东日本盛冈支社的范围之内,是所在地花卷市的主车站,设有绿窗口()与旅行服务中心「View Plaza」()等主要设施。2002年时,花卷车站以「作为庄重的城下町,与宫泽贤治、高村光太郎有渊源之地的车站」()的评价,入选东北车站百选。在1972年完全废线之前,原本还存在一个隶属于私人铁路业者花卷电铁的「电铁花卷车站」与花卷车站相邻,两铁路系统所属的月台之间尚有跨线天桥相连。侧式月台1面1线、岛式月台1面2线,合计2面3线的地面车站。两方向月台以跨线天桥连接。此外,站内设有侧线。","text2":"花卷车站有哪些配套设施?","label":1} {"text1":"慕容延钊(),字化龙。太原人。出身将门,其父慕容章官至襄州马步军都校。延钊少年以勇敢干练著称,后汉时为枢密使郭威的部下,广顺元年(951年),补西头供奉官,历官尚食副使、铁骑都虞侯。柴荣即位后,为殿前散指挥使都校,显德五年(958年),以功迁殿前副都指挥使、为当淮南节度使。恭帝即位,改镇宁军节度。宋太祖即位,加殿前都点检。建隆二年(961年),改为山南东道节度、西南面兵马都部署。干德元年(963年)一月,宋太祖决定向荆湘地区发动进攻,以延钊为湖南道行营前军都部署、枢密副使李处耘为都监,以讨张文表,出兵湖南,顺便向高继冲借道荆南。不久宋军灭荆南,渡江南下,向潭州(今湖南长沙)进发。这时张文表已经被湖南周保权的属下杨师璠所杀,周保权拒绝宋军入境湖南。慕容延钊分兵两路,水陆并进,陆路出澧州(今湖南澧县),水师东趋岳州(今湖南岳阳)。一月底,水师占领江陵城,二月底,大破武平军于三江口(今岳阳北),斩首四千余级;李处耘由陆路至澧州,楚军望风而遁。三月,占领岳州(今湖南岳阳),三月,攻克朗州(今湖南常德),斩杀张崇富,将其首级悬挂于闹市高杆之上。当时慕容延钊抱病在身,太祖命他“肩舆即戎事。”李处耘与慕容延钊素不和,延钊的士兵一旦有错,李处耘直接杀伐决断,造成延钊一病不起。北宋干德元年(964年)卒。","text2":"是什么造成延钊一病不起?","label":1} {"text1":"弹鼓(英文:Drum magazine)是一种圆型的供弹具,因为类似鼓而得名,于冲锋枪、步枪或机枪上较为常见。也可亦称弹盘。弹鼓的设计较传统的直排弹匣为复杂,运作时弹药依靠旋转的内部拨弹轮由内至外到达供弹口。弹鼓最大的优势是在无须更换供弹具的情况下直接发射更多弹药,在全自动武器上使用可具有更持续的连射火力。相对地,容纳更多弹药自然有比弹匣更大的重量和体积,弹簧力度亦更为强劲,部份弹鼓装填弹药时也需要专门附件,根据武器口径的分别,弹鼓的内部设计也相对不同。一般常见的弹鼓可分作两种类型,历史悠久的单室型及1980年代出现的双室型,著名的C-Mag就是双室型的代表作,C-Mag是一种左右对称排列的双室型弹鼓,采用塑料制造,两个弹室中间以弹匣适配器来连接,具有一百发弹药容量,比金属制造的单室型弹鼓更轻更紧凑。弹鼓的一个缺点是一旦发生故障,需要较久的时间排除故障,使用上没有弹链来得可靠。","text2":"弹鼓在哪里比较常见?","label":1} {"text1":"服务研习(SERVICE-LEARNING)为一种将「服务」与「学习」相互结合的一门课程,以事先规划的社会服务活动与结构化设计的反思过程,让学生运用课堂所学贡献社区;同时可透过服务的过程中得到启发及省思,学习课堂中学不到的知识与经验,服务与学习二者在课程中具有某种程度的平衡关系,它是一种「从做中学」(Learning by doing)的学习经历,强调服务与学习目标同等重要,对所有服务与被服务的人都能加强其完成目标,以达「互惠」之功效。据文献记载,「服务研习」一词最早是由美国南部地区教育董事会(Southern Regional Educational Board)于一九六七年首先提出,经过美国各级学校的推广发展后,衍伸出许多的解释和做法。而我国比较被广泛接受的定义为:「由学校与社区结合,共同协助学生应用所学知能去服务他人,并且在服务过程中不断的学习成长。所以服务-学习是学校教育的一环」(林胜义,2002)。","text2":"服务与学习跟学校教育有什么关系?","label":1} {"text1":"《中山世谱》是琉球国的汉文官修史书。康熙三十六年(1697年),摄政尚弘才等人奉尚贞王之命,以蔡铎为中心,对《中山世鉴》进行汉译和改订,于1701年完成初版《中山世谱》(被称为蔡铎本《中山世谱》),献给清朝康熙帝。蔡铎本《中山世谱》记载了琉球国自天孙王朝至尚益王代发生的事情。共正卷5卷、附卷1卷。雍正二年(1724年),由摄政尚彻主持,以蔡温为中心,对蔡铎本《中山世谱》进行加笔、修正,于翌年完成第二个版本的《中山世谱》,后世称之为蔡温本《中山世谱》。现存的《中山世谱》共有本卷14卷(主要是收录有关中国的事项)、附卷7卷(主要是收录有关萨摩藩的事项),蔡温本《中山世谱》的改订只到本卷九卷,其后是由史官继续修订,直到1876年才被迫停止。附卷的部分可能也是由蔡温所编修,后来又经唐荣都通事郑秉哲之手改订,成为现在的7卷。","text2":"谁对《中山世鉴》进行汉译和改订?","label":1} {"text1":"韩通(),字仲达,并州太原(今属山西太原)人。后周军事将领。曾祖韩莹,曾祖母京兆郡第五氏,封汧国夫人。祖赠太尉,祖母清河郡太君张氏,封卫国夫人。父亲韩章,赠太子太师,母谯郡太夫人李氏,封陈国太夫人。少年应募从军,勇于作战,被提拔为骑军队长。后晋开福元年(947年),投靠刘知远,讨杜重威有功。干祐元年(948年),随郭威出兵,身上有六处创伤,被提拔为都虞侯,担任了天雄军马步都校。韩通性情刚直,人称“韩瞪眼”。显德七年(960年)正月,殿前都点检赵匡胤在陈桥发动兵变,黄袍加身之后回军开封。韩通抵抗,被军校王彦升杀死,全家皆被屠尽。韩通是陈桥兵变中唯一试图抵抗而被杀的后周重臣。宋太祖即位后,追赠为中书令。虽然宋太祖赠韩通中书令,但是宋史周三臣传记述,后来太祖到开宝寺,见到韩通与其儿子画像,下令除去该画。故有说法认为宋太祖早就忌讳韩通。前夫人董氏,封陇西郡夫人,有子二人;继室蒋氏,卫国夫人,生一子。有女二人:","text2":"宋太祖即位后追赠韩通了什么?","label":1} {"text1":"基隆厅,原淡水厅鸡笼所属,为台湾清治时期暨台湾日治时期行政区划。1876年1月16日(清光绪元年12月20日)沈葆桢奏请新设台北府,设立时将淡水厅分割成淡水县、基隆厅与新竹县而来,隶属于台北府之下,此行政区划名称持续至1895年4月17日《马关条约》签定,清朝将台湾割让予日本,台湾总督府将「台北府基隆厅」改设「台北县基隆支厅」为止,所辖范围约为今基隆市及基隆北海岸以及新北市的汐止区等区域。日本明治三十四年(1901年),台湾总督府再设基隆厅,明治四十二年(1909年)则再并入台北厅内。清光绪元年12月20日(1876年1月16日),奏准设基隆厅,置通判,兼理通商煤务。光绪十三年(1887年)九月八日,奏准将东北四堡,即基隆堡、石碇堡、金包里堡、三貂堡拨归基隆厅管辖。以「南港口」为淡水县、基隆厅界,并以草岭为基隆厅、宜兰县交界。光绪十四年(1888年),四堡之地及附属岛屿正式归基隆厅管辖。明治三十四年(1901年),台湾总督府以办务署负责地方政务,而县及厅介于总督府与办务署之间,造成行政事务上的欠缺灵活 ,而废止「三县四厅」,设「二十厅」之行政单位,将台北县属基隆支厅复成立基隆厅,管辖基隆、金包里、三貂、石碇等四堡及文山堡部分地区,划分为厅直隶及金包里、水返脚、顶双溪支厅。明治42年(1909年)10月,台湾总督府将原有之二十厅,废止合为十二厅,合并台北、基隆二厅及深坑厅之一部分为台北厅。","text2":"基隆厅原属哪个厅?","label":1} {"text1":"闵浦二桥,即沪闵路-沪杭公路越江工程,原名西渡大桥或奉浦二桥,是上海市的一座公路、轨道交通两用双层桥梁,为独塔双索斜拉桥形式;由上海市城市建设设计研究院所设计,武汉中铁大桥局集团承建。北接沪闵路,南连沪杭公路,是黄浦江上第九座大桥。上层(公路层)已于2010年5月21日通车。该桥北岸起自闵行区东川路以北,沿沪杭公路到奉贤区西渡镇西闸路以南落地,全长约5.8公里,距奉浦大桥约1.7公里。主桥全长436.55米,主跨长251.4米,轨道交通与公路叠合段长度为3.2公里。上层为双向四车道二级公路,宽度18米;下层为轨道交通5号线南延伸段,为电气化复线城市轨道交通。主桥设计为按300年一遇防洪标准,最高通航水位4.41米,最大通行3000吨级杂货船设计。","text2":"闵浦二桥的主桥全长是多少米?","label":1} {"text1":"张孝骞(,),字慎斋,湖南省善化县南门外马厂巷人,内科医学家及医学教育家。幼年在私塾学习《四书》、《五经》及古文,1914年以第1名毕业于长沙长郡中学,并以第1名的成绩被录取进入湘雅医学院。1921年复以第1名毕业,并获得金牌及美国康涅狄格州政府授予的医学博士学位。毕业后留校专攻内科,于1924年1月到北京协和医学院(今中国协和医科大学)深造1年,随后正式留任于该校,陆续担任住院医师、总住院医师。1926年9月被选送到美国约翰·霍普金斯大学进行研究,1927年7月回国续任,1930年在协和医学院组建消化专业组,1932年晋升为副教授。1933年12月再次赴美国史丹福大学研究,1934年7月回国后,担任协和内科消化专业组主管。1937年芦沟桥事变后,他放弃协和医学院的研究工作,回长沙接任湘雅医学院院长职务。1938年夏季,战火逼近长沙,他不顾美国雅礼会的反对,迁校到贵阳,再于1944年迁到重庆。抗战胜利后,于1946年8月迁校回湖南。随后应美国国务院的邀请赴美国考察医学教育和讲学。1948年4月辞去院长职务,并于9回到北京协和医学院,担任内科学教授和内科主任。中华人民共和国成立后,他续任原职。1962年9月被任命为协和医学院副校长。1966年开始的文化大革命期间,他被打成「反动学术权威」、「特务」,备受侮辱和折磨。1978年以后担任中国医学科学院副院长。1987年8月8日因肺癌病逝于北京协和医院,享年90岁。张孝骞毕生致力于临床医学、医学科学研究和医学教育工作,对人体血容量、胃分泌功能、消化系溃疡、腹腔结核、阿米巴痢疾和溃疡性结肠炎等有较深入的研究,被喻为内科专家、医学教育家、中国消化病学的奠基人。。1948年获选为第一届中央研究院院士,1955年被推选为中国科学院生物学部委员(院士)。","text2":"张孝骞在业内被冠以什么样的美誉?","label":1} {"text1":"宗亲会是联络同姓宗族的组织,宗亲会是在中国传统伦理思想下衍生出的组织。源于中国人自古以来有慎终追远;饮水思源的传统礼教,重视家庭崇拜祖先观念。现今宗亲会组织常见于华人世界暨汉字文化圈,在日本称籓族在琉球多称之为门中会。当一个宗族世代繁衍,子孙往往散居各地,为了保持宗族的完整和团结,就成立宗亲会。宗亲会通常以同姓一族为组成团体,隐含彼此之间有著共同祖先血缘关系。在台湾及海外类似团体也有称之祭祀公业、祀产公业、祖公会、丁仔会等不同的组织称呼。名称虽不相同,但其主要的功能如:调解、互助、团结、联谊、承续维持宗族壮大之传统观念是相同的。而宗亲会除了平时的联络外,还有族谱的编制,宗族祖先的祭祀,所以亦可说宗亲会的设立就是为了维系伦理而产生的。","text2":"宗亲会起源于什么文化?","label":1} {"text1":"林凤镇是中国四川省安岳县东部、四川盆地腹心的一个镇级行政单位,位于县境东部,距县城26公里。林凤镇是中国四川省安岳县东部、四川盆地腹心的一个镇级行政单位,位于县境东部,距县城26公里。位于北纬29°57'12-30°03'26、东经105°32'11-105°36'27之间。林凤镇地表地质构造均属侏罗纪沙溪庙组,表现为石英砂岩夹泥页岩,曾有恐龙化石出土。岩层倾角小,近似水平,多表现为方山丘陵。岩层非常稳定,无断裂、无地震历史。镇总体由南向北倾斜,大部分属于中浅丘地带,间有平坝。北部以平坝浅丘为主,南部以平坝及中丘为主。其中,西南的长林村、东部的独林村有相对高度超过100m的高丘地带。海拔一般在280-350m之间,最低点海拔为262m(石羊河出界处),最高点海拔401m(长林村困牛寺坡)。石羊河由南向北纵贯乡境,另有数条支流汇入。林凤镇气候温和,年平均气温17.2℃,7月平均气温为27.8℃,1月平均气温为6.4℃,极端最高气温41.3℃,极端最低气温-5.6℃,无霜期达300天。降水丰沛,年降水量约1,000mm。属亚热带常绿阔叶林,森林覆盖率在30%以上。1941年置麟凤乡,1958年改林凤公社,1984年置乡,1992年建镇。面积47.49平方公里,辖其林、独林、石滚、新坝、长林、大坡、天池、松林、横山、桥堰、山湾、玉带、学田、大月、华阳、青山、南泥、育才、七龙19个村委会和麟凤街居委会。人口3.05万(2007年)。农业主产水稻、小麦,兼产油菜子、棉花,养殖生猪。柠檬种植是全镇的特色支柱产业,种植面积已经超过一万亩。系长江上游防护林工程带乡镇之一,蚕桑业发展也很迅速。工业以家具产业为主,乡镇企业有建材、汽车修理等厂。大安公路穿境而过,另有林高路。村村通有公路。庵堂寺、刘家山林、天池山天池寺、唐宋摩崖造像","text2":"林凤镇的交通如何?","label":1} {"text1":"上海科学技术出版社位于上海市钦州南路71号,始建于1955年12月初,现设有科学编辑部、工业编辑部、农业编辑部、医学编辑部、科普编辑部、科教编辑部、国际部、合作出版编辑室、声像部等二十几个编辑部、室。建社至今,已出版各类图书11000种,累计印数达5.7亿册。该社于1955年12月初由中国科学图书公司编审处、大东书局、大中国图片出版社、新亚书店等单位联合建成,之后兼并了交流无线电出版社、北京书店上海分店、电世界出版社、世界舆地社、永祥印书馆编辑部、电工图书出版社和上海机电图书出版社,初名科学技术出版社。1957年4月,上海科学普及出版社并入;8月,又与上海卫生出版社合并,改名为“上海科技卫生出版社”,1959年3月,恢复现名。文革时期该社一度被停摆,后并入上海人民出版社。1978年1月恢复建制。","text2":"上海科学技术出版社一度兼并了哪些出版社?","label":1} {"text1":"金仁问(),新罗武烈王之子,新罗文武王的弟弟。651年,金仁问23岁时,其父将他派到中国唐朝,并得到唐高宗的信任和赏识。653年,武烈王通过金仁问取得唐朝联合攻打百济的支持。金仁问与唐朝和新罗的联军一起灭了百济。在此之后的大多时间,金仁问都留在长安,并一直都是新罗与唐朝之间的协调人。674年,在新罗统一朝鲜半岛前夕,新罗与唐朝的战争升级。唐高宗扶植金仁问为新罗君主以对付反唐的新罗文武王。在金仁问去新罗的途中遇到前来道歉的新罗来使。金仁问随后被召回到长安。据《三国史记》记载,金仁问694年病故于中国洛阳。唐朝在他死后将其遗体送回新罗。金仁问在新罗被受谥号“太大角干”,葬于新罗首都庆州的西部。","text2":"武陵王攻打百济期间,金仁问起了什么作用?","label":1} {"text1":"PCX是一种由美国乔治亚州的ZSoft公司所开发的影像档格式,原本是该公司的PC Paintbrush软体的档案格式(PCX代表PC Paintbrush Exchange),却成了最广泛接受的DOS影像标准之一,然而这种使用格式已经被其他更复杂的影像格式如GIF、JPEG、PNG渐渐取代。PCX是一种不受平台限制的像素影像格式,档案标头储存著有关显示硬体方面的资讯(萤幕解析度、颜色深度、色盘资讯、位元平面(bit planes)等),有别于后面夹带的实际档案资讯。这个资讯能让影像可以正确地传送与显示在不同硬体上的电脑系统。PCX档案一般来说,储存色盘索引的影像,范围从2、4种色彩至16与256种色彩,而且格式同样可扩充以记录真实色彩(24-bit)的影像。PCX使用区段长度编码(Run Length Encode;缩写RLE),是一种简单的演算法,可以把代表同一个颜色的多个连续位元组,简化成一个双位元组。当档案做处理时,最高位元会用来当作控制切换(旗标),用来决定所给的资料是代表一个像素中的色盘索引值、色彩数值,还是一个用来表示数个接连相同数值像素的RLE数值。由于使用两个最高位元来当作旗标,在一个具有256色盘的PCX档案里,色彩索引编号192至255(其两个最高位元都已设为1),则必须储存成RLE双位元组,即使它们在区段中只出现过一两次。而色彩索引编号0至191就可以直接储存或是存成RLE双位元组(无论哪一种较省空间)。因此实际的压缩率可以透过适当的排序色盘顺序来达到最佳化(然而若与其他影像共用色盘时则可能无法实行)。举例来说,一个色盘可以将最常出现的色彩放置于色盘索引编号的0至191的位置,而最少使用的色彩则可座落于色盘的后半部,这个RLE机制可以有不错的应用。RLE压缩演算法仅需少量的处理器资源与记忆体即可应用,以当时的电脑系统来看是非常有用的,但现今电脑的硬体与显示设备逐渐的复杂精密,RLE演算法反而变成没那么省空间,它无法压缩混色(dithered)的影像例如相片或复杂的电脑图像。","text2":"PCX代表什么?","label":1} {"text1":"王明渠(),山东省海阳县人,宫宝田之徒,八卦掌传人,并将此拳传至台湾。王明渠17岁时拜入宫宝田先生门下,学习八卦掌,在三年之间,与宫宝田朝夕相处、日夜苦练,因此得到真传。他擅长虎头钩,在宫宝田门下学习时,年纪较轻,其师兄孙富英、于友三、侯天郭与宫宝田之女等人都是一身武艺,见他练功认真,也常对他加以指导。后因国共内战,1949年跟随国民党政府军队来到台湾,定居高雄。退伍后,先在林园地区牧羊,经营皮货、茶馆,后因慕名而来学拳的弟子渐多,于是开始在高雄授拳。因他在高雄寿山(又称柴山)上教授八卦掌,所以此派又被称为寿山八卦掌。传有硬手罗汉拳、疯魔罗汉拳、伏虎罗汉拳、疯魔对拳等。弟子有金春竹、林清介、颜义宗、陈逸民、李青吉、郭朝顺等人","text2":"王明渠17岁拜入了谁的名下学习八卦掌?","label":1} {"text1":"罗金汉侯爵(Marquess of Rockingham)属于大不列颠贵族爵位,创设于1746年4月19日,授予托马斯·沃森-文特沃斯,第六代罗金汉男爵。沃森-文特沃斯家族发源于北安普顿郡的罗金汉堡,先祖刘易斯·沃森爵士(Sir Lewis Watson)率先在1645年1月29日获封世袭男爵,是为第一代罗金汉男爵。在1714年10月19日,第三代罗金汉男爵获晋封罗金汉伯爵(Earl of Rockingham)、桑德斯子爵(Viscount Sondes)及霍尔利男爵(Baron Throwley)。除罗金汉男爵爵位外,以上所有头衔在第三代罗金汉伯爵逝世后而断绝,至于该男爵爵位则由第三代罗金汉伯爵的堂弟继承。此一堂弟本名托马斯·沃森-文特沃斯,他在1746年2月26日获继承世袭男爵爵位,成为第六代罗金汉男爵以前,本身早已在1728年5月28日获封莫尔顿男爵(Baron Malton),后又于1733年11月19日获封莫尔顿伯爵(Earl of Malton)、海恩子爵(Viscount Higham)、哈洛登男爵(Baron Harrowden)及沃夫男爵(Baron Wath)。此外,他在1695年从舅公第二代斯特拉福伯爵继承大笔遗产,家族姓氏遂由「文特沃斯」改成「沃森-文特沃斯」。他后于1746年4月19日进一步获封罗金汉侯爵。第一代罗金汉侯爵在1750年12月14日死后,侯爵爵位由他唯一仍在世的儿子继承。第二代罗金汉侯爵在继承父亲爵位前,早已在1750年9月17日获封莫尔顿伯爵(Earl Malton)及莫尔顿男爵(Baron Malton),但此爵位属于爱尔兰贵族爵位。第二代罗金汉侯爵曾两度出任英国首相,他在1782年7月1日逝世后,所有贵族爵位因绝嗣而断绝,而他的遗产后来由外甥第四代菲茨威廉伯爵继承。","text2":"第几代罗金汉侯爵曾两度出任英国首相?","label":1} {"text1":"文洁若(),汉族,在北京出生,祖籍贵州贵阳,从事文学翻译。从小在日本接受日语教育,回到中国后在清华大学外国语言文学系完成本科学业,专业是英语。文洁若一生翻译出版了14部长篇小说,18部中篇小说和100多篇短篇小说,校订了150余部外国文学名著。1927年,文洁若出生在北京,籍贯贵州省贵阳市。其父亲文宗淑是中华民国驻日本横滨总领事。6岁时,文洁若被父亲接到日本东京。在东京时,父亲文宗淑给她买了一套日文《世界小学读本》,并且对她说:“要是你刻苦用功搞翻译,以后在书上印上自己的名字,该有多好”。2年后,文洁若回到北京,在圣心学校和辅仁女中读书,父亲给其一本日文版《岩波文库》的《尤利西斯》,并且指着书说:“要是以后你能翻译这本书,才算有出息”,因为该书夹杂德文、法文、西班牙文、希腊文、梵文等多种文字。抗日战争胜利之后,文洁若考入清华大学,在外国语言文学系学英文。1990年8月,译林出版社社长李景端找到文洁若和萧干,请求他们翻译《尤利西斯》,萧干认为自己80岁开外了,拒绝了李景端的请求,但是文洁若“(可我却)擅自答应了下来”。为了翻译这本“天书”,译林出版社社长李景端此前曾找遍中国大陆所以英语界的一流学者,均遭到婉拒,作家叶君健说:“中国只有钱锺书能译《尤利西斯》,因为汉字不够用,他能边译边造汉字”,李景端找钱锺书,钱锺书却回答:“八十衰翁,若译此书,无异于别开生面的自杀”。1990年到1994年,文洁若和丈夫萧干翻译《尤利西斯》,为了达到翻译界的“信、雅、达”译书标准,两人做了数万张卡片,同时向国内外发出大量的请教信。经过起早贪黑的四年,这对七十岁和八十岁的老夫妻终于翻译成该书。参与编辑的译稿有周作人译的《古事记》、《枕草子》、《狂言选》、《浮世澡堂》、《浮世理发馆》、《平家物语》(未完成译者便去世),钱稻孙译的《近松门左卫门作品选》、《井原西鹤作品选》,丰子恺译的《源氏物语》(周作人、钱稻孙校订)等。译作有芥川龙之介短篇小说集《海市蜃楼·橘子》、三岛由纪夫长篇小说《天人五衰》等。英语文学译作有爱尔兰詹姆斯·乔伊斯的《尤利西斯》、新西兰凯瑟琳·曼斯菲尔德的短篇小说等(与爱人萧干合作)。《尤利西斯》、《海市蜃楼·橘子》已在台湾出版繁体字版。","text2":"文洁若主要从事什么工作?","label":1} {"text1":"马维祺(),河北定兴人,董海川之徒,八卦掌代表人物之一。马维祺家中在北京城东便门开设维记德煤炭厂,绰号「煤马」。同治年间,董海川因年老,离开肃王府,曾经住在马维祺家中三年,这段时间,马维祺专心跟随董海川练功,功力精进,因而以武艺闻名北京。他擅长风轮掌、点腿及翻背捶,有「铁腿煤马」之称,与尹福、程廷华、梁斋文齐名,被认为是董海川门下四大弟子之一。经尹福推荐,曾在佛音尼布家担任护院。他性情暴烈,好与人斗,与师兄尹福的徒弟马贵,被人合称为「二马」。每日清早都会到天坛一带的武馆与人比试,也曾远赴河北、河南、山东一带向人挑战,与不少人结下梁子。他年纪很轻(29岁)就过世了,在家中抱著孙女清晨散步时,突然吐血而亡,死因众说纷纭。传说因为他犯了武林规矩,一位董海川不知名的秘传弟子上门执行家法,将他打死。也有传说他是被人下毒而亡。马维祺并没有收弟子,他的风轮掌只有传给尹福弟子刘栋臣的弟弟刘彩臣。马维祺死后,刘彩臣拜程廷华为师,因此程廷华所传的柔掌中,加入了风轮掌的元素与腿法。","text2":"马维祺与谁被人合称为「二马」?","label":1} {"text1":"《科学战队炸药人》(原题:)是日本东映公司在1983年制作的「超级战队系列」第7部特摄作品。在日播放期间为1983年2月5日-1984年1月28日,全51集。远古时代,坠落于地球上的陨石中残存的生命物质,在地底下经过长年的进化成了有尾人一族-邪进化帝国。因尾巴的数量代表其身分的象征的特有文化,将利用它们超乎人类想像的科学力征服地上的人类,以让日本各地火山喷发为第一步,开始了侵略地上世界的行动。然而,及早察觉到这件事的科学家·梦野久太郎,集合了五名智慧与体力都相当优秀的年轻科学家到自己所营运的发明中心来,也正式宣布科学战队的成军。五人著装了梦野博士开发的战斗强化服后,成了炸药人并衔命全力阻止邪进化帝国侵略的邪恶野心。第9回后的播出时间皆缩短至25分钟。梦野久太郎于远古时代坠落到地球上的陨石中生命物质,长久以来寄居于地底进行著自主进化,并成为了能自由操纵火山的地底支配者「有尾人一族」们的帝国。属于卵生,故于蛋中出生,会将尾数加于名字,且尾数越多代表在帝国中越高。十分藐视没有尾巴的人类(地上人),并视为「下等生物」,在无法透过实验让人类长出尾巴后,便乘著有如怪物一般外观的巨大攻击要塞格兰吉兹摩,向地表上的人类展开侵略行动。亚顿卡亚洁诺比亚梅基度奇梅拉进化兽机械进化兽大爆炸光束小尾兵格兰基兹摩基兹摩斯奇托大木星 DyJupiter炸药马赫 Dyna Mach炸药战车 Dyna Mobile炸药拖车 Dyna Garry炸药猎鹰 Dyna Falcon炸药吉普车 Dyna Machine第一话登场。当炸药红喊出『合体! Grand Slam!』后,由炸药马赫、炸药战车、炸药拖车三体合而为一的巨大机器人。由梦野博士开发制造。于剧场版『199英雄大决战』中登场。武装招式必杀技以下时间以当地时间(日本时间)为准本作品的战队服装曾经被台湾自制特摄节目《太空战士》作为参考。片头曲片尾曲插曲","text2":"《科学战队炸药人》是日本东映公司哪一年制作的作品?","label":1} {"text1":"丁卡人(Dinka people),是南苏丹的一个民族,根据2008年苏丹人口调查,丁卡总人口约450万。丁卡人是非洲其中一个最古老的民族。在加札尔河州、上尼罗州以及白尼罗河流域的广袤地域上,分布着大大小小二十多个丁卡部落,每个部落的聚居虽然彼此毗邻,但是受到自然环境的阻隔而分隔开来。这些部落往下又分为几“分支部落”(或称之为“子部落”),分支部落之下又可细分为由几个家庭组成临时或长久一起放牧的“群体”(或称之为“小部落”),他们各自占据着可以为们的牛群提供丰沛的水源和牧场。而这些居住在一个部落或者之下子部落甚至小部落的人来说,他们之间不都存在血缘关系。氏族是丁卡人眼中最大的血亲戚的联合体,几个氏族之间会承认在父系上有关系。一个氏族之下分为几个宗,这种宗族是源自于一个共同祖先的不同的继嗣分支,可将这些分支暂称大宗。把最大宗族之下的分支称为小宗,把小宗之下的分称为门中,把门中之下的分支称为房。房就是当个人被问及他是哪个宗族的时候,他所提及的那个。这样看来,宗族就是组或已去世、或仍在世的男性亲属,在他们之间,可以从谱系上推溯亲属关系。丁卡族的孩子,从小就被教育要以自己的氏族为荣。只要一个丁卡的小男该会说话了,他的家人就会教给他不断重复父亲一方祖先的名字。所有的孩子都能往上说出十几代祖先的名字。一个孩子的全名——同时也是对他身份的一个完整表述——就是他的祖先们,即所有声称是氏族创始人后裔的全部名字的组合。由于有这个风俗,丁卡人才能同朋友和邻居追宗论祖。每个氏族都有其明显的特征,如尊敬各自不尽相同的图腾(帕蒂普氏族的图腾是狐狸,帕蒂库尔氏族的图腾是狮子)和祭拜各自先祖的神灵,以及在成人仪,给受礼的男孩的脸颊纹刻上象征着本部落或本氏族的纹面图案。尽管存在些细微方面的差异,但所有的丁卡人在文化上是同种同源的,他们信奉唯一个至上神、造物主——‘Nhialic’。","text2":"居住在一个部落或者之下子部落甚至小部落的人之间全都存在血缘关系吗?","label":1} {"text1":"《决战玄武门》()是香港电视广播有限公司拍摄制作的古装武打剧集,全剧共20集,监制王天林。此剧于1985年10月下午,1988年4月及1990年10月中午在翡翠台重播。以下所有影碟发行均由电视广播(国际)有限公司授权:香港发行代理商现代音像(国际)有限公司于2001年推出发行了《决战玄武门》VCD影碟零售版本,此影碟将已播放的集数并制作成12只碟作为片段完整及无删剪版本,每只碟跟电视剧的每集片长约60分钟一样,设有粤语及国语发音版本并配上繁体中文字幕。台湾发行代理商弘音多媒体科技股份有限公司于2007年推出发行了《决战玄武门》DVD影碟零售版本,此影碟烧录VCD香港版的12只碟作为集数,设有粤语及国语发音版本并配上非隐藏繁体中文字幕。","text2":"《决战玄武门》电视剧版每一集大约有多少分钟?","label":1} {"text1":"李清楠(),台湾省彰化县人,柔道八段,合气道八段。他将合气道引进台湾并加以推广,其门下称他为大老师。他也是合气太乙拳的创始者。曾任中华民国合气道协会理事长、财团法人中华合气道总部道馆董事长、亚洲合气道总会会长、国际合气道联盟总会副会长、中华民国合气道协会永久会长。李清楠台湾省彰化县人,在中学时代开始学习柔道,毕业于台湾大学法学院。1959年,代表中华民国参加第三届亚洲运动会柔道比赛获得金牌。在日本东京参加比赛时,接触到合气道,对于它的护身法深为吸引,回国后即致力于合气道的学习及推广。经过友人介绍,认识了日本大阪合气会的富田诚治老师,并邀请他来台教授合气道。经日本大阪合气会会长田中万川师范的同意,田中万川的弟子富田诚治来到台湾,与李清楠在台北巿太平国小成立台湾第一个合气道道场。李清楠号召自己从前学习柔道的学生们,如游能松、李练达、黄显谟、叶武光、曾现财等人,一同加入合气道的学习,合气道因此得以在台湾推广扎根。民国62年(1973年)在台北市成立中华民国合气道协会,成为当时台湾合气道惟一的领导团体。民国80年〈1991年〉之后,合气道协会中许多人士为了追求理想,继续推展及维护道统,遂成立中华民国合气道推广训练协进会。民国85年〈1996年〉,他总结了一生习武的心得,发表了合气太乙拳,又名合气道球转入身转换术。民国97年(2008年)1月20日上午十时过世。享寿八十八岁。著有《合气道--基础篇》《合气道--应用篇》。","text2":"李清楠是什么拳的创始者?","label":1} {"text1":"萧规曹随,汉语中的一个成语,形容继任人沿用前任定下的规则、做法等等,不作更改。萧何与曹参两人少年担任秦朝小官吏时,便已是好友,后来西汉建国,萧何身为宰相,曹参身为大将,地位不凡的两人却反而有了嫌隙。萧何担任相国时,参考前朝文献制订典章及制度。萧死前,推荐曹参继任。曹参上任后,认为萧何订下的法令已很完备,所以继续沿用而不作改动。曹参就任汉相国期间,整日饮酒食肉,政治上清静无为,继续执行萧何留下的政策,不予变动。汉惠帝认为自己被曹参轻视,于是命其子御史大夫曹窋劝谏,曹参把曹窋鞭笞了两百下并赶出门外,汉惠帝于是亲自责问曹参。曹参摘帽,向皇帝俯首谢罪:「陛下您认为,您与先帝相比,谁较为英明神武?」皇帝回道:「我怎敢与先帝比?」曹参又问说:「我跟萧何比,谁较贤能?」皇帝说道:「您好像不太比得上他。」曹参接著说:「陛下说得对,且高祖跟萧何平定了天下,法令都健全具备。陛下只要垂拱而治,我们这些官吏坚守岗位,遵守他们的法令而不犯过失,不是很适当吗?」时人歌颂:「萧何制定法律,调和整齐如一;曹参继任相国,遵法而不犯过失。施载清净无为的政策,人民因而安宁统一。」史称「萧规曹随」。据前汉书记载,曹参与萧何于沛县时为挚友,但萧何担任相国时反生嫌隙。不过曹参为汉相国时,却对萧何担任相国时之制度一无变更。现今中文使用中,世人所熟知的「萧规曹随」有率由旧章、依样葫芦、一成不变、沿袭旧规等等意涵,然而曹参为汉相国时,却不是此成语所指称之无所事事,反而是选拔贤才、注重吏治,各官吏均选任老成持重之人,若有好名之人,即使其去职。可见「萧规曹随」的传统见解对曹参为汉相国的治术略有误解。曹参担任相国时,对于下属有极小过错,反而为其隐忍不公开。原典内容如下:司马迁著《史记·卷五十四·曹相国世家》:「参代何为汉相国,举事无所变更,一遵萧何约束。……惠帝怪相国不治事,以为『岂少朕与』?……参免冠谢曰:『陛下自察圣武孰与高帝?』上曰:『朕乃安敢望先帝乎!』曰:『陛下观臣能孰与萧何贤?』上曰:『君似不及也』。参曰:『陛下言之是也。且高帝与萧何定天下,法令既明,今陛下垂拱,参等守职,遵而勿失,不亦可乎?』惠帝曰:『善。君休矣!』……百姓歌之曰:『萧何为法,顜若画一;曹参代之,守而勿失。载其清净,民以宁一。』」扬雄著《法言》·〈渊骞第十一〉:「萧也规,曹也随。」","text2":"现在人们所理解的“萧规曹随”还有什么意思?","label":1} {"text1":"澳门巴士2路线是一条由澳门公共汽车有限公司经营,往返永宁广场和妈阁的循环巴士路线。受到发夹弯弯位影响,巴士公司很少派出10米或以上巴士行走,但现时每日均有10.84米的大巴行驶于本线。1989-1990年8月:1990年7月中:2007年 - 2011年7月31日:本线虽然是祐汉、黑沙环往返中区和水坑尾的路线之一,但沿途有较多路线选择(如6A线、12线、19线和回程的22线),加上12线及19线比本线还要快,行车时间更短,本线客量明显上较12线及19线逊色,不过黑沙环往返中区一带较有优势(10线需经新口岸才到中区;6A线要行经高士德、沙梨头才到中区),而黑沙环是人口密度较高地区,不少黑沙环居民会利用本线往返中区通勤,因此尖峰时间都常有客满情况出现。虽然12线及19线行车时间比本线快,本线晚上班次仍较12线及19线频密(12线及19线每10分钟一班;而本线则8分钟一班),是本线的优胜之处之一。本线整体客量维持在中高水平。2008年8月15日,一辆2路线苏州金龙KLQ6930G大巴,在荷兰园大马路近若翰亚美打街街口停车后,巴士未能再启动,期间车尾发出响声,有气体喷出,车尾底部漏出大量的红色液体。车长基于安全考虑,立即疏散巴士内20多名乘客到附近的行人路,并安排乘客转乘下一班同线路巴士,意外中无人受伤。巴士停在马路时令现场交通受阻,须由交通警员在场指挥交通。而新福利表示事件怀疑因有管道漏波箱油所致,车长基于安全考虑,所以才疏散巴士上乘客,公司对事件表示关注。","text2":"澳门巴士2路线是由哪个公司经营的?","label":1} {"text1":"《爱在暴风的日子》(英语:),香港电视广播有限公司怀旧爱情温情电视剧,于1998年9月28日至1998年10月23日首播,共20集,监制王心慰。主要演员有黎美娴、方中信、林文龙、魏骏杰、杨羚、夏韶声等等。主题曲《火种》及插曲《扑火是我》是由夏韶声主唱。《爱在暴风的日子》以五十年代警队龙蛇混杂的香港为背景,女主角一生遇上两段刻骨铭心的爱情,剧情荡气回肠,扣人心弦!孤女林青儿(黎美娴)为救落难父亲,受尽澳门探长洪坤(夏韶声)与黑帮头目雷金发(沈威)蹂躏,幸得好友张楚文(林文龙)冒死相救。二人偷渡至香港,楚文不幸被警方逮捕,青儿承诺等他出狱,怎知噩耗传来,青儿接获楚文在服刑期间的死讯,在悲痛之余,青儿惟有独自面对将来。三年后,青儿与好友方月娥(杨羚)邂逅了廉洁正义的探长骆俊(方中信),骆俊钟情于青儿,但二人间之感情波折重重。后骆俊接手调查连串杀人事件,并在过程中发现该杀手原来是楚文,青儿为救楚文,与骆俊起争执,并欲与楚文远走他乡,然而青儿仍心系骆俊,楚文知悉后大感失落,决意寻死,诱骗骆俊对其开枪,以结束痛苦的三角关系。骆俊因有感于警界贪污受贿情况严重,与下属周铁柱(魏骏杰)合力打击以洪坤为首的警界败类,但洪坤阴险狡诈,屡次皆能化险为宜,势力更逐渐坐大,因此署长决定成立反贪污特别小组,并指派骆俊及铁柱专责处理,但就在此时,楚文的尸体被发现,骆俊自首,在种种证据皆对骆俊不利下,有人偷偷放走骆俊,但骆俊并未逃逸,反而选择为理想而死,枪杀了洪坤,被判处环首死刑。骆俊与青儿终于在狱中结婚,在骆俊行刑时,青儿亦选择跟随骆俊,在另一个空间,延续这份爱。","text2":"《爱在暴风的日子》是何时首播的?","label":1} {"text1":"环球电台(英语:Trans World Radio,缩写为:TWR)是一个跨国基督教福音广播电台。环球电台从14个国家,使用中波或大功率调频发射器、短波以及通过当地电台、电缆、人造卫星和互联网,达到160个国家、230种语言和方言区的数百万人。环球电台拥有许多节目,其中一些由电台职员制作,另一些来自其他机构,例如“Thru the Bible”和“Insight for Living”。环球电台的国际总部位于美国北卡罗来纳州卡瑞。总部和世界各地的职员来自美国、加拿大和各个广播目标国。2006年,其英国总部从巴斯迁往曼彻斯特。环球电台开始于1952年,当时保罗·弗里德(Paul Freed)从摩洛哥向西班牙发射广播。后来,环球电台被驱逐出摩洛哥,转移至摩纳哥。在摩洛哥曾经多年使用的大功率发射器是二战后纳粹所抛弃的。其他主要的发射地点还有关岛、博奈尔岛、斯里兰卡、塞浦路斯和斯威士兰。环球电台使用230种语言向全球广播。汉语普通话、广州话、蒙古语、维吾尔语、藏语、彝语、英语、日语、韩语、越南语、高棉语、乌兹别克语、阿拉伯语、土耳其语、法语、意大利语、葡萄牙语、马拉加斯语等等。环球电台的中文广播开播于1954年。平均每日播出6小时,由关岛发射,呼号为KTWR-SW。其中文广播总部设在香港。※时间=UTC+8","text2":"环球电台是如何运作的?","label":1} {"text1":"费明仪((英文:Barbara FEI Ming-yee,),天津人,祖籍江苏苏州。香港声乐家,曾赴法国深造声乐,1964年创立「明仪合唱团」,在音乐艺术界德高望重,是最早关注中国民族音乐的香港女高音,对中西音乐的修养颇深。父亲为名导演费穆,亦是《大公报》社长费彝民的姪女。费明仪曾任香港艺术发展局音乐组主席、明仪合唱团音乐总监及指挥、香港合唱团协会主席、香港大学亚洲研究中心名誉研究院士、乐府音乐委员会主席、香港民族音乐学会会长、香港音乐专科学校校董及校长、华人女作曲家协会名誉主席、香港康乐及文化事务署音乐顾问、香港中乐团资深顾问等。同时也担任北京中央音乐学院客座研究员、中国艺术研究院音乐研究所客座研究员、中国音乐学院客座教授、天津音乐学院声乐系教学顾问客座教授、吉林艺术学院客座教授、上海音乐学院名誉教授、廿世纪华人音乐经典艺术委员会委员和中华民族文化促进会的副主席等。中国歌剧方面,费明仪曾担任监制及女主角,在香港演出改编自山西民间故事《哑姑泉》之歌剧《甜姑》及国内创作喜歌剧《第一百个新娘》。费女士热衷推动音乐文化,分别于北京、上海、香港设立音乐奖学基金、赞助香港校际音乐比赛等。费女士积极致力香港与中国内地及国际性的音乐学术交流。","text2":"在中国歌剧方面,费明仪做了怎样的工作?","label":1} {"text1":"美国马萨诸塞州一共有14个县,其中的8个县没有自己的政府,只有5个县拥有自己的独立政府,分别是巴恩斯特布尔县、布里斯托尔县、杜克斯县、诺福克县和普利茅斯县。而楠塔基特县则拥有一个县级和市级的混合政府。尽管如此,司法和执法区仍然以县之间的边界划分,且这些县在地理上仍被视为一个地理实体。在法律方面,不同县份的刑事诉讼案件是由不同区域律师负责。在埃塞克斯县,其刑事诉讼案件是由东区区域律师负责;而在米德尔塞克斯县,案件则是由北区区域律师负责。中区区域律师负责伍斯特县的案件;海角和海岛区区域律师负责杜克斯县、巴恩斯特布尔县和楠塔基特县的案件;而西北区区域律师则负责富兰克林县和汉普希尔县的案件。伯克希尔县、布里斯托尔县、汉普登县、诺福克县、普利茅斯县和萨福克县则拥有自己的区域律师。有11个马萨诸塞州县份已不复存在,而其中的大部份是因其领土被新罕布什尔州或缅因州占领而消失。马萨诸塞州现存最具历史的县份为埃塞克斯县、米德尔塞克斯县和萨福克县,它们是于1643年自马萨诸塞殖民地的诺福克县(马萨诸塞殖民地的诺福克县跟马萨诸塞州的诺福克县没有关系)独立。诺福克县在后来被新罕布什尔州占领。当这三个县成立时,它们仍然属于马萨诸塞殖民地。相反,最迟成立的县份是于1812年成立的汉普登县 。大部分马萨诸塞州的县份取名自英国地名,全因马萨诸塞州在早期是大英帝国的殖民地。郡镇(shire town)或县治(county seat)是指拥有县法院和行政办公室的城镇。一个县可以拥有多个郡镇。美国马萨诸塞州共有14个县。","text2":"在马萨诸塞州,大部分县的名字由来是什么?","label":1} {"text1":"繁星鲆(学名:)俗名比目鱼、扁鱼,为辐鳍鱼纲鲽形目鲽亚目鲆科的一种。本鱼分布于印度西太平洋区,包括东非、塞席尔群岛、斯里兰卡、印度、中国、日本、台湾、韩国、越南、菲律宾、印尼、澳洲等海域。该物种的模式产地在日本高知。水深10至30公尺。本鱼体极扁平,呈卵圆形;口小,双眼位于左侧。体色随环境而改变,大致上呈棕褐色,具有许多美丽的圆形花斑,鱼体中后具三角形大斑。雄鱼的胸鳍鳍条延长,有时达到尾部,且两眼间具较雌鱼宽。背鳍软条87至97枚;臀鳍软条61至73枚,体长可达27公分。本鱼常出现在珊瑚礁海域的礁沙混合区,将鱼体埋于沙中,活动力差,仔稚鱼期身体左右对称,随成长而变态。属肉食性,以底栖性无脊椎动物为食。食用鱼,用油炸成扁鱼酥。","text2":"繁星鲆的俗名是什么?","label":1} {"text1":"奈舍(瑞典语:)是瑞典延雪平省的一座城市。奈舍有很长一段时间以农业作为主要产业,但自从瑞典铁路系统于1864年完工,并通过了奈舍,由于其在瑞典的地理位置,成为了一个重要的交汇点。行驶到卡特琳娜霍尔姆的铁路于1874年打通,同一年也开放了奥斯卡港线;1882年开放了到哈尔姆斯塔德的铁路;卡尔马则是在1914年完工。如今,奈舍是瑞典唯一一个通往六个不同方向的交汇处。工业搬到了奈舍,人口也跟著增加了。由于森林覆盖了整个延雪平省,所以最重要的行业是林木工业,这里制造的木椅也称为温莎椅。奈舍在1914年被授予宪章,成为瑞典城市之一。1971年,奈舍成为了奈舍市的行政中心。它的人口在20世纪持续增加。20世纪40年代开始安装电气设备,而长期以来是奈舍的最大工业公司。摇滚乐队和金属乐队Kongh都来自奈舍。北欧最大的庞氏骗局,其主嫌Hannu Kailajärvi于2008年12月10日在奈舍被捕,据说曾在那里住了一段时间。","text2":"奈舍现在的经济结构为什么会发生变化?","label":1} {"text1":"亚历山大港的丢番图(希腊语:,生卒年约公元200~214至公元284~298),有“代数之父”之称;也有人认为此称谓应与比他大约晚出生五百年的一位波斯数学家花拉子米共享。人们对丢番图的生活知之甚少。在罗马时代,他住在埃及的亚历山大港,大概从公元200年到214年,到284年或298年。丢番图曾被历史学家描述为希腊人,非希腊人,希腊化的埃及人,希腊化的巴比伦人,犹太人,或者是迦勒底人。我们对丢番图生活的了解,来自于一个5世纪的希腊文集。他作著的丛书《算术》(Arithmetica)处理求解代数方程组的问题,但其中有不少已经遗失。后来当法国数学家费马研究《算术》一书时,对其中某个方程颇感兴趣并认为其无解,说他对此「已找到一个绝妙的证明」,但他却没有写下来,三个世纪后才出现完整的证明,详见费马大定理。在数论中常常能看到他的名字,如丢番图方程、丢番图几何、丢番图逼近等都是数学里重要的研究领域。丢番图是第一个承认分数是一种数的希腊数学家--他允许方程中的系数和解为有理数,这是在数学史中具有开创性的。不过在今天,丢番图方程一词通常指以整数作为系数的代数方程,而其解也要求是整数。丢番图在数学符号方面也作出了贡献。","text2":"亚历山大港的丢番图有什么称号?","label":1} {"text1":"陈苇绫(),改名前为陈春如,绰号超人,台南市中西区仙草里人,台湾女子举重运动员。2004年夏季奥林匹克运动会第一次代表台湾出赛,而成绩还不如台湾另一名同一量级参赛选手陈涵彤,只获得第11名。2008年北京奥运会女子举重48公斤级比赛中代表中华台北获得金牌,总合196公斤。亦为该队于京奥获得的第一枚奖牌。2009年世界运动会健力赛轻量级,陈苇绫以蹲举,硬举,以及总合三破世界纪录夺得金牌。陈苇绫以总合177公斤在2011年彰化全国运动会中完成女子举重3连霸,赛后宣布退休。陈苇绫在2016年里约奥运女子举重48公斤级获得7名,总合181公斤。2016年6月,2008年夏季奥林匹克运动会举重女子48公斤级银牌得主西贝尔·厄兹坎遭到国际举重总会公布药检呈阳性反应。2016年8月,2008年夏季奥林匹克运动会举重女子48公斤级金牌得主陈燮霞遭到国际举重总会公布药检呈阳性反应。2017年3月,陈苇绫确定递补金牌。","text2":"陈苇绫首次获得金牌是在什么赛事上?","label":1} {"text1":"丁丁在美洲(法语:Tintin en Amérique;英语:Tintin in America)是《丁丁历险记》的第三部作品。作者是比利时漫画家埃尔热。丁丁在美洲于1931年9月3日开始在比利时报刊二十世纪小伙伴上连载。1932年集结成册出版。1945年,进行彩色化,并精简重编为62页的标准格式。美国版于1973年出版。故事的舞台是在1930年代,黑帮与盗匪横行的美国城市芝加哥。从踏足芝加哥的第一刻开始,匪帮已经牢牢盯著丁丁,一心要除之而后快!一场接一场的正邪搏斗旋即展开,丁丁不断在鬼门关外徘徊:被绑架掳劫、被囚禁、被毒气弄昏、被投进水去淹死、被嫁祸吊死、甚至被缚在铁路轨上给火车碾过!见义勇为的丁丁与好拍挡米路如何逃过一次次的劫数?如何把匪帮头子一个个地绳之于法?","text2":"《丁丁在美洲》是何时进行彩色化的?","label":1} {"text1":"基姆高撞击坑必须提到Tüttensee湖,这是位于德国东南方阿尔卑斯山的山麓小丘,邻近基姆湖的坑穴,被认为是在全新世遭到陨石撞击形成的。发现者是一个自称为基姆高撞击研究小组(Chiemgau impact research team,CIRT)的爱好考古学团队,为德国的地质学和考古学协会带来了一些争议,因为迄今除了CIRT之外,没有其他单位接受此一论点。Tüttensee湖的座标位置如下: 。依据假说,在30 × 60平方公里的区域内散布著80个直径超过3米的独立坑穴。在基姆高散布的区域被发现之后,CIRT在Tüttensee湖火山口和他的周围完成了许多地质上的田野调查和原始石块的分析。湖的直径是400米,深17米,有直径500米高8米的外缘。根据CIRT的放射性碳-14数据和考古学上的发现,撞击事件发生在西元前500年。","text2":"依据假说,在30 × 60平方公里的区域内散布著多少个直径超过3米的独立坑穴?","label":1} {"text1":"14街是纽约市曼哈顿一条东西向横贯道路,重要商业区。在过去14街曾是高档地点,但城市重心北移后失去部分魅力与地位。在百老汇, 14街构成联合广场的南部边界,也是格林尼治村和东村的北部边界,和切尔西, 熨斗区和Gramercy的南部边界。14街标志着曼哈顿方格路网的南部末端。14街以北的街道几乎完全按序号排列成完美的方格网。在14街以南,在格林尼治村和曼哈顿下城其他地方,方格网变得散乱,到休斯顿街以南就不复存在。西14街开始于格林尼治村东北的纽约州9A公路交流道。在交流道末端,与第十大道交汇,不远又与第九大道和哈德孙街交汇。14街–第八大道车站在下一个十字路口,A线、C线、E线和L线停靠此站。在第七大道十字路口,14街车站有1号线, 2号线和3号线停靠。在下一个十字路口,14街与第六大道(美洲大道)交汇。穿越第五大道后,称为东14街。在联合广场东南部与第四大道和百老汇大道交汇。14街再穿越几个路口后,结束于东河边的罗斯福快速道路。","text2":"14街结束在什么地方?","label":1} {"text1":"沈云山()是在香港九龙东部的一个山丘,高177米,地图上多没标示。该山俗称为「水塘山」。沈云山跟其西北面的平山形成佐敦谷。沈云山东北面为大上托(安达臣道石矿场),两山之间的山谷已开发成顺天邨。沈云山北面是顺利邨、顺安邨和飞鹅山,南面则为鳄鱼山和牛头角。沈云山的山顶上,建有水务署的观塘上食水配水库和观塘上海水配水库。配水库平台上建有由康乐及文化事务署管理的观塘上配水库花园,是区内的晨运热点。沈云山山脚有数座政府建筑物,北为佐敦谷海水配水库和前佐敦谷水塘,南为福建中学和乐华天主教小学,西为沈云山海水抽水站及沈云山食水抽水站。市民可从入口位于北面顺利邨道的车路,或从南面振华道经「振华道晨运径」直上山顶。在观塘上海水配水库的后面,有一个被认为是前防空洞的山洞。山洞已有闸门封锁,从闸门的窗口看进,里面有生锈之水管及一些设备,而闸门之上则刻有金属字「HKWW 1965」。","text2":"闸门之上则刻有什么?","label":1} {"text1":"卡尔-奥古斯特·威廉·恩斯特·弗里德里希·格奥尔格·约翰·阿尔布雷希特(\"Carl-August\" Wilhelm Ernst Friedrich Georg Johann Albrecht,),萨克森-魏玛-艾森纳赫大公储,1923年起为萨克森-魏玛-艾森纳赫大公家族族长。卡尔-奥古斯特是萨克森-魏玛-艾森纳赫末代大公威廉·恩斯特与夫人萨克森-迈宁根的费奥多拉的儿子,1912年7月28日生于巴伐利亚王国上弗兰肯行政区的威廉斯塔尔()。一出生便成为萨克森-魏玛-艾森纳赫的大公储,同时还是荷兰王位第三顺位继承人。1918年,德国爆发革命。卡尔-奥古斯特的父亲在革命中于被推翻,卡尔-奥古斯特失去了继位大公的机会。随着1922年荷兰修宪,他也失去了继承荷兰王位的机会。卡尔-奥古斯特的父亲于1923年去世,他继承父亲成为萨克森-魏玛-艾森纳赫大公家族族长,韦廷家族首领。若德国仍是君主制国家,他将是萨克森-魏玛-艾森纳赫大公卡尔·奥古斯特二世。卡尔-奥古斯特于1988年10月14日在巴登-符腾堡州博登湖畔的希恩嫩去世,终年76岁。他的长子米夏埃尔-本尼迪克特继位成为家族族长。卡尔-奥古斯特1944年10月5日在瓦尔特堡与汪根海姆-温特尔施泰因女男爵伊丽莎白(Elisabeth Freiin von Wangenheim-Winterstein)结婚,有一子二女:","text2":"哪一年起萨克森-魏玛-艾森纳赫大公家族族长?","label":1} {"text1":"红梅花雀(学名:)是梅花雀科红梅花雀属的一种,分布于多米尼加共和国(引进种)、老挝、印度尼西亚、东帝汶、斐济(引进种)、波多黎各(引进种)、埃及(引进种)、中国大陆(云南、贵州、海南)、菲律宾(引进种)、伊朗(引进种)、文莱(引进种)、瓜德罗普(引进种)、巴基斯坦、马来西亚(引进种)、马提尼克(引进种)、孟加拉国、瓦努阿图(引进种)、意大利(引进种)、印度、巴林(引进种)、斯里兰卡、越南、尼泊尔、柬埔寨、美国(引进种)、日本(引进种)、缅甸、墨西哥(引进种)、西班牙(引进种)、泰国、留尼汪(引进种)和葡萄牙(引进种)。该物种的保护状况被评为无危。红梅花雀的平均体重约为9.2克。栖息地包括亚热带或热带的(低地)湿润疏灌丛、亚热带或热带的湿润低地林、耕地、湿地、种植园和亚热带或热带的(低地)干草原。该物种的模式产地在印度。","text2":"红梅花雀属于什么科什么属?","label":1} {"text1":"LED背光液晶显示电视(LED-backlit LCD television),是使用发光二极体(LED)作为背光源的液晶显示电视。部分电视机生产厂商,如三星、松下、东芝、飞利浦、乐金及索尼等将其称为LED电视,但此并不是指使用发光二极管作图像显示的电视。随著发光二极管电视市场竞争日趋激烈,加上发光二极管应用逐渐成熟,各家电视厂商纷纷积极导入发光二极管背光液晶显示电视,企图在家用电视改朝换代之际能够拔得头筹。发光二极管背光技术可分为直下式(direct back-lit或 full LED array back-lit)与侧照式(edge LED back-lit)两种,而当中直下式所使用的发光二极管有分为白光发光二极管与红、绿、蓝三色发光二极管两种。直下式发光二极管背光技术是把多枚发光二极管排成阵列,放在散光片及LCD后面,直接照射LCD。如此,直下式可以依从画面不同部份的光度变化,快速地微调发光二极管的明暗,大为提高动态对比度达至等离子显示器的水平。缺点也就是需使用数量较多的发光二极管,耗电、体积厚、价格较高。直下式发光二极管背光所使用的有白光发光二极管,也有使用红、绿、蓝三种单色发光二极管(RGB LED)的,使用RGB LED可以有更阔光频谱,也即有更广色域。侧照式发光二极管背光技术是把白光发光二极管放在LCD的四边,LCD后有一与LCD大小相近的反光片,发光二极管从LCD与反光片之间的缝中照进去,反光片上特别设计的微纹能把LED照来的光作90°反射后照向LCD的背部。此等反射片精细得在不需散光片的情况下也能使发光二极管的光平均地分布地照到LCD背部。相对RGB LED,白光发光二极管较为耗电,加上反光片有损耗及LED照射角等因素,侧照式LED原先的耗电会较高,但因为没有散光片,省去了散光片的损耗,侧照式LED背光的耗电可以做到与直照式同级。侧照式可以比直下式做的更轻薄,但色域及对比度都不及直下式。以往侧照式发光二极管背光不能依从画面不同部份的光度变化快速地调整光度,但最近有厂商成功以侧照LED技术有限度的依从画面变化快速地调整光度。以往薄膜电晶体液晶显示器的光源是采用冷阴极萤光灯作背光光源,(如图2)。两者的分别在于:","text2":"侧照式跟直下式相比有什么优势?","label":1} {"text1":"营业税是流转税制中的一个税种,其课税范围和纳税依据可以是商品生产、商品流通、转让无形资产、销售不动产、提供应税劳务或服务等等的营业额,特殊情况下也有不计价值而按商品流通数量或者服务次数等计税的。营业税涉及范围广,因而税源充足;且以营业额作为课税对象,实行简单的比例税率,税收收入不受纳税人营运成本、费用高低的影响,纳税人难于转嫁税负,使计税和征收方法简便直接,极具直接税的特征。在商品的生产和流通中,在最终到达商品的消费和使用者手中之前,其所经过的环节越多,各环节所累计的营业额也越多,这就使按营业额计收的营业税有可能成为商品成本的主要增长因素。所以,在不同国家或同一国家不同的历史时期,营业税的课税范围是有所不同或有所变动的,如商品生产和流通及劳务服务等,不再征收营业税,而以在流通环节税负较低的增值税代之。一、初颁营业税法(民国20年)民国初期,我国营业税之课征,系沿袭清末之厘金、牙税、当税、屠宰税、盐税、酒税,及新创立之烟酒特许牌照税、特种营业税与普通商业牌照税所构成。民国(下同)20年乃根据各种大纲办法细则,制定营业税法13条,于同年6月13日公布施行,各省亦先后将所拟营业税征收章程报财政部核准,呈请行政院备案,我国一般营业税于焉诞生。同时,将各省原有牙税、当税、屠宰税等,依照原有税率改课营业税,故当时有普通营业税、牙行营业税、典当营业税及屠宰营业税等之分别。目前(2016年)的税率为 5%。每两个月为一期申报一次,但可申请每月申报。可使用纸本申报(401, 403),或使用软体电子申报。特种行业适用。适用行业包括小规模营业人、银行业、保险业、证券业、夜总会、有娱乐节目之餐饮店、酒家。依照行业与业务不同,税率为0.1%~25%中国大陆的营业税主要根据《营业税暂行条例》征收,课税税目包括建筑业、文化体育业、金融保险业、服务业、土地及自然资源转让、销售不动产以及娱乐业。建筑业、文化体育业:3%。金融保险、服务业、不动产及无形资产转让:5%。(已纳入营改增的交通运输业、电信业征收增值税)娱乐业:根据地方不同情况执行5%-20%的浮动税率。2016年5月1日起,营业税取消,中国大陆地区所有行业所有企业均按照增值税标准征收增值税。新增值税的税收额度由中央和地方各享有一半。","text2":"为什么营业税的税源充足?","label":1} {"text1":"皮埃尔·马克斯·杜布瓦(,),法国作曲家,出生于Graulnet。他在巴黎音乐学院(Conservatoire de Paris)就读期间受到法国六人团(Les Six)成员之一达律斯·米约(Darius Milhaud)的指导,并将六人组的新音乐主张带入二十世纪中期,虽然知名度不高却被受尊敬。曾于获1955年的罗马大奖及1964年的巴黎音乐大奖。其作品仍保留传统调性与旋律,运用有趣的和声与织度造就明亮的听觉效果。为木管乐器创作为数不少的作品,尤其是萨克斯管,其中包括许多独奏与室内乐作品。作品主要有《F大调萨克斯管四重奏》(Quartet for Saxophones in F )以及为中音萨克斯管创作的《性格小品组曲》(Pieces caracteristiques en forme de suite)等。他于1995年逝世于法国罗康库(Rocquencourt)。","text2":"皮埃尔·马克斯·杜布瓦的音乐以哪种乐器为主?","label":1} {"text1":"双江镇位于湖南省长沙县东北角,地处长沙县、浏阳市和平江县三县市交汇处。地缘上双江乡东南部与浏阳市沙市镇、社港镇接壤,东北部与平江县瓮江镇为邻,西部与金井镇交界,全乡总面积65平方公里,总人口19,093人(2000年人口普查);辖10个行政村、1个社区;乡政府驻罗代社区。今双江镇1949年属承祖乡,1956年为双江乡;1958年属金井公社,1961年析置双江公社,1983年双江公社改为双江乡,(年份?)后双江乡改为双江镇。双江镇在1995年长沙地区撤区并乡镇之前为金井区双江乡,1995年福临区撤销后双江乡直属县政府。2004年村级区划调整,调整之前为15个村,分别为大桥村、农裕村、唐家铺村、团山村、青山村、桃花村、光华村、石湾村、双江村、赤马村、石板村、金花园村、山笔村、石井村和跃进村;区划调整后为10个村、1个社区。","text2":"1958年,双江镇属于哪里管辖?","label":1} {"text1":"曼西人(),旧称“沃古尔人”(),是西伯利亚西部,鄂毕河流域的原住民族。曼西人分布在乌拉尔山以东的鄂毕河流域,即俄罗斯联邦秋明州的汉特-曼西自治区及其周边地区,也分布在秋明州。根据俄联邦2002年的人口统计,总人口数量为11432。各地曼西人的人口数据如下:曼西人和汉特人在族源、语言、历史和文化等方面有诸多共同点。其族源可追溯到1世纪中叶的额尔齐斯河流域的草原乌戈尔部落。后来一些部落迁至鄂毕河下游,逐渐形成新的民族,以捕鱼和狩猎为生,生活方式为半定居半游牧。他们是15-16世纪的西伯利亚汗国臣民,但也有自己酋长国。16世纪末期,曼西人被沙俄哥萨克叶尔马克·齐莫菲叶维奇征服(是第一个)。与汉特人相比,曼西人受俄罗斯同化程度更深。1930年,前苏联建立奥斯恰克-沃古尔民族区,1940年改称汉特-曼西民族区。1960年代以来,该地区成为重要的石油、天然气产地。曼西语和汉特语相近,也与匈牙利语有关,这些语言共同组成乌拉尔语系的乌戈尔语支。曼西语、汉特语和俄语都是汉特-曼西自治区的官方语言。","text2":"汉特-曼西自治区的官方语言是什么?","label":1} {"text1":"威廉·杰克逊·胡克(,)是英国植物学家。胡克出生于诺里奇。父亲是一位神学家,精通德国文学,尤其喜好种植稀奇的植物。他在诺里奇中学毕业后,学习了鸟类学和昆虫学,后来开始对植物感兴趣。1809年夏季,他到冰岛考察,搜集了许多标本,但回程时发生了火灾,差点丧命,标本几乎都被毁,但他凭记忆写出了《冰岛记游》,记录了冰岛的植物和居民生活。1814年,他用了9个月时间到法国、瑞士和意大利考察植物,1816年,出版了第一部学术著作《英国叶苔属》,1820年,他受聘担任格拉斯哥大学的植物学教授,第二年,出版了《苏格兰植物》,发起成立了格拉斯哥皇家植物学会和格拉斯哥植物园。1841年,他被任命为皇家植物园首任园长,在他的主持下,皇家植物园从4公顷扩大到30公顷,种植面积达1.1平方千米,建立了许多新温室和一个经济植物博物馆。他的园长职务后来由他的儿子,同样著名的植物学家约瑟夫·道尔顿·胡克接替了。","text2":"威廉·杰克逊·胡克的父亲尤其喜好种植什么?","label":1} {"text1":"托尼·埃斯唐盖(Tony Estanguet,),法国皮划艇激流回旋男子运动员,曾在2000年夏季奥林匹克运动会、2004年夏季奥林匹克运动会和2012年夏季奥林匹克运动会上夺得单人项目金牌。托尼是皮划艇运动员亨利·埃斯唐盖的儿子,亨利在20世纪70年代赢得世界锦标赛奖牌。他的哥哥,帕特里斯·埃斯唐盖[1996年夏季奥林匹克运动会|1996年亚特兰大奥运会]]赢得了一枚铜牌。托尼·埃斯唐盖共在奥运会上获得3枚C-1项目金牌,分别为2000年、2004年和2012年。2004年的金牌很有争议,裁判判罚米哈尔·马尔季坎2秒的惩罚,使其屈居亚军,其最后成绩仅比落后埃斯唐盖0.12秒。埃斯唐盖是法国在2008年北京夏季奥运会开幕式的旗手。他在C-1半决赛获得第9位,因此被最终淘汰。2012年伦敦奥运会,他成为法国第一位在同一项目获得3枚金牌的运动员。他在皮划艇世界锦标赛一共获得12枚奖牌,其中5枚金牌(C-1:2006年,2009年,2010年;C-1团体:2005年,2007年),6枚银牌(C-1:2003年,2005年,2007年;C-1团体:1997年,2003年,2009年),一枚铜牌(C-1团体:1999年)。2012年,他入选为国际奥委会运动员委员会,他将担任国际奥委会委员,任期为8年。2012年10月30日,他宣布退役。","text2":"托尼·埃斯唐盖获得过哪些赛事的金牌?","label":1} {"text1":"乌卢鲁(意为「土地之母」),又被称为艾尔斯岩,位于澳大利亚北领地的南部,是在澳大利亚中部形成的一个大型砂岩岩层,位于乌鲁汝-卡塔楚塔国家公园 之内。乌鲁汝位于离它最近的大镇爱丽丝泉南部的335公里处,连接两地的公路长450公里。乌鲁汝是(该地区的澳大利亚原住民)的圣地。乌鲁汝被列为世界遗产。乌卢鲁位于澳洲的近乎地理中央处,是世界上仅次于奥古斯特山的第二大单体岩石。由于形成乌卢鲁的砂岩含有较多铁粉,氧化之后的铁粉呈现红色,使得乌卢鲁呈现红色外观。原住民在距今1万年之前已经开始在乌卢鲁一带居住。这里发现的壁画遗迹中历史最久的距今已有超过1000年的历史。乌卢鲁现在由原住民组织所有,在法律上该组织将乌卢鲁一带的土地借给澳洲政府至2084年。现在进入乌卢鲁一带地区需要支付25澳元的入场费。","text2":"乌卢鲁的入场费是多少?","label":1} {"text1":"南雅镇是中国福建省南平市建瓯市南部的一个镇,位于建溪中游,与延平区及顺昌县相邻。《建瓯县志》记载“东汉献帝建安初年,即隶属建安县,因系建瓯南面门户,初名‘南瓦口’。后人因这里的山水相依,四季山青水秀,环境幽雅,因而更其名为南雅。”。南雅镇属中低山地,境内高山有云龙岩、龙岗岩、打四门山、莲花山、老保岗、通天阳、九洞岩、高世林岗等。溪河属建溪流域,主要有高阳溪、集北溪、白沙溪、中湾溪等8条。南雅镇距离建瓯市区23公里,距离南平市区45公里。205国道、横南铁路从南雅镇经过,境内设有火车货车站和浦南高速公路互通口。南雅镇共辖1个居委会,21个行政村,2个镇办场,117个自然村,分别是:四龙居委会、南雅村、新建村、伊村村、新村村、小雅村,十字街村、集瑞村、杉溪村、小康村、叶康村、皇康村、大康村、梅村村、鲁口村、白水源村、房村村、仁墩村、黄园村、山安村、白沙村、太平村。南雅镇的主要产业有林业、茶叶、柑桔、葡萄及锥栗等,素有“柑桔之乡”、“南国葡萄之乡”、“闽北竹乡”之称。竹制品加工、茶叶加工、食用菌加工、绿色食品加工、奶牛饲养为南雅镇的五大龙头产业。","text2":"为什么后来会改名为“南雅镇”?","label":1} {"text1":"张镈(),表字叔农,中国建筑家,祖籍山东无棣,出生于广州市,为末任两广总督张鸣岐次子。张镈1930年考入东北大学建筑系,受教于梁思成、林徽因等人,次年九一八事变后转入国立中央大学建筑系学习,1934年毕业后进入基泰公司,受到前辈杨廷宝的影响很深。1941年,张镈主持实测了北京市内大量古建筑。1949年,张镈为避兵火,一度随基泰公司移居香港。1951年年初,张镈辞职,返回新成立的中华人民共和国,成为北京市建筑设计院的总工程师之一。在此后的十余年间,张镈牵头设计了北京友谊宾馆、民族文化宫、人民大会堂等重要建筑。文化大革命开始后,张镈以间谍的罪名被批斗。1972年复出,负责了北京饭店东楼的设计工作。1990年,受其故乡无棣县政府之托,张镈设计了新海丰塔,新塔在原塔基遗址之南重建。1999年,张镈因患癌症,在北京逝世。骨灰安葬在无棣县车王镇张氏族茔。","text2":"1999年,张镈因患癌症,在北京逝世。骨灰安葬在哪里?","label":1} {"text1":"少年队()是日本杰尼斯事务所旗下的三人组合,为日本1980年代非常受欢迎的偶像团体,领导的是锦织一清(东山只在少年队梦这个节目内领导)。少年队原名是B Team,后期更名为Johnny's少年队,在1981年组成。当时成员是锦织一清、植草克秀及松原康行,负责为近藤真彦及田原俊彦伴舞。后来松原康行退出,1982年东山纪之加入,成为现在的少年队。经过3年训练及演出后,他们在1985年12月12日正式出道。初次亮相时的广告标语是「日本出发,前往世界」。1986年获得第28回日本唱片大赏「最优秀新人赏」。出道一年后,1986年少年队开始有他们的舞台剧Playzone,并在每年都在夏天举行,直至现在他们仍然举办Playzone,近年来演出者更有Johnny's Jr.及J-STARS成员。在2007年时,他们已经公演达900次。基本上Playzone每一次是新故事,但唯独是2004年时,为了纪念杰尼斯事务所成立50年就重演了「梦断城西」。目前除了Playzone之外,成员都是各自以个人身份在演艺圈活跃中。","text2":"2004年,少年队为什么要重演“梦断城西”?","label":1} {"text1":"对食,最早是指宫女之间的女同性恋,与宦官与宫女结合的菜户不同。 。对食一辞最早源自汉朝,《汉书》记载:「官婢曹晓、道房、张弃,故赵昭仪御者于客子、王偏、臧兼等,皆曰宫(曹宫)即晓子女,前属中宫,为学事史,通《诗》,授皇后。房(道房)与宫(曹宫)对食。」,应邵注曰:“宫人自相与为夫妇名对食,甚相妒忌也。”,可见对食最早是指女同性恋,但也可能是境遇性性行为,由于宫女在宫中寂寞,又没有接触异性的机会,就与其他宫女发展出性关系。汉武帝的皇后陈氏也有类似记载,“巫著男子衣冠帻带,素与皇后寝居,相爱若夫妇。”“事发,楚服伏辜,皇后废处长门宫。”,但此事皆记载于野史小说,真实性存疑。后来「对食」一词变成宦官和宫女的恋爱和性行为,所谓“宫掖之中,怨旷无聊,解馋止渴,出此下策耳。”。隋唐五代时期的《宫词》有-{云}-:“莫怪宫人夸对食,尚衣多半状元郎。”徐鼐《小腆纪传》:“常中郭氏名良璞,故阉夏国祥之对食也。”对食可以是宦官与宫女,或是宫女与宫女之间的爱情,但多为短暂交往;而菜户则专指宦官与宫女的长久稳定恋爱关系,有如夫妻。唐朝时宦官可在外娶妻,如高力士娶妻吕氏,吕氏则与其他权贵之妻一样同受命妇册封,封为国夫人。明朝宦官在外娶妻是被严格禁止,主要为防止外臣干涉宫廷事务。南和伯方瑛妾许氏改嫁御用监左监丞龙闰。成化五年(1469年)事发,除二人离婚外,龙闰送司礼监处置。故宦官与宫女结成菜户,在明朝宫廷中是为常见。到清朝,宫女不再是终生服役,通常在25岁离宫后,宫女会进行婚嫁,且清代宫女为八旗女子,宦官多充汉族,民间就少有满族肯婚配汉族者,遑论宦官。中国古代女同性恋性行为多称为磨镜,因双方相互以厮磨或抚摩对方下体得到性满足,古代的春宫画也有一人女扮男装,在腰间系一假阳具和对方进行性交的。由于宦官缺乏性能力,一般的性爱方式仅限于爱抚、口交和假阳具。古今文史半月刊第九期杨静盦的〈记阉人〉记载:「阉人近女,每喜手抚口啮,紧张移时,至汗出即止。盖性欲至此已发泄净尽,亦变态也。」。《万历野获编》记载:「近日都下有一阉竖比顽,以假阳具入小唱谷道不能出,遂胀死。法官坐以抵偿。」有些宦官为恢复性能力,不惜杀人,食用小儿脑,《万历野获编》「对食条」-{云}-:「近日福建税当高策,妄谋阳具再生,为术士所惑,窃买童男脑啖之,所杀稚儿无算,则又狠而愚矣!」","text2":"为什么严格禁止明朝宦官在外娶妻?","label":1} {"text1":"乌克兰国徽为蓝色的盾徽,中间有金色的三叉戟。这个图案最初是留里克王朝的印章图案,1918年3月22日曾被立为乌克兰人民共和国国徽。1992年2月19日经乌克兰最高拉达采纳作为国徽。西乌克兰人民共和国国徽启用于其独立时,与今日国徽略有不同,中间为金黄色的狮子。乌克兰人民共和国国徽启用于其独立时,与今国徽相似。乌克兰宪法曾提及要设计大国徽, 但从来没有被采纳。 然而,大国徽又被刊载在不同文献之中。 在这个变体中,盾牌由左侧的加利西亚徽章和身着传统服装的科扎克(Kozak)支持,手持一个滑膛枪,是科扎克的象征。哥萨克将军的职权在右边。 徽章上加有弗拉基米尔大帝的冠冕,象征着乌克兰主权,底部装饰着荚和小麦。乌克兰议会必须通过大国徽的正式通过,得到乌克兰议会三分之二多数的赞成。在1917年迈克哈尔洛·拉什维斯基提出武器大大衣通过与橄榄枝鸽子淋上单个屏蔽件的形式。 盾牌分成五种方式。 在它的中心有一个小盾牌描绘[犁]作为由老乌克兰的国家象征包围着富有成效的和平工作的一个符号:弗拉基米尔伟大沃洛大王侯武器(tryzub) 利特无髯带有金色狮子,带着步枪的哥萨克,基辅的十字弓和利沃夫的狮子。-->","text2":"西乌克兰人民共和国国徽与现在的国徽有何不同?","label":1} {"text1":"黑姑鱼,又称黑,俗名黑口、黑喉、乌喉等,为辐鳍鱼纲鲈形目石首鱼科的其中一个种。本鱼分布在印度西太平洋区,包括莫桑比克、印度、缅甸、泰国、日本、朝鲜、中国、越南、菲律宾、印尼、澳洲等国海域。水深45至200公尺。本鱼吻不突出,口裂大,端未,倾斜。上颌稍长于下颌,上颌骨后缘伸达瞳孔中央下方。上颌外列齿较大,为犬状齿,越前端齿越大,在前端有二大犬齿。下颌内列齿较大越后端越大。体侧灰褐色。背鳍褐色,尾鳍黑褐色。鳃腔、口腔皆为黑色。背鳍硬棘10至11枚、背鳍软条27至33枚;臀鳍硬棘2枚、臀鳍软条7枚。体长可达45公分。本鱼为近海中下层鱼类,主要栖息在较深的水域,肉食性,以小鱼、甲壳类为食。每年5至7月为繁殖期,则会大量群聚。食用鱼,肉质佳,适合油炸、糖醋、清蒸食用。台湾人重视此鱼类,列为十大美味鱼类之一,称一鯃、二红沙、三鲳、四马加、五𩾃、六嘉𫚭、七赤鯮、八马头、九乌喉、十春子。","text2":"黑姑鱼的俗名是什么?","label":1} {"text1":"斑鳍彭纳石首鱼(学名:Pennahia pawak),又称斑鳍白姑鱼俗名春子、帕头,为辐鳍鱼纲鲈形目石首鱼科的其中一个种。本鱼分布在西太平洋区,包括泰国、台湾、中国东海、南海、香港、越南、印尼等海域。水深3至50公尺。本鱼头呈钝尖形,口裂大,端位,倾斜,上颌稍长于下颌,口闭时,上颌外列齿外露。上颌外列齿扩大为犬齿,内列细小绒毛状齿。体侧白胸鳍基以上为褐紫色,会有紫色光泽反射,胸鳍基以下为银白色,自第七到第十根硬棘部份具有一黑色班,软条部中间1\/3鳍高有一纵走白色带,另外上下1\/3为浅褐色。鳃腔黑色,口腔白色。背鳍硬棘11枚、背鳍软条25至28枚;臀鳍硬棘2枚、臀鳍软条7至8枚。体长可达22公分。本鱼为近海中下层鱼类,主要栖息在沙泥底质海域。肉食性,以甲壳类、小鱼为食。食用鱼,肉质佳,适合油炸和清蒸食用。为台湾地区重要的底栖鱼获物。","text2":"斑鳍彭纳石首鱼主要分布在什么地方?","label":1} {"text1":"1918年教育法(\"Education Act 1918\"),又称为费舍教育法(\"Fisher Act\"),是英国国会通过的一项法案。草拟者为文教大臣赫伯特·费舍。1918年教育法将从学校毕业的年龄提高到14岁,并计划扩展高等教育。1918年教育法的另一特色是提供辅助设施(幼儿学校、体格检查和特殊需要儿童中心等)到1920年代,年幼儿童的教育日益成为政治家与教育家关注的问题。由于公众争论的升级,当时的政府委托哈多爵士负责进行教育调查,哈多委员会在1926年、1931年和1933年,发表了3个非常重要的报告。这些报告带来初等教育的重大改变。例如,导致了5-7岁儿童的幼儿学校和7-11岁的小学被分开实施。报告还建议班级规模不超过30人。这些建议标志进步主义教育思想胜过了传统教育思想,更多为政策制定者和教师们所喜爱。","text2":"1918年教育法的主要内容是什么?","label":1} {"text1":"Ubuntu Studio 是一个Ubuntu官方认可的Ubuntu作业系统衍生版本 。这个版本主要是为一般的多媒体制作而设计。原始的版本是基于Ubuntu 7.04,在2007年5月10日发布。Ubuntu Studio 8.04包含的即时内核(real-time kernel)是专门为繁重的影音、绘图作业修改的。排程器使得应用程式可以取得立即的CPU时间,大大地减少音频延时 。 8.10版的发布则没有这个即时内核。Ubuntu Studio同时包含了一个艺术性主题,黑底蓝按键的主题与ubuntu本身的棕桔色形成了鲜明的对比。Ubuntu Studio目前已有Live CD版本,因此使用者可以透过图形化安装介面来进行安装。此外,映像档的大小约是2.7GB,一般的CD容纳不下,因此通常是由DVD安装。Ubuntu Studio也可以透过APT从网路安装。","text2":"Ubuntu Studio 是什么?","label":1} {"text1":"卢世荣(),名懋,字世荣,出生于大名府,元朝中书省官员。卢世荣原为中书平章政事阿合马的属下。阿合马死后,元廷因财政需求需要继任者,总制院使桑哥于是“荐卢世荣有才术,谓能救钞法,增课额,上可裕国,下不损民”。元世祖下令召见,“奏对称旨”,而且卢世荣在与中书省右丞相和礼霍孙等人的廷辩中取得胜利,遂任命卢世荣为中书右丞,并任命他所推荐的史枢为中书左丞,而安童则重新起用中书右丞相。他上任当天即开始整治钞法,“官吏奉行不虔者罪之”。他还重新起用了一些已被罢黜的阿合马重臣。为增加自己在朝廷中的力量,他奏升六部为二品官衔,得到元世祖的批准。不过,以皇太子真金为首的儒臣派不满卢世荣“以言利进”的主张。真金说:“财非天降,安得岁取赢乎。恐生民膏血,竭干此也。岂惟害民,实国之大蠹”(《元史·裕宗传》)。与此同时,卢世荣的理财措施有许多因触犯了富豪势要的利益,导致难以实行。次年(1285年),监察御史陈天祥上章弹劾他过去有贪赃劣迹,而且上任后的措施多无成效。元世祖于是下令安童集诸司官吏、老臣、儒士及知民间事者,与卢世荣和同赴上都对质和审问。卢世荣在审问中承认了一些错误,如“不白丞相安童,擅支钞二十万锭;擅升六部为二品;不与枢密院议,调三行省万二千人置济州,委漕运使陈柔为万户管领”等。元世祖于是下令将他收押下狱。同年十一月,卢世荣被处死。","text2":"卢世荣做了什么去增加自己在朝廷中的力量?","label":1} {"text1":"澄海县是中国广东省历史上的一个县,始置于明嘉靖四十二年正月丁未即正月初十(1563年2月20日),由潮州府海阳县的上外莆、中外莆、下外莆等三都,揭阳县的蓬州、鮀江、鳄浦等三都,饶平县的苏湾都组成,县名取“澄靖海氛”或“海宇澄清”之意,治所设于下外莆都之辟望村(今澄海区中心城区),大致范围为今汕头市金平区、龙湖区、澄海区三区(现隆都镇和莲华华镇部分是在新中国成立划归澄海县的)。澄海县历史上曾三度废止,第一次由于康熙时期在东南沿海颁布的“迁界令”,于康熙五年(1666年)六月迁界,撤县并入海阳县,康熙八年(1669年)三月复置,其后于1959年1月也曾撤县并入汕头市郊区直至同年11月复置。1994年4月18日,澄海县撤销,原县域设立澄海市(县级,之后改制为澄海区)。澄海县亦为潮州八邑之一。这里也是泰国吞武里王朝建立者、达信大帝郑信的家乡。澄海现在有著名玩具实业:伟宝实业、著名教育机构:金叵罗教育","text2":"澄海县的治所在哪里?","label":1} {"text1":"17173网是一家位于中国大陆,旨在提供电子游戏及电子游戏产业类新闻与资讯的网站。同时17173网还提供供游戏玩家交流的论坛、视频分享及下载服务等内容。2001年3月,17173网成立。最初网站名为“网游天下”,且为个人网站。2001年5月,17173网被网龙公司收购,收购金额不详。2003年10月,搜狐斥资2050万美元收购17173网。2004年12月,17173网推出首届17173游戏动漫嘉年华活动。2005年5月,17173网标志和吉祥物诞生。2007年3月,17173网发起首届世界网游日。2008年5月,17173网举行首届中国网游竞技大赛。2011年12月,畅游以1.625亿美元收购17173.COM。自畅游收购17173网后,有分析师认为17173网在游戏厂商广告兼具双重身份或将导致广告主的放弃, 同时也引发业内对于17173网未来公正性和客观性的质疑。2013年5月15日,17173员工猝死曾引发过有关“过劳死”的讨论,但事后证明员工猝死与17173无关。2006年期间,17173曾因旗下产品涉嫌侵权而被天空游戏网诉至法庭,但并无下文。","text2":"17173网的创办宗旨是什么?","label":1} {"text1":"浙江春晖中学位于浙江省绍兴市上虞区白马湖畔,为浙江省一级重点中学。其由经亨颐策划,当地富商陈春澜捐资创建,创办于民国十年(1921年)。二十年代初,朱自清、夏丏尊、丰子恺等曾执教于此。现今春晖中学现有65个班级,在校师生3400余人(包括春晖外国语学校),校园面积35万平方米,建筑面积约10.8万平方米。校内早期建筑包括一字楼、科学馆、图书馆、曲院等,各建筑间均有长廊相连,保存较好,同时在学校的北门外侧还有当年经亨颐校长所居“长松山房”,丰子恺所居“小杨柳屋”,夏丏尊所居“平屋”等建筑。早年春晖中学曾有夏丏尊、朱自清、朱光潜、丰子恺、刘薰宇、张孟闻、范寿康等名师先后执教,开创了浙江中学届男女同校的先河,有“北南开,南春晖”之说。1919年12月2日,成立私立春晖中学校董会。1920年1月,春晖中学校董会推经亨颐为首任校长,定址白马湖。2008年10月18日,一百周年校庆。春晖中学主页","text2":"浙江春晖中学位于何地?","label":1} {"text1":"大卫·克劳斯(,)是一名德国男演员。大卫·克劳斯出生在德国汉堡以北的小镇Henstedt-Ulzburg。他的第一部电影是2002年的《变身一团糟》(Hilfe, ich bin ein Junge)。在2006年和2008年相继主演了电影《黑道新鲜人》(Knallhart) 和Krabat。2008年,大卫·克劳斯在电影《为爱朗读》中扮演男主角,15岁的男孩米歇尔·伯格,在二战后的德国,爱上了一位36岁的妇女,汉娜·舒密士。克劳斯还扮演7年后的米歇尔·伯格,22岁的法律系学生,出席汉娜·舒密士的战争罪行审判。出演该片的大部分演员都是知名的老资格明星,而克劳斯只有17岁,不为人知,而且只会有限的英语,必须学习英语,才能在这部完全使用英语的电影中演出。","text2":"大卫·克劳斯的第一部电影是什么?","label":1} {"text1":"卡尔-奥古斯特·威廉·恩斯特·弗里德里希·格奥尔格·约翰·阿尔布雷希特(\"Carl-August\" Wilhelm Ernst Friedrich Georg Johann Albrecht,),萨克森-魏玛-艾森纳赫大公储,1923年起为萨克森-魏玛-艾森纳赫大公家族族长。卡尔-奥古斯特是萨克森-魏玛-艾森纳赫末代大公威廉·恩斯特与夫人萨克森-迈宁根的费奥多拉的儿子,1912年7月28日生于巴伐利亚王国上弗兰肯行政区的威廉斯塔尔()。一出生便成为萨克森-魏玛-艾森纳赫的大公储,同时还是荷兰王位第三顺位继承人。1918年,德国爆发革命。卡尔-奥古斯特的父亲在革命中于被推翻,卡尔-奥古斯特失去了继位大公的机会。随着1922年荷兰修宪,他也失去了继承荷兰王位的机会。卡尔-奥古斯特的父亲于1923年去世,他继承父亲成为萨克森-魏玛-艾森纳赫大公家族族长,韦廷家族首领。若德国仍是君主制国家,他将是萨克森-魏玛-艾森纳赫大公卡尔·奥古斯特二世。卡尔-奥古斯特于1988年10月14日在巴登-符腾堡州博登湖畔的希恩嫩去世,终年76岁。他的长子米夏埃尔-本尼迪克特继位成为家族族长。卡尔-奥古斯特1944年10月5日在瓦尔特堡与汪根海姆-温特尔施泰因女男爵伊丽莎白(Elisabeth Freiin von Wangenheim-Winterstein)结婚,有一子二女:","text2":"哪一年德国爆发革命?","label":1} {"text1":"科斯岛战役(Battle of Cos),发生在公元前258年,是马其顿王国与托勒密王国在爱琴海的一场海战。在爆发第二次叙利亚战争后,马其顿国王安提柯二世与塞琉古国王安条克二世决议成为同盟,一同对付托勒密王国的托勒密二世。此时,托勒密埃及的海上势力几乎笼罩爱琴海各岛屿,拥有当时爱琴海的制海权,还拥有一支约300艘且主要由腓尼基人所组成的舰队,同时也和海军强国罗德岛同盟。托勒密的舰队先前在第一次叙利亚战争发挥很大的作用,在克里莫尼迪兹战争时,托勒密的舰队还封锁安提柯二世的沿岸。为此,安提柯二世在科林斯重新建造一支舰队。科林斯向来以建造大型战舰著名,安提柯二世在那建造许多的大型战舰,并在船只设计上强化登舰作战的能力,命名为「科林斯舰」。安提柯二世还与安条克二世成功说服罗德岛转变立场,一同对抗托勒密二世。然而托勒密的舰队仍是相当强大,尽管于公元前259年,罗德岛的舰队曾在以弗所附近击败一支埃及海军分队。因为大部份关于第二次叙利亚战争的资料均已遗失,对科斯岛战役发生的年代不是很确定。时间主要是从战役当年曾举办地峡运动会所推得出来的。两军相遇在东爱琴海的科斯岛附近,战争详细不清楚,已知安提柯二世亲自率领马其顿舰队,且埃及的舰队几乎是他的两倍。尽管如此,在安提柯付出惨烈的代价后嬴得这场胜利。战后,安提柯在许多方法庆祝这场胜利,更在爱琴海的新领土建造纪念物。此战使托勒密王朝丧失许多爱琴海岛屿,使第一次叙利亚战争所获得的疆土几乎丧失,削弱了托勒密的海上军力。","text2":"谁的海上势力几乎笼罩爱琴海各岛屿,拥有当时爱琴海的制海权","label":1} {"text1":"《伯爵与妖精》()是谷瑞惠所写的日本轻小说系列作品,发行于集英社的Cobalt文库。其插图由高星麻子绘画。此作品在2007年6月和2008年3月,发行了广播剧CD。另外,漫画版从2008年10月号开始在「The Margaret」上连载,单行本全4卷。电视动画在日本从2008年10月播放至同年12月。另外,由5pb.所发售的游戏也于2009年4月发售。故事发生在19世纪(维多利亚时代)的英格兰。主角是17岁的少女莉迪雅・克鲁顿。她可以和妖精说话,因此被称为「妖精博士」。为了和父亲共度复活节,她和伙伴尼可一起乘船前往伦敦。在船上他们遇到了自称是传说中的「青骑士伯爵」的美貌青年爱德格。爱德格为了得到宝剑,而向莉迪雅寻求帮助。莉迪雅虽然觉得爱德格不大可信,但还是听信了他的花言巧语,加入他的队伍。从此莉迪雅被卷入了围绕宝剑的争夺之中。广播剧CD和电视动画的配音员相同。在日本从2008年10月开始在日本千叶电视台、独立U局、AT-X播放。","text2":"哪个出版社发行的《伯爵与妖精》?","label":1} {"text1":"《回到未来第三集》()是一部经典美国科幻、喜剧、西部电影,回到未来系列的第三集及最后一集,于1990年5月25日上映,由米高·J·霍士、基斯杜化·莱特、玛丽·史汀柏格及汤马士·法兰西斯·威尔逊主演。电影在美国加利福尼亚州和亚利桑那州拍摄。马帝(由米高·J·霍士饰演)与博士(由基斯杜化·莱特饰演)的三度时光旅行,这次来到1885年的老西部时代,是《回到未来》三部曲完结篇。在第二集的结尾,马帝依据博士在七十年前留下的信,马帝和1955年的博士找到了藏在矿穴中的时光机。马帝无意间发现在1885年的博士在写信后不久就被杀,于是又乘著时光机赶往1885年9月2日。马帝成功抵达,但油箱却不慎破损。马帝遇上了他的祖先谢默斯并受到照顾,他化名“克林特·伊斯特伍德”来到了建立之初的小镇,却遇上了毕夫的祖先“疯狗”布福德·谭能(Buford Tannen),并受到迫害。在危急之际,博士救了马帝。博士得知了将要发生的事情并准备和马帝一起回到未来,但油箱的破损却使时光机无法行驶到所需的速度。博士想出了利用火车的方法,并在实地勘测时救下了女教师克莱儿,两人一见钟情。隔天,在晚会上,马帝从谭能手下救出了博士,但他却接受了与谭能决斗的挑战。博士与喜欢科幻小说的克莱儿感情日深,但必须回到未来一事却使他颇伤脑筋。在回到未来的前一天夜里,博士去与克莱儿告别。在克莱儿的追问下,博士说出了时间旅行的秘密,却被克莱儿误解。博士万念俱灰,前往酒馆欲借酒浇愁,而伤心的克莱儿也要离开小镇。马帝醒来后找到了博士,但正要离开时博士却因醉酒而昏睡,使马帝面临与谭能的决斗的危机。马帝弄醒博士后想逃之夭夭,但博士却落入了谭能的手里。马帝急中生智,利用铁板护身,打败了谭能。马帝和博士开始实行他们的计划,而克莱儿在火车上得知了博士对她的真情后,也赶来寻找博士。博士的计划顺利实施,但克莱儿赶到后却陷入了危境。为了救克莱儿,博士错过了回到未来的时机。时光机成功地把马帝送回了1985年,但却不幸被火车撞毁,马帝无法再去接回博士。他找到了女友珍妮佛和她一同出游,由于时间旅行的经历改变了马帝经不起刺激的个性,使马帝避免了未来因车祸导致的灾难。马帝和珍妮佛来到时光机被毁的地方,对博士无法回来而感到忧伤,但就在此时,博士却驾著新的时光机出现在马帝和珍妮佛面前。他已经和克莱儿结了婚并有了两个小孩。","text2":"《回到未来第三集》什么时候上映的?","label":1} {"text1":"《伊斯坦布尔:一座城市的记忆》(土耳其文:İstanbul: Hatıralar ve Şehir)大体上是奥尔罕·帕慕克一部忧伤的自传回忆录,于2005年被译成英文,同年获得德国书业和平奖。该书是一段撼动了整个土耳其的巨大文化变迁的记录——现代化与不断退却的传统之间的斗争,它也是一逝去家庭传统的挽歌,更多的,它还是一本博斯普鲁斯,以及伊斯坦布尔和这海峡之间历史的书。动手写此书时,帕慕克处于抑郁症爆发的边缘。在一次访谈中,他说:“当时我的生活,因为很多事情,处在一场危机之中;细节就不多说了:离婚,父亲去世,职业上的问题,这个问题,那个问题,所有的事都很糟糕。如果我软弱的话,一定会得抑郁症的,但每天我起来洗个冷水澡,坐下来,回忆然后写作,永远记得要把这本书写得美。”他的家庭对这本书很是不满,特别是他的哥哥。帕慕克说因为这本书他失去了他的哥哥,他还承认这本书也伤害了他母亲的感情。书中作者个人的回忆和与伊斯坦布尔紧密相关的作家、艺术家交织在一起。书中有一整章献给了十九世纪制作过君士坦丁堡版画的西方艺术家梅林。帕慕克本人最喜欢的伊斯坦布尔作家,他灵感的来源,雅哈雅、科丘、希萨尔和坦皮纳都被他写入了书中。他最钟爱的西方行旅作家对他而言,就像是他个人的奈瓦尔、歌德和福楼拜。本书所采用的插图大部分是由古勒拍摄的,由帕慕克亲自挑选的。帕慕克认为古勒的照片有一种忧伤的韵味。此外,帕慕克还加入了一些家庭照片。","text2":"本书所采用的插图大部分是由谁拍摄?","label":1} {"text1":"范廷颂枢机(,),圣名保禄·若瑟(),是越南罗马天主教枢机。1963年被任为主教;1990年被擢升为天主教河内总教区宗座署理;1994年被擢升为总主教,同年年底被擢升为枢机;2009年2月离世。范廷颂于1919年6月15日在越南宁平省天主教发艳教区出生;童年时接受良好教育后,被一位越南神父带到河内继续其学业。范廷颂于1940年在河内大修道院完成神学学业。范廷颂于1949年6月6日在河内的主教座堂晋铎;及后被派到圣女小德兰孤儿院服务。1950年代,范廷颂在河内堂区创建移民接待中心以收容到河内避战的难民。1954年,法越战争结束,越南民主共和国建都河内,当时很多天主教神职人员逃至越南的南方,但范廷颂仍然留在河内。翌年管理圣若望小修院;惟在1960年因捍卫修院的自由、自治及拒绝政府在修院设政治课的要求而被捕。1963年4月5日,教宗任命范廷颂为天主教北宁教区主教,同年8月15日就任;其牧铭为「我信天主的爱」。由于范廷颂被越南政府软禁差不多30年,因此他无法到所属堂区进行牧灵工作而专注研读等工作。范廷颂除了面对战争、贫困、被当局迫害天主教会等问题外,也秘密恢复修院、创建女修会团体等。1990年,教宗若望保禄二世在同年6月18日擢升范廷颂为天主教河内总教区宗座署理以填补该教区总主教的空缺。1994年3月23日,范廷颂被教宗若望保禄二世擢升为天主教河内总教区总主教并兼天主教谅山教区宗座署理;同年11月26日,若望保禄二世擢升范廷颂为枢机。范廷颂在1995年至2001年期间出任天主教越南主教团主席。2003年4月26日,教宗若望保禄二世任命天主教谅山教区兼天主教高平教区吴光杰主教为天主教河内总教区署理主教;及至2005年2月19日,范廷颂因获批辞去总主教职务而荣休;吴光杰同日真除天主教河内总教区总主教职务。范廷颂于2009年2月22日清晨在河内离世,享年89岁;其葬礼于同月26日上午在天主教河内总教区总主教座堂举行。","text2":"范廷颂是于何时何地出生的?","label":1} {"text1":"德国BR45型蒸汽机车是一种货运机车,亦是德国“标准蒸汽机车”之一。45型机车是德国有史以来制造过的功率最大的蒸汽机车。该型机车由汉寿尔工厂于1936到37年之间生产,在1940年之前全部入役。本来还有103辆45型机车的订单,但这些订货于1941年取消。这是由于二战爆发后德国机车工业集中生产力生产类似于BR52这样的简化版“战争机车”。45型机车共有28辆,编号从45 001 -到45 028。战后,BR45型机车开始频繁出现由于超压而造成的锅炉损害。为此,德国联邦铁路从1950年起为45 010, 45 016, 45 019, 45 021以及45 023 等机车安装了有燃烧室的下火箱以及下给加煤机。东德也对45型进行了改造,不过他们的方法是提高锅炉压力。然而这种改造没有收到预期的效果,并且使这些机车在1959年就早早退役(东德有很多蒸汽机车都使用到了7、80年代)。45型机车其实从设计上就有问题,主要是锅炉结构十分脆弱,就像之前的06型。在锅炉和主要部件更换以后,该型机车的真正实力才显现出来。在45型使用的最后阶段,其主要任务就是在44型难以担当的重型货运中发挥重要作用。到1968年,德国联邦铁路仅有3辆45型仍在使用(45 023 、45 010、 45 019),这些机车编为045型。45型退役后只有45 010号保存了下来,被收藏于纽伦堡交通博物馆。2005年,该馆发生严重火灾,这唯一的45型在大火中严重损毁。目前,有关方面正计划对该车进行修缮。","text2":"45型机车共有多少辆?","label":1} {"text1":"《沙乡年鉴》或译《沙郡年记》是由美国的生态学家和奥尔多·利奥波德写的一本非小说类书籍(1949年)。它描述了利奥波德在威斯康辛州索克县的家周围的土地和他关于发展一种“土地伦理”的思考,在利奥波德因心脏病发作而死后一年,他的儿子把它整理出版了。这本短文合集被视为美国中划时代的书。该书印刷超过两百万册,已被译为九种语言。它活跃和改变了环境运动,并促进了作为一门科学的生态学的广泛影响。《沙乡年鉴》是自然历史、以文写景与哲学的结合。也许最使它著名、且定义了他的土地伦理的是下列引文:“当一个事物有助于保护生物共同体的和谐、稳定和美丽的时候,它就是正确的,当它走向反面时,就是错误的。”营养级的概念是“像山那样思考”一节提出的,在其中利奥波德意识到,杀死一只掠食动物——狼,会对生态系统的其余部分产生重大的牵连影响。在最初的版本中,该书是以真正的“沙乡年鉴”作为开端的,它分为十二节,每节对应一个月。其中有有关植物区系和动物区系对季节作出的反应的轶事与观测,此外还提及了保护的主题。书的第二部分“随笔——这儿和那儿”探讨加拿大、墨西哥和美国的其他几处荒野,以及保护(或保护的缺乏)是如何影响它们的。全书以“土地伦理”结尾,其中利奥波德在“生态学意识”一节中深入探究了资源保护。他写道:“资源保护是人和土地之间和谐一致的一种表现。”利奥波德觉得,通常认为需要更多的教育,然而其分量和内容值得讨论。他相信,土地不是一个被支配的日用品,相反地,人为了不破坏它,必须与地球有相互的尊重。他还提出了这样的思想:人若无野性之处可供徜徉则永无自由。利奥波德之家——于1978年入选美国国家史迹名册(U.S. National Register of Historic Places)。","text2":"在《沙乡年鉴》中利奥波德坚信什么理念?","label":1} {"text1":"《阴道独白》()是美国女作家伊芙·恩斯勒()所著的戏剧,1996年由作者本人首演于纽约外百老汇,获1997年奥比奖最佳剧本奖。该剧至今至少已被翻译成50种语言,在140个国家上演过。该剧至少已有5个不同的中文译本。《阴道独白》2004年曾在北京和上海被双双禁演;但几年来,该剧已在中国大陆的至少十所高校被未经授权的学生搬上过舞台。中国大陆的授权中文首演是2009年3月薪传实验剧团在北京9个剧场的5场公演,由新浪潮戏剧人王翀翻译并导演,至今已在北京、上海、深圳、长沙、杭州、天津演出30余场,全部提前售罄,引起凤凰卫视、《北京青年报》、《青年周末》、《时代周刊》、《侨报》、《南方都市报》、《南方周末》等中美媒体的关注。美国《时代周刊》将中国大陆的首演评价为“一系列的售罄演出,造成轰动。”磨铁图书有限公司曾经签下王翀译本的版权,但因为无法通过图书审查,至今未能出版。V日义演是《阴道独白》的作者恩斯勒发起的反对对妇女暴力的活动。这个全球性的活动于2010年已经进入了第12个年头,已经筹资超过7000万美元。《阴道独白》V日义演规则:2013年11月7日,北京外国语大学17名女生为了宣传《阴道独白》在校园演出,在人人网上发布一组持牌照片,牌子上写着“我的阴道说:我要自由”等以“我的阴道说”开头的文字,引起中国社交网站上许多负面评价。","text2":"《阴道独白》获得过什么奖项?","label":1} {"text1":"238号州际公路(Interstate 238,简称I-238),是美国的一条州际公路,全线位于加利福尼亚州旧金山湾区,是加州高速及快速公路系统的一部份。该公路自圣利安卓880号州际公路分出后向东,于卡斯楚谷衔接580号州际公路,全长。虽然公路是东西走向,标志却以南北向标记。由圣利安卓往卡斯楚谷记为南向,反之为北向。238号州际公路是加州中谷地区通往奥克兰港的主要道路,由于580号州际公路在卡斯楚谷至奥克兰路段不准货车通行,所有货车必须由238号州际公路转880号州际公路进出奥克兰,因而造成该公路常有拥塞情形。I-238在1983年被纳入州际公路系统。但这个编号并不遵守州际公路系统的编号规则(38号州际公路并不存在),如果依规则,这条公路的尾数应为80,目前在加州地区未使用的编号有I-180和I-480,但180号加州州道仅离本路段约100哩(在弗雷斯诺附近),480号加州州道计划在1991年才取消(原路线位于旧金山),为避免用路人混淆,加州公路管理局决定暂不使用这二个编号,而使用公路升级前的编号238。238号州际公路在1983年以前是238号加州州道的一部份,238号加州州道大体上是南北走向,此乃238号州际公路被编为南北向的原因。I-238在与I-580衔接处有另一出口继续接往238号加州州道向南通往海沃德。238号州际公路在1956年升级成为高速公路,但在1964年以前,这个路段并未指定任何编号。原本的计划是在费利蒙境内的238号加州州道至680号州际公路段亦会升级成高速公路(与880号州际公路平行),但在1968年向联邦申请未果,且数度上诉失败后,这个计划无疾而终。在880号州际公路被纳入州际公路系统的同时,这段路亦被纳入系统。美国州公路和运输官员协会在1983年7月7日批准了I-238和I-880这二个编号。I-238二端的系统交流道随即开始改建,加入缺少的I-238往I-880南向的匝道(原本以作为联络道之用)。1990年代,随著湾区捷运都柏林/普莱森顿─密尔布瑞线的动工,I-238的分隔岛交由捷运局作为舖设铁道之用。2009年,全线拓宽为六线道,拥塞情形稍有改善。\"注意:哩程数前方若无字母,表示1964年的量测值,由于路线有稍做更动,哩程可能不准确。\"238号州际公路全线位于阿拉米达县。","text2":"该公路为什么常有拥堵情况?","label":1} {"text1":"李重进(),沧州(今河北省沧州市)人,生于太原,五代时后周禁军统帅之一,太祖郭威第四姊福庆长公主之子。李重进在后晋天福中,入仕为殿直。后汉初年,随舅舅郭威征讨于河中。后周建立的广顺元年(951年),太祖郭威以李重进为内殿直都知、领泗州刺史,女婿张永德为内殿直小底四班都知;又升李重进为小底都指挥使,而以张永德接任内殿直都知。翌年(952年),郭威以李重进为大内都点检兼马步都军头,张永德为小底第一军都指挥使;后又以李重进为殿前都指挥使,张永德为殿前都虞候,掌管殿前亲军。两年后,郭威病危,传位于发妻柴氏的侄子、养子柴荣,临终前特命李重进向柴荣行君臣之礼,以免其觊觎皇位。显德元年(954年),世宗柴荣即位,以姑表兄李重进为侍卫亲军马步军都虞侯,妹夫张永德接任殿前都指挥使,分掌侍卫亲军和殿前亲军。李重进、张永德本以姻亲之故,在数年间不次擢升,但后来都在战争中展现出过人的军事才能。在决定后周生死存亡的高平之战后,李重进以战功加使相衔,升侍卫亲军都指挥使,母福庆长公主追封燕国大长公主;而张永德以战功加检校太傅,授义成军节度使,妻寿安公主进封晋国长公主。北宋建隆元年(960年),宋太祖赵匡胤即位,加李重进为中书令,命令韩令坤代替李重进,将重进移镇至青州(治所在今山东省青州市),李重进拒绝调动,派遣幕僚翟守珣说服李筠起兵抗命,翟守珣却将此事泄露给宋太祖,于是太祖要求翟守珣拖延李重进出兵,以防止李重进与李筠南北呼应。翟守珣回去后,向重进诋毁李筠不足与谋事,重进果然中计,错失良机。李筠四月起兵反宋,六月兵败,自焚死。同年九月李重进起兵,十月,太祖亲征,带领石守信、王审琦、李处耘平叛,十一月,到达扬州城下,即日入城,李重进举家自焚。","text2":"郭威病危,为什么特命李重进向柴荣行君臣之礼?","label":1} {"text1":"宋少帝陵,早期称宋少帝墓,是南宋最后一个皇帝宋少帝赵昺的陵墓,位于中国广东省深圳市南山区的赤湾小南山上,少帝路、赤湾公园附近。宋少帝陵是广东省境内少数的一座皇帝陵寝之一 。广东省内另有永福陵(宋端宗墓,即宋帝昰,乃宋帝昺的兄长)位于江门市,及南汉二陵位于今广州市,时称兴王府,是深圳市第一批重点文物保护单位。景炎三年(1278年),宋端宗病死,当时陆秀夫在𥐻州梅蔚(今香港大屿山梅窝)拥立赵昺做皇帝,改元「祥兴」,并逃往崖山。元朝命令大将张弘范大举进攻崖山的赵昺小朝廷。宋军水师在张世杰的指挥下进行顽抗,在崖门海域里与元朝军队交战,史称「崖门战役」,这场战役关系到南宋流亡小朝廷的兴亡,结果宋军全军覆灭。西元1279年3月19日,丞相陆秀夫见大势已去,便背着这位刚满八岁的小皇帝赵昺跳海殉国,两人壮烈牺牲,宋王朝灭亡。据《赵氏族谱》记载,“后遗骸漂至赤湾,有群鸟飞遮其上。山下古寺老僧偶往海边巡视,忽见海中遗骸漂荡,上有群鸟遮居,设法拯上,面色如生,服式不似常人,知是帝骸,乃礼葬于本山麓之阳”。而民间传说,当时赤湾海滩漂来一具身著黄袍龙衣的童尸,而赤湾海边天后庙(即今天位于赤湾的天后博物馆)的一根栋梁却突然塌下,庙祝与乡绅父老急忙焚香问卜,得知童尸为少帝遗骸,塌下的栋梁是天后娘娘送少帝做棺材的材料,当地百姓于是礼葬赵昺于天后庙西边的小南山脚下。宋少帝陵墓碑刻有“大宋祥庆少帝之陵”,两旁有对联“黄裔于今延宋祀,赤湾长此筑皇陵”字样。墓前有祭坛和祭台,左右有两只石狮。墓的东侧立有一块1米宽、2米多高的泉州白石墓碑,碑文用篆体阴文,内容为“宋帝昺陵墓碑记”,为书法家商承祚所写。碑背面刻有“崖海潜龙,赤湾延帝”八个大字,为书法家秦萼生所书。西侧有花坛,坛旁建有四柱黄琉璃瓦顶休息亭。同时还有大理石碑刻的宋少帝陵的简介。在宋少帝陵入口处有“陆秀夫负帝殉海像”的雕塑,附近还有一个小卖部和几个焚香炉。墓的四周都有仿古的三角旗。1911年,香港赵氏后裔修葺宋少帝陵。1963年,赤湾守军发现了此陵墓。到了1984年初,香港赵氏宗亲会和蛇口工业区旅游公司,捐资四十多万港元,对陵墓进行修葺扩建,由原来的五十多平方米扩大到四千四百多平方米,并且开筑公路直达墓前,还增建一座陆秀夫背负少帝殉海塑像。","text2":"丞相陆秀夫和小皇帝是以什么方式殉国的?","label":1} {"text1":"广州市第一人民医院(简称市一医院)是中国广州市一所三级甲等医院。市一医院创立于清光绪二十五年(1899年)城西方便所。1952年12月31日,当局把广州方便医院和广州市市立医院合并组成“广州市人民医院”。1954年2月1日更名为“广州市第一人民医院”。1993年被卫生部评定为三级甲等医院,为广州市首批三甲医院,1997年定为广州医学院附属医院。广州方便医院的前身是由商人陈惠普等人发起自筹资金在清光绪二十五年(1899年)夏天所创立的“城西方便所”,光绪二十七年(1901年)更名为“城西方便医院”,中华民国三十七年(1948年)改称“广州方便医院”。中华人民共和国建政后,1950年被中共当局接管。不过,根据新加坡《新国民日报》1926年9月18日的报道,当时新加坡的商会和慈善家正在筹办筹款活动,号召南洋华侨援助面对经济困境的广州方便医院。因此,广州方便医院于1926年已经存在,而实际存在的年份,应该更早。广州市市立医院的前身是由广东巡警道在光绪三十七年(1907年)12月创立的“广东巡警医院”。民国二年(1913年)改名为“警察医院”。民国四年(1915年)改名为“广东医院”(隶属省署)。民国九年(1920年)9月划归广州市卫生局管辖。民国十年(1921年)4月改名为“广州市市立医院”,并划归市政厅管辖。1938年至1945年广州沦陷期间被关闭,抗日战争完结后医院被国民革命军陆军新编第一军三十八师野战医院驻扎。民国三十四年(1945年)10月收回复办市立医院,1949年10月被中共当局接管。","text2":"广州市第一人民医院创立于何时?","label":1} {"text1":"新西兰鸫鹟(\"Turnagra capensis\")是纽西兰的一种雀。新西兰鸫鹟已知有两个亚种,包括南岛指名的\"T. c. capensis\",及斯蒂芬岛较细小的\"T. c. minor\"。有指\"T. c. minor\"是在岛内独自演化的亚种,但牠们适于飞行及斯蒂芬岛与大陆相近的距离,令人很难相信此说法。不过,于1894年斯蒂芬岛上就有过百只鸟类,而\"T. c. minor\"似乎并不怎么活跃,很少在岸边出没。一直以来,新西兰鸫鹟都被认为与北岛的\"Turnagra tanagra\"是同一物种,但后来确实牠们是不同的物种。新西兰鸫鹟的体型中等,主要呈橄榄褐色,翼上及尾巴呈红褐色,胸部有斑点。牠们是纽西兰其中一种悦耳的鸣禽。牠们是杂食性的,相对地不怕人类。新西兰鸫鹟在纽西兰南岛曾经很普遍,直至1863年牠们就开始衰落。到了1880年代,牠们的数量大幅下降,主要原因是受到猫及大家鼠的掠食,并受到栖息地破坏的影响。1888年牠们在纽西兰已很稀有,1905年已接近灭绝。另外,亦有多次未确认的观察报告。新西兰鸫鹟的斯蒂芬岛亚种\"T. c. minor\"亦于1897年灭绝,同样是受到野猫的掠食所致。最后的标本是于1897年采集的,到了1898年就已没有再见到牠们。现时只有12个标本,分别存放在德累斯顿国家博物馆、伦敦的自然历史博物馆、利物浦世界博物馆、多伦多的安大略省皇家博物馆、匹兹堡的卡内基博物馆及不莱梅的海外博物馆。","text2":"为什么到了1880年代,新西兰鸫鹟的数量大幅下降?","label":1} {"text1":"詹姆斯·切斯特(,),是英格兰出生的威尔斯职业足球运动员,司职中后卫,出身曼联青训系统,现时效力英冠球会阿士东维拉。切斯特8岁时就被曼联邀请到青训营接受训练,2005年7月正式签约。2007\/08年赛季,他是曼联预备队出场最多的球员和后防绝对主力,因此进入一线队。2009年1月20日,他在联赛杯半决赛对阵德比郡的比赛中替补出场,第一次代表曼联参加正式比赛。2月2日被外借到英甲球队彼德堡一个月,期间上阵5场。2009年9月15日切斯特再获外借到英冠球会普利茅夫3个月,但仅作赛3场便因伤提前被送返曼联。2010年8月3日切斯特获外借到英甲球会卡素尔直到年底才完结。期间表现出色,在各项赛事上阵23场及射入3球,于借用期满前表示希望继续留队,于2011年元旦主场对哈德斯菲尔德近门顶入为球队扳平2-2完场。赛后曼联召回切斯特,转会窗重开后动作多多的英冠球会侯城提出约30万英镑罗致旗下。","text2":"切斯特被外借到英甲球队彼德堡一个月,上阵几场?","label":1} {"text1":"长洲北帝庙,又名长洲玉虚宫,位于香港离岛区长洲北社街北端,建于清朝乾隆四十八年(1783年),有二百多年历史,已获古物咨询委员会评定为一级历史建筑物,是全香港最具规模的北帝庙,除北帝外,庙内亦供奉太岁、观音及列圣。长洲北帝庙早期由长洲居民负责管理,于1929年交由华人庙宇委员会管理。长洲是原籍惠州、潮州及广州的渔民聚居地,他们信奉海神北帝祈求获得神明庇荫,使舟楫平安。据称,在乾隆四十二年(1777年)长洲发生疫症,当地渔民远赴惠阳县玄武山北帝庙迎请北帝神像到长洲镇压才除去疫症。因此,地方乡绅林煜武于乾隆四十八年(1783年)领导长洲原籍惠州、潮州居民集资建庙奉祀北帝。北帝庙是传统三进式建筑,正殿宽敞,神坛置中,两侧设偏殿,左右对称。殿前建有一道花岗石梯阶,门外置有一个石香炉及两对石狮子,而庙的瓦面和簷角,亦有很多狮子,成为全港庙宇中,狮子最多的一间。庙宇曾分别于道光二年(1822年)、道光十八年(1838年)、咸丰八年(1858年)、光绪廿九年(1903年)及1989年重修。最近一次由华人庙宇委员会斥资逾1,300百万港元进行重修,工程于1999年展开,直到2003年才竣工。北帝诞在每年的农历三月初三,善信敬拜祈求合境平安。而在每年一度的长洲太平清醮期间,长洲玉虚宫前地球场,会架设戏棚、抢包山等,好不热闹。庙内的古物珍品包括一柄早年由渔民网得奉于庙内的宋朝大铁剑及鱼骨、一座制于光绪二十五年的銮舆、一个咸丰十一年制的石香炉及乾隆四十九年的铜钟。此外,为纪念英国玛嘉烈公主及斯诺敦伯爵(Earl of Snowdon)伉俪于1966年到访,一位钟姓女士特别打造了一顶金冠以志盛况;而华人庙宇委员会于1959年铸造了一对铜香炉,纪念护督白嘉时于当年莅临参观。","text2":"长洲北帝庙建于何时?","label":1} {"text1":"九龙巴士69M线是香港新界的一条巴士路线,来往天水围市中心及葵芳站。九龙巴士69P线为69M的特别班次,于星期一至六上午繁忙时间单-{向}-由天耀开往葵芳站。全程:$11.1全程:$10.1乘客登上本线后150分钟内以同一张八达通卡转乘以下路线,或从以下路线登车后150分钟内以同一张八达通卡转乘本线,次程可获$4.2折扣优惠:本线使用4辆富豪B9TL12米、1辆亚历山大丹尼士Enviro 50012米(ATEU)及14辆富豪超级奥林比安12米,均属元朗车厂。天瑞开经:天恩路、天城路、天华路、天瑞路、天瑞巴士总站、天瑞路、天湖路、天耀路、天福路、朗天路、唐人新村交汇处、元朗公路、青朗公路、大榄隧道、屯门公路、青山公路—荃湾段、青山公路—葵涌段、葵涌道、葵富路、葵仁路及葵义路。葵芳站开经:葵义路、葵富路、葵涌道、青山公路—葵涌段、青山公路—荃湾段、屯门公路、汀九交汇处、大榄隧道、青朗公路、元朗公路、唐人新村交汇处、朗天路、天福路、天耀路、天湖路、天瑞路、天华路、天城路及嘉恩街。经:天耀路、天湖路、天城路、天福路、朗天路、唐人新村交汇处、元朗公路、青朗公路、大榄隧道、屯门公路、青山公路—荃湾段、青山公路—葵涌段、葵涌道、葵富路、葵仁路及葵义路。虽然本线只服务天水围新市镇内较早期落成的屋邨屋苑,范围不广,早年班次频密及路线直接是本线的优点,再加上乘坐本线前往大榄隧道转车站的转乘客亦有一定比例,因此客量曾经维持高水平,但九广西铁于2003年底通车后,导致本线及同日开办的64M线乘客量受到沉重打击,因此于2004年4月4日起64M线永久停驶,本路线增设天耀邨往葵芳站之分段收费,亦吸引部分天瑞邨、嘉湖山庄乘客步行至天耀邨乘搭本线以节省车资。经过路线重组,本线总站延长至天水围市中心后,繁忙时间仍有大量乘客乘搭本线。在下午的繁忙时间,巴士常常因满座而飞站。","text2":"九龙巴士69M线的特别班次是什么?","label":1} {"text1":"三斑天竺鲷,为辐鳍鱼纲鲈形目鲈亚目天竺鲷科的其中一种,俗名三线天竺鲷、大面侧仔。本鱼分布于西太平洋区,包括可可群岛、泰国、香港、日本、越南、马来西亚、新加坡、澳洲、菲律宾、印尼、巴布亚纽几内亚、马绍尔群岛、马里亚纳群岛、密克罗尼西亚、索罗门群岛、帛琉、东加、萨摩亚群岛等海域。该物种的模式产地在Buro。水深1-34公尺。本鱼体延长而侧扁,眼大,口大略下位。鱼体粉红色或橙色,体具褐色不规则斑纹,被栉鳞,鳞片具深色边缘,侧线明显,尾鳍截形,背鳍硬棘7枚;背鳍软条9枚;臀鳍硬棘2枚; 臀鳍软条8枚,体长可达14.2公分。本鱼栖息于近海珊瑚礁区,白天躲藏洞穴中,夜间出来觅食,属肉食性。繁殖期时,雄鱼具有口孵习性,卵约7日化成仔鱼,由雄鱼吐出,具短暂的仔鱼飘浮期。不具任何经济价值。2.FishBase","text2":"三斑天竺鲷一般是怎样繁殖的?","label":1} {"text1":"大卫·克劳斯(,)是一名德国男演员。大卫·克劳斯出生在德国汉堡以北的小镇Henstedt-Ulzburg。他的第一部电影是2002年的《变身一团糟》(Hilfe, ich bin ein Junge)。在2006年和2008年相继主演了电影《黑道新鲜人》(Knallhart) 和Krabat。2008年,大卫·克劳斯在电影《为爱朗读》中扮演男主角,15岁的男孩米歇尔·伯格,在二战后的德国,爱上了一位36岁的妇女,汉娜·舒密士。克劳斯还扮演7年后的米歇尔·伯格,22岁的法律系学生,出席汉娜·舒密士的战争罪行审判。出演该片的大部分演员都是知名的老资格明星,而克劳斯只有17岁,不为人知,而且只会有限的英语,必须学习英语,才能在这部完全使用英语的电影中演出。","text2":"大卫·克劳斯出生在哪儿?","label":1} {"text1":"乌卢鲁(意为「土地之母」),又被称为艾尔斯岩,位于澳大利亚北领地的南部,是在澳大利亚中部形成的一个大型砂岩岩层,位于乌鲁汝-卡塔楚塔国家公园 之内。乌鲁汝位于离它最近的大镇爱丽丝泉南部的335公里处,连接两地的公路长450公里。乌鲁汝是(该地区的澳大利亚原住民)的圣地。乌鲁汝被列为世界遗产。乌卢鲁位于澳洲的近乎地理中央处,是世界上仅次于奥古斯特山的第二大单体岩石。由于形成乌卢鲁的砂岩含有较多铁粉,氧化之后的铁粉呈现红色,使得乌卢鲁呈现红色外观。原住民在距今1万年之前已经开始在乌卢鲁一带居住。这里发现的壁画遗迹中历史最久的距今已有超过1000年的历史。乌卢鲁现在由原住民组织所有,在法律上该组织将乌卢鲁一带的土地借给澳洲政府至2084年。现在进入乌卢鲁一带地区需要支付25澳元的入场费。","text2":"原住民最早何时在乌卢鲁居住?","label":1} {"text1":"中线天竺鲷(学名:)为辐鳍鱼纲鲈形目鲈亚目天竺鲷科的一种,俗名开银天竺鲷。本鱼分布于西北太平洋区,包括日本、中国、菲律宾、香港、台湾等海域。该物种的模式产地在日本Kii。水深0-50公尺。本鱼体延长而侧扁,眼大且大于吻,口大略下位。体上半部为银褐色,下半部为银白色,体侧中央有一深色纵带从吻不一直延伸至尾鳍,尾鳍略凹。背鳍硬棘7枚;背鳍软条9枚;臀鳍硬棘2枚;臀鳍软条8枚,体长可达8公分。本鱼栖息于礁石区,白天躲藏于洞穴中,夜间出来觅食,属肉食性,以小型无脊椎动物或小鱼为食。繁殖期时,雄鱼具有口孵习性,卵约7日化成仔鱼,由雄鱼吐出,具短暂的仔鱼飘浮期。不具经济价值。2.FishBase","text2":"中线天竺鲷的俗名是什么?","label":1} {"text1":"萨克森-迈宁根(Sachsen-Meiningen)是韦廷家族在图林根的一个邦国。1680年由萨克森-哥达分裂产生。1871年加入德意志帝国。1918年德国君主制覆亡,1920年成为魏玛共和国图林根州的一部分。萨克森-哥达公爵兼萨克森-阿尔滕堡公爵恩斯特一世有12个儿子,其中有七人活到成年。1675年,恩斯特一世去世,他活着的最年长的儿子弗里德里希继承了他的领地。但遭到其他儿子的反对。1680年,弗里德里希一世与六个弟弟通过协议瓜分了父亲的领土。其中恩斯特一世的第六子博恩哈德得到了迈宁根,并于次年称萨克森-迈宁根公爵。随着1699年萨克森-科堡和1710年萨克森-勒姆希尔德的绝嗣,萨克森-迈宁根扩大了国土。1735年,萨克森-科堡的松讷贝格归于萨克森-迈宁根,1753年,勒姆希尔德的三分之二归于萨克森-迈宁根。19世纪初,萨克森-迈宁根先后加入了莱茵邦联和德意志邦联。1825年,萨克森-哥达-阿尔滕堡绝嗣,归于萨克森-科堡-萨尔费尔德和萨克森-希尔德布尔格豪森。1826年,恩斯廷系诸邦国领地重新划分。萨克森-迈宁根得到了希尔德布尔格豪森、萨尔费尔德、坎堡、克拉尼希费尔德等领地,领土大大增加。1829年,萨克森-迈宁根公国实施宪法。1833年,图林根诸邦加入德意志关税同盟。1866年的德意志内战中,萨克森-迈宁根与奥地利帝国和巴伐利亚王国结盟,与普鲁士王国作战。结果是普鲁士王国获胜,萨克森-迈宁根公爵博恩哈德二世被迫逊位于其子格奥尔格二世。萨克森-迈宁根成为北德意志邦联的一个邦。1871年,萨克森-迈宁根加入德意志帝国。1918年德国革命推翻了君主制,萨克森-迈宁根公国成为萨克森-迈宁根自由州,并于1920年并入图林根自由州。1826年后的萨克森-迈宁根公国分为四个县:","text2":"萨克森-迈宁根什么时候加入德意志帝国?","label":1} {"text1":"《决战玄武门》()是香港电视广播有限公司拍摄制作的古装武打剧集,全剧共20集,监制王天林。此剧于1985年10月下午,1988年4月及1990年10月中午在翡翠台重播。以下所有影碟发行均由电视广播(国际)有限公司授权:香港发行代理商现代音像(国际)有限公司于2001年推出发行了《决战玄武门》VCD影碟零售版本,此影碟将已播放的集数并制作成12只碟作为片段完整及无删剪版本,每只碟跟电视剧的每集片长约60分钟一样,设有粤语及国语发音版本并配上繁体中文字幕。台湾发行代理商弘音多媒体科技股份有限公司于2007年推出发行了《决战玄武门》DVD影碟零售版本,此影碟烧录VCD香港版的12只碟作为集数,设有粤语及国语发音版本并配上非隐藏繁体中文字幕。","text2":"《决战玄武门》共有多少集?","label":1} {"text1":"福井舞()是出身于京都府京都市的日本女创作歌手,23岁时出道,血型A型,所属唱片公司为J-more。2004年,与WADAGAKI、SHINO组合地下音乐队Poplar,发表了两张专辑,天照和梦死物语。在2006年时退出,2007年10月加入了Avex独立发展,2008年2月下旬自京都上东京。2010年8月20日,以单曲「爱之歌」初次亮相,同年9月同曲的原唱铃声下载数目超过50万。11月19日,作为第二张的单曲发表「Lucky」成为了初次亮相的契机的。使用小时学钢琴的琴进行作曲,据说作词是先以英语写完,之后转换成日语完成的,作词及作曲都是以「Maifukui」的名义发表。","text2":"2010年初次亮相时的单曲叫什么?","label":1} {"text1":"新加坡仙姑殿成立于农历壬申年十月二十三日(1992年11月17日),也正是仙姑下凡普渡众生的日子。原址是在后港3巷大牌251的住家庙,当时只供奉八仙,主神为女仙何仙姑,其余的七仙是 - 铁拐李、吕洞宾、张果老、曹国舅、汉钟离、韩湘子、蓝采和。由于仙姑的名声大振,慕名而来的香客源源不断,常把怎个地方挤得水泄不通。在短短的四年,这个办事处已经不能容纳上门祈求和膜拜的香客。以至在农历丙子年(1996年),仙姑决定把仙姑殿搬迁到芽笼25巷一栋两层楼高的私人建筑以方便她办事和应付更多香客。平时上门膜拜的香客已经占据了里外的仙姑殿,每年到了八月初八的仙姑寿诞日“八仙瑶池大会”,更要在德福巷大广场举行庆祝活动,以便应付千人宴席。从仙姑殿成立以来的10年里,她拥有许多信徒,香火鼎盛,因得到香客们的鼎力相助和支持,仙姑殿终于在农历壬午年六月十八日(2002年7月27日)正是搬迁到坐落在淡滨尼路一栋仙姑自己所构思和设计的5层楼高大殿。这栋耗资800万的新建庙宇宏伟壮观,以莲花为焦点,殿前的三个2.5公尺高的金字“仙姑殿”更受瞩目。殿内供奉高达十尺的主神何仙姑,和高达八尺的其余七仙。此外还有五尺和三尺高的其他道教天神、阴神、佛祖、拿督公。到仙姑殿祈求,也不可不看看殿内的三宝,虎虎生威的虎爷公、千斤黄铜制成的玉皇大帝香炉和一块6吨重的花岗石所围城的许愿池。","text2":"新加坡仙姑殿的原来地址是在哪里?","label":1} {"text1":"并州刺史部,是汉朝十三州刺史部之一,州治在晋阳县(今山西省太原市西南)。西汉州域范围大致是今日的山西省大部,东汉时扩大至陕西省北部及内蒙古自治区中部一带。「并州」一词的出现始于春秋以后,后人在《尚书·禹贡》九州基础上,增加幽、-{并}-、营3州,大体指常山郡以北的区域。《汉书·地理志》载:「正北曰并州。:其山曰恒山,薮曰昭余祁,川曰虖池、呕夷,寖曰涞、易;其利布帛;民二男三女;畜宜五扰,谷宜五种。」西汉时,领太原郡、代郡、上党郡、云中郡、雁门郡、定襄郡6个郡国。东汉时,领太原郡、上党郡、上郡、西河郡、五原郡、云中郡、雁门郡、定襄郡、朔方郡9个郡国,东汉末置新兴郡、乐平郡2郡。","text2":"并州刺史部是什么朝代的刺史部?","label":1} {"text1":"高鼻鱼,又称丝尾鼻鱼,俗名剥皮仔、打铁婆,为辐鳍鱼纲鲈形目刺尾鱼亚目刺尾鱼科的其中一个种。本鱼分布于印度太平洋海域,包括东非、红海、模里西斯、塞席尔、马尔地夫、留尼旺、马达加斯加、圣诞岛、罗德豪岛、斯里兰卡、安达曼群岛、日本、台湾、中国沿海、菲律宾、印尼、新几内亚、新喀里多尼亚、澳洲、新几内亚、马里亚纳群岛、马绍尔群岛、密克罗尼西亚、帛琉、索罗门群岛、斐济群岛、万那杜、夏威夷群岛、法属玻里尼西亚、诺鲁、吉里巴斯、吐瓦鲁、东加、复活节岛等海域。水深4至50公尺。本鱼体长卵形,侧扁;口小,端位,上下颌各具一列齿,齿稍侧扁且尖锐,两侧或有锯状齿。体呈黑褐色,体侧上半部及腹部有许多蓝色小点散布,下半部则有许多断续的蓝色横纹分布,两眼间有一蓝带相连,尾柄部有2个深色骨质板。而其最明显特征乃吻上方向前突出超过吻前端,尾鳍末端略突,上下叶延长如丝。雄性个体在生殖期体色会变得较淡,但尾鳍则为暗色。背鳍硬棘6枚、背鳍软条26至27枚、臀鳍硬棘3枚、臀鳍软条27至29枚。体长可达65公分。本鱼栖息于较深的礁湖或向海礁坡上,白天通常成群聚在中水层食动物性浮游生物。大鱼可食用,亦可为观赏用。鲜度差时,腥味重,可剥皮切片,沾芥末食之。","text2":"高鼻鱼什么形状?","label":1} {"text1":"斑柄天竺鲷(学名:)又称黄身天竺鲷,俗名大目侧仔,为天竺鲷科天竺鲷属的鱼类。该物种的模式产地在太平洋。本鱼分布于印度洋-西太平洋区,包括东非、南非、红海、塞席尔群岛、印度、斯里兰卡、缅甸、马来西亚、中国、台湾、日本、越南、印尼、菲律宾、巴布亚纽几内亚、澳洲、密克罗尼西亚等海域。水深1-30公尺。本鱼体延长而侧扁,眼大,口大略下位。鱼体呈金黄色,吻端至眼下具蓝色纵纹,尾柄处有一黑色圆形斑块。背鳍硬棘8枚;背鳍软条9枚;臀鳍硬棘2枚;臀鳍软条8枚;脊椎骨24个。体长可达12.5公分。本鱼生活于热带海域,喜群游于独立礁上半部之边缘区或大型礁洞中。属肉食性,以底栖甲壳类为食。繁殖期时,雄鱼具有口孵习性,卵约7日化成仔鱼,由雄鱼吐出,具短暂的仔鱼飘浮期。可食用,通常做下杂鱼处理,制成鱼粉当饲料,亦可当作观赏鱼。","text2":"斑柄天竺鲷一般分布在什么地方?","label":1} {"text1":"中国-苏丹关系是中国和苏丹之间的外交关系。苏丹于1956年独立。中华人民共和国与苏丹共和国于1959年2月4日建立外交关系。早在1964年,中国总理周恩来、副总理陈毅与苏丹总统易卜拉欣·阿布德()进行互访,最近的一次最高级别的互访为2006年奥马尔·巴希尔总统访问中国和2007年国家主席胡锦涛访问苏丹。中国自1970年开始向苏提供一定数量的经济援助,1971年起向苏丹开始提供医疗援助,1995年中苏二国开始合作开采石油项目,1997年两国外交部建立定期政治磋商机制。2007年中苏双边贸易额56.6972亿美元,苏丹仅次于安哥拉和南非,为中国非洲第三大贸易伙伴。","text2":"苏丹于何年独立?","label":1} {"text1":"东山文化()是位于越南北部红河河谷的一个铁器时代史前文化。它约繁盛于公元前700年到公元前后,它影响的区域包括东南亚其他地区乃至以及印度-马来亚群岛。古东山人具有熟练的稻米栽培技术和水牛、猪等动物的饲养技术,使用长独木舟进行航行和渔猎。东山文化最突出的文化遗产是东山铜鼓等青铜器制品,它们在东南亚和今中国南部的广大地区都有发现。越南北部在公元前几千年就已经产生了新石器时代史前文化。该地区的地理气候使得大规模的农业和畜牧业发展成为可能,并为东山文化的产生和发展创造了条件。东山文化并非独立产生,它与藏缅语系民族、傣族、孟-高棉南亚语系民族以及老挝的查尔平原的古文化均存在联系。东山文化的核心位于越南北部的红河河谷,但它与周边地区维持着紧密的商业关系,以铜鼓作为主要商品,其商品出口范围包括东南亚的大部分地区和印度尼西亚。从大约公元前1000年开始,东山文化成为东南亚最早进入铜器时代的古代文明,在东山文化的古代墓葬中曾发现了铜鼓等物品。东山铜鼓使用失蜡法铸造,其高度最高可达1米,重量最重可达100千克。东山铜鼓被作为宗教仪式用品和音乐器具,通常刻有几何图形、日常生活、战争、动物、鸟类以及船只的形象。其中的船只形象表明了贸易在东山文化中的重要性,而铜鼓本身也被作为贸易物品和家传宝物。在当地原始文化中,拥有这些铜鼓即意味着拥有掌控万物运转的力量,也表明了富有的家庭有能力支付那些大型铜鼓的制造。在作战时铜鼓被用力击打,产生的巨大声音可以吓退敌人。东山铜鼓分布十分广泛,它们在今中国南部、越南、以至印度尼西亚东部都有发现。对于东山文化衰落的原因并无确切定论。可能与气候环境的改变、由于农业活动造成当地生态环境的恶化、外族入侵、周边地区的政治环境变化等多种因素有关。在大约公元后200年,东山文化已经全面衰落。当地人在很大程度上已经忘却了东山文化的存在,他们将东山铜鼓的铸造归于某些神话人物或民间传说中的英雄。但是作为越南历史上最重要的古代文化之一,其科技文化成就对越南文明的发展产生了深远影响。","text2":"东山文化最突出的文化遗产是什么?","label":1} {"text1":"口器(Arthropod mouthparts),位于节肢动物口两侧的器官,有摄取食物及感觉等作用。昆虫口器由头部后面的3对附肢和一部分头部结构联合组成,主要有摄食、感觉等功能。蛛形纲口器包括两对附肢,昆虫的口器包括上唇一个,大颚一对,小颚一对,舌、下唇各一个。上唇是口前页,一块(其内有突起,叫上舌)。舌是上唇之后、下唇之前的一狭长突起,唾液腺一般开口于其后壁的基部。大颚、小颚、下唇属于头部后的3对附肢。甲壳亚门之外肢演变为演化为口器、吸盘和生殖器等非常不同的器官,并于附近有被称为颚足的附属物。其营养方式是以咀嚼植物或动物的固体组织为食。如:蜚蠊、蝗虫、豆娘等。口器构造复杂。初大颚可用作咀嚼或塑蜡外,中舌、小颚外页和下唇须合并构成复杂的食物管,借以吸食花蜜。如:蜜蜂等。口器形成了针管形,用以吸食植物或动物体内的液汁。这种口器不能食固体食物,只能刺入组织中吸取汁液。如:蚊、虱、椿象等。其主要部分为头部和以下唇为主构成的吻,吻端是下唇形成的伪气管组成的唇瓣,用以收集物体表面的液汁;下唇包住了上唇和舌,上唇和舌构成食物道。舌中还有唾液管。如:蝇等。口器呈吸管状,是以小颚的外叶左右合抱成长管状的食物道,盘卷在头部前下方,如钟表的发条一样,用时伸长。如:蛾、蝶等。","text2":"口器有什么作用?","label":1} {"text1":"《多宝塔碑》是唐代重要碑刻,是书法中楷书代表作品。此碑是颜真卿书,是为颜真卿早期楷书代表作品,也是颜体书法的代表帖。《多宝塔碑》,又称作《多宝塔感应碑》,全称为《大唐西京千福寺多宝塔感应碑》,《多宝塔碑》立于唐玄宗天宝十一年(752年)四月二十日,该年颜真卿正值四十四岁,担任朝议郎尚书,奉旨书写此碑。《多宝塔碑》的碑文共有三十四行,满行有六十六个字,全文两千多字,由相门后裔南阳岑勋撰文,越州徐浩以隶书提额,河南史华刻字。碑体总高285公分,宽度102公分,唐朝时立碑于长安安定坊千福寺,宋代时移到西安碑林,清朝康熙年间碑体折断,现收藏于西安碑林博物馆,属于国宝级文物。","text2":"《多宝塔碑》现收藏于哪里?","label":1} {"text1":"《丁丁与丛林战士》(法语:\" Tintin et les Picaros \")是《丁丁历险记》的第23部作品,也是《丁丁历险记》中最后一部完整作品。作者是比利时漫画家埃尔热。于1976年初版。这部作品与《丁丁在苏联》和《丁丁在刚果》并为丁丁历险记中最受争议的作品。但与前两部不同,这次的争议主要是在艺术上而非政治上。为营救身陷囹圄的毕安卡等好友,丁丁一行流落森林,遇上阿卡扎尔将军。丁丁决定冒险一拼──协助阿卡扎尔和他的丛林战士,推翻塔比奥卡政权。不料丛林战士却中了塔比奥卡的奸计,溃不成军,弄得阿卡扎尔一筹莫展。丁丁凭著图纳思的新发明,戏剧性地令军队重振士气、直捣黄龙。","text2":"丁丁凭借什么反败为胜?","label":1} {"text1":"南峇山()位于马来西亚柔佛州居銮,座落在居銮市东侧约5公里处,主峰高度海拔510米。那里的森林保留地是当地的旅游景点。南峇山的山路径清晰,适合初次攀爬热带山林的旅客。山可分为南山和北山,各有一条山道,登山者可选择其中一条路径,也可以同时攀北、南两峰和两个小峰,全程需时2到4个小时。北山后山有一棵圆周十多人的黄叶子巨树,是南峇山的亮点。前往后山的山路上,可见山藤和双层飞瀑。其中北山山高海拔505米,可由度假村水池旁的洋灰道,经情人桥和石山水道到半山,或由度假村后山路直上半山。半山宽阔的平台矗立着一座“欢迎登上南峇山”的牌楼,还有一座“茶亭”,石凳石桌,山水水喉,两个水池。北峰顶约500平方呎的光秃秃土地,向东可俯瞰丰盛港路工业区和山林,西边因被大树遮挡,无法看见居銮市容。南山和北山同样高度,较隐蔽。从度假村水池旁右拐,即是登南山的山径,一路上有许多树木,山顶大树林立。南峇山位于令金森林保护区内,山区里热带木种繁多,可在当地找到东革阿里,一种罕见可用药的树木。山脚斜坡也设有休闲中心。休闲中心由10余座独立小楼和一个泳池组成。","text2":"南峇山坐落在哪里?","label":1} {"text1":"斐豹蛱蝶(学名:\"Argynnis hyperbius\")亦称黑端豹斑蝶、斐胥,是豹蛱蝶属的一种蛱蝶。斐豹蛱蝶的前翅膀表面呈橙黄色,后翅膀表面则呈淡橙黄色。前翅膀表面翅室有短的横纹,内侧布满阔圆点,外缘弯曲;后翅膀上有隐密的窄汶,同样有圆斑点向外伸延。牠们的前翅膀底面呈淡枣红色,向主轴渐变成赭色,与表面有相似的斑点;后翅膀的底面呈赭色、赭褐色及银白色斑驳。触角表面呈褐色,底部呈赭红色。头部、胸部及腹部呈黄褐色,但底部则呈淡赭色。牠们阔约80-98毫米。斐豹蛱蝶幼虫的头及脚都呈黑色,身体呈黑色而满布橙红色斑纹。头上有四条水平的刺,腹部上的刺端呈淡粉色,尾部的刺则呈粉红色而尖端有细小短毛。斐豹蛱蝶蛹上的头部及翅鞘呈淡红色,背上有10个淡金属色的斑点。腹部呈深粉红色,刺端黑色。斐豹蛱蝶分布在喜玛拉雅山区,由印度旁遮普邦至锡金邦、阿瓦德土邦、阿格拉、孟加拉的曼巴及阿萨姆邦;卡西丘陵、上缅甸、中国大陆及台湾、苏门答腊及爪哇。","text2":"斐豹蛱蝶的翅膀表面是什么颜色的?","label":1} {"text1":"Bungie是美国著名的电子游戏软件制作商,是由两名芝加哥大学的本科生Alex Seropian和Jason Jones于1991年5月以Bungie Software Products Corporation的名称成立。公司最初设立在芝加哥,在成立的最初九年里,公司主要致力于Macintosh平台游戏的开发,他们开发了深受玩家喜爱的《马拉松》()和《杀无赦》()系列以及《奥妮》()。2000年,Bungie被微软收购,他们的《最后一战:战斗进化》()成为第一人称射击游戏及微软的游戏主机Xbox的首发游戏。《最后一战》(Halo)从此成为了Xbox的杀手级软件,它的两款续作也有著百万级的销量。2007年10月,Bungie把最后一战系列的IP权让渡给微软,恢复独立身份。2013年2月发表MMOFPS《天命》(Destiny),交由动视发行。","text2":"2000年,Bungie被哪个公司收购","label":1} {"text1":"委内瑞拉蚁鸫(学名:'),又名塔奇拉鸫或委内瑞拉蚁八色鸫,是一种神秘的雀。牠们暂时被分类在蚁鸫科内,有待审定。牠们曾只于1955年至1956年间被见到及采集,被怀疑可能已经灭绝。直至2016年,科学家在埃尔塔马国家公园重新发现了委内瑞拉蚁鸫。委内瑞拉蚁鸫约长17厘米。上身呈褐色,冠及颈部呈灰色。展翼有黑色斑纹,喉咙及耳朵呈褐色,颊上有白色斑条。下身呈白色,两侧及胸部有灰色的斑纹。于1955年至1956年间,在委内瑞拉塔奇拉州西南部共采集了4个委内瑞拉蚁鸫标本。此后60年里它们都没有被再次发现,被怀疑可能已经因丧失栖息地而灭绝。2016年,一组考察队于埃尔塔马国家公园重新发现了它们,这也是第一次记录下委内瑞拉蚁鸫的声音和拍摄到它们的活体照片。","text2":"委内瑞拉蚁鸫被怀疑可能灭绝的原因是什么?","label":1} {"text1":"意大利天主教是普世罗马天主教的一部分,接受罗马教宗和意大利主教团的精神指导。基督教在公元1世纪来到意大利半岛,可能是由不知名的旅行者、商人或士兵传入。保罗的罗马书证实了第1世纪基督徒在罗马的存在。使徒彼得和保罗都曾前往罗马传教,接触当地的基督徒,并最终在那里殉道。罗马主教克雷芒一世,约在96年写《革利免一书》。在2000年的历史中,意大利教会的规模和影响都有所增长,产生天主教一些最重要的领袖和宣传者(有些人因此殉道),包括:由于意大利文艺复兴的原因,意大利的教会艺术相当非凡,包括列奥纳多·达芬奇、米开朗基罗、济安·贝尼尼、桑德罗·波提切利、丁托列托、提香、拉斐尔、乔托等人的作品。意大利教堂建筑相当壮观,而且对于西方文化具有重要的历史意义,特别是罗马圣伯多禄大殿,威尼斯圣马可教堂,和伯鲁乃列斯基的佛罗伦萨圣母百花大教堂,包括吉伯提所作洗礼堂铜门上的“天国之门”青铜浮雕。大约90%的意大利人是天主教徒,其中三分之一是活跃的成员。意大利有225个教区和总教区,数目之多在世界上仅次于巴西。意大利还拥有世界上最多的本堂区(25,694)、女性神职人员(102,089)、男性神职人员(23,719)和神父(44,906)。意大利的主教们组成意大利主教团,合作完成根据教会法规定的某些职能。与大多数主教团不同,意大利主教团主席由教宗任命,地位为意大利的主教长。自2007年3月起,主教团主席为 Angelo Bagnasco 枢机。派往意大利的教廷大使也是驻圣马力诺大使。从2007年1月起,大使是意大利总主教Giuseppe Bertello。","text2":"意大利非凡的教会艺术有哪些?","label":1} {"text1":"索西比乌斯(希腊语:Σωσιβιoς),生活于西元前三世纪,是埃及托勒密王朝托勒密四世的重臣。关于索西比乌斯的家世没有详细记载,有可能是托勒密二世期间的索西比乌斯之子,也缺乏关于索西比乌斯从政经历的描述,但可知他对年轻的托勒密四世有相当大的影响力,并实际掌握政务。根据波利比乌斯的描叙,索西比乌斯利用巧妙的政治手腕,在不用直接煽动托勒密四世的情形下,使托勒密四世的叔叔利西马科斯、兄弟马格斯、母亲贝勒尼基二世接连遭到处死。而当时流亡到托勒密王朝的斯巴达国王克里昂米尼三世可能会影响索西比乌斯所掌握佣兵的部队,索西比乌斯因而利用当时的局势使克里昂米尼三世遭到软禁。在托勒密四世沉迷于享乐不理朝政之后,王国的实权逐渐落到索西比乌斯的手中,使国家的财政和军事防卫衰败,邻近的塞琉古国王安条克三世趁机发动第四次叙利亚战争,随即占领柯里叙利亚等地。索西比乌斯利用精湛的外交协商使安条克三世的征服行动延误,让埃及有足够时间招募佣兵和训练部队,索西比乌斯更从本土埃及裔中招募士兵,并依马其顿方阵的战术来训练,弥补托勒密军中希腊裔的缺乏。在前217年,索西比乌斯陪同托勒密四世进军巴勒斯坦,于拉菲亚战役击败安条克三世获得战争胜利,之后,托勒密四世派索西比乌斯与安条克交涉一个相当满意的和约。之后托勒密四世的期间索西比乌斯依然握有大权,但某种程度上与阿加托克利斯分享权力,对于随后的过程不是很清楚,可确定索西比乌斯和阿加托克利斯策划谋杀了阿尔西诺伊三世。索西比乌斯后来可能遭到反对派处死或被撤职,然而关于这些详细史料都已散失,因而不是很确定。","text2":"索西比乌斯生于什么年代?","label":1} {"text1":"台西镇 (),中国青岛市旧区划名,位于胶州湾东岸现市南区台西半岛中部,铁路青岛站以西。台西镇是德国胶州湾租借地要塞防线内界规划建设4个区域之一,也是2个华人劳工居住区之一。德租时期的台西镇的范围大致是台西一路、磁山路、台西四路和贵州路合围成的四边形区域。关于“台东”“台西”之名的来源,习惯性说法是:贮水山因明代建有烽火台而曾被称为“烽台岭(亦称风台岭)”,而台东、台西因地处烽台岭之东西两侧而得名。然而根据德租时期的手绘中文地图来看,“风台岭”被标注在今青岛山(“炮台山”)与贮水山之间的一个山丘上,而该山丘(根据德文地图高度在75-80米之间)的位置在青岛山北侧,胶宁高架、广饶路、登州路与青岛山山体的合围区域内,而非贮水山。至于“以烽台岭为界”“山上有明代烽火台”的说法,至今未找到可靠记载。清帝国时代该地区原有一村庄,名为“小泥洼”,德租初期音译作,后又转译为“小梅坞”。1901年,德属胶澳总督府高价收购了原村庄,开始建设台西镇,后逐渐形成了中国人居住的市镇。1910年,德属胶澳总督府设置台西镇为德属胶澳保护领青岛市的四区之一。1929年9月,中华民国青岛市政府撤销台西镇,成立第一区。1935年5月,青岛市政府重划市乡区域并改定名称,撤销第一区,成立台西区。","text2":"1901年,开始建设台西镇?","label":1} {"text1":"挖客是基于WEB2.0的互联网内容挖掘、分享社区。该词汇来源于美国的digg网站,后被中国IT业内人士引进并翻译成挖客,国内有挖客网。挖客这类网民主要热衷于分享在网络上发现热门以及有价值的信息。一般他们提供发现信息的URL地址,然后通过挖客社区把这些信息分享以便更多的挖客可以浏览。这些挖客推崇的精神是:挖客社区的内容由挖客自己决定。挖客社区的规则。当挖客分享或创作一条新信息到挖客社区时,这条信息将被列在社区的“潜水信息”栏中,等待其他挖客来“挖它”(投票)。挖客社区会有一套规则并结合挖客的“挖”来决定那些是有价值或者热门的信息从而可以被挖出水到“出水信息”栏,即到社区的首页。从而让更多的挖客可以快捷的浏览到热门以及有价值的信息。目前挖客社区采用分值制度,其中Karma值较流行,这样能够较好的保证挖客在分享信息时的合理性和公正性。","text2":"挖客这个词汇来源于什么地方?","label":1} {"text1":"个人气象站是由非气象专门机构所操作使用的气象测量仪器。使用者可能为个人、俱乐部及协会等基于兴趣而业余使用,或例如农场等基于业务经营的需要而使用。个人气象站的质量和数量差异很大,在于如何安置仪器,到获取准确资料,或有意义和可比较的数据,也可以是差异很大。典型的仪器包括风速仪、风向标、温度计、湿度计(获得相对湿度)、晴雨表和雨量计。更先进的站也可以测量紫外线指数、太阳辐射、叶面湿度、土壤水分、土壤温度和池塘、湖泊、小溪、河流的水温与其他数据。今天的个人气象站也通常涉及数码化数据收集。这些气象站可接驳到个人电脑中,数据可显示、储存并上传到网站或数据分析系统之中。个人气象站除了可供教育用途外,更是作为业余兴趣者的装置,但许多个人气象站也与他人分享他们的数据,通过使用互联网或业余无线电分享成果。公民天气观测计划是个人提交的数据,通过使用软件,透过互联网或业余无线电传送予国家气象局作为的天气或气候预测模型。","text2":"什么是个人气象站?","label":1} {"text1":"赵敬,涿州(今河北省涿州市)人,赵朓的孙子,赵珽的儿子,赵弘殷的父亲,赵匡胤的祖父。在唐朝为官,历任营州(今辽宁朝阳市)、蓟州(今河北蓟县)、涿州三州刺史。另一说法指赵敬本来在后梁出任官职,后唐建立后,其子赵弘殷为躲避父祸,逃难到今洛阳郊外的夹马营。但这一种说法找不到正面史料支持,只是推测。而且《东斋笔记》卷一记载:“刘尚书涣尝言:宣祖初自河朔南来,至杜家庄院,雪甚,避于门下。久之,看庄院人私窃饭之。数日,见其状貌奇伟兼勤谨,乃白主人。主人出见而亦爱之,遂留于庄院累月。家人商议,欲以为四娘子舍居之婿。四娘子即昭宪皇太后也。”指出赵弘殷是来投靠的镇州的赵王王镕势力,在领兵帮助李存勖后才到洛阳禁军任职。现在也没有关于赵敬如何去世的记载,年幼的赵弘殷则是在保州长大,赵敬妻子的家族所在地。960年,赵匡胤建立宋朝,追尊赵敬简恭皇帝,又被宋真宗加尊为简恭睿德皇帝,庙号翼祖。墓地号为定陵。干兴元年(1022年)七月改靖陵。陵在幽州,在宋朝统治疆域之外,没有维护。议谥,翰林学士窦俨撰;册文,中书舍人赵逢撰。大中祥符五年(1012年)闰十月加谥册文,参知政事王曾撰。","text2":"赵敬是哪里人?","label":1} {"text1":"布袋线,为位于台湾台南市新营区与嘉义县布袋镇间,由台湾糖业股份有限公司新营总厂经营之轻便铁路,今既废止。此线为糖业铁路客运化之始,开办于1909年。纵贯线铁路在选线时,在嘉义曾文溪间即有经过盐水港(今盐水)或新营庄(今新营)之议。详见纵贯线 (南段)。当时虽曾研议另建新营经由盐水至北门屿(今北门区)之官线铁道支线弥补不足,唯此案未成真。新营=盐水=布袋间铁道运输稍后由制糖会社完成,属于盐水港制糖兼办之客运业务。据铁道部之资料,新营庄=盐水港间五哩三分于1909年(明治42年)5月20日开始营业,亦为台湾首条糖业铁路定期营业线。然而,根据同年3月4日《台湾日日新报》资料,在官方核准开办营业线之前,当时新营-{庄}-=岸内-{庄}-间即对外办理客运(一日4往复),车资内地人(即日本人)15钱、本岛人(台湾人)10钱;1913年(大正2年)3月8日,营业区间延至布袋嘴(今布袋)。战后布袋线曾进行数次改动。包括糖铁新营车站(原址位于台铁车站旁,今已成停车场)因破烂不堪,1950年迁移百余米至今址。另外,布袋车站因位于市区之外,曾应民众要求,利用盐业铁路(台盐公司所有)延伸营业间区750m至贴近市区的半路店、但仅维持数年。沿线各车站亦有重修,大部份皆非日治时期原貌。布袋线为762mm狭轨铁路,但在新营=岸内间,另有一条并行之新岸线,为762mm及1067mm轨距之三线轨道。后者可允许台铁货车驶至岸内。新营 - 厂前 - (南信号所) - 工作站前 - 东太子宫 - 太子宫 - 南门 - 盐水 - 岸内 - 义竹 - 埤子头 - 安溪寮 - 前东港 - 振寮 - 布袋 (- 半路店)支线:东子宫 - 纸浆厂","text2":"布袋线是哪家公司经营的轻便铁路?","label":1} {"text1":"软颚或者软腭(医学拉丁语术语:或)是哺乳动物中硬腭延长形成的一个双褶皱形状的口腔部分。它斜挂在或者垂直挂在舌根上方,是由咽腭肌和其它肌肉的延伸组成的。它是分隔口腔与咽的组织的一部分,因此有分隔呼吸系统与消化系统的作用。软颚由一层没有角质的上皮覆盖。许多哺乳动物的软腭的粘膜里有淋巴组织()人和食肉目动物没有这样的淋巴结。咽缩肌()、腭帆张肌()和腭帆提肌()可以运动软颚。从软颚的边缘伸展出两道双皱褶组织,即腭弓()。猴和人的软颚中部有一个被称为腭垂的下垂,大多数其它哺乳动物没有这个组织。软颚的神经是从舌咽神经和迷走神经延伸出来的。软颚的基线被称为“啊线”,因为它在说“啊”的时候可见。在软颚末端悬挂的小舌在部分语言中用于发出小舌音。软颚有分隔呼吸道和消化道的作用。在下咽的时候在咽缩肌的作用下软颚后壁会亚在气管上。在下咽或者打哈欠的时候从耳咽管伸出来的腭帆张肌和腭帆提肌有平衡中耳和外部的压力差的作用。在发音的时候软颚抬起,把鼻腔与口咽腔分隔开来,这样一来从肺里传出来的气流只能流进口腔来产生口音。在发鼻音的时候口腔被封住,气流流过鼻腔。在发鼻化元音的时候软颚下沉,让气流同时通过口腔和鼻腔。软颚在打呼噜的时候也有作用。触碰小舌或者软腭的深处,对大多数人来说都会引起强烈的呕吐反射,属于一种自然的保护机制。在校正唇颚裂的整容手术以及肿瘤手术中软颚有很大的意义。","text2":"哪些组织可以运动软腭?","label":1} {"text1":"西澳州政府体-{制}-是隶属于澳大利亚宪法的政府体-{制}-,成立于1890年,百余年来经历多次修订。自1901年起西澳大利亚州成为澳大利亚联邦(Commonwealth of Australia)的一个州,与联邦的关系亦首先尊重西澳宪法。根据澳洲宪法,西澳除了被联邦分割部分立法权和司法权,其它方面保持完全独立。西澳是根据西敏制的精神——英国式议会政治体-{系}-,采三权分立。西澳州议会设于首府珀斯,政府功能方面由州议会负责监督。往昔隶属珀斯市的各市区今划分为三十余个地方政府(LGA),由于政治、经济、自然环境等皆有密切的关系,所以通称为「珀斯大都会」。西澳州政府官方网页","text2":"自1901年起西澳大利亚州成为哪里的一个州?","label":1} {"text1":"M2迫击炮为美国于1940年代制造的滑膛前装式迫击炮。美国于1920年代开始进行迫击炮规格审查,主要目的为进行新型步兵轻型支援武器测试,在经过各种迫炮的测试以后决定购买法国兵器工程师Edgar Brandt设计的轻型迫击炮。1930年代后期接受美国军方设计,1940年1月第一批1500门迫击炮交付美军服役,由于这是美国陆军第二种采用的迫击炮,因此正式代号为M2迫击炮。到1945年二战结束时,M2迫击砲总产量达60,000门左右。二战美军标准编制一个步兵团下辖27门60迫炮,使用单位除了团直属迫炮连之外,各步兵排也有直属迫炮班配发3门60迫炮提供火力支援。二战中,M2迫击炮的地位为步兵排级支援武器,介于81迫击炮与手榴弹间的火力空白地带。二战后到韩战期间M19迫击砲开始换装,不过M19的弹著精度美军认为不如M2,因此M2迫击炮一直留用到越战时期,到1980年代后才以M224迫击砲取代。M2迫击炮主要使用下列几种弹药:","text2":"M2迫击炮名称是怎么来的?","label":1} {"text1":"能源开发()是为满足文明的各种需求而不断努力提供足够一次能源和二次能源的能量形式的过程。能源开发不但涉及到成熟技术的采用,同时也包括建立新的能源相关技术。在能源规划当中,主要的考虑事项包括资源枯竭、、供应的安全性、成本、对于空气污染和水污染的影响以及相应的能源是否属于可再生能源。可再生能源通常被定义为能量来源于在人类时间量度里它们让大自然补充的能源,例如太阳光,风,雨,潮汐,波浪,地热等。可再生能源取代了四个不同领域的传统燃料:发电,热水\/供暖,汽车燃料和乡村(离网)电力服务。化石燃料(主要的不可再生的化石)来源燃烧煤或烃类燃料,这是植物和动物的分解的遗体。有三种主要类型的化石燃料:煤炭,石油,和天然气。","text2":"能源开发是什么?","label":1} {"text1":"济南革命烈士陵园,也称英雄山革命烈士陵园,位于济南市南部英雄山(原名四里山)南麓、五里山西麓,建于1949年11月至1968年,为山东省规模最大的烈士陵园。其建设目的为埋葬1948济南战役中战死的华东野战军一方军官及士兵,并对济南市民免费公开开放,以达到“爱国主义”教育目的,与济南市市区内解放阁景观遥相呼应。该陵园建筑群体分三部分,北部为位于英雄山山巅的革命烈士纪念塔,通高约34米,塔身使用乳白色花岗岩砌成,顶部嵌有以红色花岗岩刻成的五角星,中部南北两面为“革命烈士纪念塔”七个馏金大字,由毛泽东题写,塔基为双层,南北两面镌刻有花圈图案浮雕;中部为建于五里山西麓的烈士骨灰堂及事迹陈列室(已扩建为济南战役纪念馆);南部为位于六里山西北的烈士墓区,中共一大代表、山东地方党组织创始人之一王尽美和早期中共山东省委书记刘谦初的墓也迁建于此。","text2":"济南革命烈士陵园在什么地方?","label":1} {"text1":"维珍尼亚·阿普伽(, ),美国医生,专科麻醉及儿科。她是麻醉学及畸形学的领先学者,被认为是新生婴儿科的始创人。阿普伽新生儿评分是全球最广泛使用的新生儿评估方法,此法大幅减少婴儿死亡率,亦令阿氏广为人知。1929年毕业于曼荷莲学院,1933年哥伦比亚大学医学院毕业,1949年成为该院正教授。 1959年,于约翰·霍普金斯大学获公共衞生学硕士。1953年首次推出阿普伽新生儿评分,用作快速为新生儿评估健康状况。阿普伽是哥大医学院首名女教授,在很长时间内亦是唯一的女教授。阿氏无结婚,1974年8月7日病逝于哥大医院。阿氏在生时获奖无数,1994年,美国发行一款以阿氏肖像为图之邮票。","text2":"1959年,阿普伽获得了什么专业的硕士?","label":1} {"text1":"路姜(),中国足球运动员,司职中场。1990-1996年路姜在北京什刹海体校学习,1996年加入北京国安青年队。路姜在2000年进入北京国安一线队,曾入选国青队、国奥队,曾经是一名问题球员,2002年因泡吧事件与国安队友张帅一同被开除出国奥队,但同年路姜还入选了中国国家足球队在与叙利亚的友赛中为国家队上场比赛。在2008年中超联赛第18轮北京国安主场与武汉光谷比赛的第93分钟,武汉队员李玮峰准备解围,结果皮球打在路姜身上弹出,李玮峰收不住脚,左脚踩在了已经倒地的路姜的胸口上,而就在李玮峰转身后,右脚又有个蹬踏的动作踩到了路姜,路姜极为恼火,立即起身掐住李玮峰的脖子将其放倒,这个动作被当值主裁判看到,并出示红牌将路姜直接罚出场外。事后,两人都得到足协8场停赛,罚款8000的处罚。这处罚也令武汉光谷不满而退岀中超联赛。2009年,路姜帮助北京国安获得中超联赛冠军,是球队首座顶级职业联赛冠军。2012年,路姜加盟中甲联赛球队湖南湘涛。2013年,转投另一支中甲联赛球队北京八喜。足协重罚京汉战冲突当事人 李玮峰路姜各禁赛八场","text2":"路姜的职业是什么?","label":1} {"text1":"仙人洞是中国江西省庐山上的一处著名景点,位于牯岭西南,御碑亭东侧,锦绣谷西侧。仙人洞为一天然砂岩石洞,高7米,宽10米,深14米。古称“佛手岩”,明桑乔《庐山记事》载:洞内“岩石参差如人手,故名佛手”。洞内深处从洞顶有水下滴,终年不竭,名为“一滴泉”,又称“叮咚泉”。南宋朱熹《山北纪行》载:“佛手岩,僧缘岩架屋以居,临锦绣谷”,直到清嘉庆九年(1804年)舒天香在《游山日记》中所载,此处仍为僧所居。因传说唐末吕洞宾在庐山得道成仙,不知何人于何时在佛手岩下建“吕祖神龛”。嘉庆年间,道士晏纯一改“吕祖神龛”为“纯阳殿”。光绪三十一年(1905年)静阳子在仙人洞旁建“老君殿”。1925年“云仙道人”罗理松到处居住,并将原洞西侧的方形门改建为圆门。现在的仙人洞内有一石质神龛,上有吕洞宾身背宝剑的石雕像,两旁有两副对联:“称师亦称祖,是道仍是儒”、“古洞千年灵异,岳阳三醉神仙”。老君殿为歇山式单层建筑,整个建筑显得庄重而又轻巧,内供太上老君骑牛雕像。仙人洞内及周边有大量历代摩崖石刻,仙人洞石刻已于1959年列为江西省文物保护单位。在仙人洞西侧的圆门内侧有怪石,名为蟾蜍石,石上长有古松。东侧有游仙石及观妙亭,是远眺云海、群峰甚至远处的长江及俯瞰近处锦绣谷美景的佳处。此外还有朱元璋敕建访仙亭。1961年毛泽东为江青所拍摄《庐山仙人洞照》题诗:“暮色苍茫看劲松,乱云飞渡仍从容。天生一个仙人洞,无限风光在险峰”。","text2":"暮色苍茫看劲松,乱云飞渡仍从容。天生一个仙人洞,无限风光在险峰”。这首诗是谁所写?","label":1} {"text1":"别里古台(;生卒年月不详)是也速该与速赤吉勒所生,成吉思汗铁木真的异母弟,蒙古帝国建国时的一名大将。另有一兄别克帖儿,幼年时别里古台与别克铁儿友好,两人与铁木真失和,争夺猎物,铁木真刺杀之,从此别里古台顺服铁木真。据《元史》记载他为人忠厚,不喜欢喧哗,但同时领兵带将足智多谋,勇武过人。铁木真征服蒙古各部之前有一次他会宴手下各部。有人刺杀别里古台,重伤他的臂膀。铁木真要纠察凶手。别里古台劝铁木真不要这样做,以免挑拨不和。蒙古帝国建立后别里古台被封为王,领三千户,以及广宁路(今辽宁省西部)和恩城。后又加封信州路和铅山州。别里古台有三个儿子:罕秃忽、也速不花(子爪都)和口温不花(孙广宁王彻里帖木儿、曾孙定王薛彻干)。","text2":"《元史》记载中,别里古台是个什么样的人?","label":1} {"text1":"贺万里( ),中国著名画家,美术理论家,中国装置艺术理论研究奠基人。生于安徽合肥,籍贯河北。以中国山水画泼彩山水泼墨闻名。南京艺术学院美术学博士毕业,后任职于扬州大学艺术学院 教授、硕士生导师。中国美术家协会会员。江苏省美术家协会会员。他兼任清代扬州画派研究会会长,以及石河子大学新疆大山水画研究所副所长,还有石河子大学西部绘画研究所研究员。主要研究领域涉及中国古代绘画理论、中西美术史、中国现代艺术、动漫理论等,尤以装置艺术理论见长。20世纪90年代以来,在《文艺研究》、《美术观察》、《美苑》、《艺术探索》、《中国画研究》、《宁夏大学学报》等公开出版的有影响的中文核心期刊或专业性刊物上发表学术论文50余篇。《装置艺术研究》 1999 中国文联出版社《光影与色彩》(译著) 2001 江苏美术出版社《永远的前卫:中国现代艺术的反思与批判》 2003 郑州大学出版社《谁在附庸风雅》2005 上海书画出版社《鹤鸣九天:中国古代绘画的功能论》(其博士毕业论文)《美术刀客:贺万里艺术论评集》2005 天津人民美术出版社《中国山水画通鉴:维扬异趣》 2006 上海书画出版社《中国当代装置艺术史(1979-2005)》2008 上海书画出版社其著作《装置艺术研究》详细,系统地首次向中国读者以及中国艺术家介绍阐明了 装置艺术 的主要内涵和特点,为中国的装置艺术研究奠定了理论基础。在2001年获得海峡两岸书画名家邀请展金奖。在2006年纪念孙中山诞辰140周年中国书画名家作品大展金奖。多幅作品被个人、企事业和美术馆收藏,在《美术报》、《中国书画报》、《美术大观》、《书与画》、《艺术当代》、《国画家》等刊物发表,并被专文推介。1.海峡两岸书画名家交流展,2000年,台北、南宁。一等奖2.纪念孙中山诞辰140周年中国书画名家邀请展,2001,台北、南宁。金奖3.当代著名书画家精品年展,2002,台湾阿里山4.贺万里泼彩山水画暨学术文献展,2006,扬州5.美术刀客----贺万里泼彩艺术精品展,2007,广州6.千岛湖名家名湖书画精品展,2007,杭州淳安7.首届中国美术教师艺术作品年度展,2007,沈阳。7.走近番禺·2008全国书画名家新春雅集,2008,广州8.周积寅教授从艺五十周年暨七十华诞师生书画展,2008,南京","text2":"贺万里现在主要研究什么?","label":1} {"text1":"罂粟花(),为罂粟科开花植物的泛称。罂粟花为草本植物,其花朵有各种鲜艳的颜色,可作为观赏及亭园造景之用。其中最有名的,就是可做鸦片原料的罂粟。但在罂粟之外,包括花菱草、虞美人等,也可以称为罂粟花。在第一次世界大战期间,在法兰德斯的壕沟战发生在开满罂粟花的田野中,因此罂粟花被当成在战争中伤亡士兵的纪念物。在中华人民共和国,罂粟花有时单指罂粟的花。属于罂粟科之下,被称为罂粟花的植物包括了罂粟花经常被当成园艺植物。加州罂粟为美国加利福尼亚州的州花。每年的4月6日为加州指定的加州罂粟日。在第二次世界大战时代,为了解决中共的财政问题,八路军三五九旅在陕西北部南泥湾种植罂粟。在欧美及英联邦各国的国殇纪念日,当地官员及民众会在胸前衣襟佩带罂粟花以作悼念,称为国殇罂粟花。英国首相卡麦隆于2010年11月率团访华,正值国殇纪念日前后,英方全体访华成员在胸前衣襟上都佩带上了纪念一次和二次大战阵亡将士的鲜红的虞美人小花。在中国,罂粟被联想到鸦片战争。这个错误引发中英两国间一场外交风波。卡梅伦与阁员在北京人民大会堂时,拒绝中方对他们除下罂粟花的要求。一名首相府人员说:「中方人员对我们说佩戴罂粟花是不恰当的,因为两国曾经发生鸦片战争。我们告诉他们这花对我们有重大意义,我们所有人会继续佩戴。」","text2":"罂粟花有什么作用?","label":1} {"text1":"天主教托莱多总教区(拉丁语:Archidioecesis Toletana、西班牙语:Archidiócesis de Toledo)是天主教会在西班牙设立的一个总教区,主教座堂设在托莱多的托莱多圣母主教座堂。根据教会传统,该教区于1世纪由圣雅各创立。313年在《米兰敕令》颁布后升格为总教区。托莱多总主教被尊为西班牙主教长。2011年,在当地729,200人口中,有教友638,500人,占辖区总人口87.6%、270个堂区、465名司铎、81名修士、877名修女。现任总主教为Braulio Rodríguez Plaza,荣休总主教为弗朗西斯科·阿尔瓦雷斯·马丁内斯枢机。","text2":"现任总主教是谁?","label":1} {"text1":"戈伟(),中国足球运动员,司职前锋,现效力于中乙球队江西联盛。2004年,15岁的戈伟刚刚初中毕业,他所在的中山东路体校足球队面临解散。体校的教练认为戈伟是个人才,那时候戈伟身高就达到了1米92,于是他们便向江苏舜天推荐。江苏舜天教练先让戈伟接受测试,但结果令江苏舜天的教练有些失望。除了身高、技术还可以外,戈伟的其他指标均不行。不过,江苏舜天还是看到了戈伟的潜力,还是毅然决然地把他留了下来。在2007年的青年联赛是戈伟最辉煌的时候,这却不引起国字号球队的重视,因为骨龄测试戈伟始终过不了关,这也使得他无法在大赛中出场。尽管骨龄测试困扰着戈伟,但江苏舜天没有放弃戈伟在2008年主教练裴恩才还把他升入江苏舜天一队。因为骨龄测试不过关,2009年戈伟还曾被中国足协禁赛半年。2010年5月26日,戈伟在江苏舜天1比0击败天津泰达的比赛中替补出场,完成自己职业生涯首秀。 7月24月,他在和南昌衡源的比赛中打进职业生涯首个进球,帮助球队1比0击败对手。戈伟凭借在联赛中的出色表现,得以入选中国国奥队。但9月24月,戈伟在江苏舜天和杭州绿城的比赛中替补出场五分钟即因和杜威拼抢受伤离场,赛后检查发现他的右膝十字韧带出现撕裂,赛季报销。2011赛季又在和南昌衡源的季前热身赛中旧伤复发,恢复后在和天津泰达的预备队联赛再次因伤离场。7月,戈伟进行手术治疗,并在2012年4月开始有球训练,5月15日在江苏舜天1比1战平长春亚泰的预备队联赛替补出场,回归赛场。2014年2月,戈伟离开江苏舜天,加盟中乙联赛球队江西联盛。","text2":"戈伟的职业是什么?","label":1} {"text1":"耶德逊纪念教堂Judson Memorial Church是美国纽约市的一座教堂,位于曼哈顿格林尼治村的华盛顿广场公园南侧,属于美北浸礼会和联合基督教会。该教堂由杰出的布道家爱德华·耶德逊(Edward Judson)创建于1890年,得到洛克菲勒等知名浸信会教徒的支持。作为纪念他的父亲艾多奈拉姆·耶德逊(Adoniram Judson),最早前往缅甸的新教传教士之一。爱德华·耶德逊为教堂选址在华盛顿广场公园南侧,以便接触邻近的社区。耶德逊纪念教堂是一座雄伟的大厦,由著名建筑师Stanford White和花窗玻璃大师约翰·拉法奇(John La Farge)设计,意大利文艺复兴风格。其特色是用非常优雅的彩色石膏手工仿制的人造大理石。雕塑家Augustus St. Gaudens设计了浸礼池的大理石饰带(frieze)。总体而言,耶德逊纪念教堂的外部形状据说类似于罗马的圣母大殿,而大门据说受到了建于1480年的卢卡的文艺复兴式圣Alessandro教堂的启发。教堂的14扇花窗玻璃窗户是全美国最大的拉法奇作品收藏s<\/nowiki>。1999年,面对经济困难,董事会将教堂后面的耶德逊堂区大楼出售给纽约大学法学院,新建了 Furman 大楼。新大楼高11层,现在远远超过教堂的高度和华盛顿广场公园,在兴建时引发社区相当多的争议。耶德逊教堂的办公室和小聚会厅现在占据新大厦角落的一套房间,位于汤普森街239号,与大教堂毗邻。","text2":"耶德逊纪念教堂位于什么地方?","label":1} {"text1":"阿拉伯文数字是指阿拉伯文形式的印度-阿拉伯数字。这种数字被大多数使用阿拉伯文的国家使用,这包括阿拉伯国家和大多其他西亚国家。这种数字是由印度在中世纪引入阿拉伯,当时已经有很大的改变。在中世纪时,北非也使用了这种数字。之后,这种数字又从北非引入意大利,进入欧洲国家,慢慢的成为当今通用的阿拉伯数字。阿拉伯语把这些数字称为“印度数字”( ')。Unicode把它称为阿拉伯-印度文数字(Arabic-Indic Digit)。现在世界上主要有两种阿拉伯文数字:同阿拉伯文不一样,当代的阿拉伯文数字一般从左向右排列。而在一些古典作品里是从右向左排列的。在埃及,“二”通常用另一种写法。","text2":"现在世界上主要有几种阿拉伯文数字?","label":1} {"text1":"侯丽节(,),也叫洒红节、欢悦节、五彩节、胡里节、荷丽节、好利节,是印度人和印度教徒的重要节日,其地位仅次于屠妖节,也是印度传统新年(新印度历新年于春分日),传统上定于每年的12月()的月圆日。在印度、尼泊尔等地都是重要节日,特别是对年轻人而言。传说从前有一个国王金床因为修习苦行得到金刚不坏之身,便心生骄傲,要臣民只崇拜他,不允许人民信奉大神毗湿奴。他的儿子钵罗诃罗陀却坚持敬奉大神。金床尝试改变儿子的想法,但多次失败后,决心杀死儿子,于是他指使自己的妹妹、女妖侯丽卡烧死王子。侯丽卡诱骗钵罗诃罗陀和自己同坐火中,而侯丽卡有一件避火斗篷。在火中,避火斗篷突然飞起,罩在钵罗诃罗陀身上,侯丽卡被当场烧死,化作灰烬。这是大神毗湿奴保佑的结果,人们便将七种颜色的水泼向王子以示庆贺。毗湿奴则现身,处死金床。捉弄人和尽情欢乐是洒红节的精神所在,通常较低种姓的人将粉和颜料洒向高种姓的人,暂时忘记阶级的差异。洒红节的第二天,人们便用水和各种颜料互相泼撒、涂抹。夜晚,人们把用草和纸扎的侯丽卡像抛入火堆中烧毁。印度人在洒红节期间还要喝一种称为Bhang的乳白色大麻饮料,据说可保来年平安健康。在尼泊尔,庆典的开始是竖竹竿仪式。节日为期一周,人们互相抛洒红粉,投掷水球。第八天时,人们将竹竿烧掉,节日结束。在节日期间,大家互相投掷彩色粉末和有颜色的水,以示庆祝春天的到来。近年世界各地不少地区模仿洒红节庆典的形式,以色彩节(Festival of colour)在世界各地举行庆祝活动,如美国盐湖城、洛杉矶、澳洲雪梨、柏斯、墨尔本、俄罗斯莫斯科、圣彼德堡、土耳其伊斯坦堡、越南河内等地,以宣扬爱与平等。","text2":"在节日为期一周的时间里会做什么?","label":1} {"text1":"朱慈炯(),明思宗朱由检第三子、母庄烈愍皇后周氏。崇祯十六年(1643年)封为定王,别名朱三太子。崇祯十七年(1644年),甲申之变,流寇攻入京师,李自成封朱慈炯为定安公,李自成败退时不知所终,南明年间追封为定哀王。身为一个失踪的明朝皇室嫡系,他的绰号朱三太子更为有名,清兵入关以后,有关他起兵反清复明的谣言一直困扰著清初政府。查继佐《明书》(罪惟录)言思宗第三子永王慈焕、第四子定王慈炯(一作慈灿),俱田贵妃出,不详其所本。果如其言,则周皇后所生惟太子、怀隐王与二公主。朱慈焕未死甲申以前,则民间三太子之谜遂解。《罪惟录》言,思宗成人三子。《启祯宫词》云,甲申之前,田贵妃疾笃,以皇子付懿安张皇后。张皇后下落亦成谜。","text2":"朱慈炯别名什么?","label":1} {"text1":"伊拉克航空(),是伊拉克的国家航空公司,也是中东地区历史最悠久的一家航空公司,总部设在伊拉克首都巴格达。其枢纽机场为巴格达国际机场。伊拉克航空是阿拉伯航空运输组织的成员。伊拉克航空成立于1945年,是中东地区历史最悠久的一家航空公司。在50年代-60年代由于与苏联亲近所以购买了一批苏制客机,1970年代改为购买美制客机。1974年购买3架波音707,并在1976年接手波音727与波音747,并开通到北京及东京航线与大量欧洲航线。在1980年代由于美伊关系更加亲近,所以订购了更多的波音727与波音737和欧洲的空中客车A310。在两伊战争期间伊拉克航空中断了部分航线,并受到美国的制裁以至于美国未向伊拉克交付部分波音727。两伊战争结束后伊拉克国民经济受到损伤,由于美国与伊拉克关系战后一直紧张伊拉克航空也难以在购买外国客机并直接导致1990年海湾战争爆发,伊拉克航空受到联合国制裁,伊拉克航空转移了部分客机直至1992年复航。伊拉克战争之后,伊拉克航空宣布重启国际航线的计划,并于2004年10月3日重开巴格达至安曼的航线。此后,伊拉克航空向其他航空公司租借客机以恢复部份航线,并在2012年宣布订购30架波音737-800和10架波音787,同年接收一架波音777-200LR以及一架空中客车A330。2013年3月5日,伊拉克航空复航伦敦,同年4月25日使用波音777-200LR复航法兰克福。2014年8月23日,伊拉克航空使用波音777-200LR开通巴格达-广州-埃尔比勒-广州-巴格达航线。截2015年5月,伊拉克航空拥有以下机队:","text2":"在1980年代为何订购了更多的波音727与波音737和欧洲的空中客车A310?","label":1} {"text1":"姜紫蓝(Color Keung),香港模特儿。2005年主持有线电视《Channel 27》节目。2006-07 年其中一名主持亚洲电视本港台《诗游记》节目。2006年入亚洲电视工作,亦参与演出不少亚洲电视剧集,电影,例如在电影《金钱帝国》当中饰演陈细九(陈奕迅)的七奶,她也曾在2008年参选亚洲小姐,2009年6月远赴台湾垦丁拍摄其个人写真散文集,是同期第一本含有散文的少女写真集。2012年出版的《弥留人生》,更被台湾书商赏识,全线诚品上架,登上腾讯网首页。现创立自家手作品牌Miss Ginger网店。现为全听摄影师,作家。摄影作品得到美国Viewbug网站颁发「Rockstar 」奖项。2016年获香港尖沙咀K11商场邀请开办首个个人摄影展覧。姓名:姜紫蓝(Color Keung)学历:香港专业教育学院产品设计系,现于中文大学进修中文写作国籍:中国生日:1984年4月3日嗜好:画画,写作,摄影,跳舞和看电影","text2":"姜紫蓝有什么爱好?","label":1} {"text1":"《皇黎一统志》(越:),又名《安南一统志》,是一部成书于约18世纪末19世纪初的越南汉文历史小说。作者是吴氏三兄弟的吴时俧、吴时悠、吴时任。小说描述了越南后黎朝的灭亡和西山朝的崛起这段时期的历史故事。《皇黎一统志》的作者及编者为吴氏三兄弟的吴时俧、吴时悠、吴时任,越南方面有时将其合称为“吴家文派”()。小说的前七回由吴时俧所撰写,吴时俧又名吴俧,字学逊,号渊密,越南山南青威人(今属河内市),为吴时任之弟,领乡荐亚元,官历佥书平章事。第八回至十四回,为其从弟吴时悠所续。吴时悠又名吴悠,字征甫,号文博,有诗文集传世。这部小说后经吴时任编辑。吴时任又名吴任,字希尹,号达轩。30岁中进士,授户科都给事中,33岁兼太原督同行参政,后为郑王世子棕日讲,世子废,擢任为工部右侍郎,曾撰写或参与编写过《十七史撮要》、《四书说谱》、《海洋志略》等书。《皇黎一统志》的最后三回未注作者,是否为吴时任所补不得而知。《皇黎一统志》共十七回,约十二万字。小说反映了公元十八世纪末期黎郑王朝内部的斗争史。小说始于郑森宠邓妃,废嫡立少,致骄兵为变,卒招西山之兵,灭郑扶黎。因朝廷空虚郑氏诸王再次向皇帝要权,黎氏少主(即昭统帝黎维祁)重用权臣阮整,与南方军发生冲突,阮整被杀。昭统帝逃离京城,沿途招募义勇与西山党领袖阮光平对抗,屡为南方军所败,最后不得不向清政府求救,乾隆皇帝命两广总督孙士毅率清军南下以扶黎氏,孙士毅骄傲轻敌,为北平王阮光平所败,只得撤离升龙城。昭统帝亦逃到南宁、桂林,后与皇太后及皇妃被清政府召到燕京(今北京),由于清政府承认阮氏政权,昭统帝被软禁,于乾隆五十八年冬十月(1794年)郁郁而死,时年二十八岁。不久,越南最后一个王朝阮朝的帝王阮福映乘西山朝内部分裂之机回国,夺取嘉定(柴棍的另一名称),打败了阮光平之继任者阮光缵,建立阮朝,越南自此南北统一。小说最后以昭统帝归葬越南作为全书的结尾。","text2":"《皇黎一统志》的作者是谁?","label":1} {"text1":"素面是中国、日本、朝鲜、琉球传统面食,在中国习惯上称之为“挂面”,用小麦面粉制成。与乌龙面一样,素面规格严谨,面身直径通常要求小于1.3毫米。素面味冷而清,可浸于肉汤或丸子中,亦可下至酱油中,而不浇于其他食物之上。酱油通常是鲣海鲜味,可以与大葱、生姜以及青蒜。面用热水煮熟后,使用流动的冷水降温,以使面更加爽滑可口。这样做出的冷却的素面是日本夏季面料理的代表。一到夏季日本各酱油和食品生产商都会生产大量的“素面用”调味剂。此外,荞麦汁比砂糖、味醂更受青睐。关西地方的冷面()一般会添加火腿和黄瓜。药味素面则采用葱、山葵丝、生姜丝、海苔和胡麻等材料。流水素面()是日本一种放在竹制的樋里漂流著,用筷子夹取的素面。可算得上是夏天的风物诗。在鹿儿岛县和宫崎县的一些地方,这也叫「そうめん流し」。发祥地是宫崎县的高千穂峡和鹿儿岛县指宿市的唐船峡。鹿儿岛各地也有专门制作出来以便在餐桌上使用的流水素面器具。鲷素面是濑户内地方壹岐岛的料理。沙拉素面或者素面沙拉,是在素面上加西红柿、生菜等拌成的沙拉的料理,也加蛋黄酱做调味。","text2":"素面是什么样子的?","label":1} {"text1":"高体四长棘鲷(学名:\"Argyrops spinifer\"),又称长棘鲷,俗名盘仔,是辐鳍鱼纲鲈形目鲷科的其中一种。本鱼分布于印度西太平洋区,包括东非、马达加斯加、红海、波斯湾、留尼旺、模里西斯、塞席尔群岛、马尔地夫、斯里兰卡、印度、安达曼海、泰国、缅甸、马来西亚、菲律宾、印尼、日本、台湾、中国沿海、新几内亚、澳洲等海域。水深5至150公尺。本鱼和二长棘犁齿鲷十分相似,蛋头高只有体高的3\/5。背鳍硬棘11枚,其中第一硬棘较短,第二~五或第六硬棘均显著延长,软条10枚;臀鳍硬棘3枚,软条8至9枚。侧线鳞片数50至53枚。体长可达70公分。本鱼为底栖性鱼类,栖息在大陆棚具沙泥底质海域,肉食性,以甲壳类、棘皮动物为食。美味的食用鱼,可做成生鱼片、清蒸或红烧。","text2":"此鱼以什么为食?","label":1} {"text1":"马渡松子()日本歌手、作词家、作曲家。宫崎县都城市出生。代表曲『幽游白书』的OP主题歌「微笑みの爆弾」,第一代ED「ホームワークが终わらない」,第二代ED「さよならbyebye」终代ED「デイドリームジェネレーション」等。在出道前担任过美梦成真的支援乐手,是特殊的同时担任合唱和键盘两个工作的角色(马渡离开之后,美梦成真的合唱就只负责合唱,回到一般惯例。)之后,制作了代表曲『幽游白书』的OP主题歌「微笑みの爆弾」,第一代ED「ホームワークが终わらない」,第二代ED「さよならbyebye」终代ED「デイドリームジェネレーション」等。现在独立制作活动著。和同乡在福冈有活动的电台DJ首藤真吾有往来,曾出演首藤的节目。","text2":"现在马渡松子的工作状态是怎样的?","label":1} {"text1":"橙草(Orangegrass)是一支台湾的摇滚乐团,来自台北,成立于2004年。现由克拉克(主唱兼吉他手)、小蓝(贝斯手)、鸟人(鼓手)三人组成。橙草,成立于2004年,由克拉克Klark(钟礼学)、阿白Jojowhite(白盛方)、阿汤Larsq(汤宗勋)所创成的台北indie rock三人制式乐团。后来团员更动有所更动,加入了小蓝(刘晓蓉)、鸟人(黄士玮)成为了现在的橙草。团员阿白因为种种因素在2010年11月份离开橙草。团员阿汤因为个人因素在2012年7月份离开橙草,热汤计划的演出也是他与橙草最后一次的合作。  如同他们所在的多雨城市,音乐里拥有磅礡淋漓的节奏以及疏离的空气质感。人生的际遇却比这复杂的多。那些值得雀跃的转折,亟欲忘却的伤痕,以及看似平淡却伏流汹涌的每分钟,交相沉积在回忆的行囊里,与一句歌词或几枚音符暗通款曲。一旦正面撞上当下的某个场景或某种情绪,旋律便泉涌而出,在脑海里狂奔绕圈。是一支快速崛起且最受瞩目新世代乐团,广获各大音乐祭邀请演出,并担任2007年美国The Appleseed Cast乐团、德国Maximilian Hecker台北演唱会开场乐团。","text2":"橙草来自哪个地区?","label":1} {"text1":"造纸胡蜂(学名:\"\")是欧洲一种很普遍及著名的胡蜂。牠们在加拿大及美国是入侵物种。过冬后的造纸胡蜂蜂后会在春天的一个时间先行筑巢,并产子。最先出生的雌蜂会成为工蜂继续筑巢。雄蜂会在后来出生,一些雌蜂此时会与雄蜂交配,并离开蜂巢,在下一季成为新的蜂后。蜂群于夏天末就会消失,只余下雄蜂及未来的蜂后聚集在一起过冬。牠们很少会在原有的蜂巢过冬。在形态上,造纸胡蜂蜂后与其他负责繁殖的雌蜂没有分别。不过,由于在蜂群相互作用下不同的角色,牠们在行为上却有著分别。一般而言,蜂后是蜂群之首,牠会产大部份的卵,并且吃下参差的卵。蜂后会花大部份时间在蜂群的相互作用,而其他雌蜂则更多在觅食及照顾雏蜂。这种行为上的差别并非永恒:若蜂后离开了蜂巢,另一只雌蜂(第二大的)就会取代牠的位置。事实上,每只造纸胡蜂都会按牠们的个别的级别位置而变更其行为模式。一些研究显示蜂后会透过行为来压抑其他雌蜂卵巢的发育。蜂后相信是透过摆动腹部来发出支配的讯号,但有研究发现被去卵巢的蜂后失去了限制其他雌蜂繁殖的能力,但却仍维持其蜂后的地位。另外,雌蜂们的身份亦可以由其他因素所影响。若将可能成为蜂后的雌蜂放在蜂巢较早筑成的部份,这只雌蜂就会转而成为工蜂,相反亦然。这显示蜂群在阶级地位上有明显的灵活性。造纸胡蜂蜂后在外表上与工蜂有所分别,蜂后会以腹部打击蜂巢的表面,且会较其他下属多留在蜂巢中,故蜂巢的气味大部份是来自蜂后的。造纸胡蜂的分布地覆盖大部份的南欧及北非、亚洲的温带至中国。牠们亦被引入澳洲、北美洲及南美洲。造纸胡蜂的学名很多时会被误写为\"Polistes gallicus\",但这其实是柞蚕马蜂的学名。造纸胡蜂最初于1791年被命名为\"Vespa dominula\"。种小名的\"dominula\"意为「小妇人」,但一直以来很多学者都将之误并为\"dominulus\"。","text2":"蜂群在什么时候就会消失?","label":1} {"text1":"林极限是在给定的质量被压缩成恒星时的半径最大值。当一颗恒星达到完全的流体静力平衡时—情况是向内的重力和电浆体向外的压力是相配的,这时恒星的大小不会超过林极限所定义的半径。这在恒星的演化上有著重要的涵义,不仅是收缩阶段的公式化,还有稍后经由核融合消耗掉绝大部分供应的氢。赫罗图显示的是恒星的表面温度对应于光度的关系。在图中,林极限大约形成在3,500K的垂线位置。低温的恒星完全都是对流层,而恒星模型对在极限右边的恒星,因为始终在对流中而无法提供恒星平衡的解答(对表面温度更低的恒星),因此,只有所有的期间都在极限左边的恒星能达到流体静力平衡,而在极限右边的区域就形成了\"禁制带\"。但是,在林极限还是有例外,这些包括塌缩的原恒星,因为磁场干扰了恒星内部对流层能量的传输。红巨星是核心进行氦融合反应而使外面的气壳层膨胀的恒星,这会使恒星在赫罗图上向上和向右移动。但是,由于林极限的抑制,它们的膨胀有一定的半径限制。林极限的名称取自于日本天文物理学家林忠四郎()。","text2":"为什么林极限还是有例外情况的?","label":1} {"text1":"威廉·凯姆勒(,),美国纽约水牛城出身,是历史上第一个被以电椅进行死刑处决的人。凯姆勒在1889年3月29日以短柄斧杀害自己的情妇泰莉·齐格勒(Tillie Ziegler),随后被判处死刑,定于1890年8月6日早上七点于纽约的奥本监狱(Auburn Prison)执行。然而因为电流不足,首次电击后威廉·凯姆勒仍有呼吸,第二次电击让他的皮下血管破裂流血、身体着火却仍然不死。直至第三次电击才死去,但已是行刑后第8分钟。一位在场目击的记者形容这是「一个可怕吓人的场面、远比绞死还糟」(an awful spectacle, far worse than hanging.)。凯姆勒在1889年3月29日以一把短柄斧谋杀了他的情妇玛蒂尔达·“泰莉”·齐格勒,被判处死刑,在奥本监狱以电椅执行。他的律师提出上诉,认为电椅执行死刑是一种残酷和不寻常的惩罚。赞同将交流电作为主要的电力供应标准的支持者之一,乔治·威斯汀豪斯也表示支持他的上诉。但是上诉失败,部分原因是由于托马斯·爱迪生的州政府立场支持(爱迪生是直流电电力供应的支持者,他被推测希望利用电椅来向人们宣传交流电的危险性)。","text2":"威廉·凯姆勒的律师为什么上诉失败?","label":1} {"text1":"盘龙江又名滇池河,是云南省昆明市的一条河流,发源于嵩明县梁王山西麓的白沙坡,上游也叫牧羊河,在盘龙区松华街道注入松华坝水库,后由北向南流经昆明市区,最终在官渡区六甲街道洪家大村附近流入滇池,被昆明市民称为昆明的母亲河。盘龙江全长108公里,流域面积847平方公里,平均流量7.17立方米\/秒,年径流量2.75亿立方米,松华坝水库以上河段平均宽8米,以下平均宽35-40米,松华坝水库以下河段由人工改直。2005年昆明市区四个区(五华区、盘龙区、官渡区、西山区)重新划分时,盘龙江被作为东西分界线,得胜桥是昆明市主城四区的交界点。盘龙江上共有51座桥梁,从源头到河口依次为:龙川桥、幸福桥、神龙桥、大花桥、鱼龙桥、翠龙桥、巨龙桥、北仓桥、飞龙桥、蟠龙桥、升龙桥、霖雨路桥、霖雨桥、吴上村老桥、吴上村新桥、金色大道、跃龙桥、马村桥、官房桥、环北桥、敷润桥、圆通桥、桃源捌角桥、博润桥、南太桥、宝尚桥、得胜桥、小人桥、双龙桥、环南桥、永安桥、永平桥、官南立交桥、南坝桥、\"新建未命名\"、锦苑桥、乘龙桥、谷昌桥、博汇桥、向日葵忠字桥(陈家营)、广福路跨江桥、向日葵忠字桥(张家庙)、小钢桥(张家庙)、向日葵忠字桥(叶家村)、向日葵忠字桥(严家村)、向日葵忠字桥(金家社区梁家村)、小钢桥(梁家村)、向日葵桥(金家村)、环湖路桥、洪闸桥。元朝初期,赛典赤·赡思丁曾率人整治盘龙江,盘龙江上最早的龙川桥建于公元1280年前,相传为瞻思丁修建。","text2":"盘龙江的流域面积有多大?","label":1} {"text1":"吕方(Lui Fong;),生于南京,出道于香港的资深歌手,也是前香港无线电视合约艺人。吕方于1962年出生,曾就读于庇护十二学校及模范英文中学,毕业后,在1983年参加无线电视及华星唱片合办的「第二届新秀歌唱比赛」,凭《我是中国人》一曲夺冠而入行。1984年再于「第二届香港新人歌唱大赛」获得冠军。吕方与张学友出道早期都受到唱片公司重视,两人于1985年分别推出专辑《听不到的说话》与《smile》,并于1986年联袂在红磡体育馆举行一连数场的《86双星演唱会》——2007年,张学友担任吕方演唱会的嘉宾,与之合唱《李香兰》,唱完后盛赞吕方的歌唱实力,说:「香港歌手必须懂跳舞才可以出成绩,而吕方是一个只需要唱歌,不跳舞都可以成功的歌手。」吕方早期事业发展不俗,《你令我快乐过》、《每段路》、《只因我太痴》、《弯弯的月亮》、《雪山飞狐》、《心语》等歌曲皆广为人知。其唱腔清澈响亮,但形象不独特,令他在乐坛经营多年却始终没有突破,而且曾有一段长时间没有推出唱片和歌曲。他与香港著名演员及主持人郑裕玲的恋情反而成为众人之焦点。2005年,吕方和郑裕玲公开表示他们有「不婚,不生育」的共识。三年后,二人宣告分手,结束长达16年的恋情,并表示这是彼此长谈后的决定,并没有撕破脸,将来仍是好朋友。据香港报纸报道,二人分手并非因第三者,也与金钱无涉。郑裕玲接受杂志访问时,透露自己已年近半百,想在往后的人生做自己想做的事、有自己的空间。由于受到更年期的困扰,郑裕玲经常会暴跳如雷,吕方只要晚一点回家,她就会发疯似的赶他下楼。而吕方也想搬出去住,所以达成分手协议时并没流泪也无不悦。吕方至今未婚。((1))代表两星期冠军","text2":"2007年的演唱会,谁担任吕方演唱会的嘉宾?","label":1} {"text1":"瑞士联邦外交事务部(英语:Federal Department of Foreign Affairs,法语:Département fédéral des affaires étrangères,德语:Eidgenössisches Departement für auswärtige Angelegenheiten,意大利语:Dipartimento federale degli affari esteri) ,是瑞士联邦政府所属的七个部之一,主要负责维护瑞士联邦的外交关系事务,开展有关的外事活动和往来。瑞士联邦委员会的七名成员之一的外交部长负责本部门工作。该部原名瑞士政治部,1978年改为现名。现任外交事务部部长是联邦委员会委员,社会民主党成员米舍利娜·卡尔米-雷伊(Micheline Calmy-Rey)。瑞士联邦外交事务部下设总秘书处和国务秘书处,该部门的相关局和下属机构主要包括政治事务局、企业管理局、公共国际法局、瑞士发展与合作署等。联邦外交事务部总秘书处,主要负责配合联邦外交事务部部长管理与协调部内的相关工作,计划与协调部内相关各个部门之间的活动和工作。总秘书处还负责有关公共关系、督导瑞士联邦驻外国使领馆的工作,协调部内的通讯联络信息和技术等。","text2":"一般是由谁来负责这个部门的日常工作?","label":1} {"text1":"西萨塞克斯(郡)(),英国英格兰东南部的郡,南临英伦海峡。奇切斯特是郡治。以人口计算,西萨塞克斯没有包含单一管理区,无论把它看待成非都市郡还是名誉郡,其面积都是1,991平方公里(非都市郡第27、名誉郡第30),人口都是770,900(非都市郡第9、名誉郡第27)。1189年,萨塞克斯在地理上被分为东萨塞克斯、西萨塞克斯两部分。《1888年地方政府法案》在1888年4月1日生效后,东、西部分升格为行政郡,有独立的郡议会。不过,法案没有废除萨塞克斯郡,如把它看待成名誉郡,它仍包含东、西萨塞克斯两部分。《1972年地方政府法案》在1972年4月1日生效后,萨塞克斯郡连仅有的名誉郡地位也丧失掉,被正式废除;《1888年地方政府法案》新设的行政郡被全部废除,原有行政郡地位的郡改获其他郡级地位。东、西萨塞克斯都获名誉郡、新设的非都市郡地位。在1888年起属于东萨塞克斯郡西部的海沃兹希思、伯吉斯山、东格林斯蒂德等地,改归西萨塞克斯郡管辖。以下经济数据来源是英国国家统计办公室西萨塞克斯实际管辖7个非都市区:沃辛(Worthing)、阿伦(Arun)、奇切斯特(Chichester)、霍舍姆(Horsham)、克劳利(Crawley)、中萨塞克斯(Mid Sussex)、埃德(Adur)。下图显示英格兰48个名誉郡的分布情况。西萨塞克斯东北与肯特郡相邻,东与东萨塞克斯郡相邻,南临英伦海峡,西与汉普郡相邻,北与萨里郡相邻。布莱克当海拔280米,是郡内最高点。西萨塞克斯超过一半的土地都是受保护的郊野,适合市民、旅客散步和踏单车。","text2":"西萨塞克斯在哪里?","label":1} {"text1":"广州国际女子网球公开赛是一项由女子网球联合会(WTA)在中国广州举行的室外硬地网球巡回赛事。该项赛事为WTA三级赛,从2004年起每年举办1次,总奖金为22万5千美元。参赛选手根据比赛成绩计算积分,累计至女子网球联合会的排名系统。赛事的首个单打冠军是中国运动员李娜于2004年夺得,这同时也是中国网球运动员获得的首个单打冠军头衔。广州国际女子网球公开赛采用《ITF网球比赛规则》和《WTA赛事规则》,为单打正选赛32人,双打正选赛16队的单淘汰赛。每场比赛采用三盘两胜制,每盘比赛在局数比分是6比6时,采用决胜局制(七分先胜利);双打第三盘按决胜盘制(十分先胜利)。","text2":"赛事的首个单打冠军是谁?","label":1} {"text1":"《黄金B段班》()是台湾中天娱乐台于2008年7月21日至2009年2月19日,每星期一至星期五(2008年11月7日起改为每星期一至星期四)晚间九点播出的谈话性综艺节目。该节目找来最当红的「B咖」主持,以艺人角度为出发点,探讨艺人辛苦在演艺圈中奋斗且不为人知的一面。「B咖」是在台湾演艺圈最近兴起的新名词,「咖」字在台语中与「脚」同音,与英文的B字组合起来后,意思就是二线演员、配角。「黄金B段班」在意义上具有黄金等级的B咖所开的班级。2008年7月21日,《黄金B段班》开播,主持人小钟、小马、沈玉琳、小甜甜、佩甄及韦汝开始推辞上9点谈话节目;模仿康康出名的黄镫辉,则要在《黄金B段班》中扮演综艺B咖中的B咖,立志作个有质感的谐星,要与陈汉典抢饭碗!《黄金B段班》企图开创另类谈话节目型态,加入类戏剧元素,给6位主持人合乎自己性格的角色,节目暂定八人挑梁主持,除小钟、小马、沈玉琳、小甜甜、佩甄及韦汝6人确定外,还有两名人选难产中。制作单位透露,会由戏剧界去寻找新面孔,缺乏一个美艳的谐星,还有可爱的傻大姊。 佩甄对新节目收视率有信心,「你看小钟负责客家族群,我负责带入婆婆妈妈收看,韦汝吸引年轻人,小马也有非洲支持者,8个人一个人只要负责0.25(%)的收视率,轻轻松松就能破2(%)。」有时候会以三男\/三女主持,会称另外三位为黄金三宝,加了口号『黄金B段班! 旺旺!』,每集片尾由沈玉琳独挑大梁的「沈语录」是该节目特色之一。





","text2":"《黄金B段班》企图做出什么不同于以往的特色?","label":1} {"text1":"美眉校探()是一部美国青春剧及侦探剧,编剧是罗伯·托马斯。该剧2004年9月22日首播,在UPN播放两年,并在2007年5月22日结束。该剧由克莉丝汀·贝尔主演。该剧主角是韦罗妮卡,一位高中毕业生,她在侦探父亲的指导下成为了一名私人侦探。编剧托马斯原本将该书写成一本年轻人的小说,将一位男性设定为主角。章节(剧集)呈一种特殊的结构:韦罗妮卡在解决一些毫不相同的较小案件的同时还在试图解决一个贯穿一季的谜团。该电视剧系列的前两季包含一个贯穿一季的谜团,在这一季的第一集中被详细介绍并最终在结尾被解开。第三季的编排与前两季相比有所不同,主要集中在一个延续几集的谜团上。广受好评的第一季(22集)在美国获得了每集250万人的收视反响。该剧曾多次登上各种最佳电视剧名单,并多次获得各种奖项及提名。在播映期间,它曾获得两项卫星奖提名,四项土星奖提名,五项青少年选择奖提名,并在2005年美国电影协会年度电视节目中成为重点。在CW电视台2007年度预告中,CW电视台董事长宣布该片停播。","text2":"美眉校探的编剧是谁?","label":1} {"text1":"双屿港,位于今天浙江省舟山市六横岛,十六世纪远东与西方贸易的重要港口,也是明朝海盗集团王直等人的根据地。大约在十六世纪20年代由明朝海盗与葡萄牙人所建,设有市政厅、教堂、医院及1000幢民居,人口3000以上。“佛朗机之来,皆以其地胡椒,苏木,象牙,苏油,沉、东、檀、乳诸香,与边民交易,其价甚平,其日用饮食之资于吾民者,如米、面、猪、鸡之数,其价皆倍于常,故边民乐与为市。”。由于明朝的锁国封闭政策,实行海禁,嘉靖二十七年1548年,浙江巡抚朱纨派都指挥卢镗率兵进攻双屿港,港口以木石淤塞,城市被大火烧光。数百葡萄牙人被杀,侥幸活命的乘船逃走。后来,葡萄牙人往南撤往广东澳门,明朝广东官员默许葡萄牙人在澳门借地开埠经商。","text2":"双屿港位于什么地方?","label":1} {"text1":"中国政法大学弑师案,是指2008年10月28日晚发生在中国政法大学的一场由于师生恋而引发的血腥惨案。该校法学院教授程春明在该校端升楼201教室内,被该校男学生付成励用菜刀当众袭击身亡。此案引发巨大的社会轰动和深刻的道德反思。2009年10月20日,北京市第一中级人民法院一审宣判,以故意杀人罪判处23岁的付成励死刑,缓期两年执行。2007年4月,付成励结识了中国政法大学女研究生陈某,交往了几个月后,2007年8月,付成励与初恋女友陈某正式确立了恋人关系。在后来二人去北戴河游玩期间,陈某把自己曾和中国政法大学教师程春明发生过一年多性关系的事告诉付成励。知道这件事后,付成励的情绪受到沉重打击,遂对程春明动了杀机。","text2":"中国政法大学弑师案发生于什么时候?","label":1} {"text1":"利府车站()是一由东日本旅客铁道(JR东日本)所经营的铁路车站,位于日本宫城县宫城郡利府町森乡字柱田。利府是JR东日本东北本线利府支线的终点站,管辖上位于JR东日本仙台支社的范围之内,由岩切车站管理,设有绿窗口()。利府车站在初设时,原本只是东北本线沿线的一个中途站而已,但1944年起被称为「海线」、坡度较缓的新线通车之后,原本经由利府的「山线」就成了使用率越来越低的旧线。终于在1962年时,在利府~(旧)松岛之间的山线路段废线之后(至于松岛~品井沼之间的路段则在此三个月前就已废线),利府成了剩余路段的终点站。配合这改变,原本是东北本线主线的山线剩余路段,也正式改为利府支线继续营运。港湾式月台2面2线的地面车站。","text2":"利府车站是哪家公司经营的车站?","label":1} {"text1":"2008年世界大学棒球锦标赛(World University Baseball Championship 2008)为第四届世界大学棒球锦标赛(World University Baseball Championship)赛事。该赛事于2008年7月17日至7月27日于捷克的布尔诺、俄斯特拉发、布兰斯科(Blansko)、丘曾(Choceň)与奥洛穆克举行。总共有8个国家要参加这个赛事,分别为加拿大、台湾、捷克、日本、美国、立陶宛、印度与韩国。其中,印度与立陶宛是首度参加此赛事。本届赛制为八队打循环赛,取出前四名进入复赛,复赛则是采单淘汰排名赛。","text2":"此赛事的举办日期是多久?","label":1} {"text1":"废太子即被废黜太子地位的皇子。他们是因为某些原因如犯罪、生母失宠、品行不端、本人失宠、有更佳的继承人、父皇被权臣威胁被逼废黜太子等而被废去太子之位。他们的身份除了降为一般的皇子外,也可能被封其他爵位如亲王、郡王、公等,而妻儿的地位也会下降,如正妻由太子妃降为王妃、公夫人等,原本有位阶的侧室如良娣等失去位号,若废太子降为王爵,侧室统称为侧妃,但她们并没有正式位号。而犯了大罪的废太子则会被贬为庶人甚至处死。另一种情况是,王朝、君主制灭亡或父亲被废黜君主之位,太子本人形同被废。在欧洲,失去继位可能的储君,称为王位觊觎者。明代宗独子朱见济在被立为太子的次年夭折。数年后,伯父明英宗复辟,将之追废为世子。而之前废除太子之位的明英宗长子朱见深也因父亲重新登基,复立为太子。清康熙年间,太子胤礽两立两废。其后雍正帝设立秘密建储制度,终清一朝再没有公开册立太子。","text2":"犯了大罪的废太子会受到什么刑罚?","label":1} {"text1":"地热谷是一处硫气及温泉的出口,位在台湾北投中山路的阳明山山谷洼地,当地人又称「磺水头」或「鬼湖」 ,因长年蒸气弥漫、热气腾腾,使人联想成恐怖的地狱。早年亦曾有游客失足跌入高温泉水中,被活活烫死,因而另有别名「地狱谷」。在日治时期,地热谷被评为台湾八景十二胜之一,有「磺泉玉雾」美誉。原因是日出晨光照射在温泉冒起的蒸气,幻化出变化多端的光影,如身处如梦似幻的仙境当中。泉水清澈微绿似玉,所以有人称为「玉泉谷」。地热谷的泉水属酸性硫磺泉,水温高达摄氏90度,水质清澈且呈微绿似玉的色泽,俗称青磺泉。水中酸碱度达PH值1.4~1.6,腐蚀性极高,因此不宜久泡。虽然如此,地热谷温泉亦为北投温泉的源头之一,原因是泉中含微量放射性元素镭,对人体有保健的功效,主要是供应为新北投公园一带的温泉浴室或旅馆,包括历史悠久的泷乃汤。早期当地民众常携带鸡蛋到地热谷煮「温泉蛋」,后来发生烫伤及环境保护考量封闭。维护管理的台北自来水事业处也表示地热谷温泉属于硫酸盐泉、硫酸盐氯化物泉,含有重金属,有危害人体健康之虞,不建议引水煮蛋。现在政府重整地热谷景观,导引泉流,蓄水成湖,并以步道、扶栏环绕四周,步道一旁设有赏景凉亭,周边还有以石子与水泥筑成亲水沟渠,游客可免费将双脚泡在温泉中,市政府已将新北投公园连同地热谷规划为「北投温泉亲水公园」。","text2":"地热谷的水质如何?","label":1} {"text1":"杨元龙()祖籍江苏吴县(今苏州),香港知名爱国实业家,溢达集团创始人。先后获得上海圣约翰大学化学硕士学位和美国麻省洛厄尔纺织学院纺织化工硕士学位。杨元龙爱好养马,八十年代曾被冠以「马王」美誉。1986年卷入造马案,杨元龙在案件开审后承认控罪,但却以患上末期骨癌为由,并且得到多位医生联合证明他只余几星期寿命,以此为理由向法官求情免刑。结果,杨元龙被判监两年,缓刑两年,避过牢狱之苦,但罚款五百三十万元。杨元龙随即往美国医病,1989年返港,并经常神采飞扬地出席名流及商界等场合,并从事地产投资,杨元龙八十岁大寿时曾于港岛一间五星级酒店举行盛大豪门夜宴。1949年后,杨元龙和夫人杨蔡咏芳在香港从事纺织制衣业。1961年至1973年,连续12年担任香港棉业咨询委员会(后易名为纺织业咨询委员会)委员,代表香港政府参与“关税及贸易总协定”及“多种纤维协定”的制订;兼活跃于中华厂商联合会、香港工业总会、香港棉纺业同业公会、贸易发展局等行业组织。1978年创办溢达集团,一家纺织及制衣企业,全棉衬衫制造商和出口商。杨元龙名下兼创立侨美旅游事业有限公司,投资开办北京香山饭店。夫人杨蔡咏芳,长女杨敏德(溢达集团主席,曾任行政会议成员,现任全国政协委员、香港理工大学校董会主席),次女杨敏贤。","text2":"杨元龙回美国治病后,什么时候返港?","label":1} {"text1":"阿尔塔薛西斯一世(古波斯楔形文字: ; \"Ardeshir\";;), 是波斯帝国的国王(前465年—前425年在位),泽克西斯一世之子。禁卫军司令阿尔达班谋杀泽克西斯之后,把他扶上王位,几个月后阿尔塔薛西斯亲手杀掉阿尔达班。在他统治期间,帝国曾发生几次叛乱。与希腊在欧里梅敦战役之后,帮助雅典人的敌人对抗雅典。前451年,阿尔塔薛西斯在埃及大败埃及-希腊联军,并摧毁了希腊包括援军在内的所有军队,雅典因丧失了对地中海的绝对统治而感到恐惧,便将提洛同盟的金库从提洛岛移入雅典卫城,这不可避免地导致雅典与波斯帝国在公元前450年重开战端,但客蒙率领雅典军队对塞浦路斯岛上的奇提翁城邦(今拉纳卡)的围困无果而终,客蒙本人也于同年去世。塞浦路斯的萨拉米斯之战后,双方均无力再战,随即于公元前449年签订卡里阿斯和约。阿尔塔薛西斯一世曾庇护被雅典放逐的地米斯托克利,并任命他为马格尼西亚总督。","text2":"泽克西斯是被谁谋杀的?","label":1} {"text1":"裸狐鲣(学名:),又称裸䲠,俗名长翼,为辐鳍鱼纲鲈形目鲭亚目鲭科的一种。本鱼分布于印度太平洋区,包括南非、东非、红海、叶门、模里西斯、马达加斯加、葛摩、塞席尔群岛、留尼旺、马尔地夫、斯里兰卡、印度、缅甸、安达曼群岛、圣诞岛、可可群岛、中国、日本、韩国、台湾、菲律宾、越南、印尼、澳洲、斐济、新喀里多尼亚、东加、吐瓦鲁、密克罗尼西亚、萨摩亚群岛、法属波里尼西亚等海域。水深0至100公尺。本鱼体纺锤形,横切面近圆形,两背鳍几乎相连,在第二背鳍后方有6至7个离鳍,而在臀鳍后方有6个离鳍。尾鳍末缘呈弧形凹入,侧线完全,侧线后半部呈波浪状起伏,且在尾柄处和隆起的棱嵴相连。体背侧呈蓝紫色,腹部灰白色,身上无明显花纹或斑点。上下颔均具尖锐犬齿,背鳍硬棘13至15枚;背鳍软条12至14枚;臀鳍硬棘0枚;臀鳍软条12至13枚;脊椎骨38个,体长可达248公分。本鱼为沿岸中层洄游性鱼类,常一大群一起出现,属肉食性,主要以小鱼为食。为美味的食用鱼,属经济鱼种,可煮汤、盐烧或制成罐头,有雪卡鱼中毒之纪录。","text2":"裸狐鲣主要以什么为食?","label":1} {"text1":"俺巴孩(;),《元史》作咸补海罕或罕布海,是蒙古诸部落统一前蒙兀国的“合罕”(即“汗”)之一。俺巴孩是成吉思汗曾祖父合不勒汗的堂兄弟和继承者。他原本是泰亦赤兀惕系的首领。《元朝秘史》和《拉施特书》将其尊称为“汗”。在合不勒汗时期,蒙古同塔塔儿因为一个巫医引发的事件发生冲突。或为平息冲突,俺巴孩登上汗位后,答应将其女儿嫁给一个塔塔儿的部落(主因人)首领,但在护送时和合不勒的长子斡勤巴儿合黑一起被塔塔儿人抓住,送与金朝献给金熙宗。俺巴孩被抓住后曾托人对合达安太子说:“今后以我为戒,你每将五个指甲磨尽,便坏了十个指头,也与我每报仇。”熙宗将俺巴孩钉在木驴上处死,这是“专惩治游牧叛人”的刑罚。之后成吉思汗在讨伐金国之前曾祭腾格里时曾说,讨伐金国是为了报其祖先俺巴孩等人的仇恨。据《元朝秘史》记载,俺巴孩死后,汗位由侄子忽图剌继承。","text2":"俺巴孩原本是哪个系的将领?","label":1} {"text1":"徐大椿(1693年-1771年),原名大业,字灵胎,号洄溪,江苏吴江人。清朝医学家。性通敏,喜豪辩。自《周易》、《道德》、《阴符》家言,以及天文、地理、音律、技击等无不通晓,尤精于医。初以诸生贡太学。后弃去,往来吴淞、震泽,专以医活人。乾隆二十五年(1760年)大学士蒋溥患病,乾隆帝命徐大椿入京医治,大椿直言蒋之病已不可治。乾隆三十六年(1771年),奉诏回京,当时大椿已重病在身,由其子徐爔陪同,勉强支撑到京城。第三天,自拟对联:“满山芳草仙人药,一径清风处士坟”。当夜病逝。大椿著书颇多,有《兰台轨方》、《医举源流》、《论伤寒类方》等,都为医学之籍。他的歌曲有《洄溪道情》三十余首。","text2":"徐大椿是哪里人?","label":1} {"text1":"旁系群在支序分类学是指一个演化支上某个单系群最亲近的分类群,又称为姐妹群。如下图所示:单系群A是单系群B的旁系群,同时B也是A的旁系群,两者从一个最近共同祖先演化而来,而关系较远的C就是它们的外类群。A和B也可以组成一个单系群单元D,此时这个单元D就是C的旁系群,反之亦然……,以此类推,用以概括更远的旁系群。在支序分类学的定义上,A、B、C可以是生物样本、物种或分类单元。在描述物种时,有时会以“姐妹种”()来称呼。在种系发生学的亲缘关系研究中使用“旁系群”这个概念时,一般只考虑研究中所列举的样本,例如:在现存物种当中,鳄鱼是鸟类最亲近的旁系群,但这仅仅是对现生种而言;如果将已灭绝的种类算进来,鸟类作为恐龙的直系后代,鸟类和鳄鱼之间还隔着许许多多的已灭绝的旁系近亲。所以,需留意“旁系群”这个概念只是一个相对的概念,仅仅用来描述研究中所列出的类群、物种或样本之间相对的亲疏远近关系。","text2":"单系群A和单系群B可以组成一个独立的什么?","label":1} {"text1":"棘眦海蛇属(学名:),是蛇亚目海蛇科下的一种有毒单型蛇属,属下只有棘眦海蛇(\"Acalyptophis peronii\")一种海蛇,主要分布于南中国海、澳洲等海域。棘眦海蛇主要分布于泰国湾、越南、南中国海、印尼、菲律宾、新畿内亚、澳洲(包括新南威尔士、北领地、昆士兰、西澳州)等地区的海域及海岸。该物种的模式产地在澳大利亚。Duméril, ANDRÉ MARIE CONSTANT 1853. Prodrome de la classifcation des reptiles ophidiens. Mém. Acad. Sci., Paris, 23: 399-536","text2":"棘眦海蛇属是否有毒?","label":1} {"text1":"时间片()又称为“量子()”或“处理器片()”是分时操作系统分配给每个正在运行的进程微观上的一段CPU时间(在抢占内核中是:从进程开始运行直到被抢占的时间)。现代操作系统(如:Windows、Linux、Mac OS X等)允许同时运行多个进程 —— 例如,你可以在打开音乐播放器听音乐的同时用浏览器浏览网页并下载文件。事实上,由于一台计算机通常只有一个CPU,所以永远不可能真正地同时运行多个任务。这些进程“看起来像”同时运行的,实则是轮番穿插地运行,由于时间片通常很短(在Linux上为5ms-800ms),用户不会感觉到。时间片由操作系统内核的调度程序分配给每个进程。首先,内核会给每个进程分配相等的初始时间片,然后每个进程轮番地执行相应的时间,当所有进程都处于时间片耗尽的状态时,内核会重新为每个进程计算并分配时间片,如此往复。通常状况下,一个系统中所有的进程被分配到的时间片长短并不是相等的,尽管初始时间片基本相等(在Linux系统中,初始时间片也不相等,而是各自父进程的一半),系统通过测量进程处于“睡眠”和“正在运行”状态的时间长短来计算每个进程的交互性,交互性和每个进程预设的静态优先级(Nice值)的叠加即是动态优先级,动态优先级按比例缩放就是要分配给那个进程时间片的长短。一般地,为了获得较快的响应速度,交互性强的进程(即趋向于IO消耗型)被分配到的时间片要长于交互性弱的(趋向于处理器消耗型)进程。","text2":"一台计算机能否同时运行多个任务?","label":1} {"text1":"触手冠动物(Lophophorata),一切具有触手冠(口周围成扇形排列的一圈具纤毛之触手)的无脊椎动物。包括苔藓动物门(Bryozoa)、腕足动物门(Brachiopoda)、帚形动物门(Phoronida)。主要靠纤毛运动时将带有食物粒子的水流送入口中来摄取养分。与担轮动物(Trochozoa)缩合而成冠轮动物总门。包括传统上的輭体动物门(Mollusca)、环节动物门(Annelida)、纽形动物门(Nemertea)、星虫动物门(Sipunculida)以及触手冠动物。此外根据18S rRNA序列,轮形动物门(Rotifera)和新近发现的环口动物门(Cycliophora)也很可能属于冠轮动物。","text2":"触手冠动物包含哪些?","label":1} {"text1":"泉州府是明清时期福建省下辖的一个府,范围大概是今日的泉州、厦门与金门。明朝洪武初年(1368年),泉州路改为泉州府,隶属福建行省,辖晋江、惠安、南安、同安、安溪、永春、德化七县。洪武九年,福建行省改为福建布政使司,泉州隶之。洪武二十年(1387年),撤澎湖巡检司。洪武二十五年(1392年)九月,置漳泉道。嘉靖四十二年(1563年),重设澎湖巡检司。南明永历九年(清朝顺治十二年,1655年),郑成功在厦门置思明州。康熙十九年(1680年)思明州废。康熙二十五年(1686年),以同安县嘉禾屿等地置厦门厅。雍正十二年(1734年),升永春县为永春直隶州,划德化县并延平府大田县归其管辖。乾隆四十年(1775年),从同安析置马巷厅,驻马家巷(今翔安区马巷)。中华民国元年(1912年),取消府、州、厅,实行省、道、县三级地方政制,下辖各县归属南路道。民国3年(1914)废除清制,置兴泉永道(治所泉州),原清朝兴泉永道之泉州府、永春直隶州、兴化府所辖各县隶之。思明府改名厦门道,厦门成立思明县,金门隶属思明县,派分治员驻金门。《泉州府志》最初于南宋嘉定年间(1208年-1224年)编撰。现存最早的一部泉州志书是明朝万历四十年(1612年)泉州知府阳思谦聘请原礼部尚书黄凤翔、行省解元李光缙所编修。","text2":"《泉州府志》最初于什么时候编撰?","label":1} {"text1":"澎湖玄武岩自然保留区位于台湾澎湖县,包含锭钩屿、鸡善屿及小白沙屿3个无人岛,主要的保育对象为当地特殊的玄武岩地形景观。澎湖群岛是台湾三大火山群之一,除了花屿之外,各岛大多由玄武岩组成。火山熔岩流在冷却时体积收缩,形成玄武岩的柱状节理,呈现六角柱或多角柱结构。随后由于海蚀及其他地形作用的影响,形成许多高低起伏、变化多端的柱状玄武岩。保留区位于黑潮支流、南海季风流、及潮汐流交会处,邻近海域蕴藏丰富的渔类资源。鸟类大多为冬季候鸟和过境鸟,常见鸟种包括黄足鹬、大苇莺和家燕等。夏季候鸟以苍燕鸥和白眉燕鸥较多,其次为小燕鸥和红燕鸥。澎湖县野鸟学会2014年6月在保留区内的鸡善屿发现有黑嘴端凤头燕鸥在此繁殖,是马祖列岛、韭山列岛后,确认的第三个繁殖地。「世界遗产」登录工作有许多前瞻性的保存观念,为使国人保存观念与国际同步;2002年初,文化部(原:行政院文化建设委员会)陆续征询国内专家及函请县市政府与地方文史工作室提报、推荐具「世界遗产」潜力点名单;其后于2002年召开评选会选出11处(2009年经会勘后,增加至17处)台湾世界遗产潜力点,澎湖玄武岩自然保留区便是其中之一。澎湖玄武岩自然保留区是由地底流出的火山熔岩冷却形成后,形成各式的柱状玄武岩。由玄武岩组成的岛屿受到海蚀作用形成海崖、海蚀洞、海蚀柱、海蚀沟等天然美景,在亚洲地区群岛中更是少见,正符合世界遗产登录标准的第七项。澎湖玄武岩自然保留的地质年代是台湾海峡火山熔岩最活跃的年代,至今仍保留非常独特与优美的玄武岩地景,其雄伟柱状节理及丰富的地形变化符合世界遗产登录标准的第八项。澎湖玄武岩自然保留位处偏远,海流湍急、岩壁陡峭,人迹罕至,因此,每年4月至9月已成为保育类珍贵稀有鸟类的繁殖天堂,2002年更发现濒临绝种的海洋野生动物—绿蠵龟上岸产卵,极具研究与保育价值,符合世界遗产登录标准的第十项。","text2":"澎湖县野鸟学会2014年6月在保留区内的鸡善屿发现了什么?","label":1} {"text1":"《万有引力》()是村上真纪的漫画作品。在《你与我》连载。另外还有电视动画、OVA、广播剧CD等作品。第1部漫画全12卷、第2部漫画《万有引力EX》现在连载中,已出版单行卷2本、相关连载还有《新堂家的事情》(原名《新堂家の事情》、连载、7话完结、未出单行本。)跳脱出同志作品给人悲伤和不可告人的既定印象,作品主要是偏向喜剧的方式进行,而男主角愁一和瑛里的关系在本作也是公开的并获得其他配角的支持,这在同类型的作品当中算是非常少见的。主角新堂愁一是「BAD LUCK」的主唱。一天晚上,愁一在公园里与超美型人气小说家由贵瑛里命运地相遇。对于批评他写的歌词的由贵,愁一一直耿耿于怀。","text2":"《万有引力》的作者是谁?","label":1} {"text1":"副剑齿虎(学名:Paramachairodus)是一种已灭绝的剑齿虎亚科剑齿虎。分布于1500万至900万年前中新世末期的欧洲和亚洲。副剑齿虎是已知最古老的剑齿虎,大量副剑齿虎化石发现于西班牙的马德里附近的一个中新世晚期化石点Cerro Batallones。有两具豹形化石比较出名,一具是大约处在瓦西里期到Turolian期的“Paramachairodus ogygia”,另一具则是完全处在Turolian期“Paramachairodus orientalis”。大多数学者认为发现的第三个物种“Paramachairodus maximiliani”可能和第一具化石是同种的。副剑齿虎肩高约58厘米,与美洲豹相似,但体型更加轻盈。四肢形状显示牠们是灵活的攀爬者,且可以猎杀相对大型的猎物。","text2":"哪里发现了大量副剑齿虎化石?","label":1} {"text1":"黄贵妃(),本名、出身不详。宋光宗贵妃。黄氏在淳熙末年时为宋孝宗后宫嫔妃,封为和义郡夫人,黄氏姿色美丽,温和贤慧。宋光宗为皇太子时,因身边除了太子妃李凤娘以外,别无妾侍,孝宗便将夫人赐给太子为妾,得到太子专宠。太子在1189年即位后,便封黄氏为贵妃。绍熙二年十一月辛未(1191年12月14日),传出黄贵妃怀孕的消息,引起李凤娘嫉妒,决定将她害死,光宗因郊祭的关系前往斋宫居住,李凤娘便趁机将黄贵妃杀害,宣称贵妃暴卒。第二天在光宗祭祀的现场,风雨大作,祭坛上的蜡烛都被吹熄,无法继续进行祭典。光宗返宫以后,听闻贵妃暴卒,又因为祭典上风云变色之事,从此一病不起,不视朝政,自此政事多取决于李皇-{后}-。宋光宗另有侧室张贵妃、符婕妤,她们则都被送出宫外,嫁给寻常百姓。","text2":"黄氏是何时被封为贵妃的?","label":1} {"text1":"安加尔战役第二次世界大战太平洋战争中,于1944年9月17日至9月30日发生在帛琉群岛中安加尔的一场战役。安加尔是一个细小的火山岛,大约3英里(4.8公里)长,与贝里琉相隔6英里(9.7公里)的海峡,岛上人口非常少,居民从事农业、捕鱼及磷酸盐开采,到1944年中岛上共有驻军1,400人,由帛琉地区司令部司令井上贞卫中将统一指挥。帛琉群岛的脆弱防守及群岛可作为兴建机场的潜力令这些岛屿成为美军在攻占马绍尔群岛后的另一个目标,但由于缺乏登陆艇,令进攻帛琉群岛的行动必须要在美军攻占马里亚纳群岛后才能展开。炮轰安加尔的行动由田纳西号战列舰、数艘巡洋舰及由胡蜂号航空母舰起飞的SBD无畏式俯冲轰炸机于1944年9月11日实施,6天后的9月17日,由保罗·梅拿指挥的美军第81步兵师在岛的东北及东南海岸登陆,水雷及登陆滩头的挤塞状况比日军的反攻带来更大的麻烦,但当美军前进到盆地后抵抗便变得顽强,日军计划利用岛上西北部玉石湖附近的一座山头作为最后的抵抗据点,从9月20日起,第322营反复进攻盆地,但为数750名的守军在火炮、迫击砲及机关枪的支援下坚强抵抗,最终饥饿、饥渴及美军的炮轰及轰炸敲向了日军的丧钟,9月25日美军攻占盆地,战斗中美军使用了推土机封闭洞穴入口以减少伤亡数字,到9月30日该岛完全为美军占领。岛上的机场在战斗未停上前已经在建造中,但延迟实施帛琉行动即表示岛上的机场到1944年10月时进攻菲律宾群岛的行动时还未能使用,威廉·海尔赛海军上将曾争论入侵帛琉的军事行动是不必要的,而很多军事历史学家均同意其观点,唯一的好处是使第81步兵师获得一些战斗经验。第81步兵师立即转而参加贝里琉岛战役以支援美国海军陆战队第1师,当时海军陆战队在岛中央高地遭遇顽强抵抗。","text2":"后人如何评价这一场战役?","label":1} {"text1":"《协奏曲》(英语:)是一出2009年推出的台湾电视剧,2009年1月28日至2009年3月10日间每星期一至星期四晚间10时(台湾时间)于台视主频道播出。浩瀚建设总裁袁瀚生(傅雷饰),她的掌上明珠袁喆(吴佩慈饰),从学生时代即暗恋著同父异母的弟弟袁浩(彭于晏饰)的家教冯尚宁(李威饰),而袁瀚生的续弦周崇兰(陈谊饰)竟有勾引冯尚宁的企图,亲妈的作为让袁浩羞愤的离家出走,此时,一个好傻好天真的小女孩高苓(刘品言饰),出现在袁浩的生命中,原本各自独奏的故事,就此开始响起…。袁喆远赴美国求学,与左亚夫(陈宇凡饰)相恋并育有一子,但彼此却阴错阳差的分离了;多年后,袁喆回到台湾,发现冯尚寕已失志地离开浩瀚集团,当年浩瀚同事兼好友的梁亦诚(白吉胜饰) 却不放弃他,认为冯尚宁仍是袁喆的最爱,也是浩瀚最佳的驸马爷人选,梁亦诚的一头热,招惹了冤家毛家敏(路嘉欣饰),而冯尚宁的感情世界则周旋在袁喆与高苓间,对尚宁而言,与冯尚羽(东明相饰)重拾兄弟情,也是另一个人生课题 …。每个人的生命乐章,包括家人都是从各自的独奏开始,一路上会遇上不同的独奏者,怎样才能让独奏的二人、三人奏出幸福的协奏曲呢?曲目:1. Rain & Tears(片头曲)(白吉胜)(原Rain & Tears)2. 红色警戒(白吉胜&林道远)3. 多亏有你(陈宇凡&路嘉欣)4. 小草的梦想(预告插曲)(蔡佳莹)妳离开我的那一天(原Rain & Tears)(片头曲)(白吉胜)多亏有你(陈宇凡&路嘉欣)台湾星势力娱乐股份有限公司http:\/\/www.pgestar.com\/","text2":"台湾星势力娱乐股份有限公司的网址是什么?","label":1} {"text1":"天主教格拉纳达总教区(拉丁语:Archidioecesis Granatensis、西班牙语:Archidiócesis de Granada)是罗马天主教在西班牙南部设立的一个总教区,主教座堂设在格拉纳达的布尔戈斯圣母主教座堂。3世纪成立教区,1492年升格为总教区,附属教区有天主教阿尔梅里亚教区、天主教卡塔赫纳教区、天主教瓜迪克斯教区、天主教哈恩教区和天主教马拉加教区。2011年,在当地793,400人口中,有教友764,300人,占辖区总人口96,3%、267个堂区、385名司铎、2名执事、197名修士、669名修女。现任总主教为Francisco Javier Martínez Fernández。","text2":"什么时候成立的教区?","label":1} {"text1":"兔子四部曲,是美国作家约翰·厄普代克的作品。1960年厄普代克出版《兔子,快跑》(Rabbit, Run),书以「兔子」哈利·安斯特朗(Harry 'Rabbit' Angstrom)为主角,是一个二十六岁的年轻人,长得又高又帅,这位兔子有些自恋又脆弱,高中时曾是学校中的篮球明星,曾破纪录投篮得分62分,他的妻子Janice大他两岁,酗酒、不理家事,育有一子Nelson,一女。他结交了篮球教练介绍的妓女Ruth,不久Ruth怀孕,妻子淹死女婴,兔子不知所措,于是匆忙逃走了。1971年厄普代克继续创作有《兔子归来》(Rabbit Redux), 描写1969年7月16日兔子回家后,成为了一个产业工人,妻子Janice跟情人同居,他接连遇上种族对峙、性革命、反文化运动等,兔子还收留了黑人逃犯斯基特。1981年又发表《兔子富了》(Rabbit Is Rich),故事是兔子继承了岳父的日本汽车事业,经济开始富裕,兔子加入乡村俱乐部、打高尔夫球、到加勒比海度假。1990年发表的《兔子安息》(Rabbit at Rest)写兔子年老病患,岛田先生(Mr. Shimada)非常不满意兔子的作为,兔子用心倾听日本老板的严厉批评,儿媳妇Pru因为长期闺房失和,晚上主动要求和他上床,Pru从睡袍口袋里掏出保险套,两度达到性高潮,兔子在事发之后离家出走,前往佛罗里达州,在篮球场试图重温当年风采时,结果因心脏病发作去世。兔子四部曲记录了美国自二战后40年来的社会历史的全貌,描绘出了一幅中产阶级的历史画卷,这些白领阶级有点铁石心肠,共塑造了约一百五十个大小不等的鲜活人物,内容涉及越南战争、登陆月球、女权运动、能源危机,有人称之为「美国断代史」。2000年,厄普代克又写了一部182页的中篇小说《兔子回忆》(“Rabbit Remembered”),交代兔子去世的后续情节,Pru跟Nelson离了婚,带着孩子回Ohio,兔子Ruth所生的女儿Annabelle以护士的身份出现,Nelson看著她想到自己的妹妹,大家共进感恩节晚餐,却因政治话题吵架收场。“兔子”的骨灰罐遗留在一家汽车旅馆的橱架里。本篇故事收录短篇小说集《Licks of Love》,有人说这是第五部曲,亦有人批评是狗尾续貂。","text2":"兔子四部曲,是谁的作品?","label":1} {"text1":"德国BR45型蒸汽机车是一种货运机车,亦是德国“标准蒸汽机车”之一。45型机车是德国有史以来制造过的功率最大的蒸汽机车。该型机车由汉寿尔工厂于1936到37年之间生产,在1940年之前全部入役。本来还有103辆45型机车的订单,但这些订货于1941年取消。这是由于二战爆发后德国机车工业集中生产力生产类似于BR52这样的简化版“战争机车”。45型机车共有28辆,编号从45 001 -到45 028。战后,BR45型机车开始频繁出现由于超压而造成的锅炉损害。为此,德国联邦铁路从1950年起为45 010, 45 016, 45 019, 45 021以及45 023 等机车安装了有燃烧室的下火箱以及下给加煤机。东德也对45型进行了改造,不过他们的方法是提高锅炉压力。然而这种改造没有收到预期的效果,并且使这些机车在1959年就早早退役(东德有很多蒸汽机车都使用到了7、80年代)。45型机车其实从设计上就有问题,主要是锅炉结构十分脆弱,就像之前的06型。在锅炉和主要部件更换以后,该型机车的真正实力才显现出来。在45型使用的最后阶段,其主要任务就是在44型难以担当的重型货运中发挥重要作用。到1968年,德国联邦铁路仅有3辆45型仍在使用(45 023 、45 010、 45 019),这些机车编为045型。45型退役后只有45 010号保存了下来,被收藏于纽伦堡交通博物馆。2005年,该馆发生严重火灾,这唯一的45型在大火中严重损毁。目前,有关方面正计划对该车进行修缮。","text2":"该型机为什么在1941年被取消订单?","label":1} {"text1":"资产重置成本定价模型(英文: Asset Based Pricing Model), 又称格雷厄姆-多德法(Graham Dodd Method),是由哥伦比亚大学教授、著名投资家格雷厄姆及其助手多德提出并完善的资产定价模型,曾经广泛应用于对企业股权等金融资产的定价。资产重置成本定价模型与现金流折现模型、股息折现模型、市盈率比较法等基于利润、现金或股息的定价方法不同,集中关注企业拥有的净资产的重置成本,即在现有市场环境下重新购置这些资产所需要付出的价值。假设一家企业拥有厂房、机器和土地,在现有市场环境下可以出售换取1亿美元现金,格雷厄姆-多德法即认为该企业的价值为1亿美元,减去债务的市场价值即为企业股东所获得的价值。以上思路清晰明了,但具有很强的保守性。企业拥有的无形资产很难估价,格雷厄姆-多德法经常将其忽略,从而造成对企业价值的低估。即使对于土地和厂房等固定资产,由于并不存在一个流动性很强的市场,很难得出令所有投资者公认的估价。如果在估计固定资产价值时过于保守,将错过许多投资机会;如果过于激进,又可能蒙受损失。在现金流折现模型诞生之后,资产重置成本定价模型逐渐衰落,目前主要用于自然资源企业、重工业企业等资本密集型企业。","text2":"资产重置成本定价模型主要应用在哪些地方?","label":1} {"text1":"有壁车站()是一由东日本旅客铁道(JR东日本)的所经营铁路车站,位于日本宫城县栗原市金成有壁。有壁是JR东北本线沿线一个无人车站,属JR东日本盛冈支社管辖范围,并由一之关车站代管。比较特殊的是,有壁虽然属于盛冈支社管辖,但实际位置却在宫城县境内,而隔邻的石越与一之关两站则都位于岩手县。这主要是因为石越~一之关之间的东北本线实际上是在宫城与岩手两县的县界周围徘徊,总共跨越县境交界三次。在日本全国,类似的状况除了此处之外,就只有横滨线长津田~相原间路段,与三江线宇都井~香淀间路段存在。有壁是江户时代奥州街道沿线的宿场町之一,因此在车站附近,还可以见到旧有壁宿本阵建物。侧式月台2面2线的地面车站。起初为2面3线,后来撤去中线。各月台以跨线天桥连接。","text2":"在日本全国,类似特殊的状况除了有壁车站外,还有什么地方?","label":1} {"text1":"特伦特河畔斯托克(),又称史笃城、斯托克(),英国英格兰西米德兰兹区域的二级行政区(区级),拥有城市、单一管理区、自治市镇地位,有239,700人口,占地92.74平方公里,行政总部位于Stoke-upon-Trent。若视斯塔福德郡为名誉郡,则史笃城为斯塔福德郡一部分;若视斯塔福德郡为非都市郡,则史笃城不为斯塔福德郡一部分。史笃城之瓷器生产非常有名,号称「英国瓷都」,知名品牌包含 Mintons, Coalport, Davenport, Royal Crown Derby, Royal Doulton, Wedgwood and Worcester。2015年,在英国旅游局举办的#英国等你来命名#活动中,“万博陶瓷”一名被评选为“奢华购物”主题里“最受欢迎命名”之一。","text2":"特伦特河畔斯托克又叫什么?","label":1} {"text1":"爱晨径(英文:Oi Sen Path)是香港九龙中部的一条行人径,位于红磡以北,何文田以南,是十二号山的南坡,靠近港铁东铁线路轨和爱民邨。北端连接治民街;南端连接支路较多,可经行人天桥往香港理工大学育才道或经行人隧道往红磡温思劳街。此外,近温思劳街处可抄小径至忠孝街。此路早在20世纪初已建成,为当时唯一由红磡通往何文田或旺角的道路,但因道路多山坡,在雨季时容易爆发山泥倾泻,长而令道路封闭,加上汽车量大增,做成红磡通往何文田的不便。佛光街建成后,问题才得以解决,但亦令爱晨径的重要性大减。1990年代曾有匪徒藏于树上乘机把途人洗劫并把人绑于树上。由2012年12月21日起,因为沙田至中环线工程的展开,爱晨径暂时封闭,至今仍未有解封的时间表。香港街道映照-爱晨径","text2":"爱晨径的英文名是什么?","label":1} {"text1":"《狐狸和鹤》是伊索寓言中一个著名的故事。狐狸和鹤是一对工作上的搭挡。他们合作无间,令公司赚得很多钱。但是有一天,他们因为工作上的琐事而发生争执,双方不欢而散。几日后,鹤觉得自己的语气太重了,便去找狐狸道歉。狐狸虽然表面上原谅了鹤,还表示要请他吃饭以示和好,但其实狐狸是心怀诡计,要戏弄鹤,鹤却不知就里答应了。很快便到了晚上。狐狸一面奸笑,一面将煮好的肉汤倒进深碟中。鹤来到后,狐狸便摆出一副诚意相来招待鹤。这时鹤才发现狐狸是在戏弄自己。眼见狐狸轻松地用舌头喝干净了肉汤,而自己喝汤时,却只能吃下细碎的肉粒、杂豆粒,而且汤汁不断从喙边渗出,鹤心中很不是味儿,他决意要给狐狸一个教训。如是者又过了几日,鹤「礼尚往来」地请狐狸去吃饭。狐狸到后,鹤将鱼汤倒入高杯中,结果这次换鹤吃得津津有味,狐狸却因为杯子又高又窄,没有办法喝到汤,最终只能干发呆。当狐狸问鹤为什么这样做时,鹤答道:「我这样做,是想给你知道给作弄的滋味。你要知道,你想别人尊重你,你就得先尊重别人。」要别人尊重自己,先要懂得尊重别人。","text2":"鹤为什么要找狐狸道歉?","label":1} {"text1":"《烟台条约》(又称《滇案条约》、《芝罘条约》;),是1876年(即光绪二年)清廷在英政府外交压力下与之签订的不平等条约,约文中文共30页,约文原存于中华民国外交部,现典藏于台北外双溪国立故宫博物院。1875年(同治十四年)2月21日,英国人马嘉理在云南被当地人杀死,演变成外交事件。清廷令海关总税务司赫德斡旋,1876年8月21日北洋大臣李鸿章与英国公使威妥玛在山东芝罘(即今烟台)谈判,9月13日,双方签约。英国取得进入云南及西藏特权。条约共三端一则:包括清政府对马嘉理事件及以前中英之间的案件各赔偿20万两白银;洋货在中国内陆免收厘金;增加开放宜昌﹑芜湖﹑温州﹑北海等为通商口岸;以及正式遣使道歉。这成为中国外交史上派出驻外长期代表的开始。1885年清廷派遣曾纪泽前往伦敦签订中英烟台条约续增专条,中英文共14页,规定鸦片进口至中国只能缴纳进口税30两及厘金最多80两,此后在中国各处行销不必再缴纳其他税捐,是清廷丧失外国货物在内地征收税捐的滥觞。约文仍存于台北外双溪国立故宫博物院。","text2":"《烟台条约》又称什么?","label":1} {"text1":"《美好的一年》()是一部2006年的浪漫喜剧电影,由罗素·高尔、亚伯特·芬尼及玛莉安·歌迪雅等主演,雷利·史考特执导。本片内容改编自一本2004年的同名小说,作者是彼得‧梅尔()。年轻时的麦斯·史金纳(Max Skinner,费迪·夏摩亚饰)在他叔叔亨利(亚伯特·芬尼饰)位于法国东南方普罗旺斯的葡萄园里过暑假时,从叔叔身上学习该如何体验生活中更美好的事物。麦斯长大后(罗素·高尔饰),在伦敦的证券交易所工作,勤勉且雄心勃勃,赚取金钱的方式游走在犯罪及法律边缘。当他收到他叔叔死讯以及得知他是庄园的唯一继承人后,他飞往普罗旺斯,准备将葡萄园快速卖掉。他到达不久后,政府开始调查他一笔可疑的投资交易,公司高层亦叫他立即回伦敦作解释。机缘巧合下,他未能及时赶回伦敦,最后被公司停职一个礼拜。他只好留在法国,和庄园的酿酒人法兰西斯·杜夫(Francis Duflot),一起为庄园做必要的修复。期间,麦斯发觉有不少人其实不希望庄园被卖掉,包括害怕失去心爱葡萄树的杜夫与当地的一个咖啡店老板芬妮(Fanny Chenal,玛莉安·歌迪雅饰)。在他犹豫不决之际,一个来自加州纳帕县的品酒师克莉斯·劳勃(Christie Roberts)突然出现,并宣称她是亨利的私生女。","text2":"本片内容改编自哪里?","label":1} {"text1":"黑龙江省大兴安岭林业集团公司暨黑龙江大兴安岭林业管理局,是中国首批57家试点企业集团之一,隶属于中华人民共和国国家林业局。其上级不是黑龙江省。黑龙江大兴安岭林业管理局与黑龙江省大兴安岭地区行政公署为一套班子、两块牌子。林管局局长兼任地区行署专员、大兴安岭地委委员、地委副书记;副局长兼任副专员。1996年3月,国务院正式批准大兴安岭林业管理局成立大兴安岭林业集团公司。黑龙江省大兴安岭林业集团公司辖10个林业局,61个林场。  加格达奇林业局(驻加格达奇区)辖10个林场,即古里林场、白桦林场、东风林场、翠峰林场、跃进林场、多布库尔林场、达金林场、大黑山林场、大子扬山林场、罕诺河林场。  松岭林业局(驻小扬气镇)辖5个林场,即绿水林场、古源林场、大扬气林场、壮志林场、新天林场。  新林林业局(驻新林镇)辖8个林场,即塔源林场、宏图林场、新林林场、大乌苏林场、碧洲林场、翠岗林场、塔尔根林场、富林林场。  呼中林业局(驻呼中镇)辖6个林场,即雄关林场、呼源林场、苍山林场、宏伟林场、呼中林场、碧水林场。  塔河林业局(驻塔河镇)辖6个林场,即瓦拉干林场、蒙克山林场、塔林林场、绣峰林场、盘古林场、沿江林场。  阿木尔林业局(驻漠河县劲涛镇)辖6个林场,即长山林场、依林林场、龙河林场、兴安林场、红旗林场、青松林场。  图强林业局(驻漠河县图强镇)辖7个林场,即奋斗林场、二十八站林场、育英林场、壮林林场、潮河林场、卧龙河林场、潮满林场。  西林吉林业局(驻西林吉镇)辖5个林场,即河湾林场、前哨林场、河东林场、古莲林场、金沟林场。  十八站林业局(驻塔河县十八站乡)辖6个林场, 即十八站林场、查班河林场、小根河林场、白银纳林场、永庆林场、双河林场。  韩家园林业局(驻呼玛县韩家园镇)辖1个林场,即韩家园主伐林场。","text2":"加格达奇林业局(驻加格达奇区)下辖哪些林场?","label":1} {"text1":"小洋山是浙江省舟山群岛和嵊泗县内的一个重要岛屿。位于舟山群岛西北部。距该县县城菜园镇西南39公里。通过东海大桥与上海市浦东新区相接。唐代即有关于该岛的记载,为舟山群岛较早有人居住的岛屿之一。时岛上建有供奉隋炀帝的庙,是舟山合境早期重要庙宇。清代江浙分洋时,小洋山划归江苏省,而其隔壁大洋山则仍属浙江省定海县。洋山建港以前,小洋山为一悬水岛,面积仅1.76平方公里,少平地多秃岩。1934年建立小洋乡。岛上有民众约3000多人,祖籍多为宁波、温州及下属各县。洋山开港后,通过促淤围陆与周围小岛连接,小洋山的面积成倍扩增,目前已经超过14平方公里,且随着洋山港的建设,还在不断增长。原来是孤岛,目前已经连成一体成为小洋山的一部分的岛屿有(自西向东排列):未来还将成为小洋山一部分的有:洋山开港后,原小洋山居民被全部动迁,上海市承担此项工作。这些居民一部分选择定居到原南汇区,另一部分则统一居住在大洋山上新建的云鹭苑居住区。根据上海市的政策,小洋山动迁居民可以全部拥有上海市户口,料当地居民可能全部或大部将户籍由浙江省移往上海市。","text2":"洋山建港以前面积有多大?","label":1} {"text1":"《真实的朝鲜》是中国作家叶永烈在2008年出版的长篇纪实作品,属于“叶永烈世界观”丛书之一,由天津教育出版社出版发行。书中作者以游客身份在丹东参加当地朝鲜旅行团赴朝旅游,并写下此游记。作者也曾在朝鲜驻中国大使馆申办签证但不成功。书中以一个普通中国游客的角度去观察朝鲜的现状,介绍了在朝鲜看到的奇特社会和政治现象,并刊载了大量在朝鲜拍摄到的图片,其中不乏在行进中的火车和客车上随机拍摄的作品,因这种行为不被朝鲜当局认可,为此多次与导游和随团的朝鲜国安人员争执。本书的出版并不顺利。作者在离开朝鲜回国时受到严格的半小时重点检查,但仍完整带回在朝鲜的记录,并回到上海写出此书。但由于中国与朝鲜本身特殊的历史关系,虽然多次被出版社邀稿,但都因顾忌政治因素而没敢出版。后广东杂志《同舟共进》分两期摘引此书的部分章节发表题为“告诉你一个真实的朝鲜”的文章,反应热烈,但仍然没有出版商敢于出版。后由北京新华立品图书有限公司在删除三节内容后最终在中国出版。作者虽对此书保持低调,但由于新浪、搜狐等门户网站刊载,北京人民广播电台全文连播而导致官方的注意,并且在朝鲜大使馆致函中华人民共和国外交部要求停止发行此书后,中国国家新闻出版总署对此书发出禁令,但只是在大型书店下架禁售,其他地方仍有销售,在朝鲜方面再次致函外交部后,出版总署才对全国所有书店下架禁售。作者认为这是为了顾及中国与朝鲜的兄弟同盟关系而必要牺牲的作为。此书被禁后除了书店禁售,一些网络图书销售网站也撤下此书,网站的连载也被迫停止。但在2008年末当当网店庆的时候曾以原价处理此书6折的价格销售,目前已再无销售,但部分小型网络书店仍有销售。据作者撰文说,韩国有出版社有意出版此书的无删节版。目前,在香港,新东方出版(香港)有限公司已出版该书的无删节版,名为《解密朝鲜(大陆禁书《真实的朝鲜》完整版)》,价格为港币98元。2008年8月作者为此写下“《真实的朝鲜》怎样成为禁书”一文,刊载在2008年九月号的香港开放杂志。","text2":"《真实的朝鲜》是哪个出版社出版发行的?","label":1} {"text1":"翼善冠,又称翼蝉冠。是冠的一种,是明代皇帝、藩王、亲王、郡王等所着之首服,也是朝鲜王朝国王及王世子、越南皇帝、明郑延平王及王世子与宁靖王、琉球国国王的首服。翼善冠发展自唐代的幞头,唐代男子兴头戴软脚幞头身穿圆领袍服,幞头专门用以缠裹头发及发髻。而唐代的幞头发展到宋代出现了官员公服的展脚幞头,外形方正。到了明代,朱元璋极力恢复唐宋时期汉人的服制。明代官员所着公服便沿袭宋代。而常服则是圆领袍搭配展翅的幞头,俗称乌纱帽,帽胎或竹或木,或纸,两片展开的帽翅则是薄黑纱。因而君与王所戴的则是两翅向上折的,故称“翼善冠”。明代的皇帝和如朝鲜国王等藩王所戴的翼善冠在外形的尺寸和颜色上有所区别,不得僭越。","text2":"明代的皇帝和如朝鲜国王等藩王所戴的翼善冠有什么区别?","label":1} {"text1":"狮头社事件(内外狮头社事件),发生于1875年,为台湾清治时期的台湾原住民排湾族与清朝之战役,地点是屏东县狮子乡一带,被视为沈葆桢在台期间规模最大的战争。此事件发生的远因是长久以来存在的汉清朝对原住民政权的侵扰,近因则是同治13年秋狮头社人民被刺桐脚庄民杀害,双方交恶,且又临日军侵犯原住民,及狮头社逮捕入侵部落之庄民和王开俊麾下营夫,以致清朝将领王开俊率领官兵数千名进犯,但为排湾军所阻,后王开俊中伏而亡,且随后全军覆没,是以引战。清国钦差大臣沈葆桢在牡丹社事件后开始「开山抚番」的政策,计划打通往恒春和后山之路,遂于1875年(光绪元年)2月为强行开辟道路,借口保护庄民,发动淮军攻击内狮头社、外狮头社、竹坑社、本武社和草山社,并发生清军屠杀原住民之事,战事约持续3个月,出兵数万人,清军最后虽胜,但损失惨重,残兵所剩无几。狮头社事件后,沈葆桢于地图上,片面将中文当地之地名之竹坑社改永平社,本武社改永福社,草山社改永安社,内、外狮头社为改内、外永化社。就实际成效而言:番学大多荒废,招垦事务弊端多,表面上,番目与代管者即便协助官方,以维护部落治安,但事实上,当地生活习俗依旧,除了改变部落传统的经济结构外,并未真正动摇大龟艾社群的内在基础。是故,狮头社事件结束后,清廷在台湾南部番地所实施的开山抚番政策成效是有限的。在屏鹅公路南下狮子乡南世村前路旁的王太帅镇安宫,为祭祀王开俊,旁边还有一座三层白塔。匾额是前枋寮地区农会总干事李增隆1985年所立。","text2":"牡丹社事件后清政府采用什么政策?","label":1} {"text1":"徐大椿(1693年-1771年),原名大业,字灵胎,号洄溪,江苏吴江人。清朝医学家。性通敏,喜豪辩。自《周易》、《道德》、《阴符》家言,以及天文、地理、音律、技击等无不通晓,尤精于医。初以诸生贡太学。后弃去,往来吴淞、震泽,专以医活人。乾隆二十五年(1760年)大学士蒋溥患病,乾隆帝命徐大椿入京医治,大椿直言蒋之病已不可治。乾隆三十六年(1771年),奉诏回京,当时大椿已重病在身,由其子徐爔陪同,勉强支撑到京城。第三天,自拟对联:“满山芳草仙人药,一径清风处士坟”。当夜病逝。大椿著书颇多,有《兰台轨方》、《医举源流》、《论伤寒类方》等,都为医学之籍。他的歌曲有《洄溪道情》三十余首。","text2":"徐大椿的医学著作有哪些?","label":1} {"text1":"魏东(),中国湖南省永顺县人,中央财政金融学院毕业,硕士学位。生前曾创办涌金集团并担任董事长。2008年4月29日突然在北京居所跳楼自杀身亡。魏东大学毕业后曾经担任中华人民共和国财政部科员,中国经济开发信托投资公司证券部主管。1994年,魏东在北京创办了北京涌金财经顾问公司,从事投资顾问的工作;次年,他又在上海创建了上海涌金实业有限公司;并于1999年,以1.8亿人民币成立了湖南涌金投资(控股)有限公司。至此,名噪一时的涌金系初具规模。此后,魏东开始利用旗下公司大举杀入实业领域。2002年初,涌金系正式受让九芝堂集团股份,成为该公司第一大股东;后又利用九芝堂大量增持千金药业股份,成为其第二大股东;2005年,涌金系旗下公司成功控股成都证券,并更名为国金证券,并于2007年实现借壳上市。2008年4月,魏东又以自然人股东的身份持有了浙江大华技术股份有限公司5.525%的股份,他的涌金集团则持有3.45%的股份。至此,魏东和他的涌金系达到了巅峰。2008年4月29日下午,魏东突然在北京的寓所跳楼身亡。受其影响,国金证券和九芝堂在4月30日都临时停牌。据相关报道,魏东曾在自杀前数日,受到有关部门的召见谈话调查,并传言其“被限制出境”;但也有传言说其患有抑郁症,不能排除是病症发作导致其最终选择自杀。后该传言也在魏东的遗书中得到进一步确认。魏东死后,与其关系密切的原中国国家开发银行副行长王益遭到“双规”,使魏东的死因再次变得扑朔迷离。九芝堂","text2":"魏东什么时候成立了湖南涌金投资(控股)有限公司?","label":1} {"text1":"《丁丁与字母艺术》(法语: Tintin et l'alph-art ;英语: Tintin and Alph-Art ),又译《丁丁与阿尔发艺术》,是《丁丁历险记》的第24部作品,也是最后一部作品,作者是比利时漫画家埃尔热。故事的主要题材是现代派艺术,埃尔热在1983年执笔这部作品过程中不幸逝世,导致这部作品成为未完成的作品。为了逃避毕安卡的爱情之访,哈达克意外地结认新派黑人艺术家拉莫·纳许(Ramó Nash),并替丁丁约定艺术专家弗卡特(Fourcart)向他「爆料」,然而不久就意外身亡。在几次逃过被枪毙的险境下,丁丁不但揭发弗卡特是被杀,并发现了新兴邪教的教主恩达迪涅·阿卡斯(Endaddine Akass),就是支助纳许的艺术赝品贩售家。不久,毕安卡邀请丁丁及哈达克前往阿卡斯位于意大利的豪宅,阿卡斯却把丁丁囚禁,准备把他用热塑胶活活灌死,造成凯撒扩展式的艺术品...收到米卢求救纸条的哈达克及时救回丁丁,然而他们却无法脱离阿卡斯的魔爪,其后阿卡斯向二人表明自己就是拉普洛斯。庆幸机警的纳许及时报警,并制止拉普洛斯把丁丁及哈达克吊死,然而拉普洛斯却因而失足跌下山坡至死。另外,在埃尔热的原稿出现的艺廊职员Martine Vandezande,郤在Rodier同人版的结尾向丁丁表示好感,引起丁丁迷一点点争议。","text2":"《丁丁与字母艺术》出自哪部长篇连载漫画?","label":1} {"text1":"资产挂钩票据(英文:Equity-Linked Note,简称ELN)是一种包含股票成份,并以赚取高利息为招徕的短线衍生性金融商品,所以俗称高息票据。它是一种场外期权,组成部分包括一张无风险的票据或债券和一个与股票挂钩的股票期权。投资者可选定某一只蓝筹股之ELN,由银行订出行使价、结算日、到期日及息率等条款。到期日一般介乎1至6个月,而息率一般为年息10至30厘不等,要视乎挂钩股票的波动及风险等因素而定。当票据到期时,如被挂钩股票的股价高于或等于行使价,票据持有人须收回票据面值,赚取利息;如股价低于行使价,票据持有人须以行使价买入股份。当被挂钩的股票价格在到期日前高于或等于行使价,投资者可以赚取利息,但能否做到这一点就要考考投资者对大市或股票后市走势的眼光。即使到期日股价跌至低于行使价被迫换取股票,投资者可选择持有股票作中长线投资。当被挂钩的股票价格在到期日前急泻,并跌穿行使价,投资者须以高于市价的价格换取股份,导致账面上的损失,投机者或需要止蚀离场。为控制风险,投资人在选择ELN时,须注意下列若干方面:","text2":"资产挂钩票据的到期日是多久?","label":1} {"text1":"统计图形,又称为统计图、统计学图形、图解方法、图解技术、图解分析方法或图解分析技术,是指统计学领域当中用于可视化定量数据的信息图形。有时,人们也把统计图形与各种统计学表格统称为统计图表或统计学图表。统计学与数据分析过程可大致分为两个组成部分:定量分析方法(Quantitative techniques)和图解分析方法()。定量分析方法是指那套产生数值型或表格型输出的统计学操作程序;比如,包括假设检验、方差分析、、可信区间以及最小二乘法回归分析。这些手段以及与此类似的其他技术方法全都颇具价值,属于是经典分析方面的主流。另一方面,还有一大套我们一般称之为图解分析方法的统计学工具。这些工具包括、直方图、、()、箱形图、以及双标图。(,EDA)就密切地依赖于这些手段以及与此类似的其他技术方法。图解分析操作程序不仅仅是在EDA背景下才使用的工具;在检验假设、、、估计量()选择、关系确定、因素效应判定以及检出方面,此类图解分析工具还可以作为最佳捷径,用来深入认识数据集。此外,优质的统计图形还可以作为一种令人信服的沟通手段,用来向他人传达存在于数据之中的基本讯息 。图解式统计学方法具有四个方面的目标:⑴ 探究数据集的内容;⑵ 用于发现数据之中的结构;⑶ 检查统计学模型之中的假设;⑷ 沟通传达分析结果。如果不采用统计图形,也就会丧失深入认识数据基础结构之一个或多个方面的机会。统计图形的起源可以追溯到人们最早试图分析数据的活动,而如今这种技术方法已经成为科学发展的关键手段之一。早在十八世纪,人们就采用了许多为我们当前所熟悉的统计制图手段和形式,如、示意地图、条图以及坐标纸。人们对于下列四个问题的关注推动了统计图形技术方法的发展:自1970年代以来,随着计算机图形学及其相关技术方法的复兴,统计图形目前已经东山再起,再度成为一种重要的分析工具。下列人物是一些著名统计图形的设计者:","text2":"定量分析方法是指?","label":1} {"text1":"奥奈银鲈,又称奥奈钻嘴鱼,俗名碗米仔,为辐鳍鱼纲鲈形目钻嘴鱼科的其中一个种。本鱼分布于印度西太平洋区,包括东非、马达加斯加、模里西斯、留尼旺、塞席尔群岛、马尔地夫、红海、波斯湾、阿拉伯海、斯里兰卡、印度、孟加拉湾、安达曼海、泰国、缅甸、柬埔寨、马来西亚、菲律宾、印尼、日本、台湾、中国沿海、新几内亚、马里亚纳群岛、帛琉、密克罗尼西亚、马绍尔群岛、诺鲁、斐济群岛、万那杜、萨摩亚群岛、东加、澳洲、所罗门群岛、新喀里多尼亚等海域。水深1至30公尺。本鱼体延长卵圆而侧扁,体色银白色,体侧有6至8条不明显的横带,其标准体长为其体高2.5至3倍。背鳍棘前半部黑色;胸鳍不达臀鳍基部,尾鳍深分叉。背鳍硬棘9枚、软条10枚;臀鳍硬棘3枚、软条7枚。体长可达30公分。属于热带、亚热带内湾及沿岸沙泥底栖性鱼类,但也可在岩质潮池建其踪迹,肉食性,以底栖无脊椎动物为食。为中小型食用鱼,多刺,适合以油炸的方式食用。","text2":"奥奈银鲈有着怎样的身体特征?","label":1} {"text1":"霍洛威(英语:Holloway )是伊斯林顿伦敦自治市下的一个区域。根据2001年的人口普查,霍洛威共有人口11,214人,其中47%是男性,53%是女性。霍洛威是伦敦人口最稠密的地区之一,同时拥有非常多元的文化。直到19世纪,这一地区还多是乡村。但随著伦敦范围的扩大,霍洛威迅速发展起来。在第二次世界大战期间,霍洛威的部分地区由于靠近国王十字火车站而遭受了纳粹德国的猛烈轰炸。2006年,英超劲旅阿森纳将自己的主场球场由已经使用93年的海布里球场搬到了在霍洛威新建的酋长球场。酋长球场可容纳60,355名观众,是英超球场中仅次于曼联主场老特拉福德球场的第二大球场,也是伦敦的第三大球场(仅次于温布利球场和特威肯纳姆体育场)。","text2":"霍洛威是哪个地方的一个区域?","label":1} {"text1":"惠州广播电视台(简称惠州台)是中国城市广播电视台,成立于1986年7月1日,隶属惠州广播电视传媒集团(南方广播影视传媒集团下属分公司),台址位于广东省惠州市惠城区江北东江三路55号。目前惠州电视台共有5个电视频道,分别为:惠州广播电视台的网站为东江传媒网,于2011年12月31日下午正式上线。作为惠州广播电视台第四媒体的东江传媒网——立足惠州,东江视角,全球视野;以惠州广电视听资源为基础,以视听、互动、文化为特色,突出本土化、平民化。网站可收看惠州广播电视台的在线直播以及过往节目回看和点播等。惠州电视台覆盖(全境)惠城区、惠阳区、大亚湾区、惠东县、博罗县、龙门县、及广州、深圳、东莞、河源、汕尾等毗邻地区约1300平方公里,覆盖人口约800多万。2014年5月28日,珠三角八市电视互相落地,佛山、肇庆、东莞、清远、珠海、江门、中山地区的有线电视观众可接收惠州台一套节目。惠州台一套和二套为惠州电视台制作和管理,其余均为惠州市有线广播电视台(现惠州广播电视台网络信息中心)管理。而惠州台一套新闻综合频道为无线广播,信号全部由位于惠城区的高榜山电视塔32频道发射,惠州台二套公共生活频道则通过市线光缆覆盖惠州两区三县,其他频道均只在惠城区有线电视中播出。惠州台控制的惠州有线电视在数字电视强制用户观看机顶盒内置广告,并以此作为卖点招商。","text2":"惠州台控制的惠州有线电视的招商卖点是什么?","label":1} {"text1":"蒙田大街(Avenue Montaigne)是法国巴黎第八区的一条街道。蒙田大街最初称为寡妇巷(allée des Veuves),因为妇女们在此聚集哀悼,但是自从18世纪初起这条街已经发生很大变化。这条街现在的名称得名于法国文艺复兴作家蒙田。在19世纪,这条街因周六夜晚五彩闪烁的Mabille球赢得了一些名声。蒙田大街拥有许多高级时装店,如克里斯汀·迪奥、香奈儿和华伦天奴,以及宝格丽等珠宝店,和雅典娜广场酒店(Plaza Athénée)等高档场所。到1980年代,蒙田大街被认为是法国高档时装及配件街道的“伟大的女性”(la grande dame),其重要性被认为超过了圣奥诺雷郊区街。一些服装设计师驻扎于此,例如路易威登。路易威登为这条街带来了投资和国际关注,设计师和公司,如席琳,洛伊,路易威登,Inès de la Fressange和以前的克里斯汀·拉克鲁瓦 ,都在这条街大量投资房地产。加拿大大使馆设在蒙田大街35号。蒙田大街15号坐落着香榭丽舍剧院(Théâtre des Champs-Élysées)。2008年12月4日,蒙田大街29号的Harry Winston精品店在打烊前遭到三四名武装劫匪抢劫,损失价值超过8000万欧元(约1亿美元)的“钻戒、项链和高档手表”,其中至少两名劫匪穿戴着假发和妇女的衣服。2007年10月这里也遭遇过类似的抢劫案,损失了2000万欧元。","text2":"蒙田大街最初称为寡妇巷是因为什么?","label":1} {"text1":"罗彦瓌(923年-969年),太原(今山西太原)人,五代、北宋时军事将领。生于后梁末帝龙德三年(923年),父罗全德,官后晋泌州刺史,彦瓌得父荫补内殿直。广顺三年(953年),枢密使王浚因骄横跋扈被贬官,罗彦瓌被视为王浚的同党贬为邓州教练使。周世宗时起用为伴饮指挥使,改马步军都军头。显德七年(960年)一月,爆发陈桥兵变,初三赵匡胤自陈桥还京,范质率王溥、魏仁浦责问赵匡胤,帐前罗彦瓌拔剑厉声:「三军无主,众将议立检点为天子,再有异言者斩!」。范质等皆畏惧,降阶听命。建隆二年(961年),任彰德节度使。干德二年(964年),改安国节度使,与昭义节度使李继勋破契丹。四年春,又与阁门使田钦祚破太原军,斩杀千余人于静阳,擒获其将鹿英,获马三百匹。卒于宋太祖开宝二年(969年)四月。","text2":"陈桥兵变,罗彦瓌起到了什么作用?","label":1} {"text1":"氧化加成反应及还原消除反应是有机金属化学中的两种重要的反应。 二个反应都和配合物有关。氧化加成反应中,参与反应的配合物其中心金属有空轨域且氧化态较低,氧化的方式则是增加一个和中心金属键结的共价键(X-Y)。反应之后,金属的形式氧化态 n 和配合物的电子个数皆加 2。 虽然加入各种不同的共价键都会产生氧化加成反应,最常见的是加入 H-H 及碳(sp)-卤素键。sp 混成的碳(如乙烯基),也出现在氧化加成反应。还原消除反应是氧化加成反应的逆反应。 反应之后产生 X-Y 化合物,若新形成的 X-Y 键度强度较强,反应会往还原消除反应的方向移动。不过 X 和 Y 原来需要在相邻的位置和中心金属原子配位,才会产生还原消除反应。以下用Vaska配合物, \"trans\"-IrCl(CO)[P(CH)] 和氢的反应来说明氧化加成反应。此反应的生成物配体中有三个阴离子:Cl及二个H,因此可看出中心金属被氧化,由 Ir(I) 变成 Ir(III)。反应物的金属配合物有16个价电子,其配位数为4,而反应后有18个价电子,其配位数为6。此反应也可以以逆反应进行,反应物失去氢,而金属配合数被还原,即为还原消除反应。氧化加成反应及还原消除反应出现在许多有催化剂参与的反应中,如有使用威尔金森催化剂(Wilkinson's catalyst)的蒙山都法及烯的氢化反应。","text2":"氧化加成反应后金属会有什么变化?","label":1} {"text1":"水腺毛草(学名:\"Byblis aquatica\")为腺毛草科的一年生食肉植物。通常称作「彩虹草」。本种描述是于1998年由艾伦·劳瑞以及发表,归类到澳大利亚北部一年生的物种,曾被认为是「亚麻花腺毛草的复合体」。生长是以半水生的状态,并会运用长在茎上的黏液腺体。水腺毛草的是由艾伦·劳瑞于1988年4月首次采得。栽培本种时被视为是亚麻花腺毛草的生态型,并命名成\"Byblis\" aff. \"liniflora\" \"Darwin\"。这样的处理直到贝利·梅耶斯-莱斯(Barry Meyers-Rice)清楚证明本种的生殖隔离,1996年扬·弗利兹(Jan Flisek)提议改成一个新物种的分类单元来描述。以至于1998年劳瑞处理了一部分他的澳大利亚北部种类的修订。","text2":"水腺毛草通常被称之为什么?","label":1} {"text1":"穆罕默德·乌尔德·阿卜杜勒-阿齐兹( \"Mohamed Ould Abdel Aziz\";),毛里塔尼亚现任总统。1956年,生于因希里省首府阿克茹特。2005年8月参与政变,废黜毛里塔尼亚总统马维亚·乌尔德·西德·艾哈迈德·塔亚。2008年8月,发动政变,推翻总统西迪·乌尔德·谢赫·阿卜杜拉希和总理叶海亚·乌尔德·艾哈迈德·瓦格夫。此后,阿卜杜勒·阿齐兹成为国务委员会主席,行使国家元首的权力。阿齐兹1977年加入毛里塔尼亚军队,并在摩洛哥的梅克内斯皇家军事学院学习,毕业以后在毛军队获得迅速上升。他创建了精锐的总统卫队,并且平息了2003年6月的未遂政变,以及2004年8月的军事叛乱,因此获得了国家最高军事奖。2005年8月,国家安全局长埃利·乌尔德·穆罕默德·瓦勒等人发动政变,成立“争取公正与民主军事委员会”接管政权,作为总统卫队指挥官的阿齐兹是政变的参与者和执行者。2007年3月25日举行大选,独立候选人阿卜杜拉希当选总统。2008年8月6日,以总统府特别参谋长阿齐兹为首的军人,发动政变,扣押阿卜杜拉希总统,成立“国务委员会”,接管政权。2009年4月15日辞去「国务委员会主席」职务,竞选总统。参议院院长巴·马马杜(Ba Mamadou Mbaré)将履行临时国家元首职责,负责处理过渡时期日常国务。2009年7月18日,穆罕默德·乌尔德·阿卜杜勒-阿齐兹总统选举中击败前国家元首瓦勒、民主力量联盟主席达达赫和现任国民议会议长布勒海尔等9位候选人,获得52.58%有效选票,并于8月5日在努瓦克肖特奥林匹克体育场宣誓就职。2014年6月,在反对派强烈抵制选举下,阿齐兹以82%得票率再度连任茅利塔尼亚总统。","text2":"阿齐兹总统8月5日在哪里宣誓就职?","label":1} {"text1":"半甲齿龟(学名:)又译半壳龟,是种史前乌龟,属于齿龟科(Odontochelyidae)。化石发现于中国贵州省,年代属于三叠纪,约2亿2000万年前。半甲齿龟的生存年代,早于同为三叠纪晚期,陆生的原颌龟(\"Proganochelys\"),是目前已知最古老的乌龟。半甲齿龟是在2008年由李淳、吴肖春等科学家叙述、命名,学名意为「半个甲壳」+「有齿的乌龟」。目前已发现三个标本,编号分别为IVPP-V-15639、IVPP-V-13240、IVPP-V-15653。半甲齿龟的身长约40公分。半甲齿龟与现代乌龟有许多不同特征。举例而言:半甲齿龟的上下颌具有牙齿,而现代乌龟的颌部已演化出角质构成的喙嘴。许多史前乌龟已具有完整的龟壳;但是,半甲齿龟只有腹部具有甲壳,背部无壳。半甲齿龟的肋骨宽广,类似现代乌龟的胚胎。科学家据此推测,乌龟的腹甲是由腹肋演化而成;而特化的脊椎与加宽的肋骨逐渐连在一起,演化出背甲。除了嘴部的牙齿、缺乏背甲以外,半甲齿龟还具有许多原始特征。牠们的脊椎与肋骨的接触面,不同于后来的乌龟。同时,半甲齿龟的颅骨比例也不同于后来的乌龟,其眼睛之前的部分较长。尾椎的横突并没有愈合,与后来的乌龟不同。肩胛骨缺乏肩峰突(Acromion Processes)。科学家根据上述特征,发现半甲齿龟不仅是已知最古老的乌龟,也是已知最原始的乌龟,牠们可能是种过渡化石。半甲齿龟的化石发现于海相沉积层,该地层还发现牙形石、菊石等化石,因此牠们可能是水生爬行动物。由于半甲齿龟只有腹部具有甲壳,科学家也推测最早期的乌龟是水生爬行动物,而腹甲可以防御腹部遭到其他水生动物攻击。","text2":"半甲齿龟的身长有多长?","label":1} {"text1":"在数学中,单李群是不含非平凡的连通正规李子群的连通李群。另一个等价的定义是:单李群是对应到单李代数的连通李群。单李群是李群理论中的基本构件,依照其李代数的复化,可以分成三族典型群,与有限个例外李代数。前者在几何学与数论中的应用有悠久历史,而后者则涉及数学中的某些特殊配置与当代理论物理学。在应用上,我们通常会考虑更一般的半单李群或约化群。约化群的表示是当前数学的热点之一。单群的分类法是先考虑其李代数的复化,并分类相应的根系。为了从复数域回到实数域,下一步是分类复李代数的实形式,这可藉 Vogan 图完成。最后,李代数一一对应到单连通李群,为了从李代数层次回到李群层次,还须要计算单连通单李群的中心。复单李代数的分类如下,以下的 formula_1 代表邓肯图的顶点个数:","text2":"在数学中,单李群是什么意思?","label":1} {"text1":"东南亚闽南语,又称东南亚福建话,是闽南语泉漳片的一种域外分支。这种闽南语是在17世纪到20世纪初叶由泉州、漳州等地移民因过番到南洋(东南亚的旧称)而带过去的语言。广义上东南亚闽南语一词也包括若干个潮汕片在东南亚的域外变体。这种闽南语的域外变体有以下两个特点:1)这种闽南语是由早期中国福建省的移民带到南洋的,因此保留了许多比较古雅的词汇和用法,有些在中国闽南本土已经消失了。2)也因为是早期带过去的语言,东南亚闽南语在发展过程中因缺乏很多现代生活所需要的词汇(尤其是科技、商业、东南亚特殊生活环境用语)而向东南亚若干个本地语言和方言借词,形成多种不同掺杂了土语词汇的闽南语。在新加坡,福建话指的是闽南话,新加坡人把闽南话当成是福建话。早期新加坡福建话受到马来语的影响,后来又受到英语和华语影响,已经发展成独特的闽南方言,虽然英语和华语普遍使用使得它面临被淘汰的厄运,但是老一辈的华人习惯上还是使用福建话作为沟通工具。一般来说,新加坡华人把泉漳片福建话当成一种没有文字的语言使用。东南亚闽南语属于派生的闽南语次方言。由于东南亚各国原本都有自己的国语,加上英语在现今社会还拥有者相当重要的经济地位,许多东南亚国家政府的教育政策都偏重让国人学习通用语如马来语(星国国语)、英语以及越来越普遍的华语(普通话),汉语方言在东南亚的发展前景如何依然是个未知数。","text2":"在新加坡,福建话指的是什么?","label":1} {"text1":"《西斯廷圣母》(Sistine Madonna),亦称《希斯汀圣母》,为义大利画家拉斐尔圣母像中的代表作,装饰于为纪念教宗西斯笃一世而重建的皮亚琴察圣西斯托教堂,最初它被放在教堂的神龛上,至1574年为止,一直保存在圣西斯托教堂,故得此名。现为德国德累斯顿的历代大师画廊(萨克森州立艺术博物馆的一部分)收藏。画中表现圣母抱著圣子从云端降下,两边帷幕旁画有一男一女,身穿金色锦袍的教宗西斯笃一世向圣母、圣子做出欢迎的姿态。而稍作跪状的圣芭芭拉,她虔心垂目,侧脸低头,微露羞怯,表示了对圣母、圣子的崇敬和恭顺。位于中心的圣母体态丰满优美,面部表情端庄安详,秀丽文静,在更高的起点上塑造了一位人类的救世主形象。扒在下方的两个小天使睁著大眼仰望圣母的降临,稚气童心跃然画上。","text2":"《西斯廷圣母》是谁的代表作?","label":1} {"text1":"6,8-二硫辛酸,简称硫辛酸(lipoic acid),为含硫八碳脂酸,在6、8位上有二硫键相连,有氧化、还原二型。6、8位上巯基脱氢为氧化型硫辛酸,加氢变成还原型称为二氢硫辛酸。硫辛酸虽然不属于维生素,但其可作为辅酶参与机体内物质代谢过程中酰基转移,起到递氢和转移酰基的作用,具有与维生素相似的功能(类维生素),因此也被生物化学教材列入维生素中讲述。硫辛酸以闭环二硫化物形式和开链还原形式两种结构混合物存在,这两种形式通过氧化-还原循环相互转换,像生物素一样,硫辛酸事实上常常不游离存在,而是以其羧基同酶分子(如二氢硫辛酸乙酰转移酶)中赖氨酸残基的ε-NH2基以酰胺键共价结合(结构上与生物胞素十分相似)。催化形成硫辛酰胺键的酶需要ATP,并且作为反应产物产生硫辛酰胺-酶偶联物、AMP和焦磷酸。硫辛酸1951年被L. J. Reed等分离成晶体。因是一种含硫的脂肪酸,故有人将其归属于脂溶性维生素。但由于在体内代谢中与TPP、NAD等辅酶一起参加生化反应,因此,根据结构与功能的统一性,也有人将其归入B族维生素。硫辛酸是既具水溶性(微溶)又具脂溶性的淡黄色晶体,外消旋硫辛酸熔点在60~61℃,沸点为160~165℃。硫辛酸作为辅酶,在两个关键性的氧化脱羧反应中起作用,即在丙酮酸脱氢酶复合体和α-酮戊二酸脱氢酶复合体中,催化酰基的产生和转移。硫辛酸可以接受酰基与丙酮酸的乙酰基,形成一个硫酯键,然后将乙酰基转移到辅酶A分子的硫原子上。形成辅基的二氢硫辛酰胺可再经二氢硫辛酰胺脱氢酶(需要NAD)氧化,重新生成氧化型硫辛酰胺。α-硫辛酸含有双硫五元环结构,电子密度很高,具有显著的亲电子性和与自由基反应的能力,因此它具有抗氧化性,具有极高的保健功能和医用价值(如抗脂肪肝和降低血浆胆固醇的作用)。此外,硫辛酸的巯基很容易进行氧化还原反应,故可保护巯基酶免受重金属离子的毒害。硫辛酸在自然界广泛分布,肝和酵母细胞中含量尤为丰富。在食物中硫辛酸常和维生素B1同时存在。人体可以合成。目前,尚未发现人类有硫辛酸的缺乏症。","text2":"硫辛酸主要分布在哪里?","label":1} {"text1":"沙希德·阿亚图拉·达斯特盖卜国际机场()(波斯语:')是伊朗伊斯兰共和国法尔斯省省会设拉子的国际机场,本名设拉子国际机场(波斯语:'),后为了纪念在该城出生并死于爆炸(1981年)的宗教领袖阿亚图拉而改现名(“沙希德”是伊斯兰教中的殉教者)。沙希德·阿亚图拉·达斯特盖卜国际机场于2005年完成了修缮和扩建,已经成为仅次于首都德黑兰伊玛目霍梅尼国际机场的全国第二大机场。沙希德·阿亚图拉·达斯特盖卜国际机场由四个大楼组成,其中两个供国际航班的乘客使用,另外两个供国内航班的乘客使用。另外,亦有一座供国际航班乘客使用的大楼正在兴建,并预计于2020年建成,建成后,现原有的客运大楼会改为朝觐 (伊斯兰教)航班之用。","text2":"沙希德·阿亚图拉·达斯特盖卜国际机场位于哪个城市?","label":1} {"text1":"1944教育法令(Education Act 1944),此法令改革了英格兰与威尔士中学的教育系统。法令常以保守党政治家拉布·巴特勒命名,他为中学教育引入了三分系统),并变中学教育为免费。法令在1996年11月1日被1996教育法令废除。三分系统将中学分为三类:文法学校、与。它允许了的创办,这种学校是前三种学校的结合,但是这种学校最初只创办了几所。它也引入了一个新的系统,,在这个系统下,一些学校接受教育部的直接补助(不同于普通学校接受地方教育局补助),作为交换,他们需要预留一些免费学位。法令规定向英国所有18岁以下学童免费提供三分之一品脱牛奶,后来,哈罗德·威尔逊的工党政府不再向中学生提供免费牛奶。这个政策在1971年被撒切尔夫人(后任教育大臣)延伸,她不再向七岁以上学童提供免费牛奶。法令的其中一个重大结果是教育、动员了妇女与工人阶级。它使得中学向妇女开放,接受专上教育的工人阶级比率大大增加。受过教育的工人阶级发现他们在社会中处于不利地位,使得工人阶级与中产阶级间出现更严重的阶级分化。约翰·奥斯本在20世纪50年代末期的戏剧作品提及了如此的分化。负责教育的部门由教育委员会(Board of Education)更名为教育部(Ministry of Education),并给予它更大的权力与更多的预算,结束了国立学校需要交学费的时期。在将离校年龄提升至15岁后,它赋予了政府提升离校年龄至16岁的权力,“首相才刚刚因为它(将离校年龄提升至15岁)是切实可行的而感到满意”,虽然这个措施在1973年前仍未落实。法令也提供了社区学院(Community College),向儿童与成人提供教育,但措施只有少数地方教育局跟进,如剑桥郡。法案强制国家资助的学校每日进行祈祷。此条款被1988教育改革法令修订,法令指出祈祷大致上应在基督徒间进行,而不适用于特定学校或儿童。修订也指出,祈祷可以在课室进行,而不是在集会中进行。为了决定学童应该升读哪所学校,他们设立了一种被称为的考试。升学学童需要参加这个考试,分别到最适合他们“能力与兴趣”的学校,但是实际上,一些文法学校因为倾向于学术成绩所以仍未改革,而工业学校、综合学校只创办了几所。结果,不管是否合适,大部分学童升读现代中学,意味着大部分教育资助流入现代中学。","text2":"法令造成重大影响之一是什么?","label":1} {"text1":"杨清(,)是越南反抗唐朝的起义军首领。史书未记载生年。据吴士连的《大越史记全书》记载,杨清交州龙编人,其祖先「世为蛮酋」。唐朝开元年间,杨清被唐朝任命为驩州刺史。当时的安南都护李象古以「贪纵苛刻而失众心」,见杨清据守驩州,心中忌之,于819年召杨清至安南都护府任自己的牙将,令其带三千人兵前往镇压黄洞蛮。杨清见人心忿怒,暗与城中的杜士交、杨志烈勾结,率领所部乘夜袭破安南都护府,杀李象古和他的家属。据《旧唐书》记载,杨清起兵后,820年,唐朝任命唐州刺史桂仲武为新任安南都护,奉命讨伐,并许诺赦免杨清,封之为琼州刺史。杨清不从,据城反抗。桂仲武遣人招降杨清的属下。因杨清残暴不仁,部将纷纷献城投降。杨清与其子杨志贞被捕处决。杜士交拥立其子杨志烈为主,退往长州的凿溪(在今越南宁平省)继续抗唐。但不久即因势单力薄,率军投降唐朝。然而《大越史记全书》却与《旧唐书》的记载截然不同。《大越史记全书》称,杨清击败了唐朝桂仲武军,随后连结安南地区的各少数民族一起反唐,并连合南方的占城国攻击唐朝。杨清又于828年、841年两次败唐军的韩约、武浑部,此后《大越史记全书》便突然不见了关于杨清的记载。据《》:「(元和十五年三月)辛未,杨清伏诛。」以及《》:「(元和十五年三月)辛未,安南将士开城纳桂仲武,执杨清,斩之。」","text2":"杨志烈为什么率军投降唐朝?","label":1} {"text1":"盐水港厅为台湾日治时期行政区划,设立于明治三十四年(1901年),台湾总督府以办务署负责地方政务,而县及厅介于总督府与办务署之间,造成行政事务上的欠缺灵活 ,而废止「三县四厅」,设「二十厅」之行政单位,划分台南县为嘉义、盐水港、台南、蕃薯寮、凤山、阿猴等六厅,盐水港厅范围东至三脚南山系,西至海,南至曾文溪,北至八掌溪、冻仔顶山系,厅治设于盐水港街,下辖店仔口、前大埔、北门屿、麻豆、六甲、萧垄、新营庄(后改设布袋嘴)等七支厅,支厅下设盐水港、顶潭、菁寮、旧营、新营、查亩营、果毅后、店仔口、安溪寮、海丰厝、番仔丰、番社、前大埔、蔴荳、寮仔部、茅港尾、安业、佳里兴、萧垅、沤汪、下营、西港仔、塭仔内、北门屿、学甲、中洲、六甲、官佃、内庄等29区。明治42年(1909年)10月,台湾总督府将原有之二十厅,废止合为十二厅,盐水港厅废止,盐水港、店仔口、前大埔、布袋嘴等地并入嘉义厅,北门屿、麻豆、六甲、萧垄等地并入台南厅。1920年成立台南州,盐水港支厅、店仔口支厅、前大埔支厅为新营郡,盐水港设立盐水街,亦代表纵贯线铁路经过的新营已接替盐水之地位,成为区域中心。","text2":"1920年成立了哪个州?","label":1} {"text1":"彰化基督教医疗财团法人,系由财团法人台湾基督长老教会于西元1995年10月23日捐助设立的公益法人,创立时名为财团法人彰化基督教医院,西元2013年4月3日更名为彰化基督教医疗财团法人。院史可溯源自西元1896年,英格兰长老教会海外宣道会(Foreign Missions Committee of the Presbyterian Church of England)所派遣的宣教师,梅鉴雾牧师及兰大卫医师在彰化市开设的诊所,同时进行医疗传道。其所属彰化基督教医院,为彰化地区唯一的医学中心。前身为彰化市的一间诊所,于西元1896年由梅鉴雾( Rev. Campbell Moody)牧师及兰大卫( David Landsborough III)医师建立,西元1907年10月19日正式命名为「英立彰化基督教医院」。彰化基督教医疗财团法人为深入就近服务社区,近几年已发展为体系运作方式,除了自身筹办之彰化基督教医院、二林基督教医院、鹿港基督教医院、鹿东基督教医院,接手承办的云林基督教医院外,为提供民众可近性医疗服务,也进一步与彰化县员林市的员生医院、南投县南投市的南基医院、南投县草屯镇的佑民医院进行医疗合作,密集提供医疗服务,形成紧密之彰基医疗服务体系。而员生医院已于2016年8月31日正式结束合作关系。云林基督教医院原名为慈爱综合医院,创立于1896年8月8日,座落在云林县西螺镇,在2007年通过新制医院及教学医院评鉴,成为区域教学医院,2009年3月1日,改制为彰化基督教云林分院,又随彰化基督教医院改制,更名为云林基督教医院。资讯部、医工部、工务部、新建工程室、体系人力资源处、国际医疗事业部、医疗勤务部、秘书室、体系财务处、体系稽核辅导处、体系资材运筹管理处、院史文物馆、公共关系部、体系营运中心、医疗事务部、体系营养暨膳食部。内科部、外科部、妇产部、儿科部、耳鼻喉暨头颈部、牙科部、中医部、急诊医学部、其他科别、医疗中心、社区医学部教学部、研究部、医疗品质部、医学图书部、教研创新行政中心、实证医学中心、实验动物中心、医学伦理中心、人体试验委员会1896年11月29日,来自英国的传教士兰大卫医师和梅监雾牧师在台湾中部进行医疗传道工作以来,就开始聘任牧师的院牧工作。目前主要发展的工作:","text2":"彰化基督教医疗财团法人是谁在何时创立的?","label":1} {"text1":"东涌道北是位于离岛区东涌。由东涌逸东邨至东涌马湾涌。东涌道在1966年9月28日,由港督戴麟趾主持开通典礼。早期为一条单线双程行车的道路,途中设有多个避车处,及后因青屿干线通车及北大屿山新市镇首期入伙,令居民及游人改选使用东涌道进出大屿山南部地区,使东涌道的车流量超出负荷。故在1998年开始,路政署在东涌道进行多项小型的改善工程,包括扩阔部份路段及增加避车处数目;及后政府于2004年6月开展更全面的「东涌道改善工程」,当中包括将龙井头至伯公坳一段长3.6公里的东涌道改为双线不分隔道路;在伯公坳与长沙䃟石湾之间筑建一条长2.6公里的全新双线不分隔道路,包括总长度为750米的高架道路构筑物;在东涌道沿线辟设21个避车处/巴士停车处;以及在䃟石湾基督教女青年会营地附近辟设一个回旋处,与屿南道交汇。工程已于2009年1月完成,并于2009年2月6日早上十时起正式开放通车。此外,随着东涌西部的开发(兴建逸东邨),东涌道亦被分为两段,第一段由长沙大桥至东涌坝尾村,第二段由东涌松仁路回旋处至东涌马湾涌(旧码头),后者在2008年7月11日开始已改称为东涌道北;而断开的一部份路段后来改名为翠群径。","text2":"「东涌道改善工程」哪年竣工的?","label":1} {"text1":"中华人民共和国第十一届全运会的射箭比赛在2009年10月17日至10月23日于青岛市体育运动学校射箭场进行,共设4个小项,产出4面金牌。参与本届全运会的运动员必须通过预赛取得参赛资格。本届全运会射箭预赛会先举行团体预赛,并进行个人预赛。于团体预赛中晋身准决赛的队伍可参与十一运会射箭项目的团体小项。同时,晋身全运动的队伍中,其成员(每队3人)均可直接取得个人小项的参赛资格,而其余40个名额,则在其后的个人预赛中决赛,成绩最好的40人会取得最后的参赛资格。于女子个人预赛中,北京奥运女子个人小项金牌得主张娟娟取得资赛资格,铜牌得主姜林于初赛意外出局,无缘全运会。","text2":"第十一届全运会的射箭比赛的举行地点是哪里?","label":1} {"text1":"穴醚是一类人工合成的,可以与阳离子发生配位的双环和多环多齿配体。“穴醚(\"cryptand\")”一词是指该配体形如空穴,将底物分子容纳在里面。整个分子是一个三维的结构。因此与单环的冠醚相比,穴醚配合物更加稳定,对底物分子的选择性也更强。形成的复合物具有脂溶性。唐纳德·克拉姆、让-马里·莱恩和查尔斯·佩特森通过对穴醚和冠醚进行研究,开创了超分子化学的先例,并因此获得了1987年的诺贝尔化学奖。最为常见且最为重要的穴醚是N[CHCHOCHCHOCHCH]N(右图),IUPAC名称为1,10-二氮杂-4,7,13,16,21,24-六氧杂双环[8.8.8]二十六碳烷,俗称[2.2.2]<\/nowiki>-穴醚。方括号内的数字表示在两个氮桥头之间每个桥上的氧原子个数。全胺穴醚对碱金属阳离子具有极高的亲和力,通过穴醚与碱金属作用,可以成功得到含K等碱金属阴离子的盐类。穴醚的三维内部空腔可以和外来离子紧密结合,形成的复合物被称为穴状化合物。结合能力最强的是较硬的阳离子,包括NH(铵离子)和镧系元素、碱金属、碱土金属的阳离子。穴醚利用分子中的氮和氧与这些离子配位,由于不同的离子与不同三维结构的穴醚结合能力不同,通过选取适当的穴醚,可以将不同的碱金属阳离子区分或分离出来。与冠醚类似,大环穴醚一般也是利用胺和卤代烃的缩合反应制备的。但由于环系更为复杂,穴醚的产率通常不高。穴醚的制备较为困难,且价格昂贵;但是,与冠醚之类的其他配位剂相比,穴醚与碱金属离子结合更紧密,选择性更强。 通过配位,穴醚可以将一般情况下不溶于有机溶剂的盐类溶于另一相中,用作相转移催化剂,加快化学反应的速率; 也可以稳定碱金属负离子,使碱化物和电子盐得以合成。另外,穴醚还可以帮助Sn之类津特耳离子(Zintl ion)的结晶。在制作核医学需要的显像用药物氟代脱氧葡萄糖(简称 FDG)时,会使用[2.2.2]穴醚来络合反应物KF中的钾离子,提高放射性F离子的亲核性,以便将F连接到脱氧葡萄糖中。","text2":"谁开创了超分子化学的先例?","label":1} {"text1":"米基尔·文奴斯·莫桑( ,)出生于马德里,逝世亦在马德里。是一名西班牙足球运动员及教练。他职业身涯大部份时间都在皇家马德里,球员时代司职中场,代表皇马上阵超过200场,其中1955年9月欧洲冠军杯决赛对西维迪入了致胜球赢得该届欧洲冠军杯冠军,后来1956-57年成为球会队长,1958年退休,两年后执后皇马,带领球队赢得1960年和66年欧洲冠军杯冠军,是首位以球员和领队(或教练)的身份赢得欧洲冠军杯或欧洲联赛冠军杯冠军,现时只有齐达内、查柏东尼、告鲁夫、安察洛堤、列卡特、瓜迪奥拉和他能做到。文奴斯年少时在马德里几队青年球队效力,包括艾斯高纳普斯(Escolapios)、宾拿维斯达(Buenavista)、柏云(Pavon)、恩柏里奥(Imperio)和基洛特(Girod),但吸引不到皇马的注意。他先后效力CD洛格隆尼斯(CD Logroñés,1943-44年)、桑坦德竞技(1944-46年)和切尔达(1946-48)。1948年,文奴斯和主将柏轩奴(Pahiño)帮助切尔达取得西甲第四名及打入西班牙国王杯决赛。而他亦助球队在1:4大败给西维尔的决赛中取得唯一入球。翌季,他和柏轩奴一起加盟皇家马德里。1948年至1958年,共为皇马上阵347场。他亦曾7次代表西班牙国家足球队。文奴斯在1959年上任皇马一队教练前曾执教皇马预备队CF帕斯欧查(Plus Ultra CF)。他带领球队进入全盛时期,他带领皇马赢得9次西甲冠军,包括五连冠(1961-65年)及三连冠(1967-69年)。其后他任教共他西甲球队,包括CF格拉纳达(Granada CF,1975-76年)、靴格里斯(Hércules CF,1976-77年)、拉斯彭马斯(UD Las Palmas,1977-79年)及西维尔(1979-82年)。他亦曾带领西班牙国家足球队两段时期。先在1969年代任领队出战4场,而在1982-88年共领西班牙59场赛事,其中曾带领球队打入1984年欧洲国家杯决赛及1986年世界杯最后八强。","text2":"米基尔·文奴斯·莫桑逝世在什么地方?","label":1} {"text1":"李瑞东(公元 1851年-1917年),字树勋,号文侯。河北省武清县人,清末武术名家,李氏太极拳的创始者。李瑞东因为鼻子扁平,绰号“鼻子李”。家中经营药材生意,开设济生堂药舖,父亲李小歧,任职县衙为小吏,家境不错。自幼爱好拳术,初练少林等外家拳术,青少年时代跟随河北饶阳戳脚门名师李老遂求艺九年,河北名拳戳脚门拳法。青年时代与大刀王五(王子斌)结义金兰,互换拳艺,得王五所传“山东教门弹腿”之精妙。李瑞东先生天资聪颖,甚至有过目不忘之能,且练艺十分刻苦,到了成年时,已经练出了一身超群的武功,而且经常与善扑营的布库比试摔跤,多胜之,所以在京津二地很有些名气。光绪六年,端王府管事王永泰(字兰亭)出京办差,因为与李家为世交,途中经过武清县,顺道至李家拜访,因而与李瑞东结识。王兰亭早年拜在周岳图门下,学习十二连拳(心意六合拳的一个分支)。后又拜杨露禅为师,学杨氏太极拳。在杨露禅死后,又拜入董海川门下。李瑞东因此机缘,在王兰亭门下学习太极拳、八卦掌,又因王兰亭的介绍,至端王府、恭亲王府担任武术教师,与尹福、程廷华等人过从甚密,也曾得到董海川本人的指导。光绪二十年,至宫中为慈禧表演,以轻功得到赏识,被誉为「神鹰」,因而留在宫中,任四品带刀侍卫。父子在恭王府教拳,溥心畬兄弟是李子廉学生。在八国联军之后返乡,又拜甘淡然为师,学习八卦奇门拳。之后结合所学,创立李氏太极拳。因为他个性豪爽,交游广阔,在京津一带名声甚大,曾与李存义、李书文义结金兰,被推为天津中华武士会之首。但也因为不善理财,又爱接济朋友,因此而耗尽家财。在民国初年,曾任袁世凯大总统亲卫队武术教习,也曾主持天下武林英雄会,任裁判长。先生有弟子百余人,著名者近二十来人。先生有三子二女均精李派拳法:长女奇英、长子伯英、次子仲英、次女菊英、三子季英,弟子项润田、李进修、李子廉、张滔、陈继先、程安和尚、蒋万和、蒋万良、文实权、罗子鸣、王凤鸣、刘子鸣、高瑞周等人。原本是通背门的名家王荣标(人称“大枪王荣标”者)先生,在李瑞东先生中煤气毒逝世后,因仰慕先生之名,曾经与先生长子伯英结为师兄弟,从伯英学李派拳法。李瑞东曾向六大师习艺,又喜爱与人交换武术心得,因此他的武术特色即是博采众家、注意实用。","text2":"李瑞东青年时代与谁结义金兰?","label":1} {"text1":"胡燕泳(),香港新闻从业员。2001年胡在香港中文大学新闻与传播学院毕业,曾经从事公关工作。2003年转职香港有线电视新闻主播,并于同年转职到24小时亚视新闻台。2006年获奖学金,留学伦敦大学亚非学院,念国际研究与外交硕士。及后开始于免费频道本港台报道新闻,曾任担任本港台六点钟新闻、夜间新闻主播一职。2007年与黄珊、黄雅宇共同担任《主播天下》的主持及策划职务。2010年3月,她辞职并离开亚视新闻的播报工作,至2012年7月以兼职身分重返亚视新闻工作。2012年11月在报章撰文批评亚视高层干涉新闻部运作后,遭亚视解雇。2013年4月起,任无线新闻外电编辑,同年9月尾开始成为下午《新闻提要》主播。2013年12月7日起,担任《午间新闻》主播。2015年3月2日起,担任《环球新闻档案》旁白。2015年中,辞职并离开无线新闻,转职到香港经济日报。亚视受管理层连番操控,声称收视与TVB「四六开」惹来公众、广告业界人士和学者质疑,动员员工反对政府发出新免费电视牌照的举措也令全城非议。胡在2012年11月于《明报》发表题为《来自亚视的声音》的文章,力斥管理层王征和盛品儒于多次事件的处事手法都很有问题,批评王在亚洲电视误报江泽民逝世事件中没有任何承担;在政府总部举行的「大集会」王盛二人的言论及行为「着实超乎常人所能接受的尺度」,更严重的是「人们对亚视的鄙视,似乎并不规限于个别人士,而是整个亚视遭殃,新闻部也不能幸免」。另外,胡指出,过去她与同事制作《主播天下》时亚视给予新闻部很大自由度,但如今已今非昔比;原本《主播天下》的后续节目《ATV焦点》本来同样由新闻部记者制作,但最终已变质成为高层公器私用的平台;她和《ATV焦点》撰稿作者(前《大公报》执行总编辑雷竞斌)「在新闻部碰口碰脸」并不稔熟,但「因为他的个人大作,却引来四万宗投诉、网民的唾骂、前线记者在政总采访受滋扰;一人创作,新闻部百人当灾,我不甘心看到这境况」。2012年11月26日,亚视突然解雇七名员工,胡是其中之一;对她被辞退是否与批评公司有关,亚视公关部称不会回应。","text2":"2015年3月2日起,胡燕泳担任什么职务?","label":1} {"text1":"司马模(),字-{}-元表,西晋高密王司马泰的第四子。少好学,与琅琊王司马睿及范阳王司马虓在宗室中有贤名。初封平昌公,晋惠帝末年,拜冗从仆射,累迁太子庶子、员外散骑常侍。成都王司马颖奔长安,东海王司马越以弟弟司马模为北中郎将,镇守邺城。永兴初年,成都王司马颖原来的部下公师籓、楼权、郝昌等攻邺城,广平太守丁邵率众来救,范阳王司马虓也遣兖州刺史苟晞救援,公师籓逃散。之后,迁镇东大将军,镇守许昌,进爵南阳王。永嘉初年,转征西大将军、开府、都督秦雍梁益诸军事,代河间王司马颙镇守关中。东海王司马越征司马模为司空,遣中书监傅祗代替镇守关中。司马模谋臣淳于定建议司马模继续留守长安,和哥哥司马越一个在内、一个在外,遣世子司马保为西中郎将、东羌校尉,镇守上邽,司马越进司马模进位太尉、大都督。311年,永嘉之乱,镇守蒲坂的牙门赵染,求冯翊太守不得,率众投降刘聪。刘聪派其子刘粲和赵染攻长安,司马模派淳于定抵御,被赵染击败。司马模投降赵染。赵染坐在那里数落司马模的罪行,送到刘粲处,刘粲杀了他,以司马模的妃子刘氏赐给胡张本为妻,子司马保继立为南阳王。","text2":"司马模为什么在总是中有贤名?","label":1} {"text1":"古川车站()是一位于日本宫城县大崎市古川车站前大通1丁目,由东日本旅客铁道(JR东日本)所经营的铁路车站。古川车站是JR东日本所属的东北新干线与地方交通线、陆羽东线的交会车站,因此是邻近地区非常重要的交通枢纽。两条路线在古川这里以垂直方向交会,其中陆羽东线的在来线月台位于地面上,新干线月台则是高架设计。在2002年之前,JR货物原本在古川车站内也设有货运专用车站,但在货运站废站之后改以古川货柜中心()取而代之,并在2006年时根据新的政策,将货柜中心改名为古川线外货运站(,或简写为「古川ORS」)。东北新干线与陆羽东线的轨道在本站是以接近垂直的角度立体交差。其中,在来线的陆羽东线为岛式月台1面2线的地面车站,由于月台的容量是根据1980年客运设施迁至目前位置时的需求决定,因此只有约8节车厢的长度。至于新干线月台的部分,是设置于3楼高度的高架车站。虽然构造上采用的是正中央夹著作为通过线用的本线之2面3线构造,但因13号月台并没有铺设轨道,因此实际上只有2面2线配置。","text2":"在来线月台和新干线月台在设计上,有何不同?","label":1} {"text1":"李冲(),原名思冲,字思顺,魏孝文帝替他改名冲,陇西郡狄道县(今甘肃省定西市临洮县)人,北魏镇北将军、敦煌宣公李宝的幼子,北魏官员,陇西李氏仆射房始祖。李冲好交游,北魏孝文帝初年,任秘书中散、内秘书令、南部给事中。迁中书令,加散骑常侍,仍兼给事中。随后转南部尚书,赐爵顺阳侯。李冲提出均田制、三长制,孝文帝采纳,以五家为一邻,五邻为一里,五里为一党,各设一长。北魏国力大增。改置百司,开建五等,以冲参定典式,封荥阳郡开国侯,食邑八百户,拜廷尉卿。寻迁侍中、吏部尚书、咸阳王师。立太子后,拜为太子少傅。李冲受文明太后所宠幸,每月赏赐多达数十万钱,进爵陇西公,太后又密赠珍宝、御物到其府第,外人不得而知。李冲原本家境清贫,自此成为富室;然而李冲谦虚自牧,慷慨散财,从族人姻亲到乡里邻人,都分及财物。493年,北魏迁都洛阳,任镇南将军、侍中、少傅,负责营建新都,封阳平郡开国侯。迁尚书仆射、仍领少傅,改封清渊县开国侯。太子元恂被废时,李冲罢领少傅。李冲年四十余岁时,鬓发班白,姿貌丰美,未有衰老之状。李冲因自己提拔的李彪跟自己利益相背,而跟其他贵族一起攻讦李彪。李冲个性素来温柔,但一旦暴怒时,便发病狂悸,言语错乱,且会扼腕叫骂,称李彪为小人。医药无法治疗,医师或有诊断李冲为肝藏受伤破裂的。十余日后卒,卒年四十九,赠司空,谥曰文穆。葬于覆舟山。杜预早生李冲150多年,二人并称李杜。六世孙李元亨,安邑县令。李元亨生李昌庭,唐朝棣王友。李昌庭孙朝议郎、滑州酸枣县令李宙。李宙夫人卢氏之母郑氏为郑善果来孙。","text2":"他成为富户之后是怎么做的?","label":1} {"text1":"飞鹅岭位于广东省惠州市惠城区鹅岭东路和南湖路交界,东临惠州西湖之一的南湖,惠州市汽车客运站即在其山脚下。飞鹅岭高70多米,因形似飞鹅,外加古有仙人骑飞鹅降临于此的传说,故称飞鹅岭,而惠州的别称“鹅城”也是因此而来。飞鹅岭因是当时惠州城最高的山体,故历来是兵家必争之地。国民革命军东征时,孙中山、蒋介石、周恩来等人曾在飞鹅岭指挥战斗。现飞鹅岭仍完好保存着当年东征时的战斗碉堡和战壕,而山顶建有山顶广场、览胜亭、东征军战士群雕、东江阵亡将士纪念碑,往西门方向山腰则常年摆设一架曾参与抗美援朝战争的退役米格-15战斗机,西门附近还有一东征史料展览馆。目前飞鹅岭已辟为飞鹅公园,纳入惠州西湖风景名胜区。门票为1元。","text2":"飞鹅岭现在属于哪个风景区?","label":1} {"text1":"姜英勋(,),韩国陆军中将、驻外大使、韩国总理,在任日期为1988年12月5日至1990年12月27日,并于1988年12月16日前出任韩国的临时总理。日治朝鲜时期平安北道昌城郡出身。本贯晋州(),号青农()。1936年宁边农业高中毕业,1940年广岛高等师范大学毕业。后留学满州国,考入建国大学经济系。1946年5月1日成为军事英语学校第1期生,并曾参加韩战。1951年起担任韩国国防部经理局长、管理局长,1952年担任韩国驻美国大使馆武官,1954年以少将阶级担任联合参谋本部本部长,1955年任国防部动员助理次官,1956年任陆军本部管理副参谋长,1958年至进修,1959年任。1960起担任陆军官校校长,但因其反对官校生参与支持五一六政变的示威活动,而作为「头号反革命将领」()被关入西大门监狱服刑。1961年获释出狱后以陆军中将阶级退役。1970年前往美国南加州大学留学,获得政治学博士学位。1978年年初进入外务部任外交安保研究员院长,1981年1月13日任兼爱尔兰大使(至1984年)和驻梵蒂冈大使(1985年至1987年)1988年4月26日第13届总选挙中当选国会议员(1988年5月至12月),同年12月16日任第21代国务总理,1990年去职。2016年5月10日逝世。","text2":"姜英勋在1978年年初担任了什么职位?","label":1} {"text1":"突厥汗国(古代突厥如尼文:;)是原先在柔然统治下的阿史那氏部族于552年在中国以北地区建立的古代汗国,一度控制漠北、中亚等柔然故地。后分裂成东突厥汗国和西突厥汗国,7世纪时先后为唐朝所灭。而东突厥复国后形成的后突厥汗国亦在8世纪为回纥灭亡。有人认为阿史那部最先是生活在咸海边的塞种,后东走至叶尼塞河南方,受铁勒同化;阿史那部于6世纪初年游牧于金山一代(今阿尔泰山),归附于柔然,为其炼铁奴。柔然由于长期与铁勒(柔然人称其为高车人,因为他们高大的车轮而得名)战争而削弱;546年,阿史那部首领脱离柔然,并于550年在首领土门的带领下击败铁勒,552年又打败柔然,建立了政权,自称伊利可汗;同时期的中国将其音译为“突厥”。553年木杆可汗在都斤山(又作“郁都军山”、“乌德鞬山”,今蒙古国杭爱山)建立王庭。全盛时,其疆域东至大兴安岭,西抵西海(咸海),北越贝加尔湖,南接阿姆河南,建立了官制,有立法,有文字。583年,隋将长孙晟用离间之计,使突厥汗国分裂为东西两部,后在屡次与隋朝的战争中战败而走向衰落(参见隋与突厥之战)。唐太宗贞观四年(630年)与薛延陀攻灭东突厥,唐高宗显庆四年(658年)唐又灭西突厥,余部西迁中亚(参见唐灭东、西突厥之战)。在唐高宗末年(682年),再度建立后突厥汗国,最后在745年,后突厥帝国亡于回纥。","text2":"突厥汗国曾一度控制过哪些地方?","label":1} {"text1":"阿特罗巴特斯(希腊语:Aτρoπάτης 古波斯语义:被火保护的;)出身于波斯的贵族,曾效力于大流士三世、亚历山大大帝,最终建立一个以他为名的独立国家和王朝。在波斯阿契美尼德王朝末期,阿特罗巴特斯担任米底行省总督,前331年高加米拉战役时,阿特罗巴特斯统率米底、高加索阿尔巴尼亚、萨咖森(Sacasene在今日亚美尼亚一部分)等地区部队,战后大流士三世逃到米底首府埃克巴坦那,在那受到阿特罗巴特斯热情招待,大流士三世重新组建一支新的军队,但受到亚历山大大军逼近,前330年七月大流士不得不撤离埃克巴坦那,在大流士三世逝世后一个月阿特罗巴特斯向亚历山大投降。最初,亚历山大命令欧克索达提斯(Oxydates)为米底总督,但亚历山大失去对他的信任,改任命阿特罗巴特斯重新担任米底总督。当亚历山大至印度远征归来,进驻帕萨尔加德时,阿特罗巴特斯捆绑反叛者来见亚历山大,随后前324年于亚历山大所举办的苏萨集体婚礼中,阿特罗帕提斯的女儿与佩尔狄卡斯成亲。不久亚历山大率领军队抵达埃克巴坦那过冬,据一些记载,阿特罗巴特斯当时送给亚历山大100名妇女,但被回绝了,而古希腊历史学家阿利安不相信有这回事。在这期间,赫费斯提翁在此与世长辞。当亚历山大大帝逝世后的巴比伦分封协议里,因米底行省形势重要且疆域过大,故把北方较小的部分(今日阿拉斯河盆地)至米底分割出去,并任命阿特罗巴特斯为当地新总督,培松则为米底总督,而阿特罗巴特斯的女婿佩尔狄卡斯当上了帝国摄政,当佩尔狄卡斯被谋杀之后,阿特罗巴特斯就拒绝听从之后继业者的命令,并使他的领地成为独立王国。阿特罗巴特斯建立的王朝存在数个世纪之久,虽然向塞琉古帝国和安息附庸仍可保持其独立性,而他的王国在后世的希腊人称呼为阿特罗帕特尼王国,随著时间变化,约七世纪时,当时的伊朗语称呼这地区为'Azerbaijan',成为今日阿塞拜疆的由来。","text2":"阿特罗巴特斯担任什么职务?","label":1} {"text1":"努马·德罗茨(Numa Droz,)是一位瑞士政治家。努马·德罗茨是瑞士自由民主党成员。努马·德罗茨出生于瑞士西北部纳沙泰尔州的拉绍德封(La Chaux-de-Fonds),1871年当选纳沙泰尔州议会议员,1872年当选瑞士国家委员会委员。后成为瑞士联邦委员会委员(1875年-1890年)。他于1875年12月18日当选为联邦委员会委员,至1892年12月31日卸任。在任期内,他主要主持领导了以下部门的工作:他于1881年和1887年两度出任瑞士联邦总统。努马·德罗茨1899年在瑞士伯尔尼去世。在纳沙泰尔,有以他的名字命名的努马·德罗茨广场。在拉绍德封,一条大街被命名为努马·德罗茨大街。","text2":"努马·德罗茨出生于哪个地方?","label":1} {"text1":"磁州,又作慈州、礠州,中国古代的州。隋朝开皇十年(590年)置,治所在滏阳县(即今河北省磁县)。因州西北有慈石山,出磁石,又为磁石集散地,故名。大业二年(606年)省。唐朝武德元年(618年)分相州的滏阳县、临水县、成安县复置。四年(621年),割洺州的临洺县、武安县、邯郸县、肥乡县来属。六年(623年),置磁州总管府,领六州:磁州、邢州、洺州、黎州、相州、卫州。其年,废总管府。临洺县、武安县、肥乡县仍属洺州,磁州领滏阳县、成安县、邯郸县。贞观元年(627年)又省,滏阳县、成安县还隶相州,邯郸县隶洺州。永泰元年(765年)六月,昭义节度使薛嵩请于淦阳复置磁州,领四县:滏阳县、邯郸县、武安县、昭义县(临水县)。辖境相当今河北省邯郸市、磁县、武安市等市县。因为新置州,未计户口帐籍。天祐三年(906年),因与慈州同音,改名惠州。五代后唐时,复名磁州。明朝洪武初年,省滏阳县入州,辖境相当今磁县、涉县及武安市等地。清朝时,不辖县。1913年降为磁县。","text2":"磁州名字的由来是什么?","label":1} {"text1":"白脸树鸭(\" Dendrocygna viduata\")是分布在撒哈拉以南非洲及南美洲的一种树鸭属。白脸树鸭数量丰富。牠们主要是留鸟,但也会有达100公里以上的地区性移动。牠们会在地上筑巢,每次会产8-12只蛋,有时也会在树上筑巢。白脸树鸭喜欢栖息在淡水湖或水塘,主要吃种子及植物的其他部份。白脸树鸭是群居的,在一些位点可以聚居达过千只。白脸树鸭的喙是灰色的,头部及脚都很长。牠们的面部及冠都是白色的,后枕则是黑色的。背部及双翼都是深褐色至黑色,下身是黑色,两侧有白色斑纹。颈部是栗褐色。雏鸟头部的斑纹不怎么明显。白脸树鸭是受到《非洲-欧亚大陆迁徙水鸟保护协定》所保护的。","text2":"白脸树鸭一般分别在什么地方?","label":1} {"text1":"《烟台条约》(又称《滇案条约》、《芝罘条约》;),是1876年(即光绪二年)清廷在英政府外交压力下与之签订的不平等条约,约文中文共30页,约文原存于中华民国外交部,现典藏于台北外双溪国立故宫博物院。1875年(同治十四年)2月21日,英国人马嘉理在云南被当地人杀死,演变成外交事件。清廷令海关总税务司赫德斡旋,1876年8月21日北洋大臣李鸿章与英国公使威妥玛在山东芝罘(即今烟台)谈判,9月13日,双方签约。英国取得进入云南及西藏特权。条约共三端一则:包括清政府对马嘉理事件及以前中英之间的案件各赔偿20万两白银;洋货在中国内陆免收厘金;增加开放宜昌﹑芜湖﹑温州﹑北海等为通商口岸;以及正式遣使道歉。这成为中国外交史上派出驻外长期代表的开始。1885年清廷派遣曾纪泽前往伦敦签订中英烟台条约续增专条,中英文共14页,规定鸦片进口至中国只能缴纳进口税30两及厘金最多80两,此后在中国各处行销不必再缴纳其他税捐,是清廷丧失外国货物在内地征收税捐的滥觞。约文仍存于台北外双溪国立故宫博物院。","text2":"约文现存于哪里?","label":1} {"text1":"《星战》是香港歌手古巨基的录音室专辑,2005年10月20日由金牌娱乐发行。这张专辑共收录十一首新歌,前十首的歌名乍看之下,是七至九十年代的粤语经典流行歌曲-然而,这并不是一张翻唱专辑。为了向香港乐坛致敬,制作单位借用了这些经典歌曲的歌名,再重新作曲及填词。新曲中的部分歌词亦有和原曲相对应。专辑中第一首发表的歌曲为《Monica》,本是张国荣的经典舞曲,但在重新作曲后变为慢板情歌。MV中与香港小姐冠军曹敏莉饰演情侣。另一首主打歌为《天才与白痴》,MV邀得吴佩慈客串演出。2005年度亚洲游戏展,古巨基被委任为亚洲游戏展大使。专辑中的《明星》成为大会主题曲,MV中更加入PS2游戏侠盗银河的画面。为了配合大会主题,一间英国游戏公司更为他度身订造了「银河猎人」的造型,并成为本张专辑的封面。粗体表示四台冠军歌四大电子传媒其他媒介","text2":"这张专辑共收录了多少歌?","label":1} {"text1":"赤汤车站()是一个位于日本山形县南阳市郡山,由东日本旅客铁道(JR东日本)与地方第三部门铁路业者山形铁道所共用的铁路车站。赤汤是JR东日本所属的山形新干线与奥羽本线(山形线路段),与山形铁道所属的花长井线()之交会车站,虽然在过去几条铁路线在赤汤皆有相连,但自从奥羽本线的轨道经过升级改为三轨的迷你新干线用规格之后,就不再与花长井线之间有任何直接的连结。为配合铁路系统的不同,赤汤车站的车站本体划分为东西两半,其中东口方向是由JR东日本所使用的站区,为配合山形新干线的启用而修筑的新站体,使用了来自滑翔翼的造型灵感。相反的,由山形铁道所使用的西口方向,却使用了颇有古风的原木屋(Log house)设计。两边的站区虽有相连,但旅客必须实际通过检票口之后,利用月台区的连络通道才能行抵另一侧的站区。赤汤车站除了是所在地南阳市的主车站之外,同时也是当地著名的观光点、赤汤温泉的主要进出门户。侧式月台、岛式月台混合3面4线的地面车站。","text2":"赤汤车站在哪里?","label":1} {"text1":"《建康实录》,中国六朝史料集,唐代许嵩撰。全书共20卷,现已残缺。这是一部记述三国吴、东晋、南朝宋、南朝齐、南朝梁、南朝陈六朝事的史籍。因六朝皆建都建康(今江苏南京),故以此名。作者在《序》中说,此书「具六朝君臣行事,事有详简,文有机要,不必备举。若土地、山川、城池、宫苑,当时制置,或互兴毁,各明处所,用存古迹。其有异事别闻,辞不相属,则皆注记,以益见知。使周览而不烦,约而无失者也。」《建康实录》对研究魏晋南北朝历史及南京地区历史地理都很有参考价值。虽作于《三国志》、《晋书》、《宋书》、《南齐书》、《梁书》、《陈书》及《南史》之后,但保存著某些正史不载的史实和轶事。常为后来考证六朝史事者所引征。","text2":"为什么叫《建康实录》?","label":1} {"text1":"刘定逌(),字叙臣、叔达,号灵溪,广西武缘(今武鸣)人,壮族,清朝政治人物,进士出身。雍正二十年(1734)时于县试中案首,进入学宫。乾隆六年(1741)考取贡生,乾隆九年(1744年)参加广西乡试,中举人第一名(解元)。乾隆十三年(1748年)登戊辰科进士,授翰林院编修。但因故触怒了和珅,被诬以“大考论事不如式”被劾而归。乾隆二十二年(1757年)“载书五车”归乡,写下“千年成此恨,耿耿不能磨”的诗句。此后先在隆安县驮厚村教私塾,后在家乡葛阳社学授徒,不久被思恩府聘为阳明书院长。乾隆二十八年(1763年)主讲浔州府(今桂平)浔阳书院,任浔州府山长。乾隆四十年(1775年)掌教桂林秀峰书院,任秀峰书院山长。期间,制定了《秀峰书院学规》,写《“三难”通解训言》悬于书院讲堂,作为学生行动准则长达百年之久。嘉庆六年(1801年)主讲宾州(今宾阳县)书院,任书院山长。著有《论语讲义》、《四书讲义》、《读书六字诀》、《刘氏族谱》、《刘灵溪诗稿》、《灵溪文集》等。嘉庆九年(1804年)于八十四岁高龄受邀以乾隆甲子科乡试解元身份赴鹿鸣宴。广西巡抚张百龄手书“玉清尊宿”表其门,尊他为德高望重的学界前辈。死后,武缘县壮族乡亲于县城孔庙的乡贤祠中设立牌位祀奉他。纪念他为当地的教育事业和壮族文化所做出的重要贡献。为桂林秀峰书院撰联:","text2":"刘定逌是哪里人?","label":1} {"text1":"绿党(捷克文:\"Strana zelených\")是捷克的一个政党。绿党成立于1989年12月,他们为了能够在捷克政治中获得影响力,奋斗很长一段时间。在2002年选举中,绿党获得2.4%的得票率。内部冲突及争执不断登上媒体版面,掩盖过他们原来的政治目的。2005年9月,党代表大会决议改变党的政策。新任党主席,前环境部长马尔廷·布尔西克(Martin Bursík)将重心放在他们的政治主张及媒体宣传上。党内的部分反对派离开绿党并试图筹组新政党。绿党在参议院有一位议员。2006年选举,绿党得票率6.3%,在众议院赢得了6个席次。选举前一个礼拜,部分左派党员因公开呼吁将票投给捷克社会民主党(ČSSD)而遭到开除。2007年1月,绿党与公民民主党(ODS)和基督教民主联盟-捷克斯洛伐克人民党(KDU–ČSL)共同组成执政联盟。2010年选举,绿党得票率2.4%,所以没有进入新一届议会。党主席翁德热·利什卡(Ondřej Liška)辞去职务,不过他在2010年和2012年的全国代表大会再被选为党主席。","text2":"2005年绿党的政策重心有何改变?","label":1} {"text1":"澎湖玄武岩自然保留区位于台湾澎湖县,包含锭钩屿、鸡善屿及小白沙屿3个无人岛,主要的保育对象为当地特殊的玄武岩地形景观。澎湖群岛是台湾三大火山群之一,除了花屿之外,各岛大多由玄武岩组成。火山熔岩流在冷却时体积收缩,形成玄武岩的柱状节理,呈现六角柱或多角柱结构。随后由于海蚀及其他地形作用的影响,形成许多高低起伏、变化多端的柱状玄武岩。保留区位于黑潮支流、南海季风流、及潮汐流交会处,邻近海域蕴藏丰富的渔类资源。鸟类大多为冬季候鸟和过境鸟,常见鸟种包括黄足鹬、大苇莺和家燕等。夏季候鸟以苍燕鸥和白眉燕鸥较多,其次为小燕鸥和红燕鸥。澎湖县野鸟学会2014年6月在保留区内的鸡善屿发现有黑嘴端凤头燕鸥在此繁殖,是马祖列岛、韭山列岛后,确认的第三个繁殖地。「世界遗产」登录工作有许多前瞻性的保存观念,为使国人保存观念与国际同步;2002年初,文化部(原:行政院文化建设委员会)陆续征询国内专家及函请县市政府与地方文史工作室提报、推荐具「世界遗产」潜力点名单;其后于2002年召开评选会选出11处(2009年经会勘后,增加至17处)台湾世界遗产潜力点,澎湖玄武岩自然保留区便是其中之一。澎湖玄武岩自然保留区是由地底流出的火山熔岩冷却形成后,形成各式的柱状玄武岩。由玄武岩组成的岛屿受到海蚀作用形成海崖、海蚀洞、海蚀柱、海蚀沟等天然美景,在亚洲地区群岛中更是少见,正符合世界遗产登录标准的第七项。澎湖玄武岩自然保留的地质年代是台湾海峡火山熔岩最活跃的年代,至今仍保留非常独特与优美的玄武岩地景,其雄伟柱状节理及丰富的地形变化符合世界遗产登录标准的第八项。澎湖玄武岩自然保留位处偏远,海流湍急、岩壁陡峭,人迹罕至,因此,每年4月至9月已成为保育类珍贵稀有鸟类的繁殖天堂,2002年更发现濒临绝种的海洋野生动物—绿蠵龟上岸产卵,极具研究与保育价值,符合世界遗产登录标准的第十项。","text2":"澎湖群岛是台湾三大火山群之一,除了花屿之外,各岛大多由什么组成?","label":1} {"text1":"帕特里夏·派伯敏特·佩蒂·雷查德(),漫画人物,是漫画家查尔斯·舒兹从1950年代起连载的漫画作品《花生漫画》中的重要角色之一。派伯敏特·佩蒂是主角查理·布朗的同级生,不过不同班。运动少女,个性豪放随和,上课时必打瞌睡,因此常遭老师责骂。但对于运动则极有天分,自己组建棒球队,在球季之间无往不利,与查理·布朗那支只败不胜的球队形成对比。同时也有从事网球、划艇、榄球、花式溜冰、游泳等运动的记录,皆成绩斐然。平日的形象是一头乱发,基本上是随意的半长发。脸大,鼻子大,有雀斑,阳光笑容。习惯穿绿色衬衫和短裤,脚踏破旧凉鞋。个性直来直去,有话直说,纯粹的户外系。但也有心思细腻的一面,乐意照顾他人,因此颇留得住朋友。被查理·布朗那罕见的温顺性格所吸引,暗中颇怀好感,平时对查理·布朗非常照顾,表面上是哥们关系。将查理布朗昵称作“查克”。与玛西是死党。虽然一文一武、一动一静,思维方式亦南辕北辙,却相处融洽。基本上是佩蒂负责捧哏\/发呆役,玛茜负责吐槽。由于相识时,佩蒂是玛茜的夏令营领队,所以玛茜叫佩蒂作“SIR”。对史努比的运动能力非常折服,是英雄惜英雄的关系。她一直误会史努比是一个人,称他是查理·布朗的大鼻子好友。","text2":"派伯敏特·佩蒂是谁?","label":1} {"text1":"23街是横贯美国纽约州纽约市曼哈顿区的一条主要街道,为曼哈顿格子式街道计划中少数能双向通行的街道。23街以第五大道为界(交汇于麦迪逊广场公园),分为东23街与西23街。自1999年起,麦迪逊广场公园附近23街以北的区域被称为NoMAd,取英文麦迪逊广场公园北方之意(NOrth of MADison square park).。23街现在从东河到第十一大道结束,过去曾经可以一直通到哈德逊河畔。西23街为哈德逊河到第五大道的段落。西23街贯穿雀儿喜区。十九世纪后期西23街位于雀儿喜的路段有著许多著名的剧院,例如皇宫歌剧院()、派克歌剧院()、与普洛克特剧院(),是当时美国剧院的指标,其意义就如今日的百老汇一般。二十世纪初期其西端为位于雀儿喜码头北边的63号码头帕翁尼亚渡船口(),亦造就了西23街雀儿喜区的繁荣。在百老汇大道上的帝国剧院()开幕也象征百老汇剧院区逐渐取代位于西23街的雀儿喜剧院区的地位。雀儿喜旅馆位于西23街第七与第八大道中间的路段,于1884年建成,是纽约市第一个合作公寓大楼,一直到1902年皆为纽约最高的大楼。东23街为第五大道至东河(罗斯福快速道路)。东23街为葛莱姆西公园区(Gramercy Park)的主要道路。大都会人寿保险的总部就位于东23街与麦迪逊大道的交接口。大都会人寿位于麦迪逊大道1号的大楼为曼哈顿最早的摩天大楼之一,上面的钟为其特色;麦迪逊大道11号大楼原本设计的比现在还要高,但是因为经济大萧条的缘故,大都会人寿只好缩减大楼高度,但是其装饰风艺术设计仍是非常精美。1966年10月17日东23街发生了一场大火灾,两栋楼房遭祝融之灾,十二名消防员命丧火场。二十三街火灾()为911事件之前纽约消防史上消防员死伤最严重的一场火灾。东23街与百老汇大道交叉口为著名的熨斗大厦所在地。所有经过23街的纽约地铁路线皆有在23街设慢车停靠站:纽新航港局过哈德逊河捷运在23街也有设站。纽约市公车的M23行经23街全线。","text2":"西23街横穿过哪个区?","label":1} {"text1":"假想粒子,理论物理学家提出的物理模型中假想的一些粒子,基本没有切实的实验依据,很有可能宇宙中根本就不存在这些粒子。这些粒子的提可能只是为了给某些物理现象作一种可能的解释,或者是因为这种粒子如果存在也不会破坏现有的物理定律,因此没有理由相信它们一定不存在。暗物质相关假说中的假想粒子有大质量弱相互作用粒子(WIMP)、惰性中微子(sterile neutrino)、加速子(Acceleron)。超光速理论研究中的运动速度远超光速的快子(tachyon,也叫速子、迅子)。弦理论中一些仅带有北极或南极单一磁极的磁单极子,以及X玻色子和Y玻色子。作为标准模型的补充提出的超对称理论认为,每一种基本粒子都匹配一种被称为超对称伙伴(Superpartner)的粒子,玻色子的伙伴如重力微子(gravitino)、光微子(photino)、胶微子(gluino),而费米子的伙伴叫超粒子(sparticle),就是在每种费米子前加一个s,如超电子(selectron)、超夸克(squark)、超中微子(neutrilino)。此外还有其它一些理论提出的轴子、引力子等等。在标准模型理论基础上还有一些可能的新强子态复合粒子被提出,由2个胶子或3个胶子组成的胶球(glueball),由胶子和夸克组成的混杂态(hybrid state),由4个夸克组成的四夸克态,由5个夸克组成的五夸克态,由6个夸克组成的六夸克态(双重子态)等等。","text2":"由胶子和夸克组成的混杂态有哪些?","label":1} {"text1":"《今日VIP》(),是香港电视广播有限公司的一个娱乐访谈节目,于2009年2月2日起开始播出,前身是娱乐资讯节目《娱乐直播》。本节目主要是名人及TVB旗下的当红艺人的专访。本节目于2009年2月2日开播时逢星期一至五15:05于翡翠台播映,并于翌日深夜01:15重播。另外,高清翡翠台亦会01:45及04:30重播节目(但在2009年6月15日起,将改为当天17:15-17:50及翌日05:00-05:30重播)。节目同时于tvb.com内myTV提供节目内容足本重温。由2010年12月20日起,本节目于翡翠台改于14:15-14:45播映,并于翌日深夜01:25-01:55重播,节目长约半小时(连广告)。此外,本节目于高清翡翠台改于17:15-17:45(但于2012年7月16日起取消)及翌日05:10-05:35(2012年9月25日起改为05:00-05:30)重播。由2012年9月25日起,翡翠台深夜重播改为01:20-01:45。2012年11月29日起,深夜重播时段改为01:45-02:15。2013年3月19日起,翡翠台深夜重播改为01:25-01:55。由2016年2月22日起,由于高清翡翠台改名为J5关系,不再于J5播出。如遇上特别事件,如政府记者会或部分重要的立法会会议,则改为只于深夜播出或全日暂停播映本节目,是TVB的节目中较少出现的安排。本节目逢每年新春假期(农历正月初一至农历正月初三)暂停播映。2016年3月7日起,更换新片头。现任:前任:","text2":"《今日VIP》的前身是什么?","label":1} {"text1":"巴黎第三区()是法国首都巴黎市的20个区之一。该区位于塞纳河右岸,面积1.171km²,是巴黎市第二小的区。该区包括中世纪风格的玛莱区北面较安静的部分(玛莱区较活跃的南部,包括巴黎的同性恋区,属于第四区)。巴黎保存至今最古老的私人房屋,建于1407年,位于第三区的蒙莫朗西路()。虽然小但是扩展迅速的中国城,居住着来自中国温州的移民,聚居在市长路()。附近是国立巴黎工艺技术学院(),坐落在中世纪Saint-Martin-des-Champs小修道院中。该区的人口峰值在1860年区划调整之前。1999年,该区人口为34,248人,工作岗位 29,723个。人口峰值出现于1860年区划调整以前。","text2":"巴黎第三区包括哪几个部分?","label":1} {"text1":"杜文辉(),中国足球运动员,司职前锋。1998年,杜文辉正式进入北京国安三队。1999年,杜文辉和邵佳一、崔威、王硕等四人去法兰克福培训半年。2002年,杜文辉进入北京国安一线队,杜文辉在热身赛中因骨折养伤3个月。2002年北京国安成绩非常好,所以伤愈后全年他一场联赛没打。2008年3月30日,中超首轮最后一场比赛北京国安主场迎战河南建业。比赛进行到第78分钟,北京国安发动反击,陶伟传球时被对手得到,然后河南建业的防守队员竟然出现致命失误,球被杜文辉断到,杜文辉随即劲射破网,将场上比分扩大为2-0。杜文辉这一进球也成为了中超联赛历史上第2000个进球。2011年,杜文辉与北京国安合同完结并正式转会江苏舜天,在当年实行自由转会后,成为第一位以自由球员身份加盟到其他球队的球员。2012赛季结束后,杜文辉离开了江苏舜天。2013年7月,中甲球队湖南湘涛正式宣布与杜文辉签约。","text2":"杜文辉什么时候正式进入北京国安三队?","label":1} {"text1":"彩荣路(),是一条位于香港九龙观塘区平山的两线双向行车道路。该路由彩兴路与彩荣路交界开始,经过彩霞道交界,后向彩福邨方向伸展,终点尽头位于彩福邨彩欢楼、彩荣路公园前。彩荣路最首次于1999年5月28日出版的「牛头角及九龙湾分区计划大纲图S\/K13\/13」出现,该计划大纲图已可见彩荣路的雏型。于彩云道及佐敦谷毗邻的发展计划中出现的编号E1、E2及G1道路,则与1999年规划的有所出入,相信已作出修改。E1道路由彩兴路与彩荣路交界为起点,终点尽头于彩霞道与彩荣路交界;E2道路则指由彩霞道与彩荣路交界起至彩荣路与彩荣里交界为止的路段;G1道路指由彩荣路与彩荣里交界起至彩荣路回旋处止的路段。彩荣路首段已于2008年7月11日刊宪,其后于2010年7月16日将其余一段彩荣路刊宪。现在彩荣路分别设有5个巴士站,其中在彩福邨附近设置了两个巴士站。彩荣里(),是香港牛头角佐敦谷内的一条道路。该路主要连接彩云道海水配水库及彩云道食水配水库。该路的路旁设至一条通往佐敦谷登山径的入口。","text2":"现在彩荣路设有几个巴士站?","label":1} {"text1":"牛蒡(学名:)是菊科牛蒡属的植物,又名东洋参、东洋牛鞭菜、白肌人参、吴某、吴帽、夜叉头、牛菜、鼠粘、蒡翁菜、便牵牛、蝙蝠刺、牛旁、便南牛,果实别名恶实、大力子、黑风子、大牛子、鼠粘子、鼠尖子。二年生大型草本,肉质根;广卵形至心脏形叶子,背面密生白毛;头状花序簇生,具有先端呈钩刺状的总苞片,夏秋开紫红色管状花;长椭圆形或倒卵形瘦果,先端有刺毛一束。分布于欧亚大陆、美洲大陆寒温带,生长于海拔750米至3,500米的地区,一般生于林缘、林中、灌木丛中、山谷、村庄路旁、山坡、河边潮湿地和荒地。台湾食用牛蒡的风气主要是受到日本影响,原来并没有野生种,但经由引进种植,在屏东县屏东市的归来地区(湖西里 湖南里 归心里 顶柳里 顶宅里)人工种植成功后,被列为当地名产之一。日本在太平洋战争期间俘虏英美战俘,因战时粮食短缺,日本人强迫英美战俘吃牛蒡,而英美战俘在饮食文化上不知道牛篣是一种食物而当作树根吃而误会为虐待战俘,这件事件也被英美战俘控告到国际法庭之上。(此事件在佐伯俊的料理漫画《食戟之灵》〈实地实习篇〉亦有引用,由四宫小次郎说明。)台湾人习惯使用牛蒡细长的根部切块与排骨炖煮成「牛蒡排骨汤」,油炒牛蒡丝或是切片后泡茶,或切丝为凉拌菜,据信有减肥和养生等效用。在欧洲中古世纪,牛蒡被当成蔬菜食用,现已不再普遍。东欧和北欧等国还习惯使用其花部做成洗发精、润发精或点心,亦加在茶中入味。牛蒡的成熟果实(牛蒡子)是常用的中药。牛蒡子性味辛苦寒,可疏散风热、透疹、消肿利咽,用于银翘散、牛蒡汤、透疹汤、瓜蒌牛蒡汤、普济消毒饮等方剂中。可做为抗氧化剂。可借由帮助控制细胞出现突变,进而抵抗癌症。协助过多的液体、尿酸及毒素排出体外。具有抗细菌及抗霉菌的特性。纯化血液、恢复肝脏及胆囊的功能、刺激消化及免疫系统作用。帮助疖以及面疱这类的皮肤问题,可减轻痛风以及更年期的症状。牛蒡根可以用来滋润头发,以促进头皮及头发的健康。","text2":"牛蒡生长的地区海拔约是多少米?","label":1} {"text1":"黄尾魣(学名:),又称黄尾金梭鱼,俗名针梭、竹梭,为辐鳍鱼纲鲈形目鲭亚目金梭鱼科的其中一种。本鱼分布于印度西太平洋区,包括东非、红海、模里西斯、阿曼、印度、泰国、圣诞岛、可可群岛、马来西亚、日本、台湾、越南、印尼、菲律宾、巴布亚纽几内亚、香港、澳洲、马里亚纳群岛、新喀里多尼亚、关岛、萨摩亚群岛、东加等海域。该物种的模式产地在红海。水深3至50公尺。本鱼体延长呈鱼雷状,横切面几近圆柱形,侧线直而不弯曲;尾鳍深分叉,胸鳍末端达第一背鳍起点,腹鳍起点远前于第一背鳍起点。鳃盖具一尖棘,上颔骨远前于眼前缘。背部暗绿色,腹部银白色,尾鳍镶黑边,在侧线下具一暗色纵带,背鳍硬棘6枚;背鳍软条9枚;臀鳍硬棘2枚;臀鳍软条9枚,体长可达60公分。本鱼常成群地在礁湖或向海礁区的上方游走,属肉食性,以鱼类为食。为食用鱼,适合各种烹调方式。","text2":"黄尾魣的体长可达到多少?","label":1} {"text1":"双斑长鳍天竺鲷(学名:),又称黑尾长鳍天竺鲷、暗体长鳍天竺鲷,俗名大面侧仔、大目侧仔,为辐鳍鱼纲鲈形目鲈亚目天竺鲷科的一个种。本鱼分布于西太平洋区,包括日本、印尼、巴布亚纽几内亚、菲律宾、澳洲、帛琉、马里亚纳群岛、斐济、关岛、萨摩亚群岛、索罗门群岛、万那杜、东加等海域。水深0-33公尺。本鱼体呈圆形侧扁,眼大。体呈淡红褐色,背部颜色略深,腹部银白色。被栉鳞且具红边缘,头部有一深褐色直条纹从眼睛延伸至鳃盖下缘,在塞概后上方靠近额部之地方有一黑色眼斑,尾柄部也具一黑色眼斑,尾鳍凹入,臀鳍延伸,背鳍硬棘7枚;背鳍软条9枚;臀鳍硬棘2枚;臀鳍软条16-17枚,体长可达11公分。本鱼栖息于海湾与潟湖,喜群游,属肉食性,以浮游生物和底栖性无脊椎动物为食。雄鱼与雌鱼交配后,雄鱼会将卵含在口中保护,孵化之。无任何经济价值。","text2":"双斑长鳍天竺鲷是怎样繁殖的?","label":1} {"text1":"东京KAWAII TV(日语:,英译:Tokyo Kawaii TV)是日本放送协会的潮流节目,于日本时间(UTC+9)每逢星期三0时10分至0时39分播出。2010年5月起,播放时间变更为每逢星期日23时30分至23时59分播出。2010年9月起,播放时间变更为每逢星期六23时00分至23时29分播出。2011年3月起,播放时间再变更为每逢星期六23时05分至23时34分播出。现在的主持人是泽村一树和BENI。Kawaii TV的固定模特儿是纱耶。由于该节目以日本潮流指标为节目内容,在日本以外地区亦有播出。2008年7月起,开始于香港无线电视J2台播出(目前较日本慢约两个月);同年12月于香港无线生活台播出。日本于2013年3月23日播放完毕。制造这些流行时尚的是10至20多岁的普通东京女孩子。她们发挥自己潮流触觉,成为潮铺店员兼生招牌,业余模特儿甚至建立自己品牌……Tokyo Kawaii TV 节目每周播出不同种类的歌曲。(详情请浏览官方网页)","text2":"播放时间变更为每逢星期六23时00分至23时29分播出是从什么时候开始?","label":1} {"text1":"约翰尼·卡尔·埃马纽埃尔·申克( Johann Karl Emmanuel Schenk ,)是一位瑞士牧师、政治家,瑞士联邦委员会委员(1863年-1884年)。截至2008年,他仍然是瑞士联邦历史上任期最长的联邦委员会委员。申克于1823年12月1日出生于瑞士伯尔尼一个基督教家庭。11岁时申克成了孤儿,在德国巴登-符腾堡州路德维希堡县生活。1839年到1842年期间,他在伯尔尼学习,后修神学。1845年,在他22岁时成为一名牧师,开始在瑞士伯尔尼州许普芬(Schüpfen)任教区牧师。1848年,他与埃利斯·卡尔(Elise Kähr)结婚,他们共育有九个孩子,其中两个年幼夭折。他于1863年12月12日当选为联邦委员会委员,任职期长达31年,他于1895年7月18日死于任内。卡尔·申克是瑞士自由民主党成员。在任期内,他主要主持领导了以下部门的工作:期间,卡尔·申克先后六次出任瑞士联邦总统,分别是在1865年、1871年、1874年、1878年、1885年和1893年。卡尔·申克1895年在伯尔尼去世,他被安葬在伯尔尼的许希利胡斯(Chüechlihus)。","text2":"卡尔·申克是什么时候出生的?","label":1} {"text1":"记忆树是利用关联性记忆法,有效的将大量的资料整理并且快速的在脑中留下印象,特色是会以一个主题当作是主干,与其相关联的资料会采上下半辐射状依序连结出,呈现出类似树状图的图像,故称为记忆树,属于心智图应用的一种。记忆树的主要原理是采放射性思考的笔记方式,与传统的笔记方式中条列重点的方式相异,是用图像式地从记忆主干向外展开的记录方式来写。利用关联和所谓的强调去加强对事件的记忆程度,平常我们的大脑分左右去处理事情,人在记忆的时候用左脑处理文字和语言、而右脑则是处理影像,所以若在记忆的同时用图画在旁强调提示,当左脑遗忘了文字和语言,右脑反而会记下自己所画的图画。也就是说,记忆树的原理在于当单调的语言或是文字被我们的左脑所遗忘时,看到图像的右脑可以及时发挥功效,进而连结到左脑去唤起被遗忘的文字。整体关系是经过联想的,所以不但容易记忆,而且有助于思考。如此的记忆方式,也受到其他诸多国家采用。采用又画图又画线的记忆方式连来连去,记忆的物件就不单只是文字了,若是加进了很多别的东西,自然也会用到别的大脑区位,顺便正好可以刺激平常用不到的大脑区位,分散了记忆的工作。有些安养院会提供各种不同物品的图片去刺激老人的大脑,使老人的脑部因为刺激而减缓退化,在治疗阿兹海默症的非药物疗法中也是利用这个原理,将有纪念意义的照片记念品等放置在病人旁边,去延缓病人脑细胞的破坏。在日剧以及漫画的东大特训班也有提到了记忆树的画法,其中颠覆了学生对于传统抄写笔记的观念,一般的学生都认为在抄写笔记时,应力求版书的工整以及字迹的漂亮与否。然而在东大特训班中特别提到了\"关联\"才是最重要的部分。字丑一点没有关系,图画的不好看也无所谓,只要能够帮自己建构出事件与事件当中的关联性即可。以法国大革命为例有可以简单的画出记忆树的软体。FreeMind:以开放原始码的方式撰写开发的软体,可以画出思维导图,也就是心智图。","text2":"为什么说记忆树方法不但容易记忆而且有助于思考?","label":1} {"text1":"斯诺登站(、)位于加拿大魁北克省蒙特利尔市,是蒙特利尔地铁2号线及5号线的换乘站,设有上下4个月台,并以岛式月台排列,跨月台转车站的转乘设计让乘客只要走到对面月台便可转线。斯诺登站设有上下两层4个月台,上层为2号线往禾度岭及5号线终点落客月台,主要供乘客由5号线月台步行过对面月台,转乘2号线往禾度岭沿途各站。下层为2号线往蒙莫朗西(经市中心)及5号线往圣米歇尔月台。由于2号线及5号线亦有另一个转线站庄塔隆站,因此此设计方便从禾度岭及斯诺登之间出发的乘客步行往对面月台乘坐5号线往东区及拉华尔市,从而避免绕经市中心一带所浪费的时间。此设计是为斯诺登站设计时提出的5号线恩典圣母延长线而设的,方便恩典圣母地区的乘客快速转线前往市中心。斯诺登站亦为蒙特利尔地铁系统中仅有的两个跨月台转车站之一(另一个为莱昂内尔·格鲁站)。斯诺登站主要用作转线用途,加上附近多为住宅区,所以只设有一个出口。斯诺登站是2号线中途站,亦为5号线的总站,乘客可以在上述路线之间互相转乘,来往蒙特利尔岛上各处。斯诺登站附近设有多条蒙特利尔交通局巴士路线的巴士站,供地铁乘客转乘接驳,来往附近一带区域。斯诺登站作为2号线的西面总站于1981年9月7日,当时5号线尚未竣工,然而该线的月台经已建成。5号线于1988年1月4日通车。","text2":"斯诺登站是蒙特利尔地铁系统仅有的什么类型的车站?","label":1} {"text1":"上田车站()是一位于日本长野县上田市天神1丁目,由东日本旅客铁道(JR东日本)、信浓铁道()与上田电铁所共用的铁路车站。上田车站是JR东日本所经营的北陆新干线与两条地方铁路线信浓铁道线()与上田电铁别所线之交会车站。配合全面高架化的新干线路线,JR东日本所属的月台区是位于车站二楼(但检票口位于一楼),至于信浓铁道与上田电铁的部分,检票口皆位于二楼的桥上站屋内,但月台与铁路线则分别设置于二楼(信浓铁道线)与一楼(别所线)处。对向式月台2面2线的高架车站。。是北陆新干线轻井泽至长野段唯一的高架车站。由于不设待避线,因此设有月台闸门。起初计划为岛式2面4线,但根据通车后的需求预测轻井泽与长野之间不需要待避设备,成为现时对向式月台2面2线。对向式月台(下行本线)与岛式月台(中线与上行本线)2面3线的地面车站。侧式月台1面1线的高架车站。1998年3月29日由地面车站改建为高架车站。","text2":"由地面车站改为高架车站是在什么时候?","label":1} {"text1":"管海马(学名:)为辐鳍鱼纲棘背鱼目海龙科海马属的鱼类,俗名库达海马。其繁殖过程独特,交配时雌性将卵子转移到雄性管海马的育儿袋中。受精作用何时进行仍然不明,但经受精后的合子至少会留在受儿袋中四至五星期,直至雄性管海马将它们喷出为止。分布于日本、印度洋、新加坡、菲律宾、夏威夷群岛、澳洲及非洲东部、台湾、香港以及渤海、东海、南海等海域。该物种的模式产地在新加坡。本鱼体延长,口小,吻突出呈管状,头大。鳞片特化成骨板,颈前板上有冠状突起。体色多变,从黄色、褐色、黑色甚至橘红色都有,无尾鳍。本鱼栖息于藻类或海草丛茂盛的海域,适应力强,略能忍受盐度的变化,常以尾部缠绕海草、珊瑚或石块上,游泳能力弱,体色随环境变化。属肉食性,以无脊椎动物为食,繁殖期为春夏季,雌鱼将卵产在雄鱼的孵卵囊中,由雄鱼负责照顾。古希腊和古罗马相信海马是波塞冬或海王星的海神属性,被认为是象征力量和权力。 欧洲人相信海马携带死去水手的灵魂到地府,给他们安全通道和保护,直到他们达到自己的灵魂的目的地。观赏鱼,可做为中药材。","text2":"为什么说管海马繁殖过程独特?","label":1} {"text1":"参加2008年北京夏季奥林匹克运动会的中国游泳队一行共计70人,其中男运动员17人,女运动员31人,官员与工作人员22人,领队尚修堂。中国除男子4X100米混合泳接力外,在全部六项游泳接力赛中取得了五项参赛资格;中国在游泳项目的参赛人数仅次田径。参加本次奥运游泳队共计22名官员与工作人员,其中教练员17人,领队尚修堂、副领队江斌波。·教练员(17):张亚东、么正杰、叶瑾、韩冰岩、陈映红、刘海涛、石晓铭、朱志根、潘佳章、姚颖、徐惠琴、徐国义、陈勤、常谊春、魏亚平、冯真、吕森·管理:陆一帆,医生:郭清华和巴震。男运动员男运动员共计17人,分别是蔡力、陈祚、张恩剑、张琳、孙杨、孙晓磊、邓健、薛瑞鹏、赖忠坚、石峰、陈寅、吴鹏、曲敬宇、黄绍华、吕志武、施浩然和辛桐。女运动员女运动员共计31人,分别是朱颖文、徐妍玮、庞佳颖、朱倩蔚、谭淼、李茉、李玄旭、尤美宏、赵菁、陈燕燕、齐晖、孙晔、罗男、陈慧佳、周雅菲、刘子歌、焦刘洋、李佳星、唐奕、王丹、杨雨、刘京、哈思楠、汤景之、高畅、洪文文、王然迪、徐田龙子、王群、李哲思和方晏乔。","text2":"08年北京奥运游泳队的领队是谁?","label":1} {"text1":"丝鳍高身天竺鲷(学名:)为天竺鲷科高身天竺鲷属的鱼类,俗名丝鳍圆天竺鲷、丝鳍天竺鲷。分布于日本的西表岛、热带印度西太平洋、台湾岛等。该物种的模式产地在Manado、苏拉威西岛。可长至8公分。体色灰白,第一背鳍下有一宽黑横纹,后半部分身体有黄棕色圆点,第二背鳍延长成丝状。夜行性,栖息于6到14米深的珊瑚礁区域,群聚在珊瑚的枝缝内。夜间会游到靠近水面处。食物主要是动物类浮游生物。能够适应饲养环境,可以冷冻磷虾等投喂,甚至会进食浮片饲料。在观赏鱼的发展中已经有人工繁殖的纪录。本种和S. orbicularis(斑带天竺鲷)大致相似,其区别方式可根据体侧的横带宽度,和横带后方的圆点密度区别,本种白宽频较宽;圆点密度较大。口孵性鱼类,雌雄差别较小,不易分辨。繁殖时雄性会将卵含在嘴里进行孵化,约1周左右可孵化出幼体。幼体个体较小,成活率低。繁殖时需提供安静环境,以防雄性受惊后吐卵。卵吐出后较难孵化。","text2":"它们的模式产地是哪里?","label":1} {"text1":"《Just can't help it.》(,\"Just can't help it.\")是东京事变的第五张DVD,于2006年9月6日发行。发行当周即卖出3.0万张,总计销售额5.1万张,名列2006年年度销售榜第30位。初回限定为「Digipak纸盒仕样」。此张DVD是收录东京事变《\"DOMESTIC!\" Just can't help it.》 巡回演唱会的其中一站,于2006年5月26日在NHK会馆(NHK HALL)所举行的,其演唱的歌曲及幕后花絮。演唱会中的新曲「Mirror Ball」、翻唱林檎给友阪理惠「机器少女」和「停电」于8月21日开始接受线上下载的服务,同一天也成立了DVD特设网站。在日本09月6日发行DVD,台湾则是等到09月26日才发行进口盘。《\"DOMESTIC!\" Just can't help it.》为二期东京事变第一次巡回演出,总共在十六个城市举办二十一场的演唱会,场场皆吸引爆满的观众前往感受东京事变的魅力。演唱会演唱的曲目以第二张专辑《大人》为主,演唱会的开场便是日本传统乐的歌曲雪国,身穿和服的林檎,抓住了众人的目光,还有服务这首歌,全体团员拿著大声公演唱,中途搭配上团员们台上换装的片段,带起一波高潮。而演唱会利用投影的方式,搭配团员们的的动作与歌曲的内容,让人惊叹,像是中场时,鼓手刄田缀色凭空踢球的效果,还有脸孔这首歌时,钢琴手伊泽在钢琴独奏时,便把改编的英文版本歌词投影至舞台,这种投影的效果,不但提高了演唱会的可看性,更增加了与台下观众互动的机会。另外由于椎名林檎在专辑《大人》发行接受访问中提到「今后,她想要专心作词和演唱,作曲则会交给其他成员。」因此,在巡回演唱会上发表了浮云所创作的曲目「Mirror Ball」,借此希望之后的活动能以团员的创作曲目为中心。 椎名林檎 - 主 唱・吉他手 龟田诚治 - 贝斯手 刄田缀色 - 鼓 手  浮云  - 吉他手 伊泽一叶 - 钢琴手・吉他手","text2":"在演唱会中运用投影带来了什么效果?","label":1} {"text1":"在厘米-克-秒单位制(CGS制)里,静库仑(statC)或(esu)是电荷的物理单位。它是一种衍生出来的单位,表达为在国际单位制里,则采用库仑(C)为电荷的物理单位。转换公式为这公式乃精确的(但是,请参阅后面正确使用方法的警告)。在公式右边的数值是光速(CGS制)的十分之一。双方向转换大约为静库仑定义为:假若两个固定的,相距1 cm的点电荷各自带有电量1 statC,则它们彼此互相排斥的静电力是1 达因(dyne)。这排斥力是由库仑定律给出,在CGS制里,表达为其中,formula_2是力量,formula_3、formula_4分别是两个点电荷的电量,formula_5是它们之间的距离。做一个因次分析。formula_2的因次是[质量][长度][时间]。所以,在CGS制里,电荷的因次必须是[质量] [长度] [时间]。在国际单位制里,这句话并不正确。因为,库仑定律的形式不同:其中,formula_8是真空电容率。由于formula_8本身有因次。与静库仑不同,库仑的因次不是[质量] [长度] [时间]。实际上,我们无法只用质量、长度和时间来表达库仑的因次。因此,以下这公式必须非常小心地使用,公式两边的单位并不一致:我们不能直接地使用这转换因子在库仑与静库仑之间做转换,就好像我们在公分与公尺之间做转换一样。更确切地说,这公式应该诠释为:\"1 coulomb\"对应于\"2997924580 statC\"。换句话说,假若某一个带电体的电量是1 coulomb,那么,它的电量也是 2997924580 statC。特别注意,以下这转换的因次完全一致,对于国际单位制和CGS制之间的转换,非常有用:其中,formula_8是CGS制的真空电容率,大约为8.85 As gcm。","text2":"静库仑在国际单位制里面采用什么形式?","label":1} {"text1":"《开心大发现2009》(,曾暂名为Family Wisdom 2009),为香港亚洲电视一个生活智慧节目,其目的表示为普罗大众带来开心。于逢星期一至星期五晚上20:05-20:30在亚洲电视本港台,于2009年4月20日起首播,每集大约30分钟(连广告)。其后于亚洲电视数码频道亚洲高清台星期六深夜00:00-01:30及星期日深夜00:50-01:50重播。节目是继承2004年亚洲电视受欢迎的《开心大发现》,但由于多位原主持陆续离开亚洲电视,故除鲍起静外,所有主持均属首次主持此系列;而于6月14日,亚洲电视宣布加入三位主持,原主持人林隽健则退出主持阵容。另外,2009版本亦停止播放外购的日本片段,全由亚洲电视制作。2010年,亚洲电视于暑假期间再次制作《开心大发现》节目系列,并命名为《开心大发现2010》。为香港观众提供的大发现,会播放观众4:3片段,多为生活大发现。该环节主要是于第二节播放,讲解有关烹调食物时的「大发现」,多以主持人作有否使用「大发现」作对比。而由于节目由德国宝香港有限公司赞助,故所有电器均使用其品牌,而该环节则明显显示电器中的品牌。该环节主要是于第二节播放,其「大发现」为旧两辑曾经介绍,现再重新介绍。由于被网民指有「炒冷饭」之嫌,现时的「经典大发现」己取消。这与Yahoo! 知识+及3香港合作,由观众提供「大发现」,通过文字、影像上载至3香港或Yahoo! 知识+,若一经采用,会得到现金奖。这与Yahoo! 知识+及3香港合作,会以互动形式予观众参与,每日均会发表片段,让观众猜猜如何解决问题,胜出者会得到奖品。以下为该节目于亚洲电视首播之收视纪录:资料来源:CSM媒介研究(一个收视点代表63,600名观众)亚洲电视授权Wiser出版于2009年7月推出发行了《开心饮食大发现》书籍零售版本,此书籍可以把《开心大发现2009》之《饮食大发现》内容改编而录成书籍,重新编写,重要添加拍摄片中内容资料图片。","text2":"该节目源自于哪个节目?","label":1} {"text1":"婆罗洲战役是第二次世界大战中同盟国在西南太平洋战区最后一场主要军事行动,在一系列由5月1日至7月21日期间的两栖登陆行动中,由指挥的攻击占领该岛的日本军队,盟国的海空军力量,主要是由托马斯·金凯德指挥的美国海军第7舰队、及美国陆军航空军第13航空队亦在此战役中扮演重要的角色,在战役中,他们受到了由海军中将指挥、位于婆罗洲东南部的日本陆军和日本海军,以及由陆军中将指挥、位于西北部的日本第37军的抵抗。虽然该战役当时在澳洲受到批评及在以后时间被认为是没有意义或浪费士兵的生命,它仍然达到一系列目的如增强对占领荷属东印度大部份地区的日军的孤立、攻占主要石油供应地区释放盟军战俘,当时这些战俘的处境已十分恶劣(参见及)。盟军在婆罗洲一系列行动计划代号为双簧管及是属于蒙特克莱尔行动的第2阶段,该行动的目的是消灭当地的日军及收复荷属东印度、菲律宾南部及英属婆罗洲北部,蒙特克莱尔行动的第1阶段,代号为胜利,目的是在菲律宾的彭内岛、宿务岛及内格罗岛登陆及已在1945年中完成。原本双簧管行动分为6个阶段:双簧管1号行动是攻打打拉根;双簧管2号行动是攻打峇里巴板;双簧管3号行动是攻打马辰;双簧管4号行动是攻打泗水或巴达维亚(雅加达);双簧管5号行动是攻打荷属东印度东部;及双簧管6号行动是进攻英属婆罗洲(沙巴),最后只有进攻打拉根、峇里巴板及英属婆罗洲—纳闽及文莱被实施,整个战役由1945年5月1日实施的拉开序幕,于打拉根的东北部海岸实施登陆,之后在1945年6月10日实施: 在婆罗洲北部的纳闽及艾文莱海洋实施登陆,1个星期后澳军根据双簧管6号行动的计划,之后盟军主力转移至东面中部海岸,于1945年7月1日实施,在峇里巴板实施第二次世界大战中最后一次大规模两栖登陆作战。这些行动最终亦是澳军队对日军的最后一系列行动。婆罗洲日据时期","text2":"原本双簧管行动分为几个阶段?","label":1} {"text1":"张磊(),中国足球运动员,司职守门员。张磊13岁时入选广东宏远少年队,随后担任东莞南城的主力门将,2006年转会到深圳队。他入选了08之星国奥队,并进入2005年世青赛的大名单。2009年北京国安用替补门将程月磊加上40万元,与深圳队交换张磊,这也是2009赛季中超第一次转会市场上北京国安队最后一笔内援引进交易。因为北京国安有杨智把关,所以张磊没有得到出场机会,结果被北京国安挂牌,最终在2010年转会到失去主力门将宋振瑜的长沙金德,成为球队新的主力门将。2011年,张磊转会到重庆力帆成为球队的主力门将。效力了五年期间,前三年打主力,后两年渐为替补。2016年转会到杭州绿城。2017年,张磊从杭州绿城租借到丽江飞虎。","text2":"张磊在哪一年转会到深圳队?","label":1} {"text1":"林 芳正(),日本政治家,出身于山口县下关市,为自民党的参议院议员(4次当选),历任防卫大臣、内阁府特命担当大臣(经济财政担当大臣)、农林水产大臣等要职。在自民党内属于岸田派(宏池会)。1984年3月东京大学法学部毕业,4月三井物产株式会社入社(1989年5月退职),1989年6月Sanden交通株式会社入社(1990年6月退职),1990年7月山口合同煤气株式会社入社(1993年1月退职)。1992年12月任众议院议员林义郎秘书,1993年2月任大藏大臣秘书官,至1993年8月自民党失去政权。1994年1月任众议院议员林义郎政策秘书(~1995年6月)。6月美国哈佛大学肯尼迪行政大学院结业,1995年7月当选参议院议员(第17次通常选举)。1999年10月任大藏政务次官至2000年7月。2001年7月连任参议院议员(第19次通常选举)2004年10月任参议院外交防卫委员长,至2005年11月。2006年9月任安倍内阁府副大臣,至2007年8月27日。2007年7月29日,连任参议院议员(第21次通常选举)。2008年8月任防卫大臣,至9月。2009年3月任参议院政府开发援助等相关特别委员长,7月任内阁府特命担当大臣(经济财政政策担当)至9月自民党失去政权。2012年12月自民党重新执政后,任安倍内阁的农林水产大臣。","text2":"林芳正属于哪个党派?","label":1} {"text1":"周德威(),字镇远,小名阳五,朔州马邑(今山西朔县)人。骁勇而擅骑射,最初事李克用,为帐中骑督。久在云中(今山西大同),胆略出众,据称见烟尘即能判断兵力。干宁中,随李克用攻王行瑜。899年,后梁军围太原时,朱温听说周德威之名,曾下令能生擒周阳五者,可拜为刺史。梁军勇将陈章,号「陈夜叉」,主动请缨,身披红甲,骑白色骏马,李克用劝德威要小心,德威笑曰:“ 陈章好大言耳,安知刺史非臣作邪?”。结果陈章被周德威挥铁檛擒获。天祐三年(906年),与李嗣昭攻取潞州。后梁开平二年(908年),随李存勗解潞州之围,救出李嗣昭,以功授振武节度使、同中书门下平章事。干化三年(913年),进攻幽州(今北京),消灭“大燕”刘守光,授检校侍中、卢龙节度使。贞明三年(917年),坚守幽州,契丹军强攻两百余日竟不能破。贞明四年,周德威父子于胡柳陂(今河南濮阳西)之战中战死。","text2":"被李克用选中后做了什么官?","label":1} {"text1":"王崇愚(),满族,物理学家,中国科学院院士。主要研究领域为计算凝聚态物理及计算材料物理,在金属合金领域作出重大贡献,是中国多尺度研究领域的开拓者。王崇愚1950年秋—1952年夏在北洋大学读书,因为高校院系调整,他1952年夏—1953年秋在清华大学读书,1953年秋—1954年夏,转入北京钢铁学院(今北京科技大学)读书。王崇愚长期在冶金部钢铁研究总院工作,在金属电子缺陷领域作出重要贡献,1993年他当选中国科学院学部委员。1999年进入清华大学物理系担任教授。 是中科院咨询委员会委员,中科院技术科学部常委,中科院国际材料物理中心学术委员会委员,中国金属学会理事。","text2":"他1952年夏—1953年秋在哪所大学读书?","label":1} {"text1":"银河系有一些较小的受到重力约束的星系,它们是银河系次星系群的一部分,这个次集团又是本星系群的一部分。大约有30个小星系已经证实是在银河系的420 千秒差距(140万光年)之内,虽然不是所有的小星系都必定在轨道上。其中,肉眼可见的只有大麦哲伦星系,小麦哲伦星系,自从史前时代从已被观察到。哈勃太空望远镜于2006年的测量表明麦哲伦星系可能移动得太快而不是在绕银河系的轨道上。其中,在已证实是在轨道上的星系中,最大的是人马座矮椭球星系,这是一个直径有20000光年(6100秒差距),或大约是银河系直径的五分之一。银河系的卫星星系包括下列的:人马座矮星系是处于正在被银河系被吸收的过程中,并预计在未来1亿年内穿过它。人马座星流是在极轨道上绕银河系的人马座矮星系的吸出星流。室女座星流是被认为曾经是一个在轨道上围绕银河系的矮星系但已经被银河系的引力完全分散开了的星流。","text2":"所有的小星系都必定在轨道上吗","label":1} {"text1":"瓮盘狸藻节(学名:Utricularia\" sect. \"Calpidisca)为狸藻属下的一节。该节的10个物种皆为小型的陆生食虫植物。其分布于非洲,其中1个物种分布延伸到墨西哥以及亚洲印度。1916年,约翰·亨德利·巴尔哈特将该组作为一个独立的属进行了最初的描述。经过小宫定志在其1973年的分类修订中将其降为狸藻属下的一组。彼得·泰勒在其1986年发表的狸藻属分类学专著《狸藻属——分类学专著》中,将该组继续置于狸藻亚属下。更多现今的种系发生学资料和修订使双瓣狸藻亚属恢复且让本节置于它的范围内。经过进一步的系统发生学数据的支持,在恢复双瓣狸藻亚属的同时,也将瓮盘狸藻组置于其下。","text2":"小宫定志对瓮盘狸藻节是如何划定的?","label":1} {"text1":"伊莱恩是一个澳洲维多利亚的城镇,位于柏拉瑞特和吉朗之间的中部公路之上。根据2006年的人口普查,伊莱恩包括邻近地区合共有677人口。1862年,吉朗至柏拉瑞特的铁路线连接到伊莱恩,并于短时间内开设了火车站(但是今天只剩谷物和载货火车使用这一条铁路线)。当地的邮局于1859年3月1日开始营业,命名为「杜兰山邮局」(Mount Doran Post Office)。其后于1872年更名为「伊莱恩邮局」(Elaine Post Office)。位置离旧邮局较接近火车站的「伊莱恩火车站邮局」(Elaine Railway Station Post Office)于1873年4月14日开始服务。1877年,新邮局取代旧邮局被命名为「伊莱恩邮局」,而旧邮局则恢复「杜兰山邮局」的原称。","text2":"伊莱恩的邮局什么时候开始营业?","label":1} {"text1":"《安阳婴儿》是一部由中国第六代导演王超的电影作品。该片的导演、制片人兼编剧都是王超,他曾经是导演陈凯歌的助手。1996年-2001年曾发表三部短篇小说,一部中篇小说。影片《安阳婴儿》根据王超同名小说改编。在《安阳婴儿》之后,王超又指导了电影《日日夜夜》(2004年)及《江城夏日》(2006年)。影片通篇使用静止的镜头和自然光拍摄,体现了独特的效果。在一个冬天,中国河南安阳的下岗工人肖大刚(孙桂林饰)在路边面条摊上吃饭时,得知摊主拾到一名弃婴。肖大刚发现这个婴儿身上有一张纸条,说收养这个孩子的人可以每个月得到二百元人民币的抚养金,还有一个传呼机号。肖大刚决定收养婴儿。他给传呼机主人打电话,发现婴儿的母亲是来自中国东北的娱乐厅坐台小姐冯艳丽(祝捷饰)。两人在一家清真面馆吃了面条后分手。冯艳丽在娱乐厅和一个黑道老大(岳森谊饰)发生争执失去工作。肖大刚在住家附近街边修理自行车。肖大刚向冯艳丽表明要把孩子还给冯。但两人随后发生性行为之后肖大全提议他继续抚养婴儿,并让冯用他的家从事性交易。两人在不工作的时候一起抱着孩子上街,并且晚上在一起生活。冯艳丽计划开春之后停止从事性工作。这种暂时的平静不久被打破。黑道老大身患血癌不久于人世,找到冯艳丽,要承认婴儿是自己的孩子。冯否认婴儿是他的孩子,但也没有回答是谁的孩子。肖大刚坚持要在旁边听两人的谈判,惹恼了黑道老大,两人发生武力冲突。冲突结果没有明说,但肖大刚被警方逮捕。冯艳丽带着孩子去看守所看望肖大刚。肖大刚说如果他死了,要冯善待这个孩子,因为他是肖大刚的后代。冯点头应允。一个月后,在一次警方扫黄行动中,冯艳丽抱着孩子仓皇奔逃,情急之下把孩子塞给一个路人。躲过警察之后她又跑回来找孩子,被留下的便衣警察抓获遣返。在火车闷罐车厢内,冯艳丽幻觉那个被塞给孩子的路人就是肖大刚。影片《安阳婴儿》在美国第37届芝加哥国际电影节上获得费比西(FIPRESCI AWARD)国际电影评论大奖。 并入选54届戛纳国际电影节“导演双周”单元,收到广大的好评。","text2":"《安阳婴儿》是由谁执导的电影作品?","label":1} {"text1":"保罗·策雷索莱(Paul Cérésole ,)是一位瑞士政治家,瑞士最高法院大法官(1867年-1870年),瑞士联邦委员会委员(1870年-1875年)。他于1870年2月1日当选为联邦委员会委员,任职至1875年12月31日卸任。保罗·策雷索莱是瑞士自由民主党成员。在其联邦委员会委员的任期内,他主要主持领导了以下部门的工作:他的儿子皮埃尔·策雷索莱(Pierre Cérésole)是著名的和平主义者。保罗·策雷索莱于1905年1月7日在瑞士洛桑去世。在瑞士小城沃韦(Vevey)有以他的名字命名的保罗·策雷索莱大道(Avenue Paul-Cérésole)。","text2":"保罗·策雷索莱在哪里去世?","label":1} {"text1":"吴甡(),字鹿友,晚号柴庵。江苏兴化人。明末政治人物。吴甡万历四十一年(1613年)成进士。先后任福建邵武、晋江及山东潍县知县。天启二年,由知县升为御史,追论崔文升、李可灼罪。依附赵南星,成为东林党人,天启七年(1627年)二月因反对魏忠贤被削籍革职,罢官归里。崇祯间复出,任河南、陕西等省巡抚,又曾出任山西巡抚。大学士温体仁告诉吴甡:“流贼癣疥疾,勿忧也。”崇祯四年以吴甡为赈济御史,奉命携带十万两的帑银前去赈济陕北,吴甡描述当时鄜州,“一县之内,十室九空”,延长县“流贼数万,屯聚城下”,清涧县,“荒乱尤甚,县中之民,半化为盗,因之饥馑,人心汹汹……城十里外皆是贼巢”。三边总督杨鹤指出: “非救荒乃救乱也”。崇祯十三年(1640年)以左侍郎协理戌政。崇祯十五年(1642年)官至礼部尚书兼东阁大学士。杨嗣昌死后,督师无人,帝命吴甡上前线督师,晋吴甡为太子少保、户部尚书兼兵部尚书、文渊阁大学士。崇祯十六年(1643),李自成克襄阳、荆州、承天,建立大顺政权。崇祯命吴甡督师湖广。吴甡请拨精兵三万“自金陵赴武昌”,仅凑残兵万余。吴甡因手中无兵,一再找借口拖延,至四月仍未出京。五月初一,张献忠陷汉阳,武昌大震。五月三十日张献忠陷武昌。商毅收复汉阳,吴甡下令黄得功和刘良佐从黄州渡江,协同商毅围剿张献忠。八月,武昌失守,以“故延师期,以致楚省不守,根本震邻”罪,令锦衣卫把吴甡逮捕入京,被革职查办,交法司议罪。南京兵部尚书史可法得知此事,驰书援救,帝不允。十一月,被遣戍云南金齿(今云南保山)。途中得知李自成破北京事,肝胆俱摧。崇祯十七年(1644年)五月福王建弘光,下旨赦还。卒于康熙九年(1670年),《明史》评价:“抑时势实难,非命世才,固罔知攸济也。”有《柴庵疏集》20卷,清代被列为禁书。","text2":"吴甡字什么?","label":1} {"text1":"曼哈顿下城(,或称)是美国纽约市曼哈顿最南端的部分,其最常见的边界是北到14街,西到哈德孙河,东至东河,南到纽约港(也称为上纽约湾)。当提到曼哈顿下城商务区时,通常指北到14街以南约1.5英里,和该岛最南端以北一英里,从哈德孙河附近的钱伯斯街(Chambers Street),东到布鲁克林大桥出入口和立交桥。其他两个主干道有时也被确定为曼哈顿下城的北部边界:钱伯斯街以北大约半英里的运河街(Canal Street),和14街以北大约半英里的23街。曼哈顿下城商务区构成钱伯斯街以南区域的核心,包括金融区—通常称为华尔街,和世界贸易中心遗址。在该岛最南端是炮台公园(Battery Park),在西侧,TriBeCa区横跨钱伯斯街,在街的东端是庞大的曼哈顿市政大楼。纽约市政厅就位于金融区以北。钱伯斯街南面还有炮台公园城规划区域和南街海港历史区域。钱伯斯街和布鲁克林大桥以北、运河街以南是纽约最早的唐人街,许多法院大楼和其他政府办公楼也位于这一区域。下东城跨运河街。运河街以北、14街以南有SoHo、肉类加工区(Meatpacking District)、西村、格林尼治村、小意大利、Nolita和东村。介于14街和23街之间是下切尔西、联合广场、熨斗区(Flatiron District)、Gramercy和大型住宅项目Peter Cooper Village—Stuyvesant Town。曼哈顿下城商务区是全美第四大中央商务区,仅次于曼哈顿中城、芝加哥的卢普区和华盛顿哥伦比亚特区。此前曾经是美国第三大商务区,之所以下降到第四位是因为九一一事件爆发后,该地损失了世界贸易中心16,000,000平方英尺的办公大楼楼板面积,相当于辛辛那提全市办公大楼的总和。但是兴建自由塔兴建高盛财务公司的新总部后,预计该地区将恢复其排名第三位。","text2":"炮台公园位于什么地方?","label":1} {"text1":"李重进(),沧州(今河北省沧州市)人,生于太原,五代时后周禁军统帅之一,太祖郭威第四姊福庆长公主之子。李重进在后晋天福中,入仕为殿直。后汉初年,随舅舅郭威征讨于河中。后周建立的广顺元年(951年),太祖郭威以李重进为内殿直都知、领泗州刺史,女婿张永德为内殿直小底四班都知;又升李重进为小底都指挥使,而以张永德接任内殿直都知。翌年(952年),郭威以李重进为大内都点检兼马步都军头,张永德为小底第一军都指挥使;后又以李重进为殿前都指挥使,张永德为殿前都虞候,掌管殿前亲军。两年后,郭威病危,传位于发妻柴氏的侄子、养子柴荣,临终前特命李重进向柴荣行君臣之礼,以免其觊觎皇位。显德元年(954年),世宗柴荣即位,以姑表兄李重进为侍卫亲军马步军都虞侯,妹夫张永德接任殿前都指挥使,分掌侍卫亲军和殿前亲军。李重进、张永德本以姻亲之故,在数年间不次擢升,但后来都在战争中展现出过人的军事才能。在决定后周生死存亡的高平之战后,李重进以战功加使相衔,升侍卫亲军都指挥使,母福庆长公主追封燕国大长公主;而张永德以战功加检校太傅,授义成军节度使,妻寿安公主进封晋国长公主。北宋建隆元年(960年),宋太祖赵匡胤即位,加李重进为中书令,命令韩令坤代替李重进,将重进移镇至青州(治所在今山东省青州市),李重进拒绝调动,派遣幕僚翟守珣说服李筠起兵抗命,翟守珣却将此事泄露给宋太祖,于是太祖要求翟守珣拖延李重进出兵,以防止李重进与李筠南北呼应。翟守珣回去后,向重进诋毁李筠不足与谋事,重进果然中计,错失良机。李筠四月起兵反宋,六月兵败,自焚死。同年九月李重进起兵,十月,太祖亲征,带领石守信、王审琦、李处耘平叛,十一月,到达扬州城下,即日入城,李重进举家自焚。","text2":"决定后周生死存亡的高平之战后,李重进晋封了什么官职?","label":1} {"text1":"2010年亚洲运动会棒球比赛,于2010年11月13日至11月19日,在广州奥林匹克体育中心运棒球场举行。参赛队伍有日本、南韩、中华台北、中国、中国香港、泰国、蒙古、巴基斯坦等八队。比赛需要打9局,如打成平局,继续比赛,第10局开始采用突破僵局制,一律从无人出局一、二垒有人时开始进行,直至决出胜负。如双方比分相差10分及以上时,7局可结束比赛;双方比分相差15分及以上时,5局可结束比赛。下列所有的比赛时间皆是北京时间(UTC+8)11月13日 12:0011月13日 13:0011月18日 12:0011月18日 18:0011月19日 12:0011月19日 18:00日本面对蒙古和泰国的比赛中,出现了「无安打比赛」,但是两者都因为比分差距在15分以上而在五局提前结束,而且都是投手接力完成。","text2":"如双方比分相差10分及以上时,几局可结束比赛?","label":1} {"text1":"天文航空(Astro Air International Inc.)是一家菲律宾廉价航空公司,基地设于邦板牙省克拉克自由港区的克拉克国际机场(Clark International Airport,又称「马嘉柏臬国际机场」,Diosdado Macapagal International Airport,DMIA)。天文航空的前身为马尼拉精神航空(Spirit of Manila Airlines),主要航线皆集中于东南亚地区。2011年冬季,该公司由于内部进行重组和融资,因此航班处于停飞状态。该公司已于2013年5月1日恢复台北(台湾桃园国际机场)到长滩岛(卡利博国际机场)的航线 。","text2":"天文航空是哪个国家的航空公司?","label":1} {"text1":"米诺地尔()是一种钾通道开放药,这类药物在降压时常伴有反射性心动过速和心输出量的增加。对于血管扩张的作用具有选择性,见于冠状动脉,胃肠道血管和脑血管,而不扩张肾和皮肤血管。它还可以减低或停止掉发并且促进毛发再生。现在一般的门诊病人不需要处方就可以取得,用于。米诺地尔本来是用于控制高血压。后来偶然发现有一个有趣的副作用,就是扭转或减慢秃头过程。Upjohn推出了含2%的米诺地尔的外用药液,用来治疗秃头及脱发。该药液在美国及加拿大以Rogaine名称销售,在欧洲及亚太区则使用Regaine名称,在台湾译为「落建」。5%浓度的药液专供男性使用,而2%浓度的药液则适合女性使用。在台湾贩售的含5%浓度的米诺地尔成分生发药液产品如:落建生发系列的落健生发液、萌发生发液等。米诺地尔的专利保护期在1996年2月11日已经到期。5%泡沫配方的米诺地尔,跟液体配方一样有效。米诺地尔促进毛发生长的机转目前还未完全了解。此药物的结构式中有一氧化氮基团且可能做为一氧化氮的致效剂。它同样是,造成细胞膜的去极化。对于大面积的掉发,米诺地尔就比较没有效果。另外,对于年轻男性有五年以下的掉发困扰,它有非常显著的效果。目前,米诺地尔只用于的掉发。它同样也是个血管扩张剂。理论上,借由扩张血管及开通钾离子通道,使更多的氧气及养分到毛囊,会促使毛囊从休止期到生长期。常见的副作用有对眼睛的烧灼感及刺激性,治疗部位搔痒、红肿及刺激性,非预期位置的毛发生长。如果使用者有以下的严重副作用,立即寻求医疗照护:严重的过敏反应(红疹、荨麻疹、搔痒、呼吸困难、胸闷等);胸痛;晕眩;火烧心;立即且非预期的体重增加;手或脚的胀大。酒精和丙二醇可能会使头皮干燥,造成头皮屑和。有些配方改用奈米液滴为的就是减少因为酒精和丙二醇所造成的接触性皮肤炎。米诺地尔对猫来说是致命的。","text2":"后来米诺地尔发现了具有怎样的副作用?","label":1} {"text1":"叶齿鳄(学名:\"Phyllodontosuchus\")是种原始鳄形类动物,属于喙头鳄亚目。目前已发现一个颅骨、下颌化石,发现于中国云南省,年代为侏罗纪晚期。叶齿鳄的牙齿形状为叶状,类似草食性恐龙的牙齿,显示牠们是草食性动物,与其他鳄形类动物不同。叶齿鳄的正模标本(编号BVP568-L12)是个被压碎的颅骨与下颌,发现于中国云南省的下禄丰组,年代属于侏罗纪晚期的锡内穆阶。同一地点还发现一个摩尔根兽的颅骨。叶齿鳄的颅骨长7.14公分,保存状态不佳。这个标本最出被认为是个早期鸟臀目恐龙的化石。颅骨的骨头间没有缝合处,显示这个小型动物已经是个成年的个体。叶齿鳄是在2000年由Jerald Harris等人所叙述、命名,模式种是禄丰叶齿鳄(\"P. lufengensis\")。属名意为「叶状牙齿的鳄鱼」;种名则是以发现化石的禄丰组为名。叶齿鳄的上颌两侧各有17或18颗牙齿。最前5、6颗牙齿呈圆锥状,微弯,而后方12颗牙齿呈叶状、铲状。牙齿的形状并不一致,显示牠们是异齿型动物。叶齿鳄的牙齿型态类似某些原蜥脚类、或早期鸟臀目恐龙。但是,叶齿鳄的牙齿缺乏棱脊,与早期鸟臀目恐龙(例如赖索托龙)不同;叶齿鳄的牙齿也缺乏锯齿状边缘,与上述草食性恐龙不同。叶齿鳄也缺乏鸟臀目恐龙的前齿骨。科学家根据上述特征,认为叶齿鳄最类似喙头鳄类。目前已发现数种异齿型的喙头鳄类,例如:裂头鳄、黄昏鳄、\"Pedeticosaurus\"、喙头鳄。这些喙头鳄类的颌部牙齿有类似的型态,前方皆为尖状、微弯的牙齿,中间的牙齿略尖,而后段是接近叶状的牙齿。另外,有数种后期的小型鳄形类也具有类似异齿型牙齿,例如:贫齿鳄、喀迈拉鳄、马拉威鳄。科学家认为上述的鳄形类动物可能是草食性动物,而非一般鳄形类的肉食性、食鱼性食性。如果叶齿鳄是草食性动物,牠们可能咬碎植物直接吞下,而不经由研磨的阶段。","text2":"叶齿鳄的最前5、6颗牙齿有什么特征?","label":1} {"text1":"卡尔-奥古斯特·威廉·恩斯特·弗里德里希·格奥尔格·约翰·阿尔布雷希特(\"Carl-August\" Wilhelm Ernst Friedrich Georg Johann Albrecht,),萨克森-魏玛-艾森纳赫大公储,1923年起为萨克森-魏玛-艾森纳赫大公家族族长。卡尔-奥古斯特是萨克森-魏玛-艾森纳赫末代大公威廉·恩斯特与夫人萨克森-迈宁根的费奥多拉的儿子,1912年7月28日生于巴伐利亚王国上弗兰肯行政区的威廉斯塔尔()。一出生便成为萨克森-魏玛-艾森纳赫的大公储,同时还是荷兰王位第三顺位继承人。1918年,德国爆发革命。卡尔-奥古斯特的父亲在革命中于被推翻,卡尔-奥古斯特失去了继位大公的机会。随着1922年荷兰修宪,他也失去了继承荷兰王位的机会。卡尔-奥古斯特的父亲于1923年去世,他继承父亲成为萨克森-魏玛-艾森纳赫大公家族族长,韦廷家族首领。若德国仍是君主制国家,他将是萨克森-魏玛-艾森纳赫大公卡尔·奥古斯特二世。卡尔-奥古斯特于1988年10月14日在巴登-符腾堡州博登湖畔的希恩嫩去世,终年76岁。他的长子米夏埃尔-本尼迪克特继位成为家族族长。卡尔-奥古斯特1944年10月5日在瓦尔特堡与汪根海姆-温特尔施泰因女男爵伊丽莎白(Elisabeth Freiin von Wangenheim-Winterstein)结婚,有一子二女:","text2":"谁继位成为家族族长?","label":1} {"text1":"广州府,明清时广东省的府。清代隶广肇罗道。元朝时为广州路,明朝洪武十一年(1378年),改广州府。下领一州:连州(领阳山县、连山县),十五县:南海县、番禺县、顺德县、东莞县、新安县、三水县、增城县、龙门县、香山县、新会县、新宁县、从化县、清远县、阳山县、连山县。治所在南海县、番禺县。清朝时,两广总督旧驻肇庆,乾隆十一年徙。光绪二十四年裁巡抚,寻复。三十一年,仍与粤海关监督、粮道同裁。布政、提学、提法、盐运四司,巡警、劝业二道,广州将军,满洲、汉军副都统,广东水师提督驻。明领县十三。康熙中增置花县。领县十四:南海县、番禺县、顺德县、东莞县、从化县、龙门县、新宁县、增城县、香山县、新会县、三水县、清远县、新安县、花县。1912年中华民国成立后废州府,成立广东军政府。","text2":"广州府是明清时期哪个省的府?","label":1} {"text1":"周以真(英文名Jeannette M. Wing,),美国计算机科学家。卡内基梅隆大学教授。美国国家自然基金会计算与信息科学工程部助理部长。ACM和IEEE会士。她的主要研究领域是形式化方法、可信计算、分布式系统、编程语言等。1993年她与图灵奖得主芭芭拉·利斯科夫合作,提出了著名的Liskov代换原则,是面向对象基本原则之一。1979年6月在麻省理工学院获得学士和硕士学位,导师中有图灵奖得主Ronald Rivest。1983年获得该校的博士学位。1983—1985年,在南加州大学任助理教授。1985年起,任教于卡内基梅隆大学。2004—2007年间,曾担任该校计算机系主任。2006年3月,在美国计算机权威期刊《Communications of the ACM》杂志上给出,并定义的计算思维(Computational Thinking)。认为:计算思维是运用计算机科学的基础概念进行问题求解、系统设计、以及人类行为理解等涵盖计算机科学之广度的一系列思维活动。周以真教授曾担任卡内基—梅隆大学计算机学院院长、美国国家科学院计算机科学与通讯部门主席、美国国防部高级研究计划署(DARPA)信息科学与技术委员会、美国国家科学基金科学顾问委员会以及Sloan 研究基金程序委员会的成员。2007年她出任美国国家科学基金会(NSF)计算机与信息科学与工程部(CISE)副部长,负责管理NSF在信息科学和计算机研究领域的基金。CISE每年的预算高达5.27亿美元,占整个美国联邦政府对计算机科学研究资助经费的86%。","text2":"周以真是哪个大学的教授?","label":1} {"text1":"《最后一个人》是一部由鲍里斯·萨迦尔导演的科幻电影,于1971年上映,影片由查尔顿·赫斯顿主演,以李察·麦森的小说《我是传奇》为蓝本。电影剧本为约翰·威廉和乔伊斯·卡灵顿之作。影片用彩色印片法和单声道录制,共98分钟。故事首先被拍成由文森特·普林斯主演的《地球上最后一个人》。第三部曲是威尔·史密斯主演的《我是传奇》(2007年)。故事发生在1977年,在中苏生物武器战争之后两年,世界上几乎所有人都灭绝了,空气中散播着瘟疫。幸存者中包括美军中校Neville,M.D.(查尔顿·赫斯顿),他给自己注射了疫苗,因此对瘟疫免疫。在洛杉矶, 几百个白化病人也幸存了下来,他们创立了一个名叫“家庭”的组织,但是他们变成了疯狂,暴力,昼伏夜出,害怕光线的变异人。“家庭”认为科学是造成这个悲剧的主要原因,所以应该消灭一切科学的迹象,包括Neville。他们自己也不用什么高科技的东西,穿着打扮效法中世纪,最高科技的东西就是手电筒和弹弓车(他们也不使用枪)。Neville决心消灭这个邪恶的组织,白天他到处搜索食品衣物,捕杀“家庭”成员;晚上住在一个配备军火库的公寓,旁边装了很多灯,所以“家庭”成员无法靠近,但是他们却在楼下大山大叫,鬼哭狼嚎地烦扰neville, 甚至使用弹弓车投掷火球。Neville有一天去一家酿酒厂时遭到“家庭”成员埋伏而被拿获,当他们正要烧死他时,一个年轻男子和一个黑人女子救了他,这个女子名叫lisa,他曾在一家衣店遇见她。lisa带他来到她们家,neville发现居然还有人幸存。但是这些幸存者也感染了瘟疫,只是病情还没有恶化,他想既然自己免疫的话,用自己的血浆制造疫苗说不定可以治好这些人。他们还计划如果都康复之后可以一起去野外开始新生活。于是他开始制造疫苗。但是其中一个叫richie的人由于同情“家庭”,而拿着疫苗给他们,但是他们却不领情,反而杀害了她。这时, Lisa病情恶化,于是她帮助“家庭”袭击neville的住宅, 在战斗中neville被“家庭”领袖飞茅杀死。","text2":"《最后一个人》是根据哪个人的小说改编的?","label":1} {"text1":"斑鳍彭纳石首鱼(学名:Pennahia pawak),又称斑鳍白姑鱼俗名春子、帕头,为辐鳍鱼纲鲈形目石首鱼科的其中一个种。本鱼分布在西太平洋区,包括泰国、台湾、中国东海、南海、香港、越南、印尼等海域。水深3至50公尺。本鱼头呈钝尖形,口裂大,端位,倾斜,上颌稍长于下颌,口闭时,上颌外列齿外露。上颌外列齿扩大为犬齿,内列细小绒毛状齿。体侧白胸鳍基以上为褐紫色,会有紫色光泽反射,胸鳍基以下为银白色,自第七到第十根硬棘部份具有一黑色班,软条部中间1\/3鳍高有一纵走白色带,另外上下1\/3为浅褐色。鳃腔黑色,口腔白色。背鳍硬棘11枚、背鳍软条25至28枚;臀鳍硬棘2枚、臀鳍软条7至8枚。体长可达22公分。本鱼为近海中下层鱼类,主要栖息在沙泥底质海域。肉食性,以甲壳类、小鱼为食。食用鱼,肉质佳,适合油炸和清蒸食用。为台湾地区重要的底栖鱼获物。","text2":"这种鱼有着怎样的生活习性?","label":1} {"text1":"《丁丁与丛林战士》(法语:\" Tintin et les Picaros \")是《丁丁历险记》的第23部作品,也是《丁丁历险记》中最后一部完整作品。作者是比利时漫画家埃尔热。于1976年初版。这部作品与《丁丁在苏联》和《丁丁在刚果》并为丁丁历险记中最受争议的作品。但与前两部不同,这次的争议主要是在艺术上而非政治上。为营救身陷囹圄的毕安卡等好友,丁丁一行流落森林,遇上阿卡扎尔将军。丁丁决定冒险一拼──协助阿卡扎尔和他的丛林战士,推翻塔比奥卡政权。不料丛林战士却中了塔比奥卡的奸计,溃不成军,弄得阿卡扎尔一筹莫展。丁丁凭著图纳思的新发明,戏剧性地令军队重振士气、直捣黄龙。","text2":"《丁丁与丛林战士》是《丁丁历险记》系列的第几部?","label":1} {"text1":"天主教阿维尼翁总教区是罗马天主教在法国南部设立的一个总教区,范围相当于普罗旺斯-阿尔卑斯-蓝色海岸大区的沃克吕兹省。现任总主教Jean-Pierre Marie Cattenoz。阿维尼翁教区成立于4世纪,1475年升格为总教区,附属教区有卡庞特拉教区、Vaison教区和Cavaillon教区。1801年这些教区合并,并降格为阿维尼翁教区,附属于天主教艾克斯总教区。1822年恢复阿维尼翁总教区,附属教区为维维耶教区、瓦朗斯教区、尼姆教区和蒙彼利埃教区。2002年12月16号,阿维尼翁总教区的教省解散,并被降格为马赛总教区的教省附属教区,原先的附属教区则分到其他教省。2009年才重新获得总教区的头衔,不依然从属于马赛总教区。","text2":"阿维尼翁教区成立于什么时候?","label":1} {"text1":"钱天白(),中科院计算机网络信息中心客座研究员,CNNIC工作委员会副主任委员,国务院信息办安全专家组成员。他曾为中国的互联网建立作出过贡献。钱天白1945年出生在无锡的一个书香门第之家,1963年他以平均分93分的高分进入清华大学无线电系无线电技术专业3班。毕业后因政治成分原因曾被分配到继电1990年钱天白加入CANET项目。1990年10月10日,王运丰与措恩在卡尔斯鲁厄大学商讨,王运丰定下“CN”两个字母作为域名。1990年11月28日,措恩在德国卡尔斯鲁厄大学内建立.cn顶级域名服务器,在SRI-NIC(Stanford Research Institute’s Network Information Center)注册登记中国的顶级域名.CN及开通.CN的国际电子邮件服务,将当时负责行政联络的钱天白教授登记为管理联系人员。1992年底中科院院网(CASNET)建成,CN服务器移入该网络。1997年5月30日,国务院信息化工作领导小组办公室发布《中国互联网络域名注册暂行管理办法》,授权中国科学院成立中国互联网络信息中心(CNNIC),该工作由钱天白主持。1998年5月,钱天白心脏病突发去世,享年53岁。1987年9月,CANET在北京计算机应用技术研究所内正式建成中国第一个国际互联网电子邮件节点,曾有说法称钱天白于9月14日发出了中国第一封电子邮件:“Across the Great Wall we can reach every corner in the world.(越过长城,走向世界)”。但据时任中国兵器工业计算机研究所所长的李澄炯与卡尔斯鲁大学教授维尔纳·措恩回忆,该邮件为李澄炯和当时的机电部科学研究院副院长王运丰连同措恩合作发出,1987年钱天白仍在美国美国CST公司学习。另外在他去世后,媒体报道的文章将一些国内互联网早期发展的功绩归在他身上,但从现有的文献参考看,很多工作应为互联网早期工作者的共同成果。而他更多参与一些联络及行政管理方面的工作。他身体力行地大量使用互联网电子邮件作为工作及生活的辅助手段,并不断向身边的人介绍互联网。为国内的互联网发展做出巨大贡献。他对兄弟姐妹十分关照。他妻子于婚后不久发现患上重病,他20多年不离不弃照顾妻子直至其去世,之后又尽力照顾其父母。","text2":"什么疾病导致钱天白去世?","label":1} {"text1":"《麻姑仙坛记》(全称《有唐抚州南城县麻姑山仙坛记》),俗谓《麻姑帖》,是唐朝书法家颜真卿,年六十三时所书,号称真卿楷书代表之作,有人称此帖是天下第一楷书,但清代考据学者遍寻众书,不见此看法,普遍认为九成宫醴泉铭乃正书第一。大历六年四月,真卿在抚州刺史任上,因仕途受挫,故而心向道教与佛教,游览南城县麻姑山,撰文记述麻姑得道成仙之事,刻文在江西建昌府南城县西南二十二里山顶。麻姑是道教著名的女仙,在瑶池服侍西王母。传此帖本有大、中、小三种,因原石均佚,故难寻佳本。传世宋刻本就有张之洞、何子贞、端方、罗振玉藏本和戴熙、赵子谦跋本等数种。马子云《碑帖鉴定》称:「闻何绍基(子贞)藏宋拓本,后为颜韵伯藏,现不知为何人所藏。又称原石为雷火所破,元建昌知府梁伯达重建。然元刻本今也罕见。以后又有唐晏云本,忠义堂何氏本、黄氏本,惟唐氏刻本最善,何氏本最劣。《校碑随笔》云:上海原石石印本,即罗振玉所藏,有张廷济跋,现也不知存于何处。」","text2":"清代考据学者遍寻众书普遍认为什么是正书第一?","label":1} {"text1":"海蟑螂(学名:\"Ligiidae\")是一种常见的岸栖甲壳类,鲜少在海中活动,但遭遇危险会逃入海中,以生物尸体及有机碎屑为食,为食腐动物。有七对步足,及一双复眼,因貌似蟑螂而得名。特征与习性 :体长1.5-2.5cm左右。除了头外,身体有十三节,其中胸部七节,腹部六节。有七对脚分布在胸部的七节。尾部腹面是呼吸器官,有 12块薄膜。头上有一对大眼,一对触须和嘴。在水中或陆地都靠保持湿湿的尾部腹面薄膜呼吸。海蟑螂有抱卵的习性。 每年11月为抱卵孵化最多时期。早上或傍晚,就从栖身处向有食物的地方列队前进。 海蟑螂是生活在高潮带的生物。冬天常躲在岩石缝里。由于海蟑螂的主食为海藻类,所以沿海居民并不将其与肮脏做连结,也常用海蟑螂作为钓饵使用。","text2":"海蟑螂是什么类型的海洋生物?","label":1} {"text1":"《即刻毁灭》(\")是一部由科恩兄弟导演的黑色喜剧电影,于2008年9月12日在美国上映,由约翰·马克维奇、乔治·克隆尼、法兰西丝·麦朵曼、布莱德·彼特和蒂妲·丝云顿主演。本片曾于2008年8月27日在第65届威尼斯国际电影节首映 。根据导演科恩兄弟表示,剧情主题是关于美国中央情报局(CIA)、华盛顿特区的健身教练,以及网路交友约会。本片是这对兄弟导演再以《险路勿近》(\")赢得奥斯卡后的第一部新作,演员之一的蒂妲·丝云顿形容本片「...像是一种很夸张的荒谬喜剧。所有演员都很夸张,真的很夸张。电影充满了荒谬之处。比《险路勿近》要轻松多了。」奥斯朋·考克斯(,约翰·马克维奇 饰)是一名中央情报局分析师,他在因为酗酒问题被降职后辞去工作。他决定开始撰写在中情局期间的回忆录。他的妻子凯蒂·考克斯(,蒂妲·丝云顿 饰)想要和奥斯朋离婚,在与离婚律师开会时,她从奥斯朋的电脑中拷贝了所有的个人财务资料,同时也包括了他正撰写到一半的回忆录。存有这些资料的光碟辗转来到「硬汉健身房」,在此工作的查德·菲尔德海墨(,布莱德·彼特 饰)发现了这片光碟,并企图要勒索考克斯,","text2":"《即刻毁灭》这部电影是由谁导演的?","label":1} {"text1":"旁系群在支序分类学是指一个演化支上某个单系群最亲近的分类群,又称为姐妹群。如下图所示:单系群A是单系群B的旁系群,同时B也是A的旁系群,两者从一个最近共同祖先演化而来,而关系较远的C就是它们的外类群。A和B也可以组成一个单系群单元D,此时这个单元D就是C的旁系群,反之亦然……,以此类推,用以概括更远的旁系群。在支序分类学的定义上,A、B、C可以是生物样本、物种或分类单元。在描述物种时,有时会以“姐妹种”()来称呼。在种系发生学的亲缘关系研究中使用“旁系群”这个概念时,一般只考虑研究中所列举的样本,例如:在现存物种当中,鳄鱼是鸟类最亲近的旁系群,但这仅仅是对现生种而言;如果将已灭绝的种类算进来,鸟类作为恐龙的直系后代,鸟类和鳄鱼之间还隔着许许多多的已灭绝的旁系近亲。所以,需留意“旁系群”这个概念只是一个相对的概念,仅仅用来描述研究中所列出的类群、物种或样本之间相对的亲疏远近关系。","text2":"在支序分类学的定义上,A、B、C可以是什么?","label":1} {"text1":"《碧血剑》()是香港电视广播有限公司与中国中央电视台旗下的中国国际电视总公司合作摄制的古装武侠电视剧,由林家栋、江华、佘诗曼及吴美珩领衔主演,以及由李添胜担任监制。此剧根据金庸所著的武侠小说《碧血剑》为基础,再把小说改编剧集,然后进行拍摄制作。此剧由无线电视继1985年版《碧血剑》之后,无线电视为拍摄制作第二部同名剧集。此剧在无线电视翡翠台首播前的电视预告片及宣传片中,为无线电视所选播的33周年台庆剧。电视广播(国际)有限公司授权台湾弘音多媒体科技股份有限公司于2008年推出发行了《碧血剑》DVD影碟零售版本,此影碟收录全套35集,共7只碟,设有粤语及国语发音版本并配上非隐藏繁体中文字幕。","text2":"《碧血剑》DVD影碟有多少集,多少碟片?","label":1} {"text1":"尖鳍魣(学名:),又称尖鳍金梭鱼,俗名针梭、竹梭,为辐鳍鱼纲鲈形目鲭亚目金梭鱼科的其中一种。本鱼分布于印度太平洋区,包括南非、东非、红海、留尼旺、马达加斯加、塞席尔群岛、印度、日本、台湾、菲律宾、越南、香港、澳洲、马绍尔群岛、马里亚纳群岛、纽西兰、诺克福岛、库克群岛、夏威夷群岛、法属波里尼西亚等海域。该物种的模式产地在印度。水深20至50公尺。本鱼体延长呈鱼雷状,横切面几近圆柱形,体呈银色,背部为银蓝色,侧线鳞列数110枚以上,延长的鳃耙数1,腹鳍起点较背鳍起点的垂直线为后,侧线下具一细纵带,上颔骨未达眼前缘,下颔骨尖端有一肉质突起,尾鳍分叉,背鳍硬棘6枚;背鳍软条8至9枚;臀鳍硬棘2枚;臀鳍软条8枚,体长可达80公分。本鱼栖息在近海沿岸的礁石区、潟湖或内湾,喜成群游动追捕猎物,属肉食性,以各种鱼类幼鱼、沙丁鱼、头足类为食。为鲜美的食用鱼,适合煎、烤食用。","text2":"尖鳍魣的模式产地是哪里?","label":1} {"text1":"茅湖山观测台位于香港西贡区将军澳茅湖山上,行山人士及街坊称其为茅湖废堡,确实建筑年份不可考,但从现有历史文献中推算,建筑物早于1898年英国租借新界前已建成,估计为清朝佛堂洲海关的观测台,作观测佛堂门航道之用。基于其独特性,观测台已被古物古迹办事处于2009年12月18日列为一级历史建筑。茅湖山观测台与佛堂洲海关有密切联系,观测台能监察整个佛堂洲及其海域船只出入的情况,估计当时观测台的官员透过信号灯,又或是透过烟火,与佛堂洲总部及海关船只联系。现时茅湖山的遗址包括一座圆形石砌塔楼的下半部,以及一座长方型的乡村式石屋。两栋建筑物皆由深灰色的火山岩建造,估计由当地附近开采而来。圆形塔楼应为一座瞭望塔,观测人员能透过半圆形的窗户毫无阻碍地观测佛堂门航道。该塔楼原有两层高,现时第二层已倒塌,现存一条石楼梯,通到已倒塌的二楼。历史照片显示塔楼原有一个类似中式帽子的拱顶,现时塔顶部分已不存在,原因不明。所有门及窗户已散失,只剩下石制窗框和门框。建筑物塌下的碎片早已被区议会移走,小部分则用作建造休憩长椅。塔后的单层建筑物估计为驻守该建筑官员的总部或住所,以本地乡村村屋模式设计,与圆形塔楼的风格极不相同。现时该建筑物的天花已塌下。圆形塔楼的设计令人联想起古时欧洲防御海岸用的圆形石堡或城堡,在香港极为罕见,类似的圆形塔楼建筑仅见于几座灯塔及尖沙咀前香港水警总部的报时塔,此外昂船洲军营内也有两座瞭望塔,但它们是八角形的建筑而不是圆形;新界不少村落亦设有瞭望塔,但它们多以砖建成,形状则多为正方形。香港审计署在2013年3月28日发表的第六十号报告书「古迹及历史建筑物的保育」中指出,截至2013年2月,该项一级历史建筑长期缺乏打理。由于缺乏妥善的维修保养,引致破旧失修,加上四周并无围封,可能会对游人安全带来风险。2013年3月,地政总署告知审计署,地政总署会联络古物古迹办事处,为安全起见把调景岭茅湖山观测台围封,作为过渡措施,以待当局定出计划,予以善用。","text2":"圆形塔楼应为一座什么塔?","label":1} {"text1":"《沙乡年鉴》或译《沙郡年记》是由美国的生态学家和奥尔多·利奥波德写的一本非小说类书籍(1949年)。它描述了利奥波德在威斯康辛州索克县的家周围的土地和他关于发展一种“土地伦理”的思考,在利奥波德因心脏病发作而死后一年,他的儿子把它整理出版了。这本短文合集被视为美国中划时代的书。该书印刷超过两百万册,已被译为九种语言。它活跃和改变了环境运动,并促进了作为一门科学的生态学的广泛影响。《沙乡年鉴》是自然历史、以文写景与哲学的结合。也许最使它著名、且定义了他的土地伦理的是下列引文:“当一个事物有助于保护生物共同体的和谐、稳定和美丽的时候,它就是正确的,当它走向反面时,就是错误的。”营养级的概念是“像山那样思考”一节提出的,在其中利奥波德意识到,杀死一只掠食动物——狼,会对生态系统的其余部分产生重大的牵连影响。在最初的版本中,该书是以真正的“沙乡年鉴”作为开端的,它分为十二节,每节对应一个月。其中有有关植物区系和动物区系对季节作出的反应的轶事与观测,此外还提及了保护的主题。书的第二部分“随笔——这儿和那儿”探讨加拿大、墨西哥和美国的其他几处荒野,以及保护(或保护的缺乏)是如何影响它们的。全书以“土地伦理”结尾,其中利奥波德在“生态学意识”一节中深入探究了资源保护。他写道:“资源保护是人和土地之间和谐一致的一种表现。”利奥波德觉得,通常认为需要更多的教育,然而其分量和内容值得讨论。他相信,土地不是一个被支配的日用品,相反地,人为了不破坏它,必须与地球有相互的尊重。他还提出了这样的思想:人若无野性之处可供徜徉则永无自由。利奥波德之家——于1978年入选美国国家史迹名册(U.S. National Register of Historic Places)。","text2":"《沙乡年鉴》是什么类型的书籍?","label":1} {"text1":"《科学战队炸药人》(原题:)是日本东映公司在1983年制作的「超级战队系列」第7部特摄作品。在日播放期间为1983年2月5日-1984年1月28日,全51集。远古时代,坠落于地球上的陨石中残存的生命物质,在地底下经过长年的进化成了有尾人一族-邪进化帝国。因尾巴的数量代表其身分的象征的特有文化,将利用它们超乎人类想像的科学力征服地上的人类,以让日本各地火山喷发为第一步,开始了侵略地上世界的行动。然而,及早察觉到这件事的科学家·梦野久太郎,集合了五名智慧与体力都相当优秀的年轻科学家到自己所营运的发明中心来,也正式宣布科学战队的成军。五人著装了梦野博士开发的战斗强化服后,成了炸药人并衔命全力阻止邪进化帝国侵略的邪恶野心。第9回后的播出时间皆缩短至25分钟。梦野久太郎于远古时代坠落到地球上的陨石中生命物质,长久以来寄居于地底进行著自主进化,并成为了能自由操纵火山的地底支配者「有尾人一族」们的帝国。属于卵生,故于蛋中出生,会将尾数加于名字,且尾数越多代表在帝国中越高。十分藐视没有尾巴的人类(地上人),并视为「下等生物」,在无法透过实验让人类长出尾巴后,便乘著有如怪物一般外观的巨大攻击要塞格兰吉兹摩,向地表上的人类展开侵略行动。亚顿卡亚洁诺比亚梅基度奇梅拉进化兽机械进化兽大爆炸光束小尾兵格兰基兹摩基兹摩斯奇托大木星 DyJupiter炸药马赫 Dyna Mach炸药战车 Dyna Mobile炸药拖车 Dyna Garry炸药猎鹰 Dyna Falcon炸药吉普车 Dyna Machine第一话登场。当炸药红喊出『合体! Grand Slam!』后,由炸药马赫、炸药战车、炸药拖车三体合而为一的巨大机器人。由梦野博士开发制造。于剧场版『199英雄大决战』中登场。武装招式必杀技以下时间以当地时间(日本时间)为准本作品的战队服装曾经被台湾自制特摄节目《太空战士》作为参考。片头曲片尾曲插曲","text2":"《科学战队炸药人》一共多少级?","label":1} {"text1":"董景昌(),原籍山东,台湾著名针灸师,是董氏奇穴的创始者。董景昌早年从军,随著国民政府撤退至台湾。在台湾传授他家传的针法,因为由他传授的穴位并不在传统内经所记载的范围内,但疗效显著,因此被称为董氏奇穴。董景昌自成一套独特的经络思想,与传统的十四经络循行路线不同,针灸的穴位配伍极为简明,在放血治疗方面特别擅长,他门下弟子甚多,在台湾中医界极负盛名。但是他并不敌视传统技术,经常鼓励弟子去学习传统十四经针灸手法,常推许台湾传统古方派名医孙培荣,临床时也常以十四经穴道搭配董氏奇穴而发挥奇效。他的弟子有杨维杰、巴顿 (Dr. Dachen Palden)、赖金雄、满萱春、胡文智等人。著作有《董氏针灸正经奇穴学》。","text2":"董景昌对待传统技术的态度是怎么样的?","label":1} {"text1":"刘雪庵(),笔名有晏如、吴青、苏崖。重庆铜梁人,作曲家、音乐教育家。民国时代流行歌曲《何日君再来》、《长城谣》、艺术歌曲《红豆词》的曲作者,也是中华民国空军军歌的曲作者。1905年11月12日生于重庆铜梁。1930年考入国立音乐专科学校,1934年毕业,在中央航空学校任教,后在上海音乐艺文社《音乐杂志》作编辑。抗战胜利后在苏州社教学院任教。1949年留在大陆,先后在苏南文化教育学院、江苏师范学院、华东师范大学、北京艺术师范学院、中央音乐学院任教。1957年因《何日君再来》歌曲被打成右派,被剥夺讲课资格,任图书管理员。1969年被押送河北军粮城农场劳改。1975年因几乎双目失明,失去劳动能力,被遣送回北京家中。1979年被口头平反。然而书面结论直到1982年才交给他本人,结论中仍将《何日君再来》与《红豆词》定为黄色歌曲。1985年3月15日病逝于北京医学院第一附属医院。上海音乐学院的老师这样描述刘雪庵的晚年:「他已被打得失明,每天坐在一张特制的椅子上,椅子挖了个洞,下面放个马桶,椅子中间架个扶手,用一根木棍横挡著,以免他摔下来,扶手上挂著几个馒头,让老人家饿了可以吃。」","text2":"刘雪庵1930年考入哪个学校?","label":1} {"text1":"金勤()是一位台湾演员。在国立台北艺术大学修得教育课程后,任教于公立学校,并于国立台北艺术大学教授电影表演课程。金勤小学三年级时曾参与台视戏剧演出,并陆续在何平执导的《感恩岁月》和金鳌勋执导的《过河小卒》中亮相。
国中毕业后,金勤以男生第一名的成绩考入国光艺校,之后又进入国立台北艺术大学戏剧系、剧场艺术研究所就读。
金勤在2002年因出演公视电视剧《孽子》里的小玉一角而受到注目。金勤陆续演出了电影《国士无双》、《十七岁的天空》、偶像剧《爱就宅一起》、《旋风管家》等,皆获得好评。
在演出的同时,金勤在国立台北艺术大学取得了教师资格,并逐渐淡出舞台。自2008年起,金勤开始担任电影与电视剧的表演指导,包含杨雅喆导演的《囧男孩》、戴立忍导演的《不能没有你》、钟孟宏导演的《第四张画》、林育贤导演的《翻滚吧!阿信》、Rocky Jo导演的《恋爱恐慌症》等作品。金勤并成为演艺圈许多演员及新人的表演老师,目前所知指导过的学生有林依晨、陈柏霖、王阳明、周汤豪、黄靖伦、大庆、五月天的怪兽、[ 钱佑升]等。金勤个人最新演出作品为台湾八大电视台与日本合拍、耗资6千万的台湾偶像剧,改编自日本漫画家仲村佳树的畅销同名漫画作品《华丽的挑战》与Super Junior的崔始源、李东海、及台湾新生代演技派女星陈意涵共同演出。","text2":"金勤从何时开始担任电影与电视剧的表演指导?","label":1} {"text1":"在物理学中,费米气体(Fermi gas),又称为自由电子气体(free electron gas)、费米原子气体,是一个量子统计力学中的理想模型,指的是一群不相互作用的费米子。费米气体是理想气体的量子力学版本。在金属内的电子、在半导体内的电子或在中子星里的中子,都可以被视为近似于费米气体。处于热力平衡的费米气体里,费米子的能量分布,是由它们的数目密度(number density)、温度、与尚存在能量量子态集合,依照费米-狄拉克统计的方程式而表征。泡利不相容原理阐明,不允许两个或两个以上的费米子占用同一个量子态。因此,在绝对零度,费米气体的总能量大于费米子数量与单独粒子基态能量的乘积,并且,费米气体的压力,称为「简并压力」,不等于零。这与经典理想气体的现象有很明显的不同。简并压力使得中子星或白矮星能够抵抗万有引力的压缩,因而得到稳定平衡,不致向内爆塌。在低温下,玻色原子气体可以形成玻色-爱因斯坦凝聚(Bose-Einstein condensation, BEC),这是由爱因斯坦在1925年的理论而预言的。费米子由于泡利不相容原理,不能形成BEC。但可通过Feshbach共振,利用磁场调节费米原子间的相互作用,使费米子配对转变成玻色型粒子而形成BEC。由于前述定义忽略了粒子与粒子之间的相互作用,费米气体问题约化为研究一群独立的费米子的物理行为的问题。这问题本身相当容易解析。一些更深奥,更进阶,更精密的理论,牵涉到粒子与粒子之间的互相作用的理论(像费米液体理论或相互作用的微扰理论),时常会以费米气体问题为研究的开端。","text2":"简并压力有什么作用?","label":1} {"text1":",是1972年香港邵氏公司出品,由张彻及鲍学礼联合执导,陈观泰主演之武打电影。影片取材于清末拳师马永贞之事迹,并将年代向后推移自民国年间。而自此片起不但掀起黑帮片热潮,更开创了上海滩争霸的戏路,后以此为题材之影视作品更层出不穷。1990年代影星金城武重拍此片再演马永贞。山东青年马永贞(陈观泰饰)自幼练得一身好功夫,为谋求生活,只身来到十里洋场的大上海打拼。由社会低下阶层的苦力做起,因缘巧合结识黑帮头子谭四(姜大卫饰),两人惺惺相惜,马永贞更向谭四看齐,暗誓必要获得如其一般之地位。后马永贞于一壶春茶馆打去上海另一黑帮老大杨双(姜南饰)手下恶棍,夺得一壶春一带之地盘;另外又在杨双所招之擂台上打倒来自俄国的大力士。自此马永贞声名大噪,意气风发,为扩张地盘更夺去杨双一处赌馆,但此举却也使杨双对其怀恨在心。为夺取谭四地盘及利益,杨双与谭四手下勾结,使计将谭四杀害,其后杨双便将目标放诸马永贞。而马永贞在得知谭四为杨双杀害后,亦决心替谭四报仇,两人遂约于青莲阁相见。岂知杨双早命手下暗伏于青莲阁,马永贞不知遭袭,身负重伤,但凭一股劲力,终将杨双一伙尽数败于拳下,然而马永贞却也气衰力竭,倒于血泊之中。","text2":"马永贞(1972年电影)这部电影由谁主演?","label":1} {"text1":"少年队()是日本杰尼斯事务所旗下的三人组合,为日本1980年代非常受欢迎的偶像团体,领导的是锦织一清(东山只在少年队梦这个节目内领导)。少年队原名是B Team,后期更名为Johnny's少年队,在1981年组成。当时成员是锦织一清、植草克秀及松原康行,负责为近藤真彦及田原俊彦伴舞。后来松原康行退出,1982年东山纪之加入,成为现在的少年队。经过3年训练及演出后,他们在1985年12月12日正式出道。初次亮相时的广告标语是「日本出发,前往世界」。1986年获得第28回日本唱片大赏「最优秀新人赏」。出道一年后,1986年少年队开始有他们的舞台剧Playzone,并在每年都在夏天举行,直至现在他们仍然举办Playzone,近年来演出者更有Johnny's Jr.及J-STARS成员。在2007年时,他们已经公演达900次。基本上Playzone每一次是新故事,但唯独是2004年时,为了纪念杰尼斯事务所成立50年就重演了「梦断城西」。目前除了Playzone之外,成员都是各自以个人身份在演艺圈活跃中。","text2":"少年队正式出道的时间是什么时候?","label":1} {"text1":"陈秋盛,生于1942年7月9日,台湾苗栗县人。台湾指挥家,曾担任台北市立交响乐团团长及音乐总监。陈秋盛早年负笈德国慕尼黑音乐院学习小提琴演奏,1969年学成归国,开始在参与交响乐团演出并担任一些教学工作。1971年,陈秋盛为了学习指挥再次赴欧洲深造,在奥地利的维也纳音乐大学向斯瓦罗夫斯基学习,1973年他回到台湾,先在台湾省立交响乐团工作,最后落脚台北市立交响乐团。在北市交服务期间,陈秋盛极力推广歌剧作品,并数度率领北市交至美国、日本、新加坡、香港、菲律宾、俄罗斯、法国、西班牙、奥地利等地巡回演出。除此之外,他对本地作曲家的作品发表不遗余力,国内很多作品都经由他的指挥首演,在国外的音乐会总是尽可能的推荐演出本地作曲家的作品。2003年9月,陈秋盛被检举在北市交团长任内有多项弊端,因此被台北市文化局解除团长职务,最后他主动申请退休。退休后的陈秋盛,将音乐重心从乐团转移到他所擅长的教学,台湾中生代指挥家吕绍嘉、简文彬都曾是他的学生,除了在国立台湾师范大学、台北艺术大学、台南艺术大学等校任教以外,他有时也应邀客席国家交响乐团与国立台湾交响乐团。","text2":"退休的陈秋盛有什么转变?","label":1} {"text1":"侧带天竺鲷(学名:)为辐鳍鱼纲鲈形目鲈亚目天竺鲷科的一个种,俗名侧身天竺鲷、大目侧仔。本鱼分布于印度西太平洋区,包括东非、马达加斯加、塞席尔群岛、模里西斯、台湾、越南、菲律宾、巴布亚纽几内亚、印尼、关岛、澳洲、密克罗尼西亚、帛琉、新喀里多尼亚、马里亚纳群岛、万那杜、索罗门群岛、萨摩亚群岛、法属波里尼西亚等海域。该物种的模式产地在Vanicolo。水深0-2公尺。本鱼体延长而侧扁,眼大,口大略下位。体呈银色,身体纵深, 头部轮廓些微地凹曲,深色的狭窄纵线从鳃盖延伸至尾柄,第一背鳍的前端与顶端黑色,尾柄部有黑色小细点,背鳍硬棘7枚;背鳍软条9枚;臀鳍硬棘2枚;臀鳍软条8枚,体长可达11公分。本鱼栖息于河口或河川下游处,为广盐性鱼类,白天躲藏于近海岩礁或河口中,成群行动,夜间出来觅食,属肉食性,以小型底栖甲壳类为食。繁殖期时,雄鱼具有口孵习性,卵约7日化成仔鱼,由雄鱼吐出,具短暂的仔鱼飘浮期。可食用,通常做下杂鱼处理,制成鱼粉当饲料。","text2":"侧带天竺鲷的体长可以达到多少?","label":1} {"text1":"BA-6()是苏联在1930年代早期研制的装甲车。BA-6使用的45毫米炮(BT炮塔)和T-26坦克一样,另装备1挺7.62毫米DT机枪于车身中部,一挺DT同轴机枪于炮塔。BA-6重5.1吨,车长4.65米,宽2.1米,高2.2米,乘员4名。BA-6使用美国制GAZ-AAA底盘,这种底盘只适合在公路等良好路况下行驶。这情况后来在增加一对适合路轮时得到一点改善。BA-6外形接近BA-3,但BA-6取消了右后方一扇车门。BA-3装甲过于笨重,而BA-6装甲较薄(10毫米)且性能更好。BA-6最后被更佳的BA-10取代。在30年代早期,BA-6等装甲车可轻易击毁大部份同期装甲车,但其薄薄的装甲还是容易被小口径炮击破。BA-6参加了早期东线的战斗。由于其装甲不足以应付德军火力,所以其侦察角色被T-60坦克及T-70坦克等取代。","text2":"BA-6是什么?","label":1} {"text1":"方顺吉(),台湾高雄市凤山区人,台湾男歌手、男演员、主持人。方顺吉是小童星起家,中华艺术学校毕业。在校期间留级,霸凌同学。1992年参加台视五灯奖(与当时的冠军打成平手)。1993年参加三立《二十一世纪新人歌唱排行榜》儿童组歌唱比赛,闯关成功。1994年10月举辨「方强强滚反毒演唱会」,创国内年纪最小开三万人演唱会之歌手。1994年6月15日出版第一张个人专辑《翘脚髯嘴须》(与方婉真(方宥心)、萧玉芬两人),创国内年纪最小专辑破一百万张之歌手。1994年9月24日出版第二张个专辑《强强滚》,专辑销售破50万张。1994年11月14日与同比赛出身的童星歌手钟旎菱、邬兆邦、萧玉芬、张俊豪、慈袖静等人出合辑《美梦成真》,销售破25万张。其后又在方婉真(方宥心)《彩虹》专辑中与其合唱《彩虹》一曲。1995年1月26日出版第三张个人比赛歌曲专辑,销售破30万张。1995年6月29日出版第四张人专辑《彩色精灵》专辑,销售破30万张。1996年1月31日参与《让爱无所不在》合辑,其中收录强强滚,翘脚髯嘴须。2000年大旗唱片出版《大旗2000超级音乐快递》合辑,收录了《让爱无所不在》与大人变声版的《鼓励》两首歌曲。2002年12月参与中视闽南语连续剧《漂浪之女》记者招待会,于剧中饰演男主角「蔡顺吉」。2003年2月22日服食摇头丸被查缉,勒戒24天。2003年5月20日在高雄县燕巢乡看守所进行勒戒,6月12日完成勒戒。2003年8月19日贴钱发专辑《鼓励》,举行发片记者会,并发誓不再碰毒。2003年9月6日接拍大爱戏剧《后山姐妹》。2004年2月出版第6张个人专辑《异乡情梦》。2004年7月演出民视《蓝色水玲珑》借种大丈夫。2004年8月演出民视《亲戚不计较》。2004年8月演出台视《兄弟姊妹》。2006年卷入华视《快乐星期天》评审包小柏被殴事件,带嫌犯进摄影棚,被疑教唆打人,同年携大麻被捕,因未吸食以5万元交保。2012年1月加入天帝海国际股份有限公司顺瑛堂生技电视主持人。","text2":"方顺吉的第一张个人专辑名字叫什么?","label":1} {"text1":"《皇黎一统志》(越:),又名《安南一统志》,是一部成书于约18世纪末19世纪初的越南汉文历史小说。作者是吴氏三兄弟的吴时俧、吴时悠、吴时任。小说描述了越南后黎朝的灭亡和西山朝的崛起这段时期的历史故事。《皇黎一统志》的作者及编者为吴氏三兄弟的吴时俧、吴时悠、吴时任,越南方面有时将其合称为“吴家文派”()。小说的前七回由吴时俧所撰写,吴时俧又名吴俧,字学逊,号渊密,越南山南青威人(今属河内市),为吴时任之弟,领乡荐亚元,官历佥书平章事。第八回至十四回,为其从弟吴时悠所续。吴时悠又名吴悠,字征甫,号文博,有诗文集传世。这部小说后经吴时任编辑。吴时任又名吴任,字希尹,号达轩。30岁中进士,授户科都给事中,33岁兼太原督同行参政,后为郑王世子棕日讲,世子废,擢任为工部右侍郎,曾撰写或参与编写过《十七史撮要》、《四书说谱》、《海洋志略》等书。《皇黎一统志》的最后三回未注作者,是否为吴时任所补不得而知。《皇黎一统志》共十七回,约十二万字。小说反映了公元十八世纪末期黎郑王朝内部的斗争史。小说始于郑森宠邓妃,废嫡立少,致骄兵为变,卒招西山之兵,灭郑扶黎。因朝廷空虚郑氏诸王再次向皇帝要权,黎氏少主(即昭统帝黎维祁)重用权臣阮整,与南方军发生冲突,阮整被杀。昭统帝逃离京城,沿途招募义勇与西山党领袖阮光平对抗,屡为南方军所败,最后不得不向清政府求救,乾隆皇帝命两广总督孙士毅率清军南下以扶黎氏,孙士毅骄傲轻敌,为北平王阮光平所败,只得撤离升龙城。昭统帝亦逃到南宁、桂林,后与皇太后及皇妃被清政府召到燕京(今北京),由于清政府承认阮氏政权,昭统帝被软禁,于乾隆五十八年冬十月(1794年)郁郁而死,时年二十八岁。不久,越南最后一个王朝阮朝的帝王阮福映乘西山朝内部分裂之机回国,夺取嘉定(柴棍的另一名称),打败了阮光平之继任者阮光缵,建立阮朝,越南自此南北统一。小说最后以昭统帝归葬越南作为全书的结尾。","text2":"小说描写了一个什么故事?","label":1} {"text1":"嵊山镇是浙江省舟山市嵊泗县下属的一个镇。曾为嵊泗县和舟山渔场驻地。嵊山镇位于嵊泗县东部,境内的海礁(童岛)既是浙江省的最东端,也是中国大陆领海基点之一。全镇陆地总面积为7.13平方公里,海域面积广达2900平方公里。2000年人口普查时嵊山镇有人口10807人,壁下乡有739人,合计11546人,其中3\/4为渔业人口。常年有流动人口2000人,多为到此地抲鱼停泊的苏浙闽沿海各省渔民。2010年人口普查时总人口0.85万。2001年壁下乡撤消,并入嵊山镇。目前共设4个渔农村新型社区和7个行政村,即:渔业是嵊山镇的传统产业,嵊山曾为舟山渔场场部所在地。嵊山渔港为舟山渔场的重要渔港,浙江省最大的鲜活海水产品出口基地,也是国家一级渔港和二类开放口岸。","text2":"境内的海礁有什么特别之处?","label":1} {"text1":"伯奇还原反应(Birch还原)是指用钠和醇在液氨中将芳香环还原成1,4-环己二烯的有机还原反应。此反应最早由澳大利亚化学家Arthur John Birch (1915–1995)在1944年发表。 Birch还原的重要性在于:尽管剩下的双键(非芳香性)更为活泼,该反应却能停留在环己双烯上,而不继续还原。反应中的钠也可以用锂或钾取代,使用的醇通常是甲醇或叔丁醇。使用Birch还原的一个例子是还原萘:其他人也发表了很多篇关于此反应的综述。钠溶于液氨中会形成一个电子盐的亮蓝色溶液,化学式为[Na(NH)] e。溶剂化电子会与芳香环加成,形成一个自由基负离子。溶液中的醇此时作为质子试剂提供一个氢原子。对大多数反应物来说,氨上的氢酸性还不够。如果是取代芳香化合物,当取代基是羧基等吸电子基时,能够稳定碳负离子并生成最少取代的烯烃; 当取代基是供电子基时,则生成取代最多的烯烃。 热力学不稳定的非共轭1,4-加成产物往往产率超过热力学稳定的1,3-加成产物,这是由于共轭的戊二烯负离子中间体HOMO的最大轨道系数是在中间那个碳原子上,导致生成的1,4-环己双烯没有办法经过平衡移动而生成更加热力学稳定的产物,因此,生成的是动力学稳定产物。在卤代烃的存在下,上文提到的碳负离子也可以发生亲核取代反应生成新的碳-碳键。如下图所示,在Birch还原中生成的负离子中间体可以被一个合适的亲电试剂捕获,例如卤代烃:根据逆合成分析,前者即是后者的合成子。在下图所示反应中,1,4-二溴丁烷被加入到苯甲酸叔丁酯中,最后生成烷基化的1,4-环己双烯产物。","text2":"Arthur John Birch是在何时发表伯奇还原反应的?","label":1} {"text1":"真心之音广播电台为台湾唯一以儿童、青少年为为收听对象的民营非营利广播电台,节目内容以生活教育为主䌷。电台位于台北地区,目前尚为未立案电台,即一般俗称「地下电台」,2011年调频无线停播,转为网路直播电台,成立发音至今(2015)19年。创办人为媒体人 戴静远 先生,现41岁,23岁时成立广播电台,从事电视后制工作近二十年。改名为真心之音联播网,持续约半年即告终止,后又与马祖地区新成立之马祖生活资讯电台进行全时段联播,但随马祖电台的易手宣告终止。公益非营利跨宗教性电台,以十八岁以下儿童及青少年为收听对象,节目内容以生活教育为主;节目制播、工程之成员皆为志工性质,每年并经常举办社区公益活动,至今已结合相当多之公、民营机关团体及基金会。任何一个电台经营本身皆需消耗庞大资金,维持电台的开支以广告为主,原理上真心之音广播电台的属性于广播市场上是当难以生存的,因其公益、教育、收听对象均较不为广告主喜爱,真心之音纵然受青少年听众及其师长喜爱,但广告收入方面依然难以推展,但可贵的是真心之音广播电台之节目制播人员、工程人员为志工性质,不收任何费用甚至拿钱赞助电台开支,而器材供应商更以低于市场价格或赞助方式供给真心之音。这应是台湾大力推展近用媒体近二十多年来的典范。","text2":"真心之音广播电台之节目制播人员、工程人员是什么性质的?","label":1} {"text1":"桃园捷运棕线为桃园捷运计划路线之一,自1990年代便展开推动,路线能够衔接桃园捷运机场线、桃园捷运绿线、台铁捷运红线(纵贯线)、台北捷运中和新芦线、台北捷运万大树林线,并串连台北捷运路网。桃园捷运棕线(回龙线)由桃园车站起,沿桃园市桃园区万寿路三段与龟山区万寿路二段、东万寿路、万寿路一段、新北市新庄区中正路布设,最后与台北捷运中和新芦线、台北捷运万大树林线于回龙站交会,路线长11.5公里,共设8座高架车站,轨道机电系统采用轻轨捷运系统。目前设有桃园客运及中坜客运营运之桃园捷运棕线先导公车,其起讫站为捷运回龙站-桃园车站(代号:BR)以及延伸服务并有站站停车的601路,起迄点为捷运回龙站-内坜。回龙线曾考虑采单轨捷运系统,不过后来决定使用轻轨运输系统。截至目前为止的进度:","text2":"桃园捷运棕线从哪一年代开始推动?","label":1} {"text1":"叶灵凤(),原名蕴璞。江苏南京人。中国现代作家。叶灵凤幼时于上海美术专门学校学习。1925年加入创造社,开始文学创作,小说内容重视性心理分析。笔名有临风、亚灵、霜崖、秦静闻、佐木华、雨品巫、柿堂、南村、任诃、任柯、风轩、燕楼等。曾主编《洪水》半月刊。1926年与潘汉年合办过《幻洲》。1929年创造社被封,被捕,不久释放。代表作有《女娟氏之遗孽》、《菊子夫人》、《红的天使》、《香港方物志》等。中日战争爆发后初期,叶灵凤于上海参加由夏衍主持的《救亡日报》工作。其后上海沦陷,随报纸迁至广州。1938年以后在香港定居,1947年起于《星岛日报》撰写〈香港史地〉专栏,有系统地介绍香港历史和方物知识,1958年由中华书局出版《香港方物志》。迁居香港后,叶灵凤亦发表读书随笔,曾出版《北窗读书录》、《文艺随笔》、《晚晴杂记》等著作,1960至1970年代于香港《新晚报》撰写〈霜红室随笔〉等。1975年病逝。其后由丝韦(罗孚)为中华书局编辑整理叶灵凤三本著作《香港的失落》、《香海浮沉录》和《香岛沧桑录》,合称「叶灵凤香港史系列」,1989年出版。叶灵凤𡥧女是1970年代末至1980年代初在电视台任职编剧的叶中娴。","text2":"叶灵凤的笔名叫什么?","label":1} {"text1":"半领彩鹬(学名:')是一种涉禽,属于鸟纲鸻形目彩鹬科,是半领彩鹬属\"'下的单属种。其所属的彩鹬科(\"Rostratulidae\")也是一个小科,只有另一属另外两个物种。头及颈为深红棕色并在鸟冠上带有黄色条纹,上半身深灰棕色配白点;下半身全白。此科的雌鸟一般体型较大及毛色较亮丽,但此种对比起另外两种,两性异形的情况并不明显。有一相对较长及笔直的喙部,趾上有蹼等特征都使此种与另外两种区为不同的属。体长19至23厘米,重65至86克。此种分布于南美洲的南部,包括巴西南部、巴拿圭、乌拉圭、智利及阿根廷等地,生境为低地区的淡水湿地及湿润草原等。杂食性,于黄昏时间探头于湿润的泥中吞食无脊椎动物及种子等。一夫一妻制,筑巢于湿地旁不起眼的草丛中,浅薄的半盖状,一丛约生2至3枚蛋。繁殖期约为7月至翌年2月。沙哑叫声,在饲养种中曾录得\"wee-oo\"般的低呜。半领彩鹬是在智利及阿根廷等地是民间传统上的赛鸟,因此多有被捕猎的纪录。在其广泛分布的区域内并不是一种常见鸟种,淡水湿地也而受到被抽干池水的威胁。但到目前为止,仍未有文献指其种群数目有太大的威胁,因此在IUCN红色名录内评为无危物种。","text2":"半领彩鹬的上半身是什么颜色的?","label":1} {"text1":"苏保罗(Paul O'Sullivan、),人称「阿苏」、「Paul苏」,是纽西兰出生的练马师,现时于香港从练。苏保罗1976年开始练马事业,其父是纽西兰传奇练马师苏礼云,胞弟苏利云为骑师,他与父亲共同名义训练马匹,其胜出的主要大赛有:1989年凭好利时赢得日本杯、1986年「Our Waverley Star」赢日本富士锦标。此外亦凭滑浪天堂赢得1991年觉士盾。曾胜出澳洲多项一级赛,而在纽西兰,差不多所有大赛都赢过,曾经训练出三匹纽西兰全年度马王以及十一匹不同组别的冠军赛驹。于纽西兰期间,总共有11年成为了练马师冠军。1998年,其父苏礼云光荣退休,苏保罗独立展开其练马事业,夺得2002-2003年度纽西兰冠军练马师。2004-2005年马季开始,苏保罗获得香港赛马会发给练马师牌照,展开于香港的练马事业,家族在纽西兰的事业,便交由胞弟苏利云接管,并于2004年10月30日,凭同欢畅团体名下的「领奖」取得在港从练首场头马。2006-2007年马季成为亚军练马师,2007年凭活力金刚赢得香港打吡大赛,赢得自来港开仓以来第一个分级赛冠军。2010-2011及2011-2012年度,马房成绩陷入低潮,之后成绩渐见起色。2013-2014年度马季,苏保罗训练的「友莹格」取得一季五连捷佳绩。2015年3月29日,苏保罗凭「友莹格」为香港夺得首次高松宫纪念赛冠军,扬威日本。5月17日,苏保罗凭「友莹格」为香港赛驹连续三届胜出KrisFlyer国际短途锦标冠军,扬威新加坡。苏保罗在2014-2015年马季于港夺得39场头马,并位列练马师榜的前六名。2015年11月14日,苏保罗凭「尚华盛甲」取得在港从练的第三百场头马。2015-2016马季成绩突出,取得50场头马位列练马师榜的第五名。","text2":"苏保罗从事什么职业?","label":1} {"text1":"国民乐派,是广义古典音乐的一支,但没有明确的定义和共同的风格,经常被视为「浪漫主义音乐」或「现代音乐」的分支。大致就是指会在古典音乐的曲式中,使用了本国的民间音乐旋律和特别的乐器,并通常宣扬爱国主义或民族主义,主要在十九世纪中到二十世纪中活跃。一般认为十九世纪中前期,西方音乐的传统心脏的德语和义大利地区以外的欧洲和美洲国家,作曲家需要属于本地的音声。反之德语或义语的作曲家即使用民间音乐素材,仍然只是会被认为浪漫主义音乐,如小约翰·施特劳斯的圆舞曲原型虽然来自民间,却被当时的维也纳视为国际性的艺术。国民乐派起源众说纷纭,较流行有两说。一是十九世纪俄国作曲家格林卡的《爱国歌》,另一说是法国作曲家圣桑成立国家音乐社团组织,两人分别影响了本国的作曲家组织,交流了其对民间音乐的资料和心得,以及对于教育本国的音乐界作出重大的贡献。其他尚有肖邦和弗朗兹·李斯特之说,因为是他们首先采用故乡的民间音乐素材,但他们并未有组织本地作品家活动,也没有系统地使用和搜集民间音乐元素。在十九世纪后期其他欧洲国家也产生了自己国民乐派作曲家和作品,如英国的艾尔加的《威风凛凛进行曲》(Pomp and Circumstance Marches),捷克的斯美塔那的《我的祖国》。还有一些作曲家为自己尚末被认可为国家的故乡,创作鼓吹独立的作品,如芬兰的西贝柳斯的芬兰颂,挪威的格里格的培尔·金特组曲等。而美洲的作曲家也开始活跃起来,如美国的约翰·菲利普·苏萨的进行曲《星条旗永远飘扬》,而同期斯蒂芬·福斯特更有开创流行音乐的先河。到二十世纪初巴西的海托尔·维拉-罗伯斯,匈牙利的柯达伊,美国的查尔斯·艾夫斯等,新国民乐派作曲家开始向现代音乐的过渡。到二十世纪中后期起,因为流行音乐的兴起,也少有未被古典音乐使用的民歌,所以在欧美国民乐派渐趋于沈寂,但在古典乐传统地区外的亚洲,却有新型的国民乐派作品面世,如中国的冼星海的黄河大合唱,何占豪和陈钢的梁祝小提琴协奏曲等,代表了欧美以外的民族主义兴起。二十一世纪初,虽然因为再很少未独立的殖民地和国际主义盛行,政界不再需要常以音乐推广爱国思想,但国民乐派音乐并未因此而丧失了其地位,反而把这种音乐从政治服务中解放过来,受到本来创作国家外的欢迎。","text2":"国民乐派具体指什么类型的音乐?","label":1} {"text1":"宏安集团有限公司,(Wang On Group Limited,),简称宏安集团,创立于1987年 ,是香港上市公司位元堂的控股公司,持有位元堂28.2%股权,亦为卢森堡大药厂(即珮夫人咳药水品牌的持有人)。另外宏安集团是万有街市的经营者,其外判承包:荃湾丽城花园、马鞍山颂安邨、东涌富东邨等多处街市经营权。宏安亦持有天水围俊宏轩商场、西营盘荟溱及油塘四山街13及15号工业地皮等。2012年5月31日宏安集团成功收购深水埗营盘街140-142号旧楼,地盘合共占地约4624方呎,以地积比率九倍计,料重建楼面约41616方呎,预计将发展与旗下红磡荟点相若类型的住宅。2013年1月29日土地注册处资料显示,宏安(01222)旗下旺角弥敦道724至726号银座式商厦8楼、11楼及12楼分别以6600万、5610万及5610万元易手。宏安于2010年以3.2亿元向科达地产主席汤君明购入汉英大厦,然后改建为一幢25层高银座式商厦,并拆售其中7至25楼,合共16层楼面,每层面积约2200方呎,预期以餐厅、零售等作主题,并于2014年落成。2014年6月25日宏安集团伙拍锦华实业,成功投得沙田市地段第599号地皮。8月13日,双方再次合作,成功投得沙田市地段第598号地皮。2015年1月7日,宏安集团成功投得沙田市地段第587号地皮。2017年6月14日旭辉控股以约6.645亿港元,向宏安地产收购油塘四山街13及15号50%权益,项目土地面积41,080平方呎,可建楼面面积约27.2万平方呎。2017年6月22日宏安地产以转让公司形式元向亚洲联合基建控股90%和俊和发展集团10%购入马鞍山白石耀沙路地皮,即沙田市地段第601号的所有股权,占地面积252954平方呎,地积比率约1.53倍,预计总楼面面积可达约387,500平方呎,双方都未有披露成交金额,市场估计应接近29亿元,楼面地价约7,500元。2017年9月12日宏安地产以24.4125亿元向碧桂园旗下全资附属公司恒宙国际出售马鞍山白石耀沙路地皮,即沙田市地段第601号的60%股权,交易完成后,宏安地产将仍持有地皮40%权益。","text2":"宏安集团的外判承包包括哪些方面?","label":1} {"text1":"语义网堆栈()或称语义网蛋糕、语义网千层饼()指语义网的体系架构。语义网堆栈利用图示解释是不同层面的语言所构成的层级结构;其中,每一层面都将利用下游层面的能力。该图说明的是针对语义网而加以标准化的种种技术究竟是如何组织起来,从而让语义网变为可能的。同时,该图还说明,语义网究竟是如何成为经典超文本网络的一种扩展,而不是取代者。语义网堆栈图最初是由Tim Berners-Lee所创建的。目前,随着图中不同层面之间的协调,该堆栈仍在不断地发展演变正如语义网堆栈图所示,语义网的创建是采用下列语言或技术来实现的。当前,从堆栈底层向上直至OWL的那些技术已经实现标准化并获得认可,可以用来构建语义网应用程序。不过,究竟对于该堆栈顶层部分会如何加以实现,目前尚不清楚。要获得完整意义上的语义网,则需要实施该堆栈的所有层面。堆栈图中的底部层面,包括超文本网络方面众所周知的那些技术以及一些未加改动的技术,奠定了语义网的基础。中间的那些层面-{}-包含的是那些W3C已经加以标准化的技术,旨在使语义应用程序得以构建。顶部层面则包含的是那些目前尚未实现标准化的或者仅仅包含的是为了实现语义网而应当加以实现的想法或概念。","text2":"语义网堆栈又被称为什么?","label":1} {"text1":"张家界市第一中学,简称张家界市一中,位于中国湖南省张家界市城区,成立于1938年,是湖南省重点中学、省首批示范性高中。现任校长余国忠。有5800多名学生,包含初中部和高中部,教职工340多人,建有可容纳1500余床的宿舍。张家界市第一中学成立于1938年中国抗日战争时期,初名「湖南省私立孔道中学大庸分校」。1939年为躲避日军轰炸,迁往大悲庵(今荷花机场)。1940年,学校更名为「大庸县立初级中学」,下学期接省教育厅批覆,学校停办一年。1942年,湖南省教育厅核准大庸县立初级中学。1943年,除了原本的初中部,增收一个四年制简易师范班和一个一年制特师科班,培育小学师资。1949年,大庸县解放。中国国民党第122军军长张绍勋在校内的总务室内被俘。1998年,大庸一中更名为张家界市一中。","text2":"张家界第一中学成立于哪一年?","label":1} {"text1":"巴鲁剌思氏是蒙古部落,源出尼伦蒙古,祖先可追溯至哈剌察儿,与成吉思汗同宗(起源于孛端察儿),这部落在草原时代著名人物是忽必来,他降服了葛逻禄人。在钦察汗国也有他们。主要分布于伊朗与中亚。蒙古征服花剌子模国后,察合台出镇西域,乃蛮、克烈、札剌亦儿、巴鲁剌思、阿鲁剌惕、杜格拉特也有被分配,以巴鲁剌思最被器重。他们后来突厥化与伊斯兰化,采用察合台语,用阿拉伯文与波斯文,他们在察合台汗国中期发迹,帖木儿的父亲塔剌海控制了渴石,帖木儿后来成为秃忽鲁帖木儿的辅臣,在他死后,赶走也里牙思火者,袭杀忽辛,自称苏丹,建立帖木儿帝国。在帖木儿帝国解散后,巴布尔在印度北部建立蒙兀儿帝国。现在有一些巴鲁剌思穆斯林生活在巴基斯坦与印度,乌兹别克。他们语言也变成乌兹别克语。","text2":"巴鲁剌思氏为什么会采用察合台语,用阿拉伯文与波斯文?","label":1} {"text1":"\"Creative声卡,请参见x-fi\"XFI是一个10 吉比特每秒的芯片间电气接口规范,是作为XFP多源协议的一部分定义的,由XFP MSA小组开发。XFI的主要应用是10 Gigabit Ethernet,10 吉比特每秒的光纤通道,SONET OC-192, SDH STM-64, 10 吉比特每秒的OTN OTU-2,以及并行光纤链接。XFI提供了采用64B\/66B encoding模式下运行于10.3125吉比特每秒的单通道。XFI有时候也被读作\"X\" \"F\" \"I\"或\"ziffie\".在2006年中,多数10 G以太网产品使用XAUI接口,采用8B\/10B encoding有4条运行于3.125Gbit\/s的通道。大多数10吉比特以太网产品要求SerDes设备完成XAUI到XFI的传输。有些10 GbE设备集成了XAUI和XFI两种接口。","text2":"\"Creative声卡是由哪个小组开发的?","label":1} {"text1":"《多宝塔碑》是唐代重要碑刻,是书法中楷书代表作品。此碑是颜真卿书,是为颜真卿早期楷书代表作品,也是颜体书法的代表帖。《多宝塔碑》,又称作《多宝塔感应碑》,全称为《大唐西京千福寺多宝塔感应碑》,《多宝塔碑》立于唐玄宗天宝十一年(752年)四月二十日,该年颜真卿正值四十四岁,担任朝议郎尚书,奉旨书写此碑。《多宝塔碑》的碑文共有三十四行,满行有六十六个字,全文两千多字,由相门后裔南阳岑勋撰文,越州徐浩以隶书提额,河南史华刻字。碑体总高285公分,宽度102公分,唐朝时立碑于长安安定坊千福寺,宋代时移到西安碑林,清朝康熙年间碑体折断,现收藏于西安碑林博物馆,属于国宝级文物。","text2":"《多宝塔碑》是谁所书?","label":1} {"text1":"单角鼻鱼,又称长吻鼻鱼(学名:),俗名剥皮仔、打铁婆、独角倒吊,为辐鳍鱼纲鲈形目刺尾鱼亚目刺尾鱼科鼻鱼属的鱼类。单角鼻鱼分布于印度太平洋海域,包括东非、红海、模里西斯、塞席尔、马尔地夫、留尼旺、马达加斯加、斯里兰卡、安达曼群岛、日本、台湾以及西沙群岛、中沙群岛、舟山群岛等中国沿海、菲律宾、印尼、新几内亚、新喀里多尼亚、澳洲、新几内亚、马里亚纳群岛、马绍尔群岛、密克罗尼西亚、帛琉、索罗门群岛、斐济群岛、万那杜、夏威夷群岛、法属玻里尼西亚、诺鲁、吉里巴斯、吐瓦鲁等海域。水深1至80公尺。本鱼体呈椭圆形而侧扁;成鱼头顶有角状突起,其长度与吻长略同,吻背朝后上方倾斜,直到角突处为止。体色为蓝灰色,腹侧则为黄褐色,尾柄上的骨质板为青黑色。雄鱼除角突外,尾鳍截形,上下叶延长成丝状,上叶较长。背鳍硬棘6枚、背鳍软条27至30枚、臀鳍硬棘2枚、臀鳍软条27至30枚。体长可达70公分。本鱼多半在礁沟,礁坡或有涌浪处成群活动,交配时成对出现,属杂食性,幼鱼以藻类为食,成鱼则以浮游生物为食。大鱼可为观赏用。亦可食用,此鱼不新鲜时,腥味极重,故一捕获,即需去除内脏,尾柄有硬棘,须注意。多煮汤,尤其煮味噌汤,味美。","text2":"单角鼻鱼什么时候腥味极重?","label":1} {"text1":"董雷,北京人,是中国职业足球守门员。董雷从乙级联赛开始初露头角,2003年被八一签下,师从著名前国门李松海,结果还在绝对主力李松海之子李雷雷受伤期间成了八一的正选门将。2003赛季结束后,俱乐部解散,董雷被珠海中邦签下。董雷随中邦打入中超联赛,不但逐渐成为主力门将,更加曾经被列入国家队大名单。但是之后在2006年的联赛中曾经被撞导致右肾出血,休战了相当长的时间。2007年,他因为申花联城合并而进入上海申花,面对王大雷、张晨等的竞争,最终被俱乐部租借到新加坡联赛辽宁广原。但是他却在新加坡陷入了假球案,最终被判刑和罚款,回国后连年被俱乐部挂牌。2009年初,董雷租借至刚刚搬迁回上海的浦东中邦。2010年,他又与原申花第三门将张晨交换,再次返回了上海申花。2010年11月6日,上海申花在联赛最后一轮客场出战已经提前夺冠的山东鲁能,由于王大雷上调国奥队,邱盛炯受伤,董雷第一次也是最后一次代表申花出战正式比赛,不过球队以2-5告负。2012年,董雷转会到中甲球队北京八喜,在加盟球队的首个赛季联赛得到5次上场机会。2013赛季逐渐成为球队的主力门将,2014赛季随着姜波的租借加盟,再次沦为替补门将。","text2":"董雷最后一次代表申花出战正式比赛的战果如何?","label":1} {"text1":"野山羊(\"Capra aegagrus\")是一种野生的山羊,分布在欧洲及小亚细亚至中亚及中东。群族中野山羊的数量可以多达500头。较老的公羊会在发情期躯使年幼的公山羊进到母群中交配。平均妊娠期为70日。母山羊一般会产下一只幼山羊。幼山羊出生后会立即跟随母山羊,在6个月后断奶。母山羊在岁半至两岁半时就达至性成熟,公山羊则要到三岁半至四岁。牠们的寿命介乎12至22岁之间。虽然野山羊在一些地区是原生种,但做为入侵物种则会是非常具破坏性的。在加拉帕戈斯群岛,外来的野山羊大量啃食当地的植物,破坏了生态的平衡,后来人们只能将牠们猎杀至岛内灭绝来解决问题。野山羊分布在阿富汗、亚美尼亚、阿塞拜疆、孟加拉国、塞浦路斯、格鲁吉亚、希腊、印度、以色列、伊朗、伊拉克、义大利、黎巴嫩、阿曼、巴基斯坦、克什米尔、俄罗斯、斯洛伐克、叙利亚、土耳其及土库曼斯坦。","text2":"幼山羊出生多久后会断奶?","label":1} {"text1":"仙女座星系-银河系碰撞是预计四十亿年后,在本星系群中两个最大的成员星系──银河系和仙女座星系之间发生的星系碰撞 。在星系碰撞的有关模拟研究中仙女座星系-银河系碰撞常常被用来当作此类现象的范例。事实上,在这种星系碰撞中星系中所包含的恒星等天体并不会真的发生物理上的碰撞接触,因为星系本身是非常弥散的——作为距离太阳最近的恒星,比邻星与地球间的距离也有太阳直径的三千万倍之遥。(如果太阳按比例缩小为一枚25美分硬币大小,那么比邻星则在700千米之外。)如果这个理论正确,那么在大约三十亿年后仙女座星系内的恒星与气体将能够在地球上用肉眼看到。如果仙女座星系与银河系发生了碰撞,两个星系将在大约七十亿年后最终合并为一个更大的椭圆星系。对于这一合并后星系的命名有多种提议,其中最广为接受的是「」,亦即「银河系」()和「仙女座星系」()的英文合称。要指出的是,现在还无法确定这场碰撞是否一定发生。仙女座星系相对于银河系的径向速度可以通过对星系中恒星光谱的多普勒效应的观测来测量,但其横向速度 (即自行运动的速度)无法直接测量。这样,虽然我们已知仙女座星系正以每秒120千米的速度向银河系接近,但依然无法得知届时它会和银河系碰撞,还是错过。目前对仙女座星系横向速度的最佳估计是小于每秒100千米,这暗示着星系的暗物质晕将会发生碰撞,而星系盘则可能不会。欧洲空间局计划在2011年将发射一艘新的航天器盖亚,试图通过测量仙女座星系中恒星的位置来精确测定仙女座星系的横向速度。空间望远镜研究所的弗兰克·萨默斯()根据凯斯西储大学的克里斯·米霍斯()教授和哈佛大学的拉尔斯·赫恩奎斯特()教授的研究制作了描述这一预期事件的计算机图像。这种星系碰撞在宇宙是相当普遍的。例如一般认为仙女座星系在过去就曾经和其他星系至少发生过一次碰撞。理论上我们的太阳系在这场碰撞过程中也有一定的可能从合并形成的新星系中释放出来,而在星系碰撞的初期甚至有可能成为仙女座星系的一部分。由于恒星间的距离非常遥远,这种事件对太阳系本身不会有什么负面影响(特别是在五六十亿年后太阳将进入红巨星阶段后),对太阳及其行星本身的任何形式的扰动更是非常微小的。","text2":"这种碰撞将会带来什么?","label":1} {"text1":"高升泰()是北宋年间云南大中国皇帝,滇东白蛮领袖。大理点苍山莲花峰芒涌溪人氏。1080年,杨义贞弑大理皇帝段廉义,自立为帝。时为鄯阐(昆明)侯的高升泰受其父高智升之命率滇东乌蛮三十七部兵马攻灭杨义贞,拥立段寿辉为大理皇帝。1081年,高智升与高升泰父子逼迫上明帝段寿辉退位出家,接著便拥立段正明继位。高升泰封为善阐侯。宋哲宗绍圣元年(1094年),一直以来都在专权的高升泰废段正明,自立为帝,国号“大中”,年号上治。分封诸子弟:其弟高升祥的势力在善阐府(昆明),幼子高泰贤的势力在永昌府(保山)、高泰运的势力在腾冲府(腾冲)、高泰慧的势力在鄯巨府(永胜)。1096年,高升泰病逝。由于云南诸部的反对,高升泰临终嘱咐其子高泰明还政段氏,于是高泰明拥立段正明的弟弟段正淳为大理皇帝,但其后大理国柄掌握在高氏手中,世称高国主。高升泰谥号富有圣德表正皇帝。在金庸小说《天龙八部》中,高升泰是段正明、段正淳的亲近臣子和好兄弟,段正明将皇位传给段誉。小说中高升泰、段正淳并没有当皇帝。","text2":"高升泰是谁?","label":1} {"text1":"《714航班》(法语:Vol 714 pour Sydney ;英语:Flight 714)是《丁丁历险记》的第22部作品,也是倒数第二部完整作品。作者是比利时漫画家艾尔吉,于1968年初版。故事的主要舞台是在南太平洋,特色之一是外星生物的登场。丁丁、米路、哈达克船长和图纳思教授前往澳洲悉尼,参加世界航空大会,却误上了被绑匪看中的百万富翁卡达斯的私人飞机。更糟的是,绑匪头子正是丁丁的宿敌。在南太平洋的小荒岛上,绑匪头子下药逼富翁说出银行密码,并打算杀人灭口,丁丁等人也在顺带消灭之列。怎样从绑匪布下的天罗地网中脱身?神庙壁画和雕像,透露了甚么秘密?丁丁等人遇上火山爆发,濒临死亡异境,是谁救他们逃出荒岛?这一切,与离奇的声音又甚么关连?","text2":"丁丁及其伙伴们前往澳洲的原因是什么?","label":1} {"text1":"《碧血剑》()是香港电视广播有限公司与中国中央电视台旗下的中国国际电视总公司合作摄制的古装武侠电视剧,由林家栋、江华、佘诗曼及吴美珩领衔主演,以及由李添胜担任监制。此剧根据金庸所著的武侠小说《碧血剑》为基础,再把小说改编剧集,然后进行拍摄制作。此剧由无线电视继1985年版《碧血剑》之后,无线电视为拍摄制作第二部同名剧集。此剧在无线电视翡翠台首播前的电视预告片及宣传片中,为无线电视所选播的33周年台庆剧。电视广播(国际)有限公司授权台湾弘音多媒体科技股份有限公司于2008年推出发行了《碧血剑》DVD影碟零售版本,此影碟收录全套35集,共7只碟,设有粤语及国语发音版本并配上非隐藏繁体中文字幕。","text2":"这部电影是根据谁的同名小说改编的?","label":1} {"text1":"富纳角箱鲀(学名:)为箱鲀科角箱鲀属的鱼类,俗名福氏角箱鲀。分布于印度洋-太平洋海域的热带地区,从东非和南非至台湾岛、日本南部、夏威夷、豪勋爵岛等。该物种的模式产地在莫桑比克。它喜欢珊瑚礁和珊瑚礁外区的沙地或者石地。最深的生活区为130米的深处。它生活在离海底很近的水域。富纳角箱鲀的一个特征是在它背部正中有一根大刺。它的颜色是朱红至淡棕色,占据地盘的雄性是黄色的,带有蓝色的线条。组成装甲似的骨头往往明显显示出是多边形的。体长在23至25厘米间。雄性富纳角箱鲀的地区性非常强烈,它们占据约500平方米的地域。在这个地域上他们可以与至四条雌性富纳角箱鲀一起生活。有时地域上也会有一些未成年的雄性鱼。对富纳角箱鲀的繁殖的研究很彻底。交配在日落后不久的黄昏中。雌鱼和雄鱼并行向上游。雄鱼发出嗡嗡声。在它们游到最上方的时候排卵和精。一些雄鱼也会变色成雌鱼的颜色,游到其它地域里,假如它在那里遇到要排卵的雌鱼,它会变色回到雄鱼的颜色,然后和雌鱼交配。","text2":"富纳角箱鲀分布于哪一海域的热带地区?","label":1} {"text1":"葡萄牙国徽(),为盾徽,中间白盾绘有五个呈十字状排列的小蓝盾,蓝盾绘有五个呈X形十字分布的白色圆点。外面红盾绘有七座金色城堡。置于浑天仪上,再以用红绿两色绶带束起的橄榄枝装饰。此徽颁布于葡萄牙第一共和国成立后的1911年6月30日,且沿用至今。代表葡萄牙的徽章在大概在1000年前已经出现。最早可以追溯到勃艮第的亨利(葡萄牙开国君主亚丰素·殷理基的父亲)之银盾蓝色十字。经过多个世纪的改朝换代,葡萄牙国徽的图案内容也有所增减。1910年10月5日革命后,成立葡萄牙第一共和国,国徽则在翌年6月30日开始使用。葡萄牙王国在1139年宣告独立,自1143年签订萨莫拉条约正式从卡斯蒂利亚王国分离,象征著王权与铸币权的银色圆点便被加在勃艮第的旗帜上,以宣示主权的独立。经过中世纪长年累月战争的洗礼,当桑乔一世继承他父亲亚丰素的徽章时,盾牌上的图案已由十字被磨损成五个小盾牌。各小盾牌内的圆点起初有11个,后来塞巴斯蒂昂在位时把圆点的数量减至5个。这在现代被解读成与耶稣受难时的有关。红框和里面的城堡是亚丰素三世统治时期加上的。源自卡斯蒂利亚的城堡代表在收复失地运动中从摩尔人夺取的城池。当亚丰素三世夺取其兄桑乔二世的王位时,由于他是亚丰素二世的次子,并无准备登基成为国王。他的纹章融合了父亲葡萄牙式的盾牌和卡斯蒂利亚式的红色框界和金色城堡。尽管初时城堡的数量由8至12座不等,而亚丰素四世曾经定为12座,但后来塞巴斯蒂昂最终把数量定为现时的7座。与西班牙式的城堡不一样的是,西班牙式城堡的城门绘上的是蓝色(保持开放);葡萄牙式城堡的城门绘上的是金色闩门(没有开放)。浑天仪自15世纪起成为代表葡萄牙的其中一个元素,在许多葡萄牙的殖民地旗帜上(特别是巴西)也可轻易找到。它在航海的重要用途(辨认方向和测量距离)意味著地理大发现时代对葡萄牙以及其众多殖民地(即使在共和后也是如此)的重要性。相对王室的冠冕,虽然浑天仪被认为「共和」的象征,不过其实早在葡萄牙-巴西-阿尔加维联合王国时期经已在国徽上出现。这是由于巴西也是联合王国的一部分的关系。","text2":"葡萄牙国徽颁布的时间是?","label":1} {"text1":"尼康 D300 是一款 12.3 百万像素的专业级 Nikon DX 格式 数码单镜反光相机 (dSLR) ,与尼康全画幅相机D3一同发布于2007年8月23日。它被尼康定位于DX格式最顶尖的相机。它很类似于尼康 D3,最大的区别在于感光元件尺寸的不同,D3为全画幅。但与D3相比D300更实用,兼有高分辨率和高速的优势(连拍速度为6张每秒,而当附加MB-D10电池模块时能够达到8张每秒)。尼康 D300 发布于2007年8月23日。 初步试用报告来自于杂志和户外摄影师, Shutterbug 杂志, 以及英国的杂志, \"What Digital Camera\". Imaging resource, 一家摄影网站同样也发布了它的初步试用报告一份全面的D300使用报告发布于DCR网站 和Camera Labs网站上。同时来自加拿大的Digital Review Canada将其与早先的D200做了比较 by 摄影师 Ken Rockwell在他的个人网站上也做了同样的比较。流行摄影杂志赋予尼康 D300 \"2007年最佳相机\"称号。 此杂志同样也发布了有关于D300的评测,同时注明\"Nikon Capture NX\"随机附送。MB-D10手柄兼电池匣作为一个可选的电池盒附件同时提供了竖拍支援,竖拍位置附设快门按钮、主功能转盘、辅功能转盘以及AF对焦钮。通过MB-D10,D300能够使用额外的EN-EL3e锂电池或是AA型电池,若要使用使用EN-EL4a\/El4锂电池,还须另购BL-3电池仓盖。相机能够设定优先使用机身的EN-EL3e锂电池或是 MB-D10手柄,这样仅当主电池耗尽时使用另一块电池。当D300附加内置AA电池或是EN-EL4\/EL4a锂电池MB-D10手柄时,连拍速度能够提升到8张\/秒,否则相机的连拍速度限制在6张\/秒。2008年1月15日, 尼康为D300额外添加了三种相片控制模式,能够让其模拟D2X\/D2XS的色彩模式I, II 以及 III。2008年2月14日,尼康释出了一个固件升级,解决了部分D300在超过8秒的曝光时间后,画面会出现不正常的垂直条纹的问题。","text2":"尼康是如何定位这款 D300 相机的?","label":1} {"text1":"1944教育法令(Education Act 1944),此法令改革了英格兰与威尔士中学的教育系统。法令常以保守党政治家拉布·巴特勒命名,他为中学教育引入了三分系统),并变中学教育为免费。法令在1996年11月1日被1996教育法令废除。三分系统将中学分为三类:文法学校、与。它允许了的创办,这种学校是前三种学校的结合,但是这种学校最初只创办了几所。它也引入了一个新的系统,,在这个系统下,一些学校接受教育部的直接补助(不同于普通学校接受地方教育局补助),作为交换,他们需要预留一些免费学位。法令规定向英国所有18岁以下学童免费提供三分之一品脱牛奶,后来,哈罗德·威尔逊的工党政府不再向中学生提供免费牛奶。这个政策在1971年被撒切尔夫人(后任教育大臣)延伸,她不再向七岁以上学童提供免费牛奶。法令的其中一个重大结果是教育、动员了妇女与工人阶级。它使得中学向妇女开放,接受专上教育的工人阶级比率大大增加。受过教育的工人阶级发现他们在社会中处于不利地位,使得工人阶级与中产阶级间出现更严重的阶级分化。约翰·奥斯本在20世纪50年代末期的戏剧作品提及了如此的分化。负责教育的部门由教育委员会(Board of Education)更名为教育部(Ministry of Education),并给予它更大的权力与更多的预算,结束了国立学校需要交学费的时期。在将离校年龄提升至15岁后,它赋予了政府提升离校年龄至16岁的权力,“首相才刚刚因为它(将离校年龄提升至15岁)是切实可行的而感到满意”,虽然这个措施在1973年前仍未落实。法令也提供了社区学院(Community College),向儿童与成人提供教育,但措施只有少数地方教育局跟进,如剑桥郡。法案强制国家资助的学校每日进行祈祷。此条款被1988教育改革法令修订,法令指出祈祷大致上应在基督徒间进行,而不适用于特定学校或儿童。修订也指出,祈祷可以在课室进行,而不是在集会中进行。为了决定学童应该升读哪所学校,他们设立了一种被称为的考试。升学学童需要参加这个考试,分别到最适合他们“能力与兴趣”的学校,但是实际上,一些文法学校因为倾向于学术成绩所以仍未改革,而工业学校、综合学校只创办了几所。结果,不管是否合适,大部分学童升读现代中学,意味着大部分教育资助流入现代中学。","text2":"1944教育法令在哪方面做出了改革?","label":1} {"text1":"维拉拉维拉海战(,;1943年10月7日)是一场发生于所罗门群岛维拉拉维拉岛海域,在日本和美国舰队之间爆发的一场海战。在新乔治亚战役和维拉湾海战失败后,日军已放弃了中所罗门地区,撤出了驻扎在该地区的部队。然而至1943年10月,还有600名日本官兵驻扎在维拉拉维拉岛北部的观察点。日本海军调集了9艘驱逐舰,在伊集院松治少将的指挥下前往维拉拉维拉岛疏散其陆军部队,同时一并拖走停泊在岛北部的日本驳船。与此同时,附近海域也游弋着一支美国舰队。美国第4驱逐舰中队包括6艘驱逐舰,分为2个部分,正在弗兰克·沃克上校的指挥下沿维拉拉维拉西岸北上,对临近海域进行巡逻。22时30分,日本瞭望发现第一支由三艘驱逐舰组成的美国编队。沃克上校没有等待另外三艘驱逐舰的支援,决定立即接敌战斗。23时左右,双方几乎同时开火,同时都发射了鱼雷。日本夕云号驱逐舰排在战列首位,故遭到美国人的集中攻击。其舵机被摧毁,后又被一枚鱼雷命中,于23时10分左右沉没。然而夕云号发射的一枚鱼雷击中了美国切瓦立号驱逐舰(DD-451),造成后者的弹药舱发生爆炸。跟进的奥班农号(DD-450)又意外同切瓦立号相撞,两舰一时无法分开。23时6分,塞尔弗里奇号(DD-357)又被一枚鱼雷击中,丧失了战斗力。此时美军增援部队尚有15分钟才能到达战场。然而日本人此时却转向撤走了。切瓦立号于次日3时左右沉没。日本人完成了其撤退任务,标志着盟军第二阶段车轮行动的结束。在长达三个月的战事中,盟军占领了中所罗门地区,损失了6艘舰艇,击沉了16艘日本军舰。","text2":"这场海战发生于哪两个国家之间?","label":1} {"text1":"尖吻单棘鲀(学名:),又称尖吻鲀,俗名尖嘴砲弹、玉黍砲弹,为辐鳍鱼纲鲀形目单棘鲀科的一种。本鱼分布于印度太平洋区,包括东非、模里西斯、红海、马尔地夫、塞席尔群岛、留尼旺、斯里兰卡、印度、泰国、越南、日本、中国、台湾、菲律宾、印尼、澳洲、新几内亚、关岛、圣诞岛、马绍尔群岛、帛琉、萨摩亚群岛、东加、万那杜等海域,属于暖水性鱼类。水深0至30公尺。本鱼体极侧扁,略呈长椭圆形;吻部延长突出。第一背鳍棘特化成强棘,表皮呈绒毛状粗糙,无鳞片。体呈蓝绿色,布满呈黄色斑点,背鳍硬棘2枚; 背鳍软条31至35枚;臀鳍软条29至32枚,体长可达12公分。本鱼栖息于在内湾或向海的礁坡,喜藏于轴孔珊瑚中,并以此种珊瑚虫为食,属肉食性。常以略呈头上尾下的倾斜姿势游泳,筑巢于长有海藻的死珊瑚基部。为色彩鲜艳的观赏鱼,不供食用。","text2":"尖吻单棘鲀常以怎么样的方式游泳?","label":1} {"text1":"琵琶湖是台湾一个淡水湖泊,位于台东县台东市东北处的市郊,卑南溪出海口南边,台东森林公园之内。琵琶湖的形成是卑南溪地下涌泉不断冒出的结果。琵琶湖原本是不存在的,现在之所以可以形成湖泊,这是因为它的出海口已经被沙嘴阻挡了,即所谓「没口」。此湖泊因形状恰似琵琶,而取名为「琵琶湖」。琵琶湖是一个珍贵的河口海滨湿地,湖泊里有著各式各样台湾原生淡水鱼类。湖泊之水清澈见底,周围树木茂密,环境清幽。周围木麻黄林又称「黑森林」,林间栖息著小白鹭、小水鸭、翠鸟等候鸟。琵琶湖周围主要是木麻黄为主的防风林地,也普遍可见热带河口的植物,另外还有海檬果分布。琵琶湖周围兴建了许多造景,例如赏鸟小屋、凉亭、树屋、全长1.8公里的自行车道、森林步道、原木栈道等,另外每年的南岛文化节皆在此处举办。琵琶湖周围尚有人工兴建的活水湖、原为沼泽湿地,栖息许多鹭鸶、野鸭、树蛙、雉鸡、鱼类的鹭鸶湖等其他湖泊。适合在周末假日时前来游憩、骑乘自行车。","text2":"琵琶湖可以形成湖泊是因为什么?","label":1} {"text1":"乔治·贝瑞·奥图尔(George Barry O'Toole),生于美国俄亥俄州,是一名天主教本笃会神父,天主教激进派联盟(Catholic Radical Alliance)创始者,天主教辅仁大学首任校长。他是「基于宗教主张拒服兵役」(Conscientious objector)的重要阐释者,其神职生涯始自于路易斯安那州的教区,亦曾于第一次世界大战时任美军神父。他在圣文森学院(St. Vincent College)、宾州的拉筹伯大学(Latrobe University)与薛顿丘大学(Seton Hill University)教授哲学,也是辅仁大学的首任校长,以及杜肯大学(Duquesne University)哲学系的领导人。他是天主教激进派联盟(Catholic Radical Alliance)的创始者,也是匹兹堡最早的传教组织支援干部,尤其重要的是创立了匹兹堡圣文森之家慈善医院(Saint Joseph's House of Hospitality)。在1939年,他断定义战将是不可能的,因为「全体征兵制度的当代滥用」在很大程度上制造了无理的战争,此后他尝试在参议院听证会召开之前反对Selective Training and Service Act of 1940--一场悬而未决的征兵制度会议。","text2":"乔治·贝瑞·奥图尔,生于哪里?","label":1} {"text1":"韩国围棋联赛地区性围棋对抗赛改名韩国梦幻联赛,只有6支队伍,总规模为3.6亿韩元改名韩国围棋联赛,发展到了有8家企业俱乐部参加,总预算达到10亿韩元。每队4人共32人中,2003年度奖金排名前16名的棋手自动获得参赛权,其他16个名额由其余的180名棋手竞争获得。此外还设立了1000万韩元的最佳棋手奖和300万韩元的多胜奖。分为季前赛、常规赛和季后赛三个阶段。奖金总规模15亿韩元,从4月28日起开始正式比赛。冠名“KB国民银行”,常规赛棋队排名将依次按照积分--胜局数--两队间胜负--两队间比赛净胜局--主将至第5台各台次胜局数的累计情况来确定。  1、BGF retail队 主教练:白大铉九段  正规:姜东润九段、李志贤五段、李元荣五段、柳珉滢四段、崔精五段  次级:李昌锡二段、洪茂镇初段  2、tbroad队:主教练:李相勋九段(大)  正规:朴廷桓九段、李东勋五段、金升宰六段、姜儒泽七段、朴珉奎四段  次级:金东昊五段、李鱼德东初段  3、韩国物价信息队 主教练:韩钟振九段  正规:元晟溱九段、白洪淅九段、安国铉五段、朴承华七段、韩态熙五段  次级:洪旼杓九段、宋泰坤九段  4、SK能源队: 主教练:崔圭丙九段  正规:朴永训九段、安成浚六段、闵详然四段、李泰贤六段、姜升旼四段  次级:黄宰渊三段、崔显宰二段  5、新安天一盐队主教练:李相勋九段(小)  正规:李世石九段、赵汉乘九段、睦镇硕九段、申旻埈三段、李浩范五段  次级:安正己初段、朴炫洙初段  6、Kixx队:主教练:金荣桓九段  正规:金志锡九段、尹畯相九段、许映皓九段、金起用七段、崔宰荣初段  次级:宋知勋初段、吴长煜二段  7、浦项队: 主教练:金成龙九段  正规:崔哲瀚九段、罗玄六段、卞相壹四段、尹灿熙五段、柳秀沆四段  次级:金民浩二段、朴材根初段  8、正官庄皇真丹队:主教练:金荣三九段  正规:申真谞五段、李昌镐九段、金明训三段、韩升周二段、朴进率七段  次级:洪基杓七段、李炯珍三段  华城koriyo队:主教练:李廷宇九段  正规:李映九九段、洪性志九段、金庭贤五段、朴正祥九段、安祚荣九段  次级:高根台九段、朴河旼初段","text2":"韩国围棋联赛地区性围棋对抗赛一开始改成什么名字?","label":1} {"text1":"梅芳姑是金庸小说《侠客行》里头的人物。梅芳姑是丁不四和梅文馨的私生女,习得丁梅两家武学,年纪轻轻就成为梅花拳掌门人,除此也善于文学和烹饪,加上长相美丽,是个才貌出众的女强人。梅芳姑心仪上清观门人石清,无奈石清却钟情于虽然温柔美貌,但却样样不如梅芳姑的师妹闵柔。梅芳姑失恋后上门报复情敌,将年幼的石中坚夺去,石清夫妇后来看到被以为是石破天的死婴,把梅芳姑视为杀子仇人决计雪恨。梅芳姑则隐居在熊耳山枯草岭,为了泄愤,自毁容貌。根据后文推断,石破天极有可能就是被夺去的石中坚,被梅芳姑收为养子后改名为狗杂种,而且梅芳姑对他态度十分恶劣,幸好石破天本性善良,反而对她十分孝敬。几年后的某天梅芳姑不知何故外出许久未归,下山寻母的石破天因此开始涉入江湖。后来寻女的梅文馨在侠客岛上得知梅芳姑居处,辗转被史小翠告知石清夫妇,石清夫妇、助拳的雪山派众人和不知情的石破天跟著前往讨公道时,才知道梅芳姑和石破天是母子关系。石清向梅芳姑表达出他为何选择闵柔的原因后,梅芳姑伤心下自杀身亡。梅文馨抱起梅芳姑的遗体,露出点有守宫砂的手臂,众人发现梅芳姑仍是处子。","text2":"梅芳姑是怎么死的?","label":1} {"text1":"尚真(;)是琉球国第二尚氏王朝第三代国王、第九代琉球国王,1477年(成化十三年)至1526年(嘉靖五年)在位。他是第二尚氏王朝第一代国王尚圆王之子。童名真加户樽金(),神号于义也嘉茂慧()。尚真未即位时被封为久米中城王子,1477年接受叔父尚宣威的禅位,并派遣长史梁应、使者吴是佳、通事梁德等至明朝,以尚圆王讣告,请求袭封为琉球国王。明宪宗遂在1479年(成化十五年)派兵科给事中董旻、行人司右司副张祥前往琉球,册封尚真为王。尚真王在位期间,不断进行对外扩张。1500年平定了八重山群岛的远弥计赤蜂之乱。1506年相继征服了久米岛按司势力和具志川按司势力,在1522年镇压「鬼虎之乱」,征服与那国岛。此后琉球的统治势力扩张至宫古、八重山,基本确定了琉球南部的版图。尚真王统治的50年被认为是琉球历史上最强盛的时期,确立了琉球的官员品秩、朝仪制度、神官制度、赋税制度、行政划分,扩建了首里城,废除了殉葬习俗,并于1526年召各按司赴首里居住,禁止私人拥有兵器,加强了中央集权,此后琉球进入稳定发展的时期。琉球群岛著名的建筑圆觉寺(1492年)、玉陵(1501年)、圆鉴池(1502年)、辩财天堂(1502年),都建于尚真王时期。用于国王处理政务、接待册封使的首里城北殿(1508年),也是在这一时期建造的。","text2":"尚真王期间建立了琉球群岛哪些比较著名的建筑?","label":1} {"text1":"在计算机显示器的图形用户界面上,工具栏放置了界面按钮、图标、菜单或其它输入\/输出元素。早期的工具栏样式和功能由程序员定义,因此带按钮的工具栏与一排按钮之间没什么区别。大部分现代程序和操作系统,允许终端用户根据个人需要自定义工具栏,对工具栏中的按钮等项目进行添加、删除和调整位置。例如GNOME和KDE桌面环境的面板就是很好的自定义工具栏,这些面板的功能从应用程序的可扩展菜单和按钮、窗口列表、通知区域、时钟和资源监控到音量控制以及天气预报控件。有些应用程序,如图形编辑软件,允许工具栏分离并在窗口或其它工具栏之间移动。工具栏在办公软件套装上很常见,如OpenOffice.org、图形编辑软件以及网页浏览器如Inkscape和Mozilla Firefox。浏览器的第三方工具栏,通常增加了一些方便终端用户操作的功能选项。浏览器使用自己的功具栏进行基本的浏览导航(后退、停止、刷新),而外部工具栏提供附加功能(附加的搜索框、表格填写、链接到热门站点等)。常见第三方浏览器工具栏有:虽然第三方工具栏通常只提供功能给用户,却有可能给将缺陷引入相关浏览器。更有一些工具栏专为此目的而生。这些顽固的特洛伊木马程序伪装成搜索栏,除了用广告轰炸用户以外,还为其它病毒和间谍软件进入电脑大开方便之门。这种工具栏的另一个共同特点是,不能用相关浏览器或操作系统提供的传统办法禁止或卸载。但是,许多浏览器工具栏是无害的。","text2":"工具栏在哪些办公软件套装上很常见?","label":1} {"text1":"斯蒂芬岛异鹩(\"Xenicus (Traversia) lyalli\")是纽西兰斯蒂芬岛的一种雀,主要吃昆虫。牠们都是岛内的居民,且已经灭绝。斯蒂芬岛异鹩的学名是为纪念灯塔的助理David Lyall,因为是他发现这种雀鸟的。原先斯蒂芬岛异鹩是分类在独立的\"Traversia\"中,这个属是为纪念Henry H. Travers,因他购买了多个标本,而后来牠们被分类在刺鹩属中。牠们并非真正的鹪鹩科,只是一种外表相似的原始雀鸟,属于刺鹩科。根据考古学的发现,斯蒂芬岛异鹩是分布在纽西兰的主岛中。牠们的消失可能是因波利尼西亚鼠的掠食。但是牠们不懂得飞行,如何能跨越3.2公里的海洋到达斯蒂芬岛就不得而知了。流传较广的有关斯蒂芬岛异鹩灭绝的原因常是错误的,如传说于1894年,被灯塔看更的猫所消灭,莱昂内尔·沃尔特·罗斯柴尔德(Lionel Walter Rothschild)指一只猫就杀死了所有的斯蒂芬岛异鹩。根据历史纪录,蒂芬岛异鹩并非由守灯塔人所携带的单一猫只灭绝,而是由守灯塔人所饲养的家猫逃逸后繁殖为野化家猫群体,从而掠食并于1895年消失的。另外,岛内原初都是森林,因开发而令牠们失去栖息地也是成因之一。斯蒂芬岛异鹩只有15个标本,分别如下:","text2":"导致斯蒂芬岛异鹩灭绝的原因可能是什么?","label":1} {"text1":"丹北镇是江苏省丹阳市老北门外的一中心镇,2014年由原来的埤城镇、后巷镇、新桥镇合并成立。埤城位于江苏省丹阳市老北门外一中心镇。全镇面积73.5平方公里,人口3.8万余人。北部紧邻镇江大港新区,东接后巷镇,南邻云阳镇,西接丹阳市经济技术开发区。境内的水晶山,范围约4平方公里,主峰海拔166.1米,是全市最高峰。泰山水库为丹阳蓄水量最大的水库,水质极好。埤城原名卑林,三国时为东吴孙权养马之地,明嘉靖年间,此地曾设营寨、筑长城,以防倭寇,因此得名“埤城”。境内有南朝齐代齐宣帝萧承之永安陵、齐高帝萧道成泰安陵、齐景帝萧道生修安陵和齐废帝鬰林王萧昭业陵4座,今存石刻6尊,其中天禄、麒麟各2尊、辟邪2尊,均为全国重点文物保护单位。S338纵向、通江大道和丹界公路横向贯穿镇区,宽阔的硬质路面通向每个自然村。与丹阳市、镇江市有公交化便利交通。紧邻沪宁高速公路,最近处距离丹阳入口处仅2千米。泰州长江大桥和规划中的五峰山过江通道建成以后,丹北将成为沟通长江南北、江苏东西的交通枢纽。埤城工农业都很发达,2010年全镇实现GDP28亿元。主要工业企业有沃得集团、晶谷集团、华宇等企业集团,其中沃得精机已在新加坡上市。","text2":"丹北镇位于什么地方?","label":1} {"text1":"在南非,每年有10,000人死于道路意外。在居民区,时速限制为50或60公里每小时,在公路时速限制为100公里每小时,国道/高速公路时速限制为120公里每小时。在1996年,国家的公路总长358,596公里,当中59,753公里(17%)已铺平(包括1,927公里是快速公路)。在2000年,南非铁路网总长20,384公里,所有都是窄轨。20,070公里是1.067米宽(9,090公里被电气化),其他的314公里是0.610米宽。现可行性研究是建设720公里标淮轨,由约翰内斯堡至德班双层车箱列车。铁路网会连结博茨瓦纳、莱索托、纳米比亚、斯威士兰和津巴布韦。铁路网会连结莫桑比克是正在复修中。南非主要港口是开普敦、德班、东伦敦、莫塞尔贝、伊利莎白港、里查兹贝和萨尔达尼亚湾。在2006年,有新的港口开放:Ngqura 和库哈。库哈距离伊利莎白港东北方20公里。管理和运作港口设施,是由两家公司:国家港务局和南非港口行动。南非输油管总长931公里,1,748公里是石油运输,322公里是天然气。南非曾经拥有电车系统,但最后系统(约翰内斯堡)于1961年关闭。","text2":"南非的主要港口有哪些?","label":1} {"text1":"微指令的编译方法是决定微指令格式的主要因素.考虑到速度,成本等原因,在设计计算机时采用不同的编译法 .因此微指令的格式大体分成两类:水平型微指令和垂直型微指令。一次能定义并执行多个并行操作微命令的微指令,叫做水平型微指令.水平型微指令的一般格式如下:控制字段 判别测试字段 下地址字段按照控制字段的编码方法不同,水平型微指令又分为三种:一种是全水平型(不译法)微指令,第二种是字段译码法水平型微指令,第三种是直接和译码相混合的水平型微指令。微指令中设置微操作码字段,采用微操作码编译法,由微操作码规定微指令的功能,称为垂直型微指令。垂直型微指令的结构类似于机器指令的结构.它有操作码,在一条微指令中只有l-2个微操作命令,每条微指令的功能简单,因此,实现一条机器指令的微程序要比水平型微指令编写的微程序长得多 .它是采用较长的微程序结构去换取较短的微指令结构。","text2":"什么是水平型微指令?","label":1} {"text1":"《时代漫画》(Modern Sketch)是一份民国时期的漫画杂志,于1934年创刊至1937年因抗战终刊。出版者为时代图书公司,创刊人为漫画家张光宇和鲁少飞。该杂志上刊登传统的讽刺漫画、都会写生和连环漫画等,作为民国时期漫画传播的一大平台,集结了一大批当时有名的漫画家如鲁少飞、张光宇、张正宇、曹涵美、叶浅予等,而廖冰兄、丁聪和华君武等也在刊中崭露头角。1934年张光宇与邵洵美组成时代图书公司,创办了《时代漫画》,由鲁少飞担任主编,于1937年终刊。由于当时正值日本进攻中国,因此书刊除了发表都会百态以外还发表宣传抗日的作品。以单幅的漫画为主,多为讽刺漫画,都会写生,人物速写。画面配上文字,以风趣幽默取胜。鲁少飞、张光宇、张正宇、曹涵美、叶浅予、张乐平、胡考、张仃、特伟、丁聪、华君武、黄尧、廖冰兄等。","text2":"《时代漫画》分别是什么时候创刊和终止?","label":1} {"text1":"伯克尔坑是全新世撞击工作群发现,被假设为撞击坑的一个海底地形。它们认为是一颗彗星或陨石在近代(西元前2,800-3,000年)的撞击事件所造成的,估计直径有30公里(18英里),因此比陨石坑大25倍。它的位置在印度洋的南部,马达加斯加的东部和西澳大利亚州的西方,就在西南印度洋脊。在2006年,同一个小组使用存在于澳大利亚和马达加斯加的史前V形沙丘,以三角测量,测量出它的位置和证实期存在。但是这些V形沙丘是由海啸形成的理论已经受到地质学家Jody Bourgeois和R. Weiss的挑战。使用一个电脑模型来模拟海啸,他们认为风积过程更符合这种结构;其他的地球科学家也质疑这些沙丘是否起源于海啸。伯克尔坑位于,位于印度洋,并且在海面下3,800公尺(12,500英呎)。还没有利用辐射分析伯克尔坑内的沉积物。全新世撞击工作群认为这个坑是在5,000年前(西元前2,800-3,000年)形成的,当时正是地质年代的全新世。当一颗彗星撞击海洋,和那巨大的大海啸创造的沙丘,可以指出坑穴的方向。据报告,在临近坑穴的附近和关联影响的喷出物或热水沉淀物有异常的方解石(CaCO)晶体、半透明的碳小球体、玄武岩的玻璃和天然的金属碎片(自然铁和镍)。在如此深的坑穴,海水会快速的掩埋和保存这些晶体,而不管这些方解石是否不饱和。不只是圣经,许多古老文字记载的各种各样文化都有大洪水,可能这些传说都与此一事件有所联系。","text2":"伯克尔坑是什么?","label":1} {"text1":"素面是中国、日本、朝鲜、琉球传统面食,在中国习惯上称之为“挂面”,用小麦面粉制成。与乌龙面一样,素面规格严谨,面身直径通常要求小于1.3毫米。素面味冷而清,可浸于肉汤或丸子中,亦可下至酱油中,而不浇于其他食物之上。酱油通常是鲣海鲜味,可以与大葱、生姜以及青蒜。面用热水煮熟后,使用流动的冷水降温,以使面更加爽滑可口。这样做出的冷却的素面是日本夏季面料理的代表。一到夏季日本各酱油和食品生产商都会生产大量的“素面用”调味剂。此外,荞麦汁比砂糖、味醂更受青睐。关西地方的冷面()一般会添加火腿和黄瓜。药味素面则采用葱、山葵丝、生姜丝、海苔和胡麻等材料。流水素面()是日本一种放在竹制的樋里漂流著,用筷子夹取的素面。可算得上是夏天的风物诗。在鹿儿岛县和宫崎县的一些地方,这也叫「そうめん流し」。发祥地是宫崎县的高千穂峡和鹿儿岛县指宿市的唐船峡。鹿儿岛各地也有专门制作出来以便在餐桌上使用的流水素面器具。鲷素面是濑户内地方壹岐岛的料理。沙拉素面或者素面沙拉,是在素面上加西红柿、生菜等拌成的沙拉的料理,也加蛋黄酱做调味。","text2":"素面通常是如何食用的?","label":1} {"text1":"瓦维尔主教座堂()是位于波兰克拉科夫瓦维尔山上的教堂,也是天主教克拉科夫总教区的主教座堂。其正式名称为圣达尼老圣文策老圣殿总主教座堂()。在过去9百年的时间里,瓦维尔主教座堂是波兰的国家圣殿,波兰历代君主在此举行加冕仪式,14世纪以后的波兰历代君主也多安葬于此。1946年11月2日,教宗若望保禄二世在教堂的地下室主持了他晋铎后的第一台弥撒。2010年4月,坠机遇难的波兰总统莱赫·卡钦斯基及夫人玛丽亚·卡钦斯卡亦安葬于此。教堂南端的西吉斯蒙德小教堂(Kaplica Zygmuntowska),被誉为“意大利以外最纯净的文艺复兴建筑”,兴建于1517-33年。方形的小教堂有着金色的圆顶,安葬着国王西吉斯蒙德一世和西吉斯蒙德二世。","text2":"瓦维尔主教座堂的正式名称是什么?","label":1} {"text1":"脂溢性脱发又称雄激素源性脱发或早秃,是指头皮部出现油脂分泌旺盛,头发倍感油腻,在中医中称为柱发癣。多见于男性,或脑力劳动多者。现代医学研究发现,男性脱发主要是遗传造成。机理为:男性油脂分泌旺盛与和雄性激素有关,男性脱发的根源是从睾酮衍生出来的二氢睾酮。大约5%的睾酮会被一种叫5α还原酶的催化剂转化成二氢睾酮,二氢睾酮量虽少,威力却大,和雄性激素受体结合的能力是睾酮的十到几十倍。它们在前列腺和头顶的头皮这两个地方最为活跃,由于遗传的缘故,有些人天生就对它们特别敏感。在前列腺,二氢睾酮量和雄性激素受体的结合刺激细胞的增殖,能导致前列腺肥大。而在头皮,二氢睾酮量和雄性激素受体的结合让毛囊萎缩,毛囊的生长周期变得越来越短,长出来的头发越来越细,最终,头发细得肉眼看不见或完全消失。因为头皮油脂分泌过剩,细菌或真菌大量滋生,可能导致头发出现脂溢性皮炎,后伴有脱发症状。","text2":"男性脱发主要是由什么造成?","label":1} {"text1":"三色龙胆(学名:)为龙胆科龙胆属的植物,是中国的特有植物。分布于中国大陆的青海、甘肃等地,生长于海拔2,200米至3,200米的地区,多生在湖边漫滩草地、河滩草地、沼泽化草甸、林下以及路边,目前尚未由人工引种栽培特征一年生草本,高3-5厘米,茎紫红色,下部光滑,上部具细乳突,在基部多分枝,枝铺散,斜上升。基生叶大,在花期枯萎,宿存,倒卵圆形,长7-11毫米,宽5-6.5毫米,先端钝圆,具短小尖头,边缘光滑,软骨质,两面光滑,中脉软骨质,在下面突起,叶柄膜质,光滑,长2-3毫米,连合成长0.5毫米的筒;茎生叶对折,疏离,长于或等于节间,矩圆披针形至线状披针形,长4-7毫米,宽1.5-2.5毫米,愈向茎上部叶愈长,先端钝至渐尖,边缘膜质,狭窄,平滑,两面平滑,中脉在下面呈脊状突起,叶柄背面有细乳突或光滑,连合成长2-2.5毫米的筒。花数朵,单生于小枝顶端;花梗紫红色,具细乳突,长2-5.5毫米,藏于上部叶中;花萼倒锥形,长7-8毫米,光滑,裂片披针形或三角形,长2.5-3毫米,先端急尖,边缘膜质,狭窄,平滑,两面光滑,中脉绿色或白色,在背面呈脊状突起,并向萼筒下延,弯缺窄,截形;花冠常闭合,上部淡蓝色或蓝色,下部黄绿色,外面具绿色宽条纹,筒形,长11-13毫米,裂片卵形或卵状椭圆形,长1.2-2毫米,先端钝,具短小尖头,褶卵形,长1.5-2毫米,先端钝,具极短小尖头,全缘或有不整齐细齿;雄蕊着生于冠筒中部,整齐,花丝丝状钻形,长4-4.5毫米,花药直立,稀微弯曲,矩圆形,长约1毫米;子房狭椭圆形,长2.5-3.5毫米,两端渐尖,柄粗壮,长2-2.5毫米,花柱线形,连柱头长约2毫米,柱头2裂,裂片矩圆形。蒴果外露,倒卵状矩圆形,长5.5-6.5毫米,先端钝圆,具宽翅,两侧边缘具狭翅,基部渐狭成柄,柄粗壮,长至15毫米;种子黑褐色,矩圆形,长1.1-1.4毫米,表面具细网纹。花果期6-8月。","text2":"三色龙胆的花冠有什么特点?","label":1} {"text1":"马来闭壳龟(学名:)又名安布闭壳龟,为龟科闭壳龟属的爬行动物。分布于印度尼科巴群岛、孟加拉、缅甸、泰国、寮国、柬埔寨、越南中部及南部、马来西亚西部、新加坡、菲律宾及印尼,并可能分布至中国广东及广西,常见于平原地区的沼泽、湿地、池塘、河流中的水荡以及水稻田等水流缓慢、底质松软的水域。该物种的模式产地在印度尼西亚。幼体几乎完全水栖,成体偏好陆栖。草食性为主,吃水生植物和真菌,也吃一些昆虫。如要以马来闭壳龟作为宠物龟,饲养者要深入研究饲养方式,并要做好一般护理和照顾。考虑以马来闭壳龟作为宠物时,要了解它是一种热带、非冬眠的物种,比许多其他箱龟需要更温暖的水(约82华氏度)。完全成熟的成龟通常需要半水陆的饲养环境。闭壳龟马来闭壳龟比其他箱龟更能适应在水中生活。不幸的是,这往往导致被错误地放在陆地中饲养。虽然他们是拙劣的游泳者和比其他半水栖龟需要较浅水,但他们往往会比较喜欢在水中。在野生环境中,他们往往会住在几乎完全在水晒太阳和产卵,他们甚至可以在水中交配。","text2":"考虑以马来闭壳龟作为宠物时,需要了解什么?","label":1} {"text1":"汪大燮内阁成立于民国11年(1922年)11月29日,结束于民国11年(1922年)12月11日。汪大燮成为国务总理,是因为黎元洪大总统利用总统特权提出;汪大燮内阁的阁员也是黎元洪提名的,而内阁名单一出台,就惹来曹锟、吴景濂、张伯烈等人的反对,因为汪大燮、许世英、高恩洪等都与罗文干案关系密切。曹锟的反对声明再一次得到各地都督的相应,使汪大燮进退两难,内阁阁员纷纷托辞不肯赴任,最后只有两名阁员出席,其它各部均由次长代理。吴佩孚也再一次为罗案而通电认错。12月5日,众议院通过决议案查办高恩洪、罗文干涉嫌舞弊渎职受贿一案;黎元洪也通电解释组织汪大燮内阁是为了让接收青岛时可以有国务总理的副署,并且表示同意由津保派的张绍曾组阁。12月11日,汪大燮请辞。他本来只和黎元洪约定10日的总理期限,由于短短10日内受到各方不断的非难,届满之后,汪不肯延期,他敦请王正廷代理总理职务。","text2":"为什么汪大燮能成为国务总理?","label":1} {"text1":"梅西尔72(也称为M72或NGC 6981)是在宝瓶座的依个球状星团,在1780年8月29日被皮埃尔·梅香发现。梅西尔随后也在10月4日和5日观测到,并将之编入星表中的第72号。当时这两位都认为他是一个暗淡的星云,而不是现在所认知的球状星团。即使使用10吋(25公分)口径的望远镜观察这个星团,依然只能看出模糊与微弱的影象,但使用20英吋的望远镜就能提高解像力。M72距离地球大约53,000光年,与银心也有一段的距离。但也有资料认为M72与地球的距离是62,000光年,直径为42光年。虽然一般的星团都有一些最老年的恒星,但M72拥有一些蓝巨星,因而被认为是一个年轻的星团。","text2":"梅西尔72是哪个星座的星团?","label":1} {"text1":"正鳄鲬,又称鳄形牛尾鱼、鳄鲬为辐鳍鱼纲鲉形目牛尾鱼科的其中一种。本鱼分布于南日本、印度洋、西太平洋、红海、东非、索罗门群岛、澳洲等海域。水深5至100公尺。本鱼鳃盖主骨边无皮瓣;眼前骨中部有1锐棘。背鳍、臀鳍均有鳍条11枚,背鳍硬棘部具宽暗缘,软条部具暗色小点。体延长,平扁,向后渐细尖,纵剖面略呈圆柱状。体呈暗紫灰色,有4、5条宽横带,头顶、背部体侧有若干小黑点散布。第一背鳍边缘黑色,第二背鳍有2至3列黑点,尾鳍颜色多变,通常具有多条暗色斑纹或成列的斑点。;臀鳍鳍膜暗色;胸鳍下半部暗色,并散有黑色小点。体长可达50公分。属底栖鱼类,喜砂泥质海底,以小鱼、甲壳类为食。食用鱼,肉质美味,以油煎、清蒸为宜。","text2":"正鳄鲬纵剖面略呈什么形状?","label":1} {"text1":"帕特里夏·派伯敏特·佩蒂·雷查德(),漫画人物,是漫画家查尔斯·舒兹从1950年代起连载的漫画作品《花生漫画》中的重要角色之一。派伯敏特·佩蒂是主角查理·布朗的同级生,不过不同班。运动少女,个性豪放随和,上课时必打瞌睡,因此常遭老师责骂。但对于运动则极有天分,自己组建棒球队,在球季之间无往不利,与查理·布朗那支只败不胜的球队形成对比。同时也有从事网球、划艇、榄球、花式溜冰、游泳等运动的记录,皆成绩斐然。平日的形象是一头乱发,基本上是随意的半长发。脸大,鼻子大,有雀斑,阳光笑容。习惯穿绿色衬衫和短裤,脚踏破旧凉鞋。个性直来直去,有话直说,纯粹的户外系。但也有心思细腻的一面,乐意照顾他人,因此颇留得住朋友。被查理·布朗那罕见的温顺性格所吸引,暗中颇怀好感,平时对查理·布朗非常照顾,表面上是哥们关系。将查理布朗昵称作“查克”。与玛西是死党。虽然一文一武、一动一静,思维方式亦南辕北辙,却相处融洽。基本上是佩蒂负责捧哏\/发呆役,玛茜负责吐槽。由于相识时,佩蒂是玛茜的夏令营领队,所以玛茜叫佩蒂作“SIR”。对史努比的运动能力非常折服,是英雄惜英雄的关系。她一直误会史努比是一个人,称他是查理·布朗的大鼻子好友。","text2":"派伯敏特·佩蒂个性如何?","label":1} {"text1":"西澳州政府体-{制}-是隶属于澳大利亚宪法的政府体-{制}-,成立于1890年,百余年来经历多次修订。自1901年起西澳大利亚州成为澳大利亚联邦(Commonwealth of Australia)的一个州,与联邦的关系亦首先尊重西澳宪法。根据澳洲宪法,西澳除了被联邦分割部分立法权和司法权,其它方面保持完全独立。西澳是根据西敏制的精神——英国式议会政治体-{系}-,采三权分立。西澳州议会设于首府珀斯,政府功能方面由州议会负责监督。往昔隶属珀斯市的各市区今划分为三十余个地方政府(LGA),由于政治、经济、自然环境等皆有密切的关系,所以通称为「珀斯大都会」。西澳州政府官方网页","text2":"西澳州议会设于何地?","label":1} {"text1":"天文学计算研究所(德语:\"Astronomisches Rechen-Institut\",缩写为\"ARI\"),是位于德国海德堡的一所科学研究机构,现在隶属于海德堡天文中心和海德堡大学,在合并入海德堡天文中心之前隶属于德国巴登符腾堡州。天文学计算研究所是德国天文学家哥特弗里德·基尔希(Gottfried Kirch)于1700年创建于柏林。二战后的1945年,研究所被美军搬迁到美军在海德堡的总部附近,搬迁工作于研究所300周年庆典2000年5月完毕。自2005年1月1日起,研究所正式并入海德堡天文中心,天文中心的另外两个组成部分是理论天体物理研究所(ITA)和州立天文台(LSW)。历史上,研究所一直为德国的授时机构。直到今天,还仍然承担着整理天文学文献、出版星表和确定天文参照系FK4、FK5的恒星天文测量学的任务。今天的天文学计算研究所,不仅先于出版恒星星表,同时在众多的天文学研究领域贡献力量。目前研究所涉及到的研究方向包括引力透镜、星系演化、恒星动力学和宇宙学,同时研究所加入了包括GAIA在内的众多空间天文项目。研究所有收集和整理出版天文星历表、文献和发布时间、坐标系统的任务。其出版物包括:","text2":"自什么时候开始研究所正式并入海德堡天文中心?","label":1} {"text1":"「天神小子」(),是日本快乐快乐月刊从2007年1月号到2010年9月号连载的的作品。台湾快乐快乐亦从2007年10月号开始连载。是作者预告「能够超越『可乐小子』的作品」。主角伽利略以成为天神为目标,挑战高第天神学校的入学考试。人物名多是以历史上出名的人物为范本(夹注号内为受取材的历史人物)。在4班待遇最差,是低于标准的放牛班。可是由于大家被期待的标准大多相同,所以是天神学校中十分有趣的班级。G技:G铁拳,W.G铁拳、神之G铁拳D技:D铁拳GD技:GOD铁拳G技:女神的吐息,女神的喷嚏G技:岩窟王G技:光之翼G技:毕沙罗的墙壁G技:热焰制敌拳G技:神之手,凤凰箭雨G技:枭枭烟雾,突刺之烟,G.烟怒,G.烟怒陨石G技:荆棘暴龙、荆棘暴龙猛冲、荆棘暴龙大雷疱G技:人间大砲、人间大砲MAXG技:绝对指挥棒、雨中的镇魂曲、艾西莫夫最终乐章G技:极限巨影G技:弹簧式G技:乙女的束缚G技:分身术G技:G技:石油危机G技:动物变化G技:超速射线G技:LA.SHA-BORZ现任天神。天神学校的教师。1班的G技教师。G技:注意我无天神大人的宠物。D技:千斩鬼牌、恶魔之手恶魔王,杀死了伽力略的双亲,以笛福的身份在恶魔学校最强5人之中生活。D技:暗黑流星","text2":"「天神小子」,是指什么?","label":1} {"text1":"中国人民解放军总政治部,简称总政治部、总政,是中国人民解放军曾经存在的的政治工作机关,由中华人民共和国中央军事委员会领导。其前身是1930年创建的中共中央军委总政治部,1954年正式改称中国人民解放军总政治部。总政治部负责中国人民解放军的宣传、思想政治、青年、组织、纪律工作,并负责保卫及对外联络工作。总政治部与总参谋部、总后勤部、总装备部合称为四总部。2016年1月撤销,改组成立中央军事委员会政治工作部。中国人民解放军总政治部,可追溯于1930年创建的中共中央军委总政治部,1954年正式改名为”中国人民解放军总政治部”。1966年,文化大革命爆发,总政治部受到严重破坏。1967年7月25日,林彪和江青等人提出“彻底砸烂总政阎王殿”后,总政治部主任、副主任等领导干部都被关押、批斗和审查,总政治部机关陷于瘫痪。1967年11月,军委办事组政工组,取代了总政治部领导。1968年10月,对总政治部实行军事管制。1969年10月,开始重新组建总政治部。1969年12月,总政治部恢复办公。1977年,总政治部增设中国人民解放军军事法院和中国人民解放军军事检察院。1978年中共十一届三中全会以后,总政治部得到了恢复和发展。总政治部根据中央军委的决定,调整了机构:设组织部、干部部、宣传部、文化部、保卫部、联络部、群工部、军事检察院、军事法院和办公厅、直政部、管理局。1982年,总政治部的机构又作了精简,总政领导班子再次进行调整,总政办公厅改为总政办公室。1985年,总政管理局和直政部合并为直工部,总政办公室改为总政办公厅。2016年1月,在深化国防和军队改革中,总政治部改为中央军委政治工作部。以下是2016年中国人民解放军总政治部撤销时的机构设置:所有派出机关均接受双重领导中国人民解放军总政治部除使用军旗和军徽外,还拥有自己的臂章,于2007年启用。臂章呈盾形,绿地,绘有八一军旗、两把交叉的步枪与两支交叉橄榄枝的图案,并在上方写有“中国人民解放军总政治部”的字样。臂章主要佩戴在总政治部人员的常服的左臂上。","text2":"总政治部的领导机构是什么?","label":1} {"text1":"沧龙亚科(Mosasaurinae)是有鳞目沧龙科的一科,生存于白垩纪晚期。在1967年,戴尔·罗素(Dale Russell)将沧龙亚科定义为具有以下特征:前上颌骨前方的喙小,或是没有、齿骨与上颌骨具有14颗以上的牙齿。第五、六、七对脑神经穿越耳后的两个孔、枕骨底部或基蝶骨没有基底动脉的血管沟、方骨的上镫骨突末端扩张、至少有31节荐前椎,多为42到45节、荐前的脊椎总长,长于荐后的脊椎总长、后端尾椎的神经棘长,形成尾鳍、四肢关节的表面平顺、距骨与腕骨高度骨化。在1997年,G. L. Jr. Bell出新的沧龙超科的系统发生学研究,仍将沧龙亚科保持为一个演化支的地位,但将倾齿龙族改归类于沧龙亚科,并新建圆齿龙族。沧龙亚科的化石可发现于各大洲,除了澳洲、南美洲以外。沧龙亚科首次出现于土仑阶,并持续生存到马斯垂克阶末期。沧龙亚科的体型差距大,\"Carinodens\"的身长约3到3.5公尺,硬椎龙的体长约7公尺,而霍氏沧龙的身长可达17公尺,是最长的沧龙类之一。许多沧龙亚科是鱼食性动物,或者猎食对象广泛,以鱼类或其他海生爬行动物为食。其中的圆齿龙族演化出适合压碎的牙齿,可咬碎菊石或海生乌龟的硬壳。","text2":"沧龙亚科的化石在哪些大洲有过发现?","label":1} {"text1":"三司官,亦称法司,是琉球国朝廷的最高执政机构,也是这个机构所有官员的官职名称。琉球共设三司官三人,称号为「某某亲方」。三位三司官分别监督「用意方」(管理国家财产和山川的治理)、「给地方」(管理给与役人俸禄和旅费)、「所带方」(管理租税和国库的出纳)三个物奉行所。三司官的官阶由正一品至从二品,这是琉球士族中最高的官阶,相当于中国和日本的三公或三师。虽然在三司官之上还有摄政,但摄政一般不参与政治活动,因此三司官是琉球国实质上的最高级别官员。三司官由王族、上级士族共约二百余名投票选举产生。王族成员只有选举权,没有被选举权。所有三司官的候选人必须是出生在首里城的上层阶级,拥有较高的道德修养,而且精通文学、伦理、经济、治国的儒家学者。这与中国古代的科举制度有些相像,但三司官的选举没有科举考试那样严格。第二尚氏王朝时期,翁氏、马氏、毛氏池城家和毛氏丰见城家四家三司官辈出,时人将他们与王族的分家向氏并称为「五大姓」、「五大名门」。据《中山世谱》记载,三司官一职早已有之,原称「阿司多部」,至明末,改称「法司」。但三司官拥有较高的政治权力始于第二尚氏王朝时期。1556年,年幼的尚元王登上王位。因尚元王是哑巴,群臣推举德高望重的大臣毛龙唫等三人代理尚元王管理国家。从此以后,三司官成为执政之臣。1571年尚元王死去,但三司官这一统治机构却被保留了下来。1609年萨摩入侵琉球之后,萨摩藩派兵监督琉球国政,琉球王的权力被完全架空。1611年,萨摩与琉球签订了《掟十五条》(),规定三司官一职只能由倾萨摩派人士出任。此后,三司官名义上是独立琉球国的高级官员,实际上已沦为萨摩藩统治琉球的代理人。1879年琉球处分,琉球被日本灭掉,改为冲绳县。三司官的权力被完全剥夺,但明治政府同意琉球国上层阶级维持他们的特权。华族令颁布后,原琉球的三司官都被列入日本华族。","text2":"哪本文献指出三司官一职早而有之?","label":1} {"text1":"郭咏燊(Kwok Wing Sun,),生于香港,是一名职业足球员,绰号「炮仔」。担任右后卫。现时效力于香港甲组足球联赛球队花花。郭咏燊除防守技术全面外,准绳的「飞铲」拦截更为其拿手好戏,队友李树燊曾表示郭咏燊的飞铲为自己最想得到的技术之一。加上在场上经常指挥和提点队友以及演出水平稳定,郭咏燊成了队中不可多得的成员。郭咏燊曾效力流浪、东方、和富大埔、晨曦、屯门。他效力大埔以来,郭咏燊水平非常稳定,演出亦拼劲十足。教练陈浩然曾表示郭咏燊为队中其中一名值得入选港队的球员。2009年6月6日,大埔于足总杯决赛中以4-2击败天水围飞马,升班三年后获得首个锦标。在甲组浮沉十载,郭咏燊表示当中有喜有悲,而十年转眼已过,等了十年终能一尝冠军滋味。而他亦感谢父亲,十年来差不多每一次他的比赛父亲都会到场观看。郭咏燊曾入选香港奥运足球代表队以及于2002代表港队参与釜山亚运足球。香港著名教练黎新祥曾表示,郭咏燊具有潜质,但因工作关系才令进度停留下来,未有把潜力完全发展。郭咏燊经营海鲜生意的父亲暱称为「阿炮」,因此郭咏燊便被称为「炮仔」。","text2":"郭咏燊的优势是什么?","label":1} {"text1":"杨守亮(),本姓訾,唐朝军事人物。杨复光之养子,改姓杨,早年为扈跸都将,官至山南西道节度使,全靠养伯杨复恭提拔。景福元年(892年),杨守亮与西川节度使王建在四川东部交战,史称梓州之战。大顺二年(891年),杨复恭失势,十一月八日,昭宗令天威都将李顺节等人逮捕杨复恭。张绾、杨守信奋死抵抗,九日禁军刘崇望前兵增援,杨复恭趁混乱之际前去兴元依靠守亮。静难节度使王行瑜,镇国节度使韩建,同州节度使王行约,秦州节度使李茂庄,争先上书,说杨守亮藏匿杨复恭。唐昭宗命李茂贞为招讨使,与邠宁节度使王行瑜合力进取兴元。李茂贞攻取兴元,杨守亮同杨复恭、杨守信、杨守贞等人奔逃阆州。干宁元年(894年)七月,杨守亮欲投奔河东节度使李克用,至华州,被韩建擒获。杨复恭跟韩建不和,对其辱骂,韩建大怒,将杨复恭、杨守亮一行人押解至长安。杨守亮请求活着被解送长安,希望能通过陈述先人功绩得免死,韩建答应,以槛车解送杨守亮,押送官吏用布帛绑缚他,用球堵住他的嘴。昭宗御延喜楼审问,杨守亮不能说话只能颔首,左右说杨守亮是服罪了,于是杨复恭、杨守亮等被执献太庙,斩于独柳下,枭首于市。","text2":"杨守亮的养父是谁?","label":1} {"text1":"尼古洛巴托总统国际机场(,),之前称科摩罗国际机场(),机场位于东帝汶帝力。因尼古·洛巴托在今日视同东帝汶国家英雄,之后东帝汶便将国际机场命名为「尼古劳·洛巴托总统国际机场」,除此之外帝力部分道路与总统府也使用其名字做为纪念。目前为止,机场跑道还无法起降比波音737或C-130运输机更大的飞机。但在2008年1月,由葡萄牙包机航空公司欧洲大西洋航空运营的从里斯本至帝力的直达航班,使用了架波音757执飞,该航班载有140位葡萄牙全国共和国卫队成员。在葡萄牙统治时期,包考机场有一条长于本机场跑道,曾经用作国际航班起降,但在1975年的印度尼西亚入侵东帝汶事件后,包考机场被印尼军方接管并停止民航运营。(按航空公司英文名称首字母顺序排列)","text2":"东帝汶为什么要将国际机场命名为「尼古劳·洛巴托总统国际机场」?","label":1} {"text1":"平泉车站()是一由东日本旅客铁道(JR东日本)的所经营铁路车站,位于日本岩手县西磐井郡平泉町平泉字泉屋。平泉是JR东北本线沿线车站之一,属JR东日本盛冈支社管辖范围,由一之关车站管理,设有绿窗口(),是个营运上由JR东日本子公司Jaster()代为经营的业务委托车站。平泉车站周围座落有包含中尊寺、无量光院遗迹与毛越寺在内,曾经在12世纪时繁荣一度的奥州藤原氏所留下之历史遗迹。2002年时,平泉车站以「与藤原三代时繁华一度的古都平泉相辉映,采用平安朝样式屋顶的车站站舍」()之评语,入选东北车站百选。对向式月台2面2线的地面车站。曾经2号月台对面设有3号月台,现在轨道已经拆去。","text2":"平泉车站位于哪里?","label":1} {"text1":"《我愿意为妳朗读》(德语:\"Der Vorleser\")是德国法律教授和法官本哈德·施林克于1995年撰写的小说。本书是第一本登上《纽约时报》畅销榜的德语书籍。本书1995年在德国出版,1997年由卡露·布朗·珍妮维(Carol Brown Janeway)所翻译的英语版本于美国发行。2008年电影《生死朗读》系根据本书改编。这个故事是个类比, 比喻德国二战后的世代在理解犹太人大屠杀所面临的困境。露丝·富兰克林写道:这本书特别针对的世代是贝托尔特·布莱希特所称的「后人」(德语:\"Nachgeborenen\")。《我愿意为妳朗读》一书是属于\"Vergangenheitsbewältigung\"(德语,字面意义:处理过去)的文类,\"Vergangenheitsbewältigung\"这类作品主要内容都是关于面对过去时所面临的矛盾与挣扎。《我愿意为妳朗读》探讨战后的世代该如何面对战争时期参与或目睹各种暴行的世代。随著二战时期的受害者以及目击者的消亡,那些血淋淋的记忆也渐渐淡去,因此20世纪末及21世纪初,犹太人大屠杀的文学作品越发探讨这类的问题。《我愿意为妳朗读》在各地广受好评,荣获许多奖项。德国《明镜》周刊评论此书堪称继君特·格拉斯的《铁皮鼓》后最为成功的德国文学作品。《我愿意为妳朗读》在德国热销50万册,并在2007年的一次电视民调上,被德国读者选为百名榜上最喜爱的书籍当中的第14名。此书在1998年荣获德国的文学奖 Hans Fallada Prize,也成为史上第一部荣登纽约时报畅销榜的德国文学。","text2":"本书的英语版是谁翻译的?","label":1} {"text1":"苏凡凌,生于1955年4月29日,台湾作曲家,台湾新竹人,华人女作曲家协会副主席,现任国立新竹教育大学 专任教授,行政院客家委员会委员。苏凡凌于2007年1月获得国立台北艺术大学音乐艺术(作曲)博士学位,不仅是台湾第一位获得国内大学音乐艺术博士学位者,也是台湾第一位获得国内大学艺术类博士学位者,在台湾音乐教育史及培养音乐艺术人才上具有指标意义。她曾获得奥地利市立维也纳音乐学院理论与作曲毕业文凭,亦于奥地利国立维也纳音乐学院电子音乐音响应用研究所毕业,另外也获得国立台湾师范大学音乐研究所作曲硕士学位。在国际舞台上,苏氏的作品经常在世界各地演出,如美国、加拿大、香港、德国、奥地利、波兰、立陶宛、马来西亚及中国北京、上海等地区。华人女作曲家协会副主席、国立新竹教育大学音乐学系专任教授、国立台湾艺术大学音乐学系兼任教授、辅仁大学音乐学系兼任教授、行政院客家委员会委员。国立台北艺术大学音乐艺术(作曲)博士,师事潘皇龙教授;国立台湾师范大学音乐研究所作曲硕士,师事陈茂萱教授;奥地利国立维也纳音乐学院电子音乐音响应用研究所毕业;奥地利市立维也纳音乐学院理论与作曲毕业文凭,师事Reinhold Portisch教授。多次受委托创作大型国乐曲《 菩提无语》、《 佛曰不可说》、《 一笑君王》 、《 粉墨登场》、客家风协奏曲《 我从山中来》。文建会委托创作合唱作品《 竹枝词》。","text2":"苏凡凌博士毕业于哪所大学?","label":1} {"text1":"阿拉伯文数字是指阿拉伯文形式的印度-阿拉伯数字。这种数字被大多数使用阿拉伯文的国家使用,这包括阿拉伯国家和大多其他西亚国家。这种数字是由印度在中世纪引入阿拉伯,当时已经有很大的改变。在中世纪时,北非也使用了这种数字。之后,这种数字又从北非引入意大利,进入欧洲国家,慢慢的成为当今通用的阿拉伯数字。阿拉伯语把这些数字称为“印度数字”( ')。Unicode把它称为阿拉伯-印度文数字(Arabic-Indic Digit)。现在世界上主要有两种阿拉伯文数字:同阿拉伯文不一样,当代的阿拉伯文数字一般从左向右排列。而在一些古典作品里是从右向左排列的。在埃及,“二”通常用另一种写法。","text2":"阿拉伯文数字被大多数使用阿拉伯文的国家使用,这包括哪些国家?","label":1} {"text1":"桂绍彬(),安徽省六安县人。中国人民解放军海军将领,开国少将。1931年参加红四方面军,历任独立第15团营部书记,红25军第73师第218团营部书记,第271团政治处秘书长,红31军第91师政治部秘书长等,并参加长征。抗战中任八路军第129师第386旅第771团司令部政治指导员,第772团政治处宣传股长,太岳军区政治部组织部部长,太岳军区政治部主任,晋冀鲁豫野战军8纵政治部主任,华北军区野战第一兵团8纵队政治部主任,陆军第60军政治部主任、副政委等。中华人民共和国成立后,他历任中国人民解放军海军航空学校政委,中南军区海军副政委兼政治部主任,南海舰队副政委、海军顾问,是中国海军航空兵的创建人之一。1955年,被授予海军少将军衔并荣获二级八一勋章、二级独立自由勋章、一级解放勋章。中共七大代表。2014年去世。","text2":"1955年桂绍彬被授予了哪些荣誉?","label":1} {"text1":"降解体(Degradosome)是存在于大多数细菌中的一种多亚基蛋白质复合物,其功能为参与核糖体RNA的加工以及信使RNA的降解,故又可称为RNA降解体。1994年,瑞士日内瓦大学的Henry Krisch所领导的实验室和英国剑桥大学的Christopher Higgins所领导的实验室分别发表文章提出了细菌中存在着一种可以降解RNA的蛋白质复合物,也就是降解体;他们是在对大肠杆菌的RNase E和PNPase的研究和鉴定过程中得到这一发现。降解体的主要成分为核糖核酸酶E(RNase E)、多核苷酸磷酸化酶(PNPase)、RNase螺旋酶B(RhlB)和烯醇酶(enolase)。","text2":"降解体的功能是?","label":1} {"text1":"铃木重秀(、1546年-1586年)是日本战国时代的铁炮佣兵习团杂贺众的头领,人称杂贺孙巿、孙一。为铃木佐大夫的儿子。石山合战时,杂贺众与本愿寺派均有多数人员参战,导致织田信长陷入苦战。重秀与下间赖廉并称为「大坂之左右大将」。在「本愿寺文书」有「铃木孙一于石山合战非常活跃」的文字记录。之后孙一为协助毛利军将军粮补给送进本愿寺,而前往播磨国,并于织田信长讨伐杂贺城时,率领铁砲僧兵队数千人据守,导致信长久攻不下。荒木村重反叛信长并据守有冈城时,也加入协防。后来其父死于藤堂高虎的计谋(一说),石山合战讲和交涉的阶段,「三下间」(赖廉、仲之、赖龙)的起请文中,也出现「杂贺一向宗」四宿老与「杂贺众」七人(包括重秀)等人连署的文字记录。在织田与本愿寺双方的文书中,对于重秀的军事才能都有很高的评价,并受到显如重用。后来本愿寺显如与信长讲和退出石山后,重秀投降信长。天正9年(1581年),信长与杂贺庄的土桥守重渐渐产生冲突,隔年天正10年(1582年)1月23日重秀派刺客将守重暗杀,同年2月8日取得完全胜利。天正12年(1584年)小牧长久手之战时,根来众・杂贺众等纪州势力多数跟随织田信雄、德川家康,但重秀为羽柴秀吉方,担任铁砲首领,领兵200。天正13年(1585年)跟随秀吉进攻纪州,3月25日担任使者前往太田城劝降,战后将儿子送至秀吉处当人质。此后史料不见重秀的踪迹,据考证可能死于大坂,儿子后来也担任铁砲首领,名为「铃木孙一郎」。","text2":"为什么信长久攻不下?","label":1} {"text1":"净闲寺()是位于日本东京都荒川区南千住的净土宗寺院。在吉原游廓附近,因无依无靠的妓女死后「投入寺」知名。因为地处「三轮」,所以也叫做「三轮净闲寺」。1655年(明历元年)创建,比吉原游廓(1657年)早2年。据说于吉原妓院街病死的妓女被送到净闲寺并冠以「某某卖女」的戒名埋葬,事实上没有这样的文字记载,近年的研究表明只有「自杀」、「胡乱发誓」、「逃跑」、「私通」、「吸食鸦片」等坏了吉原规矩的妓女才被会冠以「某某卖女」的戒名。这种情况下,死人被剥去衣服用破草席包裹扔到净闲寺。因为怕死人作祟,所以被当作猫狗处理,使其灵魂落入畜生道。吉原380年的历史中共有25,000名妓女埋葬与此。净闲寺的网页上说,净闲寺是安政大地震(1855年)埋葬大量遇难妓女之后才被称为投入寺的。东京都荒川区南千住2-1-12","text2":"净闲寺在哪里?","label":1} {"text1":"醉猿(\"Dionysopithecus\")是一属史前的灵长目,生存在中新世的中国及巴基斯坦。醉猿的化石最初是由李传夔于1978年在中国江苏的双沟发现,化石包括了一个左上颌骨连同几颗牙齿。由于当时李传夔是住在双沟的一间酒厂内,故将这些化石以希腊神话的酒神狄俄倪索斯来命名,并以双沟为种小名。李传夔同时间亦发现了宽齿猿的化石。后来于1990年发现了另一些醉猿化石,并名为东方醉猿。在巴基斯坦亦有发现一些醉猿的遗骸。醉猿最初被认为是属于原康修尔猿科,但更多的化石发现显示醉猿的牙齿更像树猿,故被分类在上猿科中。醉猿的化石属于中新世早期,比原康修尔猿的更早。不过学者一般都不认为醉猿就是现今长臂猿的祖先,而是介乎猿与人类之间的分支。有些学者认为醉猿的祖先是生活在非洲,并迁徙至欧洲演化为上猿。而上猿亦进而扩散至亚洲,成为醉猿及宽齿猿。","text2":"将化石以酒神和双沟为名的原因是什么?","label":1} {"text1":"Xenomai是一个Linux内核的实时开发框架。它希望通过无缝地集成到Linux环境中来给用户空间应用程序提供全面的,与接口无关的硬实时性能。Xenomai项目始于2001年8月。2003年它和RTAI项目合并推出了RTAI\/fusion。RTAI\/fusion是Linux平台上的具有工业生产级别的实时自由软件开发平台,它基于Xenomai的抽象实时操作系统内核。2005年的时候RTAI\/fusion项目又从RTAI中独立出来作为Xenomai项目。Xenomai基于一个抽象的实时操作系统核心,它可以被用来在一个有通用实时操作系统调用的核心上构建任意的实时接口。用来给用户程序提供接口的任意多个(可以是不同的)实时操作系统的接口被构建在同一个核心上。所有通用的系统调用都是由这个核心来实现的。这些用户接口又被称作“皮肤”。虽然这两个项目有不少共同的想法,而且都支持RTDM实时驱动模块,但是实际上他们有很多不同之处。这些不同之处主要是由他们不同的目标和各自不同的实现方式造成的。RTAI项目致力于技术上可行的最低延迟;Xenomai除此之外还很看重扩展性,可移植性以及可维护性。Xenoami项目将对Ingo Molnar的PREEMPT RT实时抢占补丁提供支持,这又是与RTAI项目的一个显著的不同。","text2":"2003年Xenomai和RTAI项目合并推出了什么?","label":1} {"text1":"《战神》(')是由索尼电脑娱乐发行的动作游戏,由索尼电脑娱乐旗下公司圣塔莫尼卡工作室开发。游戏是根据希腊神话改编。按情节来看是《战神》系列第三作。2007年,被游戏网站IGN评价为PlayStation 2最优秀游戏第4名。SCEA于2009年11月17日推出重制《战神》与《战神II》战神合辑版,并针对PS3硬体加以强化移植,解析度提升为720p,也支援PS3奖杯系统。《战神III》于2010年3月推出,及后于2015年7月以高清重制方式在Playstation 4平台上推出。最新作《战神》计划于2018年年初推出,故事将由希腊神话转至北欧神话。游戏主角克雷多斯原本是一名斯巴达将军。克雷多斯在一次与野蛮人巴比伦王展开战争之时不敌,在接近被杀而命悬一线时,克雷多斯以自身灵魂作为代价,成功呼召了初代战神阿瑞斯降临人世,阿瑞斯赐予克雷多斯由冥府火焰所打造而成的混沌剑,得到混沌剑的克雷多斯轻易击杀巴比伦王。自此克雷多斯成为阿瑞斯在人间的忠实奴仆及追随者同时也是人间的一名完美战士,在之后的战争中战无不胜。好景不常,阿瑞斯为了让克雷多斯放弃人间的所有拖累,精心安排地设局令克雷多斯杀了自己的妻女。自此克雷多斯受到诅咒,他妻女的骨灰嵌入了克雷多斯的身体并每晚受到恶梦的折磨,自此克雷多斯化身为斯巴达的鬼魂并展开了他向阿瑞斯的复仇之旅。","text2":"《战神》是由哪个单位发行的动作游戏?","label":1} {"text1":"溪生战役(、英:),美军代号为“苏格兰行动”(Operation Scotland)和“飞马行动”(Operation Pegasus),是越南战争中爆发于越南共和国(南越)广治省西北部的一次大规模战役,时间为1968年1月到4月间。在此战役中,三个整师规模的越南人民军部队在重炮、迫击炮和火箭炮支援下猛烈围攻了和越南共和国陆军驻守的。而美军则在优势空中火力的支援下坚守基地。激战77天后,越南人民军被击退。该地名使用罗马字母拼写的越南语中为「」。由于现代越南使用罗马字表记,而鲜用其原越南汉字或喃字表记该地名,使得现代汉语对地名的翻译产生含糊。事实上,其罗马字名称中的「」对应汉字「」(此处为「溪」的喃音,其标准汉越音为:「」),而「」对应越南汉字为「」,即“溪生”。有部分材料根据越文罗马字进行音译,将其错误地译为「山」。实际「」字在越南语中的读音为「」,与「」差异很大。未发表的美国政府档案已发表的美国政府档案","text2":"溪生战役中越南军队什么时间撤退?","label":1} {"text1":"佛罗里达山狮(学名:'),又名佛罗里达美洲狮或美洲狮佛罗里达亚种\"'。过去被认为是美洲狮的一个亚种,近年支持将其合并在北美山狮内(\"Puma concolor couguar\")。栖息在美国佛罗里达州南部的松林、针叶林及沼泽。雄狮重约150磅,栖息在大落羽衫国家自然保护区(Big Cypress National Preserve)、大沼泽地国家公园及美洲狮国家野生动物保护区(Florida Panther National Wildlife Refuge)。牠们是美国东部唯一的美洲狮,只占有以往牠们分布地的5%。牠们现存的数量估计只有80-100只。佛罗里达山狮一直都被认为是美洲狮的亚种,学名为\"Puma concolor coryi\"。于1967年牠们被美国渔业与野生动物局(United States Fish and Wildlife Service)列为濒危,且是一直最为受到保护的猫科群族。根据美洲狮线粒体DNA的研究显示,佛罗里达山狮与很多美洲狮亚种都很相似,建议将牠们合并重新分类为北美山狮(\"Puma concolor couguar\")。自此,一些经典的文献都不将佛罗里达山狮看为独立的亚种,而是与其他亚种合并为北美山狮。不过在研究方面,尤其是有关佛罗里达山狮的保育工作,仍然会将牠们列为独立的亚种。然而重新分类的建议却仍未有公认的做法。在美国佛罗里达州已进行保育工作来挽救余下的佛罗里达山狮。不过,这项工作因佛罗里达山狮所需的生活面积而变得不容易:每一个繁殖单位,即1只雄狮及2-5只雌狮就需要约200平方公里的栖息地。一群240只佛罗里达山狮就需要8000平方公里的栖息地,并足够的遗传多样性来避免近亲繁殖。从德克萨斯州引入的8只雌性美洲狮近亲似乎能成功地减低近亲繁殖的问题。佛罗里达州南部是一个快速发展的地区,天然环境的开发影响著佛罗里达山狮的生存。牠们的死亡率最多是来自交通意外及为地盘而自相残杀,这都是因失去栖息地、栖息地的退化及分裂。在近那不勒斯开发的圣母镇(Ave Maria)有指是牠们的主要栖息地。不过,佛罗里达山狮的分布地却被受争议,这引发影响地产商及环保组织的争议。由于以往美国渔业与野生动物局使用了错误的数据,误会了佛罗里达山狮的分布地,致使批核了一些发展项目。","text2":"佛罗里达山狮现存的数量有多少?","label":1} {"text1":"沈阳造币厂隶属于中国印钞造币总公司,是中国政府指定的专门从事法定货币生产的特殊企业,主要生产流通硬币、金银纪念币、铜合金坯饼和工业金银材,是中国目前铜合金硬币生产基地,占地149147平方米,现有职工1500余人。现址为沈阳市大东区大东路138号。始建于1896年(清光绪二十二年),因沈阳地区制钱短缺,光绪帝根据盛京将军依克唐阿的奏请,批准设立“奉天机器局”铸造银元。1896年8月23日,依克唐阿在沈阳东关外、大东边门内,开工兴建“奉天机器局”,是东北第一家机器工厂,开创沈阳近代工业先河。1931年“九一八事变”后,日军占领造币厂并更名为“满洲中央银行造币厂”。1945年日本投降后,该厂由国民政府税务局接收。1948年11月沈阳解放后,“满洲中央银行造币厂”更名为“东北银行工业处”,后改称“东北银行工业处总厂”,集中印制东北银行地方流通券、银元、有价证券等。1987年,厂名改为现名“沈阳造币厂”。","text2":"沈阳造币厂隶属于哪个公司?","label":1} {"text1":"位于伊丽莎白女王奥运园的体育场() 是一座位于英国伦敦斯特拉特福地区的体育场 ,通称“奥林匹克体育场”(),中文俗称“伦敦碗”。奥林匹克体育场是2012年夏季奥林匹克运动会的主场馆。在奥运期间,它曾设置大约80,000个座位。该体育场的规划开始于2007年夏天,并于2008年5月22日正式开工建造,最后于2011年3月29日完工,于2012年4月收工。由2016\/17年度球季起,该球场成为英超球队韦斯咸的新主场。该体育场的设计开始于2007年11月7日。根据伦敦奥委会所说,伦敦奥林匹克体育场是“独一无二的80,000座的体育馆,它将会是2012年奥林匹克运动会的中心,用于主持奥运会的开幕典礼、闭幕典礼和田径项目。残奥结束后将被改建为25,000座的永久体育馆,成为田径运动员们的新家,并作为其它运动、交流和教育的场地。”建造该体育馆田径场时挖出的泥土将被堆积在其周围,形成一个25,000座的看台。伦敦奥林匹克体育场的外围架设有一个可拆卸的轻质铁架作为附加的55,000座的看台。该看台由缆索构成,承担大约三分之二的座位,看台将在2012年奥运会结束后被拆除并被再利用。中文媒体广泛将奥林匹克体育场昵称为“伦敦碗”,据一些资料解释此名称来源于体育场的主体结构如碗状。有趣的是,“伦敦碗”名称为中文媒体独创:虽然非中文背景的英文资料也将体育场主体结构形容为“碗”()状,但没有将体育场本身称为 “伦敦碗”的,倒是有中文背景的媒体从此中文昵称倒翻译出“中式英语”——“London Bowl”。","text2":"伦敦奥林匹克体育场的中文俗称是什么?","label":1} {"text1":"冯兴(,),是8世纪越南反唐起义军的军事首领。据越南吴士连撰写的《大越史记全书》记载,冯兴出生在当时唐林(越南语:Đường Lâm) 地区(今越南河内市山西市社)的一个富裕的家庭,身强体壮。在791年(唐贞元8年),冯兴与他的弟弟冯骇接受杜英翰的建议,率众起义。唐安南都护高正平发兵镇压,为冯兴所败,高正平忧惊而死。他去世,其子冯安继立。冯安为他上了尊号,称为「布盖大王」()。「布盖」一词,在越南语中是「父母」的意思。冯兴此人,在中国史籍《唐书》和《新唐书》中都未提及,《唐书》与《新唐书》都把这场起义的首领写作杜英翰。越南人相信冯兴「英灵显赫」,将之视作神灵。据《粤甸幽灵集》所说,10世纪时的吴权与南汉对战时曾忧心冲冲,在梦中遇见冯兴协助,结果取得胜利,因而从吴朝时起便深受尊崇。陈朝时,朝廷又对冯兴陆续加上「孚祐」、「彰信」、「崇义」等字,因而又称「布盖孚祐彰信崇义大王」。在现代,越共学者则把冯兴的故事视为「一次规模巨大的反抗统治者的起义」。<\/div>","text2":"哪里记载说冯兴出生在当时唐林(越南语:Đường Lâm) 地区(今越南河内市山西市社)的一个富裕的家庭?","label":1} {"text1":"澄海县是中国广东省历史上的一个县,始置于明嘉靖四十二年正月丁未即正月初十(1563年2月20日),由潮州府海阳县的上外莆、中外莆、下外莆等三都,揭阳县的蓬州、鮀江、鳄浦等三都,饶平县的苏湾都组成,县名取“澄靖海氛”或“海宇澄清”之意,治所设于下外莆都之辟望村(今澄海区中心城区),大致范围为今汕头市金平区、龙湖区、澄海区三区(现隆都镇和莲华华镇部分是在新中国成立划归澄海县的)。澄海县历史上曾三度废止,第一次由于康熙时期在东南沿海颁布的“迁界令”,于康熙五年(1666年)六月迁界,撤县并入海阳县,康熙八年(1669年)三月复置,其后于1959年1月也曾撤县并入汕头市郊区直至同年11月复置。1994年4月18日,澄海县撤销,原县域设立澄海市(县级,之后改制为澄海区)。澄海县亦为潮州八邑之一。这里也是泰国吞武里王朝建立者、达信大帝郑信的家乡。澄海现在有著名玩具实业:伟宝实业、著名教育机构:金叵罗教育","text2":"澄海县最先是什么时候设置的?","label":1} {"text1":"《小马王》()是梦工厂于2002年发行的一部动画电影。该片讲述了一匹直到影片结束时仍没有名字、不愿被驯服的年轻马的历险过程。本片不同于其它同类动画片,片中的动物们不再开口说话。卡森伯格说,如果我们让动物说话,那么无论你再作任何努力,这都将使本片更像是一部滑稽的喜剧,这与他所竭力想表现的主题是冲突的。所以,梦工厂接受了挑战,一个没有语言的主人公,将通过它的神情、简单的旁白和适时的音乐来展现它的内心世界。19世纪原始的美国西部,一个原始、野蛮但同时也是充满了自由空气的理想国度。一匹充满野性的不驯小马,就出生在这个美丽的地方。片名斯比尔特(Spirit),象征着向往自由、不肯屈服的灵魂。它是与生俱来的领导者;是群落里的希望与未来之星。所以它的成长道路上,也将与所有为成功而奋斗的生命一样,面对无尽的磨难。但是,尽管需要时刻提防着在邪恶的角落里准备征服和占有西部地区的骑士团上校(詹姆斯·克伦威尔配音),但是在它的印第安人朋友克里克(丹尼尔·斯塔蒂配音)的帮助和自己顽强的努力下,在那遥远的美丽的西部,又一个伟大的英雄,从此诞生。汉斯·季默作曲,布莱恩·亚当斯演唱","text2":"《小马王》讲述了什么?","label":1} {"text1":"《柳生忍法帖》,是日本小说家山田风太郎的「忍法帖」系列作之一(其它作品尚有《甲贺忍法帖》、《伊贺忍法帖》、《女忍忍法帖》等)。小说本创作于1964年,是山田风太郎笔下的「柳生十兵卫三部曲」其中一部,1998年曾拍成电影,2005年推出漫画《Y十M~柳生忍法帖~》。《柳生忍法帖》的故事起点是由一所在江户时代号称「绝缘寺」的东庆寺,因收留了七名刚强的女性而展开一场劝善惩恶的讨伐战所开始的。作者指出本作是一篇「想为弱者讨伐强者」的物语,不过要七名女性战胜如魔鬼般的强敌毕竟是困难的,于是天下第一的剑豪柳生十兵卫就成为七名女性的救星了。与山田风太郎的其它「忍法帖」作品有所不同,本作中并没有涉及忍者、忍术等概念,而且亦不如惯例般以悲剧收场,相反是大团圆结尾,这与山田的一贯风格有所出入。另外,柳生十兵卫在山田风太郎的作品中是拥有最高的人气的角色,本作与《魔界转生》、《柳生十兵卫之死》合称山田的「十兵卫三部曲」。故事时间设定于日本宽永19年(即公元1642年),当时会津藩的第二代藩主加藤明成被称为当世的暴虐领主,他把反抗其暴政的家老堀主水一族处以灭族极刑(详见\"会津骚动\")。堀一族的男性为免牵连族中的女性,于是把她们寄托在以救助女性闻名的比丘尼寺院「东庆寺」中,可是加藤明成手下嗜杀成性的会津七本枪(模仿贱岳七本枪的加藤家臣)却率领士卒到东庆寺残杀堀一族的女性,最后只剩下以堀主水女儿千绘为首的七名女性,得天树院出手相救幸免于难。千绘等七名女性为向加藤明成及七本枪报此血海深仇,决定拜由当代高僧泽庵宗彭引荐的柳生十兵卫为师,誓要向其习得一身武艺,亲自挑战武艺高强的会津七本枪。《Y十M~柳生忍法帖~》——本作的漫画版。","text2":"《柳生忍法帖》故事时间设定于哪一年?","label":1} {"text1":"王延禀(?-931年),原名周彦琛,五代十国时期闽太祖王审知的养子。王审知墓志铭称其为次子。闽嗣王王延翰(王审知长子)在位时,王延禀出任建州刺史。王延翰大量搜括民女以充后庭,王延翰的亲弟王延钧上书反对;延禀对王延翰的搜括民女命令也不满,回复王延翰的书信中语气不恭,导致双方关系紧张。后唐明宗天成二年(927年)十二月,延禀、延钧联合出兵福州,王延禀顺著闽江先抵福州,打败福州指挥使陈陶,陈陶自杀。王延禀活捉王延翰,历数其罪,并指他谋害先主(王审知),将他斩于紫宸门外。王延钧后至,王延禀推延钧为威武留后。之后王延禀返回建州,和王延钧送别时,对延钧说:「善守先人基业,勿烦老兄再下!」延钧虽然道谢,但面色大变,对王延禀加以防范。后唐明宗长兴二年(931年)四月,王延禀任奉国节度使兼中书令,听闻闽王王延钧有病,认为有机可乘,于是任命次子王继升为建州留后,和另一位儿子王继雄一同领水军进攻福州。王延禀攻西门,王继雄攻东门,王延钧派楼船指挥使王仁达对抗,仁达将士兵埋伏在舟中,挂起白帜向王继雄请降。继雄大喜,摒退左右登上仁达的战船纳降,反被仁达杀死。王仁达将继雄的人头挂于西门。延禀刚刚纵火攻城,见到儿子的人头,不禁大哭,军心动摇,仁达趁机击之,王延禀大败,后被生擒。王延钧笑著对延禀说:「果烦老兄再下!」将延禀囚于别室,派使者到建州招安延禀的党羽。党羽杀死使者,和延禀的两个儿子继升和继伦逃到吴越。五月闽王延钧斩王延禀于市,并恢复其原名,周彦琛。","text2":"王延禀原名叫什么?","label":1} {"text1":"沈云山()是在香港九龙东部的一个山丘,高177米,地图上多没标示。该山俗称为「水塘山」。沈云山跟其西北面的平山形成佐敦谷。沈云山东北面为大上托(安达臣道石矿场),两山之间的山谷已开发成顺天邨。沈云山北面是顺利邨、顺安邨和飞鹅山,南面则为鳄鱼山和牛头角。沈云山的山顶上,建有水务署的观塘上食水配水库和观塘上海水配水库。配水库平台上建有由康乐及文化事务署管理的观塘上配水库花园,是区内的晨运热点。沈云山山脚有数座政府建筑物,北为佐敦谷海水配水库和前佐敦谷水塘,南为福建中学和乐华天主教小学,西为沈云山海水抽水站及沈云山食水抽水站。市民可从入口位于北面顺利邨道的车路,或从南面振华道经「振华道晨运径」直上山顶。在观塘上海水配水库的后面,有一个被认为是前防空洞的山洞。山洞已有闸门封锁,从闸门的窗口看进,里面有生锈之水管及一些设备,而闸门之上则刻有金属字「HKWW 1965」。","text2":"沈云山俗称什么?","label":1} {"text1":"扁头豹猫(学名:'),或简称扁头猫,是一种体型较小的野外猫科动物,分布在泰国南部、马来西亚、汶莱及印尼的森林。牠们因失去栖息地及污染而濒危。只有两只扁头猫受到饲养,都是在马来西亚的动物园。像很多细小的猫科,牠们都曾被分类在猫属之中。扁头豹猫体长41-50厘米,尾巴很短,只长13-15厘米,重1.5-2.5公斤。牠们一般呈深赤褐色,头部更红,腹部较白。除了面部上淡淡的斑纹,牠们可谓没有斑纹。牠们四肢较短,耳朵短而圆。趾间的蹼可以帮助牠们在泥泞及水中活动。牠们的头颅骨稍长,头顶扁平。扁头豹猫栖息在次生林及原生林,很多报告都指是在近水区见到牠们。牠们主要猎食青蛙、鱼类及甲壳类,也会捕捉大家鼠及鸡。牠们的前臼齿较长,爪不能伸缩。这与半水中生活的鼬科相似,随时可以进到水中。整体而言,牠们野外的生活资料不详。妊娠期约56天,每胎产1-2只幼猫。饲养的扁头猫寿命可以超过14岁。一般相信牠们是夜间活动的,但观察显示牠们是黄昏及破晓出没的。扁头豹猫被世界自然保护联盟列为濒危,且列在《濒危野生动植物种国际贸易公约》附录一中。牠们的成年数量相信少于一万只,其下的亚群没有一个有多于1000只成年。牠们受到失去栖息地及水污染的威胁。牠们除了在在汶莱外都受到完全的保护。在野外很少会见到牠们。","text2":"扁头豹猫为什么濒临灭绝?","label":1} {"text1":"显存全称显示记忆体,亦称帧缓存,它是用来存储显示芯片处理过或者即将读取的渲染数据。如同计算机的内存一样,显存是用来存储图形数据的硬件。在显示器上显示出的画面是由一个个的像素点构成的,而每个像素点都以4至64位的数据来控制它的亮度和色彩,这些点构成一帧的图形画面。为了保持画面流畅,要输出和要处理的多幅帧的像素数据必须通过显存来保存,达到缓冲效果,再交由显示芯片和中央处理器调配,最后把运算结果转化为图形输出到显示器上。从上公式可预知使用高分辨率和开启全屏抗锯齿、各向异性过滤、物理效果模拟、多边形顶点数据运算、海量三维函数运算等高级画面效果后,计算速度较慢的显示芯片需要更多的显存空间才能流畅显示。早期显卡的显存容量只具有512KB、1MB、2MB等极小容量时,显存容量成为最大分辨率的一个瓶颈;但目前主流显卡已经淘汰512MB的显存容量,主流中低端级显卡的显存容量是1GB或2GB,某些高端显卡甚至已经具有4GB或6GB的显存,在这样的情况下,显存容量早已经不再是影响最大分辨率的因素。现在显卡性能越来越强,这当然不能忽略显示芯片的发展速度快的事实,但显存的频率、位宽与带宽也制约着显卡的总体性能。高级的显存能让显示芯片全力工作,令输出的画面更流畅。显示芯片性能的日益提高,其数据处理能力变得更强,使得显存数据传输量和传输率的要求变得更高,显卡对显存的要求也更高,现时大部分显卡使用的是GDDR2至5代规格的显存。虽然各个类型的显存因厂家与规格而不同,而GDDR5规格的显存频率、位宽与带宽都普遍比其他规格高,少部分显卡使用更高频宽和位宽的HBM,HBM为现时最高价规格。","text2":"显卡对显存的要求更高了?","label":1} {"text1":"本条目为欧洲足协杯赛冠军球会领队列表,纪录各主教练带领曾执教的欧洲足协所属成员国的足球俱乐部,在欧洲足协官方认可的球会级杯赛赢得冠军的数量。这些杯赛包括欧冠杯(1955年至1991年,欧洲联赛冠军杯前身)、欧联(欧洲联赛冠军杯,1991年至现今)、欧洲杯赛冠军杯 、欧洲足协杯(1971年-2009年)、欧霸杯(2009年至现今)、图图杯和欧洲超级杯。截止2014年意大利籍领队查柏东尼和苏格兰籍领队费格逊是最成功的领队,分别带领球队赢得6项赛事冠军。截至2014年,意大利领队赢得最多杯赛冠军,合共38项,英格兰领队则紧随其后,有25项。当欧洲博览会杯被欧洲足协杯取代前,欧洲足协并没有将其视为官方赛事,所以此杯的冠军将不被在此列表中记录。而1972年的欧洲超级杯,同被视为「非官方赛事」。说明此表是纪录各国领队带领球队夺冠的总数:","text2":"截至2014年,哪个领队赢得最多杯赛冠军","label":1} {"text1":"结构性失业(Structural unemployment)是指市场竞争的结果或者是生产技术改变而造成的失业,通常由于就业市场并不平衡,某些行业正扩张,另一些则衰退,造成部分工人失业。结构性失业通常较摩擦性失业持久,因为失业人员需要再训练或是迁移才能找到工作。结构性失业的出现是因为经济结构、体制、增长方式等的变动,改变了工作技能的要求,导致失业的发生。由于失业工人并不具有合适的技能,因此,若失业工人并没有接受再培训或进一步的教育,他们便不能再获聘任,结构性失业问题因而会持续,影响长远的经济发展。摩擦性失业和结构性失业被归类为自然失业,经济学家视这两种失业为正常现象。劳动人口中,属于该两种失业的劳动人口数目占劳动人口总数的百分比等于自然失业率。自然失业率:失业率(%)=(摩擦性失业 + 结构性失业)÷劳动人口×100%","text2":"什么是结构性失业?","label":1} {"text1":"杨廷艺(,,部份文献写成「杨延艺」()),越南史上亦称为「杨正公」,交趾爱州人,原为静海节度使曲氏将领,当其主曲承美被中国南汉灭亡后,杨廷艺奋起抵抗,夺取了交趾的统治,因而被视为是越南自立建国的重要人物之一。杨廷艺是中国五代十国时期爱州人士,曾任交趾静海节度使曲颢的将领。当时,位于中国岭南的南汉与曲氏势成水火,曲颢之子曲承美继位后,于大有三年,930年(《大越史记全书·外纪全书·南北分争纪》作923年),南汉君主刘䶮出兵,擒获曲承美,占领交趾地区,并派骁将梁克正、交州刺史李进驻守,而杨廷艺则获南汉授以爵命,名义上成为了南汉的臣僚。据《安南志略》,刘䶮虽已得胜,但洞悉到「交趾民好乱,但可覊縻而已」,认为当地民众不会服从其统治。而杨廷艺亦为驱逐南汉而作好准备,《大越史记全书》提到,到了大有四年(931年)农历十二月,南汉官员李进向刘䶮告发「杨廷艺养假子三千人,图恢复」,廷艺随即先发制人,攻破李进所在的大罗城,李进逃归。适值南汉派遣程宝(《新五代史·南汉世家·刘隐》作「程宝」,《大越史记全书·外纪全书·南北分争纪》作「陈宝」)率兵赶来,包围杨廷艺,杨廷艺出战,击杀程宝。杨廷艺取得军事胜利后,便自称节度使,领州事。关于杨廷艺得胜后与南汉的关系,中国学者郭振铎、张笑梅作以下分析:「杨廷艺虽战败南汉军,领有交州。但他深恐南汉再次派大军来攻,为了本身的政权不受威胁,只好臣服南汉,服从南汉的诏谕,接受为交州节度使,刘䶮和杨廷艺之战宣告结束。」而在交趾方面,杨廷艺与本地的土豪结好,例如把女儿嫁给「世为贵族」的牙将吴权。南汉大有十年(937年)农历三月,杨廷艺被牙将矫公羡所杀。矫公羡行凶的原因,则是「恶廷艺所为,起所部攻杀廷艺,而代为节度。」矫公羡遂向南汉臣服,但其后亦被杨廷艺另一员牙将吴权所杀,并继续争取交趾政权的独立。","text2":"杨廷艺是哪里人?","label":1} {"text1":"普鲁士公国(、)位于普鲁士地区东部,存在时间为1525年至1701年(其中1618年-1701年间以勃兰登堡-普鲁士形式存在)。普鲁士公国是历史上第一个新教国家,人口中德语使用者为主,亦有为数不少的波兰语和立陶宛语人口。1525年宗教改革中,条顿骑士团大团长阿尔布雷希特宣布条顿骑士团国世俗化,改宗路德宗,建立普鲁士公国,向波兰国王效忠,首都柯尼斯堡。1618年,来自霍亨索伦家族的勃兰登堡侯国选帝侯约翰·西吉斯蒙德继承公国王位,勃兰登堡-普鲁士建立。1657年,根据《韦劳条约》,大选帝侯腓特烈·威廉获得公国完全主权,1660年的《奥利瓦条约》对此进一步确认。1701年,普鲁士公国升级为普鲁士王国。","text2":"普鲁士公国位于什么位置?","label":1} {"text1":"伯根-贝尔森集中营 (,又称Bergen-Belsen或Belsen) ,是纳粹德国在德国西北部下萨克森建立的一座集中营,该地毗邻伯根和策勒。1943年到1945年,统计约有50,000名苏联战俘和超过50,000名犯人死于其中。他们中超过35,000人死于1945年初流行的斑疹伤寒。该集中营于1945年4月15日被英国第十一装甲师解放。当时约有60,000名犯人尚在其中,绝大多数都已身患重病,另外还有13,000具未处理的尸体。〈由于二战末期纳粹陆续把欧洲各地集中营的犯人迁移到德国境内的集中营,导致各集中营极度拥挤,再加上当时斑疹伤寒极为严重,集中营内死亡人数增加迅速,党卫军来不及处理尸体,才导致尸体任意堆叠在营区内〉。BBC的随军记者理查德·丁布尔比这样形容当时的景象:英国准将看到该集中营的惨状时相当生气,拿元帅杖殴打投降的德国空军元帅艾尔哈德·米尔希,元帅杖也被打断。在伯根-贝尔森集中营里死去的15岁犹太女孩安妮·法兰克,她的日记成为二次大战期间纳粹消灭犹太人的最佳见证,日记中展现了惊人的勇气与毅力。","text2":"BBC的随军记者理查德·丁布尔比是怎样形容当时景象的?","label":1} {"text1":"胡贝雅(Blair Cornelia Waldorf)又译布莱儿·华道夫,美国热门电视剧《花边教主》中的主角,她在第三季读了纽约大学。第四季转学至哥伦比亚大学。布莱尔·华朵芙出生于纽约上东城(Upper East Side)的一个富裕家庭,她爸爸是哈洛·华朵芙(Harold Waldorf),妈妈是爱琳诺·华朵芙(Eleanor Waldorf)。布莱尔为家中独女,所以总是娇滴滴的,很“大小姐”脾气。中学时,她父亲爱上了一个男人并跟他到法国居住,使她大受打击。后来,她又多了位是犹太继父,叫罗曼(Cyrus Rose),并带来一名“哥哥”Aaron Rose。布莱尔家中还有一名女仆叫朵洛塔(Dorota)。","text2":"胡贝雅在第三季中就读于哪个大学?","label":1} {"text1":"徐珂(),原名昌,字仲可,浙江杭县(今属杭州市)人。光绪举人。后任商务印书馆编辑。参加南社。1901年在上海担任了《外交报》、《东方杂志》的编辑,1911年,接管《东方杂志》的“杂纂部”。与潘仕成、王晋卿、王辑塘、冒鹤亭等友好。编有《清稗类钞》、《历代白话诗选》、《古今词选集评》等。光绪十五年(1889年)举人。后任商务印书馆编辑。参加南社。曾担任袁世凯在天津小站练兵时的幕僚,不久离去。","text2":"1911年,徐珂担任什么职务?","label":1} {"text1":"坦佩谷()是希腊色萨利大区北部一个峡谷的古名,它位于奥林帕斯山以南,俄萨山以北,被希腊诗人誉为“阿波罗和缪斯喜爱的去处”。山谷长10千米,狭窄处仅宽25米,两旁山崖高近500米,匹尼奥斯河于其间蜿蜒而过流向爱琴海。皮尼奥斯河右岸伫立着一座阿波罗神庙,周围月桂丛生,古时人们用它做成桂冠嘉奖皮西安竞技会的得胜者。坦佩谷也曾一度是阿波罗与库瑞涅之子阿里斯泰俄斯的居所,他也正是在此处追赶俄尔甫斯之妻欧律狄刻,以致她在逃走时被蛇咬死。公元13世纪山谷里建起了一座敬献给圣帕拉斯凯韦的教堂。坦佩关是希腊的战略关隘,因为它是从拉里萨穿山到达海岸的主要路线,也正是由于这一点,从古到今许多战役都在这里打响。如要避开此关,也可取道稍迂回的撒伦多普罗关。公元前480年,一万雅典人与斯巴达人试图在坦佩关阻止波斯国王泽克西斯一世的入侵,但波斯人取道撒伦多普罗从而绕开了希腊军队。公元前164年,在第三次马其顿战争期间,罗马人冲破马其顿国王珀尔修斯的防御,之后在彼得那战役中取得了胜利。公元前148年,安德里斯库斯的另一场斗争也在这里上演。无论在标志着罗马时代在希腊的终结的异族袭击中,还是在拜占庭和奥特曼时代,这里也不乏争战。今天对于大多数希腊人来说,坦佩也以其十分恶劣的路况和可怕的交通事故而臭名昭著。例如,在2003年,由于所乘客车与一辆满载木材的货车相撞,二十一名来自马克拉荷里的十一年级一个班的中学生全部死亡。尽管货车司机已尽力在几秒钟的时间内减速,但木材带着巨大冲力从货车后部飞出,从客车左部砸进,当场导致许多学生死亡。美国亚利桑那州的坦佩和澳大利亚新南威尔士的坦佩市都是由此命名。南非东开普省的一个农场也叫坦佩,还有着两个名为奥林帕斯和俄萨的农场邻居。另外,马来西亚槟城沿着一条小峡谷而设的坦佩谷路也是以它命名。","text2":"坦佩谷是什么的名称?","label":1} {"text1":"降解体(Degradosome)是存在于大多数细菌中的一种多亚基蛋白质复合物,其功能为参与核糖体RNA的加工以及信使RNA的降解,故又可称为RNA降解体。1994年,瑞士日内瓦大学的Henry Krisch所领导的实验室和英国剑桥大学的Christopher Higgins所领导的实验室分别发表文章提出了细菌中存在着一种可以降解RNA的蛋白质复合物,也就是降解体;他们是在对大肠杆菌的RNase E和PNPase的研究和鉴定过程中得到这一发现。降解体的主要成分为核糖核酸酶E(RNase E)、多核苷酸磷酸化酶(PNPase)、RNase螺旋酶B(RhlB)和烯醇酶(enolase)。","text2":"降解体谁被那两个实验室发现的?","label":1} {"text1":"小高鳍刺尾鱼(学名:),又称小高鳍刺尾鲷,俗名黑三角倒吊、褐三角倒吊,是辐鳍鱼纲鲈形目刺尾鱼亚目刺尾鱼科的一种。本鱼分布于印度太平洋区,包括东非、红海、马达加斯加、模里西斯、留尼旺、葛摩、塞席尔、马尔地夫、斯里兰卡、安达曼群岛、印度、日本、中国沿海、台湾、菲律宾、马来西亚、印尼、新几内亚、泰国、澳洲、豪勋爵岛、圣诞岛、新喀里多尼亚、马里亚纳群岛、马绍尔群岛、帛琉、密克罗尼西亚、索罗门群岛、斐济群岛、万那杜、诺鲁、夏威夷群岛法属玻里尼西亚、吉里巴斯、吐瓦鲁、复活节岛、东加、美属萨摩亚等海域。水深1至60公尺。本鱼体呈卵圆形而侧扁。口小,端位,上下颌齿较大,齿固定不可动,扁平,边缘具缺刻。体为深橄榄棕色到几乎全黑,除胸鳍具有明显的橘红色软条及暗色边缘纹外,其余鳍条均为暗橄榄色。体侧中央部分有一道黄色平行纵条纹。幼鱼期,鱼体布满窄的垂直白色斑纹。背鳍硬棘4至5枚、背鳍软条23至25枚、臀鳍硬棘3枚、臀鳍软条19至21枚。体长可达20公分。本鱼喜栖息于有隐蔽处所的珊瑚礁茂盛区,常单独或三两成群活动。繁殖配对时可能是一对一,也可能为群体进行。为藻食性。因造型极体色鲜艳是受欢迎的水族观赏鱼类,也可食用。可做豆腐味增汤。尾柄棘会伤人,须注意。","text2":"小高鳍刺尾鱼分布在何地?","label":1} {"text1":"伯奇还原反应(Birch还原)是指用钠和醇在液氨中将芳香环还原成1,4-环己二烯的有机还原反应。此反应最早由澳大利亚化学家Arthur John Birch (1915–1995)在1944年发表。 Birch还原的重要性在于:尽管剩下的双键(非芳香性)更为活泼,该反应却能停留在环己双烯上,而不继续还原。反应中的钠也可以用锂或钾取代,使用的醇通常是甲醇或叔丁醇。使用Birch还原的一个例子是还原萘:其他人也发表了很多篇关于此反应的综述。钠溶于液氨中会形成一个电子盐的亮蓝色溶液,化学式为[Na(NH)] e。溶剂化电子会与芳香环加成,形成一个自由基负离子。溶液中的醇此时作为质子试剂提供一个氢原子。对大多数反应物来说,氨上的氢酸性还不够。如果是取代芳香化合物,当取代基是羧基等吸电子基时,能够稳定碳负离子并生成最少取代的烯烃; 当取代基是供电子基时,则生成取代最多的烯烃。 热力学不稳定的非共轭1,4-加成产物往往产率超过热力学稳定的1,3-加成产物,这是由于共轭的戊二烯负离子中间体HOMO的最大轨道系数是在中间那个碳原子上,导致生成的1,4-环己双烯没有办法经过平衡移动而生成更加热力学稳定的产物,因此,生成的是动力学稳定产物。在卤代烃的存在下,上文提到的碳负离子也可以发生亲核取代反应生成新的碳-碳键。如下图所示,在Birch还原中生成的负离子中间体可以被一个合适的亲电试剂捕获,例如卤代烃:根据逆合成分析,前者即是后者的合成子。在下图所示反应中,1,4-二溴丁烷被加入到苯甲酸叔丁酯中,最后生成烷基化的1,4-环己双烯产物。","text2":"溶剂化电子会与芳香环加成会形成什么?","label":1} {"text1":"熏豆腐通常指曲阜产熏豆腐,又称孔府熏豆腐。孔府熏豆腐是地方传统小吃。顾名思义,这种豆腐是熏制而成的。关于熏豆腐的由来,民间有一个传说:旧时孔府有许多佃户,其中有一个姓韩的豆腐户家住曲阜城东北书院村,他家祖祖辈辈给孔府送豆腐。至清代乾隆年间,韩家的兄弟俩每天各给孔府送一方豆腐。有一年三伏期间遇到连阴天,韩老二家做的豆腐没卖完,放着怕馊了,他就把豆腐切成许多小块,摆在秫秸帘子上晾着。谁知他家不慎失火,晾豆腐的帘子也被烧着了,由于天阴、帘子湿,豆腐块有的被烤煳,有的被熏成了棕黄色。韩老二舍不得将这些豆腐扔掉,便取了一些放在盐水里煮了煮,一吃味道还挺不错。于是,他便送了些这样的豆腐到孔府,让衍圣公品尝。衍圣公让厨师在炖豆腐时放了桂皮、花椒、辣椒粉等作料,待豆腐被炖煮好之后,一尝味道甚好。从此,韩老二就专门给孔府制作、呈送熏豆腐,熏豆腐遂成为孔府的一道美味佳肴。传说有一年乾隆皇帝来曲阜,孔府摆了个豆腐宴招待皇上,其中就有一道熏豆腐。皇上吃着十分可口,大加赞赏。后来,熏豆腐的做法在民间逐渐流传开来,成为当地的一种独具特色的风味小吃。这种熏豆腐不仅曲阜本地人爱吃,而且到曲阜的外地人吃过之后也都爱上了它。据说,当年曲阜师范大学里有一位南方籍的教授,常步行几里路到集市上去吃熏豆腐,几十年乐此不疲。先将白豆腐切成长宽五六厘米、厚1厘米左右的方块,放在铁箅子上,下面燃以松、柏、梨、枣、苹果木锯末或谷糠,待豆腐被锯末或谷糠油熏烤至棕黄色、泛起油亮光泽即成。这种熏制而成的豆腐表面呈棕黄色,表层结实,里面细白鲜嫩,吃起来带有浓郁的木香或谷糠香味.熏豆腐可凉拌、炖熏,也可切成薄片,稍加鲜辣椒炒制。另外,集市上设有许多熏豆腐锅,把熏豆腐与肉块放入铁锅内,加水没盖,在加整辣椒、茴香、花椒、桂皮等佐料炖煮,即成“五香油辣熏豆腐”。上市者多以熏豆腐为佳肴饮酒,佐餐。当然,最正宗的做法还是曲阜当地的“五香油辣熏豆腐锅”:把熏豆腐与肉块放入铁锅内,加水,再加入辣椒、茴香、花椒、桂皮等作料炖煮。这种熏豆腐锅一般要炖煮很长时间,以使肉及作料的香味皆浸入豆腐内部,再加上熏豆腐自身特有的烟熏味,吃起来风味十分独特。","text2":"熏豆腐又称什么?","label":1} {"text1":"菲利普·克雷文,MBE(Sir Philip Craven,),出生于英国英格兰大曼彻斯特郡博尔顿,退役轮椅篮球运动员,现任国际残疾人奥委会主席。菲利普·克雷文于16岁时因攀爬意外致残,导致失去行走能力,他称意外之前喜欢踢足球,脚法虽有欠灵活,但眼手协调能力比较突出,成为日后投身轮椅篮球运动的原因。1972年毕业于曼彻斯特大学地理学系。已婚,并育有两子。他曾于1986年至1991年间担任英国煤炭公司行政主管。克雷文自1972年夏季残奥会至1988年夏季残奥会五度代表英国参加轮椅篮球项目,他更于1972年夏季残奥会参加田径及游泳项目。他曾在1973年及1975年举行的世界轮椅篮球锦标赛中,夺得金牌及铜牌;在1971年及1974年的欧洲锦标赛两度获得金牌,并在1993年获得银牌。他并获得1994年欧洲冠军杯及1970年英联邦运动会夺得金牌。克雷文曾三度担任英国轮椅篮球协会主席(1977年至1980年;1984年至1987年;1989年至1994年),并先后担任国际斯托克曼德维尔轮椅运动联合会篮球小组委员会主席(1984年至1988年)、国际轮椅篮球协会主席(1988年至2002年)及执行长(1994年至1998年)、英国男子轮椅篮球队执行总监(1988年至2002年)等公职。自2001年起,克雷文担任国际残疾人奥委会主席。克雷文于2003年获委任为国际奥林匹克委员会委员,并先后出任国际奥委会体育及环境委员会成员(2002年至2004年)、北京奥运会协调委员会成员(2002年至2008年),并于2005年起担任文化及奥运教育委员会成员。","text2":"导致菲利普·克雷文残疾的原因是?","label":1} {"text1":"《Rule\/Sparkle》(规则/光采)为日本歌手滨崎步发行的第45张单曲,2009年2月25日于日本发售,同月27日于台湾发售。本作是十周年纪念单曲以后的第一张单曲,但是仍然延续著之前的销售模式,多版本搭配收录不同的混音以刺激销量。封面设计以鲜艳颜色、塑胶及瑜珈动作为特色,是滨崎步「新起步」创新的第一乐章,电子音乐和摇滚曲的乐风、以及前述夸张的封面都是这张单曲的新尝试。另外,《Rule》一曲作为七龙珠电影版的全球主题曲,七龙珠原作者鸟山明特地为滨崎步绘制似颜绘(以容貌为目的所绘的图样),并印刷于CD及DVD封面作为本张单曲的特典。这首也是第60回NHK红白歌合战的演唱歌曲。全曲作词:滨崎步","text2":"这首单曲封面设计有何突破?","label":1} {"text1":"《山水有相逢》(英语:)是香港电视广播有限公司拍摄制作的怀旧温情喜剧剧集。剧中讲述1950年代香港电影界的百态,和三个当时红透半边天的演员的温情生活。剧情从1980年开始。当时,香港一间电视台需要找两位年纪比较大的演员,在一套剧名为「双天斗至尊」的大制作里,饰演两位大家族的老奶奶。一名在电视台工作的小员工 (贾思乐饰) 就建议电视台找他的姑母,五十年代曾当红一时的粤剧演员梅剑仙 (黄韵诗饰) 。他也满以为他的姑母梅剑仙可以与她当年一同拍下无数电影的拍档梅妹 (李司棋饰) 一起复出。但是,那职员并不知道梅剑仙与梅妹有一段非常复杂的关系。原来当年,梅剑仙与梅妹被一间名为「天山影业公司」的电影公司聘请,来香港拍电影。多年以后,两位新人已经成为红透半边天的电影演员。但是,她们要好的感情就被花花公子靳玉楼 (黄锦燊饰) 破坏。两个人就为了情和利翻脸。多年后,梅妹就过著有如「少奶奶」的生活,但是当年出入表现的非常富贵的梅剑仙就只住在钻石山木屋区,过著非常贫穷的生活。当电视台出面找这两位资深演员复出拍剧后,她们当年的误会就开始被一一化解。甘国亮与香港苹果日报在2006年时做过一篇访问。访问里,他说当年选择黄韵诗饰演「梅剑仙」,在初期受到无线高层反对。甘国亮说这是因为当年佳艺电视开始广播时,黄韵诗抛弃无线,过档佳视。在佳视倒闭后,这些当年过档佳视的前无线演员就被无线当成「叛将」,不能再返回无线。但是,黄韵诗在「山水有相逢」的表现被无线看好,也为黄韵诗日后在无线带来非常多的拍剧机会。","text2":"简述这部剧的主要内容?","label":1} {"text1":"萨克森-希尔德布尔格豪森(Sachsen-Hildburghausen)是韦廷家族在图林根的一个邦国。1680年由萨克森-哥达分裂产生。1702年升为公国,1826年并入萨克森-迈宁根。萨克森-哥达公爵兼萨克森-阿尔滕堡公爵恩斯特一世有12个儿子,其中有七人活到成年。1675年,恩斯特一世去世,他活着的最年长的儿子弗里德里希继承了他的领地。但遭到其他儿子的反对。1680年,弗里德里希一世与六个弟弟通过协议瓜分了父亲的领土。其中恩斯特一世的第九子恩斯特得到海尔德堡、艾斯菲尔德和希尔德堡豪森,称萨克森-希尔德布尔格豪森亲王。1702年,萨克森-希尔德布尔格豪森升为公国。1705年得到松讷费尔德,1714年得到贝林根。萨克森-希尔德布尔格豪森的公爵们在希尔德堡豪森修建了法式的宫殿和许多奢华的建筑。1806年,萨克森-希尔德布尔格豪森加入莱茵邦联,1816年加入德意志邦联。1818年,萨克森-希尔德布尔格豪森宪法正式实施。1825年,萨克森-哥达-阿尔滕堡绝嗣。哥达归于萨克森-科堡-萨尔费尔德;阿尔滕堡归于萨克森-希尔德布尔格豪森。1826年,恩斯廷系诸邦国领地重新划分。萨克森-希尔德布尔格豪森将本土交给萨克森-迈宁根,成为萨克森-阿尔滕堡公国。萨克森-希尔德布尔格豪森公爵弗里德里希改称萨克森-阿尔滕堡公爵。","text2":"萨克森-希尔德布尔格豪森公爵弗里德里希的改称是什么?","label":1} {"text1":"四季名花,即兰花、荷花、菊花、梅花四种名花,又称“中国四季名花”。兰花代表春季,荷花代表夏季,菊花代表秋季,梅花代表冬季,是花文化在群众中发展而形成的,从被人格化的花卉身上提炼出来的称号。春兰夏荷秋菊冬梅虽各开一季,但它们皆被中国人赋予一致的特征:高尚的品质、高洁的品性和不屈不挠的精神内涵。四季名花深受文人墨客喜爱,各有名句流传:兰花“如入芝兰之室,久而不闻其香”;荷花“出淤泥而不染”;菊花“此花开尽更无花”;梅花“暗香浮动月黄昏”,历史流传下来的关于四季名花的诗词数量之多,形式之丰,质量之高,不胜枚举。书画史上:画兰花者如郑板桥、八大山人、任伯年等;画荷花如张大千;画菊花如郑思肖;画梅花如王冕、汪士慎。在中国传统花艺中,将兰、荷、菊、梅同插一瓶,寓意“四季平安”。当代又有《中华字经》:“奇花异卉,艳丽荣秧,兰荷菊梅,四季芬芳。”","text2":"画荷花的大家是谁?","label":1} {"text1":"裸盖鱼(学名:')又名银鳕鱼、裸头鱼,是辐鳍鱼纲鲉形目黑鲉亚目裸盖鱼亚目黑鲉科的其中一种,经常被与真正的鳕鱼混为一谈。后在南极海域发现南极犬齿细鳞鱼的鱼类,外观虽与裸盖鱼完全不同,但也被称为银鳕鱼,且市价更高。本鱼分布于北太平洋,包括日本北部、勘察加半岛的白令海海岸、阿拉斯加、加利福尼亚州等海域。一般栖于近底层(水深0-2740米),属冷水性底栖鱼类。水深0至2740公尺。本鱼体相当延长而稍侧扁,呈圆筒形。头尖形;口中大,上颌略长于下颌。齿细小,呈齿带,上下颌、锄骨、腭骨均具齿。头部及身体均被弱小圆鳞。背鳍分离,具有硬棘19至27枚、软条15至19枚;臀鳍硬棘3枚、软条15至19枚;尾鳍深叉型。体背深蓝灰色或绿灰色;体腹灰色或淡色;除第一背鳍后,各鳍均具有黑色外缘。体长可达120公分。冷水域之深海鱼类,本鱼卵为大洋性漂浮卵,随著成长逐渐由水表面而不断迁移至较深水域。杂食性,大都以虾、蠕虫或小鱼为食。食用鱼,一般皆以烟薰食之。其肝具有丰富的维他命A及D。在中国大陆市场上被以“鳕鱼”的名称出售。然而,另一种较常见的名称“油鱼” ,即“异鳞蛇鲭”,常被拿来冒充裸盖鱼,但肌肉及内脏富含蜡质油脂,多吃会导致腹泻。","text2":"裸盖鱼是属于什么科目的物种?","label":1} {"text1":"宽脸长鼻袋狸(\"Potorous platyops\")是已灭绝的长鼻袋狸。牠们最初于1844年由约翰·古尔德(John Gould)所描述,发现的时间候其数量已很稀少。目前只有少量的标本,最后一个于1875年采集。牠们在欧洲殖民澳洲前就已经差不多灭绝了。从亚化石遗骸得知,牠们原本广泛分布在南澳州半干燥的沿岸区域至西澳州海岸,最北可能至西北角。对于宽脸长鼻袋狸的习性差不多完全不明。但有一点肯定的是牠们不会进入其近亲长鼻袋鼠及长脚袋鼠所栖息的森林。宽脸长鼻袋狸较其他长鼻袋狸细小,约长24厘米,而尾巴长18厘米。上身呈灰色,而下身呈白色,体形像大家鼠。牠们的耳朵细小及较圆,吻短,颊明显肥胖。","text2":"宽脸长鼻袋狸的习性是什么样的?","label":1} {"text1":"荷兰公共安全与司法部(Ministerie van Veiligheid en Justitie)是主掌荷兰司法的政府部门,由司法大臣担任部门主官,设有一位国务秘书(Staatssecretaris)协助大臣。公共安全与司法部原名为「司法部」(Ministerie van Justitie),吕特内阁上任后,由于原内政部掌管的部分公共安全职权转移至司法部,10月14日正式改名。司法部的法定责任有:另外,也负责反恐政策的协调。由于司法部与内政及王国关系部共同分担许多职责,也位在同一栋大楼,因此有双子部会之称。司法部首长为司法大臣与一位国务秘书,30,000名公职人员分布在海牙(本部所在地)及荷兰各地。司法部与内政及王国事务部位于同一栋大楼。正副秘书长率领整个公职部门,下辖三个总司:检察长会议(Raad van Procureurs-Generaal)率领整个检察部门(Openbaar Ministerie),是一个相对独立的组织,为司法系统的一部分。荷兰法医研究所(Nederlands Forensisch Instituut)是司法行政及执法总署底下的自治部门。","text2":"为什么司法部与内政及王国关系部有双子部会之称?","label":1} {"text1":"颜信辉为台湾会计师及会计学家,现任淡江大学教授与会计长,曾多次参与制定中华民国的财务会计准则与参予国际财务报告准则(IFRS)翻译搞的审阅,多年来持续宣导堆动中华民国全面直接使用国际会计准则(T-IFRS),放弃中华民国过去数十年来自行制定并参杂美国会计准则(FASB)的财务会计准则(T-GAAP),以原则基础(Principle-based)会计取代规则基础(Rule-based)会计。颜信辉,拥有十分完整的会计专业背景,1986年(民国75)于淡江大学会计系毕业,1988年获政治大学会计所硕士学位,他回到淡江担任讲师一职,为了让自己的视野更宽广,他选择留职停薪至美国伊利诺大学厄巴纳-香槟分校校区攻读会计硕士,回国后继续在本校担任教职,并同时攻读台大会计博士,1996年获台湾大学会计学博士学位。曾任中华民国财团法人会计研究发展基金会财务会计委员会顾问,委员,中华民国考试院典试委员等职务,现为淡江大学会计系教授兼会计室主任,以及会计研究发展基金会的财务会计准则委员会委员。其妻为金管会证期局第六组组长詹静秋。","text2":"颜信辉的妻子从事什么工作?","label":1} {"text1":"株式会社khara(,)是一家以动画的开发和制作为主要经营项目的日本企业。khara成立于2006年5月,其执行董事是GAINAX的动画企划庵野秀明。主要经营业务是动画作品的企划、原作、脚本、设计等开发工作以及动画作品的制作。自2006年成立开始,khara便开始《福音战士新剧场版》的制作工作。庵野秀明是GAINAX的创始人之一,离开GAINAX继而创建khara之际,在一份《新世纪福音战士新剧场版》的公开声明中写道:khara的假名是「」,与英文单词「color」(色彩,彩色)的假名相同,因此khara的意思常被认为是彩色。但实际上khara来自,意为高兴、欢笑,对外常使用制作部门名义。此外,公司的标志也是由安野梦洋子设计的。","text2":"谁是GAINAX的创始人之一?","label":1} {"text1":"净闲寺()是位于日本东京都荒川区南千住的净土宗寺院。在吉原游廓附近,因无依无靠的妓女死后「投入寺」知名。因为地处「三轮」,所以也叫做「三轮净闲寺」。1655年(明历元年)创建,比吉原游廓(1657年)早2年。据说于吉原妓院街病死的妓女被送到净闲寺并冠以「某某卖女」的戒名埋葬,事实上没有这样的文字记载,近年的研究表明只有「自杀」、「胡乱发誓」、「逃跑」、「私通」、「吸食鸦片」等坏了吉原规矩的妓女才被会冠以「某某卖女」的戒名。这种情况下,死人被剥去衣服用破草席包裹扔到净闲寺。因为怕死人作祟,所以被当作猫狗处理,使其灵魂落入畜生道。吉原380年的历史中共有25,000名妓女埋葬与此。净闲寺的网页上说,净闲寺是安政大地震(1855年)埋葬大量遇难妓女之后才被称为投入寺的。东京都荒川区南千住2-1-12","text2":"该寺庙因什么而出名?","label":1} {"text1":"黑背鼻鱼,又称颊吻鼻鱼、颊纹双板盾尾鱼,俗名天狗倒吊、剥皮仔、打铁婆,为辐鳍鱼纲鲈形目刺尾鱼亚目刺尾鱼科的其中一个种。本鱼分布于太平洋海域,包括日本、台湾、中国沿海、菲律宾、印尼、新几内亚、新喀里多尼亚、澳洲、新几内亚、马里亚纳群岛、马绍尔群岛、索罗门群岛、斐济群岛、万那杜、夏威夷群岛、法属玻里尼西亚、诺鲁、吉里巴斯、美属萨摩亚、吐瓦鲁等海域。水深0至90公尺。本鱼体呈卵圆形而侧扁,且不随年龄而改变;鱼体最高处在鳃盖末端,成鱼体色由灰绿到墨绿色皆有。口小,端位,上下颌各具一列齿,齿稍侧扁略圆,两侧或有锯状齿。尾柄两侧各有2个橘色斑点,分别落在尾柄棘上。成鱼斑点较大,而又鱼较小。另成鱼眼到口角处有一显著的橘黄色弧线且吻与此线同色。尾鳍末端略凹入,成鱼的上下叶延长成丝状。又鱼体成淡橘色。背鳍硬棘6枚、背鳍软条28至31枚、臀鳍硬棘2枚、臀鳍软条29至31枚。体长可达70公分。本鱼栖息于珊瑚礁、石块、碎石堆或向海坡面上。以马尾藻、棱藻等叶状藻为食。可做为食用鱼类,小鱼则适合于水族箱观赏。可剥皮切片,再浸于流状米糠内,然后冰冻一日,再食之。尾柄棘会伤人,须注意。","text2":"黑背鼻鱼分布于什么海域?","label":1} {"text1":"尚清王(;)是琉球国第二尚氏王朝的第四代国王,为第三代国王尚真王的第五王子。1527年至1555年在位。神号天续之按司添()。尚清童名真仁尧樽金(),原为中城王子,是尚真王的庶子,华后所生。华后为使尚清成为王世子,不断向尚真王进谗言,诬称世子尚维衡对她欲行非礼。尚真王大怒,废尚维衡,立尚清为世子。1526年12月11日,父王尚真薨,他于翌年即位,同时派正议大夫郑绳往明朝,请求袭封。但郑绳在途中遭遇台风溺死。尚清重新派出长史蔡瀚、马吾喇等人赴明。1534年(嘉靖十三年),明嘉靖帝派吏科左给事中陈侃、行人司行人高澄为正副册封使,前往琉球国,册封尚清为王。1537年,第四代尚清王率军北伐,攻取奄美群岛。琉球王国终于将势力扩张到整个琉球列岛,确定了北起喜界岛、奄美大岛,南至宫古、八重山群岛的疆界,即琉球史书中所称「三省并三十六岛」。同年奄美大岛酋长向尚清王报告,诬称与湾大亲欲发动叛乱。尚清王发兵前往镇压。与湾大亲自缢身亡,尚清王掳其子糠中城而归。1542年,琉球商船与中国海船发生争执并发生斗殴,明廷命令琉球勿轻易中国商船贸易。1547年,修建大美御殿。1553年,尚清王在那霸港附近修建屋良座森城,以加强了对倭寇的防备。1555年6月29日,尚清王薨去,享年59岁。临终前遗命法司毛龙吟、和为美、葛可昌三人辅佐尚元。但他死后,和为美、葛可昌突然变心,欲拥立尚鉴心为新王。最后,在毛龙吟的帮助下,尚清王生前指定的继承人尚元在斗争中胜利,并在翌年继位。|-style=\"text-align: center; background: #FFE4E1;\"","text2":"尚清王在位时间是什么时候?","label":1} {"text1":"红梅花雀(学名:)是梅花雀科红梅花雀属的一种,分布于多米尼加共和国(引进种)、老挝、印度尼西亚、东帝汶、斐济(引进种)、波多黎各(引进种)、埃及(引进种)、中国大陆(云南、贵州、海南)、菲律宾(引进种)、伊朗(引进种)、文莱(引进种)、瓜德罗普(引进种)、巴基斯坦、马来西亚(引进种)、马提尼克(引进种)、孟加拉国、瓦努阿图(引进种)、意大利(引进种)、印度、巴林(引进种)、斯里兰卡、越南、尼泊尔、柬埔寨、美国(引进种)、日本(引进种)、缅甸、墨西哥(引进种)、西班牙(引进种)、泰国、留尼汪(引进种)和葡萄牙(引进种)。该物种的保护状况被评为无危。红梅花雀的平均体重约为9.2克。栖息地包括亚热带或热带的(低地)湿润疏灌丛、亚热带或热带的湿润低地林、耕地、湿地、种植园和亚热带或热带的(低地)干草原。该物种的模式产地在印度。","text2":"红梅花雀的保护状况如何?","label":1} {"text1":"本届奥运会耗资430亿美元,故此要开发市场。奥运会市场开发由国际奥委会的市场开发计划和举办城市奥组委的市场开发计划组成。国际奥委会市场开发计划由奥运会电视转播权计划和奥林匹克伙伴计划(TOP计划)以及相关计划组成。本届奥运会赞助商共64家,创下历届之最,其中包括8家大型中国企业。但是由于自身的相关经验不足和其它的非奥运赞助商企业的激烈竞争,这些赞助商在获得相关赞助商权力后表现平平,只有少数业绩满意。人们甚至把一些原本未获得奥运赞助商资格的企业当成了奥运赞助商。对北京2008年奥运会的赞助包括国际和国内两个方面:2008年北京奥运会赞助商计划将赞助商又分为2个级别,即合作伙伴和赞助商,它们均享有规定产品(服务)类别的奥林匹克市场营销排它权。北京奥组委合作伙伴和赞助商销售工作预计于今年第4季度开始。合作伙伴和赞助商的市场营销期为4年左右。奥林匹克全球合作伙伴,不仅得到了全球范围内使用奥林匹克知识产权、市场营销的权力以及其它相关权益,还得到了指定产品和服务类别的排它权。北京奥运会供应商计划分为2类,包括独家供应商和供应商。独家供应商享有规定产品(服务)类别的奥林匹克市场营销排它权。同一或相似类别产品(服务)2家以上的供应商共享规定产品(服务)类别的奥林匹克市场营销排它权。供应商的市场营销期为3年左右。2008年北京奥运会特许计划是指由北京奥组委授权企业生产、制造、销售带有北京奥运会标志、吉祥物和中国奥委会商用标志的产品。特许企业以交纳特许使用费的形式对奥运会做出贡献,不享有赞助商和供应商的权益,不能进行使用奥林匹克标志的市场营销活动。在各地都开设有北京2008年奥运会特许商品及授权零售店,特许商品零售价由北京奥组委确定,属于特许经营的商品只能在奥组委指定的特许商品零售店买到。这些特许商品零售店具有统一的店面形象,统一悬挂北京奥组委颁发的特许零售店标识。","text2":"2008年夏季奥林匹克运动会赞助商为何在获得相关赞助商权力后表现平平,只有少数业绩满意?","label":1} {"text1":"哈斯木(),哈萨克汗国第二代可汗,他是贾尼别克的儿子,巴兰杜黑汗的堂兄弟。他之前是巴兰杜黑手下一位苏丹(王子),娶了蒙兀儿斯坦羽奴思汗的女儿后,势力更大,在1508年-1509年,昔班尼大举入侵哈萨克南部,哈斯木迎战,乌兹别克人听到哈斯木汗来临,不敢应战,昔班尼一直撤退,去到波斯萨非王朝边界,被伊斯迈尔一世杀害,哈斯木在1511年放逐了巴兰都黑,他在1511年死在乌兹别克一带。哈斯木汗是哈萨克汗国力量的真正奠基人,叶尔羌汗国的米尔咱·马黑麻·海答儿(哈斯木是其姨父)说,哈斯木在钦察草原上号令一切,势力极大,除了术赤汗没有人有他那么大的权力,他在外交上联盟蒙兀儿斯坦,严防瓦剌、乌兹别克,得到塞兰城,但围攻塔什干失败,内政上,制定第一部哈萨克成文法,当时人口一百万,军力30万,由阿尔泰山至伏尔加河西岸,也有哈萨克人居住。新疆民族辞典","text2":"哈斯木是哪个国家的可汗?","label":1} {"text1":"法比安·恩斯特(Fabian Ernst,),出生于德国汉诺威,是德国足球运动员,司职防守中场,效力土耳其联赛班霸比锡达斯。出身于德国小型球会汉诺威96,曾于1998-2000年间效力汉堡。效力汉堡时共上阵48场,没有进球。恩斯特终其球员生涯,效力最大的球会乃云达不莱梅。这也是恩斯特最成功之时。他于2000-05年间效力,一共出赛152场德甲,打入11球。这段期间恩斯特随球会赢得2004年德甲和德国杯双料冠军,并于翌年打入欧洲联赛冠军杯。2005年他转会到沙尔克04,可是却只能取得两届德甲亚军(2005、2007)。2009年,恩斯特征战海外,转至土耳其班霸比锡达斯。恩斯特乃于2002年首次进入德国国家队。惟迄今仅参加过2004年欧洲国家杯一项大型赛事。截至2007年,他代表国家队出场24次,打入1球。","text2":"截至2007年,恩斯特代表国家队出场多少次?","label":1} {"text1":"柯灵()是中国电影理论家、剧作家、评论家。原名高季琳。原藉浙江绍兴,生于广州。少年失学,自修走上文学道路。长期从事报刊编辑、电影、戏剧工作。1926年,在上海《妇女杂志》发表第一篇作品。1932年,参加左翼电影运动,为影评小组成员。1933年任明星影片公司宣传主任,1937年后相继主编《文汇报》、《大美报》、《正言报》副刊,此期间至解放前夕,写作了电影剧本,改编了大量舞台剧。建国后历任上海法政学院新闻专修科教授,《文汇报》总编辑兼副社长,《万像》、《周报》主编,文化部上海电影剧本创作所所长,上海电影艺术研究所所长,上海电影局顾问等职。据张爱玲的自传小说《小团圆》指出,柯灵本来对她很好,而她们夫妻也曾在其落难时搭救他(胡兰成提过这件事,柯灵承认是自己)。但在胡兰成落难后张爱玲随之变为「汉奸的女人」时,柯灵却伺机于公车上对之性骚扰。小说集著作剧作","text2":"柯灵在哪里发表了它的第一篇作品?","label":1} {"text1":"崔真实(,),已故韩国女演员。20岁时以拍摄MBC剧集《朝鲜王朝500年-恨中录》出道,1995年凭借电影《杀妻秘笈》获得第33届韩国大钟奖最佳女主角奖及第31届韩国百想艺术大奖人气奖。2008年10月2日,因深受被指迫害安在焕烧炭自杀谣言困扰,在住所浴室利用绷带上吊自杀,享年岁。事后韩国政府拟于11月立法信息通信网法施行令修正案,防止匿名网民发放流言。曾经与日本职棒选手赵成珉于2000年结婚,因此息影,育有2名子女。之后在2002年10月因为家庭暴力,崔真实怀孕被推落楼梯,而在2004年9月离婚。2008年5月崔真实的两名子女依照当地的户籍法改随母姓。生前崔真实最后的日子与母亲同住。其弟崔真永亦为演员,2010年3月29日自杀身亡,得年39岁。2013年1月6日,其前夫赵成珉被发现在家自杀身亡。2013年11月27日,前经理人朴尚浩自杀。","text2":"崔真实是哪个国家的女演员?","label":1} {"text1":"裂带天竺鲷(学名:),为辐鳍鱼纲鲈形目鲈亚目天竺鲷科的一种,俗名粗体天竺鲷。本鱼分布于印度西太平洋区,包括马来西亚、印尼、越南、琉球群岛、菲律宾、所罗门群岛、台湾、澳洲、密克罗尼西亚、万那杜、帛琉等海域。该物种的模式产地在菲律宾。水深2至20公尺。本鱼体延长而侧扁,眼大,口大略下位。体色呈白色或略带桃色,在体侧边上有大约6条红褐色的斑纹;在尾鳍基底有3至4个深色斑点。眼具蓝色圆圈包围。稚鱼有黄色的尾梗且在中心有一黑色斑点,背鳍硬棘8枚;背鳍软条9枚;臀鳍硬棘2枚;臀鳍软条9枚,体长可达12公分。本鱼栖息于珊瑚礁区,白天躲藏于岩洞中,夜间出来觅食,属肉食性,以软体动物或其它底栖甲壳类为食。繁殖期时,雄鱼具有口孵习性,卵约7日化成仔鱼,由雄鱼吐出,具短暂的仔鱼飘浮期。可做为观赏鱼。","text2":"裂带天竺鲷分布在什么地方?","label":1} {"text1":"共和国广场(捷克语:Naměstí republiky)是捷克城市比尔森古城的一个广场,规模为552 x 627 英尺 ,是捷克最大的中世纪广场之一。自13世纪末比尔森创建城市起,该广场和已经存在。在这个长方形广场周围,是由直角相交的街道划分出的规则的住宅街区。考古学家鉴定,此处自13世纪铺设木质地面。1859年,广场 铺设了鹅卵石,后来改铺沥青。周边建筑以哥特式和文艺复兴风格占据优势。在东部的庭院内,仍保留有中世纪城墙的残余。广场的南部保护得最好。许多建筑物设有2或3层地窖,用于保存食物、水井或污水坑。最重要的地标为圣巴尔多禄茂主教座堂(1295年兴建,1993年成为主教座堂)、比尔森市政厅(自1496年起)和圣母玛丽亚黑死病纪念柱(1681年),全部位于广场的北部。","text2":"共和国广场在哪里?","label":1} {"text1":"泡泡浴()()直译为肥皂乐园,亦有译为泡泡澡,是存在于日本的一项性服务,曾经被命名为土耳其浴(),后来由于抗议改为现名,内容类似泰国浴。具体上来说,就是由一名性工作者向嫖客提供性服务的一种方式。在很多知名的成人电影作品当中,泡泡浴通常有性暗示的意味,即妓女通常会帮客人先抹上肥皂,而且方式与一般方式(鸳鸯浴)的有所差异,妓女会以身体的任何部位(如大腿、胸部甚至阴部)透过身体互相摩擦的方式抹上肥皂,借由全身的力量达到高级情趣的意味。在日本风俗店中,泡泡浴属于店舖型性风俗营业1号。根据统计,日本全国的泡泡浴店舖,目前有1,249家(平成20年,2008年)。另外,某些泡泡浴店仅限日本人前往消费而谢绝外国人。泡泡浴最早被日本叫做土耳其浴(日语:),原意是指中东奥斯曼帝国的一种公共浴室,与伊斯兰教的教义规定祈祷之前必先沐浴有关。土耳其浴巧妙的被日本人理解运用,后因土耳其留学生向厚生省诉求。1984年12月19日,「东京都特殊浴场协会」公开寻求改名,借用英文Soapland改为泡泡浴(ソープランド)的现名只对外国人开放的日本第一而且唯一的泡泡浴在2017夏天开放了","text2":"泡泡浴最早被日本叫做什么?","label":1} {"text1":"协和桥(Pont de la Concorde)是法国巴黎一座跨越塞纳河的拱桥,介于协和广场的堤道(quai des Tuileries)(右岸)和奥赛堤道(quai d'Orsay)(左岸)之间。它在过去曾称为路易十六桥(pont Louis XVI)、革命桥(pont de la Révolution)、协和桥,波旁复辟时期(1814年)复称路易十六桥,1830年再度恢复协和桥名称,直至今日。附近有地铁的国民议会站和协和站。1787年,建筑师Jean-Rodolphe Perronet受命建造新桥。计划开始于1755年,当时开始兴建路易十五广场(今协和广场),以取代此处的渡口。在动荡的法国革命中工程继续进行,利用了拆卸巴士底狱的石材建造,完成于1791年。1810年,拿破仑一世在桥的置于两旁安放法兰西第一帝国8位阵亡将军的雕像。波旁复辟时期改为12座巨大的大理石雕像,其中包括4名大臣Suger、苏利、黎塞留、柯尔贝尔,4名皇家将军(杜·盖克兰、贝亚德、孔代, 蒂雷纳子爵)和4名航海家勒内杜高依、迪凯纳、Suffren、图尔维尔.然而,这些雕像对于桥梁太过沉重,路易-菲利普一世将它们转移到了凡尔赛宫。过桥交通变得十分拥挤,1930年到1932年,该桥加宽到原来的2倍。工程师德瓦尔和马利特仔细维护新古典主义建筑原貌。1983年最后一次修复。今天,这座桥是巴黎的道路交通要冲(环城大道的桥梁除外)。","text2":"1787年,谁受命建造新桥?","label":1} {"text1":"韩通(),字仲达,并州太原(今属山西太原)人。后周军事将领。曾祖韩莹,曾祖母京兆郡第五氏,封汧国夫人。祖赠太尉,祖母清河郡太君张氏,封卫国夫人。父亲韩章,赠太子太师,母谯郡太夫人李氏,封陈国太夫人。少年应募从军,勇于作战,被提拔为骑军队长。后晋开福元年(947年),投靠刘知远,讨杜重威有功。干祐元年(948年),随郭威出兵,身上有六处创伤,被提拔为都虞侯,担任了天雄军马步都校。韩通性情刚直,人称“韩瞪眼”。显德七年(960年)正月,殿前都点检赵匡胤在陈桥发动兵变,黄袍加身之后回军开封。韩通抵抗,被军校王彦升杀死,全家皆被屠尽。韩通是陈桥兵变中唯一试图抵抗而被杀的后周重臣。宋太祖即位后,追赠为中书令。虽然宋太祖赠韩通中书令,但是宋史周三臣传记述,后来太祖到开宝寺,见到韩通与其儿子画像,下令除去该画。故有说法认为宋太祖早就忌讳韩通。前夫人董氏,封陇西郡夫人,有子二人;继室蒋氏,卫国夫人,生一子。有女二人:","text2":"为什么韩通被称为韩瞪眼?","label":1} {"text1":"云起游戏代理有限公司(Run Up Game Distribution Limited),简称云起游戏,于2002年成立,主要业务为营运线上游戏以及代理销售其相关产品。云起游戏扎根于香港,乃香港少数的本土游戏代理公司之一,直至近年积极于开拓海外的游戏市场,业务已经拓展至亚洲多个地区,而且更先后于上海、深圳、台湾、马来西亚及新加坡等地开设分公司。云起游戏于2006年开始进军线上游戏研发业务,从基本的游戏概念、系统策划、甚至人物设定等皆主动参与,终于在2007年12月成功将第一个著手参与研发的作品《幻想学园Online》打进港澳市场,并计划将游戏推广到台湾及中国大陆等地。","text2":"云起游戏主要开拓了哪一地区的游戏市场?","label":1} {"text1":"朝鲜马鲛(学名:),又称高丽马加䲠,俗名阔腹、白北,为辐鳍鱼纲鲈形目鲭亚目鲭科的其中一种。本鱼分布于印度西太平洋区,包括印度、巴基斯坦、孟加拉、斯里兰卡、中国、台湾、香港、日本、韩国、印尼、马来西亚、缅甸、越南、泰国等海域。该物种的模式产地在日本。水深0至50公尺。本鱼体形较短,横断面略侧扁,腹部甚宽,头高大于头长,尾鳍大而深分叉;第二背鳍、臀鳍及尾鳍发达,背部灰绿色,腹部银白色,体侧有4至5个纵列暗色斑,背鳍硬棘14至17没;背鳍软条20至24没;臀鳍硬棘0没;臀鳍软条20至24枚;脊椎骨46至47个,体长可达1.5公尺。本鱼为暖海域之洄游鱼类,常在表层海域追食沙丁鱼等鱼类;游速快,喜栖息于海流湍急的岛礁附近。为高经济价值鱼类,体型大富脂肪,适合各种烹饪方式食用。","text2":"朝鲜马鲛的模式产地在哪里?","label":1} {"text1":"威廉·亨利·“斯马什”·帕克(William Henry \"Smush\" Parker,),绰号“死神”,美国职业篮球运动员,身高1.93米,场上位置组织后卫。帕克出身于洛克公园,是著名的街头篮球明星,2002年弃学成为职业球员,但在当年的NBA选秀中落选。克里夫兰骑士队为其提供了一年合同,但在次年将其放弃。帕克之后前往希腊打球。之后重返NBA,先后短暂效力于底特律活塞队和菲尼克斯太阳队。2005年赛季开始前,帕克意外地得到了洛杉矶湖人队传奇主教练菲尔·杰克逊的垂青,不但获得了湖人队的合同,还一举成为其主力组织后卫,同科比·布莱恩特成为后场搭档。但是在2007年季后赛开始前,杰克逊开始培养新人乔丹·法玛尔,帕克丢失了主力位置。不满于忽然的失宠,帕克在新赛季开始前转投迈阿密热火队,但是在热火队帕克并无出色表现,在2008年3月10日被热火队裁员。其后洛杉矶快船队和丹佛掘金队都对其表示过兴趣,但最终都放弃了。之后帕克栖身于NBDL。2009年1月,帕克加盟CBA的广东东莞银行队,并帮助广东华南虎队夺得了连续两个赛季的CBA总冠军。2010年9月,帕克与俄罗斯球队签约一年。","text2":"威廉·亨利·“斯马什”·帕克的职业是什么?","label":1} {"text1":"《王菀之Ivana首张国语创作专辑》为王菀之在2007年推出的首张全国语创作专辑,台湾版发行日期为2007年12月6日,香港版发行日期为2007年12月18日,第二版发行于2008年2月15日。第一版包括一只CD,共收录了十一首新歌及两首Bonus Track。除了四首歌曲为全新创作外,其余歌曲均从王菀之旧作改编。香港版特别收录全新广东歌《奇异恩典》;第二版包括一只CD及一只DVD,DVD中收录了三首新歌的MV。全碟歌曲均由王菀之作曲(《奇异恩典》除外)。全碟由王菀之作曲,除了《奇异恩典》由William Walker作曲及王菀之附加旋律。备注:","text2":"《王菀之Ivana首张国语创作专辑》的香港版发行的日期是?","label":1} {"text1":"氯化铬(化学式:CrCl)也称三氯化铬、氯化铬(III),是常见的铬(III)化合物之一。有无水物和六水合物两种。无水物为强烈发光的紫色结晶,几乎不溶于水。六水物 formula_1 是一个配合物,有三种水合异构体:formula_2formula_3,分别为紫色、浅绿色和暗绿色的固体。一般买到的都是暗绿色的异构体。三氯化铬晶体中含有连接成层的 CrCl 八面体单元,结构中存在螺旋状位错,不含有金属-金属键。它与三碘化铬同构。氯化铬是较硬的路易斯酸。其中含 d 构型的三价铬,对于配体置换反应来说是惰性的。为了提高其活性,可以加入少量的还原剂(如锌 \/ 盐酸),将其还原为氯化亚铬,很快发生配体交换反应,然后使其与 CrCl 通过氯桥发生电子转移,获得三价铬的配合物,并再生成少量活性的Cr(II),直至所有Cr(III)都发生了取代反应。无水三氯化铬几乎不溶于水,但在还原剂(如锌)存在时会缓慢溶解,原因可能是生成了电荷转移的桥连配合物 [Cr-X-Cr…X]。溶解的产物是紫色的 [Cr(HO)] 离子。如果配体是吡啶,则产物是 [CrCl(CHN)]。Cr(III)的配位数为6的配合物大多都是八面体型的。与碱金属氯化物(如氯化钾)熔融时,三氯化铬生成含八面体型 [CrCl] 离子,以及多聚生成的 CrCl 等离子的盐类。无水三氯化铬可通过单质高温化合制备。或者800°C时,用三氧化二铬与氯气在碳存在下反应也可以得到无水三氯化铬。六水合三氯化铬在650°C时与四氯化碳蒸汽反应,可以得到不含 CrOCl 的无水三氯化铬。也可以用亚硫酰氯脱水。水合三氯化铬可由金属铬与盐酸反应得到。无水三氯化铬是有机金属化学中的重要原料。以它为原料可以制得很多有机铬化合物,比如结构上与二茂铁类似的二苯铬(下图)。三氯化铬也是很多铬(III)配合物的起始原料。有机合成中,CrCl 被原位还原生成的 CrCl 是常用的有机还原试剂之一。它可以(A)将 C-Cl 键还原为 C-H 键,也可以(B)与醛作用,将其还原为烯基卤化物。第二个还原反应中通常用2:1摩尔比的三氯化铬和氢化铝锂。三氯化铬的路易斯酸性可以用于催化某些反应,例如用亚硝基化合物作亲双烯体的狄尔斯-阿尔德反应。","text2":"六水物 formula_1 是什么?","label":1} {"text1":"动物扮演是指人加上一些道具去扮演一些实际存在的动物或是幻想的动物,并且模仿该动物的肢体行动,例如扮演小狗则以四肢著地地爬行。扮演动物的理由很多,例如Cosplay、情趣游戏、商户吸引顾客等,有的只是外表动物化,心理还是保持人的状态,但是稍稍受所扮演的动物所影响,好比说扮演兔子、小猫就以比较温驯的姿态。扮演花豹、小狗,则是比较狂野姿态。有的是连心智都要表现得像动物,常见于BDSM的游戏中,而伴侣的角色也可能是扮演动物,或是驯兽师、宠物主人或是骑师。扮演动物者,受到支配者以对待动物的方式对待获得满足。好比扮演小猫、小狗者被主人以项圈拴住,或是关进兽笼,以宠物的姿态生活进食。或是扮演马,穿著马具拉马车,承受伴侣的鞭子。无性暗示的动物扮演则常见于原始部落文化,像是北美洲的原住民文化以及史前传说中,半人半兽的动物在他们的文化仪式中有很重要的地位。这些动物如神兽般地被尊敬,有的时候也拿来教育儿童,要求学习动物的求生智慧,以及运动姿态。扮演动物的原因,往往与参与的人有大的歧异。不单只是一般的角色扮演,这牵涉到很多戏剧、小说、神话、传说以及心理戏剧的观点来分析这样的现象。就如同一般的情趣游戏或是角色扮演,动物扮演的理由跟参与者的喜好心情相关。可能从只是简单的模仿马的嘶叫声、狗吠声,或是小猫小狗活动的姿态。到被主人喂食、被当作宠物把玩。有点类似年龄游戏(Ageplay),透过游戏可以当一些别于日常生活所扮演的角色。更复杂的可能如母马扮演时,加上一些马具、嘴里衔住口衔、加上马尾以及穿上模仿马蹄的靴子。扮演母猫则可能加上猫耳、项圈、猫蹼以及猫尾。西方式的小猫游戏,主要是从性感内衣为出发点,外观上较为性感,小猫泛指所有的猫科动物,如花豹、老虎等皆有人扮演。东方式或是日式的小猫游戏受卡通、ACG影响较深,因此比较属于可爱形式。不论是东方还是西方,最终目的是要充分令自己显出猫的特征。以下简单地并依重要性及普遍度列出:","text2":"扮演动物的理由有哪些?","label":1} {"text1":"莫斯克内斯岛(Moskenesøya)是挪威罗弗敦群岛中的一个岛,这座186平方公里的岛上有莫斯克内斯和弗拉科斯塔两个城镇。该岛有很多冰山,其中最高峰是海拔1,029米的Hermannsdalstinden山。岛从西南向东北,长约40公里,宽约10公里。其海岸线很崎岖。它与附近的弗拉克斯塔德岛通过科肯桥(Kåkern Bridge)相连,欧洲E10公路通过该桥,其终点是岛上的奥镇。岛上有很多村庄。弗拉科斯塔镇位于岛的北部,其下辖村庄包括Fredvang、Selfjord和Krystad。莫斯克内斯镇位于岛的南部,其下辖村庄包括奥镇、Hamnøy、莫斯克内斯、雷讷、Sakrisøy、Sørvågen和 Tind,所有的村庄位于岛的东侧。西海岸曾经有人居住,但是由于风暴,1950年代最后一个居住地被废弃。","text2":"莫斯克内斯岛上有几个城镇?","label":1} {"text1":"沈果孙(),中国围棋职业棋手和记者,江苏常熟人,出身书香门第,后来加入山西围棋队。是中国首批获得段位的十名棋手之一。1962年获得全国围棋个人赛第五名,1964年获第三名。1973年在中日围棋对抗赛上,沈果孙执白中盘战胜当时日本顶尖棋手坂田荣男九段,成为第一位战胜日本围棋现任冠军的国内棋手。1982年被授予七段。在20世纪80年代初因故离开国家队后,沈果孙曾担任中国体育报社《棋牌周报》的主任记者,他的《围棋基本定式》、《围棋常型百例》、《棋理与要诀》、《围棋的正着与俗手》、《围棋定式以后》等十多本围棋著作,发行量超过100万册,在广大棋迷中有很好的口碑。后长期从事围棋教育工作。学生包括邵震中九段、江铸久九段、周逵三段、彭立峣三段、付冲二段、赵威六段、曾扬杰六段等围棋高手。2006年10月在徐州开办沈果孙围棋道场。近来为了让现役国手摆脱“恐韩症”,他把自己多年来研究中韩对局的成果结集,写成了《围棋的道与魔》。","text2":"沈果孙有哪些围棋著作?","label":1} {"text1":"交通咨询委员会(简称交咨会)(英文:Transport Advisory Committee)是香港一个公营机构,现时由香港行政长官委任,负责就香港的交通运输事宜向行政长官会同行政会议提出意见。该会于1965年12月成立。现任主席是广西政协郭琳广律师。交通投诉组(英文:Transport Complaints Unit (TCU))在1980年成立,隶属交通咨询委员会,负责接受和处理市民就交通事宜提出的建议和投诉,并会将有关的投诉及建议,转交运输署、警务处、其他有关部门以及公共交通机构调查及跟进。投诉组处理的个案内容包括公共交通服务(如小巴及的士等)、交通情况(如交通挤塞及泊车设施等)、道路维修(如交通标志及设备等)及法例执行(如违例泊车等)。投诉人可透过电话、电邮、传真、信件或交通投诉组投诉表格提出。","text2":"投诉人可透过什么方式向交通咨询委员会提出投诉?","label":1} {"text1":"预科学校是一种中等教育机构,通常为私立,专为学生升入大学或学院而设立。一些学校也包括小学。该名称通用于北美洲。在欧洲许多地方,例如德国、前奥匈帝国国家、低地国家和斯堪的纳维亚,专门把升大学预备的中等学校称为文科中学(\"Gymnasium\",又译文理中学)。在美国和加拿大,有3种类型的预科学校。设有生活区(宿舍、餐厅)者称为寄宿学校;大部分为走读学校(day school),还有少数学校兼有两种方式,香港以往的预科学制沿袭英国制度,共有两个学年(中六及中七),学生完成后会应考香港高级程度会考以升读大学。而在这学制下,一些学校专门提供预科课程,并保送学生参加高级程度会考,进入大学,较著名的有恒生商学书院和保良局庄启程预科书院等,这类学校可视为预科学校。从2009年9月开始,因应三三四学制正式推行,高中的学段逐步改为三年,大学则扩至四年,并逐步停办预科。香港预科课程于2012年正式取缔,这些学校亦要另寻出路,如转型为高中学校或高等教育机构。","text2":"美国和加拿大有哪3种类型的预科学校?","label":1} {"text1":"港铁巴士K68线是由港铁巴士营运的西铁线及轻铁接驳巴士,属循环线,行走元朗工业邨及元朗公园之间,途经朗屏邨、元朗市中心等地区。本路线于2004年1月18日起配合656线分拆为两条路线而投入服务,方便往来元朗工业邨至元朗市及元朗公园的乘客。最初路线往元朗公园方向是以顺时针方向行走,但是绕经朗屏站公共运输交汇处迂回,加上在妈横路或元朗安乐路已能够接驳西铁,因此于2004年4月25日便不再绕经朗屏站公共运输交汇处,往元朗公园方向也改以逆时针方向行走。配合元朗南进一步发展,由2009年10月12日起,往元朗公园方向绕经十八乡路(翘翠峰、南元朗官立小学)及榄口村路,成为唯一一条全日服务该带的公共交通服务。此外,本路线是现时唯一一条只于元朗新市镇内(包括元朗工业邨和元朗南)服务的路线。本线初期使用富豪奥林比安(2xx)、富豪B10M(4xx)单层巴士和丹尼士三叉戟三型(7xx)行走,但后来引入亚历山大丹尼士Enviro 200 Dart单层巴士,把富豪B10M(4xx)调走,单层用车改派亚历山大丹尼士Enviro 200 Dart单层巴士(9xx),但现时本线用车大多以亚历山大丹尼士Enviro 500 MMC(5xx)及富豪B9TL(3xx)为主,亚历山大丹尼士Enviro 200 Dart数目渐渐减少,令本线用车都是双层巴士,而丹尼士三叉戟三型(6xx)、Enviro 400(14x)、亚历山大丹尼士Enviro 500 11.3米(825-833)间中会行走本线。由于本路线两边总站位置都是偏远的,加上并非驶经元朗区心脏地带,除繁忙时间外客量只属一般,不过由于沿途没有其他巴士路线竞争,接近独市生意,所以仍有一定需求。现时本线主要客源是出入元朗工业邨、横洲杨屋村、朗屏邨、赵聿修纪念中学、元朗公立中学以及公园北路一带住宅的乘客。","text2":"港铁巴士K68线为什么要拆分成两条线路?","label":1} {"text1":"Boost C++ 函式库(Libraries)是一组扩充C++功能的经过同行评审(Peer-reviewed)且开放源码程式库。大多数的函式为了能够以开放源码、封闭专案的方式运作,而授权于Boost软体授权条款(Boost Software License)之下。许多Boost的开发人员是来自C++标准委员会,而部份的Boost函式库成为C++的TR1标准之一。为了要确保函式库的效率与弹性,Boost广泛的使用模板(template)功能。而它是针对各式领域的C++使用者与应用领域(Application Domain)上,包含的函式库类别从像smart_ptr 函式库这种类通用函式库,到像是档案系统的作业系统抽象层,甚至能够利用Boost来开发额外的函式库或是给进阶的C++使用者利用,像是MPL。现有的 Boost 包含大约150种不同的函数库,以下面几项做范例:Boost 包含了 uBLAS 线性代数函数库,能够借由基本函数库子函数(BLAS)来支持向量与矩阵形运算。Boost 也提供独立分布的模拟随机与 PRNG 独立性的机率分布,而这些能够具体的建立产生器。更详细的说明请参阅 Boost 随机数库。范例码演示建立执行绪:","text2":"Boost的开发人员主要来自哪里?","label":1} {"text1":"刘芸(),长沙人,中国女演员,现居于北京。毕业于中央戏剧学院2000级表演系。2001年,还在读大一的刘芸通过出演《命运的承诺》进入演艺圈,2007年她还成为继章子怡、李宇春之后第三位登上美国《时代周刊》的中国艺人。刘芸出生在长沙,11岁时进入广州舞蹈学院少女舞蹈团学习舞蹈。1997年毕业后被分配到广东南方歌舞团。但她希望考上大学,通过一番努力,2000年她考入了中央戏剧学院表演系。在大学,刘芸运气不错,虽说中央戏剧学院不准许大一学生拍戏,但因人情关系,她获得了许可,从《命运的承诺》起,她拍了一连串的戏,而且多数都是女一号。但拍戏太多,耽误学习也导致她差点没能毕业。刘芸演艺起点虽然不低,但她在之后的发展却并不顺利,被媒体关注更多的也只是她的感情生活。刘芸和知名演员聂远于2003年拍摄《汗血宝马》时开始相恋,这段地下感情维持了三年,最终因二人个性好强而分手,也有人爆料称二人分手是有“第三者”介入。对于这段恋情,当事人没有否认,但都表示二人之间已是过去式了,没必要再提起。因拍摄《大汉天子2》和《鹿鼎记》与黄晓明合作,被传出二人相恋,但二人都给予了否认,表示只是朋友。2010年7月,刘芸在一个访谈节目中承认她已和著名摇滚歌手郑钧结婚,二人于5月26日在西安注册结婚。据报道二人2007年开始相恋。而对于有人指责她是“第三者”(郑钧有前妻)、郑钧抛妻无情无义的说法,刘芸表示这只是误解,并表示每个人都有过去,自己和老公都问心无愧。同年10月23日,刘芸在美国洛杉矶产下儿子,两个月后回国。对于能登上2007年9月封面,刘芸觉得自己很幸运。虽然名气不大,但《时代周刊》认为她长了一张非常东方的脸,“我这次只是靠脸蛋取胜的幸运儿。因为他们觉得我的脸形适合上《时代周刊》”。《时代周刊》对她的评价则是“刘芸代表了中国刚刚成长起来的新一代。他们年轻、快乐、不愁吃穿、无拘无束、真实自然”。","text2":"2007年刘芸还成为继谁之后第三位登上美国《时代周刊》的中国艺人?","label":1} {"text1":"白阿绣,是金庸小说《侠客行》里头的人物,小说中只称呼为「阿绣」,「白阿绣」不一定为其本名。白阿绣是雪山派掌门白自在的孙女,美丽秀气,凌霄城上下将她像小公主一样疼爱。但是在她十三岁时,被十五岁的师兄石中玉绑住手脚脱光衣服企图强奸,幸亏仆人发现即时挽回名节,她仍然不堪耻辱,哭了三天后跳下山崖自尽。从雪山派出走的奶奶史小翠在山下救起白阿绣后,祖孙二人到中原散心,在武昌府偶遇对史小翠痴恋的丁不四。她和奶奶不愿接受丁不四的要胁上碧螺山,于是加紧修练武功但导致身子动弹不得,后来石破天的误打误撞,将祖孙二人救到紫烟岛。众人见到石破天均以为他是石中玉,但是白阿绣凭借直觉认定他跟石中玉不一样,让石破天犹如含冤得到昭雪,又在紫烟岛的相处,两人进而相惜相恋,日后也得到雪山派的成全。","text2":"白阿绣是哪部作品里面的人物?","label":1} {"text1":"罗马治世(拉丁语:Pax Romana),又称罗马和平,是指罗马帝国存在的五百多年间,前二百年比较兴盛的时期,亦即盛世。公元前30年,屋大维消灭埃及托勒密王朝,结束了罗马内战。一般将这一年视为罗马和平时期的开始,也有一个说法是从公元9年(罗马基本上停止侵略日耳曼地区)开始算起。公元161年,五贤帝中的最后一个─马尔库斯·奥列里乌斯即位,不久后遇上帕提亚入侵亚美尼亚,紧接著又发生瘟疫,他本人更在征伐日耳曼的途中病逝。期间,罗马帝国的财务状况恶化,日耳曼人也开始骚扰边境,帝国由盛转衰。从屋大维统一罗马至马尔库斯·奥列里乌斯过世(公元180年),长达200年左右的时间罗马大致富强稳定,没有较大的战乱,因此史称“罗马治世”。","text2":"为什么一般公元前30年一般将这一年视为罗马和平时期的开始?","label":1} {"text1":"蕊蕊(,),原名陈蕊蕊,香港女歌手、演员及前舞蹈员。前无线电视舞蹈艺员。蕊蕊中学时就读乐善堂梁植伟纪念中学,其后毕业于EF国际语言学校(伦敦分校),主修语言为英语。2004年入读无线电视第9期舞蹈艺员训练班,并签约无线电视。至2009年因演出该台综艺节目《耳分高下》,且担任「Sing之天使」其一「柑桔」,使她渐渐受人注目。同年11月约满后,与唱片公司自由行娱乐(Freeway Entertainment)签约,经理人是汉洋。2011年2月,她发表首支派台歌曲《Make Up Baby》。2014年4月,蕊蕊与邓咏雪、陶枳樽和何丽娟组成女子歌唱组合「HunterZ」。2016年,她离开自由行娱乐和签约新公司。蕊蕊与台湾艺人金刚于2013年开始交往,但两人在2016年4月因性格不合而结束恋情。","text2":"2009年蕊蕊与那个公司签约?","label":1} {"text1":"陈思荣(Chan Sze Wing,),生于香港,香港足球运动员,可司职左后卫和左中场,优点是速度快,不论快速短传或是长传能力亦属上乘。陈思荣自小学六年级加入香港体育学院。四年后足球部解散,陈思荣加入东联二合青年军,两年后转踢流浪预备组,2000年被提升上甲组队,效力一年后决定退出全职足球,只以业余身份效力南华预备组。两年后转而加入大埔足球会,在大埔升上甲组以后,陈思荣仍然以兼职球员身分出赛。2009年6月6日,大埔于决赛中以4-2击败天水围飞马,升班三年后获得首个锦标。陈思荣赛后称:「以业余球员身份去取得冠军可以说是难上加难,我要放弃很多私人时间,以及家人的支持,才有动力继续我的足球事业。可以协助大埔由丙组踢到甲组,并且在甲组比赛夺冠是对自己能力的肯定,现在的心情难以形容,很想多谢大埔街坊多年支持。」陈思荣与队友陈旭智、吕志兴等自幼相识,他的正职为眼镜销售员,而之前的正职则是贸易公司文员。另,他拥有D级足球教练牌照。","text2":"陈思荣球技的优点是什么?","label":1} {"text1":"巴黎第三区()是法国首都巴黎市的20个区之一。该区位于塞纳河右岸,面积1.171km²,是巴黎市第二小的区。该区包括中世纪风格的玛莱区北面较安静的部分(玛莱区较活跃的南部,包括巴黎的同性恋区,属于第四区)。巴黎保存至今最古老的私人房屋,建于1407年,位于第三区的蒙莫朗西路()。虽然小但是扩展迅速的中国城,居住着来自中国温州的移民,聚居在市长路()。附近是国立巴黎工艺技术学院(),坐落在中世纪Saint-Martin-des-Champs小修道院中。该区的人口峰值在1860年区划调整之前。1999年,该区人口为34,248人,工作岗位 29,723个。人口峰值出现于1860年区划调整以前。","text2":"巴黎第三区的人口峰值出现在何时?","label":1} {"text1":"曾有翼(),字子敬。奉天城南红菱堡人。中华民国政治人物。曾有翼幼年因家庭贫困而没能上学,常常在学堂窗外偷听。后来他经过自学,于清朝末年中秀才,后成为光绪三十二年(1906年)丙午科优贡。1907年,他考入京师大学堂。毕业后,他历任奉天蒙文学堂监督,奉天省立第一中学历史教员,提学司议绅,奉天省教育厅谘议,教育总会副会长、会长等职。中华民国成立后,1912年他进入政界,当选北京临时参议院议员。此后,他到1922年共三次出任国会众议院议员。他还历任奉天行政公署教育咨议官,东北盐运使兼山海关监督,奉天督军公署秘书等职务。1923年,奉天市政公所筹备处成立。同年,奉天市政公所成立,曾有翼任奉天市首任市长,兼奉天电灯厂厂长。1926年9月,他辞去奉天市市长等本兼各职,到哈尔滨投靠吴俊升,任东三省铁路督办公署参赞兼秘书长,1932年改任顾问。1936年,曾有翼逝世。","text2":"他共几次出任国会众议院议员?","label":1} {"text1":"汪文言(),谱名守泰,南直隶徽州府歙县(今安徽歙县)人。明朝官员。官至中书舍人,与李漕抚(李三才)被称为「漕汪二贤」,朝有李漕抚,野有汪文言。又与黄尊素并称「黄汪」,又称「东林党二智囊」,在东林党争中,汪遭魏忠贤下狱,魏党的锦衣卫指挥使许显纯将他拷打而死。出身游侠,少年为狱吏,饶具谋略,因监守自盗,逃奔燕京大宦官王安门下。和东林党党人杨涟、左光斗、魏大中过从甚密。刑部郎中于玉立派遣他入京师探事,以银钱捐监生,用计离间齐楚浙三党,从此东林党独大。后世对他评价: “以布衣之身操控天下。”天启元年(公元1621年)九月,王安死,顺天府府丞邵辅忠弹劾汪文言。又被御史梁梦环弹劾,被逮下狱。后查明无罪,释放。天启五年(1625年),宦官魏忠贤专权,以杨涟等人透过汪文言,收受熊廷弼贿赂,将汪文言下锦衣卫镇抚司诏狱,都指挥佥事许显纯对汪文言严刑拷打,施以“械、镣、棍、拶、夹棍”,汪文言外甥设法探监,见其伤痕累累,难过大哭。汪文言骂他没出息。汪文言遍体鳞伤,厉叱:“天乎冤哉!”却仍不肯诬蔑东林党人。许显纯于是自己编造了汪文言的自白书,文言垂死大呼:“你不要乱写,我到时候(死后)一定当面跟你对质!”许显纯把他打死灭口。","text2":"后世对汪文言做出了何种评价?","label":1} {"text1":"天主教康布雷总教区(拉丁文:Archdiocesis Cameracensis)是罗马天主教在法国北部设立的一个总教区,范围相当于北部省的Avesnes-sur-Helpe、康布雷、杜埃、瓦伦谢讷专区。康布雷教区成立于第6世纪,管辖范围包括今比利时。1559年西班牙国王分出梅赫伦总教区和另外11个教区以抵抗宗教改革,康布雷教省的范围大为缩减,附属教区还有Saint Omer教区、图尔奈教区和那慕尔教区。1802年,康布雷失去所有附属教区,自身也附属于巴黎总教区。1817年,教宗和法国国王希望成立里尔教区,遭到康布雷主教Louis de Belmas的激烈反对。1841年,康布雷恢复总教区,附属教区为阿拉斯教区。现任总主教为2000年就任的 François Charles Garnier。2002年,康布雷失去教省地位,转而附属于天主教里尔总教区。","text2":"天主教康布雷总教区是哪里的一个总教区?","label":1} {"text1":"马永贞()回族,山东省临清直隶州邱县陈村(今河北省邱县陈村回族乡陈村)人。清朝末年拳师。马永贞自幼习武,擅长查拳、弹腿功,还善于驯马、骑马。离开家乡后,靠武艺生活,曾在河南周口、江苏扬州多次和人比武。后到上海卖艺收徒,是最早在上海传授查拳的武师。他在上海还以贩马维生,曾多次赴蒙古、热河、察哈尔贩马。咸丰十一年(1861年)秋季,在上海跑马厅(1851年建,今人民广场和人民公园)举办的赛马会上,马永贞战胜了前两届赛马冠军洋人史蒂夫。马永贞曾任清朝松江府正营武备教官。光绪五年三月二十二日(1879年4月13日),马永贞应邀到上海“一洞天”茶楼(原址位于今南京东路金华路)吃茶,遭马贩顾忠溪纠集打手暗算,经朋友送到上海体仁医院抢救无效,当夜死亡。1927年电影《山东马永贞》中,还虚构了马永贞的胞妹马素贞得知后,只身到上海手刃杀兄仇人而回的情节。","text2":"马永贞在什么地方战胜了前两届赛马冠军洋人史蒂夫?","label":1} {"text1":"少年队()是日本杰尼斯事务所旗下的三人组合,为日本1980年代非常受欢迎的偶像团体,领导的是锦织一清(东山只在少年队梦这个节目内领导)。少年队原名是B Team,后期更名为Johnny's少年队,在1981年组成。当时成员是锦织一清、植草克秀及松原康行,负责为近藤真彦及田原俊彦伴舞。后来松原康行退出,1982年东山纪之加入,成为现在的少年队。经过3年训练及演出后,他们在1985年12月12日正式出道。初次亮相时的广告标语是「日本出发,前往世界」。1986年获得第28回日本唱片大赏「最优秀新人赏」。出道一年后,1986年少年队开始有他们的舞台剧Playzone,并在每年都在夏天举行,直至现在他们仍然举办Playzone,近年来演出者更有Johnny's Jr.及J-STARS成员。在2007年时,他们已经公演达900次。基本上Playzone每一次是新故事,但唯独是2004年时,为了纪念杰尼斯事务所成立50年就重演了「梦断城西」。目前除了Playzone之外,成员都是各自以个人身份在演艺圈活跃中。","text2":"少年队的原名是什么?","label":1} {"text1":"挖客是基于WEB2.0的互联网内容挖掘、分享社区。该词汇来源于美国的digg网站,后被中国IT业内人士引进并翻译成挖客,国内有挖客网。挖客这类网民主要热衷于分享在网络上发现热门以及有价值的信息。一般他们提供发现信息的URL地址,然后通过挖客社区把这些信息分享以便更多的挖客可以浏览。这些挖客推崇的精神是:挖客社区的内容由挖客自己决定。挖客社区的规则。当挖客分享或创作一条新信息到挖客社区时,这条信息将被列在社区的“潜水信息”栏中,等待其他挖客来“挖它”(投票)。挖客社区会有一套规则并结合挖客的“挖”来决定那些是有价值或者热门的信息从而可以被挖出水到“出水信息”栏,即到社区的首页。从而让更多的挖客可以快捷的浏览到热门以及有价值的信息。目前挖客社区采用分值制度,其中Karma值较流行,这样能够较好的保证挖客在分享信息时的合理性和公正性。","text2":"目前的挖客社区采用了什么制度?","label":1} {"text1":"国民年金保险,简称国民年金、国保,法源依据《国民年金法》,于2007年7月20日经立法院三读通过,是中华民国于2008年10月1日开始实施的一项社会保险制度,主要的纳保对象为未参加军保、公教保、劳保、农保的25岁以上未满65岁中华民国国民。国民年金主管机关为卫生福利部(2013年7月23日卫生福利部成立以前为内政部),并委托劳动部劳工保险局为保险人。国民年金的法源依据《国民年金法》,于2007年7月20日经立法院三读通过,2007年8月8日总统令公布,2008年10月1日施行。国民年金目的在保障25岁以上、未满65岁,且未参加军人保险(军保)、公教人员保险(公保)、劳工保险(劳保)、农民健康保险(农保)之国民,于发生老年、身心障碍、生育及死亡事故时,本人及遗属之基本经济生活。国保之保险事故分为老年、生育、身心障碍及死亡4种,保险给付项目计有:老年年金给付、生育给付、身心障碍年金给付、丧葬给付及遗属年金给付等5种。具津贴性质的年金项目为:身心障碍基本保证年金、老年基本保证年金、原住民给付。自得请领之日起,应于5年请求权时效内提出申请。以下对于国民年金的各项给付的条件做说明。被保险人在国民年金保险加保期间,遭受伤害或疾病,达重度以上领有身心障碍手册(非重大伤病卡),经治疗终止症状固定或是再行治疗不能期待复原,需经身心障碍医疗机构评定无工作能力。如同时符合劳、公教保、军保、农保的身障给付,仅能择一请领。身心障碍基本保证年金是被保险人在参加国保前,已领有重度以上身心障碍手册(非重大伤病卡)或证明,经身心障碍鉴定医疗机构评定为无工作能力,且在国内居住,每年居住超过满183天。无下列任何情形之一者,得申请每月4,872元(105年1月起由4,700元调整至4,872元)的身心障碍基本保证年金。","text2":"国保之保险事故分为哪四种?","label":1} {"text1":"《第一季单车二势力》为2008年台湾年代MUCH台所制播的户外活动性节目,主持人为黑涩会美眉瑶瑶以及演艺圈新进MIA两位台湾艺人共同主持。该节目以主持人骑乘单车造访台湾各处知名景点,透过活泼风趣介绍当地有趣的人事物。《第二季单车二势力》全台湾走透透、吃透透、玩透透,此季单车二势力以校园周边美食为主轴,每集由主持人MIA及PARTY率领两所大专院校学生推荐的周边美食来PK,在介绍最好吃美食的同时,在节目中还加入了两校学生推荐PK的元素,让节目不只是提供吃喝玩乐讯息,还有竞赛性的趣味,增加内容张力。同时,用学生推荐美食也可以增加节目的互动性,拉近与一般观众的距离,节目也不将再是纯粹的单向介绍美食给观众。","text2":"《第一季单车二势力》是哪一年 的户外活动性节目?","label":1} {"text1":"伊拉克航空(),是伊拉克的国家航空公司,也是中东地区历史最悠久的一家航空公司,总部设在伊拉克首都巴格达。其枢纽机场为巴格达国际机场。伊拉克航空是阿拉伯航空运输组织的成员。伊拉克航空成立于1945年,是中东地区历史最悠久的一家航空公司。在50年代-60年代由于与苏联亲近所以购买了一批苏制客机,1970年代改为购买美制客机。1974年购买3架波音707,并在1976年接手波音727与波音747,并开通到北京及东京航线与大量欧洲航线。在1980年代由于美伊关系更加亲近,所以订购了更多的波音727与波音737和欧洲的空中客车A310。在两伊战争期间伊拉克航空中断了部分航线,并受到美国的制裁以至于美国未向伊拉克交付部分波音727。两伊战争结束后伊拉克国民经济受到损伤,由于美国与伊拉克关系战后一直紧张伊拉克航空也难以在购买外国客机并直接导致1990年海湾战争爆发,伊拉克航空受到联合国制裁,伊拉克航空转移了部分客机直至1992年复航。伊拉克战争之后,伊拉克航空宣布重启国际航线的计划,并于2004年10月3日重开巴格达至安曼的航线。此后,伊拉克航空向其他航空公司租借客机以恢复部份航线,并在2012年宣布订购30架波音737-800和10架波音787,同年接收一架波音777-200LR以及一架空中客车A330。2013年3月5日,伊拉克航空复航伦敦,同年4月25日使用波音777-200LR复航法兰克福。2014年8月23日,伊拉克航空使用波音777-200LR开通巴格达-广州-埃尔比勒-广州-巴格达航线。截2015年5月,伊拉克航空拥有以下机队:","text2":"伊拉克航空总部设在哪里?","label":1} {"text1":"法老的雪茄 (法语: Les Cigares du pharaon;英语:Cigars of the Pharaoh)是丁丁历险记的第四部作品。作者是比利时漫画家埃尔热。故事的主要舞台是在埃及,故事主线围绕著贩毒展开。在本作中,丁丁后面历险故事的两位主角拉斯泰波波罗斯(故事的头号反派角色)和汤姆森汤普森首次亮相。实际上丁丁历险记的第五部作品,也是该作的下一步作品蓝莲花可视为是本作的延续。丁丁第一次来到埃及,无意间发现毒品走私犯罪集团的贼窝,更在阴差阳错下到了阿拉伯和印度,与恶势力一次次周旋,将毒品走私帮一点点曝光。究竟法老、雪茄和贩毒有甚么关系?丁丁又能否成功打击罪犯?……","text2":"丁丁后面历险故事的两位主角是谁?","label":1} {"text1":"高桥宏(,),本名高桥裕幸,日本歌手、作词家、作曲家。1984年开始加入「POPSICLE」乐团开始活动。之后被财津和夫提拔,1987年从乐团TULIP的第三期加入开始职业音乐生涯。乐团不只专辑曲,连有些主打单曲也让高桥制作,受到器重。1989年,TULIP解散后暂时再结成「POPSICLE」,从独立制作发表专辑后,1993年开始个人活动生涯。成为「幽游白书」片尾曲的「不平衡之吻」(アンバランスなKissをして)(销售破50万),「当太阳再度闪耀时」(太阳がまた辉くとき)取得大成功,也在音乐节目露面。不过,由于所属唱片公司的解散,之后缩小歌手活动,活动重心转移为幕后作曲者。以后,主要作曲动画音乐,舞台音乐,电视广播音乐。2005年度的「ニッポン放送ショウアップナイター」节目音乐也是他的作品(持续到使用到2008年度)。生前很喜欢北海道的静内町(现在的新日高町),并受启发制作作品「SIZUNAI~我喜欢的风景~」(SIZUNAI~仆の好きな风景~)。过世后城市当地志愿者成立「CD制作实行委员会」将乐曲CD化,在本地CD店舖和旅游协会被销售。2005年11月4日,由于腹膜后肿瘤并发的多重器官衰竭逝世,得年41岁。在音乐节目MUSIC STATION提到在中学生时代听塔摩利在广播中批评软派系音乐,所以放弃了软派系音乐。在HEY!HEY!HEY!节目中提到「女心」和「北の街」是国小5年级时创作的曲子。2010年3月15日,逝世四年后,发行「高桥ひろ ベスト・コレクション」(高桥宏 BEST COLLECTION)CD,收录1993年至1995年间发行的经典作品。","text2":"高桥宏什么时间开始活动的?","label":1} {"text1":"乔治·温斯顿(,),是一位美国钢琴家兼作曲家。乔治·温斯顿出生在密歇根州,并在蒙大拿州和密西西比州长大,他毕业于佛罗里达州戴兰德市的斯特森大学,居住在加利福尼亚州的旧金山 。他的大多数作品因为能使听众感到大自然的风景,并且反映了四季的精髓,给听众闲适、轻松、怡然、清新的感觉。乔治·温斯顿被誉为“新世纪音乐之父”,而他本人却回避这个说法,乔治·温斯顿称自己的音乐为“乡村田园钢琴曲”。乔治·温斯顿于1972年录制了自己的第一张专辑为《民谣与布鲁斯》(Ballads and Blues),当时并没有得到多大的反响。而直到1979年,威廉·阿克曼(William Ackerman)与他谈起为他的新唱片公司录制专辑的事情。起初,乔治·温斯顿只是用夏威夷吉他弹奏了一些简单的音乐作品,之后有为电台的夜间节目做背景音乐,这样也就成为了乔治·温斯顿的专辑《秋日》(Autumn)。在音乐中,他将秋日描述成为万圣节的颂歌。这张专辑《秋日》后来被听众大受好评。之后他的专辑《十二月》(December)和《辞冬》(Winter into Spring)都获得了白金唱片的称号,并在1996年的的格莱美奖中获得最佳新世纪音乐专辑。由于他对史努比花生漫画的喜爱,他在1996年制作了音乐专辑《Linus & Lucy-The Music of Vince Guaraldi》,这张专辑是献给格拉第(Vince Guaraldi)为花生漫画所做的音乐。在2002年他将19世纪60年代门户合唱团(The Doors)的音乐制作成专辑《Night Divides the Day - The Music of the Doors》,与其不同的就是音乐使用的是钢琴的独奏,并且音乐更加的恬静怡然。在乔治的音乐会上,乔治本人并不是一个特别爱表现自己的人,而是衣着十分简单,法兰绒衬衫和牛仔裤。秃顶、一脸的胡子,根本不像是在音乐会为听众表演的演员,反而经常被观众认为是为钢琴调音的技术人员。除了作曲和演出,在音乐会上乔治也会给观众演奏口琴和夏威夷吉他。他演奏这些乐器主要在音乐会上,并非在专辑中。在专辑《Remembrance - A Memorial Benefit》可以听得到这两种乐器的使用。他为自己的唱片公司Dancing Cat Records制作过夏威夷吉他的录音作品。","text2":"乔治·温斯顿的大多数作品给听众什么感觉?","label":1} {"text1":"安卡拉战役(Battle of Ancyra)发生约在前239年,也可能在前240年,是塞琉古帝国内战-兄弟战争的一场决定性战役。当塞琉古国王塞琉古二世与托勒密王朝结束第三次叙利亚战争,一场内战紧接著在塞琉古帝国爆发,塞琉古二世的母亲劳迪丝要求他给予弟弟安条克·伊厄拉斯共同摄政权以及塞琉古领土中安那托利亚的管治权,安条克企图掌握整个帝国,便向塞琉古二世宣战,爆发兄弟战争。占据小亚细亚的安条克向附近的本都王国国王米特里达梯二世、卡帕多细亚王国国王阿里阿拉特三世和加拉太人同盟。对于安条克·伊厄拉斯的叛乱,塞琉古二世立即入侵安那托利亚,并在初期的军事行动中都有进展,并把矛头转向米特里达梯二世,而安条克率领一支包含许多加拉太人的大军来对抗,两军在Ancyra(安卡拉旧称)附近交战。战役结果塞琉古二世大败,损失了20,000名兵马且仅仅勉强逃脱。之后,战争仍维持几年,双方似乎签定和约,塞琉古二世失去了帝国安那托利亚部分和塔尔索西部。","text2":"安卡拉战役发生在哪一年?","label":1} {"text1":"洛多庇斯(古希腊语:ροδωπις;英语:Rhodopis)是一名在古埃及嫁给法老的希腊少女。她的经历见于古希腊历史学家斯特拉波于公元前1世纪写成的《地理志》(Geographica)及罗马作家克劳狄俄斯·埃利安 (Claudius Aelianus)的《史林杂俎》(Various Historia)。从书中的记载,洛多庇斯的经历与童话故事灰姑娘有部分相似之处,所以有人认为她是灰姑娘的原型。在《地理学》中,斯特拉波指出根据古希腊女诗人莎芙的记述,指出多莉查(Doricha)是于诺克拉提斯(Naucratis)港口经商的卡拉克索斯(Charaxus)的情人,其他人通常称她为「洛多庇斯」。据说有次她在洗澡的时候,一只鹰飞过,从洛多庇斯的侍从手上攫走了她的鞋子,后来那只鹰将鞋子带到孟斐斯,把鞋子掉到正在审讯案件的法老的大腿上。法老惊讶于鞋子的模样,亦觉得这件事别具深意,便派人去找这只鞋子的主人。最后,人们在诺克拉提斯找到洛多庇斯,把她带到法老面前,法老便娶她为妻。洛多庇斯是个美丽的交际花(Hetaera,详见古希腊卖淫业)。有一次她在洗澡的时候,一只鹰向下俯冲,攫去了她的一只鞋子,然后将鞋子带到孟斐斯,掉在法老普萨美提克(Psammetichus)的大腿上。法老惊叹鞋子精细的手工,便派人搜遍埃及,人们在找到洛多庇斯后,法老就娶她为妻。在古希腊历史学家希罗多德的《历史》中亦记载著一个同样名为「洛多庇斯」的希腊女子,但她完全是另一个人物,这个洛多庇斯是希腊的一名交际花,她曾与伊索一同沦为雅德蒙(Iandom)的奴隶。古时的希腊人还以为孟卡拉金字塔是她兴建的,希罗多德在书中驳斥这说法。","text2":"在《地理学》中,斯特拉波指出根据古希腊女诗人莎芙的记述,指出多莉查(Doricha)是于诺克拉提斯(Naucratis)港口经商的卡拉克索斯(Charaxus)的情人,其他人通常称她为什么?","label":1} {"text1":"冈山路竹延伸线,为高雄捷运规划路线之一,预定设置8个车站(不含南冈山站),采用标准轨距重运量系统,路线将会使用(叠式工法)并采用侧式叠式月台,经南冈山站与既有之红线直通行驶营运,全长13.22公里。依据最新市政府计划,本路线将采两阶段推行建造,第一阶段将优先兴建南冈山至冈山间,尽早服务广大冈山地区。第二阶段行经冈山、路竹、湖内等区域,并串联境内重要的产业廊带。本线原仅规划设置6站,于2011年1月7日重新评估后,认为应在路竹市区增设一站以扩大服务范围,因而增设路竹站,并将原本的东丰站改名南路竹站;最新计划则又增设大湖一站。自2001年起至2011年间,高雄市政府规划以BOT模式兴建营运,前后总共送件14次于交通部审核,但均以退件收场。经历多次退件后,市政府舍弃BOT模式,于2012年改回政府自办。
","text2":"市政府为什么舍弃BOT模式,于2012年改回政府自办?","label":1} {"text1":"拜耳张力学说(Baeyer张力学说,或张力学说)是由阿道夫·冯·拜尔于1885年用以解释不同环烷烃的稳定性而提出的一个理论。这个学说认为,所有环状化合物都具有环平面结构,由于键角(即多边形内角)与\"sp\"杂化轨道正常键角(109°28')有差别,因此所有环系都存在角张力。这个偏转角可以用(\"sp\"杂化轨道正常键角 - 多边形内角)÷ 2 来计算。各常见环烷烃的偏转角可以依此计算出来,见下表。根据这些数据,可以认定大环化合物与小环化合物一样,环系越偏出五元环,偏转角越大,张力越大。由于张力越大,分子能量越高,分子越不稳定,故小环的环丙烷环系容易开环。这便是拜耳张力学说对不同环烷烃稳定性的解释。事实上,大环化合物是稳定的。除三元环和芳香环具有平面结构外,其他环都不是真正的平面结构,因此自然也就不存在所谓“偏转角”,拜耳张力学说是错误的。但它所提出的当分子内键角偏离正常键角时会产生张力的现象,却是存在的。这种张力称为角张力。","text2":"拜耳张力学说虽是错误的,但是有怎样的意义值得后人学习呢?","label":1} {"text1":"国际乒乓球联合会(简称国际乒联,,ITTF)是一个国际性乒乓球管理组织。1926年1月,在德国柏林,德国、奥地利、匈牙利、瑞典等国的代表决定成立国际乒联。同年12月,国际乒联正式成立,并在伦敦举行第一届世界乒乓球锦标赛。国际乒联的总部设在瑞士洛桑,至2016年共有222个成员协会,现任主席是德国籍。惯例: MT\/WT: 男团\/女团; MS\/WS: 男单\/女单; MD\/WD: 男双\/女双; XD: 混双不像其他比较受欢迎的运动组织那样,国际桌球总会倾向承认一些在国际上不被普遍承认或领土有争议性的政府组织,例如:国际桌球总会目前就承认科索沃联邦桌球队,即便科索沃在大部分的国际热门运动都被排斥在外,他也在1970年承认当时不被国际承认的中华人民共和国,并且因而引发后续美国总统尼克森到中国访华的事件,史称乒乓外交。","text2":"国际桌球倾向于承认什么样子的组织?","label":1} {"text1":"凤凰卫视资讯台于2001年1月1日正式开播,是凤凰卫视全日24小时播放来自全球各地时事新闻与财经资讯的频道。重点在两岸以至全球华人地区的新闻资讯报道及评论,2003年在中国大陆落地。而广东有线仅有中文台,没有资讯台。凤凰卫视资讯台于2013年开播高清版,此时标清版为切边播出。2014年起开始标清16:9播出。另外,凤凰卫视资讯台会在每晚20点档《凤凰正点播报》开始前播放国歌,播放版本与香港电视台主流频道一致。凤凰卫视资讯台于亚洲3号S卫星通过C波段向整个亚洲、澳洲及非洲北部广播,以数码制式的DVB-S调制及MPEG-2编码,香港部分大厦使用卫星电视共用天线系统将讯号转为PAL制式模拟信号供住户收看。中国国际电视总公司境外卫星代理部接收凤凰卫视资讯台信号,通过亚太6号卫星(东经134度)发射KU波段信号。该服务一般提供给中国境内三星级或以上的涉外宾馆酒店、外国人居住区、领事馆及大使馆。部分中国城市更可以在有线电视上免费或额外付费观看。注:部分记者会兼以粤语于香港台作报导。","text2":"凤凰卫视资讯台与香港电视台主流频道保持一致,体现在哪?","label":1} {"text1":"松雪泰子,日本女演员,佐贺县鸟栖市出身。自小家境富裕,父亲松雪秀敏先纯是鸟栖瓦斯的社长。泰子就读于佐贺县立鸟栖商业高中,在学期间曾组乐团担任主唱,高中毕业即前往东京,加入演艺界,身高165公分的她在「MEN'S NONO」担任模特儿。1991年,参演电视连续剧《热血!新入社员宣言》,正式成为一名女演员。1993年以《白鸟丽子》一剧走红,随后又参演了《太阳不西沉》、《怪医黑杰克SP》和《真夜中的雨》,以及《砂之器》,原著小说《砂之器》中并没有松雪泰子饰演的「成濑麻美」这一个角色。松雪泰子2006年以《扶桑花女孩》的舞蹈老师平山圆香一角,获「2006年日刊体育电影大奖」最佳女主角奖。2008年,因演出《重金摇滚双面人》和《嫌疑犯X的献身》获日本电影金像奖优秀助演女优奖。2010年,以话题电视剧《Mother》获得第65回日剧学院赏最佳女主角奖。松雪泰子的弟弟为歌手。1998年,松雪泰子与吉他手Gaku结婚,2001年生下一子。2004年12月离婚,松雪泰子独自扶养儿子。","text2":"松雪泰子因为什么正式成为一名演员?","label":1} {"text1":"莙荙菜(\"Beta vulgaris\" subsp. \"cicla\"),即叶用甜菜,俗语又叫牛皮菜、厚皮菜、猪乸菜,苋科菾属的耐寒性一年生或二年生草本叶菜,是甜菜的一个变种。常见于地中海料理。莙荙菜原产欧洲南部,公元5世纪由阿拉伯传入中国。虽然菜叶常绿,但其叶柄颜色多变,有绿色、红色等多种颜色。莙荙菜的营养价值非常高,而且经过裁培后,菜的营养都聚集到叶片,而不是像甜菜那样聚在根茎。因此,莙荙菜被誉为是其中一种最健康的菜,常用于健康餐单里。莙荙菜虽然味道微苦,但在世界各地方的料理均有出现。在北美及欧洲亦有用来做西式馅饼的馅料;华南地区一般都是灼熟、用蒜头煮或煮汤,用以去掉苦味,口感也媲美烫菠菜。莙荙菜的幼苗可以用来拌色拉。","text2":"莙荙菜什么时候进入中国?","label":1} {"text1":"《麻雀变公主》()是盖瑞·马歇尔执导的一部喜剧电影,由茱莉·安德鲁斯、安海瑟威、曼蒂·摩尔主演,于2001年9月1日上映。该电影改编自同名英语小说。蜜亚(Mia Thermopolis)是个聪明、但极害羞又笨手笨脚的少女,她的人生目标是保持校园生活低调、避免撞板尴尬。可是,随著严厉祖母的突然到访,出乎意料地透露了她的真正身份是一名公主,是小国捷诺维亚的王位继承人,令她「被当透明人」的心愿终于幻灭。在十万个不愿意的情况之下,她参加了「公主培训班」,并要在三星期内作出毕生最重大的决定。期间,她经历了连串搞笑的「变身公主」课程,又惊觉自己成为传媒焦点,同时引来同窗妒忌,甚至牵涉一宗夺国阴谋……","text2":"《麻雀变公主》的主演是谁?","label":1} {"text1":"《群青日和/遭难》(,\"Ideal Days for Ultramarine \/ A Distress (First Run Limited Edition)\")是东京事变的第一张黑胶唱片,跟第一张专辑《教育》,一起于2004年11月25日发行。总计销售额2,281张。本张专辑为东京事变首张黑胶唱片,采完全限定发行,购买者必须要在10月23日以前预约至唱片行预约,11月25日再来拿取唱片。而这张以25公分的黑胶盘收录了单曲《群青日和》和《遭难》中的曲目,封面则是与首张专辑《教育》有异曲同工之妙,专辑《教育》是红色的纸鹤、红色的框,到了黑胶唱片整个就变成白色的复刻版,十分有趣。","text2":"《群青日和/遭难》是关于什么的照片?","label":1} {"text1":"黑背鼻鱼,又称颊吻鼻鱼、颊纹双板盾尾鱼,俗名天狗倒吊、剥皮仔、打铁婆,为辐鳍鱼纲鲈形目刺尾鱼亚目刺尾鱼科的其中一个种。本鱼分布于太平洋海域,包括日本、台湾、中国沿海、菲律宾、印尼、新几内亚、新喀里多尼亚、澳洲、新几内亚、马里亚纳群岛、马绍尔群岛、索罗门群岛、斐济群岛、万那杜、夏威夷群岛、法属玻里尼西亚、诺鲁、吉里巴斯、美属萨摩亚、吐瓦鲁等海域。水深0至90公尺。本鱼体呈卵圆形而侧扁,且不随年龄而改变;鱼体最高处在鳃盖末端,成鱼体色由灰绿到墨绿色皆有。口小,端位,上下颌各具一列齿,齿稍侧扁略圆,两侧或有锯状齿。尾柄两侧各有2个橘色斑点,分别落在尾柄棘上。成鱼斑点较大,而又鱼较小。另成鱼眼到口角处有一显著的橘黄色弧线且吻与此线同色。尾鳍末端略凹入,成鱼的上下叶延长成丝状。又鱼体成淡橘色。背鳍硬棘6枚、背鳍软条28至31枚、臀鳍硬棘2枚、臀鳍软条29至31枚。体长可达70公分。本鱼栖息于珊瑚礁、石块、碎石堆或向海坡面上。以马尾藻、棱藻等叶状藻为食。可做为食用鱼类,小鱼则适合于水族箱观赏。可剥皮切片,再浸于流状米糠内,然后冰冻一日,再食之。尾柄棘会伤人,须注意。","text2":"黑背鼻鱼以什么为食?","label":1} {"text1":"郑义(),原名郑光召。生于重庆,原籍四川省双流县,中国当代作家、海外异议人士。幼年在北京上学。曾参加红卫兵运动和大串连,1968年毕业于清华大学附属中学,去山西省太谷县插队,后流浪于中国东北地区、内蒙古自治区的森林草原。1974年到煤矿当木工,1977年考入晋中师专中文系。1979年以红卫兵运动中武斗为背景写出处女作《枫》,揭示了一代人的痛苦和挣扎。1981年毕业分配到晋中文联,任文学期刊编辑。以后,写有中篇小说《远村》和《老井》等,其中《远村》写的是太行旱区人打井寻源为主线表现了黄河人民的艰辛。1984年底到山西省作协筹办大型文学刊物《黄河》,时为山西作协专业作家。他的作品持有浑厚沉郁的风格和宏阔的历史感,在反映当前农村的变革生活和民族性格方面,有新的开掘和大胆的探索。六四事件后遭通缉,与妻子北明在大江南北四处逃亡,1992年至香港,翌年前往美国。2001年参与创办独立中文笔会,2002-2003年任副主席,2005年起为荣誉理事,2007-2009年任第三届会长。根据《美国新闻与世界报导》报导,郑义曾于《亚洲周刊》撰文,称林彪在文化大革命爆发后不久,从陶铸处获悉毛泽东对自己关于“政变”的讲话有猜疑,林陶二人在1966年末派黄埔四期同学萧正仪赴香港,秘晤居住香港的前国军华南补给区中将司令-{周}-游(周、林、陶、萧都是黄埔四期学生)并转交一秘函,林彪在函中倾诉自己「处危疑之局,遇多疑猜忌之主,朝荣夕枯,诡变莫测,因思校长(蒋介石)爱护学生无微不至,苟有自拔之机,或不责已往之错……」。1966年11月,该函由-{周}-游送交国防部特呈蒋经国。台湾方面对此做出秘密响应,表示欢迎,但与林彪之间联系随后中断。","text2":"郑义的文学作品有什么特点?","label":1} {"text1":"太的黄鲫(学名:\"\"),为辐鳍鱼纲鲱形目鲱亚目鳀科的其中一种。本鱼广泛分布印度西太平洋区,包括印度、孟加拉湾、斯里兰卡、缅甸、泰国、马来西亚、柬埔寨、越南、中国南海、台湾、印尼苏门答腊、婆罗洲、爪哇岛附近等海域。水深5至50公尺。本鱼体甚扁,背缘窄,腹缘有强锐的棱鳞。口大倾斜,口裂窄长。上颌稍长于下颌。上颌骨细长,不达鳃盖开口。胸鳍第一鳍条延长为丝状,向后达臀鳍起点。体背部青灰色,体侧和腹部银白带有黄色。吻部和头侧中部淡金黄色,鳃盖内面橘黄色。背鳍前方至头顶黑褐色。背鳍和尾鳍金黄色,背鳍末端、尾鳍上缘和后缘灰黑色。腹鳍白色,尖端黄色。体长可达20公分。本鱼近海洄游性鱼类,偶会进入岸边、港湾觅食。食用鱼,但体薄肉少,常晒成鱼干或是当下杂鱼处理。","text2":"太的黄鲫体长最长可达多少?","label":1} {"text1":"宏安集团有限公司,(Wang On Group Limited,),简称宏安集团,创立于1987年 ,是香港上市公司位元堂的控股公司,持有位元堂28.2%股权,亦为卢森堡大药厂(即珮夫人咳药水品牌的持有人)。另外宏安集团是万有街市的经营者,其外判承包:荃湾丽城花园、马鞍山颂安邨、东涌富东邨等多处街市经营权。宏安亦持有天水围俊宏轩商场、西营盘荟溱及油塘四山街13及15号工业地皮等。2012年5月31日宏安集团成功收购深水埗营盘街140-142号旧楼,地盘合共占地约4624方呎,以地积比率九倍计,料重建楼面约41616方呎,预计将发展与旗下红磡荟点相若类型的住宅。2013年1月29日土地注册处资料显示,宏安(01222)旗下旺角弥敦道724至726号银座式商厦8楼、11楼及12楼分别以6600万、5610万及5610万元易手。宏安于2010年以3.2亿元向科达地产主席汤君明购入汉英大厦,然后改建为一幢25层高银座式商厦,并拆售其中7至25楼,合共16层楼面,每层面积约2200方呎,预期以餐厅、零售等作主题,并于2014年落成。2014年6月25日宏安集团伙拍锦华实业,成功投得沙田市地段第599号地皮。8月13日,双方再次合作,成功投得沙田市地段第598号地皮。2015年1月7日,宏安集团成功投得沙田市地段第587号地皮。2017年6月14日旭辉控股以约6.645亿港元,向宏安地产收购油塘四山街13及15号50%权益,项目土地面积41,080平方呎,可建楼面面积约27.2万平方呎。2017年6月22日宏安地产以转让公司形式元向亚洲联合基建控股90%和俊和发展集团10%购入马鞍山白石耀沙路地皮,即沙田市地段第601号的所有股权,占地面积252954平方呎,地积比率约1.53倍,预计总楼面面积可达约387,500平方呎,双方都未有披露成交金额,市场估计应接近29亿元,楼面地价约7,500元。2017年9月12日宏安地产以24.4125亿元向碧桂园旗下全资附属公司恒宙国际出售马鞍山白石耀沙路地皮,即沙田市地段第601号的60%股权,交易完成后,宏安地产将仍持有地皮40%权益。","text2":"宏安集团是什么的简称?","label":1} {"text1":"广东住血线虫(学名:)是住血线虫属(\"Angiostrongylus\")的一个物种,可引起的病名称为广州管圆线虫病。本物种由Chen (1935)首度于中国广东省的鼠只发现;而在人体则是于1944年在当时仍是日治的台湾由Nomura & Lim (1944)在病人的脑脊髓液里发现。他们留意病人曾经吃过未经煮熟的食物,而这些食物很可能为鼠只所感染。1955年,Mackerass & Sanders (1955)确认了这种线虫在鼠只体内的生命周期,并以腹足纲物种作为中间宿主,透过血液、脑部及肺部传染鼠只。2013年,美国佛罗里达州首度发现这种线虫透过非洲大蜗牛(\"Lissachatina fulica\")来散播,使人关注这物种肆疟的范围在扩展。在大鼠肺动脉所产下的卵,于微血管中孵化成第一期幼虫,经气管食道在粪便排出体外。软体动物(例如:非洲大蜗牛)可摄食入第一期幼虫,在体内发育成第三期幼虫,人或老鼠(尤其是大鼠)吃下软体动物或受污染的饮水或生菜而得病。在囓齿类,幼虫穿过肠管进门脉循环后,移至宿主脑部经脱皮两次,回到宿主肺脏。在人体中,幼虫进入脑部不再发育。在动物的临床症状:轻度感染不显性,大鼠可见肢体无力,步伐障碍、呼吸促迫,发绀、流鼻血; 血样泪水。严重时,会昏睡、脑僵直、麻痺、死亡。在人临床症状常见有脑炎、脑膜炎、恶心、呕心、发烧、角膜炎 、虹膜炎、视网膜水肿、严重会昏睡及死亡。MRI定位仪雷射可观察患部;腰椎穿刺抽取脑脊髓液、检测嗜酸性球增加数量亦可作为诊断,用血清以及脑脊髓液进行ELISA抗原-抗体测试。除了扑灭鼠类外,应该要移除其第一中间宿主,像是螺蛳、非洲大蜗牛等,食物应该确实煮熟,及在-10℃下冷冻保存达24小时以上。不食用未煮熟之螺蛳、蛞蝓、淡水明虾、生菜等中间宿主;要注意非洲大蜗牛的接触史,以防其粪便或是黏液中带有虫卵,并且应全面清除;也需要注意储虫宿主的大鼠是否造成排泄物污染食物的可能。此防治措施施实背景以台湾为主,其他地方应参考当地可能的野生动物,而修正公卫策略。人类感染发生于整个大洋洲(特别是夏威夷、萨摩亚群岛、索罗门群岛、巴布亚新几内亚、印尼、菲律宾和澳大利亚北部);东南亚(特别是泰国、台湾、日本以及印度);一些感染曾报告于非洲和中南美洲,亦有报告存在美国纽奥良和路易斯安那。","text2":"广东住血线虫由Chen (1935)首度在什么动物上发现?","label":1} {"text1":"二氧化三碳是一个无色刺激性气体,化学式为CO,分子中含有四个累积双键。它与CO、CO、CO等其他碳氧化物有重要联系。1873年,Brodie通过对一氧化碳放电,首次制得了二氧化三碳,Marcellin Berthelot创造了低氧化碳(carbon suboxide)这个名称, 而后来Otto Diels表示,更贴近有机化学的名称,如二羰基甲烷(dicarbonyl methane)与二氧代丙二烯(dioxallene)也是正确的名称。二氧化三碳可以通过加热干燥的五氧化二磷(分子式为PO)和丙二酸或丙二酸酯衍生物的混合物来制备。 所以二氧化三碳也可视为丙二酸的酸酐,为丙二酸酐以外的另一种丙二酸的酸酐,即丙二酸的\"第二酸酐\"。此外,丙二酸分子间脱一分子水生成的酸酐也是存在的。其他有关二氧化三碳的合成和反应信息可在Reyerson于1930年出版的综述中找到。二氧化三碳自发聚合生成红、黄或黑色的固体,结构类似于2-吡喃酮。1969年时,曾一度认为火星表面的颜色是由存在二氧化三碳导致的,但后来海盗号的研究推翻了这个说法。二氧化三碳可以作为丙二酸的制剂,同时它还可以增强皮革染料与皮革的亲和力。","text2":"是谁首次制得了二氧化三碳?","label":1} {"text1":"是博彩的其中一种投注方式,英文为Flexi Betting,在香港「灵活玩」,由香港赛马会于2008-09年马季开始举办。基本上「灵活玩」不是一种新彩池,投注者只需自订一个投注总额,电脑系统便会按照顾客的投注指示,自动计算投注组合数目,以及将投注总额平均分配于每个投注组合上。如获中彩,彩金将按每注投注金额的比例计算。「灵活玩」适用于特别彩池(包括单T、孖T、三T、三重彩、三宝、四连环、四重彩及六环彩),以及所有过关及过关投注彩池(包括独赢、位置、连赢、位置Q及单T过关)。最初实施时,在投注各彩池时,只要投注总额超过$100,每注最低投注额低至$5, 三T及六环彩每注更可低至$2。而投注总额超过$300,每注最低投注额划一低至$2。2012-13年马季开始,「灵活玩」的每票\/每注最低投注金额进一步调低,让顾客能更灵活地运用投注本金。如每票\/每注的投注总额达$100或以上,三重彩、四连环、单T、孖T、三宝,以及过关\/混合过关注项的每注金额可低至$1;三T及六环彩的每注金额则维持$2。因应马会复办四重彩,由2014年1月19日开始,四重彩每票\/每注的投注总额达$24或以上,每注金额可低至$1。由于「灵活玩」每注金额可以是小数点后两位,而马会派彩是以十元为单位,故彩金计算出来便有机会为小数点后两位。马会派彩是会以彩金计算至最接近的对上或对下的伍角整数。「灵活玩」及位置赔率新计法 新马季首个赛马日推出__强显目录__","text2":"灵活玩的玩法是怎样的?","label":1} {"text1":"四线天竺鲷(学名:),为辐鳍鱼纲鲈形目鲈亚目天竺鲷科天竺鲷属的一种。本鱼分布于印度西太平洋区,包括马达加斯加、留尼旺、塞席尔群岛、斯里兰卡、台湾、泰国、越南、印度、日本、马来西亚、印度尼西亚、菲律宾以及中国南海及东海、澳洲、斐济等海域。该物种的模式产地在印度Pondicherry。水深0至80公尺。本鱼体延长而侧扁,眼大,口大略下位。鱼体黄褐色,腹部银白色,体侧具4条深色细纵带,其中一条从吻部穿过眼睛延伸至尾鳍底。背鳍硬棘8枚;背鳍软条9枚;臀鳍硬棘2枚;臀鳍软条8枚,体长可达13公分。本鱼栖息于珊瑚礁或岩礁区,白天躲藏于岩架下或岩洞中,夜间出来觅食。繁殖期时,雄鱼具有口孵习性,卵约7日化成仔鱼,由雄鱼吐出,具短暂的仔鱼飘浮期。不具经济价值。","text2":"四线天竺鲷一般分布在什么地方?","label":1} {"text1":"委内瑞拉蚁鸫(学名:'),又名塔奇拉鸫或委内瑞拉蚁八色鸫,是一种神秘的雀。牠们暂时被分类在蚁鸫科内,有待审定。牠们曾只于1955年至1956年间被见到及采集,被怀疑可能已经灭绝。直至2016年,科学家在埃尔塔马国家公园重新发现了委内瑞拉蚁鸫。委内瑞拉蚁鸫约长17厘米。上身呈褐色,冠及颈部呈灰色。展翼有黑色斑纹,喉咙及耳朵呈褐色,颊上有白色斑条。下身呈白色,两侧及胸部有灰色的斑纹。于1955年至1956年间,在委内瑞拉塔奇拉州西南部共采集了4个委内瑞拉蚁鸫标本。此后60年里它们都没有被再次发现,被怀疑可能已经因丧失栖息地而灭绝。2016年,一组考察队于埃尔塔马国家公园重新发现了它们,这也是第一次记录下委内瑞拉蚁鸫的声音和拍摄到它们的活体照片。","text2":"2016年,科学家在哪里发现了委内瑞拉蚁鸫?","label":1} {"text1":"张燮林(),中国上海市人,祖籍江苏省镇江市,乒乓球运动员,以灵活多变的直板长胶削球打法为其特色。张燮林早年自学成才,1958年获得上海市比赛亚军后被调入上海队,1960年进入中国国家乒乓球队。曾参加过多届世界乒乓球锦标赛,先后获得过男团、男双和混合双打冠军,并获得过男子单打第三名。1972年至1995年期间担任中国国家女子乒乓球队教练,邓亚萍等著名运动员均出自其门下,后升任国家体育总局乒乓球羽毛球运动管理中心副主任。张燮林是1987年乒乓球世界锦标赛“何智丽让球事件”的幕后主教练。他要求中国乒乓球女运动员何智丽在比赛中故意输掉二分之一决赛给另一位中国女运动员。何智丽没有听从他的指示,赢得了比赛并最后赢得了世锦赛冠军。随后他禁止了世界排名第一的何智丽参加任何国际性大赛包括奥运会。何智丽在巅峰时期无赛可比而被迫远走日本,加入日本国籍,代表日本出赛。张燮林作为违反体育公平竞赛的幕后操纵者没有受到中国体育局的任何处罚。","text2":"张燮林哪一年进入国家乒乓球队?","label":1} {"text1":"乔·马汀·祖马亚 (Joel Martin Zumaya,1984年11月9日生于加州丘勒维斯塔)曾是美国职棒大联盟底特律老虎队的一名中继投手,也曾经是大联盟球速最快纪录保持人(104.8mph)(此记录在2010年被古巴籍投手阿罗鲁迪斯·查普曼打破(105.1mph)。祖马亚2002年选秀会被老虎在第11轮第320顺位选进,本来以先发为主,后来转为牛棚投手,经过3年磨练于2006年登上大联盟,8月对双城的比赛投出104英哩(约167公里)的球速。于当年某一场比赛,祖马亚于第7局登板,面临2出局满垒,领先3分的情况,投出二次快达104英里的速球,不料被打者Ken Griffey J.R. 击成了逆转的满贯全垒打。同年老虎队晋级季后赛,到美联冠军赛期间却缺席三场比赛,后来总经理邓布罗斯基(Dave Dombrowski)表示,他是玩「」造成手肘受伤才缺席。2010年6月29日对双城,因手肘骨折,自此开始与伤病搏斗,从大联盟消失3年。2012年转至双城队,但伤势未见起色,3月底遭释出。2014年2月决定退休。大联盟生涯总计出赛171场,战绩13胜12败61中继5救援,防御率3.05。","text2":"大联盟球速最快纪录被谁打破?","label":1} {"text1":"刘定国(),本名刘玉云,中华民国(台湾)军人及政治人物,苗栗县人,乙未战争遗孤。少年于台湾受李钟萼抚养并学习汉文,15岁赴日本留学,加入由丘逢甲之子丘念台所创的同乡读书会,并随丘氏前往中国大陆,考入中央陆军官校(黄埔军校)就读。曾参加对日抗战,战后回台湾任职保安司令部,并担任新竹县防卫司令。1951年参选第一届苗栗县县长选举,因第一轮无人过半而由前两名进行第二轮选举。刘定国虽以63,627票击败黄运金的60,814票,但因仍具军人身分,随后被判决当选无效。1954年代表国民党参选第二届县长选举,获得当选。1957年当选连任。1960年当选为台湾省议会第二届议员,1972年底当选为第一次增额国民大会代表。刘定国是为苗栗县地方派系刘派创始人。","text2":"抗战之后,刘定国回台湾担任什么职务?","label":1} {"text1":"麻黄杏仁甘草石膏汤,出自《伤寒杂病论》。简称麻杏甘石汤或麻杏石甘汤。麻黄四两(去节) 杏仁五十个(去皮尖) 炙甘草二两 石膏半斤(碎、棉裹)上四味,以水七升,先煮麻黄减二升,去上沫,纳诸药,煮取二升,去滓,温服一升,日再服。太阳病汗出而喘,无大热者。麻黄、杏仁宣肺平喘,石膏清肺热,炙甘草补中。此方较麻黄汤少桂枝多石膏,显见其主攻寒邪由表入肺化热。本方主治证是由风热袭肺,或风寒郁而化热,壅遏于肺所致。肺中热盛,气逆伤津,所以有汗而身热不解,喘逆气急,甚则鼻翼煽动,口渴喜饮,脉滑而数。此时急当清泄肺热,自然热清气平而喘渴亦愈。所以方用麻黄为君,取其能宣肺而泄邪热,是“火郁发之”之义。但其性温,故配伍辛甘大寒之石膏为臣药,而且用量倍于麻黄,使宣肺而不助热,清肺而不留邪,肺气肃降有权,喘急可平,是相制为用。杏仁降肺气,用为佐药,助麻黄、石膏清肺平喘。炙甘草既能益气和中,又与石膏合而生津止渴,更能调和于寒温宣降之间,所以是佐使药。综观药虽四味,配伍严谨,用量亦经斟酌,尤其治肺热而用麻黄配石膏,是深得配伍变通灵活之妙,所以清泄肺热,疗效可靠。","text2":"麻黄杏仁甘草石膏汤的简称是什么?","label":1} {"text1":"八神浩树(,),日本男性漫画家。出身于新潟县柏崎市。毕业于。代表作有《》、《G-taste》及《灌篮少年》。1987年开始在讲谈社的旗下漫画杂志《月刊少年Magazine》上连载出道漫画《》。1989年《麻辣双娇》连载终了后,同样在《月刊少年Magazine》开始连载《灌篮少年》。同作品于2003年4月7日至9月29日在日本东京电视台与AT-X播放全26集的电视动画。1996年开始至2004年在《Mister Magazine》和(途中移籍)《》连载《G-taste》。1997年《灌篮少年》连载终了期间,短期连载《灌篮少年国中篇》。同年开始至2008年连载续篇《灌篮少年ACT2》。2007年,《灌篮少年ACT2》荣获第31回讲谈社少年部门漫画奖。2009年至2015年连载《灌篮少年ACT3》。正篇连载结束之后,2016年至2017年同样在《月刊少年Magazine》集中连载《灌篮少年特别篇》。全由讲谈社发行,连载于《月刊少年Magazine》(G-taste除外)。台湾中文版除了《G-taste》由尖端出版代理之外,所有作品都由东立出版社正式授权出版发行。","text2":"《灌篮少年ACT2》获得过什么奖项?","label":1} {"text1":"参加2008年北京夏季奥林匹克运动会的中国摔跤队共计26人,其中运动员16人,官员与工作人员10人,领队周进强。摔跤是中国自2000年悉尼奥运会开始的参赛的项目,2004年雅典奥运会首次获得1枚金牌。运动员(16人)男运动员(12人):覃和、王强、斯日古楞、王赢、梁磊、 焦华锋、盛江、李岩岩、常永祥、马三义、姜华琛、刘德利女运动员(4人):黎笑媚、许莉、许海燕、王娇官员与管理人员(10人)领队:周进强,副领队:董生辉教练员(6人):许奎元、曲忠东、于涛、盛泽田、李国、尼克奇医生:侯希贺,管理:袁海东(2008年8月12日)男子古典式摔跤55公斤级中国有焦华锋参加该项目,1\/8决赛被淘汰。1\/8决赛焦华锋Vs格鲁吉亚拉沙·戈吉塔泽,技术得分以0:2被淘汰。(2008年8月12日)男子古典式摔跤60公斤级中国有盛江参加该项目,资格赛失败后复活赛胜2场,铜牌决赛对哈萨克斯坦努巴克特·田吉兹巴耶夫,以3:8失败名列第4位。资格赛盛江Vs阿塞拜疆维塔利·拉希莫夫,盛江1:5失败后进入复活赛。复活赛第1轮盛江Vs罗马尼亚欧塞比乌·扬库·迪亚科努,以3:1胜出。复活赛第2轮盛江Vs保加利亚阿尔缅·纳扎里安,以3:1胜出。铜牌决赛盛江Vs哈萨克斯坦努巴克特·田吉兹巴耶夫,以3:8失败列第4位。(2008年8月13日)男子古典式摔跤55公斤级中国有李岩岩参加该项目,1\/4决赛淘汰。1\/4决赛李岩岩Vs哈萨克斯坦达尔汉·巴亚赫梅托夫 ,以6:11遭淘汰。(2008年8月13日)男子古典式摔跤55公斤级中国有常永祥参加该项目,1\/8决赛淘汰。1\/8决赛常永祥Vs保加利亚亚沃尔·亚娜基耶夫,以4:3胜出。半决赛常永祥Vs白俄罗斯奥列格·米哈伊洛维奇,常永祥胜出。冠亚军决赛常永祥Vs格鲁吉亚马努恰尔·克维克利亚,常永祥失败获得亚军。","text2":"参加2008年北京奥运会的中国摔跤队有多少运动员参赛?","label":1} {"text1":"原子球塔(Atomium)是为1958年比利时布鲁塞尔举办世界博览会而建的金属结构的纪念性建筑物,André Waterkeyn设计,高102米,包括9个直径18米的球体,与连接圆球的钢管构成相当于放大1650亿倍的α铁的正方体晶体结构。目前(2008年)三个球体向游客开放,顶部的球体提供观察布鲁塞尔全景的观景地。.12条管道连接著不锈钢球形成一个立方体,另外立方体中心球体连接8条管道通往立方体外围8个球体。每个连接的管道内都有电动扶梯与电梯,连接包含展览厅与公共场所等的六个可开放的球体。顶部球体可以观赏布鲁塞尔全景。美国有线电视新闻网把它命名为欧洲最怪异的建筑。原子球塔的整修作业开始于2004年3月;同年10月,为了整修而开始休馆,直到2006年2月18日才重新开放,整修包括将原子球体上已退色的铝板更换成不锈钢材料。为了能筹措装修费用,馆方将更换的铝板当纪念品贩售。约2公尺长,三角形形状的铝板以1000欧元售出。","text2":"原子球塔是为了什么活动而建的?","label":1} {"text1":"卫笑堂(),原名延桐,字梓生,山东栖霞县人,民国时期著名武术家,八步螳螂拳第三代传人,将八步螳螂拳传至台湾。卫笑堂在年青时即喜欢武术,其父卫稽云是地方上有名的武术家,他曾学习地趟拳,后随冯环义先生学习八步螳螂拳,尽得其妙。八步螳螂拳是民国初年时,由山东烟台三位武师-「通臂拳」师陈德善、「八卦拳」师王中庆、「螳螂拳」师姜化龙,三人所创,后由姜化龙传授给冯环义,而又传至卫笑堂。他在上海时曾至精武体育会,结识吴鉴泉先生,两人决定换艺,相互学习,因此学得吴氏太极拳。1949年(民国38年),因国共内战,只身赴韩国,因在韩友人与当地韩人发生冲突,为友人助拳,在混乱恶斗中击毙三个韩国小混混,重伤数人,隔天于当地报纸得知此消息后,卫笑堂得友人帮助复回山东。于1950年(民国39年)冬辗转到台湾,在台北开设饺子馆为生,闲暇之余在植物园教授拳法,弟子逾千人,遂使此拳在台湾及美加各地流传。但是八步螳螂拳在发源地山东反而因为文化大革命而消失了。民国73年(1984年),因心脏病发作过世。《实用螳螂拳》 卫笑堂著 首著序中,首次落款日期为民国56年秋天,民国57年九月由当时华联出版社刊印。民国69年6月再版。民国85年6月五洲出版社亦发行、民国93年亦再版。《实用螳螂拳续集》卫笑堂著 卫师父序里落款日期为民国60年10月10日,印行出版日期为民国61年5月,卫笑堂兼任发行人,福美彩色公司印刷。《实用螳螂拳秘笈》卫笑堂著 卫师父序里落款日期为民国65年仲冬,印刷出版日期为民国66年3月,卫校堂自力发行,福美彩色公司印刷。民国70年7月由当时任台湾大学国术会会长经由卫笑堂首肯,勘误前版部分内文后再版刊行,也由福美彩色公司印刷张老板印制。[1][1]卫笑堂老师著作资料参考《实用八步螳螂拳秘笈》p.337页 左显富‧杨清容校释,民国100年逸文出版社出版。知名弟子有、王秋雄、苏昱彰、林钧福、温送珍、陈国钦、吴而立、张家璜、郑荣贵、荀广龙、廖宏志、廖宏达、陈义华、左显富、朗咸平、颜锦福、张光宏、林松贤、彭韩萍、王杰、田贝康广等人。卫笑堂由中国大陆至台湾的这段历程,被台湾小说家张大春写进他的小说《城邦暴力团》中。","text2":"卫笑堂的父亲是谁?","label":1} {"text1":"《银河骑士》(The Adventures of the Galaxy Rangers)是美国与日本东京ムービー于1986年所合作,自1986年9月14日~12月11日之间,共播映65集(每周一~周五带状播出)的动画作品。台湾于1987年至1988年间于每周五下午5:30~6:00时段由台视引进播出。公元2086年两位外星人和平使节来到地球寻求人类的合作与协助,也为了报答人类的对于合作的好意,他们慷慨捐出一项非常珍贵的礼物——『超驱动能(hyperdrive)』,使人类首次克服时空之限制,能够更轻易跨入大宇宙中的银河,展开了外太空探索的崭新一页。在地球本身方面,为了地球自身的生存安全与大宇宙的命运前途,立即组成了『宇宙防卫总部』,担负起打击恶魔的神圣使命,以便共同对付一帮存心在大宇宙中破坏和平安宁的恶徒。每位队员都有配戴藏有能源传继系统的『第五系列太空警徽(Series Five Implant)』,只要当负担打击来敌时按动所佩带的警徽,立刻就可以使自己的「能」力因扩增而达于极点,拥有风驰雷掣难有匹敌者之力。","text2":"宇宙防卫总部的作用是什么?","label":1} {"text1":"植村直己(),日本男冒险家。1941年2月12日生于日本兵库县城崎郡国府村(现为日高町)上乡。毕业于明治大学。他是第一个站上世界最高峰圣母峰的日本人,也是世界第一个成功攀登五大陆最高峰者。植村挑战完高山后,把目标转向极地。为了横越南极这终极目标,他先后完成了徒步纵走日本列岛3000公里、格陵兰3000公里的单独雪橇之旅、北极圈12000公里的单独雪橇之旅、世界最初的北极点雪橇单独行,以及雪橇纵走格陵兰、攀登严冬期的阿空加瓜山等。1984年2月12日,植村直己成功挑战冬季独自登上北美最高峰、标高6168公尺的德纳利山((北阿萨巴斯卡语支、英语:Denali,\/dᵻˈnɑːli\/,直译「高山」),1917年至2015年官方名称为麦肯尼山(另译作麦金利峰,英语:Mount McKinley))后,隔天在下山途中失联,当地曾出动飞机寻找,于当月16日看到疑似植村的人在挥手,但由于天气恶劣且视线不佳而未能成功救援。植村的母校明治大学登山社曾两度组队上山搜索,在山顶附近找到植村所插的日本国旗,以及过夜的营地所留下的物品,但仍找不到植村本人。以当时恶劣的天候,几乎已确定植村的生还机率为零。同年6月19日获颁国民荣誉赏。","text2":"植村直己在挑战高山上有什么成绩?","label":1} {"text1":"皇家泰国空军博物馆()位于泰国首都曼谷廊曼县的拍凤裕庭路171号、廊曼机场国内航站楼6区(wing)南侧。该馆于1952年成立,目的为保存自暹罗发展航空工业已来的各种航空设备,以及泰国皇家空军使用过的军用飞机及物件。博物包详细介绍了泰国在二战中扮演的角色。当日本帝国军队于1941年12月8日在泰国南部登陆,在几个小时发生了几次小的战斗后,泰国就停止与日本作战,并于1942年1月向英国和美国宣战,同时与日本结盟,直到1945年8月日本投降。博物馆中有一些图画表现了泰国战斗机截击美国的B-29、P-38和P-51。非假日:上午8:30至下午4:30;假日关门。","text2":"皇家泰国空军博物馆位于什么地方?","label":1} {"text1":"白翅栖鸭(\"Cairina scutulata\"),又名白翼木鸭,是一种鸭。白翅栖鸭属大型栖鸭,羽毛黑色,脚短,脚爪强而尖,双翼有白色的覆羽。头颈都是白色,满布黑点。雄鸭及雌鸭的体型及颜色相似,但雌鸭较雄鸭细小,色泽亦较暗。牠们日间会在大树的叶丛中休息,晚间会于森林中幽暗隐蔽及布满杂草的小潭和水流缓慢的溪流觅食。牠们在沼泽附近的树孔内筑巢,每次会生6-13颗蛋,孵化期为33至35天。白翅栖鸭是杂食性的,主要吃种子、水生植物、昆虫、蠕虫、软体动物、青蛙及细小的鱼类。白翅栖鸭是与疣鼻栖鸭一同分类在栖鸭属中。不过,根据粒线体DNA的细胞色素b及NADH脱氢酶亚基2的分析,并生物地理学的分布地模式显示,白翅栖鸭与疣鼻栖鸭的相似性正在减少。故此,有指应将牠们分类在单型的属中,是与疣鼻栖鸭无关,但与潜水鸭是近亲。白翅栖鸭以往广泛分布在印度东北部(阿萨姆邦,阿鲁纳恰尔邦)及孟加拉,经东南亚I至爪哇及苏门答腊。于2002年,牠们的数量只有800只,其中200只在老挝、泰国、越南及柬埔寨,150只在苏门答腊,450只在印度、孟加拉及缅甸。白翅鸭生活在茂密的热带常绿森林,河流和沼泽附近。由于失去栖息地、数量稀少及被猎杀,白翅栖鸭被世界自然保护联盟列为濒危物种,且受到《濒危野生动植物种国际贸易公约》附录一的保护。","text2":"它们以什么为食?","label":1} {"text1":"「桥立」()是西日本旅客铁道(JR西日本)一列行驶于京都~天桥立之间的特别急行列车,沿途行经隶属于JR西日本的山阴本线与第三部门业者北近畿丹后铁道所属的宫福线、宫津线,是北近畿地区跨业者联营特急列车群北近畿大X网路()的组成份子之一。「北近畿大X网路」下属的诸列车全都是采跨社联运的方式运作,因此桥立号的车资实际上是由JR西日本与北近畿丹后铁道共同拆分,而与桥立号相对应、隶属于北近畿丹后铁道的特急列车是丹后发现号(),但不同的是丹后发现号在驶抵天桥立之后尚会继续行抵终点站丰冈才折返。桥立号的列车名源自位于终点站所在地的著名名胜、日本三景之一的天桥立。列车头牌上的代表色为红色,起源于耸立在宫福线沿线的大江山地区著名的「赤鬼」传说。这是桥立1・3・4・6・7・10号的编成这是桥立2・5・8・9号的编成桥立","text2":"桥立号列车是西日本旅客铁道(JR西日本)一列行驶于什么地方之间的特别急行列车?","label":1} {"text1":"四川博物院是一座中国四川省成都市城西青羊区,浣花溪公园旁,始建于1941年,是中国西南地区规模最大的综合性博物馆,藏品数量在中国大陆各类博物馆中位居第六,现有院藏文物26万余件,其中珍贵文物5万余件,国家一级文物1399件。2012年,四川博物院被国家文物局核定为第二批国家一级博物馆。四川博物院藏品分为陶瓷类、砖石类、金属类、钱币类、书画类、民族民俗类、碑帖类、近现代史类八大类三十余小类文物,具有浓郁的巴蜀地方特色。这些藏品非常多样,上至25000年前旧石器时代的“资阳人”头骨化石,下至近几年中国著名艺术家的作品,为研究四川各时期的历史、文化、经济、政治、军事提供资料。四川博物院展览面积达12900平方米,15个展厅,可一次性展出文物3300余件,可举办大中小各类文物和艺术品展,并拥有多媒体陈列展示。此外,博物院设有现代化的对外文化交流中心和多功能学术报告厅,可为博物馆学术研究与文化交流提供平台。","text2":"四川博物馆能为博物馆学术研究与文化交流提供哪些平台?","label":1} {"text1":",是1977年台湾六福影业有限公司出品,由李嘉执导、倪匡编剧之武侠片,改编自古龙同名武侠名著《飘香剑雨》。故事叙述江湖中有一邪教名为天争,教主武功极为高强,且手段残酷,因欲称霸武林,不时令教众屠杀异己,其现身时均着一面具,故无人知晓其真实身分,更令江湖中人心惶惶。侠客吕南人(田鹏饰)生于名门世家,家传有剑诀飘香一剑闻名江湖。一日探知因窥视于吕家家传剑诀之秘,天争教意突袭于好友凌北修(白鹰饰)一家,遂邀友人三心神君(马骥饰)赶往相助,岂知抵达凌家时凌北修已被掳,而吕南人归宅后更发现其妻薛若壁(唐宝云饰)竟为天争教主所欺。吕南人悲愤万分之下终寻至祖上藏置剑诀处,欲以祖传之飘香一剑对抗天争教,岂料传闻中飘香一剑实乃虚构。吕南人心灰意冷之余,幸得隐居多年的南偷北盗(葛小宝、陈慧楼饰)传以绝艺,自此努力学艺,决心消灭天争教。在凌北修为天争教所擒后,因不禁美色诱惑而加入天争教,更在天争教主命下与吕南人为敌,但始终敌不过武艺大进之吕南人。凌北修死前将一切经过告知吕南人,并表白自己加入天争教之缘由,而在此番告白中,吕南人境意外发现天争教主的真正身分……。","text2":"《飘香剑雨》电影是由哪部小说改编的?","label":1} {"text1":"京岘山位于江苏省镇江市市区东南郊,山高300米左右,受爆破采石以及市区拓宽影响,现已被铲平为新区的一部分。山上靠近陈家湾方向原有宋朝著名将领宗泽和夫人陈氏合葬冢,在市区拓宽时被移至今天的纪念碑处。宗泽安葬夫人时曾赋诗“一对龙湖青眼开,乾坤倚剑独徘徊,白云是处堪埋骨,京岘山头梦未回。”京岘山的来历据说是在周朝,有风水先生看到这里云雾缭绕,紫气升腾,就断定这一带要出王侯将相。到了公元前221年,秦始皇最统一中国。为保其一统江山万古千秋,秦始皇传旨,急驱3000多名赭衣囚徒,星夜赶往京岘山,名为修筑驰道,实是劈山削岭,断其 “龙脉”,败其“风水”,并煞费苦心地将谷阳更名为“丹徒”,顾名思义,即赤衣囚徒之地。对此,清朝乾隆帝在下江南时曾经写《京岘山》诗:“凿京岘命赭衣徒,名象虽存迹久芜,未及临洮东属海,长城筑罢竟亡胡。” 对秦始皇不会引导使用人才进行嘲讽。","text2":"关于京岘山的来历有什么传说吗?","label":1} {"text1":"詹姆士·维南·泰勒(James Vernon Taylor,),美国音乐人,吉他演奏家。泰勒出生于马萨诸塞州波士顿,在北卡罗莱纳州的卡波罗市(Carrboro)长大。泰勒的事业生涯起于1960年代中期,但直到70年代才逐渐受到欢迎。他多演唱感性柔和的歌曲。同时期蔚为风潮的自弹自唱者包含琼尼·米歇尔、Tom Rush、Cat Stevens、Carole King、John Denver、Jim Croce、Don McLean、Gordon Lightfoot及 Jackson Browne,同时还有 Carly Simon,后来成为泰勒的妻子。他在1976年出了张钻石级唱片红极一时,总共售出超过一千一百万张,他在90年代及00年早期也出过几张销售成绩斐然及获奖无数的歌曲,仍保有相当大量的听众。","text2":"詹姆士·维南·泰勒的妻子是谁?","label":1} {"text1":"国务院参事室是中华人民共和国国务院的正部级直属机构,于1954年9月设立(前身为1949年11月成立的中央人民政府政务院参事室),具统战性及咨询性,是国务院直属的主管政府参事工作的机构,政府决策的智力支持机构。1949年4月7日,上海解放前夕,毛泽东主席电示华东局领导人,提出上海解放后应设参事室,为建设新上海出谋划策。1949年11月11日国务院参事室成立,各省区市政府参事室也于1950年左右相继成立,政务院制定了《政务院参事室暂行工作简则》。1951年7月,经毛泽东倡议,设立了中央文史研究馆。目前两者合署办公,在中共国务院参事室党组领导下开展工作。2008年11月起,经国务院领导批准,国务院参事室设立了特约研究员制度。2010年1月,《政府参事工作条例》(简称《条例》)正式实施,促进参事工作进一步规范化。根据《条例》,参事室的主要职责是参政议政、建言献策、咨询国是、民主监督、统战联谊。政府参事是直接为政府领导同志服务的,由行政首长直接聘任,不代表任何党派、部门的利益。与各种智囊机构不同,参事室作为国务院直属机构,拥有建言献策的「绿色通道」,即参事建议直达直批、无障碍呈送。参事和馆员由国务院总理聘任。参事、馆员、特约研究员多数是民主党派成员和无党派人士,也有中共专家、学者和富有经验的领导干部。除国务院设立参事室外,省级人民政府和许多省辖市人民政府也都设有政府参事室。凡设立参事室的地方人民政府,大都设有文史研究馆。地方文史研究馆的馆长和馆员亦由地方政府首长聘任。近年来,国务院参事室积极开展同世界一些国家智库的业务交流与合作。截至目前,已同22个国家和地区的33家智库建立了不同形式的合作关系。  国务院参事室、中央文史研究馆下设办公室、参事业务一司、参事业务二司(合作交流司)、文史业务司和机关服务中心。此外,还下辖中央文史研究馆书画院、中华书画家杂志社、中国国学中心、中华诗词研究院、华鼎国学基金会等单位。政务院参事室时期:国务院参事室时期:本资料更新于2017年7月28日:","text2":"参事室对谁负责?","label":1} {"text1":"曾健明(Leo Tsang,)歌影视三栖艺人,现为香港无线电视部头合约艺员。曾独得市政局年度全港舞台剧《最优异演员奖》,出道于1977年丽的电视第三期艺员训练班。1978年《丽的艺训班》毕业,成为全职艺员。1980年代开始在TVB拍电视剧,最擅长演骗子的角色,可以说骗人骗到出神入化,由于拍骗人的戏实在太深入民心,导致现实生活中,很多人都分不清他说的是真话还是假话,骗人骗到真假难辨,绝对称得上是TVB的骗子王,于综艺节目《Sunday好戏王》获颁「另类好戏王」,演技大受肯定。2016年更得到《骗子王》奖项,被誉为《最有价值演员首位》。
及后随知名歌唱家,如秦燕老师,韦秀娴老师,叶惠康教授,程路禹教授等等,钻研乐理及歌唱技巧。近年更拜梁月玲老师门下习艺,唱功更是突飞猛进、一日千里。1990年代曾为香港电台电视制作部拍摄教育电视,当时以扮演医生、成展权的父亲居多,又经常参演《警讯》及《绳之于法》,做尽一切坏人及骗徒。1994年『永盛影业』剧本赛,得《最佳故事银奖》。曾知天份得天独厚,乃半由人力半由天,遂以义工身份,到各大院舍,献唱怀旧金曲,以娱一众长者老友记。拿手曲目包括:《风流梦》、《卖花曲》、《热带情歌》、《盼知音》、《百花亭之恋》、《相思泪》、《秋风吹谢了春红》、《相逢恨晚》、《怀旧》、《彩蝶双飞》、《一心想玉人》等等,不计其数。","text2":"曾健明拿手的曲目有哪些?","label":1} {"text1":"日本的首都圈(),指的是以首都东京为中心的都会区,也称东京圈()或东京都市圈()。一般包括东京都、神奈川县、千叶县、埼玉县,因此又称为一都三县,与南关东的范围相同;日本政府为统合东京及周边区域发展而制定的《》,则将首都圈的定义范围涵盖整个关东地方(茨城县、栃木县、群马县、埼玉县、千叶县、东京都、神奈川县以及山梨县)。根据日本总务省2017年的统计,总人口为36,312,851人(一都三县内),2008年,日本首都圈的GDP(国际汇率)总量达到19,000亿美元,规模同样位居世界主要大都会区中的第一位,如果把首都圈视为一个国家,其经济规模在当时也仅次于美国、日本、中国、德国、法国、英国、意大利7个国家。","text2":"日本总务省2017年统计的总人口为多少?","label":1} {"text1":"斑鱵(学名:)为辐鳍鱼纲鹤鱵目鱵科的一种。本鱼分布于印度西太平洋区,包括东非、留尼旺、塞席尔群岛、阿曼、印度、泰国、越南、马来西亚、模里西斯、中国、日本、台湾、印尼、澳洲、密克罗尼西亚、新喀里多尼亚、帛琉、万那杜、菲律宾、东加、萨摩亚群岛等海域。其主要栖息于水的中上层。该物种的模式产地在红海。水深0-30公尺。本鱼体略呈长枪状,稍侧扁。吻细长且上颌短于下颔,眼大,体背部青黑色,腹部银白,鳞片大但易脱落。侧线上具5-6个明显黑斑,下颔尖端为红色。尾鳍深叉,体长可达45公分。本鱼栖息于亚热带海域的水表层,常栖息在水流平静的内湾,受惊时会越出水面。属杂食性,以藻类及浮游生物为主。繁殖期至河口产卵。属高级食用鱼,适合油煎食用。2.FishBase3.台湾鱼类资料库","text2":"斑鱵的模式产地是哪里?","label":1} {"text1":"海军陆战队两栖侦搜大队,又称中华民国蛙人部队或中华民国陆战蛙人,为中华民国海军陆战队指挥部直属大队之一。在成为正式侦搜队员之前,每位学员都必须在集训队中先受8到12周不等的「两栖专长训」,训练课程大致分为跑步、游泳、操舟等三大部分。在这8到12周的集训生活中,是每位侦搜队员最痛苦却也是最美好的回忆。在集训阶段完成前,每一期的集训队都必须通过「综合考验周」(又俗称为「克难周」)的洗礼。综合考验周共分为22堂课,这22堂课也就是先前在集训队受训期间所学到的所有课程。每位学员必须在这短短的六天五夜中,将受训的成果展现出来。综合考验周的最后一关,俗称为「天堂路」,因为-{只}-要通过该项考验就能结束犹如地狱般痛苦的训练,宛若飞上天堂一样,而得名。天堂路的考验在军教片及新闻媒体的宣传下,成为台湾民众对海军陆战队训练的一项基本印象。此部队与陆军海龙蛙兵训练内容大至相同。若要比较差异,陆战队两栖较注重毅力以及忍耐度的训练,例如爬天堂路、或是跳粪坑用粪便刷牙;陆军海龙蛙兵较注重近身搏斗及作战技能部份,例如自由搏击、格斗刀术等。天堂路是一条以棱角尖锐的咾咕石铺成、长约50公尺的路面,全身仅著游泳短裤的海军陆战队员们必须在这样的路况上以匍匐前进、翻滚等方式通过,还要做出各种指定的战技动作,而且-{只}-要教官认定动作不合格便必须重来,沿途还会被泼洒盐水。所以凡通过天堂路的队员没有不遍体鳞伤的,但许多合格的队员们都将这些伤疤视为军人荣誉的象征与珍贵的人生经历。侦搜大队目前编制共有七个中队,分别如下(括弧内是精实案前分别所属国军特种单位,现已不存)两栖侦搜中队,拥有陆海及空中侦查渗透能力,蛙人的座右铭是「高山向我低头,海水为我让路」一旦战争爆发,蛙人是最早投射到敌人阵营的部队。特勤中队,俗称黑衣部队,格言「忠孝 勇敢 敏捷 机智」,行动隐密,快速打击敌方,负责台湾南部反劫持,反恐及反劫船任务,一旦接到上级任务,特勤队能够在15分钟内完成战备整备,每位成员必须能够独立作战,三分之二队员具有七年以上的资历。爆破中队,早年接受美国海军指导,负责爆破清除水中障碍,渗透特战任务。","text2":"海军陆战队两栖侦搜大队又称为什么?","label":1} {"text1":"吴少诚(),唐朝申蔡节度使。少年从军,因节度使庾准的信任,成为牙门将。后来成为李希烈手下大将,任淮西兵马使。贞元二年(786年)七月,吴少诚杀陈仙奇,自为留后,唐德宗拜为申蔡节度使。贞元十五年(799年)三月,吴少诚叛乱,德宗下诏削其官爵,出兵讨伐,但一战溃败。贞元十六年(800年)二月再讨之,五月诸军又大败。朝廷见无法讨平吴少诚,下诏赦免,承认其地位,加封检校仆射。唐顺宗年间又加同中书门下平章事,唐宪宗年间又迁检校司空,封濮阳郡王。吴少诚宠任大将吴少阳,收为契弟,署为军职,累迁申州刺史。后少诚病危时,少诚家奴「鲜于熊儿」勾结吴少阳,软禁少诚,杀少诚之子吴元庆,少诚忧愤而卒,吴少阳自立为留后,朝廷讨伐无功,遂赐诏书真除。吴少诚有小女婿董重质,后投降朝廷,任夏绥节度使。","text2":"董重质最后被任命为什么官职?","label":1} {"text1":"本条目为欧洲足协杯赛冠军球会领队列表,纪录各主教练带领曾执教的欧洲足协所属成员国的足球俱乐部,在欧洲足协官方认可的球会级杯赛赢得冠军的数量。这些杯赛包括欧冠杯(1955年至1991年,欧洲联赛冠军杯前身)、欧联(欧洲联赛冠军杯,1991年至现今)、欧洲杯赛冠军杯 、欧洲足协杯(1971年-2009年)、欧霸杯(2009年至现今)、图图杯和欧洲超级杯。截止2014年意大利籍领队查柏东尼和苏格兰籍领队费格逊是最成功的领队,分别带领球队赢得6项赛事冠军。截至2014年,意大利领队赢得最多杯赛冠军,合共38项,英格兰领队则紧随其后,有25项。当欧洲博览会杯被欧洲足协杯取代前,欧洲足协并没有将其视为官方赛事,所以此杯的冠军将不被在此列表中记录。而1972年的欧洲超级杯,同被视为「非官方赛事」。说明此表是纪录各国领队带领球队夺冠的总数:","text2":"为什么欧洲博览会杯的冠军杯不在欧洲足协赛事冠军球会领队列表中被记录?","label":1} {"text1":"欧迈尼斯一世,(约)是小亚细亚的帕加马统治者(前263年—前241年在位)。父亲是欧迈尼斯,而奠定帕加马阿塔罗斯王朝的菲莱泰罗斯是他父亲的兄弟,无子的菲莱泰罗斯把欧迈尼斯一世过继在自己名下,并认定他为继承者。尽管帕加马在菲莱泰罗斯时名义上属于塞琉古帝国统治,却享有高度自治。在欧迈尼斯一世统治时,可能受到托勒密王朝托勒密二世的搧动,当时托勒密王朝和塞琉古帝国发生叙利亚战争,帕加马趁机对塞琉古的反叛。前261年,欧迈尼斯一世更在吕底亚的首府萨第斯附近击败塞琉古国王安条克一世,使帕加马从帝国独立出来,更扩大不少领地。欧迈尼斯一世在伊达山北方的山脚下建造一个要塞,命名为菲莱泰罗利亚来纪念他的养父,在推雅推喇西北方莱克斯河源头附近建造另一个要塞,命名为安塔利亚来纪念他的祖父,往南扩展到卡伊科斯河Cyme那。欧迈尼斯还自行发行钱币,上有菲莱泰罗斯的肖像,这成为帕加马独立的证据之一。在帕加马发生对塞琉西的反叛之后,并没有历史记载帕加马之后在欧迈尼斯的统治期有无战争发生,尽管托勒密王朝和塞琉西帝国持续在战争,而加拉太塞尔特人在小亚细亚四处劫掠,如果帕加马没有受到塞尔特人侵扰,那欧迈尼斯可能付出不少贡品。虽然欧迈尼斯从未自立为国王,但他许多行为仿效其他希腊化国王的作为。不确定欧迈尼斯是否有儿子,一些资讯透露他可能有一个儿子菲莱泰罗斯,但死于欧迈尼斯一世逝世前。最后欧迈尼斯收养了年轻的堂姪阿塔罗斯一世,认定他为继承者。","text2":"欧迈尼斯为什么没有付出很多贡品?","label":1} {"text1":"赵贞吉,字孟静,号大洲,四川成都府内江县桐梓坝人,明朝政治人物。官至文渊阁大学士。南宋右丞相赵雄十世孙。幼时聪慧,六岁能读经,嘉靖十四年(1535年)成进士,初任庶吉士,授翰林院编修;王敏肃对他十分称赞。后任右春坊右中允,管司业事,嘉靖二十九年(1550年)升任左春坊左谕德,兼河南道监察御史。之后因得罪严嵩,而被贬为贵州荔波县典史,量移徽州通判,后历任吏部文选司主事、郎中,光禄寺少卿、通政使司参议、右通政,光禄寺卿、户部右侍郎,皆在南京。嘉靖四十年(1561年)始入为户部右侍郎,又因得罪严嵩遭到罢职,隆庆改元,起用为吏部侍郎兼翰林院学士,掌詹事府事。上幸学,暂掌祭酒事,出为南京礼部尚书;召入兼翰林院学士,协管詹事府事,寻拜文渊阁大学士。隆庆年间,以礼部尚书兼文渊阁大学士入阁,因与高拱不睦,遂辞官返家,在桂湖街讲学,与杨慎、任瀚、熊南沙合称“蜀中四大家”,著有《赵文肃公诗文集》。万历四年(1576年)三月十五日过世,享年69岁,赠少保,谥文肃。","text2":"嘉靖二十九年(1550年)赵贞吉担任什么职位?","label":1} {"text1":"董景昌(),原籍山东,台湾著名针灸师,是董氏奇穴的创始者。董景昌早年从军,随著国民政府撤退至台湾。在台湾传授他家传的针法,因为由他传授的穴位并不在传统内经所记载的范围内,但疗效显著,因此被称为董氏奇穴。董景昌自成一套独特的经络思想,与传统的十四经络循行路线不同,针灸的穴位配伍极为简明,在放血治疗方面特别擅长,他门下弟子甚多,在台湾中医界极负盛名。但是他并不敌视传统技术,经常鼓励弟子去学习传统十四经针灸手法,常推许台湾传统古方派名医孙培荣,临床时也常以十四经穴道搭配董氏奇穴而发挥奇效。他的弟子有杨维杰、巴顿 (Dr. Dachen Palden)、赖金雄、满萱春、胡文智等人。著作有《董氏针灸正经奇穴学》。","text2":"董景昌的著作名称是什么?","label":1} {"text1":"刘氏鳄属(学名:\"Lewisuchus\")是主龙类爬行动物的一属,身长约1公尺,生存于三叠纪中期(拉丁尼阶)的阿根廷,约2亿900万年前。刘氏鳄属于恐龙形类的西里龙科,这是一群恐龙的原始近亲。属名是以美国化石挖掘家Arnold Lewis为名。某些原本的刘氏鳄化石,现已重新归类到其他属,而剩余部份可能属于伪兔鳄。科学期刊《自然》在2010年的一份文章,指出刘氏鳄、伪兔鳄可能是相同物种,两者的体型接近、化石被发现相同地层,但是两者的化石重复部位少,无法做出详细鉴定、结论。由于刘氏鳄的命名时间,比伪兔鳄早了15年,如果两者是相同动物,刘氏鳄将具有学术上的优先权,而伪兔鳄是刘氏鳄的次异名。","text2":"刘氏鳄属是恐龙形类的哪一科?","label":1} {"text1":"拉多斯瓦夫·西科尔斯基,Radosław Tomasz Sikorski,()(1963年2月23日出生于比得哥什)是一个保守的波兰政治家和记者,现任波兰共和国外交部长。西科尔斯基积极参与了二十世纪70年代末团结工会的动乱,并在1981年3月成为比得哥什的学生罢课委员会主席。1981年12月当波兰实施戒严时,他滞留在英国。在牛津大学彭布罗克学院学习了哲学、政治学、经济学。随后他成为一名自由记者。1984年他获得了英国国籍。二十世纪80年代中期西科尔斯基成为一名在阿富汗和安哥拉工作的战地记者。1988年他在阿富汗拍摄的一张照片获得了世界新闻摄影比赛奖。1990年他成为默多克在波兰的投资顾问。回到波兰之后,1992年他很快成为了政府的副国防部副部长。1998年至2001年他在政府中任外交部副部长一职。2007年出任外交部长。","text2":"1998年拉多斯瓦夫·西科尔斯基获得了世界新闻摄影比赛奖的照片是在哪拍摄的?","label":1} {"text1":"P. 1000 巨鼠重型巡航坦克(\"Landkreuzer P. 1000 Ratte\")是纳粹德国计划开发的一种重量达1000吨的超重型坦克,由德国的克虏伯公司研制,但计划在1943年被放弃,没有一辆P-1000被生产出来。此超重型坦克非常巨大,高度有11米,炮台也有两个平常人的高度。1942年初,德国武器局第5处计划在“大西洋防线自行280毫米海岸炮”计划的基础上开发P-1000超重型坦克。当时纳粹德国的军事发展以及力量已经达到了巅峰程度,阿道夫·希特勒本人在开始的时候对这种研发超重型坦克的项目也很感到兴趣,他觉得坦克愈大就愈强劲,并完全没有考虑到很多实际问题,就批准了由德国的克虏伯公司承担研发超重型坦克的工作。1942年6月23日,在克虏伯公司由Grote博士作主要指导,及Hacke博士协助,研发超重型坦克的研究开发工作正式开始。研发这一辆超重型坦克的研发代号为P-1000,称为「巨鼠」坦克(Ratte),如果研发完成后,设计重量可高达1000吨。同年该公司完成了P-1000「巨鼠」超重型坦克的设计书,并向希特勒汇报。他也很欣赏这个研发计划,并批准研发。在研发超重型坦克过程上,通常要考虑很多要点,例如在不同的地面或地形上,这种庞然大物是否会陷入沙地或泥地,地势、经费、运输、重量大小、零件可靠性、机动性、驾驶技术、攻击力以及研发的技术等等也没有考虑清楚。超重型坦克因为行动不方便,例如缺乏可以承托它们的桥梁,而它也会把公路压坏。而且P-1000的体积使它极其容易被轰炸机摧毁。最后,它的成本相当于80辆黑豹坦克,建造时间更是天文数字。P-1000超重型坦克的研发在1943年被军备和战时生产部长阿尔伯特·斯佩尔终止,没有一辆P-1000被生产出来。车体长度为35米,宽度是14米,高度是11米,为了承受本身的巨大重量,坦克每侧的履带有3.6米宽,由3条1.2米宽的分履带组合而成。P-1000超重型坦克动力系统是使用8台Daimler-Benz MB501型20缸的柴油发动机,一台柴油发动机马力为2000匹马力,输出动力可达到16000匹马力,就可以推动到这架超重型坦克,而行走最高速度是40公里/小时。P-1000超级坦克3D游戏观看影片(战地风云1942 模组)","text2":"P-1000超重型坦克的研发被谁给终止?","label":1} {"text1":"约翰尼·卡尔·埃马纽埃尔·申克( Johann Karl Emmanuel Schenk ,)是一位瑞士牧师、政治家,瑞士联邦委员会委员(1863年-1884年)。截至2008年,他仍然是瑞士联邦历史上任期最长的联邦委员会委员。申克于1823年12月1日出生于瑞士伯尔尼一个基督教家庭。11岁时申克成了孤儿,在德国巴登-符腾堡州路德维希堡县生活。1839年到1842年期间,他在伯尔尼学习,后修神学。1845年,在他22岁时成为一名牧师,开始在瑞士伯尔尼州许普芬(Schüpfen)任教区牧师。1848年,他与埃利斯·卡尔(Elise Kähr)结婚,他们共育有九个孩子,其中两个年幼夭折。他于1863年12月12日当选为联邦委员会委员,任职期长达31年,他于1895年7月18日死于任内。卡尔·申克是瑞士自由民主党成员。在任期内,他主要主持领导了以下部门的工作:期间,卡尔·申克先后六次出任瑞士联邦总统,分别是在1865年、1871年、1874年、1878年、1885年和1893年。卡尔·申克1895年在伯尔尼去世,他被安葬在伯尔尼的许希利胡斯(Chüechlihus)。","text2":"他是哪个党的党员?","label":1} {"text1":"郭诩(),字仁弘,号清狂道人、清狂逸叟,明代山水人物画家,江西泰和南寮(今澄江镇郭义井)人。擅长画山水、人物,兼有粗笔和细笔两种面貌。郭诩年少时应科举不第后,转而遍游名山大泽,专志于诗文书画创作中。他在游历当中领悟到“岂必谱也,画在是矣”的现实主义创作思想,“肆意手摹,即成山水”。其画作多有题跋,诗画结合,隽永有致,以致“天下竞传其画,购之百金”。弘治年间,郭诩应诏入京城皇家画院,与江夏吴伟(弘治初授画状元)、姑苏沈周、北海杜堇齐名,延颈原交。为人淡泊名利,宁王朱宸濠、中常侍萧敬等人试图拉拢,都被他其婉言辞谢。士林领袖李梦阳称赞他“居业洁良,不愧逸民”,一代心学大师王阳明也与他交好。郭诩之画题材广泛,尤擅长山水人物、花鸟牛马,兼有粗笔和工细两种面貌。其水墨写意人物画,风格豪放,采用白描手法,运笔如行云流水,不着颜色而光彩照人。又善杂画小品,用没骨法画花鸟草虫和人物,信手点染,笔墨简括却变化无穷,野趣盎然。七十三岁时,还创作了一幅《磨镜图》。郭诩传世作品不多,现存的有:","text2":"郭诩擅长什么?","label":1} {"text1":"叶绍德(Yip Shiu-Tak,),人称德叔,籍贯广东东莞,资深粤剧作家及填词人。叶绍德1930年出生于香港,自幼喜爱粤剧; 1949年起参与粤剧音乐社;1951年起编曲;翌年追随王粤生并代写电影插曲,及后因王粤生介绍而认识唐涤生。1956年,薛觉先去世后,在颜耀林的指引下整理薛觉先名剧《花染状元红》以及为何非凡创作《红楼金井梦》而成名。1960年起和白雪仙合作并整理《帝女花》唱片版本;翌年于仙凤鸣剧团出任《白蛇新传》编导组成员,并负责为该剧编成初稿。1962年、1966年分别整订《再世红梅记》及《紫钗记》。叶绍德于1971年正式转为粤剧编剧,在雏凤鸣剧团、颂新声剧团、庆凤鸣剧团等多个粤剧剧团作编剧。七十年代起他亦开始参与电视剧歌曲的填词工作,如电视剧《啼笑因缘》和《芸娘》的同名主题曲,以及前者的插曲《想郎》。1980年代于香港电台担任《戏曲天地》的主持。2002年起,叶绍德除了是香港艺术发展局的艺术顾问外,也是戏曲组的审批员。2006年,叶绍德于肝胆之间患上癌症;2009年4月4日香港时间中午12时20分,他因癌症引起并发症逝世于九龙播道医院;香港八和会馆于2009年4月24日假世界殡仪馆为他举行丧礼,翌日出殡。由叶绍德编剧、创作的粤剧作品多达70多出,著名的有《朱弁回朝》、《楼台会》、《洛水梦会》、《李后主》、《红楼梦》、《三夕恩情廿载仇》、《西河会妻》、《赵氏孤儿》、《碧血写春秋》等。1995年,叶绍德因《洛神之洛水梦会》而获香港作曲家及作词家协会颁发最广泛演出金帆奖(戏曲);1998年至2001年、2004年至2007年、2009年、2010年、2013年至2016年,合共十四年均以《李后主之去国归降》而获得最广泛演出金帆奖(戏曲)奖项。香港演艺学院于2007年6月27日向叶绍德授予「荣誉院士」殊衔;香港作曲家及作词家协会将向叶绍德追颁音乐成就大奖。","text2":"叶绍德因何病逝?","label":1} {"text1":"张容轩,台湾男子篮球运动员,目前效力于金门酒厂篮球队。张容轩从小在金门长大,高中就读新荣高中,2006年代表出赛HBL获得MVP。高中毕业后目标就读师范体系,原锁定高师大,后因师大有名额因此往台北发展。在球队选择上,因大专条款最终选择SBL而放弃大专联盟,效力台银队。张容轩当时决定效力台银的主要原因有:一、相较于其他明星球队,在台银的上场机会较多。二、台银的入行条件很吸引他,为篮球生涯退休后的生活做准备。张容轩的体能条件不算特别优异,特别的是他用脑打球,理解比赛的能力过人,台银教练韦陈明就说张容轩是个模范球员,从不迟到、态度正确也很受教。在99学年度UBA大专篮球联赛帮助台师大夺得大专篮球联赛(UBA)男子组冠军,缔造队史首度3连霸,并获选为冠军赛最有价值球员MVP。 张容轩赛后说:「想把这个奖分享给队上每个人,台师大每个人都有资格拿MVP!」成为台湾极少数拥有高中大学两大联赛的最有价值球员得主。加入金门酒厂篮球队后,以平均15.2分的成绩拿下第八季SBL年度最佳进步奖。张容轩于第8季超级篮球联赛转入金门酒厂篮球队,第9季超级篮球联赛转到新任总教练贾凡领军的台湾大篮球队。第11季SBL超级篮球联赛以场均13.1分、3.4篮板、1.4助攻入选「年度第1队」,甚至继第8季后,再度夺下「年度最佳进步奖」,张容轩表示,「对我来说,『进步奖』才是我认为的最大奖,这代表大家都看到你持续进步,每年我都以此奖为目标持续努力成长。」在兵多将广的台湾大突破重围,一举抱回生涯第二座年度进步奖并入选为年度最佳第一队,最佳五人的肯定,创下自己生涯的高峰。台湾大篮球队并在第11季SBL超级篮球联赛打进队史的第一次总冠军赛。球季过后入选中华白队,代表参加第五届亚洲杯男篮锦标赛,拿下历史以来最佳的银牌成绩。台湾啤酒篮球队总教练阎家骅今天证实,台啤和张容轩签4年合约,张容轩已随队练球。第十三季SBL超级篮球联赛台湾啤酒篮球队击败璞园建筑篮球队拿下总冠军,缔造篮球生涯大满贯(HBL,UBA,SBL)都有冠军的纪录。","text2":"张容轩当时为什么决定效力台银?","label":1} {"text1":"过海隧道巴士960S线是香港一条途经西区海底隧道巴士路线,由屯门(富泰邨)单-{向}-开往湾仔北,由九巴独自经营。全程:$20.8乘客登上本线后150分钟内以同一张八达通卡转乘以下路线,或从以下路线登车后150分钟内以同一张八达通卡转乘本线,次程可获车资折扣优惠:本线主要是由66M、66X及960抽调车辆行走。经:屯贵路、青山公路(岭南段、新墟段)、兆康站(南)公共运输交汇处、青山公路(新墟段、青山湾段)、屯兴路、屯门公路、屯门公路巴士转乘站、屯门公路、青朗公路、青衣西北交汇处、长青公路、长青隧道、青葵公路、西九龙公路、西区海底隧道、干诺道西、干诺道中、民吉街、统一码头道、民吉街、干诺道中、红棉路支路、金钟道、轩尼诗道、菲林明道、天桥、菲林明道、会议道及鸿兴道。","text2":"此线路是由何处开往何处的?","label":1} {"text1":"尖鳍魣(学名:),又称尖鳍金梭鱼,俗名针梭、竹梭,为辐鳍鱼纲鲈形目鲭亚目金梭鱼科的其中一种。本鱼分布于印度太平洋区,包括南非、东非、红海、留尼旺、马达加斯加、塞席尔群岛、印度、日本、台湾、菲律宾、越南、香港、澳洲、马绍尔群岛、马里亚纳群岛、纽西兰、诺克福岛、库克群岛、夏威夷群岛、法属波里尼西亚等海域。该物种的模式产地在印度。水深20至50公尺。本鱼体延长呈鱼雷状,横切面几近圆柱形,体呈银色,背部为银蓝色,侧线鳞列数110枚以上,延长的鳃耙数1,腹鳍起点较背鳍起点的垂直线为后,侧线下具一细纵带,上颔骨未达眼前缘,下颔骨尖端有一肉质突起,尾鳍分叉,背鳍硬棘6枚;背鳍软条8至9枚;臀鳍硬棘2枚;臀鳍软条8枚,体长可达80公分。本鱼栖息在近海沿岸的礁石区、潟湖或内湾,喜成群游动追捕猎物,属肉食性,以各种鱼类幼鱼、沙丁鱼、头足类为食。为鲜美的食用鱼,适合煎、烤食用。","text2":"尖鳍魣体长可达到多少?","label":1} {"text1":"琵琶湖特快()是西日本旅客铁道(JR西日本)营运的一个特别急行列车(特急)的名称。列车在东海道本线(琵琶湖线、JR京都线)行驶,来往米原和大阪。琵琶湖特快的前身是1988年开办的专线列车「琵琶湖Liner」()。当时以485系在平日提供服务。2003年起改用新型的681系或683系后,升格为特急班次。2007年3月17日,东日本旅客铁道(JR东日本)和东海旅客铁道(JR东海)营运的东海道本线特急列车「东海」停止服务后,琵琶湖特快成为东海道本线内唯一在日间行驶的特急班次。米原站 - 彦根站 - 近江八幡站 - 野洲站 - 守山站 - 草津站 - 石山站 - 大津站 - 山科站 - 京都站 - 新大阪站 - 大阪站京都列车区","text2":"琵琶湖特快途径哪些站点?","label":1} {"text1":"克里斯汀·玛利尔·莉莉(Kristine Marie Lilly,)是一名著名的美国女子足球运动员。她从1987年开始为美国国家女子足球队效力,现在是美国国家队的队长。她在美国国家队的位置是前锋,在这之前她一般是中场。莉莉在康涅狄格州的Wilton出世,现在居住在马萨诸塞州的布鲁克莱恩 (马萨诸塞州)。莉莉嫁给了当地的一个消防员David Heavey。David Heavey在大学时代是一名著名的曲棍球和高尔夫球员。她的女儿Sidney Marie Heavey 在2008年7月22出世。莉莉曾参加过HBO电视台的纪录片《Dare to Dream: The Story of the U.S. Women's Soccer Team》。1991年,莉莉在北卡罗来纳大学教堂山分校被评选为全美最佳大学生球员()莉莉是世界上为国家队出战A级比赛次数最多的球员。1998年5月21日,她第152次为国家队出场,打破了挪威的Heidi Støre的记录。2006年1月18日,莉莉为美国国家队出场达到300次。2008年,莉莉为为美国国家队出场达到342次。2006年,莉莉在世界足球小姐的评选中位列第二,仅次于巴西的玛塔。尽管没有当选世界足球小姐,但莉莉连续两年被评为了美国最佳女足运动员(2005和2006)。2007年9月11日,在参加完2007年女子世界杯足球赛对朝鲜队的比赛之后,莉莉成为了第一位参加了5次世界杯的女足运动员(第三位参加了5次世界杯的足球运动员)。在2007年9月22日对英格兰队的比赛中,莉莉打入一球,成为了在女足世界杯上进球的年纪最大的球员。由于产期,莉莉错过了2008年北京奥运会。2008年10月,莉莉重新加入美国国家队。","text2":"莉莉是哪个国家的女子足球运动员?","label":1} {"text1":"陈鸿平(Chan Hung Ping,),外号「嚤啰平」,是已退役香港足球运动员,司职中场,曾入选中华民国代表队参加国际赛。退役后曾执教多支甲组球队,1990年代初带领东方连夺三届香港甲组足球联赛冠军,创造「东方皇朝」,并曾经两度兼任香港足球代表队教练,现时担任香港超级足球联赛球队理文守门员教练。陈鸿平出身于香港老牌球会南华青年军,1960年转投愉园,并获提升上甲组,展开职业球员生涯。其后效力过元朗、怡和、精工、海蜂等多支香港甲组足球联赛球队,到1980年,以38岁之龄宣布挂靴。国际赛方面,陈鸿平于1960年协助中华民国赢得亚青杯冠军。其后亦入选中华民国代表队,参加过亚洲杯足球赛、默迪卡杯等赛事,协助中华民国赢得1965年默迪卡杯足球赛冠军,以及1968年亚洲杯殿军,并入选决赛周最佳十一人。1981年8月,由香港足球总会资助,与文锦棠一同前赴英国参加初级足球教练训练班。1982年担任香港甲组足球联赛「升班马」菱电助教,开始展开教练生涯,先后执教过海蜂、星岛、东方、民信、愉园、公民等多支香港甲组足球联赛球队。最辉煌成绩是1992-93年度至1994-95年度球季,带领东方连夺三届香港甲组足球联赛冠军,创造「东方皇朝」。其中在1992-93年度,更创下香港甲组足球联赛首循环赛事全胜兼不失一球的辉煌纪录。由2009-10年度球季开始,担任四海流浪教练,2011年夏天转任领队,与教练巴贝利合作带领球队。陈鸿平并曾经先后两度担任香港足球代表队教练。陈鸿平有过两段婚姻,皆离婚收场,育有三女一子,儿子陈念萱亦曾是愉园预备组球员。2011年8月6日傍晚,于元朗参加一场元老足球赛时晕倒,昏迷不醒送院,在博爱医院深切治疗部留医,至8月8日上午苏醒过来。8月15日接受心脏通血管手术后已出院,并于8月18日恢复工作。2012年12月4日,昔日队友李方辉、施维山、黄德祥,以及艺人尹扬明等,一起于筲箕湾南平坊硬地足球场举办一场小型足球赛,为他提前庆祝七十大寿。","text2":"陈鸿平为什么创造「东方皇朝」?","label":1} {"text1":"水力资源,是由水流体含有的能量天然资源,转化为人类利用的能源,例如水力发电。自古以来,人类就使用各种形式的水车等工具利用水利资源里的可再生能源,进行农田灌溉和各种机械装置的操作,例如面粉厂,锯木厂,织布,码头起重机,升降机和矿石磨等。从19世纪末期开始,人类开始建造水电站利用水利资源发电。在诺森伯兰郡的克拉格塞德是第一个住宅于1878年以水力发电供电,而第一个商用水力发电厂在尼亚加拉大瀑布于1879年建成。在1881年,在尼亚加拉瀑布城的路灯是由水力发电供电。自20世纪初,水力资源这个术语已被用于几乎完全结合水力发电的现代化发展。国际机构例如世界银行检视水电作为促进经济发展的手段而无需添加大量的碳到大气中,但在某些情况下有大坝的环境问题。水力资源的水能蕴藏在河流的水位落差和流量,其数值成正比例。单位数为电力常用的千瓦或马力。此外,自然的水力资源还有来自潮汐和海浪的流体力学。现时,人类十分依赖非再生能源,所以水力资源多数有待开发。","text2":"为什么如今水力资源多数有待开发?","label":1} {"text1":"若泽·博辛瓦·达·席尔瓦(,),葡萄牙足球运动员,于2012年夏季加盟英超球队昆士柏流浪。保辛华出生于刚果民主共和国(他的父亲是葡萄牙人,母亲则为刚果人),幼时随家人移居到葡萄牙。他原先是效力葡超球会博阿维斯塔,至2003年获葡萄牙班霸波图看中,获邀加盟该会。效力博阿维斯塔首季保辛华上阵机会不多,在葡超仅出场11次。他于2003年9月6日,波图作客赛和柏迪逊的比赛中初次出战欧联。该届欧联赛事波图最终夺冠而回,晋级过程中保辛华合共上阵8场。04至05球季,随着队友保罗费拉拿及Carlos Secretário离队他投,防守中埸出身的保辛华成为波图正选右闸,协助波图夺得五年内第四次葡超冠军。他在这几年间崭露头角,于2007年获选入葡萄牙国家队。当时已有不少顶级班霸争相招揽这名球员,结果他于2008年5月宣布加盟英超车路士。保辛华加盟车路士时,转会费公布为一千六百万镑,合约为3年。球会本于2008年7月16日,公布保辛华将穿16号球衣,然而到同年8月8日,保辛华却与队友史葛·冼佳亚交换球衣号码,改穿17号球衣。保辛华于2008年9月16日,车路士对波尔多的欧联赛事中初次为新球队上阵。他于2008年9月27日对史笃城的英超联赛中射入他在车路士第一球。2012年5月24日,保辛华于6月份约满,车路士虽有优先续约权,但最后放弃,保辛华可以自由转会。2012年8月17日,保辛华以自由身加盟英超球会昆士柏流浪签约三年。","text2":"若泽·博辛瓦最先效力于哪支球队?","label":1} {"text1":"霍氏树懒(学名:\"\")是中美洲及南美洲的一种树懒。牠们独居,夜行性及栖于树上,生活在次生雨林及落叶林。牠们的毛长,爪巨大,行动缓慢。牠们重5.5-7公斤,长60厘米,很适合攀树。与二趾树懒两种构成整个二趾树懒科。霍氏树懒分布在中美洲及南美洲,由尼加拉瓜南至玻利维亚,并由秘鲁东至巴西。霍氏树懒大部份时间都栖于树上,有时也会因要走到另一颗树而落到地上。牠们也是游泳能手。牠们完全是夜间活动的,在冠层缓慢的移动及吃叶子。牠们虽然行动缓慢,但并非如名般懒惰,而是为配合低能量的食性。牠们的新陈代谢率是同一大小哺乳动物的一半。牠们的视觉及听觉很差,差不多完全依赖触觉及嗅觉来寻找食物。霍氏树懒以其巨大的爪来挂在树上,其爪长约2-3吋。牠们差不多所有的行为都是在树上进行,有时甚至死后也挂在树上。牠们唯一会走到地上是在排泄时,但差不多每五天才一次。霍氏树懒的天敌有美洲豹、鹰及蛇。若受到威胁,牠们会以其爪或咬对方来防御。就算受到一些对哺乳动物来就是致命的伤势,牠们也能存活下来。由于牠们行动缓慢,毛皮上有藻类覆盖,令掠食者远距离很难发现牠们。牠们在树顶上的巢亦非所有掠食者可以到达。牠们长及粗糙的毛可以防晒及防雨水。霍氏树懒与褐喉树懒重叠了一些栖息地。由于霍氏树懒较为大只及数量较少,所以较少活动及较多于夜间出没。雌性霍氏树懒会成群的生活,而雄性则较多独居。在野外,雌性的数量较雄性多11倍。妊娠期为11.5个月,每胎会产一只幼懒。幼懒最少要有9个月的哺育,其后2年也会跟随母亲,约到4-5岁就会达至性成熟。霍氏树懒差不多只吃叶子,有时也会吃果实及花朵。牠们用唇来撕开食物,其牙齿没有珐瑯质及不断生长。牠们的胃像偶蹄目般有多个胃腔,充满共生的细菌来帮助消化纤维素。牠们可能要一整个月才能消化一餐的食物,其体重的三分之二都是胃内的叶子。霍氏树懒数量下降的原因可能是失去栖息地。牠们被列为《濒危野生动植物种国际贸易公约》附录三的保护物种。","text2":"雌性霍氏树懒和雄性霍氏树懒有什么区别?","label":1} {"text1":"记忆树是利用关联性记忆法,有效的将大量的资料整理并且快速的在脑中留下印象,特色是会以一个主题当作是主干,与其相关联的资料会采上下半辐射状依序连结出,呈现出类似树状图的图像,故称为记忆树,属于心智图应用的一种。记忆树的主要原理是采放射性思考的笔记方式,与传统的笔记方式中条列重点的方式相异,是用图像式地从记忆主干向外展开的记录方式来写。利用关联和所谓的强调去加强对事件的记忆程度,平常我们的大脑分左右去处理事情,人在记忆的时候用左脑处理文字和语言、而右脑则是处理影像,所以若在记忆的同时用图画在旁强调提示,当左脑遗忘了文字和语言,右脑反而会记下自己所画的图画。也就是说,记忆树的原理在于当单调的语言或是文字被我们的左脑所遗忘时,看到图像的右脑可以及时发挥功效,进而连结到左脑去唤起被遗忘的文字。整体关系是经过联想的,所以不但容易记忆,而且有助于思考。如此的记忆方式,也受到其他诸多国家采用。采用又画图又画线的记忆方式连来连去,记忆的物件就不单只是文字了,若是加进了很多别的东西,自然也会用到别的大脑区位,顺便正好可以刺激平常用不到的大脑区位,分散了记忆的工作。有些安养院会提供各种不同物品的图片去刺激老人的大脑,使老人的脑部因为刺激而减缓退化,在治疗阿兹海默症的非药物疗法中也是利用这个原理,将有纪念意义的照片记念品等放置在病人旁边,去延缓病人脑细胞的破坏。在日剧以及漫画的东大特训班也有提到了记忆树的画法,其中颠覆了学生对于传统抄写笔记的观念,一般的学生都认为在抄写笔记时,应力求版书的工整以及字迹的漂亮与否。然而在东大特训班中特别提到了\"关联\"才是最重要的部分。字丑一点没有关系,图画的不好看也无所谓,只要能够帮自己建构出事件与事件当中的关联性即可。以法国大革命为例有可以简单的画出记忆树的软体。FreeMind:以开放原始码的方式撰写开发的软体,可以画出思维导图,也就是心智图。","text2":"记忆树的主要原理是什么?","label":1} {"text1":"加西特王朝,是西亚两河流域古代王朝。公元前16世纪由加喜特人,克喜特人,喀西特人(Kassites)建立。又称加喜特巴比伦、巴比伦第三王朝。加喜特人可能原居于札格罗斯山脉中部,其语言由于语料缺乏而仍未归类,但其部份统治者有印欧人名字。公元前16世纪初加西特人占据巴比伦,建加喜特王朝。此时约当阿古姆二世时期。布尔纳布里亚什二世至卡什提里亚什四世期间(约前14世纪中~前13世纪后半),为其繁荣时期,与埃及新王国、赫梯帝国、亚述同为并立的大国。公元前13世纪后半期,亚述人和埃兰人入侵。约公元前1157年,加喜特王恩利尔纳丁·阿基被埃兰人拐走,加喜特王朝灭亡。加喜特人在入主两河流域后,将侵占的土地分配给本族的朝臣、贵族和官吏,形成大地产私有主。其政治体制基本上是贵族政治,后期地方贵族势力日趋强大,王权削弱。加喜特人恢复了两河流域的秩序、和平和统一,特别是在其统治的中后期,社会经济有所发展,巴比伦、尼普尔、西巴尔等城市经济相当繁荣。人们在两河流域推广用于牵引的马和战车,并在建筑物上以砖刻浮雕代替石雕。这一风格为以后的新巴比伦王国和阿契美尼德王朝所继承。加喜特人全面接受两河流域固有的文化、宗教和阿卡德语言文字,迅速塞姆化。其国王重建和修饰尼普尔、拉尔沙、乌尔、乌鲁克等地的神庙。这一时期的重要遗物“界碑”,实际是国王所授地产的凭证。史称此时为界碑时代,即得名于此。界碑的右面或上部刻神像或神的象征,如以圆盘象征太阳神沙马什,以月牙象征月神欣,以锄头象征马尔杜克等;左面或下部的铭文多为王授土地情况。","text2":"加西特王朝是什么时候建立的?","label":1} {"text1":"命理师,即民间所称的算命先生、半仙等,是利用术数来算命的古老职业。其刻版印象多为打著铁口直断的旗帜、掐指一算、留著八字须。除算命外,命理师一般对其他传统术数也有涉猎,因此有不少兼任风水师。现代有些命理师甚至有涉猎其他地区的占卜术如西洋占星术、塔罗牌等。在台湾,西元2000年以前政风较为保守,命理风水被官方视为怪力乱神,因此命理师形象往往成为综艺节目中串场的搞笑形象;而也常于新闻报章上看到神棍以改运等诱因行骗财骗色之时,因此扭曲其命理师的公众形象,并有诸多不利命理师职业的传言,例如:妻财子禄必有一破。但在西元2000年以后,传媒尺度大开,许多命理节目风行,如:开运鉴定团、命运好好玩,造成命理的风行与正当化。借由网路时代的来临,算命行为逐渐公开化与网路化,加上上述的媒体包装,命理师被包装成艺人一类。在时代的巨变中,许多古老职业的社会地位逐渐反转。例如:现代社会的艺人在过去并非中高社经地位族群,现在则为热门的职业,命理师也有此现象。","text2":"命理师是依靠什么手段被包装成艺人的?","label":1} {"text1":"鲣(拼音:jiān;学名:),又称正鲣、烟仔、小串、柴鱼,为辐鳍鱼纲鲈形目鲭亚目鲭科的其中一种。本鱼分布于全球各大洋的温暖海域。有时鲣鱼会跟随洋流迁徙至高维度海域,例如日本海。水深0至260公尺。本鱼体呈纺锤型,横断面几近于圆形,头稍大,尾柄细强,尾鳍分叉。除胸甲与侧线有鳞外,全体均光滑无鳞。背部暗青色,腹部银白色,侧线下方有4至10条暗灰色之纵纹,背鳍硬棘14至16枚;背鳍软条14至15枚;臀鳍硬棘0枚;臀鳍软条14至15枚;脊椎骨41个,体长可达108公分。本鱼喜好清澈的温暖的表层水域,在热带海域中定居,而在温带海域则呈季节性移栖洄游,小型鱼在前上方;老成鱼在后下方,有靠近船舷猛吃洒饵的现象,不食死饵。为相当重要性的经济鱼类,味甚佳,可制成柴鱼片、罐头及生鱼片。2013年于香港,曾有雪卡鱼中毒而引起的神经毒素食物中毒之纪录。","text2":"鲣分布在哪里?","label":1} {"text1":"葡萄牙国徽(),为盾徽,中间白盾绘有五个呈十字状排列的小蓝盾,蓝盾绘有五个呈X形十字分布的白色圆点。外面红盾绘有七座金色城堡。置于浑天仪上,再以用红绿两色绶带束起的橄榄枝装饰。此徽颁布于葡萄牙第一共和国成立后的1911年6月30日,且沿用至今。代表葡萄牙的徽章在大概在1000年前已经出现。最早可以追溯到勃艮第的亨利(葡萄牙开国君主亚丰素·殷理基的父亲)之银盾蓝色十字。经过多个世纪的改朝换代,葡萄牙国徽的图案内容也有所增减。1910年10月5日革命后,成立葡萄牙第一共和国,国徽则在翌年6月30日开始使用。葡萄牙王国在1139年宣告独立,自1143年签订萨莫拉条约正式从卡斯蒂利亚王国分离,象征著王权与铸币权的银色圆点便被加在勃艮第的旗帜上,以宣示主权的独立。经过中世纪长年累月战争的洗礼,当桑乔一世继承他父亲亚丰素的徽章时,盾牌上的图案已由十字被磨损成五个小盾牌。各小盾牌内的圆点起初有11个,后来塞巴斯蒂昂在位时把圆点的数量减至5个。这在现代被解读成与耶稣受难时的有关。红框和里面的城堡是亚丰素三世统治时期加上的。源自卡斯蒂利亚的城堡代表在收复失地运动中从摩尔人夺取的城池。当亚丰素三世夺取其兄桑乔二世的王位时,由于他是亚丰素二世的次子,并无准备登基成为国王。他的纹章融合了父亲葡萄牙式的盾牌和卡斯蒂利亚式的红色框界和金色城堡。尽管初时城堡的数量由8至12座不等,而亚丰素四世曾经定为12座,但后来塞巴斯蒂昂最终把数量定为现时的7座。与西班牙式的城堡不一样的是,西班牙式城堡的城门绘上的是蓝色(保持开放);葡萄牙式城堡的城门绘上的是金色闩门(没有开放)。浑天仪自15世纪起成为代表葡萄牙的其中一个元素,在许多葡萄牙的殖民地旗帜上(特别是巴西)也可轻易找到。它在航海的重要用途(辨认方向和测量距离)意味著地理大发现时代对葡萄牙以及其众多殖民地(即使在共和后也是如此)的重要性。相对王室的冠冕,虽然浑天仪被认为「共和」的象征,不过其实早在葡萄牙-巴西-阿尔加维联合王国时期经已在国徽上出现。这是由于巴西也是联合王国的一部分的关系。","text2":"浑天仪在航海的重要用途是?","label":1} {"text1":"洪龙浩(,;),是天主教平壤教区原主教。出生于朝鲜半岛平壤,1933年5月25日(基督升天日)晋升司铎。1944年3月24日被教宗庇护十二世任命为平壤宗座代牧区宗座代牧(vicar apostolic),领衔奥济亚,并于同年圣伯多禄及圣保禄宗徒节(6月29日)被担任过平壤宗座署理的德源自治会院区院牧波尼法爵‧苏尔主教祝圣为主教。1962年3月10日,当天主教平壤教区成立后,被教宗若望二十三世任命为该教区首任主教。他于1949年被金日成政府囚禁后失踪。根据平壤教区的前任宗座署理、首尔总主教郑镇奭枢机称:没有消息证明有司铎在1940年代末的迫害中存活下来,当时166名司铎以及修士被杀或被绑架。教廷年鉴仍然将当时的平壤主教,主教洪龙浩,列为失踪,他若活到现在已经是100岁了。这是圣座的姿态,以指明朝鲜教会经受并仍在发生的悲剧。2013年公布的《宗座年鉴》,洪主教被宣布辞世,教区主教之位随即从缺。韩国主教团已请求梵蒂冈册封圣人部发出「无异议」许可,从而为洪主教及其他八十位殉道者启动宣福程序。","text2":"洪龙浩出生于什么地方?","label":1} {"text1":"芦欣,天津人,是一名已经退役的中国足球运动员,司职中场,曾经短暂入选过国家队。芦欣长期在天津效力,1994、1995赛季效力于天津三星队,但出场机会不多。1996赛季被调入乙级联赛的天津万科队,帮助球队打入甲B联赛,但俱乐部不幸于1997赛季末降级。1998年初天津三星与天津万科两队合并,芦欣与刘云飞、田玉来、朱艺和王军被调整进入天津泰达。2003年,芦欣凭借在联赛中长期的出色表现,曾经入选过中国国家队,当时已经29岁。2005年,他以先租借后转会的方式加盟厦门蓝狮,帮助球队打入了中超联赛。2007年初,他加盟另一支中超球队上海联城,但是不到一个月球队即与上海申花合并,芦欣又来到了上海申花。赛季中前期,33岁的芦欣仍然获得了一定的出场机会,在同时期的中国球员中已属难得,到赛季末他正式宣布退役。2013年他受昔日队友,国家女足U-19青年队新任主帅王军的邀请加入教练组。","text2":"芦欣在哪一年一何种方式加入了厦门蓝狮?","label":1} {"text1":"黑岛 (法语:L'Île Noire;英语:The Black Island)是丁丁历险记的第七部作品。作者是比利时漫画家埃尔热。本作于1937年4月开始在比利时报纸《二十世纪小伙伴》上连载,并随后出版了黑白版。随后又在1943年和1966年出版了两个彩色版本。故事的舞台是在英国的苏格兰。内容主线是丁丁破获试图扰乱英国经济的伪钞集团。丁丁第一次来到英国,在误打误撞之下,竟从古怪的米勒医生家中庭园里,发现一个伪钞集团的贼踪!为了打击匪帮,丁丁不畏艰险,一直追踪到苏格兰北部海岸,闯进叫人闻风丧胆的黑岛。在这一册,当然少不得笨警探杜邦德和杜邦特兄弟\"拔刀相助\"、引爆笑弹了。究竟丁丁能否一举捣破伪钞集团?神秘的黑岛,又暗藏几许怪事?","text2":"《黑岛》是何时进行彩色化的?","label":1} {"text1":"雅典埃莱夫塞里奥斯·韦尼泽洛斯国际机场(,),是位于希腊斯帕塔的一座民用机场,2001年3月28日开通,为雅典及阿提卡大区服务。1995年,希腊政府与德国建筑施工公司豪赫蒂夫集团(HOCHTIEF)签订了一项合作发展计划,在斯帕塔地区修建一座国际机场。雅典国际机场是一家私营公司,并根据合作发展计划以及希腊法律,在30年内拥有对全部设施的“设计、融资、建造、完善、试营、维护、运作、管理以及发展”的特许权。希腊政府持有机场的55%的股份,其余的45%由三家私人股东持有。机场建筑工程总投资达到2.2亿欧元,费时四年完成。2001年3月28日,雅典国际机场正式投入营运。雅典国际机场位于雅典东南,离市区20公里,离市中心45公里。它占地17.5平方公里,一共有一座航站楼、两条跑道,89个停机位,24个旅客登机桥以及48个登机口。","text2":"雅典埃莱夫塞里奥斯·韦尼泽洛斯国际机场是哪里的民用机场?","label":1} {"text1":"长流村,为台湾南投县之村落,位在国姓市区北方,行政区划划分为国姓乡,今称水长流,旧称写法甚多:水冇流、水无流、水波流或水无楼。长流村之名有七种来由:至于水波楼是推判原由同水冇流,但仅是种读音不同。长流村全境面积10.6798平方公里。今日的水长流泛指长福村、长丰村、大旗村以及包含本村,其中大旗村具有争议,大旗村昔称大旗尾,距水长流村落甚远,且历史独具一体。水长流位在地处河阶平原之上,从中被水长流溪贯穿,向南流入北港溪,于柑子林与南港溪汇流后称为乌溪。长流村主要依赖台21线与县道133线进出,但县道133线并无穿越全境,自长流国小附近与台21线分歧,开始向南通往国姓市区,终点是衔接中潭公路;台21线自水长流溪东岸行经,与县道133公路分歧,向东驶往北港村,是联络台中、埔里的省道。自921大地震震毁中横公路,除了中潭公路,来往台中、花莲必须行走台21线至埔里,转接埔雾公路上中横公路,因此交通是甚为重要。","text2":"大旗村为何在水长流中最具有争议?","label":1} {"text1":"刘松伟(Liu Songwei,),生于中国上海,前中国足球运动员,司职左后卫,他于2017年6月遭廉政公署落案起诉他连同等5名前香港飞马球员涉嫌于3场预备组联赛中打假波操控赛果,现为通缉犯。刘松伟生于中国上海,于2008年球季季初开始便在南华于中国大陆集体训练时随队跟操,直到2009年正式来到港加盟南华不过当时被暂时放驻于预备组中磨练,最终他在2009年5月10日对和富大埔的联赛煞科战中于67分钟后备入替首次在甲组联赛亮相,在球季结束后刘松伟未有获南华续约转投四海流浪,于2012年随原有四海的班底加盟标准流浪并在6月1日以1–0击败的国际友谊赛首次代表香港参与国际A级赛,直至2016年季中离队返回中国加盟业余联赛球队无锡银阳。不过刘松伟在2017年3月23日中国国家队于世界杯外围赛主场以1–0击败南韩后,他在个人社交网站上载一幅中国队球员于大宝顶入全场唯一入球后庆祝的照片并留言批评香港队及教练金判坤的话后引来过百名香港网民不满留言百反击,其后刘松伟再于社交网站上载一幅分开了一半的香港身份证和往来港澳通行证又指没拿香港身份是这辈子做的最正确的决定(已经违反《人事登记规例》条例属刑事罪行可处$25,000罚款及监禁2年)而他的香港身份证显示虽然他在2009年2月起已居住香港满7年刘松伟并没有成为香港永久性居民,事隔三个月后刘松伟遭廉政公署落案起诉他于2016年2月24日至5月13日期间连同等5名前香港飞马球员涉嫌于3场预备组联赛中打假波操控赛果、诈骗香港飞马及香港足总,他现为通缉犯。","text2":"廉政公署落案起诉刘松伟什么事?","label":1} {"text1":"花煎()是一种朝鲜半岛传统的甜煎米饼,上面可食用的时花制成。韩语固有词又称为、、。在不少传统节日如上巳节、重阳节等都会食用。从高丽王朝起,朝鲜人在一些节日里有玩「花煎游戏」(,)的传统习俗,并会食用花煎。每逢上巳节,妇女就会带著叫燔铁()的一种厚煎板去郊游踏青,她们会采摘杜鹃花或其他盛开的花来做花煎,当中以杜鹃花煎(固有词为)最具代表性。花煎常会与加入同种花瓣的花菜(,一种把水果加入蜜糖水或五味子汁里制成的朝鲜传统甜品)一同食用,例如以杜鹃花煎与杜鹃花菜同吃。到了重阳节,人们也会举行花煎游戏,这时食用的花煎会以菊花瓣制作,亦即菊花煎(),配以菊花酒()或柚子花菜()。除了杜鹃花煎和菊花煎外,任何可吃的时令花卉都可用作制造花煎,例如春季常见以沙梨花制成的梨花煎()、樱花煎(韩语固有词:)、堇菜花煎(固有词:)。初夏时则多会食用蔷薇花煎()。鸡冠花煎(韩语固有词:)则多在秋季食用。当没有适合制作花煎的花时,会以水芹、艾草、石耳、红枣等做成花形装饰放在煎饼面团上。","text2":"花煎一般在什么时候食用?","label":1} {"text1":"海因里希·约翰·李凯尔特(Heinrich John Rickert,),德国哲学家和历史家,新康德主义弗赖堡学派的代表人物。1863年5月25日出生于但泽(今波兰格但斯克),1936年7月28日逝于海德堡。19世纪可以说是兰克史学发展的蓬勃期,但一些人并不认同兰克史学的「绝对客观」是存在的,尼采在一篇名为〈历史的用处及其弊病〉中说:「按兰克以及科学派的历史主义思想而言,那么,历史是没有生命的,是完全为一堆死的材料、死的东西。」但他是哲学家,所以这段看法没有受到大家的重视,直到他提出类似的看法才为人所接受。李凯尔特把科学分成四类:第一种是不涉价值又讲通则的(non-valuing and generalizing),即纯粹的科学,自然科学属之。第二种是不讲价值但讲个别性的(non-valuing and individualizing),强调个别的特殊性,地质学、生物学属之。第三种是涉及人的价值又讲通则的(valuing and generalizing),有发展规律人文活动,社会学、经济学属之。第四种是有价值问题且有个别性的(valuing and individualizing),即是历史。","text2":"海因里希·约翰·李凯尔特是哪一派的代表人物?","label":1} {"text1":"1980年代,因东线铁路改线及台东新站的设置,许多石板棺及陪葬品被发现,因陪葬品完整且精美,引起民众的注意及争相盗掘。于是国立台湾大学人类学系接受政府委托由连照美带领台大师生前往挖掘,宋文薰也前往指导。此次行动共抢救出二万四千多件文物,并由他们定名为卑南遗址史前古物,此后更进行十多次的抢救工作。学者普遍认为这是台湾新石器时代中晚期十分重要且具代表性的遗址,也是太平洋沿岸与东南亚地区规模最大的石板棺墓葬群遗址。连照美以此发表了很多篇论文。史前馆筹备处在1990年成立,首任筹备处主任就是因研究卑南文物而知名的连照美。但借调满两年后就辞去该职务,而由她保管的文物则跟著留在台大人类学系。因原订展示的卑南遗址史前古物几乎都在台大人类学系,史前馆盖好后一直没有主要的展示品而遭人诟病。于是台东地方人士积极的向台大人类学系追讨这批文物。由于连照美坚持这是当时没有人要,是她抢救来的,再加上史前馆位处地震带又曾发生火灾,于是拒绝将文物交给史前馆。2005年3月15日,在台东地方人士组成的「讨壶联盟」强势的作为下,台大人类学系趁连照美出国时,将部份文物交给史前馆派来的点收人员,成为第一批到史前馆的卑南文物。","text2":"史前馆筹备处在哪年成立?","label":1} {"text1":"白兰氏(英语:Brand's),是一家食品公司的品牌,著名于鸡精饮料,产品已经有170年历史以上。白兰氏品牌由食益补太平洋有限公司(英文名:Cerebos Pacific Limited)所有,而食益补是新加坡交易所主版上市公司,在1990年已经由日本三得利公司收购其股权。在1820年,白兰氏鸡精由英国白金汉宫御厨韩温白兰(H.W.Brand)发明,之后白兰先生于1835年创立白兰氏公司,在英国设厂生产白兰氏鸡精。2003年,白兰氏健康博物馆于台湾彰化县鹿港镇彰滨工业区开馆,除了企业相关主题外,其鸡精制造工厂以「空中走廊」的方式,开放一般民众参观。鸡精是白兰氏的主要产品,除传统鸡精外,亦开发添加冬虫夏草、花旗参等传统保健药材的鸡精。白兰氏另开发多种保健食品,饮品类包括红胶原青春饮、燕窝、蚬精及养参饮等。锭类包括维他命、深海鱼油、五味子芝麻锭。","text2":"白兰氏鸡精的发明者是谁?","label":1} {"text1":"斑鱵(学名:)为辐鳍鱼纲鹤鱵目鱵科的一种。本鱼分布于印度西太平洋区,包括东非、留尼旺、塞席尔群岛、阿曼、印度、泰国、越南、马来西亚、模里西斯、中国、日本、台湾、印尼、澳洲、密克罗尼西亚、新喀里多尼亚、帛琉、万那杜、菲律宾、东加、萨摩亚群岛等海域。其主要栖息于水的中上层。该物种的模式产地在红海。水深0-30公尺。本鱼体略呈长枪状,稍侧扁。吻细长且上颌短于下颔,眼大,体背部青黑色,腹部银白,鳞片大但易脱落。侧线上具5-6个明显黑斑,下颔尖端为红色。尾鳍深叉,体长可达45公分。本鱼栖息于亚热带海域的水表层,常栖息在水流平静的内湾,受惊时会越出水面。属杂食性,以藻类及浮游生物为主。繁殖期至河口产卵。属高级食用鱼,适合油煎食用。2.FishBase3.台湾鱼类资料库","text2":"斑鱵一般栖息在水的什么位置?","label":1} {"text1":"风吕敷是日本传统上用来搬运或收纳物品的包袱布。起源不清楚,但正仓院的藏品中有类似的物品。以前被称为衣包、平包。在室町时代末期被称为风吕敷,大名洗澡的时候将脱掉的衣服放在展开的平包上包起来,也可能是做为擦脚布,但具体的用途不明。最早的文字记录,见于骏府德川家分赠遗物的记载。此后,在江户时代随着澡堂的普及,风吕敷也在民间普及起来。此外,平包还作为风吕敷的包裹方法之一残留下来。这种布因为能用多种包裹方式来包裹各种形状与不同大小的物件而普及,为了搬抬各种形状和大小的东西,制作成了厚布,尺寸大的甚至有被褥大小。明治时代以后,从西欧引进了皮包后,风吕敷的使用率减少。现代,就算在日本街头也很少看到。但近几年,在保护环境等问题中,风吕敷被建议用来替代塑料袋。","text2":"风吕敷以前被叫做什么?","label":1} {"text1":"猪毛蒿(学名:)为菊科蒿属的植物。分布于匈牙利、印度、欧洲、日本、巴基斯坦、俄罗斯、朝鲜、土耳其、伊朗、阿富汗以及中国大陆遍及全国等地,生长于海拔2,800米至4,000米的地区,一般生长在路旁、黄土高原、林缘、荒漠边缘地区都有、草原、半干旱、半温润地区的山坡和局部地区构成植物群落的优势种,目前尚未由人工引种栽培。石茵陈、山茵陈、西茵陈、北茵陈(本草纲目),野同蒿、白蒿(救荒本草、植物名实图考),扫帚艾(广州植物志),土茵陈(南方省区俗称)、东北茵陈蒿(东北、华北省区俗称),滨蒿(西北省区俗称),白头蒿(河北),香蒿(河北陕西),臭蒿(河北、内蒙古),米蒿(内蒙古),棉蒿、沙蒿(山西),白毛蒿、灰毛蒿、毛滨蒿(吉林),黄蒿(内蒙古、黑龙江、吉林),小白蒿(陕西),迎春蒿、黄毛蒿(甘肃),白茵陈、白青蒿同、毛毛蒿(四川),绒蒿(广西),“阿各弄”、“伊麻干-沙里尔日”、“雅曼-沙里尔日”(蒙语名),“亚布泉”(维吾尔语名),“阿仲”(四川西部藏语名),“察尔旺”(青海藏语名)","text2":"猪毛蒿分布在哪些地区?","label":1} {"text1":"芦潮港站,是浦东铁路上的一个客运站,位于中国上海市浦东新区南汇新城镇(临港新城)附近,距离洋山深水港32公里,距离浦东国际机场37公里,距离虹桥国际机场80公里。于2005年12月1日正式投入营运,是中国的第一个铁路集装箱中心站。芦潮港站曾经每天有2对往来上海南站的旅客列车K8351\/8352、K8353\/8354次列车经停,该2对列车同时停靠海湾站。由于离危险品仓库过近,铁路芦潮港客运中心站自2015年8月27日起暂停运营。现在每周有上海开往合肥、南昌、苏州等地的集装箱班列。由于中国铁路运力严重不足,目前洋山港仅有0.4%的集装箱通过铁路运输。周边交通线路包括1059路、芦杜专线、泥城3路等。","text2":"芦潮港站是什么站?","label":1} {"text1":"开角龙亚科(Chasmosaurinae)又名角龙亚科(Ceratopsinae),是角龙下目角龙科的一个亚科,角龙科的另一个亚科是尖角龙亚科。开角龙亚科是群大型四足动物,前肢略弯,短于后肢。牠们的体型壮硕,四肢粗壮。头骨后方的头盾大,由顶骨、鳞状骨构成,头盾长度可相当于头骨本身。喙骨与前齿骨构成长、狭窄的喙状嘴。牠们具有多列的齿系,会不断地生长、取代磨损的牙齿。开角龙亚科与尖角龙亚科的最明显差异在于头盾与角的形状。与尖角龙亚科相比,开角龙亚科的鼻角较短、额角较长、头盾较长、头盾具有大的洞孔。但是,三角龙的头盾短,头盾没有洞孔。五角龙与牛角龙的头盾可达2公尺长,是已知具有最大头部的动物。头盾周围的颈盾缘骨突(Epoccipital)短小、或没有,与尖角龙亚科不同。如同其他角龙科,开角龙亚科生存于白垩纪晚期的北美洲西部。化石发现于坎潘阶与马斯垂克阶地层,年代为8,000万到6,500万年前。开角龙亚科是在奥塞内尔·查利斯·马什(Othniel Charles Marsh)在1888年建立。其名称来自于角龙属,但角龙的化石过于稀少,目前状态为疑名。因此劳伦斯·赖博(Lawrence Lambe)在1915年建立开角龙亚科(Chasmosaurinae)以取代角龙科。三角龙的头骨较短,产生分类学上的争议,较晚期才被归类于角龙亚科。","text2":"开角龙亚科又叫什么?","label":1} {"text1":"田钦祚,宋朝颍州汝阴(安徽省阜阳市)人。父田令方,官汉虢州团练使,被伶人靖边庭叛杀。钦祚早年任后周的殿直,不久改供奉官,从柴荣征淮南,为前军都监。入宋,拜为阁门通事舍人,后擢升为西上阁门使,平南唐后,以功加领汾州防御史。970年,契丹六万骑南下,田钦祚出征,赵匡胤指示他:“敌至即战,勿与追逐”。钦祚以三千人打败了契丹六万人,将其逼退,宋太祖喜曰:“契丹数入寇边,我以二十匹绢购一契丹人首,其精兵不过十万人,止费二百万绢,则敌尽矣。”。钦祚“性阴狡,多所忤犯,好狎侮同列,人皆恶之。”郭进与田钦祚素不和,屡揭其丑事。太平兴国四年,宋太宗命郭进守石岭关,派田钦祚作监军。郭进被田钦祚逼死。田钦祚在石岭关,“恣为奸利诸不法事”,宋太宗知道后,降其为睦州防御使,仍护石岭关屯兵。太平兴国六年秋,改房州团练使,一年后,又改柳州,当时柳州未开化,多瘴气,因此感染疾病,钦祚上表求返京,迁郢州团练使。太宗征幽州时,命钦祚与宣徽南院使郭守文为排阵使,但时钦祚有重疾,当晚因太高兴而过世。有子田承诲、田承说。","text2":"田钦祚的性格如何?","label":1} {"text1":"对食,最早是指宫女之间的女同性恋,与宦官与宫女结合的菜户不同。 。对食一辞最早源自汉朝,《汉书》记载:「官婢曹晓、道房、张弃,故赵昭仪御者于客子、王偏、臧兼等,皆曰宫(曹宫)即晓子女,前属中宫,为学事史,通《诗》,授皇后。房(道房)与宫(曹宫)对食。」,应邵注曰:“宫人自相与为夫妇名对食,甚相妒忌也。”,可见对食最早是指女同性恋,但也可能是境遇性性行为,由于宫女在宫中寂寞,又没有接触异性的机会,就与其他宫女发展出性关系。汉武帝的皇后陈氏也有类似记载,“巫著男子衣冠帻带,素与皇后寝居,相爱若夫妇。”“事发,楚服伏辜,皇后废处长门宫。”,但此事皆记载于野史小说,真实性存疑。后来「对食」一词变成宦官和宫女的恋爱和性行为,所谓“宫掖之中,怨旷无聊,解馋止渴,出此下策耳。”。隋唐五代时期的《宫词》有-{云}-:“莫怪宫人夸对食,尚衣多半状元郎。”徐鼐《小腆纪传》:“常中郭氏名良璞,故阉夏国祥之对食也。”对食可以是宦官与宫女,或是宫女与宫女之间的爱情,但多为短暂交往;而菜户则专指宦官与宫女的长久稳定恋爱关系,有如夫妻。唐朝时宦官可在外娶妻,如高力士娶妻吕氏,吕氏则与其他权贵之妻一样同受命妇册封,封为国夫人。明朝宦官在外娶妻是被严格禁止,主要为防止外臣干涉宫廷事务。南和伯方瑛妾许氏改嫁御用监左监丞龙闰。成化五年(1469年)事发,除二人离婚外,龙闰送司礼监处置。故宦官与宫女结成菜户,在明朝宫廷中是为常见。到清朝,宫女不再是终生服役,通常在25岁离宫后,宫女会进行婚嫁,且清代宫女为八旗女子,宦官多充汉族,民间就少有满族肯婚配汉族者,遑论宦官。中国古代女同性恋性行为多称为磨镜,因双方相互以厮磨或抚摩对方下体得到性满足,古代的春宫画也有一人女扮男装,在腰间系一假阳具和对方进行性交的。由于宦官缺乏性能力,一般的性爱方式仅限于爱抚、口交和假阳具。古今文史半月刊第九期杨静盦的〈记阉人〉记载:「阉人近女,每喜手抚口啮,紧张移时,至汗出即止。盖性欲至此已发泄净尽,亦变态也。」。《万历野获编》记载:「近日都下有一阉竖比顽,以假阳具入小唱谷道不能出,遂胀死。法官坐以抵偿。」有些宦官为恢复性能力,不惜杀人,食用小儿脑,《万历野获编》「对食条」-{云}-:「近日福建税当高策,妄谋阳具再生,为术士所惑,窃买童男脑啖之,所杀稚儿无算,则又狠而愚矣!」","text2":"对食和菜户有什么区别?","label":1} {"text1":"托尔尼奥(,,)是芬兰拉普兰区靠近瑞典边境的一座城市。托尔尼奥是托尔讷河谷的中心城市及芬兰与瑞典之间的重要过境关卡。托尔尼奥的人口数量为21939(2017年8月31日),面积为1348.83平方公里(2017年1月1日),其中1188平方公里为陆地面积、40.83平方公里为淡水区域、其余的120平方公里为海面。在1960年罗瓦涅米设市之前,托尔尼奥曾是芬兰最北部的城市。设立于1621年的托尔尼奥是拉普兰区历史最悠久的城市。托尔尼奥周边市镇为凯米、凯明马、泰尔沃拉和上托尔尼奥。托尔尼奥和瑞典一边的边境城市哈帕兰达保持紧密的合作。托尔尼奥位于波的尼亚海湾北岸芬兰和瑞典的交界线处。托尔尼奥市中心位于苏恩岛(Suensaari)上,由于陆地上升及人工填土的缘故岛屿已经扩展至芬瑞边境线上。芬瑞划界的一个原则是沿着穆奥尼奥河及托尔讷河的主航道来划分的,但在托尔尼奥市中心处却是个例外。尽管处于不同的国家及时区,托尔尼奥与河对岸的哈帕兰达形成了一对孪生城市。凯米位于托尔尼奥东南方向25公里处,与周围其它市镇一起,它们组成。除了市中心外,托尔尼奥还有其它区域位于托尔讷河的河岸上。托尔尼奥南北最长处有83公里,东西最宽处有44公里。除去海域面积外,城市面积有1228.38平方公里(包括41.37平方公里的淡水面积)。广阔的地域使得人口密度变小:每平方公里只有18.9人。托尔尼奥位于北方针叶林地带。主要的树种为欧洲云杉和欧洲赤松,常见的落叶树为桦树。整个市镇区域内一半以上为沼泽。托尔尼奥最高处海拔为189米。由于陆地上升的缘故,沿海地区每年上升6至8毫米。海岸多石,最靠海的部分为波的尼亚海湾国家公园的一部分。","text2":"托尔尼奥的主要植被是什么?","label":1} {"text1":"阿拉伯文数字是指阿拉伯文形式的印度-阿拉伯数字。这种数字被大多数使用阿拉伯文的国家使用,这包括阿拉伯国家和大多其他西亚国家。这种数字是由印度在中世纪引入阿拉伯,当时已经有很大的改变。在中世纪时,北非也使用了这种数字。之后,这种数字又从北非引入意大利,进入欧洲国家,慢慢的成为当今通用的阿拉伯数字。阿拉伯语把这些数字称为“印度数字”( ')。Unicode把它称为阿拉伯-印度文数字(Arabic-Indic Digit)。现在世界上主要有两种阿拉伯文数字:同阿拉伯文不一样,当代的阿拉伯文数字一般从左向右排列。而在一些古典作品里是从右向左排列的。在埃及,“二”通常用另一种写法。","text2":"阿拉伯语把这些数字称为什么?","label":1} {"text1":"真心之音广播电台为台湾唯一以儿童、青少年为为收听对象的民营非营利广播电台,节目内容以生活教育为主䌷。电台位于台北地区,目前尚为未立案电台,即一般俗称「地下电台」,2011年调频无线停播,转为网路直播电台,成立发音至今(2015)19年。创办人为媒体人 戴静远 先生,现41岁,23岁时成立广播电台,从事电视后制工作近二十年。改名为真心之音联播网,持续约半年即告终止,后又与马祖地区新成立之马祖生活资讯电台进行全时段联播,但随马祖电台的易手宣告终止。公益非营利跨宗教性电台,以十八岁以下儿童及青少年为收听对象,节目内容以生活教育为主;节目制播、工程之成员皆为志工性质,每年并经常举办社区公益活动,至今已结合相当多之公、民营机关团体及基金会。任何一个电台经营本身皆需消耗庞大资金,维持电台的开支以广告为主,原理上真心之音广播电台的属性于广播市场上是当难以生存的,因其公益、教育、收听对象均较不为广告主喜爱,真心之音纵然受青少年听众及其师长喜爱,但广告收入方面依然难以推展,但可贵的是真心之音广播电台之节目制播人员、工程人员为志工性质,不收任何费用甚至拿钱赞助电台开支,而器材供应商更以低于市场价格或赞助方式供给真心之音。这应是台湾大力推展近用媒体近二十多年来的典范。","text2":"真心之音广播电台的收听对象是谁?","label":1} {"text1":"望安岛是澎湖群岛第四大岛屿,位于澎湖县南方的海域,离澎湖本岛距离约有18海浬之远,而整个面积大约有6.7413平方公里,望安地势较平坦,最高高度54公尺,海岸弯曲多,岛上附近海域大多为暗礁。望安地理位置约在东经119度18–59分至北纬23度12–23分之间,附近还有40个大小岛屿及礁塭。从天台山上可以居高临下地眺望望安岛,山上有一著名的仙迹岩,其实是一块状似人类脚印的玄武岩。传说中仙迹岩是当年八仙过海时吕洞宾突然喊起肚子痛,可又临时找不到可供如厕的场所,只好一脚踏在望安的天台山上另一脚踩在花屿东边的海崖上,就开始方便起来了,而那些落下的黄金就化为澎湖诸岛中的一些小岛了。中社古厝原名为花宅,建筑风格仍保有清末民初的特色,吸引各地画家、摄影家及建筑科系的教授与学生来此勘查研究。因此,中社古厝巡礼至今也成为观光客必到景点。每年总会有绿蠵龟到回游至望安岛上产卵,为了保护这些稀客,岛上特地规划了多处绿蠵龟产卵栖地保护区,也砸下巨资设立了绿蠵龟保育中心。鸳鸯窟为日治时期,日军的重要军事站,当时被计划为快艇基地,后来因为日军战败而打消念头,至今仍留著当时所挖掘的两个巨大洞窟。其地区海流缓和,海水清澈,政府在那兴建了凉亭、公厕,也规划了露营区。早期的望安岛原称为八罩岛,因古时郑成功抵达七美屿,在高处望见了八罩岛,又因其海域清澈且平静,望之而心安,当下易名为望安,别名又称为网埯、网埯澳。在近年来被观光游客称为『蜜月岛』。居民的维生大都靠渔业,农业为辅。近年来海上渔业资源日渐枯竭,导致人口往外发展情况越来越严重。望安岛盛产文石、珊瑚、九孔、海胆、紫菜、海苔、小管干等物产。往来望安的交通,有民营的交通船光正陆号(客货两用)及光正参号、光正捌号(载客快艇)。平均每天皆有往返马公的船班。澎湖车船管理处有南海之星、恒安一号交通船,往返马公、七美途中有停经望安。空中交通,有德安航空以小型飞机于高雄-望安间来回直航。","text2":"这座岛屿名称的由来?","label":1} {"text1":"内大臣府是日本曾经设立以辅佐天皇、负责宫廷之文书等事务之政府机关。创立于1885年,1945年废止。其最高长官为内大臣,并不等同于日本古代律令制度下的同名太政官职位。明治政府在1885年将太政官制改为内阁制度。作为内阁成员之一,内阁总理大臣职务从国务大臣原负责之职务分离,内大臣一职则负责内阁权限之外宫廷事务而再度设立。明治政府下所设立之内大臣平常在宫中辅佐天皇,负责玉玺保管和诏令与其他宫廷文书之事务。内大臣负责执行天皇之决定,而非直接对民众负责,其职务与权力的范围不甚明朗,缺乏准确之定义,通常是天皇非常信赖、并通常有特殊关系之人。当初太政官制废止之时,前太政大臣三条实美出任内大臣。内大臣空缺之紧急情况由枢密院之议长临时代理内大臣之职务。这始自二二六事件之后枢密院议长一木喜德郎紧急代理内大臣(虽然当日即辞去职务)之先例。另外、还有身为皇族之贞爱亲王出任内大臣(1912年 - 1915年)之先例。当初之内大臣在昭和时代由元老取代,在政府中有非常大的权力。太平洋战争结束后,驻日盟军总司令(SCAP)于1945年11月24日命令废除内大臣府。1907年以降,内大臣府由11人名职员构成。至废止前由以下的人员构成。※括号内为任期","text2":"1885年明治政府将太政官制改为什么?","label":1} {"text1":"艾列高·沙基(),出生于富西尼亚诺,是一名意大利足球教练,曾在1991年至1996年带领意大利国家足球队。球会方面,沙基曾两次带领AC米兰,分别为1987年至1991年和1996年至1997年。任教练之前,沙基只是低组别联赛球会之业余球员,正职则是鞋店的推销员。其后断断续续于小型球会执起教鞭,但只属辅助性质。1985年正式成为教练,首家球会为帕尔马,当时只属丙组。其后于1987年转投AC米兰,首季就成功赢得意甲联赛冠军。当时他一手收购了古列治、列卡特、云巴士顿等名将,并提拔了如马甸尼等后卫。到1989年、1990年,更连续两届赢得欧洲冠军球会杯。1991年开始执教意大利国家队,曾带领国家队打入1994年世界杯决赛,仅于互射十二码中不敌当届冠军巴西。到1996年卸任,随即重返AC米兰,但因战绩欠佳而下课,且仅执教了一年。此后沙基把重心放在西甲,如曾于1998年-99年间任马德里体育会主教练;2004年又任皇家马德里足球总监一年。","text2":"转投AC米兰后艾列高·沙基做出过什么成就?","label":1} {"text1":"鳢鱼(学名:)又称鲖鱼、斑鳢、南鳢、鳢、台湾鳢鱼,俗称雷鱼、乌鱼或南方蛇头鱼。鳢鱼在鳢科中算是较大型的种类,一般成年鳢鱼体长可达10至40公分之间,而最大纪录也有60公分,与泰国鳢()体型差不多大。鳢鱼的身体修长,呈圆筒型或棒型,头顶平,身体后方侧扁,口部大,并较其他鳢科鱼类来地短而钝,下颚稍微突出,瞳孔稍小,尾鳍以及胸鳍呈圆形,背鳍基部长,侧边腹鳍短小,身体大部分区域覆盖著中小型鳞片,近又头部部份覆盖著大型鳞片。鳢鱼的体色通常为暗灰色或者为灰黑色,侧边无侧线,但有2纵列黑色斑纹,两边约各有10至12个,腹部则呈现灰黑色,各鱼鳍颜色均较为淡。鳢鱼为次级淡水鱼,喜爱栖息于河川中下游流域、水库、湖泊、池塘、沼泽、沟渠等水流和缓与水生植物繁多的浅水区,并喜在水底游动,躲在树枝底下或水草丛中。鳢鱼可以爬上岸,能在缺氧或淤泥中生活,但无法长期脱离水中,且在陆上不太灵活,可能是鳢鱼的身体过于粗壮及脊椎数过少导致的。鳢鱼分布不广,原产于中国、日本、菲律宾等亚洲国家。原本被引入美国当做宠物鱼或观赏鱼的鳢鱼,现今被一些饲主非法弃养,流入美国东部沿岸河流、密西根湖及西海岸加州的银木湖(Silverwood Lake)等地。鳢鱼的性情凶猛,掠食性极为强,对食物并不挑剔,主要食物为小鱼、两栖类以及一些甲壳纲动物(如:螃蟹、虾类等)。鳢鱼被人们视为一种十分高级的食用鱼,在大部分餐厅都能见到鳢鱼。此外,鳢鱼汤也是术后进补的好食材。","text2":"鳢鱼汤是什么时候进补的好食材?","label":1} {"text1":"纳忠(),字子嘉,中国回族人,出生于云南通海县一个穆斯林家庭,在学术上曾编撰及翻译多部有关阿拉伯文化的书籍,推动了中阿文化交流及友好。此外,他曾担任中华人民共和国全国政协委员及中国伊斯兰教协会的常委、顾问等职务。纳忠生于云南通海县纳家营一个传统的回族穆斯林家庭,是元代忽必烈派驻云南任平章政事的赛典赤·乌马尔·赡思丁的后裔。到纳忠的祖父一代,已是普通的农民,并且仍笃信伊斯兰教。青年时代,纳忠在求学上已有突出的表现。他在1929年毕业于昆明明德中学(昆明高等中阿双语学校),并且是第一位以公费留学埃及艾资哈尔大学的中国回族学生。他在艾资哈尔大学取得最高传统学位「学者证书」,接著考进该校的研究院,专攻阿拉伯历史与伊斯兰文化。1940年纳忠学成归国后,历任昆明明德中学教务主任、代校长,并任《清真铎报》主编、国立中央大学(南京大学)、云南大学、外交学院、北京外国语大学教授、系主任。纳忠除了在教育界作育英才外,还投身社会,参与多种工作。他曾任六届全国政协委员、中国伊斯兰教协会常委及顾问、中国非洲史研究会会长及名誉会长等等。2008年1月24日13时25分(中午)纳忠因病医治无效去世,享年99岁。他的葬礼,在1月26日8时30分(上午)于北京牛街中国伊斯兰教协会礼堂举行。纳忠至晚年仍勤于钻研笔耕,因此留下专著和译著甚丰,有二十多部,发表论文亦有二百余篇。其中一部份如下:纳忠曾荣获不少殊荣,如中华人民共和国国务院评为第一批「对高等教育事业有突出贡献」的专家;2001年又获得联合国教科文组织颁发首届「沙迦阿拉伯文化贡献奖」的殊荣。<\/div>","text2":"纳忠获得过哪些殊荣?","label":1} {"text1":"视觉化程式设计语言(,以下简称VPL),又称『图形化程式语言』、『视觉化程式编成语言』。系使用者利用图形化元素进行程式设计;相异于文字式程式设计。VPL以视觉表达为基础,利用『文法』或是某种『辅助标记』进行图形与文字的排列。许多VPL建基于『方块与箭头』的概念之上,以方块或萤幕上的物件为本体,以箭头相连接,以直线段与弧线段代表相互之间的关系。更进一步的分类VPL,依据类别与视觉表达延伸的使用,分为:图示式程式语言、表格式程式语言以及图表式程式语言。视觉化程式设计环境(Visual programming environments)提供图形与图示的元素让使用者操弄,以便符合特定空间文法以进行程式建构。一种经视觉化转换的程式语言是利用视觉表达叠套在非视觉程式语言作成。视觉化程式的视觉表达天性是没有文字式程式语言的例子可供比拟的。视觉化程式设计在目前的发展是尝试整合视觉化程式设计的研究,一是对于状态式程式设计进行线上除错,二是程式产生自动化与文件产生自动化。另外还有资料流程式语言所带来的平行自动化,那将会是未来一项重大的程式设计挑战。注意:Microsoft Visual Studio 及其包含的程式语言(Visual Basic,Visual C#,Visual J#,等。)通常会被混淆成视觉化程式设计语言,但它们不是。这些程式语言都是文字式程式语言。MS Visual Studio 是一种视觉化程式设计环境。不要混淆成视觉化程式设计语言。\"This article was originally based on material from the Free On-line Dictionary of Computing,used with 。Update as needed.\"","text2":"许多VPL建基于什么的概念之上?","label":1} {"text1":"卜公体育会(HK Blake Garden A.A. Ltd,简称卜公),是香港一支带有地区色彩的足球队,由上环卜公花园一班热爱足球的人士组成,曾培育出包括胡国雄与梁帅荣两位香港足球代表队队长等球星。卜公曾经升上香港甲组足球联赛角逐,现时已被淘汰出局。位于上环太平山街的卜公花园足球场,在战后初期至1970年代,曾经是香港岛小型足球的热门场地,卜公花园一班热爱足球的青年,组成卜公足球队,至1971年正式注册社团。1970年代,卜公于小球界称霸,在小型球总会的七个比赛中赢了六个,于是便决定加入为香港足球总会会员。卜公多年来孕育出无数球星,早期有莫振华、区彭年、邝演英,1960年代有「卜公三宝」胡国雄、施建熙、崔永生,以及毕伟康、黎新祥等,1970年代亦有尹志强、梁帅荣、邓锦添等不少本港足球人材。1975–76年,卜公以丙组球队身份,杀入初级银牌决赛,并于决赛凭左翼李剑虹射入十二码,越级挑战成功以 1–0 击败乙组强队怡和,勇夺冠军。1976–77年,卜公获得香港乙组足球联赛亚军,得以首次升上香港甲组足球联赛作赛。1977–78年,卜公在甲组的首个球季,便成功杀入足总杯决赛,可惜最终不敌精工,只能够屈居亚军。1978年11月4日,在花墟球场上演的一场甲组联赛卜公对元朗,两队球员比赛中途大打出手,最后要警察到场调停,卜公外援球员马文拿更受伤需要送院治疗。1979年5月28日,甲组联赛卜公于花墟球场对警察,在天雨下只得8名观众购票入场,创下香港甲组足球联赛最低入场人数纪录。1978–79年,卜公在甲组联赛名列包尾,在甲组只角逐了两个球季,便要降回乙组作赛,自此未再回升甲组。2008–09年球季,卜公在香港丙组(甲)足球联赛 19 战 8 胜 7 和 4 负得 31 分,以第 8 名完成赛季。2010年3月11日晚上,卜公为出身于该会青年军的已故亚洲第一中锋尹志强,于湾仔修顿球场举办纪念赛,并颁赠纪念品予尹志强父亲及女友米雪,并以鼓掌一分钟来怀念尹志强。香港足球总会于2012–13年,重组丙组联赛,并复办丁组联赛。卜公在上一届香港丙组(甲)联赛名列第 17 名,故需降落丁组联赛角逐。卜公在复办的首届丁组联赛成绩欠佳,经过半季之后,在15队中排名包尾,被淘汰出局。","text2":"卜公体育会在什么地方?","label":1} {"text1":"接景是用来创造虚拟摄影棚和数码外景地的一种技术”。可以用来创建整个场景,或者对已有场景进行扩充。传统的接景是在一片玻璃上用颜料或者水彩进行绘制,然后再通过光学方法与原始镜头合成。两块玻璃板平行摆放保持恰当的距离。摄像机位于他们前面。在后面的玻璃板上,是相对粗糙的背景地形,比如绘制的丛林。在前景玻璃板上,绘制细节丰富的元素,比如小植物,石头,等等。在玻璃板之间,应该就是移动或者静止的影片模型。如今,接景借助电脑制作,并以绘图板作为绘画设备。在数码环境里,接景 也可以制作与三维环境,允许三维摄像机运动。传统的接景是被画家用水彩或颜料绘制在玻璃板上的,然后再将其与实际拍摄的镜头合成(如果实际拍摄了镜头的话)。第一个接景镜头是Norman Dawn (ASC)在1907年的电影《加利福尼亚任务》里即兴绘制的“摇摇欲坠的加州任务”。值得注意的传统接景镜头有多萝西在《绿野仙踪》中绘制的“翡翠城”,《公民凯恩》中查尔斯·福斯特·凯恩绘制的“世外桃源”,还有《星球大战第四集:新希望》里那看似深不见底的“牵引光束”镜头。到20世纪80年代中期,在计算机图形程序的进步下,磨砂画家开始转向了到数字领域工作,于是“接景”这一工作也就逐渐与“后期特效”融合到了一起。第一个数字接景镜头是由画家克里斯·埃文斯在1985年的电影《少年福尔摩斯》中使用CG,绘制的一个骑士从彩绘玻璃窗口跃起的场景。埃文斯先用丙烯酸树脂画出了窗子的样子,然后把这幅画扫描到了卢卡斯电影公司的皮克斯系统,来进行进一步的数字操纵。电脑动画完美融合了数字接景,达到了传统的接景技术所不可及的高度。在整个20世纪90年代,传统的磨砂画仍然在使用,但更经常与数字合成。《虎胆龙威2:死得再惨点儿》(1990)是将传统的玻璃磨砂喷漆与已拍摄和扫描到计算机中的现场片段使用数字合成的第一部电影。那是机场跑道上发生的最后一个场景。在九十年代末,手绘绘景的时代已经接近尾声,虽然在1997年后期,一些传统绘画仍然派上了用场,尤其是出自詹姆斯·卡梅隆的《泰坦尼克号》的的绘画“卡帕提亚救援船”,由克里斯·埃文斯绘制。","text2":"现如今的接景以什么作为绘画设备?","label":1} {"text1":"《暴走兄妹》(;另译康子与健儿)是日本漫画家或子所画漫画作品。在集英社发行的女性漫画杂志《Deluxe Margaret》的2004年1月号发表后,于《别册Margaret》2005年4月号至2006年11月号连载。单行本全5册。2008年由日本电视台改编拍成电视剧,7月到9月于每周六播出,由松冈昌宏(TOKIO)、广末凉子、多部未华子、大仓忠义等人主演。另外在《Deluxe Margaret》从2009年1月号开始、发表番外编的「暴走兄妹」。6年前的12月24日,双亲因为交通事故而死,于是冲康子和原暴走族总长的哥哥冲健儿开始住在一起。但是由于哥哥的阻挠,使得她总是无法好好谈恋爱。于是兄妹因此每天过著吵架的日子,但康子却某天遇上了资优生桩纯而爱上他,但纯的老姐却是以前常和健儿吵架的另一原暴走族总长惠理香,因此康子除了要让纯喜欢上她外,还要努力改善健儿和惠理香这对欢喜冤家的关系,故事也在此展开了一段非常暴走的生活。从《Deluxe Margaret》2004年1月号开始连载。收录有『三篇和红色脚踏车』。短篇版的设定,康子16歳(高校1年生)、健儿是23岁。另外健儿的助手摩斯和爱迪达也有登场,但长相有些微的不一样、也没有名字。另外伴随著戏剧版化、在《Deluxe Margaret》2008年9月号再度连载。《暴走兄妹》电视剧集于2008年7月12日到9月20日在日本电视台播出。播出时间是周六的21:00〜21:54(JST)。主演的是TOKIO的松冈昌宏,是松冈自从主演连续剧2006年播放的《夜王》(TBS系)以来,阔别两年三个月参与连续剧演出,而演出同台的连续剧则是自从2004年播放的《秀逗男护士》的四年以来,再度演出。本剧2009年在香港无线剧集台首播,香港版本译名为《暴走冤家》。收视率为日本关东地区,由Video Research所调查。","text2":"《暴走兄妹》2008年由日本电视台改编拍成电视剧,由哪些人主演?","label":1} {"text1":"天主教巴黎总教区(;)是天主教会在法国设立的23个总教区之一,位于法国首都巴黎。创立于3世纪,首位主教是圣德尼。1622年10月20日升格为总教区。巴黎教省包括整个法兰西岛大区,附属教区有克雷泰伊教区(瓦勒德马恩省)、埃夫里-科尔贝埃索纳教区(埃松省)、莫城教区(塞纳-马恩省)、楠泰尔教区(上塞纳省 )、蓬图瓦斯教区(瓦勒德瓦兹省)、圣但尼教区(塞纳-圣但尼省)和凡尔赛教区(伊夫林省)。礼仪中心位于巴黎圣母院,总主教住所位于巴黎第六区的Barbet de Jouy路,教区办公室位于该市其他地方。2011年有教友1,340,463人,占辖区总人口60.0%、113个堂区、1,345名司铎、111名终身执事、1,054名修士、1,762名修女。现任教区总主教为安德烈·阿芒·万-特鲁瓦枢机。","text2":"天主教巴黎总教会位于哪里?","label":1} {"text1":"卡路士·艾坚哈顿·艾华斯(,)是一名千里达足球员,担任右翼,现效力英冠球队米禾尔。卡路士·艾华斯出道自千里达球会Defence Force。他在2000年以25万镑加盟-{zh-hans:雷克瑟姆; zh-hk:域斯咸;}-。艾华斯在域斯咸度过五年,上阵165次。在2005年,他免费加盟英冠球会卢顿。他成了球迷的宠儿。在2007年1月2日日,艾华斯以150万镑加盟新特兰,签约三年半。他在新特兰的表现抢镜,接连在对锡菲联、高云地利及伯明翰城取得入球。在对修咸顿的比赛更射入致胜一球。在对般尼的比赛入球,更把新特兰带返英超。2008\/09年球季由于马白兰基加盟影响艾华斯的出场机会,于10月2日被外借到狼队3个月。11月20日仅为狼队出赛6场后被提早召回。2009年9月1日艾华斯转投英冠球会叶士域治,转会费135万英镑。2012年3月30日艾华斯与叶士域治续约留效多一季。卡路士·艾华斯曾代表千里达出战2006年德国世界杯。在对瑞典,他的表现令他成为全场最佳球员。2011年10月10日艾华斯在法庭承认于停牌期间驾驶,事发于9月26日,前一天他的妻子因怀孕并发症而入院,艾华斯送女儿上学,但未能截获的士而决定亲自驾驶。其妻于10月8日早产诞下只有两磅重的双胞胎。10月13日艾华斯被判入狱12星期,但缓刑12个月,同时罚款2,500英镑和需要进行200小时社会服务,他亦被所属球会处罚两星期薪金。","text2":"艾华斯加盟新特兰后个人表现如何?","label":1} {"text1":"因贾兹(阿拉伯文:,意为“成功”、“成就”),出生于2009年4月8日是一头雌性单峰骆驼,也是世界上第一头克隆骆驼。 阿拉伯联合酋长国迪拜骆驼繁殖中心的兽医及胚胎学专家尼萨尔·艾哈迈德·瓦尼博士于2009年4月14日宣布,经过克隆代孕母体378天“不太复杂的”妊娠,小骆驼成功诞生。此次克隆计划获得了阿联酋副总统兼总理、迪拜酋长国埃米尔穆罕默德·本·拉希德·阿勒马克图姆的亲自批准和资金支持。在此之前,迪拜酋长国已经进行过数次不成功的克隆骆驼试验。因贾兹是由一头2005年被宰杀的成年骆驼的肉中提取的细胞克隆出来的。这些细胞在组织培养皿中生长,后被置于液氮中低温冷冻保存。之后,科学家将其中一个细胞的细胞核用显微注射法植入代孕母骆驼的一个被移除了细胞核的卵母细胞内,并用电流和化学方法促使细胞融合并发生细胞分裂。合成的胚胎经过一周的培养,又被植回代孕母骆驼的子宫内。二十天后,科学家用超声波监测出母骆驼怀孕了,而且在她的整个妊娠期间科学家都会用相同的方法进行监测。因贾兹出生后,它的DNA被送往迪拜的分子生物学与遗传学实验室进行鉴定,证实它确实是原骆驼细胞的复制品,从而证明因贾兹克隆成功。赛驼在阿联酋是一项有利可图的产业。骆驼繁殖中心的科学主任卢卢·斯基德莫尔博士对此评论说,因贾兹的成功克隆“为将来保护我们参加赛驼和产奶的良种骆驼的有价值的遗传科学提供了方法” 。","text2":"因贾兹是由谁克隆出来的?","label":1} {"text1":"超实数系统是为了严格处理无穷量(无穷大量和无穷小量)而提出的。自从微积分的发明以来,数学家、科学家和工程师等(包括牛顿和莱布尼兹在内)就一直广泛地用无穷小量等概念。超实数集,或称为非标准实数集,记为formula_1,是实数集  的一个扩张;其中含有一种数,它们大于所有如下形式的数:formula_2这可以解释为无穷大;而它们的倒数就作为无穷小量。 满足如下性质:任何关于  的一阶命题如果成立,则对  也成立。这种性质称为传达原理。举例来说,实数集的加法交换律是关于  的一阶命题。因此以下命题同样成立:也就是说超实数集同样满足加法交换律。无穷小量的概念是否严格呢?此问题可以追溯到古希腊数学:数学家们如欧几里得、阿基米德等,为了在一些证明里绕开无穷小量的争议以保证严格性,而采用了穷竭法等其它说明方式。而亚伯拉罕·鲁滨逊在1960年代证明了,换句话说,如果对实数的使用没有怀疑,那也可以放心使用超实数。在处理数学分析的问题时对超实数、尤其是传达原理的使用,通称为非标准分析。","text2":"无穷小量等概念一直被哪些人广泛使用?","label":1} {"text1":"《扶桑花女孩》是一部真实事件改编的日本电影,改编自日本常磐市Hawaiian Center(现为Spa Resort Hawaiian)的成立过程,由日本导演李相日执导。故事背景是在昭和40年(1965年),日本福岛县常磐以煤矿维生的小镇逐渐没落,而镇长打算兴建「夏威夷度假中心」来拯救小镇的失业危机。度假中心的负责人吉本(岸部一德 饰)特地从东京聘请一位舞蹈老师平山圆香(松雪泰子 饰)教导矿工的女儿们练习草裙舞,但此事受到当地居民的大力阻挠。谷川纪美子(苍井优 饰)与木村早苗等人却努力跟著舞蹈老师学舞,不久早苗的父亲被解雇,早苗因为家庭因素不得不放弃自己的梦想。其后随着大家的努力,扶桑花女孩逐渐打开知名度,纪美子的母亲看到自己的女儿练舞的神情而产生了怜惜。她终于了解到,练习跳舞也是一种努力的付出。为了借用暖炉,她替渡假村四处奔走,表演当天还在舞台前面忘情的为自己的女儿加油。第30届日本电影金像奖:2006年蓝丝带奖:电影旬报十佳奖","text2":"《扶桑花女孩》改编自什么故事?","label":1} {"text1":"《猎鹰》(),香港电视广播有限公司时装警匪电视剧,于1982年首播,由刘德华、陈敏儿、叶德娴及刘江领衔主演,监制李添胜。此剧乃刘德华首度出任主角的剧集。是港剧少有其中一个首演主角而一炮而红并从此演艺事业走上康庄大道的特例,另一著名例子就是过客之于黄日华。此剧于1996年、2003年及2012年在翡翠台、2005年8月在广东广播电视台珠江频道、2011年9月18日及2014年3月31日在TVB星河频道重播。描写新一代警官成长的过程:一个充满理想的大学生投身警界,后来成为卧底神探,伪装为毒贩头子的得力助手进行工作。经过长期的辛苦工作,他搜集到了贩毒头子的大量犯罪证据,终于“猎鹰”成功。","text2":"《猎鹰》描写了什么故事?","label":1} {"text1":"高雄展览馆站位于台湾高雄市苓雅区,为高雄捷运环状轻轨的捷运车站。车站代码为C8。站区位于新光码头公园内。设置二侧式月台。此站原计划站名「新光路站」,行政院已于2008年3月20日通过环状轻轨兴建案,2009年5月11日公告招商流标,2010年1月21日二次公告招商再度流标。之后高雄市政府决定采OT模式办理,并将环状轻轨与水岸轻轨两案整并,此站计划名称改名「新光公园站」。后于环状轻轨第一阶段兴建时,高雄市政府捷运工程局举行征名活动,活动第一阶段共有「市立图书馆站」、「高雄展览馆站」、「新光码头站」等三个建议站名,第二阶段则以「高雄展览馆站」、「市立图书馆总馆站」、「新光码头站」三名称进行票选,最终于2015年3月29日公告以「高雄展览馆站」为确定名称。于2016年6月26日启用。","text2":"高雄展览馆站正式启用的时间是?","label":1} {"text1":"雁门乡是位于中国四川省阿坝藏族羌族自治州汶川县的一个乡,素有汶川县“东大门”之称,是冉尤大道的重要关口。雁门乡因“负山临江,两岸壁立,中通一线只有鸿雁可以飞越”而得名。乡境东连茂县南新乡,南面与彭州市相连,西邻威州镇和西北的克枯乡。全乡幅员面积144.56平方公里,耕地面积5287亩,人均有地0.85亩,现有农户1305户,人口6197人,2002年全乡工业总产值817万元,人均纯收入702元。雁门乡共有9个行政村,21个村民小组。全乡羌、藏、回、汉等多种民族,其中羌族占97%,是羌民族的聚居地。雁门乡属岷江上游典型的干旱河谷地带,西北低走势,境内村寨最高海拔2080米,最低海拔1350米。雁门乡资源丰富,境内有多种野生动植物及矿产资源。岷江河穿境而过,雁门沟内水量丰富。","text2":"“雁门乡”得名的由来是什么?","label":1} {"text1":"李重进(),沧州(今河北省沧州市)人,生于太原,五代时后周禁军统帅之一,太祖郭威第四姊福庆长公主之子。李重进在后晋天福中,入仕为殿直。后汉初年,随舅舅郭威征讨于河中。后周建立的广顺元年(951年),太祖郭威以李重进为内殿直都知、领泗州刺史,女婿张永德为内殿直小底四班都知;又升李重进为小底都指挥使,而以张永德接任内殿直都知。翌年(952年),郭威以李重进为大内都点检兼马步都军头,张永德为小底第一军都指挥使;后又以李重进为殿前都指挥使,张永德为殿前都虞候,掌管殿前亲军。两年后,郭威病危,传位于发妻柴氏的侄子、养子柴荣,临终前特命李重进向柴荣行君臣之礼,以免其觊觎皇位。显德元年(954年),世宗柴荣即位,以姑表兄李重进为侍卫亲军马步军都虞侯,妹夫张永德接任殿前都指挥使,分掌侍卫亲军和殿前亲军。李重进、张永德本以姻亲之故,在数年间不次擢升,但后来都在战争中展现出过人的军事才能。在决定后周生死存亡的高平之战后,李重进以战功加使相衔,升侍卫亲军都指挥使,母福庆长公主追封燕国大长公主;而张永德以战功加检校太傅,授义成军节度使,妻寿安公主进封晋国长公主。北宋建隆元年(960年),宋太祖赵匡胤即位,加李重进为中书令,命令韩令坤代替李重进,将重进移镇至青州(治所在今山东省青州市),李重进拒绝调动,派遣幕僚翟守珣说服李筠起兵抗命,翟守珣却将此事泄露给宋太祖,于是太祖要求翟守珣拖延李重进出兵,以防止李重进与李筠南北呼应。翟守珣回去后,向重进诋毁李筠不足与谋事,重进果然中计,错失良机。李筠四月起兵反宋,六月兵败,自焚死。同年九月李重进起兵,十月,太祖亲征,带领石守信、王审琦、李处耘平叛,十一月,到达扬州城下,即日入城,李重进举家自焚。","text2":"为什么李重进能在数年间不次擢升?","label":1} {"text1":"巴黎第三区()是法国首都巴黎市的20个区之一。该区位于塞纳河右岸,面积1.171km²,是巴黎市第二小的区。该区包括中世纪风格的玛莱区北面较安静的部分(玛莱区较活跃的南部,包括巴黎的同性恋区,属于第四区)。巴黎保存至今最古老的私人房屋,建于1407年,位于第三区的蒙莫朗西路()。虽然小但是扩展迅速的中国城,居住着来自中国温州的移民,聚居在市长路()。附近是国立巴黎工艺技术学院(),坐落在中世纪Saint-Martin-des-Champs小修道院中。该区的人口峰值在1860年区划调整之前。1999年,该区人口为34,248人,工作岗位 29,723个。人口峰值出现于1860年区划调整以前。","text2":"巴黎第三区的中国移民聚居在何处?","label":1} {"text1":"银鲳(学名:)通称平鱼、白鲳、灰鲳、燕尾鲳,为鲳科鲳属的其中一种,坊间所谓的白鲳,即此种鱼,非圆白鲳。由于台湾人逢年过节时,习惯使用白鲳祭祖、围炉或宴客,造成白鲳价格居高不下,近年来金鲳或黑鲳也受到台湾消费者的青睐。还有鱼贩用中国的斗鲳来混充白鲳,鱼目混珠,大发其财。本鱼分布于印度西太平洋区,包括波斯湾、伊朗、巴基斯坦、印度、斯里兰卡、孟加拉、泰国、越南、马来西亚、缅甸、日本、中国、韩国、柬埔寨、菲律宾、印尼等海域。该物种的模式产地在广东虎门。由于苏伊士运河开通后出现雷赛布迁移,本种得以由红海经苏伊士运河而进入地中海。水深5至110公尺。本鱼体呈椭圆形而侧扁,鳞甚小且易剥离。口小,鳃孔狭小,无腹鳍、背鳍及臀鳍前方鳍条特长,呈镰刀状。背部呈淡墨青色,腹面呈银白色,各鳍略带黄色及淡墨色边缘,背鳍硬棘0枚;背鳍软条37至43枚;脊椎骨34至37个,体长可达60公分。本鱼为外洋性鱼类,栖息于海洋中层,6月左右为产卵期,此时会成群靠近沿岸群游中层海域,产卵后秋天再出外海,孵化后的幼鱼成长至3公分时,即向外海游出。为高级的食用鱼,适合各种烹饪方式食用。一般所谓的白鲳鱼,因为肉质细致又少刺受到消费者喜爱。台湾人重视此鱼类,列为十大美味鱼类之一,称一鯃、二红沙、三鲳、四马加、五𩾃、六嘉𫚭、七赤鯮、八马头、九乌喉、十寸子。每逢华人四大节日,台湾流行以白鲳祭拜神灵、祖先,故白鲳价格居高不下。例如2014年的春节假期,台湾白鲳的拍卖价都在每公斤新台币2300元,有时甚至高达2800元,春节结束后,价格跌五成。潮州人称「好鱼马鲛、鲳,好菜芥蓝样,好戏苏六娘」。","text2":"鱼贩用什么鱼来混充白鲳大发其财?","label":1} {"text1":"1960年美国总统选举,由民主党参议员约翰·肯尼迪以极微弱优势击败时任美国副总统理查德·尼克松成为总统。1960年,随著德怀特·艾森豪两任总统任期的即将结束。艾森豪的副总统理查德·尼克松成为共和党总统候选人,在电视辩论举行前,尼克森占尽优势。民主党提名马萨诸塞州参议员约翰·肯尼迪。他是参选的主要政党的第二位罗马天主教徒总统候选人(前一位是1928年的民主党人艾尔·史密斯)。在竞选的电视辩论中,肯尼迪指责在艾森豪和尼克松的共和党统治下,在冷战中,美国在军事上和经济上都落到了苏联后面,如果他当选总统,将“使美国再次运行起来”。尼克松作出回应说,如果他当选,将会继续艾森豪带给这个国家的“和平与繁荣”,国家正处于冷战之中,肯尼迪担当总统责任太年轻,缺少经验。最终肯尼迪在总统选举中以大约十万票(0.1%)的差距赢得选举。肯尼迪赢得303张选举人票,选举人票比1916年美国总统选举以来的任何一次总统选举都更接近,肯尼迪在普选中超出的票数也是美国历史上最接近的。而尼克森也成为首位赢得过半数州分,但输掉选举的总统候选人。一些历史学家对1960年选举是否有选举欺诈及舞弊提出争议,因为肯尼迪赢出的州中,很多州都不超过五个百份比,有数个州的选票十分接近,尼克森当时绝对可以提出要求重点选票,但他并没有这样做。只要重点选票后赢得其中两个州得克萨斯州和伊利诺州的选举人票,那么尼克森便成为总统了。这也是阿拉斯加州和夏威夷州首次参加总统选举,它们刚刚在前一年的1月3日和8月21日获得州的地位。这次选举中,夏威夷州支持民主党的肯尼迪,而阿拉斯加州支持共和党的尼克森,在此后的总统选举中大部分时间皆维持此格局,夏威夷州支持民主党,阿拉斯加州支持共和党。","text2":"尼克松是哪个党派的总统候选人?","label":1} {"text1":"文森特·孔帕尼(Vincent Kompany,),是一名比利时足球运动员,司职中后卫。孔帕尼身材高大而且身体素质出众,是「欧洲最有天份的球员」之一。孔帕尼现时为英超球队曼城与比利时国家队双料队长。孔帕尼防守能力出众且组织后防能力极强,曼城有无孔帕尼的后防线强度天差地远。但是非常容易受伤,是标准玻璃人,也因此大大影响孔帕尼的总体评价。2008年奥运会,他入选了比利时国奥队,2008年8月12日,汉堡主教练祖尔在球会网页发表声明,要求甘宾尼在奥运分组赛第二场赛事后回到汉堡。最后,甘宾尼和队友费莱尼在首场对巴西国奥队的赛事中,两人均两黄一红被逐离场,结果二人均在第二场小组赛停赛。甘宾尼其后依循汉堡的指示回到球会,准备在8月15日的德甲首战对拜仁慕尼黑,成功助球队以2-2迫和拜仁。2008年8月21日,汉堡官方网页宣布已与曼城达成协议,甘宾尼将转投该英超球队。","text2":"2008年8月21日,汉堡官方网页做出了什么决定?","label":1} {"text1":"怀仁堂,是中国北京市中南海内主要建筑之一,位于中海西门内,丰泽园东北。清朝光绪11年(1885年),慈禧命醇亲王奕𫍽在此处开始营造仪銮殿,费用主要由各地海关关税中提取,光绪14年(1888年)落成。此后,仪銮殿成为慈禧日常起居之所,由于她才是清朝实际统治者,故仪銮殿也因而取代养心殿,成为清朝实际权力中心。1900年,八国联军攻占燕京,仪銮殿被联军统帅,德国的瓦德西元帅占用,成为联军指挥部,但在1901年4月18日,该殿因为瓦德西所雇中国厨师用火不当,起火焚毁,导致一名德国人丧生。但也有指责此火为德国人所为者。1902年慈禧回都后,耗资500余万两白银对该殿加以重建,改名佛照楼。1908年,慈禧病逝于此。1911年,中华民国成立后,大总统袁世凯将此楼改名为怀仁堂,并在此接见外宾、接受元旦朝贺。袁世凯死后,灵堂设于此处。其后黎元洪、徐世昌也沿用怀仁堂。曹锟就任总统后,将怀仁堂改为眷属居住场所。北洋政府结束后,怀仁堂长期闲置,成为当时北平市政府举办集体婚礼的场所。1949年,中国人民政治协商会议第一届全体会议在怀仁堂召开。中华人民共和国成立后,怀仁堂成为中央政府的礼堂,经常举行各种政治会议和文艺晚会。1953年,为筹备亚太和平会议,周恩来亲自制定了翻修方案,将怀仁堂改建为中式二层楼阁样式礼堂。其后又多次进行翻修。怀仁堂正门位于南侧,进门后为前厅,北接礼堂,可供上千人开会。礼堂东西两侧各有一休息室。礼堂北有一门,进门为怀仁堂正厅,正厅北门通往后花园。","text2":"谁将仪銮殿改名为怀仁堂?","label":1} {"text1":"DVD播放程式(DVD Player)是Mac OS X的默认DVD播放器。它支持所有的DVD标准功能,比如多重音频、视频及字幕轨道以及杜比音效、DVD@ccess URL和关闭字幕。在某些情况下,用户可以选择其中的VOB文件并将其打开。这个文件的地址是「\/Applications\/DVD Player.app」。只有当有一个DVD驱动器连接到计算机时,DVD播放程式才会被安装到OS中(例如,当一部电脑的引导盘磁盘映象发现计算机上只有CD-ROM而没有DVD-ROM驱动器,将不会启动DVD播放程式的安装)。DVD播放程式也完全兼容DVD Studio Pro和iDVD所撰写的DVD,包括用DVD Studio Pro撰写的HD DVD。DVD播放程式顺从大部分著作权法,并因此将执行最严格的限制DVD措施,如区域编码限制和限制用户行为(「停用行为」)。它甚至会令苹果的屏幕截取程序失效,直到退出DVD播放程式程序。这有效的防止了用户从屏幕上捕捉DVD内容(但用户也能通过使用Terminal command间接得到图片。当然,其它的一些软件也能达到此目的,比如VLC media player。另外,DVD区域编码限制也能通过MacTheRipper这个软件去除)。","text2":"什么是DVD播放程式?","label":1} {"text1":"费明仪((英文:Barbara FEI Ming-yee,),天津人,祖籍江苏苏州。香港声乐家,曾赴法国深造声乐,1964年创立「明仪合唱团」,在音乐艺术界德高望重,是最早关注中国民族音乐的香港女高音,对中西音乐的修养颇深。父亲为名导演费穆,亦是《大公报》社长费彝民的姪女。费明仪曾任香港艺术发展局音乐组主席、明仪合唱团音乐总监及指挥、香港合唱团协会主席、香港大学亚洲研究中心名誉研究院士、乐府音乐委员会主席、香港民族音乐学会会长、香港音乐专科学校校董及校长、华人女作曲家协会名誉主席、香港康乐及文化事务署音乐顾问、香港中乐团资深顾问等。同时也担任北京中央音乐学院客座研究员、中国艺术研究院音乐研究所客座研究员、中国音乐学院客座教授、天津音乐学院声乐系教学顾问客座教授、吉林艺术学院客座教授、上海音乐学院名誉教授、廿世纪华人音乐经典艺术委员会委员和中华民族文化促进会的副主席等。中国歌剧方面,费明仪曾担任监制及女主角,在香港演出改编自山西民间故事《哑姑泉》之歌剧《甜姑》及国内创作喜歌剧《第一百个新娘》。费女士热衷推动音乐文化,分别于北京、上海、香港设立音乐奖学基金、赞助香港校际音乐比赛等。费女士积极致力香港与中国内地及国际性的音乐学术交流。","text2":"费明仪的父亲是做什么的?","label":1} {"text1":"瑞士地处欧洲几个主要文化的十字路口。这对其国家的语言和文化产生了极大的影响。瑞士有四种官方语言:德语、法语、义大利语 (邦联的三种官方语言) 和罗曼什语(格劳宾登州的官方语言)。很多瑞士人都会操一种以上的语言,英语在瑞士也相当普及。超过75%的瑞士人居住在中央平原。居住在瑞士的外国人和外国劳工占人口的20%。 瑞士人口的识字率几乎达到100%,大约25%的瑞士成年人有高等教育文凭。10.4个新生儿\/1,000人8.75人\/1,000人1.38人\/1,000人4.53 \/1,0001.46个新生儿\/1名妇女
1.33个新生儿\/1名瑞士籍妇女
1.86个新生儿\/1名非瑞士籍妇女
","text2":"对瑞士的语言和文化产生了极大的影响的重要因素是什么?","label":1} {"text1":"格伦·科恩(,),美国乒乓球运动员。他不仅球技好,更重要的是在乒乓外交事件中促进中美冰封二十年的外交关系重新解冻,并成为首批到访中华人民共和国的美国人,他也是美方的代表。1971年日本名古屋举行第31届世界乒乓球锦标赛。周恩来召集多名中国球手,要他们接触多个国家的球手,并希望他们可以邀请他们到中国去。而在第一天,当中国乒乓球队乘巴士时,美国运动员科恩上来乘车。这时,中国运动员庄则栋主动和他握手和谈天数句,并送他一块中国杭州的织锦给他。这个细节立刻成为轰动的新闻。1971年4月6日,中共领导人毛泽东在比赛闭幕前夕决定邀请美国队访华。4月7日,美国国务院向白宫报告。美国总统尼克森得知这个消息后,立即同意中国的邀请。1971年4月10日,美国乒乓球代表团和科恩抵达北京,成为自1949年以来第一批进入中华人民共和国的美国人。4月14日,中国乒乓球队到美国访问。乒乓外交结束了中美两国22年来人员交往隔绝的局面,使中美和解随即取得历史性的突破。2004年科恩因心脏搭桥手术失败病逝,享年52岁。","text2":"格伦·科恩是什么运动员?","label":1} {"text1":"书香文化节为澳门大型书展活动之一。由澳门出版协会主办,从2005年起,每年分春、秋两季举办两次书展。在2005年以前,澳门主要有三间书店每年举办大型书展,包括星光书店的阅读文化节书展、澳门文化广场的两岸四地(澳门)书展及一书斋的书市嘉年华,当时阅读文化节书展及两岸四地(澳门)书展皆不定期、举办地点亦各有不同,只能靠宣传才能知道在甚么时候举办。直到现在的主办单位澳门出版协会成立以后,才由2005年开始定期举办「书香文化节──两岸四地(澳门)书展2005」,每年书展始分为春、秋两季进行。春季书展于每年4月举行,由星光书店统筹承办,于澳门理工学院体育馆举行;而秋季书展则于每年10月举行(早年由于展场问题延于12月举行),由澳门文化广场统筹承办,于澳门塔石体育馆举行。2016年秋季开始,变更名称由「书香文化节──两岸四地(澳门)书展2016」改为「书香文化节──2016澳门书展」,以后沿用新名称。书香文化节为澳门春、秋两季澳门的大型书展。因澳门的书展并非由政府全资主办,而是由个别非牟利组织申请支助或个别书店主办,而且澳门的出版社数量也较少,因此即使是澳门的大型书展,规模上与周边地区的书展相距甚远。展出的书籍则是集合本土及周边各地区的中、英、葡文书籍,因此两岸四地同时会有出版商参展,尤以香港出版商居多,为周边地区的出版商之间提供了一个业务交流的机会。不少澳门出版书籍都会选择在书展时首发,主办单位有时还会邀请这些首发书籍的作者出席签名会、亦会邀请两岸四地知名作家及知名人士出席签名会。由于是免费入场,而且购书亦可获优惠折扣(一般是八折或九折)或特价优惠,因此每年都会吸引大批书迷到场选购书籍。据主办单位表示,过往书展约有四至五万人次参与。弘扬中华文化;推动本澳阅读风气,鼓励持续进修、终身学习,从而达至提升本澳居民的文化素质;促进两岸四地图书出版事业的交流互动。书香文化节主要由主办方面提出各项主题活动,吸引各界文化爱好者到场参观及交流。如有:每届书展均会请来新书首发人士发表作品相关的介绍及写作分享,以及邀请文化界及知名人士与读者对话。","text2":"书香文化节的春季书展和秋季书展有什么不同?","label":1} {"text1":"汪嵩(),中国足球运动员,司职前锋,现效力于江苏苏宁。在2007年中国足球甲级联赛中,汪嵩为成都谢菲联攻入17粒入球,在射手榜上排列第二位,帮助球队冲超成功。凭借自己出色的表现,汪嵩入选了中国国家足球队,这也使得他成为成都谢菲联队历史上第一名入选国家队的队员。2008年1月12日,中国国家队在迪拜与汉堡足球俱乐部进行比赛时,汪嵩登场亮相24分钟,这是他第一次为国家队出场。1月20日中国国家队与黎巴嫩进行比赛,汪嵩登场20分钟。不过,在这之后的世界杯预选赛中他却未能获得出场机会。2010年2月9日,汪嵩以接近300万元的身价加盟杭州绿城。2014年12月10日,杭州绿城通过官方微博宣布不再与汪嵩续约。2014年12月15日,广州富力宣布汪嵩自由转会加盟球队。2017年2月28日,江苏苏宁宣布汪嵩转会加盟球队,签约三年。","text2":"汪嵩是谁?","label":1} {"text1":"望安岛是澎湖群岛第四大岛屿,位于澎湖县南方的海域,离澎湖本岛距离约有18海浬之远,而整个面积大约有6.7413平方公里,望安地势较平坦,最高高度54公尺,海岸弯曲多,岛上附近海域大多为暗礁。望安地理位置约在东经119度18–59分至北纬23度12–23分之间,附近还有40个大小岛屿及礁塭。从天台山上可以居高临下地眺望望安岛,山上有一著名的仙迹岩,其实是一块状似人类脚印的玄武岩。传说中仙迹岩是当年八仙过海时吕洞宾突然喊起肚子痛,可又临时找不到可供如厕的场所,只好一脚踏在望安的天台山上另一脚踩在花屿东边的海崖上,就开始方便起来了,而那些落下的黄金就化为澎湖诸岛中的一些小岛了。中社古厝原名为花宅,建筑风格仍保有清末民初的特色,吸引各地画家、摄影家及建筑科系的教授与学生来此勘查研究。因此,中社古厝巡礼至今也成为观光客必到景点。每年总会有绿蠵龟到回游至望安岛上产卵,为了保护这些稀客,岛上特地规划了多处绿蠵龟产卵栖地保护区,也砸下巨资设立了绿蠵龟保育中心。鸳鸯窟为日治时期,日军的重要军事站,当时被计划为快艇基地,后来因为日军战败而打消念头,至今仍留著当时所挖掘的两个巨大洞窟。其地区海流缓和,海水清澈,政府在那兴建了凉亭、公厕,也规划了露营区。早期的望安岛原称为八罩岛,因古时郑成功抵达七美屿,在高处望见了八罩岛,又因其海域清澈且平静,望之而心安,当下易名为望安,别名又称为网埯、网埯澳。在近年来被观光游客称为『蜜月岛』。居民的维生大都靠渔业,农业为辅。近年来海上渔业资源日渐枯竭,导致人口往外发展情况越来越严重。望安岛盛产文石、珊瑚、九孔、海胆、紫菜、海苔、小管干等物产。往来望安的交通,有民营的交通船光正陆号(客货两用)及光正参号、光正捌号(载客快艇)。平均每天皆有往返马公的船班。澎湖车船管理处有南海之星、恒安一号交通船,往返马公、七美途中有停经望安。空中交通,有德安航空以小型飞机于高雄-望安间来回直航。","text2":"为什么近年来岛内居民外流?","label":1} {"text1":"家铉翁(1213年-?)号则堂,眉山(今属四川)人。先祖自唐末迁居入蜀,是眉州望族。宋理宗时以荫补官,官常州(今江苏武进县)知县,政誉翕然,迁浙东提点刑狱,入京担任大理少卿。咸淳八年(1272年),权知绍兴府、浙东安抚提举司事。德祐二年(1276年),赐进士出身。是年二月,元军攻临安,丞相吴坚、贾余庆发文命各地献城投降。铉翁独不降。后奉诏出使元朝,被强留馆中。闻知宋亡,日夕哭泣,不仕,自号则堂,被羁縻于河间(今属河北省),以《春秋》教授弟子,前后长达十七年之久。元成宗即位(1294年),赐衣服并放还,年纪已八十多岁。卒于家。著有《说易》、《春秋序例》、《孝经解义》等书皆失佚。另有《则堂集》二十卷,今存《永乐大典》本六卷。","text2":"家铉翁先祖什么时候迁居四川的?","label":1} {"text1":"克罗地亚航空(Croatia Airlines d.d.),是克罗地亚的国家航空公司,总部设在萨格勒布。该公司以萨格勒布机场为枢纽,经营国内及欧洲航线,重点城市为杜布罗夫尼克和斯普利特,是星空联盟的区域成员。2008年克罗地亚航空录得3800万库纳的纯利,载客量为187万人次。克罗地亚航空于1989年7月20日以萨格勒布航空之名成立,用一架塞斯纳C-402替UPS运输货物。在克罗地亚的首次民主选举后,萨格勒布航空于1990年7月23日更名为克罗地亚航空。1991年克罗地亚航空与亚德里亚航空达成协议,租用一架MD-82执行国内航线萨格勒布至斯普利特。由于与军事空域的冲突和空域关闭,很快地克罗地亚航空被迫停止营运。复航后,克罗地亚从汉莎航空取得3架波音737及成为国际航空运输协会的成员,同时亦开始了首条国际航线往法兰克福。1993年,取得2架波音737和ATR 42,并在不同的欧洲城市设立办事处,同时航空公司亦买下旅行社Obzor。1994年,克罗地亚航空接送了第100万名乘客。不久,教宗若望·保禄二世乘拾克罗地亚航空前往克罗地亚。1995年接收了另一架ATR 42和运载第200万名乘客。1996年波斯尼亚战争后,克罗地亚航空是首架恢复塞拉耶佛航线的公司。1997年接收了首架A320并命名为Rijeka,1998年接收了首架A3190并命名为Zadar。同年克罗地亚航空加入了欧洲航空公司协会。1999年接收了2架空中巴士飞机,并开始卖掉老旧的波音737,同年克罗地亚航空接送了第500万名乘客。2000年克罗地亚航空再接收2架空中巴士飞机,并开始自动售票系统。2001年取得了从德国联邦航空局发出的维修及技术保养认证。于2004年11月18日加入了星空联盟。2009年7月克罗地亚航空接受了第2000万名乘客 ,而自2000年开始年载客量超过100万人。克罗地亚航空有以下机型:至2016年4月,克罗地亚航空航空的平均年龄为11.7年。2008年10月22日,空中巴士公司与克罗地亚航空宣布增购4架132座的A319,于2012年交付。","text2":"克罗地亚航空重点城市是哪里?","label":1} {"text1":"东莞县是中国广东省珠江三角洲东岸昔日的一个县。始于东晋咸和六年(331年)的宝安县,至唐朝至德二年(757年)更名东莞县。县治由当时的南头迁至涌,即今日的莞城。在唐朝及宋朝大部份时间内,东莞县所辖领地含今增城市、东莞市、深圳市、中山市、珠海市、澳门特别行政区及香港特别行政区,至南宋绍兴三十一年(1161年),将珠江口对侧新出水土地划香山县止。明朝万历元年(1573年),东莞县的南部被分拆为新安县,范围包括今天的香港及深圳。及至1985年,东莞县经国务院批准列为珠江三角洲经济开发区,同年9月撤县建市,即今天的东莞市。东莞相传是东官的音转,亦有人说因地处广州东面及盛产水草(莞草)而得名,或曰昔日有侨东莞郡人居此,以为相似于其乡,故名。","text2":"东莞什么时候撤县建市的?","label":1} {"text1":"德瑞莎·麦德罗(英语:Teresa Medeiros)是一位1960年代出生,美国女性罗曼史作家。目前出版23部作品,多次荣登纽约时报畅销书榜。作品题材多为历史罗曼史,亦有数本穿越时空、吸血鬼题材,等超自然罗曼史作品。德瑞莎·麦德罗出生于1960年代美国,处女作于21岁时出版。在成为知名小说家之前,职业是一名护士。截至2015年为止,已出版23 部作品,翻译至十七种语言,总印量超过一千万册。目前与她的丈夫和两只猫,住在美国肯塔基州。麦德罗作品多次荣登纽约时报畅销书榜,赢得PRISM奖两次,亦获得两次Waldenbooks Award,同时也是Romance Writers of America创始会员,亦为肯特州罗曼史写作家和小说家协会会员之一。作品题材多为历史罗曼史。有少数系列作品为超自然罗曼史,例如:有穿越时空剧情的女巫系列或吸血鬼题材的The Lord of Midnight Series等。作者亦有现代罗曼史、西部罗曼史作品各一,可谓多方尝试写作方向。德瑞莎·麦德罗在台湾的中文译作大多由林白\/果树出版社出版代理,早期作品出版代理较多。近期作品目前无台湾代理。最新一本中文译作,亦是唯一本由高宝集团代理的作品,为2004出版之《一夜绯闻》(One Night of Scandal)。该出版社将作者名称误值为Terest Medeiros,译名为泰瑞莎‧梅戴洛斯。","text2":"德瑞莎·麦德罗是谁?","label":1} {"text1":"《ADULT VIDEO》(,\"ADULT VIDEO\")是东京事变的第四张DVD,跟《ADULT VIDEO Original Sound Track》,一起于2006年3月23日发行。发行当周即卖出1.6万张,总计销售额3.2万张,名列2006年年度销售榜第52位。初回限定为「Digipak纸盒仕样」。本音乐录影带是东京事变第二张映像集,共收录六首音乐录影带,包括两首来自单曲《修罗场》内的曲目,四首来自第二张专辑《大人》的曲目。首先是取材自大人的团员介绍作品「歌舞伎」,然后是改编专辑版本的「秘密 FOR DJ」,再来是第三张单曲,同样是改编版本的「恋爱幻梦 FOR MUSICIAN」,与以白色为基调的「修罗场」,然后是鼓手刄田缀色大秀舞技的喧哗上等,最后则是改编专辑版本的「黄昏泣 FOR MOTHER」做为结尾,总计六首。早于3月6日起,即在MTV JAPAN拨出「秘密 FOR DJ」音乐录影带,同一天,「秘密 FOR DJ」也开始接受歌迷线上下载。等到3月20日,官方网站特地开设「特设网站」。日本则于3月23日发行DVD,台湾则是等到4月4日才发行日版DVD才发行进口盘。《ADULT VIDEO Original Sound Track》(,\"\"ADULT VIDEO\" Original Sound Track\")是东京事变的第二张黑胶唱片,跟《ADULT VIDEO》DVD,一起于2006年3月23日发行。总计销售额1,065张。这张黑胶唱片为东京事变第二张黑胶唱片,主要是将《ADULT VIDEO》收录的曲目音源化的一张黑胶唱片,因此也可以说这是《ADULT VIDEO》的原声带,采完全限定生产。早于3月6日起,「秘密 FOR DJ」开始接受歌迷线上下载。日本则于3月23日发行黑胶唱片,台湾则是完全没有进口。","text2":"本张唱片共收录了几首音乐录影带?","label":1} {"text1":"小行星8991,即团结小行星(8991 Solidarity),是太阳系的小行星之一。其公转轨道位于火星和木星之间。小行星8991于1980年8月6日由欧洲南方天文台(ESO)下属的拉西拉天文台(La Silla)发现。2001年10月9日,国际天文联合会(IAU)下属的小天体命名委员会(Committee on Small Body Nomenclature)发布公告,将小行星8990、小行星8991和小行星8992分别命名为同情(Compassion)、团结(Solidarity)和宽容(Magnanimity),以悼念2001年9月11日在美国纽约和华盛顿发生的九一一恐怖袭击事件中的罹难者和受害者。小行星8991被命名为团结(Solidarity)的原因是希望世界人民与九一一恐怖袭击中的罹难者和幸存者团结起来,以将恐怖主义从这个世界上消灭为目标。","text2":"小行星8990、小行星8991和小行星899分别被命名为什么?","label":1} {"text1":"台中工业区(Taichung Industrial Park),为台湾中部的一个大型工业区,隶属经济部工业局管理,位于台中市西郊大肚台地东侧,总面积580公顷,于1980年开发完成,北临东海大学与台湾大道,南邻台中市精密机械科技创新园区。工业区内目前计有921家工厂,从业员工数约40,000人,年产值约新台币3,299亿元。新进驻厂商大都属高科技产业,如:光电、电子与精密机械…等等,已逐渐从传统行业中转向高科技工业迈进。劳动部劳动力发展署中彰投分署与台中世贸中心亦设立于此。东距国道一号台中交流道与南屯交流道约1.5公里以及纵贯铁路台中车站9公里、西距国道三号龙井交流道3公里以及台中港15公里、南距高铁台中站5.5公里、北距台中国际机场8公里,交通非常便利。台中工业区周边往台中市区与国道的联外道路台湾大道、五权西路,一到通勤时间就会严重塞车。近年来大肚台地东侧陆续设立中部科学园区台中基地、台中精密机械园区,使得原本已严重塞车的周边道路(主要为台湾大道)雪上加霜,使得中央与地方政府加速开发交通建设已纾解此区的交通问题,陆续兴建南屯交流道、龙井交流道以及陆续完成跨越筏子溪的朝马路、福科路、中科路与特三号道路大型交通建设,并著手规划市政路的延伸案。","text2":"台中工业区位于哪儿?","label":1} {"text1":"基隆拿国际机场(,)位于加拿大不列颠哥伦比亚省基隆拿市,为一单跑道机场,并提供前往温哥华、西雅图、维多利亚、卡加里、爱民顿、乔治王子城和多伦多的定期航班。季节性航点还包括坎昆、巴亚尔塔港、洛斯卡沃斯和拉斯维加斯。基隆拿国际机场的2014年总旅客量达1,602,899人次,为加拿大国内最繁忙机场之一。基隆拿市选民于1946年以466票对460票通过市政府方案,授权当局以2万加元购入艾利逊湖()以南约320英亩土地来兴建机场。艾利逊机场()于1947年开幕,当时草坪跑道全长。联邦交通部于1950年代出资让跑道改以碎石铺设,而随著加拿大太平洋航空于1958年开展每日以道格拉斯DC-3型号飞机来回温哥华的航班,市政府遂向联邦政府寻求拨款进一步改善跑道。跑道于1960年延长至英尺并改以沥青铺设。当局于1960年代在跑道南端一带增建新客运大楼,而新航空管制塔则于1970年代初落成。机场客运量于1997年突破80万人次。为了应付日益增长的航空交通,市政府从1998年起斥资2千万加元扩建机场设施,当中客运大楼楼面面积倍增至,机场可应付的客流量遂从每小时150人次增至450人次。","text2":"哪一年跑道延长至英尺并改以沥青铺设?","label":1} {"text1":"马里亚纳群岛及帛琉战事是在太平洋战争期间美国军队在1944年6月至11月攻击在马里亚纳群岛及帛琉的日本帝国军队,美军的攻势在切斯特·威廉·尼米兹的指挥下,紧跟随吉尔伯特及马绍尔群岛战事而展开,目的是攻占日军在中太平洋的军事基地,以支援同盟国重返菲律宾群岛,及提供基地对日本本土实施战略轰炸行动。在攻势开始时,美国海军陆战队和美国陆军在美国海军的支援下,于1944年6月在塞班岛实施登陆,作为回应,日本帝国海军联合舰队对支援登陆的美国海军舰队实施打击,在6月19日–6月20日的菲律宾海海战航空母舰战役中,日本海军被打败,舰载机及岸基飞机遭到惨重损失。之后,美军于1944年7月分别在关岛及天宁岛实施登陆,经过激烈的战斗后,美军在7月攻占塞班岛及在8月占领关岛和天宁岛,美军在塞班岛及天宁岛修建机场,至第二次世界大战结束前作为B-29超级堡垒轰炸机对日本本土实施战略轰炸的基地,包括原子弹轰炸广岛及长崎。与此同时,为了保障美军攻击在菲律宾日军时的侧翼安全,1944年9月美国海军陆战队和美国陆军在帛琉的贝里琉及安加尔实施登陆,在贝里琉经过激烈的战斗后,该岛在1944年11月终于被美军占领。经过了在马里亚纳群岛及帛琉的登陆战役后,盟军继而成功地在1944年10月于菲律宾群岛和1945年1月在硫磺岛及硫球群岛实施登陆","text2":"美军分别在几月攻占塞班岛及关岛和天宁岛?","label":1} {"text1":"林德曼-魏尔斯特拉斯定理()是一个可以用于证明实数的超越性的定理。它表明,如果  是代数数,在有理数  内是线性独立的,那么formula_1在  内是代数独立的;也就是说,扩张域formula_2在  内具有超越次数 。一个等价的表述是:如果  是不同的代数数,那么指数  在代数数范围内是线性独立的。这个定理由林德曼和魏尔斯特拉斯命名。林德曼在1882年证明了对于任何非零的代数数α,\"e\"都是超越数,因此推出了圆周率是超越数。魏尔斯特拉斯在1885年证明了一个更一般的结果。这个定理,以及格尔丰德-施奈德定理,可以推广为Schanuel猜想。\"e\"和π的超越性是这个定理的直接推论。假设α是一个非零的代数数,那么{α}在有理数范围内是线性独立的集合,因此根据定理的第一种表述,{\"e\"}是一个代数独立的集合,也就是说,\"e\"是超越数。特别地,\"e\" = 1(参见欧拉公式)也是超越数,这与1是代数数的事实矛盾。把这个证明稍微改变以下,可以证明如果α是一个非零的代数数,那么sin(α)、cos(α)、tan(α)和它们的双曲函数也是超越数。\"p\"进数林德曼-魏尔斯特拉斯猜想,就是这个定理在p进数中也成立:假设\"p\"是素数,α...,α是\"p\"进数,它们都是代数数,且在Q内线性独立,使得对于所有的\"i\",都有formula_3。那么p进指数formula_4在Q内是代数独立的。","text2":"林德曼-魏尔斯特拉斯定理表明了什么?","label":1} {"text1":"向象贤(;)即羽地按司朝秀()、羽地王子朝秀(),琉球国政治家。字文英,号通外。他是琉球第二尚氏的王族,原姓「吴」(),名乘「重家」。直到他死后的1691年,琉球王府将王族的分支统一改姓为「向」,名乘改为「朝」字辈,象贤的名乘才被改为「朝秀」。初为按司,后升王子。1666年至1673年任琉球国摄政。亲萨派人物,是「日琉同祖论」的提出者。1617年出生于琉球王族羽地御殿。根据伊波普猷和真境名安兴共著的《琉球五伟人》一书的说法,向象贤自幼就有非凡的才能,得到王子尚亨(具志川王子朝盈)的赞赏,试图将其培养为自己的继任者。1640年继任羽地御殿家督,领有羽地间切之地。后赴萨摩藩留学。1650年奉命修撰《中山世鉴》,这是琉球历史上最古老的一部史书。1658年,以年头使的身份前往萨摩,同年归国。1664年,庆贺康熙帝即位的进贡船在福州遇风搁浅,赏赐物品被盗。1666年,萨摩藩藩主岛津光久得知此事,十分愤怒,胁迫摄政尚亨辞职,以向象贤代之;并将进贡的王舅向国用(北谷亲方朝畅,时任法司)、英常春(惠祖亲方重孝)投入狱中。次年,萨摩藩将向国用、英常春二人处决,引起琉球政坛的混乱,史称「北谷惠祖事件」。为了巩固在琉球的统治,萨摩使亲萨的毛国栋(具志头亲方安统)续任法司,并支持向象贤进行一系列的改革。此后,向象贤奉行「财政再建,政教分离」的政策,对琉球进行改革。他废止了国王对久高岛的祭祀和巡礼,销弱了琉球传统信仰的影响。同时他要求琉球官员们都要学习茶道、花道等日本诸艺。他的改革在一定程度上打击了宗教势力,增加了财政收入,但对杣山和田地过度的开发也为后来琉球的资源短缺埋下了伏笔。1673年辞官退隐,两年后死去。他被以国葬级的礼仪下葬,尚贞王亲自出席了他的葬礼。冲绳民俗学家伊波普猷认为向象贤的改革使琉球得以复兴,因此积极的评价此次改革,并将向象贤当作琉球的伟人看待。但是中华民国历史学家陶元珍认为向象贤在编撰《中山世鉴》中,「伪造出日人源为朝乃舜天王之父亲的谬说」、「实极可恨」;其改革最终导致琉球文化被日本逐渐同化,所以向象贤「是个琉奸」。","text2":"向象贤的家庭背景如何?","label":1} {"text1":"文理中学()部份地区也称,是中等学校教育体系中的一种为进入大学准备的学校。何时进入这种学校,以及这种学校的学习时长,依不同的教育体制而有所区别。在德语中,文理中学的学生被叫做。“文理中学”一词为古希腊语(拉丁转写:)的拉丁语化形式。在古希腊,体育馆()是一个给青年男子是训练公共竞赛中的竞赛者的设施,也是进行社会化和从事智力辩论的地方。在体育馆中,所有人须裸体训练,而体育馆()的词源 (拉丁转写:)亦有裸体之意。Lyzeum这一名字也来自于古希腊语,并可以追溯到亚里士多德开创的学院。这一名字之后被引申为高等教育的标志,在奥地利与南德较为常用。文理中学在德语国家的发展:在现代,最初的授课始于中世纪的修道院学校与市立学校。当时的课程设置基本上都是宗教性的,主要是为了培养神职人员。16世纪开始的宗教改革使得这些在新教地区的学校逐渐转变成拉丁语学校,而这些的管理责任则转移到了当地的侯爵与市政官员身上。这时学校主要还是以教授拉丁语为主,逐渐加入希腊语作为圣经教材。很多文理中学与更高一级别学校有直接联系,可以直接升学。直到18世纪文理中学才开始教授德语和其他现代外语(主要是法语)以及自然科学学科。这期间学校的教学计划经历了多次改革。直到19世纪末,文理中学在德国才对女性开放,同一时期自然科学与现代外语的课程也正式被列入教学计划。自1900年起,德国的三种中学(humanistische Gymnasium、Realgymnasium、Oberrealschule)的学位被认为同等级。第二次世界大战前后,文理中学的常规教育时间被缩短,学生必须在12年级结束后毕业。在德国分裂以后,西德实行了9年的文理中学制度,而东德则改制为4年Oberschule。文理中学分为不同类别。如:除这几种常见的文理中学外,还有:不同国家的教学系统不同,所以文理中学在许多国家的定位也有或多或少的差异。拥有文理中学的国家有德国、奥地利、瑞士、卢森堡、意大利、列支敦士登、立陶宛、法国、荷兰、波兰、斯洛文尼亚、捷克、斯洛伐克、希腊、塞浦路斯、土耳其、英国、美国、加拿大等。本页面主要介绍的是德国的文理中学。","text2":"文理中学到什么时候才开始教授其他外语和自然学科?","label":1} {"text1":"金刚(King Kong)是电影中的虚构怪兽,一种巨大的猩猩,最早出现在1933年的电影《金刚》之中,之后1976年及2005年各有一部同名的重拍电影,日本和香港邵氏的电影公司也拍过相关题材的作品。目前「金刚」的角色使用权在环球影业。「金刚」最早由美国电影制片创造,在1933年的原作中,这个生物的名字其实是「刚」(Kong),是骷髅岛上原住民对牠的称呼,「金」(King,王者、国王之意)是片中人物将牠捕获、并带到纽约展示时加上的头衔,然而Kong就是丹麦语的国王之意,与King相同字源。<\/nowiki>身长:30m体重:未知登场电影:2017年《金刚:骷髅岛》\/2020年《金刚大战哥吉拉》","text2":"在1933年原作中,”金“是什么意思?","label":1} {"text1":"全球海平面观测系统(Global Sea Level Observing System, GLOSS)是一个政府间海洋学委员会的计划,其目的是测量海平面全球长期气候变化的研究。自2004年印度洋地震后,该计划的目的已经改变为收集海平面的实时数据。该项目目前正在升级超过290台观测站,以使他们能够通过卫星向新设立的国家海啸中心传送实时数据。他们还在组装太阳能电池板,令观测站即使在恶劣天气下仍能够继续运作。全球海平面观测系统与另一计划深海评估和报告海啸(Deep-ocean Assessment and Reporting of Tsunamis, DART)两者并非相互竞争,因为大部份GLOSS传感器都设于近岸地方,而DART传感器则设于深海地方。","text2":"为什么全球海平面观测系统要组装太阳能电池板?","label":1} {"text1":"丹北镇是江苏省丹阳市老北门外的一中心镇,2014年由原来的埤城镇、后巷镇、新桥镇合并成立。埤城位于江苏省丹阳市老北门外一中心镇。全镇面积73.5平方公里,人口3.8万余人。北部紧邻镇江大港新区,东接后巷镇,南邻云阳镇,西接丹阳市经济技术开发区。境内的水晶山,范围约4平方公里,主峰海拔166.1米,是全市最高峰。泰山水库为丹阳蓄水量最大的水库,水质极好。埤城原名卑林,三国时为东吴孙权养马之地,明嘉靖年间,此地曾设营寨、筑长城,以防倭寇,因此得名“埤城”。境内有南朝齐代齐宣帝萧承之永安陵、齐高帝萧道成泰安陵、齐景帝萧道生修安陵和齐废帝鬰林王萧昭业陵4座,今存石刻6尊,其中天禄、麒麟各2尊、辟邪2尊,均为全国重点文物保护单位。S338纵向、通江大道和丹界公路横向贯穿镇区,宽阔的硬质路面通向每个自然村。与丹阳市、镇江市有公交化便利交通。紧邻沪宁高速公路,最近处距离丹阳入口处仅2千米。泰州长江大桥和规划中的五峰山过江通道建成以后,丹北将成为沟通长江南北、江苏东西的交通枢纽。埤城工农业都很发达,2010年全镇实现GDP28亿元。主要工业企业有沃得集团、晶谷集团、华宇等企业集团,其中沃得精机已在新加坡上市。","text2":"丹北镇的面积有多大?","label":1} {"text1":"麦可·派翠克·金(,),美国电视节目导演、剧作家和制作人,曾赢得艾美奖。其知名的作品为HBO影集《欲望城市》,第二季后的结尾和开头都是他撰写的。金最近也担任了同名电影的导演。他也替HBO另一影集《喜开二度》(\"The Comeback\")撰写剧本,还有其他影集,如《威尔与格蕾丝》、《Cybill》和《风云女郎》(\"Murphy Brown\")。1980年代,金搬到了纽约,制作栋笃笑(stand-up comedy),并写剧本。最后他搬到了洛杉矶,写了《风云女郎》的剧本,因此获得艾美奖的提名。他是一位出柜的同性恋者,拥有自己的制作公司(Arcade Productions)。","text2":"金凭借哪部作品获得了艾美奖?","label":1} {"text1":"姚翰林(),中国足球运动员,司职中场。目前效力于武汉卓尔足球俱乐部。职业生涯始于前武汉光谷1985年龄段梯队。2001年被俱乐部选派到英国斯托克港足球俱乐部留学。因表现良好,对方曾经发出将他引进的邀约;不过最终因故没有成行。2004年进入武汉光谷一线队开始参加中甲联赛并夺得该赛季冠军升入中超联赛。2005至2008四年间作为替补球员在中超比赛中一共出场37次。2008赛季,武汉光谷发生退赛事件并被足协取消俱乐部参赛资格。姚翰林于2009赛季以租借形式加盟武汉队前任主帅裴恩才带领的中超球队江苏舜天,并在该赛季作为重要成员联赛出场28次。2010年租借期满回归刚刚升入中甲联赛的湖北东方国旅。","text2":"姚翰林目前效力于哪儿?","label":1} {"text1":"闵浦大桥,是位于中国上海黄浦江上的一座公路双层斜拉桥,是跨越黄浦江的第八座大桥,上层S32公路段于2009年12月31日建成通车,下层(普通公路层)则于2010年4月20日通车。建成时为世界上跨径最大的双塔双索面双层公路斜拉桥。闵浦大桥西岸位于闵行区吴泾镇放鹤路,东岸位于闵行区浦江镇沈杜公路,大桥位于徐浦大桥和奉浦大桥之间,全长约3.61公里。主桥全长1212米,主跨长708米。通航能力为2万吨级。于2005年9月1日开工建设,2009年7月19日开始合龙,2009年7月22日大桥主桥提前合龙。闵浦大桥上层设双向八车道及紧急停车道,是S32申嘉湖高速公路的跨黄浦江设施,下层为普通公路,设双向六车道。","text2":"闵浦大桥在建成时有何重要性?","label":1} {"text1":"两女一杯(2 Girls 1 Cup)是巴西色情片《饥饿婊子》(\"Hungry Bitches\")的预告片,由MFX Media发行。预告片中包含两嗜粪癖女子相互爱抚,吞吃对方的排遗物,然后呕吐到对方的嘴里的情节。该片在网路通过病毒传播,引起巨大反响,成为网路迷因,在各大网志和留言板上传播。2007年10月中旬左右,YouTube上的反应和声讨达到顶峰。尽管2008年后方才逐渐被禁,该影片仍有传播的余地,甚至部分人特地为此开设站点。2013年1月16日,美国法院以涉嫌拍摄吃屎、人兽交和恋物癖等淫秽影片的罪名,将导演萨克斯判刑4年,全案仍可上诉,其辩护律师戴蒙(Roger Diamond)表示:「他的客户作品明显有别于主流A片,应受美国宪法第一修正案保护。」","text2":"美国法院认为该片导演涉嫌哪些罪名?","label":1} {"text1":"运动系统是动物体用来进行诸如移动,抓取,进食,眼动以及言语等骨骼肌运动的器官组成的功能整体。运动系统的主要部分包括位于外周的骨骼肌和神经以及位于中枢的脑和脊髓。脑和脊髓支配所有骨骼肌的运动。从脊髓的前角投射到肌肉的运动神经元与骨骼肌形成神经肌肉接头,通过动作电位和乙酰胆碱的突触来使肌肉发生收缩。肌肉一般通过肌腱与骨骼相连。肌肉的收缩导致骨骼之间角度的变化,从而使得身体的结构发生形状变化。骨骼肌内含有两种感受器,称为高尔基腱器官和肌梭。两种器官通过感觉纤维(Ia, Ib和II型纤维)与脊髓相连,提供关于肌肉的张力,长度和收缩速度的信息。这些感觉纤维在脊髓内形成神经元回路,来支配肌肉的反射,例如伸张反射。此类低级反射的主要作用是保证肢体姿势的稳定性。脑是较脊髓更高一级的运动中枢。支配脊髓运动神经元的输入来做多个脑内的结构。包括脑干内的下行网状结构,小脑深部核团,中脑的红核,以及大脑皮质的运动区。除了这些结构,基底核也与运动功能有密切的关系。从大脑皮质的运动去直接投射到脊髓运动神经元的通路称为皮层脊髓束。这种非常简单直接的通路与灵长类对手指的精确控制有关。除了支配肢体运动,眼球的运动也是运动系统的重要一部分,与动物的视觉感知,注意力和抓取等功能密切相关。中脑的上丘内含有眼动神经元,直接支配控制眼球运动的肌肉。常见的运动系统疾病包括瘫痪,运动失调症,帕金森氏病,亨廷顿氏病等","text2":"为什么脑是较脊髓更高一级的运动中枢?","label":1} {"text1":"安东尼·刘易斯·约翰·卢彻涅特(Antoine Louis John Ruchonnet,)是一位瑞士政治家,瑞士联邦委员会委员(1881年-1893年)。他于1875年12月10日被首次选为联邦委员会委员,但是他回拒了选举结果,没有履职。1881年3月3日他被再次选为委员,并接受这一任命,任职至1893年9月14日在任内去世。刘易斯·卢彻涅特是瑞士自由民主党成员。在任期内,他主要主持领导了以下部门的工作:刘易斯·卢彻涅特于1883年和1890年两度出任瑞士联邦总统。在洛桑,一条大道被以他的名字命名为刘易斯·卢彻涅特大道(Avenue Louis-Ruchonnet)。刘易斯·卢彻涅特的肖像画由瑞士出生的美国艺术家阿道夫·穆勒-乌里(Adolfo Müller-Ury,1862年-1947年)作于1888年,现收藏于洛桑美术学院博物馆。这幅画像是1894年赠送给他的儿子欧内斯特·卢彻涅特(Ernest Ruchonnet)的礼物。","text2":"刘易斯·卢彻涅特在什么时候被首次选为联邦委员会委员?","label":1} {"text1":"杰里德·贝勒斯(Jerryd Bayless)1988年8月20日生于亚利桑那州凤凰城,美国职业篮球运动员。他在2008年NBA选秀中第一轮第11顺位被印第安纳步行者队选中,随即被交换到波特兰开拓者队。作为一年级新人的贝勒斯得到了球队最高的场均19.7分、2.7个篮板和4.0次助攻。 他的球队在那个赛季取得了一个18胜13负的成绩,在太平洋十校联盟中取得的8胜10负,球队在NCAA首轮比赛中被西弗吉尼亚大学所淘汰。 在完成大学一个赛季的球赛后,贝勒斯决定提前参加2008年NBA选秀。在2008年NBA选秀中,波特兰开拓者队在与印第安纳步行者队的5人大交换后得到了贝勒斯。这次交易完成于2008年6月26日,但是直到7月9日开拓者总经理凯文·普理查德才确认此事,他表示“贝勒斯将可以扩充球队后场的活力,他是一位得到证明的射手和有力的竞争者。”在同年7月20日,他取得了2008年NBA维加斯夏季联赛最有价值球员的荣誉,在夏季联赛中他得到了场均29.8分,带领开拓者取得了3胜2负的成绩。2010年10月23日,贝勒斯被交易到新奥尔良黄蜂换未来的第一轮选秀权。2010年11月20日,贝勒斯被交易到了多伦多猛龙以及斯托贾科维奇和一些钱,换来了杰瑞特·杰克、马库斯·班克斯和大卫·安德森。在他的首次为多伦多猛禽出赛,贝勒斯得到13分, 2个篮板和2次助攻。12月11日,2010,贝勒斯追平职业生涯最高的31分,要配5个篮板和7次助攻,帮助猛龙反弹从一个25分的劣势以120:116击败底特律活塞。2012年7月6日,多伦多暴龙退出了420万美元的报价合同,以杰里让他不受限制的自由球员2010年11月20日,贝勒斯签下了一份多年合约加盟孟菲斯灰熊队。2013年6月,​​他行使球员选项310万美元。2014年1月6日,波士顿凯尔特人与曼斐斯灰熊进行双方交易,考特尼·李被交易到灰熊队,杰里德·贝勒斯则是被交易到凯尔特人。2014年7月17日,贝勒斯将与公鹿签下两年600万美元合约。","text2":"杰里德·贝勒斯生于哪里?","label":1} {"text1":"短喙鼻鱼又称短吻鼻鱼,俗名剥皮仔、打铁婆、独角倒吊,为辐鳍鱼纲鲈形目刺尾鱼亚目刺尾鱼科的其中一个种。本鱼分布于印度太平洋海域,包括东非、红海、模里西斯、塞席尔、马尔地夫、斯里兰卡、安达曼群岛、日本、台湾、中国沿海、菲律宾、印尼、澳洲、新几内亚、马里亚纳群岛、马绍尔群岛、索罗门群岛、斐济群岛、万那杜、夏威夷群岛、法属玻里尼西亚、诺鲁、加拉巴哥群岛等海域。水深4至46公尺。本鱼体呈椭圆形而侧扁;体呈均匀的褐色,体侧在侧线下方有许多红褐色细横纹,尾鳍上有一橘红色弧形横带。头顶上的角状突起,超过吻长甚多。尾柄上有2枚骨质盾板,板中央有一突出颇高的锐脊。背鳍硬棘6枚、背鳍软条27至29枚、臀鳍硬棘2枚、臀鳍软条27至30枚。体长可达70公分。本鱼常成群在礁区水层中巡游,特别是在礁坡、礁外缘。幼鱼以藻类为食,成鱼则以浮游生物为食。食用鱼类,内脏及头部可能有毒性累积,故避免食用此部位。亦可供观赏。此鱼去腥味之方法乃以尖刀将鱼体侧面次下一、二刀后将鱼体弯曲以使血液流出,再将腹部内脏去除。尾柄棘会伤人,需注意。","text2":"食用鱼类为什么要避免食用内脏及头部?","label":1} {"text1":"黄宗仰(),俗名黄浩舜,别号乌目山僧,笔名黄中央。中国民主革命家、建筑设计师、佛教僧人。1865年,黄宗仰出生于中国江苏省苏州府常熟县商人家庭,1880年在常熟清凉寺出家,1884年在镇江江天寺受戒。1899年黄宗仰前往上海。1901年,受犹太富商哈同的华籍夫人罗迦陵聘请,设计建造爱俪园,并在其内讲授佛经。1902年4月,黄宗仰与章太炎、蔡元培等联合组建“中国教育会”,开始参与政治活动。1903年6月苏报案发,邹容和章太炎被上海公共租界工部局逮捕,黄宗仰多方营救章太炎未成后,逃亡日本横滨,在那里结识孙中山,并出资帮助他得以前往檀香山。1904年黄宗仰回上海。1914年,黄宗仰离开爱俪园,回到镇江任江天寺首座。1920年,黄宗仰募集巨款,使毁于太平天国战争的南京栖霞寺得以恢复。1921年7月,黄宗仰在栖霞寺圆寂,安葬于栖霞山,年56岁。","text2":"黄宗仰在哪里结识了孙中山?","label":1} {"text1":"《少年犯之七人》是作家安部让二原作、漫画家柿崎正澄作画的日本青年漫画。自2001年起在小学馆发行的《周刊YOUNG SUNDAY》2001年51号上开始连载,2008年7月31日因连载杂志休刊,自第四部起移到《Big Comic Spirits》2009年29号(6月15日)上连载,至2010年5、6号(1月4日)连载完毕。单行本共发行22册,日文版皆已发行完毕,累计销量超过330万部。2006年获第51回小学馆漫画赏大赏。台湾繁体中文版由尖端出版代理,目前已发行至第20册(2009年10月14日出版)。香港繁体中文版由玉皇朝发行。原作者安部让二曾称之为「爱与勇气的故事」,这部作品亦是安部自身的经验所写成的。2010年4月,日本电视台将播映电视动画。昭和30年(1955年),6个16岁少年因犯罪而进入「湘南特别少教所」的二舍六房。他们对同屋里较年长的樱哥的教导铭记心中,出狱后,面对世间的痛苦、不合理,坚强地开始各自的人生。","text2":"《少年犯之七人》最初的发行日期是?","label":1} {"text1":"翠群径是位于离岛区东涌。由东涌松仁路至东涌北大屿山医院。东涌道在1966年9月28日,由港督戴麟趾主持开通典礼。早期为一条单线双程行车的道路,途中设有多个避车处,及后因青屿干线通车及北大屿山新市镇首期入伙,令居民及游人改选使用东涌道进出大屿山南部地区,使东涌道的车流量超出负荷。故在1998年开始,路政署在东涌道进行多项小型的改善工程,包括扩阔部份路段及增加避车处数目;及后政府于2004年6月开展更全面的「东涌道改善工程」,当中包括将龙井头至伯公坳一段长3.6公里的东涌道改为双线不分隔道路;在伯公坳与长沙䃟石湾之间筑建一条长2.6公里的全新双线不分隔道路,包括总长度为750米的高架道路构筑物;在东涌道沿线辟设21个避车处/巴士停车处;以及在䃟石湾基督教女青年会营地附近辟设一个回旋处,与屿南道交汇。工程已于2009年1月完成,并于2009年2月6日早上十时起正式开放通车。此外,随着东涌西部的开发(兴建逸东邨),东涌道亦被分为两段,第一段由长沙大桥至东涌坝尾村,第二段由东涌松仁路回旋处至东涌马湾涌(旧码头),后者在2008年7月11日开始已改称为东涌道北;而断开的一部份路段后来改名为翠群径。","text2":"东涌道在1966年9月28日由谁主持了开通典礼?","label":1} {"text1":"天恒邨公共运输交汇处(英文:Tin Heng Estate Public Transport Interchange)是位于香港新界元朗区天水围北天瑞路天恒邨多层停车场底层的公共运输交汇处,现时有6条巴士路线以此为总站。由九龙巴士营运的一条路线,提供天恒、天富苑、天瑞及天耀往来上水的巴士服务。于2001年4月10日起开办。取代276/276P线于天水围的巴士服务。由九龙巴士营运的一条路线,提供天水围北、天水围市中心、天耀及天慈往来长沙湾、旺角及大角咀的巴士服务。开办时由于本站尚未启用,以天富苑作临时总站,于2001年3月25日起开办。后于同年4月1日本站启用随即迁往。由九龙巴士营运的一条路线,提供天水围北、美湖居及天慈往返荃湾、葵涌、丽瑶邨的巴士服务。于2001年4月1日起开办于2014年12月1日配合元朗区M线服务重组而开办,途经葵兴、大窝口站、荃湾、屯门公路、大榄隧道、元朗公路、天慈邨、嘉湖山庄(美湖居、丽湖居、景湖居)、天晴邨、天逸邨及天泽邨,为九龙巴士269M线的特别班次,只于平日下午繁忙时间服务。于2004年4月18日开办,取代原有A70及A74线,途经天富苑、天晴邨、嘉湖山庄(美湖居、丽湖居、景湖居)、天水围游泳池(只限回程)、天慈邨、朗屏邨及元朗广场。于2003年11月24日开办,最初是一条来往西铁兆康站(北)及天恒的临时路线,2004年7月4日缩短至由此开出并改为循环线,2005年3月27日取消循环线运作,途经天恩邨、天富苑、天泽邨及天逸邨。","text2":"现时有多少条巴士路线以天恒邨公共运输交汇处为总站?","label":1} {"text1":"杨林(),是中国足球运动员,身材高大,主要出任高中锋,也可出任中后卫,曾经入选过中国国家队。2002年,杨林加盟甲B球队大连赛德隆。2003年,大连赛德隆被转卖到珠海,更名珠海中邦,杨林随队。2004年,球队又搬到上海,更名为上海中邦,杨林继续作为球队主力。2007年,上海申花与上海联城合并后,杨林转会到了河南建业,表现依然稳定。2009年,杨林从河南建业租借至家乡的大连实德效力,不过被视为中后卫的人选。2010年,杨林加盟新组建的乙级联赛球队大连阿尔滨,前半赛季和联城时代的老队友常琳搭档球队的中后卫,表现稳定,后半赛季顶替离队的徐一文出任球队的高中锋,并攻入3粒进球。2011年,在大连阿尔滨大力引援的情况下,杨林失去了主力位置,不过做一名老将,杨林有效的丰富了球队的板凳深度,继续为阿尔滨队发挥着余热。最后更新:2011年10月30日","text2":"何年大连赛德隆被转卖到珠海?","label":1} {"text1":"安提贞尼斯(希腊语:Aντιγένης;)是亚历山大大帝麾下将领之一。从腓力二世时就为马其顿王国效力,公元前340年时在围攻色雷斯的皮林塔斯时失去一只眼。当亚历山大大帝逝世后,第一次继业者之战爆发,安提贞尼斯跟随帝国摄政佩尔狄卡斯攻打埃及的托勒密,当佩尔狄卡斯抵达尼罗河三角洲最东部的城市佩鲁修姆(Pelusium)企图渡过尼罗河,遭受到失败。佩尔狄卡斯斯转道三角洲顶部的孟菲斯,但部分部队在渡河时又被河水冲走,部队士气低下,在佩尔狄卡斯的严厉惩处下,部队被激怒而发生哗变。银盾兵指挥官之一的安提贞尼斯与培松、塞琉古联手杀害了佩尔狄卡斯,并与托勒密谈和。在特里帕拉迪苏斯分封协议中,安提贞尼斯因此担任埃兰总督。在第二次继业者之战中,安提贞尼斯和其他东部行省总督加入欧迈尼斯一方来对抗安提柯,并在帕莱塔西奈战役、伽比埃奈战役两场会战中率领银盾兵都有良好表现,而当银盾兵在伽比埃奈战役后发生哗变,安提贞尼斯随著欧迈尼斯落入安提柯的手中,最终被活活烧死。","text2":"在第二次继业者之战中,安提贞尼斯的结局如何?","label":1} {"text1":"久里滨线()是一条连结日本神奈川县横须贺市堀之内站与同县三浦市三崎口站,属于京滨急行电铁(京急)的铁路线。车站编号使用的路线记号为KK。普通车只在平日早上繁忙时段行驶。现在普通车是不会行走京急久里滨以南的地方(1996年4月至1999年7月只在平日行驶)。特急在早上至早上繁忙时间、平日晚上至深夜,在该时段的列车全为快特。只有平日朝上运行,抵达横滨站7:30 - 8:30前往品川的最前的车卡是女性专用车。为方便前往成田山新胜寺参拜的乘客,在1969年至1972年间开办了来回三浦海岸 - 京成成田站的通宵列车。使用的车辆为1000形,停车站与京急线、都営线内特急一样,而在京成线内则以「临时特急」运行(京成津田沼站 - 京成成田间不停车,这个与押上站至京成津田沼间之间运行的特急有少许差异)。原定此线也会在1973年运行,但是由于东京都交通局就劳动争议问题而反对通宵行驶,最终亦停办此线。","text2":"特急是指哪一个时间段?","label":1} {"text1":"溪头龙蜥(学名:),又名牧氏攀蜥,俗称竹虎,旧称牧茂氏攀蜥,为飞蜥科龙蜥属下的物种。仅分布于台湾,正模标本采集自南投县的针叶林中,由日本动物学家牧茂市郎采集。其生存的海拔范围为1500至1500米。体长 10~25 公分,最大可达 27 公分。头部有一条粗黑的过眼线,嘴部外缘与腹部体色较浅。雌雄体色差异甚大,体色会随环境而小幅度改变;雄性最显著特征在于身上有许多粗而黑的斑块(斑块颜色有个体差异),鬣鳞明显;雌性体色会因个体差异而有所不同,虽然大多以绿色为主(有些个体也有与雄性相似的粗斑块,但颜色较接近绿色)但有些个体除了斑块外,自头部经背脊中央至尾部均为暗褐色,形成一条宽纵带,称之为「棕背型」。为树栖性生物,日行性,白天常在森林边缘空旷处活动,且常在树丛面向光的叶面。领域性强,雄蜥遇惊扰时喉部会扩张,并做出类似伏地挺身的姿势,多以小型无脊椎动物为食","text2":"正模标本是有谁采集的?","label":1} {"text1":"文化区(Cultural area)是指文化较接近的地区。此概念与另一个类似的概念「文化圈」(德文:)的意思并不尽相同,后者现时在日本非常流行。「文化区」与「文化圈」,两者都是与文化相关的地域名词,两者同样是在第二次世界大战之前提出的,意思亦有少许类似,但并不尽相同。首先,「文化区」是由美国的Clark Wissler和阿弗烈·克鲁伯,两位人类学家提出的文化人类学概念;「文化圈」则是由德国的Fritz Graebner和奥地利的Wilhelm Schmidt,两位民族学家提出的。另外,「区(Area)」的意思非常清晰,就是指地域。「圈(Circle)」的意思却很模糊,例如「演艺圈」的圈字就有「界」的意思,「华人圈」的圈字则是指社群,而「文化圈」按字面并没有地域的意思,但现时流行于日本的「文化圈」概念,事实上却是一个地域名词。不过因为日本用「圈」字代替了「区」字,使其地域性质变得不明显,一般人便不容易察觉其背后所隐含的地域性质。这类概念亦最容易被利用和变质,最明显的例子就是「大东亚共荣圈」。至于「文化区」则没有此问题,因为意思本来就很清晰,很难被利用。最后,「文化区」所著重的是地理,很少会著重历史。但现时流行于日本的「文化圈」概念,所著重的却是「历史发展」与「地域统合」。","text2":"现在流行在日本的文化圈注重的是什么?","label":1} {"text1":"《江湖三女侠》(共三册)是梁羽生于1957年4月8日至1958年12月10日于《大公报》「小说林」连载成书,为「天山系列‧七剑下天山」续篇,除了易兰珠外,其他有关「七剑」者多为子弟及徒弟,如唐晓澜、冯瑛、冒广生、李治、桂华生等。清初,民间反清之心不平,一师从天山七剑之首凌未风的大侠在中原传下武艺,由经人至唐晓澜手中,经年不想遭命丧子弟家中,变故频生,清廷开始大规模铲除意图谋反之人,吕四娘亦受文字狱牵连,拜独臂神尼为师;唐无力护卫师门亲属(包括冯瑛,冯琳这对孪生姐妹),冯琳被劫走,他送冯瑛至天山学艺,自己另求名师;又过十多年,又有变革,冯氏姐妹长大……本篇在天山系列中,篇幅较长,情节波澜不惊,刻画人物不少,以小说的形式体现了江湖豪侠和清廷的恩怨。","text2":"《江湖三女侠》(共三册)是谁的书?","label":1} {"text1":"亚历山大港的丢番图(希腊语:,生卒年约公元200~214至公元284~298),有“代数之父”之称;也有人认为此称谓应与比他大约晚出生五百年的一位波斯数学家花拉子米共享。人们对丢番图的生活知之甚少。在罗马时代,他住在埃及的亚历山大港,大概从公元200年到214年,到284年或298年。丢番图曾被历史学家描述为希腊人,非希腊人,希腊化的埃及人,希腊化的巴比伦人,犹太人,或者是迦勒底人。我们对丢番图生活的了解,来自于一个5世纪的希腊文集。他作著的丛书《算术》(Arithmetica)处理求解代数方程组的问题,但其中有不少已经遗失。后来当法国数学家费马研究《算术》一书时,对其中某个方程颇感兴趣并认为其无解,说他对此「已找到一个绝妙的证明」,但他却没有写下来,三个世纪后才出现完整的证明,详见费马大定理。在数论中常常能看到他的名字,如丢番图方程、丢番图几何、丢番图逼近等都是数学里重要的研究领域。丢番图是第一个承认分数是一种数的希腊数学家--他允许方程中的系数和解为有理数,这是在数学史中具有开创性的。不过在今天,丢番图方程一词通常指以整数作为系数的代数方程,而其解也要求是整数。丢番图在数学符号方面也作出了贡献。","text2":"《算术》主要是解决什么问题的?","label":1} {"text1":"亲子丼(おやこどん、-どんぶり(Oya Ko Donburi),汉语拼音:qin1 zi3 dan3),又称滑蛋鸡肉饭,是日本的一种米食类料理。以鸡肉、鸡蛋、洋葱等覆盖在饭上,再以碗盛装而成的丼物,「亲子」的名称是因为丼物里面同时包含鸡肉与鸡蛋而来。亲子丼由一间叫「玉日出」的餐厅开始,本是贩售「军鸡锅(军鸡就是斗鸡,其肉质耐嚼且鲜美多汁)」,也就是将鸡肉煮熟后,再淋上蛋汁一起食用的料理,当时日本的鸡蛋价格非常昂贵,客人认为若因军鸡锅吃不完而丢弃是很浪费的事,所以就把剩下的料理和白饭拌在一起食用,没想到味道相当可口,来店的客人与日遽增,一直到第五代老板上任后,才正式将鸡肉覆盖蛋汁,再淋在饭上来销售,并命名为亲子丼(OYAKODON)。同时包含鱼与鱼卵的丼物也称为「海鲜亲子丼」,如「鲑亲子丼」。","text2":"「鲑亲子丼」具体被称为哪种丼物?","label":1} {"text1":"白金汉郡(,简称:Bucks),英国东南英格兰的郡。米尔顿凯恩斯是第一大镇(Town),海威科姆排第二,艾尔斯伯里排第三(亦是郡治)。以下经济数据来源是英国国家统计办公室. 数字经四舍五入,与实际数据或有偏差. 包括:狩猎、林业. 包括:能源、建筑. 包括间接统计出来的金融仲裁服务白金汉郡的人口、面积、人口密度随不同的郡级地位而不同,如看待成名誉郡,其人口(km²)、面积、人口密度(人/km²)会较多、大、高:白金汉郡是非都市郡,实际管辖4个非都市区(「#政治」段落右图编号1、2、3、4);如看待成名誉郡,它名义上包含多1个单一管理区(编号5)。白金汉郡内的5个非都市区被划分为7个英国国会选区,下方两图可作比较。选区分为集中于市区的自治市镇选区(borough constituency,简称:BC)和集中于郊区的郡选区(county constituency,简称:CC)两种。白金汉郡没有城市(City),但有19个镇(Town)。以下19个镇按人口多寡编号,在下图中分别代表她们在伯克郡的位置(沃本桑兹未有人口数据,唯有按英文字母编为第15)。点击图上编号即可进入该镇的主条目。注:米尔顿凯恩斯的数据已包含以下四镇:布莱奇利(Bletchley)、芬尼斯特拉特福(Fenny Stratford)、斯托尼斯特拉特福(Stony Stratford)、沃尔弗顿(Wolverton)。下图显示英格兰48个名誉郡的分布情况。白金汉郡位于英格兰东南区域的,北与北安普敦郡相邻,东(上半部)与贝德福德郡相邻,东(下半部)与赫特福德郡相邻,东南与大伦敦相邻,南与伯克郡相邻,西与牛津郡相邻。","text2":"白金汉郡有多少个镇?","label":1} {"text1":"圣乔治角海战(,1943年11月26日)是一场发生于布干维尔岛水域,在日本和美国舰队之间爆发的海战。这也是在所罗门群岛海域进行的最后一次海上战斗。1943年10月,所罗门群岛战役已接近尾声。11月1日,美军在布干维尔岛上登陆,威胁到了位于布干维尔北部布卡岛上的日本基地。日本海军调遣了3艘运输驱逐舰和2艘驱逐舰,在香川清登上校的指挥下运载900名陆军官兵前往该岛增援。美国海军获悉此行动后,调集了5艘弗莱彻级驱逐舰,在阿利·伯克上校的指挥下前往拦截日本舰队。日本舰队在布卡岛卸载了兵员和补给品后,又装载了一些海军人员返回拉包尔。1时40分,美军雷达发现日本舰队。由于拥有雷达优势,美国人得以接近日本舰队而不被对方发现。1时55分,美军发射鱼雷。日军旗舰大波号被数枚鱼雷命中,当即沉没,香川上校阵亡。另一枚鱼雷击中卷波号驱逐舰,使其失去战斗力,随后被炮火击沉。其余三艘运输驱逐舰四散奔逃。伯克下令追击夕雾号,于3时30分左右将其击沉。圣乔治角海战标志着持续长达一年之久的东京快车行动的终结,也宣告了日本在所罗门群岛战役中的失败。美国的雷达技术在这一系列的海上交锋中扮演了重要的角色。在圣乔治角海战之后的半年时间里,太平洋战场再未发生海上战斗,直到1944年6月的菲律宾海海战为止。","text2":"发生海战的原因是什么?","label":1} {"text1":"过冷(Supercooling,又译超冷冻)是一种物理现象,透过降低液体或气体的温度,但不使其凝固的过程,能做到让水瞬间凝冰的效果。冰的形成其实是一个结晶的过程,如果原本水中已存在结晶核(冰晶),低于其标准熔点的液体的水分子就会在核周围形成结晶结构,依附在冰晶上,结成冰的过程就可以进行。但是有些水太洁净,没有尘埃和杂质,容器又非常干净而平滑,缺少这样的晶核,冰晶就因无处可依附而不能形成,于是温度轴上的液体部分可以一直延伸,即使温度低于冰点(标准压力下formula_1),水仍然会维持在液体的状态,直到均匀核化结晶生成。如要进行超冷冻的现象要符合以下三个条件:把一瓶纯水或蒸馏水放进冰箱数小时,令水的温度低至0°以下,但未至于结冰,即过冷水,把水瓶拿出冰箱,然后向瓶一拍,就可看到水会开始结冰,由一点迅速扩散至整瓶水,最终整瓶水会成为冰。但现象可能受很多因素影响而未能成功,因素包括水的温度,水瓶质料,大小等。拍向这瓶超冷冻的过冷水,会使到水中的水分子受压,水分子之间的距离变得更大,触发水分子变成固体。当有一点的地方结冰,其他的水分子就会依附结成的冰,过程迅速进行,瞬间整瓶超冷冻水都变成冰的现象就会出现。","text2":"怎么制作过冷水?","label":1} {"text1":"茅湖山观测台位于香港西贡区将军澳茅湖山上,行山人士及街坊称其为茅湖废堡,确实建筑年份不可考,但从现有历史文献中推算,建筑物早于1898年英国租借新界前已建成,估计为清朝佛堂洲海关的观测台,作观测佛堂门航道之用。基于其独特性,观测台已被古物古迹办事处于2009年12月18日列为一级历史建筑。茅湖山观测台与佛堂洲海关有密切联系,观测台能监察整个佛堂洲及其海域船只出入的情况,估计当时观测台的官员透过信号灯,又或是透过烟火,与佛堂洲总部及海关船只联系。现时茅湖山的遗址包括一座圆形石砌塔楼的下半部,以及一座长方型的乡村式石屋。两栋建筑物皆由深灰色的火山岩建造,估计由当地附近开采而来。圆形塔楼应为一座瞭望塔,观测人员能透过半圆形的窗户毫无阻碍地观测佛堂门航道。该塔楼原有两层高,现时第二层已倒塌,现存一条石楼梯,通到已倒塌的二楼。历史照片显示塔楼原有一个类似中式帽子的拱顶,现时塔顶部分已不存在,原因不明。所有门及窗户已散失,只剩下石制窗框和门框。建筑物塌下的碎片早已被区议会移走,小部分则用作建造休憩长椅。塔后的单层建筑物估计为驻守该建筑官员的总部或住所,以本地乡村村屋模式设计,与圆形塔楼的风格极不相同。现时该建筑物的天花已塌下。圆形塔楼的设计令人联想起古时欧洲防御海岸用的圆形石堡或城堡,在香港极为罕见,类似的圆形塔楼建筑仅见于几座灯塔及尖沙咀前香港水警总部的报时塔,此外昂船洲军营内也有两座瞭望塔,但它们是八角形的建筑而不是圆形;新界不少村落亦设有瞭望塔,但它们多以砖建成,形状则多为正方形。香港审计署在2013年3月28日发表的第六十号报告书「古迹及历史建筑物的保育」中指出,截至2013年2月,该项一级历史建筑长期缺乏打理。由于缺乏妥善的维修保养,引致破旧失修,加上四周并无围封,可能会对游人安全带来风险。2013年3月,地政总署告知审计署,地政总署会联络古物古迹办事处,为安全起见把调景岭茅湖山观测台围封,作为过渡措施,以待当局定出计划,予以善用。","text2":"观测台已被古物古迹办事处于2009年12月18日列为几级历史建筑?","label":1} {"text1":"《暴走兄妹》(;另译康子与健儿)是日本漫画家或子所画漫画作品。在集英社发行的女性漫画杂志《Deluxe Margaret》的2004年1月号发表后,于《别册Margaret》2005年4月号至2006年11月号连载。单行本全5册。2008年由日本电视台改编拍成电视剧,7月到9月于每周六播出,由松冈昌宏(TOKIO)、广末凉子、多部未华子、大仓忠义等人主演。另外在《Deluxe Margaret》从2009年1月号开始、发表番外编的「暴走兄妹」。6年前的12月24日,双亲因为交通事故而死,于是冲康子和原暴走族总长的哥哥冲健儿开始住在一起。但是由于哥哥的阻挠,使得她总是无法好好谈恋爱。于是兄妹因此每天过著吵架的日子,但康子却某天遇上了资优生桩纯而爱上他,但纯的老姐却是以前常和健儿吵架的另一原暴走族总长惠理香,因此康子除了要让纯喜欢上她外,还要努力改善健儿和惠理香这对欢喜冤家的关系,故事也在此展开了一段非常暴走的生活。从《Deluxe Margaret》2004年1月号开始连载。收录有『三篇和红色脚踏车』。短篇版的设定,康子16歳(高校1年生)、健儿是23岁。另外健儿的助手摩斯和爱迪达也有登场,但长相有些微的不一样、也没有名字。另外伴随著戏剧版化、在《Deluxe Margaret》2008年9月号再度连载。《暴走兄妹》电视剧集于2008年7月12日到9月20日在日本电视台播出。播出时间是周六的21:00〜21:54(JST)。主演的是TOKIO的松冈昌宏,是松冈自从主演连续剧2006年播放的《夜王》(TBS系)以来,阔别两年三个月参与连续剧演出,而演出同台的连续剧则是自从2004年播放的《秀逗男护士》的四年以来,再度演出。本剧2009年在香港无线剧集台首播,香港版本译名为《暴走冤家》。收视率为日本关东地区,由Video Research所调查。","text2":"《暴走兄妹》在哪个刊物上连载?","label":1} {"text1":"巴林国际机场(,),是巴林王国全国唯一的民用机场,位于该国首都麦纳麦东北方向约7公里处的穆哈拉格岛上。它是海湾航空主要的枢纽机场,也是海湾地区第一座国际机场。巴林国际机场与阿布扎比国际机场、素万那普国际机场、多哈国际机场、迪拜国际机场、奥利弗·坦博国际机场、科威特国际机场、伦敦希思罗机场和马德里-巴拉哈斯机场一起被公司评为三星级机场。第一架预定抵达巴林的商务班机是1932年从伦敦飞往德里的名为“汉尼拔”的汉德利·佩季·H·P·42型飞机。这架飞机只搭载了24名乘客,从伦敦出发后用了好几天才抵达目的地,时速100英里。这条航线固定后,巴林建起了海湾地区第一座国际机场。","text2":"被称为三星级机场的机场是哪些?","label":1} {"text1":"蒙纳黑体是蒙纳公司(Monotype Corporation)的一款中文无衬线字体。其衍生字体包括蒙纳正线体、蒙纳中黑体及蒙纳粗黑体等。由于蒙纳黑体价钱较贵,所以并不多企业使用,私人使用更是少之又少。1996年,九广铁路公司开始使用蒙纳黑体作为其全新格式的字体。香港特别行政区政府的香港品牌标志亦使用蒙纳黑体作为主要中文字体。另外,领展于屋邨商场内新安装的指示牌也用上了蒙纳黑体。汇丰银行近年采用的「环球金融,地方智慧」标语,也使用蒙纳黑体,但字元比例比一般要宽(多于 100%)。香港大部分报章、书刊和杂志也有采用蒙纳黑体的,例如苹果日报、新Monday、Yes!等等。香港国际机场于2015年10月更换了最初落成时用过的蒙纳黑体作为导向系统的字体。同样由 Monotype 拥有的 Arial Unicode MS 字型, 采用蒙纳黑体作为汉字字体, 但默认的字形是适用于日本的风格, 不适合中文排版。尽管如此, 用户仍可透过利用该字型提供的 OpenType 特性, 获得恰当的地区专用的汉字字形。","text2":"蒙纳黑体是哪家公司的字体?","label":1} {"text1":"在厘米-克-秒单位制(CGS制)里,静库仑(statC)或(esu)是电荷的物理单位。它是一种衍生出来的单位,表达为在国际单位制里,则采用库仑(C)为电荷的物理单位。转换公式为这公式乃精确的(但是,请参阅后面正确使用方法的警告)。在公式右边的数值是光速(CGS制)的十分之一。双方向转换大约为静库仑定义为:假若两个固定的,相距1 cm的点电荷各自带有电量1 statC,则它们彼此互相排斥的静电力是1 达因(dyne)。这排斥力是由库仑定律给出,在CGS制里,表达为其中,formula_2是力量,formula_3、formula_4分别是两个点电荷的电量,formula_5是它们之间的距离。做一个因次分析。formula_2的因次是[质量][长度][时间]。所以,在CGS制里,电荷的因次必须是[质量] [长度] [时间]。在国际单位制里,这句话并不正确。因为,库仑定律的形式不同:其中,formula_8是真空电容率。由于formula_8本身有因次。与静库仑不同,库仑的因次不是[质量] [长度] [时间]。实际上,我们无法只用质量、长度和时间来表达库仑的因次。因此,以下这公式必须非常小心地使用,公式两边的单位并不一致:我们不能直接地使用这转换因子在库仑与静库仑之间做转换,就好像我们在公分与公尺之间做转换一样。更确切地说,这公式应该诠释为:\"1 coulomb\"对应于\"2997924580 statC\"。换句话说,假若某一个带电体的电量是1 coulomb,那么,它的电量也是 2997924580 statC。特别注意,以下这转换的因次完全一致,对于国际单位制和CGS制之间的转换,非常有用:其中,formula_8是CGS制的真空电容率,大约为8.85 As gcm。","text2":"我们应该如何诠释转换公式?","label":1} {"text1":"张燮林(),中国上海市人,祖籍江苏省镇江市,乒乓球运动员,以灵活多变的直板长胶削球打法为其特色。张燮林早年自学成才,1958年获得上海市比赛亚军后被调入上海队,1960年进入中国国家乒乓球队。曾参加过多届世界乒乓球锦标赛,先后获得过男团、男双和混合双打冠军,并获得过男子单打第三名。1972年至1995年期间担任中国国家女子乒乓球队教练,邓亚萍等著名运动员均出自其门下,后升任国家体育总局乒乓球羽毛球运动管理中心副主任。张燮林是1987年乒乓球世界锦标赛“何智丽让球事件”的幕后主教练。他要求中国乒乓球女运动员何智丽在比赛中故意输掉二分之一决赛给另一位中国女运动员。何智丽没有听从他的指示,赢得了比赛并最后赢得了世锦赛冠军。随后他禁止了世界排名第一的何智丽参加任何国际性大赛包括奥运会。何智丽在巅峰时期无赛可比而被迫远走日本,加入日本国籍,代表日本出赛。张燮林作为违反体育公平竞赛的幕后操纵者没有受到中国体育局的任何处罚。","text2":"张燮林以什么打法为特色?","label":1} {"text1":"伊藤加奈惠(,),是日本的女性声优。隶属青二Production(声优事务所)以及Lantis(歌手事务所)。长野县出身。身高147cm、血型是B型。AMUSEMENT MEDIA ACADEMY毕业。中学时因为收听『HUNTER×HUNTER』的广播节目-竹内顺子和三桥加奈子主持的「HUNTER×HUNTER R」而对竹内顺子抱有憧憬,继而自己也想成为声优。小学时隶属垒球部。高中时同时隶属于包括演剧部在内的多个社团。高中毕业后进入AMUSEMENT MEDIA ACADEMY,2007年毕业。在AMUSEMENT MEDIA ACADEMY就学时参加了声优组合こえっち,不过于2005年11月解散。之后,又参加了由こえっち成员组成的声优组合りらちっち,不过也于2007年1月28日解散。2007年,在『守护甜心!』中第一次主演,出演了日奈森亚梦一角,之后又出演了该系列作品的动画、游戏、广播。2007年,出演电视剧『幼獣マメシバ』中演员出道,在杂志『保育のひろば』上模特出道,之后又完成了歌手出道,拓宽了活动范围。在2010年3月6日公布的声优奖上获得新人女声优奖、2011年3月5日的第5回声优奖上获得女配角奖。※主角及主要人物以粗体识别。2006年2007年2008年2009年2010年2011年2012年2013年2014年2015年2016年2017年2008年2009年2010年2011年2012年2013年2015年2016年2017年电视动画其他","text2":"伊藤加奈惠毕业于什么学校?","label":1} {"text1":"共和国广场(捷克语:Naměstí republiky)是捷克城市比尔森古城的一个广场,规模为552 x 627 英尺 ,是捷克最大的中世纪广场之一。自13世纪末比尔森创建城市起,该广场和已经存在。在这个长方形广场周围,是由直角相交的街道划分出的规则的住宅街区。考古学家鉴定,此处自13世纪铺设木质地面。1859年,广场 铺设了鹅卵石,后来改铺沥青。周边建筑以哥特式和文艺复兴风格占据优势。在东部的庭院内,仍保留有中世纪城墙的残余。广场的南部保护得最好。许多建筑物设有2或3层地窖,用于保存食物、水井或污水坑。最重要的地标为圣巴尔多禄茂主教座堂(1295年兴建,1993年成为主教座堂)、比尔森市政厅(自1496年起)和圣母玛丽亚黑死病纪念柱(1681年),全部位于广场的北部。","text2":"共和国广场的规模有多大?","label":1} {"text1":"恒安国际有限公司(),简称恒安国际、恒安集团,是中国最大的妇女卫生巾和婴儿纸尿裤生产商,从事生产及销售「安尔乐」、「心相印」、「安乐」、「安儿乐」、「安而康」等品牌的妇幼卫生用品和生活用纸。公司在1985年由主席施文博先生及副主席兼行政总裁许连捷先生共同创办,1998年在香港交易所上市。总部设在福建晋江。2011年6月7日,恒安国际与友邦保险获纳入恒生指数成份股。恒安国际发行54.34亿元零息可换股债券,于2018年到期,换股价每股120.0825元,较2013年5月21日收市价88.75元溢价达35.3%,倘悉数换股共逾4,525万股,占扩大后股本约3.5%。集资用作还债及资本开支等。","text2":"恒安国际什么时候在香港交易所上市?","label":1} {"text1":"司马模(),字-{}-元表,西晋高密王司马泰的第四子。少好学,与琅琊王司马睿及范阳王司马虓在宗室中有贤名。初封平昌公,晋惠帝末年,拜冗从仆射,累迁太子庶子、员外散骑常侍。成都王司马颖奔长安,东海王司马越以弟弟司马模为北中郎将,镇守邺城。永兴初年,成都王司马颖原来的部下公师籓、楼权、郝昌等攻邺城,广平太守丁邵率众来救,范阳王司马虓也遣兖州刺史苟晞救援,公师籓逃散。之后,迁镇东大将军,镇守许昌,进爵南阳王。永嘉初年,转征西大将军、开府、都督秦雍梁益诸军事,代河间王司马颙镇守关中。东海王司马越征司马模为司空,遣中书监傅祗代替镇守关中。司马模谋臣淳于定建议司马模继续留守长安,和哥哥司马越一个在内、一个在外,遣世子司马保为西中郎将、东羌校尉,镇守上邽,司马越进司马模进位太尉、大都督。311年,永嘉之乱,镇守蒲坂的牙门赵染,求冯翊太守不得,率众投降刘聪。刘聪派其子刘粲和赵染攻长安,司马模派淳于定抵御,被赵染击败。司马模投降赵染。赵染坐在那里数落司马模的罪行,送到刘粲处,刘粲杀了他,以司马模的妃子刘氏赐给胡张本为妻,子司马保继立为南阳王。","text2":"司马模投降赵染之后有着怎样的下场?","label":1} {"text1":"本条目为欧洲足协杯赛冠军球会领队列表,纪录各主教练带领曾执教的欧洲足协所属成员国的足球俱乐部,在欧洲足协官方认可的球会级杯赛赢得冠军的数量。这些杯赛包括欧冠杯(1955年至1991年,欧洲联赛冠军杯前身)、欧联(欧洲联赛冠军杯,1991年至现今)、欧洲杯赛冠军杯 、欧洲足协杯(1971年-2009年)、欧霸杯(2009年至现今)、图图杯和欧洲超级杯。截止2014年意大利籍领队查柏东尼和苏格兰籍领队费格逊是最成功的领队,分别带领球队赢得6项赛事冠军。截至2014年,意大利领队赢得最多杯赛冠军,合共38项,英格兰领队则紧随其后,有25项。当欧洲博览会杯被欧洲足协杯取代前,欧洲足协并没有将其视为官方赛事,所以此杯的冠军将不被在此列表中记录。而1972年的欧洲超级杯,同被视为「非官方赛事」。说明此表是纪录各国领队带领球队夺冠的总数:","text2":"欧洲足协赛事冠军球会领队列表记录了什么?","label":1} {"text1":"黑头海蛇(学名:\"Hydrophis melanocephalus\")是蛇亚目眼镜蛇科海蛇属下的一种有毒海蛇,主要分布于南中国海、澳洲及日本之间的海域,是日本的受保护动物。目前未有任何亚种被确认。黑头海蛇体长约为80至140公分,体型幼长。体色以浅黄色为主,身上布有黑色纹带。特征为其黑色的头部,其学名中的「melanocephalus」意思就是「黑色的头」。黑头海蛇主要分布于以下海域:标准产地为印度洋。黑头海蛇主要栖息于热带至亚热带的海域,是完全水栖型的蛇类,于陆地上活动困难。牠们多于日间出没,以捕食鱼类为主。繁殖方面,黑头海蛇属卵胎生蛇类,雌蛇每年约于十月时生产,每次能诞下约四至五条幼蛇。虽然大部分海蛇都没有主动攻击人类的倾向,然而黑头海蛇却属于较具攻击性的成员,而且牠们遇到人类时,不会采取纯回避的消极态度。每年在日本冲绳一带,均会有数宗黑头海蛇咬死人类的事件,因此在遭遇野生黑头海蛇的时候必须谨慎小心。黑头海蛇的毒素以神经毒素为主要成分,人类中毒后会出现全身疲惫乏力、肌肉刺痛、活动能力出现障碍、呼吸困难、麻痺等征状,最恶劣情况可能导致死亡。倘若肢体被黑头海蛇所咬,应立即将伤口近心脏一端的肢体紧缚起来,阻缓血液间的毒液运行,而且尽量将毒血抽出,同时尽快求医接受适当治疗。","text2":"黑头海蛇体色以什么颜色为主?","label":1} {"text1":"亚历山大·瓦西里耶维奇·苏沃洛夫(,),俄国大元帅,神圣罗马帝国伯爵、雷姆尼克伯爵、意大利亲王。苏沃洛夫是俄国史上最杰出的将领之一,并在其漫长的军事生涯中保有不败的名声,著有军事学名著《制胜的科学》。苏沃洛夫的全部头衔为:亚历山大·瓦西里耶维奇·苏沃洛夫,意大利亲王、雷姆尼克伯爵、神圣罗马帝国伯爵、撒丁尼亚伯爵、俄罗斯陆海军大元帅、奥地利和撒丁尼亚陆军元帅。1942年7月29日,苏联以其名字设立了苏沃洛夫勋章,以表彰在进攻中歼灭敌人优势兵力的军事首长和指挥员。1729年,亚历山大·瓦西里耶维奇·苏沃洛夫出生于莫斯科家中。当时,父亲瓦西里·伊万诺维奇·苏沃洛夫是普列奥布拉任斯科耶团准尉,后成为上将参政员,学识渊博,曾编纂了第一部俄国军事词典。母亲是亚美尼亚贵族的后裔。苏沃洛夫从小体弱,父亲也希望他将来从事一些公共服务工作。但是他却酷爱军事,崇拜查理十二世、彼得大帝、恺撒、汉尼拔、大孔代、蒂雷纳、欧根亲王等杰出的统帅。也努力通过严酷的锻炼来改善体质,在其父亲的指导下研究炮兵学、筑城学和军事史。1742年11月,苏沃洛夫被录为禁卫谢苗诺夫团火枪兵,在这期间阅读了希腊史学家普卢塔尔赫的著名的古罗马统帅恺撒的回忆录,学习了奥地得军事家蒙特库科利的《论军事艺术》。研究过沃尔夫和莱布尼茨的哲学原理。苏沃洛夫除用法语外,还掌握了德语。1748年5月6日,首次获得提升,升任下士。1748年1月12日,开始在该团正式服役,任班长。1754年5月第一次被授与中尉军官军衔,并被派往英格曼兰步兵团任职。1756—1758年在陆军院供职。","text2":"苏沃洛夫是哪个国家的?","label":1} {"text1":"《思草纸》(琉球语:,汉字表记:,罗马字:\"Omoro Sōshi\"),琉球第二尚氏王朝时期的一部祝词集,也被冲绳学学者当作是历史上最早的一部琉歌集,1531年至1623年间由琉球国朝廷主持,编纂而成。原文是以日本平假名记录的琉球语(其间夹带少量汉字)。据冲绳学学者伊波普猷考证,「」在琉球语中是「思考」(琉球语:,汉字表记:)的意思,是古代琉球群岛特有的一种祝词;而「」一词来源于日本语,是「草纸」的意思。这部祝词集收集了自12世纪以来琉歌作品,共22卷,记载有关国王、祝女、英雄、诗人、航海者的事迹,以及与风景、天象、战争、神话有关的琉歌,还有少量的爱情诗作,有重要史实价值。《思草纸》中所收录的祝词,最短的仅有两行,最长的有四十行,皆为韵文,盛行对句,因此被认为是后世琉歌的起源。但是,《思草纸》中的语言多为琉球古语,在现代的琉球语中已经不使用了。《思草纸》的编纂,共分有两个时期。最早是在尚清王时期的1532年,为颂扬先王尚真王的丰功伟业,尚清王下令编纂第一卷。此后70年间编辑中断。直到1613年,琉球国沦为萨摩藩的附庸国,琉球朝廷处处受到萨摩藩的制约,在政治上无所作为。为维持琉球人的传统文化和历史,尚丰王下令编纂第二卷。1613年至1623年间,又陆续编纂至22卷。※「/\」为重复记号,实际在竖排文字中写作〳〵,形如拉长的假名く。","text2":"《思草集》的地位是什么?","label":1} {"text1":"同济路站()是广佛地铁的一座车站,位于广东佛山禅城区汾江中路同济路口的地底,邻近季华、祖庙两大商业区,也有相当的休闲娱乐设施 ,车站已在2010年11月3日启用。本站共有两层。地面为汾江中路、同济路、金马剧院及其它住宅或建筑;地下一层为广佛地铁站厅;地下二层为广佛地铁站台层。同济路站站厅中西侧被划分为收费区,周围被行人通道以及车站商店包围,站厅收费区内设有专用电梯(升降机)、扶手电梯和楼梯前往车站月台。商店及自助服务方面,本站设有便利店、面包糕饼店、中国银行自动柜员机以及自动售货机。同时也设有“好易”机,可充值羊城通(含广佛通)、购买彩票、缴纳水电费以及交通罚款等服务。本站设有一个岛式月台,位于汾江中路地底。同济路站目前设有3个出入口。同济路站是广佛线的中途站,乘客可以乘搭往魁奇路或西朗列车,来往佛山市区及广州市区各地。同济路站附近以较为密集的住宅楼为主,周边有佛山乐园以及各所中小学。但现时佛山地铁线网没有完善,因此车站在平日时期客流一般,但在上放学时期的早晚高峰车站会稍显拥挤。2002年广佛地铁的走向初步确定,同济路站确定为广佛地铁佛山段的其中一个中途站。2007年6月28日,广佛地铁首通段正式动工。2009年6月28日,本站主体结构封顶。2010年10月28,29日,本站随广佛地铁首通段全线开放给广佛两市的市民进行试乘。11月3日,广佛地铁首通段开通试运营,本站正式启用。","text2":"同济路站位于什么地方?","label":1} {"text1":"卡尔·考克斯(Carl Cox,)国际知名的技术性电子音乐(techno)和浩室电子音乐(house)的DJ。考克斯出生于奥尔德姆,兰开夏郡,英格兰。 幼年时就读位于摩登镇(Modern)格拉斯顿伯里男子中学(Glastonbury High Boys)。 考克斯于八零年代展开他的职业DJ生涯,创作的音乐类型是以Hardcore dance与acid house类型为主。1988年时考克斯开始尝试在同一个时间内使用三个唱盘,他称自己为「三盘巫师」。他也曾经在 The Eclipse, Edge, Shelly's, Sterns Nightclub, Heaven, Angels, The Haçienda等俱乐部内演出Rave、Dreamscape与Amnesia风格的浩室舞曲。至今考克斯创立了Intec Records 与 23rd Century Records这两间唱片公司。2007年11月3日考克斯在委内瑞拉的一场演出中爆发了一场枪战。四人死亡,五人受伤。当时有人将考克斯面临枪战时的反应给录了下来,并且将影片内容公开于网际网路上。对此,考克斯在自己的网站上发表了一段谈话,表达遗憾之意。卡尔考克斯是第一位在「DJ杂志」所举办的全球百大DJ投票中获选为第一名的DJ。","text2":"考克斯有哪些杰出的成就?","label":1} {"text1":"路易斯-恩里克·马丁内斯·加西亚(,),是一名已经退役的著名西班牙足球运动员,以场上位置多变而闻名,职业生涯中曾经出任过除了守门员与中后卫之外的所有位置。目前为足球教练,曾执教西甲球队巴塞隆拿。1996年,路易斯·恩里克从皇家马德里自由转会至巴塞罗那,在双方球迷中都引起轩然大波。不过他最终凭借出色表现征服了巴塞罗那的球迷,之后更加成为俱乐部的队长。2008年,他开始担任俱乐部二队的主教练,并带领巴塞B队连续2年升班及取得西乙季军。2011年安历基确认不与巴塞B队续约,并于夏季正式担任意甲球会罗马主教练。他仅执教一季后离任,在2013年起担任西甲球会切尔达主教练,担任一季后离任,在2014年5月重返巴塞隆拿担任主教练,并且于2015年获得「五冠王」的成绩。2017年3月1日在带队6-1战胜希洪竞技后的新闻发布会上,宣布将不会续约并不会担任下一赛季巴塞罗那队的主教练。球员时期与名帅瓜迪奥拉是巴萨及西班牙国家队队友。","text2":"路易斯·恩里克以什么闻名?","label":1} {"text1":"宿雾太平洋航空(简称宿雾太平洋;英文:)是菲律宾的一家廉价航空公司,亦是该国最具代表性的航空公司之一,专门营运国内和国际航班。宿雾太平洋是目前菲律宾领先的国内航班运营公司,拥有最多的国内目的地以及航班数量,并且配备了现代化的机队。宿雾太平洋航空是JG顶峰控股公司的全资子公司,由菲律宾最富有的华裔家族之一 Gokongwei家族管理。宿雾太平洋目前由吴奕辉(John Gokongwei,顶峰控股的名誉主席)的假定继承人Lance Gokongwei领导。截止到2007年3月,公司拥有1182名员工。2010年,宿雾太平洋航空在推出安全设备简介舞圣诞节特别版本,备受国际赞扬,但也有少数妇女团体抗议。宿雾太平洋航空表示,有许多旅客因不看安全设备简介而使自己受伤,宿雾太平洋航空此举除了增添机上活泼元素,也让旅客注意自己的安全。宿雾太平洋航空也同时表示,空服员会在安全设备简介舞圣诞节特别版本前预先示范正常版。2012年7月3日,宿雾太平洋航空宣布将提供可在机上付费使用Wi-Fi的服务。。2014年4月10日,欧盟宣布将宿雾太平洋航空从禁制名单上除名,意味著宿雾太平洋可以开展往返欧洲的航线 。宿雾太平洋航空飞往34个国内目的地,以及14个国家的27个国际目的地。","text2":"宿雾太平洋航空在推出安全设备简介舞圣诞节特别版本此举主要是为了什么?","label":1} {"text1":"成人教育(),是以成人为对象的教育,通常指在经历一般正常教育后的再学习,例如在的社区大学、进修学校等,所学不一定是一般所认知的科学或人文教育,如语言、文化、艺术、兴趣方面的课程,也有些是技能培训或考取专业资格或受到业界承认的证书等。1798年,英国设立成人学校于诺丁汉;各西方国家在19世纪中叶后实施成人教育。联合国教科文组织(UNESCO)于1976年所举办的成人教育会议中对成人教育所下定义为:『代表具有全部有组织的教育历程、而不管其内容、水准及方法如何;是正式或非正式;是学校教育的代替或学校教育的延长;其目的在提供社会中的成人,发展潜能、充实新知、改进技术与提升专业资格,导引其新的发展或使其在态度或行为上产生改变』(杨国赐,1989)。欧洲经济合作开发组织(1977)为成人教育下的定义是:『指已逾规定离校年龄者,提供各种精心设计的活动或方案,以满足个人生活中的学习兴趣或需要,其范围包括非职业性的、职业性的、一般的、正式的、非正式的学习与教育』(杨文彬,2003)。台湾教育部的『发展与改进成人教育五年计划纲要』则将成人教育定义为:『指提供不再参与一般学校教育的成人有关基本教育、进修教育、职业进修教育、生活及休闲教育等活动机会,以增长成人生活基本知识;增进工作知能;扩展生活领域,提升民主素养及充实精神内涵,好充分发挥全民教育,终身教育的功能』(教育部,1991)。","text2":"1798年,英国在什么地方设立成人学校?","label":1} {"text1":"龙舌兰酒效应原意指1994年墨西哥所爆发的金融危机。1990年代开始,金融市场迈向全球化趋势之下,当某一地区或国家产生严重金融震荡甚至是危机时,会对临近地区而至于全球造成影响,媒体为增加报导吸引力,于是会从此地区或国家取一代表物名并直接冠于其上。而当1994年墨西哥爆发严重的金融危机时,媒体便用当地著名的代表物-龙舌兰酒(Tequila)-冠上名号,从此金融学家与媒体便普遍的将此次金融危机称做「龙舌兰酒效应」(Tequila Effect)。造成此次墨西哥金融危机的原因:一般金融界将「龙舌兰酒效应」指为金融债券不稳甚至大量倒债造成开发中国家整体市场的大波动。之后部分媒体引申将「龙舌兰酒效应」扩大为:遭到与墨西哥相关的事物(国家、比赛队伍……)所击溃,如同被龙舌兰酒狂灌而失去应有的水准,","text2":"媒体为什么会有从此地区或国家取一代表物名并直接冠于其上的习惯?","label":1} {"text1":"湖南路街道是中国上海市徐汇区下辖的一个街道办事处。,位于徐汇区东北部,东以陕西南路与黄浦区瑞金二路街道相邻,西以兴国路毗邻长宁区,北以长乐路、华山路毗邻静安区,南临复兴中路、淮海中路,面积1.73平方公里,有居委会16个,户籍人口5.34万人(2008年)。办事处设在复兴西路62号。湖南路街道辖区原为上海法租界西部的高级住宅区,至今仍保留700多幢花园住宅,居住此区域的居民中,知名人士、离休干部多。湖南路街道下辖淮中居委会、安福居委会、兴武居委会、金波居委会、武康居委会、春华居委会、华康居委会、复永居委会、新乐居委会、张家弄居委会、复襄居委会、复中居委会、东湖居委会、陕新居委会、淮海居委会、延庆居委会。","text2":"湖南路街道位于什么地方?","label":1} {"text1":"海军陆战队两栖侦搜大队,又称中华民国蛙人部队或中华民国陆战蛙人,为中华民国海军陆战队指挥部直属大队之一。在成为正式侦搜队员之前,每位学员都必须在集训队中先受8到12周不等的「两栖专长训」,训练课程大致分为跑步、游泳、操舟等三大部分。在这8到12周的集训生活中,是每位侦搜队员最痛苦却也是最美好的回忆。在集训阶段完成前,每一期的集训队都必须通过「综合考验周」(又俗称为「克难周」)的洗礼。综合考验周共分为22堂课,这22堂课也就是先前在集训队受训期间所学到的所有课程。每位学员必须在这短短的六天五夜中,将受训的成果展现出来。综合考验周的最后一关,俗称为「天堂路」,因为-{只}-要通过该项考验就能结束犹如地狱般痛苦的训练,宛若飞上天堂一样,而得名。天堂路的考验在军教片及新闻媒体的宣传下,成为台湾民众对海军陆战队训练的一项基本印象。此部队与陆军海龙蛙兵训练内容大至相同。若要比较差异,陆战队两栖较注重毅力以及忍耐度的训练,例如爬天堂路、或是跳粪坑用粪便刷牙;陆军海龙蛙兵较注重近身搏斗及作战技能部份,例如自由搏击、格斗刀术等。天堂路是一条以棱角尖锐的咾咕石铺成、长约50公尺的路面,全身仅著游泳短裤的海军陆战队员们必须在这样的路况上以匍匐前进、翻滚等方式通过,还要做出各种指定的战技动作,而且-{只}-要教官认定动作不合格便必须重来,沿途还会被泼洒盐水。所以凡通过天堂路的队员没有不遍体鳞伤的,但许多合格的队员们都将这些伤疤视为军人荣誉的象征与珍贵的人生经历。侦搜大队目前编制共有七个中队,分别如下(括弧内是精实案前分别所属国军特种单位,现已不存)两栖侦搜中队,拥有陆海及空中侦查渗透能力,蛙人的座右铭是「高山向我低头,海水为我让路」一旦战争爆发,蛙人是最早投射到敌人阵营的部队。特勤中队,俗称黑衣部队,格言「忠孝 勇敢 敏捷 机智」,行动隐密,快速打击敌方,负责台湾南部反劫持,反恐及反劫船任务,一旦接到上级任务,特勤队能够在15分钟内完成战备整备,每位成员必须能够独立作战,三分之二队员具有七年以上的资历。爆破中队,早年接受美国海军指导,负责爆破清除水中障碍,渗透特战任务。","text2":"综合考验周的最后一关,为什么叫「天堂路」?","label":1} {"text1":"邝其志,(),香港政府前高官,暱称「神童志」。曾任库务司(1995-1997)、库务局局长(1997-1998)、资讯科技及广播局首任局长(1998-2000),后转任香港交易所行政总裁,因马时亨引发「仙股事件」遭牵连,负行政责任辞职。在细价股事件后,他遗下一句「公道自在人心」,成为他的名言。2007年起,出任澳门励骏创建主席。当2008年亚洲小姐竞选后亚洲电视行政总裁王维基在内部会议质疑亚洲小姐赛果,要求有关职员提交资料,两日后他发现实际票数与赛果不符,于是通知张永霖。张永霖其后在记者会向公众及监票委员会主席霍震霆致歉,并宣布成立独立调查委员会,由前资讯科技及广播局局长邝其志担任主席,向亚洲电视提交报告彻查事件。于2009年11月21日他的儿子邝之彦与未婚妻孙凯宁结婚,邝其志以自己的平治豪华房车「KC1300」作为花车用途。于2011年6月,他在山光道会所泳池与刘鸣炜发生争执。当时邝、刘二人同于泳池畅泳,其间刘鸣炜疑因「游过线」与他发生肢体触碰,其后刘游到池边「转塘」时,邝其志将他的头压入水中。刘鸣炜当场与邝其志理论,并要求对方道歉。事件惊动在场救生员及马会保安,但邝其志否认出手。两人各执一辞,最终刘鸣炜向马会作正式书面投诉,马会翻看泳池闭路电视录像,发现邝其志确曾出手。马会事后曾尝试约见邝其志,他一直否认其事。在收到书面投诉后,马会召开聆讯,但邝其志未有出席。马会有感邝其志的品德及操守严重不当,于是决定重罚,宣布暂停其会员权益 12个月。","text2":"邝其志曾任什么职位?","label":1} {"text1":"信贷挂钩票据(Credit-linked note),是一种连动债和结构性产品。作为一种特别的金融衍生工具,虽然名称为债券,事实上是一种高风险的金融衍生产品。这类产品的发行商一般利用来自投资者的款项买入抵押品,然后作出若干“掉期”财务安排,从而令发行商得以将产品的潜在整体回报提升至高于传统银行存款的回报水平。这些“掉期”安排通常与若干拥有高信贷评级的参考机构有关连。在香港、台湾及新加坡等地曾有名为和等一般被认为属于信贷挂钩票据的产品发售,但认为发行的迷你债券在其三层结构内隐藏了不同于一般连结资产的债务抵押债券的,因此并不应被算作信贷挂钩票据。迷你债券以期权和套利为基础,即是以保险金的原理,赚取中间的差价。虽被称为「债券」,但实际上只是一堆信贷挂钩票据。它先成立一间空壳公司作为迷你债券的发行人,然后再向投资者销售一大堆信贷挂钩票据,包括一些债务抵押债券(),当中有许多高风险的信贷违约掉期,再混入一些较稳建的蓝筹股,以杠杆方式抵消其风险。发行人利用客户的资金投资,赚取这些证券定期产生的现金流,客户则可收取比一般银行存款较高的利息。","text2":"信贷挂钩票据是什么?","label":1} {"text1":"复线铁路,或双线铁路,是指在同一时间,两个相对的通行方向的列车互不干扰的铁路。有别于单线铁路,单线铁路通常每隔一段距离就设置一个会车时使用的车站,称为“会让站”,用于相对行驶的列车停车避让。复线铁路有点类似于“分道行驶”的公路,通过合理的设置,没有正面相撞的危险,可以大大提高运输的效率。但是复线铁路显然需要更宽的路基,在山区等地形困难的地段,有时不得不仍然只建设单线铁路,甚至部分仅供会车使用的车站不得不建在隧道中和桥梁上。复线铁路解决了单线铁路的会车的问题,但并未解决同方向不同速度的列车的超车问题。快速列车要超越慢速列车有两个方式:各国的繁忙铁路干线、高速铁路、城市轨道交通系统为了实现高密度和高效率,大部分都是复线铁路。平行复线是指两条铁路线路相距很近(中国铁路规定普速复线相距不小于4米,高速复线相距不小于4.4米),走向大致平行,沿途接轨的各车站相同。一般在地势简单或者一次新建成复线的情况下采用这种情况,具有占地节省、投资节省、共用车站的技术设备与行车设施等优点。所经过的桥梁、隧道往往都是复线桥、复线隧道。非平行复线是指两条线路线路虽然也是分为上下行固定行车方向,但是相距可能较远,线路走向可能不一致,沿途接轨的车站也可能不一致的情况。这是因为沿线地势复杂困难,难以布置双线;或者是既有单线运量饱和而新增建第二线改为复线行车,既有老线与第二线由于建设年代不同导致设计、施工技术差别巨大,往往是既有老线沿山麓河谷曲折展线,而新建第二线大量采用桥梁、隧道的捷径线。西康铁路的秦岭特长隧道就是因地势与施工限制,采取两条单线隧道。而丰沙铁路、陇海铁路的宝鸡——天水段,都是在既有单线技术条件下非常落后的情况下,新增二线采取了非常高的线路标准,桥隧比很大。双线铁路有复线与单线并列两种形式,差别在于列车行驶的安排,复线的情况列车在各别行车线只会向固定方向行驶,而单线并列则是各别行车线都可以双向运行。单线并列在台铁称为“双单线”,而台湾高铁称为“双线”。双线铁路并非指有两条铁轨的“双轨铁路”,而单线铁路也非指只有一条铁轨的“单轨铁路”。","text2":"单线铁路为什么被称为“会让站”?","label":1} {"text1":"广济寺位于中国安徽省芜湖市镜湖区赭山西南麓半山腰上,始建于唐代干宁年间(传为纪念金乔觉曾在此结茅庵驻锡而建),光化年间署名永清寺,又称广济院,北宋大中祥符年间改今名。因朝拜九华山的信徒必先到此进香,故又俗称“小九华”。建筑屡毁屡修,今尚存北宋治平二年(1065)所建赭塔(又名赭山塔,为五层六面的楼阁式塔,高28米,塔内供奉地藏菩萨坐像,赭塔晴岚为古芜湖八景之一)和民国八年(1919)所建滴翠轩(原名桧轩,即黄庭坚读书处,后用于供奉文昌帝和本地乡贤),其它建筑均为近年重建,分山门、天王殿、大雄宝殿和地藏殿(九华行宫)四进,其布局亦仿九华山,地藏殿西为滴翠轩(玉佛楼),东为药师殿,北为赭塔。寺内藏有唐至德二年(757)以砂金所铸地藏利成金印。","text2":"传说广济寺因何而建?","label":1} {"text1":"CVCGo(原名:CVC中文广播电台,已经停止广播服务。转型为原创视频拍摄)是CV传播机构(总部设在英国)的组成部分,隶属于设在澳大利亚的亚太媒体中心。CV是一个全球性、非营利的媒体组织,通过多种媒体向全世界许多地区传播信息,自90年代初成立以来,很快就成长为一个主要的国际性媒体,并用6种语言--普通话、英语、西班牙语、葡萄牙语、印地语,以及印度尼西亚等语言,针对不同文化背景的听众播出广播节目。2001年12月22日,CVC以“国际之声广播电台”为名,首次通过普通话、英语和印度尼西亚语进行短波广播,其中发射基地设在澳大利亚的普通话节目主要面向中国大陆听众。2005年7、8月间更名为“CVC中文广播电台”。2008年7月10日,位于新加坡的播音室开幕。2010年2月1日,CVC中文广播电台结束短波广播,并向网络多媒体平台转型,更名为CVCGo。CVCGo目前在澳大利亚和新加坡设有两个媒体制作中心,通过多种风格的原创视频短片和多种富媒体形式,依托社交网络和手机等多种方式,在互联网和手机网络上面向全世界华人,全方位、立体化的传播我们“相系 明天”的理念。短波广播:每个星期六和日,北京时间(UTC+8)06:00-06:30,15525千赫播出《快乐早点-张雪莉主持》、《心灵早餐-流星主持》。联系方式:(手机短信:1314-7586686)2011年3月27日,CVC宣布停止短波15525千赫的广播,标志着CVC中文电台正式停播。","text2":"CV是一个什么样的媒体组织?","label":1} {"text1":"保加利亚是一个位于欧洲东南部巴尔干半岛上的国家,毗邻罗马尼亚、塞尔维亚、马其顿共和国、希腊、土耳其和黑海。多瑙河构成保加利亚同罗马尼亚之间的边界(直到锡利斯特拉为止)。保加利亚国土面积110,550平方公里,略大于冰岛或美国的田纳西州。 保加利亚位于黑海西海岸。北与罗马尼亚,西同塞尔维亚和马其顿,南与希腊和土耳其接壤。虽然保加利亚的国土面积不大,但地形却十分多样。横贯保加利亚国土中央的巴尔干山脉将保加利亚的河流分为两大水系。以北的河流多注入多瑙河,以南的则多注入爱琴海。面积:\"总计:\"110,910 km²\"陆地:\"110,550 km²\"水域:\"360 km²面积比较:略大于田纳西州。陆地边界\"总计:\"1,808 km\"接壤国家:\"希腊 494 km;马其顿共和国 148 km;罗马尼亚 608 km;塞尔维亚 318 km;土耳其 240 km海岸线:354 km最低处:黑海 0 m最高处:穆萨拉峰 2,925 m","text2":"保加利亚国土面积有多少?","label":1} {"text1":"美瑾·德妮丝·霍丝(,台湾与中国大陆译为:梅根·福克斯,)是美国女演员、模特儿,在电影《变形金刚》和电视情景喜剧《希望和信仰》的演出最为人熟悉。她被英国男士杂志《男人帮》选为2008及2009年度「百大性感美女」第一名。1986年,美瑾·霍丝生在美国田纳西州罗恩郡橡树岭,拥有法国人、爱尔兰人、印第安人血统。她小时家境并不富裕,父母离婚后与母亲和继父一起居住,对她管教颇为严格要她不得交男友,念天主教学校,家中还有一位姐姐。她在5岁起学习舞蹈和戏剧。10岁时她移居至佛罗里达州圣露西港,在学校常被其他女生排挤,所以她说她只有一位女性朋友。2010年1月,她与足球巨星基斯坦奴·朗拿度一同取代大卫·碧咸夫妇,成为奢侈时尚品牌乔治·阿玛尼最新的形象代言人。梅根成名后也曾自己出钱拍公益广告片批评加州政府删减教育预算减少教师人数。2010年6月24日她与美国演员布莱恩·奥斯汀·格林于夏威夷举行私人婚礼,结为夫妻。2012年9月27日生下了第一个儿子。2014年2月14日生下了第二个儿子,此时已拍完2014年电影《忍者龟:变种世代》。","text2":"美瑾·霍丝的职业是什么?","label":1} {"text1":"角箱鲀(学名:),俗名长牛角、箱河鲀、牛角、牛角狄,为辐鳍鱼纲鲀形目箱鲀科的一种。本鱼分布于印度洋及太平洋的热带海域,包括东非、南非、马达加斯加、模里西斯、红海、塞席尔群岛、留尼旺、马尔地夫、印度、斯里兰卡、泰国、中国、韩国、日本、台湾、菲律宾、越南、泰国、马来西亚、印尼、新几内亚、澳洲、密克罗尼西亚、马里亚纳群岛、新喀里多尼亚、萨摩亚群岛、东加、万那杜等海域,属于热带近海底层鱼类。该物种的模式产地在印度。水深10至100公尺。本鱼体略呈正立方体;口小唇厚。鳞片特化成骨质盾板,表皮粗糙。幼鱼似黄底黑点的小圆球;成鱼在眼上方与腹部下方各长出一对长棘,背鳍软条 8至9枚;臀鳍软条8至9枚,体长可达46公分。本鱼鳍息在水流缓慢的沿岸地区,幼鱼偶尔会进入河口。属肉食性,常将海底的沙吹散以寻找食物。受惊吓时,体会分泌毒素。为观赏性鱼类。","text2":"角箱鲀什么时候会分泌毒素?","label":1} {"text1":"玛黑区(法语:Le Marais,意为沼泽,又译为\"玛黑区\")是法国巴黎的一个区域,横跨巴黎右岸的第三区和第四区,传统的布尔乔亚区域。19世纪末和20世纪上半叶,蔷薇路(Rue des Rosiers)周边地区成为许多来自东欧犹太人的家园。因此在法国沦陷期间,玛黑区也成为纳粹攻击的目标。1944年以后,该区一度衰落。1969年,安德烈·马尔罗使玛黑成为第一个保护区(\"secteur sauvegardé\"),该区拥有许多博物馆,美术馆和历史遗迹,大量文学家、艺术家居住在这里,玛黑区的艺术、文化氛围因此浓厚。不同于巴黎其他地方,这里原有的商店出售改业后,门面却不更换,这种保护措施导致奇怪的结果,如“面包店”销售时尚物品,或变成了豪华酒店。直至2013年,法国老佛爷百货集团公司旗下的巴诗威百货公司(LE BHV MARAIS)正式启动一项企业文化更新计划,使玛黑区又一次成为了巴黎时尚最前沿的街区,同时也成为了巴黎周末人气最旺的地方。近年来,随着国际游客到巴黎旅游的人数增加,玛黑区也开始成为文化、艺术旅游、巴黎式购物以及年轻化生活方式的震中。","text2":"近年来,随着国际游客到巴黎旅游的人数增加,玛黑区有什么变化?","label":1} {"text1":"东非狒狒(学名:\"Papio anubis\",英文:\"Olive Baboon\";),亦被叫做阿努比斯狒狒和橄榄狒狒,猴科猕猴亚科(Cercopithecinae)狒狒属的一种,主要分布在马利、衣索比亚、坦桑尼亚北部。天敌为狮子、豹等栖息于非洲之肉食性动物。在非洲,数量颇多,分布也很广泛。因为牠们会吃农作物,所以当地的农民非常厌恶东非狒狒,常常射杀牠们。橄榄狒狒是因为它的外表看起来很类似橄榄色。他的另一个名字,阿努比斯狒狒则是来自埃及的神--阿努比斯,它的头部从侧面看上去就像是阿努比斯的真实写照。雄性橄榄狒狒有别于雌性橄榄狒狒主要在体型的大小,重量和犬齿大小方面;平均而言,雄性橄榄狒狒长度约70厘米,平均为24公斤。雌性长度约为60厘米,平均14.7公斤。最大的个体头到身体的长度可以到114厘米,可能重达50公斤。头躯干长508-1,143毫米,尾长456-711毫米,雄性重约24-45公斤、雌性12-28公斤。毛色为橄榄色,脸部黑色无毛,雄性的颈部及肩膀部份毛较长,亦较强壮,口鼻部延长似狗,尾巴成倒U形。杂食性,主要食植物之各部位,以取食方便、供应充足为主要的考量因素。另外亦吃面包、猴饲料、香蕉、胡萝卜、地瓜、玉蜀黍、番石榴、蕃茄等物。也吃肉(如:羚羊、野兔)。东非狒狒是日行性动物,主要栖息于草原、草地、开阔的林地(woodlands)、石砾山地等区域,通常在地面活动,亦会爬树。群居动物,每群的大小通常为30-60只,雌性数量较多,但是雄性地位较高。雄性的东非狒狒借由打架来决定彼此的阶级,只有高阶的雄狒狒才能与发情中的雌狒狒交配。雄性大约在3岁时可交配,雌性则在3-5岁时进入发情期,直至24岁左右。怀孕期154-193日(五-七个月),通常只有一胎。于六个月大始断奶,约一岁时可自立。","text2":"东非狒狒主要分布在哪里?","label":1} {"text1":"穴醚是一类人工合成的,可以与阳离子发生配位的双环和多环多齿配体。“穴醚(\"cryptand\")”一词是指该配体形如空穴,将底物分子容纳在里面。整个分子是一个三维的结构。因此与单环的冠醚相比,穴醚配合物更加稳定,对底物分子的选择性也更强。形成的复合物具有脂溶性。唐纳德·克拉姆、让-马里·莱恩和查尔斯·佩特森通过对穴醚和冠醚进行研究,开创了超分子化学的先例,并因此获得了1987年的诺贝尔化学奖。最为常见且最为重要的穴醚是N[CHCHOCHCHOCHCH]N(右图),IUPAC名称为1,10-二氮杂-4,7,13,16,21,24-六氧杂双环[8.8.8]二十六碳烷,俗称[2.2.2]<\/nowiki>-穴醚。方括号内的数字表示在两个氮桥头之间每个桥上的氧原子个数。全胺穴醚对碱金属阳离子具有极高的亲和力,通过穴醚与碱金属作用,可以成功得到含K等碱金属阴离子的盐类。穴醚的三维内部空腔可以和外来离子紧密结合,形成的复合物被称为穴状化合物。结合能力最强的是较硬的阳离子,包括NH(铵离子)和镧系元素、碱金属、碱土金属的阳离子。穴醚利用分子中的氮和氧与这些离子配位,由于不同的离子与不同三维结构的穴醚结合能力不同,通过选取适当的穴醚,可以将不同的碱金属阳离子区分或分离出来。与冠醚类似,大环穴醚一般也是利用胺和卤代烃的缩合反应制备的。但由于环系更为复杂,穴醚的产率通常不高。穴醚的制备较为困难,且价格昂贵;但是,与冠醚之类的其他配位剂相比,穴醚与碱金属离子结合更紧密,选择性更强。 通过配位,穴醚可以将一般情况下不溶于有机溶剂的盐类溶于另一相中,用作相转移催化剂,加快化学反应的速率; 也可以稳定碱金属负离子,使碱化物和电子盐得以合成。另外,穴醚还可以帮助Sn之类津特耳离子(Zintl ion)的结晶。在制作核医学需要的显像用药物氟代脱氧葡萄糖(简称 FDG)时,会使用[2.2.2]穴醚来络合反应物KF中的钾离子,提高放射性F离子的亲核性,以便将F连接到脱氧葡萄糖中。","text2":"“穴醚”一词指的是什么?","label":1} {"text1":"广州国际女子网球公开赛是一项由女子网球联合会(WTA)在中国广州举行的室外硬地网球巡回赛事。该项赛事为WTA三级赛,从2004年起每年举办1次,总奖金为22万5千美元。参赛选手根据比赛成绩计算积分,累计至女子网球联合会的排名系统。赛事的首个单打冠军是中国运动员李娜于2004年夺得,这同时也是中国网球运动员获得的首个单打冠军头衔。广州国际女子网球公开赛采用《ITF网球比赛规则》和《WTA赛事规则》,为单打正选赛32人,双打正选赛16队的单淘汰赛。每场比赛采用三盘两胜制,每盘比赛在局数比分是6比6时,采用决胜局制(七分先胜利);双打第三盘按决胜盘制(十分先胜利)。","text2":"广州国际女子网球公开赛是什么时候开始举办的?","label":1} {"text1":"温泉关战役(Battle of Thermopylae),是前191年罗马共和国在罗马-叙利亚战争中与塞琉古帝国交战的一场战役,此战的结果结束在希腊半岛的战事。塞琉古国王安条克三世率领一支规模不算大的军队登陆希腊半岛,他听信阿塔马尼亚人亚历山大的进言,希望能获得其他希腊势力的支援一同对抗罗马,然而大部分的希腊城邦都站在罗马阵营,就连前盟友马其顿国王腓力五世也愿意援助罗马。根据阿庇安记载,当安条克三世进军色萨利并来到库诺斯克法莱,看到在先前在库诺斯克法莱战役中许多阵亡的马其顿士兵遗骸散落四处,安条克三世在那举办隆重的葬礼并指责马其顿国王腓力五世,使当时摇摆不定的腓力听到这些消息后决心加入罗马阵营,罗马将军巴埃比乌斯立即派遣阿庇乌斯·克劳狄率领2,000名步兵通过马其顿进入色萨利。 此时安条克三世正在围攻色萨利重镇拉里萨,而阿庇乌斯·克劳狄在附近升起火光使安条克以为腓力和巴埃比乌斯来到,匆忙撤往卡尔基斯,并无所事事度过整个冬天。当罗马执政官马尼乌斯·阿基利乌斯·格拉布里奥率领20,000名步兵和2,000名骑兵进入希腊后,很快的与马其顿收复色萨利。面对如此庞大的军队,安条克三世决定在温泉关抵挡罗马的进攻,除了这里地势险要外,并确保盟友埃托利亚同盟的联系不被切断以及保护海军基地卡尔基斯。安条克三世亲自率领军队防御温泉关,并让投石兵布署在高处,步兵驻守完善的防御工事,埃托利亚同盟的部队防守左方侧翼,其中2,000名步兵固守的特拉喀斯,另外2,000名步兵防卫阿索波斯要塞和当年波希战争温泉关战役所使用的山路。当罗马指挥官马尼乌斯看到塞琉古如此布署,分别派兵趁夜明前攻取波斯所用小道的两侧山头,同时准备在天明时进攻。会战开始,安条克三世先派轻装步兵突袭罗马军,但遭到击退。而先前派往攻击埃托利亚人罗马军队,终于在老加图领导的第二次攻击下成功击破埃托利亚人,便一路追击直至塞琉古军后方的大营,在不知后方有多少敌军又担心遭到背面攻击的恐慌蔓延整个塞琉古军,塞琉古军逐渐溃散,而罗马军立即在后扩大战果,安条克三世只能重整500骑兵逃出,罗马军获得战役胜利。据记载罗马仅损失100多人,而塞琉古几乎全军覆没,无可奈何下安条克三世只能撤回小亚细亚,而埃托利亚同盟不久后向罗马投降。","text2":"温泉关战役的结果是什么?","label":1} {"text1":"在奥斯曼帝国扩张(土耳其语:Osmanlı Devleti yükselme dönemi)的过程中,奥斯曼帝国在西南不断扩张直至北非地区,在东部则和萨非王朝发生战争。巴耶塞特的父王穆罕默德二世在1481年去世后,巴耶塞特与弟弟杰姆为王位继承权而开战,杰姆战败后逃到罗德岛,巴耶塞特登上王位。随后在1499年,他发动对威尼斯的战役,并攻占了威尼斯人在伯罗奔尼撒最后的据点和一些在亚得里亚海沿岸的城市。而在1511年,巴耶塞特镇压了了由伊斯迈尔一世和马木留克组成的反叛军。在巴耶塞特晚年的时候,他的儿子塞利姆和自己的哥哥艾哈迈德同样为了继承权而开战,塞利姆战败逃亡克里米亚。当艾哈迈德正准备加冕王位时,支持塞利姆的土耳其禁卫军杀死了他,并强迫巴耶塞特二世在1512年4月25日退位,由塞利姆继承皇位。退位后,巴耶塞特离开首都,前往出生地方隐居,却于途中去世,死后葬于今伊斯坦布尔的巴耶塞特二世清真寺旁。在塞利姆的统治下,奥斯曼帝国的领土大大地向南部和东部扩张。他打败了马木留克,并将叙利亚、黎巴嫩、巴勒斯坦和埃及并入帝国版图,包括圣城耶路撒冷以及开罗。至此,塞利姆已能够宣称他就是哈里发了。","text2":"奥斯曼帝国在西南不断扩张直至哪里?","label":1} {"text1":"周瘦鹃(),中国当代作家、园艺家,属于“鸳鸯蝴蝶派”代表人物之一。周瘦鹃原名周祖福,字国贤,系江苏省苏州府吴县人,1895年6月30日(清光绪二十一年闰五月初八)出生于上海,父亲是一名职员。6岁时父亲因病去世,靠母亲为人缝补维持家计。先后就读于上海储实两等小学和老西门民立中学。1912年中学毕业前患病,毛发脱光,得校长器重,留校任教,不久改行为职业作家。他在中学时代即开始从事写作,用笔名“泣红”,在商务印书馆出版的《小说月报》上发表了自己的处女作,话剧《爱之花》。1915年,周瘦鹃参加南社。此后任职于中华书局,翻译出版《福尔摩斯侦探案全集》和《欧美名家短篇小说丛刊》(其中包括高尔基作品中最早的中译),获得稿费后,得以和胡凤君结婚。自1920年起,周瘦鹃任《申报》副刊《自由谈》编辑,直到1932年底。同时,他还主编或与人合编《礼拜六》周刊、大东书局《半月》杂志(后改名《紫罗兰》和《新家庭》)、《紫兰花片》、《良友画报》等多种报刊。1931年,周瘦鹃迁居苏州,在王长河头辟紫兰小筑,人称周家花园。他往来于苏州与上海之间,继续负责申报的《春秋》副刊。1937年淞沪会战爆发,周瘦鹃与程小青两家经浙江南浔避居安徽黟县山区。次年《申报》在租界复刊,周瘦鹃也回到上海复职,居愚园路田庄,并在海格路售卖盆景维持生计。经同学蒋保厘介绍,周瘦鹃加入上海中西莳花会,在比赛中曾两度夺魁,获得彼得葛兰奖杯。第三届仅获次奖,遂愤而退出。至太平洋战争爆发,日军占领租界,周瘦鹃辞去《申报》副刊编务。1946年起,周瘦鹃再度隐居苏州,闭门研究盆景。1950年代以后,陈毅、周恩来、叶剑英等都前去拜访。此后曾发表有关花草园艺、游记的散文集《行云集》、《花花草草》、《花前琐记》、《花前续记》等。在文化大革命中,周瘦鹃受到张春桥的点名批判。1968年8月11日(农历七月十八日)投井身亡,年73岁。1973年获苏州有关方面平反并落实政策。当年被抄家没收的二百多个盆景未予实物归还,由有关部门折价补偿,合每盆二元五角人民币。被抄走的字画包括明代画家沈周的一幅中堂,后折价补偿七百元人民币。","text2":"周瘦鹃是谁?","label":1} {"text1":"洪兴社是香港漫画及电影《古惑仔》中出现的一个虚构的黑社会社团,由蒋震创立,也是主角陈浩南等人所属的社团。该组织名早在1989年一部香港电影《我在黑社会的日子》就出现过,但只是名字一样,和《古惑仔》系列漫画或电影中的洪兴社全无关系。客家帮派,1930年由中国宝安县蒋氏家族后人蒋震开山立派。1950年代移民潮,蒋震带领洪门门生南下,在香港湾仔海皮一带当苦力,深受该区「陀地」(本地帮派)欺压,为了反抗,是以「洪兴社」旗帜再度飘扬。1975年蒋震病逝,其长子蒋天生继任,将「洪兴社」企业化管理,创立「十二揸FIT人」位置,分区管理数以万计会员。1992年「洪兴社」会员人数已达五万人。由于蒋震将「洪兴社」龙头宝座世袭传予儿子蒋天生,这直系传位惹来数名得力下属不满,遂萌生去意分裂出「洪安社」、「洪义社」及「洪乐社」。","text2":"洪兴社是什么时候开山立派的?","label":1} {"text1":"羽毛球于1972年慕尼黑奥运会始设为表演项目,至1992年巴塞罗那奥运会正式设为比赛项目。中国羽毛球队渐渐上升为世界最强队位置。1992年巴塞罗那奥运首次获1银4铜共计5枚奖牌的成绩,2000年悉尼奥运会羽毛球项目的比赛获得4金1银3铜共计8枚奖牌,为历届奥运会羽毛球比赛的最好成绩;2004年雅典奥运会,中国羽毛球队获得3金1银1铜共计5枚奖牌。参加2008年北京夏季奥林匹克运动会的中国羽毛球队一行共计33人,其中男运动员9人,女运动员10人,官员与工作人员14人;领队李永波,副领队李卫国。官员与工作人员(14人):领队:李永波,副领队李卫国;教练(9人):田秉毅、钟波、陈兴东、李志锋、汤仙虎、贺向阳、唐学华、张军、翁建德;管理:胡学兵,医生:李建平、李全意。运动员(19人):男运动员(9人):林丹、鲍春来、陈金、蔡赟、傅海峰、郑波、谢中博、郭振东、何汉斌女运动员(10人):张宁、谢杏芳、卢兰、杨维、张洁雯、张亚雯、魏轶力、高崚、于洋、杜婧","text2":"羽毛球什么时候正式设为比赛项目?","label":1} {"text1":"紫荆(学名:),又名裸枝树(中国主要植物图说)、紫珠(本草拾遗),为豆科紫荆属的植物。落叶乔木,高达15米,胸径50厘米,但一般作为灌木或小乔木栽植,有变种垂花紫荆花大,为粉红色;黄山紫荆花为粉红或乳白色。叶近圆形,长6-14厘米,叶端急尖,叶基心型,全缘,两面无毛。花4-10朵簇生于老枝上。荚果长5-14厘米,沿腹缝线有窄翅。花期4月,叶前开放;果10月成熟。中国的特有植物,分布于湖北西部、辽宁南部、河北、陕西、河南、甘肃、广东、云南、四川、广西、山东、江苏、浙江等地,生长于海拔150米至1,400米的地区。北京的清华大学校花是紫荆花和丁香花,但通常也仅指紫荆花,这是由于紫荆花与校旗、校色的颜色一致,也是由于紫荆花在4月校庆日前后盛开。洋紫荆是香港法定代表花卉。在《香港基本法》中,市花洋紫荆的「洋」字被略去而误称为“紫荆花”,中国大陆媒体也依照“紫荆花”来宣传,故中国大陆民众多称香港市花为「紫荆花」,与一般意义上的“紫荆花”相混淆。","text2":"紫荆主要分布在中国的哪些地区?","label":1} {"text1":"张致(),锦州人。金朝民变领袖。1214年,张鲸、张致兄弟聚众反金朝,投降蒙古帝国大汗铁木真,次年,张鲸又企图背叛蒙古,事情败露,张鲸被蒙古监军石抹也先所杀。张致遂据锦州自立,自称汉兴皇帝,建年号兴隆(或作兴龙),号称黑军。他又联络当地地主武装,攻城夺地,很快即攻占附近的平州(今河北省卢龙县)、滦州(今河北省滦县)等数十州,并自称瀛王。但张致深感自己无法同蒙古帝国对抗,遂于1216年再次投降金朝并攻下兴中府(今辽宁省朝阳市)。蒙古派将领木华黎带兵讨伐张致,交战中张致节节败退。七月,木华黎攻克兴中府,张致只好退守锦州。部将高益背叛,将张致绑缚送往蒙古,张致在蒙古军前被杀。","text2":"张致在锦州自立后,号称什么军?","label":1} {"text1":"《纯情罗曼史》是一部由漫画家中村春菊所绘的BL漫画,后来被改编为广播剧、小说、电视动画以及游戏。与同为中村春菊所作之漫画「世界第一初恋」享有共同世界。《纯情罗曼史》内容分为三部分:此漫画的总销量目前己超过四百万册,在同类型的漫画作品里可说是异数。此系列的改编小说《纯爱罗曼史》和《自我中心纯爱》声称是「把漫画主角之一创作的妄想小说搬到现实来」的小说。生日:11月22日 血型:A型。 属性:傲娇受。同名电视动画于2007年宣布开始制作,于2008年4月开始播放。第二季电视动画则于2008年10月开始播放,片尾插图由中村春菊担任,动画风景制作则使用实景照片。相隔6年之后,于2014年7月宣布决定制作第3季,预定2015年7月开始播放。","text2":"《纯情罗曼史》是由谁所绘?","label":1} {"text1":"郭家梁,台湾著名中医师、针灸师。从军,随国民政府撤退至台湾,服务于高雄左营军队中,因个人兴趣,开始研究中医。退伍后,在左营经营中药行,组织中国针灸学会并任理事长,对于台湾中医学的推广有著相当重大的贡献。他也是台湾最早提倡食疗药膳的先驱。其子郭啸天也是著名中医师,以只针合谷穴,而能治万病,继承其父郭家梁的衣钵。据郭啸天医师统计,缓减癌症疼痛最主要且常用的穴道即为合谷与三阴交,上述两穴位是阴阳二总穴,而癌症本就常见气阴双虚,癌症疼痛更是易见气阴双虚证;有时,也会采合谷、太冲以开四关,或先用一阴一阳作为诊断,而后再依各证不同继续加穴辅助。郭啸天教授98年曾在台湾中医皮肤科医学会理监事会中,以唐朝“摸得较”之手法,轻摸一位同道,五分钟后,脖子下如小指节之瘤即变小,成一时佳话。2010年曾在山东针灸大会上即席演示郭家消瘤针法,令在场的一位中医师肋下的瘤迅速消散,技惊四座而声名大噪。其次子郭靖海为前马偕医院心脏内科医师,在93年8月7日发生车祸而往生。《中国针灸经穴学》为郭家梁 编著;郭靖海 增订。著有《中国针灸经穴学》、《针灸精蕴》等书。","text2":"郭家梁其子郭啸天擅长什么?","label":1} {"text1":"天主教圣地亚哥-德孔波斯特拉总教区(拉丁语:Archidioecesis Compostellanus、西班牙语:Archidiócesis de Santiago de Compostela)是罗马天主教在西班牙西北部加利西亚设立的5个教区之一,圣地亚哥-德孔波斯特拉教省的中心。成立于1120年。其主教座堂是相传耶稣十二门徒之一的雅各伯安葬于此,是著名西班牙朝圣地圣地亚哥-德孔波斯特拉主教座堂。2011年,在当地1,299,880人口中,有教友1,191,800人,占辖区总人口91,7%、1,071个堂区、771名司铎、11名执事、254名修士、1,035名修女。现任总主教为Julián Barrio Barrio。","text2":"天主教圣地亚哥-德孔波斯特拉总教区成立于哪一年?","label":1} {"text1":"山原猫(学名:'),又名南美山猫或安第斯山猫,是一种细小的野生猫科。牠们是传统上两种没有亚种的猫科之一。牠们的数量估计少于2500只,但却只有有限的保育工作。山原猫的外观像雪豹,栖息在海拔3500-4800米的地方,远远高于林木线。牠们约有家猫的大小,但尾巴较长,毛色呈银灰色,有斑纹及斑点。牠们体长约60厘米,尾巴长42厘米,肩高36厘米,体重5.5公斤。山原猫是所知最少及最为稀少的猫科之一,对牠们的认识都是来自仅有的观察及一些标本。牠们未被驯养。估计牠们只生活在秘鲁、玻利维亚、智利及阿根廷安地斯山脉的高山上。山原猫喜欢栖息在高山环境,其分布亦受当地的猎物情况所影响。总有效群体大小可能是低于2500只,因失去猎物及栖息地、被猎杀及没有多于250只的亚群等缘故而有下降的趋势。由于牠们只居于高山上,人类居住的山谷成为了屏障,分隔了牠们的群族。牠们在玻利维亚及智利因祭祀用途而受到猎杀。牠们的数量稀少,较同样生活在高海拔的南美草原猫更为稀疏。在其分布地,牠们的遗传多样性非常低。","text2":"山原猫的主要栖息地在哪?","label":1} {"text1":"塞拉纳岛(Serrana Bank)是大西洋众多小环礁中的一个,它大部分都是低于水面的暗礁,大概有50公里长和13公里宽,由众多的珊瑚礁和小岛组成,最突出的是西南部的珊瑚礁。塞拉纳岛是哥伦比亚领土,不过在1981年以前一直被美国占领,做为美国的一个军事基地,现在主要的访问该岛的人是捕龙虾的渔民。塞拉纳岛的名称一般认为是取自西班牙坐船遇难的人佩德罗·塞拉诺,这个名称在1545年首次出现在荷兰的地图里面,然后在1660年英国更广阔地在地图上标注了该岛。在1972年9月8日,美国和哥伦比亚签订了一项协议,协议承认哥伦比亚拥有对Quita Sueño Bank、Roncador Cay和塞拉纳岛的主权,协议于1981年9月17日生效。在2007年9月3日,5级飓风费利克斯的风眼经过了塞拉纳岛。","text2":"塞拉纳岛是哪个国家的领土?","label":1} {"text1":"尚纯()琉球国第二尚氏王朝第十一代国王尚贞王之长子,童名思德金。据《中山世谱》载,尚纯资质聪敏,好学崇道,亲近贤者并且重视有才能的人。《中山诗文集》收录了尚纯所写的绝句《咏双松》。尚纯于清顺治十七年农历正月初五出生,为父亲尚贞王之长子,清康熙八年二月初六封为佐敷王子,领佐敷间切,同年闰十二月十四加领中城间切,立为世子,封中城王子,并加领知行高千三百斛。康熙十五年(1676年)五月初八加领久米具志川间切,同时加领知行高四百斛。康熙二十八年(1689年)十二月二十八,尚纯的采地有所更动,由领佐敷间切改为领久米仲里间切。尚纯共有一妃四妻,育有三子一女,妃毛氏于康熙十七年(1678年)十月廿五生长子尚益(后为第二尚氏第十二代国王)。康熙三十年(1691年)八月初四,妻贺氏生长女国场翁主。两年后的(1693年)六月十一,妻景氏生次子尚监,康熙四十一年(1702年)的八月十七,妻曦氏生三子尚盛。康熙四十五年农历十二月廿三,身为世子的尚纯未及即位而薨逝,享年四十七岁,追尊为尚纯王葬于玉陵,奉祀于圆觉寺宗庙。","text2":"尚纯活了多少岁?","label":1} {"text1":"大连金州体育场,座落于辽宁省大连市金州区,是一座可承办足球、田径等运动项目的综合性体育场。总占地面积为7.9万平方米,其中,主体及内场占地3万平方米。现在该体育场可容纳观众30,776人。以下内容中新金州体育场将被称作金州体育场。1958年,老金州体育场竣工。当时这座体育场仅能容纳观众12,000人。以现今的标准来看,老金州体育场难以达到目前体育比赛,尤其是足球比赛的要求。随着中国体育赛事随着改革开放的深入而日渐增多,老金州体育场已经很难满足现代比赛的需求。20世纪90年代,大连市政府决定重建金州体育场。1996年10月26日,新金州体育场动土兴建;1997年6月18日竣工。而在之后,2007年,大连市政府又出资重新整修体育场,包括修缮电子屏幕等设施,使体育场达到了现今的面貌。该体育场承办的最重量级赛事就是1998年世界杯预选赛亚洲区十强赛,1997年10月31日中国队主场迎战卡塔尔队的比赛。在这场关乎出线的关键大战中中国队以2:3败北。而最终中国队在本次比赛中以3胜2平3负的战绩名列小组第三,未能冲入1998年世界杯决赛圈,也从此留下“金州没有眼泪”的说法。后来,该体育场成为了中国足球超级联赛大连实德足球俱乐部的主场。在这座体育场中,大连实德曾经多次获得甲A、中超以及中国足协杯的桂冠;这里也成为了大连足球甚至大连体育的象征之一。如同中国其他大型体育场,金州体育场也经常承办演唱会、大型文艺晚会等演出活动。","text2":"大连金州体育场有多大?","label":1} {"text1":"米高(\"迈克\")·安东尼奥·鲍威尔(,)是一位美国的田径运动员。目前为男子跳远(8米95)的世界纪录保持者。鲍威尔出生于宾夕法尼亚州的费城。在1991年世界田径锦标赛上,鲍威尔以5cm的优势打破了由鲍勃·比蒙保持23年之久的男子跳远世界纪录,成绩为8.95m。目前,该项纪录仍在保持,使得鲍威尔成为1900年以来保持世界纪录超过20年的第四个人。鲍威尔在1988年汉城奥运会和1992年巴塞罗那奥运会上均获得银牌,在1993年世界田径锦标赛和1995年世界田径锦标赛中分别获得金牌和铜牌。鲍威尔于1996年亚特兰大奥运会后退役,之后于2001年复出,并希望能够参加2004年雅典奥运会,但是没有如愿。","text2":"迈克·鲍威尔是哪个股国家的田径运动员?","label":1} {"text1":"裸狐鲣(学名:),又称裸䲠,俗名长翼,为辐鳍鱼纲鲈形目鲭亚目鲭科的一种。本鱼分布于印度太平洋区,包括南非、东非、红海、叶门、模里西斯、马达加斯加、葛摩、塞席尔群岛、留尼旺、马尔地夫、斯里兰卡、印度、缅甸、安达曼群岛、圣诞岛、可可群岛、中国、日本、韩国、台湾、菲律宾、越南、印尼、澳洲、斐济、新喀里多尼亚、东加、吐瓦鲁、密克罗尼西亚、萨摩亚群岛、法属波里尼西亚等海域。水深0至100公尺。本鱼体纺锤形,横切面近圆形,两背鳍几乎相连,在第二背鳍后方有6至7个离鳍,而在臀鳍后方有6个离鳍。尾鳍末缘呈弧形凹入,侧线完全,侧线后半部呈波浪状起伏,且在尾柄处和隆起的棱嵴相连。体背侧呈蓝紫色,腹部灰白色,身上无明显花纹或斑点。上下颔均具尖锐犬齿,背鳍硬棘13至15枚;背鳍软条12至14枚;臀鳍硬棘0枚;臀鳍软条12至13枚;脊椎骨38个,体长可达248公分。本鱼为沿岸中层洄游性鱼类,常一大群一起出现,属肉食性,主要以小鱼为食。为美味的食用鱼,属经济鱼种,可煮汤、盐烧或制成罐头,有雪卡鱼中毒之纪录。","text2":"裸狐鲣的横切面近似什么形状?","label":1} {"text1":"维珍尼亚·阿普伽(, ),美国医生,专科麻醉及儿科。她是麻醉学及畸形学的领先学者,被认为是新生婴儿科的始创人。阿普伽新生儿评分是全球最广泛使用的新生儿评估方法,此法大幅减少婴儿死亡率,亦令阿氏广为人知。1929年毕业于曼荷莲学院,1933年哥伦比亚大学医学院毕业,1949年成为该院正教授。 1959年,于约翰·霍普金斯大学获公共衞生学硕士。1953年首次推出阿普伽新生儿评分,用作快速为新生儿评估健康状况。阿普伽是哥大医学院首名女教授,在很长时间内亦是唯一的女教授。阿氏无结婚,1974年8月7日病逝于哥大医院。阿氏在生时获奖无数,1994年,美国发行一款以阿氏肖像为图之邮票。","text2":"阿普伽为什么会广为人知?","label":1} {"text1":"全球海平面观测系统(Global Sea Level Observing System, GLOSS)是一个政府间海洋学委员会的计划,其目的是测量海平面全球长期气候变化的研究。自2004年印度洋地震后,该计划的目的已经改变为收集海平面的实时数据。该项目目前正在升级超过290台观测站,以使他们能够通过卫星向新设立的国家海啸中心传送实时数据。他们还在组装太阳能电池板,令观测站即使在恶劣天气下仍能够继续运作。全球海平面观测系统与另一计划深海评估和报告海啸(Deep-ocean Assessment and Reporting of Tsunamis, DART)两者并非相互竞争,因为大部份GLOSS传感器都设于近岸地方,而DART传感器则设于深海地方。","text2":"DART传感器一般设在什么地方?","label":1} {"text1":"昙无谶(梵文:Dharmakṣema,385年—433年\/439年?),又音译为昙摩忏、昙无忏,意译竺法丰,中天竺人(《魏书》说他是罽宾人),南北朝佛教高僧,为著名译经家,是涅槃宗的始祖。昙无谶,中天竺国人。六岁时父死,独与母居。见沙门达摩耶舍。以谶为其弟子。习学小乘。后遇白头禅师遂业大乘。至年二十,诵大小乘经二百余万言。谶从兄善能调象,谶明解呪术,所向皆验,西域号为「大呪师」。谶后往罽宾,欲演大乘,彼国不合,乃东适龟兹(音丘慈),顷之,复进到姑臧,止于传舍,虑失经本,枕之而寝。有人牵之在地,谶警觉,谓是盗者。如此三夕。闻空中语曰:「此如来解脱之藏,何以枕之?」谶乃惭悟,别置高处。夜有盗之者,数过提举,竟不能动。明旦谶持经去,不以为重。盗者见之,谓是圣人,悉来拜谢。敦煌此时被河西王蒙逊攻下,昙无谶受到蒙逊的保护,开始译经。蒙逊尊为国师,并将军国大计与之商讨。关于沮渠蒙逊杀昙无谶的原因有两种说法。北魏太延五年(公元439年)始,魏太武帝(庙号世祖)听到昙无谶的名声,要求蒙逊将他送到长安,蒙逊于是杀害他。《魏书》与《北史》:「…昙无谶以男女双身法教授妇女,蒙逊诸女、媳妇皆往受法。蒙逊不遣,遂揭发此事,拷讯杀之。…」《梁高僧传》说因昙无谶执意要西行求《涅槃经》后分,宋元嘉十年(北凉义和三年\/公元433年)沮渠蒙逊遣刺客于路杀之、谶春秋四十九。。《大般涅槃经》《方等大集经》 《方等大云经》 《悲华经》 《菩萨地持经》 《优婆塞戒》 《金光明经》四卷 《海龙王菩萨戒本》","text2":"昙无谶是谁?","label":1} {"text1":"帚石楠(学名:')是指杜鹃花科帚石楠属的植物,别名「苏格兰石楠」。在欧洲,帚石楠属与其同科近类欧石楠属经常遭到混淆,被互作训解的称号亦有不少(这种情况在汉语领域中更为明显)。虽然它们在生物分类上界线清晰,但在文化理解上却曾造成不少误会。两种花属的主要分别是帚石楠的花萼的裂片遮盖著花瓣,而欧石楠的花瓣则反过来盖著萼片。。帚石楠的学名\"Calluna\"源自希腊语,词义是「扫除」,意指用以制作扫帚的植物,因此被翻译成中文的「帚石楠」。英语对石楠花有「\"Heath\"」及「\"Heather\"」两种称谓,当中的「\"Heather\"」实际所指的正是帚石楠。帚石楠是挪威的国花,同时亦是容易令人联想到苏格兰的重要标志。帚石楠是一种多年生灌木,普遍高度有20至50厘米(亦有少数种类高至一米),大致分布在欧洲及小亚细亚地区,适合种植于有足够水分及阳光的酸性土壤上,习性耐寒,较难于高温潮湿的地区生长。帚石楠品性强韧,能存活于经历有限度放牧的地区,亦有机会在遭受山火焚焦后依然持续繁衍。帚石楠是欧洲一种著名地貌(\"Heath\")植被的主要品种,广泛生长于酸土草原()及酸性泥炭湿地()。另外,帚石楠是多种反刍动物(如羊、鹿)与及部份鸟类(如松鸡)和昆虫(如有「帚石楠甲虫」之称的\"\")的主要食粮。帚石楠是相当受欢迎的观赏植物,许多花园及原野都种植了大量的帚石楠,在欧洲尤其闻名,曾经是西欧及北欧许多荒地的主要植被,因此有「山中薄雾(Mountain Mist)」的称号。当中最著名的帚石楠原野是位于德国北部的吕讷堡石楠草原。另一方面,帚石楠可制花茶,其花蜜亦具备药效价值。由于帚石楠的花卉部份相当美观,受到赏花者的普遍喜爱,因此出现了经由人工改良的帚石楠栽培种,衍生出不少在颜色、花瓣形态及成长习性方面都迥然不同的新品种。大部份新品种的帚石楠都以紫色为主,另外亦有白色、粉红色甚至红色的品种。北半球的帚石楠,开花期本来在七月的下旬,如今亦被人工扩展至十一月左右。而且部份帚石楠在冬季仍能保存,除了颜色会转变成棕色之外,花卉依然可以保持不凋谢的状态。帚石楠后来传入新西兰,却在当地成为了入侵物种。尤其在东格里罗国家公园里,帚石楠的过份繁衍更令当地政府感到忧虑。为解决帚石楠丛生的困扰,当局特意驱动大量「帚石楠甲虫(\"\")」进食帚石楠,最终成功处理相关问题。","text2":"帚石楠大致分布在欧洲哪些地区?","label":1} {"text1":"挖客是基于WEB2.0的互联网内容挖掘、分享社区。该词汇来源于美国的digg网站,后被中国IT业内人士引进并翻译成挖客,国内有挖客网。挖客这类网民主要热衷于分享在网络上发现热门以及有价值的信息。一般他们提供发现信息的URL地址,然后通过挖客社区把这些信息分享以便更多的挖客可以浏览。这些挖客推崇的精神是:挖客社区的内容由挖客自己决定。挖客社区的规则。当挖客分享或创作一条新信息到挖客社区时,这条信息将被列在社区的“潜水信息”栏中,等待其他挖客来“挖它”(投票)。挖客社区会有一套规则并结合挖客的“挖”来决定那些是有价值或者热门的信息从而可以被挖出水到“出水信息”栏,即到社区的首页。从而让更多的挖客可以快捷的浏览到热门以及有价值的信息。目前挖客社区采用分值制度,其中Karma值较流行,这样能够较好的保证挖客在分享信息时的合理性和公正性。","text2":"挖客是什么?","label":1} {"text1":"雕纹海龙(学名:)又称猪海龙,为辐鳍鱼纲棘背鱼目海龙科曲海龙属的鱼类。本鱼分布于印度太平洋区,包括东非、印度、安达曼群岛、中国、日本、台湾、菲律宾、圣诞岛、可可群岛、印尼、马来西亚、澳洲、新几内亚、斐济、帛琉、马里亚纳群岛、密克罗尼西亚、萨摩亚群岛、法属波里尼西亚等海域。该物种的模式产地在斐济群岛。水深2至3公尺。本鱼体延长,口小,吻突出呈管状,鳞片特化成骨板,体为褐色,尾鳍圆形。背鳍软条27至34枚;臀鳍软条4枚,体长可达8.5公分。本鱼常出现在潮池,躲藏于海藻从中,游泳能力弱,尾部不能卷曲。属肉食性,以浮游动物为主。5至6月为繁殖期,雌鱼将卵产于雄鱼孵卵囊中,由雄鱼负责照顾。无任何经济价值。","text2":"雕纹海龙有着怎样的生活习性?","label":1} {"text1":"元件细分化接取(Unbundled access)是1996年美国电信法的重要措施,既有地方电信业者(incumbent local exchange carriers,ILEC)必须提供互相竞争的电信业者可以接取地方电信业者个别的网路元件。1996年美国电信法提供三个方式让相关公司可以进入新的电信市场中,包含「以设备为基础的进入」(facilities-based)、「以转售为基础的进入」(resale-based)、「元件细分化接取的进入」(unbundled access based)。元件细分化接取的定义为:「在252条的规定中,于元件细分基础上有责任去提供、去要求电信业者给予电信服务、非歧视接取网路元件,并且在技术可行下,费率、条款、条件上要保持公平、合理、非歧视性。」既有地方电信业者应该提供元件细分的网路元件让其他的竞争业者可以合并一些元件以利提供电信服务。.这个措施让已经拥有客户、相关建置、维护人员、付款流程的相关公司,如有线电视、无线网路业者,以及电力、瓦斯公共事业业者,去租赁其没有的网路元件(如音讯交换器、电力提供系统等)以完备其地方的电话系统。由此看来,1996年电信法命令传统垂直阶层的电信产业分割数个部分,也就是元件细分网路元件(Unbundled Network Elements,UNE),已让新进业者能够选择他们有需要的部分,提供地方电信服务。.","text2":"我们要怎么定义元件细分化接取?","label":1} {"text1":"赵世良,中国著名图书馆学家,最早的留苏图书馆学者。曾任东北师范大学图书馆学教授、黑龙江省图书馆《图书馆建设》杂志主编。赵世良先生1931年6月3日出生于北京。1961年入黑龙江省图书馆,先后在科技阅览室、总库出纳台、书目咨询室、文献检索室和外文编目组等多个部门工作。1979至1981年,赵世良先生在东北师范大学图书馆学系任教。1982年,赵世良先生回到黑龙江省图书馆工作,任《黑龙江省图书馆》(现《图书馆建设》期刊)专职编辑、副主编、主编。他带领大家刻苦钻研,努力拼搏,使刊物从内部交流跃升为国内公开发行,从季刊变为双月刊,确立了中国图书馆学专业学术期刊的重要位置。赵世良先生2009年3月7在哈尔滨去世。","text2":"赵世良曾担任过哪些职务?","label":1} {"text1":"纽约三一教堂()位于纽约市曼哈顿下城的百老汇大道79号(百老汇大道与华尔街的交汇处),是圣公会纽约教区的一座古老的堂区教堂。1696年,英国圣公会购买这块土地兴建新教堂。目前的教堂祝圣于1846年5月1日基督升天节,当时是曼哈顿下城最高的建筑,是进入纽约港船只的欢迎灯塔。它被认为是哥特复兴式建筑的经典实例,1976年被列入国家史迹名录。1696年,州长本杰明·弗莱彻通过了在曼哈顿下城卖地建设新的教堂。1697年五月六日威廉三世国王以每年60桶(一桶约等于8加仑) 麦子为租金, 租给了教区。第一位主教是威廉姆维希,他在这个教堂任职49年,死于1746年。2001年9月11日,世界贸易中心的坍塌撞倒了教堂院内生长了一个世纪的无花果树。三一教堂是纽约市最大的土地拥有者之一。","text2":"纽约三一教堂在什么地方?","label":1} {"text1":"泡泡浴()()直译为肥皂乐园,亦有译为泡泡澡,是存在于日本的一项性服务,曾经被命名为土耳其浴(),后来由于抗议改为现名,内容类似泰国浴。具体上来说,就是由一名性工作者向嫖客提供性服务的一种方式。在很多知名的成人电影作品当中,泡泡浴通常有性暗示的意味,即妓女通常会帮客人先抹上肥皂,而且方式与一般方式(鸳鸯浴)的有所差异,妓女会以身体的任何部位(如大腿、胸部甚至阴部)透过身体互相摩擦的方式抹上肥皂,借由全身的力量达到高级情趣的意味。在日本风俗店中,泡泡浴属于店舖型性风俗营业1号。根据统计,日本全国的泡泡浴店舖,目前有1,249家(平成20年,2008年)。另外,某些泡泡浴店仅限日本人前往消费而谢绝外国人。泡泡浴最早被日本叫做土耳其浴(日语:),原意是指中东奥斯曼帝国的一种公共浴室,与伊斯兰教的教义规定祈祷之前必先沐浴有关。土耳其浴巧妙的被日本人理解运用,后因土耳其留学生向厚生省诉求。1984年12月19日,「东京都特殊浴场协会」公开寻求改名,借用英文Soapland改为泡泡浴(ソープランド)的现名只对外国人开放的日本第一而且唯一的泡泡浴在2017夏天开放了","text2":"具体上来说,泡泡浴是什么?","label":1} {"text1":"阿史那皇后(),北周武帝宇文邕的皇后。突厥木杆可汗阿史那俟斤之女。为得突厥之助灭北齐,宇文邕于565年遣使迎娶木杆可汗之女。568年三月立阿史那氏为皇后。皇后美姿貌,善举止,武帝深敬之。但武帝担心因此会被突厥控制,疏远了皇后。武帝的外甥女窦氏劝周武帝说:“四边未静,突厥尚强,愿舅抑情抚慰,以苍生为念。但须突厥之助,则江南、关东不能为患矣!”这时窦氏只有六七岁年纪,能有如此见解,顿时语惊四座。周武帝正色以对,立即接纳甥女的进谏,从此对阿史那皇后态度大为转变。578年六月,武帝驾崩,阿史那皇后被尊为皇太后。579年二月,改为天元皇太后。580年二月又尊为天元上皇太后。宣帝宇文赟驾崩,静帝宇文阐尊她为太皇太后。隋朝开皇二年(582年)殂,年三十二。与宇文邕合葬孝陵,谥号武成(墓志作“武德”)皇后。","text2":"阿史那皇后的谥号是什么?","label":1} {"text1":"上海光源(,缩写为)是一台高性能的中能第三代同步辐射光源。工程包括三大加速器,分别是一台150MeV的电子直线加速器、一台能在0.5秒内把电子束能量从150MeV提升到3.5GeV的全能量增强器和一台周长432米的3.5GeV高性能电子储存环,由中国科学院上海应用物理研究所,位于上海浦东张江高科技园区,工程于2004年12月25日动工,于2009年4月完成调试并向用户开放。上海光源工程首批还建造了生物大分子晶体学线站、XAFS线站、硬X射线微聚焦及应用线站、X射线成像与生物医学应用线站、软X射线谱学显微线站、衍射线站和X射线小角散射线站等七条光束线与实验站。光源能量位居世界第五,仅次于日本SPring-8、美国APS、欧洲PETRA III、ESRF四台高能同步辐射光源。","text2":"上海光源工程包括哪三大加速器?","label":1} {"text1":"钱烈宪是一名在中国知名的博客作者,也是一位作家,曾经从事媒体行业,担任记者、编辑、评论员;现任果壳网副总裁。自称江苏人,爱好神话谶纬。钱烈宪并不是他的真名,而是写博客时用的名字,博客名字叫做“钱烈宪要发言”。而他发表正式文章时则使用“徐来”的笔名,其作品曾获得第三十届香港青年文学奖小说高级组冠军。而在网上发表评论时,则一般使用“拇姬”或“Moogee”这个名字。他的真名则并不被公众知晓。钱烈宪为其博客用户名,博客名称“钱烈宪要发言”(取自“前列腺要发炎”的谐音),格言是“内部交流 供造谣用”(ProState In Flames)系牛博网主力牛博之一。博客内容绝大多数以转贴为主,并辅以自行杜撰的标题,将其想法隐晦地表达出来。其转帖内容涵盖各个方面,大多数与时事和意识形态相关。2008年11月24日被从牛博网牛博名单中清除,原因不详。但很快又在牛博网注册账号“拇姬”,并重新开通博客“钱烈宪继续发言”(普通博客,有别于先前的牛博)。牛博网于2009年1月9日被关闭之后,博客搬迁至凤凰博报。《想象中的动物》(ISBN 9787802255432)为钱烈宪的第一部作品,曾以专栏的形式发表于《上海一周》、《新京报》等多家报刊。该书被认为在笔法上模仿博尔赫斯,以亦真亦幻的手法描述远古传说中的动物。按照书中的内容,《想象中的动物》分为五个章节,分别为禽部、兽部、鳞部、虫部、附录。2008年11月14日,当时南方报业旗下的《新京报》文化版编辑徐来在其博客“钱烈宪要发言”上发表文章《传孔庆东被拘审查》,称孔庆东因组织“北大主体思想学习小组,为朝鲜提供情报被警方带走”。2009年2月14日,当徐来在朝阳区万达广场举办新书《想象中的动物》读者见面会时,被孔庆东的助理杨春持刀刺成重伤。杨春以故意伤害罪判处有期徒刑4年6个月。2009年2月14日,钱烈宪作为嘉宾参与北京市单向街书店主题为“钱烈宪的小趣味”讲座。讲座结束后钱烈宪上卫生间,两名不明人士尾随之。亦有说是两男子将他架进男厕所。其一持匕首将他刺伤,另一持菜刀欲砍向他的手,未遂。事后两名行凶者迅速逃逸。警方已经介入调查。由于“钱烈宪要发言”的内容常常触及一些敏感方面,所以“钱烈宪要发言”的读者广泛猜测其遇袭与博客有关。当天钱烈宪在北京市朝阳医院接受手术。","text2":"钱烈宪的博客内容主要包括什么?","label":1} {"text1":"尹福(),绰号「瘦尹」、「尹铁镯」(铁镯尹)。清代直隶省(今河北)冀县北漳淮村人,董海川之徒,清末八卦掌名家,开创尹氏八卦掌。尹福幼年因家乡盗匪为祸,流落至北京生活,到剪刀行当过学徒,后以卖油条、麻花卷度日。原习罗汉拳、弹腿等,师从著名镖师秦凤仪。在秦凤仪死后,拜肃王府太监董海川为师,因入门最早,为其大弟子。尹福天资聪颖,事师以礼,时时跟随在董海川身边,师兄弟皆对他十分佩服。艺成后以护院为生,曾任「崇外税务司」巡检、「善扑营」教习,后经满人佛音尼布(汉名叶诗梦)举荐供职皇宫担任侍卫,宫女、太监皆称其为老师。因为身形瘦长,外表斯文,而有「瘦尹」的绰号。他的掌力惊人,人一旦被他扣住就无法脱身,所以又被称为「铁镯子」、「尹铁镯」。他身处皇宫之中,行事低调,弟子多为皇宫侍卫,他也负责教导许多皇族成员武功。其师弟程廷华,在北京城中,与江湖人物来往密切,门徒遍满北京城,称为程派八卦掌,形成八卦掌的两大传承。当时又称尹福为宫内派,而程廷华为宫外派。尹派的特点主要是手掌平放,又称牛舌掌。掌势较少,较为朴实无华。另一个特色,则是以罗汉拳为主要入门。其长子尹世庭早逝,长女尹金玉嫁给其徒何金奎,三子尹成璋、四子尹玉璋皆承家学,擅长八卦掌。他的弟子有宫宝田、杨名山、马贵、居庆元、李永庆、马世清、门宝珍、钟声、金增启等人。","text2":"为什么师兄弟皆对尹福十分佩服?","label":1} {"text1":"基隆街为台湾日治时期中期(1920年至1924年间)存在之行政区,位于台北州基隆郡。辖属范围包含今基隆市的中山、中正、仁爱、信义、安乐等5个区。原属基隆堡之基隆街、仙洞-{庄}-、牛稠港-{庄}-、石硬港、田藔港-{庄}-、大水窟-{庄}-、大沙湾-{庄}-、社藔-{庄}-、深澳坑-{庄}-、八-{斗}-仔-{庄}-、狮球岭-{庄}-、大武仑-{庄}-、蚵壳港-{庄}-、内木山-{庄}-、外木山-{庄}-、大竿林-{庄}-、基隆屿、花瓶屿、棉花屿、彭佳屿基隆街辖域内分为基隆、大沙湾、社寮、八-{斗}-子、深澳坑、田寮港、大水窟、石硬港、狮球岭、蚵壳港、大武仑、大竿林、内木山、外木山、仙洞、牛稠港、基隆屿、花瓶屿、棉花屿、彭佳屿二十个大字。1924年10月,基隆街升格为基隆市,成为台北州之州辖市,此后便维持此态直至日治时期结束为止。","text2":"基隆成为台北州辖市一直延续到什么时候?","label":1} {"text1":"梅花泉位于章丘市明水百脉泉公园内,处于宋代著名女词人李清照故居中,为济南七十二名泉之一。“睡起觉微寒,梅花鬓上残”,这泓被清照园内亭台楼阁、绿柳垂杨环绕的清泉,恰有五处泉眼均匀分布在泉池的东北侧,正应了“梅花五出”,泉名即由此而来。泉池面积1800平方米,正常年份的喷流量为0.5立方米/秒,水量较大,清泉喷涌,玉珠四溅,水汽氤氲,丝毫不输著名的趵突泉。更有趣的是,每处泉眼四周都生长着茂盛的翠绿色水草,被泉水冲击纷披四散,宛如花瓣随风轻摇,每处泉眼已自成一花,五处泉眼又汇成一朵白色梅花,映衬在墨绿色泉池中,美不胜收。泉水漫过锦江桥,汇入绣江河,游人站在桥上犹如立于水中,不时有泉风拂面,雨丝沾衣,加之清照园内浓郁的文化气息,只觉仿佛身临画境。有些可惜的是,近年来济南地区地下水位有所下降,其中一处泉眼水量稍有减小,影响了梅花泉整体的效果。其附近还有漱玉泉、百脉泉、墨泉等。","text2":"梅花泉在哪里?","label":1} {"text1":"水教堂(Wasserkirche)是苏黎世利马特河中的小岛上的一座教堂,位于苏黎世的2座中世纪大教堂,苏黎世大教堂和苏黎世圣母大教堂之间。该地点在古代曾是异教集会之所,中心的那块石头现在教堂的地窖内,根据中世纪传说,这是圣徒Felix 和 Regula被处死的地方。第一座教堂建于10世纪,在各个阶段进行了重建,在1486年完成的重建达到了顶峰。在宗教改革过程中,水教堂被确定为偶像崇拜的地方,被世俗化,1634年成为苏黎世第一所公共图书馆,成为学习的中心,在19世纪极大地促成了苏黎世大学的创建。1839年,该岛与利马特河右岸连接,修筑了Limmatquai。1917年图书馆并入中央图书馆(Zentralbibliothek),建筑用作谷仓一段时间后,1940年开始重建和考古发","text2":"在宗教改革过程中,水教堂有了什么变化?","label":1} {"text1":"马沙沟位于台湾台南市将军区的长沙-{里}-、平沙-{里}-,将军溪出海口南侧。有长沙与平沙两村落,拥有两座渔港及一座滨海游憩区。因近年举办海滩派对而逐渐知名。旧南瀛八景之一「绿汕帆影」之所在。将军区长沙-{里}-、平沙-{里}-的名称中的「沙」字,即是来自于马沙沟。而将军区的马沙沟与顶山子脚、下山子脚、北埔四地因为同是吴姓开垦之地,故合称「四埔」,吴姓家族对外自称为「四埔吴」。根据清朝方志的记载,马沙沟与北门屿斜对,南隔青鲲鯓沙线六里,原本应为滨外沙洲,后来才与陆地连在一起。故过去有人认为马沙沟的地名与当地原为滨外沙洲有关。但学者黄文博认为,该地名是因为「马加、鲨鱼多在水浊的沙沟底始易捕获」而来,并加上开庄者陈马洲名字中的「马」字而来。而学者杨森富则认为马沙沟之名来自西拉雅语中MASAKAU之音译,「MA」是接头语,而「SAKAU」同「SAKAN」,为渔港之意,而「AU」则为地名语尾。台61线西滨快速公路将军交流道
","text2":"为什么过去有人认为马沙沟的地名与当地原为滨外沙洲有关?","label":1} {"text1":"安雅·罗素法(,),来自俄罗斯圣彼得堡的模特儿。她是《全美超级模特儿新秀大赛》第十季的亚军。2008年,安雅宣布改回出生时的名字:安雅·罗素法(Anya Rozova),在此之前是使用安雅·冈()。安雅于俄罗斯出生,后来被一个居住在美国夏威夷群岛欧胡岛檀香山的家庭领养。安雅十七岁时曾参与香奈儿、路易·威登及芬迪(Fendi)等品牌的非正式时装秀。2007年,她于瓦伊帕胡高级中学毕业。毕业后,她当了一名售货员。她曾为Russell Tanoue拍摄照片,Russell Tanoue称赞她是「有前途的新面孔」。安雅在半准决赛面试时说她对模特儿行业充满热诚,所以参加全美超级模特儿新秀大赛。她于比赛中表现出色,曾五次首名入围,平均入围顺序更拿下历届以来最优异的成绩(2.64),另外胜出三次小挑战,分别获得与评判尼祖·百克拍照、为柠檬味道的七喜拍摄广告的机会及十万美元、和盖马蒂洛(Gai Mattiolo)设计的晚装。在最后两强中,安雅与另一名参赛者惠妮·汤姆森为范思哲走秀,但评判认为她在台上不够惠妮突出,所以选了惠妮当冠军,安雅屈居亚军(但就整体表现来说,部份网友认为安雅才是第十季名副其实的冠军。)安雅在比赛拿五次第一,也胜出多次小挑战。安雅赛后再次与Russell Tanoue合作,为2008年4月30日出版的MidWeek杂志拍摄封面及内页照。其后她参加了V杂志与Supreme模特儿公司合办的模特儿选拔赛2008。她其后更与Elite签约。最近她与香港的模特儿公司 Style International Management 签约,并在香港发展其模特儿事业。她曾在很多香港的时装杂志中任模特儿,《Jet》、《东方日报》、《Elle》等。","text2":"安雅·罗素法合作过的香港杂志有哪些?","label":1} {"text1":"刘氏鳄属(学名:\"Lewisuchus\")是主龙类爬行动物的一属,身长约1公尺,生存于三叠纪中期(拉丁尼阶)的阿根廷,约2亿900万年前。刘氏鳄属于恐龙形类的西里龙科,这是一群恐龙的原始近亲。属名是以美国化石挖掘家Arnold Lewis为名。某些原本的刘氏鳄化石,现已重新归类到其他属,而剩余部份可能属于伪兔鳄。科学期刊《自然》在2010年的一份文章,指出刘氏鳄、伪兔鳄可能是相同物种,两者的体型接近、化石被发现相同地层,但是两者的化石重复部位少,无法做出详细鉴定、结论。由于刘氏鳄的命名时间,比伪兔鳄早了15年,如果两者是相同动物,刘氏鳄将具有学术上的优先权,而伪兔鳄是刘氏鳄的次异名。","text2":"刘氏鳄的命名时间比伪兔鳄早了多久?","label":1} {"text1":"《哥斯拉·伊比拉·魔斯拉 南海大决斗》(日文原名:ゴジラ・エビラ・モスラ 南海の大决闘)是1966年12月17日上映的日本电影,哥斯拉系列电影的第7部作品,日本观众人数达到421万人。良太为了前往南海寻找自己失踪的哥哥弥太,一直想要找到一艘船,某天良太参加一场比赛,而那场比赛第一名的奖品正是一艘游艇,但是却因为报名时间已过而徒劳无功,不过在因缘际会下结识了市野和仁田,三人一同到港边去看游艇,结果在其中一艘游艇上遇到了抢劫犯吉村,而急著寻找自己兄长的弥太趁著其他三人熟睡时将游艇开出,结果在海上遇到暴风雨,四人漂流到一座岛上,在那座岛上四人发现了海里有龙虾怪兽伊比拉,然后岛上有神秘组织红竹,红竹似乎在岛上进行一项不为人知的计划,而四人正巧碰上了一位从红竹手下逃脱的一位女原住民,在那位女原住民的帮助之下四人潜入红竹的基地,没想到却误触警报,良太在惊慌之中抓住了一个气球往天空飘去,其他人为了逃避红竹的追捕于是逃到了一个山洞,没想到他们发现了哥吉拉正在那个洞里面沉睡著...","text2":"《哥斯拉·伊比拉·魔斯拉 南海大决斗是哥斯拉系列电影的第几部作品?","label":1} {"text1":"李家昕(英文名:,)是新晋的体育记者,李家昕于2003年进入香港浸会大学修读本科课程,第一年修读社会科学学士的中国研究课程, 翌年转糸至体育系,修读体育及康乐管理文学士课程, 并于2007年毕业。毕业后先后投身香港有线电视及无线电视。 她于2008年北京奥运前获委派无线电视翡翠台的「电波圣火令」节目主持之一,走遍世界追纵圣火的传递,节目并于黄金时间于无线电视翡翠台播出,令她走红,广为市民所认识。 于奥运期间,她被派往青岛采访风帆赛事。2008北京奥运完结后,李家昕留任无线电视当体育主持,主持包括体育世界等体育节目。2009年,李家昕重返香港有线电视继续工作。幼稚园: 翠华幼稚园小学: 浸信会吕明才小学 (1990-1996)中学: 浸信会吕明才中学 (1996-2003)大学: 香港浸会大学 (2003-2007)保龄球, 曾晋身香港代表队羽毛球, 曾当选香港浸会大学校队香港电台 (2006)2006年世界杯客席主持香港有线电视 (2007-2008)担任体育记者,负责采访及报道体育消息。香港无线电视 (2008)担任体育记者及节目主持,包括:电波圣火令,2008年北京奥运会,体育世界...等香港有线电视 (2009)担任体育主持。CAPDASE Marketing and PR (2012)","text2":"李家昕哪一年大学毕业?","label":1}